Download as pdf or txt
Download as pdf or txt
You are on page 1of 1316

f m m-

Jf

CLASS XI (Code 241) VOLUME-1

ow
Based on the syllabus of Applied Mathematics (Code 241) prescribed
by CBSE for Class XI under 10+2 Pattern of Senior School Certificate Examination

e
Fl
re
F
ur
f or
ks
Yo
oo

Dr. R.D. SHARMA


B

B.Sc. (Hoits) (GoldMedalist), M.Sc. (GoldMedalist), Ph.D.


Professor and Head of Department of ScierKes and Humanities
re

Delhi Skill and Entrepreneurship University (DSEU)


u

Aryabhatt DSEU Ashok Vihar Campus, Delhi


ad
Yo
d
Re
in
F

DHANPAT RAI PUBLICATIONS (P) LTD.


22, ANSARI ROAD, DARYAGANJ, NEW DELHI-110002
E-Maii: ish.dhanpat@gmail.com
Combined Price : ? 745.00 for Vol. 1 & 2
(Not to be sold separately)

To prevent a pirated book being sold to you, always ask the bookseller
to put his stamp on the first page of this book.

ISBN: 978-81-944765-7-3

w
F lo
First Edition : 2020
Reprints :2021, 2022

ee
This Reprint : 2023

Fr
Published by Isli Kapur, for Dhanpat Rai Publications (P) Ltd.
for
ur
s
ook
Yo

EVERY GENUINE COPY OFTHIS BOOK HAS A HSHF


eB

The title of the book has been Laminated by a High Security Holographic Film (HSHF) to prevent Piracy.

ImportantNote: Buying & Selling pirated books which do not carry HSHF is illegal and a criminal offence
under the Indian Penal Code. This also deprives the author and the publisher of their due earnings for
our
ad

providing you the knowledge through this book. Pirated books are also sold as old editions.
Y

© Author
Re

This book shall not, by way of trade or otherwise bo lent, resold, hired out, or otherwise circulated without the
nd

publisher's prior written consent in any form of binding or cover other than that in which it is published. No
Fi

part of this book may be reproduced or copied in any form or by any means [graphic, electronic or mechanical,
including photocopying, recording, taping, or information retrieval system] or reproduced on any disc, tape,
perforatedmedia or otherinformationstorage device, etc., without the written permission of the author and
publisher. Breach of this condition is liable for legal action.
DISCLAIMER:

Every effort has been made to avoid errors or omissions in this publication. In spite of this, some errors might
have crept in. Any mistake,erroror discrepancynoted may be brought to our notice which shall be taken care
of in the next edition. It is notified that neither the publisher nor the authors or seller will be responsible for any
damage or loss of action to any one, of any kind, in any manner, therefrom.
For binding mistakes, misprints or for missing pages, etc., the publisher's liability is limited to replacement
within one month of purchase by similar edition. All expenses in this connection are to be borne by the
purcha.ser.
All disputes are subject to Delhi jurisdiction only.
Dear Teachers & Students

w
CBSE has introduced Applied Mathematics (Code 241) as an academic elective at Senior
Secondary Level from Academic Session 2020-21 and onwards. This book of Applied
Mathematics for Class XI has been written strictly as per the latest syllabus prescribed by

e
CBSE.

e
or
r
Entire text has been divided into 30 chapters which have been divided into two volumes
for convenient handling. Volume-I consists of chapters 1-17 aiid Volume-II consists of

F
chapters 18-30. In each chapter, all concepts have been discussed in detail in a lucid manner

oF
ul
and have been explained with suitable illustrations and illustrative examples.
Some unique features of the book are:

rs
— Detailed theory with Illustrations

ko
— Algorithmic approach
— Large number of graded Illustrative Examples of
o
— Large number of unsolved Exercises
Y
rB

— Large number of Multiple Choice Questions (MCQs), Very Short Answer Questions
eY

(VSAQs) and Fill in the Blanks Questions (FBQs).


1 avail this opportunity to convey my sincere thanks to Sh. Ish Kapur of Dhanpat Rai
u

Publications (P) Ltd for his painstaking and sincere efforts in bringing out two volumes of
this book in such an excellent form.
d
o
ad
in

Although every effort has been made to keep this book error free, inspite of this some printing
errors might have crept in. I would be extremely thankful if the same are brought to my notice
Re

through email to rdsmath.feedback@gmail.com or by post.


F

With my Best Wishes


Dr. R.D. SHARMA
APPLIED MATHEMATICS - XI (Vol. I) CONTENTS

Preface to the First Edition {Hi)

w
1. NUMBERS 1.1-1.36

2. INDICES 2.1-2.26

e
ro
re
3. LOGARITHMS 3.1-3.36

F
4. APPLICATIONS OF LOGARITHMS 4.1-4.17

uFl
5. QUANTIFICATION AND NUMERICAL APPLICATIONS 5.1-5.94

sr
ko
6.1-6.42
o
6. SETS of
7. RELATIONS 7.1-7.48
o
Y
erB

8. ARITHMETIC PROGRESSIONS 8.1-8.49


uY

9. GEOMETRIC PROGRESSIONS 9.1-9.50

10. PERMUTATIONS 10.1-10.50


ad
do
in

11. COMBINATIONS 11.1-11.28


Re

STRAIGHT LINES 12.1-12.61


F

12.

13. CIRCLE 13.1-13.22

14. PARABOLA 14.1-14.8

15. FUNCTIONS 15.1-15.45

16. LIMITS 16.1-16.40

17. CONTINUTY 17.1-17.23


APPLIED MATHEMATICS-XI (Vol. 1) CONTENTS IN DETAIL

Preface to the First Edition m

1. NUMBERS 1.1-1.36

1.1 Prime Numbers 1.1

1.2 Congruences 1.10

1.3 Cryptography 1.13

1.22
1.4 The Binary Number System
1.5 Binary Arithmetic 1.24

w
1.6 Need for Complex Numbers 1.28

2. INDICES 2.1-2.26

F lo
2.1 Introduction 2.1

2.2 Integral Exponents of a Real Number 2.1

ee
2.3 Laws of Integral Exponents 2.2

Fr
2.4 Rational Exponents of a Real Number 2.3

2.5 Laws of Rational Exponents 2.5

3. LOGARITHMS
for 3.1-3.36
ur
3.1 Introduction 3.1

3.2 Definition 3.1


s
ook
Yo

3.3 Fundamental Laws of Logarithms 3.3


3.4 Some useful Results 3.5
eB

3.5 Systems of Logarighms 3.6


3.6 Standard form of Decimal 3.19

3.7 Characteristic and Mantissa of a Logarithm . 3.21


r
ad
ou

3.8 Finding log n when n is Given 3.24

3.9 Antilogarithm 3.26


Y

3.10 Use of Logarithm in Numerical Calculations 3.30


Re
nd

4. APPLICATIONS OF LOGARITHMS 4.1-4.17

4.1 Compound Interest 4.1


Fi

4.2 Population Growth 4.9

4.3 Depreciation 4.15

5. QUANTIFICATION AND NUMERICAL APPLICATIONS 5.1-5.94

5.1 Introduction 5.1

5.2 Average 5.1


5.3 Calender 5.9

5.4 Clocks 5.15

5.5 Time and Work 5,20

5.6 Time and Distance 5.33

5.7 Mensuration of Plane Figures 5.39

5.8 Mensuration of Solids 5.73

(V)
(Vi) Contents

6. SETS 6.1-6.42
6.1 Sets 6.1

6.2 Description of a Set 6.2

6.3 Types of Sets 6.7


6.4 Subsets 6.10
6.5 Universal Set 6.12
6.6 Power Set 6.12

6.7 Venn Diagrams 6.17

6.8 Operations on Sets 6.18

6.9 Laws of algebra of Sets 6.21

6.10 Some important results on number of elements in Sets 6.25

ow
7. RELATIONS 7.1-7.48
7.1 Introduction 7.1

7.2 Ordered Pairs 7.1

e
7.3 Cartesian Product of Sets 7.1

re
7.4 Some useful results
7.5 Relations

rFl 7.9
7.10

F
7.6 Types of Relations 7.21

8. ARITHMETIC PROGRESSIONS 8.1-8.49

r
ou
8.1 Sequence fo 8.1
ks
8.2 Arithmetic Progression (A.P.) 8.2

8.3 General term of an A.P 8.5


oo

8.4 Selection of terms in an A.P 8.11


8.5 Sum to n terms of an A.P 8.13
Y
eB

8.6 Properties of Arithmetic Progressions 8.31

8.7 Insertion of Arithmetic Means 8.40

8.8 Applications of A.P 8.43


r
ou
ad
Y

9. GEOMETRIC PROGRESSIONS 9.1-9.50

9.1 Geometric Progression 9.1

9.2 General term of a G.P 9.1


d

9.3 Selection of terms in G.P 9.10


Re
in

9.4 Sum of the terms of a G.P 9.15


F

9.5 Sum of an infinite G.P 9.28

9.6 Properties of Geometric Progressions 9.35

9.7 Insertion of Geometric means between two given numbers 9.40

10. PERMUTATIONS 10.1-10.50

10.1 The Factorial 10.1

10.2 Fundamental principles of counting 10.5


10.3 Permutations 10.17
10.4 Permutations under certain conditions 10.29

10.5 Permutations of objects not all distinct 10.36


10.6 Circular Permutations 10.44

11. COMBINATIONS 11.1-11.28


11.1 Introduction 11.1
Contents (vii)

11.2 Combinations 11.1


11.2
11.3 Properties of ”C^ or, C(»i, r)
11.8
11.4 Practical problems on Combinations
11.18
11.5 Mixed problems on Permutations and Combinations
12. STRAIGHT LINES 12.1-12.61
12.1
12.1 Definition of a Straight Line
12.1
12.2 Slope (Gradient) of a Line
12.3
12.3 Angle between two Lines
12.9
12.4 Intercepts of a Line on the Axes
12.10
12.5 Equations of Lines Parallel to the Coordinate Axes

w
12.6 Different forms of the equation of a Straight Line 12.12
12.34
12.7 Transformation of general equation in different standard forms ...
12.8 Point of Intersection of two Lines 12.36

12.9 Condition of Concurrency of three Lines 12.40

o
12.41
12.10 Lines parallel and perpendicular to a given Line

e
12.11 Angle between two Straight Lines when their Equations are given 12.46

re
12.12 Distance of a Point from a Line

rFl 12.49

F
12.13 Distance between Parallel Lines 12.53

13. CIRCLE 13.1-13.22

r
13.1 Definition 13.1
ou
13.2 Standard Equation of a Circle fo 13.1
ks
13.3 Some Particular Cases 13.2

13.4 General Equation of a Circle ... 13.12


oo

13.5 Diameter form of a Circle 13.18


Y
eB

14. PARABOLA 14.1-14.8


14.1 The Parabola 14.1

14.2 Equation of the Parabola in its Standard Form ...


14.2
r
ou

15. FUNCTIONS 15.1-15.45


Y
ad

15.1 Introduction 15.1

15.2 Function as a special kind of Relation 15.1


d

15.3 Function as a correspondence 15.2


in

15.4
Re

15.4 Equal Functions


15.5 Real Functions 15.9
F

15.6 Domain of Real Functions 15.12

15.7 Range of Real Functions 15.14

15.8 Some Standard Real Functions and their Graphs 15.18


15.29
15.9 Operations on Real Functions
16. LIMITS 16.1-16.40
16.1
16.1 Informal approach to Limit
16.2 Evaluation of left hand and right hand Limits 16.4

16.3 Difference between the Value of a Function at a Point and the Limit
at that Point 16.13
16.14
16.4 The Algebra of Limits
16.5 Indeterminate forms and evaluation of Limits 16.15
16.16
16.6 Evaluation of algebraic Limits
16.33
16.7 Evaluation of exponential and logarithmic Limits
(viii) Contents

17. CONTINUTY 17.1-17.23

17.1 Intutive notion of continuity 17.1

17.2 Continuity at a point 17.3

17.3 Algebra of continuous functions.... 17.4

17.4 Continuity on an interval 17.13

17.5 Properties of continuous functions 17.14

o w e
re
rFl
F
r
ou
fo
ks
oo
Y
r eB
ou
Y
ad
d
in
Re
F
CHAPTER d

NUMBERS

1.1 PRIME NUMBERS

In earlier classes, we have learnt about divisibility, divisors (factors), greatest common divisor
(GCD), least common multiple (LCM) of integers. We have also learnt about Euclidean
algorithm and its applications. We know that every integer is divisible by 1 and by the integer

w
itself. If we do not take these obvious divisors into consideration, we find that some integers like
36,54,105 have a number of divisors while others like 23,31,67, 71 have no divisors at all. The

F lo
former are called composite iniinbers and the latter prime }iumbers.
PRIME NUMBERS An integer p > 1 is called n prime number if it has no divisor except 1 ami the number

ee
itself.

Fr
Many a time we use the word prime for a prime number.
2,3,5,7,11,13,17,19,23,29 is the list of first ten prime numbers and the next 10 primes are: 31,
37,41,43, 47, 53, 59, 61, 67 and 71. for
ur
82589933
NOTE 1 is not considered a prime and 2 is the first prime number. 2 -1 is the largest prime
found till date. It has 24862048 digits in decimal system.
s
ook

the n'^’ prime by p„. Thus, p^ =l,p2= 3,


Yo

A prime number is usually denoted by the letter p and


P3 =5, p4 =7,p^ =11, =13, p7 =17 and so on.
eB

COMPOSITE NUMBERS An integer n>l is called a composite integer if it is not prime.


Thus, a composite number is one which has at least one divisor other than 1 and the niunber
r

itself.
ou
ad

We find that 12 has divisors 2,3,4,6 other than 1 and 12. So, it is a composite number. Other than
1 and 87, there are two more divisors viz. 3 and 29 of 87. Hence, it is a composite number.
Y

RELATIVELY PRIME NUMBERS Two positive integers a and b are said to be relatively prime to each other
Re
nd

or co-prime iff their ged {a, b) =\ i.e. they do not have a common factor other than 1.
Fi

We observe (25, 37) = 1, so 25 and 37 are relatively prime to each other.


Also, 16 and 35 are relatively prime to each other, because (16, 35) = 1.
PROPER DIVISORS Divisors of a number other than the number itself are called proper divisors of the
numbers.

Proper divisors of 12 are 1, 2, 3, 4 and 6. There is only one proper divisor of a prime viz 1.
1.1.1 PROPERTIES OF PRIME NUMBERS

A composite number has at least one divisor other than 1 and the number itself the smallest of
these divisors is a prime number. This and other such properties of prime numbers are stated
and proved as theorems given below;
THEOREM 1 The smallest divisor (other than 1) of a composite number is a prime number.
PROOF Let a be any given composite number. Then a has at least one divisor other than 1 and
a. Therefore, it has a smallest divisor. Let this be d. If d is composite, then d has a divisor frj other
than 1 and d.
1.2 APPLIED MATHEMATICS-XI

Now, d I (1 and d-^\d=> d^\n


Thus, d-[ <d and divides a. This contradicts the supposition that d is the smallest divisor of/?.
Hence, d cannot be composite. Consequently, dis a prime. Q.E.D.
Divisors other than 1 of 24 are 2,3,4,6,12,24. The smallest of these is 2 which is a prime.
THEOREM 2 Let p he a prime number mid n be any given integer. Then, either (p, a) = 1 or p \ a.
PROOF Since p is prime, it has two divisors 1 and p itself.
When 1 is considered as a divisor of p, then (p, /?) = 1
When p is considered as a divisor of p, then {p, /?) = p.
Thus, {p, a) - \ or (p , /?) = p.
If (p, a)=\, then the theorem is proved.
If (p, a) =p, then p I a.
Hence, either (p, /?) = 1 or p | /?. Q.E.D.

w
THEOREM 3 Let pbe a prime number and a, b be integers. Then,
p\ab^ p\n or p\b

Theorem 2, we obtain
(p, /?) = 1 and (p, b) = \
F lo
PROOF Let us assume that p does not divide any one of the two numbers a and b . Then, by

...(i)

ee
[v(.V,I/)=l=>(T,I/2)=(Ar, 2)]

Fr
Now, (p,/?)=! => (p,/7l?) =(p, 1?)
=> (p, nb) = 1 for [Using (i)l
=> p ^ ah, which is a contradiction.
and. ip,b)=l ^ip,nb)={p,a) [●.●(.Y,i/)=l=>(.v,y2)=(T,2)]
r
=> (p, ah) = 1 [Using (i)]
You
s

3^ p / ab, which is a contradiction.


ook

Thus, our supposition that p does not divide any of the two numbers a and b leads us to a
eB

contradiction. Hence, our supposition is wrong.


Consequently,p\n or p\b. Q.E.D.
ALITER If possible, let p | ci, then we have to prove that p j b.
our
ad

Now,
p / n=>{p,a)=l [●.● p is prime]
dY

^ There exist integers, x, y such that px + ay =1


Re

=> pbx + aby = b [Multiplying both sides by b]


Fin

Now,
p\nb=> p I aby
Clearly, p | pbx.
p\ pbx + aby^ p\b.
Similarly, it can be prove that if p ^ b, then p | a.
Hence, p | /?/? => p | /? or p 11?. Q.E.D.
Following theorem is the generalization of the above theorem.
THEOREM 4 Let p be a prime number and Aj, /?2,..., be integers. // p divides the product
a-], 02, Of., then prove that p divides at least one of the integers a^, 02, ■■■, aj^.
PROOF Left as an exercise.

REMARK Replacing a^, 02, —, Of. by a, we obtain


p I => p I /?
where p is prime and a is any integer.
NUMBERS 1.3

THEOREM 5 Let a, b, c be three positizv integers such that (c, a) = 1 ami c \ ab, then c \ b.
PROOF Since (c, o) = 1 i.e. c and a are relatively prime. Therefore, there exists integers x, y such
that

cx + ay = 1. b = bcx + aby
It is given that c | ab. Therefore, c \ aby. Clearly, c \ bcx
c I aby + bcx ^ c\b [Using (i)]
Q.E.D.
THEOREM 6 Let a, b be two positive integers. If there exist integers x, y such that ax + by =1. Prove that
a, b are relatively prime i.e. (a, b) =1.
PROOF Let cl be the gcd of a and b i.e. li = {a, b). Then,
d| a and d\ b
=> d I ax and d j by

w
^ d I ax + by
d\l
d =l

F lo
Hence, (rt, b) =1.
Q.E.D.

ee
THEOREM 7 Let a. b, c be positive integers such that {a, b) =1 and {a, c) =1. Then, prove that

Fr
((?, be) =1.
PROOF It is given that for
^ There exist .Yj, i/| € Z such that +bi/i = 1.
ur
{a, c) = 1 ^ There exist X2, >/2 ^ ^ such that ax2 + ci/2 = L
Now,
s
ook
Yo

rt.Yj + h/j = 1 and ax2 + cj/2 = I


{i7.Y;i+I)l/l) (fl.Y2 + Cl/2)=l
eB

a {ax-^X2 + cx-i 1/2 + hx2y-[) + be {y^ 1J2) = 1


ax + bey = 1, wherex = axiX2 + cxii/2 + l’X2yi e 2 and 1/ = yij/2 ^ ^
{a, be) =1
r
ad
ou

Q.E.D.
THEOREM 8 (FUNDAMENTAL THEOREM OF ARITHMETIC) Every composite number can be represented
Y

as the product of prime factors in one and only one zuay.


Re

Every integer n>l can be expressed uniquely in the form


nd

THEOREM 9 (Unique Factorisaion Theorem)


-Pk
Fi

where p^, p2, Pk are primes such that p^ < p2 < — < Pk and a^,a2, are positive integers.
THEOREM 10 Let n be a positive integer. If n is not dhnsible by any prime less than or equal to fn, then n
is prime.
PROOF If possible let n be not a prime number. Then, by Theorem 9,
n=p,-lp2^2 -Pk %
where ;?●), p2,..., pi^ are primes such that P\<P2 < — < Pk a^, 02,a}, are positive integers.
Now,
«1 172 ak
n=p] *p2 ... pf,

n > p-i p2
It is given that n is not divisible by any prime less than or equal to -Jn and pi, P2 are primes
dividing n.
Pi > fn and p2>'Jn
1.4 APPLIED MATHEMATICS-XI

=> p-^p2>i^y/n
=> P1P2 > n or, n < pi p2
This is impossible as n > p]p2- Hence, n is prime. Q.E.D.
THEOREM 11 If n is a composite number, then the smallest positive divisor ofn is less than or equal to
1^.
i’RQOF Let a be the smallest positive divisor ofn. Then, there exist a positive integer & such that
n = ab, where 1 <a <b.
Now, a<b=^ aa <ab:^ <ab=> a^ <n=> a <-Jn
Hence, the smallest positive divisor of n is less than or equal to -Jn. Q.E.D.
THEOREM 12 There exist infinitely many primes.
PROOF If possible, let there exist only a finite number of primes namely P\> P2- Pk

w
ascending order.
Consider the number n = p^p2 ... +1. Clearly, n > pf..
If n is a prime, then it is clear that there exists a prime greater than pj^.

Flo
So, let H be a composite number. Then,
n=piP2 ...Pk+^

e
^ Whenn is divided by any of the primes p\, P2, Pk> obtain 1 as the remainder

re
^ n is not divisible by any one of the primes pj, P2, Pf;

F
But, n being composite, must be divisible by a prime. Consequently, there exist a prime greater
than pi^ that divides n.
ur
or
Thus, in either case there exists a prime greater thanp^. But, this contradicts our assumption that
sf
there are only finite number of primes. Therefore, our assumption is wrong. Hence, there are
infinitely many primes. Q.E.D.
k
Yo

1.1.2 COMPLEXITY IN FACTORIZATION OF LARGE NUMBERS


oo

The fundamental theorem of arithmetic states that every positive integer can be expressed as the
eB

product of prime numbers. The process of expressing a number as the product of prime factors
is known as its prime factorization. In earlier classes, we have learnt about methods of prime
factorization of integers which are convenient only for small integers. If the integer is
ur

sufficiently large, no efficient integer factorization algorithm is known. Researchers are still
ad
Yo

working on factorization of large numbers. In 2019, Fabrice Boudot with his teammates factored
a 240-digit number by utilising approximately 900 core-years computing power. Thus, even
computers take inordinate amount of time in factorizing integers which are a few hundred
d
Re

digits long. Moreover, the number of operations required to perform factorization increases
in

drastically if the integer to be factored happens to be a large semi-prime (product of two large
F

prime integers). Consider two prime integers 2359603 and 195359. The product of these two
primes is the semi-prime 460969682477. To factorize this integer, the computer has to try
factorization with every integer less than or equal to the integer nearest to its square root i.e.
678947. For this the computer has to perform approximately 700,000 operations. If the number to
be factored is 400 digits, the square root of it will be about 200 digits long which means the
number of possibilities for the computer to compute is 10^*^^. Assuming a computer would test
one million factorization per second. The number of factorizations carried out by the computer
in the life time of the universe, which is estimated to be 10^® seconds, could be 10^^. So even
universe lifetime is not sufficient for a computer to factorize a 400 digits long Semi-prime
number.

1.1.3 EULER'S TOTIENT FUNCTION

In 1763, Leonard Euler, a Swiss mathematician, introduced a function which has deep
relationship to prime numbers. This function is known as Euler's totient function or phi (4»)
function and is defined as follows.
NUMBERS 1.5

EULER’S TOTIENT FUNCTION The totient function ^{n) ofa positive integer u greater than 1 is defined to
be the number of positive integers less than n that are co-prime to n.
i.e.
(j) (n) = Number of positive integers less than n and co-prime to n.
For example.
(f) (2) = Number of positive integers less than 2 and co-prime to 2
= 1 [●.● 1 is co-prime to 2]
(|) (3) = Number of positive integers less than 3 and co-prime to 3
= 2 [●-● 1,2 are co-prime to 3]
m =2 [●.● 1, 3 are less than 4 and co-prime to it]
4)(5) =4 1,2,3,4 are less than 5 and co-prime to it]

ow
m =2 [●.● 1, 5 are less than 6 and co-prime to it]
4.(7) =6 [●.● 1, 2,3,4,5,6, are less than 7 and co-prime to it]
4.(15) =8 [●.● 1,2,4,7, 8,11,13,14 are less than 5 and co-prime to it]
REMARK 4K1) is defined as 1 i.e. 4> (1) = 1-

e
Fl
re
Following are some useful properties of Euler's totient function;

F
PROPERTY 1 If p is a prime number, then 4>(p) =p -1.
PROPERTY 2 ]fm and n are co-prime, then 4> (mn) = <}) (m) 4> («)●
ur
PROPERTY 3 If n = p*, where p is prime, then 4> (») = p*’
1
or
sf
1 —
PJ
k
Yo

PROPERTY 4 If » = p/^P2*^2 Pr^’' is the prime factorisation of n, then


oo

r1 1
\ /
B

1 1
4)(n)=Ji 1 1
e

Pi / V P2 Pr)
ur

PROPERTY 5 If p and q are prime, then


ad
Yo

^ipq)={p-l){q-l)
ILLUSTRATION Evaluate the following:
(i) 4>(40) (ii) 4.(300) (iii) 4) (378) (iv) 4>(625) (v) 4>(35)
d
Re

SOLUTION (i) We have, 40 = 2^x5


in

1V 1^ 1 4
4>(40) =4.(2^x5) = 40 1-
F

1- = 40x-x- = 16
2A 5J 2 5

(ii) The prime factorisation of 300 is: 300 = 2^ x 3x5^


IV n 1 1 2 4
4.(300) = 300 1 — 1- 1- = 300 X — X—X — =80
V 2 }\ 3 )\ 5) 2 3 5

(iii) The prime factorisation of 378 is: 378 = 2^ x 3^ x 7


\ /
f n 1 1 2 6
4.(378) = 378 1- 1 — 1- - 378 X —X —X — =108
l 2 3A 7 2 3 7

(iv) We have, 625 = S'*


4.(625) =4> (5*) =5* 1 — = 500
5)
(v) 4.(35)=4.(5x7)=(5-l)(7-l)=24
1.6 APPLIED MATHEMATICS-XI

1.1.4 THE SIEVE OF ERATOSTHENES

The Sieve of Eratosthenes is a method for finding all prime numbers upto a given positive
integer n (say). In this method we construct a table of primes not exceeding n by using
Theorem 10. In order to explain the method, let us take n = 100 . The procedure is as follows:

X ® X ® X X X
X X X
X X X X X
X X X X X X X X

X X X X X X X
X X X X X X X
X M M X X X X

w
X X X X X X

F lo
X X X X
M X X X X X

STEP I Writedown all integersfrom 1 ton (=100) in the form of a rectangular table as shown below.

e
Fre
STEP II 2 is the first number in the table ami it is prime also. So, we encircle it and strike out all larger
multiple of 2. The integers thus deleted are 4, 6, 8,10,12,..., 98,100.
for
STEP lit The first integer greater than 2 which now remains undeleted in the table is 3. Encircle 3 and
strike out from the remaining integers in the table ivhich are larger multiples of 3. This zoay zve
r
strike out 9,15,21,21
You

STEP IV The first integer greater than 3 zuhich remain undeleted is 5. Encircle 5 and strike out from the
s
ook

remaining undeleted integers ivhich are larger multiples of 5. The integers, 25, 35,55, 65, 85
atid 95 are deleted in this step.
eB

STEPV The first integer greater than 5 which remains undeleted is 1. Encircle 1 and strike out from the
remaining undeleted integers which are larger multiples of 7. The integers 49, 77, and 91 are
deleted in this step.
our
ad

STEP VI In the table the first integer ivhich remains undeleted nozo is 11 zuhich is greater than
= 10. Hence, the integers in the table which remain after step V are not divisible by any
prime less than VlOO. It follows that they are all primes. Encircle all of the them. All encircled
dY
Re

integers are primes less than 100. These are:


2, 3, 5, 7,11,13,17,19, 23, 29, 31, 37, 41, 43,47, 53, 59, 61, 67, 71, 73, 79, 83, 89, 97
Fin

RAMANUJAN PRIMES The n^^‘ Ramanujan prime is the least integer for which there are at least n
primes in [.v / 2, x]for all x > R„.
The first five Ramanujan primes are 2,11,17, 29 and 41.
i.e.
=2, ^2 =11-^3 =17, ^4 =29 and R5 =41.
R^=2=> There is at least one prime in the interval [.t / 2, .v] for all .v > 2.
For .Y = 2, we find that prime 2 is in the interval [1, 2j.
For ,Y = 7.4, we find that the interval [3.7, 7.4] contains primes 5 and 7.
R2 = 11 => There are at least 2 primes in the interval [.y / 2, x] for all .v > 11,
For Y =12.8, the interval [6.4,12.8] contains primes 7 and 11.
Theorem 9 (page 1.3) suggests the following algorithm to check whether a given positive
integer is prime or composite.
ALGORITHM
STEP I Obtahi the number n (say).
NUMBERS 1.7

STEP II Fund the greatest integer less than or equal to 4n i.e. [V»]./;i other words integral part oj
Let it he ni.

STEP III Find all primes less than or equal to m.


STEP IV If n is divisible by any one of the primes obtained in step III, then it is a composite number.
Otherivise, it is a prime number.
Following examples will illustrate the above procedure.
ILLUSTRATIVE EXAMPLES

EXAMPLE 1
Which of the following numbers is a prime number?
(i) 667 (ii) 1003 (iii) 73271 (iv) 503
SOLLmON (i) Let n = 667. Then, m = [V667 ] = Integral part of V667 =25.
Prime integers less than or equal to m = 25 are 2,3, 5, 7, 11,13,17,19, 23.

low
We find that 667 = 23 x 29. Therefore, 231 667.
Thus, 667 is divisible by a prime less than or equal to 25. Hence, it is not prime,
(ii) Let n = 1003. Then, m = [v^] = [Vl003] = 31
Prime numbers less than or equal to 31 are 2,3, 5, 7,11,13,17,19,23,29 and 31.
We find that 1003 = 17 x 59. Therefore, 1711003. Hence, 1003, is not a prime number,

ee
F
(iii) Let n = 73271. Then, m = [Vn] = [V73271] = 270.

Fr
Prime numbers less than or equal to 270 are: 2,3,5, 7,11,13,17,19,23,29,31,37,41,...
Clearly, 73271 = 11 x 6661. Therefore, 11173271 for
ur
Hence, it is a composite number,
(iv) Let n = 503. Then, m = (Integer part of 4n = V503) = 22.
ks

Prime numbers less than or equal to/?/are: 2,3,5,7,11,13,17,19. None of these primes divides
Yo
oo

503. Hence, 503 is a prime.


eB

EXAMPLE 2 Let p be a prime number and N is the product of all prime numbers less than or equal to p.
Examine whether N +1 is prime or not.
SOLUTION If p = 2, then N = 2 and so N +1 =3, which is prime.
r
ou

If p = 3, then N =2x 3 =6 and so N +1 = 7, which is prime.


ad

If p =5, then N=2x3x5 = 30 and so N +1 = 31, which is also prime.


Y

For p=7, N = 2x3x5x7 = 210 and so N +1 =211, which is also prime.


nd
Re

For p=ll, N=2x3x5x7xll =2310 and so N + 1 =2311, which is also prime.


But, for p=13, N=2x3x5x7xllxl3 = 30030 and so N +1 = 30031 =59x509, which is not
Fi

prime. Hence, N +1 is not always prime.


EXAMPLE 3 If a and b are positive integers such that [a^ -b"^) is a prime number. Show that
a ^-b^ =« + &.

SOLUTION It is given that a and b are positive integers. Therefore, a> \ and b >1=> a + b >2
Now, (a^-b^) =(a-b){a + b) -(i)
^ (fl -b) I {a^ -b^) and (a + b) \ {a^ -b~)
^ {a-b) and {a + b) are divisors of {a^ -b^).
But, {a -b ) is prime. Therefore, its positive divisors are 1 and (a -b ) itself.
fl-i>=l Ivf7 + 1?>2]
Substituting // = 1 in (i), we obtain
a^-b^ =a + b.
1.8 APPLIED MATHEMATICS-XI

EXAMPLE 4 Prove that for any integer n > 1, the number +1 is not prime.
SOLUTION n^ +n"^ +1 = ]? +n^+n^ +1
= {n^ +71^ +n^)-{n^ +11^ +n) + (n^ +n + l)
= n^{n^ + n +1) -n (n^ +?? + !)+ (n^ +n +1)
= {n^ + « + l) (n^ -FZ + 1)
It is given thatn > 1. Therefore, n +n + l > 3 and n -n + l>l.
So,7-P +n"^ +\ ={n^ +n +1) (n^ -n +1)
=> +1 is product of two integers greater than 1.
Hence, ff^ + ff^ +1 is not a prime number.
EXAMPLE 5 Find all positive U7tegers a, bfor zvhich a^ + Ab'^ is prune.
SOLUTION ff^ + 4!f^ =(ff^ + 4/f^ +

w
= (a^+2b^)-{2abf

F lo
= (fF^ + 2b^-2ab) {a^ +2b^ + 2ab)
= -2ab'^ + b) + b^\ \{a^ + 2ab +b^) + b^]

ee
= {ia-bf+b~\\{a + bf+b^}

Fr
It is given that a, b are positive integers. Therefore,
(7 >1 and^F >1 => \{a + b)^ +/f^} >1
for
ur
Since «“* + 4&"* = [{a-by +b^} {{a + b) +b^} is prime. Therefore, its factors are 1 and the number
s

itself. But, {a + b)^ +b'^ >1.


ook
Yo

{a-b)^ +b^ =1^ a-b = 0 andb =1=> a =b=l.


eB

Hence, a = l,b =1.

EXAMPLE 6 Ifp and q are primes and x -px + q = 0has distmct positive mtegral roots, find p and q.
r
ou
ad

SOLUTION Let x-^ and X2 with x^ < X2 be two distinct positive integer roots of .y -px + q = 0.
Then,
Y

.Vi + X2=p and x^ A'2 = q


Now,
Re
nd

q = XiX2
Fi

^ Xi, X2 are factors (or divisors) of q.


But, q is prime. So, its factors are 1 and q itself.
[●●● ^2 > ^1]
=> X2=q [Putting Y| = 1 in ^2 = F/]
Now,
p=Xi+X2
p=l + q
p and q are two consecutive primes such that p> q.
p = 2 and q = 3.
a
EXAMPLE? Let a, b, c be non-zero mtegers, a ^ c, such that - = -. Prove that a^ +b^ + c^ cannot
c c^+b^
be prime.
SOLUTION We have
a a^ +b^
c c^+b'^
NUMBERS 1.9

ac'^ + ab^ =a^c + I'P'c


{b'^a-b'^c) + {ac^ -a^c) = 0
b^{a - c) - (1C (i7 - c) = 0
-ac) ((7- c) = 0
b'^-ac = Q [●.● a^c]
b2 = ac

<7^ + + rtc + c“ + 0 = (rt“ + 77C + c^) + {ac-b~) [Vb2 = 77C . ●. ac-b^ =0]
= {a^ + 2ac + c^)-b^ = (((? + c)^ -b^l
= (<7 + c + b) (a + c-b)
Since, a, b, c are non-zero integers. Therefore, i7 + b + c > 3.

w
If fl + b + c is prime, then only four cases are possible.
CASE I ]Nhena + c-b=\

In this case, (i) gives

Flo
a + c + b =a^ +b^ + c^
a^ +b^ + c^ =a + c + {n + c -\)

ee
[●.●77 + C-b=l .'. 77 + C-l=b]
+ b^ + c^ - 2 (a+ c) + I = 0

Fr
{a^ -2a + l)+b^ +{c^ -2c + 1) =1
{a-lf+b^+ic-lf =1 for
ur
«-l =0, c-l =0 and b =1
s
<7 =1, c =1 and b=1^77 = c= b= l
k
Yo

This is a contradiction as i7 c is given.


oo

CASE 11 Wbt’;jfl + b + c=l


eB

In this case, (i) gives


(7^ +b^ + C^ =77 + 'L-b
a^ + b^ + c'^ = n + c - {1 - a - c)
r

[V7? + b + C=l .●.b=l-77-c]


ou
ad

a'^ -2a+b^ + c^ -2c + l =0


Y

(n-l)2+b^ + (c-l)^=l
77-1 =0, b =1, c-1 =0
Re
nd

77=l,b=l,C=l=>77=C=b=l
Fi

This again a contradiction as t7 vt c.


CASE III W/7C77 77 + c-b =-l
In this case, (i) gives
77^ +b^ +C^ =-(t7 + c + b)
77^+b^+C^=-77-C-l-77-C [●.● 77 + C -b = -1]
(77 + 1)2+b^+(C+1)2 =1
77 + 1=0, b=l,c + l= 0 ^77 = c= -l
This is also a contradiction to t? 5^ c.
Similarly, t? + c + b = 1 leads us to a contradiction.
*^ 7 9
Hence, a~ +b + c cannot be prime.
EXERCISE 1.1

!. Show that the following numbers are prime numbers:


(i) 131 (ii) 167 (iii) 191 (iv) 199
(v) 487 (vi) 523 (vii) 647 (viii) 709
1.10 APPLIED MATHEMATICS-XI

2. Show that the following numbers are not prime numbers:


(i) 786 (ii) 1029 (iii) 1113 (iv) 1415
3. Which of the following are primes?:
(i) 313 (ii) 614 (iii) 421 (iv) 751 (v) 1007
Find all positive integers n for which 3/i -4, 4/i -5 and 5n - 3 are all prime numbers.
Using sieve of Eratosthenes find all primes less than or equal to 205.
6. Find the value of Euler's totient function atH =
(i) 40 (ii) 540 (iii) 143 (iv) 667
ANSWERS

ow
3. (i) prime (ii) composite (iii) prime (iv) prime (v) composite
4. »=1 or 2
5. 2,3,5,7,11,13,17,19,23,29,31,37,41,43.47,53,59,61,67,71,73,79 ,83,89,97,101,103,
107,109,113,127,131,137,139,149,151,157,163,167,173,179,181,191, 193,197,199

e
: . (i) 16 (ii) 144 (iii) 120 (iv) 616

re
HINTS TO SELECTED PROBLEMS

Frl
F
4. We find that(3n -4) + (4n-5) +(5n- 3) =12 (n-1), which is an even number. So, at least one
of these three numbers is even. Since An is always even. Therefore, An -5 is always odd. So,
either (3n - 4) is even or (5?i - 3) is even and of course the other is odd.
ou
sor
If (3;i - 4) is even, then 3« - 4 = 2, because only even prime is 2.
Now, 3?i-4 = 2^ 3n = 6 ^n = 2.
If (5« - 3) is even, then 5?i-3=2=>m=1
Hence,« = 1 or « = 2.
kf
oo
1.2 CONGRUENCES
Y

CONGRUENCE Let a, b be two integers and rn be a positive integer other than 1. Then 'a' is said to be
B

congruent to 'b' modulo m ifm divides {a-b) or, {a-b) is divisible by m.


If 'a' is congruent to 'b' modulo m, we write a=b (modulo m) or,a=b (mod m).
re
oY

Thus,
u

a=b (mod m)
ad

o
m\ {a~b)
d

<=>
There exists an integer q such that (rt-b) = mq or, a = b + mq
If 'a' is not congruent to 'b' modulo m, then we write a ^b (mod m).
in
Re

For example:
(i) 37 = 5 (mod 4), because 37 -5 = 32 is divisible by 4
F

(ii) 12 = -9 (mod 7), because12 -(-9) = 21 is divisibleby 7


(iii) 23 5^ - 4 (mod 5), because 23 -(-4) = 27 is not divisible by 5.
(iv) 9 s 29 (mod 4), 9 - 29 = -20 is divisible by 4.
(v) 13 s 29 (mod 4), because13 - 29 = -16 is divisible by 4
(vi) 5 = 29 (mod 4), because5 - 29 = -24 is divisibleby 4.
From (iv) - (vi), we observe that there are infinitely many values of a for which a=b (mod m) for
given b and m.
IILUSTK.S I !●. Find the values ofx such that x =5 (mod 7)
SOLUTION We have.
a: =5 (mod 7)
=>
X -5 is divisible by 7
x-5=7X,XsZ
x=5 + 7X,X^Z
NUMBERS 1.11

x=...,-16,-9,-2^42,19,...
Hence, the set of values of -.cis{..., -16,-9,-2,5,12,19,...}
MODULAR EXPONENTIATION Let a be an integer and m, n be positive integers such thatr^' =b (mod m),
where 0 <b < m. Then, we say that b =(^' (mod ni.)
Now,
b =a" (mod m)
<=> a" sb (mod ni), 0 <b <m
O’
rt” -b is divisible by m, where 0 < < »?
o
b is the remainder when a” is divided by ni.
The remainder when 5 is divided by 4 is 1. Therefore, 1=5 (mod 4).
When 3^ is divided by 8, the remainder is 3. Therefore, 3^ (mod 8) = 3.

w
When 2^ is divided by 10 the remainder is 4. Therefore, 2^ (mod 10) = 4.
1.2.1 SOME IMPORTAN PROPERTIES OF CONGRUENCES

F lo
Following are some useful properties of congruences which are very useful for finding private

e
key in RSA encryption (section 1.3.1).

Fre
PROPERTY 1 Let a, b, c be integers and let m be a positive integer. Then, for
(i) a = b (mod ni) <=> a and b have the same remainderwhen divided by m.
(ii) = 0 (mod m) if a is divisible by m
(iii) rtHrt(mod)»)
r
(iv) (modm) => i) sfl(mod m)
You
oks

(v) a = b (mod;/;) and b = c (mod m)^ a = c (mod m)


eBo

(vi) a = b (mod m) ^ (r? + x) = (1? + x) (mod m) for any integer x


(vii) a ^b (mod»0 => (i?-x) =(1j - .y) (mod for any integer x
(viii) a = b (mod in) => ax = bx (mod m) for any non-zero integer x
ad
our

a b m
(ix) fl=i>(mod;;0^ mod , ifx is a divisor of a, b and m
X XV X J
a bf m

(x) a^b (mod m) o — h mod , ifx is a divisor of a and b and (x, m) = d.


dY
Re

X X V d

PROPERTY 2 Let a,b,c,d be integers and m be a positive integer such that fl=b(mod»i) and
Fin

c = d (mod ni). Then,


(i) a + c = b + d (mod m)
(ii) a-c sb ~d {modm)
(iii) ac = bd (mod m)
(iv) asb + c (mod in) and c = d (mod in) ^ a = b + d (mod in)
(v) as be (mod m) and c sd (mod in) ^ a sbd (mod m)
(vi) i?s!j(modw) sb" (mod in) for any integer n.
(vii) (a^’ b‘1) (mod in) = (a^ mod m) (b'^ mod ?;»)
ILLUSTRATION Evaluate the following:
(i) 3“ (mod 7) (ii) 3^°° (mod 13) (iii) 7^^^ (mod 13)
SOLUTION (i) We find that when 3^ = 9 is divided by 7, the remainder is 2.
3^ H 2 (mod 7)
1.12 APPLIED MATHEMATICS-XI

=> (3^)^ =25(mod7)


3^® = 32 (mod 7)
3^°=4(mod7) [v 32 =4 (mod 7)1
(310)2 _ 42 (jnod7)
3^°s2(mod7) [v 4^=16 ^2 (mod 7)1
(320)3 ^2^ (mod7)
3^=1 (mod 7) [v 2^=8=l(mod7)l
Hence, 3*’" (mod7)=l.
(ii) We find that:
3^ = 3 (mod 13)

w
3^ = 3x3 =9 (mod 13)
=> 3^ = 3x9 = 27 (mod 13)
3^=1 (mod 13)
32500 ^(33)833x 31

F lo
ee
3^ =1 (mod 13)

Fr
(33)833 =(1)833 (mod 13) for
=> 33x833 = 1 (mod 13)
ur
32499 X 3 =lx 3 (mod 13)
s

32500 = 3 (mod 13)


ook
Yo

Hence, 3^^®® (mod 13) = 3.


eB

(iii) We find that


1
7" =7 (mod 13)
our
ad

7^ =7x7 =49 (mod 13)


7^=10 (mod 13) [v 49 =10 (mod 13)1
Y

7 3 =70 (mod 13)


Re

7 3 =5 (mod 13)
nd

[v 70 =5 (mod 13)1
7^ = 35 (mod 13)
Fi

=>

7^ =9 (mod 13) [v 35 = 9 (mod 13)1


7^ =63 (mod 13)
7^ = -2(modl3) [v 63 =-2 (mod 13)1
7^ =-14 (mod 13)
7^ =-l(modl3) [v -14 = -l (mod 13)1
7^58 —^76)43
7^=-l(modl3)
(7V3=(_i)43(jj^q^j^3)
7^5® =-l(modl3)
7^58 =12 (mod 13) [v -1=12 (mod 13)1
NUMBERS 1.13

EXERCISE 1.2

1. Which of the following statements are true?


(i) 35 =7 (mod 4) (ii) 2 = 65 (mod 9) (iii) -9 = -109 (mod 10)
(iv) -12 = -107 (mod 6) (v) 3^ =5 (mod 19) (vi) 2^ = 8 (mod 12)
2. If 120 s 30 (mod in), find the least and greatest values of m.
3. Ux = 7 (mod 5), find the set of values of x.
4. Find the least positive integer x to which 282 is congruent under modulo 11.
5. If n is an odd positive integer, prove that = 1 (mod 8)
ANSWERS
(i),(ii),(iii),(v),(vi) 2.2,90 3. {7,12,17,22,...} 4.7

1.3 CRYPTOGRAPHY

Considering ubiquitous internet/web which stores and moves digitized information across the
world, security of all the sensitive information has gained paramount importance. The study

w
and design of systems that aim to keep information secure in storage and transmission is called
Cryptography. The word cryptography comes from two Greek words krypton (meaning

F lo
hidden) and graphein (meaning “to write"). It is a method of storing and transmitting data in
such form that only those for whom it is intended can read and process it.
Modern Cryptography concerns itself with following 4 objectives:

ee
Fr
a. Confidentiality: The information cannot be understood by anyone for whom it was not
intended,
for
b. Integrity: The information cannot be altered in storage or transit without being detected,
ur
c. Non-repudiation: The creator/sender of the information cannot deny at a later stage his or
her intentions in the creation or transmission of the information,
s
ook

d. Authentication: The sender and receiver can confirm each other's identity and the
Yo

origin/destination of the information.


eB

As a simple example of cryptography consider two friends who have to secretively


communicate “HELLO“to each other. Between the two of them, they decide that every letter in
the message has to be replaced by a letter that is 2 letters ahead of itself. So, the sender
our
ad

transforms “HELLO" which is called message or plaintext into “JGNNQ" which is called
ciphertext. This process is called encryption.
Y

ENCRYPTION The process of converting normal message (plaintext) into something that appears to be
Re

random and meaningless (ciphertext) is called Encryption.


nd

Tlie receiver of ciphertext “JGNNQ" will have to apply inverse translation i.e. replace every
Fi

letter with a letter 2 letters prior to itself to retrieve original message “HELLO". This is called
decryption.
DECRYPTION The process of converting ciphertext into plaintext is called Decryption.
Cryptography

Plaintext Ciphertext Plaintext


Sender Encryption Decryption Receiver
HELLO' JGNNQ' HELLO"

}
Key = 2
I
Key = 2

In the aforementioned example, if the sender wishes to change encryption logic such that every
letter is to be replaced by 4th letter ahead of itself, the same logic will have to be communicated
to receiver also to allow correct decryption.
1.14 APPLIED MATHEMATICS-XI

Every encryption or decryption process encapsulates two key components which are algorithm
and key.
CRYPTOGRAPHY ALGORITHM An encryption or decryption algorithm, commonly known as
cryptography algorithm, is the speciifc sequence of steps that convert either plaintext to ciphertext or
ciphertext to plaintext.
In the aforementioned example, the process of replacing every character with another is the
cryptography algorithm. This algorithm requires a number which specifies the amount of
character displacement. This number which acts as an input to the cryptography algorithm is
called key.
Depending on the nature of the key, cryptography can be classified into following two types:
SYMMETRIC CRYPTOGRAPHY When key used by the senderfor encrypting plaintext and the receiver for
decrypting ciphertext is the same, the process is called symmetric cryptography.
As you would have guessed, the aforementioned example is a symmetric cryptography
algorithm.

w
Such algorithms offer simplicity at the risk of information theft through the loss of key to an
eavesdropping adversary while it is being communicated by sender to receiver.

F lo
ASYMMETRIC CRYPTOGRAPHY When keys used for encryption and decryption are different, although
mathematically related, the process is called Asymmetric Cryptography. It is also called Public Key
Cryptography.

e
Fre
In Asymmetric Cryptography, each sender and receiver has two keys called Public Key and
Private Key. for
PUBLIC KEY Public key, as the name suggests, is known publically and used by senders to encrypt
messages for corresponding receiver.
r
PRIVATE KEY Private key ns the name suggests, is the secret key belonging to receiver of ciphertext and is
You
oks

used to decrypt any message encrypted for the receiver.


The diagram below illustrates the functioning of Asymmetric Cryptography. It shows how the
eBo

same message plaintext "HELLO" gets encrypted into the different ciphertexts based on
intended receiver's public key. It also shows how the individual ciphertext can be decrypted
back to original message using the private key of the intended receiver only.
our
ad

Public key 1 (Receiver 1) Private key 1 (Receiver 1)


dY

Ciphertext Plaintext
Re

X O -o ’Q8W9V’ o HELLO"
Receiver 1
Fin

-j
,S u
(3
El I
c c
o o
Sender ●j:
Cl CL
Ih
U U
X o c (K
<u u D
-j
.S
c. X
Ciphertext Plaintext

-Q ’’M27NX' Q HELLO
Receiver 2

Public key 2 (Receiver 2) Private key 2 (Receiver 2)

The email systems in today’s world uses very complex encryption and decryption standards.
However, if 1 were to dramatically reduce the complexity, I would say that my email-id is the
NUMBERS 1.15

public key which anyone, who wants to send across to me message, can use to encr3^t a
message. When the same encrypted message arrives in my mailbox, I use my password, which
is my private key, to retrieve the original message.
Now that we have learnt about key concepts in Cryptography and also computational difficulty
in factorization of very large semi-primes (Section 1.1.2), let us see how the two come together in
practical use through a popular cryptography algorithm called RSA algorithm. RSA algorithm
was invented in 1978 by three mathematicians Richard Rivest, Adi Shamir and Leonard
Adleman and is named after the initials of these inventors. It is an asymmetric cryptography
algorithm i.e. each sender and receiver has to have a public and a private key which are
mathematically related.
1.3.1 RSA ALGORITHM

The RSA algorithm has the following steps:

w
STEPJ Determine public key (encryption key) by using the follozoing sub-steps:
(i) Choose two prime numbers pnnd qandfind their product to get RSA modulusn = p^q-

F lo
(ii) Compute totient (|)(n) by using the formula:
^(n) = <^ipq) ={p-1) iq-l)

ee
(hi) Find public key exponent e such that e is co-prime to (j)(n) andl <e < (j)(n).

Fr
(iv) Establish public key as public key = (n, e).
STEP II
Determine private key (decryption key) by using the following sub-steps:
(i) Find an integer d such that de = \ (modulo <K^0)-
for
ur
i.e. de-l is divisible by (j)(»)
de-\ (n)for some integer X.
s
ook
Yo

1 + X d(?3)
=> d = for some integer X
eB

Choose a value ofXfor zvhich d is an integer.The value of 'd' so obtained is the private key.
STEP 111
Encrypt message using public key (n, e) by using following sub-steps:
r
ou
ad

(i) Obtain message m.


(ii) Find ciphertext C by using C = (mod n)
Y

STEP IV
Decrypt Ciphertext C using private key'd' by using:message m = C^ (mod n).
Re
nd
Fi

ILLUSTRATIVE EXAMPLES

EXAMPLE 1 Given p = 3 and q =5, find the public key in RSA algorithm.
SOLUTION Using RSA algorithm, we obtain
RSA modulus »=px^J = 3x5=15
The totient (l)(?i) = ^(pq) =(p-l) (q-l) = (3-1) (5-1) =8
Publickey: The public key exponent cisco-prime to <t)(fi) such that 1 <e <<}i(n).Thus,e is co-prime
to 8 and lies between 1 and 8. So, the possible values of e are 3, 5, 7. Any one of these can be
chosen as the value of public key exponent. The public key is the combination of RSA modulus n
and the public key exponent. So, the public key can be any one of the following:
(15, 2), (15, 5) and (15, 7).
EXA.MPI.E 2 Given p = 5 and q = 7, find the public and private key in RSA algorithm.
SOLUTION Using RSA algorithm, we obtain
1.16 APPLIED MATHEMATICS-XI

RSA modulusn ~px q =5x7 = 35


The totieni: =^{pq) ={p-l) {q-1) =(5-1) (7 -1) = 24
Public key: The public key exponent e is co-prime to <|>(/j) such that 1 <e <^{n). Thus, e lies
between 1 and 24 and is co-prime to 24. So, the possible values of e are:
5,7,11,13,17,19, 23
The public key is the combination of RSA modulus n = 35 and the public key exponent e. So, the
public key can be any one of the following:
(35, 5), (35, 7), (35,11), (35,17), (35,19) and (35, 23).
Let us choose (35,11) as public key by taking e = 11 as the public key exponent.
Private Key: Private key d is given by
de =1 (mod (|) (n))
=> lid =1 (mod 24)
=> (lid -1) is divisible by 24.
Clearly, ford =ll,lld-l =120 is is divisible by 24. Hence, Private key = 11.

w
EXAMPLE 3 Given p =11, q =13 and message (in) = 2, find the public, private key and ciphertext.
SOLUTION Using RSA algorithm, we obtain
RSA modulus n=pq=llxl3= 143

values of e are: 7,11,13,17,..., 119


F lo
The totient c|)(m) = ^{pq) ={p-l) (ij-l) =(11 -1) (13-1) =120
for F
ree
Public key: The public key exponent e is co-prime to (j> (n) = 120 such that 1 < e < <j> («) = 120. Such

Let us choose e = 7 as the public key exponent. Thus, we have n = 143 and e = 7.
So, the public key is (», e) = (143,7).
Your

Private Key: Private key d is given by


oks

de si (mod (|)(h))
eBo

=> 7d si (mod 120)


(7d-l) is divisible by 120.
ad

=> 7d-l =120X.for some integer X.


our

=> 7d =120X.4-1 for some integer k.


Clearly, 120X +1 is a multiple of 7 for ^ = 6. For this value of X, d =103. Thus, private key = 103.
Re

Ciphertext: We have, message m = 2,n= 143 and e =11. Ciphertext C is given by


dY

C = nf (mod n)
Fin

=> C=2^^(modl43)
EXAMPLE 4 Given p =13, q =17 and message (m) = 3, find the public key, ciphertext and decrypted
message as an exponent of ciphertext.
SOLUTION We have, p= 13, 7 = 17.
RSA modulus n =px 7 = 13x 17 =221
and. Totient (|)(ji) = (t>(p7) =(p-l) (7-I) =(13-1) (17 -1) =192
Public key: Let us now find the public key exponent e which is co-prime to n = 221 and
1 < e < <f)(n) =192. Clearly, e can take values 5,7,11,13,..., 191. Let us choose e = 7.
The public key is combination of RSA modulus n and public key exponent e.
i.e. Public key = (», e). So, public key is = (221,7).
Private Key: Private key d is given by dc s 1 (mod <|> (?z)).
NUMBERS 1.17

Now,
de =1 (mod ({) («))
7d=l(modl92)
7d-l is divisible by 192.
7d-l =192?^ for some integer X
. 192?.+ 1
d = ,leZ.
7

Clearly, d is an integer for \ = 1. For this value of 'k, we obtain


192x2 + 1
d = =55
7

Public key = 55.


Ciphertext: Ciphertext C is given by C = (mod »).

w
We have, m = 3, = 221 and e = 7
Ciphertext C = 3^ mod 221

F lo
=> Ciphertext C = 2187 mod 221 = 198 [v 2187 =221x9 + 198]
Decrypted message m is given by m = mod n

ee
Message m = 198^^ mod 221

Fr
EXAMPLE 5
Given message (m) =2, prove that the RSA algorithm generates different ciphertext for
receivers having different public keys. for
SOLUTION Let us assume two receivers with the following values of p and q.
ur
Receiver 1 :p=3 and q=5, Receiver 2 : p = 3 and q=7.
s

Let us now apply RSA algorithm to encrypt message m = 2 for both the receivers.
ook
Yo

Receiver 1: p = 3 and q =5.


RSA modulus n = pi/= 3 X 5 = 15
eB

Totient (j>(n) =^{pq) =(p-l) ((/-!) = (3-1) (5-1) = 8


Public key: Public key exponent e is co-prime to <^{n) and lies between 1 and ^{n). Tliat is e lies
r
ad

between 1 and 8 and is co-prime to 8. So, e can take values 3,5, and 7. Let us choose e =7.
ou

Public key = (;;, f) = (15,7).


Private Key: Private key d is given by de = 1 (mod (|) (n))
Y

Now, de =1 (mod (j) {n})


Re
nd

7d=l (mod 8)
Fi

7d~l is divisible by 8.
7d-l = 8^for some integer X
SX+1
7d = 8X + l => fr =
7

Clearly, d is an integer for X = 6. For this value of X, we obtain fr =7. So, private key = 7.
Let us now encrypt the message in = 2 using public key (15, 7).
Ciphertext C is given by C = mod n
We have, m = 2,n= 15 and e = 7.
Ciphertext C = nf mod n
=> Ciphertext C = 2^ mod 15 = 128 mod 15 = 8
Thus, for Receiver 1, Ciphertext C = 8
Receiver 2: p = 3 and q=7
RSA modulus j? = p7 = 3 X 7 = 21
1.18 APPLIED MATHEMATICS-XI

Totient =^(pq)={p -1) ((j-l) =(3 -1) (7 -1) =12


Public key: Public key exponent e is co-prime to (|)(;i) and lies between 1 and That is e is
co-prime to 12 and 1 < e <12. So, c can take values 5, 7, and 11. Let us choose e =11.
So, Public key = (?i, e) = (21,11).
Private Kei/: Private key d is given by de = 1 (mod (|) («)).
Now,
de =1 (mod (j)(»i))
lid =1 (mod 12)
llri-1 is divisible by 12.
lld-1 =12X.for some integer X
12X + 1

low
d = ,XeZ
11

=> fr=llforA. = 10

So, private key = 11.


Let us now encrypt the message m = 2 with public key (22,11).

ee
F
Ciphertext C which is given by C = mod n

Fr
Now,
C =m^ mod n
for
ur
C = 2^^ mod 21 = 2048 mod 21 =11 [v 2048=21x97+11]
Thus, for Receiver 2, Ciphertext C =11.
ks

' Given p =13, q = 17 use RSA algorithm to encrypt 2 to a new code and then decrypt it to
Yo
oo

get 2.
SOLUTION We have, p=13,q = 17.
eB

RSA modulus n = pry = 13 X17 = 221


Totient ^(n) = <^{pq)={p-l) (q-1) =(13-1) (17 -1) =192
r
ou
ad

Public key: The public key exponent e is co-prime to 4>(/i) =192 and lies between 1 to 192. The
possible values of e are 5,7,11,..., 191. Let us choose e =5. So, the public key = (221,5).
Y

Private Key: Private key d is given by de =1 (mod (|) («))


nd

Now,
Re

de si (mod {[)(»))
Fi

5iisl(modl92)
5d-l is divisible by 192.
5d-l=192X,XeZ
1+192X
d = ,XeZ
5

Clearly, d is an integer for X = 2. For this value of X,d = 77. So, Private key = 77.
Ciphertext: Ciphertext C is given byC =m^ mod n
We have, m = 2 and e = 5

Ciphertext C = m*'’ mod n


^ C =2^ mod 221 = 32 mod 221 = 32

Decryption of ciphertext: The decryption m of the ciphertext C is given by


NUMBERS 1.19

m=C^ (modn)
m = 32^ (mod 221) = (mod 221) = (mod 221)
Now,

iP mod 221 = (2^ x 2^^) mod 221 = (2*^ mod 221) (2^^ mod 221)

Let us now compute the values of 2^^ mod 221 and 2^^ mod 221.
Computation of 2^ mod 221. We find that
2® =256 = 35 mod 221

(2^)^ = 35^ mod 221


216 =1225 mod 221
2^^ =120 mod 221

w
[v 1225 = 221x5 + 120]
(216)2 ^(j20)2 mod 221

F lo
2^2 =14400 mod 221
2^^ = 35 mod 221 [●.● 14400 = 221X 65 + 35 14400 = 35 (mod 221)]

ee
Fr
(2^2)2 =(35)2 mod221
264 =1225 mod ^21 for
ur
2^ =120 mod 221 [v 1225 = 221x5 + 120 .. 1225 =120(mod221)] ...(i)
8
s
Again 2° =256 = 35 mod 221
ook
Yo

2®x2^=35x2^mod221
eB

2^3 =(35 X 32) mod 221


2^3 =1120 mod 221
r
ou
ad

2^^ =15 mod 221 [v 1120 =15 (mod 221)] ...(ii)


From (i) and (ii), we obtain
Y

2^ X 2^3 mod 221 = 120 x 15 mod 221


Re
nd

2^ mod 221 = 1800 mod 221


Fi

iP mod 221 = 32 mod 221 [●.● 1800 = 32 mod 221]

{pPf mod 221 =(32)^ mod 221


m=(32)^ mod 221
m=(2Ymod221
m = 2^mod 221

m=(2^^x2®x2)mod221
m = (2^^ mod 221) (2® mod 221) (2 mod 221)
m = (120 X 35 X 2) mod 221
m = 8400 mod 221
1.20 APPLIED MATHEMATICS-XI

=> m=2 [v 8400= 221X 38+2]


Hence, decrypted value of ciphertext C = 32 is 2.
! I’LE" An under-cover agent and his boss decide to secretively communicate with each other using
encrypted messages. They decide to code alphabets into numbers by taking their alphabetical order and
then encrypt using RSA algorithm. Suppose the under-cover agent wants to say "HELLO" to his boss
whose public key is (221,5). Find the ciphertext received by the boss.
SOLUTION Using the alphabetical order of letters in English alphabet word "HELLO" is coded
as 8-5-12-12-15.

It is given that public key is (221,5). Therefore, n = 221 and ^ = 5. We will have to find ciphertext
for each of the digits 8,5,12,12 and 15.
Ciphertext for m = 8: Ciphertext for m = 8 is 8^ mod n
Now, 8^ mod n = 8^ mod 221 =2^^ mod 221 = (2® ● 2^) mod 221
= (2^ mod 221) (2^ mod 221)

w
= (35 mod 221) (128 mod 221) [See example 6]

F lo
= 35x128 mod 221
= 4480 mod 221
= 60 mod 221 [v 4480=28x221 + 60]

ee
Ciphertext for m = 5: Ciphertext for m = 5 is 5^ mod n

Fr
Now, 5^ mod n =5^ mod 221 = 3125 mod 221 = 31 for [v 3125=221x14+31]

Ciphertext for m = 12: Ciphertext for m = 12 is 12^ mod n


ur
Now, 12^ mod n = 12^ mod 221 = (3 x mod 221 = (3^ mod 221) (2^° mod 221)
s

= (243 mod 221) (1024 mod 221)]


ook
Yo

= (22 mod 221) (140 mod 221) [●.●1024 = 221x4 + 140]


eB

= (22x140) mod 221


= 3080 mod 221
= 207 [●.● 3080=221x13 + 207]
our
ad

Ciphertext for m = 15: Ciphertext for m = 15 is 15^ mod n


Now, 15^ mod n = 15^ mod 221 = (3 x 5)^ mod 221 = (3^ x 5^) mod 221
Y

= (3^ mod 221) (5^ mod 221)


Re
nd

= (243 mod 221) (3125 mod 221)


Fi

= (22 mod 221) (31 mod 221)


= 22x31 mod 221
= 682 mod 221=19 [●.● 682 = 221x 3 + 19]
So, the ciphertext received by agent's boss is 60-31-207-207-19.
EXAMPLE 8 Ravish and Shikha decided to communicate secretively with each other using encrypted
messages. They decided to code letters into numbers by taking their alphabetical order and then encrypt
using RSA algorithm. Suppose Ravish wants to communicate "BAD" to Shikha whose public key is
(33,3). Find the ciphertext received by Shikha. Also, show decryption by Shikha to retrieve original
message.

SOLUTION It is given that Shikha's public key is (33,3). Therefore, n = 33 and e = 3.


Now, n = 33 = 3 X11 = p X where p = 3 and q = 11.
(|)(n)=.l)(p^)=(p-l)(g^l)=(3-l)(ll-l)=20.
Private key d is given hy de=l (mod <}> (n)).
NUMBERS 1.21

Now,
de=l (mod(t)(n))
3d = 1 (mod 20)
3d -1 is divisible by 20.
3d-l=20X,XGZ
^
d =
20X+1
,XeZ
3

Clearly, d=7,X = l. So, private key d=7.


The word "BAD" is coded as 2-1-4.

Now, we shall find ciphertext for each of the digits 2,1 and 4.

w
Encryption:
Ciphertextfor m = 2: Ciphertext for m = 2 is 2^ mod n

Flo
Now, 2^modn = 2^ mod 33 = 8

e
re
Ciphertext for w = 2 is 8.
Ciphertext for m = l: Ciphertext for m = 1 is mod n

F
Now, 1 ^ mod n = 1 ^ mod 33 = 1
ur
r
Ciphertext for m = 1 is 1. fo
Ciphertextfor m = 4: Ciphertext for m = 4 is 4^ mod n
ks
Yo

Now, 4f mod n = 4^ mod 33 = 64 mod 33 = 31


oo

Ciphertext for m = 4 is 31.


B

Thus, Shikha receives 8-1-31 as ciphertext.


re

Decryption: Now, we shall decrypt the ciphertext 8-1-31. For this we will have to decrypt 8,1
and 31 separately.
u
ad

Decryption of C = 8: Decryption of C = 8 is given by m = C mod n


Yo

Now,
m=C^ modn
d
Re
in

m = 8^ mod 33 = 2^^ mod 33 = (2^° x 2^) mod 33


F

m = 2^0 mod 33 x 2^ mod 33


m = i(2^f mod 33) (2 mod 33)
m
= ((-1)^ mod 33) (2 mod 33) [v 32 = -lmod33]
=> m = (1 mod 33) (2 mod 33) = 2 mod 33 = 2
Decryption ofC = 1: Decryption of C = 1 is given by m = mod n
Now,
m=C^ modn => m=l^mod 33=> m = l
Decryption ofC = 31: Decryption of C = 31 is given by m = mod n
Now, m=C^ mod n
=> m = 3l7 mod 33 = (-2)^ mod 33 = - (128 mod 33) = - (-4) = 4.
Thus, decrypted code is 2-1-4 which when coded to letters in English alphabet becomes BAD.
1.22 APPLIED MATHEMATICS-XI

EXERCISE 1.3

1. Given totient cj) (pq) = 120, find RSA modulus.


Find the private key by taking p = 2, q-5 and c = 3.
Givenp = 2, q=ll and message m = 5, find ciphertext.
4. Given public key = (77,13) and plaintext/message m = 2, find ciphertext.
5. Taking p=2,q = ll convert 'GOD' by taking numerical values of the alphabetical order of
the letters G, O, D as encrypted code and then find its decryption.

w
Given public key = (143,13) and plaintext/message as m = 3, find expression for ciphertext
and message by using RSA algorithm.
By taking p =5 and 7 =13, encrypt '2' to a new code and decrypt it again to get 2.
R. By taking p = 3 and iy =11, convert "RAM" by taking numerical values of the alphabetical

e
order of the letters R, A, M, as encrypted code and then find its decryption.

ro
re
ANSWERS

Possible primes and corresponding RSA modulus for (fi {pq) = 120 are:

F
uFl
(i) p = 3, q = 61 and RSA modulus = 183
(ii) p =5, q = 31 and RSA modulus = 155
(hi) p=ll,q = 13 and RSA modulus = 143

sr
3 C = 5 ^ mod 22 for e = 3 or C = 5^ mod 22 for e = 7

ko
13 37
o
: 30 GOD . C = 3 mod 143, m =C
of mod 143

1.4 THE BINARY NUMBER SYSTEM


o
Y
Since our childhood we are using the decimal number system which is also known as the
erB

base-10 system. This system has 10-digits 0,1,2,3,..., 9. This system is a positional system where
each digit has a value based on its position called the place value. The place value of units
position is 1 i.e. 10^, tens position is 10^, hundreds position is 10^ and so forth. In decimal
uY

notation, we write a number as a string of digits, where each digit is one of the ten digits 0,1,2,3,
..., 9, and to interpret a decimal number, we multiply each digit by the power of 10 associated
with that digit's position.
ad
do

For example, the decimal number 8375 can be written as 8 x 10'^ +3 x 10^ + 7x 10^ + 5 x 10^
and the decimal number 259.754 can be written as 2 x 10^ + 5 x 10^ -1- 9 x 10^ + 7x10 ^ + 5x10 ^
in

+ 4x10*^.
Re

A computer can understand only the "on" and "off state of a switch/circuit. These two states are
F

represented by 1 and 0 respectively. A string of digits, where each digit is either 1 or 0 forms a
binary number in the binary number system. As two digits, namely 0 and 1, are used to
represent numbers, therefore a binary number system is also called a base-2 system. The base-2
digits 0 and 1 are termed as binary digits or bits for short. Binary number system is also
positional number system where each digit has a value based on its position called the place
value. To interpret a binary number, we multiply each digit by the power of 2 associated with
that digit’s position. For example the binary number 11011 is 1 x 2^ + 1 x 2^ + 0 x 2^ + 1 x 2^ +
1x2^ = (27)in and the binary number 1011.101 is lx 2^+ 0x2^+1x2^+ 1x2^+ 1x2 ^ +
0x2” 2+1x2'^ = (11.873)10-
1.4.1 CONVERSION FROM BINARY NUMBER TO DECIMAL NUMBER

To convert a binary number to a decimal number, we write the binary number as a sum of
powers of 2. For example, to convert the binary number 1101 to a decimal number, we observe
that the rightmost position is the ones position and the binary digit at that position is 1. So, this
right most binary digit has the decimal value of 1 x 2^. The next position to the left is the twos
position and the Binary digit in this position is 0. So, this binary digit has the decimal value
Ox 2^ The next position to the left is the fours position and the binary digit in the position is 1.
NUMBERS 1.23

So, this binary digit has the decimal value 1x2'^. The left most position is the eights position and
the binary digit in this position is 1. So, this left most binary digit has the decimal value 1 x 2^.
Thus,
(1101)2 =1>^2^+1x2^ + 0x2^1x2°=8 + 4 + 0 + 1= (13)|o
ILLUSTK.VnON 1 Convert the binary number 111011 to a decimal number.
SOLUTION (111011)2 =lx2^+lx2^+lx 2^+ 0x2^+ 1x2^+ 1x2*^= 32 + 16 +8 + 2 + 1 =(59);kj
II.LUSTRA I i: Express the binary number 1011.1011 as a decimal number.
SOLUTION (1011.1011)2 =lx2^ + 0x2^+lx2^+lx2‘^+lx2“^+0x2“^+lx2"^+ lx2'^
= 8 + 2 + 1 + 05 + 0.125 + 0.0625 = (11.6875)|o
1.4.2 CONVERSION OF DECIMAL NUMBER TO BINARY NUMBER

In order to convert a decimal number to a binary number, we use the remainder method. In this
method, we divide the number of 2 recursively until we obtain 0 as the quotient and take note of

w
each remainder. Following algorithm may be used to convert a decimal number to a binary
number.

Flo
ALGORITHM
STLP 1 Write down the decimal number.
-TEIMI Divide the number by 2 and zvrite the quotient underneath and the remainder on the right hand

ee
side. Note that the remainder zvill be either Oorl.

Fr
■STEP IIIDivide the quotkmt obtained in step II by 2 and again zvrite dozvn the quotient and remainder.
Repeat step II until the result of division is 0.
STEP 1\ The most significant binary digit is at the bottom of the column of remainders and the least
for
ur
significant binary digit is at the top. So, read the series of Is and Os on the rightfrom the bottom
up to obtain the binary equivalent of the decimal number.
s
ILI.USTRATION Convert the decimal number 53 to binary.
k
Yo

SOLUTION Dividing 53 recrussively, we obtain


oo
eB

Remainder

2 53
2 26 1
r

2 13 0
ou
ad

2 6 1
2 3 0
Y

2 1 1

0 1 Read up
Re
nd

(53)io =(110101)2
Fi

■ '●( R-\'I ' Convert the decimal number 201 to binary.


SOLUTION Dividing 201 recrussively, we obtain
Remainder
2 201

2 100 1

2 50 0

2 25 0
2 12 1

2 6 0
2 3 0

2 1 1

0 1
Read up

(201)io =(11001001)2
1.24 APPLIED MATHEMATICS-XI

CONVERSION OF DECIMAL FRACTION TO BINARY

To convert a decimal fraction to a binary number, we follow the following algorithm.


ALGORITHM

STEP 1 Multiply the given decimal fraction by 2 which is the base of the binary number.
STEP II Record the carry of in the integral position. The carry of obtained is the most significant digit of
the binary number.
STEP 111 Multiply the decimal part obtained in step II by 2 and the carry obtawed is the second most
significant digit of the binary number.
STEP 1 \ Repeat the above steps till the desired accuracy is achieved.
STEP V The equivalent binary number is obtained by arranging the binary digits from most sigyiificant
binary digits to the least significant binary digit (i.e. from top to bottom).
ILLUSTRATION ^ Convert the decimal fraction 0.35 to a binary fraction.
SOLUTION Multiply 0.35 recursively by 2 and recording carry of each time, we obtain

w
0.35 X 2 = 0.70 with a carry of 0
0.70 X 2 = 0.40
0.40 X 2 = 0.80
0.80x2 = 0.60
F lo
with a carry of
with a carry of
with a carry of
1
0
l

e
Fre
0.60 X 2 = 0.20 with a carry of 1
0.20x2 = 0.40 with a carry of 0 Top to bottom
for
The above process is continued till the desired accuracy is achieved.
(0.35)io =(0.010110
r
You

CONVERSION OF MIXED NUMBERS IN DECIMAL SYSTEM TO BINARY NUMBER


oks

To convert a decimal mixed number into the binary number, we convert the iiateger part and
eBo

decimal part separately to their binary equivalents as explained in the following illustration.
ILLUSTRATION -I Convert the decimal number 15.6875 to binary.
SOLUTION Let us convert 15 and 0.6875 into their binary equivalents separately as given
ad
our

below.

2 15 0.6875x2 = 0.375 with a carry of 1


1
dY
Re

2 7
1
0.3750x2 = 0.750 with a carry of 0
2 3 0.750 X 2 = 0.50 with a carry of 1
Fin

1
2 1 0.50 x2 = 0.00 with a carry of 1
0 1
bottom to top

(15)io =(1111)2 and(0.6875)^0 =(0.1011)2


Hence, (15.6875)|o =(1111.1011)2
1.5 BINARY ARITHMETIC

In this section, we will discuss addition, subtraction, multiplication and division of binary
numbers.

1.5.1 BINARY ADDITION

While adding decimal (base-10) numbers, we use the principle of "carrying over to the next
column" which states that if the sum of the digits in a column (of I's or lO's or lOO's ....) is not a
NUMBERS 1.25

single digit number, carry over to the next column and subtract the number being carried over
from the sum of the digits in the column to get a single digit in that particular column.
For example, when we add decimal numbers 29 and 16, we obtain 15 as the sum of the digits in
the ones column. So, we carry over a quantity of ten to the tens column (where it becomes 1 since
it is now in the tens column) and subtract ten from 15 to get 5 in ones column.
1
TO
2 9
+ 1 6

ow
4 5

The same principle is followed in binary addition.


Thus, if the sum of the digits in a column is too big to be represented as a single digit in the
binary stem, we carry over to the next column, and subtract the quantity being carried over from

e
the sum of the digits to get a digit in that particular column.

re
Using the above principle and our skill of addition, we obtain

Frl
F
1

1 0 0 1 1

+0 +1
ou +0 +1 +1

sor
1 1 F' fo' TT

Note that to find 1 + 1, we have to carry over a quantity of two to the 2's column (where it
kf
becomes 1 since it is in the 2’s column), and subtract 2 from 1 + 1 to get 0 in ones column.
oo
Similarly, while performing the addition 1 + 1+1, we carry over a quantity of two to the 2's
Y

column (where it becomes 1 since it is in the 2's column) and subtract 2 from 1 + 1+1 i.e. 3 to get 1
B

as the digit in ones column.


ILLUSTRATION 1 Perform the following additions of binary numbers:
re
oY

(i) 101101 (ii) 1010 01


u

+ 1011 + 110110
ad
d

SOLUTION

®®®®
in

(i) 10 1101 (ii) 1010 01


Re

+ 1011 +110110
F

111000 1011111

1.5.2 BINARY SUBTRACTION

Binary subtraction is similar to that of decimal subtraction with the difference that when 1 is
subtracted from 0, it is necessary to borrow 1 from the next higher order binary digit and that
digit is reduced by 1.
Thus, the rules of binary subtraction are as follows:
0-0 = 0
1-0=1
1-1=0
0-1=1 with a borrow of 1.

ILLUSTRATION 1 Perform the following subtractions:


(i) 10 01 (ii) 10 0 0 (iii) 1111011
101 1 11 -1010101
1.26 APPLIED MATHEMATICS-XI

SOLUTION
® borrow ® borrow ® borrow
(i) 10 01 (ii) 10 0 0 (iii) 1111011
101 1 1 1 -1010101
100 0001 100110

JARY MULTIPLICATIO!

Binary multiplication is similar to the multiplication of decimal system. Following are the rules
of binary multiplication.
0x0 = 0
0x1 =0

w
1x0 = 0
1x1=1

Similar to the multiplication in decimal system, the multiplication of binary numbers is carried
out by multiplying the multiplicand by one binary digit of the multiplier at a time, beginning

o
e
with the least significant digit, and the result of the partial product for each binary digit is placed

re
in such a manner that the least significant binary digit is under the corresponding multiplier

Frl
binary digit. Finally, the partial products are added to get the complete product.

F
It should be noted that a multiplication by 0 makes all the binary digits of the partial product
zero, so it does not contribute anything to the final sum of partial products. Thus, it may be
ou
ignored in the intermediate steps. Also, a multiplication by 1 leaves the digits of multiplicand

or
unchanged but shifts it towards left by one digit position. Consequently, the multiplication of
kfs
binary’ numbers is the addition of multiplicand with itself after some suitable shift depending
upon the multiplier. Following algorithm may be used to multiply two binary numbers.
oo

'LGORITHr;

Obtain the multiplicand and multiplier and ivrite the multiplier just below the multiplicand.
Y
eB

Take the right most digit of the multiplier. If it is 1 write the multiplicand as such as first
partial product. If the right most digit is 0, do not ivrite any thing.
Take the second right most digit of the multiplier. If it is 1 write the multiplicand as second
ur

partial product in such a manner that its least significant digit is just below the second right
oY

most digit of the multiplier. If the second right most digit of the multiplier is 0 do not write
ad

anything as the second partial product.


Find the intermediate sum of two partial products.
d

Take the third digit from the right of the multiplier and proceed as in step III keeping in mind
in
Re

that the partial product is to be shifted leftward suitably. Find the intermediate sum.
Perform thefollozving binary multiplications:
F

II '

(i) 10111 (ii) 10101 (iii) 10111


X 11 X 1 0 X 1101

SOLUTION
(i) 10111
X 11

10111 <- First partial product


10111
Second partial product
1000101 <— Final sum

(ii) 10101
X 1 0

0 0 000 <— First partial product


1010 1 <- Second partial product
101010 <- Final sum
NUMBERS 1.27

A LITER
10101
X 1 0
10101 <— Second partial product
101010 <r- Final sum

(iii) 10111
xlioi
10111 <- First partial product
10111 <- Third partial product
1110011 <- First intermediate sum
10111 <— Fourtln partial product
100101011 <— Final sum

w
1.5.4 BINARY DIVISION

The process of division of binary numbers is quiet similar to that used in the division in decimal
system. However, in the case of binary numbers, the operation is simpler because the quotient

F lo
can have either 1 or 0 depending upon the divisor. Following are the rules of binary division.
- =0
1

ee
1

Fr
= 1
1
0 1 for
Note that — and — are not meaningful.
0 0
ur
Following illustrations will explain the division process.
11 I.L'STRATION Perform the folhiving binary divisions:
oks
Yo

(i) 101111-111 (ii) 11001-101 (iii) 11101-1100


o

SOLUTION (i)
eB

111) 101111 (11 <-Quotient


111
our

1001
ad

111
101 <- Remainder

(ii) 101) 11001 (10 <-Quotient


Y

101
Re
nd

101
101
Fi

0 <- Remainder

(iii) 1100)11101(10
1100
101

EXERCISE 1.4

1. Convert the following decimal numbers to binary numbers:


(i) 25 (ii) 53 (iii) 108 (iv) 129
(v) 75.125 (vi) 213.75 (vii) 39.625 (viii) 217.8125
Perform the following binary additions:
10001 101101 1110 lion
(i) (ii) (iii) (iv) + 1001010
+ 11101 + 11001 + 1111

Oil
10111 1111101 11111111
(v) (vi) (vii) 111 (viii) + 1111111
+ 110101 + 10101010
+ 1011
1.28 APPLIED MATHEMATICS-XI

Perform the following binary subtractions.


1011011 1010110 1000101 101101
(i) - 10010
(ii) -101010
(iii) -101100
(iv) -loom

mono 100010110 10100111001


(V) (Vi) (vii)
-1010111 1111010 -11101011

Perform the following binary multiplications.


1010 1011 1110 non
(0 X 101
(ii) xll
(iii) X 101
(iv) xlll

non 1101011
(v) X 1001
(Vi)
X101101

Perform the following binary divisions,

w
(i) 1111100-10 (ii) 101101-101 (iii) 11011-101 (iv) 101100-1011
(v) 1000110-101

F lo
Solve the following equations, where all numbers, including x, are binary numbers:
X X
(i) — = 110 (ii) = 101 (iii) = 1011

ee
11 101 111
Find the value of (1101)^ -(101)^.

Fr
Multiply the following binary numbers and verify the product by converting the two sides
to decimal system: for
ur
(i) 1001 and 101 (ii) 11010 and 111 {iii) 101011 and 1101
Divide the following binary numbers and verify the result by converting the two sides to
decimal system:
ks

(i) 101111 by 111 (ii) 1110101 by 1001 (iii) 10000001 by nil


Yo
oo

Also, find the quotient and remainder in each case.


eB

ANSWERS

(i) (11001)2 (ii) (110101)2 (iii) (1101100)2 (iv) 10000001


(v) (1001011.001)2 (vi) (11010101.11)2
r

(vii) (100111.101)2
ou
ad

(viii) (11011001.1101)2
(i) 101110 (ii) 1000110 (iii) 11101 (iv) 1100101
Y

(v) 1001100 (vi) 100100111 (vii) 10101


(i) 1001001 (ii) 101100 (iii) 11001 (iv) no
Re
nd

(v) mil (vi) 10011100 (vii) 10001001110


(i) 110010 (ii) 100001 (iii) 1000110 (iv) 10111101
Fi

(v) iiiioon (vi) 1001011001111


(i) 111110 (ii) 1001 (iii) Quotient = 1010, Remainder = 1
(iv) 100 (v) 1110
(i) 10010 (ii) 11001 (iii) 1001101 7. 10010000
(i) 101101 (u) 10110110 (ui) 1000101111
(i) Quotient : no (ii) Quotient : 1101 (iii) Quotient : 1000
Remainder: 101 Remainder: 0000 Remainder: 1001

EED FOR COMPLEX NUMBERS


If a, b are natural numbers such that a>b, then the equation x + a=b is not solvable in N, the set
of natural numbers i.e. there is no natural number satisfying the equation x + a=b. So, the set of
natural numbers is extended to form the set / of integers in which every equation of the form
X + a=b; a, b eN is solvable. But, equations of the form xa = b, where a,b e I, a ¥: 0 are not
solvable in / also. Therefore, the set / of integers is extended to obtain the set Q of all rational
numbers in which every equation of the form xa =b, a ji: 0, a, b e I is uniquely solvable. The
equations of the form x = 2, .v^ = 3 etc. are not solvable in Q because there is no rational
NUMBERS 1.29

number whose square is 2. Such numbers are known as irrational numbers. The set Q of all
rational numbers is extended to obtain the set R which includes both rational and irrational
numbers. This set is known as the set of real numbers. The equations of the form
.v^ + 1 = 0, + 4 = 0 etc. are not solvable in R i.e. there is no real number whose square is a
negative real number. Euler was the first mathematician to introduce the si/mbol i (iota) for the square
root of - I i.e. a solution of +1=0 loitli the property i^ =-l. He also called this symbol as the
imaginary unit.
1.6.1 INTEGRAL POWERS OF IOTA (i)

Positive integral powers of i: We have, / =


●2 ●2 ; ;4
I = -1, = / XI - -I, I = (iV = (-1)^ = 1
In order to compute /" for n > 4, we divide n by 4 and obtain the remainder r. Let m be the
quotient when n is divided by 4. Then,
n = 4m + r, where 0 < r < 4
:4m + r
I 1 = f

Thus, the value of i” for > 4 is f, where r is the remainder when n is divided by 4.

w
Negative integral powers of i: By the law of indices, we have
.-1
i

:-3
I

If n > 4, tlien i
1 I
;3
:3


- 2
i

I
1
1

:3
1
= -1
■A
i

1
.4
I
l
=
r

1
= -i, i

.-4
i
F lo
1
i

1
;4

= —, where r is the remainder when n is divided by 4


■2

1
1

- = 1
1

ree
for F
; n -● r ■'
l l

NOTE i ^ is deifned as 2.
Your
ks

The above discussion suggests the following algorithm to find integral exponents of i.
eBoo

ALGORITHM

To find the value of i” for n e Z, we may follow the following steps.


STCPl
Ifn = 0, then zvrite /" = 1.
ad
our

OTPil Ifn >0, then


i, if n=l
-1, if n=2
Re

.-n
l -l, if n = 3
Y

1, if n =4
Find

if n > 4, where r is the remainder zohen n is divided by 4


MEP III Ifn < 0, thenn = -m, wherein > 0.
if n = -l
i
1 1
;-2
1 = -l. if n=-l
-2
I -1
1 I
:-3
i i if n = -3
;3 .4
i 1

●-4
1 1
i if n = -4
1
; - m
1 1
/
:III :r
where r is the remainder when m is divided by 4, if n < -4.
I I
1.30 APPLIED MATHEMATICS-Xl

ILLUSTRATIVE EXAMPLES

LXAMPLE i Evaluate the following:


;135 .-19 4n+ 3
(i) i (ii) i (iii) (iv) i-pi) ,t1€N
SOLUTION (i) 135 leaves remainder as 3 when it is divided by 4.
.135 -.3
l = l

(ii) The remainder is 3 when 19 is divided by 4.


i^^=i^=-i
.-999 .●999
(iii) We have, i = 1//
:999 _ :3
On dividing 999 by 4, we obtain 3 as the remainder. Therefore, i I .

ow
1 1 / I
Hence, - I
:999 ●3 ;4
1 I 1 1
4(1+ 3 4(1+3 :3
(iv) We have, (-^/^) = (-0 X -1 I

e
1 XAMI’LC 2 Show that:

re
2512 34l 2
(i) +
(1
= - 4

rFl (ii)
.●17
i > =2/

F
V J V J
3
24
n

r
ou
.●18 :)I+ 1 . :n+ 2 . ;}]+ 3
(iii) r + = 0 (iv) /” + / + i +1
= 0, for all n eN.
V J sfo
251 2 :3 12
k
2
I

SOLUTION (i) \ +
oo

~i +

\ I :25 l A
i l
Y
B

= [~i + /^]2 = (- / - /)2 = 4 _4


3412 2
re

(ii) < i
.●17
1
.●17
i
1
M,- 1- — i -
1
— =(/ + l)2
,
ou

.-34 ●2
(-1)
Y

V l l
ad

= (2 + 2/ + 1=-1+2/ + 1=2/
241 ^
d

(iii)
.●18 n .●18
1
1' ●2
= (-1 +1)^ =0
in

< i + r + r + -
Re

.-24
V J i 1
F

:JJ+ 1
(iv) I
;(I
+ t + l -■»+2^-«+3^ .●3

= /"(1+/ + /2+/3)
= /"(l+/-l-0= /" (0)=0
13
r\ \ MPLE 3 Evaluate Z (/" + /" ^), where n e N.
« = 1

13 .
SOLUTION Z («■" + ^)
(1=1
13
;(l
= Z (/■ + !) /
n=\

13
= (i + l) Z i
(1=1
NUMBERS 1.31

= (i + l) {i + -p +i^+...+
^●13
i -1
= (/ + !) XI
i-1

fi-1
= {P+i) = (-1+0
.●13
= i]
Ki-lJ
. \Mi-i.i; -■ Evaluatel+P+P+P+...+ P^’.
SOLUTION Let S = 1 + P +P+P+...+ .Then
S=l+i'2 + (i2)2+{i2)^+...+ {i2)"
1
2n«+1'I — (1 -1) = 0, if « is odd
S=1
l-(l-) l_(/2)»+l 1 ji+1 2
l-(-l)

w
1
1-/2 1+1 2
— (1 +1) =1, ifn is even
2

F lo
EXERCISE 1.5

Evaluate the following:


1
;457 528
(i)/ (ii) i (hi)

ee
●67
^8

Fr
l

1
(v)
;41
i +
:257
(Vi) (/77^./70 ^,-87 ^-414)3 for (vii) /30+1-40+1-60
I
ur
(viii)l^^+I^+/89+,-110
' ● Show that 1 + P^ + P^ + is a real number.
s
ook
Yo

Find the values of the following expressions:


(i)/49^-68^.89^.n0 (h),-30 + -80 +1
, ..120
eB

(hi) i + P + P + P (iv) P + + I:I5


592
590 p88 ^586 ^584
r

l +1
.-20
(vi) l+/2+/4+/6^y8^ _
ad
ou

(V)
■582 _j_ y580 ^ ^578 ^ y576 ^ -574
(vii) (i+o^+(i-o^
Y
Re

ANSWERS
nd

(V) 0 (vi) - 8 (vii) 1 (viii) 2 i


Fi

l.(i) i (ii) 1 (iu) -1 (iv) 2/


3.(i) 2/ (ii) 1 (iii) 0 (iv) -1 (V) -1 (vi) 1 (vii) -2-10i

1.6.2 IMAGINARY QUANTITIES

The square root of a negative real number is called an imaginary quantity or an imaginary
number.

For example, yj- 3, yj-9/4 etc. are imaginary quantities.


THEOREM If a, b are positive real mimbers, then x =- -Pab.
I’jtooi: We have.

yj-lx a - X yfa =i -Ja and, y[^ = yj-lxb = y[^ x yjb =i 4b


yf^x .y^ = (j yfa) (i 4b) =P {4a x 4b) =-l {4ab) =-4ab
1.32 APPLIED MATHEMATICS-XI

\()'l r I For any tivo real numbers -Ja.y S =:^f^ is true only when at least one of a and b is either
positive or zero. In other zuords, -Ja y sib = is not valid if a and b both are negative.
l; I'l z For any positive real number a, we have = f-lx a x fa =i yfa.
II LUSTRATION 1 Compute the following:

(i) ^Pl44 (ii) X


^ (i i) +
SOLUTION Using = i4a when <? > 0, we obtain
(i) =7-1 X 144 y Vl44 =12/
P9 ("3M 9
(ii) Plx = (20 — =3r=-3
II 4 2

(hi) yj~ 25 + 3 yj- 4 + 2 = 5 / + 6 / + 6 / = 17 I


ILLUSTRATION 2 A Student lorites the formula fob =4a 4b. Then he substitutes a = -l andb = -l

w
and finds 1 = -1. Explain where is he lorong?

fort? and & both negative, we have 4a 4b = - 4^.


II I.USTRATION '

SOLUTION
lVr2).V(^l = V<-2)-(-3) = V6
F lo
SOLUTION Since a and b both are negative. Therefore, 4ab cannot be written as 4a 4b. In fact,

Is the follozving computation correct? If not give the correct computation:


for
F ree
The said computation is not correct, because - 2 and - 3 both are negative and
4ab = 4a 4b is true when at least one of a and b is positive or
zero. The correct computation is
{yP2){yP3) = ii42){i43) = 4e - 46
r
You
oks

1.6.3 COMPLEX NUMBERS


eBo

COMPLEX NUMBER If a, b are two real numbers, then a number of the form a + ib is called a complex
number.

For example, 7 + 2 -1 + i, 3 - 21, 0 + 2 /, 1 + 0 2 etc. are complex numbers.


ad
our

Real and imaginary parts of a complex number: Ifz = a + ib is a complex number, then 'a' is called the real
part ofz and 'b' is known as the imaginary part of z. The real part ofz is denoted by Re (z) and the
imaginary part by Im (z).
Re
dY

If z = 3 - 4 /, then Re (2) = 3 and Im (2) = - 4.


Purely real and purely imaginary complex numbers: A complex number z is purely real if its imaginary
Fin

part is zero i.e. Im (2) = 0 and purely imaginary if its real part is zero i.e. Re (z) = 0.
Set of complex numbers: The set of all complex numbers is denoted by C i.e. C =\a + ib : a,b € R}.
Since a real number 'a' can be written as + 0 2. Therefore, every real number is a complex
number. Hence, R <zC, where R is the set of all real numbers.
1.6.4 EQUALITY OF COMPLEX NUMBERS

DEFINITION Tivo complex numbers = n-j + 2 b-^ and22 = 222 + i ^^2 equal ifa-^ = 2?2 and bi = &2-
i.e. Re (z{) = Re (22) and Im (zj) = Im (22).
Thus, 2i=Z2 Re(zj) =Re(z2) and Im (z^) =lm (29).
ILLUSTR.ATION I Ifz^ =2 -iyandz2 =a: + 3 i are equal, find x and y.
SOLUTION We have,
2i = Z2 ^ 2 - iy = x + 3i ^ 2-x and ~y = 3 ^ x = 2 and y = - 3.
ILLUSTRATION 2 If{a + b) -i{3a + 2b) =5 + 2 i, find a and b.
NUMBERS 1.33

SOLUTION We have,
{a + b) -i{3ci + 2b) =5 + 2i => a + b =5 and ~{3a + 2b) = 2 ^a = -12,b=17
1.6.5 ADDITION OF COMPLEX NUMBERS

DEFINITION Let 2^ = fl-j + i b^ and 22 = ^ ^2 complex numbers. Then their sum Zj + Z2 is


defined as the complex number + ^2) + Kh + ^2)-
It follows from this definition that the sumz^ + 22 is a complex number such that
Re (zj + Z2) = Re (zj) + Re (22) and, Im (zi + 22) = Im (Z|) + Im (22)
For example, Ifzj = 2 + 3 / and 22 = 3- 2/, thenzi +22 =(2 + 3) + (3 - 2) / = 5 + /
1.6.6 SUBTRACTION OF COMPLEX NUMBERS

ow
DEFINITION Letzi = 4- ibinndz2 = + ^^2 be two complex numbers. Then the subtraction ofZ2from
Zi is denoted bi/z-^ -Z2 and is defined as the addition ofz^ and -22.
Thus, 2i -22 -Zi + (-22) + ib{) + (-^2 -ibi) = {a-[ -^2) + ^ (^1 “^2)
For example, If Zj = - 2 + 3/ and 22 = 4 + 5/, then

e
2^-22 = (-2+3/)+(-4-50 ={-2-4) + /{3-5)=-6-2/

re
1.6.7 MULTIPLICATION OF COMPLEX NUMBERS

rFl
F
Let 2j = and 22 = ^2 +' ^2 complex numbers. Then the multiplication of Zj zuith Z2 is
denoted byz-^ 22 and is defined as the complex number{a-^ <?2 -bi ^2) + ^ ^2 ^2 ^l)-
Thus, z-j Z2 = +//>i) («2 '^2)

r
ou
Zi Z2 = (flj ^2 ~h ^’2) ' (^1 ^2 + ^2 h) fo
ks
2| Z2 = [Re(zi) Refzj) -lm(Zi) Im{z2)] + / [Re(Z|) lm{22) + Re(z2) Im(zi)]
For example. If Zj = 3 + 2/ and 22 = 2 - 3/, then
oo

2^22 = (3 + 20(2-30 = (3x 2-2x(-3))+/(3x-3 + 2x 2)=12-5/


Y

NOTE The product z-j 22 can also be obtained if we actually carry out the multiplication
eB

(i7| + / bi) (^2 + / 62) given belozv:


{a-^ + i b{) {a2 + i /’2) = ^ ^2 ^^2 h ^2
r

●2
You

= (<7j i?2 ~b-^ 1^2) ^'(^1 b2 ^2


ad

ILLUSTRATIVE EXAMPLES
d

Type I EXPRESSING A COMPLEX NUMBER IN THE STANDARD FORM a + lb


Re
in

In order to express a complex number in the standard form, we may follow the following
algorithm.
F

ALGORITHM
a + ib
■STEP I Write the complex number in the form by using fu7ida7nental operations of addition.
c + id

subtraction and multiplication.


STEP 11
Multiply the numerator and denojninator by the cotijugate of the denominator.
EX.AMPLE 1 Express the following in the form a + ib :
(I \ 1
(i)(-50 -/ (ii)(-0(20 —/

3
(iii) (50 --/ (iv)/%/19
5
- 39 4
(V) / (vi) (1 -0
1.34 APPLIED MATHEMATICS-XI

1 5 9 5 5 5
SOLUTION (i) (-50 -l --r — x-l=-=- + 0/
8 8 8 8

1 1 I 1
(ii) (-0(20 —/ = - 2i^ X - t = 0 + /

8 512 256 256 256 256

3 ^
(iii) (50 —i =-3/^=-3x-l = 3 = 3 + 0/
5

IV = (/^)^ i + (/"^)^ = (■ + /3 =/ _/ = 0 = 0 + 0 (■
39
(V) i = (/^) -^0/ = / = 0 + l./

ow
2

(Vi) = (1-2j + /^)^=(1-2/-1)^=(-20^=4/2=-4 = -4 + 0/

\ ME’U ; Express each of the following in the form a-hib:


(i) 3(7+70+((7+70 (ii) (1 -0 -(-1 + 60

e
re
Fl
\ r 1 ^
(I 2 \ 5 'I 1 7 4
(iii) -+- i - 4 +—/ (iv) \ — + — i + 4 + — i ■ — + /

F
5 5 2 ; 3 3 y V 3 ; 3
ur
SOLUTION (i) 3 (7 + 70 + / (7 + 70 = 21 + 21 / + 7i + 7^ = 21 + 21 i + 7i - 7 = 14 + 28 i

r
(ii) (1 -0 -(-1 + 60 =1 -i + 1 - 6/ = 2 -7/
n 2^ 5 'i fl 2i 5/ 19
fo 21
ks
(iii) - + — i - 4 + -i -4 + 1
Yo
5 5 2 ) 5 5 2 5 10
oo

ri 7 W 1 ( —4 I ^ (7 -[
— + i
4 1
+ i —+ 4 +i — + —
eB

(iv) — + — / + 4 + z

3 3 3 J\ { 3 j 3 3 3J 3

fl3 8 ^ 4
ur

— + — z + z

3 3 3
ad
Yo

(13
■— + —
4 + 1
17 5
z =— + — z
3 3 U 3 3
d
Re
in

tXAMl’LE 3 Express each of the following in the form a + ib:


F

1 1
(i) -+3/ (ii) -2--Z
>.3 3

(iii) (5-30^ (iv) (-V3+V^)(2V3-0

SOLUTION (i)
fl
+ 3z
'^3 ^ 1 + (30^n + 3x -1 X 3/
fl
— + 3z
1
— + 27r + 3i
^ n
-+ 3z
V3 I 3, 3 3 27 3
1 o 9 1 242
= — + 27r +i+9i^ = 27z + z -9 = - -26z
27 27 27

(ii) -2--Z
1 f = (-2)^+ ' - 1 ^\3 — z + 3 X -2 X
1
z 2
1 ^
z 8
1
— r + 2i
3 1
-2 — i
3 ) \ 3 3 K 3 J 27 3 )
1 2 2 9 1 22 107
= - 8 + — z - 4z — r = - 8 + — z - 4z + — = i

27 3 27 3 3 27
NUMBERS 1.35

(iii) (5 - 3/)^ =5^ + {- 30^ + 3 X 25 X - 3/ + 3 X 5 X (- 30^=125 + 27/-225(-135 =-10 -198/


(iv) (-V3+^) (273-0 = (-73+/72)(2 73-0=-6 + 73/ + 2V6/-V2/^
= - 6 + (73 + 276) / + 72 = (72 - 6) + (73 + 276) /
Tiipc II ON EQUALITY OF COMPLEX NUMBERS
Recall that two complex numbers z-^ and Z2 (Jre equal iff Re(Z2) = Re(z2) and Im(z-j) =Im(22)-
liXAMPi.t 4 Find the real values ofx and y, if
(i) (3x -7) + 2/y = -5y + (5 + x) i (ii) (1 - 0 X + (1 + 0 y = 1 - 3/
SOLUTION (i) We have
( 3t - 7) + 2/y = - 5y + (5 + .v) /
3.1: -7 = -5y and 2y =5 + .t
3.r + 5y=7 and x-2y=-5 .Y = -l,y = 2.
(ii) We have.

w
(l-0x + (i + 0y = i-3/
(.v + y) + /(-x + y)=l-3/
.r 4- y = 1 and - x + y = - 3

Flo
[On equating real and imaginary parts]
x=2,y=~l
EXERCISE 1.6

ee
1. Express the following in the form a + ib:

Fr
1" fl
(i) 3/x—/ (u) /(-20x -/ (iii) (1-,)^ (iv) {1+i)*
4 2
for
ur
Find real values of x and y, if
(i) (x + /y) (20 =6+3/ (ii) (3x-2/y) (1+0^ =5-4/
s
(iii) (2x + 3) + 2/y = -3y + (7 - x) / (iv) (1 + 0 X + (1 -0 y = 1 + 3/
k
Yo

ANSWERS
oo

3 1
(i) —+ 0/ (ii) 0—/ (iii) -2-2/
eB

(iv) -4 + 0/
4 4
3 2 5
(i) .t = -,y =-3 (ii) x = - -,y =- (iii) x=-27,y=17 (iv) x = 2, y =-1.
r

2 3 4
ou
ad

MULTIPLE CHOICE QUESTIONS (MCQs)


Mark the correct alternative in each of the following quesBtions:
Y

1. Upi and p2 are two odd prime numbers such that P\>P2> then - p-^, is
Re
nd

(a) an even number (b) an odd number


(c) an odd prime number (d) a prime number
Fi

If p and q are co-primes, then y^and q^ are


(a) co-prime (b) not co-prime (c) even (d) odd
3. The LCM and HCF of the numbers are equal, then the numbers must be
(a) prime (b) co-prime (c) composite (d) equal
4. If a and b are two relatively prime numbers, then their LCM is equal to
(a) a (b) b (c) ab (d) «'●
Wliich of the following numbers is not prime?
(a) 191 (b) 199 (c) 487 (d) 648
6. The sum of binary numbers 101101 and 110010, is
(a) 101111 (b) 1011111 (c) 110111 (d) none of these
The binary number (1100011)2 -(100100)2 equal to
(a) 111 111 (b) mil (c) 101111 (d) none of these
1.36 APPLIED MATHEMATICS-XI

8. The value of (1010)2 X (1000)2 is


(a) 101000 (b) 1010001 (c) 1010000 (d) none of these
9. The product of binary numbers 1001 and 101 is
(a) 101100 (b) 101101 (c) 111101 (d) 101000
10. If A =11010 and 6 =101, the remainder whenv4 is divided by B, is
(a) 10 (b) 100 (c) 11 (d) 1
A
II- If A =1010 and B =101, then is equal to
B
(a) 1 (b) 10 (c) 11 (d) none of these
12. If 101101 =(101) X X, then .Y =
(a) 101 (b) 1100 (c) 1001 (d) 1010

ow
13. The value of (1+0^/is
(a) -2+/ (b) 2-2/ (c) -2 + 2/ (d) -2+/
17 1
14. The value of r' + ,is
.401

e
I

re
(a) -/ (b) / (c) 1 (d) 0
15. The decimal equivalent of (110101)2,

rFl
F
(a) 50 (b) 52 (c) 51 (d) 53
' If S is the sum and P is the product of first four primes, then P - S is equal to

r
(a) 227 (b) 193 (c) 197 (d) 183
ou
fo ANSWERS
ks
1. (a) 2. (a) 3. (d) 4. (c) 5. (d) 6. (b) 7. (b) S. (c)
9. (b) in. (d) n. (b) 12. (c) 13. (c) 14. (d) 15. (d) 16. (b)
oo
Y

FILL IN THE BLANKS TYPE QUESTIONS (FBQs)


B

Fill in the blanks to make the following statements correct:


re

1. Every prime number is an number.

2. Tire sum of any two prime numbers is an number.


ou
Y
ad

3. The small divisor (other than 1) of a composite number is a .number.


4. The sum of first five Ramanujan primes is .
5. The sum of the divisors of a prime number p is
d

6. If S and P denote respectively the sum and product of the divisors of a prime number, then
in
Re

S - P is equal to .
F

7. Process of converting plaintext to ciphertext is called .


8. Process of converting ciphertext to plaintext is called .
9. When encryption and decryption processes use the same key, the cryptography process is
called .

10. When encryption and decryption processes use different keys, the cryptography process is
called .

11. The value of (11)2 X (10)2 is .


12. / + /^+/^+ +i
.●10
is equal to
1.3. The value of/x/ X /^ X i‘^x XJ
.●100 :
is

ANSWERS
1. odd 2. Even 3. prime 4. 100 5. p + 1 6. 1 7. Encryption
Decryption 9. Symmetric Cryptography 10. Asymmetric Cryptography
n. (110)2 12. 0 13. -1.
CHAPTER 2

INDICES

w
2.1 INTRODUCTION

In earlier classes, we have studied about powers of rational numbers and the laws obeyed by

Flo
their indices (exponents). We have also learnt about real numbers and throughout this text we
will be working with real numbers. So, it is desirable to define powers of real numbers. The

e
powers of real numbers are defined in the same way as powers of rational numbers and the

re
same laws of indices (exponents) hold as discussed in the following section.

rF
2.2 INTEGRAL EXPONENTS OF A REAL NUMBER
ur
POSITIVE INTEGRAL POWER For any real numher 'a' and a positive integer n, loe define a" as fo ;r

rt" = rt X X X... X fl (» factors)


ks
a” is called the n^^‘ pozver of a. The real number a is called the base and n is called the exponent (index)
Yo

of the n^’’ power of a.


oo

It follows from the above definition that:


B

( 3
_3 3 3_^ -SV -3 -3 -3 -3 81
re

(i) 2^=2x2x2=8 (ii) - (iii)


8 4 ) ~4''4’^4''4'256
For any non-zero real number a, we define a^ = 1.
u
ad
Yo

f ( '1'^
Thus, we have 3° = 1,5° = 1. - 1, -- = land so on.
u V 3J
d
Re

NEGATIVE INTEGRAL POWER For any non-zero real number 'a' and a positive integer n, we define
in

1
F

n
H
a

Thus, we have

fsy^ 1
(i) '
1 1 1 1
(ii) -
5^ 5x5x5 125 \ ; I 2 (3/2)^ 3/2 X 3/2 X 3/2 27/8 27

/IV^ 1 1 1
(iii) i
s 5 / (1/5)2 1/5 X1/5 1/25
= 25

Thus, we have defined a" for all integral values of n, positive, zero or negative, a" is called the
?Jth power of a. The real number a is called the base and n is called the exponent or index of
the ?ith power of a.

2.1
2.2 APPLIED MATHEMATICS-XI

2.3 LAWS OF INTEGRAL EXPONENTS

In this section, we shall study the laws of integer exponents of real numbers. These laws are
similar to the laws of integer exponents of rational numbers which we have studied in
earlier classes.

FIRST LAW If a is any real number and m, n are positive integers, then a'" xa" = a
m+n

V u Using the definition, we obtain


a'" xfl" to m factors)x(ax ax ... to n factors)
= ax ax ax.. .to {tn + n) factors

ow
m*n
= a

m+ii
Hence, a'" xa" = a

2 /2 2V
iLLUSTK.VnON t (i) 5*^x5^ =5^^^^ 5^ (ii) I X 3j 3J 3J

e
V J /

re
in

rFl
SECOND LAW Ifa is a non-zero real number and m, n are positive integers, then
a
n
=a
m-ri

F
a

We shall divide the proof into three parts:


(i) when m > n (ii) when m = n and (iii) when m < n.

r
ou
I When m > n: In this case.
fo
ks
a axaxa...tom factors
n
a ax ax a... ton factors
oo

m
a
= flx (7 X (7... TO (m-n) factors [Cancelling n factors in and D^]
Y

It
B

m
a
re

in-ii
= a
n
a
ou

When m = n: In this case, we have


Y
ad

III m
a a axaxa... m factors
II 111
a a axaxa...m factors
d

Cancelling common factors in N'^and


in

= 1
Re

= a
0
[●.● 77*’ = 1 (By definition)]
F

= a
Ill-Ill
[●.● m-m = 0]
= a
iii-ii
[v« = m]
When m < n: In this case,
III
(7 ax ax a... tom factors
II
a <7Xi7Xff... to n factors

17X77X77... to (» -m) factors Cancelling m common factors in N'^and


1
ii-m
77

1
= a
-(II-IU) —r- = 77
a
k *'(by def.)
INDICES 2.3

m
a m-n
Hence, tl
= a , whether in > n or, m = n or, m < n.
a

2 f 2'\^ 2n5-2 (2
ILLL STU-IIOn: (i) 3^-^3^ =3^"^ =3^ (ii) ^ " 3) 3) 3)

THIRD LAW Ifa is any real number and m, n are positive integers, then (/?"')"= a”"'= {a")"'.
; i Using definition, we obtain
(a"‘)'' = a"‘ X a'" x a"' x... to /i factors

ow
= {axax.. .m factors) x x x... to m factors)

x{axax...tom factors)... to ?j factors

= rtx/7xflx...to{mn)factors = a
nrn

e
re
Similarly, we obtain (rt")
m nw
= a

Frl
F
Hence, {a”')
}j tun
= a = (fl")"'.

fpV Y [2
n,4x3 /
2\12
ou
or
ILLUSTKATluv (i) (3^)^ =3
2x5 10
= 3 (ii)
3) kfs 3J K3J
FOURTH LAW If a, b are real numbers and m, n are positive integers, then
a
oo

(i) {nb)''=a”b
ri
Y

(i) Using definition, we obtain


B

(rtb)" = (ab) X (rtii) X (ab) x to n factors


re

= (fl X X ... to n factors) x (ft x fj x &... to factors)


oYu
ad

= a’'xb'‘=a"b

(ii) Using definition, we obtain


d

fa\ fa\ axaxa...ton factors


in

a a a
Re

x..Aon factors =
b)
II
/ V b bxbxb ..Aon factors b
F

2.4 RATIONAL EXPONENTS OF A REAL NUMBER

In section 2.1, we have learnt about integral powers of a real number. In tliis section, we shall
define rational powers (exponents) of a real number. This requires the knowledge of the
principal root of a real number for a positive integer n which is defined as follows:
PRINCIPAL /?' ROOT OF A POSITIVE REAL NUMBER If 'a' is a positive real number and n is a
positive integer then the principal nth root of a is the unique positive real number x such that x" = a.
The principal rootof a positive real number a is denoted by or.
Thus, or !^fa is the posi tive real number a: such that :c
17
= a.

V2
For example, 16 is the positive real number x such that x^ = 16.
In other words, 16'^^ is the positive real number .r whose square is 16.
2.4 APPLIED MATHEMATICS-XI

Clearly, such number is 4. Therefore, 16^^ = 4.


Similarly, 8^^ is the positive real number x such that =8 .
We find that such number is 2. Hence, 8^^^ = 2.
The principal fourth root of 81 i.e. is 3, because 3'^ =81.
C TL (i) The principal 2^*^ root of a positive real number'a'is is also denoted b}/ y/a and is also
knozvn as the square root of a.
(ii) The principal third of a posivite real number a is a^^^ or, }fa which is also called the cube root of
a.

iLLUSTRAl ION Find the values of


n1/3 \V5
f 8 32 f 1
(i) (ii) (hi)
1.27; 243 J 64

w
SOLUTION Using the definition of the principal n* root of a positive real number, we find
that

Flo
xl/3
3_ 8
(i)
K27J
= A positive real number x such that x

ee
2 (2 _2 2

Fr
3 y.3j " 3 "" 3"" 3 “ 27
32 V' 32
(ii) = A poisitive real number .r such that for -
ur
243 243

2 2 2 2 2 2 2 32
s
= —X —X —X — X — =
3J 3 3 3 3 3 243
ok

3
Yo

xl/6
( 1 6_ 1
Bo

(iii)
64
= A positive real number x such that
re

1 1 1 1 1 1 1 1
=—X—X—X—X—X—=
2 2J 222222 64
ou
ad

Thesi/mbol 'f~’ usedin ifa is called the radical sign.


Y

PRINCIPAL ri'- ROOT OF A NEGATIVE REAL NUMBER If a is a negative real number and n is an odd
positive integer, then the principal root of a is defined i7s -] i.e. the principal nth root of a is
nd
Re

minus of the principal n^'‘ root of \a\.


Fi

For example,
(i) (-8)^^=-8’^^=-2 v2^=8 .-.8^^^ =2

(ii) (-243)^^^ = -(243)^^^ = -3 v3^=243 .-.(243)’^-‘^ =3

(iii) (-128)^^ =-(128)^^=-2 v2^=128 .-.(128)’^ =2


REMARK If a is a negative real number and n is an even positive integer, then the principal n”' root of
a is not defined, because an even power of a real number is always positive. Therefore, {-9f^ is a
meaningless quantity, if we confine ourselves to the set of real numbers only. But, (-9)^ is
meaningful in the set of complex numbers.
P
RATIONAL POWERS (EXPONENTS) For any positive real number a and a rational number ^' where
q>0, we define = (a^' i.e. is the principal q^'' root of .
INDICES 2.5

For Example:
(i) 4^^={4^^={64:f^=8 V 8^ = 64

(ii) (-8)^ = = {64f^ = 4 V 4^ =64

(iii) {-4f^=[{-4ff^={-64f^, which is not meaningfull because there is no


real number x such that = -64.

(iv) (27= (27^ = (729= 9 9^ = 729


REMARK In the above discussion we hove introduced the concept of rational exponent of positive real
number. We can also define real exponents of a real number, but it is beyond the scope of this book.
2.5 LAWS OF RATIONAL EXPONENTS

In section 2.2, we have learnt about the laws of integral exponents of a real number. Using
these laws and the definitions of and a^’^f q>0,a>0,it can be shown that the following

w
laws hold for rational exponents:

F lo
(i) a
m
xa
n
=a
m+n
(ii) a
m
-r-a
II
=a
m-ii

-n 1
(iii) («'")
n mn
= a
(iv) a
n
a

e
Fre
m/n m

(v) a

a a
III
for
(vi) (ab)"‘=a'"b
III

(vii)
bin'
r
where a, b are positive real numbers and m, n are rational numbers.
You
oks

We shall now illustrate some applications of these laws in simplifying expressions


involving rational exponents of real numbers with the help of following examples.
eBo

ILLUSTRATIVE EXAMPLES
our
ad

Type I ON SIMPLIFYING THE EXPRESSION BY USING LAWS OF EXPONENTS


FX.-MPLE 1
Simplify each of the following:
n+2 17+1 77+3 77 + 1 77 + 1
-3x7
(ii) I9x5"-2^x5
-6x5 16x2 -4x2
71
dY

(i) T
Re

20x7"-2x7
n n (iii) «+2 n+2
16x2 -2x2
Fin

71+2 77 + 1
7 -3x7 7"x7^-3x7x7 n
49x7"-21x7
SOLUTION (i) -
20x7"-2x7
U
20x7"-2x7
tt
20x7”-2x7
n

(49-21)7
II
28x7 28 14 14
- = —x7"""= —y? 0
(20-2)7
77
18x7" 18 ^ 9 9

577+3 -6x5
77+1
5"x5^-6x5x5" 125x5"-30x5 II

(ii)
9x5"-2^x5"~ 9x5"-4x5 II
9x5"-2^x5 II

II
(125-30)5
77
95x5 95
= 19x5^ = 19x1=19
n-ii
= —X 5
(9-4)5
n II
5x5 5

77+1 II
16x2 16x2x2"-4x2
II
-4x2
(iii) 77+2 77+2
16x2 -2x2 16x2^x2"-2x2^x2 II
2.6 APPLIED MATHEMATJCS-XI

32x2"-4x2" (32-4)x2" _28x2" _ 28 _ 1


64x2"-8x2" ”{64-8)x2"'56x2" 56 2
EXAMPLE 2 Simplify:
(3" -2x3"“^)(2 -3x2^""^)
2h
4h-1
3^ X (81)"-243 x3
n
4x3
(i) (ii)
3^x9^" 3"''^-3
n
3”“^ ^4»i+3 _ 2^”^
4n-\
3^x81
n n
243x3 4x3
SOLUTION (i)
3^x9^" 3"^^-3
n

3^ x(3^)" -3^ x3
4/1-1 H

w
4x3

3^ x(3^)^" 3//+1 -3

4/I-1+5
3^ X 3^” - 3
n
4x3

lo
3^ X 3^" 3/1+1
n
-3

e
34/1+5 -3 4/1+4

re
n
4x3

^4w+3 3" X 3 - 3
n

rF
F
34/1 + 5 34/1+4
n
4x3

r
34/1+3 34/1+3
fo
u
4
4/I+5-4/I-3 4/1 + 4-4/J-3
= 3^-3-2x3°=9-3-2 = 4
ks
/1-/1
= 3 -3 X 3
2
Yo
oo

(3" -2x3'"^)(2 -3x2^"'^)


2/1

(ii)
3"'4 (4"-"^ - 2^")
B

(3"-2x3"x3"^)(2 - 3 X 2^" X 2"^)


2/1
e
ur

3/1-4 ^(22)/i+3 _22/«)


ad
Yo

3" (1-2x3"^)., 2^" (1-3x2'^)


3/1-4 ^ ^22«+6 _ 2^")
d

2^"(l-3x2~^)
Re
in

= 3"-(«-4)(i_2x3"^)x
2^"(2^ -1)
F

1-3 3^ X 7 1
= 3^ 1 -IL = 3^ X — X
7 1

9) 63 9 4x63 3^ X 2^ X 3^ X 7 4

\V4

(0.3)'" X (9)^^ X (0.81)^^


\^' J
EXAMPLE 3 Find the value of
IT" -V4
(0.9)^^ X 3-^ X X(243)
3;
\V4
1
(0.3)^^^ X X (9)^^ X (0.81)^^
27
SOLUTION
IV^
(0.9)^'X 3-^^ X ±
-V4
X(243)
3J
INDICES 2.7

\V3 A 2/3
( 3 f 1 81
^ T X(3^)^^ X 100
10

X 3-’^ X (1
9 -V4

10 3j
X
(3^)
.._§L = il = 3'* X 10"^
■ 100 10^
(3‘^" x10"^''3)(3-^)^^ (3^)^^ X (3^x10'^)^^ X

(3^ X 10"^)^^ X 3"^ X (3"^)‘^ X (3^r^^“ and—= — = 3^x10-*


10 10

(3*^ X 10“^^) (3’^)’^^ X (3^^) X ((S'*) (10-2)2/3 j


2/3
X

(3^)^^x(10“^)^^x3“’^^x3^ X 3"^^^

w
_1_4 '
------V 10"3"3
33 4 3 3
-V3 -4/3
(3^^ X 3~^^^ x3^^ X 3^^) (10 xlO

F lo
J V

(3^^^ X 3“*^^ X 3^ X 3“^^‘*)(10-^^^) ' i_i+2-5V '


33 2^ "4 10 3

ee
/ \

Fr
31 19 5 2
j31/12 X 10
-5/3
3
= 3^2 12 X 10"3^3 =3x10 -1
319/12 xlO
-2/3 10
for
ur
V2 -|V2
11/16
EXAMPLE 4 If X = 9 the value of a: a;
s
ook
Yo

1/2
nl/2 1/2
-11/2 1/2 nV2
{
11/16 -V4l 11/16 5/41
SOLUTION X X ■ X a: ● a: X X - X X{X
eB

n1/2 1/2
_ _^11/16 XXX
5/8
= X
11/16
= X 11/16 ^ _^13/16
our
ad

11+13 3 3
2x-
= x = x2 = (9)2 =3 2 = 33 = 27
Y

9" X 3^x3"-27 1
II
Re

EXAMPLE 5 // =—, prove that m - n = 1.


nd

33m x2^
SOLUTION We have.
Fi

9" X 3^x3"-27
72
1

33m ^ 23 27

(3^)"x3^x3”-(3^y 1

3^'"x2 3 27

3^" X 3^x3"-3^" 1

3^’" X 2'-' 27

22ii+2+ii -3 3/1 1

3311. x2^ 3'^


3311+2 -3 3/1 1

3^"' X 2^ 3^
2.8 APPLIED MATHEMATICS-XI

3^"x3^-3^" 1
33^ x2^ 3^

33«(32-1) ^
3^'"x8 ~3^
3^"x8 1 3n -3m
= 3 ^ => 3n-3m = -3 => n-m = -l => m-n = l
33m x8 3^' ^ ^

ow
Ti/fW II ON PROVING RESULTS USING LAWS OF EXPONENTS
EXAMPLE 6 Assuming that x is a positive real number and a, b, c are rational numbers, show that:
( f ..a V
X X X X X
=1 =1
(i) (ii)
\x^
b
X^
a
X X^ x“

e
\a^+flb+b^ \ii’+£>c+c^ f \<^ +w+a‘

re
a:" X
a
X X
(iii) = 1 =1
(iv) x^ a

Flr
X

F
SOLUTION (i) We have,

'tfx a Y
ou
={x^-y {x^-‘^f {x‘'-^f

sr
x^ / V
X
a

/ V
x^

(ii) We have.
= X
ab-ac
XX
bc-ab
XX
ac-hc
= x
ab-ac+bc~ab+ac-bc
fo
k 0
= X'^ = 1
oo
Y
X X X

^^c~a^Vac {a-b)/ab ^ ^(b-c)/bc XX (c-a)/ac


reB

X^ x^ X
n = X

1 _1 1_1 1_]
uY

b II b a c b a c 0
= x XA*^ f’XX'’ =X = X^=1
(iii) We have,
ad

^ N^J^+bc+c’ X'^ \c^-¥ca+a^


do

a
X X

X
b
X
n _ ^^a-by^+ab+b^ ^^b-c-^b^+bc+c^ +ca+a^
in

{a-b){a^ +ab+b^) (b-c){b-+bc+c^) {c-a){c^+ca+a~)


Re

= X X X X X
F

a^-b' b^-c^
= X X X XX^' -a^ = x
a^-bYb^-cYc^-a^ 0
= X'^=1
(iv) We have.
\i>+c
a f c
X X

V A*’
●*
^(^a-i>)a+fc(^b-c)l7+c(^c-a)c+a
= X (fl-6)(fl+b) ^ ^(t-c)(i'+c) XX
(c-fl){<r+a)

a‘-b^ a^-b^+b’-c’ +c^-a^ 0


= X X X = x = a"=1

EXAMPLE 7 Ifx is a positive real number and the exponents are rational numbers, show that:
>^a+b-c ^ ^ -^b+c-a ^ ^ \c+a-b
(x“ X X
=1
(i) X
b a

^ Na^+b^-ab^ \b‘+c’-bc \f’+a^-<n


X a" 2(a’+b’+c’)
(ii) -b
= x
X x-‘ x-“
INDICES
2.9

SOLUTION (i) We have.


X
\fl+b-c X j, \b+c-a X
a
X

X' x^ X
a

_ ^y.a-bY*b-c ^^b-c^b*c-a
= X
{a-b){a+b-c) (b-c)(b+c-a) {c~a){c+a-b)
XX XX

= X (a~b){a+b)-c{a-b) XX {b-c)(b+c)-a{b-c) XX {c-a){c+a)~b(c-a)

ow
a' -b^ ~ca+bc b'-c^-ab+ac a^-b'^-ca-t-bc+b^-c^-ab+ac-i-c^-a'^-bc+ba 0
= X XX = x = x'^ = l
(ii) We have.
g \a^*b'-ab sfe’+c’-bc +a^ -ca
x^ X^ )

e
X

-b

re
~a
x

Frl
(^h+C)il*+C^-{)C (^c+fl)C'+fl^-ca

F
= X
{a+b){a^+b' -ab) X X
{b+c){b^+c^-bc) (c+fl){c^ +a*-ffl)
X X
ou
fl'+i.’ i.’+e’ c'+fl' fl’+ii’+b’+c’+c*+fl’ 2(fl’+2>*+c’)

r
= X XX XX = X = x

EXAMPLES Show that:


so
kf
bY
(i)
X
= 1 jx‘'^‘’f{x^*‘f{x'^‘’y =1
(ii)
oo
^Ha-c) ● X
a
Y

SOLUTION (i) We have,


B

X
o{b-c) r..bY ab-ac
X X 1
_ ^{ab-ac)-{ba-bc) ^
re

ha~bc
^(b-a)c
a
X X
oY
u

1
_ ^ab-ac~ba+bc ^ -ac+bc ..ac-bc -ac+bc+ac-bc 0
ad

bc-ac
= x ■X = x = a:" = 1
X

(ii) We have.
d
in

(:^a+b)2(^£>+C)2(^c+a)2 ^2(a+b) ^ X ^2a+2&.. ..26+2c XX 2c+2n


X X
Re

ix^)Wix^r 4a 4b
X’ XX"XX
4c
F

2a+21i+2l)+2c+2c+2a 4a+4ft+4c
X X
4a+4ii+4c 4a+4i’+4c
=1
X X

EXAMPLE 9 Show that:


1 1 1
=1
l + x'”‘’ + x c-a
1 + X" “ + 'A
a-b x-b
1 + X' ' + X
b-c a~c

SOLUTION Multiplying the numerators and denominators of three terms on LHS by X


n
,x‘’
and respectively, we obtain
1 1 1
LHS = b~n a-b
c-a b-c
1 + x‘ ●■ + X 1 + X" “ + X 1 + X' ‘ +X
a-c

a b
X X x^
li-a+a
x^ + x"-'’^^
a c-a+a c-fe+i? a-c+c
X +X + x + x + x
APPLIED MATHEMATICS-Xl
2.10

X
a
x^’ x^ x° +x‘’ + x“^
= 1 = RHS
x” +X*’ + x'^ x^’ +x“ +x'^

EXAMPLE 10 Ifabc = 1, show that


1 1 1
=1
1+a+b ’ 1+b+c
-1
1+c+a ^

ow
SOLUTION We have,
1 1 1
= 1
1 +a + b
-1
1+b +c
-1
1+c+a ^
1 1 1

e
, 1 1 1

re
l +a + - l+b + 1 + C +-
b c a

Frl
b 1 1

F
b + ab + 1 1 + b + ab 1 ●: abc =l:.- = ab and c = —
+ - c ab
ab a
ou
sor
b 1 ab _b + l + ab =1
b + ab + 1 1 + b + ab ab + l + b b + ab + 1 kf
a
-1
a-^ 2b^
oo
EXAMPLE n Prove that:
a ^ +b ^ a
-I
h-^ b^-a^
Y

SOLUTION We have,
B

1/a 1/a _1 ab ^ 1. ab
-1 -1
a a Ua 1/a
re

a-^ + b-^ a-^-b-^ 1 1 ^ b +a a b-a


oY

- + —
a b a b ab ah
u

h b{b-a) + b{b + a) b^ -ba + b^ + ab Ib^


ad

b
b^-a^ b^-a^
d

b+a b-a (b+a) {b-a)


in

EXAMPLE 12 Ifx, y, z are positive real numbers, show that yjx~^y x Jy^ x 4z~^ = 1 ■
Re
F

SOLUTION We have.
\V2
y 1/2 ^V2 ^1/2
y\x ^yxyjy^x-J z
-1
X =
\y
— X |— X
2
Z \ X y
-T X

y
V2 ^
= 1
X y

III ON SOLVING EQUATIONS INVOLVING EXPONENTS OF NATURAL NUMBERS

EXAMPLE 13 Solve the following equations:


(i) 2*“^ =256
J.+3 *-1
(ii) 2 = 4

SOLUTION (i) We have,


2-V-5 = 256 =?> 2
x-5
= 2® => .r-5 = 8 => x = 8 + 5 => x = 13

(ii) We have,
>2^x+3 = 4
x-\

=> 2^x+3 ={2^r-^


INDICES 2.11

2-r+3
x + >? = 2{x-\) => x + 3 = 2x-2 => 2x-x = 3 + 2 => x = 5

EXAMPLE 14 Solve the following equations for x:

3V
~lx
2x 1 17
(i) 4 (ii) = 4—
32 vis 27

2^^ 8
(iii) 3 5°+t = (0.6) 3-2x (iv) 2^(5°+3^^)=8 —
27
3J

ow
SOLUTION (i) We have,

42x — => (2^f^


32
= 4^2
25
4x
= 2'^=> Ax = -5 => x = -~
4

e
[On equating the exponents]

re
(ii) We have,

rFl
F
n1-2x l-2x'l 1/2 1 - 2-v
, l-2x
(3 17 (3^ 125
3^ 2 rsV f3Y^ (3 N-3
= 4—
vis 27 s; 27
s; 3) Is s;

or
ou
l-2.r 7
= -3 ^ 1-2a: = -6 => -2x = -7 => x = —
ksf
=>
2 2
On equating the exponents
(iii) We have.
oo

2^^
Y

3-2x
3 5'’+-
B

= (0.6)
3;
re

1/3
2^^
1+- 34^"" v5«=l
oYu

3) Is,
ad

3-2x
rs
d

K3J 3)
in
Re

2/3
rs SV"^ 2 2 11 11
F

^ — = 2x - 3 ^ 2.V = — + 3 ^ 2.Y = — ^ 4: = —
13 3j 3 3 3 6

(iv) We have.
8
2^ (5°+3^^)=8 —
27

2^(1+ 3^-’^) = 8 +—
27

8
8(l + 3^^) = 8 + —
27

8 1 -3
8 + 8x3^-''=8 + —=> 8x3^* =8 + —=> 3 2i => 3
2i-
= 3"^=>2.r = -3=>x =
27 27 27 2
2.12 APPLIED MATHEMAUCS-XI

EXAMPLE 15 If =720 + 9"-'//»*^^ the value of


SOLUTION We have,
gX+2 = 720 + 9'’
9’= X 9^ =720 + 9^'
9^x9^-9" =720
720
9^(9^-1) = 720 => 9^x80 = 720 => 9^ = 80
= 9 => 9’^ =9^ => x = l

(4y)^^’"=(4xl)^ =4
.Y-l 2x-l
EXAMPLE 16 If 25 = 5 -100, fiud the value ofx.
SOLUTION We have.

w
.r-l 2.V-I
25-' =5 -100

F lo
(5^r’=5
2.V-1
100

=> 52Y-2 -5
2y-1
= -100

ee
Fr
52Y-2 -5
2.T-2
x5' =-100

52.V-2 (1-5) = -100 for


ur
52Y-2 x^ = -100
s

52Y-2 = 25 => 5
2.T-2
= 5^ => 2a--2 = 2 => 2a: = 2 + 2 => 2:c = 4 => x = 2
ook
Yo

.t+2
EXAMPLE17 Solve the equation: 3(2*+l)-2 + 5=0
eB

SOLUTION We have,
y+2
3(2’^ + l)-2 + 5=0
r
ad
ou

3x2-^+3-2'^x2^+5 = 0
Y

3x2" -4x2^+8=0
Re
nd

(3-4)2"+8 = 0 => -2"+8 = 0 => 2" =8 => 2" =2^ => ^- = 3


Fi

EXAMPLE 18 Solve the following equations for x:


(i) 2^"""* =17-2"-2^ (ii) =6-5"-l
SOLUTION (i) We have,
22y+1 = 17-2"-2^
2-2^"=17-2"-2^
2-(2")^=17{2")-8
2y' = 17y-8, where 2" = y
2y^-17y + 8=0
2y^-16y-y + 8 = 0
2y(y-8)-(y-8) = 0
INDICES 2.13

(y-8)(2y-l) = 0
1/ - 8 = 0 or, 2y -1 = 0
1
y =8 or, y = - => 2" = 2^ or, 2-'^ =2"’ => x = 3 or, a: = -1

(ii) We have,
52..1 = 6(5^)-1

5{5^^) = 6(5^')-l
5{5^)^-6(5'") + l = 0

ow
5y^ -61/ + I = 0, where y =
5y^-5y-i/ + i = o

e
5y(y-i)-(y-i)=o

re
{5y-l)(y-l) = 0

Fl
F
5y-l = 0 or, y-l = 0
ur
r
1

y 5 or, y = l 5-’^= 5
-1
fo
or, 5'’^= 5" => x = -l or, a: = 0
ks
EXAMPLE 19 Solve the following equations for x and y:
Yo
oo

4- —
(i) 3^=9x3“'fl«rf 8x2^ =4^ (ii) (^/32f ^2^-^’ 16 2_sy = 0
eB

SOLUTION (i) We have,

3^=9x3''and,8x2^=4^'
ur
ad

3"^=3^x3-^and2^x2y=(2^)"'
Yo

3" =3^^y and 2^^^'=2^-"


d
Re
in

a: = 2 + 1/ and3 + y = 2Af [On equating the exponents]


F

a: - y = 2 and 2x-y = 3

(2a: - y) - (.V “ y) = 3 - 2 and (2a: - 2y) - {2x - y) = 4 - 3

a: = 1 and - y = l => a: = 1 and y = -1


(ii) We have.
4_£
(V^)-'^+2i'^’=landl6 2_gy=o

{(25)V2|%2*"'=1 and -(2Y = 0


4 4-^
(2^^)* +2^^* = 1 and 2 2J = (23)y
2.14 APPLIED MATHEMATICS-XI

5x

2^
= 1 and =2^^
2^+1

5-v
-(y+1) and =2^^
2^ = 1

5.V
-.V-1 16-2x
_ 2^y
=> and 2
2^ = 2
0

-x-y-l=0 and 16-2jf = 3y [On equating the exponents]


2

w
=> 5a: - 2y - 2 = 0 and 2:ii: + 3y-16 = 0
Thus, we have the following system of simultaneous linear equations:

F lo
5A:-2y-2 = 0

ee
2.v + 3y-16 = 0 ...(h)

Fr
Solving these equations by cross-multiplication, we get
1
X
y for
ur
32 + 6 -4 + 80 15 + 4
s
X y \ 38 , 76
— = — =sx = — and V =
ook

=> a: = 2 and y = 4
Yo

38 76 19 19 ^ 19

EXAMPLE 20 Solve the following pairs of equations:


eB

(i) 2^^\ 2^^^^ = 8 (ii) 2^+3^^ =17


. 2-’'^^ = 16
r

2^+2 _ 3y+i ^ 5
ou
ad

SOLUTION (i) We have,


Y

2^+1 2^^^^ 8 and 2-t+2_2y+2 ^

—s 2^'^I'^3y+l 2^ and ^jt+2+y+2 _


Re
nd

—s 2-'’"^3y+2 2^ and
Fi

=> .T + 3y + 2 = 3 and a: + y + 4 = 4
=> a: + 3y = 1 and x +y = 0
On subtracting second equation from first, we obtain

2y = 1 => y = I
Putting y = ^ in any one of the two equations, we obtain x
1
2

Hence, x = y = -i is the solution of the given pair of equations,


(ii) We have.
2^ + 3-‘^ = 17

2*+2 _ 3y+i ^ 5
INDICES 2.15

=> 2" + 3-'' = 17

2^ X 2^ - 3 X 3^ = 5
=> 2-'^ + 3^ = 17

4 X 2^ - 3 X 3 " = 5
=> li + y = 17

4u -3v = 5, where u = 2^,v = 3^


Multiplying first equation by 3 and adding to the second equation, we obtain
7u = 56 => » = 8 => 2^ = 2^ X =3
Putting w = 8 in H + t? = 17, we obtain

w
u = 9 3-" = 3^ ^ y = 2
Hence, x = 3, y = 2 is the solution of the given pair of equations.

o
EXAMPLE 21 Solve the following system of equations:

e
2X+1/+Z _

re
SOLUTION We have.

rFl
F
2Z+V+Z = = 25
i’+Z
. ^x+y+2z _ ^3y+y
3J-+I/

(23^-.-53y.2 ^^52p,32x.y.2z
r
=> 2*^y
+z
ou
^ 2-'^+y+^ = 53y+2 _ ^2x+Zz. ^Ix^y+lz _ 361+2^
fo
ks
=> X + y + 2 = 3x - 3y + 32 ; 3y + 2 = 2j: + 2z; 2.r + y + 22 = 6x + 2y
oo

^ 2x - 4y + 2z = 0 ; 2x - 3y + 22 = 2 ; 4x + y - 22 = 0
Y

Thus, we obtain the following system of equations


eB

...(i)
X - 2y + 2 = 0
●●●(ii)
2x - 3y + 22 = 2
r
ou

...(iii)
Y

4x + y - 2z = 0
ad

From equation (i), we obtain


z = -X + 2y ...(iv)
d

Substituting this value of 2 in (ii) and (iii), we obtain


in
Re

2x-3y + 2(-x + 2y) = 2 and 4x + y - 2(-x + 2y) = 0


F

y = 2 and 6x-3y = 0 ■=> y=2 and x = 1


Putting X = 1, y = 2 in (iv), we obtain: 2 = -1 + 4 = 3
Hence, x = 1, y = 2 and 2 = 3.
EXAMPLE 22 Ifa,b,c are distinct positive prime integers such that c^ = 49392, find the values
of a, b and c.
SOLUTION By prime factorization, we obtain; 49392 = 2“^ x 3^ x 7^
49392

flW=2^x3^x7^

flW=3^x7^x2^
2.16 APPLIED MATHEMATICS-XI

a = 3, b = 7 and c = 4
a, b and c are primes]
Hence, a = 3,b = 7 and c = 4

Z’ -1 2
fA
-3
.r
a.b
EXAMPLE 23 // ^ y ' prove that a + b= -1, where x and y are different
positive primes.

w
SOLUTION We have,

x-yT (xy = x y
a..b

x-y

e
6.2y-3-3)V2^^y

ro
re
(x-yf\[xy^)
V2
a^.b
= xy

F
Fl
X-V3y4/3
xY^
= xy
a..b

u
sr
4 4
---4 Y a,.b

ko
X ^ 1/3 =x"y
o
-16 13
of
AT 3 y3 =x‘*y^
o
-16 13
Y
a = and b = —
3 ['.● X and y are primes]
erB

a +b = -16 ^ 13 -3
uY

3 3 " 3
1
EXAMPLE 2-1
If x = \/28 aiid y = ff27,find the value of x + y
x^ +xy + y^
ad
do

SOLUTION We have.
1
in

x + y-
x^ +xy + y^
Re

{x-y)
F

= x + y-
(x-y) {x^^xyyy'^)
{x-y)
-x + y-
x^-y^
{x-y)
= x + y-
28-27 ●: X = VtE and y = x^ = 28, y^ = 27 ’
= x + y-(A:-y) = 2y = 2x3 = 6 '■.■y^^ffT? ^{3Y^ =3
EXAMPLE 25 If X = 3, find the value of +x
-1/3

SOLUTION We have,

(X^+X~Y{X^^ + X -2/3 -1)


INDICES 2.17

-V3
XX

1 10
= =A: + a:-^=3 + 3-*=3 + ^ = ^ o
[v {a + b) (a^ -ab + b^) = a^ + b^]
nxAMPU If X = a
vs
+ a
-V3
, prove that x^ - 3x = a + -. a

SOLUTION We have,

ow
-vs
X = + a

[On cubing both sides]

e
1 1 1 1
r»3 x3

re
1
x^ = + fl 3 ^ 3j^3 3-^ ^ x^ = a + a^ + Sa'^x => .r^ - 3:t = + -
a

Flr
F
EXAMPLE 27 If x = {2 + + (2 + S)~^\ prove that
ou -3x-4 = 0-

SOLUTION Replacing by (2 + ^^) in Example 26, we obtain

sr
1
X"'* - 3x = (2 + V3)

fo
(2-V3)

-3x = {2 +s/3) +
(2 +V3) k
oo
(2 + ^/3)(2-V3)
Y

(2 + ^)
reB

x^-3x=(2 + V3) +
4-3

x^-3x = 4
uY

x^-3x-4 = 0
-vs

AL1T1-:k We have, x =(2 +Vs+ (2 +V3 )


ad
do

-1/3 3
^ .v^={(2 + V3f%(2 + V3)
in
Re

=> =|(2+^/3)’^^ +|(2 + ^/3)‘^^| +3(2 + ^/3)^''^(2 + ^/3)“^^|(2 + ^/3)^^+(2 + ^/3)


F

-V3

=> x^-(2 + V3) + 2 + V3) ^+3x


1
x3=(2+V3) 2 + ^/3
+ 3x

=> x^-3x = (2 + V3)


(2-V3)(2 + V3)
(2-V3)
=> x^-3x = (2 + n/3) 4-3

=> x^-3x = (2 + V3) + (2-V3)


=> x^-3x-4 = 0
2.18 APPLIED MATHEMATICS-XI

Find the value of: (5 l)(5 +1)


V4
EXAMPLE 28

SOLUTION We have.

(5
V4

= (5^^'* -1) ((5^*^)^ + + {5^^^) +1)


V4
= {a:-1)(x^ + .v“ +a: + 1), where:c = 5
= (a--1) (:c + 1) + {j: + 1)

= (*-l)(x + l)(.v"+l) =(x"-l)(.r" + l) (5-)^ -1=5-1=4


Tt/pv iV MISCELLANEOUS EXAMPLES ON LAWS OF EXPONENTS

w
EXAMPLE 29 If a^ = b, = c and = a, prove that xyz = 1
SOLUTION

a
We know that

xyz

xyz
F lo for F
ree
■:a^ = b

vM = c

V c = a
a^^-=a
Your
ks

a
xyz
= a^ =>xyi~\
eBoo

Ixz
EXAMPLE 30 =c‘and b^ =ac, prove that y = x + z

SOLUTION Let a'' = b-' =c^=k. Then, « = and c = k^^


ad
our

Now, =ac

1-1 + i 2 x +z Ixz
{k^yf=k^^^k^^ j^2/y _ ^V.r + Vz => y =
Re

y X z y xz x + z
Y

, 1 1 1 11
Find

EXAMPLE31 //2-"=4-’'=8= and xyz = 288, then sho7U that ^ + ^ + ^ =


SOLUTION We have.
2^ =4^= 8=

=>
2^'=(2^)^ =(2^)"=>2^ =2^5^ =2^" =>.r = 2i/ = 3z.
Let x = 2y = 3z = ?t. Then, a: = X,y = ^andz = —.
X
●●●(i)
3

xyz = 288
rx vx
- =288 =i>X^= 2 x 3 x 288 =>^^= 2x3x2= x3^ =2*^3^
2J13
>.^=(22x3^ =>X = 2^x3 = 12
a: = 12, y = 6 and z = 4 [Putting A = 12 in (i)]
INDICES 2.19

1 1 1 1 1 1 11

^’^4y’^82 24 24 32 96
.v-3 2.T-8
EXAMPLE32 Find the value ofx, if 5 x3 = 225
SOLUTION We have,
5JT-3 x3 2i-8 = 225

5.V-3 x3
2.V-8
= 5^x3^
x-3 = 2 and 2a: - 8 = 2 [On equating the exponents]
a: = 5

EXERCISE 2.1
1. Prove that:
a \C b V / .X
1 1

w
X X
= 1 =1
(i) a-b b-a
X
vfl j
l +x l +x \ X J \ X V a:
■“ Prove that:

(i) a:
\a^+ab+b^
X
/
£_
t. \6^+k+c’
X
X^

F lo = 1

ee
.x‘^ J
a
\ X \X /

Fr
1 1 1
=1
(ii) b-a
\ + X" " + X
c-a
1 + AT ' + X
a~b c-b
1 + x^~‘^ + a: a-c

3. Show that:
for
ur
-\a*b
^a{a-b) ^b(b-a) '
s

= 1
(i)
ook

<i(a+ii) ^b(b+a)
Yo

a: X
eB

1 1 1

1 \c—f JL^b -a
(ii) Xa-b X
b-c
XC-a =1
our
ad

a+b /
X
6^+c^ A b+c / X
X
_^2(fl’+i>'+cT
(iii)
Y

be
xab
ac
X X
Re
nd

1 Q+1

(V) (-■■“ ')


fl-1
Fi

(iv) (^a-t)«4b(^.b-C)b4C(^C-«)C4«^l = x

f a^.r+1 V+y f a„y*2 y-^V a„Z43 \a+b / 1, ■\i’+c/ _c


3" 3 3
= 1
(Vi) = l(vii) 3^ 3^
y4l Z42 x+3
a a a 3"

( X..a^h f X..h^c y-V X


c+a
NC-rt
X
771
X
71

4. Show that (i) = 1 (ii) Im X 77171) — X "'^=1


'1‘
^x
b I
I a:" \x
771
a:*^

m
n ir
a +~ X a —
bj b) a

w. Show that:
^771
\” bJ
b +
1
x(b~-
a J V 0 J
2.20 APPLIED MATHEMATICS-XI

' Solve the following equations for x:


j-3 S.r+3 A-+3
(i) 72x+3^^ (ii) (iii) 2 =8

2.r 1 fiY
(v) 4-^-^x(0.5)
3-2a 3x-7
(iv) 4 (Vi) 2 = 256
32 8

(vii) 2^^^+3.2-"-"’=5(viii) +3.5"*'^ = 76 (ix)


4^
7. Solve the following equations for ;r:
2.V+4 x+2
(i) + =0 (ii) 3 + 1 = 2'3

8. If 4:9392 = a'^b'c^, find the values of a, b and c, where a, b and c are different positive
primes.

ow
9. Given 4725 = 3" 5^ 7", find
(i) the integral values of a, b and c (ii) the value of 2"" 3'’7^

10. li a = xy^~^,b = xy^~^ and c = xy''~', prove that a**”" i?'"”''c^“‘^ = 1.

e
Fl
re
11. If 2" =3●■' = 12^ show that- = - +
2 y AT

F
If 2●’^=3●‘'=6"^showthati+i + - = 0.
ur
X y z

13. If (7^ =M =c'and b^ =ac, then show that y = 2 + A'


2za:
or
k sf
14. If 2^ x3'^ X 5^ = 2160, find a:, y and z. Hence, compute the value of 3^x2"^x5~3
Yo
oo

If 1176 = 2'’x3^x7*^,find the values of a, b and c. Hence, compute the value of


B

2" X 3^ x 7 as a fraction.
e
ur

If,, (i) If rt = A;'"-^V/'’ = '-^''V'andc = A:'^'Y'/ prove that a’^-’'b”~‘c‘-”'=l


ad
Yo

u+l l+m
, prove that x"'y" z' =x’‘y'z’\
m+ii
(ii) If a: =77 ,y = 77 and 2 = 77

,V3 -V3

17. If a:=(4 + 7T5 (4 + VI5 , find the value of a:^ -3a:.


d

+
Re
in

1/2 -iVl
F

18. If a: = 81, find the value of y

19. Show that: (2^^-l) (2^^Y2^+2^Yl) = 1


20. Show that:
;i

n+1 H+1
-I 1-V2 i_i 'irr
1-
rt
.t
(i) = 77
(ii)
= .t

21. (i) If Y = 2^^ + 2^^ show that - 6x -12 = 0


(ii) If Y = 3^^ + 3“’^,showthat3.Y^-9Y-10 = 0

(iii) If Y=(V2 + lj^^-|V2-l , show that x^ + 3x - 2 = 0.


INDICES 2.21

ANSWERS

6.(i) X = -~ (ii) x = 7 (iii) x = 3


r(IV)^ -v=--5

(V) ^ = 1 (vi) x=5 (vii) -1 (viii) 0

(ix) I 7. (i) .t = 2 (ii) .y = -2


63
8, n = 2,b = 3,c=7 9. {i)a = 3,b = 2,c=l (ii) ^
81 24
14 x = 4, i/=3,z = l,— 16
10. a = 3,b = l,c = 2,—
40
17. 8 18. 729

w
HINTS TO SELECTED PROBLEMS

11. Let 2' = a" = 12' = k. Then, 2 = k^\3 = k^^^ and 12 = k^^

F lo
Now, I2 = k^^

ee
1
. ^„ 2 11 v2 = ic^^^and3 = Fi'
-

=> 2^x3 = ic^^" {k^^fxk^^=k Vz


=> => -+- =-

Fr
X y z

12. Let 2^ = 3^ = = k. Then, 2 = k^^\3 = and 6 = k~^^. for


ur
Now, 6 = k-^^
2 1
s
1
1
ook

=> 2x3 = r*^^ v2 = ic^^and3 = cf^^^'


Yo

■'■ y 2
eB

13. Let = b-' =c“ =k.Then,a = k^^,b = k^^,c = k^^


2 1 1
2 1 1
r

.-. b^ = 17C^ =>- = — + -


ou
ad

y ^ 2
Y

MULVPLE CHOICE QUESTIONS (MCQs)


Re
nd

Mark the correct alternative in each of the following:


1. The value of {2-3(2-3)^}\ is
Fi

(a) 5 (b) 125 (c) 1/5 (d) - 125


●'t-y
The value of a: -y when x = 2 and y = -2 , is
(a) 18 (b) -18 (c) 14 (d) - 14
3. The product of the square root of x with the cube root of :c, is
(a) cube root of the square root of x (b) sixth root of the fifth power of x
(c) fifth root of the sixth power of x (d) sixth root of x
4. The seventh root of x divided by the eighth root of x, is
1
(a) -V (b) ^ (c) ^ (d)

5. The square root of 64 divided by the cube root of 64, is


1
(a) 64 (b) 2 (0 ^ (d) 64'^^
2.22 APPLIED MATHEMATICS - XI

-V6

6. Which of the following is (are) not equal to


(iri 7

1 1
^,nV30

(6)
1

w I
r5^5'6
(a) T \V5
V6
\ o >

V6J
7. When simplified (x ^ +y ^ is equal to
m. x+y
(a) xy (b) x + y (c) (d)
x+y

8. If 8^+1 = 64 / what is the value of ?

w
(a) 1 (b) 3 (c) 9 (d) 27

9. If (2^)^ = 4^, then 3^ =

F lo
(a) 3 (b) 6 (c) 9 (d) 27

ee
10. If X ^ =64, then x^^ + x° =

Fr
(a) 2 (b) 3 (c) 3/2 (d) 2/3
n-2/3
( 1 for
11. When simplified \ , is
ur
2/ /
1 1
(a) 9 (b)-9 (0 5 (d)-5
s
ook
Yo

12. Which one of the following is not equal to


eB

1 1
-1/2
(a) (^) (b) 8-^^
(^)
1/2 (d) >/2
r
ou
ad

n-3/2
flOO 7
13. Which one of the following is not equal to
Y

\ y J
Re
nd

n3/2
f 9 1 3 3 3 '100 100 100
(a) (b) (c) lo'^io'^io (d) X X
Fi

\3/2
UOO/ 'm 9 9 9
[ 9 J
14. If a, b,c are positive real niunbers, then is equal to
1
(a) 1 (b) ahc (c) 4^ (d) ahc

/2Y/3^^^ 81
then x =
V 3, ^2 7 16'
(a) 2 (b) 3 (c) 4 (d) 1

16. The value of { 2"^ i


IS

1 1
(a) 2 (b) 2 (c) 4 (d) 4
2.23
INDICES

17. If (7, ft, c are positive real numbers, then is equal to

(a) 5a^bc^ (b) 25 ab^c (c) 5aV (d) 125 aV

18. If ,n are positive integers, then |''/W| is equal to


(a) a
«1H
(b) a (c) (d) 1

f,/
X y
19. If x = 2 and y = 4, then +

y \^ )

w
(a) 4 (b) 8 (c) 12 (d) 2

V4
-V3
( 1

Flo
20. The value of m for which = 7"', is
I72

e
re
1 1
(c) -3 (d) 2
(a) -3 (b) i

F
The value of {(23 + 2")^= + (140 - 29f^}", is
ur
21.

(a) 196 (b) 289 (c) 324


f or (d) 400
ks
0.09
22. (256)°-'^ X (256)
Yo
oo

(a) 4 (b) 16 (c) 64 (d) 256.25


B

23. If 10^-’'= 25 ' 10“^ equals


re

1 1 1 1

(a) -3 Cb) 50
(c) 625 (d) 5
u
ad
Yo

x+2
24. If 9 = 240 + 9^^, then x =
(a) 0.5 , (b) 0.2 (c) 0.4 (d) 0.1
d
Re
in

25. If :c is a positive real number and .v^=2, then


3
.t
F

(a) V2 (b) 2V2 (c) 3^/2 (d) 4

^ = 8a.' * and a: > 0, then a: =


26. If ;,.5
V2
(b) 2^/2 (c) 4 (d) 64
(«) 1
-1/2
27. If g = +U , what is the value of g when f=64?
31 33 257
(c) 16 (d)
(a) y (b) Y 16

.r-1
28. If 4’'-4 = 24, then (2.a:)'‘ equals

(a) SS (b) Vs (c) 25V5 (d) 125


2.24
APPLIED MATHEMATICS -XI

29. When simplified (256) / IS

1 1
(a) 8 (b) 8o (c) 2
(d) 2
32X-8 53
30. If =—,then a: =
225 5
(a) 2 (b) 3 (c) 5 (d) 4

31. The value of 64"^ (64^^^ -64^), is


1
(a) 1 (c) -3 (d) -2

w
32. If V^ = 125, then 5^ =
1 1
(a) 25

F lo
(b) 125 (c) 625
5
2ar+3
33. If (16) = (64)^^^ then 42at-2^

ee
(a) 64 (b) 256 (c) 32 (d) 512

Fr
34. If 2"'”x
4r
2'« = -/
4' then -1
14 |(4'”)1/2
^ fj_'
U'”J is equal to for
ur
1 1
(a) 2 (b) 2 (c) 4 (d)
s
ook
Yo

m+«
3P ^2m+n-p
35. If - = 16, —= 81 and a = then -1 “
eB

2^—fw
1
(a) 2 (c) 9 (d) 7
8
our
ad

3^^x81^x6561
36. If = 3^, then x =
3^^
Y
Re

1 1
nd

(a) 3 (b) -3
(d) -3
Fi

37. If0<ycjc, which statement must be true?

(a) V^->/y=V^ (b) yfx+yjx=^/2x


(c) x^ = y4x (d)

^+1
38. If 10* = 64, what is the value of 10^ ?
(a) 18 (b) 42 (c) 80 (d) 81
5"+^-6x5 n+l

39.
13x5"-2x5”'"^ is equal to
5 5 3 3
(b) -3 w i (d) -3
INDICES
2.25

/ 2| --4
40. If ^2" =1024, then 3
(a) 3 (b) 9 (c) 27 (d) 81

ANSWERS
1. (b) 7. (d) 3. (b) 'i- (c) 5. (b) 6. (a) 7. (c)
8. (d) 9- (d) :n. (c) 11. (a) 12. (a) 13. (d) 14. (a)
1-^- (c) 16. (a) 17. (a) IS. (b) 19. (b) (a) -- (d)
● (a) ^(d) 24. (a) 25. (b) 26. (d) (b) (c)
(d) 30. (c) 3i. (c) 32. (a) Cb) 34. (a) 35. (a)
36. (b) 37. (d) 38. (c) 39. (b) 40. (b)

w
FILL IN THE BLANKS QUESTIONS (FBQs)

(21^-15^] is equal to

F lo
1.

2- 81’^'*x9^/2^27"^^ is equal to.

ee
Fr
1

3.
(169)-^ 48
for
-8
(196)
r
4. If :c=8^^x32“^,thenx-^ =
You
s
ook

h-3
If 6" =1296, then 6
eB

6. The value of 4x(256)“^^'^ 4-(243)’^^ is


If (6a.')^=6^’, then x=
our
ad

^/^+^/97
\n/^-V97
8. a
dY

b )
Re
Fin

lY'V 1 \p*‘i
p+ p-
p
9. If y 1 Y~'^f 1 Y+‘f
„ ' then X =
‘1 + ‘7-
PJ PJ
10. If 5”^2 = 625,then(12« + 3)‘^^ =
1
3-7a 6-2a
a
11. If Xfl

2fl 9-2a
a .y a

12. If
(24^“^^^^^*^"^^' 5:>r + 6y=
13. If 6^“^=36 and3"^^^=729, then =
APPLIED MATHEMATICS - XI
2.26

14.
4^3^ equals ●
15. The product is equal to

^(81)"^ is equal to
0.09 J
17. The value of (256fx(256) IS

ANSWERS

14V3 1
1. 36 2. 1 3. 4. 1 5. 6
13
1/8 3
1

ow
10. 3 11. 12. 0
7. 6'^' 8. 9. 2p
1
13. 12 14. 2^^^ 15. 2 16. -
9
17. 4

SHORT ANSWER QUESTIONS (VSAQs)

e
Fl
re
1● Write (625) in decimal form.

F
2. State the product law of exponents.
3. State the quotient law of exponents.
ur
4. State the power law of exponents.
If 2“* X 4^ = 16^, then find the value of x . or
sf
X

6. If 3^“^ = 9 and 4^'^^ = 64, what is the value of “ ?


k
Yo
oo

7. Write the value of ^x^/49.


-VI
B

(W
8. Write x(64)”^^ as a rational number.
9)
re

9. Write the value of ^125 x 27.


u
ad
Yo

b+c C+fl
a
X
X X

10. For any positive real number x, find the value of


d
Re
in

V4
11. Write the value of ● 5 (8^^^ + 27^^)^
F

-V2\ -V4f
12. Simplify |(b25) J
1 1 1

13. For any positive real number x, write die value of |(x'')‘'}«'’{(x'')" ((x'-')" ●
14. If (;j: _ 1)3 = 8, what is the value of (x +1)^ ?
ANSWERS
3
1. 0.2 5. 2 6. 3 7. 7 8. 9. 15 10. 1
4

II. 5 12. 5 13. x^ 14. 16


CHAPTER 3
LOGARITHMS

3.1 INTRODUCTION

Sometimes we come across numerical expressions involving multiplication, division or rational


powers of large numbers. To simplify such expressions logarithms are very useful. In fact,
logarithms are very helpful in making lengthy and difficult calculations easy. In this chapter, we

w
shall first introduce the concept of logarithms, and then apply this technique to simplify
numerical expressions.
3.2 DEFINITION

F lo
If a is a positive real number, other than 1 and x is a rational mmiber such that = N, then we say that

ee
logarithm ofN to base a is x or, x is the logarithm ofN to the base a, written as log^ N =x.

Fr
Thus, = N <=> logfl N =x. for
NOTE 1 It should be noted that “log" is the abbreviation of the word “logarithm".
ur
It follows from the above definition that:

(i) 2^ = 32 o log2 32=5 (ii) 10^ =1000 logiol000 = 3


s
ook
Yo

(hi) S'* =81 o log3 81=4 (iv)7°=l log7 1=0


eB

= 3 o loggi 3 = i (vi)128*^^=2 ‘»logi28 2 = ^.


1/4
(v) 81
our

NOTE 2 For any positive real number a, we have a* = a. Therefore,


ad

a=l.
It also follows from the above definition that:

(i) log2 64 = (A real number X such that 2^ =64) = 6


dY
Re

(ii) log5 625 = (A real number x such that 5^ = 625) = 4


Fin

1 1/2
(hi) log9 3= (A real number .r such that 9 =3) = — [●.● 9 = 3]

1
(iv) logioO-01 =(ArealnumberxsuchthatlO^ =0.01) = -2. V 10“^ = = 0.01
100
1
(v) logi5 2= (Arealnumberxsuchthatl6 =2) = — [v lb*/'* =(2“*)*/^ =2]
(vi) log25 25 = (A real number x such that 25'*^ = 25) = 1
Thus, the logarithm of a number to a given positive real number 1) as base is the index or the power to
which the base must be raised in order to make it equal to the given number.
EXAMPLE i Find the values of each of the following:
(/) logg 81 (ii) logio 0.0001 (in) log^2 ^ (iv) log2(l/32)
SOLUTION (i) log9 81 = X. Then,
logg 81 =x => 9^ =81 => 9^ =9^ => x = 2.
3.2 APPLIED MATHEMATICS-XI

(ii) Let logio 0-0001 =.t. Then,


1 1
logio 0.0001 =:c=> 10^ =0.0001 lO"" = 10000 => lO"^ = ●74 => lO-^' =10"^ => x=-4
10

(iii) Let log^ 4 = x. Then,


logy2 4 = ;c => {^/2)^ = 4 => (2^^V=2^ ^ => f
(iv) Let log2 (1/32) =x. Then,
1
log2 (1/32) =x^ 2-’^ = => 2^ = = 2”^ => x = -5
32 2^
LXAMPLt 2 Find the values of X in each of the follozuing:
(i) Iog2 X = 3 (ii) logg x = 2.5 (iii)
I log8iX=| (iv) log^.v=4

ow
SOLUTION (i) log2 x = 3 =>x = 2^=8.
(ii) log9X = 2.5 => x = 9^-^=(3^)^/^ = 3^=243.
(iii) log8i3:=| =>x=(81)^^^ => x=(3^)^^^ => x = 3^=729.

e
re
Fl
(iv) log^x = 4 => x=(V2)^ => x = (21/2)4 ^ x = 2^=4.

F
EXAMPl r - Express each of the following in exponential form:
ur
(i) log2 64 = 6 (ii) logic 0.01 =-2

r
SOLUTION (i) log2 64 = 6 2^=64 (ii) logic 0.01=-2 ^ 10"^=0.01 fo
: \AMPi.r 4 //logjo y = x,find the value of 10^^ in terms ofy.
ks
Yo
SOLUTION We have,
oo

*ogio y=x^ y=io''


eB

10^^=(10’')^=y^
a- 1
LXAMI’IJ-; 5 //logio = a, find the value of 10 in terms ofx.
ur

SOLUTION We have,
ad

a
logic X = a => X = 10
Yo

a
10 X
10"" ^ =10'" X 10 -1 [Using (i)]
d

10 10
Re

PXAMPLE b //log3 y = X and log2 z=x,find72^ in terms of y and z.


in

SOLUTION We have,
F

log 3 y = X and log2 z = x => y = 3’^ and z = 2^ ...(i) [Bydef. oflog]


72^ =(2^ X 3^)'^ =(2^)-"' X (3^)^ = 2 3.v_32.v = (2")"‘(3^)“={z)^y)"=y"z^.
fl- 1
nXAMPl.C 7 //log3 X = a, find the value of 81 in terms ofx.
SOLUTION We have,
a
log3X=rt => x = 3 ...(i)
fl -1
= 8l"x 81"^ =
81
a
(3^)'" (S'")^ x^
81 [Using (i)]
81 81 81 81

PX AMPLE //log5 X = <7 and log2 y = a, find 100^*^" ^ in terms ofx and y.
SOLUTION We have,
log5 x = a and log2 y = a ^ x =5'^ and y = 2
a
...(i)
100^""^ =(5^ X 2^)^""^ =(5^)^"“^x(2^)^‘'“^=5 4fl-2 X 2
4fl-2
LOGARITHMS 3.3

(5")'* „ (2“)
4 4 4..4
S4« 24" a: V ^*1/

5^ ^ 2^
_ X ^

5^ 22
EXAMPLE 9 Find the value of 1728.
SOLUTION Let log2y3l728= x. Then
log2^ 1728=.r
=> 1728=(2V3)^=>{12)2 ={2V3)'-" => [(2V3)^]^ =(2^3)^ => (2^3)^ =(2^3)"^^ x = 6
Hence, log2^1728 = 6.
EXERCISE 3.1

1. Write each of the following in the form of logarithms:

ow
1
(i) 2^=64 (ii) 10^ =10000 (iii) 3^=243 (iv) 3-3 27

(v) 10" 3 =0.001 (vi)7^=49 (vii) 2-6=464 (viii) 4


3/2
= 8

2 Find the values of x in each of the following:

e
re
(i) log3X = 4 (ii) log4 .V = 3 (iii) log^ x = 4 (iv) logio-v = -3
(v) log4 x=1.5

rFl
(vi) logs X=-
2
(vii) logi25-^ = -

F
3. Express each of the following in exponential form;

r
(i) logs 25 = 2 (ii) log4 64 = 3 (iii) logio 0.001 = - 3 (iv) logio 1000 = 3
ou
4. If log2 y = X, find the value of 8' in terms of y. fo
ks
2fl- 1
If logio X = a, find the value of 10 in terms of .v.
oo

ANSWERS
Y

(iv) logsf^r
B

1. (i) log264 =6 (ii) logio 1000 =4 (iii) log 3 243 =5 = -3


27
re

/ 1
(v) logio0.001=-3 (vi) log7 49 = 2 (vii) log2 77 =-6
164
ou
Y
ad

(viii) log4 8 = 3/2


2. (i) 81 (ii) 64 (iii) 9 (iv) 10" 3
d

(V) 8 (vi)4 (vii) ^/5


in

(iii) 10-3 = 0.001 (iv) 103 =100


Re

3. (i) 5^=25 (ii) 43 =64


2
F

.V
4. y3 10

3.3 FUNDAMENTAL LAWS OF LOGARITHMS

In this section, we shall take logarithms to any base «((? > 0 and a ^ 1).
FIRST LAW: Ifni, n are positive rational numbers, then
log„ (mn) = logfl m + log„ n
i.e. the log of the product of two numbers is equal to the sum of their logs.
PRCXJF Let \og(,m=x and log^ « =y-T/ten,
log„ m-x ^ a^ = m and, log„ n=y ^ a^ =n [By def. of log]
mn = a^ .a^

mn = a X + J/
[By law of indices]
logfl (mn) = X + y [By def. of log]
3.4 APPLIED MATHEMATICS-XI

logrt {mn) = log^ m + log„ n [■●● = logrt m and 1/ = log« n]


Hence, log„ (mn) = log,, m + log^ n.
GENERALISATION (i) Ifm, n, p are positive rational numbers, then
log„ (mnp) = log„ m + log„ n + log„ p
(ii) If Xj, X2> , X,, are positive rational numbers, then
iogfl (^1 ^2 ^j) = logfl ^1 + logfl ^2 + + logfl
SECOND LAW; Ifm and n are positive rational numbers, then
( m 'i
log„ - =logflm-loga«
n)

i.e. the log of the ratio of two numbers is equal to the difference of their logs.
PRCK)!- Let log(j m=x and log(j n=y. Then,
logfl m = x => = m and, log^ .n=y ^ a^ =n. [By def. of log.]

w
m _ a^
ay

F lo
n

[By laws of indices]


n

ee
(m\
log a = x-y [By def. of log]

Fr
\n J
m
logfl - = logrt- logrt ” for
\n )
ur
( m\
Hence, log^ - = log^ m - log^ «.
\n )
s
ook
Yo

THIRD LAW: //m, n are positive rational numbers, then log„ (m") = n . log a m
eB

[’ROOT Let log„ m = x. Then, = m.


Now, a^ =m
our

{a’^f =m
)}
ad

=>

n
Y

=nd
Re

log„ (m") =nx


nd

[By def. of log]


logfl K)=». log« ni
Fi

[●●● X = log„ m]
Hence, log„ (w/') =/i. log^ m
FOURTH LAW log^ 1=0. i.e. The log oft to any base is always zero.
PROOF Since = 1. Therefore, by definition of log, we obtain log^ 1=0.
FIFTH LAW logfl a = l. i.e. the log of any positive quantity of the same base is always one.
PROOF Since a^ = a. Therefore, by definition of log, we obtain log,, a=\.
SIXTH LAW Ifm is positive rational number and a, b are positive real numbers such that a ^l,b ¥=1, then
log/, m
log,, m =
log/, a
PR(X )F Let log„ m = X. Then, = m.
3.5
LOGARITHMS

Now, =tn

logj, (fl^) =logfe rn [Taking log to the base b]


xlogi, fl=log(, m
logfl m. log^ a = logi, m [v x = log„m]
logft '»
iogfl m =
logfc ^
REMARK Replacing b by m in the above result, we get
log»i'»
logrt m =
log,„ a

=> logff »' = [v log^m^l]


log,« ^

w
=> log^m.log,,, fi = l
●ogfl«
SEVENTH LAW If a is a positive real number and n is a positive rational nu7nber, then
a = n

PROOF Let logfl n = x. Then,


= n

F lo [Putting the value of x in a^ =n]

e
Fre
ILLUSTRATION (i) 3log3 8=8 (ii) 2^*°82 5 =
for =5^

(iii) 5"2‘°g5 3=5!og5 3-2 ^3-2^^


1
r
EIGHTH LAW If a is a positive real number and n is a positive rational mnnber, then
You
oks

log , nP=^ log„»


eBo

PROOF Let log quP =x and log„ n = ij.


ad
our

and a^ =n

=> a‘1^ = nP and =n

=> =nP and {aP)P =nP


Re
dY

= {aP)P a^^' =aPP ^ qx =yp x =^y => log^^ nP = ^ log^ n.


Fin

PARTICULAR CASE Putting a=n in the above law, we get

log .JJP=- log„n = -


nl q q
3.4 SOME USEFUL RESULTS
RESULT 1 If fl > 1, then

(i) logrt T < 0 for all x satisfying 0 < t < 1


(ii) log^ a: = 0 for X = 1
(iii) log„ X > 0 forx >1.
(iv) x>y => logrt X > logfl y, i.e. log^ x is an increasing function.
The graph of y = log^ x, for > 1 is as shown in Fig. 3.1.
3.6
APPLIED MATHEMATICS-XI

x' x'

Fig. 3.1 Graph of 1/ = log„ x,a>\ Fig. 3.2 Graph of 1/ = logoi', 0<a< 1
RESULT2 lfO<a<\,then

w
(i) logfl j: <0 forallx>\
(ii) log„ x = 0/orA:=l
(iii) log„ a: > 0 for all x satisfying 0 < x < 1

F lo
(iv) x>y => logfl .V < log^ y, i.e. log^ x is a decreasing function.

ee
The graph of y = log^ a:, 0 < r? < 1 is as shown in Fig. 3.2

Fr
RESULTS Iffl>l,then
(i) logfl a: > 1, if a: > (ii) logfl a: <1, if 0 < a: <fl
for
(iii) logrt a:=1, if A: = rt.
RESULT4 IfO<fl<l,then
ur
(i) logflA:>l, ifO<A:<rt (ii) log„.r=l, ifx = a (iii) logfl a: < 1, if a: > r?
s
ook
Yo

RESULT 5
x>a , where <7 > 1 0 <a: <77, if 77 >1
eB

(i) logrt a: > 1 => 0 < a: < <7, where 0 < fl < 1 (ii) logaX<l => a: > 77 , if 0 <77 <1

3.5 SYSTEMS OF LOGARITHMS


our
ad

There are two systems of logarithms which are generally used.


I. COMMON LOGARITHMS: In this system the base is always taken as 10. This system is also
Y

known as Brigg's system. In all practical calculations this system is very helpful. If no base is
Re

mentioned, the base is always taken as 10 e.g. log 25 means logig 25.
nd

Thus, log 10 = logio W=l, log 100 = logio 100 = 2, log 1000 = lognj 1000 = 3 etc.
Fi

NATURAL LOGARITHMS : In this system the base of the logarithm is taken as e, where e is an
irrational number lying between 2 and 3. The approximate value of e correct to one decimal
place is 2.7 and e is defined as an irrational number given by
,1 1
e =1 + — + ~ + — + _ +
1 1
1! 2! 3! 4!

Natural logarithm of a: is written as log^ .r or. In x


Throughout this chapter we shall be using common logarithms. Thus, log a: will mean logjQ a:.

ILLUSTRATIVE EXAMPLES

EXAMPLE I Prove that

(i) log 12 = log 3 + log 4. (ii) log 50 = log 2 + 2 log 5


(iii) log (1 + 2 + 3) = log 1 + log 2 + log 3.
LOGARITHMS 3.7

SOLUTION (i) Wehave,12 = 3x4

log 12 = log (3 X 4) = log 3 + log 4 [●.● log mn = log m + log n]


Thus, log 12 = log 3 + log 4.
ALITER RHS = log 3 + log 4 = log (3x4) [●-● log m + log n = log mn]
= log 12 = LHS
(ii) We have, 50 = 2 x 25 = 2 x 5^
log 50 = log (2 X 5^)
= log 2 + log 5^ [●-● log mn = log m + log «]
= log 2 + 2 log 5 [●.● log m" = n log m]

w
ALITER RHS = log 2 + 2 log 5
= log 2 + log 5^ [●.● nlogm = log ?n"]
= log(2x5^)
= log 50 = LHS.

F lo [●.● log w + log « = log mn]

ee
(iii) LHS = log (1 + 2 + 3)

Fr
= log6
= log (1 X 2 X 3)
for [: 6=1 X 2x 3]
= log 1 + log 2 + log 3 [●.● log (mnp) = log m + log n + log p]
ur
= RHS
s

Hence, log (1 + 2 + 3) = log 1 + log 2 + log 3.


ook
Yo

EXAMPLE 2 Express each of the following as the logarithm of a single number:


eB

(i) log 2 + 1 (ii) log 12 - log 2 - log 3


(iii) 3 log 2 +2 (iv) log 2.V + 2 log A'
our

(vi) ^ log 9 + ^ log 81 + 2 log 6 - log 12


ad

(v) log a^ ~ log a


1 1
(vii) — log 9 ~ 3 log 4 + 3 log 2 (viii) 1 - -log 64
Y
Re
nd

SOLUTION (i) log 2 + 1 = log 2 + log 10 [■■■ Iogl0=logiol0=l]


Fi

= log (2 X 10) log m + log n = log mn]


= log 20
(ii) log 12 - log 2 - log 3 = log 12 - (log 2 + log 3)
= log 12 - log (2 X 3) [: log m + log « = log mn
/12^ m
= log 12 - log 6 = log — = log 2 log m - log n = log —
V 6 j

(iii) 3 log 2 + 2 = 3 log 2 + log 100 [: Iogl00 = log;iol00=2]


= log 2^ + log 100 [●.* n log m = log w"]
= log 8 + log 100 = log (8 X 100) = log 800. [●.● log m + log n = log mn]
(iv) log 2:c + 2 log X = log 2x + log [: n log m = log m"]
= log (2x X x^) = log 2x^.
3.8 APPLIED MATHEMATICS-X!

2^
0 Cl
(v) log - log = log —
a
= log fl

1 1
(vi) - log ^ ^ log 81 + 2 log 6 - log 12

= -^ log 3^ + ^ log 3^ + 2 log 6 - log 12


= log (3^)^/^ + log (3^)^/^ + log 6^ - log 12
= log 3 + log 3 + log 36 - log 12
= log {3 X 3 X 36) - log 12 [●/ log m + log M + log p = log mnp]
r3x 3x 36^
= log
12
= log 3^ =log 27.

(vii) i log 9 - 3 log 4 + 3 log 2

w
= |log32 3 log 4 + 3 log 2

= log(3^)'/^ -log 4^ + log 2^


= log 3 - log 64 + log 8 = log
f 3
64
F lo (3
+ log 8 = log —
for F
64
ree
3
X 8 = log
33

(viii) 1 - —3 log 64 = log 10 - —3 log 4^


Your

no 3 5
-log 10-log (4^)^/^ = log 10 - log 4 = log V — =log-.
ks

4 y Z
eBoo

EXAMPLE 3 Evaluate each of the following:


(i) log 5 + log 2 (ii) log500-log5
ad
our

(in) 4 log 5 + 2 log 4 (iv) log 6 + 2 log 5 + log 4 - log 3 - log 2


(v) i log 36 + log 5 - log 30 (vi) log 5 + 2 log 05+3 log 2
Re

SOLUTION (i) log 5 + log 2 = log (5x2) = log 10 = 1.


Y

/ - -
500
Find

(ii) log 500 - log 5 = log — = log 100 = 2.


V 5

(iii) 4 log 5 + 2 log 4 = log 5^ + log 4^


= log 625 + log 16
- log (625 X 16) = log 10000 - 4 [●●● log 10000 = log 10^ = 4 log 10 = 4]
(iv) log 6 + 2 log 5 + log 4 - log 3 - log 2
= log 6 + log 5 + log 4 - log 3 - log 2
= log 6 + log 25 + log 4 - (log 3 + log 2)
= [log 6 + log 25 + log 4] - [log 3 + log 2]
6 X 25 X 4
= log (6 X 25 X 4) - log (3x2) = log = log 100 = 2
3x2

[●●● log 100 = log 10^ = 2 log 10 = 2]


LOGARITHMS 3.9

1
(v) — log 36 + log 5 - log 30
1/2
= log (36) + log 5 - log 30
= log 6 + log 5 - log 30 ^ log (6 X 5) - log 30 = log 30 - log 30 = 0.
(vi) log 5 + 2 log 0.5 + 3 log 2
= log5 + log (0.5)^ + log 2^
= log 5 + log 0.25 + log 8 = log (5 x 0.25 x 8) = log 10 =1.

ow
i:\AMPLE 4 Prove that:
50 f 9 35 15^
(i) log 2 + 2 log 5 - log 3-2 log 7 = log (ii) log + log — - log — = 0
147 14 24 16

11 14 22

e
(Hi) log —+
5
log —-log
3
—=
Id
log7.

re
SOLUTION (i) log 2 + 2 log 5 - log 3 - 2 log 7

Flr
= log 2 + log 5^ - log 3 - log 7^

F
= log 2 + log 5^ - (log 3 + log 7^)
= log (2 X 5^) - log (3 X 7^) = log
ou 2x5
2 '1
= log
50

sr
3x7^ 147

9
(ii) log — + log
35
24
15

16 fo
k
oo
9 . 35 15 f 9 35^ 15 15 ^ 15
log — = log -log = log — -log —= 0.
14 24 16 14 24 16 16 16
Y

11 14 22
reB

(iii) log y + log y - log 15


22

= [log-.log-]-log-
uY

(11 X
14
fll 14^ , 22 , 5 O , fll 14 15^
ad
do

= log - X — - log — = log — = log7


V 3 3 J ® 15 ^222/155 5 3 22
in

EXAMPLE 5 Prove that:


Re

16 25 81
F

(i) 7 log — +5 log + 3 log — = log 2


15 24 80
70 22
(ii) log —+ log - log — = 3 log 2 - 2 log 3
33 135 18
16 25 81
SOLUTION (i) 7 log — + 5 log + 3 log —
15 24 ^ 80
/25f ,
+ log —
fSl^^
= log —
1
+log
1

15 24 80

ri6f (25\^ 81
= log X X

15 24 80

= 1log f —^
4

3x5
''[2^x 3 2^x5
3.10 APPLIED MATHEMATICS-XI

28
5^0 3^2
= log X X

3^ x5^ 2^^x3^ 2^^x5^

= log 2
28 - 15 - 12
X 5
10-7
3,312-7-51^ log (2^x5 X
3°) = log 2
16 25 81
ALITER 7 log + 5 log — + 3 log
15 24 80

= 7 (log 16 - log 15) + 5 (log 25 - log 24) + 3 (log 81 - log 80)


- 7 {log 2^ - log (3 X 5)} + 5 (log 5^ - log (2^ x 3)} + 3 {log 3^ - log (2^ x 5)}
^ 7 {log 2^ - (log 3 + log 5)}+ 5 {log 5^ - (log 2^ + log 3)} + 3 {log 3“^ - (log 2^ + log 5)}

ow
= 7 {4 log 2 - log 3 - log 5} + 5 {2 log 5 - (3 log 2 + log 3)}+ 3 {4 log 3 -(4 log 2 + log 5)}
= 28 log 2-7 log 3-7 log 5 + 10 log 5-15 log 2-5 log 3 + 12 log 3-12 log 2 - 3 log 5
= 28 log 2-15 log 2-12 log 2-7 log 3-5 log 3 + 12 log 3-7 log 5 + 10 log 5-3 log 5

e
= log 2.

re
Fl 70 _22 '

F
70 22 7 , 70 22 33 "" 135'
(ii) log —+ log -log = Iog — X — - log = log
ur 7
33 135 18 33 135 18

r
18

18^ 1^8^
fo ^ T
— = log \^9J
- = log 8 - log 9 = log 2'^ - log 3^ = 3 log
ks
= log 2 - 2 log 3.
33 "" 135 7 J
Yo
oo

EXA.MPLE 6 Shozv that


1 1
(i) 3 log 4 + 2 log 5 - —3 log 64 - -2 log 16=2
eB

..1 1
(ii) — log 9 + 2 log 6 + — log 81 - log 12 = 3 log 3
ur

SOLUTION (i) 3 log 4 + 2 log 5 - -3 log 64 - ^^ log 16


ad
Yo

1/3 1/2
= 3 log 4 + 2 log 5 - log (64) - log (16)
d
Re

= 3 log 4 + 2 log 5 - log (4^)^'^ ^ -log(+)'/^


in

= 3 log 4 + 2 log 5 - log 4 - log 4


F

= log 4 + 2 log 5 = log 4 + log 5^ = log (4 x 5^) = log 100 = 2.


1 1
(ii) — log 9 + 2 log 6 + — log 81 - log 12
1/2
= log 9 + log 6^ + log (Sl)^^"^ - log 12
+ iog 6^ + log (3^)^/^-log 12
= log 3 + log 36 + log 3 - log 12
= 2 log 3 + log 36 - log 12
= log 3^ + log 36 - log 12
9x 36^1
= log 9 + log 36 - log 12 = log (9 x 36) - log 12 = log
12
= log (27) = log 3^=3 log 3.
LOGARITHMS 3.11

tXAMPLE 7 Evaluate the following:


1
(i) log 25-2 log 3 + log 18 (ii) 2 log 5 + log ^ ~2 ^
(in) 3 log2 5 + log2 10 - log2 625
(iv) log^^g 10 + logio 100 + log 10 1000 + logio 10000
SOLUTION (i) ~ log 25 - 2 log 3 + log 18
1/2
= log (25) - log 3^ + log 18
= logs-log 9 + log 18 = log - + log 18 = log — xl8 = log 10=1.
19 y V9

(ii) 2 log 5 + log S ^ 4

low
1/2
= log 5^ + log 8 “ log (4)
= log 25 + log 8 - log 2 = log (25 x 8) - log 2 = log r^l
2 J
= logl00=2.
(iii) 3 log2 5 + log2 10 - iog2 625

ee
= log2 (5^) + Iog2 10 - log2 625
F
Fr
= (log2 125 + log2 10) - log2 625
125 X 10
= log2 (125 X 10) - log2 625 = log2 = log2 2=1 for [●●● logrtfl=ll
625 )
ur
(iv) logio 10 + logio 100 + logic logic 10000
= logio 10 + logic logic + logic 10^ = 1 + 2 + 3 + 4 =10
s
[v logl0"=»]
k
Yo
oo

EXAMPLE 8 Find the values ofx in each of the following:


log 144 log 125
eB

(i) = logx (ii) = X

log 12 log 25
(iii) log^ 4 + logj 16 + log^j. 64 =12
r
ou
ad

SOLUTION (i) We have,


log 144 log 12^ = log X => 2 log 12 = log
Y

= log X => log X = 2


log 12 log 12 log 12
Re
nd

logic ^ = 2 [●-● log x = logic-v]


a; =10^ =100
Fi

=>

(ii) We have,
log 125 = x =>
log 5^ = x =>
3 log 5 = X =>
3
— = X.

log 25 log 5^ 2 log 5 2

(iii) log;j. 4 + logY 16 + logj. 64 =12


=> log,^ 2^ + log.., 2^ + log., 2^ = 12
=> 2 log, 2 + 4 log, 2 + 6 log, 2 = 12
=> 12 log, 2=12 => log, 2=1 => x^ = 2 =;> x = 2
EXAMPLE 9Find the value (s) of x in each of the following:
(/) log (x + 1) + log (x -1) = log 11+2 log 3 (ii) log (x + 3) + log (x - 3) = log 27
(iii) logx-log(x-l) =log 3
SOLUTION (i) log (x + 1) + log (x -1) = log 11 + 2 log 3
3.12 APPLIED MATHEMATICS-XI

log {(:v + 1) (j:-l)} = logn + log 3^


log {x^ -1) = log 11 + log 9
log(a;2-l)=Iog(llx9)
log {x^ -1) = log 99 => -1 = 99 => Y^ = 100 y = 10 [VY>0]
(ii) log (y + 3) + log {y - 3) = log 27
log [(y + 3) (y - 3)] = log 27

ow
log (y^ - 9) = log 27 Y^ - 9 = 27 y^ = 36 => y = 6 [v Y>0]
(iii) log Y - log (y -1) = log 3
/

=>
log =log3

e
V

re
—— = 3=> y = 3(y-1)=> y = 3y-3=> y-3y=-3=> -2y = 3 => y = 3/2.
Y -1

Flr
F
l-XAMPLE 10 Evaluate each of the following:
1/3
(/) 32-logs 4 (ii) (275)^-^ ^‘’8275 10
(iii) 8^/3+log2(121)
ou
sr
SOLUTION (i) We have.

32-log34^^32j (3-iog3 4j^(32j (3fog34


(ii) We have,
fo
k 4
oo
4+ log2.75 40 1 log2.75 10
(2.75) = (2.75)^2.75) = (2.75) (10) = 27.5 [v
Y
reB

(iii) 8^/^ + log2(121)^^3


1/3
= (8l/3j^glog2(121)
uY

= (2) |8^^^^^’°82l21]^^2) (23)4/3.log2 121 (2) (2*°S2 ^2^) = 2 X 121 = 242


ad
do

{iXAMPLE 11 Evaluate each of the following:


(i) 10^°S10 ^ '°S10 ” + 3 logjo p (ii) 10 *°S10 2+logio 3+logio 4+... + logio«
in

SOLUTION (i) We have,


Re

10^°S10'”+2logio«+ 31ogiop
F

= + logic + loglO ~iQloglO p^) = 7 3


[■:
(ii) We have
10 ^°810 2+ logio 3+ log 10 4+ ... + log 10”
^j01o810(2x3x4x...x„)^2^ 3^4^
EXAMPLE 12 If a > 0, prove that
a log^ 1 + 2 . log^2 2 + 3. log 3 3 +... + h . logy, n = nl
SOLUTION We have,
logfl 1 + 2. log 2 2 + 3 . log 3 3 +...+«. log ,, n
n

= log«l + 2^ogfl2 + -log„3 + . ■ + - logrt « n

= logrt 1 + logrt 2+... + logrt « = logrt (1.2.3...») = logfl(u!)


LOGARITHMS 3.13

a
logrt 1+2 logrt2 2+3. logflS 3+ ... + ». log^Ji n = rt log„(H!)

1 (1 1 1

log 25 3 + 32^3
+ ... 00

EXAMPLE 13 Prove that: (0.16) > =4


a
SOLUTION Using: rt +rtr + «7r^+, 1-r
, we obtain

1
1
1
1
:r + —7
1
+ .... 00 = —3_=1
3 3^ 3^ 1-^ 2
3

, 1 1 1
log2.5 7 + 32 +

3^
+ ... 00

ow
1 (-1)
= log(5/2) (1/2) = log (2/5) -1 (2-0 = log2/5 2 = logo.4 2
(-1)
, 1 1
●og25 3 + -32
+ ... + ®

e
(0.16)

Fl
re
(0.16)^"S0.4 2

F
2 '(●ogO.4 2
= (0.4) = (0.4)2'°S0.4 2 =(o.4)*°®0-4 2^ -2^ = 4
ur
EXAMPLE 14 Evaliinfe each of the following: or
sf
(i) (81)^^/^°85 3)+27'°S9 + 3(4/log7 9)
36
(ii) log9 27 - log27 9
k
Yo

(in) 2‘°S3 5_5log3 2


oo

SOLUTION (i) We have,


eB

(8l)(l/log5 3) + 27'°S9 3(4/log7 9)


36
+

= 81*Ӥ3^ + 27 36 + 341ogg7
ur

logrt&
ad
Yo

= (34)log3 5+(33)log9
36
+ 34 logg 7
= 341og35^331og3 262_^341og327
d
Re
in

3x2

= 3log3 5^ + 32 logs 6
+ 32
t log3 2 = 3log3 5^ + 3iog3 63 ^ 3108372 = 54 + 53 ^72 = 890
F

(ii) We have.
3 , . 2 , .,3 2 5
logg 27 -log27 9 = log^2 “^‘^§33 -log3 3--log3 3=--- = 6

(iii) We have.
2^08 3 5 -5I083 2
= 2<l°825.1og32) _5log32
^ 2*“832.1og25 _5log32

^ 2'°82(5)’°^3 2 _^iog3 2 _ 5log3 2 _5log3 2 _ 0 [v


1/2
I 2-[(logs 13)/(2logs 9)1
EXAMPLE 15 Evaluate:
3.14 APPLIED MATHEMATICS-XI

SOLUTION We have,
logs 13 ^
2 log5 9 “ log9 13 = ^ log^2 = \ logs 13
^ x2-{(Iog5l3)/(21og5 9)}

/● 1 \2 / 2 y 4 ^3
~JW) 1727
/ 1 w 1 y(l/8)log3 13
= 3“ ^ x(3" 3j-(l/8)log3 13 _ 3- 3 ^ 3(3/8)log3 13

ow
1/2
f 1 2-[(logs 13)/(2logs 9)]

e
1/2
3-3 ^ 3(3/8)iog3l3

re
Frl 3/16

F
= 3-3/2^ 3(3/16) logs 13 ^3-3/2 ^ 3log3 (13) = 3-3/2 X (13)3/16
EXAMPLE 16 If n = 2020evaluate
ou
r
1 1 1 1
+ + +... +

log2» log3« log4tt log2020 "


so
kf
SOLUTION We have.
1 1 1 1
oo
+ + +... +

Iog2« log3« log4« ●og2020 "


Y
eB

= log,, 2 + log,, 3 + log,, 4 + ... + log,, 2020 ■r^— = logrt b


Jog), a
ur

= log,, (2.3.4.... 2020) = log,, (2020!) = log,, « = 1


oY

EXAMPLE 17 lfx,y,z >0aud x^\,y such that


ad

Jog^ _ log y _ log z


, prove that .r' 2^ = 1
y-z 2-x x-y
d

log X ^ log y _ log z


in

SOLUTION Let = X
Re

y-z z-x x-y

log X = X (y - 2), log y = X (2 - x), log z = X (x - y)


F

X log X + y logy + 2 logz = Xx(y -2) + Xy (2 - x) + Xz(x -y) = 0


log x-^ + log y^ + log 2^ = 0 =^.- log (.x^ y^^ 2^) = 0 => x^’ y^ z^ = 1
EXAMPLE 18 =
prove that
b - c c -a a-b '

^b+c-a a-b ^a + b-c


SOLUTION Let log X _ log y _ log 2 = X
b -c c-a a-b

logx - X(l)-c), logy = X(c-fl), log2 = X(rt-fc)


(& + c - <7) log X + (c + - b) log y + (<7 + - c) log z
= X (b + c - rt) (b - c) + X (c + fl - b) (c - fl) + (£7 + b - c) X (fl - b)
= k {{b^ - c^) - a {b - c)} + k {{c^ -a^)-b{c-a)} +k{(a^ -b^)-c{a-b)}
= X{b'^ -c^ + -a'^ + a^ -b^)-k{a{b -c) + b(c-a) + c{a-b)}= kx0-kx0=^0
LOGARITHMS 3.15

.b+ c-a c+ a-b a+ b - c


log -V + logy + log2 = 0

.b + c-a
log (a-
xb^c^ y
c+ a
-b ^a + b-c
EXAMPLE 19 If a, b, c and n are positive real numbers other than unity, prove that
logfl » ● ^o$b » ● logc »
logrt n. logy n + log^ n. log^. n + log^ n. log^ n =
log«fcc «
SOLUTION We have,
LHS = log^ n. logf, n + logj, n. log,- n + log, n. log

ow
n

1 1 1
+ +

log,, a. log,, b log,, b . log,, c log,, c. log,, a


log„ a + log,, b + log,, c _ log» log„ n. log^ n ■ log, n
log,, a. log,, b . log,, c log,, a. log,, b . log,, c logak ”

e
re
EXAMPLE 20 If a, b, c are positive real numbers other than unity such that:
a{b + c-a) b{c + a-b) c{a + b-c)
prove that a^ b‘^ =b^^ =c“ a*'.

Frl
F
logfl \ogb logc
a{b + c -a) _b {c + a -b) _c{a + b -c) =_1—. Then,
SOLUTION Let
ou
logb logc X
logfl

sor
log a = Xa{b + c - a),\ogb = Xb {c + a - b), log c = Xc (a + b -c) kf
b log a + a log b = Xab {b + c - a) + Xab {c + a - b) = 2X abc
c log b + b log c = Xbc {c + a - b) + Xbc {a + b - c) = 2X abc
oo

and. a logc + c log a = Xca {a + b - c) + Xac{b + c - a) = 2 X abc


Y
B

b log + a log = clog 1; +1) log c = fl log c + c log <7


log {a’’ &“) = log {b^ c*') = log (c" fl") => b" = b^ = c"
re
oY

EXAMPLE 21 I/log 12 IS = a and log24 54 =b, prove that ab + 5{a-b)=l.


u

SOLUTION We have.
ad

fl=logi2 18 =
log 18 _ log (3^ X 2) _ 2 log 3 + log 2 _ 2y + X
d

log 12 log (2^x3) 2 log 2 +log 3 2x + y


in

log 54 _ log (3^ X 2) _ 3 log 3 + log 2 _ 3y + X where


Re

and, b - log24 54 =
log 24 log (2^x3) 3 log 2 +log 3 3x + y'
F

log 2 = X and log 3 = y.

ab + 5 {a - b) =
'x+ 2yV x+ 3y + 5 X + 2y _ X + 3y
,2x4-yJ[sx + y 2x + y 3x + y^
X + 5xy + 6y + 5
(3x^ + 7xy + 2y^) ~(2x^ + 7xy + 3y^)
(2x + y){3x + y) (2x + y)(3x + y)
X ^ + 5xy + 6y^ + 5 (x^ - y^) 6x^ + 5xy + y^ = 1.
(2x + y)(3x + y) 6x^ + 5xy + y^
EXAMPLE 22 Prove that log2 3 is not a rational nuiiiber.
m
SOLUTION If possible, let log2 3 be a rational number equal to , where m,n eN and m, n do
n

not have any common factor.


3.16 APPLIED MATHEMATICS-X!

Now, log2(3)=-=> 3 = 2'"/" n


=:» 3” =2'"

Clearly, this is impossible, because 3" is an odd natural number and 2'" is an even natural
number. Hence, Iog2 3 cannot be a rational number.
EXERCISE 3.2

Prove the following: (1-12)


1. log 108 = 3 log 3 + 2 log 2 2. log 75 = log 3+2 log 5
3. log 72 = 2 log 3 + 3 log 2 4. 2 log 5 + log 8 - i log 4 = 2
5. log 7^ +log
33 22 16 25
-log = 0 6. log 2 + 16 log +12 log + 7 log —=1
18 21 24 ^ 80

ow
50

8. 2logg)+log^^^ -log
130
7. log 2 + 2 log 5 - log 3-2 log 7 = log 147
— =log2
\91
75 32

-21og|=log2
.n I 9 , 35 , 15
9. log — + log 10. log —+ log —-log —= 0
16 243 14 24 16

e
16 .V
2 ^ 2 ^
y
11. 7 log2 ^ + 5 log2 — + 3 log2 — =1

re
12. log + log ^ + log — = 0.
15 24 80

rFl 1/2 zx
.tyj

F
Express each of the folloiving as the logarithm of a single number (13 -17):
13. log 3 + 2 14. 2 log 3 + 3
1

r
1 1
15. 1 + -log 27 16. - log 4 + - log 81 + 3 log 2 - log 6 + 2
ou
fo
ks
17. ■- log 9 + 2 log 3 - log 6 + log 2 - 2
oo

Evaluate each of the following (18-20):


18. 2 logS + — log 64
Y

19. log 21 + log 4 + 2 log 5 - log 3 - log 7


eB

20. log 15+2 log 0.5 + 3 log 2 - log 3 - log 5


Solve each of the following equations .●(21- 26)
ur

21. Iog(A: + l)-log(:t-l)=l 22. log(2% + l)-log(2.v-l)=l


ad
Yo

log —
81
23. = >r 24. log 16 + 2 log 2 _ ^
log 9 log 4
d

25. log (x + 1) + log (a: -1) = 3 log 2 + log 3, a- > 0.


Re
in

26. log (3 A + 2) + log ( 3a - 2) = 5 log 2


27. If 2 log A + 3 log y - 2 = 0, prove that A^y ^
F

= 100

28. If log
V 3
i (log a + log b), prove that«^ + b^ = 7ab.
29. lia 2.v-3^2.v ~a
6 - A ^5x^ prove that 3 log a = x log
,,, a + b] 1 ..
30. If log = - (log a + log b), prove that a=b.
V 2 ^ _
31. If logio “ ^ogio I' ^ 1/ prove that ba^ = 100.
32. Given that log 2 = 0.3010 and log 3 = 0.4771, find the value of each of the following:
1/3
(i) log 12 (ii) logs (hi) logs (iv) log 108
(v) logf-^
S'!
(vi) log 48
^8
LOGARITHMS 3.17

33. Evaluate each of the following:


(i) 3-(l/2)log3 81 (ii)
10
log 8.5
(hi) (8.5) + 1 (iv)
(v) logn (tan 0.25 tt) (vi) log2 (log 3 81)
(vii) (log 3 5). (log25 27)
34. Prove that:

(i) 1+ 21ogfl 2+ 31og„ 3+ ,.. + ii.logflU =2^ . 3^ .4^^.


It
... n

(ii) 21ogrt 2+21ogs 3+...+21ogflii = |)2


35. Prove that:

(i) logio tan iogio 2°. logig tan 3°... logig tan 50° = 0
(ii) log^o tan 1° + log^Q tan 2° +... + log^o tan 89° = 0

w
36. Prove that:

(i) log3 4. log4 5. log5 6. log5 7 . log7 8. logg 9=2

F lo
(ii) log3 2.1og4 3.!og5 4....1ogi6l5=-

ee
37. Prove that:

Fr
logfc a. log,, b . logd c. loge d. logfl e = 1
38. If n e N, prove that
1
+
1
+
1
+... +
1 1 for
ur
log2-v logs a: log,, a: log,,, a:
39. If =7rt6, prove that
s
ok
Yo

(1
log V -{a + b) 7 =-{logrt + logt}
o

3 2
eB

7^
40. Iflog3 2, log3(2'^-5)andlog3 are in A.P., find the value of x.
r
ad
ou

41. Prove that:

(i)
iogfl »
= l + logflb ,..j log„ a: ■ logj, a: = logflfc
log„ a: + logi, a:
Y

iogflb n
42. Find the value of x in each of the following:
Re
nd

... , 10 1/3
(0 (ii) log AT^ = 3
Fi

(hi) (log,. 2) (logj. 625) =(logio 16) (log,. 10)


ANSWERS

13. log 300 14. log 9000 15. log 30 16. log 800
( 9
17. log 18. 2 19. 2 20. log 2
Uoo
11 11
21. — 22. _ 23. 2 24. 3
9 18
25. 5 26.

32. (i) 1.0791 (ii) 0.6990 (iii) 0.2329 (iv) 2.0334


(v) -0.4259 (vi) 1.6812
23. (i) 1 (ii) (iii) 85 (iv) miip (V) 0
3.18 APPLIED MATHEMATICS-XI

(Vi) 2
(vii) I 41. :c = 3

34/3
42. (i) 32 (ii) (V2) (iii) 5

HINTS TO SELECTED PROBLEMS

13. log 3 + 2 = log 3 + log 100 = log 300


14. 2 log 3 + 3 = log 3^ + log 1000 = log 9000
15. 1 + 1 log 27 = 1 + log 27
1/3
= 1 + log 3 = log 10 + log 3

18. 2 log 5 + log 64 = log 5^ + log 64^^ ^ = log 25 + log 4 = log 100 - 2

ow
21. log {3: + 1) - log (x -1) = 1
.Y + 1 .t + 1
=> log = log 10 => = 10
x-1 x-1

22. log (2.V + 1) - log (2y-1) = log 10

e
Fl
re
2y + 1 2.V +1
=> log = log 10 => 2y-1 = 10

F
2a--1

27. 2 log A- + 3 log y - 2 = 0 =5> log a'^ + log y^ - log 100 = 0


ur
=> log (Y^y^) = log 100 ^ x^y^ =100 or
sf
1/1 1 I\ ... > f Cl +
28. log - (log + log ft) => 2 log — = log a + log b
k
Yo
oo

3x
29. a 2y- 3 ^2x 6~ X i.5at 3x-9 a 3x
= a => (a) =>
B

a
3x \3x
re

3.T
a 9 f ^- 9 1
= => = a
=> log - = log
b) uv
u
ad

f Cl'\ f Cl^
Yo

=> 3y log - = 9 log « =^> Y log - = 3 log fl


\bJ
1
d

30- log
^ (log fl + log &) log = log a + log b
Re

V ^ /
in

fa + b'\^ f a + b'\^
= ab ^ {a + b)^ =4nb
F

=> log — = log ab ^ —


6 2
=> (r? + b)^ -Aab = 0 => (a-b)^ =0 => a=b
35.(i) logio tan 45° = logjo 1=0
(ii) (tan 1 ° tan 2° tan 3°... tan 89°)
= (tan 1° tan 89°) (tan 2° tan 88°) ...(tan 44° tan 46°) (tan 45°) =1
36. By using log/, a. log„ c = log/, c, we get:
(i) log3 4 . log4 5 . log5 6 . logg 7 . log7 8 . logg 9 = logg 9=2
(ii) log32.1og4 3.1og54...1ogi6l5=log/62 = log^4 2=^log22=^
39. +b^ = 7ab => (n + bf = 9ah
(a-vb^ 1 j. 01 +^
3
= ab=^ log — = log ab=> 2 log = log a + log b
V 3 \ 3
LOGARITHMS 3.19

42. (iii) (log, 2) (log;, 625) = (logio ^6) (log, 10)


=> log., 625=(logiol6) (Iog2l0)
=> logv 5*^ = log2 16 => log;, 5^ = log2 2^=> 4 log.T 5 =4 log2 2=> logx5=l => a:=5
3.6 STANDARD FORM OF DECIMAL

In the previous sections we have seen that:


logjo 1=0, since 10^ = 1
logio 10 =1 , since 10^ =10

ow
logic 2 ' since 10^ = 100
logic ^ ' since 10^ =1000
logic 10000 = 4 , since 10^ = 10000
-1

e
logic 0.1 =-l , since 10 = 0.1

re
logic 0-01 = -2 , since 10“ ^ =0.01

Frl
logic 0.001 = - 3 , since 10” ^ = 0.001

F
and so on.
ou
r
In fact, if n is an integer, then log ic 10" = n. In other words, if k is an integral power of 10 i.e., k is

so
expressible in the form 1 followed by several zeros or 1 preceded by several zeros immediately
kf
to the right of the decimal point, then log k can be easily found by using the definition of
logarithm. If ^:is not an integral power of 10, then it is not easy to compute log/: just by using the
oo

definition of logarithm. In such case we take the help of logarithmic tables from which we can
read the approximate value of logarithm of any positive number between 1 and 10. These tables
Y
eB

are sufficient for us to calculate the logarithm of any number expressed in decimal form. To
calculate the logarithm of any positive number in decimal form, we always express the given
positive number in decimal form as the product of an integral power of 10 and a number
ur
oY

between 1 and 10 i.e. any positive number k in decimal form is written in the form
k = m.x 10^, where p is an integer and 1 < m < 10.
ad

This is called the standard form of k.


d

Let us now discuss some examples to express any positive number in decimal form in the
in
Re

standard form of decimal.

ILLUSTRATION 1 Express 35.7 in the standard form of decimal.


F

SOLUTION Since 35.7 lies between 10 and 100. So, we write


35.7
35.7 = xl0 = 3.57xl0l.
10
1
Thus, 35.7 in the standard form of decimal is 3,57 x 10 .

ILLUSTRATION 2 Express 1239.54 in the standard form of decimal.


SOLUTION Clearly 1239.54 lies between 1000 and 10,000. So, we write
1239.54
1239.54 = X 1000 =1.23954x10^.
1000

ILLUSTRATION 3 Express 0.0029 in the Standard form of decimal.


SOLUTION Clearly, 0.0029 lies between 0.001 and 0.01. So, we write
0.0029 = (0.0029 x 1000) x 10"^ = 2.9 x 10”^
3.20 APPLIED MATHEMATICS-Xt

ILLUSTRATION 4 Express 0.00037 in the standard form of decimal.


SOLUTION We observe that 0.00037 lies between 0.0001 and 0.001. So, we write

0.00037 =(0.00037 x 10000) x 10“^ = 3.7 x 10


-4

It follows from these examples that we divide or multiply the given number in decimal form by
a power of 10 to bring one non-zero to the left of the decimal point, and do the reverse operation
by the same power of 10, indicated separately.
The following algorithm is very helpful to express a given number in the standard form of
decimal.

ALGORITHM

STItlM Obtain the given positive number.


S I HP n If the given number is more than lOe.g. 123.49,32785.21 etc, then move the decimal point to

low
the left to bring one non-zero digit to the l^ of the decimal point. If the decimal point is moved p
places to the left, then multiply the nezv number obtained by 10^^ to express the given number in
the standard form of decimal. For example, to express 123.49 in the standard form of decimal
we will have to move the decimal point 2 places on the left to get 1.2349. So, 123.49 in the

ee
standard form of decimal is 1.2349 x 10^.
rF
Fr
ST1:P 111 If the given number is less than one, then move the decimal point to the right to obtain one
non-zero digit to the left of the decimal point. If the decimal point is moved p places to the right,
then multiply the new number by 10“ ^ to express the given number in the standard form of
r
fo
decimal. For example, to express 0.02347 in the standard form of decimal we will have to move
u
the decimal point 2 places on the right to get 2.347. So. 0.02347 in the standardform of decimal
ks
is2.347x 10“^.
Yo
oo

l[-TIV
If the given number lies betzveen 1 and 10, then multiply by lO*^ to zvrite it in the standardform
B

of decimal. For example, 3.259 in the standard form of decimal is 3.259 x 10*^.
re

The following illustrations will illustrate the above algorithm.


u
ad

ILLUSTRATIVE EXAMPLES
Yo

EXAMPLE 1 Write each of the following numbers in standard form of decimal:


(i) 312303 (ii) 0.3123 (Hi) 0.03123 (iv) 3.1235
nd
Re

(v) 31.235 (vi) 423.15 X 10^


Fi

SOLUTION (i) To express 31230.5 in the standard form of decimal, we will have to move the
decimal point 4 places on the left to get 3.12305. So, 31230.5 in the standard form of decimal is
3.12305x 10^.

(ii) To write 0.3123 in the standard form of decimal, we have to move the decimal point one
place on the right to get 3.123. So, 0.3123 in the standard form of decimal is 3.123 x 10“
(hi) To write 0.03123 in the standard form of decimal, we will have to move the decimal point
two places on the right to get 3.123. So, 0.03123 in the standard form of decimal is
0.03123 = 3.123x10“ 2.

(iv) Since 3.1235 lies between 1 and 10. So, we do not move the decimal point. Therefore 3.1235
in the standard form of decimal is 3.1235 x 10*^.

(v) To express 31.235 in the standard form of decimal, we have to move the decimal point one
LOGARITHMS 3.21

1
place to the left to get 3.1235. So, 31.235 in the standard form of decimal is 3.1235 x 10 .
(vi) To express 423.15 in the standard form of decimal, we have to move the decimal point two
places on the left to get 4.2315. So, 423.15 = 4.2315 x 10^
Hence, 423.15 x 10^ = 4.2315 x 10^ x 10^ = 4.2315 x 10^.
EXAMPLE 2
Write the following numbers in decimal form, without powers of 10 as factors:
f/)5.34x 10^ CnJ 1.2056x10^ fm) 1.2056 x 10“ ^ (iv) 4.354 xlO^

SOLUTION (i) 5.34x10^=^x10^=534x10=5340


100

(ii) 1.2056 X 10^ =i^^xl0^ =


12056
= 12056
10000 100
12056 1 12056
(iii) 1.2056x 10"^ = = 0.012056
10000 100 1000000

w
(iv) 4.354x 10^=435400

F lo
EXERCISE 3.3

1- Write each of the following numbers in standard form:


(i) 328.59 (ii) 17.143 (iii) 3.589 (iv) 73524

e
2. Which of the following are in standard forms of the decimal. Express the remaining in the

Fre
standard form of decimal:

(i) 1.25 X 10^ (ii) 0.352 X 10"^ (iii) 12.59x10^ (iv) 0.132x10^
for
3. Write the following numbers in the decimal form, without powers of 10 as factors:
r
(i) 5.6x10^ (ii) 1.425x10
-1
(iii) 35x10“^ (iv) 9.637x10^
You
oks

(v) 8.7 X lO'^ (vi) 1.039x10"^


eBo

ANSWERS

1. (i) 3.2859x 10^


0
(iv) 7.3524x10^
1
(ii) 1.7143 X 10 (iii) 3.589x10
2. (i) 1.25x10^ (ii) 3.52x10”^
8
(iv) 1.32 X 10^
ad
our

(iii) 1.259 X 10
3. (i) 5600 (ii) 0.1425 (iii) 0.035 (iv) 963700
(v) 87000 (vi) 0.1039
dY
Re

3.7 CHARACTERISTIC AND MANTISSA OF A LOGARITHM

In the previous section, we have learnt how to write a positive real number in the standard form
Fin

of decimal. Let « be a positive real number and let m x 10^ be the standard of n. Then,
n
m X 10^, where p is an integer and m is a real number between 1 and 10 i.e. 1 < m < 10
log n = log (w X 10^) = log m + log 10^
= log w + p log 10 = p + log m [v Iogl0 = logiol0 = l]
Here p is an integer and 1 < m < 10.
Now, l<m<10=> log 1 < log w <log 10 => 0<log??i<l.
Thus, the logarithm of positive real number n consists of two parts:
(i) The integral part p, which is an integer: positive, negative or zero, and (ii) the decimal part
log m, which is a real number between 0 and 1.
It follows from the above discussion that the logarithm of any positive real number consists of
two parts; the integral part and the decimal parts. The integral part is known as the characteristic
and the decimal part is called the mantissa. The characteristic is always an integer : positive,
negative or zero. The mantissa is never negative and is always less than one.
3.22 APPLIED MATHEMATICS-XI

It also follows from the above discussion that:

log n = Characteristic + Mantissa.


We may use the following algorithm to find the characteristic of the logarithm of a given
positive integer.
ALGORITHM

STHI‘ I Obtain the given number.


STEP II Write the number in the standard form of decimal. Let it be m x 10^.
STEP 111 The index of 10 in the standard form i.e. p is the characteristic of the given positive number.
ALTERNATIVE ALGORITHM

STEP I Obtain the given number.

ow
STEP il Write it in decimal form if it is not given in the decimal form.
^lEPlIl If the number is greater than or equal to 1, then obtain its characteristic by using the formula:
Characteristic = (Number of digits to the left of the decimal point) -1
If the number is less than one, then use the following formula to obtain its characteristic.

e
Characteristic = - (Number of zeros between the decimal point and the first

re
Fl significant (non-zero) digit of the number + 1).

F
ILLUSTRATION 1 Write the characteristics of the logarithyns of each of the following numbers by using
their standard forms:
ur
(i) 2315.4 (ii) 459.72 (Hi) 45.852 (iv) 5.29385

r
(v) 0.23145
SOLUTION
(vi) 0.025138 (vii) 0.001238 fo (viii) 0.0001052
ks
Yo
Number Standard Form Characteristic
oo

2315.4 2.3154x10^ 3
eB

459.72 4.5972 X 10^ 2


1
45.852 4.5852 X 10 1
ur

0
5.29385X10
ad

5.29385 0
Yo

0.23145 2.3145 X 10“^ -1

2.5138x10"^
d

0.025138 -2
Re
in

0.001238 1.238x 10"^ -3


F

0.0001052 1.052x10'^ -4

ILLUSTRATION 2 Write the characteristics of the logarithms of each of the following numbers zvithout
writing than in their standard forms:
(i) 2315.4 (ii) 459.72 (Hi) 45.852 (iv) 5.29385
SOLUTION

Number Number of Digits to the Left Characteristic

of the Decimal Point


2315.4 4 3
459.72 3 2

45.852 2 1
5.29385 1 0
LOGARITHMS 3.23

ILLUSTRATION 3
Write the characteristics of the logarithms of each of the following numbers:
(i) 0.825138 (ii) 0.08539 (in) 0.00123 (iv) 0.0000458
SOLUTION

Number of zeros between the


Number decimal point and the first Characteristic
non-zero digit of the number
0.825138 0 -1

0.08539 1 -2

ow
0.00123 2 -3

0.0000458 4 -5

3.7.1 MANTISSA OF THE LOGARITHM OF A GIVEN NUMBER

The table of logarithms is used to find the mantissa of logarithms of numbers. A table is

e
re
appended at the end of this book. It consists of 90 rows and 20 columns. Every row begins with a

rFl
two digit number from the list 10,11,12,...., 98, 99 and every column is headed by a one digit

F
number 0,1,2,3,..., 9. On the right of the table, we have a big column which is divided into 9
sub-columns headed by the digits 1, 2, 3, ..., 9. This column is called the column of mean
differences.

or
ou
Note that the position of the decimal point in a number is immaterial for finding the mantissa.
To find the mantissa of a number, we consider first four digits from the left most side of the
ksf
number. If the number in the decimal form is less than one and it has four or more consecutive
zeros to the right of the decimal point, then its mantissa is calculated with the help of the number
oo

formed by digits beginning with the first non-zero digit. For example, to find the mantissa of
Y

0.000032059, we consider the number 3205. If the given number has only one digit, we replace it
B

by a two digit number obtained by adjoining zero to the right of the number. Thus, 2 is to be
re

replaced by 20 for finding the mantissa.


SIGNIFICANT DIGITS The digits used to compute the mantissa of a given number are called its significant
oYu
ad

digits.
ILLUSTRATION
Write the significant digits in each of the following numbers to compute the mantissa
of their logarithms:
d

(i) 2.185 (ii) 7 (Hi) 0.5 (iv) 0.05


in
Re

(v) 0.0512 (vi) 25 (vii) 0.0003 (viii) 0.00031


(ix) 0.00002015
F

SOLUTION

Number Significant Digits to find the Mantissa of its


Logarithm
2.185 2185

7 70

0.5 50

0.05 50

0.0512 512

25 25

0.0003 30

0.00031 31

0.00002015 2015
3.24 APPLIED MATHEMATICS'XI

We may use the following algorithm to find the mantissa of the logarithm of a given positive
number.

ALGORITHM

Obtain the given number.


sfLlMl Obtain the number formed by the significant digits in the given number.
I'XIT iii Obtain the number formed by first two digits in the number obtained in step II.
1 i 1. Look in the row starting with the number obtained in step II in the logarithmic table.
●'I'crx In the row obtained in step IV, look at the number in the column headed by the third significant
digit. If there is no third significant digit, then look at the number column headed by zero.
sTtr\! In the same row move to the column of mean difference and look under the column headed by the

w
fourth significant digit. Obtain the number there and add this number to the number obtained
in step V. This is the required mantissa. If there is no fourth significant digit, then the number

F lo
obtained in step V is the required mantissa.
The following examples will illustrate the procedure of finding the mantissa of the logarithm of

ee
a given number.

Fr
ILLUSTRATIVE EXAMPLES

I'XAMPLE 1 ^h%d the mantissa of the logarith77J of the number 5395. for
SOLUTION To find the mantissa of log 5395, we first look into the row starting with 53. In this
ur
row, look at the number in the column headed by 9. The number is 7316. Now, move to the
column of mean differences and look under the column headed by 5 in the row corresponding to
s
ook
Yo

53. We see the number 4 there. Add this number 4 to 7316 to get 7320. This is the required

mantissa of log 5395. If we wish to find the log 5395, then we compute its characteristic also.
eB

Clearly, the characteristic is 3. Thus, log 5395 = 3.7320.


:A AMPLE 2 Find the matttissa of the logarithm of the number 0.002359.
r

SOLUTION The first four digits beginning with the first non-zero digit on the right of the decimal
ad
ou

point form the number 2359. To find the mantissa of log (0.002359), we first look in the row
starting with 23. In this row, look at the number in the column headed by 5. The number is 3711.
Y

Now, move to the column of mean difference and look under the column headed by 9 in the row
Re
nd

corresponding to 23. We see the number 17 there. Add this number to 3711. We get the number
3728. This is the required mantissa of log (0.002359).
Fi

i;XAMPLE 3 Compute the mantissa of the logarithm of the nwnber 23.598.


SOLUTION The first four digits in the given number form the number 2359. Proceeding as in
example 2, we obtain the mantissa of log 23.598 equal to 3728.
i.xAMI’LE -I Compute the mantissa of the logarithm of the number 2.3598.
SOLUTION Here also the first four digits in the given number constitute the number 2359. So,
proceeding as in example 2, we obtain the mantissa of 2.3598 as the number 3728.
: \AMPLE 5 Compute the mantissa of the logarithm of 0.023598.
SOLUTION The first four digits beginning with the first non-zero digit on the right of the decimal
point form the number 2359. So, the mantissa of log (0.023598) is 3728 (See Example 2).

3.8 FINDING log N WHEN N IS GIVEN


In the previous section, we have learnt how to find the characteristic and mantissa of the
logarithm of a given positive number. We have also seen that the logarithm of a given positive
LOGARITHMS 3.25

number is the sum of its characteristic and mantissa.


It should be noted that the numbers having
same significant digits in the same order have the same mantissa. The logarithms of such
numbers differ only in characteristic.
The following algorithm is very useful to compute the logarithm of a given positive number.
ALGORITHM

STEP I Obtain the given number. N (say).


STEP II Fmd the characteristic.
STEP III Find the mantissa.

STEP IV Use the formula log N = Characteristic + Mantissa.

w
ILLUSTRATIVE EXAMPLES

EXAMPLE 1
Use logarithm tables to find the logarithm of the following numbers:
(i) 25795 (ii) 25.795 (iii) 2.5795 (iv) 0.25795

Flo
SOLUTION (i) The characteristic of the logarithm of 25795 is 4. To find the mantissa of the

e
logarithm of 25795, we take the first four digits, The number formed by the first four digits is

re
2579. Now, we look in the row starting with 25. In this row, look at the number in the column

F
headed by 7. The number is 4099. Now, move to the column of mean differencesand look under
the column headed by 9 in the row corresponding to 25. We see that the number there is 15. Add
ur
r
this number to 4099. We get the number 4114. This is the required mantissa.
Hence, log (25795) = 4.4114
fo
ks
(ii) The characteristic of the logarithm of 25.795 is 1, because there are two digits to the left of the
Yo

decimal point. To find the mantissa, we take the number formed by first four digits. The number
oo

is 2579. Proceeding exactly in the same way as in (i) the mantissa of the log of 25.795 is 4114.
B

Hence, log (25.795) = 1.4114


re

(iii) The characteristic of the logarithm of 2.5795 is 0. The number formed by four significant
digits is same as in (i) and (ii).
u
ad

Hence, log (25795) =0.4114.


Yo

(iv) The characteristic of the logarithm of 0.25795 is-1 and the number formed by four
significant digits is same as in the first three parts. So, mantissa is same. Hence,
d
Re
in

log(0.25795) =-1 +0.4114 =1.4114


It should be noted that -1 + 0.4114 cannot be written as -1.4114. Because, -1.4114 is a negative
F

number of magnitude 1.4114, whereas -1 + 0.4114 is equal to - 0.5886. In order to avoid this confusion
we write 1/or-l and thus log (0.25795) = 1.4114.
EXAMPLE 2 Use logarithm tables to find log (0.0074).
SOLUTION Since there are two consecutive zeros between the decimal point and the first
non-zero digit on the right side of decimal point. So, the characteristic of log (0.0074) is - 3.
To find the mantissa, we take 74 and look in the row 74 under column 0. We see the number 8692
there. So, mantissa = 8692.

Hence, log (0.0074) = - 3 + 0.8692 = 3.8692.


EXAMPLE Use logarithm tables to compute log (0.005).
SOLUTION The characteristic of log (0.005) is - 3. To find the mantissa we consider the number
50, because 0.005 = 0.0050. Now, we look in the row 50 under column headed by 0. We see the
number 6990.
3.26 APPLIED MATHEMATICS-XI

Hence, log(0.005) = -3+ 0.6990 = 3.6990


NOTH 1, The mantissae of log 2, log 20, log 0.2, log 0.02 are equal.
NOTE 2. The tnantissae of log 2, log 3, log 4, etc. are same as those of log 20, log 30, log 40 respectively etc.
NOTH :> the logarithm tables the decimal point is not given before any number and is, therefore, to be
preifxed zvhile zvriting the mantissa.
EXAMPLE 4 Given log 53498 = 47283, find thefollozving :
(i) log (5.3498) (ii) log (053498) (Hi) log (0.053498)
SOLUTION Since the four digit number formed by the significant digits in 5.3498, 0.53498,

ow
0.053498 is same as in 53498. Therefore, the mantissa of each one is the same which is 0.7283.
Only the values differ in characteristics. Hence,
(i) log (5.3498) =07283 (ii) log (053498) =1.7283 (iii) log (0.053498) =27283
EXERCISE 3.4

e
1. Write the characteristics of the logarithms of each of the following numbers:

re
(i) 5970 (ii) 125.35 (iv) 7.985

Frl
(iii) 20.125

F
(v) 0.7598 (vi) 0.02539 (vii) 0.003754 (viii) 0.00003258
2. Use logarithm tables to find the logarithms of the following numbers:
(i) 1270 (ii) 12.70 (ui) 431.5 (iv) 21.58
ou
or
(v) 0.3582 (vi) 0.002598 (vii) 0.000125 kfs (viii) 0.003
(ix) 0.02 (X) 7 (xi) 71 (xii) 71.2

ANSWERS
oo

1. (i) 3 (ii) 2 (iii) 1 (iv) 0 (V) -1 (vi) -2 (vii) -3 (viii) -5


Y
B

2. (i) ai038 (ii) 1.1038 (iii) 2.6350 (iv) 1.3340


(v) 15541 (vi) 3.4147 (vii) 4.0969 (viii) 3.4771
re

(ix) 2.3010 (x) 0.8450 (xi) 1.8513 (xii) 1.8524


oYu

3.9 ANTILOGARITHM
ad

In the previous section, we have discussed the procedure of finding log n for a given value of
d

positive number n. In this section, we shall find n when log n is given.


in

ANTILOGARITHM The positive number n is called the antilogarithm of a number m if log n = m. If n is


Re

antilogarithm of m, we zorite n = antilog m.


F

Thus, n=^ antilog m <=> log«=m.


rxAMPl.E (i) log 100 = 2 antilog 2 = 100
(ii) log 431.5 = 2.6350 antilog (2.6350) =431.5
(iii) log 0.1257 = 1.993 o antilog (1.993) = 0.1257
To find the antilog of a given number, we use the antilogarithm tables given at the end of the
book. To find n, when log n is given we use only the mantissa part. The characteristic is used
only in determining the number of digits in the integral part or the number of zeros on the right
side of the decimal point in the required number.
The following algorithm will be helpful in determining the antilogarithm of a given number.
ALGORITHM

■'IHP 1 Obtain the given number.


<TFPIl Determine zohether the decimal part of the given number is positive or negative. If it is negative,
make it positive by adding one to the decimal part and by subtracting 1 from the integral part.
3.27
LOGARITHMS

For example, in - 25983 the decimal part is - 05983 ivhich is negative. So, we write
- 2.5983 = - 2 - 0.5983 = - 2 -1 + 1 - 0.5983 = - 3 + 0.4017 = 3.4017
STFPIll In the antilogarithm table, look into the row co7^taining the first tzoo digits in the decimal part of
the given number.
STEP1\ In the rozo obtained in step III, look at the number in the column headed by the third digit in the
decimal part.
STEP In the row chosen in step III, move in the column of mean differences and look at the number in
the colwnn headed by the fourth digit in the decimal part. Add this number to number obtained
in step IV.
STEP VI Obtain the integral part (Characteristic) of the given number. If the characteristic is positive
and is equal to n, then insert decimal point after {n +1) digits in the number obtained in step V.
Ifn > 4, then write zeros on the right side to get {n +1) digits.

w
If the characteristic is negative and is equal to -n or n, then on the right side of decimal point
write {n -1) consecutive zeros and then write the number obtained in step V.

F lo
Following examples will illustrate the above algorithm.
ILLUSTRATIVE EXAMPLES

ee
Fr
EXAMPLE 1 Find the antilogarithm of each of the following:
(i) 2.7523 (ii) 3.7523 (iii) 5.7523 for (iv) 0.7523
(v) 1.7523 (vi) 2.7523 (vii) 3.7523
ur
SOLUTION (i) Tlie mantissa of 2.7523 is positive and is equal to 0.7523.
Now, look into the row starting with 0.75. In this row, look at the number in the column headed
s
ook
Yo

by 2. The number is 5649.


Now in the same row move in the column of mean differences and look at the number in the
eB

column headed by 3. The number there is 4. Add this number to 5649 to get 5653.
The characteristic is 2. So, the decimal point is put after 3 digits to get 565.3.
our

Hence, antilog (2.7523) = 565.3


ad

(ii) The mantissa of 3.7523 is same as the mantissa of the number in (i) but the characteristic is 3.
Hence, antilog (37523) = 5653.0
Y

(iii) The mantissa of 5.7523 is same as the mantissa of the number in (i) but the characteristic is 5.
Re
nd

Hence, antilog (5.7523) = 565300.0


Fi

(iv) Proceeding as above, we obtain: antilog (0.7523) =5.653


(v) In this case the characteristic is 1. Hence, antilog (1.7523) = 05653
(vi) In this case the characteristic is 2 i.e. - 2. So, we write one zero on the right side of the
decimal point. Hence, antilog (2.7523) = 0.05653
(vii) Proceeding as above we find that antilog (37523) = 0.005653.
example 2 antilogarithm of each of the folloiving:
(i) 0.452 (ii) 0.3 (iii) 1.051

SOLUTION (i) The mantissa of the given number is 0.452.


So, look into the row starting with 0.45. In this row, look at the number in the column headed by
2. The number is 2831. Since the characteristic is zero. So, the decimal point is put after one digit
to get 2.831. Hence, antilog (0.452) = 2.831.
(ii) The mantissa of the given number is .3 or .30. So, we look into the row starting with .30. In
this row, the number in the column headed by 0 is 1995. Since the characteristic is 1, so the
decimal point is put after one digit to get 1.995. Hence, antilog (0.3) =1.995
3.28 APPLIED MATHEMATICS-XI

(iii) The mantissa is 0.05. So, we look into the row starting with 0.05. In this row, the number in
i:\
the column headed by 1 is 1125. Since the characteristic is 1. So, we put the decimal after two
digits to get 11.25. Hence, antilog (1.051) =11.25
EXAMPLE ^ Find the antilogarithm of each of the foUoiuing:
(i) -1.2084 (ii) -2.4325 (iii) - 3.6432 (iv) -0.62

SOLUTION (i) The mantissa of the given number is negative. So, we first make it positive as
follows;

-1.2084 = -1 - 0.2084 = -1 -1 +1 - 0.2084 = - 2 + 0.7916 = 2.7916


From the antilogarithm table we find that the number corresponding to the mantissa .7916 is
6189. Since the characteristic is 2 i.e. - 2. So, we put one zero just after the decimal point to get the
antilogarithm of the given number. Hence,antilog (-1.2084) = antilog (2.7916) = 0.06189
(u) - 2.4325 = - 2 - 0.4325 = - 2 -1 + 1 - 0.4325 = - 3 + 05675 = 3.5675

From the antilogarithm table, we find that the number corresponding to the mantissa .5675 is

w
3694. Since the characteristic is 3 i.e. - 3.
analog (-2.4325) = anhlog (35675) = 0.003694
(iii)

F lo
3.6432 = - 3 - 0.6432 = - 3 -1 + 1 - 0.6432 = - 4 + 0.3568 = 4.3568
From the antilogarithm table, we find that the number corresponding to the mantissa .3568 is
2274. Since the characterisdc is 4 i.e. - 4.

e
analog (- 3.6432) = andlog (4.3568) = 0.0002274
Fre
for
(iv) - 0.62 =-1 +1 -0.62 = -!+ 0.38 = 1.38
From the andlogarithm table the number corresponding to .38 is 2399. The characteristic is 1
r
i.e. -1. So, analog (- 0.62) = analog (1.38) = 0.2399
You
oks

EXAMPLE 4
Each of the following numbers is the logarithm of some number. Express each in the for m
p + log m, where p is the characteristic and log m the mantissa, and find the number,
eBo

(i) 1.2086 (ii) 2.5674 (iii) -1.3295


SOLUTION (i) Given number is 1.2086 =1 + .2086
ad
our

From the andlogarithm table the number corresponding to the mandssa .2086 is 1616. Since the
characterisac is 1. Therefore, antilog (1.2086) =16.16
(ii) 25674 = 2 + .5674
From the antilogarithm table the number corresponding to the mandssa .5674 is 3693. Since the
Re
dY

characterisdc is 2. So, we put the decimal after 3 digits from the left side,
Fin

analog (2.5674) = 369.3


(ui) -1.3295 = -1 - 0.3295 = -1 -1 + 1 - 0.3295 = - 2 + .6705 = 2 + .6705

From the andlogarithm table the number corresponding to the' mandssa .6705 is 4682. Since
the characterisdc is 2 i.e. - 2.
analog (-1.3295) = antilog (2.6705) = 0.04682
EXAMPLE
5 Find the value ofx in each of the follozuing:
(i) log x = -1.3273 (ii) log-r = -8.2734
SOLUTION (i) We find that,
-1.3273 = -1 -0.3273 = -2 + 1 -0.3273= -2 + 0.6727 = 16727

log.t = -1.3273 => logx = 2.6727 :=> x = andlog (16727)


From the antilogarithm tables, we find that the number corresponding to the mantissa .6727 IS
i:
4707. Since the characteristic is 2, i.e - 2. So, we put one zero on the right of the decimal point.
Hence,

X = analog (2.6727) = 0.04707 = 4.707 x 10“ ^


LOGARITHMS 3.29

(ii) We find that,


- 8.2734 = - 8 - 0.2734 - - 9 + 1 - 0.2734 = - 9 + 0.7266 = 9.7266

.'. log 'A= -8.2734 log X" = 9.7266 x = antilog (9.7266) = 0.000000007266 = 7.266 x 10 ^
EXAMPLE 6 Given that log a: = 1.4914 and log y = 2A669,fmd the value of each of the foUoiving:

(i) xij (ii) ^ (in) ~


y

SOLUTION (i) We know that


log (xy) = log X + log ij
=> log (xy) =1.4914 + 2.4669 = -1 + 0.4914 + 2.4669 = 1.9583
=> xy = antilog (1.9583) = 90.84
(ii) We know that

log - = logX-logy
lyj
/'x'l -

F low
log - =1.4914 - 2.4669 = -1 + 0.4914 - 2.4669 = - 05086 - 2.4669 = - 2.9755
yj
(x\
=>
log - = - 2 - 0.9755 = - 3 + 1 - 0.9755 = - 3 + 0.0245 = 3.0245
lyj
- = antilog (3.0245) = 0.001058
ee
y
for Fr
f X
2\
(iii) log = log x^ - log y ^ = 2 log X - 3 log y = 2 X 1.4914 - 3 x 2.4669
= 2 (-1 + 0.4914) - 3 X 2.4669 = - 2 + 0.9928 - 7.4007 = - 9.4007 + 0.9928 = - 8.4079
Your
s

= - 8 - 0.4079 = - 9 +1.04079 = - 9 + 05921 = 95921


eBo k

,v2
y
2 = antilog (9.5921) = 0.000000003909 = 3.909 x 10" ^
ad
our

i.XAMPu; 7 Evaluate ^72.3, //log 0.723 = 1.8591.


SOLUTION Let x = ^72.3. Then,
Re

1/3
log X = log (72.3)
Y
Find

=> log X =—log72.3


3

=> log X = — X 1.8591


^ 3 [●.' log 0.723 and log 72.3 have tlie same mantissa ]
log X = 0.6197 ^ X = antilog (0.6197) ^ x = 4.166
EXA.MPLF. 8 Evaluate ^10076, if log 100.76 = 2.0029.
SOLUTION Letx= ^10076. Then,
1/5
log X = log (10076)

log X = ^ log 10076


log X = ^ X 4.0029 [●.● log 10076 and log 10076 have the same mantissa ]
log X = 0.8058 => X = antilog (0.8058) => x = 6.409
3.30 APPLIED MATHEMATICS-XI

EXAMPLE 9
Find VlM, if log 0.281 =\M87.
SOLUTION Let:f= =(2.81)'^^.Then,

log ^ ^ log 2.81


log a: = — X 0.4487 [■-● log 0.281 and log 2.81 have the same mantissa]
^ 7
log a: = 0.0641 => a: = antilog (0.0641) => a: =1.159

EXERCISE 3.5

1 ● Find the antilogarithm of each of the following:


(i) 0.7523 (ii) 0.310 (iii) 0.5428 (iv) 2.752

w
(v) 1.301 (vi) -2.5428 (vii) 2.5428 (viii) -3.6423
(ix) -15082

F lo
i. Each of the following numbers is the logarithm of some number. Express each in the form
p + log m, where p is the characteristic and log m the mantissa, and find the number,
(i) 2.5403 (u) -1.2503 (ui) -2.3245 (iv) -3.3264

ee
(v) 2.4765 (vi) 0.26

Fr
3. If log a: =1.0712, find X.
-1. If log X = 2.1352, find X.
0. Find the value of x in each of the following :
for
ur
(i) log X =1.7345 (u) log x = -3.8721 (iii) log x = - 3.0078 (iv) log x = -2.1381.
6. Given that log x = 1.4914, log i/ = 2.4669, find the value of each of the following:
s
ook

x^ x2
Yo

(i) X^y2 (ii) (iii) —


y
eB

7. Evaluate the following:


(i) V843.6, if log 84.36 = 1.9261 (ii) t/0.038, if log 38 = 1.5798
r

(iii) ^0.849, if log 84.9 = 1.9289


ou
ad
Y

ANSWERS
Re

1- (i) 5.6532
nd

(ii) 2.0417 (iii) 3.4898 (iv) 564.9


(v) 0.2000 (vi) 2.865 X 10" ^ (vii) 0.3489 (viii) 2.279x10"'*
Fi

(ix) 0.0310
2.''(i) 346.9 (ii) 0.0562 (iii) 4.736 x 10" ^ (iv) 4.716 X 10" ^
(v) 299.5 (vi) 1.819
3. 11.78 4. 0.0136 5. (i) 1.843 X 10" ^ (ii) 1.343x10"^
(iii) 9.822x10"'* (iv) 7.276x 10"^
6. (i) 8251.90 (ii) 3.470 X 10"^ (iii) 3.279 X 10"
7. (i) 29.05 (ii) 0.4413 (iii) 0.9729

3.10 USE OF LOGARITHM IN NUMERICAL CALCULATIONS

In this section, we shall use logarithms to simplify lengthy numerical calculations involving
multiplication and division by using the laws of logaritlms. The procedure is explained by the
following examples. '*
LOGARITHMS 3.31

ILLUSTRATIVE EXAMPLES

EXAMPLE 1 Using logarithms find the values of the following:


(i) 6.45 x 981.4 (it) 0.0064 X 1507 (in) 3.423 x 27.74 x 9.32

SOLUTION (i) Let x = 6.45 X 981.4. Then,


log :c = log (6.45 x 981.4) = log 6.45 + log 981.4
log .x = 0.8096 + 2.9919 = 3.8015 [●.● log 6.45 = 0.8096 and log 981.4 = 2.9919]
X = antilog (3.8015) = 6331
(ii) Let X = 0.0064 x 1507. Then,
log X = log (0.0064 X 1507) = log 0.0064 + log 1507 = 3.8062 + 0.1781 = 3.9843
x = antilog (3.9843) = 0.009645

ow
(iii) Let X = 3.423 x 27.74 x 9.32. Then,
log X = log (3.423 X 27.74 x 9.32)
log X = log 3.423 + log 2774 + log 9.32 = 0.5344 + 1.4431 + 0.9694 = 2.9469
X = antilog (2.9469) = 884.9
EXAMl’LE 2 Evaluate the following:

e
2.632

re
(/)
0.0045
(ii) 0.054-216.3

rFl
(iii) 4.382 - 3.738 (iv) 0.0072 - 0.00067

F
2.632
SOLUTION (i) Letx = . Then,
0.0045

r
f 2.632
ou
log x= log = log 2.632 - log 0.0045 = 0.4203 - 3.6532
fo
0.0045
ks
=> log X = 0.4203 - (3 + 0.6532) = 0.4203 + 3 - 0.6532= 3.4203 - 0.6532 = 2.7671
.'. X = antilog (2.7671) = 584.9
oo

0.054
Y

(ii) Let X = . Then,


eB

216.3
( 0.054 ^
log X = log = log 0.054 - log 216.3 = 2.7324 - 2.3351 = - 2 + 0.7324 - 2.3351
216.3
r
ou

=> log X = 0.7324 - 4.3351 = - 3.6027 = - 3 - 0.6027 = - 4 +1 - 0.6027 = - 4 + 0.3973 = 4.3973


ad
Y

x = antilog (4.3973) =0.0002497


4.382
(iii) Let x = . Then,
d

3738
Re
in

M-382^
log X = log = log 4.382 - log 3.738 = 0.6417 - 05726 = 0.0691
3.738
F

x = antilog (0.0691) =1.172


0.0072
(iv) Let X = . Then,
0.00067
0.0072 'i
log X = log = log 0.0072 - log 0.00067 = 3.8573 - 4.8261
(0.00067

log X =(- 3 + 0.8573) - (- 4 + 0.8261) = - 3 + 0.8573 + 4 - 0.8261 =1.0312


x = antilog (1.0312) =10.74.
EXAMPLE 3 Using logarithmic tables, find the values of each of the following:
L5
(i)
520.4 X 8.065
(ii)
76.03 X 9.08
(iii)
(25.36)^ X 0.4569 (iv)
(1.23)
9753 101.2 X 63.17 8475 11.2 X 235

(v)
(7356)^x(0.0371)^
68.21
3.32 APPLIED MATHEMATICS-XI

520.4 X 8.065
SOLUTION (i) Let;c = . Then,
9753
520.4 X 8.065 ^
log= log = log 520.4 + log 8.065 - log 9753
9753

log A- = 2.7163 + 0.9066 -1.9891 = 3.6229 -1.9891 =1.6338


A = antilog (1.6338) =430.3
76.03 X 9.08
(ii) Let A = . Then,
101.2 X 63.17

76.03 X 9.08
log A = log = log (76.03 X 9.08) - log (101.2 x 63.17)
\
101.2 X 63.17

=> log A = (log 76.03 + log 9.08) - (log 101.2 + log 63.17) = (1.8810 + 0.9581) -(2.0051 +1.8005)

ow
=> log A = 2.8391 - 3.8056 = -0.9665 = -1 +1 - 0.9665 = -1 + 0.0335 = 1.0335
A = antilog (1.0335) =0.2163
15
(1.23)
(iii) Let a = . Then,
11.2 X 235

e
(1.23) 15 ^

Fl
- log (11.2 X 235) = I log 1.23 - log 11.2 - log 235

re
15
log A = log = log (1.23)
11.2 X 235

F
log A = I X 0.0899 -1.0492 -1.3711 = 0.1348 -1.0492 -1.3711 = - 2.2854 = - 2 - 0.2854
ur
log A = - 3 +_1 - 0.2854 = 3.7145 or
sf
A = antilog (3.7145) =0.005183
(25.36)^ X 0.4569 ●.
k
Yo
(iv) Let A = Then,
oo

8475

(25.36)^ X 0.4569
B

log A = log
8475
= log ((25.36)^ X 0.4569) - log (8475)
re

log A = log (25.36)^ + log 0.4569 - log 8475 = 2 log 25.36 + log 0.4569 - log 8475
u
ad

log A = 2 X 1.4041 + 1.6599 - 2.9282 = 2.8082 + 1.6599 - 2.9282


Yo

log A = 2.8082 + (-1 + 0.6599) - 2.9282 = 2.8082 -1 + 0.6599 - 2.9282


log A = 3.4681 - 3.9282 = - 0.4601 = -1 +1 - 0.4601 = -1 + 05399 = 15399
d

A = anhlog (15399) =0.3466


Re
in

(v) Let A =
(7356)^x(0.0371)^ . Then,
F

68.21

(7356)^x(0.0371) 2^1
log A = log
68.21
= log [(7356)^ X (0.0371)2] _

log A = log (7356)^ + log (0.0371)2 _ j^g ^g 21 = 3 log 7356 + 2 log 0.0371 - log 68.21
log A = 3 X (1.8667) + 2 (25694) -1.8339 = 3 x (1.8667) + 2 (- 2 + 05694) -1.8339
log A =5.6001 - 4 + 1.1388 -1.8339 =5.6001 + 1.1388 - 4-1.8339= 0.905
A = antilog (0.905) = 8.035
i:\AMPLEJ__Usin<^ logarithmic tables, find the value of each of the folloivin<^:
(i) ^0.0847 (ii) (0.09634)^ (iii) ^0.6789 (iv)
(v) (0724)2 (vi) V42.36
SOLUTION (i) LetA= ^0.0847. Then,
log A = log (^00847) = log (0.0847)^/ ^ 3
log (0.0847) = -x 2.9279
3
LOGARITHMS 3.33

= -^x (-2 + 0.9279) = - 0.6666 + 0.3093 = - 0.3573 = -1 +1 - 0.3573 = -1 + 0.6427 = 1.6427

x = antilog (1.6427) = 0.4392


(ii) Let :c= (0.09634)^. Then,
log 3: = log (0.09634) ^ = 3 log (0.09634) = 3 (2.9838) = 3 x (- 2 + 0.9838) = - 6 + 2.9514
=> log X = - 3.0486 = - 3 - 0.0486 = - 4 + 1 - 0.0486 = - 4 + 0.9514 = 4.9514
3: = antilog (4.9514) = 0.0008941
(iii) Let 3:= ^0.6789. Then,

log 3: = log ( ^0.6789) = ^ log (0.6789) = ^ (1.8318) = ^ (-1 + 0.8318)


=> log 3T = - 0.25_+ 0.2079 = - 0.0421 = -1 + 1 - 0.0421 = -1 + 0.9579 = 1.9579
3: = antilog (1.9579) = 0.9076

w
(iv) Let 3:= ^/4^. Then,
1/5 _
log 3: = log (42.7) - log 427 = - X (1.6304) = 0.3260

F lo
5 5

3: = antilog (0.3260) = 2.1187.


(v) Let 3: =(0724)^. Then,

ee
Fr
log 3: = log (0.724)^ = 3 log (0724) = 3 (1.8597) = 3 (-1 + 0.8597)
=> log 3: = - 3 +_3 X 0.8597 = - 3 + 25791 = - 0.4209 = -1 + 1 - 0.4209 = 15791
for
X = antilog (15791) = 0.3794
ur
(vi) Let X = V42.36. Then,

^ log 42.36 =~x (1.6269) = 0.8134


l/2_
s

log x = log (42.36)


ook
Yo

X = antilog (0.8134) = 6507


eB

EXAMPLE 5
Using logarithmic tables, find the value of each of the following:
41.32 X 20.18 131.42x7.192 563.4 X
(i) (ii) 3 (iii)
r

"y (0.236)2 ^(3.2)2 X (5.6)3


ad

(6.15)3
ou

12.69

(v)
(71.24)3 ^ ^
Y

\ (2.3)^ X JtI
Re
nd

41.32 X 20.18
SOLLITION (i) Letx = . Then,
Fi

V 12.69
41.32 X 20.18 41.32 X 20.18
log X = log
12.69 12.69

=> log X = i [log (41.32 X 20.18) - log 12.69] = ^ [log 41.32 + log 20.18 - log 12.69]
log X = ^ [1.6162 + 1.3049 -1.1035] = ^ (2.9211 -1.1035) = ^ (1.8176) = 0.9088
X = antilog (0.9088) = 8.106
31.42x7.192
(ii) Let x= 3 . Then,
y (0.236)2
. 1/3
31.42 X 7.192
log X = log
(0.236)2
3.34 APPLIED MATHEMATICS-XI

= i Ilog (31.42 X 7.192) - log (0.236)^|


31.42x7.192
log a: =1 log (0.236)^
log = - {log 31.42 + log 7.192 - 2 log 0.236} = - {1.4971 + 0.8568 - 2 (1.3729)}
3 3

logX = -
3
{1.4971 + 0.8568 - 2(-1 + 0.3729)} = -
3
{1.4971 + 0.8568 + 2 - 07458}

log X = - {4.3539 - 07458} = - (3.6081) = 1.2027


3 3
= antUog (1.2027) =15.95,
(45.4)^
(ui) Letj:= 3
1|(3.2)2 ^ (5^) 3. Then,
1/3

w
log x = log
f (45.4)^
(3.2)2 ^ (5^)3

Flo
1 f (45.4)2
log;. = - log =
(3.2)^ X (5.6)^
^> = I log(45.4)2 -log «3.2)2x (5.6)2)

ee
logjr= i3 log(45.4)2 -log(3.2)2-log(5.6)2 = i[2log45.4-2log

Fr
3
3.2-3log5.6)

for
log X = - [2 X 1.6571 - 2 X 05051 - 3 x 07482] = - [3.3142 -1.0102 - 2.2446]
ur
3 3
=> log x = i3 [0.0594] = 0.0198
ks

x = antilog (0.0198) =1.047


Yo
oo

563.4 X
(iv) Let X = . Then,
eB

(6.15)^
1/3
563.4 X (0.4573)
log x = log ► = log (563.4 X (0.4573)^/ - log (6.15)^
r

(6.15)2
ou
ad

=>
log X = log 563.4 + - log 0.4573 - 3 log 6.15 = 27508 + - (1.6602) - 3 (07889)
Y

3 3

log X = 27508 + i3 (-1 + 0.6602) - 2.3667 = 27508 - 0.3333 + 0.2200 - 2.3667 = 0.2708
Re
nd

X = antUog (0,2708) = 1.865


Fi

(v) Letx =
(71.24)5 ^ ^ . Then,
^ (2.3)^ X
1/2
(71.24)^ x(V^)^/^
log x = log
(2.3)’'X (21)^/^ i jlog |(71.24)5x (56)^/2j _ x
logx =
^ 5 log 71.24 + i log 56 -7 log 2.3 -1 log 2l|
5 X 1.8527 + - X 17482 - 7 x 0.3617 - - x 1.3222
2 2

log X = I [9.2635 + 0.8741 - 25319 - 0.6611] = ^ (6.9446) = 3.4723


x = antilog (3.4723) =2967.
LOGARITHMS 3.35

EXAMPLE 6
V^rite down the logarithn of 2^^ nnd use it to state the number of digits in the numeral for
2&4
SOLLITION Let x = 2^. Then,
log X = log 2^ = 64 log 2 = 64 X 0.3010 = 19.264
So, the characteristic of logarithm of 2^^ is 19. So, 2^ has 20 digits.
EXERCISE 3.6

Using logarithmic tables, find the value of each of the folloiving:


1. (i) 69.13 X 0.014 X 0.34 (ii) 0.7625 x 0.000357 (iii) 2456 x 0.000071
(iv) 352.6x 0.078x 0.5943 (v) 32.7 X 0.0425 x 494 (vi) 0.0001 X 0.0027 X 38.9 x 50.2
2. (i) 0.0953 - 3.794 (ii) 25 0.0683 (iii) 0.3865 0.000572

w
8.25 X 4.63 27 22
3. (i)
2.18
(ii) 1045 X —
49
(iii) X (3.2)2
7

(iv) |x 3.142 X (15)^


Flo
ee
1/7
4. (i) ^aoT25 (ii) ^/UlTl (iii) (0.00001427)

Fr
5. (i)
8.356 (19.42)^ J 0.7136x0.8
(ii) (iii)
2.635 8.73 X 60.4
for \ 0.0467
ur
(7356)^ X (0.0371)^ 3.279 X VL207 7.93 X (2.65)^
(iv) (V) (Vi)
V68.21 ^120000
k s
Yo
oo

81/3^73/4 (76.24)^ X (6.45)^ X (0.00034)^^ ^ x 981.4


(vii) (viii) (ix)
^ (3.2)^xVl7
eB

^(7.29)^/^ X 3.26 (9.37)^ X (8.93)^''^ X (0.0617)


6. Find the number of digits in the numeral for (875)^^.
r

ANSWERS
ou
ad

l.(i) 0.3291 (ii) 0.0002723 (iii) 0.1744 (iv) 16.34


Y

(v) 686.4 (vi) 0.0005272


2.(i) 0.02512 (ii) 366 (iii) 675.7
Re
nd

3.(i) 13.92 (ii) 575.8 (iii) 32.18 (iv) 14.15


Fi

4.(i) 1.517 (ii) 2.032 (iii) 0.2032


5.(i) 1.781 (ii) 0.7152 (iii) 2.303 (iv) 66.37
(v) 0.07303 (vi) 9.860 (vii) 1.1670 (viii) 855.5
(ix) 1963 6. 48.

MULTIPLE CHOICE QUESTIONS (MCQs)


16 25 81
1. The value of 7 log + 5 log — + 3 log —, is
15 6..
24 6gQ'
(a) log 2 (b) log 3 (c) log 5 (d) none of these

2. The value of
log 49 V7 + log 25 Vs - log 4-V2 , IS.
log 17.5
(a) 5 (b) 2
(d)|
3.36 APPLIED MATHEMATICS-XI

3. The value is

(a) log 2 (b) 1 (c) 0 (d) none of these


4. The value of is

(a) log 2 (b) 1 (c) 0 (d) none of these


5. The value of
3 + log 343 IS
^ '
1 r49 1 1
2 + log - +
2 3 125

(a) 3 (b) 2 (c)l


(d)|
6. The value of
, f. 1
+ Iog5(’l+i
1 1 f I \
logs 1 + 7 + log5 1+- +. .. + logc 1 + i
IS
\ ^ V o V / , I 624
(a) 5 (b) 4 (c) 3 (d) 2

w
7. If2'‘"Sl0 3^ = 3^'ogl0 2^then^::
1

.)|
F lo
(a)- (c) 3 (d) 2

8. If logiQ 2 = 0.3010, then log5 64 =

ee
602 233 202 633
(a) (b) (c) (d)

Fr
233 602 633 202

9. If 4*°S9 3 + gIog2 4 83^ ^^ for


(a) 4 (b) 9 (c) 83 (d) 10
ur
4 1 1

10. The value of 3*‘’S4 9 ^ 27^°§36 9 + si^°S5 3 ^ jg


s
ook
Yo

(a) 890 (b) 860 (c) 857 (d) none of these


eB

11. The value


15 15 31
(a) — (c) — (d) ^
our

16 8 32
ad

1
12. If - 2, then x =
log;, 10 l0g„ 10
Y

2 2
a a a
Re

(a)- (b) (c) - (d)


nd

100 10 100
Fi

13. If logi2 27 =fl, then log^ 16 =


3-^7 3-a 4-a 4 + a
(a) (b) 4 (c) 3 (d) 3
3 + a 3 + a 4 + a 4-a

14. The value of log2 log2 log4 256 + 2 log 73 2, is


(a) 2 (b) 3 ^tc) 5 (d) 7
15. If log j _ log y _ log 2
, then xyz is equal to
a-b b-c c-a

(a) 0 (b) 1 (c) -1 (d) 2


ANSWERS

1. (a) 2. (c) 3. (c) 4. (c) 5. (a) 6. (c) 7. (b) 8. (a)


9. (d) 10. (c) 11. (c) 12. (d) 13. (b) 14. (c) 15. (b)
CHAPTER 4

APPLICATIONS OF LOGARITHMS

4.1 COMPOUND INTEREST

w
DEFINITION If the borrower and the lender agree to fix up a certain interval of time (say, a year or a half
year or a quarter of a year etc.) so that thea^nount (= Principal + Interest) at the end of an interval becomes
the principal for the ne.xt interval, then the total interest over all the intervals, calculated in this way is

Flo
called the compound interest and is abbreviated as C.I.
Clearly, C.I. = Amount - Principal

e
re
RESULT 1 Let P be the principal and the rate of interest be r% per annum. If the compound interest is
reckoned annually, then the amount A at the end ofn years is given by

F
( r Y'
A=P 1+-— .
ur
100

r
PROOF We have : P = Principal and rate of interest is r% per annum. Since the interestis reckoned
fo
annually. Therefore,
ks
Pr
Yo

Interest after one year =


oo

100
Pr ( r
B

Amount at the end of one year = P + =P 1 + —


100 100
re

This amount is taken as the principal for the second year.


( r \ r
u
ad

Interest for the second year =


100 J100
Yo

( r ( r r
Amount at the end of second year = P 1 + — + P 1 + —
d

100 100; 100


Re
in

= P
(
1 + —
r
= P 1 +
r f
F

100 J V 100 100 j


Considering this amount as the principal for third year, we obtain
( r f r
Interest for the third year = P 1 + —
100 100

r y r r
Amount at the end of third year = P 1 + + P 1 +
100 100 100

r (
= P 1 + 1+^
100 y V 100 J \ 100

Continuing in this manner, we obtain


( r V
Amount at the end of n years = P 1 + —
100

Q.E.D.
4.2 APPLIED MATHEMATICS-XI

RESULT 2 Let P be the prmcipal and the rate of interest be r % per annum. If the compound interest is
reckoned k-times in a year, then the amount A at the end ofn years is given by
nk
( Y
A =P 1 +——
lOOA:

l»ROQF Since the rate of interest is r % per year and the interest is payable k times in a year. Therefore,
the rate of interest is -k % per interval. Hence, the amount A of principal P at the end of n years or, nk
intervals is
\nk
r/k r
A=P 1 + or, A P 1 +
100 lOOit

Q.E.D.
RESULT 3 Let P be the principal and the rate of interest ber^/ofor first year, r2%for second year, r^% for

w
third year and so on atidin the last ,rf/o for the nth year. Then, amount A at the end ofn years is given by
\ /

A=P 1 +

Flo
100 / V, 100 100

I’ROOF For the first year the rate of interest is rj%.

ee
Pri
Interest at the end of first year =

Fr
100
Pr1
Amount at the end of one year -P + 100
= P 1 +
100
for
ur
This amount is taken as the principal for the second year.
^2
Interest at the end of second year = P 1 +
s
100 J 100
k
Yo

ri ^2
oo

Amount at the end of second year = P 1 + + P 1


100 ) 100 100
eB

i l + fl
r r

= p 1 +
100 JI 100 J
r

Continuing in this manner, we obtain


ou
ad

^1 \ (
Amount at the end of n years = P 1 + 1 + ... 1 +
100 / V 100 100 j
Y

Q.E.D.
When the time is the fraction of a year, then amount is computed as follows;
Re
nd

RESULT 2 IfPis the principal and the interest is r% compounded yearly, then amount A at the end of
Fi

5 2 years is given by
r r/4
A=P 1 + 1 +
100 / V 100
We shall now illustrate the use of these results with the help of following examples.
ILLUSTRATIVE EXAMPLES

Ty/v / ON FINDING THE COMPOUND INTEREST AND AMOUNT


EXAMPLE 1Find the compound interest on ? 12000 for 10 years at the rate of 12% per annum
compounded annually.
SOLUTION We know that the amount A at the end of n years at the rate of r% per annum when
the interest is compounded annually is given by
APPLICATIONS OF LOGARITHMS 4.3

\rt

A=P 1 + —
100

Here, P=ri2000, r=12 and »=10.


nIO nIO nIO
12 25+ r28
/4=^ 12000 1 + = ? 1200011 + — = ^12000 = ?12000
100 25 25 25
xlO
28
Now, /I =12000
25

log A = log 12000 +10 (log 28 - log 25) = 4.0792 +10 (1.4472 -1.3979)
log 71 = 4.0792 + 0.493 = 4.5722

ow
A = antilog (45722) = 37350.00
So, Amount after 10 years ^ 37350.
Hence, Compound interest = ? (37350 -12000) = ? 25350.
EXAMPLE 2 Ift 2730 are invested at 10% interest, compounded every year, for 5 years, what is the

e
amount realised at the end of 5 years ?

re
SOLUTION Here, P = ^ 2730, r = 10, n = 5. Let A be the amount at the end of 5 years. Then,
' f \n

rFl
F
A=P 1 + —
100

10 Ilf

or
A=^ 2730 1 + = ? 2730
ou
100 10
ksf
log A = log 2730 + 5 (log 11 - log 10) = 3.4362 + 5 (1.0414 -1) = 3.6432
A = analog (3.6432) =4397
oo

Hence, A = ? 4397.
Y

EXAMPLES Find the compound interest on^ 7600 at 17% per annum for 2 years, reckoned halfyearly.
B

SOLUTION We know that for the principal P, rate of interest r% per annum compounded k times
in a year, the amount after n years is given by
re

r
oYu

A =P 1 +
im
ad

Here, P =7600, r=17, and n = 2.


d

Since the interest is reckoned half-yearly i.e. twice in a year. Therefore, k =2.
17 \4
n4
in

f217
Re

A =7600 1 + = 7600
200 200
F

log A = log 7600 + 4 (log 217 - log 200) = 3.8808 + 4 (2.3365 - 2.3010)
log A = 3.8808 + 0.0355 x 4 = 4.0228
A = antilog (4.0228) = 10540.00.'
Thus, A =? 10540.

Hence, Compound interest = ^ (10540 - 7600) = ^ 2940.


EXAMPLE 4
Find the compound interest on ^ 6000 for 9 months at 10% per annum, being reckoned
quarterly.

SOLUTION Wehave, P=^ 6000, r=10, = 9 months = — year = — yearandA' = 3

M =P 1 +
r Y^^
lOOA:
3
-x4
A = 6000 1 +
10 '14 = 6000
31
300 30
4.4 APPLIED MATHEMATICS-Xl

log A = log 6000 + 3 (log 41 -log40) = 3.7782 + 3 (1.6128 -1.6021) = 3.8103


A = antilog (3.8103) = 6461
Hence, Compound interest = ? (6461 - 6000) = ? 461.
i:XAMI’LL' 5 Find the difference in the compound interests on ^ 10,000for 4 years at 10% per annum
when reckoned half yearly and yearly.
SOLUTION CASE 1 When compound interest is reckoned half-yearly
P = 10,000, r = 10,« = 4 and k =2. Let Aj be the amount. Then,
r
A.=P 1 +
100 A:

ow
10 2n
A^ =10000 1 + = 10000
200 20

log = log 10000 + 8 (log 21 - log 20) = 4 + 8 (1.3222 -1.3010) = 4.1696


Ai= antilog (4.1696) =14780.

e
So, Compound interest = t (14780 - 10000) = 14780.

re
CASE II

Fl
When compound interest is reckoned yearly

F
P =10,000, r=10, n = 4, A: =1. Let A2 be the amount at the end of 4 years. Then,
ur
r

r
A2=P 1 +
lOOA ;
10 \4 in'
fo
ks
^2 =10000 1 + = 10000
Yo
100 10
oo

=>
log A2 = log 10000 + 4 (log 11 - log 10) = 4 + 4 (1.0414 -1) = 4.1656
eB

A2 = antilog (4.1656) =14640


So, compound interest = ? (14640 - 10000) = ^ 4640.
Hence, required difference = ^ (4780 - 4640) = ^ 140.
ur

EXAMiM.U 6 A Slim of money is puf at cottipound interest for 2 years at 20% per annum. It wouldfetch
ad
Yo

? 482 more, if the interest were payable half yearly than if it were payable yearly. Find the sum.
SOLUTION Let the required sum of money be ^ P.
d

CASE 1 When interest is payable yearly: In this case, let the amount be A]. Then,
Re
in

n2 x2

Aj=P 1 +
20
= pf^ 36P
F

100 J 5; 25

CASE II When interest is payable half-yearly: In this case, principal = P, r = 20, n = 2 and A = 2.
Let A2 be the amount at the end of 2 years. Then,
^ J' ,nk
Aj=P 1 +
^ lOOfc
^4
A2 = P 1 +
20

200
Pf^f
10

It is given that A2~Ai= 482


P
11
p(^ 482
10 5J

in' 12
^2
=482
P <
10 10
APPLICATIONS OF LOGARITHMS 4.5

11 \2 12 11
^2
12 f 24:1 ^ 1
P < + —
!> =482 => P = 482 => P =20,000
10 10 10 10 100 A100
/ V

Hence, the sum of money was ^ 20,000.


, ... ON FINDING THE PRINCIPAL WHEN COMPOUND INTEREST AND AMOUNT ARE GIVEN

EXAMPLE 7 What simi will amount to ? 9,000 at 12% per annum compound interest reckoned yearly
for 5 years ?
SOLUTION Let the principal be ^ P. It is given that A = 9,000 , r = 12 and n=5 .
( T
A=P 1+ —

ow
100

9000 = P 1 +
12 f
100

28
9000 = P

e
U5

re
log 9000 = log P + 5 (log 28 - log 25)

rFl
F
log P = log 9000 - 5 (log 28 - log 25) = 3.9542 -5 (1.4472 -1.3979) = 3.7077
P = antilog (3.7077) =5101

r
Hence, Principal = ^ 5,101.
ou
fo
EXAMPLE 8 What sum will amount to ^ 5,525 at 10% per aimum compounded yearly for 13 years ?
ks
SOLUTION Let the principal be ? P. It is given that
A = 5525, r = 10 and n = 13
oo

f r V'
Y

A=P 1 + —
B

100
13
10 ^
re

5525 = P 1 +
100
ou

n13
Y
ad

11
5525 =P —
10

log 5525 = log P +13 (log 11 - log 10)


d

log P = log 5525 -13 (log 11 - log 10) = 3.7423 -13 (1.0414 -1) = 3.2041
in
Re

P = antilog (3.2041) = 1600


F

Hence, required sum = ? 1600.


li/)V Hi ON DETERMINING THE INTEREST RATE WHEN PRINCIPAL, AMOUNT AND PERIOD
ARE GIVEN

EXAMPLE 9 At what rate percent compound interest would ? 30,000 amount to ^ 14,992 in 18 years ?
SOLUTION Let the rate percent be r% per annum.
We have, P=? 10,000, /l=?ri4992 and h=18
V'
A=p(l+~
100
18
14992 =10000fl + —
100
1/18
14992
= 1+-!
10000 100
4.6 APPLIED MATHEMATICS-XI

f r 1
log 1 + — = — (log 14992 - log 10000)
100 18
( r 1
log 1 + — = _ (4.1759 - 4) = 0.009772
100 18

1 + ^ = antilog (0.009772) =1.0223


-=1.0223-1 =0.0223
100
r = 2.23

Hence, the rate is 2.2% per annum.

ow
EXAMPLE 10 \Nhat rate percent per annum compound interest will ^ 2000 amount to ^ 3000 in 3 years
if the interest is reckoned half-yearly.
SOLUTION Let the rate of interest be r% per annum. If the interest is reckoned cf-times in year,
then amount at the end of n years is given by

e
r

re
A=P 1 +
WOk)

rFl
F
Here, A = 3000, P = 2000, n=3 and k=2
r
3000 = 2000 1 +

or
ou
200

3 f1 r '1^'’
ksf
+
2 200
oo

1-f
2)
Y

200
B

1 1
log l + = - (log 3 - log 2) = - (0.4771 - 0.3010) = 0.02935
200 6 6
re

1 + —
= antilog (0.02935) = 1.070 => ^ = ^^
oYu

200
ad

Hence, the rate is 14% per annum.


At zuhat rate percent per annum compound interest will ^ 5,000 atnount to ^ 9,035 in 5
d

1 XAMPLE 11

years, if the compound interest is reckoned quarterly.


in
Re

SOLUTION Let the rate of interest be r% per annum. We know that the amount A after n years of
F

the principal P is given by


r Y'^
A = P 1 + , if the interest is reckoned k times in a year.
look

Here, P = 5,000, A = 9,035, » = 5 and k = 4.


20
f r
9035=5000 1 + —
400

?
. \20 9035
1 +
400 5000

79035^^^'^
400 5000
( r
log l + -:7
400
=—
20
(log 9035 - log 5000) = —
20
(3.9559 - 3.6989) = 0.0128
APPLICATIONS OF LOGARITHMS 4.7

1 + J-
400
= antilog^ (0.0128) = 1.030 => —
400
= 0.030 => r = 12
Hence, the rate is 12% per annum.
EXAMPLE 12 At zvhat rate percent per annum will an amount double itself in six years.
SOLUTION Let the rate of interest be r% per annum. Then, amount = 2P, principal = Pand n = 6.
\n
A=P I+ —
100

r
2P=P 1 +
100

2= 1 +
100

1t^ = 2'/6
100

w
r ^ 1. , 0.3010
log 1 + = 0.0501
100 J 6 =^

1 + — = antilog (0.0501) ^ ^ ^ = ^-122 => — = 0.122 =>


100

Hence, the rate of interest is 12.2% per annum.


F lo 100 r = 12.2

ree
F
ilfpc IV ON DETERMINING TIME WHEN PRINCIPAL, AMOUNT, RATE OF INTEREST ARE GIVEN
EXAMPLE 13 In how much time, at 10% compound interest zvill^ 500 amount to ^ 665.60?
for
SOLUTION Let the required time be n years. We know that the amount A of principal P at the end
r
of n years at the rate of r% per annum compounded annually is given by
You

V'
f r
oks

A=P 1 + —
100
eBo

Here, P = ^ 500, A=t 665.60, r = 10


10 V'
665.60=500 1 +
our
ad

100

665.60

500 UO
dY
Re

n (log 11 - log 10) = log 665.60 - log 500


log 665.60 - log 500 _ 2.8232 - 2.6989 _ ^
Fin

n =

log 11 - log 10 1.0413-1

Hence, the required time is 3 years.


EXAMPLE 14 In hozv many years zoill an amount double itself at 12.2% cgnipoiinded annually?
SOLUTION Let the principal amount be ^ P, and let the required number of years be n. Then,
amount = 2P.

12.20 V’
2P=P 1 +
100

12.2
2=1 +
100 j

^_ni2.2
100 J
4.8 APPLIED MATHEMATICS-X!

log 2 0.3010
=>
log 2=n(log 112.2 - log 100) => » = = 6
log 112.2-log 100 2.0500-2
Hence, required time = 6 years.
In how many years an amount treble itself at 11% compounded half yearly?
..
SOLUTION Let the principal be P and, let the required number of years be n. Then, amount = 2P.
We know that, if the interest is compounded k times in a year. Then,
r
A=P 1 +
lOOfc

Here, k=2, r=ll.


11
A=P 1 +
200^

w
f211\^
x2n
211
2P=P => 2 = => log 2 = 2n (log 211 - log 200)

F lo
Uoo 200

n -
log 2 0.3010 0.3010
= 6.48

ee
2 (log 211 - log 200) 2 (2.3242 - 2.3010) 0.0464

Fr
Hence, required time = 6.48 years = 6 years 5 months and 25 days.
/ I//V WHEN RATE OF INTEREST VARIES AFTER EVERY INTERVAL OF TIME.
for
ur
rXAMPLF Iti
Find the compound interest (reckoned yearly) on ^ 10,000for 4 years at 10% forfirst year,
12% for the second year, 14% for the third year and 15% for the fourth year.
s
SOLUTION Let A be the amount after 4 years. Then,
ook
Yo

10 12 V. 14 15
/I =10000 1 + 1 + 1 + 1 +
100 100 100 100
eB

/ V / \

A =10000 '110Y112Y114V115^ 11 X 112 X 114x115


.looJuooJuooJliTO 1000
r
ou
ad

log A = log 11 + log 112+ log 114 + log 115 - log 1000
=> log A = 1.0413 + 2.0492 + 2.0569 + 2.0606 - 3 = 4.2080
Y

A = antilog (4.2080) = 16,140


Re
nd

Hence, the required amount is ^ 16,140.


Fi

Ti/;v ; WHEN TIME IS THE FRACTION OF A YEAR


EXAMPLE Find the compound interest on ^ 2000 at 12% per annum for 4 years 3 months.
SOLUTION Here, P = ^ 2000, r = 12, n = 4 years 3 months
- 4 ^ years.
\4 r 103
>1 =2000 1 +
12
X 1 + (1/4) X 12^ = 2000
112

100 100 100 y V 100

log A = log 2000 + 4 (log 112 - log 100) + (log 103 - log 100)
log >1 = 3.3010 + 4 (2.0492 - 2) + (2.0128 - 2) = 3.3010 + 0.1968 + 0.0128 = 33106
>1= antilog (35106) = 3240
Hence, Compound interest = ? (3240 - 2000) = ? 1240.
EXERCISE 4.1

1. Find the compound interest on ? 20,000 for 6 years at 10% per annum compounded
annually.
APPLICATIONS OF LOGARITHMS 4.9

2. Find the compound interest on ? 600 at 10% per annum for 13 years.
3. Find the compound interest on ? 8500 at 14% per annum for 2 years, reckoned half yearly.
4. Find the compound interest on f 32000 for 15 months at 12% per annum, being reckoned
quarterly.
5. Find the compound interest on ? 7250 at 14% per annum for 1 ^years, reckoned half yearly.
1
6. Find the compound interest on ? 6250 at 14% per annum for 1 — years when it is calculated
half yearly.
7. What will amount to ^ 9000 at 12% compound interest reckoned yearly for 5 years ?
8. What sum at 4% compound interest will amount to 110,000 in 18 yearsy?
9. If a sum of money amounts to ? 3000 in 20 years at 5% per annum compound interest, find

w
the sum.

10. If a sum of money amounts to ^ 100900 in 31 years at 25% per annum compound interest,

F lo
find the sum.

11. At what rate percent per cinnum compound interest will ? 8000 amount to 12260 in 3 years

ee
compounded half-yearly?

Fr
At what rate percent compoimd interest would ? 3000 amount to ? 4200 in 2 years ? Given
that the interest is compounded half-yearly.
13.
for
Find the time in which a sum of money will treble itself at compound interest at 8% per
ur
annum.

At what rate percent per annum compound interest will ? 1695 amount to ^ 4500 in 20
s
ok

years ?
Yo

15. Find the compound interest on ^ 12000 for three years, the rate of interest being 10% for the
o
eB

first year, 12% for the second year and 14% for the third year.
16. Find the compound interest on ? 2400 at 10% per annum for 2 years and 4 months.
ANSWERS
r
ad
ou

?15430 2. ? 1472 3. ?2640 4. ?5050


Y

^\632 6. ? 1408 7. ^5101 8. ^4922


^1130 10. ^100 11. 14.7% 12. 17.55%
Re
nd

13. 14.28 years 14. 5% 15. ? 4860 V. ?601


Fi

4.2 POPULATION GROWTH

When an entity (such as population, industrial production etc.) increases in magmtude over a
period of time, we say that it has grown during that period. The growth over a period of time is
defined as the ratio of the increase in magnitude of the entity during that period of time to its
initial magnitude. Thus, if Pq is the magiutude of an entity in the beginning of a unit of time and
Pi is the magnitude at the end of the unit of time. Then,
Growth in one unit of time = — ^
Po
The growth in one unit of time or growth per unit of time is called the "rate of growth"
Generally, the rate of growth is expressed as a percentage. Thus, if we say that the rate of growth
is r%, then
Pi-Pq_ r => P1 = Pq 100
Po 100 Po 100 Po 100
4.10 APPLIED MATHEMATICS-X!

Thus, if Pq is the initial magnitude of an entity and the rate of growth is r% per unit of time, then
the magnitude of the entity at the end of a unit of time is given by Pg 1 +

RESULT I //Pq is the population at the beginning of a certam year and r% per annum be the constant rate
( r V'
ofgrozoth. Then, population after n years is given by P„ = Pq 1 + .
I’KOOl
Let P[ be the population after one year. Tlien,
Rate of growth = ——
Po
But the rate of growth is r% per year.
P^-Pq_ r
Po 100

Pi1 - Pqu = — Pq => “ -^00 ~~ Pq ^ Pz1 - Pni^ ~

w
JOO ^ 1 100 0 0
i 100

Let P2 be the population after Uvo years. Then,

P2 = Pi 1 + -^
100
= P„ 1 + —
u iooJ(^
F lo
1+—
100
^ P2 = Po
(
100
r

e
Fre
Similarly, if P3 is the population after 3 years, then
r
for
P3=Pq 1 +
100

Continuing in this manner, if P„ is the population after n years, then


r
You
oks

Pn = +
100
eBo

RESULT II LetPQ be the initial popidation and the rate of growth ber-^% in the first i/ear,r2 % in the second
year, r^ % in the third year and so on and r„ % in the nth year. Then tlie population after n years is given by
ad

\ /
our

^2
P.,n =Pn0 1 + 1 + i + j!
100 / V 100 100

These results are very similar to the results for compound amoimt. So, all calculations are done
in the same way as illustrated in the following examples.
Re
dY
Fin

ILLUSTRATIVE EXAMPLES

rXAMPLH 1 In the 2001 census, the population of India was found to be 8.7 x 10^. If the population
increases at the rate of 2.5% every year, lolmt zuould be the population in 2021 ?
SOLUTION Here,Po=8.7xlO^,r = 25 and fi = 20.
Let P be the population in 2021. Then,
Y'
P = Pofl +
r

100
x20
25
P= 87x10^ 1 + = 8.7 X 10^ (1.025) 20
100

Taking log of both sides, we get


log P= log {87 X 10^ (1.025)^*^!
APPLICATIONS OF LOGARITHMS 4.11

log P = log (8.7 X 10^) + log (1.025)


20

log P = log 8.7 + log 10^ + log (1.025)


20

log P = log 8.7 + 7 log 20 + 20 log (1.025)


log P = 0.9395 +7 + 20 (0.0107)
log P = 0.9395 + 7 + 0.214
log P= 8.1513
P = analog (8.1513) = 1.4167 x 10®
EXAMPLE 2 In 1980, the population of the state of Andhra Pradesh and of France zuas roughli/ the sajjie
(about 5.4 X 10®). If the population of Andhra Pradesh grozvs at the rate of 2.4 % per year and that of

ow
France grows at the rate of 1.8% per year, shozv that in the year 2100, the population of Andhra Pradesh
will be roughly tzoice that of France.
SOLUTION LetP^ and Pp be the populadons of Andhra Pradesh and France respecUvely in the
year 2100 i.e. 120 years after 1980.

e
Computation of population of Andra Pradesh: We have, Pq =5.4 x 10®, n =120 and r = 2.4

re
P^ =5.4x10' 1 +
2.4 ^120
= 5.4x10° (1.024)
rFl 120

F
100
8
Computation of population of France: We have, Pq =5.4 x 10 , ;i = 120 andr =1.8

r
120
ou
Pp=Po 1 + 100
r
fo
ks
120
8 120
Pp =5.4x10° 1 + = 5.4 X 10” (1.018) ...(ii)
oo

V 100
Y

P^ _5.4x 10® (1.024)


120
eB

Now,
Pp 5.4 X 10® (1.018)
120

120
ur

P^ _f 1-024
Pf
ad

1.018
Yo

n120
Pa (1.024
log ^ = log [Taking log of both sides]
1.018
d

F )
Re

( Pa
in

log iA =120 (log 1.024-log 1.018) =120(0.0103 - 0.0078) = 120 [0.0025] = 0.300
[Pf J
F

— = analog (0.300) =1.995 = 2 approximately.


Pp
Pa=^Pf
EXAMPLE ,3 The population of a town is 120,000. If it increases 7% annually, zohat will be the
population of the tozon after 2 years ?
SOLUTION Let P be the population of the town after 2 years.
Here, Pq =120,000, r =7 and h = 2.
r V'
P=Pn0 1 +
100
2

P=120,000|l + -^1
100
4.12 APPLIED MATHEMATICS-XI

(107
P = 120,000
100

no7
log p = log 120,000 [Taking log of both sides]
100 j
no7
log P = log 120,000 + log = log 120000 + 2 (log 107 - log 100)
.100
log P =5.0792 + 2 (2.0294 - 2) =5.0792 + 0.0588 = 5.1380
P = antilog (5.1380) = 137,400
Thus, population after 2 years = 137,400.
EXAMPLE 4 The population of a toivn in the year 2014 was 250,500. If the annual increasing during
three successive years be at the rate of 7%, 8% and 6% respectively, find the population at the end of 2017.
SOLUTION Let P be the population at the end of 2017. Here,

w
Pq =150,500, rj =7, r2 = 8 and r^=6.
^3 1

F lo
0
1 + 1 +^ 1 +
100 / V 100 / \ 100

7 V 8 V 6 \ m
=> P =150,500 1 + 1 + 1 + — =150500
100 "" 100 ^ 100

ee
lOOA 100 / V 100

Fr
_ 1505 X 107 X 108 X 106
10000 for
1505x107x108x106'!
log P = log
ur
10^
s

4
log P = log 1505 + log 107 + log 108 + log 106 - log 10
ook
Yo

log P = log 1505 + log 107 + log 108 + log 106 - 4 log 10
eB

log P = 3.4775 + 2.0294 + 2.0334 + 2.0253 - 4 =5.2656


P = antilog (5.2656) =184,400
our

The present population of a country is 9,261,000. If it has been increasing at the rate of 3%
ad

EXAMPLE 5

annually, what was its population 3 years ago?


SOLUTION Let pQ be the population 3 years ago. Here, P = 9,261,000, r = 3 and n = 3.
dY

f r V*
Re

100
Fin

f 3 103 (103)^
9,261,000 =Po 1 + — => 9,261,000 = Pq => 9,261,000 =pQ
^ 100 100 (100)^
9,261,000X(100)^ 9261X10^
(103)^ (103)^
9261X10^
log Pq = log ● = log 9261 + 9 log 10 - 3 log 103
(103)^
log Pq = 3.9667 + 9-3 (2.0128) = 12.9667 - 6.0384 = 6.9283
Pq = antilog (6.9283) = 8,476,000
Hence, its population was 8,476,000.
EXAMPLE 6
The population of a town at the beginning of the year 2016 was 265,000. If the rate of
increase be 52 per thousand of the population, find the population at the beginning of the year 2021.
APPLICATIONS OF LOGARITHMS 4.13

52
SOLUTION We have, r = Rate of increase = xlOO =5.2%, tt =5, Po= 265,000
1000

Let the population at the beginning of 2021 be P. Then,


V'
f f
100

5.2 105.2
P= 265,000 1 + = 265,000
100 100

log P = log 265000 + 5 (log 105.2 - log 100) = log 265000 + 5 (log 105.2 - 2)
log P =5.4232 + 5 (2.0220 - 2) =5.4232 + 0.1100 =53332
P = antilog (55332) = 341,400.

ow
Hence, required population = 341,400.
EX.'^MPLE 7 The population of tivo states A and B are 8.25 x 10^ and 1.11 x 10^ respectively. If the rate
of growth in these states be 2.7% and 2.6% respectively. In how many years the population of two slates
will be equal?

e
re
SOLUTION Let the populations of two states will be equal in n years. Then,
n

rFl 2.7
= 8.25 X 10^' X (102.7

F
Population of state A in n years = 8.25 x 10 1 +
100 100 )
V’
8 2.6 V' 8 f 102.6

r
Population of state B in n years =1.11 x 10 1 + = 1.11x10
ou
100 100

r 102.6 Y'
fo
ks
8.25 X 10
7 r 102.7 V' = 1.11x10
8

100 100
oo

ri02.7Y' 102.6
8.25 = 1.11 X 10
Y

100 , 100
B

H »
(102.7) (102.6)
re

8.25 X = 11.1 X
n n
(100) (100)
ou

8.25 X (1027)" =11.1 X (102.6) [Multiplying both sides by 100"]


Y
ad

log [8.25 X (1027)"] = log [11.1 X (102.6)"] [Taking log of both sides]
d

log 8.25 + n log 1027 = log 11.1 + n log 102.6


in

n (log 102.7 - log 102.6) = log 11.1 - log 8.25


Re

n (2.0116 - 2.0112) =1.0453 - 0.9165


F

0.1288
n (0.0004) =0.1288 => « = = 322 years.
0.0004

EXAMPLE 8 The bacteria in a culture grows by 7% in theifrst hour, decreases by 6% in the second hour
and again increases by 5% in the third hour. If at the end of third hour the count of bacteria is 11,270,000,
find the original count of bacteria in the sample.
SOLUTION Let the original count of bacteria in the sample be N. Then,
6 \r 5 ^
N 1 + 1 + = 11,270,000
100 / V 100 /V 100

N 107V 94 yiQ5 = 11,270,000


100 / V looJuoo
10
N =
11,270,000 X (100)^ 1127x10

107 X 94 X 105 107 X 94 X 105


4.14 APPLIED MATHEMATICS-XI

log N = log 1127 + log 10^° - log 107 - log 94 - log 105
log N = 3.0518 + 10-2.0294-1.9731 -2.0212-13.0518-6.0237 =7.0281
N = antilog (7.0281) =10,670,000
Hence, original count of bacteria = 10,670,000.
EXAMPLE 9 A population grows at the rate of 8% per pear. How long does it take for the population to
double ?

SOLUTION Let Pq be the initial population and let the population is doubled in n years. Then,
P=2Pq. Here,r = 8%

ow
V'

100 J
8 108 Y' no8Y
2Po=Po 1 + => 2 = => log 2 = log
100 j 100 100 j

e
log 2 = >2 (log 108 - log 100)

re
n =
log 2 0.3010 _ 0.3010 _ 3010 = 9.01
log 108 - log 100 2.0334 -2 ~ 0.0334 ^

Frl
F
Hence, the population is doubled in 9.01 years.
ou EXERCISE 4.2

osr
1.
The population of a town is 80,000. If the population increases annually at the rate of 75 per
thousand, find the population of the town after 2 years. kf
2.
The population of a town in the year 2018 was 425,000. Find its population in 2020, if the
rate of increase is 4% per year.
oo
3. The present population of a town is 140,000. What will be the population of the town after
Y

2 years if it increases 5% annually ?


B

4.
The population of a town increases at the rate of 50 per thousand. Its population after
2 years will be 22050. Find its present population.
re
Y

5. The present population of a city is 9,261,000. If it has been increasing at the rate of 5%
u

annually, find its population 3 years ago.


A population grows at the rate of 5% per year. How long does it take for the population to
ad

6.
do

double?
7.
The bacteria in a culture grows by 8% in the first hour, decreases by 8% in the second hour
in

and increases by 7% in the third hour. If at the end of the third hour the count of bacteria is
Re

12,170,000, find the original count of bacteria in the sample.


F

8.
In a factory the production of scooters rose to 46,305 from 40,000 in 3 years. Find the annual
rate of growth of the product of scooters.
9.
The population of a town was 160,000 three years ago. If it had increased by 3%, 2.5% and
5% in the last three years, find the present population of the town.
10.
The population of a town was 218,405 in the year 2016. If the rate of increase is 4.5%, find the
population in the end of the year 2020.
11.
The bacteria in a culture grows by 10% in the first hour, decreases by 10% in the second
hour and again increases by 10% in the third hour. If at the end of third hour the count of
bacteria is 131,769,000, find the original count of bacteria in the sample.
ANSWERS

1. 92,450 2. 459,680 3. 154,350 4. 20,000


5. 8,000,000 6. 14.2 years 7. 11,450,000 8. 5%
9. 177,366 10. 3,819,758 n. 1.21x10^
APPLICATIONS OF LOGARITHMS 4.15

4.3 DEPRECIATION

It is a well known fact that the constant use of any machine or any other article causes wear and
tear due to which its value decreases with time. The relative decrease in the value of a machine
over a period of time is called its depreciation. Depreciation per unit time is called the rate of
depreciation. The value at any time is called the depreciated value.
RESULT 1 If Vg is the value i/of an article at a certain time and r% per annum is the rate of depreciation,
then the value at the end ofn years is given by
r r
n 0
1-
100

IM^OQF The rate of depreciation is r% per annum.

ow
Vr,0 r «

Depreciation in one year = 100


Vnr
0
Depreciated value at the end of a year = Vg - = K0
100 100

f ^ 'l r

e
Depreciation in the second year = Vg 1

re
100; 100

rFl { r ) ( r r (

F
=> Depreciated value at the end of two years = Vg \ 1 100 y
-V'g \ 1- —
100
b: = ^o 1- 100 j
7100

r r

or
ou
Continuing in this manner, we get V„ = Vg 1 - 100
ksf
Q.E.D.

RESULT 2 If Vq is the value of an article at certain time and the rate of depreciation is r-^o/^for ifrst n-^
oo

years, r2% for next »2 y^nrs and so on and ryf/o for the last iif^ years, then the value at the end of
Y

/j-j + n^ Uj. years is given by


B

Y'l f V'fc
n f'2 1 -
V=Vn0 1- 1 -
re

100 100 100 ;


oYu
ad

ILLUSTRATIVE EXAMPLES

ON DETERMINING THE DEPRECIATED VALUE WHEN RATE OF DEPRECIATION AND


d

Type I
INITIAL VALUE ARE GIVEN
in
Re

EXAMPLE 1 A neio car costs ? 360,000. Its price depreciates at the rate of 15% a year. What zoill be the
F

price of the car after 5 years ?


SOLUTION If r% per annum is the rate of depreciation of an article and Vg is its value at certain
point of time. Then its value V„ after n years is given by
r r
]] 0 1-
100

Here, Vg =? 360,000, r=15 and n=5.


Vc5^ = 360,000fl-
15 17
= 360,000 —
I 100; 20

log V5 = log 360000 + 5 (log 17 - log 20) =55563 + 5 (1.2304 -1.3010) =5.2033
V5 = analog (5.2033) =159,700
Hence, the depreciated value after 5 years will be ? 159,700.
EXAMPLE 2 A new car costs ? 360,000. Its price depreciates at the rate of 10% a year during the first two
years and at the rate of 20% a year thereafter. What will be the price of the car after 10 years ?
4.16 APPLIED MATHEMATICS-XI

SOLUTION Let price of the car after 10 years. Then,

—TJ fiV 20 f V'jt


n 1-^
V^10 = 360,000 1 - UsingV;, =Vo 1- .... 1 -
100 100 100 100 100

9
=> V^io = 360,000 —
10 / V 5)
=> log = log 360,000 + 2 (log 9 - log 10) + 8 (log 4 - log 5)
log Vio = 55563 + 2 (0.9542 -1) + 8 (0.6020 - 0.6989) = 53563 - 0.0916 - 0.7757 = 4.689
=> VjQ = antilog (4.689) =48,870
Hence, the depreciated price of car after 10 years will be ? 48,870.
IXAMPI.E:- A machine depreciates in value in a year by 6% of its value at the beginning of the year. If

w
the value of the new machine be ? 625,000, what is its depreciated value after 7 years ?
SOLUTION Let V7 be the depreciated value after 7 years.
We have, Vq = 625,000, r = 6 and n=7.
6 f

Flo
n7
f 47 r r
Vj = 625,000 1 - = 625,000 — Using :V„ = Vq 1 -

ee
100 50 100 j

Fr
=>
logV7 =log 625,000+ 7 (log 47-log50) =5.7959 + 7 (1.6721 -1.6990)
log Vy =57959 - 0.1883 =5.6076
for
ur
Vj = antilog (5.6076) = 405,200
Hence, the depreciated value after 7 years = ^ 405,200.
s
k

/ i/)v U ON DETERMINING THE INITIAL VALUE WHEN DEPRECIATED VALUE AND TIME
Yo
oo

PERIOD ARE GIVEN


eB

EXAMPLE 4 The value of a Xerox machine depreciates by 11% annually. If its present value is ^ 38,440,
find its value three years ago.
SOLUTION Let Vq be the value of Xerox machine 3 years ago. Then,
r
ou
ad

11
38440 =Vn0 1-
100
Y

89
38440 =y0
Re
nd

100
Fi

log 38440 = log Vq 3 (log 89 - log 100)


45848 = log Vq+ 3 (1.9494 - 2)
log Vq = 45848 - 3 (1.9494 -1) = 47366 => Uq = antilog (4.7366) = 54,530.
Hence, the value of the machine 3 years ago was ^ 54,530.
LX.XMPLt The value of a machine depreciates 10% annually. Its present value is ? 45,450, find its
value 5 years ago.
SOLUTION Let Vq be the value of machine 5 years ago. Then,
10 '■ 9 10
45450 = Vq 1 - => 45450 = y0 => Vq = 45450 X
100 10 V 9
log Vq = log 45450 + 5 (log 10 - log 9) = 4.6576 + 5 (1 - 0.9542) = 4.8866
Vq = antilog (4.8866) = 77001.64
Hence, the value of the machine 5 years ago was ? 77,001.64.
APPLICAT.ONS OF LOGARITHMS 4.17

: '// ON DETERMINING THE LENGTH OF DEPRECIATION PERIOD. WHEN INITIAL VALUE.


DEPRECIATED VALUE AND RATE OF DEPRECIATION ARE GIVEN
EXAMIM !
A typing machine zvorth t 12,000 is depreciated at the rate of 10% a year. It ivas sold
eventually as zvaste metal for f 200. Find the number of years during zvhich the machine zuas in use.
SOLUTION Suppose the machine was in use for n years.
We have.
r r
K,n =V"n0 1-
100 J
Here, Vg = ? 12000, V„ = ^ 200, r = 10.
10 Y'
200=12000 1 -

w
100

9 Y'
1 =60 => log 1 = log 60 + n (log 9 - log 10)
10

o
1.7781
0=1.7781 +n (0.9542-1) => 0 =1.7781 - 0.0458 » => n = = 38.8

e
0.0458

re
Hence, the machine was in use for the last 38.8 years.

rFl EXERCISE 4.3

F
1.
The value of a machine depreciates at the rate of 6% per annum. If its present value is
^ 625,000, what will be its value 7 years hence ?

r
ou
The value of a machine depreciates by 13% annually. If its present value is ? 39,304, find its
fo
value 3 years ago.
ks
3. A new car costs ? 250,000. Its price depreciates at the rate of 5% a year during the first two
years and at the rate of 10% a year thereafter. What will be the price of the car after 10
oo

years ?
Y

The value of a residential flat constructed at a cost of


^ 550,000 is depreciating at the rate of
eB

4.

15% per annum. What will be its value 5 years after construction.
A machine the life of which is estimated to be 10 years, costs ^ 10,000. Calculate its scrap
r

value at the end of its life, depreciation being 10% per annum.
ou
Y
ad

ANSWERS

1. ^ 405,200 2. ^59,690 3. f 97124 4. ? 244,037.90


d

5. ^ 3483
Re
in
F
CHAPTER 5
QUANTIFICATION AND
NUMERICAL APPLICATIONS
£.

5.1 INTRODUCTION

ow
In this chapter, we will discuss some numerical problems on averages, time, work and distance,
calender, clock, mensuration and seating arrangements.
5.2 AVERAGE

The word average is not new to us. In our day-to-day life we frequently use this word. For

e
re
Fl
example, average run score of a batsman, average salary of a group of workers, average
temperature of a month in a city, average marks of a class in mathematics etc.

F
AVERAGE //.V| X2, x^,..., area values of a variable, then the average of these values is given by
ur
Xi + .^2 + a.'3 +... + x,j

r
Average =
n

Sum of the observations or values


fo
ks
i.e.
Average =
Number of observations or values
Yo
oo

REMARK 1 The average is also called the mean.


NOTE All observations or values, whose average is computed, must be in the same unit.
eB

REMARK 2 IfX is the average of n observations .\q, X2,—, x^t Then,


X1+X2 + ... + Xji
ur

X =
n
ATi + .x'2 -i- ...+x„ =nX
ad
Yo

I.e., Sum of all observations = (Number of observations) (Average).


5.2.1 SOME USEFUL RESULTS ON AVERAGE
d
Re

If each observation in a given set of observations is increased (decreased) by k, then their


in

RESULT 1

average is also increased (decreased) by k.


F

RESULT 2 If each observation in a given set of observations is multiplied or divided by a non-zero


number k, then their average is also multiplied or divided by k.
. . . (n + l\
RESULTS The average offirst n natural numbers is —
V 2

RESULT 4 The average offirst n even natural numbers is (n +1).


RESULTS
The average offirst n odd )iatural numbers is n.
RESULT 6 If each observation in a given set of observations is decreased by their az’erage, the average of
the neio set of observations is zero.
RESULT?
If there are an odd number (say n) of consecutive natural numbers arranged in ascending or
-

descending order, then their average is the V —2 munber.


5.2 APPLIED MATHEMATICS-XI

KtiSLLT 8 If there are ai\ even number (say n) of consecutive natural numbers arranged
th
in
f It
ascending or descending order, then their average is the average of — number
V 2
th
fu
and — +1 number.
u
RESULT 9 The average of 77 consecutive natural numbers .r, .x + l, x + 2, x+3,.r+ (f7-l) is
f n-l\
x +
\ 2
RESULT 10 The average of 77 consecutive odd natural numbers x, x + 2, x + 4,... + x+ 2 (ti-l) is
X + (77-l).
RESULT i I The average of n consecutive even natural numbers x.x + 2, x + 4,..., x + 2(77-1) is
X + (77-l).
aFSL'ITI^ 2 The average of natural numbers from tt to l7 is

w
K'^rn ; The average of all even natural numbers from rt to is

F lo
KI.SL'l.I ! The average of ail odd natural numbers from 77 to is

ee
ILLUSTRATIVE EXAMPLES

Fr
I X/\Mi’Li: 1 the average offirst >1 natural numbers.
SOLUTION Let X be the average of first 77 natural numbers i.e. 1,2,3,..., n. Then, for
1 + 2+ 3 + ... + 77
ur
X =
77

77 (77+ 1) 77 + 1 77 (77 + 1)'


s
x = => x = 1 + 2 + 3 +... + 77 =
ok
Yo

277 2 2
o

\ \ Mi’Ll' 2 FitJd the average offirst n even numbers.


eB

SOLUTION First 77 even natural numbers are 2,4, 6,..., (277-2), 2n.
Let X be their average. Then,
2 + 4 + 6 +... + (277 - 2) + 277
r

X =
ad
ou

77

2\l+2+3+ ...+ (77-1) +77


Y

X =
Re
nd

77

77 (77+ 1)
Fi

X = 2' 2 77 (77+ 1)
1 + 2 + 3 + ... + (77 — 1) + 77 =
77 2

X=77+l.

; \ AMlM-i;'' Find the average offirst n odd natural numbers.


SOLUTION First n odd natural numbers are 1,3,5, 7„ ..., (2?7 -1). Let S be their sum. Then,
S=l + 3+5+7 + ... + (277-l)
5 =
^{2xl+(f7-l)x2} [●.● 1, 3,5, 7,..., (277-1) is an A.P. witli <7=1 and d = 2]
5 = 77 (1 + 77 -1) = n^
Let X be the mean of first n odd natural numbers. Then,
2
X =
1 + 3 + 5 + ... + (277-1) 77
=> X = —=>X=77.
77 77
M f
t'

1.11

M. c\

y'
is 48,
^ ^^»^Udofthele„st and the
i

w
A, V
\O^^0
; <»-ftUiab
-0/
Flo
\ >
●j
-^§e result 9'

e
■«S^.

\
\
re
F
^^ + 4=50.
droatest number =
ur
r

= 46.?0 = 2300 95, W/wl -.ithe-r^^number „ ,h,


fo

\ frv odd numbers is


con?^^^
ks

HenC^'
odd numbers x, ^ -2,1 j^-
Yo

pXA>»
oo

-.rtne
;.ty K < const*"
B

lU flO^^ ' >nd


re

.r iV X
r
X"' tV\T»rn rtUWilVI'SIS
u

<pen
ad
Yo

Idr?^ ● Yc .^rthea'-e^f
jarr~w»
^2iTyO> \
d

iUlU d-U
Re
in

w a
x + -~bL
:est

pf 4 odd nun0f^i
F

.tnaUestnui-H^^ thim the sum



^itl^btive odd i\umb I

8,ia* IS 1/2./-^
.11

\ “»

3^. '^2 'H]+2i


^^3=2^2
●●■(ij
●●●(^)
+
1^ ● yj
■ ^iiV>
A i+y3)+j/2-fe-^^.
(●

= 2(X2 + ^0-^^

■j = 2(;c2 + ^3)~l6
t o

2 [Usi.
3
— {x2 + ^3) + 18 = 2 {a'2 + .t3) -16
S.4

< \
/
\
I
I
\
\

t) ^

1
i

I i

i
Ave^
n
\

*«*rf »*’
«
' ..II L't
w
F lo
e
Fre
for
r
You
oks
eBo
our
ad dY
Re
Fin
5.5
>L applications

= 45^^=15=> i/ = 45
'' 3 3 3

tiie third number is 45.

i^XAMi’LE 12 There are three positive mimk’j's. Oite third of the average of all the three numbers is 8 less
than the value of the highest number. The average of the lowest and the second lowest number is 8. Wiat is
the highest number?
SOLUTION Let the three positive numbers in increasing order be a, b and c and the average of
these numbers be .x. Then,
a +b +c
X =
3

It is given that one third of the average x is 8 less than the highest number c.

w
a +b +c
i.e. -=c-8=> = c-8=> a+b =8c-72 ●●●(ii)
3 9

F lo
It is also given that the average of the lowest and the second lowest number is 8.
a +b
i.e. = 8=> a+b =16 ...(iii)
2

ee
From (ii) and (iii), we obtain

Fr
16=8c-72:z>8c = 88=>c=ll

Hence, the highest number is 11.


for
ur
EXAMPLE 13 The average offive consecutive numbers is x. Ifiiext two numbers are also included, then
show that the average ivill increase by 1.
s
ook

SOLUTION We know that the average of 5 consecutive natural numbers beginning with y i.e.
Yo

i/,y + l,y + 2,y+ 3, i/ + 4isi/ + — = y+2.


eB

x=y + 2 [Usingresult9] ... (i)


If next two numbers i.e. y + 5 and y + 6 are also included, then their average z is given by
our
ad

7-1
z = u +
=y+3 [Using result 9] ... (ii)
2
Y

From (i) and (ii), we obtain z = x + l i.e. the mean is increased by 1.


Re

The average of 8 numbers is 20. The average offirst two numbers is 15.5 and that of the
nd

EXAMPLE 14
64
next three numbers is — . If the sixth number is less than the seventh and eighth by 4 and 7
Fi

respectwely,find sixth, seventh and eighth numbers.


SOLUTION Let the sixth number be x, then seventh and eighth numbers are x + 4 and .r+7
respectively. The average of 8 numbers is 20.
The sum of eight numbers = 20 x 8 =160
The average of first two numbers is 15.5
The sum of first two numbers = 15.5 x 2 = 31
64
The average of next three numbers = —
64
The sum of next three numbers = — x 3 = 64
3

Sum of sixth, seventh and eighth numbers = 160 - 31 - 64 = 65


=> x+(x + 4) + (.Y + 7) =65 => 3x=54=>.v=18
Hence, the numbers are 18,22 and 25.
5.6

The average expenditure of a man for the first five moidh^^^p^ ,


EX \MI’l

seven months it is ^ 5,400. If he saves ^ 2,300 during the year, find his
SOLUTION Total expenditure of first five months = ? (5 x 5,000) = t 25,000 >
Total expenditure of next seven months = ? (7 x 5,400) = ? 37,800
Total savings during the year = ^ 2,300
Total annual income (25,000+ 37,800 + 2,300) =? 65,100
Hence, average monthly income = ^ 65,100^ = ? 5,425
12

EX.AMl’U
When 15 is included in a list of natural numbers, their average is increased by 2. When 1 ISi:
included in this new list, the average of the numbers in the new list is decreased by 1. How many numbers
were there in the original list?

w
SOLUTION Let there be n natural numbers in the given list and let x be their average. Then,
Sum of the natural numbers in the list = nx

Flo
It is given that when 15 is included in the list, the average is increased by 2.
nx +15

e
= x + 2 => x + 2n = 13 ...(i)

re
n +l

F
When 1 is included in the new list, the average is decreased by 1.
«.y + 15 + 1 nx +16
= (x + 2)-l=> = x + l=> 2a: + «=14 ...(ii)
ur
r
n +2 n + 2 fo
Solving (i) and (ii), we obtain x =5 and n = 4.
ks
Hence, there were 4 numbers in the list.
Yo

EXERCISE 5.1
oo

The average of a, b, c and d is 16. Half the sum of b, c, d is 23, find the value of a.
B

If the average of five consecutive odd numbers p, q, r, s, t is 75, then find the value of p.
re

The average of 11 numbers is 60. If the average of first six is 58 and that of the last six is 63,
find the sixth number.
u
ad
Yo

4.
The average temperature of the first four days of a week in Delhi is 40.2®C and that of the
last four days is 41.3°C. If the average temperature of the whole week is 40.6°C, then what is
the temperature of the fourth day of the week?
d
Re

D.
If the mean of 5 natural numbers x, x + 2, x + 4, x + 6 and x+8 is 11, then find the mean of
in

last three natural numbers.


F

6. Of three numbers, the first is 4 times the second and three times the third. If the average of
these three numbers is 95, find the third number.
Out of four numbers, the average of first three is 15 and tliat of the last three is 16. If the last
number is 19, then find the first number.
8,
The average of « numbers is>l. If the number/? is replaced by the number i?, then the average
b -a
becomes B. Prove that n =
B-A
9.
The average of monthly income of A and 6 is ? 60,000; that of B and C is ? 55,000; and A
and C is ? 50,000. What are their monthly incomes?
The average weight of X, Y and Z is 40 kgs. Weight of Z is 24 kgs more thanX's weight and 3
kgs less than Y's weight. What will be the weight ofX, Y, Z and T, if T weights 15 kgs less
that Z?
11-
Tlie average weight of 30 students in a class is 64 kgs. When 20 new students are admitted,
the average weight is reduced by 2 kgs. Find the average weight of new students.
5NS 5.7
... iHCATION AND NUMtRICAL APP

12. The average weight of 3 persons P, Q and R is 84 kgs. Another person X joins the group and
the average now becomes 80 kgs. If another person Y, whose weight is 3 kgs more than that
of X replaces P, then the average wcightage of Q, R,X and Y beomes 79 kgs. What is the
weight of P.
13. If n, b, c are three numbers such that ab+bc + ca = 0 and the average of a, b, c is .v. Then,
2 2 2 2
prove that the average of a , b and c is 3a: .
’ ' Five years ago the average age of P, Q, R and S was 45 years. By including X the present
average of all the five persons is 49 years. What is the present age of X?
ANSWERS
1. 18 2. 71 3. 66 4. 41.8°C 13

6. 60 7. 16 9. A: ^ 55,000, 6: ? 65,000,C; ? 45,000


10. 38 kgs 11. 55 kgs 12. 75 kgs 14. 45 years

low
HINTS TO SELECTED PROBLEM
These are either all even natural number or all odd natural numbers. So, their mean is
.v + (5-l) =a: + 4.
.■..v + 4=11=>a:=7

ee
a: + 4 + .V + 6 + .V + 8 39

F
The mean of last three numbers i.e. a: + 4, a: + 6 and a: + 8 is = 13

Fr
3 3

5.2.2 COMBINED AVERAGE


for
ur
Sometimes, averages of two or more groups or series are given and we have to find the
average of combined group i.e. the observ^ations of all the groups taken together. The
average of all the groups taken together is called the combined average.
k s
Yo

Let there be two groups Gj and G2 with the following details:


oo

Group Group G2
Number of observations :
eB

"1 ”2
Average ; X2
Then,
r

Sum of observations in Gj Sum of observations in G2


ou
ad

Xi = , X2 =
»1 ih
Y

111 X| = Sum of observations in Gj, ^2 X2 = Sum of observations in G2


nd

Sum of observation in both the groups = X^ + X2


Re

LetX be the combined average i.e. average mean of all observations in G^ and G2- Then,
Fi

X =
Sum of the observations in both the groups
Mj + ??2

X =
X^ + 112 X2
111 +112

RE.MARK ! Let _ there he a group of «i observations with average X^. Iffrom this group 112 observation
having average X2 are removed. Then, the average of remaining Ui -«2 observations is
X| — ??2 X2
111 -nj

R1:M.‘\RK2 X =
/z-j X]^ + ^2 X2
111 ”2

(zzj + ^2) X “ H2 X2 (zz-j + ZI2) X ~ ZZ| X|


Xi = and X2 =
’h »2
5.8
'..^PLlEDi^

If there are three groups of observations having averages Xj_,X2,X3 and numbeT^
observations /7|, ri2 and respectively, then the combined average X is given by
X = /ij Xj + ii2 X2 + ^3 Xj
+ H2 ^^3

and. ^ _ (Hj + II2 + Hg) X ~^2 X2 X3 _ (H| + ?i2 + 3) X — Hi Xj — U3 X3


,X2 =
”2

(Nj “t ^2 + U3) X — /Ij X] — ^2 X2


X3 =
”3

ILLUSTRATIVE EXAMPLES

w
rxAMPLE 1 If the average ofm mmibers is and that of n numbers is then find the average of
numbers (m + n) numbers.

SOLUTION Wehave,?i;i = m, =n^,ii2 =n and X2 =m^ .


Let X be the average of (m + n) observations. Then,

F lo
ee
X =
?Jj X^ + ^2 X2 X =
mn^ + nni^ run {m + n) = mn

Fr
+ «2 m + n m + n

EXAMPLE 2 The average annual income (in t) of certain agriculture zuorkers is Sand that of construction
for
workers is T. The number of agriculture workers is 11 times of that of construction xvorkers. Find the
ur
average mcome of all the workers.
SOLUTION We have, = 11^2, X;i = S and X2 =T.
ks
Yo

Let X be the average income of all the workers. Then,


oo

Xj + 7I2 X2 X = {Un2)S + n2T _}i2 (IIS + T) _llS + 7


eB

X =
n-^ +112 11/72 ^^2 ~ 1202 ~ 12
EXAMPLE The average age of section A of 40 students of Class V of a school is 10 years and the average
r
ou
ad

age of section B of 30 students of class VI of the same school is 12 years. What is the average of students in
both the sections?
Y

SOLUTION We have,/7i = 40, Xj =10, .V2 = 30, X2 =12


Re
nd

LetX be the average age of students in both the sections. Then,


/7| X-j + 7/2 X2
Fi

40 X 10 + 30 X 12 760
X = =>x = = 10.85
n-^ + ri2 40+ 30 70

Hence, the average age of students in both the sections is 10.85 years.
EXAMPLE ●! The average of 5 numbers is 60, the average of 10 numbers is 30 and the average of 15
numbers is 20. Find the average of all the 30 numbers.
SOLUTION We have, n-^ =5, Xj = 60,7/2 = 10, X2 = 30,773 = 15, X3 = 20
Let X be the average of all the 30 numbers. Then,
X = 77| X^ + U'y X2 + 7/3 X3
77-j + 772 ^^3
5 X 60 + 10 X 30 + 15 X 20 300 + 300 + 300
X = = 30.
5 + 10 + 15 30

EXAMPLE S
The average of 5 numbers is 20 and the average of three of them is 14, find the average of
remaining numbers.
QUANTIFICATION AND NUMERICAL APPLICATIONS 5.9

SOLOTION We have, n-^ =5, Xj = 20, »2 = 3, X2 = 14


Let X be the average of remaining numbers. Then,
X = Hi Xi -ti2 X2 ^5x 20 - 3x 14 _100-42 = 29
”l-"2 5-3 2

EXAMI’LE 6 /I library has an average of 510 visitors on Sundays and 240 on other days. What is the
average number of visitors per day in the month of June beginning with a Sunday?
SOLUTION In the month of June beginning with a Sunday, there are 5 Simdays and 25 other
days. Thus, we have
n^ =5,Xi =510, ?i2 = 25, X2 - 240
LetX be the average number of visitors per day. Then,
X = 111 Xi + /I2 X2 _ 5 X 510 + 15 X 240 _ 8550 = 285.
+112 5 + 25 30

w
EXERCISE 5.2
1.
In a class of 32 boys and 28 girls. The average age of boys is 14 years and that of girls is 13

F lo
years. What is the average age of whole class?
2.
If the average of six numbers is a and the average of three of these is b. If the average of
remaining three is C, then prove that 2a = b + c.

ee
3.
The average of 6 numbers is 7. The average of three numbers of them is 5. Find the average

Fr
of remaining numbers.
4.
Average score of a class of 60 students, in an examination was 43. Average score of the
for
students who had passed is 52 and that of who had failed is 16. Find the percent age of
ur
failed students.
5.
The average of 12 numbers is 15 and the average of first two is 14. What is the average of the
s

rest?
ook
Yo

6.
The average score of a class of boys and girls in an examination is .r. The ratio of boys and
girls in the class is3:1. If the average score of boys is.v+ l,find the average score of girls.
eB

/.
The average age of students of a class is 15.8 years. The average age of boys in the class is
16.4 years and that of girls is 15.4 years. In what ratio are boys and girls in the class?
8.
In a factory, the average salary of the employee is ^ 7,000. If the average salary of 12 officers
our
ad

is ^ 40,000 and that of remaining employees is ? 6,000. Find the number of employees.
ANSWERS
Y

1. 13.53 years 3. 9 4. 25% 5. 15.2


Re

6. a- -3 7. 2 : 3 8. 408
nd

HINTS TO SELECTED PROBLEM


Fi

6x7
3. Required average = ^=9
6-3

5.3 CALENDER

A calendar is a chart displaying the dates, days, weeks and months of a particular year. In this
section, we will discuss various types of problems related to calendar. Recall that there are 7
days in a week. A week begins with Monday and ends on Sunday. There are 12 months in a
year. There are some other important facts about the calendar. So, let us discuss those facts.
LEAP YEAR If a year, other than a century, is divisible by 4, then it is a leap year.
ORDINARY YEAR If a year is not a leap year, then it is called an ordinary.
If the last two digits of a year are not both zero i.e. it is not a century and the two digit number
formed by them is divisible by 4, then it is a leap year. If last two digits are both equal to zero and
the year is divisible by 400, then it is a leap year. Otherwise, it is ordinary year.
5.10 APPUED ■ ● '■^WPMATICS-Xl

For example, 2000 is a leap year, because it is divisible by 400 but 1900 is not a leap-year.
2012 is a leap year, because the number 12 formed by last two digits is divisible by 4. But, 2014 is
not a leap year.
Each of the years ; 400,800,1200,1600,2000 etc. is a leap year.
Each of the years ; 1648,1872,1904, 2012, 2016 is a leap year.
None of the years : 1400,1700,1900,1971,1994, 2021 is a leap year.
A leap year has 366 days and an ordinary year has 365 days. There are 7 days in a week and
366=52x7 + 2, 365=52x7 + 1

So, a leap year has 52 weeks and 2 extra days where as an ordinary year has 52 weeks and one
extra day.
ODD DAYS In a given period, the number ofdays more than the complete weeks are called odd days.

w
In order to find the number of odd days in a given period, divide the number of days by 7 and
get tl\e remainder. The remainder is the number of odd days.

F lo
For example, in the January month of a year, there are 31 days. Dividing 31 by 7, we obtain 3 as
the remainder. So, there are 3 odd days in January.

ee
In an ordinary year, there are 365 days. 365 =52 x 7 + 1. When wedivide365by 7, weobtain 1 as

Fr
the remainder. So, an ordinary year has one odd day.
A leap year has 366 days 366 =52 x 7 +2. On dividing 366 by 7, we obtain 2 as the remainder. So,
for
a leap year has 2 odd days.
ur
The number of odd days associated to each day of the week are listed in the following table.
s
ook

Day: Sunday Monday Tuesday Wednesday Thursday Friday Saturday


Yo

Number of
eB

0 1 2 3 4 5 6
odd days
our

Let us now find the number of odd days in a century.


ad

Number of odd days in WO years


100 years = 24 leap years + 76 ordinary year [Note that 100'” year is not a leap year]
Y
Re

.'. Number of odd days in 100 years


nd

A leap year has two odd days and an


= 24 X 2 + 76 X 1 odd days
Fi

ordinary year has one odd days


= 48 + 76=124

Now, Number of odd days in 124 days = Remainder when 124 is divided by 7 = 5
Hence, there are 5 odd days in 100 years.
Number of odd days in 200 years:
There are 5 odd days in 100 years. Therefore, number of odd days in 200 years is5 x 2 =10. But,
when 10 is divided by 7, the remainder is 3. Hence, there are 3 odd days in 200 years.
Number of odd days in 300 years:
There are 5 odd days in 100 years. So, there are 3 x 5 = 15 odd days in 300 years. But, when 15 is
divided by 7, the remainder is 1.
Hence, there is 1 odd day in 300 years.
QUANTIFICATION AND NUMERICAL APPLICATIONS 5.11

Number of odd days in 400 years:


As there are 5 odd days in 100 years and 400 = 4 x 100. So, there must be 4 x 5 odd days in 400
years. But, 400^’’ year is a leap year. Consequently, there must be 4 x 5 +1 = 21 odd days in 400
years. When 21 is divided by 7, the remainder is zero. So, the number of odd days in 400 years is
zero i.e. there is no odd day in 400 years.
RENi.MtK The number of odd days in each one ofSOQ years, 1200 years, 2000 years, etc. is zero.
The results obtained in the above discussion are put in the following tabular form:

Number of years: 100 200 300 400

ow
Number of odd days: 5 3 1 0

ILLUSTRATIVE EXAMPLES

iypr I ON FINDING THE NUMBER OF ODD DAYS IN A CERTAIN NUMBEROF YEARS

e
re
lAAMI’LE Find the number of odd days in 1985 years.

Frl
SOLUTION Since there are no odd days in 400 years. So, we express 1985 in terms of multiples of

F
400 as given below.
1985 = 400 X 4 + 385
ou
or
Number of odd days in 1985 years = Number of odd days in 385 years.
kfs
Now, 385 = 300 + 85

Number of odd days in 385 years = Number of odd days in 300 years + Number of
oo

odd days in 85 years


Y

= 1 + Number of odd days in 85 years ...(h)


eB

Let us now find the number of odd days in 85 years.


85 years = 21 leap years + 64 ordinary years
ur
oY

A leap year has 2 odd days and an ordinary year has one odd day.
ad

Number of odd days in 21 leap years and 64 ordinary years =21x2+64x1= 106
Number of odd days in 85 years = 106 ...(hi)
d

From (ii) and (hi), we find that


in
Re

Number of odd days in 385 years = 1 + 106 = 107


F

Dividing 107 by 7, we obtain 2 as the remainder


Thus, number of odd das in 385 years = 2 ...(iv)
Hence, number of odd days in 1985 years = 2 [From (i) and (iv)]
\i.rr.lilL 1985 = 400 x 4 + 300 + 21 (leap year) + 64 (ordinary year)
Number of odd days in 1985 = (0 +1 + 21 2 + 64 X 1) odd days
= 107 odd days
= 15 weeks + 2 days = 2 odd days
EXAMl’l.t 2 How many odd days are in 2119 years?
SOLUTION We find that 2119 =400 x 5 + 119

Number of odd days in 2119 years = Number of odd days in 119 years.
Now, 119=100x1 + 19 ...(i)
5.12 APPLIED MATHEMATICS-XI

Number of odd days in 119 years = Number of odd days in 100 years + Number of
odd days in 19 years
= 5 + Number of odd days in 19 years ...(ii)
Let us now find the number of odd days in 19 years.
19 years = 4 leap years + 15 ordinary years
Number of odd days in 4 leap years and 15 ordinary years = 4x2 +15x1= 23
Number of odd days in 19 years = 23 ...(hi)
From (i), (ii) and (iii), we obtain
Number of odd days in 2119 years = (5 + 23) odd days
= 28 odd days
= 4 weeks + 0 day s 0 odd day.
Hence, 2119 years do not have any odd day.

w
ALITER 2119 = 400 X 5 + 100 + 4 (leap year) + 15 (ordinary year)

F lo
Number of odd days in 2119 years =(0 + 5 + 4x 2 + 15 x 1) odd days
= 28 odd days
= (4 weeks + 0 odd day) s 0 odd day

e
Fre
Type n ON FINDING THE WEEK DAY ON A CERTAIN DATE for
EXAMPLE 3 What was the day of the week on 25^'’ Dec, 1995?
OR
r
You

On what day was the Christmas in 1995?


oks

SOLUTION We find that


eBo

Period up to 25**^ Dec 1995 = (1994 years + Period from 1 Jan, 1995 to 25*^^ Dec, 1995)
Now, 1994 = 400 x 4 + 300 + 23 (leap year) + 71 (ordinary year)
ad
our

Number of odd days in 1994 years =(0 + 1 + 23 x 2 + 71 x 1) odd days


= 118 odd days
= (16 weeks + 6 odd days)
dY
Re

= 6 odd days
Fin

Let us now find the number of odd days in the period from 1 Jan, 1995 to 25, Dec 1995.
Jan Feb March April May June July Aug Sept Oct Nov Dec
(31 +28 +31 +30 +31 +30 +31 +31 +30 +31 +30 +25)
= 359 days
359 days = ( 51 weeks + 2 days) = 2 odd days.
Total number of odd days till 25 Dec., 1995
= Number of odd days in 1994 years + Number of odd days from 1 Jan 1995 to
Ih
25“* Dec, 1995

= (6 + 2) odd days = 8 odd days = (one week + one day) = 1 odd day.
From the odd day table, we find that first odd day is Monday.
Hence, it was Monday on 25^^ Dec., 1995.
QUANTIFICATION AND NUMERICAL APPLICATIONS 5.13

th
EXAMPLE 4 What was the day of the zveekon 15 August, 1947?
SOLUTION We find that

Period upto 15*^*^ Aug, 1947 = (1946 years + Period from 1 Jan, 1947 to 15^^ Aug, 1947)
Now, 1946 = 400 x 4 + 300 +11 (leap years) + 35 (ordinary years)
Number of odd days in 1946 years = (0 + 1 + 11 x 2 + 35 x 1) odd days
= 58 odd days
= (8 weeks + 2 days) = 2 odd days
Let us now find the number of odd days in the period from 1 Jan, 1947 to 15^*^ Aug, 1947.
Jan Feb March April May June July Aug
(31 +28 +31 +30 +31 +30 +31 +15) =227 days
227 days = (32 weeks + 3 days) = 3 odd days

w
Total number of odd days till 15 Aug, 1947
= Number of odd days in 1946 years + Number of odd days from 1 Jan, 1947 to

F lo
15 Aug, 1947
= (2 + 3) odd days = 5 odd days
From the odd day table, we find that 5^*^ odd day is Friday.

ee
Fr
Hence, it was Friday on 15 Aug, 1947.
EXAMPLE 5 What ivas the day of the week on 26 Jan 1950? for
SOLUTION We observe that
ur
Period upto 26 Jan, 1950 = (1949 years + Period from Jan 1,1950 to 26 Jan, 1950)
s

Now, 1949 = 400 x 4 + 300 + 12 (leap years) + 37 (ordinary years)


ook
Yo

Number of odd days in 1949 years = (0 + 1 + 12 x 2 + 37 x 1) odd days


eB

= 62 odd days
= (8 weeks + 6 day) = 6 odd days
our

Let us now find the number of odd days in the period from Jan 1,1950 to 26 Jan, 1950. There are
ad

26 days from Jan 1,1950 to 26 Jan, 1950.


26 days = (3 weeks + 5 days) = 5 odd days
Y

Total number of odd days till 26 Jan, 1950 = Number of odd days in 1949 years
Re
nd

+ Number of odd days from Jan 1,1950


Fi

to 26 Jan.1950

= (6 + 5) odd days
= 11 odd days
= (one week + 4 days)
= 4 odd days
From the odd table, we find that fourth odd day is Thursday.
EXAMPLE 6 On zohat dates of April 2015 did Friday fall?
SOLUTION Let us first find the day on 1 April, 2015.
Period upto 1 April, 2015 = (2014 years + Period from 1 Jan 2015 to 1 April, 2015)
5.14 APPLIED MATHEMATICS-XI

Now, 2014 years = 400 x 5 + 3 (leap years) + 11 (ordinary years)


Number of odd days in 2014 years = (0+3x2 + 11x1) odd days =17 odd days
= (2 weeks + 3 days) = 3 odd days
Let us now find the number of odd days in the period from 1 Jan 2015 to 1 April, 2015.
Jan Feb March + April
31 + 28 + 31 + 1 = 91 days
91 days = (13 weeks + 0 day) = 0 odd day
Total number of odd days till 1 April, 2015 = (3 + 0) = 3

ow
From the odd day table, third odd day is Wednesday. So, on 1 April, 2015 there was Wednesday.
Consequently, first Friday fell on 3 April, 2015 and subsequent Friday were on 10^^, 17* and
24* of April 2015.

e
:< III ON FINDING THE DAY OFTHE WEEK ON A CERTAIN DATE WHEN THE DAY OFTHE WEEK ON

re
Fl
ANOTHER DATE IS GIVEN

F
"XAMP' - ● If 4^^ April, 1998 was Mouda}/, what zvas 5^^’ November, 1989?
ur
SOLUTION Let us first find the number of odd days from 4* April 1988 to 4* April 1989.1988 is a

r
leap year but we are counting from 4* April. So, from 5* April, 1988 to 4* April 1989 may be
fo
considered an ordinary year having one odd day.
ks
Yo
Now, we count the number of odd days from 4* April, 1989 to 5* Nov., 1989.
oo

April May June July August Sept. October November


eB

26 + 31 +30 +31 +31 + 30 +31 + 05 = 215 days


215 days = 30 (weeks + 5 days) = 5 odd days.
ur

So, there are 6 odd days from 4* April, 1988 to 5* Nov., 1989.
ad
Yo

It is given that 4* April, 1988 is a Monday. Adding 6 odd day to Monday, we find, from the odd
day table, that there is Sunday. Hence, there was Sunday on 5* November, 1989.
d
Re

!;XAMPLE 9 Ifl0‘^‘ May, 1997 was a Monday, zvhat was the day on October 10, 2001?
in
F

SOLUTION Let us first find the number of odd days upto 10 May, 2001 to

11 May, 1997 -10 May, 1998 Ordinary year 1 odd day


11 May, 1998-10 May, 1999 Ordinary year 1 odd day
11 May, 1999 - 10 May, 2000 Leap year 2 odd days
11 May, 2000 - 10 May, 2001 J Ordinary year 1 odd day
Total
5 odd days

Thus, the total number of odd days upto 10 May, 2001 = 5


Now, we find the number of odd days from 11 May, 2001 to 10 October, 2001.
May June July Aug Sept October

21 + 30 + 31 + 31 + 30 + 10 = 153 days
QUANTIFICATION AND NUMERICAL APPLICATIONS 5.15

153 days = (21 weeks + 6 days) = 6 odd days.


Thus, total number of odd days upto 10 October, 2001 = (5 + 6) = 11=4 odd days.
Since, 10 May 1997 was a Monday and then 4 days after Monday would be Friday. Hence, there
was Friday on 10 Oct, 2001.
. . I' ON MATCHING OF CALENDERS

i;x.-\MPLL 10 Prove that the calender for the year 2003 will same for the year 2014?
SOLUTION In order to prove that years 2003 and 2014 have the same calendar, it is sufficient to
show that there is same week day on 1 Jan, 2003 and 1 Jan, 2014. For this, we will have to show
that there is no odd day between 31.12.2002 and 31.12.2013. This period has 3 leap years (2004,
2008 and 2012) and 8 ordinary years.
.'. Number of odd days between 31.12.2002 and 31.12.2013 = 3x2 + 8x1=14 odd days
= (2 weeks + 0 day) s 0 odd day

w
Hence, calendar for the year 2003 will serve for the year 2014.

F lo
EXERCISE 5.3
1.
Find the number of odd days in:

ee
(i) 1874 years (ii) 2013 years (in) 1999 years

Fr
2.
What was the day of the week on 17'^ June, 1998?
3. What was the day of the week on 28‘^ May 2006?
for
4.
On what dates of March, 2005 did Friday fall?
r
You

5.
What was the day of the week on 16'^ July, 1776?
s
ook

6.
What was the day of the week on 16'^ June, 1999?
eB

/.
If 8 Feb, 2005 was a Tuesday, what was the day of the week on 8 Feb, 2004?
8.
If 11 April, 1911 was a Tuesday, what was the day on 17 September, 1915?
our
ad

Prove that the calender for the year 2009 will serve for the year 2015.
ANSWERS
dY

: - (i) 6 (ii) 2 (hi) 3 2. Wednesday 3. Sunday


Re

4. 4Ml‘M8'^and25'h 5. Tuesday 6. Wednesday


Fin

7. Sunday 8. Friday
5.4 CLOCKS

As we all are aware the face of a clock or a watch is a circular disc whose circumference is

divided precisely into 60 equal parts. Each part is marked as a minute. The face is often called
dial. The area of the region enclosed by the radii joining centre to the two consecutive minute
markings is called the minute space. So, the face of the clock is partitioned into 60 minute spaces.
As 360° angles is divided into 60 equal parts. So, the angle between two consecutive minute
markings is 6°.
A clock has two hands, the longer one is called the minute hand and the shorter hand is called
the hour hand. The minute hand makes one complete round in 60 minutes. So, the measurement
of angle traced by the minute hand in 60 minutes is 360° and the rate of rotation is 6° per minute.
The hour hand makes one complete round in 12 hours i.e. 720 minutes. So, the measure of the
5.16 APPLIED MATHEMATICS-XI

angle traced by hour hand in one minutes is ^360^ i.e. the rate of rotation of the hour
I720J I2.
hand is ^ per minute. When two hands are at right angle (at 3 :15, 9 : 0 hrs etc), they are 15
minute spaces apart. When two hands are in opposite direction, they are 30 minute spaces apart.

In a perfect clock, the minute hand makes one round in 60 minutes whereas the hour hand
rotates through 5 minute spaces only. This means that the minute hand gains 55 minute spaces
over the hour hand in 60 minutes. In every hour both hands coincide once. So, for both the hands
to be together i.e. to coincide, the minute hand must gain 60 minutes over the hour hand.
Now,
55 minute spaces are gained by minute hand over the hour hand in 60 minutes
60 minute spaces will be gained by minute hand over the hour hand in

w
— X 60 nun =65— min
1,55 11

F lo
Thus, the two hands of a clock meet together i.e. they

If both the hands meet together after an interval less


coincide after 65^ minutes.
than 65^ minutes, the clock is running fast
for F
ree
and if they meet after an interval greater than 65^ min., the clock is running slow.
If both hands of a clock coincide at an interval of x minutes and a: < 65— then
11'
Your

5 ^
ks

65—-X
eBoo

11
Total time gained = minutes
ad

and the clock is said to be 'fast'.


our

5
If both hands coincide at an interval of x minutes and a: > 65—, then
11

.-65^1
Re
Y

11
Total time lost = minutes
Find

and the clock is said to be 'slow'.

If the hands of a clock interchange positions i.e. the minute hand takes the place of hour hand
and the hour hand takes the place of minute hand, the sum of the angles traced by the two hands
is 360°.

Suppose it happens after x minutes. Then minute hand rotates through 6° per minute whereas
the hour hand through — per minute. Therefore,

Angle traced by minute hand in x minutes = (6x)°


fl V
Angle traced by hour hand in x minutes = — x
V.2 ,
QUANTIFICATION AND NUMERICAL APPLICATIONS 5.17

1 13 720
6x + .r = 360 => X = 360 ^ A" = = 55-
2 2 13 13

Thus, the two hands of a clock interchange positions after ever 55 minutes.
13

5.4.1 SOME IMPORTANT POINTS TO REMEMBER

(i) The minutes hand rotates through 6® per minute i.e. in one minute, the minute hand
moves 6°.

(ii) The hour hand rotates through 30° in one hour i.e. the hour hand moves through 30° per
1 °
hour or - per minutes.

w
(iii) The minutes hand rotates through 6° in one minute whereas the hour hand turns through
1 1 °
- in one minute. Therefore, in one minute the minute hand gains 5— more than the

Flo

hour hand. Consequently, the minute hand gains5 — X 60 = 330° over hour hand in one

e
hour. In other words, the minute hand gains 55 minute divisions over the hour hand in

re
one hour.

F
(iv) In every hour, both the hand coincide once.
ur
(V) In a day i.e. in 24 hours the hands coincide 22 times.
(vi) In every 12 hours, the hands of clock coincide 11 times.
(vii) In every 12 hours, the hands of clock are in opposite direction 11 times.
f or
ks
Yo
(viii) In a day, the two hands of a clock are at right angles 44 times.
oo
B

ILLUSTRATIVE EXAMPLES
re

EXAMPLE I The mimite hand of a clock overtakes the hour hand at intervals of 65 minutes of the correct
time. How much a day docs the clock gain or lose?
u
ad
Yo

SOLUTION In a perfect or correct clock the two hands coincide after 65 minutes i.e. 60 minutes
11

spaces are gained by the minute hand in 65;^ minutes. It is given that the two hands of a clock
d

11
Re
in

are together in 65 minutes.


F

5 A 5
Gain in 65 minutes = 65—-65 min =- min
11 11

5
Gain in 1 minute = mm
11 X 65

5
Gain in 24 hours i.e. 24 x 60 minutes = X 24 X 60 min =10—min
11 X 65 43

10
Thus, the clock gains 10 minutes in a day.
43

EXAMI’LE 2
How much does a clock gain per day if it hands coincide every 64 mmutes?
SOLUTION In a perfect clock the two hands coincide after every 65^11 minutes. Here, it is given

that the hands of clock coincide every 64 minutes.


5.18 APPLIED MATHEMATICS-XI

Gain in 64 minutes = 65 — - ...


64
● 16
mm - — min
.
11 11

16
Gain in 1 minute = min
11 X 64

16
Gain in a day i.e. 24 x 60 minutes = X 24 X 60 minutes = 32—minutes.
11 X 64 11

i.XAMPLI; 3 At wliat time between 2 and 3 O'clock will the hands of a clock coincide?
SOLUTION At 2 O'clock, the hour hand is at 2 and the minute hand is at 12, i.e. they are 10 minute
spaces apart. To be coincident, the minute hand must gain 10 minute spaces.
We know that

55 minute spaces are gained by minute hand over the hour hand in 60 minutes

w
^60 10
10 minute spaces will be gained in X 10 minutes =10 minutes
55 11

F lo
10
Hence, the two hands will coincide in 10 minutes.
11

ee
EX.AMPLI; ; At wliat time between 4 and 5 O'clock will the hands of a clock be at right angle?

Fr
SOLUTION At 4 O' clock, the minute hand is 20 minutes spaces behind the our hand. For two
hands to be at right angle, they must be 15 minute space apart. So, the two hands will be at right
for
angles in the following cases:
ur
! j When minute hand is 15 minute spaces behind the hour hand
s

In this case, the minute hand will have to gain (20 -15) = 5 minute spaces. We know that
ook
Yo

55 minute spaces are gained by the minute hand in 60 minutes.


eB

60 60
5 minute spaces will be gained by the minute hand in X 5 =5 = minutes
55 11 11
our
ad

Thus, the two hands will be at right angles at 5 —


11
minutes past 4 O' clock.

■ \s(- I! When the minute hand in 15 tninutes spaces ahead of hour hand.
Y

In this case, the minute hand will have to gain (20 + 15) = 35 minutes spaces.
Re
nd

Now,
Fi

55 minute spaces are gained by minute hand in 60 minutes.


60 420 2
35 minute spaces will be gained in — x 35 minutes = minutes = 38 — minutes.
{55 11 11

2
Hence, the two hands will be at right angles at 38—
11
minutes past 4 O' clock.

re-:m,-\RK In 11 to 12, or 12 to l,orl to 2 minute hand and hour hand of a clock are at right angels tiuice.
Once when the minute hand is on the left hand side of hour hand and once on its right side.
' \ \ MPLE s Hozv many times are the hour and minute hands of a clock are at right angles during their
motion from 1 : 00 PM to 10 : 00 PM?
SOLUTION Between 1:00 PM and 10:00 PM there are 9 hours. During each of these 9 hours, the
hands of a clock are at right angles twice. Hence, required number = 2 x 9 = 18.
EXAMPLE <’ At what time between 9 and 10 O'clock will the hands of a clock be together?
QUANTIFICATION AND NUMERICAL APPLICATIONS 5.19

SOLUTION For the two hands to be together between 9 and 10 O'clock, the minute hand has to
gain 45 minute spaces.
We know that

55 minute spaces are gained minute hand in 60 minutes.

45 minute spaces will be gained in — x 45 = 49— minutes


^55 11

1
Hence, the two hands are together at 49 11 minutes past 9 O' clock.

EXAMl’i i ■ A clock is 1 minute sloxv at 1 PM on Tuesday and 2 minutes fast at 1 PM on Thursday.


When did it shozv the correct time?

w
SOLUTION The clock will show correct time when it gains 1 minute.
Time from 1 PM on Tuesday to 1 PM on Thursday = 48 hours

Flo
It is given that the clock gains (1 + 2) = 3 minutes in 48 hours.
[48
It gains 1 minute in — = 16 hours.

ee
V 3

Fr
Thus, the clock shows correct time 16 hours after 1 PM on Tuesday i.e. at 5 AM on Wednesday.
EXAMPLE 8 A clock is set right 10 AM. The clock gams 10 minutes in 24 hours. What will be the true
time zohen the clock shows 3 PM on the following day? for
ur
SOLUTION Time from 10 AM on a day to 3 PM on the following day = 29 hours.
s
It is given that the clock gains 10 minutes in 24 hours. So,
k
Yo
oo

24 hours 10 minutes of the given clock = 24 hours of the correct clock


145
eB

i.e.
— hours of the given clock = 24 hour of the correct clock.

— X 29 1 hours of the correct clock


r

29 hours of the given clock = 24 x 145


ou
ad

= 28 hours 48 minutes of the correct clock


Y

So, the correct time is 28 hours 48 minutes after 10 AM i.e. 48 minutes past 2 PM.
Re
nd

EXERCISES 5.4
Fi

1.
Find the angle between the two hands of a clock at
(i) 8 :30 (ii) 4:20 (iii) 5:15
2. At what time between 9 and 10 O'clock are the hands of a clock 23 minute space apart?
At what time between 5 and 6 O'clock are the hands of a clock 3 minutes apart?
4. At what time between 7 and 8 O'clock will the hands of a clock be in the same straight line
but not together?
5. At what time between 8 and 9 O' clock will the hands of a clock be in the same straight line
but not together?
6.
In every 30 minutes the time of a clock increases by 3 minutes. After showing the correct
time at 5 AM; what time will the watch show after 6 hours?
8.
Imagine that your watch was correct at noon, but then it began to lose 30 minutes each
hour. It now shows 4 PM but it stopped 5 hours ago. What is the correct time now?
5.20 APPLIED MATHEMATICS-XI

ANSWERS
1
1. (i) 75° (ii) 10° (iii) 67-
10
2. 9 :24 3. 5 : 24, 5 : 31 — 4. 7:5- 5. 8:10 —
11 11 11
6. 11 :36 AM 7. 11 PM 8. 1 AM

HINTS TO SELECTED PROBLEM

7. Time from 5 AM on a day to 10 PM on the 4th day = 89 hours.


23 hours 44 minutes of this clock = 24 hours of correct clock

ow
356
hrs of this clock = 24 hours of correct clock
15

15
89 hours of this clock = 24 x X 89 hrs of correct clock = 90 hours of correct clock
356

So, the correct time is 11 PM

e
re
5.5 TIME AND WORK

rFl
F
In this section, we will discuss problems on Time and Work. In most of the problems on Time
and Work, either of the following basic variables is to be found:

r
Time: means number of hour or days for completing a job or number of hours or days
ou
actually worked.
fo
ks
Work: means the amount of job assigned or amount of job actually done.
oo

While solving problems on Time and Work, the amount of job assigned or the amount of work is
taken as equal to 1, unless otherwise specified.
Y
eB

Number of days to complete a work x One day's work = 1.


1
One day's work =
ur

Number of days to complete the work


ad

and.
Yo

1
Number of days to complete a work =
d

One day's work


Re
in

1
Thus, if A can finish a piece of work in days, then A’s 1 day's work =
F

"A

or. If A's one day's work = —, then A can finish the work in days.

5.5.1 SOME USEFUL RESULTS

RESULT-1 A and B can finish a piece of work in n/^ and ng days respectively. If A and B together ifnish
the work in days, then
— I ^ ^
><AB
i.e.
A’s one day's work -i- B's one day's work = (A and B togetherj's 1 day's work
l4tOOF A can finish the work in n^ days.

A’s one day's work = — => A's, days work =


"A
QUANTIFICATION AND NUMERICAL APPLICATIONS 5.21

5
B can finish the work in rig days
1 "AB
B's one day's work = — ^ B's, days work =

Since A and B together can finish the work in n^g days. Therefore,
A's n^g days work + B's days work = 1
"AB , ”/\B = 1 => 1 +
1 _ 1

1 1 1 1
REMARKl J_ +J_-J_ and —
’U >^AB ”>IB ”B "AB "a

remark: 1^11 ”A6 -


"a_!!b
"6 ”AB riA + Hg

w
RESULT-2 A, B and C can finish a piece of work alone in ^ iig and iiq days respectively. If A, B and C

F lo
together finish the work in n^g^ days, then
1 1 1 1
-f- h

"A ”B "ABC

e
Fre
i.e.A's 1 day's work -i- B's 1 day's work + C's 1 days's work = {A, B and C together)'s 1 day's
for work.

RESULT-3 A and B working togetherfinish a pieces of work in days. If A working alone takes 'a' days
more than A and B together and B working alone takes 'b' days more than A and B together. Then,
r
You

"AB =
oks

PROOF A alone can finish the work in (;i^g + a) days.


eBo

A's one day work = —^—


"ab + "
our
ad

B alone can finish the work in + b) days.

B's one day work = ——


llAB + ^
dY
Re

1 1
{A + By s one day work =
Fin

iiy^g + a -I- b
1
(A + B)' s n^g days work = +

"AB+^

But, A and B together can finish the work in n^g days.


1 1
"AB + =1
"ab ^ "AB ^
^Mb ("AB "AB + = 1
("AB + (”/\B +

iiAg {2n^g + a + b)= {n^g + a) {n^g + b)


2 (”ab)^ + "ab ^’) ~("ab)^ + "AB + ^) +
5.22 APPLIED MATHEMATICS-XI

REMARK 1 If a person A is k times efficient than another person B, then


Ratio ofzvork done by A and B = k: 1
Ratio of times taken by A and B to finish the work = 1: k
REMARK If a person A completes - part of a work x days, then Afinishes the entire work in days.
b alb

ILLUSTRATIVE EXAMPLES

EXAMPLE ! If A can complete a piece of work in 8 days and B completes the same piece of work in 6 days,
in how many days will both A and B together complete it?

w
SOLUTION We have, n^ = 8 and iig = 6
1 1 1 1 1 1

F lo
1 7 24
= — + — => — =- + —=>
-^ 'Ub - 3^
"AB 'M 'Ub 8 6 24 7 7

3
Hence, A and B together can complete the piece of work in 3— days.

e
ALITHR A's one days work = —, B's one days's work = —

Fre
8 6
for
1 1 7
(A + B)'s one day's work = —+ — = —
8 6 24
r
You

24 3
oks

A and B together can complete the work in — days i.e. 3— days.


eBo

EXAMPLE 2 A and B together can complete a piece of work in 15 days and B alone in 20 days. In how many
days can A alone complete the work?
ad
our

SOLUTION We have, n^g=15 and iig= 20


1 _ 1 1 1 1 1 1 1 ^ = 60.
'Us “a ”s 'U 'Us ”e 'U 60
Re
dY

Hence, A alone can complete the piece of work in 60 days.


Fin

1
ALll ER (A + B)'s one day's work = B's one days's work = 20
1 1
A's one day's work = —-
15 20 60

Hence, A alone can complete the work in 60 days.


EXAMPLE 3 A is twice as good a workman as B. If they zuork together, they can complete a job in 18 days.
If A alone does the job, in how many days he will complete the job?
SOLUTION Suppose A and B separately can complete the job in n^ and n^ days respectively. It is
given that A is twice as good a workman as B. Therefore, n^ = 2ji^ . It is given that A and B
together can complete the job in 18 days. i.e. ii^g =18.
11^1
QUANTIFICATION AND NUMERICAL APPLICATIONS 5.23

J__J_ + ^ 1
18 /le
J_
18 2n^
3 1

In
= 77:=^ =27
.18

Hence, A along can complete the job in 27 days.


EXAMPLE 4 A is thrice as good a ivorkman ns B and so takes 60 days less than Bfor doing a job. In how
many days A and B together will complete the sa?ne job?

ow
SOLUTION Let A and B separately complete the job in n^ and days respectively. It is given
that A is thrice as good a workman as B. This means A takes one third of the time taken by B to
complete the job.

e
= 3 ...(i)

re
It is also given that A takes 60 days less than B to complete the job.
rFl
F
»a=»B-60. -(ii)

r
Solving (i) and (ii), we obtain
ou
n^ = 30 and Hg =90
fo
ks
Suppose A and B together can complete the job in «^g days. Then,
oo

90
1 11 1 i_ J_ 1 ^ = 22^
30 ^ 90 ^ 90 ^ 4
+
2
Y

”AB 'M "B ”A6


eB

Hence, A and B together can complete the job in 22-^ days.


r

1 2
ou

EXAMPI.E 5 A can complete work in 5 days and B can complete — of the work in 10 days. In how
ad
Y

many days both A and B together can complete the ivork?


d

SOLUTION A can complete — of work in 5 days


Re
in

A can complete the work in ^ = 15 days.


F

1/3

2
B can complete “ of a work in 10 days
10
B can complete the work in = 25 days.
2/5

Thus, we obtain = 15 and ?ig = 25.


Suppose A and B together can complete the work in n^g days. Then,
1
— =— + —
1 1
^ J_=J_ —= — ”AB =
75
9^
'Ub "a ”AB " 15 25 ” 75 8 8

3
Hence, A and B together can complete the work in 9—8 days.
5.24 APPLIED MATHEMATICS-XI

fvXAMPLE (■ A and B together can complete a piece of work in n days. If A alone completes the work in
(n + 3) days ad B alone in (n + 12) days, find the value ofn.
SOLUTION We have, =n+ 3 and = n + 12

1 ^ 1 ^ 1
1 1 1
+
n n + 3 n + 12

1 In +15
r?" +15« + 36 = Ir"P + 15« ^ = 36 => « = 6
n n^ + 15« + 36
EXAMPLE 7 A alone can complete a piece of work worth f 300 in 6 days; but buy engaging an assistant B,
the work is completed in 4 days. Find the share received by the assistant.

w
SOLUTION We have, n^ =6 and n^g = 4
1 1 1 1 1 1 1 1 1 1

F lo
— = — -) => — = —I ^ — = =— «d=12
«AB «B 4 6 «B 4 6 12 ^
So, assistant B alone can complete the work in 12 days.

ee
A’s share: Assistant B's share = A's 1 day's work : B's 1 day's work

Fr
1
'^A =12:6 = 2:1
for
”B

1
ur
Hence, Assistant B's share = ? X 300 =?100.
2 + 1
s
ook
Yo

EXAMPLE A and B together can do a piece of work in 12 days; B and C together can do the same piece of
work in 15 days; C and A together can do it in 20 days. In how many days the same piece of work will be
eB

finished, if
(i) A, B and C zvork together? (ii) A, B and C each works along?
our
ad

SOLUTION Suppose A, B and C alone can complete the piece of work in n^, ng and n^ days
respectively. Further, let A, B and C together can complete the same piece of work in days.
(i) It is given that:
dY
Re

A and B together can do a piece of work in 12 days.


Fin

J_ + J_- 1 ...(i)
”B 12

B and C together can do the same piece of work in 15 days.


J- + i-=J- ● ●●11
«s ^ 15

C and A together can do the same piece of work in 20 days.


1 1 1
— + —
20
...(iii)
«C «/l

Adding (i), (ii) and (iii), we obtain


Ill 1 1
H — — + — H
't J 12 15 20
QUANTIFICATION AND NUMERICAL APPLICATIONS 5.25

2U 1
+ — + —
1 5 + 4+3
60
't

1 1 1 1
+ + ...(iv)
^ 10

1 1
= TT => ”ABC =10
”abc 10

Hence, A, B and C together can do the same piece of work in 10 days.


(ii) Subtracting (i), (ii) and (iii) successively from (iv), we obtain

w
1
±andi —=— => ih- =60, Ha =30andng =20
^t~10 12~60'«^~10 15 30 «B 10 20 20 ^ ^

Hence, A, B and C alone can complete the same piece of work in 30 days, 20 days and 60 days

o
respectively.

e
hXAMPLE 9 A and B can do a work in 8 days; B and C can do the same work in 12 days; A, B and C

re
rFl
together can finish it in 6 days. In how many days A and C together will do the same work?

F
SOLUTION Suppose A, B and C alone can do the work in n/^, Hg and n^ days respectively.
A and B together can do the work in 8 days.

r
ou
-L +J_=l fo
ks
”B

B and C together can do the work in 12 days.


oo

1 1 1
Y

...(ii)
eB

:■ — + —
”B ”C 12

A, B and C together can do the work in 6 days.


r
ou

1 J__l ...(hi)
Y
ad

«A "B ^ 6

Subtracting (i) and (ii) successively from (iii), we get


d

11 ^1 11
Re
in

nc 6 8 6 12
F

— =— and —
1
1 1 1_ J_^l
24 "A 12 ”C 12 24 ”8

Hence, A and C together can do the work in 8 days.


EXAMPLE 10 A and B together can do a piece of ivork in 12 days; B and C together can do it in 15 days. If A
is twice as good a worbnan as C,find in what time B alone can do it?
SOLUTION Suppose A, B and C separately can complete the given pieces of work in , ng and
days respectively.
A and B together can do the working 12 days.
— + — =-

«A "B 12
5.26 APPLIED MATHEMATICS-X!

B and C together can do the working 15 days.


— + — =1. ...(h)
15

It is given that A is twice as good a work man as C.


"C = 2
Putting iiQ =2 in (ii), we obtain
1
+
...(hi)
"e "l5

Subtracting (iii) from (i), we obtain


1 1 1 1 1
= — => = 30.
2n^ 12 15 2n^ 60
Putting = 30 in (i), we obtain

w
1 1 1 1
— +
— = —=j>«g=20.
30 12 ng " 12 ^ "B 20 ^
Hence, B alone can complete the work in 20 days.

F lo
EX.^MPLE 11 Rohit takes twice as much time as Mohit and thrice as much time as Sonit to complete a

ee
piece of work. If working together, they can complete the work in 4 days, find the time taken by each of them

Fr
separately to complete the zuork. for
SOLUTION Suppose Rohit can complete the given piece of work in n days. Then, Mohit and
ur
Sonit alone take — and — days respectively to complete the same piece of work. It is given that all
s

three working together can complete the work in 4 days.


ook
Yo

1 1 1 1 1 1 2 3
-=-^ n = 24
eB

4 n n n
4 n n n 4 n
2 3
our

Hence, Rohit, Mohit and Sonit alone can complete the work in 24 days, 12 days and 8 days
ad

respectively.
EXAMPLE 12 A can build a wall in the same time in which B and C together can do it. If A and B together
Y

could do it in 25 days and C alone in 35 days, in zuhat time could B alone do it?
Re
nd

SOLUTION Suppose A, B and C alone can build the wall in n^, and iiq days respectively. It is
Fi

given that A can build the wall in the same time in which B and C together can do it. Therefore,
A s one day's work = B's one day's work + C's one day's work.

...(i)

A and B together could make the wall in 25 days.

25 ...(h)
"A "B

It is also given that = 35.


Putting = 35 in (i), we obtain
I
...(hi)
iiA ug 35
QUANTIFICATION AND NUMERICAL APPLICATIONS 5.27

Subtracting (iii) from (ii), we obtain


2 1 1,2 2 =5> =175
/jg 25 35 fig 175
Hence, B alone could do it in 175 days.
EXAMPLE 13 A man nnd a boy can do a piece of work m 24 days. If the wan works alone for the last 6 days,
it is completed in 26 days. How long would the boy take to do it alone?
SOLUTION Suppose the man, the boy and both them together can do the work in
”M' ”M6 respectively.
It is given that n^g = 24 days. It is also given that
{M and boy)'s 20 days work + Man's 6 days work = 1
20 20 20^ 6 1
+ i-=l^ = 36

w
24 24
«M
1 1 1
Now,

F lo
"MB "M "B
1 1 1 1 1 1 1
— =-— + — ^ — = "B

ee
=e>
24 36 »g Hg 24 36 72

Fr
Hence, the boy alone can do the w'ork in 72 days.
EXAMPLE 14 A wan, a zooman and a boy can do a piece of work in 6, 9 and 18 days respectively. Hoio for
many boys must assist one man and one woman to do the work in 1 day?
ur
SOLUTION Suppose x boys assist one man and one woman to complete the work in 1 day. Then,
s
1 day's work of one man, one women and x boys, is
ook
Yo

1 1 .V
— + — + —
6 9 18
eB

1 1
1 h ^=1 Work is completed in 1 day]
6 9 18
r

13
^=1 _1_2 .V
ou
ad

= —=> x=13.
18 6 9 18 18

Hence, 13 boys must assist one man and one women to complete the work in 1 day.
Y

EXAMPLE 15 A child can do a pieces ofioork 15 hours slower than a ivoman. The child works for 18 hours
Re
nd

on the job and then the woman takes chargefor 6 hours. In this manner, - of the ivork can be completed. To
Fi

complete the job hoiv much time will the zvoman take?
SOLUTION Suppose the woman takes x hours to complete the job. Then, the child takes (x + 15)
hours to complete the job.
Woman's 1 hour's work = -, Child's 1 hour's work = —:
X X +15

it is given that
3
Child's 18 hour's work + Woman's 6 hour's work = —
0

18 I 6_3
X + 15 X 5

18x+6x + 90 3

x(x + 15) 5
5.28 APPLIED MATHEMATICS-XI

5 (8a: + 30) = a: (a: + 15)


-25a--150=0 (a-30){a+5)=0=> a = 30 [va + 5^ O]
3 2
Remaining work = 1 - — = -

The woman takes 30 hours to complete the work.


Time taken by the woman to complete the remaining work = — x 30 hours = 12 hours

Hence, the woman takes 12 hours to complete the job.


EXAMPLE 16 A and B can complete a piece of work in 12 and 18 days respectively. A begins to do the work

ow
and they zoork alternatively one at a timefor one day each. In hozv many days will the zuorkhe completed?
SOLUTION We have,

= Number of days taken by A to complete the work = 12


tig = Number of days taken by B to complete the work =18

e
re
Suppose A and B together can complete the work in days. Then,

rFl
F
J-=J- ■ ^ 1
s=
1
1
1
^
1 5 1

^AB 12 18 ^AB

or
ou
Thus, when each one of A and B works alternatively for 7 days they complete
— + 7- = L
ksf
part of the work.
ng “ 12 18 “ 36
oo

35
The remaining part i.e. 1 = -^ of the work is to be completed by A as A begins to do the
Y

26
B

work.
re

Number of days take by A to complete — part of the work = f — x 12 1

36^ [36 J 3
oYu
ad

Thus, A works for 7 — days and B for 7 days to complete the work. Consequently, the work ISi
1^
= 14^ days.
d

completed in 7 + 7 —
I 3
in
Re

EXAMPLE 17 A and B can do a piece of work in 45 and 40 days respectively. They began the work together
F

but A leaves after some days and B finished the remaining part of the zvork in 23 daijs. After hozv many
days did A leaves?
SOLUTION Suppose A left the work after x days. This means A and B worked together for .r days
to complete a certain part of the work.
1 ) 17a
Part of the work completed by A and B in a days = a -^ + —
U5 40 360

B completes the remaining part of the work in 23 days.


23
Remaining work = work done by B in 23 days = —
40
17a 23 17a 17
Clearly + = 1=> =— a =9
' 360 40 360 40

Hence, A left the work after 9 days.


QUANTIFICATION AND NUMERICAL APPLICATIONS 5.29

EXAMPLE 18 A can complete a work in 10 days, Bin 12 days and C in 15 days. All of them began the work
together, but A had to leave the work after 2 days of the start and C, 3 days before the completion of the
work. How long did the work last?
SOLUTION We have, =10, «g=12 and n^ =15.
A left the work after 2 days. So, A, B and C together worked for 2 days to complete
1 1 1 1
+ — + — + i
— part of the work.
10 12 15

It is given that C left 3 days before the completion


of work. C's three day's work “ ^ ^ ^

ow
Had C not left 3 days before the completion of work, he (she) would have done ^
5
part of work.
n
So, the remaining work 1 - ~ ~ = — is done by B and C.
-
SJ 10 ^

e
B and C's 1 day's work = 12

re
15 ” 20
3

rFl
F
Now , — work is done by B and C together in 1 day
3 f 20 3 ^
— work is done by B and C together in — — =2 days

or
X
ou
10 ^ I 3 10
ksf
Hence, total time taken = (2 + 3 + 2) days = 7 days.
EXAMPLE 19 A can do a piece of work in 90 days, B in 40 days ayid C in 12 days. They work for a day each
oo

in turn i.e., first day A does it alone, B does it the second day and C the third day. After that A does it for
another day, and so on. After finishing the work they get ^ 2400. If the wages are divided in proportion to
Y
B

the work done by them, find what each will get?


SOLUTION Suppose A, B and C each works for x days to complete the work. Then,
re

1 1 43.r 360 16
= 1 => X = => x = 8
oYu

X +
90 40 12 360 43 43
ad

16
So, A, B and C when work together can finish the work in 8 — days. This means when A, B and C
43
d

work alone one after the other, then each must work for 8 days to complete
in
Re

8^ J_ 1 344 part of the work.


90 40 12 J " 360
F

344 _ 16 _ 4
Remaining work = 1 -
360 " 360 " 90
344
In 8 X 3 = 24 days. A, B and C one after the other complete
360
part of the work. On 25*^ day, it is
A's turn to do the work.

Work done by A on 25*^ day = —


90

Remaining work = -J-l = J-=J_


.90 90j“90"30
On 26* day, it is B's turn.
Work done by B on 26* day = —.
40
5.30 APPLIED MATHEMATICS-XI

Remainine work =— ^
® 30 40 120
On 27^*^ day, its C's turn.
C's 1 day's work = —
12
1
— work is done by C in 1 day
12

1 f 1
work is done by C in 12 x — — day.
10
120 120

ow
Hence, the whole work is completed in 26— days out of which A worked for 9 days, B worked

for 9 days and C worked for 8 — days.


10

Ratio of wages of A, B and C

e
= Ratio of work done by A, B and C

re
1
J_x 9
\ f I W1
—X 9 : —X 8— =-
l^i

rFl
1 9 27

F
— =4:9:27
90 uo 12 10 J 10 40 40
( 4

r
A’s share = ? x 2400 =?240
ou
4 + 9 + 27
fo
ks
9
B's share = ? X 2400 = ? 540
4 + 9 + 27
oo
Y

27
B

C's share = ? x 2400 =?1620


4 + 9 + 27 ^
re

EXAMPLE 20 4 men and W women were put on work. They completed —of the zuork in 4 days. After this 2
3
ou
Y
ad

2
men and 2 women were increased. They completed — more of the ivork in 2 days. If the remaining work is

to be completed in 3 days, then how many more women must be increased?


d

SOLUTION Suppose 1 man's 1 day's work = x and, 1 woman's 1 day is work = y. Then, (4 man
in
Re

and 10 women)'s 4 days work = 4 (4.y + lOy).


F

It is given that 4 men and 10 women completed — of the work in 4 days.


3
1
4 (4.V + lOy) = - => 2.Y + 5y = ...(i)
3 24
2
6 men and 12 women completed — more work in 2 days. Therefore, work done by 6 man and
2
12 women in 2 day's is —.

2 (6a- + 12y) = I ^ 2.V + 4y =


1
i.e. ●●●(ii)
27
1
Subtracting (ii) from (i), we obtain y = —
^ 24 27 216
1 1
Putting y = in (ii),we get x =
216 108
QUANTIFICATION AND NUMERICAL APPLICATIONS 5.31

Thus, a women takes 216 days to complete the work.


Remaining work = 1 -
3 9)~9
Now, suppose k more women are added to complete this work in 3 days.
f \ 4
3 6x + {12 + k) ij =-
. / 9

18.V + 3 (12 + cf) y = ^


18 1 4 1 1
+ 3 (12 + A:) - y =
108 216 9 108 216

ow
i i A-i — =-=^ /c =8.
6 6 72 " 9 72 “ 9 3^ 72 9

Hence, 8 more women must be added.


EXERCISE 5.5

e
1. Aman can complete a piece of work in 16 days. Naman can complete the same piece of

Fl
re
work in 8 days. If both of them work together, in how many days can they complete the
same piece of work?

F
2. A, B and C can complete a piece of work in 24, 5 and 12 days respectively. Working
ur
together, in how many days can they complete the same piece of work?
3.
or
A can knit a pair of socks in 3 days. B can knit the same pair in 9 days. If they are knitting
sf
together, then in how many days will they knit two pairs of socks?
4. A and B together can complete a piece of work in 12 days. If B alone can finish the same
k
Yo

piece of work in 28 days. In how many days can A alone finish the same piece of work?
oo

0.
A and B together can complete a piece of work in 2 days; B and C together can complete the
B

same piece of work in 4 days; A and C together 2— days. In how many days A alone can
e

complete the same piece of work?


ur

6. A and B can do a piece of work in 9 days; B and C can do it in 12 days; A and C can do it in 18
ad

days. In how many days will A, B and C finish it, working together and separately?
Yo

7. A and B together can complete a work in 12 days; B and C together can complete the same
work in 8 days; A and C together can complete it in 16 days. In total how many days do A, B
d

and C together take to complete the same work?


Re
in

8. Two workers A and B working together completed a job in 5 days. If A worked twice as
efficiently as he actually did and B worked — as efficiently as he actually did, the work
F

would have been completed in 3 days. In how many days A alone could complete the
work?
9. A and B can do a piece of work in 12 days; B and C in 8 days; C and A in 6 days. How long
would B take to do the same work alone?
3
10. A and Bean do a piece of work in 7 days. A is 1 — times as efficient as B. In how many days A

alone can do the same piece of work?


7
11. A completes ^ work in 15 days. He completes the remaining work with the help of B in
4 days. Find the time required for A and B together to complete the entire work.
12. If 2 men or 6 women or 4 boys can finish a work in 99 days, how many days will one man,
one woman and one boy together take to finish the same work?
5.32 APPLIED MATHEMATICS-XI

13 8 men can complete a piece of work in 20 days, 8 women can complete the same work in 32
days. In how many days will 5 men and 8 women together complete the same work?
: George takes 8 hours to copy a 50 page manuscript while Sonia can copy the same
manuscript in 6 hours. How many hours would it take them to copy a 100 page manuscript,
if they work together?
Two workers A and B are engaged to do a work. A working alone takes 8 hours more to
complete the job than if both worked together. If B
worked alone, he would need 4 ^hours
more to complete the job than they both working together. What time would they take to do
the work together?
' ‘. A can do a piece of work in 4 hours, B and C together in 3 hours, and A and C together in 2
hours. How long will B alone take to do it?

ow
17. Three friends Anne, Bob and Chris work together to do a certain job. The time it takes them
to do the work together is 6 hour less than Anne would have taken alone, hour less that Bob
would have taken, and half the time Chris would have taken working alone. How long did
it take them to complete the job, working together?

e
18. 18 men can complete a piece of work in 72 days, 12 women can complete the same piece of

Fl
re
work in 162 days and 9 children can complete it in 360 days. In how many days can 4 man
12 women and 10 children together complete the same piece of work?

F
1 A can complete a piece or work in 20 days; B can complete the same work in 30 days and C
ur
can complete it in 60 days. If A is helped on every 3*^ day by B and C, then in how many
days the work is finished? f or
ks
ANSWERS

3. 4^ days
Yo
1. 5—days 2. 3—days 4. 21 days
oo

3 13
2
5. 3 days 6. A, B and C together in 8 days, A in 24 days, B in 14 - days, C in 72 days
B

” 7^ days 8. 6^ days
re

9. 48 days 10. 11 days

n. 13^daysl2. 108 days


u
ad

13. 16 days 14. 6—hours


Yo

7
15. 6 hours 16. 12 hours 17. 40 minutes 18. 81 days 19. 15 days
d

HINTS TO SELECTED PROBLEMS


Re
in

8.-1 + — =
1 1 and^ —1
— + ^-=1
s“3
F

5 3n
2
^ ^ 1 1 1 1 1
9. We have.
s' tlQ
Adding all the three, we obtain
J_ 1 1 3 1 1 3
+ +
16
=> — + — = — => /ig = 48
6 ng 16
1 1 1 7
10.

f 7 3 3
11. Remaining part of the work = 1 — —.A and B complete — part of the work in 4 days.
V 10 J 10 10
4 40
So, they complete the entire work in — = — days
3/10
QUANTIFICATION AND NUMERICAL APPLICATIONS 5.33

K:. nf^=l.y 99, riy^ = 99x 6, = 99x 4


1 1 1 1 1 1 1 n1 1
+ —+—
n
^MWB 2x99 6x99 4x99 198 3 2

1 J_ 11 _ 1 => "MWB =
”mwb 19s'' 6 "l08
We have, = 160, = 8 x 32 = 256. Therefore,

{5 men + 8 woman )'s 1 day's work = — + 8 _5_ 8 _ 1


160 256 16

Hence, 5 men and 8 women together can complete the work in 16 days.
(3 1 1 'l 1
19, Part of work done in 3 days = — + — + — =-.

w
^ 120 30 60 J 5
1
Part of the work done in one day = —

F lo
15

5.6 TIME AND DISTANCE

ee
In this section, we will discuss some problems related to speed, distance covered and time taken

Fr
by a moving object. Let us recall that the speed of a moving object is the distance covered per
unit time.
Distance
for
ur
i.e. Speed =
Time

The above relationship between the three quantities, distance, speed and time can also be
ks

expressed as follows:
Yo
oo

Distance
Distance = Speed x Time or, Time =
Speed
eB

Distance is usually measured in kilometers, meters, miles etc; time in hours or seconds and
speed in km/hr (denoted by kmph), or metres/second (denoted by m/s), or miles/hr (denoted
r
ou

by mph).
ad

1 km 1000 m 5 ,
1 km/hr = = — m/sec
Y

1 hr 3600 sec 18
5 18
nd
Re

V km/hr = — XV m/sec and z;m/sec = — X u km/hr


18 5 )
Fi

If a body is moving with the constant speed, then the relation: Distance = Speed x Time suggests
that the distance covered varies as time and it is written as Distance x Time. Further, if two
bodies travel for the same period of time, then the distance covered varies directly as the speed
Distance
i.e. Distance x Speed. If two bodies travel the same distance, the relation Time =
Speed
1
suggests that time varies inversely as speed i.e. Time x
Speed

AVERAGE SPEED The average speed of a body travelling at different speeds for different time periods is
defined as follows:
Total distance travelled
Average speed =
Total time taken

Consider a body travelling from a point P to a point Q with the speed u units and back from
point Q to point P with the speed v units. Then,
5.34 APPLIED MATHEMATICS-XI

Total distance covered = PQ + PQ = 2PQ


Total time taken = PQ^PQ = PQ -1 + -lA =PQ ( u + v']
ii V vj HV J
Total distalce covered 2PQ 2uv
Average speed =
Total time taken u + v
PQ
uv

u + v
REMARK 1 The average speed is not equal to — and it does not depend on the distance between P
a7id Q.
REMARK 2 If a moving bodp describes distances d^, d2,..-,d„ with different speeds vi,V2,..., v„ in times

ow
t-^, ^2 '●●●/ respectively, then the average speed of the body throughout the journey is given by
d-^ + (^2 +● ● -+ dfj
[When dj and t, are given]
ti+t2 + ...+ tf,

e
1] +1?2 ^2 +...+ v„tj,
[Wheni;,' and t, are given]

re
or.

Fl
V =

+ ^2 +--d71

F
^2 dj71
or. V =
[When dj and Vj are given]
d1
ur
+ +...+
d,7

r
^2 V ti
fo
REMARK? If a body covers a part of the journey at a speed u units and the remaining part of the journey at
ks
a speed v units and the distances of the two parts of the journey are in the ratio m : n. Then, the average
Yo
(m + n) uv units.
oo

speed for the entire journey is


mv +nv
eB

RELATIVE SPEED The speed of one moving object in relation to another jnoving object is called the
relative speed i.e. it is the speed of one moving object as observed from the second ?noving object.
ur

If two objects are moving in the same direction, the relative speed is equal to the difference of the
ad

speeds of the two objects.


Yo

If two objects are moving in the opposite direction, the relative speed is equal to the sum of the
speeds of the two objects.
d
Re

ILLUSTRATIVE EXAMPLES
in
F

EXAMPLE I A school boy, travelling at 3 km/hr, reaches his school 32 minutes late but if he travels at 5
km/hr, he reaches his school 28 minutes early. Find the distance he travels every day to reach the school.
SOLUTION Suppose the boy every day travels distance x km to reach to school in time t hours.
If he travels at 3 km/hr, he reaches his school 32 minutes late.

...(i)
3 60

If he travels 5 km/hr, he reaches his school 28 minutes early.


28
...(ii)
5 60

Subtracting (ii) from (i), we obtain


.r .r — 32 + —=>
28 2x ,
— =1=>a:
15
= — =7b
^
3 5 60 60 15 2

Hence, the boy travels 7.5 km every day to reach to his school.
QUANTIFICATION AND NUMERICAL APPLICATIONS 5.35

EXAMPLt 2 If a person travels in a car at a speed of 30 km/hr, then he would reach his destination on time.
th
( 4
He covers half of the journey in - time. What should be his speedfor the remaining part of the journey
{5
so that he reaches destination on time?

SOLUTION Let the person take t hours to reach his destination with the speed of 30 km/hr.

w
Then, length of his journey = 30 f km.
4t f 4t\ t
He covers half of the journey i.e. 15f km in — hours. In the remaining time i.e. t = - hours
5 V 5 y 5
he wants to cover the second half of the journey i.e. 15t km. So, the speed for the remaining part

e
15f
of the journey is = 75 km/hr.

e
t/5

or
r
EXAMPLE 3 A person travels 285 km in 6 hours in two stages. In the first part of the journey, he travels

F
by bus at the speed of 40 km/hr. In the second part of the journey, he travels by train at the speed of 55
km/hr. Hoio much distance did he travel by train?

oF
ul
SOLUTION Suppose he travels .v km by train and (285 - x) km by bus. He travels (285 - .r) km by
285 -X
bus at 40 km/hr. Therefore, time taken is given by fj = hr

rs
40

ko
Bus Train
B
— (285-x)—- ●-C ■t km
285 km —

Fig. 5.1
of
o
Y
He travels .v km by train at 55 km/hr. Therefore, time taken is given by
YB

f2 = 55
— hr
er

It is given that he travels 285 km in 6 hours.


u

f-j "I" ^2 — ^
d

285-.V
o

.V
ad

+ —= 6
40 55
in

285 - X .Y 285 -Y .V
+ — = 30 => + —= 30
8 11 8 8 11
Re
F

.Y X 285
- 30=> => —=15=>.y=165
8 11 8 88 " 8 11
EXAMPLE -1 The distance betzueen two stations A and B
is 100 km. Aarushi starts on a bike at 6 AM from
A and reaches Bat II AM. Mira starts at 8 AM from station B and reaches station A at 12 noon. Find at
what time ivill they meet.
SOLUTION Suppose Aarushi and Mira meet after t hours of start of Aarushi at Point C. In 5 hours
(6 AM to 11 AM) Aarushi travels 100 km. So,
Aarushi / hours (f-2) hours Mira
B
A C
Fig. 5.2
100
Aarushi's speed = — km/hr = 20 km/hr

In 4 hours (8 AM to 12 noon) Mira travelsAB = 100 km.


100
Mira's speed = — km/hr = 25 km/hr.
5.36 APPLIED MATHEMATICS-XI

Aarushi travels distance AC in t hours and Mira travels distance BC in (t - 2) hours.

AC = 20t and BC =25 {t- 2)


AC+ BC= 20f + 25 (f - 2) = 45f - 50
10
100 = 45t -50=> 45f =150=> f = hours = 3 hours 20 minutes
3

Hence, Aarushi and Mira meet after 3 hours 20 minutes of start of Aarushi.
LXAMPLE 5 Two boys Aman and Raman starts at the same time to ride from Delhi to Manesar 60 km
away. Aman travels 4 km/hr slower than Raman. Raman reaches Manesar and at once turns back meeting
Aman, 12 km from Manesar. Find Aman's speed.
SOLUTION Let Aman's speed be .r km/hr. Then, Raman's speed = {x + 4) km/hr.

ow
Delhi Manesar

P 48 km P-—12km-— Q
60 km ^
Fig. 5.3

e
re
Suppose Raman meets Aman at R such that QR = 12 km. Therefore,

rFl
Time taken by Raman to arrive at R = Time taken by Aman to reach to R from Delhi after

F
reaching Manesar.
Time taken by Raman in travelling 72 km = Time taken by Aman in travelling 48 km

r
ou
72
x + 4
48
= — =>
X
3
x + 4
- = ,r = 8
X
fo
ks
Hence, Aman's speed = 8 km/hr.
oo

i;XAMPLE 6 A person travels from P to Q at a speed of 40 km/hr and returns by increasing his speed by
Y
eB

50%. What is his average speed for both the trips?


SOLUTION We have,
ur

Uj = Speed in travelling from P to Q = 40 km/hr


ad

i>2 = Speed in travelling from Q to P = 150% of 40 = 60 km/hr


Yo

Let V be the averages speed for both the trips. Then,


d

V -
2v-yj2 _ 2 X 40 X 60 = 48 km/hr.
Re
in

Uj +V2 40 + 60
F

AMTER_ We have,

Vi = Speed in travelling from P to Q = 40 km/hr, di = PQ


V2 = Speed in travelling from Q to P = 150% of 40 = 60 km/hr, d2 = QP
Let V be the average speed. Then,
V =
^1 + ^2 _ PQ + QP 2 2x 120
= 48 km/hr.
^+ + J- + -1. 3+2
U2 40 60 40 60

i.\AMPLE 7 A train travels 450 km in 7 hours and another 740 km in 10 hours. Find its average speed.
SOLUTION Here, d-^ = 450, t^ = 7, ^2 = 740 and t2 = 10.
Let V be the average speed of the train. Then,
V -
+ ^2 _ 450 + 740 _ 1190 = 70 km/hr.
fl+f2 7+10 ”17”
QUANTIFICATION AND NUMERICAL APPLICATIONS 5.37

EXAMI’LII 8 A person travels three equal distances at speeds ofx hn/hr, y km/hr and z km/hr respectively.
What is the average speed of whole journey?
SOLUTION Let each distance be d km. Then,
Total distance = d + d + d = 3d km

,
Total time taken = — + — + —
fd,d_d] -
d{xy + yz + zx) hr
X y z xyz
3d ^xyz
Average speed = km/hr
d {xy + yz + zx) xy + yz + zx
xyz

ow
EXA.MPLE9 A man covers a certain distance on a toy train. Had the train moved 4 km/hrfaster, it would
have taken 30 minutes less. If it moved 2 km/hr slower, it zvould have taken 20 minutes more. Find the
distance.

SOLUTION Let the distance covered by toy train be d km and the speed of toy train be v km/hr.
Then,

e
re
T-1

Fl
= Time taken by the train = -V hr.

F
If the train moved 4 km/hr faster, then
ur
r
T-y =Time taken by tlie train = —^
z? + 4 fo
ks
If the train moved 2 km/hr slower, then
Yo
oo

T3 =Time taken by the train =—


eB

30 20
It is given that “^2 “ hour and -Tj =—
60
hour
ur

d d 30
and — d^^
ad

V V + 4 60 V -2 V 60
Yo

4d 1 2d 1
= - and
d

v{v +4) 2 v(v -2) 3


Re
in

y (i) + 4) and d =
v{v -2)
d =
8 6
F

v{v
__
+ 4) _v{v
_ _
-2) ^
v + 4 v-2
=> 3u +12 =4u-8=> = 20
8 4 3

v{v + 4)
Putting i; = 20 in d = ●, we obtain d = 60.
8

Hence, the toy train covered 60 km.


EXAMPLE 10 A car starts running with the initial speed of40 km/hr, with its speed increasing every hour
by 5 km/hr. How many hours will it take to cover a distance of385 km?
SOLUTION Let the car take n hours to cover 385 km. Every hour the speed of the car increases by
5 km/hr. This means that it covers 40 km in first hour, 45 km in the second hour, 50 km in the
third hour, and so on.

40 + 45 + 50 +...upto n term = 385


5.38 APPLIED MATHEMATICS-X!

-j2x40+(?j-l)x5 = 385
Using {2«(«-l} d}
n
- (5?j + 75) = 385

jj (« + 15) =77 X 2
:=>
n~ + 15j; -154 = 0 => (;i + 22) (ji -7) = 0 => “7 = 0 => n=7 [■: »=22^0]
Hence, the car takes 7 hours to cover 385 km.
fxample; ! 1 A takes 2 hours more than B to walkd km, but if A doubles his speed, then he can make it in 1
hour less than B. Hozo much time does B require for walking d km?
SOLUTION Suppose B takes x hours to walk d km. Then A takes (.t + 2) hours to walk d km.

w
d ] fd\
A's speed = km/hr, B's speed = - km/hr
x + 2 Kxj

Flo
2d
A's new speed = km/hr
x+2

ee
It is given that

Fr
d d
= 1
d\ 2d
for
ur
\xj x + 2

x + 2\
s
.r - = 1 => 2-t - .Y - 2 = 2 => .T = 4
ok

2 J
Yo

Hence, B requires 4 hours to walk d km.


Bo

EXERCISES 5.6
re

1.
A car covers 420 km with a constant speed. If its speed were 10 km/hr, it would have taken
one hour less to cover the distance. Find the speed of the car.
ou
ad

An aeroplane first flew with a speed of 440 km/hr and covered a certain distance. It still
Y

had to cover 770 km less than what it had already covered, but it flew with a speed of
600 km/hr. The average speed for the entire flight was 500 km/hr. Find the total distance
nd
Re

covered.
3. A car travels the first one third of a certain distance with a speed of 10 km/hr, the next
Fi

one

third distance with a speed of 20 km/hr and the last one third distance with a speed of
60 km/hr. Find the average speed of the car for the whole journey.
4.
Ravish travels 760 km to his home, partly by train and partly by car. He takes 8 hours, if he
travels 160 km by train and the rest by car. He takes 12 minutes more, if he travels 240 km by
train and the rest by car. What are the speeds of train and car?
Two men starting from the same place walk at the rate of 5 km/hr and 5.5 km/hr
respectively. What time will they take to be 8.5 km apart, if they walk in the same direction?
A car covers 420 km with a constant speed. If its speed were 10 km/hr more it would have
taken one hour less to cover the distance. Find the speed of the car.
7. A and B are two stations 10 km apart. A man, P starts from A and travels towards B at the
rate of 3k/hr, whereas another man Q starts from B and travels towards A at the rate of
2 km/hr. When and where do they meet?
QUANTIFICATION AND NUMERICAL APPLICATIONS 5.39

8. A train P leaves station A at 5 AM and reaches stations 6 at 9 AM. Another train Q leaves
station B at 7 AM and reaches station A at 10 :30 AM. At what time do the two trains cross
each other?
9. Amit starts from a point A and walks to another point B and than returns from B to A by his
car and thus takes a total time of 6 hours and 45 minutes. If he had driven both ways in his
car. he would have taken 2 hours less. How long would it take for him to walk both ways?
10. Two trains start from stations A and 6 and travels towards each other at a speed of
50 km/hr and 60 km/hr respectively. At the time of their meeting, the second train had
travelled 120 km more than the first. Find the distance between the two stations.
ANSWERS

ow
1. 60 km/hr 2. 2750 km 3. 18 km/hr
4. Train : 80 km/hr Car : 100 km/hr 5. 17 hours 6. 60 km/hr
3
7. After 2 hours, 6 km from A 8. 7:56 AM 9. 8 — hours
4

e
10. 1320 km

re
5.7 MENSURATION OF PLANE FIGURES

Fl
F
In this section, we will discuss problems on finding areas bounded by plane figures viz
ur
Trianglles, quadrilaterals, rectangles, parallelograms, rhombi, circles etc.

r
5.7.1 TRIANGLES
fo
ks
Following are some useful points about triangles;
Yo
(i) The sum of the angles of a triangles is 180".
oo

(ii) The sum of any two sides of a triangle is greater than the third side.
In a right triangle: (Hypotenuse)^ = (Base)^ + (Perpendicular)^
eB

(iii)
(iv) The line segment joining a vertex to the mid point of opposite side of a triangles is called a
median.
ur

(v) The point of concurrence of three medians of a triangle is known as the centroid which
ad

divides the medians in the ratio 2:1.


Yo

(Vi) A median of a triangle divides it into two triangles of the same area.
The line segment joining the mid-points of any two sides of a triangle is parallel to the
d

Vll

third side and is equal to half of it.


Re
in

(viii) In an isosceles triangle, altitude from the vertex bisects the base.
F

(ix) The four triangles formed by joining the mid-point of the sides of a triangle are equal in
area, each equal to one fourth of the area of the given triangle.
(X) The ratios of the areas of two similar triangles is equal to the ratios of the squares of their
corresponding (i) altitudes (ii) medians (iii) sides (iv) angle bisectors.
IMPORTANT FORMULAE
1
Basex Height
(i) Area of a triangle = ^ ^
Area of AABC = (s - a) (s - b) (s-c) ,

where a, b, c are the lengths of sides BC, CA and AB respectively


and s = ~{a + b + c).
2
1 1 1
Area = — be sin A = —cfl sin B =—ab sin C
2 2 2
5.40 APPLIED MATHEMATICS-XI

Vs
(ii) Area of an equilateral triangle = (Side)^, Altitude (p) = (Side), Area = .
2 v3

ow
(iii) Let ABC be an isosceles triangles such that AB = AC =a and base (BC) = b. Then,

e
re
Frl
F
ou
or
kfs
oo
Y
B

(iv) Let ABC be a right-angled isosceles triangles such that AB = BC = a (Fig. 5.6). Then,
re

AC + a
^ = V2 fl, Area of AABC Perimeter = (2 + V2) a.
oYu
ad

ILLUSTRATIVE EXAMPLES
d

The base of an isosceles triangles is 14 cm and its perimeter is 36 cm. Find its area.
in

SOLUTION Let ABC be an isosceles triangle with base BC = 14 cm and two equal sides
Re

AB = AC — a. Let AD be the altitude. Then, D is mid-point of base BC. It is given that


F

AB + BC + AC = 36 cm =^> 2fl +14 = 36 ^ a = 11 cm.

Fig. 5.8
Applying Pythagoras theorem in AADB, we obtain
AB^=AD^ + Bd'^ => ll^=AD^+7^ => AD^ =72 => AD =6^2
Area of AABC = ~ (BC X AD) = i X 14 X 6V2 = 42V2 cm^
QUANTIFICATION AND NUMERICAL APPLICATIONS 5.41

EXAMPLE 2 The lengths of the sides of a triangles are 25 cm, 39 cm and 56 cm. Find the length of the
perpendicidar from the vertex opposite the largest side.
SOLUTION Let a = BC = 56 cm, b = CA = 39 cm and c = AB = 25 cm be the lengths of sides of AABC
and s be its semi-perimeter. Then,
56+ ^9 + 25
s = = 60
2
Let A be the area of AABC. Then,
A = ^s{s -a){s - b) (s - c) =^60 (60 -56) (60 - 39) (60 -25)
=>
A = ^60x 4x 21 X 35 =^2^ X 3x5x 2^ X 3x7 x7 x5 = 2^ X 3x5x7 = 420cm^ ...(i)
Let AD be perpendicular from vertex A on the largest side BC. Then,

w
A =
^ (SC X AD) = - X 56 X 2lD = 28AD ...(ii)

o
e
re
rFl
F
r
ou
From (i) and (ii), we obtain
28AD=420 => AD =15 cm
fo
ks
EXAMPLE 3 If the perimeter of a right-angled triangle is 234 m and the hypotenuse is 97 m,find its
(i) Area (ii) other two sides
oo

SOLUTION Let ABC be a right triangle right angled at B such that b = AC = 97 m. Let other two
Y
eB

sides be fl = BC and c = AB. It is given that


AB + BC + CA = 234 m => c + + 97 = 234 => c + = 137
A
r
ou
Y
ad

c
d
Re
in
F

Applying Pythagoras theorem, we obtain


AC^ = BC^ + AB^ => = 97^

(i) We have, c + fl=137, c^ + a ^ = 97^ and we wish to find the area of AABC i.e. —2 ca.

Now, (c + a)'^ -{c^ +a^)=2 ca


1
— ca =
(c + a)^ -{c^ +a^) 137^-97^ (137 + 97) (137-97) 234x40 = 2340
2 4 4 4 4

Area of AABC = 2340


5.42 APPLIED MATHEMATICS-XI

(ii) We have, c + <7 = 137, = 97^ and we wish to find the values of c and a.

From (i), we obtain ca = 4680.


(c -n)^ =(c + a)^ -4ca
=> (c - = 137^ - 4 X 4680 = 18769 -18720 = 49
=^> c -a=7

Thus, we have
c + a= 137 and c - a = 7 => 2c = 144 and 2a = 130 ^ c = 72 and a = 65
Hence, AB =72 m and BC =65 m.

EXAMPLE 4 In Fig. 5.11, ABC is an equilateral triangle and O is a point inside it. If lengths of
perpendiculars drawn from O on sides BC, CA and AB are 8 cm, 7 an and 6 cm respectively, find the area
ofAABC.

w
V3 2.
SOLUTION Let the length of each side of equilateral triangle ABC be a. Then, its area is — a
4

F lo
Join OA, OB and OC.

ee
Fr
for
ur
s
ook
Yo
eB

Clearly, Area of AABC = Area of A06C + Area of AOCA + Area of AOAB


r
ou
ad

^-2 i(BCxOD)+^(CAxO£) + ^(ABxOf)


4
a
Y

●JS a 2
4 i (t? X 8) + ^ (n X 7) + ^ (fl X 6)
Re
nd

V3 a 2 21a => a = ^4^f3 cm


Fi

4 2
V3
Area of AABC = —
4
X (14V3)^ cm^ = 147^3 cm^
E XAMPLE 5
The perimeter of a triangle is 30 cm and its area is30cm^. If the largest side measures 13 cm,
then find the lengths of other two sides.
SOLUTION Let the length of one of the sides other than the largest be x cm. Then, the other side is
of length (17 - .v) cm.
We have, 2s = 30 and Area = 30 cm^.
Area = fs (s ~a) (s -b) (s -c)
30 = .J15(15-13)(15-a:) (15-(17 - .v))
30=fl5x2K(15-x){x-2)
900 = 30 (15-X) (x'-2)
QUANTIFICATiON AND NUMERICAL APPLICATIONS 5.43

^ 30 + 17X - 30=> -17X + 60 =0 => (A--5) (x -12) = 0 => x =5,12

Hence, length of other two sides are 5 cm and 12 cm.


EXERCISES 5.7

1. Find the area of an isosceles right triangle whose perimeter is (6 + 3^2) cm.

2. The perimeter of a right-angled triangle is 60 cm. If its hypotenuse is 26 cm long, find its
area.

3. The lengths of base and altitude of a right-angled triangle are 12 cm and 5 cm respectively.
Find the length of the perpendicular on the hypotenuse from the opposite vertex.
4. Find the area of a triangle whose sides are in the proportion 4:5:6 and the perimeter is
195 cm.

5. The lengths of the sides of a triangle are 21 cm, 20 cm and 13 cm. Find the areas of triangles
into which it is divided by the perpendicular on the longest side from the opposite vertex.

w
6. If each side of an equilateral triangle is increased by 4 cm, its area is increased by 14sf3 cm .
Find the length of each side.

AD + AC = 2 BD, find the area of AABC.

F lo
7. ABC is a right triangle right-angled at B and D is a point on AB such that BD = BC = 33 cm. If

8. Find the area of a triangle whose lengths of two sides and perimeter are 85 cm, 154 cm and

ee
324 cm respectively.

Fr
9. The perimeter of an isosceles triangle is 306 cm and length of each equal side is —8 of the base.
for
Find the area of the triangle.
r
10. If areas and heights of two triangles are in the ratio 4:3 and 3; 4 respectively, find the ratio
You

of their bases.
s
ook

11. The perimeter of an isosceles triangle is 14 cm and the lateral side is to the base in the ratio
5 :4. Find the area of the triangle.
eB

ANSWERS
1. 4.5 cm^ 2. 120 cm^ , —60 cm 4. 1676.75cm^
our

3.
ad

13

5. 96 cm^, 30cm^ 6. 5 cm 7. 726 cm^ 8. 2772cm^


2
dY

9. 3468 cm ^ 10. 16:9 11. 2V2I cm


Re
Fin

5.7.2 QUADRILATERALS

QUADRILATERAL A figure formed by the four line segments is a quadrilateral


In Fig. 5.12, ABCD is a quadrilateral with diagonals TIC and BD. On diagonal AC perpendiculars
are drawn from vertices B and D which are known as offsets.

Area of quadrilateral ABCD = ~ ^ ^ (Pi + P2^-


5.44 APPLIED MATHEMATICS-XI

PARALLELOGRAM A quadrilateral zuhose opposite sides are parallel and equal is called a parallelogram.
Each diagonal divides the parallelogram into two triangles of equal area.
Area of parallelogram ABCD = 2 (Area of AABC) = 2 (Area of AABD).

Also, Area of parallelogram ABCD = ^ (Basex Height) = ^ {AB x DL)


D C

ow
A L B

e
Fig. 5.13

re
The sum of the squares of diagonals of a parallelogram is equal to twice the sum of the squares of
adjacent sides.

Frl
F
RHOMBUS A quadrilateral whose all sides are equal is called a rhombus.
Diagonals of a rhombus bisects each other at right angle.
ou
osr
kf
oo
Y
B
re
Y
u

In Fig. 5.14, ABCD is a rhombus whose each side is equal to a. Let AC = d^ and BD = ^2 be its
ad
do

^2
diagonals intersecting at O. Then, OA = OC = — and OB = OD =
2 2
in
Re

/1
A = Area of rhombus = 2 (Area of AABC) = 2 - xACxOB
- -r - Tz ^2
F

V 2 V z y

Applying Pythagoras theorem in AAOB, we obtain


(d 1 \2 r j \2
^2
OA^ +OB^=AB^ => + = a
^ => dl + d2= 4a^.
[ 2 2J
Thus, we obtain

A =
i frj fr2- Perimeter of rhombus ABCD = 4a and, d^ + d^ = 4a^.
Now,

(d-[ + *^2)^ —dj+d^-^ 2dj dj and, (dj -1^2)^ = ~ 2d| d2


(dj + d2)^ = 4fl^ + 4A and, (dj -d2)^ =4a^ 4A

(dj + d2)^ =4{a^ + A) and, (dj - d2)^ =4{a^ - A)


QUANTIFICATION AND NUMERICAL APPLICATIONS 5.45

RECTANGLE A parallelogrnm ivhose adjacent sides are at right angle is called a rectangle.
Diagonals of a rectangle are equal and bisect each other.
In Fig. 5.15, ABCD is a rectangle such that/46 = I and AD = b. I and b are length and breadth of the
rectangle. Let AC = BD = d. Further, let A be the area of rectangle ABCD. Then,
A=lxb

D C

A I B

w
Fig. 5.15

Applying Pythagoras theorem in AABC, we obtain

AB^ + BC^=AC^^ l^+b^=d^^d = ^jl^+b^

F lo
ee
Now,

Fr
(l + bf=l^+b^ + 2lb and {I ~bf = l^ + -2lb
{l + bf=d^ + 2A and {l-bf=d^-2A for
ur
SQUARE A rectangle whose all sides are equal is a square.
Diagonals of a square are equal and bisects each other at right angle.
s
ook
Yo

Area of a square = (Side)^, Perimeter = 4 (Side), Diagonal = V2 (Side)


C
eB
r
ou
ad
Y

B
Re
nd

Fig. 5.16
Fi

TRAPEZIUM A quadrilateral havmg exactly one pair of parallel sides is called a trapezium.
A trapezium is called an isosceles trapezium, if its non-parallel sides are equal.

Area of a trapezium = ^ (Sum of parallel sides) x (Distance between parallel sides)


D C

A L B

Fig. 5.17
5.46 APPLIED MATHEMATICS-Xl

CYCLIC QUADRILATERAL A quadrilateral is a cyclic quadrilateral if its vertices lie on a circle.


If s is the semi-perimeter of a cyclic quadrilateral ABCD such that AB = a,BC = b, CD = c and DA =
d, then its area A is given by
A = ^{s-a){s-b) is~c){s-d)
In cyclic quadrilateral ABCD, we have
ZA + ZC= 180° and ZB + ZD= 180°

w
F lo
POLYGON A figure bounded by four or more line segments is called a polygon.
If all sides and all angles of a polygon are equal, it is called a regular polygon.

e
A five sided polygon is called a pentagon and six sided polygon is called a hexagon.

Fre
(2n-4)
The measure of each interior angle of n sided regular polygon is X 90°
for n

BVS . ,2
(i) Area of a regular hexagon = — (Side)^
r
You

(ii) Area of a regular pentagon = 1.72 (Side)


oks

n
(Side)^.
eBo

(hi) Area of an «-sided regular polygon = nso 0\


4 tan
n
ad
our

ILLUSTRATIVE EXAMPLES

l-XAMPLE 1 Find the area of a quadrilateral one of its diagonals is of length 50 cm and lengths of
perpendicular drawn from other tzuo vertices on it are 21 cm and 29 cm respectively.
Re
dY

SOLUTION Let ABCD be a quadrilateral in which diagonal AC = 50 cm, pi =BL= 21 cm and


Fin

P2 = DM = 29 cm.
C

P2

\Pi

Fig. 5.19

Area of quadrilateral ABCD = ^ (AC) {pi + P2) = -^ x 50 x (21 + 29) = 1250 cm^
QUANTIFICATION AND NUMERICAL APPLICATIONS 5.47

EXAMPLE 2 Find the area of a quadrilateral ABCD in which AB = 143 cm, BC = 154 cm, CD = 60 cm,
AD = 125 cm ami diagonal AC = 165 cm.
4* SOLUTION Clearly, Area of quad. ABCD = Area of AABC + Area of AACD.
' In triangle ABC, we find that
AB+BC + AC 143 + 154 + 165
$ =
2

ow
e
re
Frl
F
Area of AABC =

= ^231 (231 -143) (231 -154) (231 -165)


ou
r
so
= f231 X 88x77x 66 = -^3x7xllx 2^ X 11 x7xll X 2x 3x11
kf
= 2^x 3x7xll“=10164 cm^
oo

AC +CD+ AD 165 + 60 +125


In AACD, we find that s = = 175
Y

2 2
eB

Area of A ACD - .Js{s^^AC)(s^^^CU){s^^AD)


= ^175 (175 -165) (175 - 60) (175 -125) = ^175 x 10 x 115 x 50
ur
oY

= ^{5^ X 7) X (2 X 5) X (5 X 23) x (5^ x 2)


ad

= 5^x2 ,]7V23 = 250 = 250x 12.688 = 3172 cm^


d

Hence, Area of quadrilateral ABCD =(10,164 + 3,172) cm^ =13,336 cm^


in
Re

EXAMPLE 3 ABCD is a quadrilateral such that AB = 8 cm, BC= 8 cm, CD = 7 cm, AD = 5 cm and
F

/lABC = 60°. Find the area of quad. ABCD.


SOLUTION In AABC, we find that AB = BC =8 cm and ZABC = 60°.

ZBAC = ZBCA =
5.48 APPLIED MATHEMATICS-XI

So, &ABC is an equilateral triangle. Consequently AC = 8 cm.


73
Area of AABC = X 8^ =16^/3cm^
4
AC + CD + AD 8 + 7+5
In AACD, s - = 10 cm.
2 2

Area of AACD = .^s(s^rAC)(s”^^DHs^^AD)

w
= 710(10-8) (10-7) (10-5) = 710 X 2 X 3 X 5 = 2673 cm^.
l-XAMPLE 4 Find the area of a cyclic quadrilateral ABCD lengths of whose sides are AB = 36 cm, BC = 77
an, CD = 75 cm and AD = 40 cm.

e
SOLUTION Let A be the semi-perimeter of cyclic quadrilateral ABCD. Then,

re
o
AB + BC + CD + AD 36 + 77 + 75 + 40
s = = 114 cm.

r
2 2

F
Let A be the area of cyclic quadrilateral ABCD. Then,
A = 7(7^^A^7i^^BC)(s“^CDj7r^^^ oF
ul
A = 7(114-36) (114-77) (114-75) (114-40) =yj78 x 37 x 39 x 74

sr
A = 72 X 39 X 37 X 39 X 2 X 37 = 2 x 37 x 39 = 2886 cm^

ko
fiX.AMPLE 5 ABCD is a cyclic quadrilateral such that AB = 30 cm, BC = 40 cm, CD = 20 cm. If diagonal
of
AC is a diameter of the circumscribing circle, find the area of quadrilateral ABCD.
SOLUTION Applying Pythagoras Theorem in AABC, we obtain
o
Y
AC^ = AB'^ + BC^ => AC'^ = 30^ + 40^ = 2500 => AC =50 cm.
B

D
Y
er

C
u
d
o
ad

o
in

90°
Re

A B
F

30 cm

Fig. 5.22

Applying Pythagoras theorem in AADC, we obtain


AC^ =AD^ +DC^=> AD^ =AC^ -CD'^^ AD^ =50^ -10^ =2400 => AD = 1046 cm.

Area of quad. ABCD = Area of AABC + Area of AADC =-^ (ABx BC) + ^ (AD x DC)
= i (30 X 40) + i (2076 x 10) =(600 +10076) cm^
= 600 + 100 X 2.449 = 844.9 cm^
EXAMPLE 6 The area of a parallelogram is 338 cm"^. If its altitude is twice the corresponding base,find the
base and altitude.

SOLUTION Let & be the base of the parallelogram. Then, the corresponding altitude is 2b.
QUANTIFICATION AND NUMERICAL APPLICATIONS 5.49

Aiea=bx2b=2b^

It is given that the area is 338 cm .


= 338 => &^ = 169 => 6 = 13 cm.
Hence, the base = 13 cm and altitude = 2b = 26 cm.
EXAMPi.f- T //dj, ^2 are the lengths of the diagonals of a rhombus of side a and area = A, prove that

(i) {dl+dl)=ia^ (ii) = ^2

(iii) {di + d2)^ = 4 + A) (iv) (d^-d2f=4(a^-A)


SOLUTION (i) Leti4BCD be a rhombus of side length a i.e. AB = BC= CD = DA = a and diagonals
AC = d| and BD = ^2-
Diagonals AC and BD bisect each other at right angle at O.

w
Applying Pythagoras Theorem in AAOB, we obtain

OA'^ +OB^ =AB^

F lo
= => dj + d| =4<7^.

e
Fre
for
r
You
oks
eBo
ad
our

(ii) Area of rhombus ABCD = 4 (Area of AAOB)

A=4
1
- X OA X 06) = 2f^2 d2 1
- - did2
2 ’) I 2 2
Re
dY

(iii) From (i) and (ii), we obtain


Fin

1 9 ^
A = ^ dj ^2 srid + ^2 = 4«2

(dj + d2)^ = d^ + d| + 2djd2


(dj + d2)^ = 4fl^ + 4A di d2 = 2A and d-^ + d2 = 4fl^

(d^ + d2)^=4(fl^+A)
(iv) (di - d2f = dl + dl~ 2di d-

(d| -d2)^ =4fl^ -4A d| d2 = 2A and d-^ + d-^ = 4<?^

(di-d2)2=4(«2-/i).
5.50 APPLIED MATHEMATICS-XI

EXAMPLE 8 The area of a rhombus is 4800 cm^ and its side is 80 cm. Find the lengths of its diagonals.
SOLUTION We have,/I =4800 cm^ and a = 80 cm.
Let the lengths of diagonals be d^ and ^2. Then,
+ ^2)^ =4 ((7^ + A) and(di -^2)^ =4 (fl^ - A)
[di + djf = 4 (6400 + 4800) and (dj - = 4 (6400 - 4800)
(dj + d2)^ = 6400 (4 + 3) and (d^ - d2f - 6400 (4 - 3)
{d-^ + d2^ = 6400 X 7and (d| - d2)^ = 6400
di + d2 = 80V7 and d-^ ~~ d2 — 80
2di = 80 (V7 +1) and 2d2 = 80 {f? -1)

w
di = 40 (V7 + 1) and d2 = 40 (VZ -1)

F lo
Hence, the lengths of diagonals are 40(^/7 +1) cm and 40(%/7 -1) cm.
iiXAMPLE 9 The perimeter of a rhombus is 48 cm and the sum of the lengths of its diagonals is 26 an. Find

ee
the area of the rhombus.

Fr
SOLUTION Let the length of each sid^ of the rhombus be a cm and the lengths of its diagonals be
dj and d2-Further, let the area of the rhombus be A. It is given that
for
Perimeter = 48 cm and d-j + d2 = 26 cm
ur
=> 4n = 48 and dj + d2 = 26 ^ a =12 and d2 + d2 = 26
s
ook

We know that
Yo

(d^ + d2)^ =4(fl^ + A) 26^=4(144 + A) 676 = 576 + 4A => 4A =100 => A = 25 => A = 25
eB

●y
Hence, area of the rhombus is 25 cm .
our

EXAMPLE 10 If the area and perimeter of a rectangular grass plot are 630 m^ and 106 m respectively,find
ad

its length and breadth.


SOLUTION Let the length and breadth be I and b respectively. Then,
Y

Area = 630 m^ and Perimeter =106 m


Re
nd

lb = 630 and 2 (/+1?) = 106 => lb =630 and l + b=53


Fi

Now,

{l-bf ={l + b)^ -4lb


(I-bf =53^ -4x 630= 2809 - 2520 = 289
/-l7=17

Thus, we obtain
I + b=53 and l-b = 17 => 2/=53 + 17 and 2h =53-17 I = 35 and b=18

Hence, length and breadth of the grass plot are 35 m and 18 m respectively.
QUANTIFICATION AND NUMERICAL APPLICATIONS 5.51

EXAMPLE 11 If the diagonal and area of a rectangle are 25 cm and 168 cm respectively, find its length
and breadth.

SOLUTION Let the length and breadth of


ly
the rectangle be / and b respectively. It is given that
diagonal AC = 25 cm and area =168 cm .
D C

ow
A 1 B

Fig. 5.24

Jl^+b^ =25 and lb =168 => l^+b^= 625 and lb =168

e
Now,(l + b)^=l^+b^ + 2lb and {l-bf=l'^+b'^- 2lb

re
(I+ bf =625+ 336 and (I-bf =625^336
rFl
F
{l + bf=961 and (/-17)^=289

r
/ + i)=31 and l-b=17
ou
(l + b)+(l-b) = 31+17 and (/ + /))-(/-&) = 31 -17 fo
ks
217=48 and 2/7=14 => /=24 andb=7
Hence, length and breadth of the rectangle are 24 cm and 7 cm respectively.
oo

i-XAMPLE 12 The length of a rectangle is increased by 60%. By what percent would the width have to be
Y
eB

decreased so as to maintain the same area?

SOLUTION Let the length and breadth of the rectangle be / and b respectively. Then, its area A is
given by
r
ou

A=lb
ad
Y

601 31
Increase in length = 60% of / =
100 5
d

vr 1 ..i. » 3/ 8/
New length = / + — = —
Re
in

Let the breadth be decreased by A/?. Then,


F

New breadth = b - Ab.


81
Area of rectangle with length = — and breadth = {b - Ab) is given by

A1 - ^ib-Ab) = -^^-'^
5 5 5
Ab ...(ii)

It is given that A =
^ lb = 8lb _ 81 Ab 3lb 81 Ab 3b
3b = 8Ab => Ab = —
5~ ~5 5 5 8

3b
Ab 8
.-. Percentage decrease in breadth = — X 100 = xlOO=-xlOO=37-%
b b 8 2

L.XAMIM.L 13 Compare the areas of an equilateral triangle, a square and a regidar hexagon of equal
parameter.
5.52 APPLIED MATHEMATICS-XI

SOLUTION Let P be the parameter of each of an equilateral triangle, a square and a regular
hexagon. Let ^7 be the length of each side of an equilateral triangle of parameter P and let^lj be its
area. Then,

on P ,,
3n=P => a = — and, Ai =—a >/3 2 = — x —= =
3 ‘4 4 9 12^/3
Let A- be the length of each side of a square of parameter P and let A2 be its area. Then,
p P^
4a = P => A = — and, A-y = A'2 =
4 ^ 16

Lett/be the length of each side of a regular hexagon ofparameter Pand let A3 be its area. Then,

6a ~P=> X-
P
— and, Ao = 3V3 ..2 3^

w
X
6 ^ 2 2 36 ~ 8V3

Ai;A2 A3 =
P^ _P^_ P^ = 4:3^/3:6.

Flo
I2V3 ■ 16 ■ 8V3
7 XAMPLt 14 The cost offencing an equilateral triangular park and a square park is the same. If the area of

ee
the triangular park isl6-j3 ,find the length of the diagonal of the square park.

Fr
SOLUTION Let the length of each side of the triangular park be a metre. Then, its area is ——.
for
ur
But, it is given that the area is 16>/3 m^.
J3a^
= 16V3=>rt^=64=>fl = 8
s
4
ok
Yo

The cost of fencing the triangular and square park is same. Therefore, their perimeters are same.
Bo

Perimeter of square park = Perimeter of triangular park = (3 x 8) m = 24 m


=> Side of the square park = 6 m
re

Hence, diagonal of the square park = 6V2 m.


ou
ad

EXERCISE 5.8
Y

1. ABCD is a quadrilateral such that AB = 20 cm, BC = 13 cm, CD = 17 cm, AD = 10 cm and


AC = 21 cm. Find its area.
nd
Re

Find the area of a cyclic quadrilateral ABCD whose sides are AB = 25 cm, BC = 39 cm,
CD = 60 cm and AD = 52 cm.
Fi

3. Find the area of the quadrilateral one of whose diagonal is 28 cm and the length of
perpendiculars drawn from other two vertices on it are 12 cm and 18 cm.
Find the area of a quadrilateral ABCD in which AB = 9 cm, BC = 40 cm, CD = 28 cm,
DA = 15 cm and ZABC = 90°.
j.
The length of a rectangle is decreased by r % and the breadth is increased by (r + 5)%. Find r,
if the area of the rectangle is unaltered.
6.
Find the perimeter of a rhombus, the lengths of whose diagonals are 56 fm and 33 cm.
7. If the area and perimeter of a rhombus are 1320 cm^ and 146 cm respectively, find the
lengths of its diagonals.
8. The area of a grassy plot is 480 m^. If each side had been 5 m longer, the area would have
been increased by 245 m^. Find the length of the fence to surround it.
QUANTIFICATION AND NUMERICAL APPLICATIONS 5.53

9. The area of a rectangle is 252 cm^ and its length and breadth are in the ratio of 9 : 7
respectively. What is its perimeter?
TO. Find the height of a rhombus whose diagonals are 72 cm and 96 cm long.
11. A rectangular carpet has an area of 120 m^ and a perimeter of 46 metres. Find the length of
its diagonal.
12. Instead of walking along two adjacent sides of a rectangular field, a boy took a short-cut
along the diagonal of the field and saved a distance equal to half of the longer side. Find the
ratio of the shorter side of the rectangle to the longer side.
13. Find the area of a rhombus one side of which measure 20 cm and one diagonal is 24 cm.
14. The length of one side of a rhombus is 6.5 cm and its altitude is 10 cm. If the length of one of

w
its diagonals is 26 cm, find the length of other diagonal.
ANSWERS

1. 210 cm^ 2. 1764 cm^ 3. 420 cm^ 4. 306 cm^

Flo
e
5. 20 6. 130 cm ". 48 cm, 55 cm 8. 88 m

re
9. 64 cm 10. 57.6 cm 11. 17 m 12. 3:4

F
13. 384 cm^ 14. 5 cm
ur
r
5.7.3 CIRCLES fo
A circle is the locus of a point which moves in a plane in such a luay that its distance from a fixed point
ks
always remains same.
Yo

The fixed point is called the centre and the given constant distance is known as the radius of the
oo

circle.

CIRCUMFERENCE The perimeter of a circle is generally known as its circumference.


B

We know that circumference of a circle bears a constant ratio with its diameter. This constant
re

ratio is denoted by the Greek letter n (read as 'pi')-


Thus, if C denotes the circumference of a circle of radius r. Then,
u
ad
Yo

Circumference C
K = K = — ^ C = Inr
Diameter 2r

Here, n stands for a particular irrational number whose approximate value upto two decimal
d
Re

place is 3.14. The value of nupto four places of decimal is 3.1416 and up to eight decimal places
in

22
its value is 3.14159265. For practical purposes, wc generally take the value of k as — or, 3.14
F

approximately.
If r is the radius of a circle, then
(i) Circiunference = 2nr
Also, Circumference = Kd, where d = 2ris the diameter of the circle.
2 ( 1 2
(ii) Area = nr . Also, Area = n - (ii) Area of semi-circle = — CTr
[2 4

1 2
(iv) Area of a quadrant of a circle =

AREA ENCLOSED BY TWO CONCENTRIC CIRLES If R and r are radii of two concentric circles,
then

Area enclosed by the two circles = tuR -nr ^ =n{R^-r^)=n(R+r){R-r)


2

tv'
5.54 APPLIED MATHEMATICS-XI

Some useful results:


(i) If two circles touch internally, then the distance between their centres is equal to the
difference of their radii,

w
(ii) If two circles touch externally, then the distance between their centres is equal to the sum
of their radii,

F lo
(iii) Distance moved by a rotating wheel in one revolution is equal to the circumference of the
wheel,

(iv) The number of revolutions completed by a rotating wheel in one minute

ee
Distance moved in one minute

Fr
Circumference

SECTOR OF A CIRCLE AND ITS AREA


for
ur
Consider a circle of radius r having its centre at the point O. Let A, B, and C be three points on the
circle as shown in Fig. 5.26. The area enclosed by the circle is divided into two regions, namely,
s

OBA and OBCA. These regions are called sectors of the circle. Each of these two sectors has an
ook
Yo

arc of the circle as a part of its boundary. The sector OBA has arc AB as a part of its boundary
eB

whereas the sector OBCA has arc ACB as a part of its boundary. These sectors are known as
minor and major sectors of the circle as defined below.
MINOR SECTOR A sector of a circle is called a minor sector if the minor arc of the circle is a part of its
our
ad

boundary
In Fig. 5.26, sector OAB is the minor sector.
Y
Re
nd
Fi

MAJOR SECTOR A sector of a circle is called a major sector if the major arc of the circle is a part of its
boundary.
In Fig. 5.26, sector OACB is the major sector.
Following are some important points to remember:
(i) A minor sector has an angle 0 (say) subtended at the centre oftlie circle, whereas a major sector has
no angle.
5.55
QUANTIFICATION AND NUMERICAL APPLICATIONS

(ii) The sum of the arcs of major and minor sectors ofa circle is equal to the circumference of the circle.
(iii) The sum of the areas of major and minor sectors of a circle is equal to the area of the circle.
(iv) The boimdary of a sector consists of an arc of the circle and the two radii. In Fig. 5.26, boundary of
sector OAB consists of two radii OA, OB and arc AB.
AREA OF A SECTOR

Consider a circle of radius r having its centre at O. Let AOB be a sector of the circle such that
ZAOB = 0. If 0 < 180°, then the arc AB is a minor arc of the circle. Now, if 0 increases the length of
the arc AB also increases and if 0 becomes 180°, then arc AB becomes the circumference of a
semi-circle. Thus, if an arc subtends an angle of 180° at the centre, then its arc length is Kr.

w
F lo
ee
Fr
for 0
If the arc subtends an angle of 0 at the centre, then its arc length = 180 X nr
ur
Hence, the arc length / of a sector of angle 0 in a circle of radius r is given by
0
s

/ =
ook

X nr
Yo

180
0 0
X (Circumference of the circle)
eB

/ = X 2nr =
360 360

As discussed above, if the arc subtends an angle of 180° then the area of the corresponding sector
1 2
our

is equal to the area of a semi-circle i.e — nr .


ad

If the arc subtends an angle 0, then area of the corresponding sector


0 12 0
Y

2
= — x—nr X Tcr
180 2 360
Re
nd

the area A of a sector of angle 0 in a circle of radius r is given by


0 2 0
Fi

A = X nr X (Area of the circle) ...(ii)


360 360
0 2 A ^ 0
Also, A = X nr => A = - X Tir r=> A = -Ir [Using (i)]
360 2U80 2

REMARK Area of major sector - Area of the circle - Area of minor segment
Some useful results to remember:
(i) Angle described by minute hand in 60 minutes = 360°
Angle described by minute hand in one minute = 60

Thus, minute hand rotates through an angle of 6" in one minute.


(ii) Angle described by hour hand in 12 hours = 360°
r360'l = 30°
Angle described by hour hand in one hour = 60 )
5.56
APPLIED MATHEMATICS-XI

360'l 1
Angle described by hour hand in one minute =
60 2
nV.
Thus, hour hand rotates through - in one minute.

SEGMENT OF A CIRCLE AND ITS AREA

Consider a circle of radius r having centre at point O. Let PQ be a chord of the circle and let R and
S be two points on it as shown in Fig. 5.27. The area enclosed by the circle is divided by the chord
PQ into two segments, viz. PRQ and PSQ. Each of these two segments has an arc of the circle as a
part of its boundary. Arc PRQ is the minor one and the arc PSQ is the major one.

ow
SEGMENT OF A CiRCLET/ic region enclosed by an arc and a chord is called the segment of the circle.
In Fig. 5.28, the shaded region PRQ is a segment of the circle. The boundary of a segment
consists of an arc of the circle and the chord determining the segment. In Fig. 5.28, the boundary
of segment PRQ consists of arc PRQ and chard PQ.

e
re
Frl
F
ou
or
kfs
oo
Y
B

MINOR SEGMENT//f/ie boundary of a segment is a minor arc of a circle, then the corresponding segment
is called a minor segment.
re

In Fig. 5.28, segment PQR is a minor segment.


oYu

MAJOR SEGMENTS segment corresponding a major arc of a circle is known as the major segment.
ad

In Fig. 5.28, segment PQS is a major segment.


AREA OF A SEGMENT OF A CIRCLE
d

Draw a circle of radius r. Let O be the centre of the


circle and PQ be a chord dividing the circle
in
Re

into two segments PRQ and PSQ as shown in Fig. 5.29. Suppose we wish to find the area of the
minor segment PRQ (shaded region in Fig. 5.29). Let ZPOQ = 0.
F

0; 0
2 ; 2

ri

R
Fig. 5.29

It is evident from Fig. 5.29 that


Area of the sector OPRQ = Area of the segment PRQ + Area of AOPQ
QUANTIFICATION AND NUMERICAL APPLICATIONS 5.57

Area of the segment PRQ = Area of the sector OPRQ - Area of AOPQ
0 2
Clearly, Area of the sector OPRQ = 360
X nr

In AOLP, we have
0
cos- =
OL
and, sin
0 _ PL
2 OP 2 ^OP
0
OL = OP cos — = r cos - and, PL = OP sin - = r sin —
2 2 2 2

OL=r cos - and, PQ = 2Pl = 2r sin —


2 2
1 r 0^ 0
AOPQ=i(PQxOL)=i 2r sin — X r COS-
2 2;
2_:
= r~ sin - cos ~
2 2

low
Hence,
0 2 2 ● 0 0 71 „ . 0 0
X 0-sm—cos—
Area of segment PRQ = xnr -r sm —cos—=
2 2 360 2 2
360

REMARK Area of the ynajor segment PSQ = Area of the circle - Area of minor segment PQR.

ee
NOTE It should be noted that the area of the minor segment of a circle is always less than the area of its
rF
corresponding sector but the area of the major segment of a circle is greater than the area of its

Fr
corresponding sector.
CIRCUM-CIRCLE OF A TRIANGLLE

Let ABC be a triangle. The perpendicular bisector of its sides AB, BC and CA meet at a point O for
(say) known as the circumcentre of AABC. Point O is equidistant from three vertices of aABC i.e.
u
OA = OB = OC. Circle with centre O and radius OA = OB= OC passes through the vertices i.e.
ks

circumscribe AABC. This circle is called the circum-circle of AABC.


Yo
o

If BC = a,CA = b, AB = c and A is area of AABC, then the radius of circumcircle is given by


Bo

abc
R =
4A
re
ou
ad
Y
nd
Re
Fi

Let ZBAC=A,ZABC =
a b
R =
2 sin A 2 sin B 2 sin C

IN-CIRCLE OF A TRIANGLE The bisectors of angles of a AABC meet at a point known as the
incentre / (say) of AABC. Sides of AABC are equidistant from incentre / i.e. perpendicular
drawn from I on the sides of AABC are equal say equal to r. A circle drawn with centre at /
and radius equal to r touches the sides of AABC and is known as incircle of AABC. Its radius
r is given by
5.58 APPLIED MATHEMATICS-XI

r =
—,
s
where A is the area and s is the semi-perimeter of AABC.
A. B C
Also, r = (5 “ «) tan — , r =(s -b) tan — r = {s - c) tan —

low
ILLUSTRATIVE EXAMPLES

EXAMPLE 1
If areas of a circle and a square are equal, find the ratio of their perimeters.
SOLUTION Let r be the radius of the circle and the length of each side of the square be a. It is

ee
given that

F
Fr
r 1
nr ^ =a^ => -fur = a^ - =
a

Let and Pj be perimeters of the circle and that of a square. Then, for
ur
Pj=2Tcr and ?2=4a
s
Pi. 2nr n r K 1 7t
k

[Using (i)]
Yo

P2 4a a yfn 2
oo
eB

EXAMPLE 2 If the perimeters of a circle and an equilateral triangle are same. Find the ratiooftheirareas.
SOLUTION Let r be the radius of the circle and let the length of each side of equilateral triangle be a. It is
given that
r
ou
ad

Perimeter of circle = Perimeter of triangle


r T
Y

27rr = 3^i=>-= — ...(i)


a 2n
Re
nd

Let Ai and A2 be the areas of the circle and the triangle. Then,
Fi

n2
A1 _ nr
^ _ 4n f r 4n
—= X
3>/3
[Using (i)]
^2 V3~T"73 a
\aj V3 2n, n

Hence, Aj: A2 = SJS : n


EXAMPLES Lengths of the boundaries of two circular play grounds are 396 m and 528 m respectively. A
third circular play ground is to be made whose area is equal to the sum of the areas of tzvo given fields.
What is the length of its boundary?
SOLUTION The circumference C and area A of a circle are connected by the relation
A =
4n

LetC|, C2 be the circumference of the given fields and their areas be A j and A2 respectively. It is
given that - 396 m and C2 = 528 m.
5.59
QUANTIFICATION AND NUMERICAL APPLICATIONS

c? cl
Al = and A2=
471

Let the area and circumference of the third field be A3 and C3 respectively . Then,
ci
A3 - 471

It is given that
A3 =Aj + A2

ow
^:=^4-^=>C§=Cf+cf={
471 471 4ti;
396)^ + (528)^ = 435600 => C3 = 660 m
Hence, tlie boundary of the third field is 660 m long.
EXAMPLE 4 A road runs around a circular Shrubbery whose outer and inner boundaries are 500 m arid

e
412 m long. Find the area of the road.

re
SOLUTION Let A^ and A2 be the area of larger and smaller circles and A be the area of the road.

Frl
F
ou
sor
kf
oo
Y
B
re
oY

Then,
u

A — A-j ~ A2
ad

f'l c'2.
d

A =
4tc 47T
in
Re

A =
C? (Cl + C2) (Cl -C2) (500 + 412) (500 - 412) _ 912x88x7
F

4tu 22 4x 22
471 4x
7

A =912x7 = 6384 m^.


2
Hence, the area of the road is 6384 m .

EXAMPLE 5 The ratio of the circumference of two circles is 2 :3. What is the ratio of their areas?
SOLUTION Let Cl, C2 be the circumference and Ai, A2 be the areas of two circles. Then,
C^
Ai = and A2 = —
4tt ^ 47T
y2
Ai _ C^/4% Cl if -i
3) "9
C1 _

C2 I (given)
A2 C2^/4iz c 2>
5.60
APPLIED MATHEMATICS-XI

EXAMPLE 6 The area of a circle whose radius is 6 cm is trisected by two concentric circles. Find the radius
of the smallest circle.
SOLUTION Let the radii of the smallestand the middle circles be r and R respectively. In Fig. 5.33
it is given that

Area between circles C and = Area between circles Cj and C2 = Area of circles C2
7CX 6^ - = nR^ -nr
2
=nr
2

n(6'^-r'^)=k(R^-r^)=nr^
6^ R^=R^-r^=r^

ow
-r 2 and r2_,2^^2
2R^=6^+r^ and R^ =2r^

2(2r^)=6^+r^ => 3r“ = 36=> =12 => r = 2>/3

e
re
Fl
Hence, radius of the smallest circle is 2fS cm.

F
EXAMPLE 7 If the radius of a circle is increased by 20%, then by zuhat percent its area is increased?
SOLUTION Let the original radius of the circle be r.
ur
r
20 r
Increase in radius = 20% of r = — X r =- fo
100 5
ks
6r
New radius = r + - =
Yo
5 5
oo

A = Original area of the circle = nr^


eB

6r ^2 36 2
Ai = New area = n = — nr
K5 25
ur

Increase in area = 36 nr 2 -nr"^2 =—


-A =— H 2
ad
Yo

25 25

11 nr
2
Ml - A
d

25
Percentage increase in area = X100 % = — xloo %
Re
in

A nr
F

11
= — xlOO %=44%.
125

EXAMPLE 8 If the radius of a circle is decreased by 50%, find the percentage decrease in its area.
SOLUTION Let the original radius of the circle be r.
50 r
Decrease in radius (50% of r) = X r = —
100 2
r
Newradiu=r--
2 2

A = Original area = nr^, A^ = New area =


n nr
n
l2 4
QUANTIFiCATION AND NUMERICAL APPLICATIONS 5.61

2 nr^ 3nr^
Decrease in area =/4 - = Tir
4 4

(3
-^xlOO
1
Percentage decrease in area = xlOO 7o = %= -xlOO =757o
A nr 4

[●XAMPLE 9 The radius of a circle is so increased that its circumference is increased by 5%. Find the
percentage increase in its area.
SOLUTION Let the original radius of the circle be r. Then,
C = Original circumference = 2nr, A = Original area = nr^
Increase in circumference = 5% of 2nr = — x 2nr = — nr
100 10

w
Ttr 21
New circumference = 2nr + — = — nr

F lo
10 10

Let rj be the new radius. Then,


21 21r
2nr^ =

ee
nr=> ri =
10 20

Fr
2 21r^^ 441 2
Ai = New area = nr^ = 71 nr for
I 20 400

441 2 2 41 _„2
ur
Increase in area = - A = nr - nr Ttr
400 400
s
ook

41 2
Yo

Tir
Ml 400 %
Percentage increase in area = xlOO % =
^xlOO
eB

A Trr

41
our

41
ad

X 100 % = — % = 10.25 7o
400 4

EXAMPLE 11 Two circles touch externally. The sum of their areas is 130 sq. cm. and the distance betzveen
dY

their centres is 14 cm. Find the radii of the circles.


Re

SOLUTION If two circles touch externally, then the distance between their centres is equal to the
Fin

sum of their radii. Let the radii of the two circles be cm and r2 cm respectively. Let C, and C2 be
the centres of the given circles. Then,

C^C2 -rj +T2


14 = rj + r2 [v C1C2 =14 cm (given)]
r-i +r2 =14
2
It is given that the sum of the area of two circles is equal to 130n cm .
5.62 APPLIED MATHEMATICS-XI

2
nr.
1
+ nr.
=13071 ri^+r2^ =130 ...(ii)

Now, {r-^+Y2)^ =r-^ +r'^ +2r-^V2 => 14^ =130 + 2r|r;i => 196-130 = 2rf2 => r/2 = 33 ...(iii)
Now,

(ri-r2)^ =r-^+r2^~lr-^r2
=130-2x 33
[Using (ii) and (iii)]
(rj -r2)^ = 64 => -r2 = 8 ...(iii)

Solving (i) and (iv), we get rj =11 cm and ?2 = 3 cm.


Hence, the radii of the two circles are 11 cm and 3 cm.

LXAMPLE11 Two circles touch internally. The sum of their areas is 116 cm^ and distance between their
centres is 6 cm. Find the radii of the circles.

w
SOLUTION Let R and r be the radii of the circles having centres at O and O' respectively. It is
given that the sum of the areas is 116 cm^and the distance between the centres is 6 cm.

Flo
ee
Fr
r

B A

for
ur
k s
Yo
oo

Now, Sum of areas = 116ti cm


eB

+ Tur^ =116jr => R^+r^=116


Distance between the centres = 6 cm
r
ou
ad

00' = 6 cm => R-r = 6 ...(ii)


Now, (R+r)2+(R-r)2=2(R^+r2)
Y

(R+r)^ + 36 = 2xll6
Re
nd

(R+r)^=(2xll6-36)=196
Fi

R+r=14 ...(iii)
Solving (ii) and (iii), we get: R = 10 and r = 4.
Hence, radii of the given circles are 10 cm and 4 cm respectively.
EXAMPLE 13 In a circle with centre O and radius 5 cm, AB is a chord of length 5-J3. Find the area of
sector AOB.

SOLUTION It is given that v4S =5^3 cm.

5^/3
AL=BL = cm
2

Let ZAOB = 20. Then, ZAOL = ZBOL = 0.


In AOL4, we have
5V3
= _2_ = y[3
AL
sin 0 =
O.A 5 2
Fig. 5.36
5.63
QUANTIFICATION AND NUMERICAL APPLICATIONS

0 = 60° => ZAOB= 120°


120 2 2 25 Tl cm
_._2
Area of sector AOB = X 7TX 5 cm
\ 360
- 3

liXAMPLI: 14 An elastic belt is placed round the rim of a pulley of radius 5 cm. One point on the belt is
pulled directly away from the centre O of the pulley until it is at P, 10 cm from O. Find the length of the
belt that is in contact with the rim of the pulley. Also, find the shaded area.
SOLUTION In Figure 5.37, let ZAOP = ZBOP = 0. Clearly, portion AB of the belt is not in contact
with the rim of itre pulley in right triangle OAP, we have

5^/3 cm

low
5 cm

0
o
® 10 cm Q

S'/S cm

ee
5 cm
rF
Fr
B

Fig. 5.37

OA
for
u
cosO = e = 60°=> ZAOB=20=12O°
OP 10 2
ks
0
Yo

120 1071
Arc AB = X 271x5 cm = Using: / = X 2nr
o

cm
360 3 360
Bo

Let / be the length of the belt that is in contact with the rim of the pulley. Then,
re

/
IOti 20ti
/= Circumference of the rim - Length of arc AB = 27tx5-
cm = cm
^ 3
ou
ad

Now,
Y

120 2 2 2571 0 2
Area of sector OAQB = X TlX 5 cm cm Using: Area = 360
X Kr
360 3
nd
Re

Applying Pythagoras theorem in A OAP, we obtain


Fi

OP^=OA^ + AP^ => AP=-JoP^^^^ = Vi00^=5V3 cm

fl
Area of quadrilateral OAPB = 2 (Area of AOAP) = 2 x V —2 x OA x AP

= 5x5>/3cm^ =25^^3 cm^


Hence, Shaded area = Area of quadrilateral OAPB - Area of sector OAQB.
= 25>/3- 15tz^
3 )
cm 2 ^(3^-j,)cm^
3

EXAMPLt 15 Find the area of the segment of a circle, given that the angle of the sector is 120" and the
radius of the circle is 21 cm. (Take = 22/7)
SOLUTION The area A of a minor segment of a circle of radius r and the corresponding sector
angle 0 (in degrees) is given by
5.64
APPLIED MATHEMATICS>XI

K 0
/4 = X 0-sin
360 2

w
Here, r = 21 cm and = 120°.

F lo
Area of the segment =

ee
22 120
— X

Fr
7

22 1 V3
for
ur
21
ks
Yo
oo

462-
= ^(88-2lV3) cm^
eB

I-XAMI’LE ife ABCD is afield in the shape of a trapezium. AB\\ DC and ZABC = 90°, ZDAB = 60°
Four sectors are formed with centres A, B, C and D (See Fig. 5.S9). The radius of each sector is 17.5 m.
r
ou

Find the
ad

(i) total area of the four sectors,


Y

(ii) area of remaining portion given that AB =75 m and CD =50 m,


SOLUTION In Fig. 5.39, AB|| DC flHd ZABC =90°. Therefore ZBCD =90°. Also, ZBAD =60°.
nd
Re

ZCDA =180°-60°= 120°


[Co-interior angles]
Fi

L
A
B

60°

90° 120°

C
D

Fig. 5.39

(i) Let A be the total area of the four sectors. Then,


A = Area of sector at A + Area of sector at 6 + Area of sector at C + Area of sector at D.
QUANTIFICATION AND NUMERICAL APPLICATIONS 5.65

60 90 90 120
=> A = X 7IX (17.5)^ + xTtx(17.5)^ + X 71X (17.5)^ + X 7tx(17.5)2
360 360 360 360

=> A =
n 1 1 n
xkx(175)^ m
2
= 7TX
35 f 2 22m
35 35 m ^=962.5
U 4 4 3 2 J 7 2 2

(ii) Let DL be perpendicular drawn from D on AB. Then,


AL=AB-BL= AB-CD = {75 -50) m - 25 m
In AALD, we have
DL DL
tan 60® = V3 = => DL = 25^3 m
AL 25

w
Area of trapezium ABCD = ^ ^ ^ 2SV3m^
= 1562.5 X 1.732 = 2706.25

Flo
Let A be the area of the remaining portion. Then,

e
A = Area of trapezium ABCD ~ Area of 4 sectors = (2706.25 - 962.5) = 1743.75 m^

re
EXAMPLE 37 In Fig. 5.40, a crescent is formed by two circles xvhich touch at A. C is the centre of the

F
larger circle. The width of the crescent at BD is 9 cm and at EF it is 5 cm. Find (i) the radii of two circles
ur
(ii) the area of the shaded region.
or
SOLUTION (i) Let the radii of the larger and smaller circles be R and r respectively. Then,
sf
BD = 9cm 2R -2r =9 ^ R - r= 45
k
Yo

Join A£ and DE. Let ZCAE = 0. Then, ZAEC = 90° - 0.


oo

Now, ZAED = 90°=> ZAEC + ZDEC = 90°=> ZDEC = 90®-(90°-e) = 6.


B

Thus, in A's ACE and DCE, we have


re

ZCAE = ZCED=Q and ZAC£ = ZECD =90®


u
ad
Yo
d
Re
in

B A
D C
F

.5.40

So, by AA similarity criterion, we obtain


AAC£~ AECD

AC _ CE
EC ~CD
AC CF-EF

CF-EF BC-BD

R ^R-5 => K(R-9)=(R-5)^=>0 = -R+25=>J?=25cm


R-5~ R-9
5.66 APPLIED MATHEMATICS-XI

Substituting the value of K in (i), we get


25-r = 45=> r = 205 cm

Hence, the radii of two circles are r = 20.5 cm and i? = 25 cm.

(ii) Let A be the area of the shaded region. Then,


A=nR^ - nr ^ =k{R^ -r^)=Tz(R+r) {R -r)

A = 3.14 (25 + 205) (25 - 205) cm^ = 3.14 x 455 x 45 cm^ = 642.915 cm^
liXAMFLE- 38 In Fig. 5.41, three circles of radius 2 cm touch one another externally. These circle are
circumscribed by a circle of radius R cm. Find the value of R and the area of the shaded region in terms of
SOLUTION Clearly, A ABC is an equilateral triangle of side 4 cm.

ow
e
Fl
re
F
ur
or
sf
In ABDO, we have
k
Yo

BD
oo

cos ZOBD =
OB
B

cos 30°= —
[v ZOBD = 30°]
e

OB
ur

^/3
— => ob=4
ad

2 OB 73
Yo

4 'i
OP=OB + BP=> R = -i= + 2 cm
73
d
Re
in

Let A be the area of the shaded region. Then,


A = Area of the larger circle of radius R - 3 x Area of a smaller circle of radius 2 cm
F

+ 3 (Area of a sector of angle 60° in a circle of radius 2 cm)


- {Area of AABC - 3 (Area of sector of angle 60° in a circle of radius 2 cm)}
A = Area of the larger circle of radius R - 3 x Area of a smaller circle of radius 2 cm
+ 6 X Area of a sector of angle 60° in a circle of radius 2 cm - Area of A ABC
n2
( 4 60 73
A = -^ TU ^+ 2 -3xnx2^+6x X71x2^ - X 4^ > cm
2

V3 360 4

16 16 2
A =>! TC — + 4 + -12t:+47:-473 cm
^ 3 73

A. =-^I 71 f4
473| cm^=|y(473+l)-4^^| cm^
16
- +—
3 73
5.67
QUANTIFICATION AND NUMERICAL APPLICATIONS

EXAMPLE 20 With vertices A, B and Cofa triangle ABC as centres, arcs are drawn zvith radii 5 cm each
as shown in Fig. 5.42. IfAB = Ucm,BC = 48 cm and CA = 50 cm, then find the area of the shaded region.
(Use n = 3.14).

SOLUTION InA>4BC, wehave

w
= BC = 48 cm, & =Ci4 =50 cm and c = AB=14cm

F lo
Let s be the semi-perimeter of Ai4BC. Then,
a +b +c 48+50 + 14
s = =56 cm
2 2

ee
Let A be the area of A ABC. Then, by Heron's formula

Fr
A= (s - fl) (s -b) (s - c) = ^56 X 8 X 6 X 42 cm^ = 336 cm^ for
Let A^, A2 and A^ be the areas of sectors with sector angles A, B and C respectively and sector
ur
radius r = 5 cm. Then,
A
s

A 2 ^ 2 2 2
ook

^1 = X nr X Kx 5 cm = X 25k cm
Yo

360 360 360


eB

A 2 B x7ix5 2 cm 2 = B 2
A2 = X 7tr X 25tu cm
360 360 360

C 2 C X 71x5 0 cm 7 = C X 25ti cm 2
our

A3 -
ad

X Ttr
360 360 360

A B C 2
X 257T + X 25ti + X 25tc cm
Aj + A2 + A3 =
Y

360 360 360


Re
nd

25;: 2
= (A + B + C)x cm
360
Fi

180
x257icm
2 25
= — Tccm
2 25 X 3.14 cm 2 = 39.25 cm^
360 2 2

Let A be the area of the shaded region. Then,


A = Area of AASC - (A^ + A2 + A3) = (336 - 39.25) cm^ = 2296.75 cm^
REMARK The above solution is the general solution. In this case, is a right triangle right angled at B.
So, its area can also be computed as follows:
A = - (BC X AB) = - X 48 X14 cm^ = 336 cm^
2 2

EXERCISE 5.9

1. A plot is in the form of a rectangle ABCD having semi-circle on BC as shown in Fig. 5.43. If
AB = 60 m and BC = 28 m, find the area of the plot.
5.68
APPLIED MATHEMATICS-Xl

D C

28 m

A 60 m B

Fig. 5.43

2.
A play ground has the shape of a rectangle, with two semi-circles on its smaller sides as
diameters, added to its outside. If the sides of the rectangle are 36 m and 24.5 m, find the
area of the playground. (Take n = Tip).
3. Find the area of the circle in which a square of area 64 cm2 is inscribed. [Use n = 3.14]
4. A rectangular piece is 20 m long and 15 m wide. From its four comers, quadrants of radii

w
3.5 m have been cut. Find the area of the remaining part.
D.
A square water tank has its side equal to 40 m. There are four semi-circular grassy plots all

F lo
round it. Find the cost of turfing the plot at ? 1.25 per square metre (Take n = 3.14 ).
(). A rectangular park is 100 m by 50 m. It is surrounded by semi-circular flower beds all

ee
round. Find the cost of levelling the semi-circular flower beds at 60 paise per square metre

Fr
(Use 7t = 3.14).
/.
The inside perimeter of a running track (shown in Fig. 5.44) is 400 m. The length of each of
for
the straight portion is 90 m and the ends are semi-circles. If the track is everywhere 14 m
ur
wide, find the area of the track. Also, find the length of the outer running track.
oks
Yo
o
eB

■►90 m
our
ad

Fig. 5.44
Y

In Fig. 5.45, ABCD is a rectangle with AB = 14 cm and BC = 7 cm. Taking DC, BC and AD as
Re

diameters, three semi-circles are drawn as shown in the figure. Find the area of the shaded
nd

region.
Fi

D C

A B

Fig. 5.45

In Fig. 5.46, ABCD is a rectangle, having AB = 20 cm and BC = 14 cm. Two sectors of 180°
have been cut off. Calculate:
(i) the area of the shaded region,
(ii) the length of the boundary of the shaded region
QUANTIFICATION AND NUMERICAL APPLICATIONS 5.69

D C A D

22 cm

B C
A B

Fig. 5.46 Fig. 5.47

10. In Fig. 5.47, the square ABCD is divided into five equal parts, all having same area. The
central part is circular and the lines AE, GC, BF and HD lie along the diagonals AC and BD
of the square. If AB = 22 cm, find:
(i) the circumference of the central part, (ii) the perimeter of the part ABEF.
n. In Fig. 5.48, OACB is a quadrant of a circle with centre O and radius 3.5 cm. If OD = 2 cm,
find the area of the (i) quadrant OACB (ii) shaded region.
B

F low for Fre


0

3.5 cm

Fig. 5.48
eBo ks
Your

12. In Fig. 5.49, an equilateral triangle ABC of side 6 cm has been inscribed in a circle. Find the
area of the shaded region. (Take n = 3.14).
ad

13. A circular field has a perimeter of 650 m. A square plot having its vertices on the
our

circumference of the field is marked in the field. Calculate the area of the square plot.
14. ABCDEF is a regular hexagon with centre O (Fig. 5.50). If the area of triangle OAB is 9 cm^
Re

find the area of: (i) the hexagon and (ii) the circle in which the haxagon is incribed.

E D
Find Y

O
F C

A B

Fig. 5.50

15. Four equal circles, each of radius 5 cm, touch each other as shown in Fig. 5.51. Find the area
included between them (Take rt = 3.14).
16. Four equal circles, each of radius rt, touch each other. Find the area between them
(Take n = 22/7) .
5.70 APPLIED MATHEMATICS-XI

i7. The diameter of a coin is 1 cm (Fig. 5.52). If four such coins be placed on a table so that the
rim of each touches that of the other two, find the area of the shaded region (Take k =
3.1416).
C

A B
O

w
n

F lo
Fig. 5.53

18. In Fig. 5.53, AB and CD are two diameters of a circle perpendicular to each other and OD is
the diameterof the smaller circle. If OA = 7 cm, find the area of the shaded region.

ee
In Fig. 5.54, two circles with centres A and B touch each other at the point C. If AC = 8 cm

Fr
and AB = 3 cm, find the area of the shaded region.
for
ur
s
ook
Yo
eB
r
ad
ou
Y
Re
nd

'In Fig. 5.55 ABCD is a square of side 2a. Find the ratio between
(i) the circumferences
Fi

(ii) the areas of the incircle and the circum-circle of the square.
QUANTIFICATION AND NUMERICAL APPLICATIONS 5.71

!. In Fig. 5.56, there are three semicircles. A, B and C having diameter 3 cm each, and another
semicircle £ having a circle D with diameter 4.5 cm are shown. Calculate:
(i) the area of the shaded region
(ii) the cost of painting the shaded region at the rate of 25 paise per cm^, to the nearest
rupee.

ow
e
re
3 cm

Frl
Fig.5,56

F
22. In Fig. 5.57, ABC is a right-angled triangle, ZB = 90°, AB = 28 cm and BC = 21 cm. With AC
as diameter a semicircle is drawn and with BC as radius a quarter circle is drawn. Find the
ou
or
area of the shaded region correct to two decimal places.
kfs
oo
Y
eB
ur
oY
ad

Fig. 5.57
d

23. In Fig. 5.58,0 is the centre of a circular arc and AOB is a straight line. Find the perimeter and
in

the area of the shaded region correct to one decimal place. (Take n = 3.14)
Re

c
F

A O B

Fig. 5.58

. In Fig. 5.59, AB = 36 cm and M is mid-point of AB. Semi-circles are drawn on AB, AM and
^<1

MB as diameters. A circle with centre C touches all the three circles. Find the area of the
shaded region.
5.72 APPLIED MATHEMATICS-XI

ow
25. In Fig. 5.60,/4BC is a right angled triangleinwhichZA =90°, AB = 21 cm and AC =28 cm.
Semi-circles are described on AB, BC and AC as diameters. Find the area of the shaded
region.

e
re
Frl
F
ou
or
kfs
oo
Y
eB

If'- Figure 5.61, shows the cross-section of railway tunnel. The radius OA of the circular part is
ur

2 m. If Z/40B = 90° calculate:


oY

(i) the height of the tunnel (ii) the perimeter of the cross-section
ad

(iii) the area of the cross-section.


d
in
Re
F

27.
Figure 5.62., shows a kite in which BCD is the shape of a quadrant of a circle of radius 42 cm.
ABCD is a square and A CEF is an isosceles right angled triangle whose equal sides are 6 cm
long. Find the area of the shaded region.
28. From a thin metallic piece, in the shape of a trapezium ABCD, in which AB\\CD and
ABCD =90° a quarter circle BEFC is removed (see Fig. 5.63). Given AB = BC = 3.5 cm and
DE = 2 cm, calculate the area of the remaining piece of the metal sheet.
QUANTIFICATION AND NUMERICAL APPLICATIONS 5.73

4 B

C .● f ■ F

3.5 cm
I

n 2 cm F

Fig. 5.63

29. In Fig. 5.64, ABC is an equilateral triangle of side 8 cm. A, B and C are the centres of circular

ow
arcs of radius 4 cm. Find the area of the shaded region correct upto 2 decimal places.
(Take k = 3.142 and VI = 1.732).

e
re
rFl
F
r
ou
fo
ks
oo
Y
eB

ANSWERS

1. 1988 m^ 2. 1353.625 m2 3. 100.48 cm2 4. 261.5 m2


r

5. ^3140 6. ^5887.50 7. 6216 ml 488 m 8. 59.5 cm2


ou
ad
Y

9. (i) 126 cm^ (ii) 84 cm 10. (i) 34.88 cm (ii) 50.64 cm

n. (i) 9.625 cm^ (ii) 6.482 cm^ 12. 22.126 cm2 13. 21387 m2
d

6 .2
14. (i) 54 cm (ii) 65.23 cm^ 15. 21.5 cm2 16. -a
Re
in

7
17. 0.2146 cm2 18. 115.5 cm2 19. 122.57 cm2 20. (i) 1: V2 (ii) 1: 2
F

21. (i) 12.375 cm2 ^3 22. 428.75 cm2 23. 59.4 cm, 61.1 cm2
y
24. 45n cm 25. 294 cm2 (i) (2 + V2)m (ii) (3ti + 2V2) m (iii) (3Tt+2)m^
27. 1404 cm^ 28. 6.125 cm^ 29. 2.576 cm^

5.8 MENSURATION OF SOLIDS

In this section, we will discuss problems on surface areas and volumes of various solids.
5.8.1 CUBE AND CUBOID

CUBOID A cuboid is a solid bounded b\j six rectangular plane surfaces.


Fig. 5.65, represents a cuboid formed by six rectangular surfaces ABCD, A', B'C'D';
ABB' A', DCC D'; ADD' A' and BCC B'. These rectangular surfaces are known as faces of
the cuboid.
5.74 APPLIED MATHEMATICS-XI

D' C'

a' b'
h

I
D
b

A B

Fig. 5.65
ABCD, A' B'C'D'; ABB' A', DCCD'; ADD'A' BCC B' are pairs of opposite faces of the cuboid.
Faces ABCD, ADD' A' etc are known as adjacent faces.

w
Let I, b and h denote respectively the length, breadth and height of the cuboid. Then,

F lo
(i) Total surface area = 2 + W? +/h) square units,
(ii) Volume of the cuboid = Area of the base x Height
= Length x Breadth x height

ee
Fr
= Ibh cubic units

(iii) Diagonal = units,


for
ur
(iv) Areas of three adjacent face are : Aj = lb, A2 = bh, A3 = Ih
Total surface area = 2 (A;| + A2 + A3) square units.
oks
Yo

Volume of the cuboid ss.^A^'A^"^ cubic units,


o
eB

(v) Areas of four walls of a room = Areas of four vertical faces


= lh + Ih +bh +bh = 2{l + b) h square units.
our

CUBE A cuboid whose length, breadth and height are equal is called a cube.
ad

If the length of each edge of a cube is 'a' units, then


(i) Total surface area of the cube = 6 (Area of a face) = 6a^ square units
Y

(ii) Volume of the cube = a^ cubic units


Re
nd

(iii) Diagonal of the cube = VSa units


Fi

We also observe that:

Volume of the cube =


Diagonal
V3

Surface area of the cube = 2 (Diagonal)

ILLUSTRATIVE EXAMPLES

I \AMPLt 1 IfVis the vohimc of a cuboid oflet^gth = a, breadth = b, height = c and S is its surface area,
then prove that
1 1'
V~ s[a'^ b^ c.
SOLUTION We have, V = abc and S = 2{ab +bc + ca)
QUANTIFICATION AND NUMERICAL APPLICATIONS 5.75

Now, 2fl 1 n _ 2 Tfcc + + ab' _2(ab +bc + ca)


ca
1 S 1 1 1
b cj S abc S abc S abc abc V

LXAMPLE 2 If the areas of three adjacent faces of a cuboid are 35 cm^, 28 cm^ and 20 cm^, then ifnd he
volume of the cuboid.
SOLUTION We have, = 35 cm^, A2 = 28 cm^ and A^ - 20 cm^
Let V be the volume of the cuboid . Then,

V=fA-^ A2 A3
y = ^35 X 28x20 = ^5 X 7 X 2^ X 7 X 2^ X 5 = 2^ x 5 x 7 = 140 cm^
EXAMPLE 3 A 4 cm cube is cut into 1 cm cubes. Findthe percentage increase in superficial area.

ow
SOLUTION = Volume of 4 cm cube = 4^ cm^ = 64 cm^

V2 = Volume of 1 cm cube = 1 cm'^ = 1 cm’^


64
Number of 1 cm cubes = — =64

e
1

Fl
re
S| = Surface area of 64 small 1 cm cubes = 64 (6 x 1^) = 384 cm^

F
S2 = Surface area of 4 cm cube = 6x4 cm = 96 cm
ur
r
5i-52 fo r 384 - 96
Percentage increase in surface area = xlOO % = xlOO %
S2 96
ks
Yo
= (3x100)% = 300%
oo

Hence, the percentage increase in superfacial area = 300%.


B

EXAMPLE 4 Three cubes each of side 5 cm are joined end to end. Find the surface area of the resulting
cuboid.
re

SOLUTION The dimensions of the cuboid so formed are as under:


u
ad
Yo
d
Re
in

U
F

<0

5 cm 5 cm 5 cm

Fig. 5.66

/ = Length = 15 cm, b = Breadth = 5 cm, and h = Height = 5 cm


2
Surface area of the cuboid = 2 (15 x5 + 5x5 + 15x5) cm

Surface area of the cuboid = 2 (75 + 25 + 75) cm^ = 350 cm^


EXAMPLE 5 Two cubes each of volume 64 cm^ are joined end to end. Find the surface area and volume of
the resulting cuboid.
SOLUTION Let the length of each edge of the cube of volume 64 cm^ be x cm. Then,
Volume = 64 cm^ => = 64 => =4^ => x = 4 cm
5.76 APPLIED MATHEMATICS-XI

rw
i
4 cm
/
f / 4 cm

4 cm 4 cm
/

e
Fig, 5.67

r
The dimensions of the cuboid so formed are;

luo
L - Length = (4 + 4) cm = 8 cm, b ~ Breadth - 4 cm and, h = Height = 4 cm

F
Surface area of the cuboid = 2 {Ib+bh + Ih) = 2(8x4 + 4x4 + 8x4) cm^ =160 cm^

oF
Volume of the cuboid= Wh =8x4x4 cm^ =128 cm^

rs
EXAMPLE 6 The sum of the length, breadth and depth of a cuboid is 19 cm and the length of its diagonal is
5y/5 cm, find the surface area of the cuboid.

ok
SOLUTION Let a, b and c be the length, breadth and height of the cuboid. It is given that

fo
a + b -h c =19 and +b^ + c^ =5f5
o
Y
(a + b + c)^ =19^ and a^ +b^ + =125
Y
rB

=> a^ +b^ + c^ +2 {ab + bc + ca) = 361 and a'^ + b'^ + = 125

125 + 2 {ab + be+ ca) = 361


ue

2 (ab +bc = ca) = 236


d

Surface area = 236 cm


no
ad

EXAMPLE 7 The sum of the perimeters of the six faces of a cuboid is 72 cm and the total surface area of the
cuboid is 16 cm^. Find the length of the longest possible that canbe kept inside the cuboid.
i

SOLUTION Let the length, breadth and height of the cuboid be a, b and c respectively. Then,
Re
F

Sum of the perimeters of six faces = 2 {2{a+ b) + 2{b + c) + 2{c + a)] = 8{a+ b + c)
It is given that
8{a + b + c)=72 and 2 (ab + be + ca) =16 ^ a + b + c = 9 and ab + be + ca = 8
Now,

(a + b + c)^ = (a^ +b^ + c^) + 2 (ab +bc + ca)


=> 9^ =(a^ +b^ +c^) + 2x8
a^+b^ + c^ =81-16 = 65

■Ja^ +b^ + =V^ = 8.05 cm

Hence, required length = 8.05 cm.


EXAMPLE a If the areas of three adjacent faces of a reef angular block are in the ratio 2:3:4and its volume
is 9000 cm^, find the lengths of the edges of the box.
QUANTIFICATION AND NUMERICAL APPLICATIONS 5.77

SOLUTION Let the lengths of edges be a cm, b cm and c cm. It is given that the areas of three
adjacent faces are in the ratio 2 : 3 : 4. So, let the areas of adjacent faces be 2.v, 3.v and 4.r
respectively.
i.e.
A-i = 2.V, A2 = 3.V, and A3 = 4.v
be - 2.V, ca - 3x and ab = 4a:
(I>c) (crt) (ab) = 2.V X 3a: X 4.v
(7^ =24.v^

(abc)^ = 24x^
(9000)^ =24.y^ [●.● abc = 9000 (given)]
=9000x 375

w
=3^xl0^x 3x5^ =(3x5xl0)^=> y = 3x5x10=150
be - 300, ca = 450 and ab = 600

F lo
abc , abc abc 9000 9000 9000
Now, a = — , b - / c = => a = = 30, b = = 20, c = = 15
be ca ab 300 450 600

ee
Hence, the lengths of edges are 30 cm, 20 cm and 15 cm.

Fr
5.8.2 RIGHT CIRCULAR CYLINDER
for
RIGHT CIRCULAR CYLINDER For a right circular cylinder of base radius r and height (or length) h, we
ur
have
r O'
(i) Area of each end = Area of base = 4' B'
s
ook
Yo

(ii) Curved surface area = 2nrh


= 2nr-xh
eB

= Perimeter of the base x Height h

(iii) Total surface area = Curved surface area + Area of circular ends
r

= 2nrh + 2xr'^
ou
ad

r
= 2nr{r+h) /I B
O
Y

(iv) Volume = nr^h


Fig. 5.68
= Area of the base x Height
Re
nd

RIGHT CIRCULAR HOLLOW CYLINDER Let R and r be the external and interna! radii of a hollow cylinder
Fi

of height h. Then,
(i) Area of each end = k{R^-r^)
(ii) Curz’cd surface area of hollow cylinder = External surface area + Internal surface area R
= 2kR h + 2nrh

= 2nh (R+r)
(iii) Total surface are = 2nRh + 2itrh + 2 (nR^-nr^)
= 2Kh{R+r) + 2K(R+r)iR-r) h
= 2k{R +r)(R+h-r)
(iv) Volume of material = External volume - Internal volume
= nR^h-Kr'^h
= nh{R^-r-) R

Fig. 5.69
5.78 APPLIED MATHEMATICS-Xl

5.8.3 RIGHT CIRCULAR CONE

RIGHT CIRCULAR CONE for <7 right circular cone of height h, slant height I and radius of base r, we have
(i) i^=r^ + h^
(ii) Curved surface area = Kr I sq. units
(iii) Total surface area = Curved surface area + Area of the base
2
= nrl + %r

= nr (l + r) sq. units.
(iv) Volume = —
— nr^h
3

1
= — {Area of the base) x Height
3

5.8.4 SPHERE AND SPHERICAL SHELL

F low
SPHERE For a sphere cf radius r, we have
(i) Surface area = 4lkt
4 2
(ii) Volume = —nr
3

For a hemisphere of radius r, we have


re
for F
(i) Surface area = Inr"^
(ii) Total surface area = 2nr^ + nr"^ = 3;rr^
2 2
(iii) Volume = —nr
3
Your
s
eBook

SPHERICAL SHELL If Rand rare respectively the outer and inner radii of a spherical shell, then
ad
our
Re
Y
Find

(i) Outer surface area = ^nK^


4
(ii) Volume of material = -n(R
3

ILLUSTRATIVE EXAMPLES

EXAMPLE I The total surface area ofa cylinder is-times its curved surface area.If the height of the
cylinder is 7 cm, find its volume.
SOLUTION Let r be the radius of the base and h be the height of the cylinder. It is given that

Total surface area = ^ (Curved surface area)


QUANTIFICATION AND NUMERICAL APPLICATIONS 5.79

2nrh + 2iir^ -
I {2nrh)
2nrh + 2nr^ =5nrh
2nr'^ = Snrh
2r = 3h

2r=21 [V /;=7]
21
=> r -
2

Let V be the volume of the cylinder. Then,


V = nrh

low
22 21 ^2
y = “ X X 7 cm^ = 24255 cm^
7 V 2

EXAMPLE 2 The difference between outside and inside surfaces of a cylindrical metallic pipe 14 cm long is
44 cm-. If the pipe is made of 99 cubic centimeters of metal, find the outer and inner radii of the pipe.

ee
SOLUTION Let R cm and r cm be the external and internal radii of the metallic pipe.

F
Fr
We have, h = length of the pipe = 14 cm.
Now,
for
ur
Outside surface area - Inside surface area = 44 cm^

2nRh-2Krh=44
ks

2n{R-r)h=44
Yo
oo

y(R-r)xU = 44
2x
eB

1
R~r=- ...(i)
2
r
ou
ad

It is given that the volume of the metal used = 99 cubic centimeters.


Y

External volume - Internal volume = 99 cubic centimeters

KR^-nr^h =99
Re
nd

K(R^-r^) h=99
Fi

yx(R+r)(R-r)xl4=99
22 1
— x(K+r)x-ixl4=99 [Using (i)]
7 2

99
R+r = —
22

9
R+r = - ...(ii)
2

Solving (i) and (ii), we get R = 2.5 and r = 2.


Hence, outer radius = 2.5 cm, and inner radius = 2 cm.

EXAMPLE 3 A well with 10 m inside diameter is dug 14 m deep. Earth taken out of it is spread all around
to a zuidth of 5 m to form an embankment. Find the height of embankment.
5.80 APPLIED MATHEMATICS-XI

SOLUTION We have,

Volume of the earth dug out = 7tr^/( = — x5x5xl4m^ =1100

Area of the embankment (shaded region) = n (R'^ -r^)

Area of the embankment (shaded region) = n (10 -5^) m'^ = —


7
x 75 m'^

w
F lo
ee
Let h be the height of the embankment. Then,

Fr
Volume of the hollow cylinder = Volume of earth dugout
Tt(R^-r^) /?=1100 for
ur
22 1100x7 14
=> (100-25)x/2 =1100 => h = m = — m = 4.66 m
7 22x75 3
s
ook

Volume of the earth dugout 1100 7x1100


Yo

Height of embankment = 22
= 4.66m
Area of the embankment 22x75
x75
eB

ALITEK The earth dugout when spread around the well forms a hollow cylinder of external
and internal radii R = 10 m and r = 5 m respectively.
r
ad
ou

EXAMPLE 4 Ifh, c, V are respectively the height, the curved surface and the volume of a cone, prove that
3tzVIi^ -Ch^ + 9V^ =0
Y

SOLUTION Let r and / denote respectively the radius of the base and slant height of the cone.
Re
nd

Then,
Fi

/ = V =—7ir^/7 and C=nrl


3

3kVIi^ -CV + 9V^ = 3jix-nr^


3
x h^-{nrl)^ + 9f-
l3
nr^h

- K^r^h'^{r^ + h^) + Tpr^lP" [v +

= -nW -kW ^0

EXAMPLES A coneofheight 24 cm has a curved surface area 550 cm^. Find itsi’olume. (Take n = 22/7).
SOLUTION Let r cm be the radius of the base and / cm the slant height. Then,

/2=r2+24^ [Using: l-=r^+h^]


QUANTIFICATION AND NUMERICAL APPLICATIONS 5.81

/2=r^+576=>/=Vr^+576
Now,

Curved surface area =550 cm'^

jir/=550
22
— X rx Vr^+576 =550
7

rVr^+576 =550x-
22

ow
rVr^+576 =25x7
r^{r^+576) =(25 x 7)^
r^+576r^-(25^x7^) = 0

e
re
r^+576r^-(625x49) =0

rFl
=>

F
r^+625r^-49r2 625x49=0

r^(r^ + 625)-49(r^ + 625) = 0 => + 625) {r^-49) - 0 => r^-49 = 0 => r=7

or
ou
22
Volume = — nr'^h = — x x7x7x24 cm^ =1232 cm^
ksf
3 3 7

EXAM1’LC6 A c\/Iinder is within the cube touching all the vertical faces. A cone is inside the cylinder. If
oo

their heights are same with the same base, find the ratio of their volumes.
Y

SOLUTION Let the length of each edge of the cube be a units.


B

Then,
re

= Volume of the cube = a^ cubic units


oYu

Since a cylinder is within the cube and it touches all the vertical
ad

faces of the cube.

r = Radius of the base of the cylinder = ~,


d
in
Re

h = Height of the cylinder = a


V2 = Volume of the cylinder = nr^h
F

wV-) = —
22 X — xa cubic units
^74
11
Fig. 5.74
V2 - — cubic units
14

A cone is drawn inside the cylinder such that it has the same base and same height.
1
V3 = Volume of the cone = nrh
3

„ 1 22 fa^^
- 11 a
xa cubic units ^ Vo = — 3 U-
cubic units
■^3 7 1:2) ^ 42

V^:V2:1^3 =a^:—a^:—a^
14 42
=42: 33:11
5.82 APPLIED MATHEMATICS-XI

EXAMPLE 7 A sphere, a ci/lwder and a cone are of the same radius and same height. Find the ratio of their
curved surfaces.
SOLUTION Let r be the common radius of a sphere, a cone and a cylinder. Then, Height of the
cone = Height of the cylinder = Height of the sphere = 2r
Let / be the slant height of the cone. Then,
/ = ^|r^ +11^ ^ +4r^ = yfSr
Now, Sj = Curved surface area of sphere = 47ir^
2
$2 = Curved surface area of cylinder = 2Tur x 2r = 47tr
and. S3 = Curved surface area of cone = Krl = Kr x ^/5r =

w
Si: S2 : S3 = 4nr“ : 47i:r^ : JSnr^ = 4: 4 : VS
EXAMPLE 8 The volume of the tzoo spheres are in the ratio 64 :17. Find the difference of their surface

F lo
areas, if the sum of their radii is 7.
SOLUTION Let the radii of two spheres be r-^ cm and r2 cm respectively. Let the volumes of two

ee
spheres be and V2 respectively. Then,
Vl _ 64

Fr
27

4 3 for
ur
64 43 4^^ => ^
-
r, = —4
- => ri
4
= —rn ...(i)
4 3 27 ,.3
'2
,3
3
^ ^2j 3 ^2 3 ^ 3 ^
3 ^^2
s
ok
Yo

4 7
But, rj+r2=7 => -r2+^2 =7 =>-r2 =7 => r2 == 7x-7 = 3 cm
o
eB

4 X o3 = 4, cm
ri = —
^ 3
r
ou
ad

Let Si and S2 be the surface areas of two spheres. Then,


Sj =47tr-i^ = 4tix 4x4 = 64ti cm^ and, S2 - 471^2 = 4;i x 3 x 3 = 36n cm^
Y

Sj -S2 = 6471- 36rt = 28tt cm^ = 28 x — cm^ = 88 cm^


Re
nd

EXAMPLE 9 A sphere of diameter 6 cm is dropped in a right cirtular cylindrical vessel partly filled zvith
Fi

zuater. The diameter of the cylindrical vessel is 12 cm. If the sphere is completely submerged in zvafer, by
hoiv much zoill the level of zuater rise in the cylindrical vessel?
SOLUTION We have,

Radius of the sphere = 3 cm


^ ^ 3
Volume of the sphere = — ti x (3) cm = 36ti cm^

Radius of the cylindrical vessel = 6 cm.


Suppose water level rises by h cm in the cylindrical vessel. Then,
2
Volume of the cylinder of height h cm and radius 6 cm = (71 x 6 > h) cm^ = 3671 h cm^
Clearly, volume of the water displaced by the sphere is equal to the volume of the sphere.
367i/i = 36t:=>/; =1 cm

Hence, water level rises by 1 cm.


QUANTIFICATION AND NUMERICAL APPLICATIONS 5.83

EXAMTLE10 Determine the ratio of the volume of a cube to that of a sphere zohich will exactly fit inside
the cube.

SOLUTION Let the radius of the sphere which fits exactly into a cube be r units. Then, Length of
each edge of the cube = 2r units
Let V] and V2 be the volumes of the cube and sphere respectively. Then,

Vi=(2r)^and V2=-Ttr^
3

V1 _ 8r^
~ = — ^ Vi:V2 = 6: K
Vo2 4-nr 3 71 ^ ^
3

EXAMPLE 11 Find the maximum volume of a cone that can be carved out of a solid hemisphere of radius r.

w
SOLUTION Clearly,
Radius of the base of cone = Radius of the hemisphere = r

F lo
and. Height of the cone = Radius of the hemisphere = r

ee
r
0
B
A

Fr
r
for
ur
s
ok
Yo

V
o

Fig. 5.75
eB

1 9 1
Volume of the cone = — rcr xr = —nr cubic units
3 3
r
ad
ou

2
EXAMPLE 12
The radius of a solid iron sphere is 8 cm. Eight rings of iron plate of external radius 6—cm
3
Y

and thickness 3 cm are made by melting this sphere. Find the internal diameter of each ring.
SOLUTION We have,
Re
nd

2048 3
Volume of solid iron sphere = — 7t x 8 cm ^ = K cm
Fi

^ 3 3
2 20
External radius of each iron ring = 6— cm = — cm
3 3

Let the internal radius of each ring be r cm. Since each ring forms a hollow cylindrical shell of
'1 20
external and internal radii
^ and r cm respectively and height 3 cm.
20 \2
Volume of each ring = 71 ● -r^ IX 3 cm ^
SJ

fm 400
Volume of 8 such rings = 8rc — X 3cm^ =24tc -r2 cm
3

I 9
Clearly, Volume of 8 rings = Volume of the sphere
5.84 APPLIED MATHEMATICS-XI

400 2048
24tc K
9 3

400 2 2048 1
-r TlX
9 3 247T

2 _400 256 _ 144 = 16


~ 9 9 ~ 9
r = 4 cm

Hence, internal radius of each ring is 4 cm.


EXAMPLE 13 1/ the diameter of cross-section of a zaire is decreased by 5% how much percent will the
length be increased so that the volume remains the same ?

ow
SOLUTION Let r be the radius of cross-section of wire and h be its length. Then,
Volume = nr^h
5 r
5% of diameter of cross-section = x2r = —

e
100 10

re
New diameter = 2r

Fl r

10
19r

10

F
19r
New radius =
ur
20

r
Let the new length be h-^.Then,
fo
ks
n2
19r
Yo
Volume = n h1
20
oo

From (i) and (ii), we obtain


eB

19r 361 400


nr^h = n h^ => h hi => fh = h
20 400 361
ur

400/1 39h
Increase in length = /jj - h =
ad

-h
Yo

361 361

hi - h 39h 3900
Percentage increase in length = xl00 = xl00 = = 10.8%
d

h h 361
Re
in

EXAMPLE 14 A well, whose diameter is 7m, has been dug 22.5 m deep and the earth dugout is used to
form an embankment around it. If the height of the embankment is 1.5 m, find the width of the
F

embankment.
7
SOLUTION We have, Radius of the "well = — m = 35 m and, Depth of the well = 22.5 cm

7 7 45 3
Volume of the earth dugout = ;r x (35)^ x 225 m^ = Kx — X — X — m
2 2 2
QUANTIFICATION AND NUMERICAL APPLICATIONS 5.85

Let the width of the embankment be r metres. Clearly, embankment forms a cylindrical shell
whose inner and outer radii are 3.5 m and (r + 3.5) m respectively and height 1.5 m.

Volume of the embankment - re | (r + 35)^ -{35)^ [xl5m^=Ti(r4-7) rx-^m^


But, Volume of the embankment = Volume of the earth dugout
7 7 45
nr (r + 7)x- = JIX —X —X
3 2 2 2
49
=> r(r + 7)=—xl5

w
4

=> 4r^+28r=735
4r^ + 28r-735 =0

^ _ -28 ± ^78A + 11760 _ -28 ± _ -28 + 112 _ —


84 =105

o
e
re
8 8 8 8

Hence, the width of the embankment is 10.5 m.

Frl
F
EXAMPLE 16 Water flows at the rate of 10 metre per minute through a cylindrical pipe having its
diameter as 5 mm. How much time will it take to fill a conical vessel whose diameter of base is 40 cm and
ou
depth 24 cm?

r
SOLUTION We have.

so
Volume of the water that flows out in one minute
kf
= Volume of the cylinder of diameter 5 mm and length 10 metre
oo

5 1
Y

= Volume of the cylinder of radius — mm - cm and length 1000 cm


B

22 1 1
-xlOOO cm^
re

= X — X

7 4 4
oY
u

Volume of a conical vessel of base radius 20 cm and depth 24 cm (20)^ x 24 cm^


ad
d

Suppose the conical vessel is filled in x minutes.


in

Volume of the water that flows out in x minutes = Volume of the conical vessel
Re

^x ix lx 1000x.v=ix—x 20^x24
F

7 4 4 3 7

1 400x24x4x4 512
.t = X minutes =51 minutes 12 seconds.
3 1000 10

EXAMPLE 17 An agriculture field is in the form of a rectangle of length 20 m width 14 m. A 10 m deep


welt of diameter 7m is dug in a corner of the field and the earth taken out of the zvell is spread evenly over
the remaining part of the field. Find the rise in its level.
SOLUTION We have,
7
Radius of the well = - , Depth of the well = 10 m
x2
f7 22 7 7
Volume of the earth dug out = n — X 10 m^ X —X -xl0m^=358 m^
b I 2 7 2 2

Also, we have
Length of the field = 14 m. Breadth of the field = 14 m
5.86 APPLIED MATHEMATICS-Xl

Area of the field = 20x 14 = 280

Area of the base of the well = x (7^ 2 11 7 7


- m - — x-x-m
2 77
- m
7
7 2 2

Area of the remaining part of the field = Area of the field - Area of the base of the field
280-
77
m
2 f560-77'i_2 m
483.__
m
2
2 ) 1 1

Let the rise in the level of the field be h metres.

Volume of the raised field = Area of the base X Height = m^


But, Volume of the raised field = Volume of the earth dugout

low
483 2x 385 _ 770
xh = 385 => /; =s =1594 m
2 483 " 483
Hence, rise in the level of the field = 1.594 m.
Is A c]/lmdrical pipe has inner diameter of 7 cm and waterfloius through it at 192.5 litres per

ree
mimde. Find the rate of flow in kilometres per hour.
SOLUTION We have.
F
F
Volume of water that flows per hour = (19250 x 60) litres = (19250 x 60 x 1000) cm

or
ur
7
Inner diameter of the pipe = 7 cm ^ Inner radius of the pipe = — cm = 35 cm
sf
Let h cm be the length of the column of water that flows in one hour.
k
Yo
oo

Clearly, water column forms a cylinder of radius 3.5 cm and length h cm.
Volume of water that flows in one hour = Volume of the cylinder of radius 3.5 cm and
B

length h cm
re

fll ■>
— x{35fxh cm
3
...(ii)
l7
ou
ad

From (i) and (ii), we have


Y

22
X 35 X 35 X /; = 192.50 x 60 x 1000
7
nd
Re

192.50x60x1000x7
h = cm = 300,000 cm = 3 km
Fi

22x 35x 35

Hence, the rate of flow of water is 3 km per hour.


i:XAMPLEls Water is being pumped out through a circular pipe zuhose internal diameter is 7cm.If
the flow of water is 71 cm per second, how many litres of water are being pumped out in one hour?
7
SOLUTION We have, Radius of the circular pipe = — cm
7
Clearly, water column forms a cylinder of radius — cm. It is given that the water flows out at the
rate of 72 cm/sec.

Length of the water column flowing out in one second = 72 cm.


Volume of the water flowing out per second
7
= Volume of the cylinder of radius - cm and length 72 cm.
QUANTIFICATION AND NUMERICAL APPLICATIONS 5.87

7 7 7 ^ 22 7
= TtX x72 cm^ = TTX — X—X 72 cm = — X—X -x72cm^ =1771 cm^
2 2 2 7 2 2

Volume of the water flowing out in one hour = (2772 x 3600) cm [●.● 1 hr = 3600 sec]
9979200
= 9979200 cm 2 = litres = 9979.2 litres
1000

Hence, 9979.2 litres of water flows out per hour.


1 \ A.\IPLE 20 Water is floxvingat the rate of 3 km/hr through a circular pipe of 20 cm internal diameter
into a circular cistern of diameter 10 m and depth 2 m. In hoio much time will the cistern be [died ?
SOLUTION Suppose the cistern is filled in .y hours. Since water is flowing at the rate of 3 km/hr.
Length of the water column in y hours = 3y km = 3000y metres.
Clearly, the water column forms a cylinder of radius
90 1
—cm = 10 cm = — m and h = height (length) = 3000.V metres

w
r =
2 10 a ef
2 (11 1 1 ^3

F lo
.'. Volume of the water that flows in the cistern in Y hours = Tur/z = — x—x—x 3000y m
^ / 10 10 j

(11 x5x5x2 m^
Also, Volume of the cistern = [:.r =5 m, /; = 2 m|

e
7

Fre
The cistern is filled in y hours. for
Volume of the water that flows in the cistern in y hours = Volume of the cistern.
22 1 1 22
— X — X 3000y = — x5x5x2
r
— X

7 10 10 7
You
oks

r5x5x2xl0xl0
Y = hrs = — hours =1 hour 40 minutes.
3
eBo

3000

EXAMPLE 21 A golf ball has diameter equal to 4.1 cm. Its surface has 150 dimples each of radius 2 mm.
Calculate total surface area which is exposed to the surroundings assuming that the dimples arc
our
ad

hemispherical.
SOLUTION We observe that:

(41^^ 2
Surface area of the ball = 47ux
dY
Re
Fin

In case of each dimple, surface area equal to Tir^{r is the radius of each dimple) is removed from
the surface of the ball where as the surface area of hemisphere i.e. 27cr^is exposed to the
surroundings. Let S be the total surface area exposed to the surroundings. Then,
S = Surface area of the ball -150 x nr^ +150 x 27rr^
5.88 APPLIED MATHEMATICS-XI

S= 16.81 TT + lSOTur^
2 2
S =●! 16.81jt +150tix — > cm
10

=>
S = (16.81 Ti + 6ti) cm^ = 22.81 k cm^ = 22.81 x — cm ^=71.68 cm^
7

EXERCISE 5.10

w
2
1. A solid cylinder has a total surface area of 231 cm^. Its curved surface area is —of the total
3
surface area. Find the volume of the cylinder.
2. A well with 14 m diameter is dug 8 m deep. The earth taken out of it has been evenly spread

e
all around it to a width of 21 m to form an embankment. Find the height of the

ro
re
embankment.
3. The difference between inside and outside surfaces of a cylindrical tube 14 cm long is 88 sq.
cm. If the volume of the tube is 176 cubic cm, find the inner and outer radii of the tube.

F
Fl
Water flows out through a circular pipe whose internal diameter is 2 cm, at the rate of 6

u
metres per second into a cylindrical tank. The water is collected in a cylindrical vessel
radius of whose base is 60 cm. Find the rise in the level of water in 30 minutes?

sr
5. A cylindrical water tank of diameter 1.4 m and height 2.1 m is being fed by a pipe of

ko
diameter 3.5 cm through which water flows at the rate of 2 metre per second. In how much
o
time the tank will be filled?
of
6. The sum of the radius of the base and height of a solid cylinder is 37 ni. If the total surface
area of the solid cylinder is 1628 cm^. Find the volume of the cylinder.
o
Y

7. A well with 10 m inside diameter is dug 8.4 m deep. Earth taken out of it is spread all
erB

around it to a width of 7.5 m to form an embankment. Find the height of the embankment.
8. C| and C2 are two cylinders having equal total surface areas. The radius of each cylinder is
uY

equal to the height of the other. The sum of the volumes of both the cylinders is2.507i cm^.
Find the sum of their curved surface areas.
A cylinder and a cone have equal radii of their bases and equal heights. If their curved
ad
do

surface areas are in the ratio 8:5, show that the radius of each is to the height of each as 3:4.
in

!P. A tent is in the form of a right circular cylinder surmounted by a cone. The diameter of
cylinder is 24 m. The height of the cylindrical portion is 11 m while the vertex of the cone is
Re

16 m above the ground. Find the area of the canvas required for the tent.
F

11. A circus tent is cylindrical to a height of 3 metres and conical above it. If its diameter is 105
m and the slant height of the conical portion is 53 m, calculate the length of the canvas 5 m
wide to make the required tent.
’ Find the weight of a solid cone whose base is of diameter 14 cm and vertical height 51 cm,
supposing the material of which it is made weighs 10 grams per cubic cm.
13. A right angled triangle of which the sides containing the right angle are 6.3 cm and 10 cm in
length, is made to turn round on the longer side. Find the volume of the solid, thus
generated. Also, find its curved surface area.
14. Find the volume of the largest right circular cone that can be fitted in a cube whose edge is
14 cm.

A cone and a hemisphere have equal bases and equal volumes. Find the ratio of their
heights.
16. A cylindrical tub of radius 16 cm contains water to a depth of 30 cm. A spherical iron ball is
dropped into the tub and thus level of water is raised by 9 cm. What is the radius of the ball?
QUANTIFICATION AND NUMERICAL APPLICATIONS 5.89

17 A cylinder of radius 12 cm contains water to a depth of 20 cm. A spherical iron ball is


dropped into the cylinder and thus the level of water is raised by 6.75 cm. Find the radius of
the ball. (Use n = 22/7).
A cylindrical jar of radius 6 cm contains oil. Iron spheres each of radius 1.5 cm are immersed
in the oil. How many spheres are necessary to raise the level of the oil by two centimetres?
19. A measuring jar of internal diameter 10 cm is partially filled with water. Four equal
spherical balls of diameter 2 cm each are dropped in it and they sink down in water
completely. What will be the change in the level of water in the jar?
20. A cone, a hemisphere and a cylinder stand on equal bases and have the same height. Show
that their volumes are in the ratio 1:2:3.

A cylindrical tub of radius 12 cm contains water to a depth of 20 cm. A spherical form ball is
dropped into the tub and thus the level of water is raised by 6.75 cm. What is the radius of
the ball?

w
22. A sphere, a cylinder and a cone have the same diameter. The height of the cylinder and also
the cone are equal to the diameter of the sphere. Find the ratio of their volumes.

F lo
ANSWERS

269.5 cm^ 2. 53.3 cm 3. 1.5 cm, 2.5 cm 4. 3 metres

ee
o. 28 minutes 6. 4620 7. 1.6 m 8. lOOncm^

Fr
10. 1320 11. 1947 m 12. 26.180 kg 13. 415.8 cm^, 234.03 cm^
14. 718.66 cm^ 15. 2:1 16. 12 cm !7. 9cm
16
for
ur
18. 16 19. —cm 21. 9 cm 22. 2 : 3 :1
75
ks
MULTIPLE CHOICE QUESTIONS
Yo
oo

Mark the correct alternative in each of the folloxving:


1. The average of the first 100 natural numbers is
eB

(a) 49.5 (b) 50.5 (c) 51 (d) 50


The average of odd natural numbers upto 100 is
r

(a) 49 (b) 49.5 (c) 50 (d) 50.5


ou
ad

3. The average of 15 numbers is 42. The sum of these numbers is


Y

(a) 600 (b) 435 (c) 620 (d) 630


4. The average of two numbers is ab. If one of the numbers is a, then the other number is
nd
Re

(a) - (c) lab - a (d) 2ab-b


Fi

The average of the reciprocals of a and b is


a + b lab a + b 2{a + b)
(a) (b) (c) (d)
lab a + b ab ab

6.
The average of 5 consecutive numbers is 16. The largest of these number is
(a) 21 (b) 20 (c) 18 (d) 19
7.
If the average of 7 consecutive numbers is 20. The smallest of the numbers is
(a) 16 (b) 17 (c) 22 (d) 23
8. The average of 15 numbers is 80. If each number is multiplied by 5, then the average of new
set of numbers is

(a) 400 (b) 16 (c) 240 (d) 1200


The average of 40 numbers is calculated, it is 24. If each number is increased by 6, the
average of new set of numbers is
(a) 18 (b) 4 (c) 144 (d) 30
5.90 APPLIED MATHEMATICS-Xl

10. The average of four numbers is 60. If first number is one-fourth of the sum of the last three,
the first number is

(a) 15 (b) 42 (c) 45 (d) 48


n. If the sum of three consecutive even numbers is 44 more than the average of these numbers,
then the largest number is
(a) 16 (b) 18 (c) 20 (d) 24
12. The length of the altitude of an equilateral triangle of side 2-\/3 cm is
^A3 V3
(a) — cm (b)- cm (0- cm (d) 3 cm
2

13. If the area of a triangle is A cm'^ and its base is a, then the height is

w
2A a /I 2n
(a) (b) (c) (d) ^
a 2/1 2a A

14. ABC is a right triangle right angled B such that AB = 32 cm and AC = 25 cm. The area of

o
AABC is

e
re
(a) 2Vl^ 2 (b) 3^/TMcm“ (c) 4Vi^ cm^ (d) Vl54 cm^
cm

rFl
F
15. The area of a triangle is 216 cm*^ and its sides are in the ratio 3:4:5. The perimeter of the
triangle is

r
(a) 72 cm (b) 36 cm (c) 12 cm (d) 6 cm
ou
fo
16. The lengths of three sides of a triangular field are 20 m, 21 m and 29 m. The area of the field
ks
IS

(a) 210 m^ (b) 215 m^ (c) 230 m^ (d) 225 m^


oo
Y

17. The area of a triangle whose sides are of lengths 3 cm, 4 cm and 5 cm is
eB

(a) 12 cm2 (b) 8 cm2 (c) 10 cm2 (d) 6 cm2


18. The areas of two similar triangles are 12 cm 2 and 49 cm2, jf the height of the smaller one is
r

2.1 cm, then the corresponding height of the bigger one is


ou
Y
ad

(a) 8.4 cm (b) 4.2 cm (c) 4.41 cm (d) 2.1 cm


19. A triangle of area 9A cm2 has been drawn such that its area is equal to the area of an
equilateral triangle of side 6 cm, then A =
d

(c) V2 (d) V3
Re
in

(a) 2 (b) 3
-0. If the sides of a triangle are in the ratio 2:3:4, then the corresponding altitudes are in the
F

ratio

(a) 6:4:3 (b) 5:4:3 (c) 4 : 5 : 6 (d) 6:5:3


21. The area of a right triangle with hypotenuse h and semi-perimeter s is given by
1
(b) s(s-;i)
(a) 2
sh (c) (s-/f) h (d) (s-/j)

22. The areas of two equilateral triangles are in the ratio 25:36. Their altitudes are in the ratio
(a) 5:6 (b) 25:36 (c) 36:25 (d) 6:5
23. The base and altitude of a right angled triangle are 12 cm and 5 cm respectively. The
perpendicular distance of its hypotenuse from the opposite vertex is
60 56
(a) 7 cm (b) 5 cm (c) — cm (d) — cm
13 ^ ^ 13
. In an isosceles triangle, the measure of each of the equal sides is 10 cm and the angle
between them is 45°. The area of the triangle is
QUANTIFICATION AND NUMERICAL APPLICATIONS 5.91

25
(a) 25^3 cm2 (b) cm2 (c) 25V2 cm2 (d) 25 cm2
V2
25. If each side of a rectangle is increased by 50%, its area will increase by
(a) 50% (b) 125% (c) 150% (d) 200%
26. The area of a rectangular field is 52000 m2. This rectangular area has been drawn on a map
to the scale 1 cm to 100 m. The length is shown as 3.25 cm on the map. The breadth of the

w
rectangular field is
(a) 150 m (b) 160 m (c) 200 m (d) 300 m
27. The perimeter of a rectangular field is 480 m and the ratio between the length and breadth is

e
5 :3. The area is

(a) 1350 m2 (b) 1550 m2 (c) 13500 m2 (d) 15500 m2

e
or
28. A rectangular farm has to be fenced on one long side, one short side and the diagonal. If the

r
cost of fencing is ? 100 per m, the area of the farm is 1200 m2 and the short side is 30 m long,

F
how much would the job cost?
(a) ?7000 (b) ? 12000 (c) ? 14000 (d) ? 15000

oF
ul
29. The diagonal of rectangular field is 15 meters and the difference between its length and
breadth is 3 metres. The area of the rectangular field is

rs
(a) 9 m2 (b) 12 m2 (c) 21 m2 (d) 108 m2

ko
30. The length of a rectangular black board is 8 m more than its breadth. If its length is
increased by 7 m and its breadth is decreased by 4 rn, its area remains same. The length and
breadth of the rectangular black board are
of
o
(a) 24 m, 16 m (b) 20 m, 24 m (c) 28 m, 16 m (d) 28 m, 20 m
Y
31. A rectangular lawn 80 m x 60 m has two roads each 10 m wide running in the middle of it,
B

one parallel to the length and other parallel to the breadth. The cost of gravelling them at
Y

^ 30 per m2 is
er

(a) ? 3,600 (b) ? 3,900 (c) ? 36,000 (d) ? 39,000


u

32. Total area of 64 small squares of a chessboard is 400 cm2, jf there is 3 cm wide border
around the chessboard, the length of the side of the chessboard, is
od
ad

(a) 17 cm (b) 20 cm (c) 23 cm (d) 26 cm


in

33. The ratio of the area of a square to that of the square drawn on its diagonal, is
(a) 1:1 (b) 1:V2 (c) 1:2 (d) 1:4
Re

34. The diagonal of a square is 8^I2 cm. The diagonal of a square whose area is double that of
F

the first square, is


(a) 8 cm (b) 16 cm (c) 16V2cm (d) 32 cm
35. If each side of a square is increased by 50%, the ratio of the area of the resulting square to
that of the given square, is
(a) 4:5 (b) 5:4 (c) 4:9 (d) 9:4
36. The length and breadth of a square are increased by 40% and 30% respectively. The area of
the residting rectangle exceeds the area of the square by
(a) 35% (b) 42% (c) 62% (d) 82%
37. The area of a circle with circumference C, is
C2
(a) $-
An
(b) ^
An
(c)-^
4n^ (d)^ n

38. The circumference of a circle of area A, is

(a) iM n
(b) 2Va^ (c) ^/An (d) AVAii
5.92
APPLIED MATHEMATICS-Xi

39. If the circumference of a circle is doubled, then its area is


(a) double (b) tripled (c) quadrupled (d) unchanged
40. A circle and a rectangle have the same perimeter. If the sides of the rectangle are 18 cm and
26 cm , then the area of the circle, is
(a) 88 cm2 (b) 154 cm2 (c) 1250 cm2 (d) 616 cm2
41. If the areas of a circle and a square are equal then the ratio of their perimeters, is
(a) 1:1 (b) 2:71 (c) 7t:2 (d) V^:2
2
42. If the area of a circle inscribed in an equilateral triangle is 154 cm , then the perimeter of the
triangle, is
(a) 71.5 cm (b) 71.7 cm (c) 72.3 cm (d) 72.7 cm
43. A triangle with sides 13 cm, 14 cm and 15 cm is inscribed in a circle. The radius of the circle

ow
IS

(a) 2 cm (b) 3 cm (c) 4 cm (d) 8.125 cm


44. Area of the segment of a circle of radius r is
wd n9 1
(*>) (b) (c) — sin 0 (d) none of these
180

e
360 2

Fl
re
45. A circular park, 42 m in diameter, has a path 3.5 m wide runiiing around it on the out side.
The cost of gravelling the park at ^ 4 per m2, is

F
(a) ? 2,048 (b) ^1,652 (c) ? 1,672 (d) ? 2,002
ur
46. A skating champion moves along the circumference of a circle of radius 28 m in 44 seconds.
or
How many seconds will it take her to move along the perimeter of a hexagon of side 48 m?
sf
(a) 48 sec (b) 68 sec (c) 72 sec (d) 84 sec
47. Four circles each of radius 'a' touch one another. The area enclosed between them, is
k
Yo

2
oo

b .2
(a) - (b) 3fl^ (c)
(d) f/
B

48. The area of the shaded region in the adjoining figure, is


re

2 2
(a) fl^(Ti-l)
a a
(c) -(TT-l) (d) —in-2)
4
u
ad
Yo

A B
d
Re
in
F

D t

Fig. 5.79

49. A circle is circumscribed around a square as shown in Fig. 5.79. The area of one of the four
shaded portions is equal to —. The radius of the circle, is
1
(a) V2 (b) (c) 2 (d) 3
^/2
50. The ratio of the areas of the incircle and circumcircle of an equilateral triangle, is
(a) 1:2 (b) 1:3 (c) 1:4 (d) 1:9
QUANTIFICATION AND NUMERICAL APPLICATIONS 5.93

The circumferences of front and rear wheels of a cart are 40 ft and 48 ft respectively. When
the front wheel has done 5 more revolutions than the rear wheel, the distance moved by
cart is

(a) 850 ft (b) 950 ft (c) 1200 ft (d) 1450 ft


Water flows out through a cylindrical pipe whose internal diameter is 2 cm, at the rate of 6
metres per second into a cylindrical tank, the radius of whose base is 60 cm. The height
through which the water level rises in 30 minutes, is
(a) 2m (b) 3 m (c) 4 m (d) 5 m
53. If the length of each edge of a cube is increased by 25%, then the percentage increase in its
surface area, is
(a) 25% (b) 48.75% (c) 50% (d) 56.25%
54. If the length of each edge of a cube is increased by 507o, then the percentage increase in its
volume, is
(a) 237.57o (b) 273.5% (c) 137.57o (d) 50%

w
If the volumes of two cubes are in the ratio 64 : 27, then the ratio of their edges, is
(a) 8: 3^/3 (b) 4:3 (c) 3 : 4 (d) 3^/3:8

F lo
DO. If the volumes of two cubes are in the ratio 8 : 27, then the ratio of their surface areas, is
(a) 2:3 (b) 4:27 (c) 4 : 9 (d) 3 :2
How many cubes of 10 cm edge can be put in a cubical box of 1 m edge?

ee
D. -

Fr
(a) 10 (b) 100 (c) 1000 (d) 10000
58. If the sum of the lengths of diagonals of a cube is 12 cm, then sum of the lengths of edges of
for
the cube, is
(a) 6^3 (b) 12 cm (c) 15 cm (d) cm
ur
cm

59. An aluminium sheet 27 cm long, 8 cm broad and 1 cm thick is melted into a cube. The
s
ook

difference in the surface areas of two solids, is


Yo

(a) 296 cm^ (b) 284 cm^ (c) 286 cm^ (d) 268 cm^
eB

60. Three equal cubes are placed adjacently in a row. The ratio of the total surface area of the
cuboid so formed to that of the sum of the surface areas of three cubes, is
our

(a) 7:9 (b) 9 :1 (c) 7:18 (d) 3:2


ad

61. If the areas of three adjacent faces of a cuboidal box are 120 cm , 72 cm and 60 cm
respectively, the volume of the box, is
Y

(a) 720 cm ^ (b) 864 cm ^ (c) 7200 cm^ (d) 72cm^


Re
nd

62. If the volume of a cube is numerically equal to the sum of the lengths of its edges, then the
Fi

surface area of the cube, is


(a) 36 sq. units (b) 72 sq. units (c) 66 sq. units (d) 144 sq. units
63. If the volume of a cylinder, whose radius and height are in the ratio 5:7, is 4400 cm^. Then,
the radius of the cylinder, is
(a) 4 cm (b) 5 cm (c) 10 cm (d) 12 cm
64. The number of circular pipes with an inside diameter of 1 inch which will carry the same
amount of water as a pipe with an inside diameter of 6 inches, is
(a) 6 (b) 12 (c) 36 (d) 72

ANSWERS
● (b) 2. (c) 3. (d) 4. (c) 5. (a) (c) 7. (b) 8. (a)
9. (d) 10. (d) Tl. (d) 12. (d) 13. (a) 14. (b) 15. (a) 16. (a)
17. (d) IS. (b) 19. (d) 20. (a) 21. (b) :::. (a) 25. (c) 21. (c)
(b) 26. (b) 27. (c) 28. (b) 2^. (d) 30. (d) 31. (d) 32. (d)
5.94
APPLIED MATHEMATICS-XI

(c) (b) 3:v (d) (d) - , (b) (b) 39. (c) 40. (d)
;i. (d) M (d) (d) (c) -1=:. (d) ● (c) (c) - (b)
(a) (c) (C) 53. (b) . (d) . (a) .35. (b) 56. (c)
(c) (d) (c) 6(1. (a) 61. (a) 62. (b) 63. (c) 64. (c)

w
F lo
ee
Fr
for
ur
s
ook
Yo
eB
our
ad
Y
Re
nd
Fi
CHAPTER 6
SETS

6.1 SETS

It is a well known fact that any attempt to define set has always led mathematicians to
unsurmountable difficulties. For example, suppose one defines the term set as "o well defined
collection of objects". One may then ask what is meant by a collection. If one answers that a
collection is an aggregate of objects or things. What is then an aggregate? Perhaps then one may

w
define that an aggregate is a class of things. What is then a class? Now, one may define a class as
a collection. In this manner question after question, since our language is finite, we find that

F lo
after some time we will have to use some words which have already been questioned. The
definition thus becomes circular and worthless. Thus, mathematicians realized that there must
be some undefined (or primitive) terms. In this chapter, we start with two undefined (or

ee
primitive) terms — "element" and "set". We assume that the word "set" is synonymous with

Fr
the words "collection". aggregate", "class" and is comprised of elements. The words
element", "object", "member" are synonymous. for
\ia is an elementof a setv4, then we writer? e A and say r?belongs to A or <? is in^4 orr? is a member
ur
of A. If a does not belong to A, then we write a i A. \.\ is assumed here that if A is any set and a is
any element, then either a sAoraiA and the two possibilities are mutually exclusive. Thus,
s

one cannot say "consider the set A of some positive integers", because it is not sure whether
ook
Yo

3 € A or 3 e A.
eB

Throughout this chapter we shall denote sets by capital alphabets e.g. A, B,C,X,Y,Z etc. and the
elements by the small alphabets e.g. a, b, c, x, y, z etc.
The following are some illustrations of sets:
our
ad

.: L ' TRATION 1 The collection of vowels in English alphabets. This set contains five elements, namely,
a, e, 1, 0, u.
Y

li LI STRATION 2 The collection offirst five prime natural numbers is a set containing the elements 2,
Re

3, 5,7,11.
nd

! u I SI R.ATION 3 The collection of all States in the Indian Union is a set.


Fi

ILLI STR.ATION 1 The collection of past presidents of the Indian union is a set.
u i LstRATlON The collection of cricketers in the world who were outfor 99 runs in a testmatchis
a set.

11.1 USTR.ATION 6 The collection of good cricket players of India is not a set, since the term "good player
is vague and it is not well deifned".
Similarly, collection of good teachers in a school is not a set. However, the collection of all
teachers in a school is a set.

In this chapter we will have frequent interaction with some sets, so we reserve some letters for
these sets as listed below:
N : for the set of natural numbers.
Z : for the set of integers.
: for the set of all positive integers.
6.2
APPLIED MATHEMATICS-X!

Q : for the set of all rational numbers.


Q'*’: for the set of all positive rational numbers.
R : for the set of all real numbers.
: for the set of all positive real numbers.
C : for the set of all complex numbers.
_ EXERCISE 6.1
1 What is the difference between a collection and a set? Give reasons to support your answer?
Which of the following collections are sets? Justify your answer:
1 A collection of all natural numbers less than 50.
(ii) The collection of good hockey players in India.
(iii) The collection of all girls in your class,

ow
(iv) The collection of most talented writers of India.
(V) The collection of difficult topics in Mathematics.
(Vi) The collection of novels written by Munshi Prem Chand.
(vii) The collection of all months of a year beginning with the letter J.

e
Fl
re
(viii) The collection of all questions in this chapter,
(ix) A collection of most dangerous animals of the world.

F
(x) The collection of prime integers.
ur
3, IfA
= {0,1, 2, 3, 4,5, 6,7, 8, 9,10), then insert the appropriate symbol eor «in each of the

r
following blank spaces:
(0 4 A (ii) -4 A
fo (iii) 12 ....A
ks
(iv) 9....A (v) 0 A (vi) -2 A
Yo
oo

ANSWERS
eB

1. Every set is a collechon but a collection is not necessarily a set. Only well defined collections
are sets. For example, group of good cricket players is a collection but it is not a set.
(0/ (iii). (vi). (vii), (viii), (X)
ur

(i) € (ii) g (iii) g (iv) 6 (V) 6 (vi) 2


ad
Yo

_ HINTS TO SELECTED PROBLEMS

(iv) The collection of most talented writers of India is not a set as there is no specific
d
Re

criterion to determine whether a writer is talented or not.


in

(Vi)
The collection of all months of a year beginning with the letter / is a set given by
F

(January, June July}.


(Vii) The collection of novels written by Munshi Prem Chand is a set because one can
determine whether a novel is written by him or not.
(viu) The collection of all questions in this chapter is a set because if a question is given one
can easily decide whether it is a question of this chapter or not.
(ix) The collection of most dangerous animals of the world is not a set because there is no
criterion to determine whether an animal is most dangerous or not.
6.2 DESCRIPTION OF A SET

A set is often described in the following two forms. One can make use of any one of these two
ways according to his (her) convenience,
(i) Roster form or Tabularform (ii) Set-builder form
Let us now discuss these forms.
SETS 6.3

6.2.1 ROSTER FORM

In this form a set is described by listing elements, separated by commas, within braces { }.
...rUATlON 1 The set of vowels of English Alphabet may be described as (a, e, i, o, uj.
II I USI'KATION 2 The set of even natural numbers can be described as {2, 4, 6, Here the dots stand
for 'and so on'.
II.LUSIUAI!!
r If A is the set of all prune numbers less than 21, then A = {2, 3, 5, 7}.
^ \

makes no difference. Thus, {a, e, i, o, ul and


Theorder in zuhich the elements are written in a set

le,a, i, 0, u} denote the same set. Also, the repetition of an element has no ^fect. For example, 11,2,3,2} is
the same set as [1,2, 3}.

6.2.2 SET-BUILDER FORM

In this form, a set is described by a characterizing property of its elements a:. In such a case
the set is described by {x: P(.v) holds } or, {a: | P (a:) holds), which is read as 'the set of all x such

w
that P (.v) holds'. The symbol' |' oris read as 'such that'.
In other words, in order to describe a set, a variable .r (say) (to denote each element of the set) is

F lo
written inside the braces and then after putting a colon the common property P (a:) possessed by
each element of the set is written within the braces.

e
ILLUSTRATION The set E of all even natural numbers can be written as

Fre
£ = {a: : a: is a natural number and x = 2n for n s N)
or. £ = {a: : a: e N, a: = 2n, n eN] or, E = {x&N:x = 2n, neN]
for
ILLUSTRATION 3 The Set A -{1,2,3, 4, 5, 6, 7, 81 can be written asA= {.v e JV : a: < 8j.
r
H LUSTRATION 6 The set of all real numbers greater than -1 and less than 1 can be described as
You

{x eR:-l <x <1).


oks

! ION 7 The set A = (0,1, 4, 9,16, ...j can be zoritten as A = {x^ : e Zj.
eBo

iLir-.;

ILLUSTRATIVE EXAMPLES
our
ad

I ON DESCRIBING OR REPRESENTING SETS IN TABULAR FORM OR ROSTER FORM

● A’Li 1 Describe the following sets in Roster form:


(i) The set of all letters in the loord 'MATHEMATICS'
dY
Re

(ii) The set of all letters in the ivord 'ALGEBRA'


Fin

(iii) The set of all vowels in the word 'EQUATION'


(iv) The set of all natural numbers less than 7.
(v) The set of squares of integers.
SOLUTION (i) We observe that distinct letters in the word 'MATHEMATICS' are:
M,A,T. H,E,!,C,S
Since the order in which the elements of a set are written is immaterial and the repetition of
elements has no effect. So, required set cap be described as follows:
{M,A,T,H,E,I,C,S}

(ii) We find that the word 'ALGEBRA' has following distinct letters: A, L, G, £, B, R
Hence, required set can be described in Roster form as follows: {A, L, G, E, B, R)
(iii) Clearly, word 'EQUATION' has following vowels: A, E, I, O, U
So, required set can be described as follows: [A, E, I, O, LZ)
6.4 APPLIED MATHEMATICS-XI

(iv) Natural numbers less than 7 are: 1,2, 3,4,5,6.


Hence, required set can be described as follows: {1, 2, 3, 4,5, 6).
(v) Since square of a negative integer is same as the square of its absolute value. Tlierefore,
squares of integers are 0,1,4,9,16, 25, Hence, required set is {0,1, 4, 9,16,

w
/!//', ON DESCRIBING OR REPRESENTING SETS IN SET-BUILDER FORM
r S
Describe the foilozvhig sets in set-builder form:
●●'I PI.;

(i) The set of all letters in the zoord ‘PROBABILITY’.


(ii) The set of reciprocals of natural numbers. (iii) The set of all odd natural numbers.

e
(iv) The set of all even natural numbers.

e
SOLUTION (i) Given set in set-builderform can be described as follows:

or
[x : .T is a letter in the word 'PROBABILITY']

r
(ii) Given set can be described in set-builder form as follows:

F
1
{.Y: X is reciprocal of a natural number] or. x:x = -, £N or. -:eiV

oF
ul
n n

(iii) An odd natural number can be written in the form(2/j -1). So, given set can be described as

rs
follows {y:y = 2n-l,neN\ or, |2?i-l:neN}.

k
(iv) An even natural number can be written as 2n, wheren e N. Therefore, set of all even natural

o
numbers can be written in the form |x: y = 2n, neN] or, {2n :n sN\
of
Write the set of all integers zvhose cube is an even integer.
o
SOLUTION We know that the cube of an even integer is also an even integer. Hence, the required
Y
set is the set of all even integers which can also be written in the set-builder form as 12n: n e Z ].
B

Write the set of all real numbers ivhich cannot be written as the quotient of two integers in
Y

the set-builder form.


er

SOLUTION We know that all rational numbers are expressible as the quotient of two integers.
u

Therefore, the required set is the set of all irrational numbers which can be written as
{y : Y is real and irrational] or, {y : y e R but y gQ).
od
ad

ni ON DESCRIBING A SET IN ROSTER FORM WHEN IT IS GIVEN IN SET-BUILDER FORM


in

i Describe each of the follozving sets in Roster form


(i) |y :xis a positive integer and a divisor of 9] (ii) {y:y e Z and | y| < 2)
Re
F

n
(iii) (y :xis a letter of the word ’PROPORTION'] (iv) < y:y= andl <n <3, wheren s N ■

SOLUTION (i) Since y is a positive integer and a divisor of 9. So, y can take values 1,3,9.
(y: Yis a positive integer and a divisor of 9 ] = {1, 3, 9]
(ii) We find that y is an integer satisfying] y| < 2.
and. ]Yj = 0,1,2 => Y = 0,+ 1,±2
So, Y can take values - 2, 1,0,1,2.
(y : y e Z and j y| < 2 ] = ]- 2, -1, 0,1,2]
(iii) We find that distinct letters in the word 'PROPORTION' are P, R, O, T, N, I. So, x can be
P,R,0,T,I,N.
Hence, {y: Yis a letter in the word 'PROPORTION'} = {P, R,0,T, I,N]
(iv) We have.
n
x = wheren eN and 1 <n<3.
SETS 6.5

n
X = , where « = 1, 2, 3.
+1

1 2 3
X -
1^+1 '2^ + 1 ' 3^ + 1
1 2
=> a: = —
2'5 ' 10
n
i 2 _3
Hence, ■! a: : a: = and 1 <f? < 3, whereneN

ow
rp- +1 2'5'10

Write the set of all vowels in English alphabet which precede s.


SOLUTION The vowels in English alphabet which precede s are a, e, i, o. So, the set
A = {a, e, i, o} is the set of all vowels in English alphabet which precede s.

e
Write the set A ={x: x eZ, x^ < 20) in the roster form.

re
SOLUTION We observe that the integers whose squares are less than 20 are: 0, ± 1, ± 2,± 3, ± 4.

Flr
F
Therefore, the set/l in roster forn\ is ^ = {-4,- 3, - 2, -1, 0,1, 2, 3, 4}.
Match each of the set on the left described in the roster form luith the same set on the right
described in the set-builder form.
ou
sr
(i) [P,R,I,N,C.A,L\ (a) {a: : a: is a positive integer and is a divisor of 18 ]
(ii) (0)

(iii) {1,2,3,6,9,18)
fo
(b) {a: : X is an integer and a: - 9 = 0}
k
(c) {a: : a: is an integer and x + 1=1}
oo
(iv) {-3,3} (d) {a: :xisa letter of the word 'PRINCIPAL' j
Y

SOLUTION (i) Clearly, [P, R, /, N, C, A, L] = {P, R, /, N, C, I, P, A, L)


reB

= (a: : a: is a letter of the word 'PRINCIPAL'}


Hence, (i) matches with (d).
uY

(ii) (0) = (a: : a: is an integer equal to zero) = {a: : a* is an integer and a: + 1 = 1}


Hence, (ii) matches with (c).
ad
do

(iii) |1, 2, 3, 6, 9,18) = Set of all positive divisors of 18


= {a: : a: is a positive integer and is a divisor of 18}
in

Hence, (iii) matches with (a).


Re

(iv) Clearly, {- 3, 3) = |:c: a: is an integer and a: - 9 = 0). Hence, (iv) matches with (b).
F

ON DESCRIBING A SET IN SET-BUILDER FORM WHL; IS GlVt f:R r:

1 2345678 91.,,
Write the set — , — , —, —, — } in the set-builder form.
23456789 lOj
SOLUTION We observe that each element in the given set has the denominator one more than
the numerator. Also, the numerator begins from 1 and do not exceed 9. Hence, in the set-builder
n
form the given set can be written as ■ x:x = n , n e N, n < 9 ●.
+ 1

1
Write the setX = <\ , - 1 J_ in the set -builder form.
4'9'16 ' 25 '
SOLUTION We observe that the elements of setX are the reciprocals of the squares of all natural
numbers. So, the set X in set builder form is X=J-^:nGNl. n
6.6
APPLIED MATHEMATICS-XI

r.\^' Ml’l.r 1 Write the following sets in Roster form:


(i) A = \a„ = 3a„ and a-^ = 1) (ii) B ={7Z„ :?! eN,7Z,,^ 2 = ^! + 1 +% ' =^2 =^1
SOi.UTION (i) We have, a^= \ and iz,, + 1 = 377,, for all n eN
Putting Z3 = 1 in 2 = 3^7,,, we get
7?2 = = 3 X 1 = 3 [●●● = 1]
Putting?! = 2infl„+j = 3rt„,weget
= 3<72 = 3 X 3 = 3^ [●●■ «2 = 3]
Putting ?! = 3 in 7?,, + j = 3!?,, , we get
<?4 = 3rt3 = 3 X 3^ = 3^ [●●● «3 = 3I
Similarly, we obtain

w
7?5 - 3^4 = 3x3^= 3^, iZg = 3a^ = 3 x 3^ = 3^ and so on.
Hence, A = {a^, G2, a^, a^, ...} = {1, 3, 3^, 3"*, 3"^, 3^,

F lo
(ii) We have, = 1, «Z2 =1 and zz,,^. , = !Z„ + j + zz„ .
Putting?! = 1,2, 3, 4, in rt,,-2 =^7+1+^!/we get

ee
!Z3=fZ2+!7i=l+l=2; i?4 = a^ + 02 =2 + 1 = 3;z^ =!Z4+ZZ3 = 3 + 2=5;

Fr
zzg = !^+?Z4 = 5+ 3 = 8 and so on.
Hence, B = Itz-j , zz2 , z?3 ,774 , ?Z5 , zz^ , ) = (1,1, 2, 3,5, 8, )
for
ur
EXERCISE 6.2
Describe the following sets in Roster form:
ks

(i) {.Y: Y is a letter before e in the English alphabet),


Yo

(ii) {YeN:Y^<25}.
oo
eB

(iii) {y £ N: Y is a prime number, 10 < y < 20).


(iv) {y6N:y = 2?z, ?!£N).
(v) {y£R:y>y|.
r
ou
ad

(vi) |y : Y is a prime number which is a divisor of 60).


Y

(vii) {y : Y is a two digit number such that the sum of its digits is 8).
(viii) The set of all letters in the word 'Trigonometry',
nd
Re

(ix) The set of all letters in the word 'Better'.


Fi

Describe the following sets in set-builder form:


(i) v4 = 11,2,3, 4, 5, 6) (ii) 6 = {1, 1.2, 13, V4,l-5,...)
(iii) C= {0,3, 6, 9,12,...) (iv) D= [10,11,12,13,14,15)
(V) E={0) (vi) (1,4, 9,16,...,100)
(vii) 12,4,6,8, ) (viii) {5,25,125,625)
List all the elements of the following sets:
1
(i) >1 = ( y: Y^ <10, Y e Z 1 (ii) B = ^ Y: Y - 27?-1 , 1 <;?<5 ●

1 91
(iii) C =
Y: Y is an integer,- ^ ~2J
(iv) D = {Y: Y is a vowel in the word "EQUATION")
(v) E = {Y: Y is a month of a year not having 31 days )
(vi) f = {y : Y is a letter of the word "MISSISSIPPI" )
6.7
SETS

4. Match each of the sets on the left in the roster form with the same set on the right described
in the set-builder form:

(i) {A,P,L,E] (i) {.y: a: +5 =5, y 6 Z |


(ii) (5,-51 (ii) 1 Y: Y is a prime natural number and a divisor of 10 1
(iii) 10} (hi) {y : y is a letter of the word "RAJASTHAN" }
(iv) 11,2,5,10] (iv) {Y: Y is a natural number and divisor of 10)

w
(V) {A,H,/,R,S,r,Nl (v) 25=0)

(Vi) 12,5} (vi) |y : y is a letter of the word "APPLE"}


Write the set of all vowels in the English alphabet which precede q.

e
Write the set of all positive integers whose cube is odd.

ro
re
Write the set 1 2 _3 ^ ^ ^ in the set-builder form.
2'5 ' 10'17 ' 26 ' 37 ' 50

F
Fl
ANSWERS

(i)
(V)
{n,b,c,d} (ii) (1,2,3, 4}
(j)
u
(iii) {11,13,17,19} (iv) (2,4,6, 8,...}
(vii) {17,26, 35,44,53,62,71,80}

sr
(Vi) {2, 3,5}
(viii) {T,R,/,G,0,JV,M,£,y} (ix) (6,£,T, R}

ko
o
(i) |y:ygN,y<71 (ii) \x:x =l/n,n eN\ (iii) (y: y = 3«,eZ'^}
of
(iv) (y:ygN, 9<y<16) (v) {y:y = 0} (vi) (y^ :y gN, 1 <Y <10}
o
(vii) (y:y = 2?j,« gN} (viii) (5": JIG A/,1 <n<4}
Y
erB

(i) A=(0,+l,±2,± 3} (ii) 1 1 1 1


3'5 ' 7 ' 9
uY

(iii) C ={0,1, 2, 3, 4} (iv) D={A,£, 7,0, L/}


(v) £ = (Feb., April, June, Sept., November) (vi) f = (A4, 7, S,P]
- (i)-^(vi); (ii)-> (v); (iii) -> (i); (iv) -> (iv); (v) -> (iii); (vi) (ii)
ad
do

n
. . {n,e,i,o\ (2« + l \n gZ,ji>0} :neN, n <7 ■
■■ «2+l
in
Re

6.3 TYPES OF SETS


F

EMPTY SET A set is said to be empt}/ or mill or void set ifithnsnoclementanditisdenotedb\/<^.


In Roster method, ^is denoted In/ { ).
It follows from this definition that a set A is an empty set if the statement y e A is not true for
any y.
ull -^ikuion i \xeR:x^=-2} =^.
11.11 s I RATION : (y gN :5 <Y <6} = (}).
Il.l L's 1 RAMON 3 -The Set A given by A == (x: x is an even prime number greater than 2} is an empty set
because 2 is the only even.prime number.
A set consisting of at least one element is called a non-empty or non-void set.
NOTl I/A and B are any two empty sets, then y e A iff (if and only i/} y g B is satisfied because there is
no element X in either A or B to which the condition maybeapplied. Viiis,A = B. Hence, there is only one
empty set and we denote it by (j). Therefore, article ‘the' is used before empty set.
SINGLETON SET A set consisting of a single element is called a singleton set.
6.8 APPLIED MATHEMATICS-XI

■ -■ !{)N 4
The set {51 is a singleton set.
■ The set {x:xeN and x^ = 9} is a singleton set equal to {3}.
rlNITESE:
A set is called a finite set if it is either void set or its elements can be listed (counted, labelled)
Inj natural numbers 1, 2,3,... and the process of listing terminates at a certain natural number n (say).
.ARDINAL NUMBER OF A FINITE SET 77ie number n in the above definition is called the cardinal
number or order of a finite set A and is denoted by n(A).
INFINITE SET A set whose elements cannot be listed by the natural numbers 1, 2, 3, ..., n, for any

ow
natural number n is called an mfiiiite set.
Each one of the following sets is a finite set:
(i) Set of even natural numbers less than 100. (ii) Set of soldiers in Indian army.
(iii) Set of even prime natural numbers. (iv) Set of all persons on the earth.

e
re
Each one of the following sets is an infinite set:
.

(i) Set of all points in a plane, (ii) Set of all lines in a plane. (iii) {xeR:0<x<l\.

F
EQUiVALENTSETS

Frl
Two finite sets A and B are equivalent if their cardinal numbers are same.
i.e. n(A) = n(B).
ou
■ QU“.L SETS Two sets A and B are said to be equal if every element of A is a member of B, and every

sr
element ofB is a member of A.
If sets A and B are equal, we write A = B and A^= B when A and B are not equal. kfo
If A = {1,2,5,6} and 6 = {5,6,2,1). Theni4 = B, because each element of A is an element of B and
oo
vice-versa. Note that the elements of a set may be listed in any order.
Y

It follows from the above definition and the definition of equivalent sets that equal sets are
reB

equivalent but equivalent sets need not be equal.


For example, A = {1,2,3) and B = [a, b, c} are equivalent sets but not equal sets.
uY

ILLUSTRATIVE EXAMPLES
ad
do

ON IDENTIFYING Wnt fHER GIVEN SET IS EMPTY OR NOT

Which of the following sets are empty sets?


in

(i) A = [x:x -3 = 0 and X is rational ] (ii) B = \x: x is an even prime number ]


Re

(iii) C = {a: :4 <a: <5, a: e N} (iv) D = \x:x^ =15, and X is an odd integer]
F

SOLUTION (i) We know that there is no rational number whose square is 3. So, - 3 = 0 is not
satisfied by any rational number. Hence, A is an empty set.
(ii) We know that 2 is the only even prime number. Therefore, B = {2}. So, B is not an empty
set.

(iii) Since there is no natural number between 4 and 5. So, C is an empty set.
(iv) Since a: =5, - 5 satisfy = 25 and ±5 are odd integers. Therefore, D ={-5,5|. Thus, D is a
non-empty set.
:i ON EQUAL SETS

Find the pairs of equal sets, from the following sets, if any, giving reasons:
-4 ={0), B ={x:x>!5and x <5},C ={x: x -5 = 0],D =[x: =25}

E = {x:xis an integral positive root of the equation x^ - 2a: -15 = O).


6.9
SETS

SOLUTION We have,

= (0), B = [x : X >15 and x <5} = <^, C = (x:x - 5 = 0} = {5),


D = {x:x^ =25} = {-5,51, and, E = j5).
Clearly, C = E.
...viplh;- Which of the foUoiving pairs of sets are equal ? Justify ijour answer.
(i) .4 = {x: X is a letter in the word "LOYAL"}, B = {x; x is a letter of the word "ALLOY" ]
(ii) A={x:xeZ and x^ <81, B= |x: x g R and x^ - 4x +3 = 0}
SOLUTION (i) We have,
= |L,0,y,^, L| = {L,0,Y, Aland, B = {A,L,L,0,Y\ = {L,0,Y,A]
Clearly, A = B.
(ii) /I = {x:xgZ andx^ <81 = {-2,-l,0,l,2}and,B=lx:x gR andx^ -4x+ 3 = 0} ={1, 3).

w
We observe that 0 g /I but 0 G B. So, A^ B.

Type HI ON FINITE AND INFINITE SETS


EXAMPLE 4

F lo
Stale lohich of the following sets are finite and zvhich are infinite:
(i) A = {x:xeZandx^-5x+6 = 0} (ii) B = {x:xgZ and x^ is even }
(iii) C = {x:xgZ and x^ = 36)
for F
ree
(iv) D = {x:xeZ and x>-10|

SOLUTION (i) A = |x:xeZandx^ 5x+6 = 0| = 12,3}


So, is a finite set
Your

(ii) B = (x:xgZ and x^ is even} = - 6, - 4, - 2, 0, 2, 4, 6,...}


ks
eBoo

Clearly, B is an infinite set.


(iii) C = {x:xeZ and x^ = 36)= |6,-6)
ad
our

Clearly, 2Us a finite set.


(iv) D = (x:xeZ and x>-10} = {-9, - 8,-7,...}
Clearly, D is an infinite set.
Re

EXERCISE 6.3
Y

1. Which of the following are examples of empty set ?


Find

(i) Set of all even natural numbers divisible by 5.


(ii) Set of all even prime numbers,
(iii) {x:x^-2 = 0 and x is rational}.
(iv) (x: X is a natural number, x < 8 and simultaneously x > 12}.
(v) (x : X is a point common to any two parallel lines }.
Which of the following sets are finite and which are infinite ?
(i) Set of concentric circles in a plane, (ii) Set of letters of the English Alphabets.
(iii) {x e N : X > 5} (iv) (x eN :x< 200}
(v) |x G Z : X < 5} (vi) (x G R : 0 < X < 1}.
3, Which of the following sets are equal ?
(i) ^ = {1,2,3} (ii) B = {x G R ; x^ - 2x + 1=0)
(iii) C= (1,2,2, 31 (iv) D = (x G R : x^ - 6x^ + llx -6 = 0).
6.10 APPLIED MATHEMATICS-XI

Are the following sets equal ?


A = U : a: is a letter in the word reap}, B = {at : a: is a letter in the word paper},
C = {a: : AT is a letter in the word rope).
From the sets given below, pair the equivalent sets:
A = {1, 2, 3), B = {t, p, q, r, sj, C = (a, p, y}, D = [a, e, i, o, i/}.
Are the following pairs of sets equal ? Give reasons,
(i) A =|2, 3), B = {x:xis a solution of -t*5x + 6 = 0)
(ii) A = {x: X is a letter of the word "WOLF" },
B = {x: X is a letter of the word "FOLLOW" )
From the sets given below, select equal sets and equivalent sets.
A = {0,fl),B = {1,2, 3,4) C- 14,8,12), D = (3,1, 2, 4),£ = (1, 0), f = (8, 4,12}
G = (1,5,7,11),
Which of the following sets are equal?

w
A = {x:xeN,x<3}, B = {1,2}, C = {3,1},
D= {x:xeN, xisodd,x<5}, E = {1,2,1,!}, F = {1,1, 3}.

F lo
Show that the set of letters needed to spell "CATARACT" and the set of letters needed to
spell "TRACT" are equal.

e
ANSWERS

Fre
(iii), (iv), (v) (i) Infinite (ii) finite (iii) Infinite (iv) Finite
for
(v) Infirute (vi) Infinite.
A = C = D ■ No 3. A,C;B,D - (i) No (ii) Yes
Equal sets :B=D,C=E Equivalent sets \A,E,H; B,D,G;C,E
r
You

A=B=E, C=D = F
oks
eBo

H!NTS TO SELECTED PROBLEMS


(ii) We have,
A - {x : X is a letter of the word "WOLF") = {W, O, L, F}
ad
our

B = {x : X is a letter of the word "FOLLOW") = {W, O, L, F]


Clearly, A = B.
A = Set of letters of the word "CATARACT" = [A, C, R, T]
B = Set of letters of the word "TRACT" = [A, C,R,T\
dY
Re

Clearly, A = B.
Fin

6 ● SUBSETS

SUBSETS Let A and B be two sets. If even/ element of A is an element ofB, then A is called a subset ofB.
If A is a subset of B, we write A c B, which is read as "A is a subset of B" or "A is contained in B".
Thus, A e B iff
a e A => a sB.

The symbol "=>" stands for "implies".


If A is a subset of B, we say that B contains A or, B is a super set of A and we write B d A.
If A is not a subset of B, we write A ^ B.
Obviously, every set is a subset of itself and the empty set is subset of every set. A subset A of a
set B is called a proper subset of B if A B and we write A c B. In such a case, we also say that B is a
super set of A. An improper subset is a subset containing every element of the original set. A
proper subset contains some but not all of the elements of the original set. The empty set is a
proper subset of a given set.
SETS 6.11

Thus, if A is a proper subset of B, then there exists an element .y e B such that x € A.


It follows immediately from this definition and the definition of equal sets that two sets A and B
are equal iff A c B and B c A.
Thus, whenever it is to be proved that two sets A and B are equal, we must prove that A c B and
Be A,

ILLUSIRATION 1 Clearly /lie H, 2,31, but {1,4) a 11. 2,31


ILLUSTRATION 2 Clearly, Ncz ZczQ c R c: C, where N, Z, Q, R and C have their usual meanings.
ILLUSTRATION 3 If A is the sct of all divisors of 68 and B is the set of all prime divisors of 68, then B is
the subset of A and we write B e A.
6.4.1 SOME RESULTS ON SUBSETS

THEOREM 1 Every set is a subset of itself


PROOF Let A be any set. Then, each element of A is clearly in A itself. Hence, A c A.
THEOREM 2 The empty set is a subset of every set.

w
PROOF Let Abe any set and <j» be the empty set. In order to show that(j)c A, we must show that
every element of 4) is an element of A also. But, (|)Contains no element. So, every element of4)is
in A. Hence, 4> c A.

F lo
THEOREM 3 The total number of subsets of a finite set containing n elements is 2”.
PROOF LetA be a finite set containing » elements. Let 0 <r <J7. Consider those subsets of A that

e
Fre
have r elements each. We know that the number of ways in which r elements can be chosen out
of n elements is ● Therefore, the number of subsets of A having r elements each is . Hence,
for
the total number of subsets of A is

"Cq + "Cl + ”C2 +... + "c„ = (1 + 1)" = 2‘ [Using binomial theorem]


r
ILLUSTRATIO.N 1 Two finite sets have m and n elements. The total number of subsets of the first set is 56
You
oks

more than the total number of subsets of the second set. Find the values of m and n.
eBo

SOLUTION Let A and B be two sets having m and n elements respectively. Then,
Number of subsets of set A = 2"', Number of subsets of set B = 2".
It is given that.
ad
our

2"' - 2" = 56

1) = 2^ (2^-1)
m -11
=> 2" (2
=> n = 3 and m-n = 3
Re
dY

=> n-3 and m = 6.


Fin

6.4.2 SUBSETS OF THE SET R OF REAL NUMBERS

Following sets are important subsets of the set R of all real numbers:
(1) The set of all natural numbers N = {1, 2, 3, 4,5, 6,....}
(ii) The set of all integers Z = {... - 3, - 2, -1, 0,1, 2, 3, ...1
(iii) The set of all rational numbers Q = \ x: x = — , m, n e Z, n ^ o\.
n

(iv) The set of all irrational numbers. It is denoted by T.


Thus, T = |Y:Yel? and YgQ).
Clearly, NczZczQciR,T(zR and N (tT.
6.4.3 INTERVALS AS SUBSETS OF/?

On real line various types of infinite subsets are designated as intervals as defined below:
CLOSED INTERVAL Let a and b be tzvo given real numbers such that a <b. Then, the set of all real
numbers x such that a< x <b is called a closed interval and is denoted by la, bj.
6.12 APPLIED MATHEMATICS-XI

Thus, [a,b] -{x & R :a < X <b\.

00 + 00
a b

Fig. 6.1
On the real line, [a, b] may be graphed as shown in Fig. 6.1
Forexample, [-1, 2] ={.V6K: -1 < .r < 2} is the set of all real numbers lyingbetween -land2
including the end points. Clearly, it is an infinite subset of R.
jPEN interval If a and b are two real numbers such that a <b, then the set of all real numbers x
satisfying a<x <b is called an open interval and is denoted by (a, b) or ] a,b!.
Thus, (a,b) = {x € K : rt < .y <f)}
■o
— 00 + cc
a b

Fig. 6.2

w
On the real line, {a, b) may be graphed as shown in Fig. 6.2.
Here, encircling a and b means that a and h are not included in the set.

F lo
For example, (1,2) = {y e K : 1 < y < 2} is the set of all real numbers lying between 1 and 2
excluding the end-points 1 and 2. This is an infinite subset of R.
SEMI-OPEN OR SEMI-CLOSED INTERVAL }f a and b are two real numbers such that a <b, then the

ee
sets {(7,Z7] = {Yel? :<7<Y<f’l and [^7,fc) = {y eR :<7<y<f5} are known as semi-open or se7ni-closed

Fr
intervals, (a, b] and fa, b) are also denoted by la, bj and la, b ( respectively.
On real line these sets may be graphed as shown in Figs. 6.3 and 6.4 respectively.
for
The number i;-<7 is called the length of any of the intervals (fl, b), [a,b], [a, b) and{a,b].
ur
— CO + CO
b
s

— cc n b + oo a
ook
Yo

Fig. 6.3 Fig. 6.4


eB

These notations provide an alternative way of designating the subsets of the set R of all real
numbers. For example, the interval [0, co) denotes the set R'*' of all non-negative real numbers,
our
ad

while the interval (- co, 0) denotes the set R”of all negative real numbers. The interval (-x, x)
denotes the set R of all real numbers.

6.5 UNIVERSAL SET


dY
Re

In any discussion in set theory, there always happens to be a set that contains all sets under
consideration i.e. it is a super set of each of the given sets. Such a set is called the universal set
Fin

and is denoted by li.


Thus, a set that contains all sets in a given context is called the universal set.
II l.USTRATlo.s When zve study two dimensional coordinate geometry, then the set of all points in
xy-plane is the universal set.
ILLUSTK.ATK)!'' When we are using sets containing natural numbers, then N is the universal set.
ILLUSTRATION If A = {1,2,3}, B = {2, 4,5,61 and C = {J, 3,5, 7), then U = (1,2,3,4,5,6, 7} can be
taken as the universal set.

: u. SI KA1 i( '\ When we are using intervals on real line, the set R of real numbers is taken as the
universal set.

6.6 POWER SET

POWER SET Let A be a set. Then the collection or family of all subsets of A is called the power set of A
and is denoted by P (A).
SETS 6.13

That is, P(/\) = IS : S c A).


Since the empty set and the set A itself are subsets of A and are therefore elements of P(A). Thus,
the power set of a given set is always non-empty.
. V;:- Let A= {1,2,3}. Then,the subsets of A are: ^, {!}, {21, {3(, U/2), {1,3}, |2, 3)
and {1,2,3). Hence, P(A) = (^, {1}, [2], {3}, (1,2}, {1,3}, {2. 3}, (1, 2, 3}}.
ll.i.l'HTK.-'11::\' If A is thevoid set then P(A) has just one element ^i.e. P(<^ = {^}.
ll.LlM'K.Ai :● Show that n [P [P (P (^)]\ =4.
SOLUTION We have,
P{<t')={‘W
p{pm = \^ i(i>ii
p[p{p((},))i = {(iii(i)),(i(i)}i, (civ mi
Hence, P [P (P ((})))] consists of 4 elements i.e. « (P [ P {P {({)))] } = 4.

w
REMARK We know that a set having n elements has 2" subsets. Therefore, if A is a finite set having n
elements, then P(A) has 2” elements.

F lo
11.1 I '^TR.-\ . u / s. If A = {a, ibi ),find P (A).
SOLUTION Let B = jl?}. Then, A = [a, B).
P(A) ={^,{a},{B},\a,B}\ = [^, {«!, {{&!},{«, (Mil-

ee
Fr
ILLUSTRATIVE EXAMPLES

\Niri i; i Consider the following sets:


for
A=-\\,2\, B = {1, 4, 8},C ={1, 2, 4, 6, 8}.
Insert the correct symbol c: or <r betweei^ each of the follotving pair of sets:
r
(i) (f»...B (ii) A... B (iii) A...C (iv) B...C
You
s
ook

SOLUTION (i) Since null set is subset of every set. Therefore, ^ .c B.


(ii) Clearly, 2eA but 2 g B. So, A ct B.
eB

(iii) Since all elements of set A are in C and A ^C. So, A <zC.
(iv) Clearly, all elements of set B are in set C and 6 C. So, B c C.
r.XAMri.L 2 Let A ={a, b, c, d], B = [a,b, c] andC ={b, d\. Find all sets X such that:
our
ad

(i) X c BandX cz C (ii) X (z A and X ct B.


SOLUTION (i) We have,
dY

P(A) = {^,{a],{b],{c\,.... .), P (B) = {a\, [b], [c\, {a, b], {a, c], {b, c], [a, b, c]]
Re

and. P(C) = l(t>Hbj,ldl,tb,dll


Fin

Now, X c B and X c C

XeP(B) and XeP(C)


Xe{4),|M}
X = 4.,IM
(ii) We have.
X cz A and X <t B
X is a subset of A but X is not a subset of B
XeP(A) butXgP(B)
X = [d\, \a, b, d], {b, c, d], \a, c, dl, {a, d\, {b, d\,\c, d\, {a,b, c, d].
Let A, B and C be three sets. If A sB and 6 c C, is it true that A .cC? If not give an
example.
SOLUTION Consider the following sets: A ={o}, B ={|ff|, I?} and C = {{fl}, fj, c).
Clearly, AeB and 6 <z C. But, ActCasaeA but o g C. Thus, the given statement is not true.
6.14 APPLIED MATHEMATICS-XI

i.XAMPi.r. ● Let A ={1, 2, 3, 4), B = {1, 2, 3] and C = {2, 4}. Find all sets X satisfying each pair of
conditions:

(i) Xc 6 and XtrC (ii) X c B, X B andX cz C (iii) X c /4, X c: B and X c: C.


SOLUTION (i) We have,
X c B and X (Z C
X is a subset of B but X is not a subset of C
XeP{B) butX sB(C)
X = ID, {3}, (1,2}, {1,3}, {2, 3},|1,2, 3}
(ii) We have.
X c B, X 6 and X cz C
X is a subset of B other than B itself and X is not a subset of C
XeP(B),XgP(C)andX^B
X = ID, 13}, {1,2}, (1,3), (2, 3}
(iii) We have.

w
X .a A, X .c B and X c C
X e P (/I), X e P (B) and X e P (C)
X is a subset of A, B and C

F lo
=>

X = <}),(2).
iIXAMPLE =!
Let B he a subset of a set A and let P {A : B) =[X e P {A) -.X z:> B}.

ee
Fr
(i) Shozv that: P (A:^) =P (^}
(ii) If A -\a, b, c, d] and B = {a, b]. List all the members of the set P {A : B).
for
SOLUTION (i) We have,
r
P(A:B) = [XeP{A):X^B] = {X g P (^): B c Xj
You

= Set of all those subsets of A which contain 6


s
ook

P (A:^) = Set of all those subsets of A which contain


eB

= Set of all subsets of set A = P (A),


(ii) TfA = {a,b,c,d]andB = {rt, b}. Then,
P (A : B) = Set of all those subsets of set A which contain B
our
ad

= {a, b}, {a, h, c|, \a,b, d], \a,b, c, d}


f XAMri I- Prove that A c (|) implies A = ({).
dY

SOLUTION We know that two sets A and B are equal iff A c B and B c A. Also, we know that
Re

(}i c A
Fin

and. A c (j) [Given]


A =(|)
rxAMPLCr
In each of the following, determine whether the statement is true or false. If it is true, prove
it. If if is false, give an example.
(i) If A' G A and A e B, then x eB (ii) If A .c B and B eC, then A eC
(iii) //A c B and BczC,thenA(^C (iv) IfAzB and B <zC, then A zC
(v) Ifx € A and A (Z B, then x e B (vi) If A c B and x e B, then x e A
SOLUTION (i) False:
Consider sets A = {1} and, B={|l|/2}.
Clearly IgA and A g B, but 1 g B. So, x e A and A g B need not imply that x eB.
(ii) False:
Let A = {!}/ B = {1, 2} and C = {{1, 2), 3}. Then, we observe that A c B and B eC but
A gC.
Thus, A c B and B g C need not imply that A g C.
SETS 6.15

(iii) True:
Let X .€A. Then,
A (.z B ^ X eB => X eC BcC]
Thus, xsA^ X eC for all A'6/4. So, AciC.
Hence, Acz B and B <zC => A zC.
(iv) False:
Let A = {1,2}, B={2, 3) and C=|l,2,5). Then,/I qc B and B C. But, A c C.
Thus, A <z B and B cr C need not imply that A <z C.
(v) False:
Let A ={1, 2} and B =|2, 3, 4,5). Then, we observethat 1 e A and AcrB,butlgB.
Thus, X eA and A <2 6 need not imply that x e B.
(vi) True:
Let A c B. Then, we observe that
xeA => xeB xeB => x € A.

w
EXAMPLE 8 Write the following subsets of R as intervals:

F lo
(i) {x:.r € -4 <X <6} (ii) lx:xeR,-12<x<-10}
(iii) {.y:xgB,0<.v<7| (iv) {x:xeR, 3<x<4}.
Also, find the length of each interval.

ee
Fr
SOLUTION (i) {.T:.veR,-4<x<6) = (-4, 61-Length =6-(-4) = 10
(ii) Ix : X e B, -12 <X < -10} = (-12, -10). Length = -10 - (-12) = 2
(iii) |x : X e K, 0 <x <7} = [0, 7). Length =7 - 0 =7 for
ur
(iv) (x: X eR, 3 <x <4| = [3, 4]. Length =4 - 3 =1

Write the folloiuing intervals in the set-builder form:


s
EXAMPLE 9
ook
Yo

(i) (-7,0) (ii) [6,12] (iii) (6,12] (iv) [-20, 3)


SOLUTION (i) (-7, 0) ={x:x eR and -7<x<0|
eB

(ii) [6,12] = {x: X e R and 6<x<12(


(iii) (6,12] = {x: X e R and 6 <x <12}
r

(iv) [- 20, 3) = (x: X e R and - 20 < x < 3}


ad
ou

EXAMPLEio LetA = {1, 2, {3, 4), 5). Which of the folloiuing statements are incorrect and zvhy?
Y

(i) {3,4} c A (ii) 13,4}eA (iii) {{3,4}} c A (iv) 1 eA


(viii) {1, 2, 3} c A
Re

(v) 1 c A (vi) |1,2,5) cA (vii) 11,2,5} eA


nd

(ix) (|)gA (x) 4) c A (xi) 14>}ciA


Fi

SOLUTION {3,4} is an element of set A. Therefore, (3, 4) eA is correct and {3,4} cA is


incorrect.

So, (i) is incorrect and (ii) is correct. As {3, 4} is an element of set A. Tl^erefore, {{3, 4)} is a set
containing element | 3,4 } which belongs to A. So, {{3, 4}} c A.
Hence, (iii) is correct.
Since 1 is an element of set A. So, 1 e A is correct and 1 c A is incorrect.
So, (iv) is correct and (v) is incorrect.
Since 1,2,5 are elements of set A, Therefore, {1, 2,5} is a subset of set A.
Hence, (vi) is correct and (vii) is incorrect.
As 3 is not an element of set A. So, {1, 2, 3} c A is incorrect. The null set is subset of every set.
So, (}) c A is correct and (}i e A is incorrect. Hence, (ix) is incorrect and (x) is correct.
As 4) c A but {4>} is not a subset of A. So, (xi) is incorrect.
6.16 APPLIED MATHEMATICS-XI

EXERCISE 6.4
Which of the following statements are true ? Give reason to support your answer,
(i) For any two sets A and B either Aq B or Bq A.
(ii) Every subset of an infinite set is infinite,
(iii) Every subset of a finite set is finite,
(iv) Every set has a proper subset,

w
(v) {a, b, a, b, a, h,...} is an infinite set.
(vi) [a, b, c} and {1,2,3} are equivalent sets,
(vii) A set can have infinitely many subsets.
State whether the following statements are true or false:

e
(i) le 11,2,31 (ii) a c {b, c, fl| (iii) {«} e [a, b, c}

e
(iv) {a, b} = \a, a, b, b, a\ (v) The set {.r: x + 8 = 8} is the null set.

or
r
Decide among the following sets, which are subsets of which:

F
A = {:r: .t satisfies - 8a: + 12 = 0}, B = {2,4,6} , C = (2, 4, 6, 8,....}, D = {6}.

oF
4. Write which of the following statements are true ? Justify your answer,

ul
(i) The set of all integers is contained in the set of all rational numbers,
(ii) The set of all crows is contained in the set of all birds,

s
r
(iii) The set of all rectangles is contained in the set of all squares,

ko
(iv) The set of all real numbers is contained in the set of all complex numbers,
(v) The sets P = |<?} and B = {{«}} are equal,
(vi) The sets A = j.v: a: is a letter of the word 'XITTLE")
of
o
Y
and, B = {a: : a: is a letter of the word "TITLE" ) are equal.
rB

5. Which of the following statements are correct? Write a correct form of each of the incorrect
eY

statements,

(i) a c {a, b, c) (ii) (<?} e {a, b, c } (iii) ae[{a],b]


(iv) {«lc{|fl|,/?) (v) {b,c]ci{a,{b,c}} (vi) [a,b}:z\a,\b,c\]
u

(vii) (J)e {a,b} (viii) i^<z{a,b,c] (ix) {.r: a: + 3 = 3} = (|)


d

6. Let A = {a, b, |c, d], e]. Wliich of the following statements are false and why?
o
ad

(i) ic. d]^A (ii) {c, d\ eA (iii) He, d}l cA


in

(iv) a eA (v) rt c A (vi) {a, b, e\ c: A


(vii) [a,b,e]EA (viii) \a,b, c} c: A (ix) 4) e A
Re

(x) {(|)} c A
F

7. Let A = {(1/2, 31, {4,5}, (6,7, 8}|. Determine which of the following is true or false:
(i) IeA (ii) {1,2,31c A (iii) {6,7, 8} eA
(iv) ({4,5)1 c A (v) <j) E A (vi) 4>^^
Let A = {(j), {4>1,1, {1, 4)1, 2). Which of the following are true?
(i) 4) 6 A (ii) 14.1 eA (iii) {lie A
(iv) {2, (H c A (v) 2c A (vi) {2, {111c A
(vii) {{2), {111c A (viii) (4), {4)1, {1,4>11 c A (ix) H4)}lcA.
Write down all possible subsets of each of the following sets:
(i) Ifll (ii) {0,11 (iii) {a, b, c }
(iv) {1,{1}) (V) (4>)
1(1. Write down all possible proper subsets each of the following sets:
(i) {1,21 (ii) {1,2,31 (iii) m
What is the total number of proper subsets of a set consisting of ?i elements?
If A is any set, prove that: A c 4><=> A = 4>-
SETS 6.17

13. Prove that: Ac B, Be C and C c A => A = C.


How many elements has P (A), if A = 4>?
What universal set (s) would you propose for each of the following:
(i) The set of right triangles. (ii) The set of isosceles triangles.
ANSWERS

(i) F,A={lr2,3},B = {a,b] (ii) F, A = (1, 2} is a finite subset of N.


(iii) T (iv) F, (|> does not have a proper subset
(v) F, Givenset = (rt, (vi) T (vii) F
(i) T (ii) F (iii) F (iv) T (V) F 3. D cz A c B cC

(i) T (ii) T (iii) F (iv) T (V) F (Vi) T


(i) a£[ci,b,c] (ii) [a]c{a,b,c] (iii) In)
(iv) {1(7)1 (v) {b,c]e{a,{b,c\] (vi) \a, \b, c|i

w
(vii) (viii) ^c{a,b,c] (ix) {.V:.V + 3 = 3| <J)

F lo
6. (i) F (ii) T (iii) T (iv) T (V) F (Vi) T
(vii) F (viii) F (ix) F (X) F
(i) F (ii) F (iii) T (iv) T (V) F (Vi) T

ree
(i) T (ii) T (iii) F (iv) T (V) F (vi) T (vii) T
(viii) T (ix) T
for F
(i) <1), («1 (ii) 10), HI, {0,11 (iii) (j), {a], {b\, {c\, {a, b\, [b, c), |<7, c}, {a, b, c]
(iv) ({.,{11, mil, II, {111 (v) ({.,1(1.1.
Your

(i) ({),{1|,(2| (ii) ({.,{11, (21, {31, {1,21, {2, 31, {1,31 (iii) ^
ks
eBoo

Ii. 2" -1 14. 1

(i)The setof all triangles in a plane. (ii) The set of all triangles in a plane.
ad
our

6.7 VENN DIAGRAMS

Sometimes pictures are very helpful in our thinking. First of all a Swiss mathematician Euler
gave an idea to represent a set by the points in a closed curve. Later on British mathematician
John-Venn (1834-1883) brought this idea to practice. That is why the diagrams drawn to
Re
Y

represent sets are called Veiw-Euler diagrams or simply Verm-diagrams. In Verm-diagrams the
Find

universal set 11 is represented by points within a rectangle and its subsets are represented by
points in closed curves (usually circles) within the rectangle. If a set A is a subset of a set B, then
the circle representing A is drawn inside the circle representing B as shown in Fig. 6.5 (i). If A and
B are not equal but they have some common elements, then to represent A and B we draw two
intersecting circles. (See Fig. 6.5 (ii)). Two disjoint sets are represented by two non-intersecting
circles. (See Fig. 6.5 (iii))
/I

Fig. 6.5 (i) Fig. 6.5 (ii) Fig. 6.5 (iii)


6.18 APPLIED MATHEMATICS-XI

6.8 OPERATIONS ON SETS

In this section, we shall introduce some operations on sets to construct new sets from given
ones.

UNION OF SETS Let A and B be tzvo sets. The union of A and B is the set of all those elements which
belong either to A or to B or to both A and B.
We shall use the notation AuB (read as "A union B”) to denote the union of A and B.
Thus, AKjB={x:xeAorxGB].
Clearly, x eA'uB x e /I or x sB.
And, A* s A u 5 <=> a g A and a g 6.
In Fig. 6.6 the shaded part represents A u B. It is evident from the definition that A c A u B,
Be AuB.

w
F lo
Fig. 6.6

If A and B are two sets such that A c B, then A u B = B. Also, A u B = A, if B c A.

e
I1.LUSTKATION 1
Fre
//A = {1,2, 3} and B = {1, 3,5, 7j, then AuB = {l,2, 3, 5, 7}.
for
!I I.USTRA170N 2 If A = lx: X = 2/1 + 3, n e Z/ and B = {x: x = 2n, neZj, then
A u B = (a : A is an odd integer} u {a : a is an even integer) = {a : a is an integer} = 2.
r
NOTE If Aj, A2, ..., A„ is a finitefainily of sets, then their union is denoted bi/ u A,-
You
oks

j = 1
or, A2 UA2 UA3 ... uA n ●
eBo

ILLLSTR.ATION > Let A = {1, 2, 3}, B = (3,5},C = {4, 7, 8}. Then,A u B uC ={1, 2, 3, 4,5, 7, 8}
INTERSECTION OF SETS Let A and B be two sets. The intersection of A and B is the set of all those
ad

elements that belong to both A and B.


our

The intersection of A and B is denoted by A n B (read as "A intersection B").


Thus, A n B = {a : A e A and a e B}.
Clearly, xeAnB o asA and an B
Re
dY

In Fig. 6.7 the shaded region represents A n 6. Evidently, AnBcA, AnBcB.


Fin

Fig. 6.7
If A and S are two sets, then An6 = A, if AcB and A n B = B, if B <z A.
.NOTE //Aj, A2,..., A„ is a finite famili/ of sets, then their intersection is denoted bi/
n

r\^Aj or, A] nA2 n... n A„.


ILLUSTRA nON 4 If A = |1, 2, 3, 4, 5} and B = {1, 3, 9,12), then A n B = {1, 3).
ILLUSTR.\TI0N 5 If A = {1, 2, 3, 4,5, 6, 7), B = {2, 4, 6, 8,10} and C =|4, 6,7, 8, 9,10,11), then
A n B = {2, 4, 6). Therefore, A n B n C = {4, 6}.
SETS 6.19

ii I! STRATION h If A = Ix: X = 2n,n e Zjand B = lx: x = 3n,n e Z}, then


/4 n B = {x: ;c = 2n,n eZ] n {x:x = 3n,n e Z]
= {..., -4, -1,0,2,4,6, ...1 n 1..., -9, -6, -3,0,3,6,9,... }
= (..., -6, 0, 6,12,...} = {x : X = 6n, n g Zj.
ILLUSTRATION 7 If A = ( X : X = 3n,n gZ} and B = { x : x = 4n,n eZ }, then find A nB.
SOLUTION Clearly,
X gA nB
<=> X = 3n and x = 4n, n gZ
X is a multiple of 3 and x is a multiple of 4
X is a multiple of 3 and 4 both
<=>
X is a multiple of 12.
«● X = 12n, n gZ
Hence, A nB = [x: x = 12n,n gZ\.
DISJOINT SETS Tzvo sets A and B are said to be disjoint, if AnB = ^

w
If A n B ^ then A and B are said to be intersecting or overlapping sets.
u

F lo
ee
Fr
Fig. 6.8 for
ur
11.LUSTRATION 8 If A = {1,2,3,4,5,6}, B = {7,8,9,10,11 land C = 16,8,10,12,141, then A and B are
disjoint sets, zohile A and C are intersecting sets.
s
ook

DIFFERENCE OF SETS Let A and B be tivo sets. The difference of A and B, ivrittenasA-B, is the set of
Yo

all those elements of A zuhich do not belong to B.


Thus, A - B = {x; X e /I and x g B}
eB

or. A-B={xgA:x^B}.
Clearly, x gA - B xeA and x g B.
r

In Fig. 6.9, the shaded part represents A- B.


ou
ad

Similarly, the difference B - A is the set of all those elements of 6


that do not belong to A i.e. 6 - A = {x g B: x g A}.
Y

Fig. 6.9
In Fig. 6.10, the shaded part represents B - A.
Re
nd
Fi

Fig. 6.10
ILLUS'I'RATION 9 If A = (2, 3, 4,5, 6, 7} and B = {3, 5, 7, 9,11,131, then A-B = {2,4, 61 and
B-A = {9,11, 13}.
SYMMETRIC DIFFERENCE OF TWO SETS Let A and B be tzoo sets. The symmetric difference of sets
A and B is the set (A - 6) u (B - A) azzd is denoted by AAB.

Fig. 6.11
6.20 APPLIED MATHEMATiCS-X!

Thus, A AB ={A-B)u{B-A)= {.v:.v « n B).


The shaded part in Fig. 6.11 represents A AB.
) 11 f!- -s li If A = (1, 2, 3, 4, 5, 6, 7, 8f and B = {1,3,5,6,7,8,9}, then A-B = {2, 4},
B-A = {9I
AAB = {2,4,9].
ILLUSTRATION J1 if A = {x ^ R : 0 < X < 3} and B = (x e R : J <x < 5j, then
v4-B = [xeR:0<3:<l], B-^ = {.reR:3<x<5)
and. A AB = {x gR : 0 <A- <1| n |x e R : 3 <.v < 5} = {x eR : 0 <x <1 or 3 <x <5}.
COMPLEMENT OF A SET Let U be the universal set and let A be a set such that A<zU. Then, the

w
complement of A with respect to U is denoted by A' orA‘^orll-A and is defined the set of all those
elements of LI which are not in A.

o
e
re
rFl
F
Fig. 6.12

or
ou
Thus A' = |x G U: X g A\. Clearly, x e /I' <=> x g A.
ksf
'
:?rK.MlON 12 Let the set of natural numbers N = {1,2,3, 4, ...I be the universal set and let
( I .

A = (2, 4, 6, 8, ...j. Then A' = {1, 3, 5,...}.


oo

; LLUSTKATION 1.1 IfU = {1,2, 3,4, 5, 6, 7, 8, 9} and A = {1,3,5, 7, 9}, then = {2, 4, 6, 8}.
Y
B

Following results are direct consequences of the definition of the complement of a set:
(i) W = |x e U : X g U} = ^
re

(ii) f = {xGU:xgii)l = (i
(hi) {A')' = {xeU:xgA') = |xeU:X6^}=7l
oYu
ad

(iv) = {xeU:xGA\u{xeLl:x^A\=Ll
(v) A r\A' = (x € Lf: X e A} n {x s U: x g A) = <j).
d

EXERCISE u...
in
Re

1. If /4 and 6 are two sets such that A (z B, then Find:


F

(i) 71 n 6 (ii) A^B


- If 71 = 11, 2, 3, 4,5], B = {4,5, 6, 7, 8|, C ={7, 8, 9,10,11) and D = {10,11,12,13,14}. Find:
(i) AuB (ii) 7l uC (hi) BuC
(iv) BuD (v) 7l u 6 u C (vi) 7l w B u D
(vii) 6 uC uD (viii) 7l n(B uC) (ix) (TlnB)n(BnC)
(x) (7l uD) n(B uC).
■’ Let 7l = {x:x G N}, B = {x: X =2n, n g N}, C ={x : x =2n -1, n g N] and, D = (x: x is a prime
natural number}. Find:
(i) 71 nB (ii) 71 nC (hi) t1 n D
(iv) BnC (v) BnD (vi) C n D
4. Let 7l={3, 6,12,15,18, 21}, B = {4, 8,12,16, 20j, C = {2, 4, 6, 8,10,12,14,16} and
D = {5,10,15, 20}. Find:
(i) M'-S (ii) 71-C (hi) 71-0
SETS 6.21

(iv) B-A (V) C-A (vi) D~A


(vii) B-c (viii) B-D
Let U={1, 2, 3, 4,5, 6, 7, 8, 9}, ^ = {1, 2, 3, 4}, B = |2, 4, 6, 8} and C = {3, 4, 5, 6|. Find:
(i) 71' (ii) B- (iii) {AnCy
(iv) {AuBy (V) {A'y (vi) {B-cy
Let L7 ={1,2, 3,4,5, 6,7, 8, 9j,yl ={2, 4, 6, 8) andS=|2, 3,5, 7). Verify that:
(i) {A u By = 71' n B' (ii) {An By = A’uB'.
ANSWERC

(i) A (ii) B
(i) {1,2, 3,4,5,6,7,81 (ii) {1,2, 3,4,5,7,8,9,10,11)

w
(iii) {4,5,6,7,8,9,10,11} (iv) {4,5,6,7,8,10,11,12,13,14}
(v) {1,2, 3,4,5,6,7,8,9,10,11} (vi) {1, 2, 3, 4,5, 6, 7, 8,10,11,12,13,14)

F lo
(vii) {4,5, 6,7, 8, 9,10,11,12,13,14} (viii) {4,51 (ix) ^ (x) {4,5,10,11}
(i) B (ii) C (iii) D

ee
(iv) ^ (V) 12} (vi) D-{2]

Fr
4. (i) {3,6,15,18,21} (ii) {3,15,18,21} (iii) {3,6,12,18,21}
(iv) {4,8,16,20} (v) {2,4,8,10,14,16} (vi) {5,10,20}
(vii) {20} (viii) {4,8,12,16} for
ur
. (i) 15,6,7,8,9) (ii) jl, 3,5,7,91 (iii) {1,2,5,6,7,8,91
(iv) {5,7,9} (V) 71 (vi) {1, 3,4,5,6,7,91
ks
Yo

6.9 LAWS OF ALGEBRA OF SETS


oo

In this section, we shall state and prove some fundamental laws of algebra of sets.
eB

THEOREM 1 (Idempotent Lazos) For any set A,


(i) 7l U7l =7l (ii) AnA=A.
r
ou

(i) AuA = {.V: .r e .A or x e 71} = {.t : .r e /!} = A


ad

(ii) .A n 7l = (.r : X e A and x 6 A} = {x: x e A) = A.


Y

THEOREM 2 (identity Lazos) For any set A,


(i) A u(})= A (ii) A nii =A.
nd
Re

i.e. (j) and U are identity elements for union and intersection respectively.
Fi

r
(i) A U(|) = {x: X € A or X € (}i} ={x : X e A} = A
(ii) A nU = [x :x eA and x sU] = {x: x e A] = A
THEOREMS (Commutative Lazos) For any tzoo sets A and B
(i) A u B = B u A (ii) A n B = B nA
i.e. union and intersection are commutative.

lU,
Recall that two sets X and Y are equal iff X c Y and Y c X. Also,X c Y if every element
.

of X belongs to Y.
(i) Let X be an arbitrary element of A u B. Then,
xeAuB=> xeA or xeB=> xeBorxeA=i> xeBuA
AkjBc BkjA.

Similarly, B A^ Akj B.
Hence, A 'u B = B 'O A.
6.22 APPLIED MATHEMATICS-XI

THEOREM 4 (Associative Lmios) I/A,B and C are any three sets, then
(i) {A^B)^C=Au{BuC) (ii) A n{B nC) ={A n B) nC
i.e. union and intersection are associative.

PROOI- (i) Let .t be an arbitrary element of u 6) u C. Then,


.r e (A 6) vj C
X € (A u B) or .V € C
(x e A or X € B) or X e C
X G A or (x 6 B or X € C)

ow
X e A or X g(B uC)
X G A u(B uC)
(AuB)uCc Au(BuC).
Similarly, A u {B u C) c (A u B) l; C.

e
re
Hence, (A w B) uC = A w{B uC).
(ii) Let X be an arbitrary element of A ri (B n C). Then,

Frl
F
X G A n(6 nC)
=> X G A and x g(B nC)
ou
X G A and (x € B and x g C)

sor
(x G A and x € B) and x g C
X G (A n B) and x g C kf
X € (A n B) n C
oo
An(BnC)c(AnB)nC.
Y

Similarly, (A r\ B) n C c A n (B n C).
B

Hence, A n(B nC) =(A n B) nC.


THEOREMS (Distributive Laws) lfA,B and C are any three sets, then
re
oY

(i) Au(BnC)=(AuB)n(AuC) (ii) An(BuC)=(AnB)u(AnC)


u

i.e. union and intersection are distributive over intersection


and union respectively.
ad

I’ROOI (i) Let A'be an arbitrary element of A u (B nC). Then,


d

X eA\j{B r\C)
in

X G A or X G (B n C)
Re

X 6 A or (x G B and x g C)
F

=> (x G A or X G B) and (x g A or x g C) [●.● 'or' is distributive over 'and']


X 6 (A u B) and x g (A u C)
X € ((A u B) n (A u C))
Au(BnC) c (AuB)n(AuC)
Similarly, (A u B) n {A u C) c A u (B n C).
Hence, AuiBnC) = (A u B) n(A uC).
(ii) Let X be an arbitrary element of A n(B uC). Then,
X G A n(B uC)
X G A and x g(B uC)
=> X G A and (x 6 B or x g C)
(x G A and x g B) or (x g A and x g C)
X G (An B) or x g (A n C)
X 6 (A n B) u (A n C)
An(BuC) c (AnB)u(AnC)
SETS 6.23

Similarly, (A u B) u (/4 n C) c A n (J5 u C).


Hence, A n(6 uC) =(A n B) u(A nC).
THEOREM 6 (De-Morgmi's Laws) If A and B are any two sets, then
(i) (A^B)' =A' nB' (ii) (AnB)'=A'uB'.
PROOF (i) Let a: be an arbitrary element of (A u B)'. Then,
xe(AuB)'
=* .r e (A u B)
^ X ^ A and € 6
^ X eA' and x eB'
=> xeA'nB'.

(A n B)' c A' n B'.


Again, let y be an arbitrary element of A' n B'. Then,
y e A' n B'
^ y e A' and y e 6'

w
=> y e A and y « B

F lo
^ y g A u B.
^ y€(AuB)'
A'nB'c(AuB)'

ee
Hence, (A u B)'= A'n B'.

Fr
(ii) Let x be an arbitrary element of (A n B)'. Then,
Are(AnB)' for
a: g (A n B)
ur
=> a: g A or a: g B
X g A' or a: g B’
s
ook

X g A' u B'
Yo

=3*

(A n B)' c A' u B'.


eB

Again, let y be an arbitrary element of A' u B'. Then,


y g A' u B’
our
ad

=> y gA'or yeB'


=> ygAorygB
y & {A r\B)
dY

=> ij^iAnB)'
Re

A' u B' Q (An B)'.


Fin

Hence, (A n B)' = A' u 6'.


ILLUSTRATIVE EXAMPLES

l:\AMPU; I Ifa^N sucfi that aN = {flx: x g N). Describe the set 3N ,n7N.
SOLUTION We have, a N = |rtx: x g N|
3iV = 13x : X g Nt = {3, 6, 9,12,...,} and, 7N = {7x : x g N| = {7,14, 21, 28,.. .|
Hence, 3N n 7N = (21,42, ...| = {21x: x g N| = 21N.
liXAMPlE .? If A = {1,3, 5, 7,11,13,15,17}, B = 12,4, 6,..., 18} and N is the universal set, then find
A'u((AuB)nB').
SOLUTION Clearly, {AuB)r\B' =A [●.● A, B are disjoint sets]
A'u ((A u B) n B') = A'u A = N.
l:\AMrii For any natural number a, zoe define aN = {ax:x gN]. If b, c, d eN such that
bN kjcN = dN, then prove that d is the l.c.m. of b and c.
6.24 APPLIED MATHEMATICS-XI

SOLUTION We have,
bN =[bx:x eN} = The set of positive integral multiples of b
cN = {cx:x eN] = The set of positive integral multiples of c
bN ncN = The set of positive integral multiples of b and c both.
bN ncN = {/c.x: .v e N], where k is the l.c.m. of b and c.
Hence, d = l.c.m. of b and c.
HXAMPLf- 4 Suppose A-^, A2, ■■■ A;^q are thirty sets each with five elements and Bi, 82, ■■■, B,j are n
30 n
sets each with three elements. Let U U B: = S. Assume that each element of S belongs to
/=! /-I

exactly ten of the Ay's and exactly 9 ofBj's. Find n.


SOLUTION Since each Ay has 5 elements and each element of S belongs to exactly 10 of A^'s.
3U 1 30

ow
S =
U 'A =>
! =1
=:^ Z
10 ',-1
— (5 X 30) = 15
10

Again, each Bj has 3 elements and each element of S belongs to exactly 9 of Bj's
S =
U Bj =>n(S) = I^ j=i
Z "(Bj) = Lsn) = ^3 ...(ii)

e
1=1 . 9

Fl
re
From (i) and (ii), we get : 15 = — ^ n= 45.-

F
3

EXAMPLES Let A, B and C be the sets such that A 'u B = A uCandA nB = AnC. Show that B = C.
ur
SOLUTION We have,
or
sf
A'uB = A uC

{A u B) n C = (A u C) n C
k
Yo
oo

(AnC)u(BnC) = C Iv(AuC)nC=C]
(AnB)u(BnC) = C [.■AnC=Ar^B] ...(i)
B

Again, A u B = A uC
re

(A u B) n 6 = {A ^C)nB
B = (AnB)uiC nB) [●.● (A u B) n B = B]
u
ad
Yo

B = {A r\ B) u{B nC) ●●●(ii)


From (i) and (ii), we get B = C.
d

EXERCISE 6.6
Re
in

1. Find the smallest set A such that A u {1, 2) = {1, 2, 3,5, 9}.
2. Let A = {1,2,4, 5} 6 = {2,3,5, 6} C = {4,5, 6, 7}. Verify the following identities:
F

(i) Au(BnC)=(AuB)n(AuC) (ii) A n(B uC) =(A n B) u(A nC)


(iii) A niB-C)=(A nB)-{AnC) (iv) A-(B uC) =(A - B) r^(A-C)
(v) A-(BnC)=(A-B)u(A-C) (vi) A n (6 AC) =(A n B) A (A n C).
3. If Ii = {2, 3, 5, 7, 9) is the universal set and A = |3, 7}, B = {2, 5, 7, 9|, then prove that:
(i) (AuB)’=A'nB' (ii) (AnB)'=A'uB'.
20
4. EachsetX,contains5elementsandeachsetY,contains2elementsand U =S= U Yj-.M
r = l r = l

each element of S belongs to exactly 10 of the and to exactly 4 of then find the value
of n.

ANSWERS

-1. A = {3,5,91 7.A=(1,2),B={1, 3},C={2, 3) 4. 20


SETS 6.25

6.10 SOME IMPORTANT RESULTS ON NUMBER OF ELEMENTS IN SETS

If A, B and C are finite sets, and U be the finite universal set, then
(i) u{AKjB)=niA) + n{B)-n{An,B)
(ii) n (y4 u B) = n(/l) + o A, Bare disjoint non-void sets,
(iii) n (A - B) = n(.A) -n{A r\ B) i.e. n (/4 - 6) + n (A r\B) = n{A)
(iv) n (A A B) = No. of elements which belong to exactly one of or B
= »((^-B)u(B-A))
= n{A-B) + n{B-A) [●.● (A - B) and (B - A) are disjoint]
= - n (A n B) + u {B)-ii{AnB)

ow
= n{A) + n{B)-2niAr^B)
(v) )i (A u B u C) = u{A) + n (B) + )f{C) - »(A n B) - n(B n C) - »(A n C) + /i (A n 6 n C)
(vi) Number of elements in exactly two of the sets A, B, C
= »? (A n B) + «(B n C) + «(C n A) - 3 it{A n B n C).

e
Fl
re
(vii) Number of elements in exactly one of the sets A, B, C
= n{A) + n (B) + n(C) -2?! (A n B) - 2n (B nC) - 2ii (A nC) + 3n(A n B nC)

F
(viii) n (A' u B') = n((A n 6)') = n {(J) - n{A n B)
ur
or
(ix) n (A' o B') =;i((A u B)') -n {(J) -n (A u 6). sf
ILLUSTRATIVE EXAMPLES
k
Yo
EXAMPLE 1 If X and Y are tzvo sets such flint n (X) = 17, n (V) = 23 mid n (X u Y) = 38, find
oo

n (XnY).
SOLUTION We know that
eB

ti(XuY) = n{X)+n{Y)-n{XnY) => 38 = 17 + 23-?i (X n Y) => m (X n Y) =40 - 38 = 2.


EXAMPLE 2 Ifx and Y are two sets such that n{X) = 45,n(XuY) =76andn{XnY) = 12, findn{Y).
ur

SOLUTION We know that


ad

fi(XuY)=»(X) + n(Y)-n(XnY) => 76 = 45 + » (Y)-12 => n(Y)=43


Yo

Hence, fi(Y)=43.
;-:xAMPLE3 IfA and Bare tzvo sets such thatn{A) =17, n(B) = 23 and n{A <jB) = 38, find
d

(i) ;i(A nB) (ii) n{A -B) (iii) n(B-A) (iv) mimberofelementsisex actlyoneofAnndB.
Re
in

SOLUTION (i) We know that


F

n{A n 6) =n(A) + n(6) -;i(A uB) => »(A n B) =17 + 23 - 38 = 2


(ii) We know that
fi(A-B)=n(A)-n(AnB) ^ ;i(A - B) =17-2 =15
(iii) We know tliat
j7(B-A)=;i(B)-n(AnB) => n (B - A) = 23-2 = 21
(iv) We know that
Number of elements in exactly one of A and B = 17(A) + h(B) - 2)? (A n B) = 17 + 23 - 2 x 2 = 36
EXAMPLE 4 Let A and B be tzvo sets such that n(A) = 35, n{A n B) = 11 and n ((A u B)’) = 17. If
n(U)=57,find
(i) »(B) (ii) n(A-B) (iii) ;r(B-A)
SOLUTION (i) We have,
f!((AuB)')=17
=> ;i(L7)-n(AuB)=17
^ 57-h(AuB)=17
6.26 APPLIED MATHEMATICS-XI

=> «(/luB)=57-17=40
=> n{A)+n{B)-n{AnB)=40 ^ 35 + h(B)-11 =40 => «{B) =16
(ii) We know that
n{A-B)=n(A)-n{Ar^B) => ii(7l-B) = 35-11 = 24
(iii) We know that
n{B-A)=n{B)-n(AnB)^ n(B-A) =16-11 =5
I XAMFLC 5 Let A and B be two sets and U be the universal set such that n{A) = 25, n{B) = 28 and
n(U) =50.fmrf
(i) the greatest value of n(A u B) (ii) the least value of n{An\B).
SOLUTION (i) We know that
AuBc U => n(AuB) <n(U) => n(AuB) <50

w
Hence, the greatest value of n(A u B) is 50.
(ii) From (i), we have

Flo
n(/luB)<50 n{/l)+Ji{B)-«{2lnB)<50 => 25 +28-«(>1 n B) <50 =>n (AnB)>3
Hence, the least value of n(A n B) is 3.

e
FXAMPLn6 If A and B are two sets and U is the universal set such that n(U) = 700, n(A) = 200,

re
n(B) = 300 and n(Ar^B)= 100. Find n {A' n B').

F
SOLUTION We have. A' nB'= {A kjB)'
n{A'nB')= n{{A^ B)') =n(U)-n(A u B)
ur
r
Thus, «(A'nB')= tt(U)-[»(2l)+«(B)-n(2lnB)] =700-(200 + 300-100) = 300.
fo
EXAMPI.n 7
In a group of800 people, 550 can speak Hindi and 450 can speak English. How many can
ks
speak both Hindi and English?
Yo
oo

SOLUTION Let H denote the set of people speaking Hindi and E denote the set of people
speaking English. We are given that: n {H) = 550, n (E) = 450 and n (H u £) = 800 and we have
eB

to find «(H n £).


Now, n{H uE) = n{H)+n{E)-n{H r\E)
ur

^ n(H nE) = n{H)+n{E)-n{HuE) = 550 + 450-800 = 200.


ad

Hence, 200 persons can speak both Hindi and English.


Yo

EXAMPLES
In a class of 35 students, 24 like to play cricket and 16 like to play football. Also, each
student likes to play at least one of the tzvo games. How many students like to play both cricket and
d
Re

football?
in

SOLUTION Let C be the set of students who like to play cricket and F be the set of students who
F

like to play football. Then, C u F is the set of students who like to play at least one game and,
C n f is the set of all students who like to play both games. It is given that n (C) = 24, n (F) = 16,
tt (C u F) = 35 and we have to find n (C n F).
Now, n(C<jF) = n{C) + n{F)-n(C nF)
=> n{C nF)=n(C) + M(F)-?i(CuF) =24 + 16-35=5.
EXAMPLE 9
In a group of 50 people, 35 speak Hindi, 25 speak both English and Hindi and all the people
speak at least one of the two languages. How many people speak only English and not Hindi ? How many
people speak English?
SOLUTION Let H denote the set of people speaking Hindi and E the set of people speaking
English. Then, it is given that: n{H u £) = 50, n (H) = 35, k (H n £) = 25.
Now, h(£-H) = «(HuE)-?i(H) = 50- 35 = 15
Thus, the number of people speaking English but not Hindi is 15.
Now, «(H u £) = n (H) + M (E) -n (H n E) => 50 = 35 + rt (E) - 25 => »(£) = 40
Hence, the number of people who speak English is 40.
SETS 6.27

nxAMPLn 10 There are 40 students in a Chemistry class and 60 students in a Physics class. Find the
number of students which are either in Physics class or Chemistry class in the follotving cases:
(i) the two classes meet at the same hour.
(ii) the two classes meet at different hours and 20 students are enrolled in both the subjects.
SOLUTION Let A be the set of students in Chemistry class and B be the set of students in Physics
class. It is given that n (>1) = 40 and n (B) = 60. We have to find w (A u 6) in both the cases,
(i) If two classes meet at the same hour, then there will not be a common student sitting in both
the classes. Therefore, n{A r\B) = 0.
n{AuB) = niA) + n{B)-n{AnB) =40 + 60-0 = 100
(ii) If two classes meet at different timings then there can be some students attending both the
classes. It is given that the number of such students is 20 i.e. n{A nB) = 20.
n{AuB) = n (/I) + n (B)-n{Ar^B) = 40 + 60-20 = 80.

w
i;XAMPLE 11 In a survey of 700 students in a college, 180 were listed as drinking Limca, 275 as
drinking Miranda and 95 were listed as both drhiking Limca as well as Miranda. Find how many students

F lo
were drinking neither Limca nor Miranda.
SOLUTION Let U be the set of all surveyed students, A denote the set of students drinking Limca

ee
and B be the set students drinking Miranda. It is given that n{U) =700, n{A) =180, n{B) =275

Fr
and n {A nB) - 95. We have to find n {A' n B').
Now, f7 (71'nB') = n{Au By = n{U)-n{AuB)= n{U)-{n(A) + n{B)-n{A nB)]
=> n (71' n B') = 700 - (180 + 275 - 95) = 700 - 360 = 340.
for
ur
EXAMPLE 12 There are 200 individuals with a skin disorder, 120 has been exposed to chemical Cy 50 to
chemical C2 and 30 to both the chemicals Cj and C^- Find the number of individuals exposed to
ks
(i) chemical Cl or chemical C2 (ii) chemical Cj but not chemical C2 (in) chemical C2 but not
Yo

chemical C-\.
oo

SOLUTION Let U denote the universal set consisting of individuals suffereing from the skin
eB

disorder, 7l denote the set of individuals exposed to chemical and B denote the set of
individuals exposed to chemical C2. It is given that: n (LI) = 200,/i(7l) =120, n(B) =50 and
r

n (AnB) = 30.
ou
ad

(i) The number of individuals exposed to chemical Cj or chemical C2 is given by n (t1 u B).
Y

Now, n(A^B) = n (A) + n (B) (7l n B) = 120 + 50 - 30 = 140


Hence, required number of individuals is 140.
nd
Re

(ii) The number of individuals exposed to chemical Cj but not chemical C2 is given by «(AnB).
Now, n{AnB)= n{A)-n{AnB)= 120-30 = 90
Fi

Hence, required number of individuals is 90.


(iii) The number of individuals exposed to chemical C2 but not chemical is given by
n (AnB).
Now, n (A n B) = n (6) -n (A n B) = 50 - 30 = 20
Hence, required number is 20.
EXAMPLE 13 Out of500 car owners investigated, 400 owned Mariiti car and 200 ozoned Hyundai car;
50 owned both cars. Is this data correct?

SOLUTION Let U be the set of all car owners investigated, M be the set of persons who owned
Maruti cars and H be the set of persons who owned Hyundai cars. It is given that n (U) = 500,
n (M) =400, n{H)= 200 and n (M n H) =50.
Now, fi (M u H) = n (M) + n (H)-n{MnH)= 400 + 200 -50 = 550
But, M\jH c H.
6.28 APPLIED MATHEMATICS-XI

n (M u H) < M (U) =s> n (M u H) < 500


This is a contradication. So, the given data is incorrect.
I-XAMPLE14 In a class of 35 students, 17 have taken Mathematics, 10 have taken Mathematics but not
Economics. Find the number of students who have taken both Mathematics and Economics and the
number of students who have taken Economics but not Mathematics, if it is given that each student has
taken either Mathematics or Economics or both.

SOLUTION Let A denote the set of students who have taken Mathematics and B be the set of
students who have taken Economics. It is given that n (AuB) = 35, n (/4) = 17 and n(A -B) = 10.
Now, n(A)=n(A-B)+n(AnB) =>17 ==10 + n (A n B) => n (A n B) = 7
Thus, 7 students have taken both Mathematics and Economics.
Now, n(AuB) = n(A)+n(B)-n(AnB) => 35 = 17+«(B)-7 => n(B) = 25
Also, n(B) = n{B-A)+n(AnB) => 25 = n(B-A)+7 => n(B-A) = 18.
Thus, 18 students have taken Economics but not Mathematics.

w
i:XAMPLE 15 In a town of 10,000 families it zoas found that 40% families buy newspaper A, 20%
families buy neiospaper B and 10% families buy nezvspaper C. 5% families buy A and B, 3% buy B and C

F lo
and 4% buy A and C. If 2% families buy all the three news papers, ifnd the number offamilies zuhich buy
(i) A only (ii) B only (Hi) none of A, B and C.

ee
SOLUTION Let P, Q and R denote the sets of families buying newspaper A, B and C

Fr
respectively. Let L/ be the universal set. Then,
n{P) = 40% of 10,000 = 4000, n(Q) = 20% of 10,000 = 2000, n{R) = 10% of 10,000 = 1000,
for
)i(P n Q) = 5% of 10,000 = 500, n (Q n R) =3% of 10,000 = 300, n {R n P) =4% of 10,000 = 400
ur
n(PnQnR)= 2% of 10,000 = 200 and n(U) = 10,000.
oks

(i) Required number = n{P nQ' n R') ^n (P n(Q ^ R)')


Yo

= n(P)-n[PniQ<jR)\ [v n{AnB')=n{A)-n{AnB}]
o
eB

= n{P)-n[{PnQ)u{PnR)]
= n{P) -[n{P nQ)+n(Pr^R)-n {(P nQ) n(P n R)] ]
our

= n(P) -[«(P nQ)+n{P nR)-n(PnQnR)]


ad

= 4000 - {500 + 400 - 200) = 3300


(ii) Required number = n (P' n Q r\ R') = n (Q n P' n R') = n (Q n (P u R)')
Y
Re

= n(Q)-«(Qn(PuR)) [ ●.● n{A nB') =n(A)-n(A n B)]


nd

= n{Q)-n[{QnP)u(QnR)]
Fi

= 4Q)-[n(QnP)+n{QnR)-n{(Qr,P)n{Qr^ R)\]
= j;(Q) - [»(R ^ Q) + n (Q n R) -«(P o Q n R) ]
= 2000 - (500 + 300 - 200) = 1400
(iii) Required number = n (P' n Q' n R') = [ (P u Q u R)' ]
= n (li) - » (P u Q u R)
= n{U)- [»(P) + n(Q) + n(R) ~n(P n Q) - w (Q n R)
-n(R n P) + n{P r\Q R)]
= 10000 - [4000 + 2000 + 1000 - 500 - 300 - 400 + 200] = 4000.
ALITER It is given that n(P r\Q r\ R) = 200 and n (P nQ) =500. So, the number of families
buying newspaper A and B only is 500 - 200 = 300. Similarly, the number of families buying
news papers B and C only is 300 - 200 = 100 and news papers C and A only is 400 - 200 = 200.
SETS 6.29

Fig. 6.13

w
(i) It is given that 4000 families buy news paper A. So, the number of families buying news
paper A only = 4000 -(300 + 200 + 200) = 3300.

Flo
(ii) Similarly,
The number of families buying news paper B only = 2000 -(300 + 200 +100) = 1400

e
The number of families buying news paper C only = 1000 -(200 + 200 +100) = 500

re
(iii) The number of families buying none of the news papers

F
= n (U) -n{P\jQ<jR) =10000 - (3300 + 300 + 200 + 200 +100 +1400 + 500) = 4000
ur
A college awarded 38 medals in Football, 15 in Basketball and 20 to Cricket. If these medals

r
LXAMPLE16
fo
zvent ton total of 58 men and only three men got medals in all the three sports, how many received medals
in exactly two of the three sports ?
ks
SOLUTION Let f denote the set of men who received medals in Football, B the set of men who
Yo
oo

who received medals in Cricket. It is given


received medals in Basketball and C the set of men
that «(^') = 38,»(B)=15,ii{C)=20, n(FuBuC) =58 and n(FnBnC) = 3.
B

Now,
re

n(F uBuC) =n(F) + n (B) + n (C) -n(F r\ B) -n(B nC) -n(F nC) + n(F r) B nC)
58 = 38 + 15 + 20-»(FnB) -»(B nC)-n (F nC) + 3
u
ad

=> /j{FnB)+»(BnC)+n{FnC)=76-58 = 18
Yo

Number of men who received medals in exactly two of the three sports
= n(F n B) + «(B nC) + n(F nC) - 3 »(F n B nC) = 18 - 3 x 3= 9
d
Re

Thus, 9 men received medals in exactly two of the three sports.


in

EXAMPLE 17
In a sun>ey of 25 students, it zvas found that 15 had taken Mathematics, 12 had taken
F

Physics and 11 had taken Chemistry, 5 had taken Mathematics and Chemistry, 9 had taken Mathematics
and Physics, 4 had taken Physics and Cheznistry and 3 had taken all the three subjects. Find the number of
students that had taken

(i) only Chemistry. (ii) only Mathematics.


(iii) only Physics. (iv) Physics and Chemistry but not Mathematics.
(v) Mathe/natics and Physics but not Chemistry, (vi) only one of the subjects.
(vii) at least one of the three subjects. (viii) none of the subjects.
SOLUTION Let M denote the set of students who had taken Mathematics, P the set of students
who had taken Physics and C the set of students who had taken Chemistry. It is given that
)i(U) -25,j7(M) =15,»(F) -12,?[(C)=11, n(MnC) =5,n (M n P)= 9, n (F nC)=4
and, n (M nP nC) = 3
(i) Number of students who had opted Chemistry only
= n(M' nP'nC) = n((M u P)'nC)
6.30 APPLIED MATHEMATICS-XI

= n(C)-n((M u P) nC) [v Jj{AnB')=n{A)-n{Ar\B)]


= ?!(C)-»((MnC)u(PnC)
= w(C)-[?!(MnC)+!i(PnC)-7z(MnPnC)l= ll-(5 + 4-3) = 5
(ii) The number of students who had opted Mathematics only.
= n{M nP' nC')
= n(M n{P nC)')
= n (M) - »(M n (P u C))
= n (M) - )i ((M n P) u (M n C))
= H (M) - {n (M n P) + n (M n C) - »{M n P n C) 1 = 15 - (9 + 5 - 3) = 4
(iii) The number of students who had opted Physics only
= n{P dM' nC')
= »(P n(M uC)')

w
= n{P)-n(P n{MuC))
= n(P)-tt((PnA4)u(PnC))

F lo
= H(f’)-('i(PnM) + /i(PnC)-ji(PnMnC)l= 12-(9 + 4-3) =2.
(iv) Required number of students = 7i{P nC r\M')

ee
= n{P nC)-n{P nC nM)

Fr
= 4-3=1 [v n{Ar\B')=n{A)-n{AnB)]
(v) Required number of students = «(M n P nC') = n{M n P) ^n{M n P nC) = 9-3 = 6
for
(vi) Required number of students
ur
= n{M) +u (P) + h(C) -2 {n(Mr\P)-hn{PnC)+n{M nC) + 3n{M n P nC)}
= 15 + 12 + 11 - 2(9 +4+5) + 3x 3 = 38- 36 + 9 = 11
s
ook
Yo

(vii) Required number of students = «(MuPuC)


eB

= n (M) + « (P) + n (C) - ?i (M n P) - {P nC) -}i{M r\C) +ti{M nP ,n C)


= 15 + 12 + 11 -9-4-5 + 3 = 23

(viii) Required number of students


our
ad

= j7(M' nP' nC')= n{MuPuCy= ti{U) -;i(M uP uC) = 25-23 = 2.


MITER 1
Consider the Venn diagram shown in Fig. 6.14. Let/?, b, c, d, e, f, g denote the number
dY

of students in the respective regions.


Re
Fin

M P U

Fig. 6.14

From the Venn-daigram, we have


n (M) = a + b + d + e,n{P) = b + c + e + = d + e + f + g,
SETS 6.31

«(M n P) = b + en{P nC) = e + f n{M r\C) = d + e and, /j (M n P nC) = e


It is given that
n (M nPnC) = 3=> e = 3
n{M r\P) =9=> b + e = 9^ b + 3 = 9 => b = 6

«{PnC)=4=>^+/=4=> 3+/=4=>/=l
H (M nC) = 5=> d + e = 5 ^ t/+3=5=> d = 2
n (M) = 15 => a + b + d + e =15 => a + 6 + 2 + 3 = 15 => a = 4
«(P)=12=> b + c + e+f =12^ 6 + C+3 + 1 =12=> c = 2
?7(C) = 11=> d + e + f + g = 11 ■=> 2+3 + l+ g=ll=>^=5
Thus, we have a = A,b =6, c =2, d = 2, e = 3, f = 1 and ^ = 5.
Now,

w
(i) Required number of students = g = 5
(ii) Required number of students = a = A
(iii) Required number of students = c = 2
(iv) Required
(V) Required
(Vi) Required
(vii) Required
(viii) Required
number
number
number
number
number
of students
of students
of students
of students
of students
=
=
=
=
=
F lo
/ = 1
b = 6

ree
fl + c+ g= 4 + 2 + 5=ll
for F
a + b + c + d + e+ f + g = 23
25 - (a+ b + c + d + e+ f + g)= 25-23 =2.
ALITCR 2
It is given that 3 students had taken all three subjects and 9 had taken Mathematics
Your

and Physics. So, number of students who had taken Mathematics and Physics but not Chemistry
ks

is 6 as shown in the Venn diagram. 5 students had taken Mathematics and Chemistry and 3
eBoo

students had taken all the three subjects. So, 2 students had taken Mathematics and Chemistry
but not Physics. It is given that 15 students had taken Mathematics. So, number of students who
had taken only Mathematics =15 -(6 + 3 + 2) =4.
ad
our

U
Re
Y

4
Find

Fig. 6.15

Similarly, we compute the other values shown in the Venn diagram. It is evident from the Venn
diagram that the number of students that had taken
(i) Chemistry only is 5.
(ii) only Mathematics is 4.
(iii) only Physics is 2.
(iv) Physics and Chemistry but not Mathematics is 1.
6.32 APPLIED MATHEMATICS-XI

(v) Mathematics and Physics but not Chemistry is 6.


(vi) only one of the subjects is 4 + 2 + 5 = 11
(vii) at least one of three subjects is4 + 6 + 2 + 2 + 3 + l+ 5 = 23
(viii) none of the subjects = 25 - 23 = 2.
i:\AMlM.l; 18 In a survey of 100 students, the number of students studying the various languages were
found to be: English only 18, Etiglish but not Hindi 23, English and German 8, English 26, German
48, Gcr}7ian and Hindi 8, no language 24. Find the number of students who were studying (i) Hindi
(ii) English and Hindi (Hi) English, Hindi and German.
SOLUTION Let E, H and G be the sets of students studying English, Hindi and German
respectively. Let LI be the set of students surveyed i.e. the universal set.
In the above Venn diagram, let a,b, c, d, e,f and g denote the number of students in the
respective regions.

w
U

F lo
ee
Fr
for
r
You
s
ook

Fig. 6.16
eB

Clearly, n (U) = 100. It is given that:


Number of students studying English only = 18 => fl = 18
Number of students studying English but not Hindi =23 => a + e = 23
our
ad

Number of students studying English and German = 8 => ^+^ = 8


Number of students studying English =26 ^ a + d + e + g = 26
Number of students studying German =48=» c + e-i-g + f = 48
dY
Re

Number of students studying German and Hindi = 8 ^ g+f = 8


Thus, we obtain a= 18, a+e = 23, e+g = 8, a + e+g + d = 26, e+g + f + c = 48 and g+f = 8
Fin

=> a =18, e =5, g = 3, d =0, f =5, c = 35


It is given that 24 students study no language. Therefore, the number of students who study at
least one language is 100-24 =76
i.e. n(EuHuG)=76
a + b + c + d + e+f+ g= 76 => 18+ i? +35 + 0+5+5+3= 76 => b =10
(i) The number of students studying Hindi = l7 + d + g+/=10 + 0+ 3 + 5=18
(ii) The number of students studying English and Hindi = d + g = 0 + 3 = 3
(iii) The number of students studying English, Hindi and German =g = 3.
rXAMI’LE 19 In an university, out of 100 students 15 offered Mathematics only; 12 offered Statistics
only; 8 offered Physics only; 40 offered Physics and Mathematics ; 20 offered Physics and Statistics; 10
offered Mathen7atics and Statistics; 65 offered Physics. Find the nwnber of students who
(i) offered Mathematics (ii) offered statistics (iii) did not offer any of the above three subjects.
SOLUTION Let M, S and P be the sets of students who offered Mathematics, Statistics and
Physics respectively. Let x be the number of students who offered all the tliree subjects. It is
given that 10 students offered Mathematics and Statistics. Therefore, number of students who
SETS 6.33

offered Mathematics and Statistics but not Physics is 10 -.r. Similarly, number of students in
different regions are marked in Fig. 6.17.

low
Fig. 6.17

It is given that 65 students offered Physics.


(40-j:) + X + (20-;r) + 8=65=> 68-x = 65 => j: = 3
(i) From Fig. 6.16, we find that

ee
The number of students who offered Mathematics =15 + (10-x) +:r + 40-x

F
Fr
= 65 -a: = 65 - 3 = 62

(ii) The number of students who offered Statistics = 12 + (10 - a:) + a: + (20 - a:)
= 42-.v = 42-3 = 39
for
ur
(iii) The number of students who offered any of three subjects
= 15 + 12 + 8 + (10-A:)+(40-A-)+(20-.r) + A:
ks
= 105-2.r=105-2x3=99
Yo
oo

Number of students who did not offer any of the three subjects = 100 - 99 = 1.
EXAMPLE 20 Out of 280 students in class XII of a school, 335 play Hockey, 110 play football, 80 play
eB

volleyball, 35 of these play hockey and football, 30 play volleyball and hockey, 20 play football and
volleyball. Also, each students plays at least one of the three games. How many students play all the three
r

games?
ou
ad

SOLUTION Let H, F and V be the sets of students who play hockey, football and volleyball
Y

respectively. Let a: be the number of students who play all the three games. It is given that 35
students play hockey and football. So, number of student who play hockey and football only
nd
Re

is (35-a:).
Similarly, the number of students playing various games are written in the regions representing
Fi

them in Fig. 6.18.

Fig. 6.18
6.34 APPLIED MATHEMATICS-XI

It is given that each student plays at least one of the three games.
tt(HuFuV)=280
=> {70 + x)+(35-x) + (30-j:) + jf + {20-.v)+(55 + 3:) + (30 + x) =280 => 240 + x = 280 => >: = 40
Hence, 40 students play all the three games.
liXAMIM.nzl From 50 students taking examination in Mathematics, Phi/sics and Chemistry, 37 passed
Mathematic, 24 Physics and 43 Chemistry. At most 19 passed Mathematics and Physics; at most 29
passed Mathematics and Chemistry and at most 20 passed Physics and Chemistry. If each student has
passed in at least one of the subjects,find the largest number of students who could have passed in all the
three subjects.
SOLUTION Let M, P and C be the sets of students passing in Mathematics, Physics and
Chemistry respectively. It is given that
n{M uP uC) =50, n (M) = 37, n{P) = 24, «(C) = 43, ;t(M nP) <19, n (M nC) < 29

w
and n (P nC) < 20
We know that

Flo
tt(MuPuC)=«(M) + «(P) + »{C)-w(MnP)-tt(MnC)-«(PnC) + «(MnPnC)
50 = 37 + 24 + 43-{»{MnP) + fj{MnC) + n(PnC)} + H(MnPnC)

e
re
^ 50 =104-{?i{MnP) + »(M oC)+h(P nC)l +« (M nP nC)
^ 54 + n(M nP nC) =n{M nP) +n{M riC) + n{P nC)

F
=> 54+ji(MnPnC) <19+29 + 20
ur
r
=> H{MnPnC) <14 [●.● n (MnP)< 19, n (M nC) < 29, n (P nC) < 20]
fo
Hence, the largest number of students that could have passed in all the three subjects is 14.
ks
i;x AMPU-: 22 A school awarded 58 medals for Honesty, 20 for Punctuality and 25 for Obedience. If these
Yo

medals were bagged by a total of 78 students and only 5 students got meadalsfor all the three values, find
oo

the number of students who received medals for exactly two of the three values.
B

SOLUTION Let H, P and O be the sets of students who


bagged medals in Honesty, Punctuality
re

and Obedience respectively. It is given that n{H) =58, n{P) - 20, «(0) =25, n{H u P uO) = 78
and n(H nP nO) =5.
u
ad
Yo
d
Re
in
F

Fig. 6.19

Let X denote the number of students who got medals in Honesty and Punctuality only, y denote
the number of students who got medals in Honesty and Obedience only and z denote the
number of students who got medals in Punctuality and Obedience only.
Now, /i(HuPuO)=78
=> n(H) + «(P) + h(0) ~ n{H n P)-n{P nO) -n {H nO)+n{H nP nO) =78
=> 58 + 20 + 25 -n (H n P) (P n O) -«(H n O) + 5 =78
SETS 6.35

=> n{H nP) + n{P nO) + n{H nO) =30


(A- + 5) + (y+ 5)+ (z + 5) = 30 => x-¥y + z = \5
Hence, required number of students = :c + y + z=15
EXERCISES.?
1.
If A and Bare two sets such that«{^ u B) =50, «(A) = 28 and n(B) = 32, find n {A n B).
If? and Q are two sets such that P has 40 elements,P uQ has 60 elementsand P n Qhas 10
elements, how many elements does Q have?
3. In a school there are 20 teachers who teach mathematics or physics. Of these, 12 teach
mathematics and 4 teach physics and mathematics. How many teach physics?

ow
4.
In a group of 70 people, 37 like coffee, 52 like tea and each person likes at least one of the
two drinks. How many like both coffee and tea?
5.
Let A and B be two sets such that: n (^4) = 20, m (A u B) = 42 and ii (/I n B) = 4. Find
(i) n(B) (ii) n{A-B) (hi) n(B-A)

e
Fl
re
6.
A survey shows that 76% of the Indians like oranges, whereas 62% like bananas. What
percentage of the Indians like both oranges and bananas?

F
In a group of 950 persons, 750 can speak Hindi and 460 can speak English. Find:
ur
r
(i) how many can speak both Hindi and English
fo
(ii) how many can speak Hindi only (iii) how many can speak English only.
8.
In a group of 50 persons, 14 drink tea but not coffee and 30 drink tea. Find:
ks
Yo
(i) how may drink tea and coffee both (ii) how many drink coffee but not tea.
oo

9.
In a survey of 60 people, it was found that 25 people read newspaper H, 26 read newspaper
B

T, 26 read newspaper I, 9 read both H and /, 11 read both H and T, 8 read both T and /, 3
read all three newspapers. Find:
re

(i) the numbers of people who read at least one of the newspapers,
(ii) the number of people who read exactly one newspaper.
u
ad
Yo

10. Of the members of three athletic teams in a certain school, 21 are in the basketball team, 26
in hockey team and 29 in the football team. 14 play hockey and basket ball, 15 play hockey
and football, 12 play football and basketball and 8 play all the three games. How many
d
Re

members are there in all?


in

n.
In a group of 1000 people, there are 750 who can speak Hindi and 400 who can speak
F

Bengali. How many can speak Hindi only? How many can speak Bengali ? How many can
speak both Hindi and Bengali?
A survey of 500 television viewers produced the following information; 285 watch football,
195 watch hockey, 115 watch basketball, 45 watch football and basketball, 70 watch football
and hockey, 50 watch hockey and basketball, 50 do not watch any of the three games. How
many watch all the three games? How many watch exactly one of the three games?
i.v
In a survey of 100 persons it was found that 28 read magazine A, 30 read magazine 6, 42
read magazine C, 8 read magazines A and B, 10 read magazines A and C, 5 read magazines
B and C and 3 read all the three magazines. Find:
(i) How many read none of three magazines?
(ii) How many read magazine C orUy?
14.
In a survey of 100 students, the number of students studying the various languages were
found to be : English only 18, English but not Hindi 23, English and Sanskrit 8, English 26,
Sanskrit 48, Sanskrit and Hindi 8, no language 24. Find:
6.36 APPLIED MATHEMATICS-XI

(i) How many students were studying Hindi?


(ii) How many students were studying English and Hindi?
15. In a survey it was found that 21 persons liked product P^, 26 liked product ?2 and 29 liked
product P3. If 14 persons liked products and P2; 12 persons liked product P3 and P^; 14
persons liked products P2 and P3 and 8 liked all the three products. Find how many liked
product P3 only.
ANSWERS

1. 10 30 3.12 4. 19

ow
5. (i)26 (ii) 16 (iii) 22 6. 38% 7. (i) 260 (ii) 490 (iii) 200
8. (i) 16 (ii) 20 9. (i) 52 (ii) 30 10. 43 n. (i)600 (ii) 250 (iii) 150
12. 20, 325 n. (i) 20 (ii) 30 14. (i) 18 (ii) 3 15. 11

e
HINTS TO SELECTED PROBLEMS

re
5. (i) 11 {A<j B)= n{A) + n{B) - it {A n B) => n{B) = 26

Frl
F
(ii) n {A - B) = n(A) -ii{A nB) => n {A -B) = 16
(iii) n{B-A)=n{B)-it{Ar.B)^22
ou
r
7. (iii) Let A and B denote the sets of persons who can speak Hindi and English respectively.
Then, 11 (A u B) = 950 , =750 and n{B) = 460.
so
kf
(i) n (A nB)= n(/\) + n{ B) - (A u B) = 750 + 460 - 950 = 260
(ii) Required number = n{A-B) = «(A) - n{A n B)
oo

(iii) Required number =n(B -A) =/j(B) -n{A n B).


Y
eB

8. (ii) Let A and 6 be sets of persons who drink tea and coffee respectively. Then
n (A u B) =50, n {A - B) = 14, »(2l) = 30.
ur

(i) n(A-B) =14 => n{A)-n{A r^B) =14=>n{AnB) =n{A) -14 = 30-14=16
oY

(ii) Required number = «(B - A) =n(B)-»(/I n B).


ad

Now, It (A u B) =»(A) + j!(6) -ti{A n B) => 50 = 30 + n(B) -16 => (i(6) = 36.
d

)i(6-A)=«(B)-»(7lnB)=> fi(B-A) = 36-16=20


in

10. Let A, B and C be the sets of members of basketball, hockey and football teams respectively.
Re

Then, f7(A)=21, »(6)=26, n{C)=29, ji(AnB)=14, «(6 nC) =15, n(A nC) =12
F

and n (A n B n C) = 8.
Required number = »(A u 6 u C)
= n(A) + »(B) + »(C) -»(A n B) -«(B nC)-n{AnC) +ii{A nB nC).
11. Let A and B be the sets of persons who can speak Hindi and Bengali respectively. Then,
»(AuS)=1000, n(A)=750and fi(6)=400.
No. of persons who can speak Hindi only =;?(A - B) = n(A) -n{A n B)
No. of persons who can speak Bengali only = n (B - A) =n(B) -n(B n A)
No. of persons who can speak both Hindi and Bengali=n (A n B) =n(A) + n(B) -n(A u B).
● JV = Total number of television viewers = 500, ?j(P)=285, n(H) =195, n(F n B) = 45,
u(FnH)=70, ii(HnB)=50, 11 (F n H' n B') =50.
Now, n (F' n H' n B') =50
=> h[(FuHu6)']=50
=> N-ii(FuHuB)=50
SETS 6.37

=> 500 - HF) + n{H) + n{B) -n{F nH)-n{F r\ B)-n(H nB) + n{F n H 6)] = 50
=> n(FnHnB) =500 - 285-195-115 +70+50 + 45 -50 = 20.
Required number = h (F n H n B) = 20
Required number =n{F ,n H' r\ B') + n {F' nH' nB) +n{F' nH n B')]
= n (f) + H (H) + ji (B) - 2 [?i (f n H) + )j (H n B) + (6 n F)1 + 3 «(F n H n B)
! We have, a =18, a + b = 23, d + e = 8, a + b + d + e = 26, d + e + f + g - 48,
and,a + b + c + d + e+ f + g= 100 -24 = 76
a = 18,b = 0, c = 10,d=5,e = 3,f =5 and, ^ = 35
(i) n(H)=b + c + e+f =18
(ii) n(H nE) =b + e = 3

w
E H

Flo
c

ee
Fr
g

for
ur
Fig. 6.20
k s
Yo
oo

MULTIPLE CHOICE QUESTIONS (MCQs)


eB

Mark the correct alternative in each of the following:

1. For any set/4, (A')'is equal to


r
ou
ad

(a) A' (b) 71 (c) 4) (d) none of these


Y

Let A and B be two sets in the same universal set. Then, A - B =

(a) AnB (b) A' nB (c) AnB' (d) none of these


Re
nd

3. The number of subsets of a set containing n elements is


Fi

(a) n (b) 2" -1 (c) (d) 2'

4. For any two sets A and B, A n (A u 6) =


(a) A (b) B (c) 4) (d) none of these

If A = il, 3, 5, Bj and B = |2, 4], then


(a) 4 e A (b) {4jc:A (c) B c A (d) none of these

6. The symmetric difference of A and B is not equal to


(a) (A-B) n(B-A) (b) (A-B)u(B-A)
(c) (AvjB)-(AnB) (d) 1(A u B) - A} u {(A u B) - B)

The symmetric difference of A = {1,2,3} and B = {3,4,5} is


(a) {1,2} (b) 11,2,4,5) (c) { 4, 3 } (d) {2,5,1,4,31
6.38
APPLIED MATHEMATICS-XI

8. For any two sets A and B, {A-B) u{B-A) =


(a) {A-B)^A (b) {B-A)uB
(c) {AuB)~(AnB) (d) (A u B) n(/4 n B)

g. Which of the following statement is false:


(a) A - B = An B' (b) A-B=A-{AnB)
(c) A-B = A-B' (d) .4-B=(/luB)-B

For any three sets A, B and C


(a) n(B-C) =(AnB)-{A nC) (b) An{B-C) ={AnB)-C
(c) Au(B-C)=(/luB)n(AuC') (d) ^u(B-C)=(/luB)-(^uC).
II. Let A : a: eB, >4} and B ={a: e B : a: <5). Then, n B=
(a) (4,5] (b) (4,5) (c) [4,5) (d) [4,5]

w
]2. Let II be the universal set containing 700 elements. If A, B are sub-sets of U such that
n (A) = 200, n (6) = 300 and n {AnB)= 100. Then, n {A' n B') =
(a) 400 (b) 600

F lo
(c) 300 (d) none of these.

13, Let A and B be two sets such that«(A) =16, «(6) =14, »(A u B) = 25. Then, »(A n B) is

e
equal to

Fre
(a) 30 (b) 50 (c) 5 (d) none of these

UA = {1, 2, 3, 4,5|, then the number of proper subsets ofis


for
(a) 120 (b) 30 (c) 31 (d) 32
r
You

13 In set-builder method the null set is represented by


s
ook

(a) {} (b) O (c) {a: : a: -.d (d) {.y:a: = A'}


eB

1(, li A and B are two disjoint sets, then n{Au B) is equal to


(a) n(7l)+n(6) (b) n{A) + n{B)-n{AnB)
(c) n (i4) + n(B) -¥n{An B) (d) n{A) n{B)
our
ad

For two sets A u 6 = /I iff

(a) Be A (b) A c B (c) A ^ B (d) A = B


dY
Re

If A and B are two sets such that n (A) = 70, n (6) = 60, n (A u B) = 110, then «(A n B) is
equal to
Fin

(a) 240 (b) 50 (c) 40 (d) 20


If A and B are two given sets, then A n (A n B)*^ is equal to
(a) A (b) B (c) O (d) AnB^

If A = {.Y: AT is a multiple of 3) and, B = {y : .y is a multiple of 5), then A - B is


(a) A n B (b) A n B (c) A n B (d) A n B

In a city 20% of the population travels by car, 50% travels by bus and 10% travels by both car
and bus. Then, persons travelling by car or bus is
(a) 80% (b) 40% (c) 60% (d) 70%
If A n B = 6, then
(a) A e B (b) Sc A (c) A = O (d) B = <h
6.39
SETS

23. An investigator interviewed 100 students to determine the performance of three drinks:
milk, coffee and tea. The investigator reported that 10 students take all three drinks milk,
coffee and tea; 20 students take milk and coffee; 25 students take milk and tea; 20 students
take coffee and tea; 12 students take milk only; 5 students take coffee only and 8 students
take tea only. Then the number of students who did not take any of three drinks is
(a) 10 (b) 20 (c)25 (d) 30

24 Two finite sets have m and n elements. The number of elements in the power set of first set is
48 more than the total number of elements in power set of the second set. Then, the values
ofm and « are:
(a) 7,6 (b) 6,3 (c)6,4 (d) 7,4

25. In a class of 175 students the following data shows the number of students opting one or
more subjects. Mathematics 100; Physics 70; Chemistry 40; Mathematics and Physics 30;

w
Mathematics and Chemistry 28; Physics and Chemistry 23; Mathematics, Physics and
Chemistry 18. How many students have offered Mathematics alone?
(a) 35 (b) 48 (c) 60 (d) 22

“ ' 30 fi

F lo
26. Suppose Ai, A9,...,A3o are thirty sets each having 5 elements and B|, - are n sets

ee
each with 3 elements, let U = 0 B: = S and each element of S belongs to exactly 10 of the
/=! j=l^

Fr
A',® and exactly 9 of the then n is equal to
(a) 15 (b) 3 (c) 45
for (d) 35
ur
27 Two finite sets have m and n elements. The number of subsets of the first set is 112 more than
that of the second. The values of m and n are respectively
s
ook
Yo

(a) 4, 7 (b) 7,4 (c) 4,4 (d) 7,7


eB

2S. For any two sets A and B, A n{AuB)’ is equal to


(a) A (b) B (c) 4> (d) AnB
29. Theset(AuB')'^(BnC)isequalto
r
ou
ad

(a) A'uBuC (b) A'uB (c) A'uC (d) A'nB


30 Let Fj be the set of all parallelograms, F2 the set of all rectangles, F3 the set of all rhombuses,
Y

F4 the set of all squares and the set of trapeziums in a plane. Then Fj may be equal to
Re
nd

(a) F2 n F3 (b) F3nF4 (c) F2 u F3 (d) F2 u F3 F[


31 IfX = {8" -7w-l:neN}andY ={49n-49:ne/V}.Then,
Fi

(a) X c Y (b) YcX (c)X=Y (d) XnY=^


"^2 A survey shows that 63% of the people watch a News channel whereas 76% watch another
charmel. If x % of the people watch both channel, then
(a) X = 35 (b) x = 63 (c) 39<x<63 (d) x = 39

33. If sets A and B are defined as A = -, 0 X e R L B = {(x, y): y = -X, X e R},then


(a) AnB=A (b) AoB = B (c) A nB (d) AuB=A
20

34 Each setX^ contains 5 elements and each set Y^ contains 2 elements and X^ = S = Y^.If
each element of S belongs to exactly 10 of the ® and to exactly 4 of the Yj. ®, then n is
(a) 10 (b) 20 (c) 100 (d) 50
6.40
APPLIED MATHEMATICS-XI

3.->. Two finite sets have m and n elements respectively. The total number of subsets of first set is
56 more than the total number of subsets of the second set. The value of m and n
respectively are:
{^) 7,6 (b) 5,1 (c) 6,3 (d) 8,7
The set (^4 w6 uC) o(^4 o6' o C')' vjC' is equal to
(a) BnC (b) AnC (c) BuC (d) AnC
37. If A and B are two sets, then n (^4 vj B) equals
(a) 71 (b) B (c) (d) AnB

3N. Let S={x:xisa positive multiple of 3 less than lOOj , P=[x:x is a prime less than 20|.
Then, n(S)+n(P) is

ow
(a) 34 (b) 31 (c) 33 (d) 30
39. In a town of 840 persons, 450 persons read Hindi, 300 read English and 200 both. Then the
number of persons who read neither is
(a) 210 (b) 290 (c) 180 (d) 260

e
40. In a class of 60 students, 25 students play cricket and 20 students play tennis and 10

Fl
re
students play both the games. Then the number of students who play neither is

F
(a) 0 (b) 25 (c) 35 (d) 45
Let S = the set of points inside the square, T = the set of points inside the triangle and C = the
ur
or
set of points inside the circle. If the triangle and circle intersect each other and are contained
in a square. Then,
sf
(a)SnTnC=4> (b)SuTuC=C (c)SuTuC = S (d)SuT=SnC
k
Yo
oo

_ANSWERS

(b) 2, (c) 3. (d) 4. (a) .5. (d) (a) 7. (b) 8. (c) 9. (c)
B

' (a)/(b), (c) II. (b) 7'. (c) 1.3. (c) 14. (c) ■(c) 16. (a) r. (a)
re

18. (d) IM. (d) (b) 2i. (c)


*>A
(b) 2'. (b) 2 !. (c)
1 >

25. (c) 26, (c)


- (b) 2 7 (c) (b) 30. (d) 30. (d) 31. (a) 32. (c) 33. (c) (b)
u
ad

2 (a) 36. (a) 37. (a) 38. (b) 30. (b)


Yo

40. (b) 41. (c)

FILL IN THE BLANKS TYPE QUESTIONS (FBQs)


d
Re

1. U A and B are two finite sets, then n{A) + n(B) is equal to


in

2. If TUs a finite set containing n elements, then the number of subsets of A is


F

The set {.X e R ; 1 < a: < 2} can be written as


If A and B are finite sets such that A cz B, then u {A<uB) =
If A and B are any two sets, then A-Bis equal to
When A = c|), then the number of elements in P{A) is
When A = b, then the number of elements in P (P(A)) is
The power set of set A = {1, 2} is
For all sets A and B, A -(A n B) is equal to
in. For all sets A and B,B-{AnB) is equal to
■ ’ Given the sets A = |1, 3,5j, B = {2, 4, 6} and C = {0, 2, 4, 6, 8).Then the universalset of all the
three sets A, B and C can be
12.
If U = {1, 2, 3, 4,5, 6, 7, 8, 9,10|, A = U, 2, 3, 5), B = [2, 4, 6, 7| and C = |2, 3, 4, 8). Then,
(i) (BuC)' = (ii) (C-A)' =
If A and B are two sets, then A n(A u6)' is equal to
6.41
SETS

14. If A and B are two sets, then ((>\'u 6') isequalto


15. For any two sets A and B, - A)]' is equal to
16. For any three sets A, B and C, (A -B)-(B-C) is equal to
17. For any three sets A, 6 and C, (A - B) n (C - B) is equal to
18. If A and B are two sets, then (A n B')’ u (6 n C) is equal to
19. For any three sets A, B and C,(A uB uC) n{A n B'nC') n C' is equal to
:o. Let S = {x: x is a positive multiple of 3 less than 100| P = {.t: a: is a prime number less than 20)
Then, ji(S) +»i{P) =
21. If » (A n B) = 10, II (6 nC)= 20 and ii (A nC) = 30, then the greatest possible value of
II (A nBnC) is
22. If A, B and C are any three non-empty sets such that any two of them are disjoint, then

low
(A uB uC) n(A nBnC) =
23. If II(An6) =5,ii(AnC) =7 and ii(A nBnC) = 3, then the minimum possible value of
ii(BnC) is
3 \ A and B are any two non-empty sets and A is proper subset of B. If ii(A) =5, then the

ee
F
minimum possible value of ii (A A B) is

Fr
25. For any two sets A and B, if ii(A) = 15, ii(B) = 12, A n B ;^ (|)and S A, then the maximum
and minimum possible values ofii (A A 6) are and respectively.
for
> ii (B) and the difference of the number of
ur
26. If A and B are two finite sets such that ii (A)
elements of the power sets of A and B is 96, thenii (A) -n (B) =
s
ANSWERS
k
Yo
oo

^ 11(B) 3. A n B
1. ii(AnB)-ni(An6) 2.2' 3. [1,2)
eB

6, 1 7. 2 ■S. P(A) = {<[), ID, 12), {1,211 9. A nB’

10. Bn A' 11. [0,1,2, 3, 4,5, 6, 81


r

14. A
12. (i) (5,9,101 (ii) (1,2, 3,5,6,7,9,101
ou
ad

IS. A' uB r, BnC'


15. B 16. A-B 17. (AnC)-B
Y

21. 10 22. (j) 23. 3 24. 1


20. 41
25. 25, 5 26. 2
nd
Re

VERY SHORT ANSWER QUESTIONS (VSAQs)


Fi

Anszuer each of the following questions in one zvord or one sentence or as per exact requirement of the
question:
If a set contains ii elements, then write the number of elements in its power set.
2. Write the number of elements in the power set of null set.
3. Let A = [x: A- e N, A' is a multiple of 31 and B = [a' : a- e N and .v is a multiple of 5 1. Write
A nB.

4. Let A and B be two sets having 3 and 6 elements respectively. Write the minimum nun-1; oer

of elements that A u B can have.

5. If A = {a- eC : 'A^ =11 and B = [v eC : .v'* = 1), then write A-B and B - A.
6. If A and B are two sets such that A c B, then write 6' - A' in terms of A and B.
7, Let A and B be two sets having 4 and 7 elements respectively. Then write the maximum
number of elements that A u B can have.
6.42
APPLIED MATHEMATICS-XI

1
S. If A - , 0 x eR} and B = |(x,y) :y = -x, x eR}, then write/I nR

9. IfA = {(:c,y) :y = e^, X gR) and B = {(x,y) :y=e“^, xeR), then write/l nR

If AandB are two sets such that n{A) =20,n(B) =25 and «(/luB)=40, then write
n(A nB).

If A and B are two sets such that?i (/I) = 115,«(B) = 326, n (A~B)= 47, then write« (A u 6).

ANSWERS
1. 2” - 1
{x: X € N, X is a multiple of 15} 4. 6

A-B = ^,B-A = (/,-/} 7. 11 {(0,1)}

w
10. 5 11- 373

F lo
ee
Fr
for
ur
s
ook
Yo
eB
our
ad
Y
Re
nd
Fi
CHAPTER 7

RELATIONS

7.1 INTRODUCTION

In previous chapter, we have discussed various operations on sets to create more sets out of
given sets. In this chapter, we shall study one more operation which is known as the cartesian

w
product of sets. This will finally enable us to introduce the concept of relation.
7.2 ORDERED PAIRS

Flo
ORDERED PAIR An ordered pair consists of two objects or elements in a given fixed order.

ee
For example, if A and B are any two sets, then by an ordered pair of elements we mean a pair

Fr
{a, b) in that order, where a ^A,b eB.
NOTE ordered pair is not a set consisting of two elements. The ordering of the two elements in an
ordered pair is important and the two elements need not be distinct. for
ur
ILLUSTRATION! The position of a point in a tzuo dimensional plane in cartesian coordinates is
represented by an ordered pair. Accordingly, the ordered pairs (1,3), (2,4), (2, 3) arid (3, 2) represents
s
ok
Yo

different points in a plane.


EQUALITY OF ORDERED PAIRS Two ordered pairs fflj, b-^) and(a2,1^2) ~ ^2 h=
Bo

i.e. (ai,b{) = {^2/^2) = «2 ^1 “ ^2


re

It is evident from this definition that (1, 2) ^ (2,1) and (1,1) (2, 2).
ILLliSTRATlON 2 Find the values of a and b, if {3a -2,b + 3) ={2a -1, 3),
ou
ad

SOLUTION By the definition of equality of ordered pairs, we obtain


Y

{3a-2,b+3) = (2fl-l, 3) <=> 3a-2 = 2a-1 and b + 3 = 3 o a=l and 1?=0


nd
Re

7.3 CARTESIAN PRODUCT OF SETS

CARTESIAN PRODUCT OF SETS Let A and B be any two non-empty sets. The set of all ordered pairs
Fi

{a, b)such that a sAandb e Bis called thecartesianprodiictofthes etsAandBand is denoted by A x B.


Thus, Ax B ={{a,b):a eA andbeS)
If A = (J) or 6 = (ft, then we define Ax B = cj).
ILLUSTRATION 1 If A = {2, 4, 6} and B={1, 2{, then
A X B = |2, 4, 61 X {1, 2) = {{2,1), (2, 2), (4,1), (4, 2), (6,1), (6, 2)1
and. BxA = jl,2)x{2,4,6} = {{1,2), (1,4), (1,6), (2, 2), (2, 4), (2, 6)1
It is evident from the above illustration that to write A x 6, we take an element from set A and
form all ordered pairs with this element as first element and elements of B as second elements.
Next we choose another element from A and corresponding to each element in B we form
ordered pairs with this element as first element and elements of B as second elements. This
process is continued till all elements of A are exhausted.
ILLUSTRATION 2 IfA={a,b]andB = {l,2, 3lfindAxB,BxA,AxA,BxB,and{AxB)(^{BxA).
7.2 APPLIED MATHEMATICS-XI

SOLUTION We have, A = {a, and B = jl, 2, 3)


AxB = {{a, 1), (a, 2), (a, 3), (b, 1), {b, 2), {b, 3)1
BxA = ((l,4(l-i^).(2,fl),(2,i7),(3,<(3,/;)l
Ax A ^ {(a, rt), (a, b), {b, a), {b, t))
BxB = {(1,1), (1, 2), (1, 3), (2,1), (2, 2), (2, 3), (3,1), (3, 2), (3, 3)|
Clearly, (A x B) n (B x /I) = (j).
CARTESIAN PRODUCT OF THREE SETS Let A, B and C be three sets. Then, Ax BxC is the set of all
ordered triplets having first element from A, second element from B and third element from C.
i.e.
Ax BxC = {{a, b, c) \a s A,b ^B, c eC\
:llUSTRATION3 If A = [1, 2),B = (3, 41WC = {4,5, 6].Then,
AxB = (l,2|x {3,41 = {{1, 3), (1,4), (2, 3), (2, 4))
and. AxBxC=(AxB)xC

w
= {(1,3), (1,4), (2, 3), (2, 4)1 X {4,5, 6)
= {(1, 3, 4), (1, 3,5), (1, 3, 6), (1, 4, 4), (1, 4,5), (1, 4, 6),

F lo
(2, 3, 4), (2, 3,5), (2, 3, 6), (2, 4, 4), (2, 4,5), (2, 4, 6))
NOTE It should be noted that Ax Bx C ={A x B) x C = A x {Bx C).
If ^2, A3,A„ arensets, then the cartesian product A^ x A2 x ... x A,, of these» sets is the

e
Fre
set of alln-tuples of the form(rt|, 02, a^,..., a,,), where a^ e A, 02 g A2, e A„.
Ai X A2 X ....X Afj = {(flj, 02/ a^, ■■■, a,,): a-^ € Aj, <?2 g A2, ^36 A3,..., a,, g A„}
i.e.
for
7.3.1 NUMBER OF ELEMENTS IN THE CARTESIAN PRODUCT OF TWO SETS
r
THEOREM If A and Bare two ifnite sets, then n {Ax B) = n{A)xn{B).
You
oks

PROOF Let A = {a^, 02, a;^,..., a,„] and B = {b■^,b2,b2, ■..,b,j] be two sets having wand?i
elements respectively. Then,
eBo

AxB = {{ai/bi),{a^,b2),ia^/b2),...,{a^,bn)
i^2' ^1)' i^2' ^2)' (^2, ^3) ●●●/ {^2' ^h)
ad
our

i^m' bf), {Offi , bf), {a„j , £>3)...., (%;, b„)\


Clearly, in the tabular representation of A x B there are??? rows of ordered pairs and each row has
dY
Re

n distinct ordered pairs. So, A x B has ????? elements.


Fin

Hence, n{Ax B) = mn = n{A)xn{B) Q.E.D.


KEM.ARK (i) If either A or B is an infinite set, then A B is an infinite set.
(ii) If A, B,C are finite sets, then n{A BxC) =n (A) X ?i(B) X n (C)
7.3.2 GRAPHICAL REPRESENTATION OF CARTESIAN PRODUCT OF SETS

Let A and B be any two non-empty sets. To represent AxB graphically, we draw two mutually
perpendicular lines, one horizontal and other vertical. On the horizontal line, we represent the
elements of set A and on the vertical line, the elements of B. If e A, be B, we draw a vertical
line through a and a horizontal line through b. These two lines will meet in a point which will
denote the ordered pair {a, b). In this manner we mark points corresponding to each ordered pair
in A X B. The set of points so obtained represents AxB graphically as illustrated below.
ILLUSTRATION If A ={1,2, 3} and B = {2, 4], find A x Band show it graphically.
SOLUTION Clearly, A x B = {(1, 2), (1, 4), (2, 2), (2, 4), (3, 2), (3, 4)1-
7.3
RELATIONS

In order to represent Ay. B graphically, we draw two perpendicular lines OX and OY as


shown in Fig. 7.1. Now, we represent the set A by three points on OX and the set 6 by two points
on OY. The set A x B is represented by the sbc points as shown in Fig. 7.1.
Y

(1,4) (2,4) (3,4)


4

2
(1,2) (2,2) (3,2)

w
0 1 2 3 X

Flo
Fig. 7.1

e
7.3.3 DIAGRAMATIC REPRESENTATION OF CARTESIAN PRODUCT OF TWO SETS

re
In order to represent A x B by an arrow diagram, we first draw Venn diagrams representing sets

F
A and B one opposite to the other as shown in Fig. 7.2. Now, we draw line segments starting
from each element of A and terminating to each element of set 6.
ur
r
If A = {1, 3,5} and B = {a, b], then following figure gives the arrow diagram of A x 6.
fo
ks
Yo
oo
B
u re
ad
Yo

ILLUSTRATIVE EXAMPLES
d
Re

Ti/;»r I ON EQUALITY OF ORDERED PAIRS


in

EX.AMrLt 1 Find X and y, if(x + 3,5) =(6, 2x + y).


F

SOLUTION By the definition of equality of ordered pairs


(x+ 3,5) = (6,2.r + i/)
X + 3 = 6 and 5 ~ 2x + y
.V = 3 and 5 = 2x + y
X - 3,5 = 6 + y
X = 3 andy = -1

Type II ON FINDING THE CARTESIAN PRODUCT OF TWO SETS


EXAMPLE! If A = {1, 3,5, 61 and B = {2, 4), find A x Band B x A.
SOLUTION We have, A = {1, 3,5, 6) and B = {2, 4). Therefore,
A X B = {(1, 2), (1, 4), (3, 2), (3, 4), (5, 2), (5, 4), (6, 2), (6, 4)1
and. B X A = {(2,1), (2, 3), (2, 5), (2, 6), (4,1), (4, 3), (4,5), (4, 6)1
7.4 APPLIED MATHEMATICS-XI

EXAMi'LL3 If A ={1,2, 3],B = {3,4\andC={l, 3,5],find


(i) Ax(BkjC) (ii) Ax(Br^C) (iii) X B) n (/I X C)
SOLUTION (i) Clearly, S uC = (1, 3, 4,51
Ax{BuC}= {1,2, 3} X (1,3, 4,5)
= 1(1,1),(1, 3), (1,4), (1,5), (2,1), (2, 3), (2, 4), (2,5), (3,1), (3, 3), (3, 4), (3,5))
(ii) Clearly, B nC = {3).
Ax(BnC) = {l,2, 3}x(3)={(l, 3),(2, 3),(3, 3)1
(iii) .4 X B = {(1, 3), (1, 4), (2, 3), (2, 4), (3, 3), (3, 4)),
and. /IxC = l(l,l),(l,3),(l,5),(2,l),(2, 3),(2,5),(3,1),(3, 3),(35)1
(AxB)n{AxC) = {(1,3),(2, 3),(3, 3)).
Type III ON FINDING SETS A AND S WHEN A ● B OR SOME

w
ELEMENTS OF A ● B ARE GIVEN

EXAMPLE 4 LetA={l, 2, 3] and B = \x: x eN, x is prime less than 5). Find Ax B and Bx A.
SOLUTION We have, A = [1,2, 3\ and, B = \x: x ^ N, x is prime less than 5 } = { 2,3 }

Flo
AxB = (1, 2, 31 X (2, 3) = 1(1, 2), (1, 3), (2, 2), (2, 3), (3, 2), (3, 3)|
and, BxA = (2, 3|x(1,2, 31 = {(2,1), (2, 2), (2, 3), (3,1), (3, 2), (3, 3)1

e
re
EXAMPLE 5 IfAxB = {{a, 1), (a, 5), (a, 2), (b, 2), {b, 5), (b, l)],find BxA.

F
SOLUTION Clearly, BxA can be obtained from AxB by interchanging the entries (or
components) of ordered pair in .4 x B.
ur
BxA = \ (l,aU5,aU2,a),i2,b).i5,bUl,b)]
r
EXAMPLE fi
fo
If A = (1, 2], form the set A x A x A.
ks
SOLUTION We have, A =[1,2].
Yo
oo

AxA = I (1,1), (1,2), (2,1), (2, 2)1


and, Ax Ax A
= {(1,1,1), (1,1, 2), (1, 2,1), (1, 2, 2), (2,1,1), (2,1, 2), (2, 2,1), (2, 2, 2)}
eB

EXAMPLE
~ IfR is the set of all real numbers, zuhat do the cartesian products RxR and RxRxR
represent?
ur

SOLUTION The cartesian product of the setKof all real numbers withitselfi.e.K x Risthesetof
ad

all ordered pairs (x, y) where x,yeR. In other words, Rx R= |(:r, y): x, y e R].
Yo

Clearly, R x R is the set of all points in XY-plane. The set R x Ris also denoted by R^.
Similarly, we have
d
Re
in

Rx Rx R = {{.Y, y, 2): X, y, 2 6 R}
F

Clearly, it represents the set of all points in space. The set R x R x R is also denoted by R
EXAMPLES Express A = [{a,b):2a + b = 5, a, b sW] as the set of ordered pairs.
SOLUTION Here, W denotes the set of whole numbers (non-negative integers).
We have.
2a + b =5, where a,b eW.
a = 0=>b = 5, a=l=>b = 3 and, a = 2 => b = 1
For a> 3, the values of b given by the above relation are not whole numbers.
^ = ((0,5), (1,3), (2,1)1
EiXAMPLE 9
If AxB = [{a, 1), (b, 3), (a, 3), (b, 1), (a, 2), (b, 2)],fmd A and B.
SOLUTION Clearly, A is the set of all first components in ordered pairs inAx B and B is the set of
all second components in ordered pairs in /I x 6.
^ = [a,b] and B = (1, 2, 3}
RELATIONS 7.5

EXAMPLE 10 Let A and B be two sets such that Ax B consists of 6 elements. If three elements of A x B
are: (1,4), (2, 6), (3, 6). Find Ax B and Bx A.
SOLUTION Since (1,4), (2,6) and (3,6) are elements of A x B. It follows that 1,2,3 are elements of
A and 4,6 are elements of B. It is given that/l x 6 has 6 elements. So, A = (1, 2, 3} and B = {4, 6}.
Hence, AxB = {l,2,3}x|4,6} = |(1, 4), (1, 6), (2, 4), (2, 6), (3, 4), (3, 6)1
and. BxA = [4,6] X {1, 2, 31 = {(4,1), (4, 2), (4, 3), (6,1), (6, 2), (6, 3)|
EXAMPLE 11
The cartesian product Ax A has 9 elements among which are found (-1, 0) and (0, V.
Find the set A and the remaining elements of Ax A.
SOLUTION Since (-1, 0) e /I x /I and (0,1) e y4 x A. Therefore,
(-l,0)eAxy\ => -l,0eA and, (0,1) e A x A => 0,1 e A
-1,0,1 e A
It is given that Ax A has 9 elements. Therefore, A has exactly three elements.

low
Hence, A ={-1,0,11.
EXAMPLE12 Let A and B be two sets such that n (A) =5andn{B) = 2.1fa,b,c,d,earedistinctand
{a, 2), (b, 3),(c, 2),{d, 3),(c, 2) are in Ax B, find A and B.
SOLUTION Since(fl, 2), {b, 3), (c, 2), {d, 3), {e, 2) are elements of A x B. Therefore, fl, b, c, d, e e A
and 2, 3 e 6.

ee
It is given that n (A) = 5 and n (B) = 2
F
Fr
a, b,c,d,e eA and n (A) = 5 => A ~ {a, b, c, d, t’l
2, 3eB and ?i(6) = 2 => B = {2, 31
Tifpc IV ON GRAPHICAL AND DIAGRAMATIC REPRESENTATION OF A x B
for
ur
EXA.MPLE13 Let A ={-l, 3, 41 and B ={2, 31. Represent the following products graphically i.e. by
lattices: (i) A x 6 (ii) BxA (iii) Ax A
s
ook

SOLUTION (i) We have, A = (-1, 3, 41 and B = {2, 3).


Yo

A X B = {{-1, 2), (-1, 3), (3, 2), (3, 3), (4, 2), (4, 3)1
eB

In order to represent AxB graphically, we follow the following steps:


(a) Draw two mutually perpendicular lines one horizontal and other vertical,
(b) On the horizontal line represent the elements of set A and on the vertical line represent the
r

elements of set B.
ou
ad

(c) Draw vertical dotted lines through points representing elements of A on horizontal line
and horizontal lines through points representing elements of B on the vertical line. Points of
Y

intersection of these lines will represent AxB graphically as shown in Fig. 7.3.
Re
nd

Y
Fi

3 (3,3)
(-1,3) r ●?(4,3)

(-l,2)f 2
●t (4.2)
(3,2)

●X
-1 o 1 2 3 4

Fig. 7.3
7.6 APPLIED MATHEMATICS-XI

(ii) Clearly, BxA = {2, 3} x |-1, 3, 4| = 1(2,-1), (2, 3), {2, 4), (3, -1), (3, 3), {3, 4)1
Here, we represent B on the horizontal line and A on vertical line. Graphical representation of
B X is as shown in Fig. 7.4.

(2,4)
4 ●(3, 4)
3 f -(3,3)
(2,3):
2

w
1

O
X
i |2 ;3

lo
-1
1(3^^)

e
re
rFFig. 7.4

F
(iii) Wehave, A =11, 3, 4}

r
fo
u
= 1-1, 3, 4) X (-1, 3,4|
ks
= K-l,-l),(-l,3),(-l,4),(3,-l),(3, 3), (3, 4), (4,-1), (4, 3), (4, 4)|
Yo

Graphical representation of .A x /I is shown in Fig. 7.5.


oo
B

Y
e
ur

(3,4)
(-1,4)-f f
-t - (4,4)
ad

4
Yo

(-l,3)-f. -f - (4,3)
3
(3,3)
d
Re

2
in
F

X
-1
O 1 2 4

●r - (-1,4)
(-i,-i)r -1 ci:3)i"

Fig, 7.5

EXAxMPLEU If A = (1,3,5), B = {x, \/\ represent the follozoing products by arrozv diagrams:
(i) A X B (ii) BxA (iii) A X A (iv) B x 6
SOLUTION (i) We have, A = (1, 3, 5} and B = {x,y\
A X 6 = 11, 3, 51 X {x, y) = {(1, x), (1, y), (3, x), (3, y), (5, x), (5, y)l
Following arrow diagram represents A x 6.
RELATIONS 7.7

B A

1 X

5 y

Fig. 7.6

(ii) We have, B = {x, y\ and A 5}.


Bx A = {x, y} X {1, 3,5} = {(x, 1), {x, 3), {x, 5), (y, 1), (y, 3), (y, 5)}
It has been represented by the following arrow diagram.

w
B A

Flo
X
1

e
■3

re
y

xy

F
ur
r
Fig. 7.7
fo
(iii) We have, A = [1, 3,5}
ks
A X A = {1, 3,5} X {1, 3,5} = {(1,1), (1, 3), (1,5), (3,1), (3, 3), (3,5), (5,1), (5, 3), (5,5)}
Yo
oo

It has been represented by the following arrow diagram.


B

A A
u re
ad

●3
Yo

5
d
Re
in

Fig. 7.8
F

(iv) We have, B=[x,y\


BxB = {x, y] X [x, y] = {{x, x), (x, y),(y, x), (y, y)}
Following is the arrow diagram representing BxB.
7.8
APPLIED MATHEMATICS-XI

EXERCISE 7.1
2^ 5 1
1. (i) If l + i^b^£ find the values of a and b.
U 3 3 ' 3/
(ii) If (;f + 1,1) = (3, y - 2), find the values of x and y.
7. If the ordered pairs {x, -1) and (5, y) belong to the set [{a, b):b=2a-3}, find the values of
X and y.
3. If /7 6 {-1, 2, 3, 4,51 and b e{0, 3, 6), write the set of all ordered pairs {a, b) such that
a + b =5.

4. lia e{2, 4, 6, 9} andb g [4, 6,18, 27}, then form the set of all ordered pairs (a, b) such that a
divides!; and a <b.

5. If A =|1, 2) and B = (1, 3|, find Ax B and Bx A.


6. Let .A = {1, 2, 31 and B = (3, 41. Find Ax B and show it graphically.
7. If A =(1, 2, 3| and B = {2, 4|, what are A x B,Bx A,Ax A,Bx 6, and (A x 6) n(Bx A)?
8. If A and B are two sets having 3 elements in common. Iffi{A) = 5,n{B) = 4, find n(A x B) and
n[{A xB)n(Bx A)].
9. Let A and B be two sets. Show that the sets Ax B and Bx A have an element in common ijf
the sets A and B have an element in common.

F low
10. Let A and B be two sets such that n{A) = 3 and n (B) = 2.
If {^> 1)/ {y> 2), (2,1) are in A x B, find A and B, where x, y, z are distinct elements,
n. Let A ={1, 2, 3, 4} and R ={{a,b):a eA,b eA,a divides!;). Write R explicitly.
17. If A = j-1,1), find A x A x A.
for Fre
13. State whether each of the following statements are true or false. If the statement is false,
re-write the given statement correctly:
(i) If P = Im, »1 and Q = {n, m], then P x Q = {(m, n), {n,;»)}
(ii) If A and B are non-empty sets, then A x B is a non-empty set of ordered pairs (a:, y)
such that A* e B and y e A.
Your
eBo ks

(iii) If A = {1, 2), 6 = |3, 4|, then A x (B n (}>) - (}i


14. If A = |1, 2), form the set A X A X A.
15. If A = {1, 2, 4} and B = {1, 2, 3), represent following sets graphically:
ad

(i) A X B (ii) BxA


our

(iii) Ax A (iv) Bx B

ANSWERS
Re

1. (i) <? =2,!) =1 (ii) A = 2, y = 3 7. A=l,y=7 3. 1(-1,6),(2, 3), (5, 0)1


4. ((2, 4), (2, 6), (2,18), (6,18), (9,18), (9, 27)}
Find Y

5. Ax B = {(1,1),(1, 3), (2,1), (2, 3)|and Bx A = {(1,1),(1, 2), (3,1), (3, 2)]
6. Ax B = {(1, 3), (1,4), (2, 3),(2,4),(3, 3),(3,4)).
7. AxB = {(1,2), (1,4), (2, 2), (2, 4), (3, 2), (3, 4)1
BxA =1(2,1), (2, 2), (2, 3), (4,1), (4, 2), (4, 3))
Ax A = ((1,1),(1,2),(1, 3),(2,1),(2,2),(2, 3),(3,1),(3,2),(3, 3)1
Bx B = {(2, 2), (2, 4), (4, 2), (4, 4)).
(AxB)n(Bx A) = |(2,2)|
8. ;i(A X B) = 20, u[(A x B) n (B x A)] = 9 10. A={A,y,2),B = {l,2)
n. R={ (1,1), (1, 2), (1, 3), (1, 4), (2, 2), (2, 4), (3, 3), (4, 4) 1
17. AxAxA = K-1,-1,-1),(-1,-1,!),(-!,1,-!),(-!,1,1),(1,-1,-1),(1 ,-1,1),
(1,1,-1), (1,1,1)1
13. (i) F (ii) F (iii) T
14. A X A X A =( (1,1,1), (1,1, 2), (1, 2,1), (1, 2, 2), (2,1,1), (2,1, 2), (2, 2,1), (2, 2, 2) 1
RELATIONS 7.9

HINTS TO SELECTED PROBLEM

8- rj{A X B) = n{A)xn{B) = 5 x 4 = 20. From theorem9 on page 7.11 (f A and B have h elements in
common, then (A x B) and B x A have elements in common. Therefore,
/t[{AxB)n(6xA)] = 3^=9.
7.4 SOME USEFUL RESULTS

In this section, we intend to study some results on cartesian product of sets which are stated as
theorems without giving their proofs.
THEOREM 1 For any three sets A, B, C, prove that:
(i) Ax(BuC)=(Ax B) u(AxC) (ii) Ax(BnC) = {Ax B) n(AxC).
THEOREM 2 For any three sets A, B, C, prove that: A x (B -C) =(A x B) - (A x C).
THEOREMS If A and B are any two non-empty sets, then prove that: A x B = B x A <=> A = B.

w
THEOREM 4 IfAcB, shoiv that A x A c (A x B) n (B x A).

F lo
THEOREM 5 IfAcB, prove that AxC c BxCfor any set C.
THEOREM 6 IfA^B and C Q D, prove that AxC,c BxD.
THEOREM 7 For any sets A, B, C, D prove that: (Ax B) r\{C xD) = (A n C) x (B n D)

e
Fre
THEOREM 8 For any sets A and B, prove that (A x B) n (B x A) = (A n B) x (B n A).
THEOREM 9 For any three sets A, B, C prove that:
for
(i) A X (B' uC')'=(A X B) n(A X C) (ii) A X (B' nC')' = (A x B) u (A x C).
r
THEOREM 10 Let A and B be two non-empty sets having n elements in common, then prove that A x B
You

and B X A have elements in common.


s
ook

THEOREM 11
Let A be a non-empty set such that Ax B = AxC. Show that B = C.
eB

EXERCISE 7.2

1. Given A = {1, 2, 3}, B = {3, 4), C = {4,5, 6}, find (A x B) n (B x C).


our

7. If A = {2, 31, B = {4,51, C = (5, 6), find Ax (B'uC), Ax {B n C), (A x B) u (A x C).


ad

If A = {1, 2, 3}, B =|4|, C = (5}, then verify that:


(i) A X (BkjC) =(A X B) u(A x C) (ii) A X (B nC) =(A X B) n(A X C)
(in) Ax(B-C)=(AxB)-(AxC).
dY
Re

4. Let A = {1, 21, B = |1, 2, 3, 4}, C = {5, 61 and D = {5, 6,7, 8|. Verify that:
(ii) Ax{BnC)={AxB)n{AxC)
Fin

(i) AxC ^BxD


V. If A = {1, 2, 31, B = 13, 4) and C = |4,5, 61, find
(i) Ax(BnC) (ii) (AxB)n(AxC) (iii) A X (B u C)
(iv) (A x B) u (A x C)
6. Prove that: (i) (A u B) x C =(A x C) u(Bx C ) (ii) (A n B) x C ={A x C) n(Bx C)
7 If AxBcCxD and Ax B^ prove that A c C and B c D.
ANSWERS

{3, 4).
7. A X (B u C) = {{2, 4), (2, 5), (2, 6), (3, 4), (3,5), (3, 6)1
A x(SnC) ={(2,5), (3,5)1,
(A X B) u (A X C) = {(2, 4), (2,5), (3, 4), (3,5), (2, 6), (3, 6)1.
R ={(1,1), (1, 2), (1, 3), (1, 4), (2, 2), (2,4), (3, 3), (4, 4))
(i) 1(1, 4), (2, 4), (3, 4)1 (ii) {(1,4), (2, 4), (3, 4)1
(iii) {(1,3), (1,4), (1,5), (1,6),(2, 3), (2, 4), (2,5), (2, 6), (3, 3), (3, 4), (3,5), (3, 6)1
(iv) {(1, 3), (1, 4), (1,5), (1, 6), (2, 3), (2, 4), (2,5), (2, 6), (3, 3), (3, 4), (3,5), (3, 6)1
7.10 APPLIED MATHEMATICS-XI

7.5 RELATIONS

Let A and B denote the sets of all male and female


members in the royal family of Dasrath's
kingdom. Clearly, A - ( Dasrath, Ram, Laxman, Shatrughan, Bharat j and B = \ Kaushalya,
Kaikai, Sumitra, Sita, Urmila, Shrutkirti, Mandvi}.
If we write R for the relation "was husband of" then the fact that Dasrath was husband of
Kaushalya, Kaikai and Sumitra, Ram was husband of Sita, Laxman was husband of Urmila,
Bharat was husband of Mandvi and Shatrughan was husband of Shrutkirti can be represented
as:

Dasrath R Kaushlya, Dasrath R Kaikai, Dasrath R Sumitra, Ram R Sita, Laxman R Urmila,
Bharat R Mandvi and Shatrughan R Shrutkirti.
Now, if we omit the letter R between the pairs of names and write them as ordered pairs, then
the above fact can also be written as a set R of ordered pairs as given below:

w
R = [ (Dasrath, Kaushalya), (Dasrath, Kaikai), (Dasrath, Sumitra), (Ram, Sita),
(Laxman, Urmila), (Bharat, Mandvi), (Shatrughan, Shrutkirti)}.

Dasrath
F lo Kaushalya

e
Kaikai

Fre
Ram ~~ - Sumitra

Laxman Sita
for
Bharat● Urmila

Shatrughan
r
Mandvi
You

r~ Shrutkirti
oks
eBo

Fig. 7.10

Clearly, Rq Ax B.
our
ad

A visual representation of this relation R in the form of an arrow diagram is as follows:


Thus, we see that the relation "was husband of" from set A to set B gives rise to a subset R of
Ax B such that (.r, y) ^Riff xRy.
dY
Re

Keeping this example in mind, we may define a relation as follows.


RELATION Let A and B be two sets. Then a relation Rfrom A to B is a subset ofAxB.
Fin

Thus, R is a relation from AtoBoRcAxB.


If R is a relation from a non-void set /4 to a non-void set B and if {a, b) s R, then we write aRb
which is read as 'a is related to b by the relation R. If (a, b) g R, then we write aRb and we say that
a is not related to b by the relation R.
ILLUSTRATION 1 If A ={1,2, 3] and B = (a, b, cl, then R = ((l,b),(2,c),(2,a), (3, a)l, being a subset of
Ax B, is a relation from A to B. Here, (1, b), (2, c), (1, a) and (3, a) e R, so we write IRb, 2Rc, IRn and
3Ra. But, (2, b) g R, so xve write 2 Rb

ILLUSTRATION 2 If A = (a, b, c,d}, B = (p,q, r, s}, then which of thefoUozuing are relations from A toB?
Give reasons for your anszver.
(i) Ri={(a.pUb,r),{c,s)\ (ii) R2={{q,b),{c,s),(d,r)\
(Hi) R^=\(a,p),(a,q),{d,pj),ic,r),(b,r)] (iv) R4 ={(a, p),{q, a),(b, s),{s,b)\.
SOLUTION (i) Clearly, Rj c A x B. So, Rj is a relation from A to B.
RELATIONS 7.11

(ii) Since (cj, b) e R2 but (9, b) ^ A x B. So, R2 C A x B. Thus, R2 is not a relation from to B.
(hi) Clearly, K3 c A x B. So it is a relation from A to B.
(iv) R4 is not a relation from A to B, because (cj, a) and (s, b) are elements of R4 but {q, a) and
(s, b) are not in A x B. As such R4 ^ A x B.
TOTAL NUMBER OF RELATIONS Let A and B be two non-empty finite sets consisting of m and n
elements respectively. Then, A x B consists of mn ordered pairs. So, total number of subsets of
A X Bis 2”'”. Since each subset of A x B defines a relation from A to 6, so total number of relations
from A to B is 2'””. Among these 2"’” relations the void relation <{) and the universal relation Ax B
are trivial relations from A to B.

7.5.1 REPRESENTATION OF A RELATION

A relation from a set A to a set B can be represented in any one of the following forms;
(I) ROSTER FORM In this form a relation is represented by the set of all ordered pairs belonging
to R.

w
For example, if R is a relation from set A ={- 2, -1, 0,1, 2} to set B = (0,1, 4, 9,10} by the rule
aRb = b. Then, 0 R 0, - 2 R 4,-1 R 1,1 R 1 and 2 R 4.

(ii) SET-BUILDER FORM

F lo
So, R can be described in Roster form as R = 1(0- 0)- (-1/ 4), (1,1), (2, 4)|
In this form the relation R from set A to set B is
R = [[a, b):a eA,b eB and a, b satisfy the rule which associates a and b].
represented as

e
Fre
For example, if A = (1,2, 3, 4,5), B =
1^111
1-- .... \ and R is a relation from A to B
2 ' 3 ' 4'5 ' 6 '
for
(
given by R =■} (1,1), 2, ^l,f3, ^3j' 4,-
4j I
, 5
r
\ Z)
You

1
oks

Then, R in set-builder form can be described as: R = Ufl, &): fl e A, 1? e B and b = - >.
a
eBo

It should be noted that it is not possible to express every relation from set A to set B in set-builder
form. For example, the relation R = {{1, c), {3,b)} from set A = {1,2, 3,4} to set
B = {fl,&, c} cannot be described in set-builder form,
our
ad

(iii) BY ARROW DIAGRAM In order to represent a relation from set A to a set B by an arrow
diagram, we draw arrows from first components to the second components of all ordered pairs
belonging to R.
dY
Re

For example, relation R = {(1, 2), (2, 4), (3, 2), (1, 3), (3, 4)} from set A = {1, 2, 3, 4,5} to set
B = {2, 3, 4,5, 6,7} can be represented by the following arrow diagram:
Fin

1
2
3
4 5
5 6
7

Fig. 7.11

(iv) BY LATTICE In this form, the relation R from set A to set B is represented by darking the
dots in the lattice for A x B which represent the ordered pairs in R.
For example, if R = {(-3, 9), (-2,4), (-1,1), (0, 0), (1,1), (2, 4), (3, 9)} is a relation from set
A = {-3,-2,-1, 0,1, 2, 3} to set B = {0,1, 2, 3, 4,5, 6,7, 8, 9}, then R can be represented
by the following lattice.
7.12
APPLIED MATHEMATICS-XI

(-3,9) (3,9)
9"T
8 !
●i
7 I
6

(-2,4) 4 I (2,4)
3

2
*

w
(-1,1) 1 F*'i)
X' -3 -2 -1 (P,0)l 2 3 X

o
r

e
Fig. 7.12

re
7.5.2 DOMAIN AND RANGE OF A RELATION

rFl
F
Let R be a relation from a set ^ to a set B. Then the set of all first components or coordinates of the
ordered pairs belonging to R is called the domain of K, while the setof all second componenij

r
or coordinates of the ordered pairs in R is called the range of R.
ou
Thus, Dom (R) = ( a : {a, b) e R] and Range (R) = [b : {a, b) e R}.
sfo
It is evident from the definition that the domain of a relation from A to Bis a subset of A and its
k
range is a subset of B. The set B is called the co-domaiii of relation R.
oo

If A = {1, 3,5,7], B = (2, 4, 6, 8,10} and let R = {(1, 8), (3, 6), (5, 2), (1, 4)} be
Y

H LUSTRATION 1 a
eB

relation from A to B. Then,


Domain (R) = {1,3,5} and Range(R) = (8, 6,2,4}
r

ILl.Un i K.ATION 2 Let A = (1,3,5, 7) and B = (2,4,6,8} be two sets and let Rbea relation from AtoB
You

defined by the phrase "(x,y) eRo x>y". Under this relation R,ioe obtain 3R2,5R2,5R4,7R2,7R4
ad

and7R6.
I.e.
R =1(3, 2), (5, 2), (5, 4), (7, 2), (7, 4), (7, 6)}.
d

Domain (R) = {3,5,7} andRange(R) = {2,4,6}


Re
in

ILLUSTRATION 3 ^f ^ is a relation from set A - (2, 4,5} to set B = (1, 2, 3, 4, 6, 8} defined by


F

xRy o X divides y.
(i) \Nrite Rasa set of ordered pairs, (ii) Find the domain and the range ofR.
SOLUTION (i) Clearly, 2R2, 2R4, 2R6, 2R8,4R4, and 4R8.
R = 1(2, 2), (2, 4), (2, 6), (2, 8), (4, 4), (4, 8)}
(ii) Clearly, Domain (R) = { 2, 4 } and Range (R) = { 2, 4, 6, 8 }
RELATION ON A SET Let A be a non-void set. Then, a relation from A to itself i.e. a subset of Ax A, is
called a relation on set A.

7.5.3 INVERSE OF A RELATION

INVERSE RELATION Let A, B be tzvo sets and let R be a relation from a set A to a set B. Then the inverse
of R, denoted by R~^, is a relation from B to A and is deifned by R~^ = {(b,a):(a,b) eR\.
-1
Clearly, (a, b) g R «■ (b,a) e R
Also, Dom (R) = Range(R"^) and. Range (R) = Dom (R“^).
RELATIONS 7.13

ILLUSTRATION 1 LefA = {l, 2, 3), B = \a,b, c, d\ be two sets and let R =l(l, a), (1, c),{2, d), {2, c)}
be a relation from A to B. Then, = {{a, 1), {c, 1), {d, 2), (c, 2)} is a relation from B to A.
Also, Dorn (R) = (1,2) = Range {R \ and Range (R) = {a, c, dj = Dorn {R \
ILLUSTRATION 2 Let A be the set offirst ten natural numbers and let Rbea relation on A defined by
(x,y) sR<^ x+ 2y=10i.e.R ={(.v, y):xsA, yeAa)idx + 2y =10\. Express Rand R^^as sets of
ordered pairs. Also, determine (i) domains ofR and R~^ (ii) ranges ofR and R~^.
SOLUTION We have,
1 n _ Y

(x,y) e R + 2y =10 o y=— x, y €/I where A ={1, 2, 3, 4,5, 6,7, 8, 9,10).


10-1
Now, Y = 1 => V =
2 2

w
This shows that 1 is not related to any element in A. Similarly we can observe that 3,5,7,9 and 10
are not related to any element of A under the defined relation.

F lo
Further we find that:
„ 10-2
For .Y = 2, y= = 4 E A. Therefore, (2, 4) e R
^ 2

ee
. 10-4

Fr
For Y = 4, y= = 3 6 A. Therefore, (4, 3) e R
^ 2
, 10-6 for
For Y = 6/ y= = 2eA. Therefore, (6,2) e R
ur
10 — fi
For Y = 8, y= =leA. Therefore, (8,1) e R
s

2
ook
Yo

Tlius, R = {(2, 4), (4, 3), (6, 2), {8,1)1 = {(4, 2), (3, 4), (2, 6), (1, 8)1
eB

Clearly, Dom(R) = {2, 4, 6, 8) = Range (R“^) and. Range (R) = (4, 3, 2,1) = Dorn {R^\
r

ILLUSTRATIVE EXAMPLES
ad
ou

Ty, I ON EXAMINING WHETHER A SET OF ORDERED PAIRS REPRESENTS A RELATION OR NOT


Y

. . ;:‘LEi If A = {1,2,3j, B = 14,5,61, zvhich of the following are relations from A to B? Give reasons
Re

in support of your ansiuer:


nd

(i) Ri={ (1,4), (1,5), (1,6)1 (ii) R2 = {(1,5), (2,4), (3,6)1


Fi

(iii) R3 = { (1, 4), (1, 5), (3, 6), (2, 6), (3, 4) } (iv)R4 = {(4,2), (2, 6), (5,1), (2,4)}.
SOLUTION (i) Clearly, Rj c A x B. So, it is a relation from A to B.
(ii) Clearly, R2 c A x B. So, it is a relation from A to B.
(iii) Clearly, R3 c A x B. So, it is a relation from A to 6.
(iv) Since (4, 2) s R4 but (4, 2) ^ A x 6. So, R4 is not a relation from A to 6.
Tijpe II ON DESCRIBING A RELATION AND ITS INVERSE AS A SET OF ORDERED PAIRS AND
FINDING THEIR DOMAINS AND RANGES

EXAMPLE 2 A relation R is defined from a set A = { 2, 3, 4, 51 to a set B = I 3, 6, 7, 20 / as follozvs:


(y, y) £ R <=> Y divides y. Express Rasa set of ordered pairs and determine the domain and range ofR.
Also, find R~^.
SOLUTION Recall that a \ b stands for ‘a divides b’. For the elements of the given sets A and B, we
find that 21 6, 2 110, 3 13, 3 16, and 5 110.
(2,6) eR,(2,10) eR,(3, 3) eR,(3, 6) eR and(5,10) e R.
7.14 APPLIED MATHEMATICS-XI

Thus, R = {(2,6), (2,10), (3,3), (3, 6), (5,10)).


Clearly, Dorn (R) { 2,3,5 } and. Range (R) = { 3,6,10 }.
Also, R-l = { (6,2), (10,2), (3,3), (6,3), (10,5) ).
EXAMPLE 3
If R is the relation ‘'less than"frotn A = {1,2,3,4,5 }toB = { 1,4,5 },zvritedown the set of
ordered pairs corresponding to R. Find the inverse of R.
SOLUTION It is given that (x,y) sR o at < i/, where .v e A and y e B.
For the elenaents of the given sets A and B, we find that
1<4, 1<5, 2<4, 2<5, 3<4, 3<5 and 4 < 5
(1,4) G R, (1, 5) e R, (2, 4) g R, (2, 5) g R, (3, 4) g R, (3, 5) g R and (4, 5) g R.
Thus, R = { (1, 4), (1, 5), (2,4), (2, 5), (3, 4), (3, 5), (4, 5)).
-1
R = I (4,1), (5,1), (4, 2), (5, 2), (4, 3), (5, 3), (5, 4)) = [{x, ij): x e B, y e A and x > i/}.

w
EXAMPLE 4 A relation R is defined on the set Z of integers as: {x, y) eR o x^ + y^ = 25.
Express R and R~^ as the sets of ordered pairs and hence find their respective domains.

Flo
SOLUTION We have,

e
(a:, y) G R <r> = 25 <=> y = ± - ip-

re
We observe that from the above relation a: = 0 gives y = ± 5.

F
(0, 5) G R and (0, -5) g R
ur
Similarly, a: = ± 3 => y = yj^ -9 = ± 4
r
(3, 4) G R, (- 3, 4) e R, (3, -4) G R and (-3, -4) g R
fo
A: = ±4=i>y=.y25-16 =± 3
ks
Yo

(4, 3)eR,(-4, 3)gR,(4, - 3) g R and (-4, - 3) g R


oo

x = ±5^y = f25-25 =0
eB

(5, 0) gR and (-5, 0) e R

Wealsonoticethatfor any other integral value of a:, the value of y givenby y = ± -^25-x^ is not
ur

an integer.
ad

■■■ R=l (0,5), (0,-5), (3,4), (-3,4), (3,-4), (-3,-4), (4,3), (-4,3), (4,-3), (-4,-3), (5, 0), (-5, 0))
Yo

=> R-l = {(5, 0), (-5, 0), (4, 3), (4, -3), {-A, 3), (-4, -3), (3, 4), (3, -4), (-3, 4), (-3, -4), (0, 5), (0, -5)}
Clearly, Domain (R) = { 0, 3, -3, 4, -4, 5, -5 } = Domain(R~\
d
Re

EXAMPLE 5
in

Let R be the relation on the set N of natural numbers defined by


R ={(fl, b): a+3b =12, a^N,beN}.
F

Find: (i) R (ii) Domain of R (Hi) Range of R


SOLUTION (i) We have,
a+ 3b = 12 => a = 12- 3b

Putting/? = 1, 2, 3 respectively in the above relation, we get« =9, 6, 3 respectively.


For b =4, a =12-3b gives <7 = 0 which does not belong to N. Also, values of a given hya=12-3b
do not belong to N for all b >4.
R = {(9,1), (6, 2), (3, 3)1
(ii) Clearly, Domain of R = {9, 6, 3)
(iii) Clearly, Range of R = (1, 2, 3}
Type III ON REPRESENTING A RELATION BY USING AN ARROW DIAGRAM
EXAMPLE 6
Let A = {1, 2, 3, 4,5, 6). Define a relation R on set Aby R ={(a:, y): y = x + 1}
(i) Depict this relation using an arrow diagram (ii) Write doiun the domain, co-domain and range of R.
7.15
RELATIONS

SOLUTION (i) Putting .v = 1, 2, 3, 4,5, 6 respectively in y = .t + 1, we get y = 2, 3, 4,5, 6,7


respectively.
(1, 2) e R, (2, 3) 6 R, (3, 4) e R, (4,5) e R, (5, 6) e R and (6,7) g R.
For X = 6, we get y = 7 which does not belong to set A.
Hence, R = 1(1, 2), (2, 3), (3, 4), (4,5), (5, 6)|
The arrow diagram representing R is as follows.

ow
e
re
rFl
(ii) Clearly, Domain (R) ={1, 2, 3, 4,5), Range(R) = {2, 3, 4,5, 6).

F
EXAMPLE? Figure 7.14 shows a relation R between the sets PandQ. Write this relation R in (i) Roster
forin (ii) Set builder form. What is its domain and range?

r
ou
fo
ks
oo
Y
B
re
ou

Fig. 7.14
Y
ad

SOLUTION (i) It is evident from the figure that


d

R = {(9, 3), (9, - 3), (4, 2), (4 - 2), (25,5), (25 -5))
in

2
Re

(ii) It is evident from R, that it consists of elements (x, y), where A' is the square of y i.e. x=y .
F

Therefore, relation R in set builder form is R = {(.v, y): a = y , a e P, y € Q}.

The domain and range of R are {9, 4, 25j and (-5, - 3, - 2, 2, 3,5) respectively.
REMARK In the above example, the range of relation R is not same as the set Q. The set Q is known as the
co-domain.

Tifpe IV ON PROVING RESULTS BASED ON THE DEFINITION OF A RELATION


EXAMPLES Let R be a relation on Q defined by R ={{a,b):a,b eQ and a - h e Z}.
Sholu that:

(i) (a, a) eR for all a eQ (ii) (a, b) eR => (b, a) eR


(iii) (rt, l7) E R and (b,c)eR => (a,c)eR.
SOLUTION (i) For any eQ, we have
a - a = 0 eZ

(a, a) eR ,
'ii
«●
7.16
APPLIED MATHEMATICS-XI

Hence, {a, a) eR for all e Q.


(ii) Let {a, b) eR. Then,
(a,b) eR
a- b eZ, wherea,b eQ
^ b-a eZ
[v b -fl = -(fl-b)]
=> (b, a) eR
(iii) Let {a, b) eR and (b, c) e R. Then,
(a, b) eR and (b, c) eR
a-bEZ and b-CEZ

{a-b) + {b - c) eZ
a-c eZ

w
(a, c)eR

Let Rbea relation on N defined by R ={{a,b):a,b eN and a =b^).

F lo
rxAMPl.l'
Are the following true:
(i) (a, a) eR for all a eN (ii) {a, b) eR => {b, a) e R

ee
(iii) {a, b) e R, (b, c) e R => {a, c) e R

Fr
Justify your answer in each case. for
SOLUTION (i) We observe that <7 = is true for = 1 e M only. Therefore, (1,1) e R. But,
ur
(2, 2), (3, 3), {4, 4) etc do not belong to R. So, {a, a) eR for all « e N is not true,
(ii) We observe that(4, 2) eR, because4 = 2^. But,(2, 4) g R as 2 4^.
s
ook
Yo

So, {a, b) eR ^ (b, a) eR is not true for all a,b eN.


eB

(iii) We observe that (16, 4) g R and (4, 2) g R. However, (16, 2) g R.


So, (a, b) eR and (b, c) g R => (a, c) eR is not true for all a,b, c e N.
our

EXAMPLE 10 Let a relation R^ on the set R of all real numbers be defined as (a,b) g Rj <=> I + ab > 0/or
ad

all a,b eR.


Show that: (i) {a, a) E Ri for all a eR (ii) (o,b)€Ri => {b, a) e R-^for alt a,b e R
Y

SOLUTION (i) For any a eR, we have


Re

1 + a^ > 0 ^ (fl, fl) G Rj


nd
Fi

Thus, {a, a) e Rj for all aeR.


(ii) Let(fl, b) gR]. Then,
(fl, b) G R| => 1 + ab > 0 1 + bo > 0 => (b, o) g R 1
Thus,{a, b) G Rj => (b, a) e Rj for alla,b eR.
EXAMPLE 11 Let R be the relation on the set Z of all integers defined by {x, y) gR => x - y is divisible
by n. Prove that:
(i) {x, x) E Rfor all X eZ
(ii) (x, y) eR => {y, x) eR for all x,y eZ
(iii) (x,y)ERand{y,z)ER => (x,z) e Rfor all x, y,z e R.
SOLUTION (i) For any x eZ, we have
X-X = 0 - 0x«

=> X - X is divisible by n
^ {x, x) G R
RELATIONS 7.17

Thus, {x, x) eR for all x eZ.


(ii) Let (a:, y) sR. Then,
(x, y) e R
=> X - y is divisible by n
=> X-y = for same X&Z
y -X = {-X)n
=> y -xis divisible by it [●.● XeZ ^ -XeZ]
=> {y, x) e R
Thus, (x, y) e R => (y, x) e R for all x, y € Z.
(iii) Let (x, y) G R and {y, z) e R. Then,

w
(x, y)eR => X - y is divisible by » => x-y = ^nfor some A. gZ
(y,z)eR => y-2 is divisible by n => y-z =|i« for some y g Z
(x, y) € R and (y, z) g R

o
=> x-y = Xn and y-z ~ |ifi

e
re
=>
=>
(x-y) + (y-z) = X?i + |4n
x-z = (X + y)ji
rFl
F
X-z is divisible by n [: >. + n e Z]
=> (x, z) e R

r
ou
Thus, (x, y) e R and (y, z) sR=> (x, z) e R.
sfo
EXERCISE 7.3
k
1. Ifi4 = { 1,2,3}, B = {4,5,61, which of the following are relations from/I to B? Give reasons in
oo

support of your answer.


Y

(i) 1(1,6), (3,4), (5,2)} (ii) {(1,5), (2,6), (3,4), (3,6)1


eB

(iii) (4,2), (4,3), (5,1)} (iv) A X B.


7. A relation R is defined from a set A = (2,3,4, 5) to a set B = { 3,6, 7,10 } as follows:
r

(x, y) e R o X is relatively prime to y


You
ad

Express R as a set of ordered pairs and determine its domain and range.
3. Let A be the set of first five natural numbers and let R be a relation on A defined as follows:
(x,y)eR o x<y
d

Express R and R“^ as sets of ordered pairs. Determine also


Re
in

1
(i) the domain of R (ii) the range of R.
F

-1
4. Find the inverse relation R in each of the following cases:
(i) R = 1(1, 2), (1,3), (2, 3), (3, 2), (5, 6) I
(ii) R = l(x, y): X, y G N, x + 2y = 8}
(iii) R is a relation from {11,12,13 } to (8,10,12) defined by y = x - 3.
5. Write the following relations as the sets of ordered pairs:
(i) A relation R from the set { 2,3,4,5,6 } to the set {1,2,3 1 defined by x = 2y.
(ii) A relation R on the set (1,2,3,4, 5, 6, 7 | defined by
(x, y) G R o X is relatively prime to y.
(iii) A relation R on the set { 0,1,2,..., 10 ) defined by 2x + 3y = 12.
(iv) A relation R from a set A = | 5, 6, 7, 8 ) to the set 6 = {10,12,15,16, 18} defined by
(x, y) G R <=> X divides y.
6. Let R be a relation in N defined by (x, y) g R <=> x + 2y = 8. Express R and R ^ as sets of
ordered pairs.
7.18 APPLIED MATHEMATICS-XI

7. Let ^ ={3,5} and 6 = |7,11). Let R =((a,b): n e A, h e B, a - b is odd }. Show that R is an


empty relation from A into B.
8- Let A = {1, 2} and B = (3, 4}. Find the total number of relations from A into B.
9- Determine the domain and range of the relation R defined by
(i) R ={(.v,.r + 5)::re|0,l, 2, 3,4,5}}
(ii) R = {(.r, : .r is a prime number less than 10 )
^0. Determine the domain and range of the following relations:
(i) R = {{a,b)\aeN,a<5,b= 4}
(ii) S = \{a, b)-.b = \ a -1 \, a eZ and } rt | < 3}
11- Let A = [a, b\. List all relations on A and find their number.
12. Let A = [x, y, z] and B = \a, b]. Find the total number of relations from A into B.
13. Let R be a relation from N to N defined by R = [{a, b):a,beN and a = b^].

w
Are the following statements true?
(i) {a, rt) e R for all n g A/ (ii) {a, b) sR => {b, n) eR
(hi) {a,b)eR and (b,c)eR => (a, c) e R

Flo
14. Let A = {1, 2, 3,..., 14}. Define a relation on a set A by
R ={(;r, y): 3^:-y = 0, where .r, y e A).

e
re
Depict this relationship using an arrow diagram. Write down its domain, co-domain and
range.

F
15- Define a relation R on the set N of natural numbers by
R = {(a:, y): y = a: + 5, -t is a natural number less than 4, x, \j e N].
ur
r
fo
Depict this relationship using (i) roster form (ii) an arrow diagram. Write down the domain
and range or R.
ks
16. A ={1, 2, 3, 5} and B = {4, 6, 9}. Define a relation R from A to 6 by
Yo

R = {(a:, y): the difference between a: and y is odd, .r e A, y e B}.


oo

Write R in Roster form.


B

p Q
re

5 3
u
ad

6 4
Yo

Fig. 7.15
d
Re

17. Write the relation R = {(a:, .y^) : .y is a prime number less


in

than 10 } in roster form.


18. Let A = |1, 2, 3, 4,5, 6). Let R be a relation on A defined by
F

R = {{a, b):a,b ^A,b\s exactly divisible by a }


(i) Write Rin roster form (ii) Find the domain of R (iii) Find the range of R.
19.
Figure 7.15 shows a relationship between the sets P and Q. Write this relation in
(i) set builder form (ii) roster form. What is its domain and range?
20. Let R be the relation on Z defined by R = \{a, b) :a,b eZ, o -b is an integer}. Find the
domain and range of R.
21. For the relation Rj defined on R by the rule {a, 1?) g Rj 1 -i- ab >0.
Prove that: (n, b) g R^ and {b, c) g R^ => (<?, c) g Rj is not true for all a,b, c e R.
22. Let R be a relation onN x N defined by
(a,b) R {c, d) o (1 + d = 1? + c for all {a, b), {c, d) eN x N
Show that:
(i) (a, b) R {a, b) for all (a,b) eN x N
(ii) (a, b) R (c, d) => (c, fr) R {n, b) for all {a, b), {c, d) ehJ x N
(iii) {a, b) R (c, d) and (c, d) R{e,f)=> (a, b) R{e, f) for all {a, b), (c, d),{e,f)eNxN
7.19
RELATIONS

ANSWERS

1. (i) It is not a relation from A to B.


(ii) It is a subset ofAxB, so it is a relation from A to B.
(iii) It is not a relation from A to B as it is not a subset of ^4 x 6.
(iv) It is a relation from A to 6.
2. R = {(2,3), (2,7), (3,7), (3,10), (4,3), (4, 7), (5,3), (5,7)}
3. R={ (1,1),(1,2),(1,3),(1,4),(1,5),(2,2),(2,3),(2,4),{2,5),(3, 3),(3,4),(3,5), (4,4),(4,5),
(5,5)1

(5,5)1
Domain of R-^ = {1,2,3,4,5 1 = Range of R.
4. (i) R-1 = {(2,1),(3,1),(3,2),(2,3),(6,5)1 (ii) R-l = {(3,2), (2,4), (1, 6)1
(iii) R"l = {(8,11), (10,13)1

w
5. (i) I (2,1), (4,2), (6,3)1

(5, 7), (6,5), (6,7), (7,2), (7, 3), (7,4), (7,5), (7,6), (7, 7) 1

F lo
(iii) 1(0, 4), (3, 2), (6, 0)1 (iv) { (5,10), (5,15), (6,12), (6,18), (8,16) |
6. R= 1(2,3), (4,2), (6,1)1 R-i = {(3,2), (2,4), (1, 6)1

ee
8. 16

9. (i) DomainR=|0,l,2,3,4,51,RangeR=|5,6,7,8,9,10)

Fr
(ii) Domain R ={2, 3,5,71, Range R ={8,27,125, 343)
10. (i) Domain R =|1, 2, 3, 4), Range R ={41 for
ur
Domain S = {0, -1, - 2, - 3,1, 2, 31, Range S = {0,1, 2, 3, 4}
(ii) S = 1(0,1)), (-1, 2), (- 2, 3), (- 3, 4), (1, 0), (2,1), (3, 2)1
ks
Yo

11. 16 12. 64 13. (i) No (ii) No (iii) No


oo

14. Domain (R) = {1,2,3,41, Co-domain (R) = 7i,Range(R) = {3,6,9,12}


eB
r
ou
ad
Y
nd
Re
Fi

15. (i) R = {(1,6),(2,7),(3,8)} (ii) Domain (R) = [1, 2,3], Range (R) = { 6, 7,8 1
7.20
APPLIED MATHEMATICS-XI

16. R =((1, 4), (1, 6), (2, 9), (3, 4), (3, 6), (5, 4), (5, 6)|
17. R = {(2, 8), (3, 27), {5,125), (7, 343)}
18. (i) R = {(1,1), {1, 2), (1, 3), (1, 4), (1,5), (1, 6), (2, 2), (2, 4), (2, 6), (3, 3), (3, 6), (4, 4),
(5.5), (6, 6)}
(ii) Domain (R) ={1, 2, 3, 4,5, 6} (Hi) Range(R)={l,2, 3, 4,5,6}
\9.
(i) R={{x,ij)-.ij=x-2,xsP,ij^Q] (ii) R={(5, 3),(6,4),(7,5)}
Domain (R) = {5, 6,7|, Range (R) = 3, 4, 5
20. (i) Domain(R) = Z, Range(R) = 2

H/Wrs TO SELECTED PROBLEMS

8. We have, n{A)=2,n{B)=2
n(Ax B) = 2x2 = 4

w
So, there are 2'* = 16 relations from A to B.

F lo
9. (i) We have,
R = |(j:,x + 5):;f6(0,l,2, 3,4,5}} = {(0,5), (1, 6), (2,7), (3, 8), (4, 9), (5,10)}
Domain(R)=|0,l,2,3,4,5) and. Range (R) = |5, 6,7, 8, 9,10)

ee
(ii) We have,

Fr
R = {(.Y, Y^): Y is a prime number less than 10} = {(2, 8), (3, 27), (5,125), (7, 343)}
Domain (R) = {2, 3, 5,7}, and Range(R) = {8, 27,125, 343}
for
ur
10. (i) We have, R = {(1, 4), (2, 4), (3, 4), (4, 4)}
Domain (R) = {1, 2, 3, 4}, Range (R) = {4}
s
ook

(ii) We have,
Yo

S = {(- 3, 4), (- 2, 3), (-1, 2), (0,1), (1, 0), (2,1), (3, 2)}
eB

Domain (S) = {- 3, - 2,-1, 0,1, 2, 3}, and Range (S) = (0,1, 2, 3, 4}


12. Here A has 3 elementsand 6 has 2 elements. Therefore,total number of relations from A to
Bis 2^^^ =64.
our
ad

13. (i) No, because (2, 2) g R.


(ii) No, because (4, 2) e R but (2, 4) g R.
Y

(Hi) No, because(16, 4) e R and(4, 2) eR but(16, 2) g R.


Re
nd

14. R = {(1,3), (2, 6), (3, 9), (4,12)}


Fi

Domain (R) = {1, 2, 3, 4}, and Range(R) = {3, 6, 9,12}.


7.21
RELATIONS

15. (i) We have,


R ={(:f, y) ;y = .i; +5, is a natural number less than 4, x, ij eN] = {(1, 6), (2,7), (3, 8)}
(ii) Domain (K) =(1, 2, 3}, and Range (R) =|6,7, 8)

-6

2-

3 -8

Fig. 7.19

16. We have, A = {1, 2, 3,5}, 6 = {4, 6, 9}


^ = {(^/ y): difference between a: and y is odd, a: e A, y e 6)

w
R = {(1, 4), (1, 6), (2, 9), (3, 4), (3, 6), (5, 4), (5, 6)}
17. Wehave, R ={(a:, x^): a: is a prime number less than 10}
R = {(2, 8), (3, 27), (5,125), (7, 343)1
18. (i) We have,

F lo
ee
R = {(a, b): (], b e A, h is exactly divisible by o}, where A = {1, 2, 3, 4,5, 6).

Fr
R = 1(1,1), (1, 2), (1, 3), (1, 4), (1,5), (1, 6), (2, 2), (2, 4), (2, 6), (3, 3), (3, 6), (4, 4), (5,5), (6, 6)1
(i) Domain R = |1, 2, 3, 4,5, 6) (ii) Range R = {1, 2, 3, 4,5, 6)
for
ur
19. (i) 1 (-Y, y): y = x-2, X 6 {5, 6, 7}, y e {3, 4,5} |
(ii) 1(5, 3), (6, 4), (7,5)1
s

Domain R = {5, 6, 71, and Range R = {3, 4, 51


ook
Yo

20. The relation R on Z is defined by R ={{a,b) :a,b eZ, a- b is an intcgerl


eB

Since n -fc is an integer for alia ,b sZ. So, domain (R) = Z = Range (R).
21. We have,
f-l'l
our

1
ad

1,- — e Ri and -4 gRi asl + — > 0 and 1 + — (-4)>0.


2] ^ 2 ' 2 J \ ^ J

But, 1 + 1 X - 4 ^0. So, (1, - 4) iE Rj.


Y

22. (i) We know that


Re
nd

a + b =b + aforalla,b€N
Fi

{a, b) R {a, b) for all n, b g N


(ii) {a,b) R {c, d) ^ a + d = b + c^c + b = d + a ^ {c, d) R {a, b)
(iii) (fl, b) R (c, d) and (c, d) R{e, f)
=> a + d = b + c and c + f = d + e
=> a + d + c + f =b + c + d + e=>a + f = b + e => (a,b) R{e, f)
7.6 TYPES OF RELATIONS

In this section, we intend to discuss various types of relations on a set A.


7.6.1 VOID. UNIVERSAL AND IDENTITY RELATIONS
VOID RELATION Let A be a set. Then, cj) c y4 x /I and so it is a relation on A. This relation is called the
void or empty relation on set A.
In other words, a relation R on a set A is called void or empty relation, if no element of A is
related to any element of A.
7.22 APPLIED MATHEMATICS-X!

Consider the relation R on the set A = {1,2, 3, 4,5} defined by R = {(a,b): a-b = 12}.
We observe that 12 for any two elements of A.
{a, b) sR for any a,b eA.
^ R does not contain any element of Ax A
=> K is empty set
^ R is the void relation on A.

UNIVERSAL RELATION Let A be a set. Then, Ax A^ Ax A and so it is a relation onA.Tliis relation is


called the universal relation on A.

ow
In other words, a relation R on a set is called universal relation, if each element of A is related to
every element of A.
Consider the relation R on the set A = {1, 2, 3, 4,5, 6} defined by R = ((«, ft) e R : | a - & | > 0}.
We obser\^e that

e
I <7 - & I > 0 for all a,b e A

re
=> (a, b) e R for all (a,b) ^Ax A

Frl
F
=> Each element of set A is related to every element of set A
=> R =Ax A

^
ou
R is universal relation on set A

sor
\i vil' It is to note here that the void relation and the universal relation on a set A are respectively the
smallest and the largest relations on set A. Both the empty (or void) relation and the universal relation are
kf
sometimes called trivial relations.
oo
ILLUSTRATION Let A be the set of all students of a boys school. Show that the relation Ron A given by
R = {{fl, b): a is sister ofb) is empty relation and R’ = {(<7, b): the difference between the heights of a and b
Y

is less than 5 meters) is the universal relation.


B

SOLUTION Since the school is boys school. Therefore, no student of the school can be sister of
re

any student of the school. Thus,


oY

{a, b) g R for any a,b sA.


u
ad

Hence, R = <|)i.e. R is the empty or void relation on A.


d

It is obvious that the difference between the heights of any two students of the school has to be
less than 5 meters.
in

(a,b) eR for a\] a,b e A.


Re

R=AxA
F

=> R is the universal relation on set A.

IDENTITY RELATION Let A be a set. Tlien, the relation I^ = {(a, a): a ^ A] on A is called the identity
relation on A.

In other words, a relation on A is called the identity relation if every element of A is related to
itself only.
If A = {1,2, 3}, then the relation 7^ = {(1,1), {2, 2), (3, 3)} is the identity relation on set A. But,
relations R^ = {{1,1), (2, 2)} and R2 = {(1,1), (2, 2), (3, 3), (1, 3)} are not identity relations on A,
because (3, 3) g Rj and in R2 element 1 is related to elements 1 and 3.
7.6.2 REFLEXIVE RELATION

DEFINITION A relation R on a set A is said to be reflexive if every element of A is related to itself.


Thus, R is reflexive o {a, a) eR for all a eA.
A relation R on a set A is not reflexive if there exists an element a e A such that (a, a) g R.
RELATIONS
7.23

ILLUSTRATION 1 Let A = {1, 2,3} be a set. Then R = {{1,1), (2,2), (3,3), (1,3), (2,1)1 is a reflexive
relation on A. But, Rj = {(1,1), {3, 3), (2,1), (3, 2)) is not a reflexive relation on A, because 2eA but
(2, 2)^Rj.
{[.LUSTRATION 2 The identity/ relation on a non-void set A is always reflexive relation on A. However,
a reflexive relation on A is not necessarily the identity relation on A. For example, the relation R = {(a, a),
(b, b), (c, c), (a, b)) is a reflexive relation on set A= \a, b, c\ but it is not the identity relation on A.
ILLUSTRATION 3 The universal relation on a non-void set A is reflexive.
ILLUSTRATION 4 A relation R on N defined by {x,y) eR x>y is a reflexive relation on N,
because every natural number is greater than or equal to itself.
ILLUSTRATION 5 Let X be a non-void set and P(X) be the power set of X. A relation Ron P(X) defined
by {A, B) eR Aq B is a reflexive relation since every set is subset of itself
ILLUSTRATION 6 Let L be the set of all lines in a plane. Then relation R on L defined by
{/j, /^) e R <=> li is parallel to I2 is reflexive, since every line is parallel to itself

w
7.6.3 SYMMETRIC RELATION

F lo
DEFINITION A relation Ron a set A is said to be a symmetric relation iff
{a,b)eR => {b, a) e R for all a,b e A
i.e. aRb ^ bRa for all a,b sA.

ee
ILLUSTRATION 1 The identity and the universal relations on a non-void set are symmetric relations.

Fr
ILLUSTRATION 2 Let L be the set of all lines in a plane and let R be a relation defined on L by the rule
{x, \j)eRc:> X is perpendicular to y. Then, R is a sxjmmetric relation on L, because 1 L2 => i-2 X Li
for
i.e. (Lp L2) e R => (L2, L{) e R.
ur
ILLUSTRATION 3 Let S be a non-void set and R be a relation defined on power set P(S) by (A, B) e R
s

<=> c 6/or all A, B e P{S). Then, R is not a symmetric relation.


ook
Yo

' 11 A relation Ron a set A is not a symmetric relation if there are at least two elements a,b eA such
that (a, b) eR but (b, a) g R.
eB

ILLUSTRATION 4 Let A = 12,2,3,41 and let R^ and R2 be relations on A given by R^ = {(1,3), (1,4),
(3, 1), (2, 2) (4, 1)1 and R2 = 1(1, 1), (2, 2), (3, 3), (1, 3)1. Clearly, Rj is a symmetric relation on A.
our

However, R2 is not so, because (1, 3) e R2 but (3,1) g R2-


ad

\orK A reflexive relation on a setAis not necessarily symmetric. Fore.xample, the relation R = ((1,1),
(2,2), (3,3), (1,3)1 is a reflexive relation on set A = {1,2,31 but it is not symmetric.
Y

ILLUSTRATION 5 Prove that a relation R on a set A is symmetric iffR = R~^.


Re
nd

-1
.In
SOLUTION First, let R be a symmetric relation on set Then, we have to prove that R = R
Fi

order to prove this we have to prove that R c R and R~^ c R.


-1

Now, {a, b) e R
=> (/j,rt)eR [●.● R is symmetric]
{a.b) eR"^ [By def. of inverse relation]
Thus, {a,b)sR =>(«,/)) e R“^ for all (1,e A.
So, R e R“^ ...(i)

Now, let (a:, y) be an arbitrary element of R Then,


-1
(x, y) e R
=> (y- x) E R [By def. of inverse relation]
=> (x, y) e R [●.● R is symmetric]
-1
Thus, (x, y) e R => (x, y) € R for all x, y e A.
7.24 APPLIED MATHEMATfCS-XI

So, R
-(ii)
Thus, from (i) and (ii), we get R=R~^.
Conversely, let R be a relation on set A such that R = R"l Then we have to prove that R is a
symmetric relation on set A. Let (a, b) e R. Then,
(a,b) eR
-1
{b, rt) E R [by def. of inverse relation]
(b, a) eR [vR=R-l]
Thus, (a, b) eR^ (b, a) e R for all a, & e A.
So, R is a symmetric relation on A. Hence, R is symmetric iff R = R“^.
7.6.4 TRANSITIVE RELATION

DEFINITION Let A be any set. A relation Ron A is said to be a transitive relation ijf

w
{a, b) eR and {b, c) eR => {a, c) eR for all a,b, c e A.
i.e.. aRb and bRc => aRc for all a,b c e A.
11 LUST RA nON 1
II I.USI RAIION 2

F lo
The identity and the universal relations on a non-voidset are transitive.
The relation R on the set N of all natural numbers defined by

ee
{x, y) e R <=> X divides y, for all x,y eN is transitive.

Fr
SOLUTION Let x,y,zeNhe such that {x, y) e R and (y, z) e R. Then,
{x, y) eR and (y, z) eR for
=> X divides y and, y divides z
=> There exist p, q eN such that y =xp and z=yq
ur
=> 2 = (xp)q
s

=> 2 = x(pq)
ook
Yo

^ X divides z
[●-■ pq^N]
eB

=> (x, 2) E R
Thus, (x, y) € R, (y, z) e R => (x, 2) e R for all x,y,ze N.
Hence, R is a transitive relation on N.
our
ad

ll.l US IRATlON 3 On the set N of natural numbers, the relation R defined by xRy ^ x is less than y is
transitive, because for any x,y,zeN
X <y and y <z => x<z i.e., xRyandyRz => xRz
dY
Re

11. LUST RATION 1 Let S be a non-void set and Rbearelation defined on power set P{S) by{A, B) eR
Fin

Ac Bfor all A, B e P(S). Then, R is a transitive relation on P(5), becausefor any A, B,C e P(S)
=> {A, 6) e R and (B, C) e R => A c B and BcC=>AcC=>(A, C)eR
II l us I EtATlON ?
LetLbelJtesetofall straight lines in a plane. Then the relation "is parallel to" on L is
a transitive relation, because for any l^, l^, l^ e L.
/1 I2 and I2
li.i.LiS rUA I ION 6 The relation "is congruent to" on the set T of all triangles in a plane is a transitive
relation.
7.6.3 ANTISYMMETRIC RELATION

DEFINITION Let A be any set. A relation R on set A is said to be an antisymmetric relation iff
{a, b) eR and {b,a) eR=i> a = b for all a,b eA
It follows from this definition that if {a, b) eR but (b, a) e R, then also R is an antisymmetric
relation.

Il l US'l RATION 1 The identity relation on a set A is an antisymmetric relation.


RELATIONS 7.25

ILLUS TRATION : The universal relation on a set A containing at least two elements is not
antisymmetric, because if a ^ b are in A, then a is related to b and b is related to a under the universal
relation will imply that a = b but a=^b.
ILLUSTRATION 3 Let Rbea relation on the set N of natural numbers defined by
xRy o 'a: divides y' for all x,y
This relation is an antisymmetric relation on N. Since for any two numbers a,b eN.
a j b and b \ a ^ a = b i.e. aRb and bRa ^ a = b
It should be noted that this relation is not antisymmetric on the set Z of integers, because we ifnd that for
any non-zero integer a, aR {-a) and {-a) R a but a^ - a.
lU t '>TRATio\ ; Let S be a non-void set and Rbea relation on the power set P{S) defined by
B) e R o A c 6 for all A, Be P(S)
Then, R is an antisymmetric relation on P{S), because
(/I, B)eR a7id(B, A) eR => Ac B and Be A=> A = B

w
ILLL'STRATION - The relation < ("less than or equal to") on the set R of real numbers is antisymmetric.
because a<b and b <a ^ a=b for all a,b eR.

L\AMI’LE 1

F lo
ILLUSTRATIVE EXAMPLES

Three relations Rj, R2 and R3 are defined on set A = {a,b,c} as follows:

ee
(i) Rj = [{a, a), (a, b), (a, c), (b, b), (b, c), {c, a), (c, b), (c, c)K

Fr
(ii) R2 = \(a, b), (b, a), (a, c), {c,«)}
(iii) R3 = [(a,b),ib,c),{c,a)]. for
Find whether each ofR^, R2 and R3 is reflexive, symmetric and transitive.
ur
SOLUTION (i) Reflexive: Clearly (a, a), {b, b), (c, c) e Rj. So, R^ is reflexive on^.
s
ook

Symmetric: We observe that (a, b) e Rj but (b, a) 0 Rj. So, Rj is not a symmetric relation on A.
Yo

Transitive: We find that (b, c) e Rj and (c, a) e R| but {b, a) g Rj. So, R| is not a transitive
eB

relation on A.

(ii) Reflexive: Since (a, a), (b, b) and (c, c) are not in R2. So, it is not a reflexive relation on A.
our
ad

Symmetric: We find that the ordered pairs obtained by interchanging the components of
ordered pairs in R2 are also in R2. So, R2 is a symmetric relation on A.
Y

Transitive: Clearly {a, b) e R2 and [b, a) e R2 but (a, a) e R2.


Re

So, it is not a transitive relation on R2-


nd

(iii) Reflexive: Since none of (<7, a),{b,b)and{c, c)is an element ofR3-So,R3is not reflexiveon/1.
Fi

Symmetric: Clearly, (1j, c) e R3 but (c, i?) g R3. So, R3 is not a symmetric relation on A.
Transitive: Clearly, {a, b) e R3 and (b, c) e R3 but (a, c) g R3. So, R3 is not a transitive relation
on A.

EXAMPLE2 Show that the relation R on the set A = {1,2,3} given by


R ={(1,1), (2, 2),(3, 3),(1,2), (2, 3)} isreflexivebut neither symmetricnor transitive.
SOLUTION Since 1,2, 3 eA and (1,1), (2, 2), (3, 3) e R i.e. for each as A, {a, a) e R. So, R is
reflexive.

We observe that (1, 2) e R but (2,1) g R. So, R is not symmetric.


Also, (1, 2) e R and (2, 3) e R but (1, 3) g R. So, R is not transitive.
I XAMPI.E 3 Show that the relation R on the set A = {1, 2, 3}given by R = {(1, 2), (2,1)} is symmetric
but neither reflexive nor transitive.
SOLUTION We observethat (1,1), (2, 2) and (3,3) do not belong to R. So, R is not reflexive.
7.26 APPLIED MATHEMATICS-XI

Clearly, (1, 2) eR and (2,1) e R. So, R is symmetric.


As(l, 2) eR and (2,1) eR but{l, 1) g R. So, R is not transitive.
KXA.Ml’LE 4 Check the foUozuing relations R and Sfor reflcxivity, symmetry and transitivity:
(i) aRb iffb is divisible by a, a,b eN
(ii) /j SI2 iffli -I I2. where and I2 are straight lines in a plane.
SOLUTION (i) We have,
aRb a\b for all a,b sN.
Reflcxivity: For any a eN, we have
I fl => aRa.
Thus, aRa for all a eN. So, R is reflexive on N.
Symmetry: is not symmetric because if a | b, then b may not divide a. For example, 216 but b\2.
Transitivity: Let a,b,c eN such that aRb and bRc. Then,

w
aRb and bRc ^ a\b and b\c => fl | c => aRc.

F lo
So, i? is a transitive relation on N.
(ii) Let L be the set of all lines in a plane. We are given that
/| S /2 <=> /-j 1 I2 for all 1}, I2 ^

ee
Reflcxivity: S is not reflexive because a line cannot be perpendicular to itself i.e. /1 / is not true.

Fr
Symmetry: Let li, I2 ^ ^ ^ l2- Then,
/l S /2 ^ ^1 L /2 /2 -L ^ ^2 ^ h' for
So, S is symmetric on L.
ur
Transitive: S is not transitive, because _L ^2 arid I2 1 l^ does not imply that /j 1 l^,.
s

'■x.wii’LC 5 Let a relation K-j on the set R of real numbers be defined as (a, b) eR-^c^l + ab> 0for all
ook
Yo

a,b e R. Show tlwt is reflexive and symmetric but not transitive.


eB

SOLUTION We observe the following properties:


Reflcxivity: Let a be an arbitrary element of R. Then,
our

a eR
ad

=> l + a.a = l + a'^>0 [■■■ a^ >0 for all« g R]


=> {a, a) e R^ [By definition of Rj]
Y

Tlius, (a, a) e R^ for all a e R. So, Rj is reflexive on R.


Re
nd

Symmetry: Let (a, b) eR. Then,


Fi

{a, b) eRj
^ 1 + ab >0
=> 1 + &<7 > 0 [●-● ab =ba for a\\ a,b G R]
=> {b, a) e Ri [By definition of R^]
Thus, {a, b) eRi=> {b, a) e R| for alla,b eR. So, R^ is symmetric on R.
Transitivity: We observe that (1,1/2)gR-j and (1/2,-l)eRj but (1, -1) gRj because
1 + 1 X (-1) = 0:[» 0. So, R^ is not transitive on R.
■ \ \ \in.n 6 Determine whether each of thefolloxving relations are reflexive, symmetric and transitive:
(i) Relation R on the set A = (1, 2, 3,..., 13,14} deifned as R = {(x, y): 3.r - y = 0}
(ii) Relation R on the set N of all natural numbers defined as R = {(x, y):y = x + 5 and x <A}
(iii) Relation R on the set A = (1, 2, 3, 4,5, 6} defined as R ~ {(x, y): y is divisible by xj
(iv) Relation R on the set Z of all integer defined'Cis R = {(x, y):x~yis an integer}
SOLUTION (i) R={(a:, y):3:r-y = 0}, where x, y'G A = {1, 2, 3,..., 13,14}
R= {(1,3), (2, 6), (3, 9), (4,12)}
RELATIONS 7.27

Reflexivit.y Clearly, (1,1) g R. So, R is not a reflexive relation on A.


Symmetr.y We observe that(l, 3)eR but (3,1) gK. So, K is not a symmetric relation/I.
Transitivit.y We observe that (1, 3) e R and (3, 9) e R but (1, 9) g R. So, R is not a transitive
relation A.

(ii) R = {(.t, y):y = :r + 5 and a: < 4}, where a;, y e N.


R={(1,6), (2,7), (3, 8)}
R^exivity Clearly, (1,1), (2, 2) etc. are not in R. So, R is not reflexive.
Symmetry: We find that (1, 6) £ R but (6,1) g R. So, R is not symmetric.
Transitivity: Since (1, 6) e R and there is no order pair in R which has 6 as the first element. Same
is the case for (2, 7) and (3,8). So, R is transitive,
(iii) R = {{a:, y): y is divisible by .v}, where a:, y e A = {1, 2, 3, 4,5, 6}.
Reflexivity: We know that

w
X is divisible by x for all x £ A
(x, x) € R for all X £ A

F lo
^ R is reflexive on set A.

Symmetry: We observe that 6 is divisible by 2 but 2 is not divisible by 6. This means that
(2, 6)£Rbut(6, 2)gR.

ee
So, R is not symmetric on set A.

Fr
Trajisitivity: Let (x, y) £ R and (y, z) £ R. Then,
(x, y) £ R and (y, z) € R. for
=> y is divisible by x and, z is divisible by y
ur
=> z is divisible by x
=> (x, z) G R
s
ook

So, R is transitive relation on A.


Yo

(iv) R = {{x, y): X - y is an integer}, where x, y £ Z


eB

R^exivity: We have,
X - X = 0, which is an integer for all x e Z.
our

=> (x, x) £ R for all X G Z


ad

=> R is reflexive on Z.

Symmetry: Let (x, y) g R. Then,


Y

(x, y) e R
Re

X - y is an integer, say, X
nd

=> y-x =
Fi

^ V - X is an integer [v XgZ=> -?.gZ]


=> {y, x) G R
Thus, (x, y) G R => (y, x) £ R for all x, y e Z.
So, R is symmetric on Z.
Transivity: Let (x, y) e R and (y, z) g R. Then,
(x, y) £ R and (y, z) g R
^ x-y and y-z are integers
=> (x - y) + (y - z) is an integer [●.● Sum of two integers is an integer]
=> X - z is an integer
^ (x, z) G R
So, R is transitive on Z.
EXAMl’Lr 7 Show that the relation RonR deifned as R== {{a, b):a< b}, is reflexive and transitive but
not sym7}ietric.
7.28 APPLIED MATHEMATICS-XI

SOLUTION We have, R = {{a, b): < &}, w^here ^7, b g R


Reflexivity: For any a g R
a < a

^ {a, fl) g R for all fl g R


R is reflexive.

Symmetry: We observe that (2, 3) € R but (3, 2) g R. So, R is not symmetric.


Transitivity: Let {a, b) eR and {b, c) g R. Then,
{a, b) eR and (b, c) e R
a < b and b < c
a < c

=> {a, c) g R
So, R is transitive.

! X AMPl n '● Let S be the set of all points in a plane and R be a relation on S defined as

w
R = {{P, Q): Distancebetween P and Q is less than 2 units}.

F lo
Shozu that R is reflexive and symmetric but not transitive.
SOLUTION We observe the following properties of relation R:
Reflexivity: For any point P in set S, we find that

e
Fre
Distance between P and itself is 0 which is less than 2 units.

=> (P, P)gR


for
Thus, (P, P) g R for all P e S.
So, R is reflexive on S.
r
You

Symmetry: Let P and Q be two points in S such that (P, Q) g R. Then,


oks

{PrQ)eR
eBo

Distance between P and Q is less than 2 units.


=> Distance between Q and P is less than 2 imits
=> (Q,P)eR
our
ad

So, R is symmetric on S.
Transitivity: Consider points P, QandR having coordinates (0, 0), (1.5, 0) and (3.2, 0). We
observethat the distancebetweenP and Q is 1.5 units which is less than 2 units and the distance
dY
Re

between Q and R is 1.7 units which is also less than 2 units. But, the distance between P and R is
3.2 which is not less than2imits.This means that(P, Q) eR and (Q, R) gRbut(P, R) gR.So,R
Fin

is not transitive on S.
● -.Mi’Ll;-
LetX = ^,2, 3, ^,5, 6,7 ,^,9).LetR-Ybea relationonXgivenby R-^ = {(.T,y):x-y
is divisible by 3} and R2 be another relation on X given by R2 = {(a:, y): {.v, y} c {1, 4, 7} or
{x, y)c {2,5, 8}or{x, y] {3, 6,9}}. ShowthatR-^ = R2.
SOLUTION
Clearly, R^ and R2 are subsets of Xx X. In order to prove that R| =R2, it is
sufficient to show that Rj c R2 and R2 c; R|.
We observe that the difference between any two elements of each of the sets {1, 4, 7}, {2,5, 8} and
{3, 6, 9} is a multiple of 3.
Let {x, y) be an arbitrary element of R^. Then,
(x, y)gRi
^ X-y is divisible by 3.
^ a: - y is a multiple of 3.
=> {a-, y} c fl, 4,7} or {a:, y} c (2,5, 8} or {x, y} cz {3, 6, 9}
7.29
RELATIONS

=>

Thus, {x,i/)eRi => (x, y)ei^2-


So, Ri c ]?2
Now, let {a, b) be an arbitrary element of i^2- Then,
(a,b) ei^2
{fl,ij}c{l,4,7}or{fl,i?}c;{2,5, 8} or {fl,&} c {3, 6, 9}
=> - & is divisible by 3
(a,b)GRi
Thus, {a,b)eR2 => {a,b)^Ri
So, R2 c; Rj ...(h)

From (i) and (ii), we get: = R2


S/JOiy t/ie re/flf/on R o» the set R of all real numbers, defined as R = {{a,b): a <b } is

w
EXAMPLE 10

neither reflexive nor symmetric nor transitive.

F lo
SOLUTION Wehave, R = {{a,b):a<b^}, wherea,beR.
1 1 1
is not true. Therefore sR.
Reflexivity: We observe that 2

ee
\2'2

Fr
So, R is not reflexive.

Symmetry: We observe that -1 < 3^ but 3 ^ (-i)^ i.e. (-1, 3) e R but{3,-1) g R-


for
ur
So, R is not symmetric.
Transivity: We observe that
s
ook

2<{-3)^ and -3<l^but2$l^ i.e. (2, - 3) e R and (-3,-1) e R but (2,1) ^ R.


Yo
eB

So, R is not transitive.


EXAMPLE 11 Let A = (1, 2, 3}. Then, show that the number of relations containing (1,2) and (2,3)
lohich are reflexive and transitive but not symmetric is three.
r
ad
ou

SOLUTION The smallest reflexive relation on set A containing (1, 2) and (2,3) is
R={(1,1),(2, 2), (3, 3), (1,2), (2, 3)}
Y

Since(l,2) eRand{2, 3) eRbut(l, 3) 0 R. So, R is not transitive. To make it transitive we have


Re
nd

to include (1,3) in R. Including (1,3) in R, we get


R-l = {{1,1), (2, 2), (3, 3), (1, 2), (2, 3), (1, 3)}
Fi

This is reflexive and transitive but not symmetric as (1, 3) e Rj but (3,1) ^ Rj.
Now, if we add the pair {2, 1) to Rj to get R2 =1(1/1)/ (2, 2), (3, 3), (1, 2), (2, 3), (1, 3), (2,1)}.
The relation R2 is reflexive and transitive but not symmetric. Similarly, by adding (3,2) and (3,1)
respectively to Ry we get
R3 = {(1,1), (2, 2), (3, 3), (1, 2), (2, 3), (1, 3),(3, 2)},
R3 ^ {(1,1), (2, 2), (3, 3), (1, 2), (2, 3), (1, 3), (3,1), (3, 2)}
These relations are reflexive and transitive but not symmetric.

We observe that out of ordered pairs (2,1), (3, 2) and (3,1) at a time if we add any two ordered
pairs at a time to Rj, then to maintain the transitivity we will be forced to add the remaining
third pair and in this process the relation will become symmetric also which is not required.
Hence, the total number of reflexive, transitive but not symmetric relations containing (1,2) and
(2,3) is three.
7.30
APPLIED MATHEMATICS-X!

EXERCISE 7.4

1 Let /4 be the set of all human beings in a town at a particular time. Determine whether each
of the following relations are reflexive, symmetric and transitive:
(i) R = {(a:, y): x and y work at the same place}
(ii) R = {(a:, y): x and y live in the same locality}
(iii) R = {(a:, y): a: is wife of y}
(iv) R = {(at, y): a: is father of y}
-● Relations R-y, R2 , R3 and R4 are defined on a set = {a, b, c} as follows:
^1 = {(fl/ a)r {a, h), {a, c), {h, b), (b, c), (c, a), (c, b), (c, c)}
^2 = {(«/«)}

R4 = {{n,b),{b,c),{c,a)}.
Find whether or not each of the relations Ry R2, R 3' R4 on A is (i) reflexive (ii)

w
symmetric (iii) transitive.
3. Test whether the following relations Ry R2, and R3 are
(i) reflexive (ii) symmetric and

F lo
(iii) transitive:
(i) Ri on Qo defined by {a, &) e <=> <7 = 1/b
(ii) R2 on Z defined hy{a,b)€R2o\a-b\<5

ee
Fr
(iii) R3 on R defined by (<7,1?) € R3 <=> -4 + 3b^ =0.
LetA = {l,2,3},andletRi = |(l,l),(l,3),(3,l),(2,2),(2,l),(3,3)|, R2 = |(2,2), (3,1), (1,3)|,
for
^3 = 3), (3,3)). Find whether or not each of the relations Ry R2, R3 on A is (i) reflexive
ur
(ii) symmetric (iii) transitive.
The following relations are defined on the set of real numbers:
s
ook

(i) aRb ii a-b>0


Yo

(ii) aRb iff 1 + >0 (iii) aRb if| <?| <&.


Find whether these relations are reflexive, symmetric or transitive.
eB

Check whether the relation R defined on the set A = {1, 2, 3, 4,5, 6} as R = {{a, b):b=a + l)
is reflexive, symmetric or transitive.
our
ad

Check whether the relation R on R defined by R = {{a, b):a<b^}is reflexive, symmetric or


transitive.

H. Prove that every identity relation on a set is reflexive, but the converse is not necessarily
Y
Re

true.
nd

If A = {1, 2, 3, 4}, define relations on A which have properties of being


Fi

(i) reflexive, transitive but not symmetric,


(ii) symmetric but neither reflexive nor transitive,
(iii) reflexive, symmetric and transitive.
Let R be a relation defined on the set of natural numbers N as

R y) -x,ye N, 2x + y = 41}
Find the domain and range of R. Also, verify whether R is (i) reflexive, (ii) symmetric
(iii) transitive.
11
Is it true that every relation which is symmetric and transitive is also reflexive? Give
reasons.

An integer m is said to be related to another integer n if m is a multiple of n. Check if the


relation is symmetric, reflexive and transitive.
1.3
Show that the relation ">" on the setR of all real numbers is reflexive and transitive but not
symmetric.
RELATIONS 7.31

ij. Give an example of a relation which is


(i) reflexive and symmetric but not transitive,
(ii) reflexive and transitive but not symmetric,
(iii) symmetric and transitive but not reflexive,
(iv) symmetric but neither reflexive nor transitive,
(v) transitive but neither reflexive nor symmetric.
15. Given the relation R = {(1, 2), (2, 3)1 on the set /I = (1, 2, 3|, add a minimum number of
ordered pairs so that the enlarged relation is symmetric, transitive and reflexive.
16. Let A - {1, 2,3| and R = 1(1,2), (1,1), (2,3)} be a relation on A. What minimum number of
ordered pairs may be added to R so that it may become a transitive relation on A.
17. Let A = {a, b, c} and the relation R be defined on A as follows: R = {{a, a), {b, c), {a, b)}. Then,
write minimum number of ordered pairs to be added in R to make it reflexive and
transitive.

w
18. Each of the following defines a relation on N:
(i) x>y,x,y^N (ii) ,Y + y =10, -Y, y e N

F lo
(iii) .vy is square of an integer, .Y, y e JV (iv) x +iy =10, x, y eN
Determine which of the above relations are reflexive, symmetric and transitive.

ee
ANSWERS

Fr
1. (i) Reflexive, symmetric and transitive
(ii) Reflexive, symmetric and transitive
for
(iii) Neither reflexive, nor symmetric but transitive
ur
(iv) neither reflexive nor symmetric nor transitive
2. R^ is reflexive but neither symmetric nor transitive.
s
ook
Yo

R2 is symmetric and transitive but not reflexive.


R3 is transitive but neither reflexive nor symmetric.
eB

R4 is neither reflexive nor symmetric nor transitive.


3. (i) Rj is symmetric but it is neither reflexive nor transitive
our
ad

(ii) R2 is reflexive and symmetric but it is not transitive


(iii) R3 is reflexive but it is neither symmetric nor transitive.
■ Rj is reflexive but neither symmetric nor transitive
dY
Re

R2 is symmetric but neither reflexive nor transitive.


R3 is transitive but neither reflexive nor symmetric.
Fin

' (i) Transitive (ii) Reflexive and symmetric but not transitive (iii) Transitive neither
reflexive nor symmetric.
. Neither reflexive nor symmetric nor transitive
Neither reflexive nor symmetric nor transitive.
0. (i) R = {(1,1), (2, 2), (3,3), (4, 4), (1,2)1 (ii) R = {(1,2), (2,1)1
(iii) R = ((1,1), (2, 2), (3, 3), (4,4), (1, 2), (2,1)1
V Domain R = {1, 2, 3,..., 19, 20), Range R = {39, 37, 35,..., 7, 5, 3,1].
R is neither reflexive nor symmetric and is not transitive.
u. No. Relation R = |(1,1), (1, 2), (2,1), (2, 2)) on A = |1, 2, 31 is symmetric and transitive but
not reflexive.

1: Reflexive and transitive but not symmetric.


! (i) R = 1(1,1), (2, 2), (3, 3), (1,3), (3,1) (2,3), (3, 2)) on A- {1, 2, 3}
(ii) R = 1(1,1), (2, 2), (3, 3), (1, 3)1 on A = [1, 2, 3|
7.32 APPLIED MATHEMATICS-XI

(iii) R = {(1,3), (3,1), (1,1), (3,3)} on A = {1,2,3}


(iv) K= {(1,1), 0,3), (3,1), {2, 3), (3, 2)) on 71 =11,2,3} (v) R = [(1,1)) on 7l =(1, 2, 3}
(1,1), (2, 2), (3,3), (1,3), (2,1), (3,2), (3,1) 16. (1,3), One 17. {b, b), (c, c), {a, c)
(i) transitive (ii) symmetric (iii) reflexive, symmetric and transitive (iv) transitive.
.HINTS TO SELECTED PROBLEMS

(iv) The relation R on the set A of all human beings in a town is given by (x, y) e K iff x is
father of y.
Reflexivity: Since a person a: cannot be father of himself. So, {x, a:) g A. Consequently, R is
not reflexive.

Symmetr.y Let x,y eAhe such that (a:, y) e R. Then,


{x, y) € R
=> a: is father of y

w
^ y cannot be father of a:

F lo
So, R is not symmetric.
Transitivity: Let at, y, z e be such that (a:, y) e R and (y, z) e R. Then,

ee
{x, y) € R and (y, z) g R

Fr
=> a: is father of y and y is father of z
=> a: is grandfather of z for
=> {x, z)iR
ur
The relation R on set A = {1, 2, 3, 4,5, 6} is defined as {a, b) ^Rif{ b = a + 1. Therefore,
R={(1,2),(2, 3), (3, 4), (4,5), (5, 6)}.
s
ook

Clearly, {a, a) iR for any a eA. So, R is npt reflective on A.


Yo

We observe that (1, 2) e R but (2,1) g R. So, R is not symmetric.


eB

We also observe that (1, 2) g R and (2, 3) g R but (1, 3) g R. So, R is not transitive.
The relation R on R is defined by R = {{a,b) :a <b^}
our

We observe that (-2) e R is such that (-2) < (-2) ^ is not true. So, R is not reflexive.
ad

Since 1 <(3^'^^)^ but 3^'^^ £1 i.e.(l, 3^'^^) e R but (3^^^^, 1) g R. So, R is not symmetric.
Y

R is not transitive because (5, 2) e R and (2, ^) g R but (5, 2^^ ^) g R.


Re

Let I be the identity relation on a set A. Then,


nd

{a, fl) g / for all fl g A => / is reflexive.


Fi

Converse: The relation {(1, 1), (2, 2), (3, 3), (1,3)} is areflexiverelationonsetA= (1,2,3)
but it is not the identity relation on A.
A relation R on the set Z of integers defined by (a,b) sR a and b are both odd, is
symmetric and transitive but it is not reflexive. Because no even integer is related to itself.
For reflexivity, we must add (1,1), (2,2) and (3,3). For symmetry and transitivity we must
add (2,1), (3, 2), (1,3), (3,1) in R.
7.6.6 EQUIVALENCE RELATION

DEFINITION A relation Ron a set A is said to be an equivalence relation on A iff it is


(i) reflexive i.e. {a, a)sR for all a&A.
(ii) symmetric i.e. (a, b) e R => (b, a) e R for all a,b eA.
and, (iii) transitive i.e. {a, b) sRand{b, c)eR =>{a, c) eRforalla,b, c g A
An equivalence relation R defined on a set A partitions the set A into pair wise disjoint subsets.
Tl-iese subsets are called equivalence classes determined by relation R. The set of all elements of
A related to an element a g A is denoted by [rt] i.e. [a\ = \x sA: (a:, a) g R}. This is an equivalence
RELATIONS 7.33

class. Corresponding to every element in A there is an equivalence class. Any tv/o equivalence
classes are either identical or disjoint. The collection of all equivalence classes forms a partition
of set A.

ILLUSTRATIVE EXAMPLES

EXAMPLE 1
Let R be a relation on the set of all lines in a plane defined by (/|, I2) ^R line l^ is
parallel to line I2. Show that R is an equivalence relation.
SOLUTION Let L be the given set of all lines in a plane. Then, we observe the following
properties.
Reflexive: For each line / € L, we have

ow
/||/ => (LOeRforall/EL
R is reflexive

Symmetric: Let /j, I2 such that (/^ I2) e R. Then,


{f, I2) .eR=> li\\l2=> l2\\h => ih' h) ^

e
So, R is symmetric on L.

re
rFl
Transitive: Let/j,/2/^3 e L such that(/j,/2) s R and (/2,/3) e R. Then,

F
(/j, I2) e R and (l2, ^3) e R => IU2 hW hW h ^ h) ^ ^
So, R is transitive on L.

r
ou
Hence, R being reflexive, symmetric and transitive is an equivalence relation on L.
fo
ks
EXAMPLE 2 Shozv that the relation 'is congruent to' on the set of all triangles in a plane is an equivalence
relation.
oo

SOLUTION Let S be the set of all triangles in a plane and let R be the relation on S defined by
Y

(Aj, A2) e R o triangle is congruent to triangle A2.


B

We observe the following properties of relation R:


re

Reflexivity: For each triangle A € S, we have


A = A ^ (A, A) e R for all A e S ^ R is reflexive on S
ou
Y
ad

Symmetry: LetAi,A2eS such tliat(Aj, A2) e R. Then,


(Aj, A2) s R A^ = A2 —^ A2 = Aj {A2/ Aj) e R.
d

So, R is symmetric on S
in
Re

Transitivity: Let Aj, A2, A3 e S such that {Aj, A2) e R and (A2/ A3) e R. Then,
(Aj, A2) € R and (A2, A3) € R ^ A^ = A2 and A2 = A3 => A^ = A3 ^ (Aj, A3) e R
F

So, R is transitive on S.
Hence, R being reflexive, symmetric and transitive, is an equivalence relation on S.
EXAMPLES Show that the relation R deifned on the set A of all triangles in a plane as R - {(Tj, T2) :Tj is
similar to T2} is an equivalence relation.
Consider three right angle triangles T^ with sides 3,4,5; T2 with sides 5,12,13 and T3 with sides 6,8,10.
Which triangles among -T^, T2 and T^ are related?
SOLUTION We observe the following properties of relation R.
Reflexivity: We know that every triangle is similar to itself.
(T,T)eRforaUreA
^ R is reflexive.

Symmetry: Let(Ti,r2) eR.Then,


(-TI.T2) 6R
7.34 APPLIED MATHEMATiCS-XI

^ Ti is similar to T2
^ T2 is similar to
^ (T2,ro ek
So, R is symmetric.
Transitivity: LetT|, T2, T3 s A such that(Tj, T2) e R and {T2, T’3j e R. Then,
(T|,T2) eR and (73,73) eR
=> 7| is similar to T2 and 73 is similar to
=> 7| is similar to 73
^ (7i, 73) eR
So, R is transitive.

Hence, R is an equivalence relation on set A.

ow
In triangles 7| and 73, we observe that the corresponding angles are equal and the
3 4 5
corresponding sides are proportional i.e. ^ Hence, Tj and 73 are related.
KX.\MPLE 4 Let n be a positive integer. Prove that the relation R on the set Z of all integers numbers

e
defined by{x, y) e R

Fl
x-y is divisible by n, is an equivalence relation on Z.

re
SOLUTION We observe the following properties of relation R.

F
Refiexivity: For any a sN
ur
a-a = 0 = Oxu

a -ais divisible by n
{a, a) sR
f or
ks
Thus, {a, a) e R for all a eZ. So, R is reflexive on Z
Yo
oo

Symnetry: Let (a, b) e R. Then,


(a, b) gR
B

=> {a -b) is divisible by n


re

=> (a-b) = for some p E Z


=> b - a = n{-p)
u
ad

=> b - a is divisible by n [●.● peZ^-peZ]


Yo

{b, a) eR
Thus, {a, b) gR=> (b,a) eR for all a,b eZ.
d
Re
in

So, R is symmetric on Z.
Transitivity: Let a,b,c eZ such that (a, b) e R and {b, c) e R. Then,
F

(a, b) eR
{a - b) is divisible by n
=>
a - b =np for some p eZ
and. (b, c) e R
{b - c) is divisible by n
b - c = nq for some q eZ
(a, b) eR and {b, c) eR
a-b - npandb-c = nq
{a -b) + {b -c) = np + nq
a-c = n{p + q)
a - c is divisible by n [●.● p, q eZ=> p + q eZ]
{a, c)eR
RELATIONS 7.35

Thus, {a, b) eR and {b, c) e R:^ {a, c) e R for all a,b, c e Z.


So, R is transitive relation on Z.

Thus, R being reflexive, symmetric and transitive, is an equivalence relation on Z.


REMARK In the above example, if we take n = 2, then R can be described as
(x, y) s R o a: -y is divisible by 2
Clearly, R is an equivalence relation on Z.
Let us now find the equivalence classes determined by R.
[0] = lx € Z : (.V, 0) e R} = {a: e Z ; X - 0 is divisible by 2} = |x e Z : x is divisible by 2}
= {0,±2, + 4,±6, ,.J
[1] ={x e Z : (x, 1) E R) = (x e Z : x-1 is divisible by 2}

ow
^ [1] = {x e Z :x-l = 2A., X eZ}
=> [1] ={x E Z : X = 2A. + 1, e Z}
^ [1]=|±1,±3,±5,±5,±7,...(
[2] = |x € Z : (x, 2) E = |x € Z : X - 2 is divisible by 2}
[2] = |x € Z : X - 2 = 2X, X e Z} = {x E Z : X = 2 + 2?., A. E Z|

e
^

Fl
[2] = |0,± 2, ± 4, ± 6,.. J, which is same as the equivalence class [0]

re
[3] = |x E Z : (x, 3) E R} = |x E Z : X - 3 is divisible by 2]

F
=> [3] = {x e Z : X - 3 = 2X, A. E Z) = {x E Z : X - 3 + 2A,, X G Z1
=> [3] = |± 1, ± 3, ± 5 ± 7,.. J, which is sam as the equivalence class [1]
ur
Continuing in this manner, we find that
[0] = [2] = [4]=[6]=... or
sf
[1]=[3] = [5] = [7]=...
k
Yo
and. [0]n[l] = (liAlso,Z=[01^[l].
oo

Thus, R partitions the set Z into two pair wise disjoint sets known as equivalence classes.
B

Similarly, the relation R on Z given by


(x, y) 6 R o X -y is divisible by 3
re

partitions Z into 3 pair wise disjoints sets i.e. equivalence classes given by
[0] = {-6,-3, 0, 3, 6, 9,..j
u
ad

[!]={...,-8, -5,-2,1,4,7,10,...!
Yo

[2]={...,7,-4,-l,2,5, 8,11,..J
suchthatZ =[0]vj[1]u[2].
d
Re

EXAMPLE 5 Show that the relation R on the set A of all the books in a library of a college given by
in

R = {{^/ y) ● y sflmc number of pages}, is an equivalence relation.


F

SOLUTION We observe the following properties of relation R.


Reflexivity: For any book x in set A, we observe that
X and X have the same number of pages,
(x, x) eR
Thus, (x, x) 6 R for allx e A.
So, R is reflexive.
Symmetr.y Let (x, y) e R. Then,
(x, y) e R
X and y have the same number of pages
y and x have the same number of pages
(y, x) E R
Thus, (x, y) e R ^ (y, x) G R
7.36 APPLIED MATHEMATICS-XI

So, R is symmetric.
Trnrisitivity: Let(a‘, y) eR and (y, z) eR. Then,
(x, y) e J? and (y,z) eR
=> (a: and y have the same number of pages) and (y and z have the same number of
pages)
^ AT and 2 have the same number of pages.
^ (a:,z) eR
So, R is transitive.
Thus, R is reflexive, symmetric and transitive.
Hence, R is an equivalence relation.
EX.A.MPLE 6 Show tfiat the relation R on the set A
= {1,2, 3, 4,5}, given by

w
R = {{a, b) :\ a-b\ is even }, is an equivalence relation.
Show that all the elements of [1,3,5} are related to each other and all the elements of {2,4) are related to
each other. But, no element o/jl, 3,5} is related to any element of [2,4] ■

Flo
SOLUTION We have,
R = {(r?, b):\a - b\ is even}, where a,b g A = {1, 2, 3, 4,5}.

ee
We observe the following properties of relation R.

Fr
Reflexivity: For any a sA, we have
] - rt j = 0, which is even
for
ur
{a, a) eR for dX\a eA
So, R is reflexive.
ks
Symmetr.y Let {a, b) eR. Then,
Yo

(a, b)eR
oo

I fl - cf I is even
B

I ft - <71 is even
re

{b, a) eR
Thus, {a, b) eR => (b,a) eR
ou
ad

So, R is symmetric.
Y

Transitivity: Let {a, b) eR and {b, c) eR. Then,


(a, b) eR and {b, c) e R
nd
Re

=>
I « - 61 is even and | & - c | is even
(fl and b both are even or both are odd) and {b and c both are even or both are odd)
Fi

Now two cases arise:


■:asI: I When b is even
In this case.
(a, b) eR and (b, c) e R
I I is even and 11) - c | is even
a is even and c is even [●.● b is even]
=> I /7 - c I IS even
=> (a, c) G R
. ■ -I II When b is odd
In this case.
{a,b)eR and {b,c)eR
I fl - & I is even and | cf - c | is even
a is odd and c is odd [●.■ b is odd]
RELATIONS 7.37

I fl - c I IS even
(rt, c)eR
Thus, {a, b) and {b, c) eR => {a, c) eR
So, R is transitive.
Hence, R is an equivalence relation.
We know that the difference of any two odd (even) natural numbers is always an even natural
number. Therefore, all the elements of set (1, 3,5} are related to each other and all the elements
of {2, 4} are related to each other.
We know that the difference of an even natural number and an odd natural number is an odd
natural number. Therefore, no element of (1, 3,5} is related to any element of {2,4}.

w
EXAMPLE 7 Show that the relation R on the set A = (x e Z : 0 < x < 12}, given by R = {(a,b) : \ a-b\
is a multiple of 4} is an equivalence relation. Find the set of all elements related to 1 i.e. equivalence
class [1].

o
SOLUTION We have,
R = {(a,b) :| - b I is a multiple of 4), wherefl,6 e A = {x e Z : 0 <x <12} = 10,1, 2,..., 12}.

e
re
rFl
We observe the following properties of relation R.
Reflexivity: For any a € A, we have

F
\ a - a\=0, which is a multiple of 4.
(fl, a) eR

r
{a, a) eR for all aeA. fo
ou
Thus,
So, R is reflexive.
ks
Symmetry: Let(fl,fc) eK.Then,
{a, b) eR
oo

=> I <7 - b I is a multiple of 4


Y
B

^ I fl -j = 4X for some XeN


=> 1- fl I = 4^ for some XeN [v \a-b\ =|b-fll]
e

=> (b, a) eR
ur

So, R is symmetric.
ad

Transitivity: Let (fl, b) eR and {b, c) e R. Then,


Yo

(fl, b) eR and (b, c) eR


I fl -I is a multiple of 4 and | & - c | is a multiple of 4
d
Re

\a - b\ = 4tX and \b -c\ = 4p for some X,[ieN


in

=> a-b = ± 4k and b-c = ± 4p


F

fl - c - ± 4k± 4|i
fl - c is a multiple of 4
I fl - c I is a multiple of 4
(fl, c) eR
Thus, {a,b)eR and {b,c)eR ^ {a,c)eR
So, R is transitive.
Hence, R is an equivalence relation.
Let X be an element of A such that (x, 1) e R. Then,
I X -11 is a multiple of 4
|x-l| = 0,4,8,12
x-1 = 0,4,8,12
X = 1,5,9 [v 13 eA]
Hence, the set of all elements of A which are related to 1 is {1,5, 9} i.e. [1] ={1,5, 9).
7.38 APPLIED MATHEMATICS-XI

EXAMPLE 8 Show that the relation R on the set A of points in a plane, given bp
^={(PrQ)' Distance of the point P fro7n the origin is same as the distance of the point Qfrom
the origin},
is an equivalence relation. Further show that the set of all points related to a point P ^ (0, 0) is the circle
passing through P with origin as centre.
SOLUTION Let O denote the origin in the given plane. Then,
R={{P,Q):OP=OQ}
We observe the following properties of relation R.
Reflexivity: For any point P in set A, we have
OP =OP

=> (P, P)eP

w
Thus, (P, P) e R for all P g A
So, R is reflexive.

Flo
Symmetry: Let P and Q be two points in set A such that
Q)eP

ee
=> OP = OQ

Fr
=> OQ = OP
{Q,P)^-R
Thus, (P, Q)eR => (Q, P) e R for all P,Q&A
for
ur
So, R is symmetric.
s
Transitivity: Let P, Q and S be tliree points in set A such that
k
Yo

(P,Q)gR and (Q, S)gR


oo

=> OP=OQ and OQ-OS


eB

=> OP=OS

=> (P, S)gR


r
ou
ad

Thus, (P,Q) e R and (Q, S) G R (P, S) G R for all P,Q,SgA


So, R is transitive.
Y

Hence, R is an equivalence relation.


Re
nd

Let P be a fixed point in set A and Q be any point in set A such that (P, Q) e R. Then,
{P.Q)^R
Fi

=> OP OQ
Q moves in the plane in such a way that its distance from the origin
O (0, 0) is always same and is equal to OP.
=> Locus of Q is a circle with centre at the origin and radius OP.
Hence, the set of all points related to P is the circle passing through P with origin O as centre.
REM.ARK Let rn be an arbitrary butfixed integer. Two mtegers a and b are said to be congruence modulo
m if a - b is divisible by m and we write a=b (mod m).
Thus, a=b (mod m) a -b is divisible bym.
For example, 18 = 3 (mod 5) because 18 - 3 =15 zohich is divisible by 5. Similarly, 3 =13 (mod 2)
because 3 -13 = -10 which is divisible by 2. But 25 ^ 2 (mod 4) because 4 is not a divisor of25 -2 = 23.
|-;XAMPLE 9 Prove that the relation ‘congruence modulo m' on the set Z of all integers is an equivalence
relation.
RELATIONS 7.39

SOLUTION We observe the following properties of the given relation.


Reflexivity: Let a be an arbitrary integer. Then,
a-a = 0 = Oxm ^ n - a is divisible by=> a=n (mod??;)
Thus, n = a (mod m) for all e Z.
So, "congruence modulo is reflexive.
Symmetry: Let a,b eZ such that a=b (mod m). Then,
a = b (mod m)
=> a ~b \s divisible by m
=> n - b = Xm (or XeZ

=> b - a = {-X)m
=> -i7 is divisible by [●.● XeZ^-XeZ]
=> fj=<7(modm)

w
So, "congruence modulo m" is symmetric on Z.

F lo
Transitivity: Let n,b,c & Z such that a=b (mod m) and b = c (mod m). Then,
a = b (mod m) => /? - is divisible by m=> a- b = X^ m for some X^ eZ
b = c (mod m)=> b - CIS divisible by m =5- b - c = X-2 m for some ^2 € Z

ee
Fr
{a-b) + {b-c) = X-im + X2m = {X-[+X2)m
fl - c = X3 m, where ^3 = A.;; + X2 e Z.
for
a = c (mod m)
r
Thus, a^b (mod ;») and b = c (mod m) ^ <7 = c (mod m).
You
s

So, "congruence modulo w" is transitive on Z.


ook

Hence, "congruence modulo m" is an equivalence relation on Z.


eB

EXAMPLE 10
Show that the number of equivalence relations on theset (1, 2, 3}containing (1,2) and
(2, 1) is tivo.
our

The smallest equivalence relation Rj containing (1,2) and (2,1) is


ad

SOLUTION

Kl = {(l,l),(2, 2), (3, 3), (1,2), (2,1)}


Now, we are left with four ordered pairs namely (2,3) (3,2), (1,3) and (3,1). If we add any one,
dY

say (2,3) to R^, then for symmetry we must add (3,2) and then for transitivity we are forced to
Re

add (1,3) and (3,1). Thus, the only equivalence relation other than R-j is the universal relation.
Fin

Hence, the total number of equivalence relations containing (1,2) and (2,1) is two.
EXAMPLE li
Given a non-empty set X, consider P (X) which is the set of all subsets of X. Define a
relation in P (X) as follows:
For subsets A, B mP (X), A R Bif A <z B.Is Ran equivalence relation on P (X)? Justify your answer.
SOLUTION It is given that for any y4, B in P (X): ARB AcB
We observe the following properties of R.
Reflexivity: For any A in P (X), we have
AcA => ARA

So, R is reflexive on P (X).


Symmetry: Let A, B in P (X) be such that ARB. Then,
ARB => A^B

This need not imply that B c A. In fact it is possible only when A = 6.


Also, we know that {1, 2} c: {1, 2, 3}, but {1, 2, 3} tz {1, 2}.
So, R is not a symmetric relation on P (X).
7.40 APPLIED MATHEMATICS-XI

Transitivity: Let A, B, C be in F (X) such that


ARB and B RC => A czB and B czC ^ A aC ^ ARC
So, F is a transitive relation on P (X).
Hius, R is reflexive and transitive relahon on P (X) but it is not symmetric.
Hence, R is not an equivalence relation on P (X).

w
" ,Ml*LE ir.
Let R be the equivalence relation in the set A = {0,1,2, 3,4,51 given by
R ={{a, b): 2 divides (a-b)]. Write the equivalejice class [0].
SOLUTION Clearly, the equivalence class [0] is the set of those elementsin A which are related
to 0 under the relation R. i.e. [0] = {(a, 0) € F : a €

e
Now, (a, 0) e F

e
=> a - 0 is divisible by 2 and a e A

or
=> a sA such that 2 divides a

r
a = 0,2,4

F
Thus, [0]={0, 2, 4).

oF
ul
I XAMPLE i:
On the set N of all natural numbers, a relation R is defined as follows:
nRm <=> Each of the natural numbers n and m leaves the same remainder less than 5 when divided by 5.

rs
Show that R is an equivalence relation. Also, obtain the pairwise disjoint subsets determined by R.
SOLUTION We observe the following properties of relation F.

ko
Reflexivity: Let a be an arbitrary element of N. Then, either a is less than 5 and if a > 5, then on
dividing a by 5 we obtain a remainder as one of the numbers 0,1,2,3,4.
of
Thus, aFa for all a e N. So, F is reflexive on N.
o
Y
Symmetr.y Let a, & e N such that aRb. Then,
aRb ^ Each of a and b leaves the same remainder less than 5 when divided by 5
YB

=> Each of b and a leave the same remainder less than 5 when divided by 5
=> bRa
er

Thus, aRb => bRa for all a,b eN. So, F is symmetric.
u

Transitivity : Let a,b, c eNhe such that aRb and bRc. Then,
od

aRb => Each of a and b leaves the same remainder less


ad

than 5 when divided by 5


bRc => Each of b and c leaves the same remainder less than 5 when divided by 5
in

Each of a and c leaves the same remainder less than 5 when divided by 5
aRc
Re
F

Thus, aRb and bRc => aRc for a\la,b, c ^N.


So, F is a transitive relation on N.
Hence, F is an equivalence relation on N.
Let us now find the equivalence classes.
[l] = {xeN:xR 1|
= {x .€N :x and 1 leave the remainder less than 5 when divided by 5)
= {x eN :x leaves the remainder 1 when divided by 5)
= !1, 6,11,16, 21,...)
[2]={xeN:xF 21
= [x eN : Each of x and 2 leave the remainder less than 5 when divided by 5}
= [x eN :x leaves the remainder 2 when divided by 5)
= [2,7,12,17,22,..)
[3]={xeN:xR 3}
RELATIONS 7.41

= {x eN: Each of x and 3 leave the remainder less than 5 when divided by 5}
= {x eN :x leaves the remainder 3 when divided by 5}
= |3, 8,13,18, 23,.. j
[A] = {xeN:xR 4}
= {x 6 N: Each of x and 4 leave the remainder less than 5 when divided by 5}
= {x 6 N: X leaves the remainder 4 when divided by 5}
= {4,9,14,19,..J
[5] = {x e N : X K 5}
= {x e N: Each of x and 5 leave the remainder less than 5 when divided by 5}
= {x e N: X leaves the remainder 0 when divided by 5)

w
= {5,10,15,.,J
Proceeding in this manner we find that
[1]=[6]=[11]...

Flo
[2]=[7]=[12]...
[3]=[8]=[13]...

ee
[4]=[9] = [14]...

Fr
and. [5]=[10] = [15]=...
Thus, we obtain the following disjoint equivalence classes: for
ur
ni [2], [3], [4], [5] such thatN = [l]u[2]u[31u[4]u[5]
s
MULTIPLE CHOICE QUESTIONS (MCQs)
k
Yo
oo

Mark the correct alternative in each of the following:


■I. If = { 1,2,4 ], B = I 2,4,5 ), C = { 2,5 }, then (A-B)x (B -C) is
eB

(a) {{1,2), (1,5), (2, 5)1 (b) Kh4)l


(c) (1,4) (d) none of these.
r
ou
ad

.. If is a relation on the set ,4 = { 1,2,3,4,5,6, 7,8, 9 ) given byxRy<e>y = 3x, then R=


(a) {(3,1), (6,2), (8, 2), (9,3)1 (b) I (3,1), (6, 2), (9,3)1
Y

(c) {(3,1), (2,6), (3,9)1 (d) none of these.


Re
nd

3. Let ,4 = 11, 2,3 1, B = {1,3,5 |. If relation R from A to B is given by R = {(1,3), (2, 5), (3,3) 1.
Then, R~^ is
Fi

(a) 1(3,3), (3,1), (5,2)1 (b) {(1,3), (2,5), (3,3)1


(c) 1(1, 3), (5,2)1 (d) none of these.
If = { 1,2,3 1, B = { 1,4,6,9 1 and R isa relation from ,4 to B defined by 'x is greater than y.
The range of R is
(a) {1,4,6,91 (b) {4,6,91 (c) I 1 1 (d) none of these.
J. If R = {(x, y) :x,y eZ,x^ + y^ < 4} is a relation on Z, then domain of R is
(a) {0,1,21 (b) (0,-1,-21 (c) I-2,-1, 0,1, 2 } (d) none of these.
6. A relation R is defined from {2,3,4,5 | to {3,6,7,10 ) by: x R y <=> x is relatively prime to y.
Then, domain of R is
(a) 12,3,51 (b) {3,51 (c) {2,3,4 1 (d) {2,3,4,51.
A relation (j) from C to R is defined by x <J> y <=> | x | = y. Which one is correct?
(a) (2+ 3 0 4»13 (b) 3 4>(-3) (c) (1+0<I>2 (d) /(|)1.
7.42 APPLIED MATHEMATICS-XI

8. Let R be a relation on N defined by a: + 2 y = 8. The domain of R is


(a) {2,4,8} (b) 12,4,6,8} (c) {2,4,6} (d) {1,2,3,41.
9. R is a relation from {11,12,13j to { 8,10,12 } defined by y = .r - 3. Then, R -1 :
IS

(a) I (8,11), (10,13) I (b) {(11,8), (13,10)}


(c) I (10,13), (8,11), (12,10)1 (d) none of these.
10. If the set A has p elements, B has q elements, then the number of elements in A x 6 is
(a) p + q (b) p + q + l (c) pq (d)
11. Let R be a relation from a set /I to a set B, then
(a) R=A^B (b) R=AnB (c) R c A X B (d) R^BxA.
12. If R is a relation from a finite set A having m elements to a finite set 6 having n elements,

low
then the number of relations from /4 to B is
mu
(a) 2 (b) 2”’" -1 (c) 2mn (d) HI*’
13. If R is a relation on a finite set having n elements, then the number of relations on A is
2
(a) 2' (c) i?
n
(b) 2 (d) 22”.

ee
rF
14. Let/j(/l) =m and n{B) =?2.Then the totalnumberof non-empty relations that can be defined

Fr
from /I to B is

(a) m‘ (b) //"-! (c) nm-1 (d) 2"'"-l


for
15- Let R be a relation on the set N given by R = {{a, b) \ a = b -2, b > 6}. Then,
(a) (2,4)eR (b) (3,8)gR (c) (6, 8)eR (d) (8,7)gR
u
16. Which of the following is not an equivalence relation on Z?
ks
Yo

(a) a Rb <=> n + b is an even integer (b) a R b <:> a -b is an even integer


oo

(c) a R b a <b (d) a R b o a =b


eB

ir. R is a relation on the set Z of integers and it is given by (.t, y) g R o I I ^l.Then,R is


(a) reflexive and transitive (b) reflexive and symmetric
(c) symmetric and transitive (d) an equivalence relation
r
ou
ad

18. The relationR defined on the set .A ={1, 2, 3, 4,5}byR = - (a, b):] a^-b^ { <16 ●, is given by
Y

(a) {(1,1), (2,1), (3,1), (4,1), (2, 3)1 (b) {(2, 2), (3, 2), (4, 2), (2, 4)}
(c) {(3, 3), (4, 3), (5, 4), (3, 4)} (d) none of these
nd
Re

19. Let R be the relation over the set of all straight lines in a plane such that /j R /2 o /j 1 I2.
Fi

Tlien, R is
(a) symmetric (b) reflexive
(c) transitive (d) an equivalence relation
20. If A = {a, b, c}, then the relation R = [{b, c)l on A is
(a) reflexive only (b) symmetric only
(c) transitive only (d) reflexive and transitive only
21. Let A = {2,3,4,5,..., 17,18}. Let be the equivalence relation on A x A, cartesian product
of A with itself, defined by(<7, b) ~{c, d) iHad - be. Then, the number of ordered pairs of the
equivalence class of (3,2) is
(a) 4 (b) 5 (c) 6 (d) 7
22. Let A ={1,2, 3}. Then, the number of relations containing (1, 2) and (1, 3) which are
reflexive and symmetric but not transitive is
(a) 1 (b) 2 (c) 3 (d) 4
23. The relation 'R' in N x N such that (rt, &) R (c, d) <=>/? + fr =& + c is
(a) reflexive but not symmetric (b) reflexive and transitive but not symmetric
RELATIONS 7.43

(c) an equivalence relation (d) none of these


If= {1,2,3), 6= {1,4,6,9} and R is a relation from/I to B defined by '.t is greater thany'.
The range of R is
(a) {1,4,6,91 (b) {4,6,91 (c) HI (d) none of these
25. A relation R is defined from {2,3,4, 5} to {3,6,7, W\hy: x R y x is relatively prime to y.
Then, domain of R is
(a) (2,3,51 (b) {3,51 (c) {2,3,41 (d) {2,3,4,51
26. A relation <() from C to R is defined by 4> i/ <^ | a.* | = y. Which one is correct?
(a) (2+3i)4>13 (b) 3 4) (-3) (c) (1+0 4.2 (d) /4>1
27. Let R be a relation on N defined by a: + 2 y = 8. The domain of R is
(a) {2,4,81 (b) {2,4,6,81 (c) {2,4,61 (d) {1,2,3,41
-1 ;
28. R is a relation from {11,12,131 lo {8,10,121 defined by y = x - 3. Then, R is

w
(a) {(8,11), (10,13)1 (b) {(11,8), (13,10)1
(c) {(10,13), (8,11), (8,10)1 (d) none of these

F lo
2'5. Let R = {{a, a), {b, b), (c, c), (a, 1?)} be a relation on set A = {n, b, c]. Then, R is
(a) identity relation (b) reflexive

ee
(c) symmetric (d) equivalence

Fr
'fi. Let A = {1, 2, 31 and R = {(1, 2), (2, 3), (1, 3)1 be a relation on A. Then, R is
(a) neither reflexive nor transitive (b) neither symmetric nor transitive
(c) transitive (d) none of these
for
ur
31. If R is the largest equivalence relation on a set A and S is any relation on A, then
(a) R c S (b) ScR (c) R=S (d) none of these
s
ok
Yo

32. If R is a relation on the set A = {1,2,3,4,5,6, 7, 8, 91 given by .r R y o y = 3 at, then R =


o

(a) {(3,1), (6,2), (8,2), (9,3)1 (b) {(3,1), (6, 2), (9, 3)1
eB

(c) {(3,1), (2,6), (3,9)1 (d) none of these


33.
If R is a relation on the set A ={1,2, 31 given by R =(1,1), (2, 2),(3, 3), then R is
r

(a) reflexive (b) symmetric (c) transitive (d) all the three options
ad
ou

34. If A = {a, b, c, d], then a relation R = {{a, h), {b, a), {a, a)} on A is
(a) symmetric and transitive only (b) reflexive and transitive only
Y

(c) symmetric only (d) transitive only


Re
nd

33. If A = (1, 2, 3}, then a relation R = {(2, 3)1 on A is


Fi

(a) symmetric and transitive only (b) symmetric only


(c) transitive only (d) none of these
36. Let R be the relation on the set A = {1, 2, 3, 4} given by
R = {(1, 2), (2, 2), (1,1), (4, 4), (1, 3), (3, 3), (3, 2)}. Then,
(a) R is reflexive and symmetric but not transitive
(b) R is reflexive and transitive but not symmetric
(c) R is symmetric and transitive but not reflexive
(d) R is an equivalence relation
31 Let A = {1, 2, 3}. Then, the number of equivalence relations containing (1,2) is
(a) 1 (b) 2 (c) 3 (d) 4
3S.
Tlie relation R= {(1,1), (2,2), (3,3)1 on the set {1,2,31 is
(a) symmetric only (b) reflexive only
(c) an equivalence relation (d) transitive only
7.44 APPLIED MATHEMATICS-Xl

l
39. S is a relation over the set R of all real numbers and it is given by {a, b) eSo ab >0.
Then, Sis

f
u
(a) symmetric and transitive only (b) reflexive and symmetric only
(c) antisymmetric relation (d) an equivalence relation

i
In the set Z of all integers, which of the following relation R is not an equivalence relation?
(a) .V K y: if .Y <y (b) .Y K y : if = y
(c) .V 1? y: if A" - y is an even integer (d) .Y R y: if .Y s y (mod 3)
LetA ={1, 2, 31 and consider the relation K = {(1,1), (2, 2), (3, 3), (1,2), (2, 3),(1, 3)1. Then,
R is

(a) reflexive but not symmetric (b) reflexivebut not transitive


(c) symmetric and transitive (d) neither symmetric nor transitive
42. The relation Sdefined on the set R of all real number by the rule aSb ifia>b is
(a) an equivalence relation
F

(b) reflexive, transitive but not symmetric


(c) symmetric, transitive but not reflexive
(d) neither transitive nor reflexive but symmetric
e
»3. The maximum number of equivalence relations on the set A = |1, 2, 31 is
(a) I (b) 2
o

(c) 3 (d) 5
44. Let R be a relation on the set JV of natural numbers defined hyn R iff n divides m. Then, R is
(a) Reflexive and symmetric (b) Transitive and symmetric
(c) Equivalence (d) Reflexive, transitive but not symmetric
Let L denote the set of all straight lines in a plane. Let a relation R be defined by IR m iff I is
perpendicular to for all /, m e L Then, R is
(a) reflexive (b) symmetric (c) transitive (d) none of these
Let T be the set of all triangles in the Euclidean plane, and let a relation R on T be defined as
aRb a a is congruent iob for all n,b s T.Then, R is
(a) reflexive but not synunetric (b) transitive but not symmetric
(c) equivalence (d) none of these
Consider a non-empty set consisting of children in a family and a relation R defined as a Rb
ad

if a is brother of b. Then, R is
(a) symmetric but not transitive (b) transitive but not symmetric
(c) neither symmetric nor transitive (d) both symmetric and transitive
Re

48. For real numbers .y and y, define y R y iff a- -y + V2 is an irrational number. Then the relation
R is

(a) reflexive (b) symmetric (c) transitive (d) none of these


49. If a relation R on the set |1,2,3) be defined by R = {(1, 2)1, then R is
(a) reflexive (b) transitive (c) symmetric (d) none of these
ANSWERS

1. (b) 2. (d) 3. (a) (c) 5. (c) 6. (d) 7. (d) 8. (C) 9. (a)


10. (c) 11. (c) 12. (a) 13. (b) 14. (d) 1-. (c) 16. (c) 17. (b) 18. (d)
19. (a) 20. (C) 21. (c) ● (a) 2.‘. (c) 2-1. (C) 3- (d) (d) (c)
28. (a) 29. (b) 30. (c) 31. (b) 32. (d) 33. (d) 34. (C) 35. (C) 36. (b)
37. (b) 38. (C) 39. (b) 40. (a) 41. (a) 42. (b) 43. (d) 44. (d) 45. (b)
46. (C) 47. (b) 48. (a) 49. (b)

FILL IN THE BLANKS TYPE QUESTIONS (FBQs)


1. Let u(A) =iu andu{B) =u. Then the total number of non-empty relations that can be
defined from A to B is
RELATIONS 7.45

2. The smallest reflexive relation on a set A is the

If/I and B are two sets such thatn (A) =5 and n(B) = 7, then the total number of relations on
Ax Bis

4. A relation R on a set A is a symmetric relation iff


If R and S are two equivalence relations on a set A, then R nS is
6. If (1, 3), (2,5) and (3, 3) are three elements of A x B and n (A x 6) = 6, then the remaining
three elements of A x 6 are

The total number of reflexive relations on a finite set having » elements is

ow
If K = {(.V, y):x, 1/ £ W, = 25), then Domain (R) = and Range (R) =

The relation R = {(a:, y):x,y & Z, x^ + = 64} is

e
If A ={x : X e W, X <2), B = {x:x eN,l <x <5)andC = (3,5), then A x (BnC) =

re
11- If R ={(a:, y): where x eR and -5 < .r <5} is a relation, then range (R) =

Frl
1: If 71 (A n B') = 9,77 (A' n B) = 10 and n (A u 6) = 24, then 77 (A x B) =

F
i 3. If A = {3,5, 6, 9} and R is a relation in A defined as R = ((.r,y): .r + y < 18), then R in roster
form is
ou
r
14. If 77 (A X B) = 200 and n (A) =50, then the number of elements in P(B) is

so
kf
If A = {1, 2, 3, 4, 4,5, 6), then the number of subsets of A containing elements 2,3 and 5
IS
oo

9 9
hv IfR = (a:, y): a: +y <4, a', y e 2 ms a relation in Z, then the domain of R is
Y
eB

17 Let R be a relation in h/defined by R = {(.v,y): a + 2y = 8), then the rangeof R is


The number of relations on a finite set having 5 elements is
ur
oY

Let A ={1, 2, 3, 4} and R be the relation on A defined by {{a, h): a, b e A, nx b is an even


ad

number), then the range of R is


d

Let A = {1, 2, 3, 4,5). The domain of the relation on A defined by R ={(.t, y): y = 2a'-1), is
in
Re

rule (77, b) € Ro -b^\<5, then R


-1
If Rs a relation defined onset A =11, 2, 3) by the
F

-1
If R is a relation from A ={11,12,13} to B = {8,10,12} defined by y = x~3, then R

The smallest equivalence relation on the set A = [a, b, c, d\ is


24. The largest equivalence relation on the set A = {1, 2, 3} is
Let R be the equivalence relation on the set Z of integers given by R = {(77, b): 3 divides
a-b}. Then the equivalence class [0] is equal to
Let R be a relation on the set Z of all integers defined as (.t, y) e R x-y is divisible by 2.
Then, the equivalence class [1] is
The relation R = {(1, 2), (1, 3)) on set A = {1, 2, 3} is only.

ANSWERS
mn 35 -1
1. 2 2 Identity relation 2 4. R = R
7.46 APPLIED MATHEMATICS-XI

2
» -n
5. an equivalence relation 6. (1,5), (2, 3), (3,5) 7, 2

8. {0, 3,4,5),{0, 3,4,5) ‘' symmetric 10. {{0, 3),(1, 3)1


11. [-3,17] 12. 210

13. R = {(3,3), (3,5), (3,6), (3,9), (5,3), (5,6), (5,9(, (6,3), (6,5), (6,6), (6,9), (9,3), (9,5), (9,6)1
14. 2^ LS. 8 16. {-2, -1, 0,1, 2) 17. (1, 2, 3)
25
18. 2 19. {2,4} 20. {1, 2, 3}

21. 1(1,1), (2,1), (1, 2), (2, 2), (3, 2), (2, 3), (3, 3)) 22. 1(8,11), (10,13))
22. {{a,a),(b,b),{c,c),{d,d)\ 2!. Ax A 25. {0, ± 3, ± 6, ± 9,
26. |±1,±3,±5,±7, J 27. transitive

VERY SHORT ANSWER QUESTIONS (VSAQs)


Answer each of the following questions in one word or one sentence or as per exact requirement of the
question:

F low
1. IfA = |l,2,4),e = {2,4,5}andC={2,51,write(A-C)x(B-C).
:. If n (./I) = 3, n (B) = 4, then write n{Ax Ax B).
3. If B is a relation defined on the set Z of integers by the rule {x, y) e R
write domain of R.
for Fre x + y =9, then

4. If B = |(x, y):x, y eZ, + i/^ < 4} is a relation defined on the set Z of integers, then write
domain of B.

5. If B is a relation from set yl = |11,12,13) to set B =|8,10,12) defined by y = ,v - 3, then


write R~
eBo ks
Your

6. Let/I ={1,2, 3) and B ={(«,/?) :| -1?^ { <5, a, & eTl). Then write B as set of ordered
pairs.
ad

". Let B = {(a:, y): x, y gZ, y = 2x - 4). If {a, - 2) and (4, b^) € B, then write the values of
our

a and b.

8. If B = {(2,1), (4,7), (1, - 2),...), then write the linear relation between the components of
Re

the ordered pairs of the relation B.


9. UA = {l,3,5)andB = (2, 4),listthe elementsof R,if B ={(.r, y): r, y e .A x B and a: > y).
Find Y

10. IfB = {(.X, y): AT, y E W, 2.r + y = 8), then write the domain and range of B.
11. Let A and B be two sets such that n (A) = 3 and n (B) = 2. If {x, 1), (y, 2), {z, 1) are in A x B,
write A and B.

12. Let A = {1, 2, 3,5), B = {4, 6, 9) and B be a relation from A toBdefinedby


R = {(a:, y): X - y is odd ). Write B in roster form.
13. Write the domain of the relation B defined on the set 2 of integers as follows:
{a,b)eR o a^+b^ =25
14. If B = {(.X, y):x^ + y^ < 4; x, y e Z}is a relation on Z, write the domain of B.
15. Write the identity relation on set A = {a, b, c}.
16. Write the smallest reflexive relation on set A = {1, 2, 3,4}.
17. If B = {(x, y): X + 2y = 8} is a relation on N, then write the range of R.
18. If B is a symmetric relation on a set A, then write a relation between B and R~ .
RELATIONS 7.47

19. Let R = {(>:, y) :| | < 1} be a relation on set A = {1, 2, 3, A,5\. Write R as a set of
ordered pairs.
20. UA = (2, 3, 4}, B = {1, 3,7} and R = {(;c, \j): x e A, ij € B and .r < y| is a relation from A to B,
then write

21. Let/^ = {3,5,7}, 6 = {2, 6,10} and R be a relation from A to B defined by R = {(.r, y) and y
are relatively prime}. Then, write R and R”^
22. Define a reflexive relation.

23. Define a symmetric relation.

ow
24. Define a transitive relation.

25. Define an equivalence relation.


26. If ,4 = {3,5,7} and B = {2, 4, 9} and R is a relation given by "is less than", write R as a set
ordered pairs.
27. A = {1, 2, 3, 4,5, 6,7, 8} and if R = {(.r, y) :yisone half of .v; .r, y e/\} is a relation on A, then

e
re
write R as a set of ordered pairs.
28. Let A = {2, 3, 4,5} and B = (1, 3, 4}. If R is the relation from A to B given by a Rb iff "a is a

Frl
F
divisor of ". Write R as a set of ordered pairs.
29. State the reason for the relation R on the set {1, 2, 3} given by R = 1(1, 2), (2,1)} not to be
transitive.
ou
r
30. Let R = {(fl, n^): <? is a prime number less than 5} be a relation. Find the range of R.
so
31. Let R be the equivalence relation on the set 2 of integers given by R = ((/i, b):2 divides a -b].
kf
Write the equivalence class [0).
oo
32. For the set A = {1, 2, 3}, define a relation R on the set A as follows:
R^{(1,1),(2, 2),(3, 3), (1,3)1
Y

Write the ordered pairs to be added to R to make the smallest equivalence relation.
eB

33. Let A = {0,1, 2, 3} and R be a relation on A defined as


R ={(0, 0), (0,1), (0, 3), (1, 0), (1,1), (2, 2), (3, 0), (3, 3)|
ur
oY

Is R reflexive? symmetric" transitive?


3, 4,5} by R ={(n, b):\a^ 8). Write R
ad

34. Let the relation R be defined on the set A ={1,2,


as a set of ordered pairs.
d

35. Let the relation R be defined on N by (iRb iff 2n + 3b = 30. Then write R as a set of ordered
in

pairs.
Re

36. Write the smallest equivalence relation on the set A = {1, 2, 3).
F

ANSWERS

1. 1(1, 4), (4, 4)1 2. 36 3. Domain (R) = {- 3, 0, 3}


4. Domain (R) = |-2, -1, 0,1, 2} 5. {(8,11), (10,13)1
6. {(1,1), (2, 2), (3, 3), (1, 2), (2,1), (2, 3), (3, 2)) 7. a = l,b = ±2

8. y = 3x-5 9. 1(3, 2), (5, 2), (5, 4)1


10. Domain (R) = |0,1, 2, 3, 4|, Range(R) = (0,2, 4, 6, 8}
11. A=Ix,y,z|, B = |l,2| 12. R = 1(1, 4), (1,6), (2, 9), (3, 4), (3, 6), (5, 4), (5, 6)1
13. {0,± 3,±4,±5} 14. {0, ±1,±2} 15, {{a,a),{b,b),{c,c)}
- 1
17. [1, 2, 3} 18. R =R
16. {(1,1), (2, 2), (3, 3), (4, 4)}
-1
19. {(1,1),(2,2),(3,3),(4,4),(5,5)} 20. R = {(3, 2), (7,2), (7, 3), (7, 4)}
21. R = {(3, 2), (3,10), (5, 2), (5, 6), (7, 2), (7, 6), (7,10)}
-1
R = {(2, 3), (10, 3), (2,5), (6, 5), (2, 7), (6, 7), (10, 7)}
7.48 APPLIED MATHEMATICS-XI

:v,. R = {(3,4), (3, 9), (5, 9), (7, 9)} R= {(2,1), (4, 2), (6, 3), (8, 4)}
{(2, 4), (4, 4), (3, 3)} . (l,2)eRand(2,l)£Rbut(l,l)gR
30. (8, 27) [0]=(0,±2,±4,±6,..J
"i2 (3,1) Reflexive and symmetric
R ={(1,1), (1, 2), (2,1), (2, 2), (2, 3), (3, 2), (3, 3), (3, 4), (4, 3), (4, 4), (5,5)}
.. R ={(3,8), (6, 6), (9, 4), (12, 2)} 36. ((1,1), (2, 2), (3, 3)1

w
F lo
e
Fre
for
r
You
oks
eBo
our
ad dY
Re
Fin
CHAPTER a

ARITHMETIC PROGRESSIONS

8.1 SEQUENCE

A sequence is a function whose domain is the set N of natural nu?nbers.


It is customary to denote a sequence by a letter 'a' and the image a{n) of neN under ahyOjj. Since

w
the domain for every sequence is the set N of natural numbers, therefore a sequence is
represented by its range. The images of 1,2,3,n,... under a sequence 'a' are generally denoted

F lo
by a^, U2, a;^,... Ojj,... respectively, a-^, 02, a^,... a„,... are known as first term, second term ...,
nth term,... respectively of the sequence. If a,i is the nth term of a sequence, 'a' then we write
a = <Ofj>.

ee
REAL SEQUENCE A sequence whose range is a subset of R is called a real sequence.

Fr
In other words, a real sequence is a function with domain N and the range a subset of the set R of real
numbers.
for
ur
REPRESENTATION OF A SEQUENCE There are several zoays of representing a real sequence.
One way to represent a real sequence is to list its first few terms till the rule for writing down
s
other terms becomes clear. For example, 1,3,5,... is a sequence whose nth term is (2n -1).
ook
Yo

Another way to represent a real sequence is to give a rule of writing the nth term of the sequence.
For example, the sequence 1,3,5,7,... can be written as a,, =2n-l.
eB

Sometimes we represent a real sequence by using a recursive relation. For example, the
Fibonacci sequence is given by
r

ai =1, «2 =land 0j,+i =a„ + a„_pn>2


ou
ad

The terms of this sequence are 1,1, 2,3,5,8,


Y

n
lU.USTRATiON 1 Give first 3 terms of the sequence defined by =
n^ + 1
Re
nd

n
SOLUTION Putting n = 1, 2, 3 in n,, = , we get
Fi

1 1 2 3 3
£2j = - , £22 - ~2 “ —c and £23
l^-M 2 2^+1 5 3^+1 10

ILLUSTRATION 1 Find the first four terms of the sequence whose first term is 1 and whose {n + l)th
term is obtained by subtracting n from its nth term.
SOLUTION We are given that a^= \ and <^1 + 1 = - n.
Putting n = 1, we obtain
a2=a-^~\=> a2=^\-l=t3 IV £21 =1]
Putting n = 2, we obtain
£73 = £^2 ~ 2 ^ £?3 = 0 — 2 = ~- 2
Similarly, by putting n = 3, we obtain
£24 =£23 —3= — 2 — 3=—5
8.2 APPLIED MATHEMATICS-XI

SERIES Ifa^, a^, n„, ...is a sequence, then the expression a-^ + 02 + + a,^ + ...+ n,j+...
is a series.

A series is finite or infinite according as the number of terms in the corresponding sequence is
finite or infinite.

PROGRESSIONS It is not necessary that the terms of a sequence alzvaysfollozu a certain pattern or they
are described by some explicit formula for the nth term. Those sequences zuhose terms folhiv certain
patterns are called progressions.
In this chapter, we shall study arithmetical progressions as defined below.
8.2 ARITHMETIC PROGRESSION (A.P.)

ow
A sequence is called an arithmetic progression if the difference of a term and the previous term is alzuays
same.

i.e. an- constant (= d) for all nsN


The constant difference, generally denoted by d is called the common difference.
ILLUSTRATION 1 1,4, 7,10,... is an A.P. lohose first term is 1 and the common difference is equal to

e
Fl
re
4-1 = 3.

F
ILLUSTR.^TlON 2 11,7,3, -2,... isan A.P. zvhosefirst term is 11 and the common difference is equal to
7 -11 =-4.
ur
or
In order to determine whether a sequence is an A.P. or not when its nth term is given, we may
use the following algorithm.
sf
ALGORITHM
k
Yo

STEP 1 Obtain a,j.


oo

STEP II Replace nbyn + 1 in a„ to get Ojj ^


B

STEP III Calculate j - a,,.


re

STEP IV If(^,+ i-(^fi is independent ofn, thegwen sequence is an A.P. Otherzvise it is not an A.P.
Following examples illustrate the procedure:
u
ad
Yo

ILLUSTRATIVE EXAMPLES

EXAMPLE 1 Shozv that the sequence defined by a„ = 4;i + 5 is an A.P. Also, find its common difference.
d

SOLUTION We have, a„ = 4n + 5
Re
in

Replacing n by {n + 1), we get


F

+ 1 = 4{/7 + 1) + 5 = 4n +9
+ 1 - + 9) - {4)1 + 5) = 4
Clearly, + 1 - a„ is independent of n and is equal to 4.
So, the given sequence is an A.P. with common difference 4.
2
EXAMPLE 2 Sholo that the sequence defined by a„ =2)) +1 is not an A.P.

SOLUTION We have, a„ = 2n^ + 1


Replacing n by {n + 1) in a,j, we obtain
^^1+ 1 = 2{n + lf + 1= 2n^ + 4)i + 3
^)+l = (2)7^ + 4)7 + 3) -(2)7^ +1) =4)7 + 2
Clearly, fi,, + 1 - a„ is not independent of n and is therefore not constant. So, the given sequence is
not an A.P.
ARITHMETIC PROGRESSIONS 8.3

l'] 3 4^1
a

EXAMPLES Shoiv that thesecjiience \o§a,\og — , log ^ , log forms an A.P.

SOLUTION We have.
2^ 2 1 /
log b
logfl = log ^x-
b a
=log \bY
K J V
' 3 ^ r 2 '' ^ .3
a b I
log ^ -log ^ =log b^ =log V‘v
J
[b ) b a

(4] a a
*

log ^ -log “log


b^ b^

w
a

This shows that the difference of a term and the preceding term is always same.

F lo
Hence, the given sequence forms an A.P.
ALITER
From the symmetry, we obtain

ee
n
a

Fr
an
= log

n + 1
for
ur
a
= log n
b
s
ook

H + 1 “ 1
Yo

H+ 1 n
a a , a a

^ 1 - = log -log X
= log r
7'^ T H
eB

Clearly, is constant for all values of n.


r
ad
ou

So, the given sequence is an A.P. with common difference log ^ .


\ ^,
Y

EXAMPLE 4 Show that a sequence is an A.P. if its nth term is a linear expression in n and in such a case
Re

the common difference is equal to the coefficient ofn.


nd

SOLUTION Let a„ be the n* term of a sequence. Let a„ be a linear expression in n.


Fi

i.e. (7„ = An + B, where A, B are constants.


a,,^ 1 = A(n + 1) + B
+ 1 a,j = \A{n +1) + B1 - [An + B} = A
Clearly, n,, ^ ^ - a„ is independent of n and is therefore a constant.
Hence, the sequence is an A.P. with common difference A.
NOTE Students are advised to use the statement of the above example as a standard result.
EXAMPLE 5 The n^^’ term of a sequence is 3n - 2. Is the sequence an A.P. ? If so, find its 10th term.
SOLUTION Here, a„ = 3n - 2.
Clearly, a„ is a linear expression in n. So, the given sequence is an A.P. with common
difference 3.

Putting n = 10, we get: fl-jg = 3 x 10 - 2 = 28


8.4 APPLIED MATHEMATICS-XI

REMARK It is evident from the above examples that a sequence is not an A.P. if its nth term is not a
linear expression in n.
EXERCISE 8.1
2
1. If the term a„ of a sequence is given by a„ = n - n -f 1, write down its first five terms.
3 2
2. A sequence is defined by rt, = n -bn +1 In -b,ne N. Show that the first three terms of the
sequence are zero and all other terms are positive.
3. Find the first four terms of the sequence defined hya^=3 and, = 3^7,, _ + 2, for all
»>1.

4. Write the first five terms in each of the following sequences:


(i) = 1, a,j = + 2, fj >1

w
(ii) =1 = a2,a„ = i + fl,,„2^ ” > 2
(iii) ai = 02 = 2,a„ = -!,??>2

Flo
3. The Fibonacci sequence is defined by a^ =1 = ^2, j + /i,, _ 2 for « > 2.
^1+ 1
Find for n = 1,2, 3,4,5.

ee
Fr
6. Show that each of the following sequences is an A.P. Also, find the common difference and
write 3 more terms in each case,

(i) 3,-1,-5,-9... (ii) -1,1/4, 3/2,11/4,...


for
ur
{iii) V2, 3V2, 5^2, 77^... (iv) 9, 7, 5, 3,...
s
7. The term of a sequence is given by a,, = 2n + 7. Show that it is an A.P. Also, find its 7th
k
Yo

term.
oo

8. The n'^ term of a sequence is given by a„ = 2n +n + l. Show that it is not an A.P.


eB

ANSWERS
r

1. flj=l, ^2 = 3, ^73 =7, ^4=13, f^=21 3. = 3,(72 = «3 = 35, (?4 = 107


ou
ad

4. (i) (?i =1, (?2 = 3, (73 =5, <74 =7, =9 (ii) n■^ =1,772 =1,773 = 2, (74 = 3, =5
Y

(iii) 77| = 2,772 = 2,773 = 1,774 = 0,7I5 = -1


6. (i)-4 (iii) 2V2 (iv) -2
Re
nd

7. 21
Fi

HINTS TO SELECTED PROBLEMS

3. We have, = 3 and t7„ = 3 _ 4 + 2 for » > 1


7?2 = 3774 + 2 = 3 X 3 + 2 = 11, 773 = 3772 + 2 = 3 X 11 + 2 = 35
and, 774 = 3773 + 2 = 3x 35 + 2 = 107.
4. (iii) We have, 774 = 772 = 2 and 77,, = _ 4 -1 for 7t > 2.
773 = 7?2 -1 = 2 -1 =1, 774 =773 -1 =1 -1 =0 and, 775=774-1=0-1= -!
5. We have, 774 = 1 = 772 and 7^, = 7^, _ 4 + 7^, _ 2 for « > 2.
773 =772 + 774 =1+1 =2, 774 =773 + 772 =2 + 1 = 3
7^ =774+773 = 3 + 2=5, 77^ = 7^+774 = 5 + 3 = 8
^+ 1
We have to find for 77 = 1, 2, 3, 4,5. i.e.
a, 774 772 ^3 ^4
8.5
ARITHMETIC PROGRESSIONS

Clearly,
f2 1 =1 S- = 12 ^ 2A = -A = -and^a
r?2 ^3 2 3 ^ 5
7. We have.

Ojj = 2n + 7 ^ (iji + \= 2(n +1) + 7 = 2n + 9.


%+l = (2n + 9) -(2n + 7) = 2( a constant). So, the sequence is an A.P.
8. Show that a,, ^ ^ - fl,, is not independent of n.
8.3 GENERAL TERM OF AN A.P.
THEOREM Let a be the first term and d be the common difference of an A.P. Then, its nth term is
a + {n-l) d i.e. a„ = a + (n -1) d.
PROOf Let a-^ ,a2 ,a^ , a,„ ...be the given A.P. Then,
flj = First term = a

w
ai=a + (l -V) d.
By the definition, we have

F lo
^2 - fli = => i?2 = a-i + d-=> 02 = + d => ~ 0 + {2 -1) d
^3 -fl2=rf=>fl3 +^ + a +2d => = a +(3-1) d

ee
H ~^3 = d^ + d^ = {a + 2d) + d => = a + 3d^ = a + {4-l) d

Fr
Similarly, 175 = n + (5 -1) fr,= a + (6 -1) d,..., Ojj=a + {n-l) d.
Hence, nth term of an A.P. with first term a and common difference d is a„ =a + {n-1) d.
for Q.L.D.
ur
8.3.1 nthTERMOFAN A.P. FROM THE END th
Let a be the first term and d be the common difference of an A.P. having m terms. Then, n term
oks

from the end is (m - n +1)* term from the beginning,


Yo
o

= a + (m - n + 1 -1) fr = a + {m - n) d
th
term from the end = 7^,
eB

For finding the term from the end, we may take a,„ as the ftrst term and -d as the common
difference.
our

Taking a,„ as the first term and common difference equal tod', we find that
ad

term from the end = a,,, + (w -1) (- t/)


Y

ILLUSTRATIVE EXAMPLES
Re
nd

Ti/p ' 1 ON FINDING THE INDICATED TERM OF AN A.P.


Fi

EXAMPLE ! Show that the sequence 9,12,15,18,... is an A.P. Find its 16th term and the general term.
an A.P. with
SOLUTION Clearly, (12 - 9) = (15 -12) = (18 -15) = 3, so the given sequence is
common difference d = 3 and first term a = 9.
16th term = /ijg = a + (16 -1) fr = a + 15d = 9 +15 x 3 =54 ['.■ a„ = a + (n -V) d\
and, General term = nth term = a„ = a + {n -1) d = 9 + {n - V) x 3 = 3» + 6
EXAMPLE! Show that the sequence log a, log(ab),log {ab^), log{ab^),... is an A.P. Find its nth term.
SOLUTION We have,
I \ ab^
log (ab) - log a = log — = log b, log (ab ) - log (ab) = log = logb
ab

ab^
log(fl&^) -log(flb^) = log ab^
= logfe
8.6 APPLIED MATHEMATICS-XI

It follows from the above results that the difference of a term and the preceding term is always
same. So, the given sequence is an A.P. with common difference log b.
a„ = a + {n -1) d
11-1 12- 1
(J,
= log a + {n -1) log b = log a + log b = 10g(77l7 )
EXAMPLE 3 Wiiich term of the sequence 71, 70, 68, 66,... is 40?
SOLUTION Clearly, the given sequence is an A.P. with first term a =72 and common difference
d = -2. Let its ?7th term be 40.
i.e. = 40

a + {n-'[)d =40
72 + (n-l)(-2) = 40 [v a,, = 77+ (27-1) d]
72 - 211 + 2 = 40 => 2n = 34 => « = 17

w
Hence, 17th term of the given sequence is 40.
EXAMPLE 4 Which term of the sequence 4, 9,14,19,... is 124?

Flo
SOLUTION Clearly, the given sequence is an A.P. with first term 77 = 4 and common difference
d = 5. Let 124 be the nth term of the given sequence. Then,

ee
a, = 124=> fl + (;i-l) d = 124=> 4 + («-l)x5 = 124 => 72 = 25

Fr
Hence, 25th term of the given sequence is 124.
EXAMPLES How many terms are there in the sequence 3, 6, 9,12,..., 111?
for
SOLUTION Clearly, the given sequence is an A.P. with first term n = 3 and common difference
ur
d = 3. Let there be n terms in the given sequence. Then,
th
s
77
term = 111 => fl + (/2 -1) d = 111 => 3 + {22 -1) x 3 =111 n = 37
k
Yo
oo

Thus, the given sequence contains 37 terms.


eB

EXAMPLE 6 Is 184 a term of the sequence 3, 7,11,... ?


SOLUTION Clearly, the given sequence is an A.P. with first term 77 = 3 and common difference
d = 4. Let the 7ith term of the given sequence be 184. Then,
r
ou
ad

a, = 184=>77 + (n-l)7f = 184 => 3 + (72 -1)x 4 = 184=> 472 = 185=>« = 46-.
4
Y

Since n is not a natural number. So, 184 is not a term of the given sequence.

EXAMPLE 7 Which term of the sequence 20,19 ^ , 18


Re
nd

2 4
... is the first negative term?
Fi

SOLUTION The given sequence is an A.P. in which first term 77 = 20 and common difference
Let the 72th term of the given A.P. be the first negative term. Then,

77„ <0
=> 77 + (?2 -1) d < 0
3n 9
=> 20 + (72 -1) X (- 3/4) < 0 => — ~ < 0 => 83 - 3t2< 0 => 377 > 83 => 22 > 27 -
4 4 3

Since 28 is the natural number just greater than 27 3 So, n = 28. Thus, 28th term of the given
sequence is the first negative term.
EXAMPLE 8 Which term of the sequence 8 - 62, 7 - 4/, 6 - 2i, ...is (i) purely real (ii) purely imaginary?
SOLUTION The given sequence is clearly an A.P. with first term 77 = 8-62 and common
difference d = ~1 + 2i. The nth term of the given A.P. is given by
8.7
ARITHMETIC PROGRESSIONS

= a+ {n-l)d = 8-6/ + (n-l){-l + 2/) = (9-n) +1'(2»-8)


n,

(i) Let the nth term of the given sequence be purely real. Then, a,, is purely real.
0=> n = 4
(9 _,i) + /(2n - 8) is purely real => 2n - 8 =
So, 4th term of the given sequence is purely real,
(ii) Let the nth term of the given sequence be purely imaginary. Then, a„ is purely imaginary
(9 _ n) + /(2n - 8) is purely imaginary
=> 9-)i = 0=>n=9
Thus, 9th term of the given sequence is purely imaginary.
Typje II PROBLEMS BASED UPON n„ - >m- fr
Ifpth, qth and rth terms of an A.P. are a, h, c respectively, then show that:

ow
EX.\MPLE9

(i) a{q -r) + b{r-p) + c{p-q)=0 (H) [a-b)r+ {b-c) p+ {c-a) q = 0


SOLUTION Let A be the first term and D be the common difference of the given A.P. Then,
a = pth term => n = A + {p -1) D
...(II)

e
b = qth term =>b = A + {q -l)D

re
c = rth term ^ c = A + {r -1)D ...(iii)

rFl
(i) Substituting these values of a,b, c, in a{q-r)+b{r-p) + c (p-tj), we obtain

F
a
iq-r)+b{r-p) + c{p-q)
: + (p -1) Dl (ty - r) + (A + (g -1) D] (r - p) + {A + (r -1) D} (p - q)

r
ou
fo
: A{{q-r) + {r-p) + {p-q)]+D\{p-l){q-r) + iq-l){r-p)+{r-l){p-ci)\
ks
= A .0 + D {p iq -r) + q {r - p) + r ip - q) -{q -r) -{r - p) -{p - q)\
= A.O + D.O = 0
oo

(ii) On subtracting (ii) from (i); (iii) from (ii) and (i) from (iii), we get
Y

...(Vi)
B

a-b =ip-q)D ...(iv) (^-c) ={q~r)D ...(v) c-a =(r-p)D


{a-b)r + {b-c)p + {c-a)q= {p-q)Dr+ {q-r) Dp+ {r-p)Dq
re

= D[{p - q) r + {q -r) p + {r - p) q] = DxO = 0


ou

Show that thesum of{m + nf and {m - nf term ofan A.P. is equal to twice then!’’ term.
Y
ad

EXAMPLE 10

SOLUTION Let a be the first term and d be the common different of the AP. Then,
d

a MI + H = (ni + nf^’ term = a + {m + n -1) d and, a,„ -n = {m - nf^' term = n + (n? - n -1) d
in
Re

am + M + am - n = {a + {m + n-l)d\ + {a + {m-n-l)d\
F

= 2n + (m + n -1 + w-n -1) d
= 2a+ 2 (in -l)d
= 2 {a + (m -1) dl
= 2fl MI-

EXAMPLE 11 Ifm times the mth term of an A.P. is equal to n times its nth term, show that the (m + n)f/i
term of the A.P. is zero.
SOLUTION Let a be the first term and d be the common difference of the given A.P. Then,
Ih
m times m term = n times nth term

m a,„ = II
III {n + (in -1) d]=n[a + (n -1) d]
in {rt + (m -1) d) - n{<7 + (n -1) d} = 0
8.8
APPLIED MATHEMATICS-X!

a{m - «) + {m(m -1) - n{n -1)} =0


a{m - n) + {(m^ - n^) ~ ~n)}d = 0
a(ni -n) + (m-n) {m+ n-1) d = 0
{m-n) {fl + (m + N-l) d]=0
n + {m + n~l)d = Q
m # n]
a
m + n = 0

Hence, the (m + term of the given A.P. is zero.


t:\AMPLE 1
Ifthe pth term ofan A.P. is q and the q‘^' term is p, prove that its nth term is{p + q-n).
SOLUTION Let a be the first term and d be the common difference of the given A.P. Then,
pth term = q=> a + {p-1) d = q
qih term = p=> a + {q-l) d - p ...(ii)
Subtracting (ii) from (i), we get

w
ip-q)d = iq-p) d=-l

F lo
Putting d = -1 in (i), we get: = ip+ q-l)
a

«th term = a + {n-l)d = (p + q-I) + {n-1) x (-1) = (p + q^n)


■ AMPLE 13 If the mth term of an A.P. be 1/n, and nth tenn be 1/m then show that its (mn)th term is 1.

e
SOLUTION Let a and d be the first term and common

Fre
Then,
difference respectively of the given A.P.
1 , 1
- = iJitn term => -
for
n
= a + {m -\)d ...(i)
n

1
r
1
— = nth term => — = a + {n-l)d ...(ii)
You

m m
oks

On subtracting (ii) from (i), we get


eBo

1 1 m-n 1
— = (m-n) d=> = (m -n)d=> d =
n m mn mn

1
Putting d =
our

in (i), we get
ad

mn

1
a +
(w -1) 1 1 1 1
- =a + => a
n mn n n mn mn
dY
Re

1
(nm)th term = a + (mn -1) d = — + (mn -1) =1
Fin

mn mn

XAMI-LL 14 Determine the number of terms in the A.P. 3, 7,11,... 407. Also, find its 20th term from
the end. ^

SOLUTION Clearly, the given sequence is an A.P. with first term 3 and the common difference
4. Let there be n terms in the given A.P. Then,
407 = nth term => 407 = 3 + («-1)x 4=> 4n = 408=>n = 102
Now,

20th term from the end = [102 - 20 + l]th term from the beginning
= 83rd term from the beginning = 3 + (83 -1) x 4 = 331
iMTl Lit To find 20th term from the end, we consider the given sequence as an A.P. with first
term = 407 and common difference - 4.

20th term from the end = 407 + (20 -1) x (- 4) = 331.


8.9
ARITHMETIC PROGRESSIONS

V: How many numbers of two digits are divisible by 7?


MPLtl.'

SOLUTION First two digit number divisible by 7 is 14 and last two digit number divisible by 7 IS
i
98. So, we have to determine the number of terms in the sequence 14,21,28,..., 98. Let there be n
terms in this sequence. Then,
98 = Mth term =>98 = 14 + (m -1) x 7 => 7h = 91 => « =13
EX-. .i. Show that there is no A.P. which consists of only distinct prime numbers.
SOLUTION Let flj, <?2, ^3,..., flfi' ●●● be an A.P. consisting only of prime numbers. Let d be the
common difference of the A.P. Since the difference of two consecutive prime numbers is greater

w
than or equal to 1. Therefore, d>\.
Now,

{«! + 1)*^ term of this A.P. = dj + + 1 -1) fr = (1 + d)

o
e
+1)^ term is not a prime number

re
This is a contradiction that the A.P. consists of only prime numbers as its terms.

Frl
F
Hence, there cannot be an A.P. which consists only of distinct prime numbers.
EXA.Ml’LE 17 Show that in an A.P. the sum of the terms equidistant from the beginning and end is
ou
always same and equal to the sum offirst and last terms.

r
so
SOLUTION Let 02, a^,..., d„ be an A.P. with common difference ‘d‘. Then, kf
kth term from the beginning = = a^+ik -1) d
kth term from the end ={n-k + l)th term from the beginning
oo
and.
Y

= ^t-k+1
B

= ai+ {n -k + 1 -V) d = ai+{n-k) d


{kth term from the beginning) + {kth term from the end)
re
oY

= ^k+ ^i-k+1
u

= {(7j + {k -1) d\ + {dj + ()i -k) d] = 2di +{n-l) d= ai + {ai + {n -1) d] = ai + a^


ad

Thus, ^k+^t-k+1 - for all A: = 1, 2,... n


d

^2 ^- 1 = ^*3 - 2 = H ^t-3 - ^ti


in

Hence, the sum of the terms equidistant from the beginning and end is always same and equal
Re

to the sum of first and last terms.


F

NOT! The statement of the above example may be treated as a standard result.
EXERCISE 8.2
1. Find:
(i) 10th term of the A.P. 1, 4, 7,10,... (ii) 18th term of the A.P. >/2, 3-fl, 5yfl,...
(hi) nth term of the A.P. 13,8,3, -2,...

In an A.P., show that Oj,, + H

(i) Which term of the A.P. 3, 8,13,... is 248 ?


(ii) Which term of the A.P. 84, 80, 76,... is 0 ?
(hi) Which term of the A.P. 4,9,14,... is 254 ?
, (i) Is 68 a term of the A.P. 7,10,13,... ?
(ii) Is 302 a term of the A.P. 3,8,13,... ?
8.10 APPLIED MATHEMATICS-XI

5- (i) Which term of the sequence 24, 23^,22-!-


3
-,21--... is the first negative term ?
4 2 4
(ii) Wliich term of the sequence 12 + 8/, 11 + 6i, 10 + 4/,... is (a) purely real (b) purely
imaginary ?
6- (i) How many terms are there in the A.P. 7,10,13,... 43 ?
(ii) How many terms are there in the A.P. -1, - - 2 _1 10,
6 3' 2 3

7. The first term of an A.P. is 5, the common difference is 3 and the last term is 80; find the
number of terms.

8- The 6th and 17th terms of an A.P. are 19 and 41 respectively, find the 40th term.

ow
9- If 9th term of an A.P. is zero, prove that its 29th term is double the 19th term.
1 f*’ If 10 times the 10th term of an A.P. is equal to 15 times the 15th term, show that 25th term of
the A.P. is zero.

11- The 10th and 18th terms of an A.P. are 41 and 73 respectively. Find 26th term.

e
12. In a certain A.P. the 24th term is twice the 10th term. Prove that the 72nd term is twice the

re
34th term.

Fl
13. If(w + i)th term of an A.P. is twice the {n + l)th term, prove that(3?« + l)th term is twice the

F
(m + n + l)th term.
ur
14. If the nth term of the A.P. 9,7,5,... is same as the nth term of the A.P. 15,12,9,... find n.

r
1
fo
Find the 12th term from the end of the following arithmetic progressions:
ks
(i) 3,5, 7, 9,... 201 (ii) 3,8,13,..., 253 (iii) 1,4,7,10,..., 88
Yo
16. The 4tlr term of an A.P. is three times the first and the 7th term exceeds twice the third term
oo

by 1. Find the first term and the common difference.


eB

12. Find the second term and nth term of an A.P. whose 6th term is 12 and the 8th term is 22.
18. How many numbers of two digit are divisible by 3 ?
ur

19. An A.P. consists of 60 terms. If the first and the last terms be 7 and 125 respectively, find
32nd term.
ad
Yo

The sum of 4th and 8th terms of an A.P. is 24 and the sum of the 6th and 10th terms is 34.
Find the first term and the common difference of the A.P.
d

21. How many numbers are there between 1 and 1000 which when divided by 7 leave
Re
in

remainder 4?

22. The first and the last terms of an A.P. are a and / respectively. Show that the sum of nth term
F

from the beginning and nth term from the end isa + 1.
23. If an A.P. is such that ^ find
3
«8

ANSWERS

1- (i) 28 (ii) 35 y[2 (iii) -5n +18 3. (i) 50 (ii) 22 (iii) 51


4. (i) No (ii) No 5. (i) 34th (ii) (a) 5 (b) 13
6. (i) 13 (ii) 27 7. 26 8- 87 M- 105 14. 7
15- (i) 179 (ii) 198 (iii) 55 16. First term = 3 , common difference = 2
17. 1 5 4
<72=“ 8, i7„=5;i-18 18. 30 19. 69 20. _ 23.
2' 2 5
8.11
ARITHMETIC PROGRESSIONS

8.4 SELECTION OF TERMS IN AN A.P.


Sometimes we require certain number of terms in A.P. The following ways of selecting terms
are

generally very convenient.


Terms Common difference
Number of terms

3 a - d, a, a + d d

4
a - 3d, a - d, a + d, a + 3d 2d

5 a - 2d, a - d, a, a + d, a + 2d d

6
a -5d, a - 3d, a - d, a + d, a + 3d, a + 5d 2d

w
It should be noted that in case of an odd number of terms, the middle term is a and the common
difference is d while in case of an even number of terms the middle terms area-d,a +dandthe
common differences is 2d.

Flo
The following examples will illustrate the use of such representations.

e
ILLUSTRATIVE EXAMPLES

re
Tlie sum of three numbers in A.P. is - 3, and their product is 8. Find the numbers.

F
EXAMPLE 1

SOLUTION Let the numbers be {a - cf), a, {a + d). Then,


ur
Sum = -3 => (a-d) + a + {a + d) = - 3=>3fl = -3^rt = -1
r
Product = 8
fo
and.
ks
(fl - fr) ^ ^
Yo
oo

a{a^-d'^) = 8
B

= 8 [V A = -l]
re

= 9^d = ±3
u

When a=-l and d = 3, the numbers are -4, -1, 2. When a = -\ and d =-3, the numbers are
ad
Yo

2, -1, -4. So, the numbers are -4, -1, 2, or 2, -1,-4.


EXAMPLE z Find four numbers in A.P. xohose sum is 20 and the sum ofzvhose squares is 120.
d

SOLUTION Let the numbers be (fl - 3d), {a - d), (d + d),{a+ 3d). Then,
Re
in

Sum = 20 ^ (a - 3(1) + (fl - if) + (rt + fr) + (fl + 3rf) = 20=> 4a = 20 => (7 =5
F

and. Sum of the squares = 120


{a - 3d)^ +{a-d)^ + (a+ d)^ + {a + 3d)'^ = 120

4a^ + 20d^ = 120


/+5rf^ = 30
25+5rf^ = 30 [V «=5]

5d^ = 5 => d = ±1
If d =1, and a =5,then the numbers are 2,4,6,8. If d=-l, and a =5, then the numbers are
8, 6, 4, 2.
Thus, the numbers are 2,4,6,8 or 8,6,4,2.
8.12
APPLIED MATHEMATICS-XI

EXAMIM.E ^
Divide 32 into four parts which are in A.P. such that the product of extremes is to the
product of means is 7:15.
SOLUTION Let the four parts be (a - 3d), (a-d),(a + d) and (a + 3d). Then,
Sum = 32 => (a-3d) + {a-d) + (a + d) + (a+3d) = 32=>4a = 32=>a =

It is given that the product of extremes is to the product of means is 7 :15.


{n -3d){n+ 3d) 7

{a -d){a + d) 15

a
^-9d^ ^ 7_ 64 - 9d^ = ~=> 128d^ = 512 =>
a 15^ 64_j2 ;^5 = 4=> d = ± 2

Thus, the four of 32 parts are 2, 6,10,14.

w
When(7 = 8and d=2 four parts are: 2,6,10, and 14. Whena = 8andd = -2four parts are 14 10 6
and 2.

Flo
\ WIl’I.F I
The product of three numbers in A.P. is 224, and the largest number is 7 times the smallest.
Find the numbers.

ee
SOLUTION Let the three numbers in A.P. hea-d,a,a + d, where d > 0. Clearly, a + dis the largest

Fr
number and a — d is the smallest number.
It is given that: for
ur
Product of numbers = 224 and. The largest number = 7 (The smallest numbers)
(rt - d) rt ((7 + d) = 224 and, a +d^7 (a-d)
k s
Yo

a{a^-d^)=224 and, 6« = 8d
oo
eB

3(7
a {a^~d^)= 224 and, d
4

2 9 2
r

a a (7 = 224
[On eliminating d]
ou
ad

16

7(7^
Y

= 224
16
Re
nd

a ^=512 = 8^
Fi

(7=8.

Putting (7 = 8 in d = —, we obtain d = 6.
4

Hence, three numbers are 2,8,14.

EXERCISE 8.3
b The sum of three terms of an A.P. is 21 and the product of the first and the third terms
exceeds the second term by 6, find three terms.
Three numbers are in A.P. If the sum of these numbers be 27 and the product 648, find the
numbers.

Find the four numbers in A.P., whose sum is 50 and in which the greatest number is 4 times
the least.
ARITHMETIC PROGRESSIONS 8.13

4. The sum of three numbers in A.P. is 12 and the sum of their cubes is 288. Find the numbers.

5. If the sum of three numbers in A.P. is 24 and their product is 440, find the numbers.
6- The angles of a quadrilateral are in A.P. whose common difference is 10°. Find the angles.

ANSWERS

1. 1,7,13 2. 6,9,12 3. 5,10,15,20 4. 2, 4, 6 or 6, 4, 2


5,8,11 6. 75°, 85°, 95°, 105°

HINTS TO SELECTED PROBLEMS

Let the three numbers he a -d, a, a + d. It is given that the sum and product of these
numbers are 24 and 440 respectively. Therefore,

low
a-d + a + a + d = 24i and (a-d) n{a+ d) = 440
=> 3fl = 24 and a - d^) = 440
a = 8 and a(a^-d^)~440

ree
Now,

a{a^-d^)=440 => 8 (64 -


F = 440 => 64-d"^ =55 => fr^=9

F
=> d = ± 3

Hence, three numbers are 5,8,11 or 11,8,5.


or
ur
sf
8.5 SUM TO n TERMS OF AN A.P.
k

THEOREM SIww that the sum S„ ofn terms of mi A.P. xvith ifrst term 'a' and common difference'd' is
Yo
oo

^|2«+(»-l)d|
B
re

or.
^ (r? + /), ivhere / = last term = a + {n-l)d
ou
ad

PROP!
Let a^, /?2/ ^3,... be an A.P. with first term a and common difference d. Then
Y

rti = a, a-i = + fr, ^3 = a +2d, a^ = a + 3d,..., a„ = a+ {n-l) d


nd
Re

Now,
Fi

S,j - + ^2 + ^3 + ●●● + 1 +%
S,, = a + {a + d) +{a + 2d) + ... + {a + {n-2) d) + {a + {n -1) d] ...(i)

Writing the above series in a reverse order, we get


S,j = [a + {n -1) d] + {a + {n - 2) d\ +... + {a + d) + a ●●●(ii)

Adding the corresponding terms of (i) and (ii), we get


2^, = {2a + {n -1) fr} + [2a + (» -1) fr} + ... + {2n + {n -1) d\
2S„ = n[2a + in-l)d] [●.● 2<7 + {n -1) d repeats n times]

%= + -l)d|
Now, 1 - last term =a + {n-l)d
8.14 APPLIED MATHEMATICS-XI

^ |2r7 + {n - 1) frj = -^ + |(7 + ()i -1) fr|


n
a + I
2^

AUTER We have,

= flj + ^2 + ^3 + ... + «,,_2 + 1 + ...(i)


or. S,, = fl,i + - 1 + fl,, - 2 + -■ + fl3 + «2 + rt] ...(ii)

Adding corresponding terms in (i) and (ii), we get


2^^j = ('Jl + ‘^n) + («2 + - l) + (^3 - 2) ■●■ ■ ■ ■ +
- 1 '’2) ^1)
2^1 = (^1 + ‘^ti) + ('*1 + ^n) + («! + + ... + (^^1 + «») + + ^i)
[v + a„ =a^. + a„_ I for k = 2, 3,..., m]

ow
2^ = «(«!+««)

“ («i + ^fi)

S,j = I |fli + ni +(»-!) fr| [v a,I =«! +(«-!) fr]

e
re
n

Fl
1) frj

F
ur
In thefonnulaS^^ = ^ |2rt + (n -1) d^, therearefour qiiantitiesviz. £^, a, uandd. Ifani/three
r
\OTEi

fo
of these are known, the fourth can be determined. Sometimes two of these quantities are given, in such
ks
cases remaining two quantities are provided by some other relations.
Yo

■-■OTE 2 If the sum of n terms of a sequence is given, then nth term a„ of the sequence can be
oo

determined by the using formula: a„ = S„-S„_i


eB

ILLUSTRATIVE EXAMPLES

Type I ON FINDING THE SUM OF GIVEN NUMBER OF TERMS OF AN A.P.


ur

It
ad

Formula: S.. 2u ~ (ir 1) d or, S n


Yo

1 2

r\ AMPLE 1 Find the sum of 20 terms of the A.P. 1, 4, 7,10, ...


d

SOLUTION Let a be the first term and d be the common difference of the given A.P. Clearly,
Re
in

a = 1, d = 3. We have to find the sum of 20 terms of the given A.P. Putting rt = 1, d = 3,« = 20 in
F

2fl + (h -I) ■/ we get

20 f
S20 =Y j 2x 1 +(20-l)x 3 ■ = 10x59=590
i:\AMPLE2 Find the sum of the series :5 +13 + 21 +... + 181.
SOLUTION The terms of the given series form an A.P. with first term /? = 5 and common
difference d = 8. Let there be n terms in the given series. Clearly,
a,j = 181 => a + (n-l) d = 181 =>5 + (n-l)x8 = 181 => 8» = 184 => « = 23

Required sum = ^ (fl + 0 = ^(5 + 181) = 2139.

lAAMPl.E ■ Find the sum of all three digit natural numbers, which are divisible by 7.
SOLUTION The smallest and the largest numbers of three digits, which are divisible by 7 are
105 and 994 respectively. So, the sequence of three digit numbers which are divisible by 7 are
ARITHMETIC PROGRESSIONS 8.15

105, 112, 119, 994. Clearly, these numbers are in A.P. with first term a = 105 and common
difference d = 7.

Let there be n terms in this sequence. Then,


a = 994 « + 0z-l)d = 994=>105 + (ji-l)x7 = 994 => ii = 128

Required sum = 12n + {;( -1) d| = 12 x 105 + (128 -1) x 7 = 70336

EXAMPLE 4 find ike sum of all natural numbers between 250 and 1000 loluch are exactly divisible by 3.
SOLUTION Clearly, the numbers between 250 and 1000 which are divisible by 3 are 252, 255,
258,..., 999. These numbers are in A.P. with first term a = 252, common difference = 3 and last
term = 999. Let there be n terms in this A.P. Then,
a,, = 999=>rt + (n-l)rf = 999 => 252 + (n-1) x 3 = 999 = 250
250
Required sum = = -’^{a + 1) = ^(252 + 999) = 156375
EXAMPLE 5 Find the sum of all odd integers between 2 and 100 divisible by 3.

w
SOLUTION The odd integers between 2 and 100 which are divisible by 3 are 3,9,15,21,..., 99.
Clearly, these numbers are in A.P. with first term a =3 and common difference d = 6. Let there be
n terms in this sequence. Then,

EXAMPLE 6
F lo
a,j = 99 => a + (n-l) d => 3 + {n -1) X 6 = 99 => n =17

Required sum = ^ (/7 +/) = ^(3 + 99) = 867. for F


ree
Find the sum ofifrst 20 terms of an A.P., in which 3rd term is 7 and 7th term is two more
than thrice of its 3rd term.
SOLUTION Let a be the first term and d be the common difference of the given A.P. It is given
that
Your

<73 = 7 and Oy = 3/73 + 2


ks
eBoo

a + 2d =7 and a + 6d = 3{a + 2d) + 2


a + 2d =7 and a = -1

a = -1, d = 4
ad
our

S20 = y|2x-l+(20 -l)x4 Using: ^ | 2t7 + (« i)d}


20
Re

S20 = y(-2 + 76) = 740


Y
Find

EXAMPLE?
The sum of the ifrst four terms of an A.P. is 56. The sum of the last four terms is 112. If its
first term is 11, then find the number of terms.
SOLUTION Let there ben terms in the A.P. with first term a=\ \ and common difference d. It is
given that
Sum of first four terms = 56

^|2xll +(4 -\)d


22+3d = 28=>3d=6 =>d = 2
= 56

It is also given that


Sum of last four terms = 112

- 1 + -2 - 3 =

i(«„+77,,_3)=112 Using: =|(i7 + 0


8.16 APPLIED MATHEMATICS-XI

=> {ll+(n-l)x2} + {ll+(«-4)x2}=56


22 + 2(2«-5)=56
=> 4« = 44=>«=11.

Hence, there are 11 terms in the A.P.


2
If the sum of n terms of an A.P. is pn + qn , where p and q are constants, find the common
difference.
SOLUTION Let Sfj denote the sum of n terms and Ojj denote the «th term of the A.P. Then,
S,, =pn+qn^
=5>
Sn_l=p{n-l) + q{n-lf [On replacing n by [n -1) in 5y,]

w
Now,

a„={pn+ qn^] - [p (n -1) + ^ (n -1)^)

lo
Ofj = pn - p {n -1) + qn - q{n-l)

e
a„=p{n-{n-l)} + q{n^-in-1)^)

re
a„=p + q(2n-l)
rF
F
a„^i=p + q{2{n-l)-l] [Replacing n by (n -1) in ^^]
Let d be the corrunon difference of the A.P. Then,

r
d =Ojj — Ojj_ fo
u
=> d = {p + q{2}i-l)]-[p + q{2{n-l)-'l\]
ks
^ d={p + qi2n-l)}-\p + q(2n-3)}
Yo

=> fr = i^(2n-1 - 2n + 3) = 2f^


oo

Hence, the common difference = 2q.


B

EX.ANii’LLQ If the sum of n terms of an A.P. is 3n +5n and its mth term is 164, find the value
e

ofm.
ur

SOLUTION Let 5^ denote the sum of m terms and a,, be the »th term of the given A.P. Then,
ad
Yo

^ = 3n^ + 5«
^,_l = 3{«-l)^+5(n-l) = 3n^-«-2 [On replacing n by (« -1) in 5y,]
d
Re
in

Now, ^ ^ ~ 1
= (3n^ + 5«) - (3«^ - n-2)
F

Ofj =6n+2
Now, =164 [Given]
6m + 2 = 164 ^ 6m = 162 ^ m = 27

EN-.'v 'LL 10 Find the sum to n terms of the sequence given by a,, = 5 -6n,n e N.
SOLUTION We have, a,, =5 - 6n
1-5-6(n+1) = -1 - 6n
+ I ~a„ = (-1 - 6n) -(5- 6n) = - 6, for all« e N
Since + i is constant for all neN. So, the given sequence is an A.P. with common
difference - 6.

Putting tt = 1, in = 5 - 6«, we get: a^ = -1.


So, the sum 5^ to « terms is given by
+'’«)=“(-!+5-6«)=«(2-3n)
ARITHMETIC PROGRESSIONS 8.17

I'XAMn :
If the term of an A.P. is - and the term is —, show that the sum of mn terms is
n m

i (mn +1), zohere m ^ n.

SOLUTION Let a be the first term and d be the common difference of the given A.P. It is given
that

1 1
and a n
n m

1
Now, am - zx> rt + (m -1) d =
n n

ow
and. = ~=> a + (n~l) d = —
tn m
...(ii)

Subtracting (ii) from (i), we get


m -n 1
(m-n)d = -~ — ^{m-n)d = d =

e
n m mn mn

re
Putting d -
1
in (i), we get

rFl
F
mn

1 1 1 1 1
a + (m-l) — = -=> <7 + =- => a =
mn n n mn n mn

r
ou
^nn ^\la + (mn -1) frlJ = —2 j(mn
— + (inn-\) x —I
mnj ^ (mn +1)
fo
ks
I
Tf/pe ll FINDING THE NUMBER OF TERMS IN AN A.P. WHEN THE SUM OF ITS n TERMS IS GIVEN
oo

i:\.\MPi.m:
How many terms of the series 54,51,48,... be taken so that their sum is 513 ? Explain the
Y
B

double answer.

SOLUTION Clearly, the given sequence is an A.P. with first term a = 54 and common difference
re

d = -3. Let the sum of n terms be513. Then,


ou
Y

= 513
ad

=> - |2i7 + (7i -l)d ■ = 513


d
in
Re

=> ^h08 + (n -l)x-3[=513=> n^-37n+ 342 = 0 (n-18) (n-19) = 0 =>n=18or,19


F

Here, the common difference is negative. So, the terms are in decreasing order and the value of
19 term is 54 +(19 -l)x - 3 =0. Thus, the sum of 18 terms as well as that of 19 terms is 513.

CXAMPLt' 13 Find the number of terms in the series 20,19 — 18 — f ● .. of which the sum is 300,
3 3
explain the double answer.
SOLUTION The given sequence is an A.P. with first term a = 20 and the common difference
2
d = - —. Let the sum of n terms be 300. Then,

= 300

= 300
8.18 APPLIED MATHEMATICS-Xl

-2 2x20 + (n-l)[--
' Is
!>=300
n ^ - 61n + 900 = 0 => {n - 25) {n - 36) = 0 => = 25 or, 36
Sum of 25 terms = Sum of 36 terms = 300.

Here, the common difference is negative therefore terms go on diminishing and 31sf term
becomes zero. All terms following 31st term are negative. These negative terms when added to
positive terms from 26th term to 30f/i tcnn, they cancel out each other and the sum remains same.
Hence, the sum of 25 terms as well as that of 36 terms is 300.
EX.-VMPLE 14 Solve 1+6 + 11+16+ ... + x= 148.
= 1 and common
SOLUTION Clearly, terms of the given series form an A.P. with first term a
difference d = 5. Let there be n terms in this series. Then,

w
1 + 6 + 11+16 + ... + .V = 148
Sum of n terms = 148

^ |2n + ()i - 1) tij


F lo
= 148

^ |2 + (n - 1) X sj = 148 => 5/1^ - 3n - 296 = 0 => {n - 8) (5m + 37)

ee
= 0 => ?j = 8

Fr
th
Clearly, .v=n“‘ term for
,T = fl + (f7 -1) fr = l + (8 -1) X 5 = 36 [■.■a = l,d = 5, n = 8]
ur
Type III PROVING RESULTS RELATED TO THE SUM OF n TERMS OF AN A.P.
s

The siwi of the first p, q, r terms of an A.P. are a, b, c respectively. Show that
ook

EXAMPLE 15
Yo

- iq -r) +- {r - p) + - ip - q) = 0
eB

p q r
SOLUTION Let A be the first term and D be the common difference of the given A.P. Then,

^\lA+{q -l)D|^y = |2A+(;7 -1)D


our
ad

a = Sum of p terms => a = ...(i)

b = Sum of 7/terms = ■^|2A + (fj -1)d| =>


2b
...(ii)
Y

2A+{q -1)D
Re

9
nd

c = Sum of r terms => c |2A + (r -1) d| ^ = |2A + (r - 1) d| ...(iii)


Fi

and,

Multiplying (i), (ii) and (iii) by {q - r), (r - p) and {p - q) respectively and adding, we get
{r-p)+^{p-q)
2a 2b
(q-r) + r
P

{2A + (p -1) D) {q-r) + {2A +{q-l) D] (r -p) + {2A + (r -1) D| (p - q)


= 2A{q-r +r p + p q) + D{{p-l)(q-r)+{q-l) (r-p) + (r-l) {p-q)\
-2AxO+DxO=0

EXAMPLE 1& The sum of n, 2n, 3?j terms of an A.P. are


Si, $2,5^ respectively. Prove that:
S3=3(S2-Si).
SOLUTION Let a be the first term and d be the common difference of the given A.P. Then,

Si Sum of n terms
^ |2fl + (m -1) tfj
=> Si = - ...(i)
ARITHMETIC PROGRESSIONS 8.19

2n

$2 = Sum of 2 n terms ^ $2 =—j 2fl + (2n-1) fr ...(ii)

and, S3 = Sum of 3 n terms => S3 =


y|2fl + (3« -1) frj ...(iii)

S2 - Si = y 12fl + (2« -1) d


n
Now,
- j 2^7 + {n -1) d
=>

S2-S1 =1 2|2rt + (2«-l) d|-|2fl + (« -l)rf| ^|2(7 + (3h-1) frj

w
3(^2-Si) = y|2rt + (3n -l)rf| ...(iv)

From (iii) and (iv), we get


3(S2-Si)=S3

o
e
EXAMPLE 17 The sums ofn terms of three arithuetical progressions are Si, $2 and S3. The first term of

re
each is unity and the common differences are 1, 2 and 3 respectively. Prove that Si + S3 = 2 S2.

Frl
F
SOLUTION We have.
Si = Sum of n terms of an A.P. with first term 1 and common difference 1
ou
=> Si=y2xl + (»-l) X 1 . = ^ (ji +1)

r
so
kf
Sj = Sum of n terms of an A.P. with first term 1 and common difference 2

$2 = -^|2xl + (?; -1) X 2|


oo
2
= n
Y

S3 = Sum of n terms of and A.P. with first term 1 and common difference 3
B

$2= -^12x1+(h -1)^ sj ^(3m-1)


re
oY

S1 + S3 = ’l(n + l)+’l{3n-l) = 2n^


u
ad

Hence, Si + S3 = 2S2 IVS2


d

EXAMPLE 18 If in an A.P. the sum ofm terms is equal to n and the sum ofn terms is equal to m, then
in

prove that the sum of (m + n) terms is ~(m + n). Also, find the sum ofifrst {m~n) terms {m > n).
Re

SOLUTION Let a be the first term and d be the common difference of the given A.P. Then,
F

|2fl + (m -1) frj = n => lam + m{m -1) d = In ...(i)

and.
S,i = m => ~ j 2fl + (» -1) d| = m => Ian + n {n -1) d = 2m ...(ii)

Subtracting (ii) from (i), we get


2a {m - n) + {m (m -1) -n {n -1)] d = 2n - 2m
=> 2a {m - n) + {{m^ - n^) - {m -n)] d = - 2 {m - n)
2a + (m + n-l) d = -2 [On dividing both sides by {m - n)] ...(iii)
m + n
Now, S, n + n 2a + (m + n -1) d
2

{m + ?i)
i + n
^(-2) [Using (iii)]
8.20 APPLIED MATHEMATICS-XI

+ n = - (m + n)
From (iii), we obtain
2a = + d ● ●●(iv)
Substituting this value of 2a in (i), we obtain
- 2m-m {m + n-\) d + m (m-1) d = 2n
m + n
d = -2 ●●●(V)
mn

m + n
Putting d = -2 in (iv), we obtain
mn )
2
2a = -2 + — {m + n-\){m + n) ...(Vi)
mn

Now,

12(? + (jn-n-1) rfj

w
ii-n

2 2 ]

F lo
m-n
-2 + — (m + n-1) (m + n) (m-»-l) (m + n) \ [Using (v) and (vi)]
2 nn mn j
m-n 4?J 1
{m + ?i) ^ = — (w - n) {m + 2n)

ee
%1-n - -2 +
2 mn m

Fr
EXAMPLE 19 If the sum offirst m terms of an A.P. is the same as the sum of its first n terms, show that
the sum of its (m + n) terms is zero. for
SOLUTION Let a be the first term and d be the common difference of the given A.P. Then,
ur
%i -
s

y ^^2a + {m -1) frj |2i7 + {n -1)


ook
Yo
eB

2a {m - n) + {m (w -1) - n {n -1)} fr = 0
2a {m - n) + {(m^ - «^) - {m - 7i)} fr = 0
our

{m -n) [2a + {m + n-l)d\ = 0


ad

2a + (777 + 71-1) d = 0 [v 777 - 77 0] ...(i)


m + n 777 + 77
^i + n ~ 2a + {m + 77 -1) d X 0 = 0 [Using (i)l
Y

2 2
Re
nd

●:\.\MPLE20 The ratio of the sums of m and n terms of an A.P. is n? : ?7^. Show that the ratio of the mth
Fi

and nth terms is {2m -1): {In -1).


SOLUTION Let a be the first term and d the common difference of the given A.P. Then, the sums
of 777 and 71 terms are given by

^77 — 2a + {m -1) d ■ and, 2ti + (77 -1) fr ■ respectively.


2

It is given that
C 2
2
n

^ |2fl +(m - 1) frj 777


2

^ |277+(tI -1) frj


2
77
ARITHMETIC PROGRESSIONS 8.21

2(7 + (»/ -1) d m

2a + (/I -1) d u

[2a + {m -1) d] u = [2n + («-!) d] m


2a (71 - m) = d {(77 -1) m - {m -1) 7i)
2a (77 - 777) = d (77-777)
d - 2a

ow
Tin _ a + (777 -1) d a + (777 -1) 2(7 2 777 -1

7;, (7 + (77 - 1) fr a + (77 -1) 2(7 2 77 -1

EXAMPLE 21 The Ulterior angles of a polygon are in A.P. The smallest angle is 120° and the common
difference is5°. Find the number of sides of the polygon.

e
Let there be n sides of the polygon. Then, the sum of its interior angles is given by

re
SOLUTION

= (277 - 4) right angles = (77 - 2) x 180° ●●●(i)

Flr
Thus, the interior angles form an A.P. with first term a = 120° and common difference d = 5°.

F
^ - 2xl20° + (77-l)x5° ■ ●●●(ii)
ou
sr
From (i) and (ii), we get
(77 - 2) X 180° = -J2xl20°
2
+ (77-l)x5°
fko
oo
(71 - 2) X 360 = 77 (577 + 235)
^ - 2577 + 144 = 0 (77 -16) (77-9) =0 => 77 = 16 or,
Y
77 77 = 9
reB

For 77=16, we obtain


Last angle = a„=a + (n -l)d =120° + (16 -1) x 5 = 195° , which is not possible.
uY

Hence, n = 9.
EXAMPLE 22 Thefirst, second and the last terms of an A.P. are a, b, c respectively. Prove that the sunt is
{a + c){b + c - 2a)
ad
do

2(b-a)
in

SOLUTION Let d be the common difference of the given A.P. Then, d = b - a. Let there be n
terms in the given A.P. Then,
Re

c = 77th term
F

c = a + {n-l) d
c = (7 + (77 -1) (/? - (?) [●.● d = b-a]
- a c -a b + c - 2a
77-1 = - ^ 77 +1 => 77 =
b -a b -a b -a

Sum of the A.P. = Sum of its n terms

= -(<I + C) Using :% = | (77 + /)


_ {a + c){b + c - 2a)
~ 2(b-a)
EXAMPLE 23 Let Sfj denote the sum of the first n terms of an A.P. If S2„ = 3 5y,, then prove that = 6.
SOLUTION Let (7 be the first term and d the common difference of the given A.P. Then,
8.22 APPLIED MATHEMATICS-XI

2jj 3/1

y|2fl+(2//-l)£/| = — ■ 2a + {n-l)d-
2 {2a + (2/7 -1) frj = 3 [2a + (// -1) d]
2rt - (3/7 - 3 “ 4/7 + 2) fr = 0
2t7 -(/7 + 1) fr = 0
=> 2t7=(/7 + 1) d
3/7

^ ■ 2t7 + (3/7 -1) fr


Now, -

/7
2a + {n-l) d ■

w
3<^ (n +1) fr + (3/7-1) d ■
^37/ _
[Using (i)]
{n + 1) d +{n-l) d ■

F lo
e
Fre
bn. =1?A^ = 6.
2nd for
t ype IV ON SUM OF TERMS OF AN A.P.
EXAMPLE 24 Prove that a sequence is an A.P. iffthe sum of its n terms is of theform An^ + Bn, where A,
r
B are constants.
You
oks

SOLUTION Let 5^, be the sum of n terms of an A.P. with first term 77 and common difference d.
eBo

Then,
2

= ^| 277 + (/7 -l)7f| = 77/7 +


n n
d- — d =
^d] n 2 +
d^
a — 77
2 2 2 2
ad
our

S, = Ai?" + Bn, where A = —2 and B = 77 — ^


2

TIius, the sum of n terms of an A.P. is of the form A/7^ + Bn.


Re
dY

Conversely, let the sum of n terms of a sequence 77^, 772, a^ ...be of the form A/7^ + Bn.
Fin

Then, we have to show that the sequence is an A.P.


We have, £y, = A//^ + Bn
S:,_i = A(/7-l)^ + e(/7-l) [On replacing n by // -1]
Now, 77 ij = - -S/ - 1
an
= {An^ + Bn]-[A (//-1)^ + B(//-l)l = 2An + (B-A)
77,,^! = 2A(// + 1)+(B-A) [On replacing /7 by // + 1]
‘hi+l~^n= l2^ (/7 + 1) + 6 - A} - [2A n + {B-A)}= 2A
Clearly, (tjj + \ - a„ = 2A for all n e N. So, the sequence is an A.P. with common difference 2A.
REMARK It foUoivs from this example that a sequence is an A.P. iff the sum of its n terms is of the
form An + Bn i.e. a quadratic expression in n and in such a case the common difference is twice the
coefficient of n . For example, ifS,j = 3/i + 2n, one can easily say that it is the sum of n terms of an
8.23
ARITHMETIC PROGRESSIONS

1 Q 2 Q n is the
A.P. with common difference 6. Similarly, ^, =/iP+ —/J (/I-1) Q =—» + P 2

sum of H terms of an A.P, with common difference Q.

EXAMPLE 25 Find the sum of first 24 terms of the A.P. fl-j, U2, ay ... , if it is known that
^ ^10 ^ ^15 ^20 ^24 ~ 225.
SOLUTION We know that in an A.P. the sum of the terms equidistant from the beginning and
end is always same and is equal to the sum of first and last term i.e. a-^+Ojj = «2 + l
= ^3 + ^1-2 - ... So, if an A.P. consists of 24 terms, then Ui +024=0^+ ^20 ='^10 + ‘^15-
Now, flj + 175 + rtjo ^15 ‘^20 ^24 = 225

ow
(rt| + (?24) + (^ + <^20) + + ^15) “
3 (rtj + ^^24) “ 225
225
+ ^24 - = 75

e
24
Using S„=jiai+a„)

re
$24 = — + 1724)

rFl
F
S24 = 12 (75) = 900 [Using (i)l

r
The first term of an A.P. is a and the sum offirst p terms is zero, shozv that the sum of its
fo
ou
EXAMPLE 26
»(p + q)'?
ks
next q terms is

Let fr be the common difference of the A.P. It is gtiven that the sum of first p terms is
oo

SOLUTION
zero
Y
eB

^|2r7 +(p -l)rf|


2(7
= 0=> d=-
i.e.
Sp=0^- p-1
Let S be the required sum. Then,
ur

S = a +...+ (?
p+1'*'^p+2
ad

p+q
Yo

S={ai + n2 + ..■ + ap + ] + .. ■+^p+ q) -(^1 + «2 V


S - Sp+ q~^p
d

S =S -0 [●.● Sp = 0 (given)]
p+'7 r 1
Re
in

S=^-^ J 2(7 +{p + q-l)di


F

2(7

S = ^^- 2a + {p-i-q-l) - p-\


p + q-^
1
p-l-p-q+l {p+q)a
S = {p + q) all- = {p + q)a
p-1
P-1 P-1

EXAMPLE 27 If the first term of an A.P. is 2 and the sum offirst five terms is equal to one-fourth of the
sum
of the next five terms, find the sum ofifrst 30 terms.
SOLUTION Let (7|, (72, (73,... be given A.P. with common difference d. It is given that (7j = 2 and
the sum of first five terms is equal to one fourth of the sum of next five terms.
1
I.e. (7j + (?2 + (73 + (74 + (75 = - ((7(^ + (77 + (?8 + (79 + (7jo)

4((?2 + (72 + (73 + (?4 + (75) =((7^ + (?7 + (78 + 779 + (7io)
5 {a-[ + (?2 + 773 + (?4 + (75) = (77.1 + (?2 + . ●. + 77iq)
8.24
APPLIED MATHEMATICS-XI

5 % - Sio
5 5
-j2x2 + (5 -l)d = y|2x2 + (10 -l)d
50 (1 + fr) = 20 + 45 fr
d = -6
Thus, we have a =2 and d = -6.
30

Required sum = S30 = — 12 x 2 + (30 -1) x - 6| = - 2550.


EXAMPLE 28 The term of an A.P. is a and qth term is b. Prove that the sum of its (p + q) terms is
p + q a -b
a +b +
2 p-q

SOLUTION Let A and D be the first term and common difference respectively of the given A.P.

w
Then,
a = pth term = A+(p-l)D
b = qth term =>b = A+{q-l)D
Subtracting (ii) from (i), we get: D =
a-b

F lo ...(ii)

e
p~q

Fre
Adding (i) and (ii), we get a + & = 2A+{p + q-2)D
=>
a + b = 2A+{p + q-l)D-D
for
{a + b)+D = 2A+{p+ q-l)D
a ~b
(a + b) + = 2A+{p + q-l)D
r
...(iii)
p~q
You
oks

Now, S = Sum oi{p + q) terms


■p+q
eBo

P|2A+(p + ,-1)d|
_ p + q ■ a + b + a-b
^p+q- [Using (iii)]
2 p-q

EXAMPLE 29 The ratio of the sum ofn terms of two A.P.'s is {7n + 1): (4» + 27). Find the ratio of their
our
ad

mth terms.

SOLUTION Let ^2 t>e the first terms and d2 the common differences of the two given
A.P.'s. Then, the sums S,j and of their n terms are given by
dY
Re

+ (»-1) snd |2(72 + (»-1) ^2!


Fin

^|2/7^+(«-!) _ 2ai + {n -1) frj

"|2«2+(n-l)rf2}
2a2 +(n-l) ^2

It is given that
7n + l

4n + 27

2a-y +(n-l) d^ 7n + l

2(?2 + (n — 1) ^2 4» + 27
n-1
fli+ fri
2 ^ 7n + l
n-1^ 4n + 27 ●●●(i)
+
di
I 2
ARITHMETIC PROGRESSIONS 8.25

+(w-l) d-[
We have to find the ratio to m‘h terms of two A.P.'s i.e., . Clearly, this can be
flj +(m-l) ^2
obtained by replacing by (»i -1) on the LHS of (i). Replacing bym -1 i.e. n by (2m -1) on
both side of (i), we get
a^ + {m-l)di _ 7(2m-l) + l 14 m - 6

a2+{m-l)d2 ~4(2m-l) + 27 8m + 23

Hence, the ratio of the mth. terms of the two A.P.'s is (14m - 6): (8m + 23).

REM.ARK It is evident from the above example that if we are given the ratio of the sums ofn terms of two

ow
A.P.'s then the ratio of their m^^' terms is obtained by replacing n by (2m -1).
EXAMrLL3(l The sum ofn terms of two arithmetic progressioiis are in the ratio {3n + S):{7n +15).
Find the ratio of their 12th terms.
SOLUTION Let a^, ^2 he the first terms and d-^, d2 the common differences of the two given

e
A.P.'s. Then, the sums of their n terms are given by

re
^1=^ {2fl;i+(»-!) and, V =| {2«2 + ~ 1) ^2)
Frl
F
It is given that
5^ _ 3» + 8
ou
or
S,/ 7n +15 kfs
3n + 8
=>
7n +15
oo
Y

2flj + (n -1) _ 3n + 8
B

2a2+(n-l)d2 7n +15
re

. ‘^1
oYu

2 3n + 8
ad

n-1 7n +15
^2 di
2 ^
d
in

Replacing by 11 i.e. n by 23 on both sides, we get


Re
F

+11 d, _ 3x 23 + 8 _ 77 _ 7
fl2 + 11 d2 ~ 7 X 23 + 15 “ " 16
Hence, the required ratio is 7:16.
th
MITER If the ratio of the sums of n terms of two A.P.s is given, then the ratio of their m terms
is obtained by replacing n by (2m -1) in the given ratio. So, required ratio is obtained by
replacing n by 2 x 12 -1 = 23 in (3?z + 8): (7n +15).
Hence, required ratio = (69 + 8): (161 + 15) = 7 :16.
EXAMPLES! If there are (2n + 1) terms in A.P., then prove that the ratio of the sum of odd terms and the
sum of even terms is (n + 1): n.
SOLUTION Let a and d be the first term and common difference respectively of the given A.P.

Let a]^ denote the terms of the given A.P. Then, aj^ = a + [k -1) d.
Now, = Sum of odd terms = «i + ^3 + %+●●● + ^2/1 + 1
8.26 APPLIED MATHEMATICS-XI

n + \(
Si =
2 \
+^2ii+]
= jfl + (7 + {In + 1 -1) frj -{n + 1) (rt + mi)
and. S2 = Sum of even terms = 02 + + a(^ +... + a2j,

= ^ (i7 + d) + |fl + (2?7-l)d|


c ” f
% - T ^2 + ^2ii = ?? (n + nd)
I V

: S2 = (« +1) (t7 + mi) :«(<? + nd) = (n + l):n


EXAMPLE 32 Ifthcsum ofm terms of an A.P. is equal to the sum of either the next n terms or the next p

ow
terms, then prove that
1 1
(m + n) -(m + p)
m
Pj m n

e
SOLUTION Let a denote the first term and d the common difference of the A.P. Further, let aj^

re
denote the term of the A.P. Then,
Sum of m terms = Sum of next n terms

Flr
F
nj+a2+a;^+... + a„, = + 1 + + 2 + ■■■ + + h
2{a■^ + H2 + ... + a,„) = ai + H2+ ... + a,fi + a,,,
2S,„ = ^It
ou + 1 + + 2 + ●●■ + % 4- II

sr
+ 11

r. m ..., , ,,
2 — ■ 2i7 + {m -1) d ■ = —-— 2a + {m + n~l) d ■
, J7/ 4- IJ ^ ,

fo
k
oo
2m 2a + {m + n-l) d
Y
m + n 2a + {m - \) d
reB

2m
-1
2a + {m + 7j -1) d
1
m + n 2a + (J7J ~1) d
uY

m - n nd
777 + 77 2(7 + (777 -1) d
Similarly,
ad
do

Sum of 777 terms = Sum of next p terms


m -p pd
...(h)
in

m + p 2(7 + (777 -1) d


Re

Dividing (i) by (ii), we get


F

777 - n m + p n

777 + 77 m- p P
(777 -77) (777 + p) {m + 77) (777 - p)
n
P
(777 - 77) (777 + p) (777 + 77) (777 - p)
77777
mp
777 - 77
777 -p
(777 + P) = (777 + 77)
77777
mp

1 1 1
(777 + 77) —- = ('» + P)
p
\ ^
777 n 777

/
. J 1 1
(777 + 77) = (777 + p) -
772 p 777 n
ARITHMETIC PROGRESSIONS 8.27

EXERCISE 8.4

1. Find the sum of the following arithmetic progressions:


(i) 50, 46, 42,... to 10 terms
(ii) 1, 3, 5, 7,... to 12 terms
(iii) 3, 9/2, 6,15/2,... to 25 terms
(iv) 41,36, 31,... to 12 terms
(v) a +b, a -b, a - 3b,...io22terms
(vi) {x-y)^, {x^ + y^),{x + y)^,... ton terms
, ., X - u 3x-2y 5x -3y
,... to terms
X +y X + y X +y

2. Find the sum of the following series:

w
(i) 2 + 5 + 8 + ...+ 182
(ii) 101 +99 + 97 + ... + 47
(iii)

F lo
+ b^) + {ci + b) +,,. + [(rt +1’) + 6(ib]
3. Find the sum of first n natural numbers.

ee
Find the sum of all natural numbers between 1 and 100, which are divisible by 2 or 5.

Fr
4.

J. Find the sum of first n odd natural numbers.


6. Find the sum of all odd numbers between 100 and 200.
for
ur
7. Show that the sum of all odd integers between 1 and 1000 which are divisible by 3 is 83667.
8. Find the sum of all integers between 84 and 719, which are multiples of 5.
oks
Yo

9. Find the sum of all integers between 50 and 500 which are divisible by 7.
o

! Find the sum of all even integers between 101 and 999.
eB

n. Find the sum of all integers between 100 and 550, which are divisible by 9.
12. Find the sum of the series: 3 + 5 + 7 + 6 + 9 + 12 + 9 + 13 + 17 + ... to 3;i terms.
our
ad

13. Find the sum of all those integers between 100 and 800 each of which on division by 16
leaves the remainder 7.
Y

14. Solve: (i) 25 + 22 + 19 + 16 + ... + .v = 115 (ii) 1 + 4 + 7 + 10 + ... + A- = 590.


Re

2
nd

15. Find the rth term of an A.P., the sum of whose first n terms is 3n + 2n.
16. How many terms are there in the A.P. whose first and fifth terms are -14 and 2 respectively
Fi

and the sum of the terms is 40 ?

17. The sum of first 7 terms of an A.P. is 10 and that of next 7 terms is 17. Find the progression.
18. The third term of an A.P. is 7 and the seventh term exceeds three times the third term by 2.
Find the first term, the common difference and the sum of first 20 terms.
19. The first term of an A.P. is 2 and the last term is 50. The sum of all these terms is 442. Find
the common difference,

"ir. The number of terms of an A.P. is even; the sum of odd terms is 24, of the even terms is 30,
and the last term exceeds the first by 101/2, find the number of terms and the series.
2'*. If £^, =« p and 5^ =w p, m 5^ n, in an A.P., prove that Sp = p .
22. If 12* term of an A.P. is -13 and the sum of the first four terms is 24, what is the sum of first
10 terms ?

23. If the 5th and 12f/i terms of an A.P. are 30 and 65 respectively, what is the sum of first 20
terms ?
8.28 APPLIED MATHEMATICS-X!

24. Find the sum of terms of the A.P. whose kth terms is 5k + 1.

Find the sum of all two digit numbers which when divided by 4, yields 1 as remainder.
If the sum of a certain number of terms of the AP 25,22,19,... is 116. Find the last term.
Find the sum of odd integers from 1 to 2001.
11
How many terms of the A.P. - 6, - 5,... are needed to give the sum - 25?
2 '
In an A.P. the first term is 2 and the sum of the first five terms is one fourth of the next five
terms. Show that 20th term is -112.
30. If Si be the sum of (2n + 1) terms of an A.P. and S2 be the sum of its odd terms, then prove

ow
that: Si:S2=(2/i + 1):(/j + 1).
Find an A.P. in which the sum of any number of terms is always three times the squared
number of these terms.
1
If the sum of n terms of an A.P. is nP + -n{n-1) Q, where P and Q are constants, find the

e
re
common difference.

rFl
The sums of n terms of two arithmetic progressions are in the ratio 5/2 + 4:9/2 + 6. Find the

F
ratio of their 18th terms.
’.4. The sums of first n terms of two A.P.'s are in the ratio (7/2 + 2): (22 + 4). Find the ratio of thel;-

r
ou
5th terms. fo
ks
ANSWERS

1. (0320 (ii) 144 (iii) 525 (vi) n{(x-yf +(/T-l).Yy}


oo

(iv) 162 (v) 22(7-4401/


Y

|M2.v-y)-y| -(0
22

6 {a^ +b^ + 3ab)


eB

(vii) 5612 (ii) 2072 (iii)


2{x + y)
n (22 + 1) 2
ur

4. 3050 o. n 6. 7500 8. 50800 9. 17696 10. 246950


1
ad
Yo

'T 16425 12. 322(2/2+ 3) 13, 19668 14. (i) -2 (ii) 58 15. 6r-l
10 a=l, d=l/7 18. -1,4, 740 19. 3
d

1
8 terms, 1 — ,3,a.... n
Re

0 23. 1150 -(5/2 + 7)


in

2". 1210
2 2
F

26. 4 27. 1002001 28. 5 or 20 31. 3, 9,15,21 32. Q


33. 179:321 34. 5:1

HINTS TO SELECTED PROBLEMS


n
J Required sum =1 + 2+ 3 + ...+?2= —

4. Required sum = Sum of natural numbers betweenl and 100 which are divisible by 2
+ Sum of natural numbers between 1 and 100 which are divisible by 5
- Sum of natural numbers beh\'een 1 and 100 which are divisible by 2
and 5 both i.e. by 10
= (2 + 4 + ... + 100) + (5 + 10 + 15 +... + 100) -(10 + 20 + ... + 100)
50 20 10
= —(2 + 100) + —(5 + 100)- —(10 + 100)
2 2
= 2550 + 1050-550 = 3050
ARITHMETIC PROGRESSIONS 8.29

n 2
5. Required sum = 1 + 3 + 5 + ... + (2n -1) = -■ 1 + (2n - 1) - = n

6. Required sum = 101 + 103 + ... + 199 = y (101 +199) = 7500


127
8. Required sum = 85 + 90 + ... + 715 =
y (85 + 715) =50800
9. Required sum = 56 + 63 + ... + 497
10. Required sum = 102 + 104 + ... + 998
11- Required sum = 108 + 117 + ... + 549

ow
12. Required sum = (3 + 6 + 9 +... to n terms) + (5 + 9 +13 +... to n terms) + (7 +12 +17 +... to n
terms)
13. Required sum = 103 + 119 + 135 + ... + 791

15. Or =S;.-S;._i=(3r^ + 2r)-] 3(r-l)^+ 2(r-l)

e
= 6r-l

re
17. We have, S7 = 10 and $14 = 10 +17 = 27

Frl
F
24. We have.
flfc = 5A: +1 => flj = 6 and a„=5n + l
ou
c ”r ^

r
so
2 V /
nr
6 + 5n+l = — 5«+7
kf
2
oo

25. We have to find the stun of all two digit numbers of the form 4fc +1, it e N. Clearly, such
numbers are 13,17,21,25,..., 97 and are forming an A.P. with common difference 4. Let
Y
eB

such numbers be n in number. Then,


th
97 = n term of AP with first term 13 and common difference 4
ur

97 = 13 + (n-l)x4
oY

n-1 =21
ad

M = 22
d

Let S be the sum of such numbers. Then,


S = nf \
in
Re
F

=> S = —fl3+97 =1210


2 V
26. Let the sum of n terms of the A.P. 25,22,19,... be 116. Then,

116 = ||2x25 + (n-l)x(-3)|


232 = n(-3n + 53)
3«^-53n + 232 = 0
3n^-24n-29n + 232 = 0
(n-8)(3n-29) = 0
29
n = 8 V —
3

flg = 25 + 7 X (- 3) =4
8.30 APPLIED MATHEMATICS-XI

27. The odd integers from 1 to 2001 arel, 3,5,7,2001. Let thenumber of such integers ben.
Then,
2001 = 1 + (n -1) X 2 => n = 1001
1001
^ (1 + 2001) =
Required sum = 1001 x1001 = 1002001.

2
auter The sum of first n odd integers is n . So, the sum of odd integers 1, 3, 5, 7, ..., 2001 is
(1001)^ =1002001.
29. We have.

— 2 and + ^2 — -{fl^ + + ... + fl^o)

ow
Now,
1
fll + <72 ●●● + - — (<76 + <7/ + ... + <7iq)

4 (flj + <?2 + ... + <^) = <?6 + <77 + ... + <7^q

e
4 S5 = Sio -S5

re
5 S5 = S^o

Fl
F
s 1 10 r 1
5 |{2x2 + (5-l)rf} =y 2x2 + (10-l)d)
ur
^ (4 4- 4<^) = y (9<f + 4)
r
=> fo
ks
20 (1 + fr) = 2 (9d + 4) => 10 + lOrf = 9d + 4^ d = -6
Yo

<720 = «i + 19<f = 2 + 19x(-6) = -112


oo

2
31. Use = 3/1 and <^,=5^-^ <-1-
eB

32. We have.

= «P + in{n-l)Q=> ^ ={n-1) P + ^(«-1)(n-2)Q


ur

th
ad

Let <7„ be the n term. Then,


Yo

„p + i„(„-l)Q| _|(„_i)F + i(«-l)(n-2) q|


d

a
Re
in

=>«<!= T + ^(n-l){n-(n-2)}Q
F

<7„ = P + (n -1) Q
<7„_1=P + (7I-2)Q
Let d be the common difference. Then,
d=<7„-n,,^;i={P+{n-l)Ql-{P + (n-2)Ql=Q
ALITLK We have.

S,=nP + h{n-\)Q^ S„=^n^Q+^P-^


Q n

fy

Clearly, 5^ is of the form An + Bn. Hence, the sequence is an A.P. with common difference
1A=Q.
33. Let S„ and S'„ be tlie sums of n terms of two arithmetic progressions. Then,
_ 5n + 4
9n + 6
ARITHMETIC PROGRESSIONS 8.31

=>
9» + 6

2rtj + {11 -1) di _5« + 4


2«72 + (n -1) ^^2 +6
/
'i-l .
«l + —- fri
2 ^ 5» + 4

9m+ 6
^2 ^2
2 j

ow
Ml +17^1 _ 179
Replacing by 17 i.e. m by 35, we get
^2 "I" 17 ^2 321
th
A LITER If the ratio of the sunas of n terms is given, then to find the ratio of their n terms, we
replace n by {In -1). So, to find the ratio of 18th terms, we replace « by 2 x 18 -1 = 35 in the ratio

e
5n + 4: 9n + 6

re
rFl
Hence, required ratio is (5 x 35 + 4): (9 x 35 + 6) i.e. 179 : 321.

F
8.6 PROPERTIES OF ARITHMETIC PROGRESSIONS

In this section, we shall discuss some properties of arithmetical progressions which will be

r
frequently used in this chapter and in the subsequent chapters. fo
ou
PROPERTY 1 If a consfatit is added to or subtracted from each term of an A.P., then the resulting
ks
sequence is also an A.P. with the same common difference.
oo

PROOF Let Ml, M2, M3,... be an A.P. with common difference d, and let cf be a fixed constant
Y

which is added toeachtermof this A.P. Then, the resulting sequence is Mi + Ic, M2 +^3 + ^l...
eB

Let5„ =M„ + /c, M =1, 2,... Then, the new sequence is b^, b^.b-^,,..-
Now, b,,^ 1 =(m,,+ 1 + cf) -(m„ + k) =a,,_^ 1 -m,It = d for all M e N
ur

Thus, the new sequence is also an A.P. with common difference d.


ad
Yo

PROPERTY 2 If each term of a given A.P. is multiplied or divided by a non-zero constant k, then the
resulting sequence is also an A.P. with common difference kd or d/k, where d is the common difference of
d

the given A.P.


Re
in

PROOF Let Ml, M2, M3,... bean A.P. with common difference d and let cfbe a non-zero constant.
F

Let&i, &2/ ^3/-- . be sequence obtained by multiplying each term of the given A.P. by k. Then,
bi = Ml k, ^2 = ^11 =
Now, ^-b„ m„) k = dk for all neN [●.● m^+ 1 -a„ = d for all n e N]
ik - a,t k = {a,^+
This shows that the new sequence is an A.P. with common difference dk.
Similarly, it can be proved that on dividing each term of a given A.P. by a non-zero constant, we
obtain a sequence which is also an A.P.
PROPERTY 3 In a finite A.P. the sum of the terms equidistant from the beginning and end is always
same and is equal to the sum offirst and last term.
I.e.
1) =^i
M„ for all k=l,2, 3,..., m -1.
PROOF Let Ml, M2, M3,..., M„ be an A.P. with common difference d. We have to show that
Ml + M„ = ^2 + - 1 =^^3 + - 2 = ^4 + 3 = ■■■
I.e.
Ml + M„ = 1) for all /: = 1,2,3,..., M-1
8.32 APPLIED MATHEMATICS-XI

For any ^ =1, 2, n-1


+ % ~{k - 1) - + 1 - it
= [a-^ +{k -1) d]+[a-i+ {n + 1 - k -1) d]
= la-^ + {k ~1 + n +1 - k -1) d
= 2ai + {n -1) d = + [ai +(n-1) d] = Oi + a,,.
PROPERTY 4 Three numbers a, b, c are in A.P. iff 2b = a + c.

PROOF First, let a, 6, c be in A.P. Then,


b-a = Common difference and, c-b = Common difference
b-a = c-b

2b = rt + c

Conversely, let fl, b, cbe three numbers such that 21? - a + c. Then, we have to show thatfl, b,c are
in A.P.

w
We have, 2b=a + c ^ b -a = c -b ^ a, 1?, c are in A.P.

ILLUSTRATION

SOLUTION It is given that,


rr 2 ,
lf-,k, — are in A.P., find the value of k.
3

F lo
e
Fre
2,5 -AD TJ 2 -
—, k, — are in A.P. => 2/c = — + —
31
=> 21c = — =^k =
31
3 8 3 8 24 48
for
PROPERTY 5 A sequence is an A.P. ffits 7ith term is a linear expression in n i.e. a„ = An + B, where A,
B are constants. In such a case the coefficient ofn in a„ is the common difference of the A.P.
r
You

PROOI See example 3 on page 8.3.


oks
eBo

PROPERTY 6 A sequence is an A.P. iff the sum of its first n terms is of the for?n A + B?i, where A, B
are constants independent ofn. In such a case the common difference is 2A i.e. 2 times the coefficient of r-?.
PROOF See example 6 on page 8.18.
ad
our

PROPERTY 7 If the terms of an A.P. are chosen at regular intervals, then they form an A.P.
PROPERTY 8 lfa^i,a„+i and + 2 three consecutive terms of an A.P., then 20fj + ^ =a„ + a,,^ 2-
Re
dY

ILLUSTRATIVE EXAMPLES
Fin

Type I TO PROVE THAT THREE NUMBERS ARE IN A.P. WHEN THREE GIVEN NUMBERS ARE IN A.P.
ex.amplf: 1 If a, b, c are in A.P., prove that the following are also in A.P.
ri 1 1 _
1
(ii) b + c,c + a,a + b
be ' ca' ab
..... fl (1- + -1 .biU-1 (\ 1
(ill)
[b
, c - + -
a b
(iv) {b + c), b^ (c + a), c^ {a + b)
C) \c a

(V)
1
^ \{c + a)'^-b^ \,\{a + b)^-c^
1 1
■ib + c)^ -a
(Vi)
y|b + ffc'^^c + ffa'yfa■h^|b
SOLUTION (i) a, b, c are in A.P.
a
b e
—, —, — are m A.P.
. A ^
abc abc abc [On dividing each term by abc and using Property 2]
ARITHMETIC PROGRESSIONS 8.33

111
— , — , — are in A.P.
-AO
be ca ah
1 1 1
Thus, a, b, c are in A.P. => —, — , — are in A.P.
be ' ea ab
(ii) It is given that
a, b, c are in A.P.
=> a-{a+b + e), b-(a + b + c), c-{a-hb + e) are in A.P. [Subtracting a + b + c from each term]
=> -(& + c),-(c + fl),-(fl + b) are in A.P. [Multiplying each term by -1]

w
=> b + c,e + a,a + b arem A.P.
(iii) a, b, c are in A.P.
a b Q

abe abc
— are in A.P.
abc
[On dividing each term by abc and using Property 2]

o
e
1 1 1 are in A.P.

be ' ca' ab

re
ab + be + ca ab +bc + ca ab + be + ca

Frl
are in A.P.

F
be ca ab

[On multiplying each term by ab + bc + ca and using Property 2]


ab + be + ca
ab + be + ca ^ ab + be + ca
ou
r
-1, -1 are in A.P.
be ca ab

so
[On adding -1 to each term and using Property 1]
ab + ac ab + be be + ca
kf
are in A.P.
oo
be ca ab
fl l) . 1 1 fl- 1
Y

a - + - ,b - + - / c + - are in A.P.
B

b c
vC a a b

(iv) a^ (b + c), b^ (c + a), c^ (a + b) will be in A.P.


re

if (c + (i) - a^ (b + c) = c^ (a + b) - b^ (c + a)
oY
u

i.e. if c {b^ ~a^) + ab {b-a) = a{c^-b'^) + be {c-b)


ad

i.e. if (b - a) {ab +bc + ca) = (c -b) {ab +bc + ca)


d

i.e. if b - a = c -b
in
Re

i.e. if 2b = a + c
i.e. if fl, b, c are in A.P.
F

2 2 2
Thus, fl, b, c are in A.P. a {b + c),b (c + a), c {a + b) are in A.P.
At-ITER It is given that
fl, b, c are in A.P.
a b c
— are in A.P.
abc ' abc ' abc
Jl Jl Jl are in A.P.
be ca' ab
ab + be + ca ab + be + ca ab + be + ca
are in A.P.
be ca ab
ab + ca ab +bc bc + ca
1 + ,1 + ,1 + are in A.P.
be ca ab
a{b + c) b{a + c) c{a + b)
be ca ab
are in A.P. [Subtracting 1 from each term]
8.34 APPLIED MATHEMATICS-XI

a^{b + c) b^{a + c) c^{a + b) are in A.P.


abc ' abc ' abc
2 2 2
a {b + c),b {c + a),c («+/;) are in A.P. [Multiplying each term by abc]
(v) It is given that
a, b, c are in A.P.
-2a, -2b, -2c are in A.P. [Multiplying each term by - 2]
a + b + c-2a, a + b + c-2b, a + b + c-2c are in A.P. [Adding a + b + cto each term]
b + c-a, c +a-b, a + b-c are in A.P.
{a+b + c){b + c-a), {a + b + c){c + a -b), {a+ b + c) (a+ b-c) are in A.P.
[Multiplying each terming a + b + c]
{b + cf -a (c + a^) -b^, (a + b)^ -c^ are in A.P.

w
1 1 1
(Vi) will be in A.P.
^Jb + ^fc'^^c + y/a'^ + ^^b

Flo
1 1 1 1
if
Jc+ja ^jb + yfc yfa + y/b yfc+yfa

ee
yjb - y[a {4c--Jb)

Fr
i.e. if
(4c + 4^) (4b + 4^) (4ci + 4b) i4c + 4^)
i.e. if
4b -4^ 4^ ~4b for
ur
4b +4c 4^ + 4b
i.e. if b-a = c-b
ks

i.e. if 2b = a + c
Yo
oo

i.e. if a, b, c are in A.P.


1 1 1
eB

Thus, a, b, c are in A.P. => are in A.P.


4b + 4c'4c + 4^'4^ + 4b
AUTER It is given that
r
ou
ad

a, b, c are in A.P.
b-a =c-b
Y

\ f

4b +4^ 4b -4^ = 4c ~4b 4c +4b


nd

/ V / V
Re

/ \

4b -4^ _ 4c ~4b
Fi

4b+4c 4b+4^
\ /

4b + 4c — 4^ 4c 4c +4^ - 4b +4^
/ \

4b +4c 4b+4^
\ f \ f \ /

4b + 4c - 4c +4^ 4c +4^ - 4^ +4b


\ / \ f

4b +4c 4c +4^ 4^ +4b 4c +4^


/ V / \

1 1 1 1

4c+4^ 4b+4c 4^+4b 4c+4^


1 1 1
are in A.P.
4b + 4c' 4c+4^' 4^+4b
ARITHMETIC PROGRESSIONS 8.35

0 0 'y
EXAMPLi;:
I/a , b , c are in A.P, then prove that the follozuing are also in A.P.
1 b
(i)r^
1 a c
(ii)
b + c' c+a a +b b + c' c + a a + b

SOLUTION (i) _l 1 1_ will be in A.P.


b + c'c +a'a+b
1 1 1 1
if
c + a b + c a +b c + a

ow
b -a c -b
i.e. if
(c + a){b + c) {a + b) {c + a)

i.e. if b-a _ c-b

e
b+c a+b

re
i.e. if -a 2 =c^-b^
i.e. if 2b^=-^ -2

F
a + c

i.e. if
2
a ,b , c
2 2
are in A.P.

Frl
1
ou
1 1
a^, b^, c^ are in A.P. ^

sr
Thus, are in A.P.
b+c'c+a'a+b
ALITER
(i) It is given that
b^, are in A.P.
kfo
oo
a

b,2 -a 2 =c 2,2
-b
Y
reB

{b~a) {b + a)-{c-b) (c + b)
b-a _c-b
b+c a+b
uY

{b + c)-{a + c) _{c + a) -{b + a)


b +c a +b
ad
do

{b + c)-{a + c) _ (c + fl)-{b + a) [Multiplying both side by —


a +b
{a + c){b + c) (a + b) (a + c)
in

j. J l_
Re

a+c b+c a+b a+c


F

1 1 1
are in A.P.
b + c ' c + a ' a + b'
a b
(ii) — will be in A.P.
b + c'c + a'a + b

—+ l,-^ + lare in A.P.


a
if +1/ [On adding 1 to each term]
b + c c + a a + b

a + b + c a +b + c a +b + c
i.e. if are in A.P.
b+c ' c+a c + a

1 1 1
i.e. if are in A.P. [On dividing each term hya + b + c]
b + c' c + a' a +b
1 1 1 1
i.e. if
c+a b+c a+b c+a
8.36 APPLIED MATHEMATICS-XI

b -a c -b
i.e. if
(c + a){b + c) {a + b){c + a)
i.e. if b-a _ c-b
b + c n + b

i.e. if b^ - a " = -1,2


i.e. if 2b^ = a 2 + c
2

I.e. if a^,b^,c^ are in A.P.

ow
2 2 2 a b
Thus, a ,b , c are in A.P. ^ -— are in A.P.
b + c'c +a'a + b

ALITER It is given that


a^,b^, are in A.P.

e
b^-a^=c^~b\

re
(b+a) (b -a)={c + b) (c-b)

Frl
F
b -a _c-b
b+c a+b
ou
r
(b + c)-{o + c) _{c + a)-{b + a)

so
b +c a +b kf
{b + c)-{a + c) _ (c + fl) -{b + a)
{a + c) (^ + c) {a + b){a + c)
oo

_l
Y
B

a+c b+c a+b a+c

J. 1 l_ are in A.P.
re

b + c' c + a' a + b
oY
u

a+b+c a+b+c a+b+c


are in A.P.
ad

b +c c+a ' a+b


d

a
=> 1 + ,1 + are in A.P.
b + c c + a a +b
in
Re

a
b C -AD
b +c c+a a +b
F

b + c-a c + a-b a + b- c 1 1 1
EXAMPLES If - are in A.P., prove that - are also in A.P.
a b c a b c
b + c-a c + a-b a + b- c
SOLUTION - are in A.P.
a b c

b + c -a c + a-b a + b - c

a
+ 2\, b
+ 2) ,■
c
- + 2 ^ are in A.P. [Adding 2 to each term]
a+b+c a+b+c a+b+c
are in A.P.
a b c

1 1 1
- are in A.P
a ' b ' c Dividing each term hy a + b + c
2 2 2 1 1 1
EXA.MPLE 4 If a + 2hc, b + 2ac, c + lab are in A.P., show that , , are in A.P.
b-c c-a a-b
ARITHMETIC PROGRESSIONS 8.37

SOLUTION 0^ + 2bc, + lac, + lab are in A.P.


^ {a^ + Ibc) - {ab +bc + ca), {b^ + lac) - (ab +bc + ca), (c^ + lab) - (ab +bc + ca) are in A.P.
[On subtracting (ab +bc + ca) from each term]
2 2 "^
=> a +bc - ab -ca,b + ca-ab - be, c~ + ab - be- ca are in A.P.
^ (a - b)(a - c), (b - c) (b - a), (c - a) (c - b) are in A.P.
-1 -1 -1
are in A.P. On dividing each term by (a - b)(b - c) (c - a)
b-c'c-a'a-b
Jl 1 are in A.P. [On multipl)dng each term by -1]
b-e'e-n'a-b

ow
1 1
-— are in A.P.
EXAMPLES lf(b-c) ,{c-a) ,(a-b) are in A.P., prove that b - c ' c -a ' a -b

SOLUTION (b -c)'^,{c-a)^,{a-b)^ are in A.P.

e
^ (c -ij)^ -(b -c)^ = (a -b)^ -{c -a)^

re
=> (b - a) (Ic - a -b) = (c - b) (la -b -c)...(i)
rFl
F
=> (b - a) {(c - rt) + (c - b)] = (c-b) {(a - b) + (a- c)l
=> (b - a) (c - a) +(b - a) (c-b) = (c - b) (a -b) + (a - c) (c - b)

r
ou
=> -(a-b)(c - a) + (a -b)(b -c) =-(a -b)(b -c) + (b -c) (c-a) fo
ks
1 1 1 1
+ [Dividing throughout by (a -b)(b- c) (c - a)]
b-c c-a c-a a-b
oo

1 1 1
Y
eB

c-a a-b b-c

1 1 1
are in AP.
b-c'c-a ' a -b
r
ou

1
if (b - c)^, (c - a)^, (a -b)^ are in A.P., then —
ad
Y

Thus, -— are in A.P.


b -c ' c - a 'a-b

EXAMPLE 6 If a, b, c are in A.P., then prove that:


d

(i) (a -c)^ = -ac) (h) a^ + 4b^ + c^ = 3b(a^ + c^)


Re
in

SOLUTION (i) It is given that a, b, c are in A.P.


F

a + c
lb = a b =
1
„ . , a + c
Putting b = —— on RHS, we obtain

a + c
a2 - 4flc
RHS = 4 (6^ - flc) = 4 - -ac> = 4
4
■ =(a + c)^ -4flc =(a -c)^ =LHS
2

(ii) It is given that a, b, c are in A.P.


a + c
lb =a + c => b =
1

LHS=fl^+4f)^ + c^
^3
3 =(a^ + c^) + l{a + c)^
a + c
= a^ +4 + c
1
8.38 APPLIED MATHEMATICS-XI

= ~ 2(fl^ +c^) + (fl + c)^) ■


^ 12 (fl + c) (fl^ - (7C + C^) + (fl + c) ^ I
= i(fl + c)- 2((7^-flC + C^)+{fl + c)^ ■

= i(fl+c) 3(<7^ + c^) = 3 =3!j{fl^+c^)=RHS


ALHER LHS = + 4b^ +
= (a^ + c^) + 4b^
= (a + c)^ - 3ac {a + c)-¥ 4b^

w
= {2bf-3ac{2b)+4b^
= 12b^-6abc

F lo
= 3b{4b^-2ac) = 3b\{2])f-2ac\=3b\{a + c)^~2ac \ = 3& (<7^ + c^) = RHS

ee
Fr
EXAMPLE . If 0^ (b + c), b^ (c + a), (a + b) are in A.P., show that either a, b, c are in A.P. or
ab + be + ca = 0.

SOLUTION a^ {b + c), b^ (c + a), {a + b) are in A.P.


for
ur
(c + ii) - a^ {b + c) = {a + b) - b'^ {c + a)
s
ok
Yo

{b^a - aH) + (b^c - a^c) = {c^b - b^c) + {c^a - b'^a)


o
eB

{b - a) {ab +bc + ca) = {c -b) (ab +bc + ca)


{ab +bc + ca) {2b - a - c) = 0
ab + be + ca = 0 or 2b -a-c = 0
r
ou
ad

ab + bc + ca = 0 or a, b, c are in A.P.
p-x _p-y _p-z
Y

EXAMPLE 8 If and a, b, care in A.P. Shozu that — , - are in A.P.


ax cz
X y z
Re
nd

p-x _p-y _p-z


SOLUTION It is given that
ax
by cz
Fi

Let p-x _p-y _p-z


= X. Then,
ax cz

^=X,^ = X^ndEZl=x
ax
by cz

a = £^,6 = '^andc=L:i
Xx Xz

X X
,fc=if£-llandc=if£-l X z

It is given that a, b, c are in A.P. Therefore,


2b =fl + c

ifP-iVifE-l
X X X z
ARITHMETIC PROGRESSIONS 8.39

^=1 + 1
y X z

-=1 + 1
y X z

111- are in
— . A.r.
. p
X y z
EXERCISE 8.5

111
1. If-,-, - are in A.P., prove that:
a b c
b + c c + a a +b
(i) are in A.P. (ii) a{b + c), b{c + a), c{a + b) are in A.P.
' b ' c

ow
a

a b
— are in A.P.
2. If are in A.P., prove that b+c'c+a'a+b

3. If b, c are in A.P., then show that:

e
'2 .0 “2.
(i) a (b + c), b (c + a), c {a + b) are also in A.P.

re
(ii) b + c - c+ - b, fl + b - c are in A.P.

rFl
F
(iii) be-a ,ca-b ,ab-c are in A.P.
b + c c + a a +b

r
4. If are in A.P., prove that:
ou
-111
a
fo
ks
(i) - - are in A.P. (ii) be, ca, ab are in A.P.
a ' b ' c
oo

5. If a, b, c are in A.P., prove that:


(ii) + c^ + iac = 2{ab + bc + ca)
Y

(i) (fl-c)^=4(fl-b)(b-c)
B

(iii) + 6abc = 8b^


re

1 l') (i 1 1 1 I
- + - are in A.P., prove that a, b, c are in A.P.
ou

6. Iffl - + - ,b - + - / c
Y

a bJ
ad

b e c a

7. Show that x^ +xy + y^,:?+zx + x^ and +yz+ 2^ are consecutive terms of an A.P., if x, y
d

and z are in A.P.


in
Re

HINTS TO SELECTED PROBLEMS


F

a + c

5. (i) Put b=^^-^ on RHS (ii) Putb=^^^ on RHS (iii) Put b = —^


on RHS and LHS

6. a
1- + --l'\ ,bJl- + -I'l f, cc fl- + -l) are in
. A.P.
. p
b U a

1 (1 f1 1
a - + - - +l,b -
- + - - + - +1 are A.P. [●.● Adding 1 throughout]
b c a a b

a
a b,(l- + -1^ +-,
(1- + -O + -, b c
1
- + -
1
+ - are in A.P.
b c a yc a) b a b) c
1 1 l"! . Tl 1 1 \ f
111 -AD
=> a -+- +- ,b -+ - + - , c - + - + - are in A.P.
b e a a b a b c
8.40 APPLIED MATHEMATICS

1
a, b, careinA.P. Dividing each term by - + - + -
a b c

It is given that x, y, z are in A.P. Therefore, y-x^z-y=d (say)


Now, {x'^+zx +2^) -(x^+ xy + y^) = (-y^+z^) + x{z-y) ={z-y) (x + y + z) =d{x + y + z)
and, (z\yz + y^) -(zhzx + x^) ={y^-x'^)+z{y-x) =iy-x) (x + y + z)=d{x + y + z)
(x^ + zx + z^) - (x^ +xy + y^) = (z^ + yz + y^) - (z^ + zx + x^)
x^ + xy + y^, z^ +ZX + x^, y^ + yz +z^ are in A.P.
0.7 INSERTION OF ARITHMETIC MEANS

If between two given quantities a and b we have to insert n quantities A-y, A2, A„ such that
a, Ay, ^2,... Afj,bform an A.P., then we say that Ay, A2, A„ are arithmetic means between a and b.

w
1 i.i usi K ATION Since 15, 11, 7, 3, -1, -5 are in A.P., it follows that 11, 7, 3, -1 are four arithmetic
means between 15 and -5.

F lo
If a. A, b are in A.P., we say that A is the arithmetic mean of a and b.
8.7. i INSERTION OF ARITHMETIC MEANS

ee
Let Ay, A2,...,A„ be n arithmetic means between two quantities a and b. Then,fl, Ay, A2,...,

Fr
A„, b is an A.P. Let d be the common difference of this A.P. Clearly, it contains (n + 2) terms.
b ~a
b = {n + 2)* term =>b = a + (n + l) d => d = for
n +1
ur
b -a
Now, Ay = a + d= a +
n + 1
s
ook
Yo

2{b-a)
A2 = a +2d = a +
n + 1
eB

3{b-a)
A,o — a + 3d — a +
n+1
r
ou
ad

A„n = a + nd = a +
n{b - a)
n + 1
Y

These are the required arithmetic means between a and b.


Re
nd

8.7.2 INSERTION OF A SINGLE ARITHMETIC MEAN BETWEEN TWO NUMBERS

Let a and b be two numbers and A be the single arithmetic mean between them. Then,
Fi

a. A, b are in A.P.
A -a=b -A
a + b
2A=a+b =>A =
2
a +b
Thus, the arithmetic mean of a and b is
2

ILLUSTRATIVE EXAMPLES

I XAMPH' 1 Insert three arithmetic means between 3 and 19.


SOLUTION Let Ay, A2, A3 be 3 A.M.'s between 3 and 19. Then 3, Ai, A-,, An, 19 are in A P
19-3 i ^ j
whose common difference d is given hv d = =4
^ 3+1
Aj = 3 + d = 3 + 4 = 7, A2 = 3 + 2rf = 3 + 2x4=11, A3 =3 + 3rf =3+3x4 =15.
ARITHMETIC PROGRESSIONS 8.41

Hence, the required A.M/s are 7,11,15.


;i+ 1 «+ 1
a + b
1 XAMi’LE 2 for what value ofn, is the arithmetic mean of a and b ?
a^ + b
n

«+l «+l
a +b a +b
SOLUTION The A.M. of a and b is . Therefore, will be the A.M. of a and b, if
a'+b
n
2

n+ 1 «+ 1
a + b a +b
n , n 2
a+b

n+ 1 H+ 1
2 {a + b ) = (fl" + b”) {a + b)
n+ 1 «+ 1 «+ 1 n+ 1
2a + 2b a + a" b +b'’ a + b

w
n+ 1 n+ 1 n n
a +b = d‘b + b“ a

fl” {a-b) = b" {a -b)

Flo
=>

n ^ _\0
n n a a a a
a = 6“ => — = 1 =>

ee
= 1 => => n = 0.
b

Fr
I \ AMPLE 3 If n arithmetic means are inserted between 20 and 80 such that the ratio offirst mean to the
last mean is 1:3, then find the value of n.
for
ur
SOLUTION Let A-^, A2,, A„ be n arithmetic means between 20 and 80 and let d be the
common difference of the A.P. 20, Aj, A2,... A,,, 80. Then,
s
80 - 20 60 b -a
k
Yo

d = Using: d =
oo

n + \ n + \ n + 1
eB

60 n + 4
Now, Ai = 20 + d => Ai = 20 + = 20
n + 1 n + 1
r
ou
ad

eon An + 1
And, Afj ~ 20 + nd => Afj == 20 + = 20
n + 1 n + 1
Y

It is given that
Re
nd

20 {n + 4)
Fi

1 1 tt + 4 1
_

— => = — =>4jj + i = 3n + 12=>n = 11


An 3 20 (4n_+1) 3 4n +1 3

I-: ● l-LIi ● that the ratio of the 7 th and {m - l)th


Between 1 and 31 are inserted m arithmetic means so
means is 5:9. Find the value ofm.
SOLUTION Let A|, A2,..., A,„ be m arithmetic means between 1 and 31. Then 1,
Aj, A2,A,„ , 31 is an A.P. with common difference d given by
31 -1 _ 30 -a
d = Using: d =-
m+1 m+1 n + 1

7x30 m + 211
Now, Ay =1+ 7 d =1 + m+1 m+1

30 (m -1) 31 m - 29
and. Ajjj _ j — 1 + (ni —V)d — 1 + m + 1 m + 1
8.42 APPLIED MATHEMATICS-XI

It is given that
^7 5 m + 211
- => +1899 =155w-145 => 146m = 2044 =>m = 14
An-1 31m-29 9

rxAMPLE 5 Prove that the sum ofn arithmetic means between two nutnbers is n times the single AM.
between them.

SOLUTION Let/lj, A2, Af^hen arithmetic means between i? and b. Then,fl, Ai^, A2, A,j ,b
is an A.P. with common difference d given by d =-

n
Now, A-^ + A2 +... + A)i = - A-,+A II
2I ^

w
= ~{a + b) [v a, A-i, A2,..., A„,b is an A.P. .-. a + b-A-^ + A„]

fa +b'

F lo
^— = n X (A.M. between a and b)
-n

13

ee
EXAMPLE 6 The sum of tzuo numbers is —. An even number of arithmetic means are being inserted
6

Fr
betzveen them and their sum exceeds their number by 1. Find the number of means inserted.
13
SOLUTION Let a and b be two numbers such that a + b = —
6
for
ur
Let A-^, A2, A2„ be 2n arithmetic means between a and b. Then,
s
a +b
ook

+ A2 + ... + A2fi — 2n [Using result of Example 5]


Yo

2
eB

13
Ai + A2 +... + A2}, = n {a + b) = — n [Using (i)]
6
13
r

2h + 1 =—n [v /4;i + A2+...+A2,, =2>? + l (given)]


ad
ou

12n + 6=13
Y

n =6
Re
nd

EXAMPLE r If the A.M. betzveen pth and qth terms of an A.P. be equal to the A.M. between rth and sth
Fi

terms of the A.P., then shoiv that p + q = r + s.


SOLUTION Let a be tlie first term and d be the common difference of the given A.P. Then
Op = pth term = a + {p -1) d; a^ - qth term - a + {q -1) d
= ?'th term - a + {r -1) d and, = sth term = a + (s -1) fr
It is given that
A.M. between Up and a^j = A.M. between a,, and
^ {(Jp + V = ^
^p + (iq = ^
{a + {p-1) d\ + {a + (q-1) d} = (a + (r -1) frj + [a + (s -1) d]
{p + q-2) d = {r + s ~2) d
p + q = r + s
ARITHMETIC PROGRESSIONS 8.43

EXERCISE 8.6

Find the A.M. between:

(i) 7 and 13 (ii) 12 and -8 (iii) (x-y)and(x + y).


2. Insert 4 A.M.s between 4 and 19.
3. Insert 7 A.M.s between 2 and 17.
4. Insert six A.M.s between 15 and -13.
There are n A.M.s between 3 and 17. The ratio of the last mean to the first mean is 3:1. Find
the value of n.

Insert A.M.s between 7 and 71 in such a way that the 5*^ A.M. is 27. Find the number of
A.M.s.

w
If n A.M.s are inserted between two numbers, prove that the sum of the means equidistant
from the beginning and the end is constant.
If y, z are in A.P. and is the A.M. of x and y and Aj is the A.M. of y and z, then prove

o
that the A.M. of Aj and A2 is y.

e
Insert five numbers between 8 and 26 such that the resulting sequence is an A.P.

re
rFl ANSWERS

F
l.(i)10 (ii) 2 (iii) X 2. 7,10,13,16

or
31 23 61 19 91 53 121
ou
8 ' 4 ' 8 ' 2 ' 8 ' 4 ' 8
ksf
4. 11, 7,3,-1,-5,-9 3. 6 . 15 11,14,17,20,23
oo

HINTS TO SELECTED PROBLEMS


Y

9. Let 02, H' ^ natural numbers between 8 and 26 such that


B

8, Oi, 02, 03, 0^, 26is an A.P. Let frbe the common difference. Then,
re

26-8 - a
d = = 3
oYu

5 + 1 n + 1
ad

.'. ^2=8 + 3— ~ / ^4 ~ 20 and 0^ — 23


d

Hence, five numbers are 11,14,17,20 and 23.


in
Re

8.8 APPLICATIONS OF A.P.


F

In this section, we shall discuss some problems based upon the applications of arithmetic
progressions.
EX '● ■ The digits of a positive integer, having three digits, are in A.P. and their sum is 15. The
number obtained by reversing the digits is 594 less than the original number. Find the number.
SOLUTION Let the digits at ones, tens and hundreds place be (a-d), a and {a + d) respectively.
Then the number is

(fl + tf) X 100 + flxl0 + (fl-rf) = 111a + 99d


The number obtained by reversing the digits is
{a-d)xl00 + axl0 + {a + d) = 111a - 99d
It is given that {a - d) + a + {a + d) =15
and. lllfl - 99d =llla + 99d -594
8.44 APPLIED MATHEMATICS-XI

3a - 15 and 198d = 594 =e> /? = 5 and d = 3


So, the number isllla + 99d =111 x 5 + 99 x 3 = 852.
EXAMPLE :
Tzuo cars start together in the same direction from the same place. The first goes with
uniform speed of 10 km/h. The second goes at a speed of 8 km/h in the first hour and increases the speed by
1/2 km each succeeding hour. After hozv many hours will the second car overtake the first car if both cars
go non-stop ?
SOLUTION Suppose the second car overtakes the first car after t hours. Then the two cars travel
the same distance in t hours.

Distance travelled by the first car in t hours = 10 t km.

w
Distance travelled by the second car in t hours
= Sum of t terms of an A.P. with first term 8 and common difference 1 /2.
t 1 t{t + 31)
-j2x8 + (f-l)x- 4

o
e
re
When the second car overtakes the first car. The distance travelled by both cars IS same.

t {t + 31)

Frl
lOf = t(t-9) = 0^ t = 9

F
4
[●●● t ^ 0]

Thus, the second car will overtake the first car in 9 hours.
ou
r
EXAMPLE 3 A man repays a loan ofX 3250 by paying ? 20 in the first month and then increases the

so
payment by ? 15 every month. How long ivill it take him to clear the loan ?
kf
SOLUTION Suppose the loan is cleared in n months. Clearly, the amounts form an A.P. with
first term 20 and the common difference 15.
oo

Sum of the amounts = 3250


Y
B

-j2x20 + (?i-l)xl5^=3250
re
oY

3??^+5/1-1300 = 0 => (?i-20)(3« + 65)=0=> /i = 20


u

3/1 + 65 ^ 0
ad

Thus, the loan is cleared in 20 months.


d

EXERCISE 8.7
in
Re

A man saved ? 16500 in ten years. In each year after the first he saved ? 100 more than he
did in the receding year. How much did he save in the first year ?
F

A man saves ? 32 during the first year, ? 36 in the second year and in this way he increases
his savings by ? 4 every year. Find in what time his saving will be ? 200.
X
A man arranges to pay off a debt of ? 3600 by 40 annual instalments which form an

arithmetic series. When 30 of the instalments are paid, he dies leaving one-third of the debt
unpaid, find the value of the first instalment.
4. A manufacturer of radio sets produced 600 units in the third year and 700 units in the
seventh year. Assuming that the product increases uniformly by a fixed number every
year, find (i) the production in the first year (ii) the total product in 7 years and (iii) the
product in the 10th year.
n.
There are 25 trees at equal distances of 5 metres in a line with a well, the distance of the well
from the nearest tree being 10 metres. A gardener waters all the trees separately starting
from the well and he returns to the well after watering each tree to get water for the next.
Find the total distance the gardener will cover in order to water all the trees.
ARITHMETIC PROGRESSIONS 8.45

6. A man is employed to count ? 10710. He counts at the rate of ? 180 per minute for half an
hour. After this he coimts at the rate of ? 3 less every minute than the preceding minute.
Find the time taken by him to count the entire amount.
7. A piece of equipment cost a certain factory ^ 600,000. If it depreciates in value, 15% the first,
135% the next year, 12% the third year, and so on. What will be its value at the end of 10
years, all percentages applying to the original cost ?
8. A farmer buys a used tractor for ? 12000. He pays 16000 cash and agrees to pay the balance
in annual instalments of ? 500 plus 12% interest on the impaid amount. How much the
tractor cost him?

9. Shamshad Ali buys a scooter for ? 22000. He pays ? 4000 cash and agrees to pay the balance
in annual instalments of ? 1000 plus 10% interest on the unpaid amount. How much the
scooter will cost him.
10. The income of a person is ? 300,000 in the first year and he receives an increase of ? 10000 to
his income per year for the next 19 years. Find the total amount, he received in 20 years.

w
II. A man starts repaying a loan as first instalment of ? 100. If he increases the instalments by
? 5 every month, what amount he will pay in the 30th instalment?

F lo
- A carpenter was hired to build 192 window frames. The first day he made five frames and
each day thereafter he made two more frames than he made the day before. How many
days did it take him to finish the job?

ee
13. We know that the sum of the ulterior angles of a triangle is 180®. Show that the sums of the

Fr
interior angles of polygons with 3,4,5,6,... sides form an arithmetic progression. Find the
sum of the interior angles for a 21 sided polygon. for
14. In a potato race 20 potatoes are placed in a line at intervals of 4 meters with the first potato
ur
24 metres from the starting point. A contestant is required to bring the potatoes back to the
starting place one at a time. How far would he run in bringing back all the potatoes?
s
ook

15. A man accepts a position with an initial salary of f 5200 per month. It is understood that he
Yo

will receive an automatic increase of ?320 in the very next month and each montli
eB

thereafter,

(i) Find his salary for the tenth month,


(ii) Wliat is his total earnings during the first year?
r
ad
ou

16. A man saved ?66000 in 20 years. In each succeeding year after the first year he saved ? 200
more than what he saved in the previous year. How much did he save in the first year?
Y

17. In a cricket team tournament 16 teams participated. A sum of ? 8000 is to be awarded


among themselves as prize money. If the last place team is awarded ? 275 in prize money
Re
nd

and the award increases by the same amount for successive finishing places, how much
Fi

amount will the first place team receive?


ANSWERS

?1200 5 yrs 3. ? 51 4. (i) 550 (ii) 4375 (iii) 775


o. 3500 m 6. 89 minutes . ?105000 8, ? 16680

9. ? 39100 10. ?7900,000 H ?245 12. 12 days


13. 3420® 14. 2480 m 15. (i)? 8080 (ii)? 83520
?1400 17. ?725

HINTS TO SELECTED PROBLEMS

10. Here, a = 300,000, d =10,000 and » = 20. Let S be the total amount received in 20 years.
Then,
20
S = ?
^ (2 X 300,000 + (20 -1) X 10,000) = ? 10 (600,000 + 190,000) = ? 7900,000
8.46 APPLIED MATHEMATICS-Xl

1 Here, a = 100, d=5 and n - 30.


Amount to be paid in 30th instalment = ^30 = a h- 29rf = 100 + 29 x 5 = 245

:.1ULTIPLE CHOICE QUESTIONS (MCQs)


Mark the correct alternative in each of the following:
If 7th and 13th terms of an A.P. be 34 and 64 respectively, then its 18th term is
(a) 87 (b) 88 (c) 89 (d) 90
If the sum of p terms of an A.P. is q and the sum of q terms is p, then the sum oip + q terms
will be

(a) 0 (b) p-q (c) p+q (d) ~{p + q)


If the sum of n terms of an A.P. be 3 ii -nand its common difference is 6, then its first term
IS

(a) 2 (b) 3 (c) 1 (d) 4

w
4. Sum of all two digit numbers which when divided by 4 yield unity as remainder is

F lo
(a) 1200 (b) 1210 (c) 1250 (d) none of these.
D. In A.M.'s are introduced between 3 and 17 such that the ratio of the last mean to the first
mean is 3 :1, then the value of n is

ee
(a) 6 (b) 8 (c) 4 (d) none of these.

Fr
2
m a,m _
If ^ denotes the sum of first n terms of an A.P. < % > such that
for Y' then
n a>
2m + l 2 m
r
m-1 m +1
(a) (b) (c) (d)
You

2 71 + 1 2t7-1 77-1 77 + 1
s
ook

The first and last terms of an A.P. are 1 and 11. If the sum of its terms is 36, then the number
eB

of terms will be
(a) 5 (b) 6 (c) 7 (d) 8
If the sum of n terms of an A.P., is 3 77^ + 5 77 then which
our

of its terms is 164?


ad

(a) 26th (b) 27th (c) 28th (d) none of these.


9. If the sum of n terms of an A.P. is 2 77^ + 5 77, then its nth term is
dY
Re

(a) 4t7 - 3 (b) 3 77 - 4 (c) 4 77 + 3 (d) 3 77 + 4


Fin

If <7|, 172, ^3' ■■■● are in A.P. with common difference d, then the sum of the series sin d
[cosec i7| cosec 172 + cosec <72 cosec a-^ + .... + cosec _ 1 cosec ] is
(a) sec 77| - sec a, (b) cosec - cosec
(c) cot fl] - cot a, (d) tan 77j - tan a„
' In the arithmetic progression whose common difference is non-zero, the sum of first 3 n
terms is equal to the sum of next n terms. Then the ratio of the sum of the first 2 n terms to
the next 2 n terms is

(a) 1/5 (b) 2/3 (c) 3/4 (d) none of these


!2. If flp 172,773, —● ^re in A.P. with common difference d, then the sum of the series
sin d [ sec sec 02 + sec 172 sec 03 +.... + sec ^ sec a,^\ is
(a) sec i7| - sec a,, (b) cosec 17J - cosec %
(c) cot <7| - cot 17„ (d) tan Ofj - tan Aj
8.47
ARITHMETIC PROGRESSIONS

13. If four numbers in A.P. are such that their sum is 50 and the greatest number is 4 times the
least, then the numbers are
(a) 5,10,15, 20 (b) 4,10,16,22 (c) 3,7,11,15 (d) none of these
14. Ifnarithmetic means are inserted between 1 and 31 such that the ratio of the first mean and
nth mean is 3 : 29, then the value of n is
(a) 10 (b) 12 (c) 13 (d) 14
15. Let Sy, denote the sum of ti terms of an A.P. whose first term is a. If the common difference d
is given by d = S,i -k S„_i+ %_ 2 , then k =
(a) 1 (b) 2 (c) 3 (d) none of these
16. The first and last term of an A. P. are a and / respectively. If S is the sum of all the terms of the
then k =

w
A.P. and the common difference is given by
k-{l + a) '
(a) S (b) 2S (c) 3S (d) none of these

Flo
17. If the sum of first n even natural numbers is equal to k times the sum of first n odd natural
numbers, then k -

ee
1 n-1 n + 1 n + l
(a) - (b) (c) (d)

Fr
n n 2n n

IS. If the first, second and last term of an A.P are a, b and 2a respectively, then its sum is
ab ab 3 ab for
ur
(a) (b) (c) (d) none of these
2{b-a) b -a 2(b-a)

If, Sj is the sum of an arithmetic progression of 'n' odd number of terms and $2 the sum of
s
19.
k
Yo

Si _
the terms of the series in odd places, then
oo

S2
eB

2ji n n + l n + l
(a) (b) (d)
n + l n + l 2n n
r

^p, where denotes the sum of r terms of the A.P., then Sp


2
ou

If in an A.P., ^, = n p and S,„


ad

20. = m

is equal to
Y

(b) mn p (c) p^ (d) (m + n) p^


Re
nd

21. If in an A.P., the pth term is q and (p + q) term is zero, then the q^^ term is
Fi

(a) -p (b) p {c) p + q (d) p-q


22. The 10"’common term between the A.P.s3, 7,11,15,... and 1, 6,11,16,... is
(a) 191 (b) 193 (c) 211 (d) none of these
2
23. If in an A.P. S„=n q and = w where ^ denotes the sum of r terms of the A.P., then Sq
equals
3

(c) q^ (d) {!n^+n^)q


(a)V (b) mnq

24. Let denote the sum of first n terms of an A.P. If =3 then S3,,: is equal to
(a) 4 (b) 6 (c) 8 (d) 10
,5. If the sum of n terms of an A.P. is given by S,j = 3?j + 2n^, then the common difference of the
A.P. is

(a) 3 (b) 2 (c) 6 (d) 4


8.48
APPLIED MATHEMATICS-Xi

’● If 9 times the term of an A.P. is equal to 13 times the 13th term, then the 22"^ term of the
A.P. is
(a) 0 (b) 22 (c) 220 (d) 198

ANSWERS

1. (c) .1. (d) (a) (b) (a) 6. (b) 7, (b) s. (b)


‘ (c) ‘ ^ (c) (a) (d) (a) 14. (d) 15. (b) l(,. (b)
i: (d) .(c) :Ma) (c) (b) (a) 23. (c) i'. (b)
(d) . o. (a)

ow
FILL IN THE BLANKS TYPE QUESTIONS (FBQs)
1 The sum of the terms equidistant from the beginning and end in an A.P. is always same and
is equal to the sum of and terms.

l~x
The minimum value of 4^ + 4 , X & R, is

e
re
Fl
If the first, second and last terms of an A.P. are a, b and 2a respectively, then the sum of its
terms is

F
The number of terms in an A.P. whose first term is 10, last term is 50 and the sum of all
ur
terms is 300, is

r
The arithmetic mean of first n natural numbers is
fo
ks
The sum of first n odd natural numbers is
Yo
oo

The sum of first n even natural numbers is


If n is even, then the sum of first n terms of the series 1-2 + 3- 4 + 5- 6 +...,
eB

is

If twice the 11^^ term of an A.P is equal to 7 times of its 21®' terms, then the value of 25‘h term
ur

IS .
ad
Yo

If the sums of ti terms of two arithmetic progressions are the ratio{2« + 3): {6n + 5), then the
ratio of their 13'^ terms is
d

11. The sum of n arithmetic means between a and & is


Re
in

If the sum of n arithmetic means between 9 and 51 is 270, then the value of h
F

is.

The sum of 4 arithmetic means between 3 and 23 is


_/i+l n+l
a +b
If is the A.M. of a and b, then n =
n I j
a +b

ANSWERS

Sab « +l
1. First, Last 4 3. 4. 10
2{b-a) 2

n
6-
7. «(« + !) 8. - 9, 0 10. 53:155
2
n
\.-{a + b) 12. 9 1.'^. 52 14. 0
ARITHMETIC PROGRESSIONS 8.49

VERY SHORT ANSWER QUESTIONS (VSAQs)

Answer each of the following questions in one word or one sentence or as per exact requirement of the
question:
Write the common difference of an A.P. whose «th term is xn + y.
P 2
Write the common difference of an A.P. the sum of whose first n terms is — n + Qn.

If the sum of n terms of an AP is 2n + 3n, then write its «th term.


4.
If log 2 , log (2^ -1) and log (2’^ + 3) are in A.P., write the value of x.
If the sums of n terms of two arithmetic progressions are in the ratio 2n + 5: 3« + 4, then
write the ratio of their mth terms.

low
b. Write the sum of first n odd natural numbers.
/.
Write the sum of first n even natural numbers.
8. Write the value of«for whichnth terms of the A.P.s 3,10,17,... and 63,65,67,.... are equal.
If 3+5 + 7 +... + upto n terms
= 7, then find the value of ;7.

ee
5 + 8 +11 +.... upto 10 terms
rF
Fr
10. If ?7?th term of an A.P. is n and nth term is m, then write its pth term.
11,
If the sums of n terms of two A.P.'s are in the ratio {3n + 2): {In + 3), find the ratio of their

r
12* terms. fo
u
ANSWERS
ks
Yo

p 3. 471 + 1 4. logjS
oo

■ X

77^
B

{4m + 3): (6777 + 1) n(77 + l)


re

8. 13 35 10.
777 + 77 - p 11- 71 :49
u
ad
Yo
nd
Re
Fi
CHAPTER 9

GEOMETRIC PROGRESSIONS

9.1 GEOMETRIC PROGRESSION

A sequence of non-zero numbers is called a geometric progression (abbreviated as G.P.) if the ratio of a
term and the term preceding to it is always a constant quantity.
The constant ratio is called the common ratio of the G.P.

w
1
In other words, a sequence, a-^, a2, a^ ,a,, ,... is called a geometric progression if = constant for
a
n

F lo
all n e N.

ILLUSTRATION 1 The sequence 4,12, 36,108,... is a G.P., because — = — = — ... = 3, which is


^ 4 12 36

e
constant.

Fre
Clearly, this sequence is a G.P. with first term 4 and common ratio 3.
1 3 9 1
ILLUSTRATION 2 The sequence
for
- ,...is a G.P. with first term — and common ratio
2'4' 8 ^ 3
f 1 1 3
r
equal to -— 2
You
oks

ILLUSTRATION 3 Show that the sequence given by a,, = 3 {2\for all n eN, is a G.P. Also, find its
eBo

common ratio.

SOLUTION We have, «„ = 3 (2")


n+ 1
a„+l = 3(2
our
ad

H + 1
+ 1 3 (2 )
So, = 2, which is constant for all n e N.
an
3 (2")
dY
Re

So, the given sequence is a G.P. with common ratio 2.


GEOMETRIC SERIES If ,^2 / ^ ^ / ●●● is aG.P., then the expression + <72 +a^ +... +a,j + ... is
Fin

called a geometric series.


Note that the geometric series is finite or infinite according as the corresponding G.P. consists of
finite or infinite number of terms.

9.2 GENERAL TERM OF A G.P.


«-1
THEOREM Prove that the nth term of a G.P. with first term a and common ratio r is given by = ar
p!-H ‘F Let fl|, <?2/ ^3/ ●●● be the given G.P. Then,
1- 1
ai=a => a-i=ar

Since a^, <72, ^3,..., ... is a G.P. with common ratio r. Therefore,
«2 2-1
=r ^ ^2 = ^ <72 = <7r => <?2 = rtr

2 3-1
=> <73 = fl2^ => a^=(ar)r => a2=ar => a^=ar
<?2
9.2 APPLIED MATHEMATICS-XI

£i_ 2 3 4-1
r => n^=a^r => a^={ar )r => a^=ar => a^=ar
«3
»-l
Continuing in this manner, we get = ar Q.E.D.
It folloxvs from the above discussion that if a is the first term and r is the cornmon ratio of a G.P.,
'y H - 1 2 3 4 « -1
then the G.P. can be written as a, ar, ar ,. ar or,a,ar,ar ,ar ,ar ,...,ar ,... according as
it is finite or infinite.
9.2.1 rrth TERM FROM THE END OF A FINITE G.P.
m - n
THEOREM 1 Prove that the nth term from the end of afinite G.P. consisting ofm terms is ar , where
a is the first term and r is the common ratio of the G.P.

ow
PROOF Since the G.P. consists of m terms.
m - n
«th term from the end = {m - « + 1) th term from the beginning = ar
THEOREM 2 Prove that the nth term from the end of a G.P. ivith last term I and common ratio r is
\n -1

e
given by
h if 1
I -

re
rFl
F
I KOOl' Clearly, when we look at the terms of a G.P. from the last term and move towards the
beginning we find that the progression is a G.P. with common ratio 1/r.
(\ yi-1

r
ou
So, ;2th term from the end = I
\n fo
ks
ILLUSTRATIVE EXAMPLES
oo

Tj//h- I FINDING THE INDICATED TERM OF A G.P. WHEN ITS FIRST TERM AND THE COMMON RATIO
Y

ARE GIVEN
B

EXAMPLE 1 Find the 9th term and the general term of


the progression: ^,--^,1,-2,...
re

SOLUTION The given progression is clearly a G.P. with first term rt =1/4 and common ratio
ou

r=-2.
Y
ad

(9-1) 1 8
9th term = flg =ar
= ^(-2)
= ar = 64
d

and. General term = a.^n = ar (»-!)_ 1


in
Re

EX.AMPLE: Find the 5th term of the progression


F

1 f 3-2V2'| (5V2-7
' 2^3 ' 12 '[ 24^3 J'
SOLUTION Clearly, the given progression is a G.P. with first term a=l and common ratio
v'2-l
. So, its 5th term is given by
2yfS
r r-
(5-1) = 1 X V2-1 (V2-1)"
«5
V
2^3 /
144

EXAMPLE ' Find 4th term from the end of the G.P. 3, 6,12,24,..., 3072.
SOLUTION Clearly, the given progression is a G.P. with common ratio r =2.
,4-1 \4-l
1
4th term from the end = I = (3072) i = 384
GEOMETRIC PROGRESSIONS 9.3

Tupc II ON FINDING THE POSITION OF A GIVEN TERM IN A GIVEN G.P.


EXAMPLE 4
Which term of the G.P. 2,1, ^ ^4 ' ic J_ 7
...IS
128

SOLUTION Clearly, the given progression is a G.P. with first term a =2 and common ratio
1
r =1/2. Let the nth term be . Then,
128

1 Y' - 2 if =>
N«-l

an
1
=> ar
H - 1 1
=> 2
(1 1
,1-2=7 => n =9
128 128 2 128 UJ 12J
1
Thus, 9th term of the given G.P. is
128

w
EXAMPLE 5 Which term of the G.P. 5,10, 20, 40,... is 5120 ?
SOLUTION Clearly, the given G.P. has first term a =5 and the common ratio r =2. Let the nth
term be 5120. Then,

Flo
a„ =5120
. . H -1 n- 1 II- 1 /I - 1 10

e
^ ar = 5120 => 5(2 ) =5120 => 2 = 1024 => 2 = 2 => ?i-l =10 => n=ll

re
Thus, 11th term of the given G.P. is 5120.

rF
. \MPLE ' Which term of the G.P. 2,8,32,... is 131072 ?
ur
SOLUTION Here, <7 = 2 and r =4. Let the nth term be 131072. Then, fo
<7„II =131072
ks
H - 1 n - 1 H - 1 « -1 8
=> <7r = 131072 =>2x4 = 131072 => 4 = 65536 => 4 =4 => n-1 =8 => n = 9
Yo
oo

Hence, 131072 is the 9th term of the given G.P.


B

-1

Tifpic III PROBLEMS BASED ON THE DEFINITION OF A G.P. AND THE FORMULA ^
re

EXAMPI r “ The fourth, seventh and the last term of a G.P. are 10, 80 and 2560 respectively. Find the
first term and the number of terms in the G.P.
u
ad

SOLUTION Let a be the first term and r be the common ratio of the given G.P. Then,
Yo

6
80
<74 = 10, <17 =80 => <7r ^ = 10 and ar^ = 80 => => r ^ = 8 => r = 2.
10
d

ar
Re
in

Putting r = 2 in <7r = 10, we get:


F

a(2)^ =10=><7 = 8 ”4'


Let there be n terms in the given G.P. Then,
<7., =2560 => <7r”‘^=2560
II

H-1 n~4 h-4


) = 2560 => 2 = 256 => 2 = 2° => n-4 = 8 => n=12.

EX.*MTLE The first term of a G.P. is 1. The sum of the third and fifth terms is 90. Find the common
ratio of the G.P.
SOLUTION Let r be the common ratio of the G.P. It is given that the first term <7=1.
Now, <73 + <^ = 90
ar ^ + ar ^ =90
r^ +r^ =90
9.4 APPLIED MATHEMATICS-XI

- 90 = 0

+10 -9 - 90 = 0 => (r^ +10) (r^ - 9) = 0 ^ -9 =0 => r = ± 3.

Hence, the common ratio of the given G.P. is 3 or - 3.


●●\UM 1 -
If the 4th and 9th terms of a G.P. be 54 and 13122 respectively, ifnd the G.P.
SOLUTION Let a be the first term and r the common ratio of the given G.P. Then,
^4 =54 and flg =13122
ar ^ =54 and rtr® =13122
8
ar 13122
=> r^ = 243 => = 3^ => r = 3
ar 3 54

w
Putting r = 3 in ^ = 54, we get: fl(3)^=54 => a = 2
2 3

F lo
Thus, the given G.P. is ii, rtr, rtr ,ar, ... i.e. 2,6,18,54,...
LXAMrLCm
Find a G.P. for ivhich the sum offirst two terms is-4 and the fifth term is 4 times the third

ee
term.

Fr
SOLUTION Let a be the first term and r be the common
ratio of the given G.P. It is given that
The sum of first two terms= -4. => a-y + a2 = - 4 => a + ar = -4 ...(i)
It is also given that
for
ur
^5 = 4 => ar^ =4 ar^ => r 2=4 => r=±2
s

Putting r = 2 and - 2 respectively in (i), we get a and a = 4 respectively.


ook

=
Yo

4
Thus, the required G.P. ' 8 _16 .. or 4,-8,16,-32,...
eB

3 ' 3 '■
[ XAMPLE 11 The third term of a G.P. is 4. Find the product of its first five terms.
r

SOLUTION Let a be the first term and r the common ratio. Then,
ad
ou

2
^3 =4 => <?r =4 ...(i)
Y

Product of first five terms = <72 ^3 % ^ ^


Re

= {ar^f=4^
nd

[Using (i)]
Fi

EXA.MI’LE 12
If the pth, qth and rth terms of a G.P. are a, b, c respectively, prove that:
<7 [j(r -p) fp-q)
SOLUTION Let A be the first term and R be the common ratio of the given G.P. Then,
<7
= pt/7 term b = qth term = ~ and c =rf/i term =
Substituting the values of <7, b and c, we get
a Iq-r) _f^(r-p) _^ip-q)
r 1(^-0 (r-p) (p-4)
('7-I)
.MR

^ ^(<?-r + r-p+p-<7) ^(p-l)(i^-r)+(</~l){r-p)+(r-l)(p-<j)


9.5
GEOMETRIC PROGRESSIONS

R p{q-r)+ q{r -p) + r(p-q)-{q-r)-(r -p)-{p-q)


0
=

EXAMPLE 13 If a, b, c are respectively the and terms of a G.P., show that


(q-r) log fl + {r - p) log + (p - q) log c = 0.
SOLUTION Let A be the first term and R the common ratio of the given G.P. Then,
a - pth term =5- = AR^^~^ => log rt = log A+(p-1) log R ...(i)

lj=ijthterm => b=AR‘^ ^ => log 1? = log A + ((/-1) log R ...(ii)

c = rth term => c= AR^ ^ => log c = log A + (r -1) log R ...(iii)

Substituting the values of log a, log b and log c, we get

w
(g-r) log fl + (r-p) log b + (p- fl) log c
= {q~r) ● log A + (p -1) log R ■ + (r - p) - log A + (tjr -1) log R

o
e
+ {p - q) l\o^ A + {r -1) log R ●

re
Frl
log A-(t/-r) + (r-p) + (p-i?) ●+log R ● (p-1) (g-r) + (^/-1) -R) +(^“1)(P 0 ’

F
ou
(log A) 0 + \ p (q -r) + q{r - p) + r ip - q) -{q -r) -{r -p) -ip - q)^ log R

r
= (log A) 0 + (log R) 0 = 0.
so
kf
EXAMPLE 14 Find fournumbers forming a geometric progression in zvhich the third term is greater than
oo
the first terms by 9, and second term is greater than the 4th by 18.
■y 2
Y

SOLUTION Let the four numbers in G.P. be a, ar, ar and ar . It is given that
B

2 3
ar = a + 9 and ar=ar +18
re

=> fl(r^-l)=9 and flr(l-r^)=18


oY
u

flr(l-r^) 18 -r=2 => r = -2


ad

9
«(r -1)
d

Putting r = - 2 in fl (r^ -1) = 9, we get


in

fl(4-l)=9 => a = 3
Re

Hence, the numbers are: 3, 3 (-2), 3 (-2)^, 3 (-2)^ or, 3, - 6,12, - 24.
F

EX.A.MPLE 15 The number of bacteria in a certain culture doubles every hour. If there were 30
bacteria present in the culture originally, how many bacteria zvill be present at the end oflnd hour, 4th
hour and nth hour?
SOLUTION Clearly, munber of bacteria at the end of different hours forms a G.P. with first term
fl = 30 and common ratio r -2.
Number of bacteria present at the end of 2nd hour
= Third term of the G.P. with first term <? = 30 and common ratio r = 2
ar ^ = 30x2^=120
Number of bacteria present at the end of 4th hour
= 5th term of the G.P. with first term fl = 30 and common ratio r = 2
ar ^ =30x 2^ =480
Number of bacteria present at the end of nth hour
9.6 APPLIED MATHEMATICS-XI

th
= (« +1) term of the G.P. with first term a = 30 and common ratio r = 2

= ar" = 30 X 2
n

I XAMPLE 16 Wliat will X 500 amounts to in 10 yrars after its deposit in a bank which pays annual
interest rate of 10% compounded annually?
SOLUTION We have,
P = Principal = X 500, R = Rate of interest = 10%
PR R
Amount at the end of one year = X P + = XP 1 +
100 100

R R
Amount at the end of second year = X ■ P 1 + + P 1 +
100 100 j 100

= XP 1 +
R ^ R R
1 + = X P 1 +

w
100 / \ 100 100

n2
R (1

F lo
R R
Amount at the end of the third year = ? J P 1 + 100
— +P +
100 J 100

ee
R
= ?P 1 +

Fr
100

and so on. for


Cearly, amounts at the end of various year form a G.P. with first term and common ratio
ur
R
1 +
100
s
ook
Yo

th
Amount at the end of 10th year = 11 term of the G.P.
eB

ao
R
= X P 1 +
100
our
ad

10 10
= X500 1 +
10 '1 11 10
= ?500x = ? 500 X (1.1)
100 10
Y

LX.XMPLE 17
A manufacturer reckons that the value of a machine, which costs him X 15625, will
Re
nd

depreciate each year by 20%. Find the estimated value at the end of 5 years.
Fi

SOLUTION We have,
Initial value of the machine = Vq = ^ 15625 and, R = Rate of depreciation = 20%
Kn
0 R R
Depreciated value at the end of first year = ,V0 “ = Vo 1-
100 100

Vi1 R ^2
R R
Depreciated value at the end of second year = V^- 100
= Vl 1-
100
= Vn0 1 -
100
ans so on.

Clearly, depreciated values at the end of different years form a G.P. with first term Vn and
common ratio
^ 1
R ^
100

Depriciated value at the end of 5 years


GEOMETRIC PROGRESSIONS 9.7

R
= 6th term of the G.P. with first term Vq ( = ? 15625) and common ratio r = 1 - 100

= Vo 1-
K f = ?\15625 1 -
20
= ^■1 15625 X
r4fl = ? 5120
100 100

L'XAMPLE 1 If the first and the nth terms of a G.P. are a and h respectively and ifP is the product of the
first n terms, prove that P^ ~(ab)'^.
SOLUTION Let r be the common ratio of the given G.P. Then,
1

V-1

ow
H-i b
b = nth term = ar => r r =
a a
V /<

Now,
P = Product of the first n terms

e
2 H - 1
P = a .ar .ar ... ar

Fl
re
p ^ 2+ 3+ ... + (h- 1)

F
H(H-l)
ur
tt-1 V

or
P = fl” r 2 V 1 + 2 + 3 + ..
f n 2
ks
Yo
1 2
lW2
(b\n-l
oo

H n b n/2
P = fl'N ■
a = a
a a
B

\ j
re

p2 =\{abf‘'^ ={ab)
n
u
ad
Yo

EXAMPLE 19 In a finite G.P. the product of the terms equidistant from the beginning and the end is
alzvays same and equal to the product offirst and last term.
d
Re
in

SOLUTION Let ,02 ,a^ ,... a„ be a finite G.P. with common ratio r.
Now,
F

k-1
aj^ = /cth term from the beginning = air
xk-1
1
and. ^-k+1 = ^:th term from the end - an , where 1 <k <n

fc-1
^k ^-k +I ^ a
n = Ojj for all k satisfying 1 < cf < n.
r
\ ● /

Hence, the product of terms equidistant from the begirming and the end is always equal to
the product of first and last term.
EXERCISE 9.1

1. Show that each one of the following progressions is a G.P. Also, find the common ratio in
each case:
(i) 4,-2,l,-l/2,... (ii) -2/3,-6,-54,...
9.8 APPLIED MATHEMATICS-XI

..... 3a^ 9a^


(m) — (iv) 1/2,1/3,2/9,4/27,...
4 16
2
2. Show that the sequence defined by = — ,» e N is a G.P.
3"
3. Find:

(i) the ninth term of the G.P. 1,4,16, 64, ...


3 1 1 2
(ii) the 10th term of the G.P. — , , , ,...
4 2 3 9
(iii) the 8th term of the G.P. 0.3,0.06,0.012,...
5 5
(iv) the 12th term of the G.P. 3 3
' ax,a X ,...
a X

1 1
(v) nih term of the G.P. ^f3,
^/3 ' 3V3'"'
1 1

w
(vi) the 10th term of the G.P. ^J2,
^/2 ' 2V2' ■■■
4. Find the 4th term from the end of the G.P. —

5. Which term of the progression 0.004, 0.02,0.1,... is 12.5 ?


F lo

2 2

27 ' 9 ' 3 ' ■


2
..,162.

e
Fre
6, Which term of the G.P.:

(i) V2, J 1 l_ . 1
for
42'242'4:41' 512^2 ■
(ii) 2, 2^2,4,... is 128?
r
(iii) 4^, 3, 3^3,... is729?
You

,.,1 1 1
oks

(iv) — , - , — ...is
3 9 27 19683
eBo

512
Which term of the progression 18, -12, 8,... is ?
1
ad
our

8. Find the 4th term from the end of the G.P. - ,...,
2 6 18 54 4374

9. The fourth term of a G.P. is 27 and the 7th term is 729, find the G.P.

10. The seventh term of a G.P. is 8 times the fourth term and 5th term is 48. Find the G.P.
Re
dY

11- If the G.P.'s 5,10,20,... and 1280,640,320,... have their nth terms equal, find the value of n.
Fin

12. If 5*, 8* and 11^^ terms of a G.P. are p, q and s respectively, prove that q^ = ps.
th
13. The4thtermof a G.P. is square of its second term, and the first term is - 3. Findits7‘“ term.
14. In a GP the 3'^‘^ term is 24 and the 6^ term is 192. Find the 10*^ term.
15. If fl, b, c, d and pare different real numbers such that:
(fl^ +b'^ + c^) p^ -2 {ab +bc + cd) p + {b^ + + d^) < 0, then show that a, b, c and d are
in G.P.

16. If a + bx _b + cx c + dx
(.T 5* 0), then show that <7, b, c and d are in G.P.
a - bx b - cx c - dx

ANSWERS

1(2]
Mi)-I
,..., 3a V 2
(ii) 9 (lu) — (iv) - 3. (i) 4' (ii) - -
2[3
9.9
GEOMETRIC PROGRESSIONS

VJ- 1
1 1
(V) ^ \
41
(iii) (0.3) (0.2)^ (iv) (fl-r) (Vi)

th th th th 1
4. 6 j. 6 6. (i) ir‘ (ii) 13‘“ (iii) 12“‘ (iv) 9 9 8.
162

9. 1,3,9,... . 3, 6,12,.. 5 .1. -2187 3072

HINTS TO SELECTED PROBLEMS

6, (ii) Let term of the G.P. 2, 2 -J2, 4,... be 128. Then,


11+ 1
« +1
2(V2) = 128 => 2 2 =2^ => = 7=>?i=13
2

Thus, 13th term of the G.P. 2,2 -v/7, 4,... is 128.


th
(iii) LetJi term of the G.P. 3, 3-^3,... be 729. Then,

w
n

V3x(V3)”“^=729 => 32 =3^ => -2 = 6=>ti = 12

F lo
th
Hence, 12 term of the given G.P. is 729.
Il l ,

ee
(iv) Let 7ith term of the G.P. — , — , —,... be —— . Then,
19683 3 9 27

Fr
yi-1 1
11 => 77 ^ = 3^ => 77 = 9
3U/ 19683 for
ur
12. Let a be the first term and r the common ratio of the given G.P. It is given that
p =5**^ term, q term, s =11^^ term
s
ok

4 7 10
Yo

p=ar , q=ar , s-ar


o

2 2 14 2 14
eB

q =a r and ps = a r
2
q =ps.

Let the common ratio of the given G.P. be r. It is given that the fourth term is square of its
r

15.
ad
ou

second term.

{-3)r^ = (-3r)^ => -3r^ = 9r^ => r=-3


Y

Hence, 7’*^ term =(-3)r^ = - 3 (-3)^ = - 2187.


Re
nd

' Let the first term and common ratio of the given G.P. be <7 and r respectively.
Fi

It is given that 3'^^ term = 24 and 6^^ term = 192


ar ^ = 24 and ar^ = 192
ar 5 192
2 24
ar

= 8 => r = 2

Puttingr =2 in = 24, we get t? - 6.


=<77- ^=6x2^= 3072
th
10“‘ term

It is given that
(a^ + &^ + c^) p^ -2{ah + bc + cd) p + (b^ + + d^) <0
(a p -2abp + b^) + ib^ p^ -2 hep + c^) + {c^p^ -2cdp + d^)
/ 2 2
=> <0
9.10 APPLIED MATHEMATICS-XI

(ap-b)^ + {bp-c)^ +(cp-d)'^ <0


=> (cip-h) +(bp-c) +{cp-d)^ =0 ['.■ (rtp -c)^+{cp-tf)^ cannot be negative]
=?● ap~b=0, bp-c = 0, cp-d = 0
b c d
- = p
a b c

a, b, c, d are in G.P. with common ratio p.


16. We have.
a + bx _b + cx c + dx
a - bx b - cx c - dx

ow
Now,
a + bx b + cx
a - bx b - cx

{a + bx) + (a - bx) _{b + c.y) +{b - c.y)

e
[Applying componendo-dividendo]

re
{a + bx) - {a- bx) (b + cx) -{b - cx)
a _ b

rFl
F
bx cx a b
, & + cx c + dx c d

r
birmlarly, =
ou
b - c.r c ~ dx b c
fo
ks
-=-=—=> n,b, c, d are in G.P.
a b c
oo

9.3 SELECTION OF TERMS IN G.P.


Y
B

Sometimes it is required to select a finite number of terms in G.P. It is always convenient if we


select the terms in the following manner:
re

No. of terms Terms Common ratio


ou

a
Y
ad

3 - ,a,ar r
r

a a 3 2
4 , ar, ar
d

r
?'r
in
Re

a a 2
5 -, a, ar, ar r
2 '
r r
F

If the product of the numbers is not given, then the numbers are taken as a, ar, ar^, ar^,...
The following examples illustrate the application of the above selections.
ILLUSTRATIVE EXAMPLES

EXAMPLE 1 If the sum of three numbers in G.P. is 38 and their product is 1728, find them.

SOLUTION Let the numbers be ^ , a, ar. It is given that the product and sum of these numbers
are 38 and 1728 respectively.
Now, Product = 1728 => - (a) (ar) = 1728 => = 1728 a =12
r

and. Sum = 38
=> - + a + ar = 38
r
GEOMETRIC PROGRESSIONS 9.11

1
a - +1 + r = 38
r

1 + r +
12 = 38
r

b + 6r + 6r^ =19r => 6r^ -13r + 6 =0=> (3r-2) (2r - 3) = 0 => r = 3/2 or, r=2/3
a
Putting the values of and rin-,a, <?r, wefind that the required numbers are 8,12,18 or 18,12,8.
r

EXAMPLE 2 If the continued product of three numbers in G.P. is 216 and the sum of their products in

ow
pairs is 156, find the numbers.
SOLUTION Let the three numbers be a/r, a, ar. Then,
Product = 216 => {a/r). (i?). (or) = 216 =>i7^=6^=>/i = 6.

Sum of the products in pairs = 156

e
a a

re
- . a + a. ar + - . ar = 156

Frl
r r

2fl-

F
a + r + 1 = 156

2
ou
or
1 + r +r
36 = 156
r
kfs
1
3(r^+r + l) = 13r=> 3r^-10r+3 = 0 => (3r-l)(r-3) = 0=>r=-^ or, r= 3
oo

Putting the values of a and r, the required numbers are 18, 6, 2 or 2, 6,18.
Y
B

EXAMPLE 3 Three numbers are in G.P. ivhosesum is 70. If the extremes be each muUiplied by 4 and the
means by 5, they will be in A.P. Find the tmmbers.
re

2
SOLUTION Let the numbers be a, ar, ar . It is given that the sum of these numbers is 70.
oYu

a{l+r+r^) =70
ad

2
It is also given that 4a, 5ar, 4ar are in A.P.
d

2 (5ffr) = 4a + 4ar~
in
Re

5r = 2 + 2r^ => 2r^-5r + 2 = 0 => (2r-l)(r-2}=0 => r=2 or, r=l/2


F

Putting r = 2 in (i), we obtain a = 10. So, the numbers are 10,20,40


Putting r =1/2 in (i), we get a = 40. So, the numbers are 40, 20,10.
EXAMPLE 4 Find three numbers in G.P. lohose sum is 52 and the sum of whose products in pairs is 624.
2
SOLUTION Let the required numbers be a, ar, ar . Then,
Sum = 52^ a + ar + ar^ = 52^ a {1 + r + r^) =52 ...(i)

Sum of the products in pairs = 624


2
a. ar + ar. ar + a .ar ^ = 624
n^r{l+r + r^) = 624 ...(h)

Dividing (ii) by (i), we get


flr=12=> rt = — ....(hi)
r
9.12 APPLIED MATHEMATICS-XI

12
Putting a= — in (i), we get
r

— (l+r + r^)=52 =>


r
-lOr + 3 =0 => (3r-1) (r - 3) =0 => r =1/3 or, r = 3

Putting r = 3 in (iii), we obtain a = 4. So, the numbers are 4,12, 36.


1
Putting r = — in (iii), we get a = 36. So, the numbers are 36,12,4.

EXAMPLES The product ofifrst three terms of a G.P. is 1000. If 6 is added to its second term and 7 added
to its third term, the terms become in A.P. Find the G.P.

SOLUTION Let first three terms of the given G.P. he -r , a, ar. Then,
Product = 1000 ^ =1000 ^ rt =10.
a

w
It is given that - , a + 6, ar + 7 are in A.P.
r

a
2 (fl + 6) = - + ar + 7

F lo
r

10
32 = + lOr + 7

ee
r

10

Fr
25 = + 10r
r

5=- + 2r => 2r^-5r + 2 = 0 => (2r-l) (r-2) = 0 => r = 2,1/2 for


r
ur
Hence, the G.P. is 5,10,20,... or 20,10,5,...
EXAMPLE 6 The sum of three numbers in G.P. is 56. If we subtract 1, 7,21 from these numbers in that
s
ok

order, we obtain an arithmetic progression. Find the numbers.


Yo

2
o

SOLUTION Let the numbers in G.P. be a, ar, ar . It is given that the sum of these numbers is 56.
eB

a + ar + ar ^ =56
It is also given that
r
ad
ou

2
a -l,ar-7 and ar -21 are in A.P.

2 (ar - 7) = (n -1) + (ar^ - 21) => 2ar = ^ - 8 => a + ar^ = 2ar + 8


Y

a + ar ...(h)
Re

From (i), we obtain


nd

2
a + ar =56-ar ...(hi)
Fi

2
Substituting a + ar = 56-ar on the LHS of (ii), we get
16
2ar + 8 = 56 ar => 3ar =48 => ar =16 => r =
a

16
Putting r = in (i), we get
a

256
+16 + = 56
a

a^ +16fl+ 256=56rt => a^ -40a+ 256 = 0 => (^ - 32) (r? - 8) =0 => a = 8, 32


16
Putting fl = 8, in r = we get: r - ^ =2
a 8
16 16 1
Putting fl - 32, in r = we get: r = —
fl 32 2
GEOMETRIC PROGRESSIONS 9.13

When a 8 and r = 2, we obtain 8,16 and 32 as the numbers in G.P.


When a = 32 and r = we obtain 32,16, 8 as the numbers in G.P.
2

Hence, the numbers, in order, are 8,16 and 32 or 32,16 and 8.


EXAMPLE 7 Find three numbers in G.P. zohose sum is 13 and the sum of whose squares is 91.
2
SOLUTION Let the numbers be a, ar, ar . Then,

Sum = 13 => a + ar + ar'^ = 13 => n (1 + r + r^) =13 ...(i)

Sum of the squares = 91

ow
a
2
+ a
2
r
2
+ a
2
r
4
= 91 => rt^(l + r^+r^) = 91 ●●●(ii)

Now, a{l+r + r^) =13


/ (l+r+r^)' = 169 [From (i)]

e
Fl
re
a^ {1+r^ + r^) + 2a^ r(l + r + r^) = 169

F
91 + 2flr ■ i7(l + r + r^) -=169
ur
or
sf
91 + 2ar x 13 = 169 [Using (ii)]
[Using (i)l
k
ar = 3
Yo
oo

3
a ...(iii)
r
B

3 ●
Puttings - — in (i), we get
re

-(l+r + r^)=13
u
ad
Yo

o 1
- + 3+ 3r = 13=> 3r -lOr + 3=0 => (3r-l)(r- 3) = 0 => r = 3 or, r=-
V
r i
d
Re

Puttingr = 3 in (iii), we get a=\. So, the numbers are 1,3,9.


in

Putting r - — in (iii), we get a =9. So, the nmbers are 9,3,1.


F

Hence, the numbers are 1, 3, 9 or 9, 3,1.


EXERCISE 9.2

1. Find three numbers in G.P. whose sum is 65 and whose product is 3375.
2. Find three numbers in G.P. whose sum is 38 and their product is 1728.
3. Tlae sum of first three terms of a G.P. is 13/12 and their product is - 1. Find the G.P.
4. The product of three numbers in G.P. is 125 and the sum of their products taken in pairs is
87V^. Find them.
39
5. The sum of first three terms of a G.P. is — and their product is 1. Find the common ratio and
the terms.
9.14 APPLIED MATHEMATICS-XI

6. The sum of three numbers in G.P. is 14. If the first two terms are each increased by 1 and the
third term decreased by 1, the resulting numbers are in A.P. Find the numbers.
7. The product of three numbers in G.P. is 216. If 2, 8, 6 be added to them, the results are in
A.P. Find the numbers.

8. Find three numbers in G.P. whose product is 729 and the sum of their products in pairs is
819.

9. The sum of three numbers in G.P. is 21 and the sum of their squares is 189. Find the
numbers.

ANSWERS

ow
1. 45,15,5 or 5,15,45 2. 8,12,18

3.^ or
4. 10,5,-
, — or —, 5,10
3 ' 4 3 2 2

5.^,1,' 6. 2,4,8 or 8,4, 2 7. 18,6, 2 or 2,6,18

e
5 2

re
8. 1,9,81 or 81,9,

Fl 9. 3,6,12

F
HINTS TO SELECTED PROBLEMS
ur
r
a
3. Let the terms of the G.P. be -r , <7, nr. It is given that fo
ks
a 13 a
~ + a + ar = — and - xaxar = -l
Yo
r 12 r
oo

2
r +r + 1 13
andrt^ = -l
eB

a
r 12

13 r
ur

2
a =
-1 and a{r + r + 1) =
12
ad
Yo

2 13 r
r +r + 1 =
12
d
Re

12 + 25 r + 12 = 0
in

4 3
12r^ + 16r + 9r+ 12 = 0 => (3r + 4)(4r + 3)=0
F

=> r= — or —
3 4

Hence, three numbers are —, -1, — or, —


4 3 3 4

5. Let the terms of the G.P. be -r , <7, ar. It is given that


a 39 a
- + a + ar = — and — x ax ar =1
r 10 r

2
r +r + l 39
a and 77^ = 1
r 10

39r
a
= 1 and 7? (r^ + r + 1) = 10
GEOMETRIC PROGRESSIONS 9.15

10(r^+r + l) = 39r
10r^-29r + 10-0
=> (2r-5)(5r-2)=0
5 2
r = — or r = -
2 5
2 5
Hence, the numbers are —, 1, — or —
5 2 2 5

9.4 SUM OF THE TERMS OF A G.P.

w
THEOREM Prove that the sum ofn terms of a G.P. with first term ‘a' and common ratio ‘r’is given by
n n
r -1 1 -r
Sn a or. Sn a
r-1 1 -r

e
o
PROOF Let^, denote the sum of«terms of the G.P. with first term 'a'and common ratio r. Then,

re
2 n-2 n-l
Sn a + ar + ar +... + ar + ar

Frl
F
Multiplying both sides by r, we get
2 ii- 1 n
rS n ar + ar +... + ar" + ar ...(ii)
ou
sor
On subtracting (ii) from (i), we get
n
S„-rS n a~ar kf
oo

II
Y

1 -r
s„ provided that r
B

a
1 -r '
re

II
r -1
oY

or. S„ a
u

r-1
ad

II n
1-r r 1
d

Hence, Sn a or. Sn a
1 -r r-1
in
Re

Q.E.D
NOTE Some authors state two different formulas for S^, viz.,
F

\
n
1 — r 1
Sitn — a forr <1 and Sn a forr>l.
1 -r r-1

In fact these two are exactly identical. The only thing which must be noted is that the above formulae do
not hold for r =1. For r-1, the sum ofn terms of the G.P. is
S,j =a + a + a+...+a {n times) =na
n-l
REMARK 1 If I is the last term of the G.P., then I = ar

V
n n n -
1 -r a -ar a - {a r a-lr
Sn a
1 -r 1 -r 1 -r 1 -r

a-lr Ir - a
Thus, Sn or. Sn , r^l
1 -r r-1
9.16 APPLIED MATHEMATICS-XI

ILLUSTRATIVE EXAMPLES

Type I FINDING THE SUM OF GIVEN NUMBER OF TERMS OF A GIVEN G.P.


EXAMPLE 1 Find the sum of 7 terms of the G.P. 3, 6,12,...
SOLUTION Here, fl = 3, r = 2 and fj =7.
f 7
/-I -1
= a - 3 — = 3(128-1) = 381
r-1 2-1

ow
EXAMPLE 2 Find the sum of 10 terms of the G.P. 1,1/2,1/4,1/8...
SOLUTION Here, fl =1, r =1/2 and n =10.
10
„ r -1
SiQ - a -

e
r-1

re
10 10
(1/2)^ -1 (1024-1) _1023
L =2 2
1
^=2 1-

Flr
S;10 -1 < 10 10

F
(1/2)-1 2 2 512 512

EXAMPLE 3 Find the sum to 7 terms of the sequence


1 ^ a1 2
ou 1 2 3

sr
- + + — + , .. .

5V'W
8
5'^ 5

SOLUTION
/
The given sequence is fo
k
oo
1 _2 3 1 1 2 3

'5^[s 5^ 53J'
- +

5 52 5^ 5^
Y
reB

1 2 3 38 1
Clearly, this is a G.P. with first term a = - + and common ratio r =
5 5 5' 125 5^
uY

7 3v7 21
1 38 1-(1/5Q- 38 1 -1/5 19
^ = a - =

1 21
1 -r 125
l-(l/5^) 125 62
5 .
ad

1 -
do

125

EXAMPLE 4 Sum the series: x (x + y) + {x^ + y^) + x^ (x^ + y ^) + ...ton terms


in
Re

SOLUTION Let denote the sum to n terms of the given series. Then,
F

= .v(x + y) + x^ (x^ + y^) + x^ (x^ + y^) +... + .v” (x" + y”)


S,i = (x^ + x"^ 4- x^ +... + x^”) + (x y + x^ y^ + x^ y^ 4-... + x” y")
jxyf-i
S,r = x^ . +xy ■
X 2-1 .vy-1


X 1 (.xy)”-l
● +xy-
.X 2-1 xy-1

EXAMPLES Find the sum of the series 2 + 6 + 18 +... + 4374,


SOLUTION The given series is a geometric series in which a = 2,r = 3 and / = 4374.
(Ir-a) 4374 X 3 - 2
Required sum = = 6560.
(r-1) 3-1
GEOMETRIC PROGRESSIONS 9.17

EXAMPLE 6 Find the sum of the following series:


(i) 5 + 55 + 555 + ... to « terms (ii) 0.7 + 0.77 + 0.777 + ... to n terms
(iii) 5 + 55 + 555 + 5555 +... to n terms
SOLUTION (i) Let S be the sum of the series 5 + 55 + 555 +... to n terms. Then,
S= 5 n +11 + 111 + ... + ton terms ■

5
=> S = ■ 9 + 99 + 999 + ... + to n terms ●

s = ^9 J (10-1)4-(10^ -1) + {10^ -1)4-...+(10" -1) ●

low
S= -9 (10 + 10^+10^+...+10")-(l+l+l+... + l)
>i times

5
S= - lOx
(10"-1) -n)-
5
no
n+ 1
-10-9n
9 10 -1 81

ee
F
Fr
(ii) Let Sbe the sum 0.7 + 0.77 + 0.777 +... ton terms. Then,
S = 7 X 0.1 + 7 X 0.11 + 7 X 0.111 +... ton terms

7-^0.1 + 0.11+0.111+ ... to n terms-


for
ur
S =
s
7
ook

S = - 0.9 + 0.99 + 0.999 +... to n terms ●


Yo

7 J 9 99 9^ + ...
eB

S = to n terms
9 1 10 100 ^ 1000
7 1 4 /
1
r

S = 1 - + 1 - +... to n terms
ou
ad

9 10 J V 100 1000

7 1 1 1
Y

S = ■I 1 + 1 - + ... + 1 -
10^ y 10^
II
9 10 10
V
Re
nd
Fi

10
7 1 1 1 7 1
S = - <n~ + +... + n -

10^
n
9 10 10 9 10

10

7 1 ( 1 7
S = — <n — 1 ● 9n -1 + ●
n n
9 9 10 81 10

(iii) Let S be the sum of the series 5 + 5.5 + 5.55 + 5.555 + to n terms. Then,
S = 5 + 5.5 + 5.55 +5.555 + ...ton terms

S =5 + (5 + 0.5) + (5 + 0.55) + (5 + 0.555) + to n terms


S = (5 +5+5 +...n terms) + {0.5 + 0.55 + 0.555+ ...to(n-l) terms}
S=5n + 5 {0.1+ 0.11+ 0.11 + ...to (n-1) terms)
9.18 APPLIED MATHEMATICS-XI

S =5n + ^ {0.9 + 0.99 + 0.999 + to (n -1) terms)


c c 5(9
S =5?z+ — < — +
99 999
+...to(n-l) terms
9 1 10 100 1000
\ / \ /

C C + -5
S=5n 1
1
+ 1-
1
+ 1-
1
+ ...+ 1-
1

9 10 100 1000 (i-l


10

1 1 1 1
-U.-D-
S=5n + —
10 10^ "^10^
+...+
ii-l
10

w
1 yf-1
1-
1 10
S=5;i + -
9 10 1

Flo
1 -
10

e
re
S=5n + -

F
9 9 10
ur
EXAMPLE 7
The sum offirst three terms of a G.P. is 36 mid the sum of the next three terms is 128. Find
the sum ofn terms of the G.P.
SOLUTION Let a be the first term and r the common ratio
f or
ks
of the G.P. It is given that
Yo

a + ar + ar^ = 16 ...(i) and, ar^ + ar“^ + ar^ = 128 ...(h)


oo

a(l +r + r^) =16 and, (1 + r + r^) = 128


B

ar^ {1 + r + r^)
re

128
=> r 3 = 8 =>r = 2
a{l+r + r^) 16
u
ad

16
Yo

Putting r = 2 in (i), we get: ^ =


d

■ 4 ^ It
r -1 16 2'-l
Re

S,j = a = -{2"-D
in

r-1 7 2-1 7
F

EXAMPLE 8 Find a G.P. for which the sum of the first two terms is -4 and the fifth term is 4 times the
third term.

SOLUTION Let a be the first term and r be the common ratio of the G.P.

We have.
ai + U2=-4 and 0^=4 0^
a + ar = -4 and ar'^ = 4 ar^
a{l + r)—-4 and =4 <7(1 + r) = -4 and r = ± 2
Whenr=2,
^ 4
a(l+r) = -4 => a =—
3

Whenr = -2,
GEOMETRIC PROGRESSIONS 9.19

fl(l+r)=-4 => n = 4
4 16
Hence, required G.P. is - — , - .... or, 4,-8,16,...
3 ' 3 '
Ti/pc n FINDING VALUE(S) OF n, r AND a WHEN THE SUM OF n TERMS OF A G.P. IS GIVEN
EXAMPLE 9 Determine the number of terms in G.P. <a„ >, if = 3, <7„ =96 and S,j =189.
SOLUTION Let r be the common ratio of the given G.P. Then,
= 96=>flir”"^ = 96=> 3r
(1-1
a
n
= 96=>r
II-
^ = 32
Now, S„ = 189
It T
r -1
= 189

ow
r-1

II-1
)r-l = 189
3 —
r-1

e
re
3 ^
r-1
= 189

Fl [Using (i)]

F
32r-l
ur
= 63r-63=> 31r = 62=>r = 2

r
Putting r = 2 in (i), we get fo
2
II - 1
= 32 => 2" ^=2^ => ;i-l=5 =>n = 6.
ks
Yo
EXAMPLE 10 How many terms of the geometric series 1 + 4 + 16 + 64 +... will make the sum 5461 ?
oo

SOLUTION Let the sum of n terms of the given series 5461.


Here, =1, r = 4 and =5461.
eB

= 5461
II 1
r -I
ur

a = 5461
r-l
ad
Yo

II
4" - I
= 5461 [v £7 =l andr = 4]
d

4-1
Re
in

4" -1 = 16383 => 4" = 16384 4” = 4^ => Ji = 7


F

EXAMPLE n The sum of some terms of a G.P. is 315 whose first term and the common ratio are 5 and 2,
respectively. Find the last term and the number of terms.
SOLUTION Let there be n terms in the G.P. with first term fl = 5 and common ratio r = 2. Then,
Sum of n terms = 315
II
r 1
a = 315
r-1

II

5 ^ -1
= 315
2-1

II II
2 -1 = 63 => 2' = 64 = 2 => 71 = 6 [●.● <7=5 and r = 2)
II- 1 6-1
Last term = ar = 5x2 = 160
9.20 APPLIED MATHEMATICS-X!

EXAMPLE 12 innn increasmg G.P., the sum of the first and the Inst term is 66, the product of the second
and the last but one is 128 and the sum of the terms is 126. How man\/ terms are there in the progression ?
SOLUTION Let a be the first term and r the common ratio of the given G.P. Further, let there be n

terms in the given G.P. It is given that the sum of the first and last term is 66.
i.e.
fl-j + (7|j = 66
H-1
a + ar = 66 ...(i)

It is also given that the product of second and the second last term is 128.
n-2 -1 128
i.e. ^ ar .ar = 128=>fl^r" ’ = 128 ^ a .(ar « -1 )=128 => ar
n

Putting this value of af^ ^ in (i), we get


128

w
a + = 66 => - 66fl +128 = 0 => (i7 - 2) (rt - 64) = 0 => a = 2, 64
a

Putting fl = 2 in (i), we get

F lo
»-l »-1
2 + 2.r = 66=> r = 32

ee
Putting = 64 in (i), we get

Fr
n - 1 » - 1 1
64 + 64r = 66=> r
32
for
We reject the second value as the G.P. is an increasing G.P. and therefore r > 1. Thus, we obtain
ur
ji-1
<7 = 2 and r = 32.

Now, Si, = 126


s
ook
Yo

r"-l
2 = 126
eB

r-1

n n
r 1 32r-l
= 63 => - = 63 => = 63 => r = 2
r
ad
ou

r-1 r-1 r-1

H-1 H-1
r = 32=>2 = 2^ => » -1 =5 => n = 6
Y

Hence, there are 6 terms in the progression.


Re
nd

EXAMPLE 13
Find the sum of the products of the corresponding terms of the sequences 2,4,8,16,32 and
128,32, 8,2,-.
Fi

SOLUTION If a, ar, ar^ ,


2
and A,AR,AR ,.... are two geometric sequences, then the
sequence having terms as the product of corresponding terms of the two sequences is also a
geometric sequence with first term aA and common ratio rR.
Given sequences are geometric sequences with first terms 2 and 128 respectively and common
ratios 2 and — respectively. Therefore, the sequence formed by multiplying the corresponding
terms of the given sequences is a G.P. with first term <? = 2 x 128 = 256 and common ratio
r = 2x — = —.
4 2

Since each sequence contains 5 terms. Therefore, the sequence formed by the products of the
corresponding terms has 5 terms.
9.21
GEOMETRIC PROGRESSIONS

1
n 1 -
1

2; 32 (1 1 31
= 496
Hence, required sum = 256 ■ I =256 = 512 = 512 X
1 1 32 32
1 -
2 2

ALITER Required sum = 2xl28 + 4x 32 + 8x8 + 16x2+ 32xi


= 256 + 128+ 64+ 32 + 16

1 -
if
2) 1 31
= 256 = 512 1 =512x = 496
1 32
1 -- 32 ;
2

w
Tifpc HI ON PROVING RESULTS BASED UPON THE FORMULA FOR THE SUM OF nTERMS OF A G.P.
EXAMPLE 14 //Si, $2 and S3 be respectively the sum of n, 2n and 3n terms of a G.P., prove that

F lo
Si(S3-S2) = (S,-s/
SOLUTION Let a be the first term and r the common ratio of the G.P. Then,
f 2ji

ee
II 3/1
r -1 - r 1 j c ^ -1
, S2 = ^7 - r-1 - and S3 = - r-1

Fr
Sj = <7 r -1

Now,
for
ur
n 3/1 2||
r
II
-1
-t
r -1 r ' -1
(^3 “ ^ r-1
a
r-1
-(?
r-1
s
ook
Yo

2
eB

a 3/1 2/1
Si(S3-S2) = 2
-i)-r -1)
(r-1)
2
our
ad

<7 3/1 2/1


=> Si(S3-^) = )
(r-1)
2
Y

Si(S3-S2) =
Re
nd

(r-1)
2
Fi

/I
It <1
// r-1
=5> Si(S3-S2) = ar
r-1

2
//
II 4

r-1
and. (S2-Si)^ = ■ a
r-1
-a
r-1

2
2
2//
(S2-Si)^ =
<7
(r -l)-(r'-l)
(r-l)2
9.22 APPLIED MATHEMATICS-XI

2
2 2
n
r -1
(S2-s/ =
(? /I / H - , )7
r (r -1)1 < ar
r-1

Hence, SjlSg-Si) = (S2-Si)^


EXAMPLE 15
IfS be the sum, P the product and R the sum of the reciprocals ofn terms of a G.P., prove
f s
that - = P^.

SOLUTION Let a be the first term and r the common ratio of the G.P. Then,

2 J/-1 r -1
S = a + ar + ar +... + ar a
-(i)

w
r-1

2 II-1
+ 2 + 3 + ... + (h — 1) _ a-r

Flo
n
P = a .ar ar ... ar 2 ...(ii)
1 1 1 1
and. R

ee
- + — + +... +
a ar
2 H-1
ar ar

Fr
(1/rf -1 II
1 1 1 -r 1
for
ur
R
a a 1-r
(lA) -1
ks
Yo
oo

I!
1 r-1 1
R
r-1 H - ] ...(hi)
B

a
r
re

II
S r -1 r-1 « -1
a .a r
ou

R r-1 II
ad

r-1
Y

sr ^ ^2ii^ii(ii-l) ^ a
II
r 2 = P^
nd

[Using (ii)]
Re

R
Fi

Hence,
R

EXAMPLE 16 A person writes a letter to four of his friends. He asks each one of them to copy the letter and
mail to four different persons with instruction that they move the chain similarly. Assuming that the
chain is not broken and that it costs 50 paise to mail one letter. Find the amount spent on the postage when
8th set of letter is mailed

SOLUTION Amount spent on mailing one letter = ^ ~

Amount spent when first set of 4 letters is mailed = ? 2


Amount spent when second set of 4 x 4 = 16 letters is mailed = ^ (2 x 4) = 8
Amount spent when third set of 4 x 4 x 4 = 64 letters is mailed = ? (8 x 4) = 32
GEOMETRIC PROGRESSIONS 9.23

Clearly, 2, 8,32,... is a G.P. with first term 2 and common ratio 4.


Total amount spent when 8th set of letters is mailed = Sum of 8 terms of the G.P.
o t
r -1
a
r-1

8
4*^ -1
= ? 1 [●.* a-^ 2 and r -4]
4-1

w
65536-1
= ? 2x ● =^(2x 21845)
3

= ^ 43690

o
e
EXAMPLE 17 Find the sum to n terms of the sequence

re
2 r ^2 / \2
1^ 2 1 3 1

Frl
AT + -
' ^ +-2

F
xj X J \

SOLUTION Let £y, denote the sum to n terms of the given sequence. Then,
ou
\2 / n2

r
1 2 1 3 1 n 1
S,, - X + + X + -^ + X + + ... + X + —

so
X
2 3 n
X J kf X y

1 1 ^ 1 1
= x^+ — + 2 + x^ + —j + 2 + x^ + -^ + 2 +...+ X
2n
+ 1.2
oo
2ii
X X X
Y

= (x^-i-x‘*4-x‘^ + ... + X 2ih) + —21 + -4 +-g +-● +


1 1 1
B

2ti
+ (2 + 2 + ...)
X ^x X X ti timcjs
re

2ui
-1 1 (l/.x^-l
oY

4, = ■ + + 2n
u
ad

2/1 2n
X -1 1 1 -X
d

= x^ + + 2n
X 2-1 X
2/1
1 -X
2
in
Re

2/1 ^ . 2/1
X -1 1 X -1
F

S,, = + + 2/1
X 2-1 X
2//
X 2-1
2/1
-1
X
2
+
1 1+ 2/7

"-1 Jl
2ii
X X

n
EXAMPLE 18 Find the sum to n terms of the sequence given by a„ = 2" + 3n, n e N.
SOLUTION Let S,, denote the sum to terms of the given sequence. Then,
= /7j + /?2 + + ... +
% = (2^ + 3 X 1) + (22 + 3 X 2) + (2^ + 3 X 3) + ... + (2" + 3 x //)
S„ = (2^ + 22 + 2^ + ... + 2") + (3x 1 + 3x2+3x3+... + 3x/7)
S,, = (2^ + 22 + 2^ + ... + 2")+ 3(1 +2 + 3+... +/i)
9.24 APPLIED MATHEMATICS-XI

II
-1 3n
=> = 2 -
2-1
+ 3
= 2(2"-l)+y(n + l)
EXAMPLE 19 Prove that the sum to n terms of the series: 11 + 103 +1005 +... is,

SOLUTION Let denote the sum to n terms of the given series. Then,
= 11 +103 + 1005+... to « terms

= (10 + 1) +(10^ + 3) + (10^ +5) + ... + 110” +(2m-1)1

S,, = (10 + 10^+...+10")+ jl + 3+5 + ... +(2m-1)

ow
10(10" -1)
=
+ -(l
2
+ 2»-l) = —(10"-1)+M^
9
(10-1)
EXERCISE 9.3

e
1. Find the sum of the following geometric progressions:

re
(i) 2, 6,18,... to 7 terms (ii) 1,3, 9,27,... to 8 terms

rFl
F
a -b
(iii) 1,-1/2,174,-1/8,... (iv) {a^ -b^),{a-b) ' a +b
... to u terms

r
ou
(v) 4,2,1,1/2... to 10 terms. fo
ks
2. Find the sum of the following geometric series:
(i) 0.15 + 0.015 + 0.0015 + ... to 8 terms;
oo

1 1
(ii) V2 + + +... to 8 terms;
iji
Y
B

,..., 2 113 ,
(ill) + + ... to 5 terms;
re

9 3 2 4

(iv) (.X + ]/) + {x^ + x\j + y^) + (a:^ + 1/ + xx/" + y^) +... to /i terms;
ou
Y
ad

/ ^ 3 4 3 4 _ ^
(v) — + -r- + ^ + —r +... to 2n terms;
5 5^ 5-^ 5'^
d

a a a a
(Vi) + +... +

(1+0^ (1 + 0^
in
Re

n
1 +/ (1+0
2 3
F

(vii) 1 ,- a, a ,-a',..Aon terms {a * 1)


●> 5 7
(viii) .r‘ ,x,x ,...to if terms
(ix) , -Jli , 3^f7 ,.... to n terms
3. Evaluate the following:
10
fl I. I' _ 1
(ii) S (2^^ + 3^“ *)
II
(i) S (2+3") (iii) I
(1 = 2
4
jj= 1 k=l

4. Find the sum of the following series:


(i) 5 + 55 + 555 +... to n terms,
(ii) 7 + 77 + 777 +... to m terms,
(iii) 9 + 99 + 999 +... to « terms,
(iv) 05 + 055 + 0555 +... to;? terms,
(v) 0.6 + 0.66 + 0.666 + .... ton terms.
3069 ,
5. How many terms of the G.P. 3, 3/2, 3/4,... be taken together to make 512
9.25
GEOMETRIC PROGRESSIONS

6. How many terms of the series 2 + 6 + 18 +... must be taken to make the sum equal to 728 ?
7. How many terms of the sequence 3, S'/S,... must betaken to make the sum 39 + 13^/3?
8. The sum of n terms of the G.P. 3, 6,12,... is 381. Find the value of n.
9. The common ratio of a G.P. is 3 and the last term is 486. If the sum of these terms be 728, find
the first term.

10. The ratio of the sum of first three terms is to that of first 6 terms of a G.P. is 125:152. Find the
common ratio.

11 The 4th and 7th terms of a G.P. are — and respectively. Find the sum of n terms of the
27 729
G.P.
1
/ 2 yi- 1

ow
10
12. Find the sum: X i “ +

v.5y
n=\

13. The fifth term of a G.P. is 81 whereas its second term is 24. Find the series and sum of its first
eight terms.

e
re
14. If
+ s| = Sj (S2 + S3).
Fl
$2/ S3 be respectively the sums of n, 2n, 3n terms of a G.P., then prove that

F
15. Show that the ratio of the sum of first ?i terms of a G.P. to the sum of terms from (n + 1)‘*^ to
ur
(2«)^*’ term is^.
r
r
fo
ks
2 2
16. If and b are the roots of x - 3x + p = Q and c, d are the roots x 12.Y + q = 0, where
Yo
oo

n, b, c, d form a G.P. Prove that(c/ + p):{q -p) = 17 :15.


3 3 3069.,
eB

17 How many terms of the G.P. 3 , — , are needed to give the sum 512
2 4

18. A person has 2 parents, 4 grandparents, 8 great grand parents, and so on. Find the number
ur

of his ancestors during the ten generations preceding his own.


ad
Yo

19. If S], S2,..., are the sums of ti terms of n G.P.'s whose first term is 1 in each and common
ratios are 1,2,3,..., n respectively, then prove that
d

It

Si + S2 + 2S3 + 3S4 + ... (J! -1) =1" + 2" + 3" + ... + ;i .


Re
in

20. A G.P. consists of an even number of terms. If the sum of all the terms is 5 times the sum of
F

the terms occupying the odd places. Find the common ratio of the G.P.
ANSWERS

171 n -b (ci + bf -1 f 1
1. (i) 2186 (ii) 3280 (iii) (iv) h-2
(V) 8 1- —
256 (rt +1?) -1 1024
(a + b)
II
1 255 V2 55 1 2 X -1
2. (i) - (ii) (iii) ^ (iv) < X
x-1
-!/■
6 10 128 72 - .V

It 2ii
19 !-(-«) 3 (.v" -1)
(V) 2h
(Vi) -^7/|l -(1+0 ") (vii)
1 + rt
(viii) X
2-1
24 5 .V
9.26 APPLIED MATHEMATICS-XI

^ o/»/2
1 n+2
(xi) V7 - - 3. (i) 265741 + 3"-5) (iii)
V
Vs-1

4.(i) Jr
81
[10 n+ 1
-9?j-10] (ii) — [10
81
n+ 1
-9n-10] (iii) -110
1 n+ 1
-9«-10]

1 (.1 1 /(v). ~<n-~


6f 1
(iv)^ <n —
9 10
n
9 9 ij
10" >
5. 10 6. 6

7. 6 8. 7 9. 2 10.
3
11. ^fi- 1

5 2
3"J
10 10
65x 97

ow
12. +
13. a = l6, r=~, Sg = 17. 10
2^ 4x5
n
2 ^ 8

18. 2046 20. 4

HINTS TO SELECTED PROBLEMS

e
2. (vii) Let £^, denote the sumof ?? terms of the G.P. 1 - n, , -a^.... Then,

re
Fl 1 ^
n

F
^.=1
-a-1 1 + a
ur
r
(viii) Let 5^, be the sum of»terms of the G.P. x^, .v^, .r^,.... Then, fo
, 2.H T 2II
ks
3 (x ) -1 3 X -1
— X ■ =x
Yo
x^-1 x^-l
oo

(ix) Let S,i denote the sum of ;i terms of the G.P. ^f7, 3-Jf,.... Then,
eB

(V3)”-l 3»/2
S,,=^ = V7
' Vs-i '
ur

11 11 11 -5 11
ad

-1 11
(i) 2 (2-3")= 2 2+2 3" =2x11+ 3 -3-1
Yo

3. = 22 + -(3 -1)= 265741.


»= 1 «= 1 » = 1
2
d

4. (i) Let Sfj = 5 + 55 + 555 +.... to n terms. Then,


Re
in

= 5 (1 + 11 + 111 + .... to n terms)

=> = I (9 + 99 + 999 + , to n terms)


F

^, = I -(10-l)+{lo2-l) + (10^-l)+...+(10”-l)
=> = ^ ■ (10 + 10^ + 10^ + . . + 10")-71 ■
n
10" -1
- ^' = 9
- . 10 -n
10-1

(10"-!)-.}
n+ 1
=> “
~ (10 -9?i-10).
9 81

(ii) Proceed as in (i)


(v) Let S,j = 0.6 + 0.66 + 0.666 +.... to »terms. Then,
9.27
GEOMETRIC PROGRESSIONS

S,i = 0.6 + 0.66 + 0.666 + .... to n terms


S, = - (0.1+ 0.11+0.111+.... ton terms)

6
Sy, = -9 ( 0.9 + 0.99 + 0.999 + ....to u temis ●

1
6
1
1
+1--1 +1-^ + ...+ 1-
10^ 10^
>1
9 10 J 10”,

6 1 1 1 1
Sn = X «- — + + + ... +
9 10 10^ 10^ 10" J

ow
V'
1-f^
6 1
lOj 6 1 1
1 -
^1 = 7:9 u -
10 1 9
<n —
9
II
10

e
1 -
10

re
rFl
F
H-1
a + ar + ... + nr
rtl + i?2 + ● ● ● + ^»
15. Required ratio = II n+ 1 In- 1
^^1+! + '*«+2 + ar + ar +... + <ir

r
ou
1 -r
II
fo
ks
a
1 -r
1
oo

It
II
r
n 1 -r
ar
Y

1 -r
B

2 3
16. We have, a + b = 3, (7^ = p, c + fr = 12 and cd = c] Letb= nr, c=ar and d=nr . Then,
re

a+b = 3 and c + =12


ou
Y
ad

n{l+r) = 3 and iir^(l+r)=12


itr^(l+r) 12
d

17 (1 + r) 3
in
Re

i7(l+r) = 3 => a=l


F

Now, p-ab =a.ar = 2, q ~ cd =nr ^ nr ^ = 2^ = 32


2

q + p 32 + 2 _ 34 _ 17
q-p 32 - 2 " 30 " 15
3 3 3069
17. Let the sum of n terms of the G.P. 3, — , — be . Then,
512

1023 1 1
3
12, 3069
^l-i- = 10
=> n=10
1-^
II
512 2" 1024 2” 2
2

3069
Hence, the sum of 10 terms of the given G.P. is 512
9.28
APPLIED MATHEMATICS-XI

18. Number of ancestors during the ten generations preceding his own generation
= Sum of 10 terms of the G.P. 2, 4, 8, ....
( 10
= 2 ^ -1
= 2046.
2-1

rw
20. Let there be 2n terms in the G.P. with first term a and common ratio r. Then,

Sum of all the terms =5 (Sum of the terms occupying the odd places)
+ ^2 + ... + n2„ - 5(rti + 4- <^ + ... + ^2/1- l)
2»- 1

e
2 2;i-2
n + ar +... + ar = 5 (« + flr +... + rtr )

r
In 2oi

luo
1 -r 1 -{r )
a ■ ● =5a ■ ●=>l+r=5=> r -4
1-r^

F
1 -r

9.5 SUM OF AN INFINITE G.P.

THEOREM

oF
The sum of an infinite G.P. zvith first term a and common ratio r (-1 < r < 1 i.e., | r | < 1)

rs
a
is S =

ok
1 -r

PROOF Consider an infinite G.P. with first term a and common ratio r, where -1 <r <1 i.e.

fo
r I < 1. The sum of n terms of this G.P. is given by
n n
1 -r a ar
o
Y
S„ = a ...(i)
1 -r l-r 1-r
Y
rB

Since -1 < r < 1, therefore r” decreases as n increases and tends to zero as n tends to infinity
n
I.e. r -> 0 as n 00.
ue

II
a r
0 as n CO.
d

1 -r

Hence, from (i), the sum of an infinite G.P. is given by


no
ad

II
a ar a
S = lim S,, = Um / if |r| <1
» —>00 l-r l-r 1 -r
i

II ^QC

NOTE !fr > 1, then the sum of an infinite G.P. tends to infiniti/.
Re
F

Q.E.D.

ILLUSTRATIVE EXAMPLES

Ti/pc / FINDING THE SUM TO INFINITY OF A G.P. OR A GEOMETRIC SERIES


5 5 5
EXAMPLE 1 Find the sum to infinity of the G.P. —, — , ,...
4 16 64
SOLUTION ThegivenG.P.hasfirsttermrt =-5/4andthecommonratior =-1/4. Also, | r| <1.
Hence, the sum S to infinity is given by
a -5/4
S = = -l
l-r l-(-l/4)
EXAMPLE 2 Sum thefollozvinggeometric series to infinity:
1 1 1 1 1 1
(i) (^/2+l) + l+(V2-l)
(ii)^ + ^
3^ + 2-^
+ ... CO + + ... 00
3 2^ 3^
SOLUTION (i) The given series is a geometric series with first term a=-yj2 +1 and the common
ratio r given by
9.29
GEOMETRIC PROGRESSIONS

1 V2-I
= V2-1
'■7m"(V2+i) (V2-1)
Hence, the sum S to infinity is given by
_ V2+I ^ V2+I ^ V2+I
S =
1 -r “l-{V2-l) 2-V2 V2(V2-1)

S =
(V2+l)^ 3 + 2V2 _ 4 + 3^2
V2(V2-1)(V2+1) V2 2

(ii) We have.

w
2 3^ 2^ 3*^ 2’ 3^
1 1 1 1 1 1
—, + ~T + —r + ●●●

F lo
— + "^ + “F +●●●
2 2^ 2^ 3^ 3"^ 3^
1 1

ee
1 1
= An infinite G. P. witli (7 = — , r - — + An infinite G.P. with f? =
2^ 3'

Fr
(1/2) 2 1 ^ 19
● + -
3 ^ 8 ” 24 for
ur
l-(l/2^) l-(l/3^)
1/8
EXAMPLE 3 Prove that: x 6^^"^ x 6
ks
00 = 6.
Yo
oo

SOLUTION Clearly,
6'^^ X 6'/* X 6'/'* ^ ^|l/2+ 1/4+ 1/8+ ...«}
eB

... 00

1 1 1 1/2
^ ^l(l/2)/(l-1/2)1 ●.●— + — + — + ... to X
1-1/2
= 1
r

2 4 "
ou
ad

= 6’ = 6
Y

Typell ON PROVING RESULTS BASED UPON THE FORMULA FOR THE SUM TO INFINITY OF A G.P.
nd

b
Re

2 3
vrozfe that a = .
EXAMPLE4 Ifb=a + a +a +...x. 1 +b
Fi

SOLUTION We have,
2 3
b == a + a + a +...x

Clearly, RHS is a geometric series with first term 'a' and common ratio 'a'
b = b -ab = a^ n
I -a 1 +b
c c
rr a a _ 4. — +... CO, prove that
+●●●«=/ 1/ = 1; - -r + —
C 4-
EXAMPLES i/x=i7 + -+
'■
“2
r r‘ r r

■xy _ ab
2 c ... . . ● ,
SOLUTION Clearly, x, y and 2 are the sums of infinite geometric progressions. ^
a ar
cr
b hr c

y = and, z =
X -
r-1
,
r 1 1 +r r^-l
1 -
r
r
I rj
9.30
APPLIED MATHEMATICS-Xl
\ /

=
ar br abr^
r-1 r + 1 r^-l

xy abr^ cr
2
ab
z c

EXAMPLES lfx=l+a + a^ + ... CO,


ivhere\ < 1 andy =1 + b + + ...oo, where \ b\ <1. Prove
that:
2 2
1 + ab + a b +... co-
x + y-1
SOLUTION We have,
x: = i + a + o^ + .. .

w
00

1 1 1
.r 1 - a = -=> a =1 —

F lo
1 -a X X

and, y = 1+b +b^ + b^ + X

ee
1 1
y = 1 -b = -=>&

Fr
1 -b ...(h)
y y

1 + ab + (ab)^ + (ab) ^ + .. . X
for
1 1
r
xy
[Using (i) and (ii)]
You

1 _ f 1 - iV
^ 1 _ i1
I ~ab
x + y-1
s
ook

x)[ y
eB

Type lU FINDING REQUIRED UNKNOWN WHEN THE SUM OF AN INFINITE G.P. IS GIVEN
EXAMPl.E 7 The first term of a G.P. is 2 and the sum to infinity is 6. Find the common ratio.
our

SOLUTION Let r be the common ratio of the given G.P. It is given that, « = 2 and S„o = 6.
ad

a 2
Now, .S X = 6 = 6 => 6-6r=2 => r=2/3.
1 -r 1 -r
dY

EX/VMPLE 8 The smn of an infinite G.P. is 8, its second term is 2, find the first term.
Re

SOLUTION Let a be the first term and r the common ratio of the G.P. It is given that
Fin

Sjo = 8 and ar = 2
a
= 8 and r = ~
1 -r a

l-(2/a) [Eliminating ?●]

a^ -8a + 16 = 0=> (a - 4)^ = 0 ^ <7 = 4.

EXAMPLE 9 The sum of an inifnite G.P. is 57 and the sum of their cubes is 9747, find the G.P.
SOLUTION Let a be the first term and r the common ratio of the G.P. Then,
a
Sum = 57 => = 57
1 -r
Sum of the cubes =9747
3
a^ + a^ r^ +a^ r^ +... = 9747 => a
= 9747
...(h)
GEOMETRIC PROGRESSIONS 9.31

Dividing the cube of (i) by (ii), we get


a (1-r^) (57)^
3 "" 3 9747
(!-/■) a

l-r^
= 19
(l-rf
1+r + r^
= 19
(1-r)^
18r^-39r + 18 = 0
(3r-2)(6;--9) = 0
r = 2/3 or, r = 3/2

w
r = 2/3 [●.● r 3/2, because -1 < r < 1 for an infinite G.P.]

F lo
Putting r = 2/3 in (i), we get
a
= 57 => r? =19
l-(2/3)

ee
Fr
Hence, the G.P. is 19, 38/3,76/9,....

Tl/fW IV FINDING A RATIONAL NUMBER WHOSE DECIMAL EXPANSION IS GIVEN for


ur
EXAMPLE 10 Which is the rational number having the dechnal expansion 0.356 ?
SOLUTION We have,
s
ook

0.3% = 0.3 + 0.056+ 0.00056+ 0.0000056+ ...00


Yo

56 56 56
eB

= 0.3 + + ... CO

10"^ lO"* 10^


56
r
ad
ou

A+ 10^1 _3 56 _ 353
10
1 -
10 990 ~ 990
Y

10^
Re
nd

EXAMPLE 11 Use geometric series to express 0555... = 0.5 as a rational number.


SOLUTION We have,
Fi

0.5 = 05555...

= 05 + 0.05 + 0.005 + ... CO

= A+ 5 + 5 + ... CO
10 10^ 10^
(5/10) 5

1-(1/10) 9

Tl/pC V ON APPLICATIONS OF INFINITE G.P.


EXAMPLE 12 A square is drawn bp joining the mid-points of the sides of a square. A third square is
drawn inside the second square in thesamewapand the process is continued indefinitelp. If the side of the
square is 10 cm, find the sum of the areas of all the squares so formed.
9.32 APPLIED MATHEMATICS-XI

SOLUTION Let/^|/l2'^3^4t’ethefirstsquarewitheachsideequaltol0cm. Let6i, B2, 63, 64be


the mid-points of its sides. Then,

0102 - ^^20^ + A2B^ = + 5^ =SV2 cm.

^4 ^3

C4 C-

B4 Bz

Cl C2

/t 1
B1 ^2

F low
Fig. 9.1

Let Cl, C2/ C3, C4 be the mid-points of the sides of the square 01020304- Then,
/ n2
5y/2 5V2
C,C2=.Jb,c/ + B,Cj2= 2 +
2
= 5 cm for Fre
Similarly, the side of fourth square is -4>/2 cm and so on.

Sum of the areas of all the squares so formed


Your
eBo ks

5
● 10^ +(SV2)^ +(5)^ + 00 ^ sq. cm. [v Area = (Side)^]
IV2
ad
our

25 100
100 + 50 + 25 + — +... 00 = 200 sq. cm.
2 l-(l/2)
\f/z
Re

I 4
EXAMPLE 13 After striking a floor a certain ball rebounds — of the height from which it has fallen.
Find Y

Find the total distance that it travels before coining to rest, if it is gently dropped from a height of 120
metres.

SOLUTION Initially the ball falls from a height of 120 metres. After striking the floor it rebounds
4 4
and goes to a height of — (120) metres. Now, it falls from a height of — (120) metres and after
5 5
4
rebounding again it goes to a height of - - (120) metres. This process is continued till the ball
5 ^5
comes to rest.

x2
4 (4
The total distance traveled = 120 + 2 ●
^(120)+
5
I
1^5
(120)4-...od .
GEOMETRIC PROGRESSIONS 9.33

= 120 + 2 X
^ (120) = 120 + 960 = 1080 metres.

EXAMPLE 14 The inventor of the chess board suggested a rezvard of one grain of zvheat for the first
square, 2 grains for the second, 4 grains for the third and so on, doubling the number of the grains for
subsequent squares. Hozv many grains ivoidd have to be given to inventor ? (There are 64 squares in the
chess board).

SOLUTION Clearly, required number of grains is the sum of an infinite G.P. with first term 1

ow
and common ratio 2.
/
64
2 -1 64
Number of grains = 1 + 2 +2“+ 2^+...to 64 terms = 1 - 2-1
= 2 1.

e
Type ON FINDING REQUIRED UNKNOWN WHEN SUM OF AN INFINITE G.P. IS GIVEN

re
EX AMPLE 15

Frl
The sum of an infinite geometric series is 15 and the sum of the squares of these terms is 45.

F
Find the series.

SOLUTION Let a be the first term and r be the common ratio of the infinite geometric series.
ou
or
a
Sum =15 ^ = 15 ...(i)
1 -r
kfs
Sum of the squares = 45
oo

2
a
+ ... co) = 45 => = 45 ...(h)
Y

1 -r^
B

Dividing the square of (i), by (ii), we get


re

1-r^
2
(15)^ 1 +r 2
oYu

a
= 5=>6r = 4=>r =
ad

3
(1-r)^ « 45 1 -r
d

2
Putting r = — in (i), we get
3
in
Re

a
= 15 => <7 =5
F

1-2/3

, . . ^ 10 20 40
Hence, the required senes is 5 + — + — +
27
+ ... 00.

EXAMPLE 16
If each term of an infinite G.P. is twice the sum of the terms foUoiuing it, then find the
common ratio of the G.P.
SOLUTION Let a be the first term and r the common ratio of the G.P. It is given that
a.11 = 2[i2„+i + fl„+2+«fT+ foralhJsN
ji-1 n+ 1
ar = 2 [ar" + ar + ... oo]
II
(1-1 2 ar 2r 1
ar => 1 = r = —
1 -r 1 -r 3
9.34 APPLIED MATHEMATICS-Xl

EXERCISE 9.4

1. Find the sum of the following series to infinity:


,.,,1 1 1 + 1 + ... QC
(ii) 8+ 4^/2+4+.. . 00
3^ 3^ 3‘*
{iii) 2/5+3/5^+ 2/5^+ 3/5^+.. . X.
(iv) 10-9 + 8.1 -7.29 + ...00
,,1 1 1 1 1 1 + ... X
3 5“ 3^ 5“* 3^ 5^
2. Prove that: = 3.

3. Prove that: (2^^^. 4^^®. 8^^^^. 16^^^^.,.x) = 2.

ow
4. If Sp denotes the sum of the series 1 + r^ + +..
. to X and Sp the sum of the series
..tox, prove that Sp + Sj, = 2 S2p.

e
Find the sum of the terms of an infinite decreasing G.P. in which all the terms are positive,

re
rFl
the first term is 4, and the difference between the third and fifth term is equal to 32/81.

F
6. Express the recurring decimal 0.125125125 ... as a rational number.
7. Find the rational number whose decimal expansion is 0.423.

r
8. Find the rational numbers having the following decimal expansions:
ou
(i) 0.3 (ii) 0.231 (iii) 352
fo (iv) 0.68
ks
9. One side of an equilateral triangle is 18 cm. The mid-points of its sides are joined to form
another triangle whose mid-points, in turn, are joined to form still another triangle. The
oo

process is continued indefinitely. Find the sum of the (i) perimeters of all the triangles,
Y
eB

(ii) areas of all triangles.


10. Find an infinite G.P. whose first term is 1 and each term is the sum of all the terms which
follow it.
r
ou

^ 1. The sum of first two terms of an infinite G.P. is 5 and each term is three times the sum of the
ad
Y

succeeding terms. Find the G.P.


12. Show that in an infinite G.P. with common ratio r (| r ( < 1), each term bears a constant ratio
d

to the sum of all terms that follow it.


Re
in

ANSWERS
F

MO I (ii) 8 (2 + V2) (iii) ^ (iv) 5.263 (v) 4 5. 6, 12


24 12 3-2V2
^ 125 7 il? 8- (i) 1
231 317 31
b.
999
(ii) (iii) (iv) -H-
990 999 90 45
1 1 1 1
9. (i) 108 cm (ii) 108 -JS square cm 10. 1,-
2'4'"'
11. 4,1,
4'16'"
HINTS TO SELECTED PROBLEM

9. Sum of the perimeters = 3 j 18 + ^ + — +... X

Sum of the areas =


V3 io2
18 + —
18
to f
+
18 f +... X
4 2 4
GEOMETRIC PROGRESSIONS 9.35

9.6 PROPERTIES OF GEOMETRIC PROGRESSIONS

In this section, we shall discuss some important properties of geometric progressions and
geometric series.
PROPERTY I If all the terms of a G.P. be multiplied or divided b\j the same 7wn-zero constant, then it
remains a G.P. with the same common ratio.

PROOl Let Aj, f72, «3,..., ... be a G.P. with common ratio r. Then,
+ 1
= r, for all« G N

ow
Let khea non-zero constant. Multiplying all the terms of the given G.P. by k, we obtain the new
sequence: /c/?2, ka^^,... ka„,...
+ 1
Clearly = r for all» gN [Using (i)]
k a II a

e
n

Hence, the new sequence also forms a G.P. with common ratio r.

re
PROPERTY I The reciprocals of the terms of a given G.P. form a G.P.

F
Frl
l‘RCX)F Letfli, a2,a;^,...,a„,...hea G.P. with common ratio r. Then,
^^1+ 1
= r for all n bN
a
n
ou
osr
The sequence formed by the reciprocals of the terms of the given G.P. is
JL ^ i_ 1
kf
rtj tJ2 ^3 an
oo
For this sequence the ratio of a term and the preceding term is given by
Y

^ + 1 a,n 1
[Using (i))
B

l/«,n '^1+ 1 r
re

So, the new sequence is a G.P. with common ratio 1 /r.


uY

PROPERTY III If each term of a G.P. be raised to the same power, the resulting sequence also forms a G.P.
PROOt Let rt], ^2/ ^^3/ ●●● ●●● be a G.P. with common ratio r. Then,
ad
do

‘^1+ 1
= r for all?! g N ...(i)
a
in

Let A: be a non-zero real number.


Re

Consider the sequence whose terms are k^^ powers of the terms of the given sequence
F

k k k k
i.e.
, ^2, «3,..., a„,...

For this sequence, we have


k
^1+1 ^^1+ 1
k
= r^ for all n gN [Using (i)]
a II
a II

k k k k k
Hence, a^, a2, a;^,... a„, is a G.P. with common ratio r .

PROPERTY IV In a finite G.P the product of the terms equidistant from the beginning and the end is
always same a7id is equal to the product of the first and the last term.
PROOF Let , r?2 / ^3 /.. ●, a,j be a finite G.P. with common ratio r. Then,
fc-1
kth term from the beginning = aj^ -a-^r
n-k
kth term from the end = (n-k + l)th term from the beginning = aJ,^|^+ ^ r
9.36 APPLIED MATHEMATICS-XI

(kth term from the beginning) {kth term from the end)
k-l n-k 2/1-1 »-l
a-ir = /?! r = . Hi r = a-^ for all k = 2, 2>,...,n

Hence, the product of the terms equidistant from the beginning and the end is always same and
is equal to the product of the first and the last term.
PROPERTY V Three non-zero numbers a, b, c are in G.P. iff = ac

PROOF Clearly,
Q 2
a, b, c are in G.P. <=> — = - = (common ratio) o b = ac
a b

NOTE When a, b, c are in G.P., then b is known as the Geometric mean of a and c.
PROPERTY VI If the terms of a given G.P. arc chosen at regular intervals, then the new sequence so
fonned also forms a G.P.
PROPERTY VII If a-^,a2,a^,.. .,a„,... IS a G.P. of non-zero non-negative terms, then

w
logfl],log02,..., log .a .. is an A.P. and vice-versa.
It' ●

PROOF Letai,a2,a2,...,a.^ ...be a G.P. of non-zero non-negative terms with common ratio r. Then,

F lo
n>

\ forallneN
n -
^t =

loga„ = log flj + (>j-1) log r, forall»eN

ee
Let b„ = loga„ = log + (?j-1) log r, forall/ieN

Fr
Then, b„^ j - b„ = [log aj + n log r] - [log a^ + {n -1) log r] = log r for all n sN
Clearly, + i~b,j= log r = Constant for all n e N. for
Hence, f?2,..., ... i.e. log i7|, log U2,..., log f?„,... is an A.P. with common difference
ur
log r.
Conversely, let log a^, log 02,..., log be an A.P. with common difference d. Then,
s
ook

log (tjj + \ - log Ojj = d for all n e N.


Yo

*^1+ 1
eB

log = d for all« 6 N.


a,

1
= e^ (a constant) for all/j eN.
r
ad
ou

«//

a\, ^2' ●●●' %' ● .. is a G.P. with common ratio


Y

ILLUSTRATIVE EXAMPLES
Re
nd

Type I PROBLEMS BASED UPON FOLLOWING RESULTS;


Fi

(i) a, b, c are in G.P. iffb^ = ac


(ii) a, b, c are in A.P. iff 2b = a c.
EXAMPLE 1 Ifp, q, r are in A.P., show that the pth, qth and rth terms of any G.P. are in G.P.
SOLUTION Let A be the first term and R the common ratio of a G.P. Then,

Up =AR^ \ a^ =AR q-1 -1


and Oj. = AR''
We have to grove that a^, a^, a^ are in G.P. For this it is sufficient to show that
(V =«p-«r
q~T2
Now, = {AR )

{a/ =
(n/ = [●.● p, q, r are in A.P.2q = p + r]
(a/ = (ARP
-1
) =Op.ar
Hence, Op, a^j, a^ are in G.P.
9.37
GEOMETRIC PROGRESSIONS

EXAMPLE 2 If a, b. c are in G.P., then prove that log a\ log b'^, log t'" are in A.P.
SOLUTION It is given that a, b, c are in G.P.
= ac

{b^f = {cic)'
2n
b a c

2ji
logb = log(«" c")
log ib”)'^ = log fl" + log c"
2 log b" log rt” + log c"
log a’\ log b’\ log c" are in A.P
EXAMPLES Three numbers whose sum is 15 are in A.P. If 1,4,19 be added to them respective!}/, then
they are in G.P. Find the numbers.

w
SOLUTION Let the three numbers be a - d, a,a + d. Then,
Sum =15 => {a - d) + a + {a + d)-15 =^> fl-5.

F lo
So, the numbers are5-rf, 5,5+ fr. Adding 1,4,19 respecHvely to these numbers, we get
6-d, 9, 24+ fr. These numbers are in G.P.

ee
9^=(6-d)(24 + fr) fr^+18J-63=0 => (rf + 21) (rf- 3) = 0 => fr =-21 or, fr = 3.

Fr
Hence, the numbers are 26,5, -16 or 2, 5,8.
Ti/;>e II PROBLEMS BASED UPON PROPERTIES OF G.P. for
EXAMPLE 4 If a, b, c, d are in G.P., shozu that:
ur
(i) ib-cf+{c-af+{d-bf={a-d)^
s

(ii) (rtl’ +bc + cd)^ = {a^ +b^ + c^) {b^ + +d )


ok
Yo

Let r be the common ratio of the G.P. a, b, c, d. Then, b = ar, c =nr ^ and d =ar^.
o

SOLUTION
eB

(i) LHS = {b-cf +{c-af +{d-bf


= {ar - ar'^f + {ar^ - af + {ar^ - arf
r
ad
ou

= a^r^ (1 -rf + (r^ -1^) + a'^r'^ (r^ -1)^


fl^(r^-2r^+l) = o^(l-r^)^ = (fl-nr'') = (a-d)'^ = RHS.
3v2
=
Y

LHS = {ab +bc + cdf = (ax ar + arx ar^ + ar =«V(l+rAr^)


Re
nd

(ii)
RHS = {a^ +b^ + c^) + /)
Fi

=- + cP'r
= (1 + + r^) a\'^ (1 + + r‘^) = (1 + r“ + r^)
LHS = RHS.

EXAMPLE 5 If a, b, c, d are in G.P., prove that a+ b,b + c, c + d are also in G.P.


SOLUTION Let r be the common ratio of the G.P. a, b, c, d. Then, b - ar, c - ar ^ and d =ar^
a + b=a + ar = a{l+r), b + c=ar + ar = t?r (1 + r) and c + fr = rtr + ar ^ =nr^ (1 +r)
? 2

Now, (b + c)^ = ■ ar (1 + r) ● = A^(l +rf ■ (7 (1 + r) ■ ■ (1 + r) ■

= (fl +1?) (c + d) [●.● a + b = a {1 + r), and c + d = (1 + r)]


Hence, « + + c, c + d are in G.P.
9.38
APPLIED MATHEMATICS-XI

EXAMPLE 6 If a, h, c, d are in G.P., prove that a” + b'\ b” + c\ c" + d” arc also in G.P.
SOLUTION
Letr be the common ratio of the G.P. a, b, c, d. Then, b=ar, c=ar'^ and d=ar^.
<7''+^" (!+/■")
b'’ + c = 77 n /●ji +/r^' = a II r n ,1
II
IK II II 111 IK
(I + r ), c =a r (I + r )
Clearly, (&% =(«%/,») (e"+7f").
Hence, 77" + b’' + c", c" + / are in G.P.
EXAMPLE?
If a, b, c are in A.P. and x. y, z are in G.P., then show that ^
-a
.z
a- b
= I.

SOLUTION It is given that


a, b, c are in A.P. => 2i? = 77 + c

ow
2
a:, y, z are in G.P. => y =xz ...(h)
b-c c-a a-b
X
y z
[Using (ii)]
c-a c-a

2 2 a-b

e
b-c
X .r z z

Fl
re
c-a
b-c + c-a

F
2 a-b+
●y
= X z
ur
2b-{a-^) {c+a)-lb

r
X 2 "2 = x^o=l
fo [Using (i)]
EXAMPLES
Ifmth,nth and pth terms of a G.P. form three consecutive terms of a G.P. Prove that
ks
m
, n and pform three consecutive terms of an arithmetic sequence.
Yo
oo

SOLUTION Let 77 be the first term and r be the common ratio the G P Then
in-l n-l
p-t
= ar
and Up
eB

ar

It is given that 77,,,, 77,,, Op are in GP.


ur

=
ad

»- l,2 7W- I
(77r ) = (77r X77r^-l)
Yo

2 2/I-2
77 r
d

2)i-2
r = +P- 2
Re
in

2/7-2 = m + p - 2
F

2/7 = m + p
m, n, p are in AP.

EXERCISE 9.5
I. \ia,b,c are in G.P., prove that log a, log b, log c are in A.P.
I I
2. If 77, Z;, c are in G.P., prove that —- are in A.P.
logfl ' log;, m ' log^- m
3. Find k such that k + 9, k - 6 and 4 form three consecutive terms of a G.P.
4. Three numbers are in A.P. and their sum is 15. If 1,3,9 be added to them respectively, they
form a G.P. Find the numbers.
5. The sum of three numbers which are consecutive terms of an A.P. is 21. If the second
number is reduced by I and the third is increased by I, we obtain three consecutive terms of
a G.P. Find the numbers.
GEOMETRIC PROGRESSIONS 9.39

6. The sum of three numbers a, b, c in A.P. is 18. If a and b are each increased by 4 and c is
increased by 36, the new numbers form a G.P. Find a, b, c.
7. The sum of three numbers in G.P. is 56. If we subtract 1, 7,21 from these numbers in that
order, we obtain an A.P. Find the numbers.
8. If ft, c are in G.P., prove that:
(i) a(b^ +c^) = c(a‘^ +b'^) (ii) aVc^ J_ J_ J_
3%3 V3
= a

a c >

(iii)
(a + b + cf a +b + c 1 1 1

a
2
+b
,2
+ c
2
a -b + c (iv) ^
a b^-c^
(v) ia + 2b + 2c){a-2b + 2c) =a^ + 4c^.
9. If a, b, c, d are in G.P., prove that:
ab - cd
(i) (ii) {a + b + c + d)^ =(<? + b)^ + 2{b + c)^ +{c + d)^

w
,2 2
b -c b

F lo
(iii) (b + c) (b + fr) ={c + rt) (c + fr)
10. If a, b, c are in G.P., prove that the following are also in G.P.:
2.2 2 /... 3.3 3
(i) , b , c (ii) a ,b , c (iii) <7^ + b^, ab + be, b^ +

e
11. If fl,b, c, are in G.P., prove that:

Fre
(i) (a^ + b^), (b^ + c^), (c^ + d^) are in G.P. for
(ii) (fl^ -b\ (b^ - c^), (c^ - id^) are in G.P.
1 1
r
111
^ 2 ' ~7 2 ' -T~~^
+b^ b^ + c^ c^ + /
You
oks

(iv) (ij^ + b^ + c^), (i7b + bc + cd), (b^ + + d^) are in G.P.


eBo

12. If (fl -b), (b -c),{c - a) are in G.P., then prove that(fl + b + c) =3{nb + bc + ca)
2 2
a + ab + b b + a
13. If a, b, c are in G.P. then prove that:
our
ad

be + ca + ab c + b

14. If the 4*, 10^^ and 16*^ terms of a G.P. are a-, y and z respectively. Prove that y,z
are in G.P.
dY
Re

15. If a, b, c are in A.P. and a, b, d are in G.P., then prove that a, a -b, d - c are in G.P.
16. If pth, qth, rth and sth terms of an A.P. be in G.P., then prove thatp - q, q - r, r - s are in G.P.
Fin

1 1
17. If are three consecutive terms of an A.P., prove that a, b, c are the three
a + b ' 2b ' b + c
consecutive terms of a G.P.
ANSWERS

3. 0 or 16 6. a = -2, b =6, c = 14 or n = 46, b = 6, c = - 34 7. 8,16,32


8. 15,5,-5 or 3,5,7 9. 12,7,2 or 3,7,11 14. 2046

HINTS TO SELECTED PROBLEMS

1. a,b, care in G.P.=> b^ =ac => logb^ = log ac=> 2 logb =iog<7 + log c
2. fl, b, c are in G.P.
.-. b^ =flc=log,„ b^ =log,„ flc
2 log,,, b =log,„ + log,,, c
9.40 APPLIED MATHEMATICS-XI

2 1 1 1 1 1
=> = + ^ , are in A.P.
log^m log^m log,.?« log„m logj, m log^ m

3. It is given that k + 9, k - 6,4 arc in G.P.


=> {k -6)^ ={k + 9) X 4 k = 0,16.
14. Let the first term and common ratio of the G.P. be a and r respectively. It is given that
3 9 15
X = ar ,y=ar andz=ar
2 2 18 , 2 18
●● y r and xz=a r

2
y xz

X, y, zare in G.P.
16. Let the first term and the common difference of the AP be a and d respectively.
It is given that its p''^, and terms are in G.P. Let A be the first term and R be the

w
common ratio of the G.P. Then,
a + {p ~1) d = A ...(i)
a + {q-l) d =AR
n + {r-l)d=AR‘^
a + {s-l)d = AR^ F lo ...(ii)
...(hi)

...(iv)

e
Fre
Subtracting (ii) from (i), we get for
a + {p - 1) dV - ^ a + {q - 1) d\ = A ~AR
r
=> ip-q)d = A(l-R) ...(V)
You
oks

Subtracting (iii) from (ii), we get


eBo

■ a + (q -1) d ■ - ■ a + {r -1) d ■ =AR -AR^


iq-r) d = AR{1-R) ...(Vi)
ad
our

Subtracting (iv) from (iii), we get

■ a + (r-l)rfl-Jrt + (s-l)rfl = AR^-AR^


Re
dY

^ (r-s)d = AR^(l-R) ...(vii)


Fin

From (v), (vi) and (vii), we obtain that


iq-rf d^ = ip-q)d{r-s)d
=> = ip-
=> ip - i(J - ^)/ ir - s) are in G.P.
17. It is given that 1 1 ^ are in A.P.
a + b' 2b' b + c
2 1
+ = nc.
2b a +b b + c

9.7 INSERTION OF GEOMETRIC MEANS BETWEEN TWO GIVEN NUMBERS

GEOMETRIC MEANS Let a and b be tzvo given numbers. If n numbers G^,C2, are inserted
between a and b such that the sequence a, G-j, G2, G„, is a G.P. Then thenumbersGi, G2/ ●●● / G„ are
known as n geometric means (G.M. 's) between a and b.
9.41
GEOMETRIC PROGRESSIONS

GEOMETRIC MEAN Ifo single geometric mean G is inserted between tivo given numbers a andb, then G
is known as the geometric mean betzveen a and b.
Thus,
G is the G.M. between a and b. <=> a, C, b are in G.P. <=> G^ = rti? G = -Jab.
The geometric mean G between 4 and 9 is given by G = .^4x9 = 6.
The geometric mean G between -9 and -4 is given by G = ^-9 x - 4 = - 6.
MOTE If a and b are tzoo numbers of opposite signs, then geometric mean between them does rwt exist.
9.7.1 INSERTION OF GEOMETRIC MEANS BETWEEN TWO GIVEN NUMBERS

Let G|, G2 ..., Gfj be n geometric means between two given numbers a and b. Then,
Gi, G2,G,„ bis a G.P. consisting of {n + 2) terms. Let r be the common ratio of this G.P.

low
Then,
«+ 1
b - {n + 2)th term = ar
1

H +1 b ^b\n^\
r r

e
a a

re
x2/(/i+ 1) , yi/«+ 1
Gi ,G2 = ar rF2
...,G» ar
n
a
b

F
ar = a a

THEOREM If n geometric means arc inserted betzveen Izvo quantities, then the product of n geometric

r
means is the nth pozver of the single geometric mean betzveen the tzvo quantities. fo
u
Let Gj, G2, G3,..., G„ be n geometric means between two quantities a and b. Then,
ks
PROOF
Yo

a, G^, G2,..., G„, b is a G.P. Let r be the common ratio of this G.P. Tlien,
oo

1
B

^b]n+ 1 2 r- 3 _ II
r = and, G-j = ar, G2 - ar , G3 - ar ,..., G„ - ar .
re

1+ 2+ 3 + ... + »
G1.G2.G3 G„ = (ar)(nr^)(ar^)...(ar)=a' r
u
ad

i]{n+ 1)
Yo

] 2
»(»+ 1)
= a
n
r =a
n
<
^bV+ 1 ■
= a
n bI
= a n/2 ^n/2
d

a a
Re

\ - /
in

H
F

=1
= G" , where G = yfdj is the single geometric mean between a and b.
Q.E.D.

9.7.2 SOME IMPORTANT PROPERTIES OF ARITHMETIC AND GEOMETRIC MEANS


THEOREM 1 If A and G are respectively arithmetic and geometric means between two positive numbers
a and b, then A>G.
PROOF We have,

A =
2

A-G = ^^-4aE
2
=
a +
^-^^=l(fa-fb)^>0
2
A >G.
Q.E.D.
9.42 APPLIED MATHEMATICS-XI

THEOREM 2 If A and G are respectively arithmetic and geometric means between two positwe quantities
a and b, then the quadratic equation having a, b as its roots is - 2Ax + = 0.
PROOF We have,
and G =
2
The equation having a and b as its roots is
-x{a + b)+ab=0 or, x^ - 2Ax + =0 —-
2 and G = ^[ab
V A =
^
Q.E.D.
THEOREM 3 If A and G be the A.M. and G.M. betzueen tziyo positive numbers, then the numbers are

A±y^.

ow
I^ROOF The equation having its roots as the given numbers is

^ - 2Ax + G^=0=> X = 2A+^A^-4G^


X
2 = A±^A~-G^ Q.E.D.

e
re
ILLUSTRATIVE EXAMPLES

Frl
F
Type I INSERTION OF GEOMETRIC MEANS BETWEEN TWO NUMBERS
EXAMPLE 1 Insert 5 geometric means betzoeen 576 and 9.
ou
or
SOLUTION Let G^,G2,G3,G4,
be 5 geometric means between <7=576 and/? =9. Then,
576, G|, G2, G3, G4, C^, 9 is a G.P. with common ratior given by
kfs
1 1
1
( 9 5+ 1 { 1 6 =1 «+ 1
oo
r IT-
576 64 2
Using: r = -
a
Y
B

1
Gi = <?r=576x- = 288,
^ 2 G2^ = =576x14 = 144,
re

G3 = =576x18 =72, G4 = ar‘^ =576 x — = 36 and, G5 = ^=576x


1
oYu

ar = 18
16 32
ad

Hence, 288,144, 72, 36,18 are the required geometric means between 576 and 9.
d

I'ype II PROBLEMS BASED UPON ARITHMETIC AND GEOMETRIC MEANS


11+ 1
in

17+ 1
+ b
Re

a
EXAMPLE 2 Find the value ofn so that n 11 maybe the geometric mean betzueen a and b.
a+b
F

ti+ 1 77+ 1
a + b
SOLUTION It is given that is the G.M. between a and b.
a" + b
n

77+ 1 77+ 1
a + b
77
= 4^
a+b

1 1
77 + 77 +
77+ 1 77+ 1 2
a + b a

a
77+ 1
a
(«+t) 1/2
2 b a
1/2
b
("+ b
2
77+ 1

1 1

a {(1 0'+2^ (r
GEOMETRIC PROGRESSIONS 9.43

1 1

o a = b a 1/2 _^l/2 0, as fl 7^ & ]


1 1
\(»+ ^0
n a a 1
o =1 <=> <=> jj + — = 0o»=--
v*-
b y U b 2 2

EXAMPLE 3 Find two numbers whose arithmetic mean is 34 and the geometric mean is 16.
SOLUTION Let the two numbers ben and i?such thatr? > i?. It is given that AM and GM offland/?
are 34 and 16 respectively.
i.e.
2
= andV^=16

ow
rt + I>=68 and ab= 256

(a-b)^ = (a + b)^-4ab
(a-bf = (68)“-4x 256 = 3600
a ~b = 60 [v a>b > 0]

e
Solving +1; = 68 and a-b =60 simultaneously, we get a = 64 and b = 4.

re
Hence, the required numbers are 64 and 4.

Fl
F
ALITER Here, A = 34 and G =16.
So, the numbers are A + -G^' and A ~
ur
r
i.e. 34 + ^34^-16^ and 34-V34^-16^ or, 64 and 4. fo
EXAMPLE! If the A.M. and G.M. between t7vo numbers are in the ratio m : n, then prove that the
ks
J2 2 / 2 T
Yo
m -n :m-Jm -n .
oo

SOLUTION Let the two numbers be a and b. Let A and G be respectively the arithmetic and
eB

geometric means between a and b. Then,


A = ^ and G = ^ a + b = 2A and G^ = ab
2 ^
ur

The equation having a and b as its roots is


ad
Yo

- {a + b) X + ab = 0
or. -Y ^ -2A.v + G^ = 0 [Using (i)]
d

2A±JZ^ -4G^
Re
in

X = 2
2
X
= A±J^
F

I 2 2
So, the two numbers are a = A + JA - G and b = A -

It is given that
A :G =m:n=> A =Xm and G =>.» for some X
/ 2 2 I 2 2
Substituting the values of A and G in rt = A + JA -G and/j=A-JA -G , we get
,2 2 2 2
a
Xm + -X n a
m + m - n
2 2 2 2
^ a:b = m + 'm -n : m - 'm -n
^ Xm - -X^n^ ^ m - m
2
-n
2

EXAMPLE 5 If a is the A.M. of b and c and the tioo geometric means are Gi and G2, then prove that
cf + g| = 2abc.
SOLUTION It is given that a is the A.M. of b and c.
b + c
a b + c = 2a
2
9.44 APPLIED MATHEMATICS-XI

Since Gj andC2 are two^geometric means between b and c. Therefore, f), Gp G2, c is aG.P. with
common ratio r ~
b)
n1/3
=br =b - = and G2 = br^ = b ^^,1/3 ^.2/3
h h)
Gi = b^c and g| = bc'^
G] + g| = b^c + bc^ = bc{b + c)~ 2abc [Using (i)]
EXAMPLE 6 If one geometric mean G and tzvo arithmetic means A-^ and A2 be inserted between two
given quantities, prove that G^ - {2A-^ - A2) {2A2 - A{).
SOLUTION Let a and b be two given quantities. It is given that G is the geometric mean of a
and b

w
G - -Jab =i> G^ = ab ...{!)

It is also given that A], A2 are two arithmetic means between a and b. Therefore,, a, A-^, A2, b is
an A.P. with common difference d =
3

F lo
ee
b -a 2a + b 2{b-a) a + 2b
Ai = a + d = a + , A2 = a + 2d = a +

Fr
3 3 3 3

. 2a + b] (a + 2b'' ''2a + 1^ =b,


So, 2 - A2 = =a and 2A2 for = 3 3
3 ^ 3
ur
{2A] -A2M2A2 -A-^) = ab
(2A-i-A2){2A2-A^) = G^
s
[Using (i)]
ook
Yo

Type III PROBLEMS ON A.M. > G.M.


EXAMPLE? Ifx, y, z are distinct positive numbers, then prove that{x + y) (1/+2) (z + .r) > S.vi/z.
eB

SOLUTION Using A.M. > G.M., we obtain


.v + y i— V +z ,— , z + .r
— > ,/i/z and —
r

2 2
ad
ou

a: +
y>2.^xy, y+z>2^Jyz and z + x>2f^
Y

{x + y) (y +2) (2 + .y) > 21^ x 2^ x 2j^


Re

(x + y) (y+z) (z + .y) > 8.Y1/2.


nd

T 1 ■” V
EXAMPLES Ifx e R,find the minimum value of the expression 3+3 ' .
Fi

SOLUTION We know that A.M. > G.M.


l-.v
s'"+3
>^I3^x3
l-.v
for all X eR
2
l-x
s'" +3
> -f3 for all X 6 K
2
1-v
3-'‘ + 3 > 2^3 for all x e 7^
1-v
Hence, the minimum value of 3'^ + 3 for any x e P is 2-J3.
EXAMPLE 9 If a, b, c, d are four distinct positive numbers in A.P. then shozu that be > ad.
SOLUTION It is given that a, b, c, d are in A.P. Therefore, a, b, c arc in A.P.
b is the A.M. of a and c

The G.M. of a and c is fac.


A.M. of a and c > G.M. of a and c
GEOMETRIC PROGRESSIONS 9.45

=> b > -yjac


2
b >ac

Again, a, b, c, d are in A.P.


b, c, d are in A.P.
c is the A.M of and d.

The G.M. of b and d is -Jbd.


A.M. of b and d > G.M. of b and d
c >

>bd ...(ii)

ow
From (i) and (ii), we obtain
b^c'^ > (nc) {bd) => bc> ad.
EXAMPLE 10 If a, b, c, d are four distinct positive numbers in C.P. then shozv that a + d>b + c.
SOLUTION It is given that a, b, c, d are in G.P.

e
a, b, c are in G.P.

re
b is the G.M of a and c

rFl
F
a + c
But, A.M. of a and c is .
2

r
A.M. of a and c > G.M. of a and c
ou
a + c
>b
fo
ks
2
=> a+ c>2b ...(i)
oo

Again, a, b, c, d are in G.P.


Y

b, c, d are in G.P.
eB

c is the G.M. of b and d.


b + d
But, A.M. oib and d is
r

2
ou

A.M. of and d > G.M. offc and d


ad
Y

b + d
>c
2
d

b+d>2c ...(ii)
Re
in

Adding (i) and (ii), we obtain


a + c + b + d >2b + 2c
F

a+d>b+c

EXERCISE 9.6
1
1. Insert 6 geometric means between 27 and —.
81
1
2. Insert 5 geometric means between 16 and -.
, n . , 32 . 81
3. Insert 5 geometric means between — and —.2

4. Find the geometric means of the following pairs of numbers:


(i) 2 and 8 (ii) a^b and ab^ (iii)-8and-2
1
5. If a is the G.M. of 2 and — , find a.
4
9.46 APPLIED MATHEMATICS-XI

6. Find the two numbers whose A.M. is 25 and GM is 20.

7. Construct a quadratic in such that A.M. of its roots is A and G.M. is C.


8. The sum of two numbers is 6 times their geometric means, show that the numbers are in the
ratio(3 +2^2): (3-272).
9. If AM and GM of roots of a quadratic equation are 8 and 5 respectively, then obtain the
quadratic equation.
iO. If AM and GM of two positive numbers a and t are 10 and 8 respectively, find the numbers
11. Prove that the product of n geometric means between two quantities is equal to the nth
power of a geometric mean of those two quantities.
12. If the A.M. of two positive numbers a and b {n >b) is twice their geometric mean. Prove
that: a:b = (2 + 73):(2 - 73).
13. If one A.M., A and two geometric means and G2 inserted betw'een any two positive
G^

w
numbers, show that ^ ^ = 2A.
G2

1. 9,
3 9
J_
27
2. 8, 4,2,1,-
1
2
F lo
3.1^
3'
8,12,18, 27
ANSWERS

e
Fre
4.(i) 4 (ii) (iii) -4 6. 40, 7. - 2Ax + G^ = 0
for
9. -16X + 25 = 0 10. 4,16 or 16,4
r
You

HINTS TO SELECTED PROBLEMS


oks

8. Let the numbers be a and b. Further, let A and G denote their arithmetic and geometric
eBo

means respectively. It is given that


a +b
a +b — 6 G ^ = 3G=>A = 3G.
2
our
ad

Numbers n and b are roots of the quadratic equation


x'^ - X {a+ b)^ab = 0
dY
Re

or, .v^-2Aa- + G^ = 0
Fin

or, x'^ -6 Gx + G^ = 0 [●●● A = 3G]


6G±J36G^ -4G^
=> .V = 5
2

-r = 3 G ± 2 72 G
-r = 3 G ± 272 G
A- = (3±2 72)G
n =(3 + 272)Gand& = (3-2^)G
Hence,n:fc=(3 + 272):(3-272)
9. Let a and b be the roots of the quadratic equation. Then, the quadratic equation is
A ^ - (a + b) X + ab = 0 ...(i)
It is given that AM = 8 and GM = 5.
GEOMETRIC PROGRESSIONS 9.47

i.e. ^ 2
- 8 and VoF = 5 => a + b =16 and ab = 25

Substituting these values in (i), we obtain -16 x + 25 = 0 as the required equation.


10. We have,

^ ^ ^ = 10 and - 8=> a + b = 20 and nb = 64


2

Clearly, a and b are roots of the equation


~{n -i-b) X + nb = 0
or, .r ^ - 20 A- + 64 = 0

w
=> (a-16) (.A-4) = 0 => A = 4,16 =?■ (7 = 4,/’ = 16 or rt = 16,= 4.

MULTIPLE CHOICE QUESTIONS (MCQs)

Flo
Mark the correct alternative in each of the folloiving:

e
1. If in an infinite G.P., first term is equal to 10 times the sum of all successive terms, then its

re
common ratio is

F
(a) 1/10 (b) 1/11 (c) 1/9 (d) 1/20
2. If the first termof a G.P. a^, a2, a^, ■■■is unity such that 4 ^2 + 5 773 is least, then the common
ur
ratio of G.P. is

(a) -2/5 (b) -3/5 (c) 2/5


for (d) none of these
ks
b-c c-a _a-b ■_
Yo

3. If fi, b, c are in A.P. and a, y, 2 are in G.P., then the value of a y 2 is


oo

(a) 0 (b) 1 (c) .vy2 (d) .v" y" /


B

4. The first three of four given numbers are in G.P. and their last three are in A.P. with
re

common difference 6. If first and fourth numbers are equal, then the first number is
u

(a) 2 (b) 4 (c) 6 (d) 8


ad
Yo

5. If a, b, c are in G.P. and = c^^^, then xyz are in


(a) AP (b) GP (c) HP (d) none of these
d
Re

2
in

6. If S be the sum, P the product and R be the sum of the reciprocals of n terms of a GP, then P
is equal to
F

(a) S/R (b) R/S (c) (R/S)' (d) (S/R)'


7. The fractional value of 2.357 is

(a) 2355/1001 (b) 2379/997 (d) 2355/999 (d) none of these


8. If pth, qth and rth terms of an A.P. are in G.P., then the common ratio of this G.P. is
p-q q-r
(a) (b) (c) pqr (d) none of these
q-r p-q

9. The value of 9^^^-9^^^ ... to cc, is

(a) 1 (b) 3 (c) 9 (d) none of these


10. The sum of an infinite G.P. is 4 and the sum of the cubes of its terms is 92. The common ratio
of the original G.P. is
(a) 1/2 (b) 2/3 (c) 1/3 (d) -1/2
9.48 APPLIED MATHEMATICS-XI

11. If the sum of first two terms of an infinite GP is 1 and every term is twice the sum of all the
successive terms, then its first term is
(a) 1/3 (b) 2/3 (c) 1/4 (d) 3/4
12. The 7Zth term of a G.P. is 128 and the sum of its n terms is 225. If its common ratio is 2, then its
first term is
(a) 1 (b) 3 (c) 8 (d) none of these
13. If second term of a G.P. is 2 and the sum of its infinite terms is 8, then its first term is
(a) 1/4 (b) 1/2 (c) 2 (d) 4
14. If Cl, b, c are in G.P. and .v, i/ are AM's between a, b and b, c respectively, then
/ N 1 1 1 . . 1 1 2 1 2
(a) - + - = 2 (b) - + - (c) - + - = - (d) -+-=
->■' y X y 2 X \j a .V 1/ b

15. If A be one A.M. and p, Cj be two G.M.'s between two numbers, then 2 A is equal to

w
3 3 3 3 2 2
P
(a) (b)^ (c)
p
2 (d)^
pq P^

F lo
2
16. If p, be two A.M.'s and G be one G.M. between two numbers, then G =
fa) (2 p-i^)(p-2(7) (b) (2 p-c;) (2 7-p) (c) {2p - q) {p + 2 q) (d) none of these

ee
Fr
1 - .r
17. If J is positive, the sum to infinity of the series ■—— + ...1S

for ' (l + ,r)=’ (l + x)"*


(a) 1/2 (b) 3/4 (c) 1 (d) none of these
ur
18. If (4^)(4^)(4^)(4^^)....(4^"')=(0.0625)
-54
, the value of x is
s
ook

(a) 7 (b) 8 (c) 9 (d) 10


Yo

30
yi-1
eB

19. Given that .r > 0, the sum I equals


w = 1 .r + 1

.r .t + 1
r

(a) .V (b) .r+1 id)


ad
ou

2x +1 2a-+ 1

20. In a G.P. of even number of terms, the sum of all terms is five times the sum of the odd
Y

terms. The common ratio of the G.P. is


Re

(a)-|
nd

(c) 4 (d) none of these


Fi

3 3
y :
21. Let X be the A.M. and i/, z be two G.M.s between two positive numbers. Then, is
xyz
equal to
1
(a) 1 (b) 2
(c>5 (d) none of these

1/6 1/ 36
22. The product (32), (32) (32) ... to 30 is equal to
(a) 64 (b) 16 (c) 32 (d) 0
23. The two geometric means between the numbers 1 and 64 are
(a) 1 and 64 (b) 4 and 16 (c) 2 and 16 (d) 8 and 16
th th
24. In a G.P. if the(»? + n)"‘ term is p and {ni -n)‘" term is q, then its term is

(a) 0 (b) pq (c) ^ (d) ~ip + q)


GEOMETRIC PROGRESSIONS 9.49

25. Let Sbe the sum, P be the product and R be the sum of the reciprocals of 3 terms of a G.P.
then P^R ^ is equal to
(a) 1:1 (b) (Common ratio)" : 1
2 2
(c) (First term) (Common ratio) (d) none of these
26. If X, y, z are positive integers then value of the expression {x + y) (y + z){z + x) is
(a) =8xyz (b) > 8.TI/2 (c) <8.yt/2 (d) =4.\-yz
27. In a G.P. of positive terms, if any term is equal to the sum of the next two terms. Then the
common ratio of the G.P.
(a) sin 18 (b) 2 cos 18° (c) cos 18° (d) 2sinl8'=
28. The lengths of three unequal edges of a rectangular solid block are in G.P. The volume of
the block is 216 cm^ and the total surface area is 252 cmL The length of the longest edge is

ow
(a) 12 cm (b) 6 cm (c) 18 cm (d) 3 cm
l-.r
29. The minimum value of 4’' + 4 , .Y € R, is

(a) 2 (b) 4 (c) 1 (d) 0


30. If Y, 2y, 3zare in A.P., where the distinct numbers .r, y, z are in G.P., then the common ratio

e
of the G.P. is

Fl
re
1 1
(a) 3 (c) 2 (d)-

F
ur
ANSWERS

r
1. (b) 2. (a) 3. (b) 4. (d) 5. (a)
fo
6. (d) 7. (c) 8. (b)
ks
9. (b) 10. (a) 11. (d) 12. (a) 13. (d) 14. (d) 15. (a) 16. (b)
Yo

17. (a) 18. (b) 19. (b) 20. (c) 21. (b) 22. (a) 23. (b) 24. (c)
oo

25. (a) 26. (b) 27. (d) 28. (a) 29. (b) 30. (b)
B

FILL IN THE BLANKS TYPE QUESTIONS (FBQs)


re

1. The third term of a G.P. is the square of its first term. If its third term is 8, then the common
u

ratio is
ad
Yo

2. Tlie sum of first two terms of a G.P. is 1 and every term is twice the previous term. The first
term of the G.P. is
d

3. In a geometric progression consisting of positive terms, each term equals the sum of the
Re
in

next two terms. Then the common ratio of the progression is


4. If A-], A2 are the two arithmetic means between two numbers a and b and G|, G2 are two
F

'VlIi-
geometric means between same two numbers, then
G1G2
0. If A and G are the arithmetic and geometric means, respectively, of the roots of a quadratic
equation. Then, the equation is
6. If three positive real numbers n, b, c are in A.P. and abc = 4, then the minimum possible
value of b is

7.
The sum of infinity of the series 9-3 + 1- .^ + ., is

8. The value of the product (32) x (32)^^^^ x (32)^'^^'^ x to =0, is


2 45
9. If the sum of the series 3 + 3y + 3.v + to 00 is , than Y =
8

10. The product of u geometric means between a and b is


9.50 APPLIED MATHEMATICS-XI

-[-x
11. The minimum value of the expression 3''” + 3 , X €R, is
a-b
12. If a, h, c are in G.P. then the value of is equal to
b-c

13. The third term of a G.P. is 4, the product of the first five terms is
ANSWERS

1. 2 2. 1/3 3.^ 4. a +b
5. x^~2Ax + G^=0 6. 7.
27
2 nb 4

8. 64 9. Z 10. 2^3 12. - or- 13. 4^


15 b c

ow
VERY SHORT ANSWER QUESTIONS (VSAQs)
Answer each of the following questions in one word or one sentence or as per exact requirement of the
question:

e
1. If the fifth term of a G.P. is 2, then write the product of its 9 terms.

re
2. If {p + q)^ and (p - q)^^ terms of a G.P. are in and n respectively,

rFl then write its pth term,

F
x/2
3.
and log;, X are in G.P., then write the value of x.
9

r
4. If the sum of an infinite decreasing G.P. is 3 and the sum of the squares of its term is — , then
ou
write its first term and common difference.
fo
ks
5. If p^^, q^ and r
th
terms of a G.P. are x,y,z respectively, then write the value of
oo

X
‘1-^ ,7
V
Y
B

6. If A^, A2 be two AM's and G-^, G2 be two GM's between a and b, then find the value of
Aj + A2
re

Gj G2
ou
Y
ad

7. If second, third and sixth terms of an A.P. are consecutive terms of a G.P., write the
common ratio of the G.P.
d

8. Write the quadratic equation the arithmetic and geometric means of whose roots are A and
G respectively.
in
Re

9. Write the product of n geometric means between two numbers a and b.


F

2 3
10. If a = 1 + b + b +b +... to CO, then write b in terms of a given that 1| < 1.

ANSWERS
. a + b
1. 512 2. ^fmn 3. log^ (log;, fl) 4. a 5. 1 6.
ab
a-1
7. 3 8. x^ -2Ax + g'^ = 0 9. (ab)
n/2
10.
a
mmm 1 0

PERMUTATIONS

10.1 THE FACTORIAL

In this section, we shall introduce the term and notation of factorial which will be often used in
this chapter and the next three chapters.
FACTORIAL The continued product ofifrst n nnturnl numbers is called the "n factorial" and is denoted
bp n! or[^.
?z! = 1 X 2 X 3 X 4 X ... X (h -1) X ».

w
i.e

Thus, 31=1x2x3=6; 41=1x2x3x4 = 24, 5! = 1 x 2 x 3 x 4 x 5 = 120 etc.


Clearly, ;i! is defined for positive integers only.
ZERO FACTORIAL

F lo
As we will require zero factorial in the later sections of this chapter and it does not
make any sense to deifne it as the product of the integers from 1 to zero. So, zve deifne 0! = I.

e
Fre
NOTE Factorials of proper fractions or negative integers are not defined. Factorial n is deifned only for
whole numbers.
for
DEDUCTION We have,
)i! = lx 2x 3x4...x()i-l)xn = [lx2x 3x4...x (ji -1)]» = [(n -1) !l;i = »x (n -1)1
r
You

Thus, n! = nx (n-1)!
oks

Similarly,
eBo

nl=n (» -1) (n -2)\ = n {n -1) (h -2) {n - 3)! = »(n -1) {n -2) {n - 3) (fi -4)! and so on.
For example, 8! = 8 (7!), 51=5(4!) and 21 = 2(11)
ad
our

Following examples will illustrate the use of this property of factorial n.


ILLUSTRATIVE EXAMPLES
Re
dY

20! 10!
EXAMPLE 1 Compute: (i) (ii)
18! 6!4!
Fin

SOLUTION (i) We have,


20! 20(19!) _ 20x19x18! [v ;z! =/IX (»-!)!]
18!^ 18! 18!

= 20x19 = 380

10! 10x9x8x7x6! 10x9x8x7


(ii) = 210
614! 6!x(4x 3x2x1) 4x 3x 2x1

EXAMPLE 2 Convert the folloioing products into factorials:


(i) 6-7-8-9'10 (ii) 2-4'6-8-10
1-2-3-4-5-6-7-8-9-10 10!
SOLUTION (i) 6 ● 7 ● 8 ■ 9 ● 10 =
1-2-3-4'5 5!

(ii) 2 ■ 4 ■ 6 ■ 8 ■ 10 = (2 X 1) (2 X 2) (2 X 3) (2 X 4) (2 X 5) = 2^ X (1.2 ■ 3 ■ 4 ■ 5) = 2^ X 5!
10.2 APPLIED MATHEMATICS-XI

EXAMPLE 3 find the LCM of 41,51 and 61


SOLUTION We have, 5! = 5x4! and 6! = 6x5x4!
L.C.M. of 4!, 51, 6! = L.C.M. j4!, 5 x 41, 6 x 5 x 4!) ={4!) x 5 x 6 = 6! = 720
EXAMPLE 4
1 1 X . ,
+ — = — , fmd X.
9! 10! 11!

SOLUTION We have,

1 - X

9! "^10! ~ 11!
1 1 -T
+
9! 10x9! 11 X lOx 9!

ow
1 f 1 X 1
1 + X —

9! 10) Uixioj 9!

.V 11 .V
=> X = 11x11 = 121.
10 11x10 10 11X10

e
Fl
re
ALITER VVe have,

F
1 J_ = _X
9! 10!~11!
ur
Multiplying both sides by the LCM of 9!, 10! and 11! i.e. by 11!, we obtain
11! 11! X ,,, or
sf
9! 10! 11!
k
Yo
11x10x9! 11x10!
oo

= X
9! 10!
B

iixio+n =x

x = 121.
re

EXAMPLE 5
Find n, if:
u
ad

(i) {n + 2)[ = 2550 xn! (ii) (;j + l)! = 12 X (/?-!)!


Yo

SOLUTION (i) We have,


(rt + 2)! = 2550x/i!
d
Re

(ji + 2)(?! + l)x»! = 2550x;i!


in

(h + 2) X ()i + 1) = 2550
F

(h + 2) (/h-1) =51 x50 Expressing 2550 as the product of


two consecutive natura numbers

» + 2 = 51 or n +1 =50 [By comparing]


» = 49

(ii) We have.
(« + l)! = 12x(«-l)!
(;i + l)xnx{;!-l)!=12x(n-l)!
ji(j7 + 1)=12
(h + 1) =4x 3 ^ jj = 3 [By comparing]
?i! n!
EXAMPLE 6 If and are in the ratio 2 : l,find the value ofn.
2!(»-2)! 4!(«-4)!
10.3
PERMUTATIONS

SOLUTION We hnve.
n! n!
= 2:1
2!(m-2)! 4!(n-4)!
n! 4! (/I-4)! 2
X

2!(n-2)! n\ 1

4!(?f-4)! 2

2!0j-2)x(?j-3)x(;?-4)! 1

4x3x2! 2

2!(;t-2)(/i-3) 1

(»-2)(jj-3) = 6=:>0i-2)(ii-3) = 3x2=>n-2 = 3and»-3 = 2=>n=5

(2»-l)|2
(2n)! 1- 3-5..
II
EXAMPLE? Prove that:

w
»/!

F lo
SOLUTION We have,

(2)))! _ 1 ■2-3-4-5-6-7'8...(2»-2)(2»-l) (2h)


Hi n!

e
Fre
{l-3-5-7...(2n-l)}-{2-4-6-8...(2n-2)(2M)}
mI for
{1-3-5-7...(2/1-1)} 2” {l-2-3-4...(//-l)»}
r
nl
You
s

{l-3-5-7...(2/j-l)|-2"-?i!
ook

n
= {l-3-5-7...(2/j-l)}2
;i!
eB

EXERCISE 10.1
our

1. Compute;
ad

30! lll-lO!
(i) (ii) (hi) L.CM.(6!,7!, 8!)
28! 9!
dY

„ , 1 1 1 122
Re

2. Prove that — + — +
9! 10! 11! 11!
Fin

3. Find x in each of the following:


.... X 1 1
(iii) ^6! + 37!
X
1 1 X
(i) — + — = (ii) — = — + —
4! 5! 6! 10! 8! 9! 8!

4. Convert the following products into factorials:


(i) 5-6-7-8-9-10 (ii) 3-6-9-12-15-18
(iii) (/I + 1) (;i + 2) (// + 3)... (2/i) (iv)l-3-5-7-9...(2/1-1)
5. Which of the following are true:
(i) (2+ 3)! = 2!+ 3! (ii) (2x 3)! = 2!x 3!
6. Prove that :/i!(/i + 2) =//! + (// + 1)!
7. If(// + 2)! = 60[(/i-l)!],find//.
8. If(/i + l)! = 90[(n-l)!], find/i.
9. lf(//+ 3)! =56 [(/; + !)!], find/I.
10.4 APPLIED MATHEMATICS-XI

(2/2)! n\
10. If and are in the ratio 44 ; 3, find n.
31(2/2-3)1 2!(?7-2)I
11- Prove that:
/I I
(i) = /7 (/7-l)(/7-2)...(/7-(r-l))
(//-/-)!
//! 7l! (22 + 1)!
(ii) +
(/2-/')!r! (//-/● + l)!(r-l)l r!(?j-r + l)!

12. Prove that: (2» + l)!^, n


1 ● 3-5...(2/7-1)(2/7 + 1)
nl

_ANSWERS
10!
l.(i) 870 (ii) 100 (Hi) 8! 3. (i) 36 (ii) 100 (iii) 64 4. (i)
4!
(ii) 3^^ (6!)
(2/7)! (2//)!
(iii) (iv) 5. (i) False (ii) False 7. 3 8. 9 9. 5 10. 6

w
27! 2" 71!
HINTS TO SELECTED PROBLEMS
3. (i) We have.
1

4!"^5! 6!
1 X

F lo
ee
6! 6!

Fr
— + — = -V
4! 5! [Multiplying both sides by 61]
6x5x4! 6x5!
= -V => 6 X 5 + 6 = -t => .Y = 36
for
4! 5!
ur
(ii) We have.
JL-L L
s

10!"8l'*’9!
ook
Yo

10! 10!
X =
[Multiplying both sides by 10!]
eB

8! 9!
10x9x8! 10x9!
=> X =
8! 9!
our
ad

x = 10x9 + 10=100
(iii) We have.
1
6l'^7!"8!
dY
Re

8! 8!
— + — =x
[Multiplying both sides by 8!]
Fin

6! 7!
8x7x6! 8x7!
= -Y
6! 7!
=> 8x7+8 = x
x = 64

7. (/i + 2)! = 60(/i-l)l


=> {n + 2) (77 + 1) (77) (77 -1)! = 60 x (77 -1)!
(77 + 2) (/i + l) (/7) = 5x4x3 [Expressing 60 as the product of three consecutive integers]
^ 11 = 3
[On comparing two sides]
8. (77 + l)! = 90(/7-l)!
77 + 1 (7l) (77-l)! = 90(/7-l)!
=> (77 + 1)77 = 10 x9 [Writiiig 90 as the product of consecutive integers]
=> 77 = 9
PERMUTATIONS 10.5

9. (m+3)!=56(n + l)!
=> (n+3)(j! + 2){;M-l)!=56(;i + l)!
^ {n+3)(« + 2)=8x7 [Writing 56 as the product of consecutive integers]
=> ;i + 2= 7^?j=5

10.2 FUNDAMENTAL PRINCIPLES OF COUNTING

In this section, we shall discuss two fundamental principles viz. principle of addition and
principle of multiplication. These two principles will enable us to understand permutations and
combinations. In fact these two principles form the base of permutations and combinations.
FUNDAMENTAL PRINCIPLE OF MULTIPLICATION If there are tzvo jobs such that one of them can be
completed in m waps, and when it has been completed in any one of these m ways, second job can be
completed in n ways; then the two jobs in succession can be completed in m x n ways.
EXPLANATION If the first job is performed in any one of the m ways, we can associate with this

w
any one of the n ways of performing the second job: and thus there are n ways of performing the
two jobs without considering more than one way of performing the first; and so corresponding

F lo
to each of the m ways of performing the first job, we have n ways of performing the second job.
Hence, the number of ways in which the two jobs can be performed is m x n.
ILLUSTRATION 1 In a class there are 10 boys and 8 girls. The teacher zvants to select a boy and a girl to

ee
represent the class in a function. In hozv many ways can the teacher make this selection ?

Fr
SOLUTION Here the teacher is to perform two jobs:
(i) selecting a boy among 10 boys, and (ii) selecting a girl among 8 girls.
for
The first of these can be performed in 10 ways and the second in 8 ways. Therefore by the
ur
fundamental principle of multiplication, the required number of ways is 10 x 8 = 80.
s

REMARK The above principle can be extended for any finite number of jobs as stated belozv:
ook
Yo

If there are njobs Jj, ■■■ In Jjcan be performed independently in mizunys; i = 1,1,..., n.
Then the total number of zvays in lohich all the jobs can be performed isx nij x m3 x ... x ?n,,.
eB

FUNDAMENTAL PRINCIPLE OF ADDITION If there are tzoo jobs such that they can he performed
independently in m and n zvays respectively, then either of the tzvo jobs can be performed in (m + n)
our
ad

zvays.
ILLUSTRATION 2 In a class there are 10 boys and 8 girls. The teacher zvants to select either a boy or a
girl to represent the class in a function. In hozv many zvays the teacher can make this selection?
Y

SOLUTION Here the teacher is to perform either of the following two jobs :
Re
nd

(i) selecting a boy among 10 boys, or, (ii) selecting a girl among 8 girls.
Fi

The first of these can be performed in 10 ways and the second in 8 ways. Therefore, by
fundamental principle of addition either of the two jobs can be performed in (10 + 8) = 18 ways.
Hence, the teacher can make the selection of either a boy or a girl in 18 ways.
DIFFERENCE BETWEEN THE TWO PRINCIPLES As we haz’c discussed in the principle of multiplication
a job is divided or decomposed into a number of sub-jobs zvhich are unconnected to each other and the job is
said to be performed if each sub-job is performed. While in the principle of addition there arc a number of
independent jobs and zoe have to perform one of them. So, the total number of zvays of completing any one
of the sub-jobs is the sum of the number of ways of completing each sub-jobs.
ILLUSTRATIVE EXAMPLES

EXAMPLE 1 There are 3 candidates for a Classical, 5 for a Mathematical, and 4 for a Natural science
scholarship.
(i) In hozv many ways can these scholarships be azvarded?
(ii) In hozv many ways one of these scholarships be azvarded?
10.6 APPLIED MATHEMATICS-XI

SOLUTION Clearly, Classical scholarship can be awarded to any one of the three candidates. So,
there are 3 ways of awarding the Classical scholarship.
Similarly, Mathematical and Natural science scholarships can be awarded in 5 and 4 ways
respectively. So, by Fundamental Principle of multiplication.
Number of ways of awarding three scholarships = 3x5x4 = 60
By Fundamental Principle of addition.
Number of way of awarding one of the three scholarships = 3+ 5 + 4=12
EXAMPLE 2 A room has 6 doors. In hou> many ways can a man enter the room through one door and
come out through a different door ?
SOLUTION Clearly, a person can enter the room through any one of the six doors. So, there are
six ways of entering into the room. After entering into the room, the man can come out through
any one of the remaining five doors. So, he can come out through a different door in 5 ways.
Hence, the number of ways in which a man can enter a room through one door and come out

w
through a different door = 6 x 5 = 30.
EXAMPLE 3 The flag of a newly formed forum is in the form of three blocks, each to be coloured

possible ?

F lo
differently. If there are six different colours on the whole to choose from, how many such designs are

SOLUTION Since there are six colours to choose from, therefore, first block can be coloured in 6

ee
ways. Now, the second block can be coloured by any one of the remaining colours in five ways.

Fr
So, there are five ways to colour the second block. for
After colouring first two blocks only four colours are left. The third block can now be coloured
by any one of the remaining four colours. So, there are four ways to colour the third block.
r
Hence, by the fundamental principle of multiplication, the number of flag-designs is
You
s

6x5x4=120.
ook

EXAMPLE 4 Find the number of 4 letter words, with or without meaning, zvhich can beformed out of the
eB

letters of the word ROSE, when


(i) the repetition of the letters is not allowed, (ii) the repetition of the letters is allowed.
our

SOLUTION (i) The total number of words is same as the number of ways of filling in 4 vacant
ad

places i T] by the 4 letters. Tlie first place can be filled in 4 different ways by any one of the 4
letters R, O, S, E. Since the repetition of letters is not allowed. Therefore, the second place can be
filled in by any one of the remaining 3 letters in 3 different ways, following which the third place
dY
Re

can be filled in by the remaining 2 letters in 2 different ways; following which the fourth place
can be filled in by the remaining one letter in one way. Thus, by the fundamental principle of
Fin

counting the required number of ways is 4 x 3 x 2 x 1 = 24.


Hence, required number of words = 24.
(ii) If the repetition of the letters is allowed, then each of the 4 vacant places can be filled in
succession in 4 different ways.
Hence, required number of words = 4x4x4x4 = 256.
EXAMPLE 5 Given 4 flags of different colours, how many different signals can be generated, if a signal
requires the use of 2 flags one below the other?
SOLUTION The total number of signals is equal to the number of ways of filling in 2 vacant
places in succession by four flags of different colours. The upper cacant place can be filled
in 4 different ways by any one of the 4 flags; following which, the lower vacant place can be filled
in 3 different ways by any one of the remaining the different flags.
Hence, by the fundamental principle of multiplication, the required number of signals is
4x 3=12.
10.7
PERMUTATIONS

EXAMPLE 6 Find the number of different signals that can be generated by arranging at least 2 flags in
order (one below the other) on a vertical staff, iffive different flags are available.
SOLUTION Since a signal may consist of either 2 flags, 3 flags, 4 flags or 5 flags. Therefore,

Total number of signals = Number of 2 flags signals


+ Nnumber of 3 flags signals

+ Number of 4 flags signals

+ Number of 5 flags signals

= 5x4 + 5x4x 3+5x4x 3x2+5x4x 3x2x1


= 20 + 60 + 120 +120 = 320

In a monthly test, the teacher decides that there will be three questions, one from each of

w
EXAMPLE?

Exercises 7, 8 and 9 of the text book. Ifthere are 12 questions in Exercise 7,18 in Exercise 8 and 9 in
Exercise 9, in how many ways can three questions be selected ?

F lo
SOLUTION There are 12 questions in exercise 7. So, one question from exercise 7 can be selected
in 12 ways. Exercise 8 contains 18 questions. So, second question can be selected in 18 ways.
There are 9 questions in exercise 9. So, third question can be selected in 9 ways. Hence, three

e
Fre
questions can be selected in 12 x 18 x 9 = 1944 ways.
EXAMPLES How many words (with or without meaning) of three distinct letters of the English
for
alphabets are there ?
SOLUTION Here we have to fill up three places by distinct letters of the English alphabets. Since
r
there are 26 letters of the English alphabet, the first place can be filled by any of these letters. So,
You
ks

there are 26 ways of filling up the first place. Now, the second place can be filled up by any of the
o

remaining 25 letters. So, there are 25 ways of filling up the second place. After filling up the first
eBo

two places only 24 letters are left to fill up the third place. So, the third place can be filled in 24
ways.
Hence, the required number of words = 26 x 25 x 24 = 15600
our
ad

EXAMPLE 9 There are 6 multiple choice questions in an examination. How many sequence of answers
are
possible, if the first three questions have 4 choices each and the next three have 5 each ?
SOLUTION Here we have to perform 6 jobs of answering 6 multiple choice questions. Each one
dY
Re

of the first three questions can be answered in 4 ways and each one of the next three can be
answered in 5 different ways.
Fin

So, the total number of different sequences = 4x4x4x5x5x5 = 8000


EXAMPLE 10 Find the total number of ways ofansiuering 5 objective type questions, each question
having 4 choices.
SOLUTION Since each question can be answered in 4 ways. So, the total number of ways of
answering 5 questions is 4x4x4x4x4 = 4^.
EXAMPLE n How many three-digit numbers can beformed without using the digits 0,2,3,4, Sand 6 ?
SOLUTION We have to determine the total number of three digit numbers formed by using the

digits 1,7, 8,9. Clearly, the repetition of digits is allowed.


A three digit number has three places viz. imits's, ten's and hundred's. Unit's place can be filled
by any of the digits 1, 7,8,9. So, unit's place can be filled in 4 ways. Similarly, each one of the
ten's and hundred's place can be filled in 4 ways.
Total number of required numbers = 4x4x4 = 64.
10.8 APPLIED MATHEMATICS-XI

liXAMPLE 12 Holo many numbers are there between 100 and 1000 in which all the digits are distinct ?
SOLUTION A number between 100 and 1000 has three digits. So, we have to form all possible
3-digit numbers with distinct digits. We cannot have 0 at the hundred's place. So, the hundred's
place can be filled with any of the9 digits 1,2,3,..., 9. So, there are 9 ways of filling the hundred's
place.
Now, 9 digits are left including 0. So, ten's place can be filled with any of the remaining 9 digits
in 9 ways. Now, the unit's place can be filled with in any of the remaining 8 digits. So, there are 8
ways of filling the unit's place.
Hence, the total number of required numbers = 9x 9x 8= 648.
EXAMPLE 13 biozv mam/ numbers are there behceen 100and WOO such that every digit is either 2 or 9 ?

w
SOLUTION Every number between 100 and 1000 consists of three digits. So, we have to
determine the total number of three digit numbers such that every digit is either 2 or 9.
Clearly, each one of the unit's, ten's and hundred's place can be filled in 2 ways.
So, the total number of required numbers = 2x2x2 = 8.

o
EXAMPLE 14 How many numbers are there between 100 and 1000 such that 1 is in the unit's place.

e
re
SOLUTION Every number between 100 and 1000 is a three digit number. So, we have to form

rFl
3-digit numbers with 7 at the unit's place by using the digits 0,1,2,..., 9. Clearly, repetition of

F
digits is allowed. The hundred's place can be filled with any of the digits from 1 to 9 (zero cannot
be there at hundred's place). So, hundred's place can be filled in 9 ways. Now, the ten's place can

r
be filled with any of the digits from 0 to 9. So, ten's place can be filled in 10 ways. Since all the
ou
fo
numbers have digit 7 at the unit's place, so, unit's place can be filled in only one way. Hence, by
the fundamental principle of counting the total number of numbers between 100 and 1000
ks
having 7 at the unit's place = 9x10x1= 90.
oo

EXAMPLE 15 A gentleman has 6 friends to invite. In how many ways can he send invitation cards to
them, if he has three servants to carry the cards ?
Y
eB

SOLUTION Since a card can be sent by any one of the three servants, so the number of ways of
sending the invitation card to the first friend = 3. Similarly, invitation cards can be sent to each of
ur

the six friends in 3 ways.


So, the required number of ways = 3x3x3x3x3x3 = 3^
ad

=729.
Yo

EXAMPLE 16 How many three-digit numbers more than 600 can beformed by using the digits 2,3,4,6, 7.
SOLUTION Clearly, repetition of digits is allowed. Since a three-digit number greater than 600
d

will have 6 or 7 at hundred's place. So, hundred's place can be filled in 2 ways. Each of the ten's
Re
in

and one's place can be filled in 5 ways.


F

Hence, total number of required numbers = 2 x 5 x 5 = 50.


EXAMPLE 17 Hozu many numbers between 3000 and 4000 can beformed from the digits 3,4,5,6, 7 and
8, no digit being repeated in any number ?
SOLUTION Clearly, a number between 3000 and 4000 must have 3 at thousand's place. So,
thousand s place can be filled in only one way. Now, hundred's place can be filled in 5 ways.
Since repetition of digits is not allowed so ten's and one's places can be filled in 4 and 3 ways
respectively.
So, total number of required numbers = lx5x4x 3 = 60.
EXAMPLE 18 Hozv many numbers divisible by 5 and lying betzveen 4000 and 5000 can be formed from
the digits 4, 5, 6, 7 and 8.
SOLUTION Clearly, a number between 4000 and 5000 must have 4 at thousand's place. Since the
number is divisible by 5 it must have 5 at unit's place. Now, each of the remaining places (viz.
hundred's and ten's) can be filled in 5 ways.
Hence, total number of required numbers =1x5x5x1=25.
10.9
PERMUTATIONS

EXAMPLE19 How many four-digit numbers can beformed using the digits 0,1,2,3,4,5 if(i) repetition
of digits is not allozoed (ii) repetition of digits is aUozved?
SOLUTION (i) In a four-digit number 0 cannot appear in the thousand's place. So, thousand's
place can be filled in 5 ways. (viz. 1,2,3,4,5). Since repetition of digits is not allowed and 0 can
be used at hundred's place, so hundred's place can be filled in 5 ways.
Now, any one of the remaining four digits can be used to fill up ten's place. So, ten's place can be
filled in 4 ways. One's place can be filled from the remaining three digits in 3 ways.
Hence, the required number of numbers =5x5x4x 3 = 300.
(ii) For a four-digit number we have to fill up four places and 0 cannot appear in the thousand s
place. So, thousand's place can be filled in 5 ways. Since repetition of digits is allowed, so each of
the remaining three places viz. hundred's, ten's and one's can be filled in 6 ways.
Hence, the required number of numbers =5x6x6x6 =1080..
EXAMPLE 20 Hozv many numbers greater than 1000, but not greater than 4000 can be formed luifh the

w
digits 0,1,2, 3, 4if: (i) repetition of digits is allowed ? (ii) repetition of digits is not allozoed ?
SOLUTION (i) Every number between 1000 and 4000 is a four digit number. In thousand's place
we can

F lo
put either 1 or 2 or 3 but not 4. So, thousand's place can be filled in 3 ways. Since
repetition of digits is allowed, so each of the hundred's, ten's and one's place can be filled in 5
ways. So, total number of numbers between 1000 and 4000, including 1000 and excludiiig 4000 is

ee
3x5x5x5 = 375. But, we have to find the total number of numbers greater than 1000 but not

Fr
greater than 4000.
for
Hence, required number of numbers = 375 + 1 (for 4000) -1 (for 1000) = 375.
(ii) As discussed above thousand's place can be filled in 3 ways. Since repetition of digits is not
ur
allowed, so, hundred's place can be filled from the remaining digits in 4 ways. Now, three digits
s

are left, so ten's place can be filled in 3 ways. One's place can be filled in 2 ways.
ook
Yo

Hence, required number of numbers = 3x4x3x2=72.


eB

EXAMPLE 21 How many three digit odd numbers can beformed by using the digits 1,2,3,4,5,6 if:
(i) the repetition of digits is not allozoed ? (ii) the repetition of digits is allozoed ?
SOLUTION For a number to be odd, we must have 1, 3 or 5 at the unit's place. So, there are 3
our
ad

ways of filling the unit's place,


(i) Since the repetition of digits is not allowed, the ten's place can be filled with any of the
remaining 5 digits in 5 ways. Now, four digits are left. So, hundred's place can be filled in 4
Y
Re

ways.
nd

So, required number of numbers = 3x5x4 = 60


Fi

(ii) Since the repetition of digits is allowed, so each of the ten's and hundred's place can be filled
in 6 ways.
Hence, required number of numbers = 3x6x6= 108.
EXAMPLE 22 How many 3-digit ei’en numbers can be formed from the digits 1,2,3,4,5, 6 if the digits
can be jvpeated?
SOLUTION For a number to be even, we must have 2,4 or 6 at the unit's place. So, there are 3
ways to fill in the unit's place. Since digits can be repeated, so each of the ten's and hundred's
place can be filled in 6 ways.
Hence, required number of numbers = 3x6x6= 108.
EXAMPLE 23 Hozo many nutnbers of 3 digits can be formed zoith the digits 1,2,3,4,5 zohen digits may
be repeated ?
SOLUTION The unit's place can be filled in 5 ways. Since, the repetition of digits is allowed,
therefore ten's place can be filled in 5 ways and hundred's place can also he filled in 5 ways.
10.10 APPLIED MATHEMATICS-XI

Therefore, by the fundamental principle of counting, the required number of three digit
numbers = 5x5x5=125.

EXAMPLE 24 Find the number of numbers of 5 digits that can beformed ivith the digits 0,1, 2,3,4 if the
digits can be repeated in the same number.
SOLUTION In a five digit number 0 cannot be put in ten thousand's place. So, the number of
ways of filling up the ten thousand's place = 4.
Since the repetition of digits is allowed, therefore each of the other places can be filled in 5 ways.
So, the required number of numbers = 4x5x5x5x5 = 2500.
EXAMPLE 25 How many 4-digit numbers are there, when a digit may be repeated any number of times ?
SOLUTION In a four digit number 0 cannot be placed at thousand's place. So, thousand's place
can be filled with any digit from 1 to 9. Thus, thousand's place can be filled in 9 ways.
Since repetition of digits is allowed, therefore each of the remaining 3 places can be filled in 10
ways by using the digits from 0 to 9.

w
Hence, the required number of numbers = 9 x 10 x 10 x 10 = 9000.

allowed (ii) repetition is allowed ?

F lo
EXAMPLE 26 Hozo many three-letter words can be formed using a, b, c, d, e if: (i) repetition is not

SOLUTION (i) Clearly, the total number of three-letter words is equal to the number of ways of

ree
filling three places. First place can be filled in 5 ways. Now, four letters are left. So, the second

F
place can be filled in 4 ways. Since the repetition of letters is not allowed, so the third place can
be filled from any one of the remaining 3 digits in 3 ways.
for
Hence, total number of words =5 x 4 x 3 = 60.
r
(ii) In this case repetition of letters is allowed, so each of the three places can be filled in 5 ways.
You
oks

Hence, total number of words =5x5x5 =125.


eBo

EXAMPLE 27 In how many ways can the following prizes be given away to a class of 30 students, first
and second in Mathematics, first and second in Physics, first in Chemistry and first in English ?
SOLUTION Here we have to give prizes in four subjects and the process of distributing prizes
ad
our

can be completed by giving prizes in the four subjects.


First and second prizes can be given in Mathematics in {30 x 29) ways.
First and second prizes can be given in Physics in (30 x 29) ways.
Re
dY

First prize can be given in Chemistry in 30 ways.


Fin

First prize can be given in English in 30 ways.


Hence, the number of ways to give prizes in all the four subjects
= (30x29)x(30x29)x 30x 30=6.8121x10^
EXAMPl.E 28 In Iwiv many ways 5 rings of different types can be worn in 4 ifngers ?
SOLUTION The first ring can be worn in any of the 4 fingers. So, there are 4 ways of wearing it.
Similarly, each one of the other rings can be worn in 4 ways.
Hence, the requisite number of ways = 4x4x4x4x4=4^.
EXAMPLE 29 In Iww many zvays can 5 letters be posted in 4 letter boxes ?
SOLUTION Since each letter can be posted in any one of the four letter boxes. So, a letter can be
posted in 4 ways. Since there are 5 letters and each letter can be posted in 4 ways. So, total
number of ways in which all the five letters can be posted is4x4x4x4x4 = 4^.
10.11
PERMUTATIONS

EXAMPLE 30 Five persons entered the lift cabin on the ground floor of an 8-floor house. Suppose each of
them can leave the cabin independailli/ at am/ floor beginning with the first. Find the total number of zvai/s
in which each of the ifve persons can leave the cabin (i) at any one of the 7foors (ii) at different floors.
SOLUTION Suppose A-^, A2, At,, A4, A^ are five persons,
(i) Ai can leave the cabin at any of the seven floors. So, Ai can leave the cabin in 7 ways.
Similarly, each of /I2. 7I3, A4, Ag can leave the cabin in 7 ways. Thus, the total number of ways
in which each of the five persons can leave the cabin at any of the seven floors is
7 x7 x7 x7 x7 =7^.
(ii) A-[ can leave the cabin at any of the seven floors. So, A^ can leave the cabin in 7 ways. Now,
A2 can leave the cabin at any of the remaining 6 floors. So, A2 can leave the cabin in 6 ways.
Similarly, A3, A4, and A5 can leave the cabin in 5, 4 and 3 ways respectively. Thus, the total
number of ways in which each of the five persons can leave the cabin at different floors is
7x6>;5x4x 3 — 2520.
EXAMPLES! A mint prepares metallic calendars specifl/ing months, dates and days in the form of
types of February calendars should it prepare to

w
monthly sheets (one plate for each month). How many
serve for all the possibilities in the future years ?
SOLUTION The mint has to perform two jobs, viz.

F lo
(i) selecting the number of days in the February month (there can be 28 days or 29 days), and
(ii) selecting the first day of the February month.

e
The first job can be completed in 2 ways while the second can be performed in 7 ways by

Fre
selecting any one of the seven days of a week. for
Thus, the required number of plates = 2x7 =14.
EXAMPLE 32 For a set offive true/false questions, no student has written all correct answers, and no tzvo
students have given the same sequence ofanszvers. What is the maximum number of students in the class,
r
You

for this to be possible ?


oks

SOLUTION Since a true/false type question can be answered in 2 ways either by marking it
eBo

true or false. So, there are 2 ways of answering each of the 5 questions.
Total number of different sequences of answers = 2x2x2x2x2 = 2 =32.
Out of these 32 sequences of answers there is only one sequence of answering all the five
ad
our

questions correctly. But no student has written all the correct answers and different students
have given different sequences of answers.
Maximum number of students in the class
dY
Re

= Number of sequences except one sequence in which all answers are correct = 32 -1 = 31
EXAMPLE 33 How many numbers are there between 100 and 1000 such that at least one of their
Fin

digits is 7?
SOLUTION Clearly, a number between 100 and 1000 has 3-digits
Total number of 3-digit numbers having at least one of their digits as 7
= (Total number of three-digit numbers) - (Total number of 3-digit numbers in which 7
does not appear at all)
Total number of three-digit numbers : We have to form three-digit numbers by usmg the digits 0,1,
2,3,..., 9. Clearly, hundred's place can be filled in 9 ways and each of the ten's and one's place
can be filled in 10 ways.
So, total number of 3-digit number = 9 x 10 x 10 = 900.
Total number of three-digit number in zvhich 7 does not appear at all : Here we have to form
three-digit numbers by using tlie digits 0 to 9, except 7. So, hundred's place can be filled in 8
ways and each of the ten's and one's place can be filled in 9 ways. So, total number of three-digit
numbers in which 7 does not appear at all is 8 x 9 x 9.
10.12
APPLIED MATHEMATICS-XI

Hence, total number of 3-digit numbers having at least one of their digits as 7 is
9x10x10-8x9x9=252.

IXAMPLE 34 Hozv many numbers are there behveen 100 and WOO zuhich have exactly one of their digits
as 71

SOLUTION A number between 100 and 1000 contains 3-digits. So, we have to form 3-digit
numbers having exactly one of their digits as 7. Such type of numbers can be divided into three

w
types:
(i) Those numbers that have 7 in the unit's place but not in any other place,
(ii) Those numbers that have 7 in the ten's place but not in any other place,
(iii) Those numbers that have 7 in the hundred's place but not in any other place.

e
Required number of numbers is the total number of these three types of numbers.

e
We shall now count these three types of numbers separately.

or
r
(i) Those three-digit numbers that have 7 in the unit's place but not in any other place.

F
The hundred's place can have any one of the digits from 0 to 9 except 0 and 7. So, hundred's
place can be filled in 8 ways. The ten's place can have any one of the digits from 0 to 9 except 7.
oF
ul
So, the number of ways the ten's place can be filled is 9. The unit's place has 7. So, it can be filled
in only one way.

rs
Thus, there are 8 x 9 x 1 =72 numbers of the first kind.

ko
(ii) Those three-digit numbers that have 7 in the ten's place hut not in any other place.
The number of ways to fill the hundred's place = 8
of
(by any one of the digits from 1,2,3,4,5,6,8,9)
The number of ways to fill the ten's place = 1 (by 7 only)
o
Y
The number of ways to fill the one's place = 9 (by any one of the digits 0,1,2, 3,4,5, 6, 8,9)
B

Thus, there are 8 x 1 x 9 = 72 numbers of the second kind.


Y

(iii) Those three-digit numbers that have 7 in the hundred's place but not at any other place.
er

In this case, the hundred's place can be filled only in one way and each of the ten's and one's
place can be filled in 9 ways.
u

So, there are 1 x 9 x 9 = 81 numbers of the third kind.


od
ad

Hence, the total number of required type of numbers = 72 + 72 + 81 = 225.


in

EXAMI’LE 35 A telegraph has 5 arms and each arm is capable of 4 distinct positions, including the
position of rest. What is the total number of signals that can be made 1
SOLUTION Since each arm can be kept in 4 positions and a signal is possible when all the 5 arms
Re
F

are simultaneously placed in positions.


Total number of ways of placing the arms =4x4x4x4x4=4^.
But, this includes one inadmissible case, when all the arms are in the position of rest and then no
signal can be made.
Hence, required number of signals = (4^ -1) =1023.
EXAMI’LE 36 In how many zvays can 3 prizes be distributed among 4 boys, zvhen
(i) no boy gets more than one prize ? (ii) a boy may get any number of prizes ? (Hi) no boy gets all the
prizes ?
SOLUTION (i) The first prize can be given away in 4 ways as it may be given to any one of the 4
boys. The second prize can be given away in 3 ways, because the boy who got the first prize
cannot receive the second prize. The third prize can be given away to anyone of the remaining 2
boys in 2 ways. So, the number of ways in which all the prizes can be given away = 4 x 3 x 2 = 24.
ALITER The total number of ways is the number of arrangements of 4 taken 3 at a time. So, the
requisite number of ways = = 4! = 24.
PERMUTATIONS 10.13

(ii) The first prize can be given away in 4 ways as it may be given to anyone of the 4 boys. Tlie
second prize can also be given away in 4 ways, since it may be obtained by the boy who has
already received a prize. Similarly, third prize can be given away in 4 ways.
Hence, the number of ways in which all the prizes can be given away = 4x4x4 = 4 =64.
(iii) Since any one of the 4 boys may get all the prizes. So, the number of ways in which a boy
gets all the 3 prizes is 4.
So, the number of ways in which a boy does not get all the prizes = 64 - 4 = 60.
f;XAMl’U-:37 Find the total number of zuai/s in which n distinct objects can be put info tzoo different
boxes.

SOLUTION Let the two boxes be and B2. Weobserve thatthere are twochoices for eachof the
n objects. Therefore, by fundamental principle of counting
Total number of ways = 2 x 2 x .... x 2 = 2”

w
n - times

EXAMlM.l: .38
Find the total number of loaps in which n-distinct objects can be put into tzoo different

Flo
boxes so that no box remains empty.
SOLUTION Each object can be put either in box B-j (say) or in box 62 (say). So, there are two

ee
choices for each of the n objects. Therefore, the number of choices for n distinct objects is

Fr
2 X 2 X ... X 2 = 2". Two of these choices correspond to either the first or the second box being
II - times

empty. Thus, there are 2” - 2 ways in which neither box is empty. for
ur
EXAMPLE 39
By using the digits 0, 1, 2, 3, 4 and 5 (repetitions not allozued) numbers are formed by
using any number of digits. Find the total number of non-zero numbers that can be formed.
k s
Yo

SOLUTION Required number of numbers


oo

= Number of 1 digit number + No. of 2 digit numbers + ... + Number of 6 digit numbers
eB

= 5 + 5x5+5x5x4 + 5x5x4x 3+5x5x4x 3x2 + 5x5x4x 3x2x1


= 5 + 25 + 100 + 300 + 600 + 600 = 1630.
r

EXERCISE 10.2
ou
ad

1. In a class there are 27 boys and 14 girls. The teacher wants to select 1 boy and 1 girl to
Y

represent the class in a function. In how many ways can the teacher make this selection?
A person wants to buy one fountain pen, one ball pen and one pencil from a stationery
Re
nd

shop. If there are 10 formtain pen varieties, 12 ball pen varieties and 5 pencil varieties, in
Fi

how many ways can he select these articles?


3. From Goa to Bombay there are two roots; air, and sea. From Bombay to Delhi there are three
routes;-air, rail and road. From Goa to Delhi via Bombay, how many kinds of routes arc
there?

4. A mint prepares metallic calenders specifying months, dates and days in the form of
monthly sheets (one plate for each month). How many types of calendars should it prepare
to serve for all the possibilities in future years?
5. There are four parcels and five post-offices. In how many different ways can the parcels be
sent by registered post?
6. A coin is tossed five times and outcomes are recorded. How many possible outcomes are
there?

7. In how many ways can an examinee answer a set of ten true/false type questions?
10.14 APPLIED MATHEMATICS-XI

8. A letter lock consists of three rings each marked with 10 different letters. In how many
ways it is possible to make an unsuccessful attempt to open the lock?
9. There are 6 multiple choice questions in an examination. How many sequences of answers
are possible, if the first three questions have 4 choices each and the next three have 2 each?
10. There are 5 books on Mathematics and 6 books on Physics in a book shop. In how many
ways can a student buy : (i) a Mathematics book and a Physics book (ii) either a
Mathematics book or a Physics book?
11. Given 7 flags of different colours, how many different signals can be generated if a signal
requires the use of two flags, one below the other?
12. A team consists of 6 boys and 4 girls and other has 5 boys and 3 girls. How many single
matches can be arranged between the two teams when a boy plays against a boy and a girl

w
plays against a girl?
13.
Twelve students compete in a race. In how many ways first three pri2es be given?
14.
How many A.P.'s with 10 terms are there whose first term is in the set {1, 2, 3] and whose

o
commondifference is in the set {1,2,3,4,5}?

e
15. From among the 36 teachers in a college, one principal, one vice-principal and the

re
rFl
teacher-incharge are to be appointed. In how many ways can this be done?

F
!6. How many three-digit numbers are there with no digit repeated?
17. How many three-digit numbers are there?

r
18. How many three-digit odd numbers are there?
ou
19. fo
How many different five-digit number licence plates can be made if
ks
(i) first digit cannot be zero and the repetition of digits is not allowed,
(ii) the first-digit cannot be zero, but the repetition of digits is allowed?
oo

20. How many four-digit numbers can be formed with the digits 3,5, 7,8,9 which are greater
Y

than 7000, if repetition of digits is not allowed?


eB

21. How many four-digit numbers can be formed with the digits 3,5,7,8,9 which are greater
than 8000, if repetition of digits is not allowed?
ur

In how many ways can six persons be seated in a row?


T7
ad

How many 9-digit numbers of different digits can be formed?


Yo

24.
How many odd numbers less than 1000 can be formed by using the digits 0, 3, 5, 7 when
repetition of digits is not allowed?
d
Re
in

25. How many 3-digit numbers are there, with distinct digits, with each digit odd?
26.
How many different numbers of six digits each can be formed from the digits 4,5,6,7,8,9
F

when repetition of digits is not allowed?


27. How many different numbers of six digits can be formed from the digits 3,1,7,0, 9,5 when
repetition of digits is not allowed?
28.
How many four digit different numbers, greater than 5000 can be formed with the digits 1,
2,5, 9,0 when repetition of digits is not allowed?
29.
Serial numbers for an item produced in a factory are to be made using two letters followed
by four digits (0 to 9). If the letters are to be taken from six letters of English alphabet
without repetition and the digits are also not repeated in a serial number, how many serial
numbers are possible?
30. A number lock on a suitcase has 3 wheels each labelled with ten digits 0 to 9. If opening of
the lock is a particular sequence of three digits with no repeats, how many such sequences
will be possible? Also, find the number of unsuccessful attempts to open the lock.
PERMUTATIONS 10.15

31. A customer forgets a four-digit code for an Automatic Teller Machine (ATM) in a bank.
However, he remembers that this code consists of digits 3, 5, 6 and 9. Find the largest
possible number of trials necessary to obtain the correct code.
32. In how many ways can three jobs I, II and III be assigned to three persons A, B and C if one
person is assigned only one job and all are capable of doing each job?
33. How many four digit natural numbers not exceeding 4321 can be formed with the digits 1,
2,3 and 4, if the digits can repeat?
34. How many numbers of six digits can be formed from the digits 0,1,3,5, 7 and 9 when no
digit is repeated? How many of them are divisible by 10?
35. If three six faced die each marked with numbers 1 to 6 on six faces, are thrown find the total
number of possible outcomes.
36. A coin is tossed three times and the outcomes are recorded. How many possible outcomes

w
are there ? How many possible outcomes if the coin is tossed four times? Five times? n
times?

F lo
37. How many numbers of four digits can be formed with the digits 1,2,3,4,5 if the digits can
be repeated in the same number?

ee
38. How many three digit numbers can be formed by using the digits 0, 1, 3, 5, 7 while each

Fr
digit may be repeated any number of times?
39. How many natural numbers less than 1000 can be formed from the digits 0,1,2,3,4,5 when
a
digit may be repeated any number of times? for
ur
40. How many five digit telephone numbers can be constructed using the digits 0 to 9. If each
number starts with 67 and no digit appears more than once?
s
ook
Yo

41. Find the number of ways in which 8 distinct toys can be distributed among 5 children.
42. Find the number of ways in which one can post 5 letters in 7 letter boxes.
eB

43. Three dice are rolled. Find the number of possible outcomes in which at least one die
shows 5.
r
ou
ad

44. Find the total number of ways in which 20 balls can be put into 5 boxes so that first box
contains just one ball.
Y

45. In how many ways can 5 different balls be distributed among three boxes?
Re
nd

46. In how many ways can 7 letters be posted in 4 letter boxes?


47. In how many ways can 4 prizes be distributed among 5 students, when
Fi

(i) no student gets more than one prize?


(ii) a student may get any number of prizes?
(iii) no student gets all the prizes?
48. There are 10 lamps in a hall. Each one of them can be switched on independently. Find the
number of ways in which the hall can be illuminated.
ANSWERS

1. 378 2. 600 3. 6 4. 14 5. 625 6. 32 7. 1024 8. 999

11. 42 12. 42 13. 1320 14. 15 15. 42840


9. 512 10. (i)30 (ii) 11
16. 648 17. 900 18. 450 19. (i) 27216 (ii) 90000 20. 72 21. 48

25. 60 26. 720 27. 600 28. 48 29. 151200


22. 720 23. 9(9!) 24. 21
10.16 APPLIED MATHEMATICS-XI

30. 720,719 31. 24 32. 6 33. 229 34. 600,120 35. 216
36. 8,16, 2" 37. 625 38. 100 39. 215 40. 336 41. 5 42. 7^
19
43. 91 44. 20x4 45. 243 46. 4^ 47. (i) 5! (ii) 625 (iii) 620
48. 2^^-l.
HINTS TO SELECTED PROBLEMS

1. No. of ways = 27 X 14.


2. Required number of ways = 10x12x5 = 600.
3. No of routes = 2 x 3 = 6.
4. Total number of calendars =7x2=14.
5. Since a parcel can be sent to any one of the five post offices. So, required number of ways

ow
= 5x5x5x5 =5*^.

6. Sincetossofeachcoincanresultin2ways.So,requiredno.ofways = 2x 2x 2x 2x 2 = 2^.
8. Required no. of ways =10 x 10 x 10 -1.

e
9. Each one of the first three questions can be answered in 4 ways and each one of the next

re
rFl
three questions can be answered in 2 ways. So, total no. of sequences of answers

F
= 4x4x4x2x2x2.

11. Required no. of signals =7 X 6.


12. A boy can be selected from the first team in 6 ways, and from the second in 5 ways. So, no. of

r
ou
fo
single matches between the boys of two teams = 6 x 5 = 30. Similarly, the no. of single
ks
matches between the girls of two teams =4x 3=12. So, total number of matches
= 30 +12 = 42.
oo

13. Required no. of ways =12 x 11 x 10.


Y

14. There are 3 ways to choose the first term and corresponding to each such way there are 5
B

ways of selecting the common difference. So, required no. of A.P.'s = 3x5.
re

1.5.
Required no. of ways = 36 x 35 x 34.
16. The total no. of required numbers = 9x9x8.
ou
Y
ad

17, The total no. of required numbers = 9x10x10.


18. The total no. of required number = 9 x 10 x 5.
d

19.
(i) Required no. of licence plates = 9x9x8x7x6
in
Re

(ii) Required no. of licence plates = 9 x 10 x 10 x 10 x 10.


20. Required no. of numbers = 3x4x3x2.
F

21. Required no. of numbers = 2x4x3x2.


22. Required no. of ways = 6x5x4x3x2xl.
23. Required no. of numbers = 9x9x8x7x6x5x4x3x2.
24. An odd number less than 1000 may be a
one-digit number, two-digit number or a
three-digit number. So, required no. of numbers is
3 (one-digit nos.) + 2x 3 (two-digit nos.) -i- 2x 2x 3 (3-digit nos.).
25. Required no. of numbers = 5 x 4 x 3.
26. Required no. of numbers = 6x5x4x 3x2x1.
27. Required no. of numbers =5x5x4x 3x2x1.
28. Required no. of numbers = 2x4x3x2.
29. Here we have to perform 6 jobs. So, required number of serial numbers i
PERMUTATIONS 10.17

6x5x10x9x8x7
30. Required number of sequences =10x9x8.
Also, total number of unsuccessful attempts =10 x 9 x 8 -1
31. Number of trials =4x 3x2x1

32. Required number of ways =3x2x1


36. Since a toss of a coin can result in a head or a tail. Therefore, if a coin is tossed fi-times, then
n
the total number of outcomes is2x2x2x...x2 = 2
n-timcs

ow
41. Each toy can be distributed in 5 ways.
So, total number of ways = 5x5x5x5x5x5x5x5 =5'
42. Each letter can be posted in any one of the 7 letter boxes.
So, required number of ways = 7x7x7x7x7 =7^

e
re
43. Required number of possible outcomes
= Total number of possible outcomes - Number of possible out- comes in which 5

F
does not appear on any dice.
= 6^ -5^ =216-125=91.

Frl
44.
ou
One ball can be put in first box in 20 ways because we can put any one of the twenty balls in

sr
the first box. Now, remaining 19 balls are to be put into remaining 4 boxes. This can be done
kfo
in 4^^ ways, because there are 4 choices for each ball. Hence, the required number of ways
= 20x 4^^.
oo
10.3 PERMUTATIONS
Y

Each of the arrangements which can be made by taking some or all of a number of things is
reB

called a permutation.
For example, if there are three objects, then the permutations of these objects, taking two at a
uY

time, are
ab, bn, be, cb, ac, cn
ad
do

So, the number of permutations of three different things taken two at a time is 6.
NO!T It should be noted that in permutations the order of arrangement is taken into account;
in

when the order is changed, a different permutation is obtained.


Re

ILLUSTRATION 1 Write down all the permutations of the set of three letters A, B, C.
F

SOLUTION The permutations of three letters A, B, C taking all at a time are :


ABC, ACB, BCA, BAC, CBA, CAB.
Clearly, there are 6 permutations.
ILLUSTRATION 2 Write down all the permutations of the vowels A, E, I, O, L/ in English alphabets
taking three at a time, and starting zuith A.
SOLUTION The permutations of vowels /\, E, I, O, U taking three at a time, and starting with A
are:

AEI, AIE, AEO, AOE, AEU, AUE, AlO, AOl, AIU, Aill, AOU, AUO
Clearly, there are 12 permutations.
ILLUSTRATION 3 Write doivii all the permutations of letters A, B, C, D taking three at a time.
SOLUTION The desired permutations are :
ABC ABD BCD ACD
10.18 APPLIED MATHEMATICS-XI

ACB ADB BDC ADC

BCA BDA CBD CAD

BAC BAD CDB CDA

CAB DAB DCB DAC

CBA DBA DBC DCA

Clearly, there are 24 permutations. These permutations are obtained by first selecting three
letters out of 4 and then arranging them in all possible ways.
A NOTATION If n and rare positive integers such thatl <r <n, then the number of all permutations of n
distinct things, taken rat a time is denoted Inj the si/mbol P(n, r) or
Thus,
II
Pr or, P(n, r) = Total number of permutations of n distinct things, taken rata time.

w
In illustration 3, we have seen that there are 24 permutations, on a set of 4 letters, taken 3 at a
time. Therefore, as per our notation, we have ‘*P3

F lo
=24 or, P(4, 3) = 24.
THEOREM 1
Let rand n be positive integers such thatl <r <n. Then the number ofall permutations of n
distinct things taken rat a time is given bpn{n -1) (/; -2) (h - 3)... (n -{r -1)).

ee
P(n,r) = "P, = «(n-l)(f;-2)...0i-(r-l)).

Fr
i.e.

PROOF The number of permutations of n distinct things, taken rata time, is same as the number
for
of ways in which we can fill up r-places when we have n different things at our disposal.
ur
The first place can be filled in n ways, for any one of the n things can be used to fill up the first
place. Having filled it, there are (?j- 1) things left and any one of these (n - 1) things can be used
s

to fill up the second place. So, the second place can be filled in {n -1) ways. Hence, by the
ook
Yo

fundamental principle of counting, the first two places can be filled in n (» -1) ways. When the
first two places are filled, there are (n - 2) places left, so that the third place can be filled from the
eB

remaining {n - 2) things in (?i - 2) ways. Therefore, the first three places can be filled in n{n -1)
(h-2) ways. Continuing in this manner, we find that the first (r -1) places can be filled in
r

jj(« -1) (n-2)... (n-{r - 2)) ways. After filling up first(r -1)places, exactly;/ - (r -1) =;/ - r +1
ad
ou

things are left. So, the rth place can be filled in (»-(r -1)) ways. Hence, the r places can be filled
in n (;/ -1) (;/ - 2)...(;/ -(r -1)) ways.
Y

Hence, the total number of permutations of n distinct things, taken rata time is
Re
nd

(;/- 2) {;/- 3)... (;/-(r -1)).


Fi

Thus, P (;j, r) = ;i (;; -1) (// - 2) (;/ - 3)... (// - (r -1)).


II n!
THEOREM 2 Proz’e that: P(n,r) = Pr =
(n - r)l

PROOF We have.
P (;/, r) = //(;/ -1) (?/ -2) (« - 3)... (;i -(r -1))
;/(;;-l) {n-2) (;/ - 3)... (» -(r -1))(//-r) (/;-(r +1))... 3.2.1
P{n,r) =
{n-r) (;j-(r + l))... 3.2.1
nl
P{n,r) =
(;/-/■)!

THEOREM 3 The number ofall permutations ofn distinct things, taken all at a time is nl.
PROOF The number of all permutations of ;/ distinct things, taken all at a time is same as the
number of ways of filling n places when we have n distinct things at our disposal.
10.19
PERMUTATIONS

Proceeding as in theorem 1, we have


1
3.2.1 = n \

THEOREM 4 Prove that 0! = 1.


PROOF We have.
n!
P{n,r) =
{n-r)\
Ji!
P{>hu) = (Putting r =n]
0!

n\
n! [v P (n, n) =n! (See Theorem 3)1
0!
h!
0! = - = 1.
>?!

w
ILLUSTRATIVE EXAMPLES

7 I

EXAMPLE 1
(i)
PROBLEMS BASED UPON THE VALUE OF " P, OR PUi. n
Evaluate the following:
(ii) P(15,3)
F lo
(iii) P{5,5)

e
Fre
5! u\
"RI =
SOLUTION (i) ^P^ = (n-r)!
(5-3)!
for
5! 5x 4x 3x 2!
^^3 = = 60
r
2! 2!
You

151 15! 15x14x13x12!


oks

= 2730
(ii) P(15, 3) = 12!
(15-3)1 12!
eBo

5! 5!
(iii) P(5,5) = — = 5! = 120.
(5-5)1 0!
ON FINDING THE VALUE OF REQUIRED UN-KNOWN WHEN A RELATION CONNECTING P <n, r)
our
ad

Tifiwll
IS GIVEN

EXAMPLE 2 If 2. P(5, 3) = P(n, 4), find n.


dY

SOLUTION We have,
Re

2.P(5, 3)=P(^i, 4)
Fin

P (fj, 4) = 2 . P (5, 3)
n\I 5!
= 2
(/j-4)l 1(5-3)!
n (» -1) (H-2) (h-3) (n-4)l _ 2(5!)
(»-4)l 2!

n{n-\) (» -2) (n - 3) =5!


fi (ji -1) (ji - 2) (n - 3) = 5 X 4 X 3 X 2 X 1
H(n-l)(n-2)(ji-3) - 5 (5-1) (5 - 2) (5 - 3)
II = 5 [By comparing two sides]
EXAMPLES If p{n, 4) = 20 X P{n, 2), find n.
SOLUTION We have,
P(ii, 4) = 20 X P(n, 2)
10.20 APPLIED MATHEMATICS-XI

n! n\
= 20x
(«-4)! {71-2)1

()t-2)\ = 20x(n-4)!
(« - 2) (« - 3) (h - 4)! = 20 X (» - 4)!
(ii-2){n-3) = 20
=> (n - 2) (n - 3) = 5 X 4
n-3 =4

n = 7
[By comparing two sides]
r:XAMl’LL4
If P(5, r) = 2. P(6, r -l),find r.
SOLUTION We have,

ow
P(5,r)=2.P(6,r-l)
5! 6!
= 2.
(5-r)! (6-(r-l))!
5! 2x6x5!

e
re
(5-r)l {7-r)\

Fl
F
5! 12x5!

(5-r)! (7-r)(6-r)(5-r)I
ur
r
12
1
(7-r) (6-r)
fo
ks
Yo
(7-r)(6-r) = 12
oo

(7 -2) (6-r) =4x3


eB

7 -r = 4
[By comparing]
r = 3
ur

10
IXAMPI.LS If Pr =5040,_/7«rf the value of r.
ad
Yo

SOLUTION We have,
10
Pr =5040
d

10!
= 10 X 504
Re
in

(10-r)!
10!
F

= 10x9x8x7
(10-r)!
10! lOx 9x 8x7x 6!

(10-r)! 6!
10! 10!
=> (10-r)! = 6!=>10-r = 6=>r = 4.
(10-r)! 6!

LXAMPI.E 6 If P(u -1,3): Pin, 4) = 1; 9, find n.


SOLUTION We have,
P(»-l, 3):P(n, 4)=1:9
Pin-I, 3) 1

P in, 4) 9
PERMUTATIONS 10.21

(»-!)!
01-1-3)! 1
ul 9
0»-4)l
07-1)1 X
0i-4)! 1 . 0?-l)I 1 =>
(77-1)! 1
— => n = 9
(77-4)1 77! 9 77! 9 »-(n-l)! 9

example;? If ‘^P^+5-^Pi = '^^P„findr.


SOLUTION We have,

’fi;+5.5p4 = '“P,
9! 9! 10!
+ 5-

w
(9-5)! (9-4)! (10-r)!
9! 9! 10!
- + 5-

F lo
4! 5! (10-r)!
9! 9! 10!
+
4! 4! (10-7-)!

ee
Fr
9! 10!
2 X — -■
4! (10-r)! for
5x2x9! 10!
ur
5x4! (10-r)!
lOx 9! 10!
s
ook
Yo

5! (10-r)!
10! 10!
eB

=> (10-r)! =5!=>10-r =5=>r =5


5! (10-r)!

EXAMPLE a If 56 pr+6-;54pr+ = 30 800:l,//i7rf r.


our

3
ad

SOLUTION We have.
56
Y

+ 3 800:1
Re

54! 30800
nd

56!

(56-r-6)! (54-r-3)! 1
Fi

56! 54!
= 30800:1
(50-r)! (51-r)!
56! 30800

(50-7-)! 54! 1

56x55x54! X
(51-7-) x (50-r)! 30800

(50-r)! 54!

=> 56x55x(51 -r) = 30800 => (51 -r) = 10=>r = 41.


2h+ 1 . 2h -1
EXAMPLE y If Pn-1- P„ - 3:5, find n.
SOLUTION We have,
277 + Ip
*71-1 ●
277 -1
R,77 = 3:5
10.22 APPLIED MATHEMATICS-XI

2n+ 1
Pn-l _ 3

(2n + l)! (n-1)! _ 3


{n + 2)] {2n-i.y ~ 5
(2n + l) (2n) (2»-l)! (n-1)! ^ 3
(n + 2) 07 + 1)h0j-1)!'' (2)7-1)! “ 5
2(2/j + 1) ^ 3
(n + 2) {n + 1) 5

10(2n + l) = 3(n+2)(n + l)
3 + 9/1 + 6 = 20// + 10

w
=>
3//^ -11//-4 = 0 (//-4)(3n + l) - 0=^ // = 4 [v //^-l/3]
EXAMPLE 10 If + 1: = 11 -.52, find r.

F lo
r+ 2

SOLUTION We have,

ee
r + 1 ● ● 20pr + 2 = 11:52

Fr
22! 20!
= 11:52
(21-r)! (18-r)l
for
ur
22! (IS-/-)! 11
(21-r)!"" 20! 52
ks

22 X 21 X 20! (18-r)! 11
Yo

X
oo

(21 -/●) (20-/-) (19-/-)-(18-/-)! 20! 52


eB

22x 21 11

(21 -r) (20-r) (19-/-) 52


r

(21 -r)(20-r)(19-r) = 2x 21x52


ou
ad

(21-/-)(20-r)(19-r) = 2x 3x7x4x13
Y

(21-r) (20-/-) (19-r) =12x13x14


=> (21 -r) (20-r) (19 -/■) = (21 -7) (20 -7) (19-7)
Re
nd

/- = 7
Fi

Type III ON PROVING RESULTS RELATED TO P (ji, /) or “P,


EXAMPLE 11 Prove the following:
(i) P (//, //) = 2 P (//, // - 2) (ii) P(n,n) = P (//,//-!)
(iii) P(n,r) = P{n -I, r) + r . P(n -1, r -1) (iv) P{n,r) = n- P{n-l,r-1)
//! fn'\
SOLUTION (i) 2P (//, // - 2) = 2 2 - =nl = P{n,n)
(//-(//-2))! 2!

//! //!
(ii) P(/i,//-l) = /?! = P(n,n)
(//-(//-I))! 1!
(P-1)! {//-!)!
(iii) P{n-\,r) + r ■ P{n-1, r ~1) = + r ■
(//-I -r)! ((//-I)-(/●-!))!
(P-1)! (P-1)! (P-1)! (P-1)!
+ r + r
{n-r-l)l (//-/-)! (;/-r-l)! (//-/●) (//-/--I)!
10.23
PERMUTATIONS

(n-1)! 1 + —
ii -r + r

(n-r-1)! u-r (»-;●-!)! n - r

n n\
= P{ii,r)
(»-r-l)! n-r (»-r)!
(n-1)! /jl
(iv) ?j ● P (;i -1, r -1) =n
((n-l)-(r-l))! (n-r)l

Type III PRACTICAL PROBLEMS ON PERMUTATIONS


NOTE ALITER 2 of each of the following examples should be done after studying permutations
and combinations.

w
EXAMPLE 12 In hou) many zunys three different rings can he worn in four fingers zuith at most one in
each finger?
SOLUTION The total number of ways is same as the number of arrangements of 4 fingers, taken 3

Flo
at a time.
4! 4!

e
— = 4! = 24.
So, required number of ways =

re
(4-3)! 1!
Let Ri, R2, /?3 be three rings. Since R^ can be put in any one of the four fingers. So,

F
ALITER 1

there are four ways in which can be worn. Now, R2 can be worn in any one of the remaining
three fingers in 3 ways. In the remaining 2 fingers ring R3 can be worn in 2 ways. So, by the
ur
r
fo
fundamental principle of counting the total number of ways in which three different rings can
be worn in four fingers is 4 x 3 x 2 = 24.
ks
Out of 4 fingers, 3 fingers can be chosen in ‘*C3 ways. Now, three rings can be worn
Yo
ALITER 2
oo

in the selected three fingers in 3! ways. Hence, three rings can be worn in four fingers in
eB

“^C3x 3! = 24 ways.
EXAMPLE 13 Seven athletes are participating in n race. In how many ivays can the first three prizes be
ur

ivon ?
ad

SOLUTION The total number of ways in which first three prizes can be won is the number of
Yo

arrancements of seven different things taken 3 at a time.


7! 7! 7x6x5x41
So, required number of ways = ^P3 = 210.
d

(7-3)1 4! 4!
Re
in

ALITER 1 First prize can be won in seven ways. Second prize can be won by any one of the
remaining six athletes in 6 ways. Now, five athletes are left. So, third prize can be won by any
F

one of the remaining 5 athletes in 5 ways.


Hence, by the fundamental principle of counting, the required number of ways =7 x 6 x 5 = 210.
ALITER 2 Out of 7 athletes, 3 can be chosen for prize in ways. Now, three prizes can be
given to three chosen athletes in 3! ways.
Numbers of ways in which 3 prizes can be won = ^03 x 3! = 210
EXAMPLE 14 How many different signals can be made by 5 fags from 8 flags of different colours?
SOLUTION The total number of signals is the number of arrangements of 8 flags by taking 5
flags at a time.
8! 8! 8x7x6x5x4x31
= 6720
Hence, required number of signals = =
(8-5)1 “3! 3!

EXAMPLE 15 In hoio many zvays can 6 persons stand in a queue?


10.24 APPLIED MATHEMATICS-Xl

SOLUTION Tlie number of ways in which 6 persons can stand in a queue is same as the number
of arrangements of 6 different things taken all at a time.
Hence, the required number of ways = =6! =720.
EXAMPLE 1ft It is required to seat 8 men and 4 zuomen in a row so that the women occupy the even
places. How many such arrangements are possible?
SOLUTION In all 12 persons are to be seated in a row and in the row of 12 positions there are
exactly 6 even places viz second, fourth, sixth, eighth, tenth and twelfth. It is given that four
women are to occupy 4 places out of these six even places. This can be done in ^ P4 ways (ways of
arranging 6 women in 4 positions). The remaining 8 positions can be filled by the 8 men in ®Pg
ways. So, by the fundamental principle of counting, the number of seating arrangements as
required, is ^P4 x ®Pg = 360 x 40320 =14515200.
ALlTEHj In all 12 persons are to be seated in a row and in the row of 12 positions there are

w
exactly 6 even places viz. 2nd, 4th, 6th, 8th and 12th. It is given that 4 women are to occupy any 4

F lo
places out of these six positions. This can be done in x 4! ways. The remaining 8 positions
are to be occupied by 8 men. This can be done in ^Cg x 8! ways.
Hence, total number of seating arrangements = ( x 4!) x (®Cg x 8!)

e
Fre
= 360x40320=14515200.

EXAMPLE 17 Three men have 4 coats, 5 ivaist coats and


for
6 caps. In hoiv znany ways can they wear them ?
SOLUTION The total number of ways in which three men can wear 4 coats is the number of
r
arrangements of 4 different coats taken 3 at a time. So, three men can wear 4 coats in “^Pg ways.
You
oks

Similarly, 5 waist coats and 6 caps can be worn by three men in ^Pg and ^Pg ways respectively.
Hence, by the fundamental principle of counting, the required number of ways as desired
eBo

= ^Pg X 5pg X ^Pg =(4!) X (5 X 4 X 3) X (6 X 5 X 4} =172800

EXAMPLE 18 How many different signals can be given using ariy number offlags from 5flags of different
our
ad

colours ?

SOLUTION The signals can be made by using at a time one or two or three or four or five flags.
The total number of signals when r flags arc used at a time from 5 flags is equal to the number of
dY
Re

arrangements of 5, taking r at a time i.e. ^ P^. Since r can take values 1, 2,3,4, 5. Hence, by the
fundamental principle of addition, the total number of signals
Fin

= ^P| + ^P2 + ^Pg + ^P4 + ^Pg


= 5+ 5x4 + 5x4x3+5x4x3x2 + 5x4x3x2x1=5+ 20+ 60 + 120 + 120 = 325

EXAMPLE 19 How many numbers lying between 100 and 1000 can be formed with the digits 2,2,3,4,5
if the repetition of digits is not allowed ?
SOLUTION Every number lying between 100 and 1000 is a three digit number. Therefore, we
have to find the number of permutations of five digits 1,2,3,4, 5 taken three at a time.
5! 5!
Hence, the required number of numbers = ^P 3 - — = 5x4x 3 = 60
(5-3)1 2!
EXAMPLE 20 Hozv many four digit numbers are there zoith distinct digits ?
SOLUTION The total number of arrangements of ten digits 0,1, 2, 3, 4, 5, 6, 7, 8, 9 taking 4 at a
time is ^P4- But, these arrangements also include those numbers which have 0 at thousand's
place. Such numbers are not four digit numbers. When 0 is fixed at thousand's place, we have to
arrange remaining 9 digits by taking 3 at a time. The number of such arrangements is ^Pg.
PERMUTATIONS 10.25

So, the total number of numbers having 0 at thousand's place = ^^3.


Hence, the total number of four digit numbers = - ^^3 =5040 -504 = 4536.
EXAMPLE 21 In how many ways 7 pictures can be hung from 5 picture nails on a wall ?
SOLUTION The number of ways in which 7 pictures can be hung from 5 picture nails on a wall is
same as the number of arrangements of 7 things, taking 5 at a time.
7! 7!
Hence, the required number = = = — = 2520.
(7-5)! 2!

EXAMPLE 22 Determine the number of natural numbers smaller than lO"^, in the decimal notation of
luhich all the digits are distinct.
SOLUTION The required natural numbers consist of 4 digits, 3 digits, 2 digits and one digit.

w
Total number of 4 digit natural numbers with distinct digits =
Total number of 3 digit natural numbers with distinct digits = ^^P^ - ^P2

Flo
Total number of 2 digit natural numbers with distinct digits =

e
re
Total number of one digit natural numbers = 9

F
Hence, the required number of natural numbers =(’^P4 P2) +^
ur
r
= 9x9x8x7 + 9x9x8 + 9x9 + 9 = 5274.
fo
EXAMPLE 23 How many zvords, zuith or without meaning, can beformed using all the letters of the word
EQUATION, using each letter exactly once.
ks
Yo

SOLUTION There are eight letters in the word 'EQUATION'. So, the total number of words is
oo

equal to the number of arrangements of these letters, taken all at a time. The number of such
arrangements is ^Pg = 8! Hence, the total number of words = 8!
eB

EXAMPLE 24 Ho'w many -i-letter words, with or without meaning, can be formed out of the letters of the
word, 'LOGARITHMS', if repetition of letters is not allowed?
ur
ad

SOLUTION There are 10 letters in the word 'LOGARITHMS'.


Yo

So, the number of 4 - letter word = Number of arrangements of 10 letters, taken 4 at a time
10
P4 =5040.
d
Re

EXAMPLE 25 In an examination hall there are four roxus of chairs. Each row has 8 chairs one behind the
in

other. There are two classes sitting for the examination with 16 students in each class. It is desired that in
F

each row, all students belong to the same class and that no tiuo adjacent rows are allotted to the same class.
In hozo many zvays can these 32 students be seated?
SOLUTION Let the two classes be Cj and C2 and the four rows be R2, -^3 ^4- There are 16
students in each class. So, there are 32 students. According to the given conditions there are two
different ways in which 32 students can be seated:
R1 Pi P3 Pa
I Cl C2 c1 C2
II C2 c1 C2 c1

Since the seating arrangement can be completed by using any one of these two ways. So, by the
fimdamental principle of addition.
Total number of seating arrangements = No. of arrangement in I case + No. of arrangements in
II case.
10.26 APPLIED MATHEMATICS-XI

16
In case I, 16 students of class can be seated in Kj and R3 in Pgx8! = 16! ways. And 16
16
students of class C2 can be seated in R2 and K4 in Pg X 8! =16! ways
Number of seating arrangements in case I = 16! x 16!
Similarly, Number of seating arrangements in case II =16!x 16!
Hence, Total number of seating arrangements = (16!x 16!) + (16!x 16!) = 2 (16!x 16!)
EXAMl’l.E 26 Ten dijferent letters of an alphabet are given. Words ivith five letters are formedfrom these
given letters. Determine the number of zvords which have at least one letter repeated.
SOLUTION Thenumber of 5-letter words which can be formed from 10 letters when one or more
of its letters is repeated =10 x 10 x 10 x 10 x 10 =10^.
Tl-\e number of 5-letter words which can be formed when none of their letters is repeated
10
= Number of arrangements of 10 letters by taking 5 at a time = P5 = 30240

w
Hence, the number of 5-letter words which have at least one of their letters repeated is
10^-30240=69760.

F lo
EXAMPLE 27 Find the sum of all the numbers that can be formed with the digits 2, 3,4,5 takc7i alt at a
time.

ee
SOLUTION The total number of numbers formed with the digits 2, 3, 4, 5 taken all at a time

Fr
= Number of arrangement of 4 digits, taken all at a time = ^^P4 = 4! = 24.
for
To find the sum of these 24 numbers, we will find the sum of digits at unit s, ten s, hundred s
ur
and thousand's places in all these numbers.
Consider the digits in the unit's places in all these numbers. Each of the digits 2,3,4,5 occurs in
ks

3! (= 6) times in the unit's place.


Yo
oo

So, total for the digits in the unit's place in all the numbers = (2 + 3 + 4 + 5) x 3! = 84.
eB

Since each of the digits 2,3,4, 5 occurs 3! times in any one of the remaining places.
So, the sum of the digits in the ten's, hundred's and thousand's places in all the numbers
= (2 + 3 + 4+5)x 3! = 84.
r
ou
ad

Hence, the sum of all the numbers = 84 (lO'^ +10^ + 10^ +10^) = 93324.
Y

EXERCISE 10.3
Re

1. Evaluate each of the following:


nd

(ii) ^0P4 (iii) *’P6


8
(i) "P3 (iv) P(6,4)
Fi

2. If P (5, r) = P (6, r -1), find r.


3. If 5 P(4, n) = 6. P (5, n - 1), find n.
4. If P (», 5) = 20 . P(n, 3), find n
5. If " P4 = 360, find the value of n.
6. IfP(9,r) = 3024,findr.
7. IfP{ll,r)=P(12,r-l) find r.
8. IfP(n,4)=12.P0i,2),find/i.
9. IfP(»-l, 3):P(n,4)=l:9,find n.
10. If P (2fi -1, n): P (2)1 + 1, ?i -1) = 22:7 find n.

n. If P ()i, 5): P ()i, 3) =2:1, find)?.


PERMUTATIONS 10.27

12. Provethat;! .P{1,1) + 2.P(2,2) +3.P{3, 3) +... + .P (»,n) = P(;/+ 1,+ 1) -1.
13. If P (15, r -1); P (16, r - 2) = 3:4, find r.
n(n-l) ,,.,3
14. If n + 5p» + 1 - ,Pj, find n.
2

15. In how many ways can live children stand in a queue?


16. From among the 36 teachers in a school, one principal and one vice-principal are to be
appointed. In how many ways can this be done?
17. Four letters E, K, S and V, one in each, were purchased from a plastic warehouse. How
many ordered pairs of letters, to be used as initials, can be formed from them?
18. Four books, one each in Chemistry, Physics, Biology and Mathematics, are to be arranged

ow
in a shelf. In how many ways can this be done?
19. Find the number of different 4-letter words, with or without meanings, that can be formed
from the letters of the word 'NUMBER'.

20. How many three-digit numbers are there, with distinct digits, with each digit odd?

e
Fl
re
21. How many words, with or without meaning, can be formed by using all the letters of the
word 'DELHI', using each letter exactly once?

F
22. How many words, with or without meaning, can be formed by using the letters of the word
ur
'TRIANGLE'?

or
23. There are two works each of 3 volumes and two works each of 2 volumes; In how many
sf
ways can the 10 books be placed on a shelf so that the volumes of the same work are not
k
separated?
Yo
oo

24. There are 6 items in column A and 6 items in column B. A student is asked to match each
item in column A with an item in column B. How many possible, correct or incorrect,
B

answers are there to this question?


re

25. How many three-digit numbers are there, with no digit repeated?
26. How many 6-digit telephone numbers can be constructed with digits 0,1,2,3,4,5,6,7,8,9
u
ad

if each number starts with 35 and no digit appears more than once?
Yo

27. In how many ways can 6 boys and 5 girls be arranged for a group photograph if the girls are
to sit on chairs in a row and the boys are to stand in a row behind them?
d
Re

28. If (1 denotes the number of permutations of (a: + 2) things taken all at a time, b the number of
in

permutations of .v things taken 11 at a time and c the number of permutations of .r - 11


F

things taken all at a time such that n = 182 be, find the value of .r.
29. How many 3-digit numbers can be formed by using the digits 1 to 9 if no digit is repeated?
30. How many 3-digit even numbers can be made using the digits 1,2,3,4,5,6,7, if no digits is
repeated?
31. Find the numbers of 4-digit numbers that can be formed using the digits 1, 2, 3, 4, 5, if no
digit is repreated? Howmany of these will be even?
32. All the letters of the word 'EAMCOT' are arranged in different possible ways. Find the
number of arrangements in which no two vowels are adjacent to each other.
ANSWERS

1. (i) 336 (ii) 5040 (iii) 720 (iv) 360


2. 4 3. 3 4. 8 5. 6

6. 4 7. 9 8. 6 9. 9
10.28 APPLIED MATHEMATICS-XI

10 11. 5 13. 14 14. 6,7


15. 120 16. 1260 1^. 12 18. 24

19. 360 ?.n. 60 21. 120 22. 8!

23. 3456 24. 720 25. 648 26. 1680

27. 86400 27. 12 29. 504 30. 90

120,48 32. 144

HINTS TO SELECTED PROBLEMS

2 We have

P(5, ?-)=P(6,r-l)

ow
5! 6!

(5-r)! {6-(r-l)}!
5! 6x5!

(5-r)l (7-r)I

e
re
1 6

rFl
F
(5-r)l (7-r)(6-r){5-r)i
6
1 = => (7-r) (6-r) = 3x 2=> 7-r = 3=> r = 4

r
(7-r)(6-r)
fo
ou
P{n-1, 3) 1 _ (»-4)i 1 1=1 ^n = 9.
ks
. P{n-1, 3):P0z,4)=l;9 =>
P(n,4) “9^{n-4)!'' n\ 9 n 9
oo

1.5. The total no. of ways = No. of arrangements of 5 things, taken all at a time =
Y
eB

36
16. Total no of ways = No. of arrangements of 36 things taken two at a time = Pi-
17. Thetotalno.oforderedpairs = No.ofarrangementsof41etters,take ntwoata time = ^P2.
ur

taken all at a time = ^P^.


ad

1 .s. No. of ways = No. of arrangements of 4 books,


Yo

Total no. of words = No. of arrangements of 6 letters, taken 4 at a time = ^P^.


d

20. Required number of numbers = Number of arrangements of digits 1, 3,5,7,9 by taking


Re
in

3 at a time = ^ P3 .
F

Let W11, ^13 ■ ^21' ^22^ ^23 . ^31' ^32 . ^41' ^42 be 4 works. These 4 works can
Wi W3 W2 W4
be arranged in4! ways. Now, volumes of each work can be arranged in the following ways:
W| —> 3! ways; W2 —> 31 ways, W3 2! ways, W4 —> 2! ways.
Hence, total no. of ways to arrange all books = 4!(3lx 3!x 2!x 2!) = 3456.
24. Each answer to the given question is an arrangement of the 6 items of column B keeping the
order of items in column A fixed. Hence, the total number of answers = Number of
arrangements of 6 items in column B = ^ P^ = 61.
“'7. Total number of three digit numbers with distinct digits = ^^P^ ~ ^P2-
. Required number of telephone numbers = ^P^^.
PERMUTATIONS 10.29

Five girls can sit on chairs in a row in^p5 =5! ways. Also, 6 boys can stand behind them in a
row in = 6! ways. Hence, the total number of ways =5! x 61
’. The total number of 4 digit numbers formed by using the digits 1, 2,3,4,5 is same as the
number of arrangements of 5 digits taken 4 at a time.
5!
So, required number of numbers = = = 120
(5-4)!
An even number will have 2 or 4 at its unit's place. So, unit's place can be filled in 2 ways
and the remaining three places (tens, hundreds and thousands) can be filled with
remaining 4 digits in ways. Hence, total number of 4 digit even numbers formed by
using the given digits is ^^P3 x 2 = 48.

low
10.4 PERMUTATIONS UNDER CERTAIN CONDITIONS

In this section, we shall discuss permutations where either repetitions of items are allowed or
distinction between some of the items are ignored or a particular item occurs in every

e
arrangement etc. Such type of permutations are known as permutations under certain

re
conditions as discussed below.

rF
F
THEOREM 1
Prove that the number of all permutations of n different objects taken r at a time, when a
H - 1
particular object is to be ahunys included in each arrangement, is r. Pr-V

r
PROOl Here we have to find the number of ways in which r places can be filled with n given
fo
u
objects such that a particular object occurs in each arrangement. Suppose the particular object is
placed at the first place. Then, the remaining (n -1) places can be filled with remaining (r -1)
ks
H - 1
Yo
objects in P^ _ 1 ways. Similarly, by fixing the particular object at the second, third, fourth,...,
oo

n -
rth places, we find that the number of permutations in each case is - V
B

Hence, by the fundamental principle of addition,


re

» -1 M-l n - 1 « - 1
The required number of permutations = Pr-] + Pr _! + ...+ ^r-1 -r.
Pr-l-
u

Q.E.D.
ad
Yo

THEOREM 2 Prove that the number of permutations of n distinct objects taken r at a time, when a
n-l
particular object is never taken in each arrangement, is Pr-
d

PKOOr Since one particular object out of n given objects is never taken. So, we have to
Re
in

determine the number of ways in which r places can be filled with {n -1) distinct objects.
F

II- 1
Clearly, the number of such arrangement is .P-
Q.E.D.
THEOREM 3 Proi>e that the number of permutations ofn different objects taken rata time in zvhich tzoo
II-2
specified objects aizvays occur together is 2! (r -1) Pr-2-
PROOF First let us leave out the two specified objects. Then the number of permutations of the
»-2
remaining (/i -2) objects, taken (r - 2)at a time, is Pr 2- Now, we consider two specified
n-2
objects temporarily as a single object and add it to each of these Pr-2 permutations which
n-2
can be done in (r -1) ways. Thus, the number of permutations becomes (r -1) P^ _ 2- But two
specified things can be put together in 2! ways.
II-2
Hence, the required number of permutations is 2!. (r -1). Pr-2-
Q.E.D.
10.30 APPLIED MATHEMATICS-X!

ILLUSTRATIVE EXAMPLES

EXAMPLE 1 In how tnani/ ways can the letters of the word PENCIL be arranged so that (i) N is always
next to E ? (ii) N and E are always together ?
SOLUTION (i) Let us keep EN together and consider it as one letter. Now, we have 5 letters
which can be arranged in a row in ^ P5 =5! = 120 ways. Hence, the total number of ways in which
N is always next to E is 120.
(ii) Keeping E and N together and considering it as one letter, we have 5 letters which can be
arranged in =5! ways. But, E and N can be put together 2! ways (viz. EN, NE).
Hence, the total number of ways =5! x 2! = 240.
EXAMPLE 2 How many different zoords can beformed with the letters of the zoord EQUATION so that
(i) the words begin with E ? (ii) the zoords begin zvith E and end with N ?

w
(iii) the words begin and end zuith a consonant ?
Clearly, the given word contains 8 letters out of which 5 are vowels and 3

F lo
SOLUTION
consonants.

(i) Since all words must begin with E. So, we fix £ at the first place. Now, remaining 7 letters can

ee
rj
be arranged in Py=7! ways.

Fr
So, total number of words = 7 !
for
(ii) Since all words must begin with E and end with N. So, we fix E at the first place and N at the
ur
last place. Now, remaining 6 letters can be arranged in = 6! ways.
Hence, the required number of words = ^Pg = 6!
s
ook
Yo

(iii) There are 3 consonants and all words should begin and end with a consonant. So, first and
last places can be filled with 3 consonants in ^P2 ways. Now, the remaining 6 places are to be
eB

filled up with the remaining 6 letters in ^Pg ways.


Hence, the required number of words = ^P2 x ^P^, = 6 x 720 = 4320
our
ad

EXAMPLE 3 Hozu many words can be formed from the letters of the word, 'TRIANGLE' 7 Hoiu many of
these will begin with T and end with E ?
Y

There are 8 letters in the word TRIANGLE'. The total number of words formed
Re

SOLUTION
nd

with these 8 letters is the number of arrangements of 8 items, taken all at a time, which is equal to
Fi

® P5 = 8! = 40320. If we fix up T in the beginning and £ at the end, then the remaining 6 letters can
be arranged in ^P^, =6! ways.
So, the total number of words which begin with T and end with £ = 6! = 720.
EXAMPLE 4 Hozv many zoords can be formed zoith the letters of the zoord 'ORDINATE' so that vozvels
occupy odd places?
SOLUTION There are 4 vowels and 4 consonants in the word 'ORDINATE'. We have to arrange
8 letters in a row such that vowels occupy odd places. There are 4 odd places viz. 1,3,5,7. Four
vowels can be arranged in these 4 odd places in 4! ways. Remaining 4 even places viz. 2,4, 6, 8
are to be occupied by the 4 consonants. This can be done in 4! ways. Hence, the total number of
words in which vowels occupy odd places = 4! x 4! =576.
EXAMPLES In hozo many zonys 5 boys and 3 girls can be seated in a roio so that no tzvo girls are
together?
10.31
PERMUTATIONS

SOLUTION The 5 boys can be seated in a row in =5! ways. In each of these arrangements 6
places are created, shown by the cross-marks, as given below:
xBxBxBxBxBx

Since no two girls are to sit together, so we may arrange 3 girls in 6 places. This can be done in
ways i.e. 3 girls can be seated in ways.
Hence, the total number of seating arrangements = x ^P3 =5!x 6 x 5 x 4 =14400.
EXAMPLE 6 In how mani/ zuai/s can the letters of the ivord 'DELHI' be arranged so that the vowels
occupy only even places?
SOLUTION There are 5 distinct letters in the word 'DELHI'. We wish to find the total number of
arrangements of these 5 letters so that vowels occupy only even places. There are two vowels E
and I and 2 even places viz 2"^ and 4^‘\ These two vowels can be arranged in the two even places

low
in 2! ways. The remaining three letters (D, L, H) can be arranged in 3 places (viz 1st 3rd, 5th) in 3!
ways. Hence, by the fundamental principle of counting the total number of arrangements
= 3!x2! = 12.

EXAMPLE 7 How many words can be formed from the letters of the word 'DAUGHTER' so that
(ii) the vowels nei>er come together ?

ee
(i) the vowels always come together ? rF
SOLUTION There are 8 letters in the word 'DAUGHTER', including 3 vowels (A, U, E) and 5

Fr
consonants (D, G, H, T, R).
(i) Considering three vowels as one letter, we have 6 letters which can be arranged in P^, = 6!
for
ways, But, corresponding each way of these arrangements, the vowels A, U, E can be put
u
together in 3! ways.
ks
Hence, required number of words = 6! x 3! = 720 x 6 - 4320
Yo
o

(ii) The total number of words formed by using all the eight letters of the word 'DAUGHTER is
Bo

8
P3=8! = 40320.
re

So, the total number of words in which vowels are never together
= Total number of words - Number of words in which vowels are always together
ou
ad

= 40320-4320 = 36000
Y

EXAMPLE 8 In how many ways can 9 examination papers be arranged so that the best and the ivorst
papers are never together?
nd
Re

SOLUTION The number of arrangements in which the best and the worst papers never come
together can be obtained by subtracting from the total number of arrangements, the number of
Fi

arrangements in which the best and worst come together.


The total number of arrangements of 9 papers = ^Pg = 9!
Considering the best and the worst papers as one paper, we have 8 papers which can be
arranged in ^Pg = 8! ways. But, the best and worst papers can be put together in 2! ways. So, the
number of permutations in which the best and the worst papers can be put together = (2! x 8!).
Hence, the number of ways in which the best and the worst papers never come together
= 9!-2!x 8! = 9x 8!-2x 8! = 7 X 8! = 282240.
EXAMPLE 9 In hoiv many ways can 5 children be arranged in a row such that
(i) two of them, Ram and Shyam, are always together?
(ii) two of them, Ram and Shyam, are never together?
SOLUTION There are five children including Ram and Shyam.
10.32 APPLIED MATHEMATICS-XI

(i) Considering Ram and Shyam as one child, there are four children. They can be arranged in
ii'. a

row in 4! ways. But Ram and Shyam can be arranged together in 2! ways.
Hence, the required number of arrangements = 4! x 2! = 48.
(ii) Total number of arrangements of 5 children in a row = 5! = 120.
.'. Total number of arrangements in which Ram and Shyam are never together
= Total number of arrangements - Number of arrangements in which Ram and Shyam
are together
= 120-48 = 72.

EXAMPLE 10 A code zuord is to consist of two distinct English alphabets followed by two distinct
numbers from 1 to 9. For example, CA 23 is a code zuord. Hozv many such code zvords are there? Hozo
many of them end zuith an even integer ?
SOLUTION There are 26 English alphabets. So, first two places in the code word can be filled in

w
P2 ways. In last two places we have to use two distinct numbers from 1 to 9. So, last two places
can be filled in ^^2 ways. Hence, by the fundamental principle of counting, the total number of

Flo
26
code words = ?2 X = 650 X 72 = 46800.

ee
Number of code words ending with an even integer.
In this case, the code word can have any of the numbers 2,4,6,8 at the extreme right position. So,

Fr
the extreme right position can be filled in 4 ways. Now, next left position can be filled with any
one of the remaining 8 digits in 8 ways and the two extreme left positions can be filled by two
English alphabets in ^^P2 ways. for
ur
Hence, the total number of code words which end with an even integer = 4 x 8 x ^^Pj
s
= 4 X 8 X 650 = 20800.
ok
Yo

EXAMPLE 11
The Principal zvants to arrange 5 students on the platform such that the boy 'SALIM'
Bo

occupies the second position and such that the girl, 'SITA' is aizuays adjacent to the girl 'RITA'. Hozo
many such arrangements are possible ?
re

SOLUTION Since SALIM occupies the second position and the two girls RITA and SITA are
always adjacent to each other. So, none of these two girls can occupy the first seat. Thus, first seat
ou
ad

can
be occupied by any one of the remaining two students in 2 ways. Second seat can be
Y

occupied by SALIM in only one way.


Now, in the remaining three seats SITA and RITA can be seated in the following four ways:
nd
Re

I II in IV V
Fi

1. X SALIM SITA RITA X

2. X SALIM RITA SITA X

3. X SALIM X SITA RITA

4. X SALIM X RITA SITA

Now, only one seat is left which can be occupied by the 5th student in one way.
Hence, the number of required type of arrangements = 2x4x1= 8.
EXAMPLE 1:
Hozv majiy numbers betzveen 400 and 1000 can be formed zuith the digits 0,2,3,4,5,6 if
no digit is repented in the same number ?
SOLUTION Number between 400 and 1000 consist of three digits with digit at hundred's place
greater than or equal to 4. Hundred's place can be filled, by using the digits 4, 5, 6 in 3 ways.
Now, ten's and unit's places can be filled by the remaining 5 digits in ^P2 ways.
PERMUTATIONS 10.33

5 5!
Hence, the required number of numbers = 3 x = 3 x — = 3 x 20 = 60.

KXAM I’LE 13 In a class of 10 students there are 3 girls A, B, C. In how many different ways can they be
arranged in a row such that no tzuo of the three girls are consecutive,
●j
SOLUTION There are 7 boys and 3 girls. Seven boys can be arranged in a row in /^ = 7! ways.
Q

Now, we have 8 places in which we can arrange 3 girls in P3 ways.


Q

Hence, by the fundamental principle of counting, the number of arrangements = 7! x P3


= 7!x 336.

EXAMPLE 14
When a group photograph is taken, all the seven teachers should be in the first row and all

ow
the twenty students should be in the second roio. If the two comers of the second row are reserved for the
two tallest students, interchangeable only between them, and if the middle seat of the front row is reserved
for the Principal, hoio many arrangements are possible?
SOLUTION Since the middle seat of the front row is reserved for the Principal, the remaining 6

e
teachers can be arranged in the fron t row in ^ = 6! ways.

re
The two corners of the second row are reserved for

rFl
the two tallest students. They can occupy

F
these two places in 2! ways. The remaining 18 seats may be occupied by the remaining 18
students in 18! ways.

r
Hence, by the fundamental principle of counting, the total number of arrangements
ou
= 6!x(18!x2!)=18!xl440.
fo
ks
EXAMPLE 15
How many even numbers are there loith three digits such that if 5 is one of the digits, then
oo

7 is the next digit?


SOLUTION We have to determine the total number of even numbers formed by using the given
Y
eB

condition. So, at unit's place we can use one of the digits 0,2,4,6,8. If 5 is at ten's place then, as
per the given condition, 7 should be at unit's place. In such a case the number will not be an even
number. So, 5 cannot be at ten's and one's places. Hence, 5 can be only at hundred's place.
r
ou

Now two cases arise.


ad
Y

CASE I
When 5 is at hundred’s place:
If 5 is at hundred's place, then 7 will be at ten's place. So, unit's place can be filled in 5 ways by
d

using the digits 0,2,4,6, 8.


Re
in

So, total number of even numbers =1x1x5= 5.


F

CASE n
When 5 is not at hundred's place:
In this case, hundred's place can be filled in 8 ways (0 and 5 cannot be used at hundred's place).
In ten's place we can use any one of the ten digits except 5. So, ten's place can be filled in 9 ways.
At unit's place we have to use one of the even digits 0,2,4,6,8. So, units place can be filled in 5
ways.
So, total number of even numbers = 8x9x5 = 360
Hence, the total number of required even numbers = 360 + 5 = 365.
EXAMPLE 1(>
How many four digit numbers divisible by 4 can be made with the digits 1,2,3,4,5 if the
repetition of digits is not allozved ?
SOLUTION Recall that a number is divisible by 4 if the number formed by the last two digits is
divisible by 4. The digits at unit's and ten's places can be arranged as follows:
Th H T O
1 2
10.34 APPLIED MATHEMATICS-XI

2 4

3 2
5 2

Now, corresponding each such way the remaining three digits at thousand's and hundred's
places can be arranged in ^P2 ways.
Hence, the required number of numbers = '^P2 x 4 = 3! x 4 = 24.
I.XAMI’LE 17
Find the number of ways in which 5 boys and 5 girls be seated in a row so that
(i) No two girls may sit together. (ii) All the girls sit together and all the boys sit together.
(iii) All the girls are never together.
SOLUTION (i) 5 boys can be seated in a row in =5! ways. Now, in the 6 gaps 5 girls can be

w
arranged in ways.
Hence, the number of ways in which no two girls sit together = 5! x =5!x 6!

Flo
(ii) The two groups of girls and boys can be arranged in 2! ways. 5 girls can be arranged
among themselves in 5! ways. Similarly, 5 boys can be arranged among themselves in 5!
ways. Hence, by the fundamental principle of counting, the total number of requisite

ee
seating arrangements = 2!(5! x 5!) = 2 (5!)^.

Fr
(iii) The total number of ways in which all the girls are never together
= Total number of arrangements - Total number of arrangements in which all the girls are
for
ur
always together
= 10!-5!x 6!
s
EXAMPLE 18
Five hoys and five girls form a line with the boys and girls alternating. Find the number of
ok
Yo

ways of making the line.


Bo

SOLUTION 5 boys can be arranged in a line in =5! ways. Since the boys and girls are
alternating. So, corresponding each of the 5! ways of arrangements of 5 boys we obtain 5 places
re

marked by cross as shown below:


(i) X ^2 ^ 63 X S4 X ^ X (ii) X X 82 ^ S3 S4 X B5.
ou
ad

Clearly, 5 girls can be arranged in 5 places marked by cross in (5! + 5!) ways.
Y

Hence, the total number of ways of making the line =5!x (5!+ 5!) =2(5!)^
nd
Re

EXAMPLE 19 In how many ways three girls and nine boys can be seated in two vans, each having
numbered seats, 3 in the front and 4 at the back? Hoiv many seating arrangements are possible if 3 girls sit
Fi

together in a back row on adjacent seats ?


SOLUTION Total number of persons = 3 girls + 9 boys = 12.
Total number of numbered seats = 2x 3 + 4x2 = 14

So, total number of ways in which 12 persons can be seated on 14 seats


= Number of arrangements or 14 seats by taking 12 at a time =
Three girls can be seated together in a back row on adjacent seats in the following ways:
1,2,3 or 2,3,4 of first van
and, 1, 2,3 or 2,3,4 of second one.

In each way the tluree girls can interchange among themselves in 3! ways. So, the total number of
ways in which three girls can be seated together in a back row on adjacent seats = 4x3!
11
Now, 9 boys are to be seated on remaining 11 seats, which can be done in P9 ways.
10.35
PERMUTATIONS

Hence, by the fundamental principle of counting, the total number of seating arrangements is
11
P9 X 4x 3!.
EXAMf’LE 20 A ten party is arranged for 16 persons along tzvo sides of a long tabic ii'ith 8 chairs on each
side. Four piersons wish to sit on one particular and two on the other side. In how many xvays can they he
seated?

SOLUTION Let the two sides be A and B. Assume that four persons wish to sit on side A. Four
persons who wish to sit on side A can be accommodated on eight chairs in ^^4 ways and two

persons who wish to sit on side B can be accommodated on 8 chairs in ^P2 ways. Now, 10
persons are left, who can sit on 10 chairs on both the sides of the table in 10! ways.
8 8
Hence, the total number of ways in which 16 persons can be seated P4X P2 X 10!
EXERCISE 10.4

In how many ways can the letters of the word 'FAILURE' be arranged so that the

w
1.

consonants may occupy only odd positions?


In how many ways can the letters of the word 'STRANGE be arranged so that

F lo
2.

(i) the vowels come together ? (ii) the vowels never come together? and
(iii) the vowels occupy only the odd places?

ee
3. How many words can be formed from the letters of the word 'SUNDAY'? How many of

Fr
these begin with D?
4. How many words can be formed out of the letters of the word, ORIENTAL , so that the
for
vowels always occupy the odd places ?
ur
5. How many different words can be formed with the letters of word SUNDAY ? How many
of the words begin with N? How many begin with N and end in Y?
s
ook
Yo

6. How many different words can be formed from the letters of the word 'GANESHPURI ? In
how many of these words:
eB

(i) the letter G always occupies the first place?


(ii) the letters P and I respectively occupy first and last place?
our

(iii) the vowels are always together?


ad

(iv) the vowels always occupy even places?


7. How many permutations can be formed by the letters of the word, VOWELS , when
Y

(i) there is no restriction on letters? (ii) each word begins with E?


Re

(iii) each word begins with O and ends with L? (iv) all vowels come together?
nd

(v) all consonants come together?


Fi

8. How many words can be formed out of the letters of the word 'ARTICLE', so that vowels
occupy even places?
9. In how many ways can a lawn tennis mixed double be made up from seven married
couples if no husband and wife play in the same set?
10. m men and n women are to be seated in a row so that no two women sit together. If m > n
m\{ni +1)!
then show that the number of ways in which they can be seated as (hi - II +1)!

11. How many words (with or without dictionary meaning) can be made from the letters in the
word MONDAY, assuming that no letter is repeated, if
(i) 4 letters are used at a time? (ii) all letters are used at a time?
(iii) all letters are used but first is vowel?
10.36
APPLIED MATHEMATICS-XI

12. How many three letter words can be made using the letters of the word 'ORIENTAL'?

ANSWERS
1. 576 2. (i)1440 (ii) 3600 (iii) 1440 3. 720,120 4. 576
720,120,24 6, 10! (i) 9! (ii) 8! (iii) 7!x 4! (iv) 5!x 6!
7. (i) 720 (ii) 120 (iii) 24 (iv) 240 (v) 144 8. 144 9. 840
11. (i) 360 (ii) 720 (iii) 240 12. 336

10.5 PERMUTATIONS OF OBJECTS NOT ALL DISTINCT


So far we were discusing permutations of distinct objects (things) by taking some or all at a time.
In this section, we intend to discuss the permutations of a given number of objects when objects
are
not all different. For example, the number of arrangements of the letters of the word
MISSISSIPPI, the number of six digit numbers formed by using the digits 1,1,2,3,3,4 etc. The
following theorem is very helpful to determine the number of such arrangements.

w
THEOREM The number of miitiinlly distinguishable permutations of n things, taken all at a
time, of
. n\
ivhicJi p are alike of one kind, q alike of second such that p + q = n IS

F lo
p\q\

PROOF Let the required number of permutations be Consider one of these .v permutations.

ee
Now, replace p alike things in this permutation by p distinct things which are also different from

Fr
others. These p different things may be permuted among themselves in pi ways without
changing the positions of other things. Similarly, if we replace q alike things by q distinct things,
for
which are also different from others, then they can be permuted among themselves in ql ways.
ur
Thus, if both the replacements are done simultaneously, then we find that each one of the :r
permutations give rise to pi x q\ permutations. Therefore, x permutations give rise to xxplx ql
s
ook

permutations. Now, each of these xx pi x ijl permutations, is a permutation of n different things,


Yo

taken all at a time.


eB

XX plx g! = Number of permutations of n different things taken all at a time=ji!


nl
Hence, X

plql
our
ad

Q.E.D.
RF-MARK 1
The number of permutations of n things, of which pi are alike of one kind; p2 are alike of
Y

second kind; p^ are alike of third kind;...; p^ are alike ofrth kind such that p-^ + p2 + ... + Pr = n,is
Re
nd

nl

Pilp2^-P2,'--Pr !
Fi

KFMARK2 The number of permutations ofn things, ofzohich p are alike of one kind, q are alike of second
nl
kind and remaining all are distinct, is
plql

REMARtU Suppose there are r things to be arranged, allowing repetitions. Letfiirtherpi,p2,...p^


the integers such that the ifrst object occurs exactly p^ limes, the second occurs exactly p2 times, etc. Then
the total number of permutations of these r objects to the above condition is + - + Pr)!
Pl-PZ-P^'—Pr'-'
ILLUSTRATIVE EXAMPLES

EXAMPLE] How many diferent words can be formed luitli the letters of the word 'MISSISSIPPI' ? In
how many of these permutations four Vs do not come together?
PERMUTATIONS 10.37

SOLUTION There are 11 letters in the given word, of which 4 are S's, 4 are Ts and 2 are P's. So,
total number of words is the number of arrangements of 11 things, of which 4 are similar of one
11!
kind, 4 are similar of second kind and 2 are similar of third kind i.e.
4!4!2!
11!
Hence,the total number of words = = 34650.
4!4!2!

Considering 41's as one letter, we have 8 letters of which 4 are S's and 2 are P's. These 8 letters
8!
can be arranged in ways.
4!2!
8!
Number of words in which 4 I's come together = 4!2!
= 840.

Hence, number of words in which 4 I's do not come together = 34650 - 840 = 33810.
EXAMPLE 2 How many permutations of the letters of the word 'APPLE' are there?
SOLUTION Here there are 5 letters, two of which are of the same kind. The others are each of its
5! 120

w
own kind. So, the required number of permutations is = 60.
211I1I1! 2

EXAMPLE
thrice ?
SOLUTION F lo
How many words can be formed using the letter A thrice, the letter B twice and the letter C

We are given 8 letters viz. A, A, A, B, B, C, C, C. Clearly, there are 8 letters of which


three are of one kind, two are of second kind and three are of third kind.
So, the total number of permutations =
8!

312! 3!
= 560.
for F
ree
Hence, the requisite number of words = 560.
EXAMPLE 4
Find the number of dijferent permutations of the letters of the word BANANA ?
Clearly, there are six letters in the word 'BANANA' of which three are alike of one
Your

SOLUTION
ks

kind (3 A's), two are alike of second kind (2 N's) and one of its own kind.
eBoo

6!
Total number of their permutations = 312I1I
= 60.

Hence, the requisite number of words = 60


ad
our

EXAMPLES (i) Hoiv many different words can be formed with the letters of the word HARYANA?
(ii) Hoiv many of these begin with H and end with N?
(iii) In hoiu many of these H and N are together?
Re

SOLUTION (i) There are 7 letters in the word 'HARYANA' of which 3 are A's and remaining all
Y

are each of its own kind.


Find

7! 7!
So, total number of words = — = 840.
3!1!1!1!1! 3!

(ii) After fixing H in first place and N in last place, we have 5 letters out of which three are alike
i.e. A's and remaining all are each of its own kind.
5!
So, total number of words = — = 20.
3!
(hi) Considering H and N together we have 7 - 2 + 1 = 6 letters out of which three are alike i.e.
A's and others are each of its own kind. These six letters can be arranged in — ways. But H and N

can be arranged amongst themselves in 2! ways.


6!
Hence, the requisite number of words = —3!
x 2! = 120 x 2 = 240.
EX ^ ^
Hoio many different words can be formed by using all the letters of the word
'ALLAHABAD' ?
(i) In how many of them votoels occupy the even positions ?
(ii) In how many of them both L do not come together ?
10.38 APPLIED MATHEMATICS-XI

SOLUTION There are 9 letters in the word 'ALLAHABAD' out of which 4 are A's, 2 are L's and
the rest are all distinct.
9!
So, the requisite number of words = = 7560.
4!2!

(i) There are 4 vowels and all are alike i.e. 4 A's. Also, there are 4 even places viz 2nd, 4th, 6th
41
and 8th. So, these 4 even places can be occupied by 4 vowels in —
4!
= 1 way. Now, we are left with
5!
5 places in which 5 letters, of which two are alike (2 L's) and other distinct, can be arranged in ^
2!

ways.

w
Hence, the total number of words in which vowels occupy the even places = — x — = — = 60.
2! 4! 2!

(ii) Considering both L together and treating them as one letter we have 8 letters out of which A

o
8!

e
repeats 4 times and others are distinct. These 8 letters can be arranged in — ways.
4!

re
8!

Fr l
So, the number of words in which both L come together = = 1680.

F
4!

Hence, the number of words in which both L do not come together


= Total no, of words - No. of words in which both L come together = 7560 -1680 = 5880.

or
ou
EXAMPLE 7 Find the number of arrangements of the letters of the word INDEPENDENCE. In hozv
kfs
many of these arrangements
(i) do the -words start with P? (ii) do all the vozuels alxuays occur together?
oo

(iii) do all the vowels never occur together? (iv) do the zvords begin with 1 and end in P?
SOLUTION In the word 'INDEPENDENCE' there are 12 letters of which 3 are N's,4are E's and
Y

2 are D's. Therefore,


B

12!
Total number of arrangements = = 1663200
re

31412!

(i) After fixing the letter P at the extreme left position, there are 11 letters consisting of 3 N's,
oYu

11!
ad

4E's and 2D's. These 11 letters can be arranged in 3!4!2! = 138600


11!
d

Number of words beginning with P = = 138600


3!4!2!
in
Re

(ii) There are 5 vowels in the given word of which 4 are E's and one I. These vowels can be put
5!
F

together in ways. Considering these 5 vowels as one letter there are 8 letters (taking 7
4!1!
8!
remaining letters) which can be arranged in ways (as there are 3 N's and 2D's). Since
3!2!
8!
corresponding to each arrangement of 5 vowels there are ways of arranging remaining 7
3!2!

letters and one letter formed by 5 vowels.


Hence, by fundamental principle of multiplication, the required number of arrangements IS
i
— =16800
3!2! 4!1!

(iii) The required number of arrangements


= The total number of arrangements - The number of arrangements in which all the
vowels occur together
= 1663200 -16800 =1646400
10.39
PERMUTATIONS

(iv) Let us fix I at the extreme left end and P at the extreme right end. Now, we are left with 10
letters of which 3 are N's, 4 are E's and 2 are D's. These ten letters can be arranged in 4!3!2!

ways.
10!
= 12600.
Hence, required number of arrangements = 4!3!2I

EXAMPLE 8 In how mmii/ zoaijs can the letters of the word PERMUTATIONS be arranged if(i) the
zuords start with P end with S (ii) vowels are all together.
SOLUTION (i) There are 12 letters in the given word of which 2 ^'s and the remaining are
distinct. Remaining 10 letters between P and S can be arranged in 2!
ways.

10!
= 1814400
Total number of words starting with P and ending in S - 2!

w
(ii) There are 5 vowels in the given word. These vowels can be put together in 5! ways.
Considering these 5 vowels as one letter, we have 8 letters (7 remaining letters and one letter

F lo
formed by 5 vowels) of which 2 are T's. These 8 letters can be arranged in - ways.
Hence, by the fundamental principle of multiplication, required number of words i_
is

ee
Fr
5 !x —= 2419200.
2!

EXAMPLES Hozv mam/ numbers greater than a million can beformed with the digits 2,3,0,3,4,2,3 ?
for
SOLUTION Any number greater than a million will contain all the seven digits.
ur
Now, we have to arrange these seven digits, out of which 2 occur twice, 3 occurs twice and the
s
ook

rest are distinct.


Yo

7!
= 420.
The number of such arrangements =
eB

2!x 3!

These arrangements also include those numbers which contain 0 at the million s place.
Keeping 0 fixed at the millionth place, we have 6 digits out of whkh 2 occurs twice, 3 occurs
our
ad

thrice and the rest are distinct. These 6 digits can be arranged in = 60 ways.
Y

Hence, the number of required numbers = 420 - 60 = 360.


Re

EXAMPLE 10 There are six periods in each working day of a school In how znany ways can one arrange 5
nd

subjects such that each subject is allowed at least one period?


Fi

SOLUTION Since each subject is allowed at least one period. So, we first select one|ubject for
the left out period. This can be done in ways. Now, six subject can be arranged in- ways.
5 6!
Hence, the total number of arrangements = Cjx —=1800
EXAMPLE 11 How many arrangements can be made with the letters of the xvord 'MATHEMATICS ? In
how many of them vowels are together ?
SOLUTION There are 11 letters in the word 'MATHEMATICS' of which two are M s, two are A s,
two are T's and all other are distinct. So,
11!
= 4989600
Required number of arrangements = 21x 2!x 2!
10.40
APPLIED MATHEMATICS-XI

There are 4 vowels viz. A, E, A, 1. Considering these four vowels as one letter we have 8 letters
(M, T, H, M, T, C, S and one letter obtained by combining all vowels), out of which M occurs
8!
twice, T occurs twice and the rest all different. These 8 letters can be arranged m
i ways.
2!x 2!

But, the four vowels {A, E, A, 1) can be put together in — ways.

rw
4!
Hence, the total number of arrangements in which vowels are always together = X —
2!x2! 2!

= 10080x 12=120960.
I \ AMPI.E 12
If all the letters of the word 'AGAIN' be arranged ns in a dictionary, what is the fiftieth

e
word?

e
SOLUTION In dictionary the words at each stage are arranged in alphabetical order. Starting

lo
with the letter A, and arranging the other four letters GAIN, we obtain 4! = 24 words.

r
Thus, there are 24 words which start with A. These are the first 24 words.

F
Them|tarting with G, and arranging the other four letters A, A, 1, N in different ways, we obtain

u
2!
oF
= — = 12 words. Thus, there are 12 words, which start with G.

rs
Now, we start with I. The remaining 4 letters A, G, A, N can be arranged in - = 12 ways. So,

ko
there are 12 words, which start with I.
Thus, we have so far constructed 48 words. The 49th word is NAAGI and hence the 50th word is
NAAIG. of
o
Y
EXAMPLE 13 letters of the word 'RANDOM' are -written in all possible orders and these words are
written out as in a dictionary. Find the rank of the word 'RANDOM'.
rYB

SOLUTION In a dictionary the words at each stage are arranged in alphabetical order. In the
given problem we must therefore consider the words begiruring with A, D, M, N, O, R in order.
A will occur in the first place as often as there are ways of arranging the remaining 5 letters all at
ue

a time i.e. A will occur 5! times. Similarly, D, M, N, O will occur in the first place the same
number of times.
od
ad

Number of words starting with A = 5! = 120


Number of words starting with D = 5! = 120
in

Number of words starting with M = 5! = 120


Number of words starting with N = 5! = 120
Re
F

Number of words starting with 0 = 5! = 120


Number of words beginning with R is 51, but one of these words is the word RANDOM. So, we
first find the number of words beginning with RAD and RAM.
No. of words starting with RAD = 3! = 6
No. of words starting with RAM = 3! = 6
Now, the words beginning with 'RAN' must follow. There are 3! words beginning with RAN.
One of these words is the word RANDOM itself.
The first word beginning with RAN is the word RANDMO and the next word is RANDOM.
Rank of RANDOM = 5x 120+ 2x6 + 2 = 614.
EXA.MPLE 14 different permutations of the word, 'EXAMINATION' are listed as in a dictionary,
how many items are there in the list before the first zuord starting xoith E?
SOLUTION In a dictionary the words at each stage are arranged in alphabetical order. In the
given problem we have to find the total number of words starting with A, because the very next
word will start with E. ^
PERMUTATIONS 10.41

For finding the number of words starting with A, we have to find the number of arrangements
of the remaining 10 letters, EXMINATION, of which there are 21's, 2 N's and the others each of
its own kind.
10!
The number of such arrangements = = 907200.
212!

Hence, the required number of items = 907200.

EXERCISE 10.5

1. Find the number of words formed by permuting all the letters of the following words:
(i) INDEPENDENCE (ii) INTERMEDIATE (iii) ARRANGE
(iv) INDIA (v) PAKISTAN (vi) RUSSIA

ow
(vii) SERIES (viii) EXERCISES (ix) CONSTANTINOPLE
2. In how many ways can the letters of the word 'ALGEBRA' be arranged without changing
the relative order of the vowels and consonants?

3. How many words can be formed with the letters of the word 'UNIVERSITY', the vowels
remaining together?

e
Fl
re
4. Find the total number of arrangements of the letters in the expression a b c when

F
written at full length.
How many words can be formed with the letters of the word 'PARALLEL' so that all L's do
ur
o.

not come together?


or
b. How many words can be formed by arranging the letters of the word 'MUMBAI' so that all
sf
M's come together?
k
Yo
7. How many numbers can be formed with the digits 1, 2,3, 4, 3, 2,1 so that the odd digits
oo

always occupy the odd places?


8. How many different signals can be made from 4 red, 2 white and 3 green flags by arranging
B

all of them vertically on a flagstaff?


e

9. How many number of four digits can be formed with the digits 1,3,3,0?
ur

10. In how many ways can the letters of the word 'ARRANGE' be arranged so that the two R's
ad

are never together?


Yo

11. How many different numbers, greater than 50000 can be formed with the digits 0,1,1,5,9.
12, How many words can be formed from the letters of the word 'SERIES' which start with S
d

and end with S?


Re
in

13. How many permutations of the letters of the word 'MADHUBANI' do not begin with M
F

but end with I?

14. Find the number of numbers, greater than a million, that can be formed with the digits 2,3,
0, 3, 4, 2, 3.
15. There are three copies each of 4 different books. In how many ways can they be arranged in
a shelf?

16. How many different arrangements can be made by using ail the letter in the word
'MATHEMATICS'. How many of them begin with C? How many of tlaem begin with T?
17. A biologist studying the genetic code is interested to know the number of possible
arrangements of 12 molecules in a chain. The chain contains 4 different molecules
represented by the initials A (for Adenine), C (for Cytosine), G (for Guanine) and T (for
Thymine) and 3 molecules of each kind. How many different such arrangements are
possible?
18. In how many ways can 4 red, 3 yellow and 2 green discs be arranged in a row if the discs of
the same colour are indistinguishable?
10.42 APPLIED MATHEMATICS-Xl

19. How many numbers greater than 1000000 can be formed by using the digits 1, 2, 0, 1,
4, 2, 4?
20. In how many ways can the letters of the word ASSASSINATION be arranged so that all the
S's are together?
21. Find the total number of permutations of the letters of the word 'INSTITUTE'.
22. The letters of the word 'SURITI' are written in all possible orders and these words are
written out as in a dictionary. Find the rank of the word 'SURITI'.
23. If the letters of the word 'LATE' be permuted and the words so formed be arranged as in a
dictionary, find the rank of the word LATE.
24. If the letters of the word 'MOTHER' are written in all possible orders and these words are

ow
written out as in a dictionary, find the rank of the word 'MOTHER'.
25. If the permutations of a, b, c, d, e taken all together be written down in alphabetical order as
in dictionary and numbered, find the rank of the permutation debac.
26. Find the total number of ways in which six '+' and four signs can be arranged in a line

e
such that no two'-' signs occur together.

re
27.

rFl
In how many ways can the letters of the word "INTERMEDIATE" be arranged so that:

F
(i) the vowels always occupy even places?
(ii) the relative order of vowels and consonantsdo not alter?

r
The letters of the word 'ZENITH' are written in all possible orders. How many words are
ou
28.
fo
possible if all these words are written out as in a dictionary ? What is the rank of the word
ks
'ZENITH'?

ANSWERS
oo
Y

1. (i) 1663200 (ii) 19958400 (iii) 1260 (iv) 60


eB

(v) 20160 (vi) 360 (vii) 180 (viii) 30240


14!
ur

(ix) 2. 72 3. 60480 4. 1260


24
ad
Yo

D. 3000 6. 120 7. 18 8. 1260

9. 9 10. 900 11. 24 12. 12


d

11! 10! 10!


14. 360 12!/(3!)'* 16.
Re

13. 17640 15.


in

2!2!2!" 2!2!2l" 2!2!


F

17. 369600 18. 1260 19. 360 20. 151200

9!
21. 22. 236 23. 14 24. 309
2! 3!

25. 93 26. 35 27. (i) 21600 (ii) 21600


28. 616
HINTS TO SELECTED PROBLEMS

2. The consonants can be arranged among themselves in 4! ways and the vowels among
31 31
themselves in —
2!
ways. Hence, the required number of arrangements = 4! x —
2!
=72.
9!
4. There are 3 a's, 2 b's and 4 c's. So, the total number of arrangements = = 1260.
3!2!4!
PERMUTATIONS 10.43

7. There are 4 odd digits 1, 1, 3, 3 and 4 odd places. So, odd digits can be arranged in odd
4!
places in 2! 2! ways. The remaining 3 even digits 2, 2,4 can be arranged in 3 even places in
3! 4! 3!
ways. Hence, the requisite number of numbers = 2!2!
X — =lb.
2!

8. We have to arrange 9 flags, out of which 4 are of one kind, 2 are of another kind and 3 are of
9!
third kind. So, total number of signals =
4!2! 3!

4! 3!
9. Required number of numbers = —
^ 2! 2!

11. Numbers greater than 50000 will have either 5 or 9 in the first place and will consist of 5
digits.

w
4!
Number of numbers of with digit 5 at first place = —
2!

F lo
4!
Number of numbers with digit 9 at first place = —
2!
4' 4'

ee
Hence, the required number of numbers = ^
2!
+ Z: = 24.
2!

Fr
(4+3 + 2)! 9!
= 1260
18. Required number of ways = 4! 312! 4! 3!2! for
ur
19. Number of numbers greater than 1000000 that can be formed by using the digits 1,2,0,2,4,
2,4.
s

= Number of numbers formed by given digits - Number of numbers having 0 as left


ook
Yo

most digit
eB

7! 6! 71-6! 6x6!
= 360
312! 312! 312! 312!

Considering all S as one letter there are 10 letters containing 3A's, 2I's, 2N's, IT, lO which
r

20.
ou
ad

10!
can be arranged in = 151200 ways.
31212!
Y

21. Tlnere are 9 letters in the word INSTITUTE containing 2Ts, 3T's, IN, IS, lU and IE. These
Re
nd

9!
letters can be arranged in = 21040 ways.
2! 3!
Fi

6!
26. Six '+' signs can be arranged in a row in 6!
= 1 way. Now, we are left with seven places in
which four different things can be arranged in ^ P4 ways but as all the four - signs are
7 p,
identical, therefore, four' - signs can be arranged in = 35 ways.
Hence, the required number of ways =1 x 35 = 35.
10.6 CIRCULAR PERMUTATIONS
So far we have discussed permutations of objects (or things) in a row. This type of permutations
are
generally known as linear permutations. If we arrange the objects along a closed curve viz. a
have seen in the earlier
circle, the permutations are known as circular permutations. As we
sections of this chapter that every linear arrangement has a beginning and an end, but there is
nothing like beginning or end in a circular permutation. Thus, in a circular permutation, we
10.44 APPLIED MATHEMATICS-XI

consider one object as fixed and the remaining objects are arranged as in case of linear
arrangements.
THEOREM
The number of circular permutations ofn distinct objects is (n -1)1
P_RQQF Let ^2/^3/●●●/ 1' t)e n distinct objects. Let the total number of circular
permutations be x. Consider one of these x permutations as shown in Fig. 10.1.
Clearly, this circular permutation provides n linear permutations as given below.
^2' ^3' ●●●'

<’5
^3, ^4, a^,...,a„^ a-i, ai a
n~2 «4
^4' %/●●●/ «n/ <^1/ (^2' ^3 a
n-1
'3

w
n
a-i 2
Fig, 10.1

F lo
^2’ ^3' ■■■' ...

Thus, each circular permutation gives n linear permutations. But, there are x circular
permutations. So, total number of linear permutations is xn. But, the number of linear

ee
permutations of n distinct objects is n\

Fr
«!
x/i=??! => x = —=(//-!)! Q.E.D
n


for
In the above theorem anti-clockioisc and clockwise order of arrangements are considered as
ur
distinct permutations.
DIFFERENCE BETWEEN CLOCKWISE AND ANTI-CLOCKWISE ARRANGEMENTS
s

Consider the
ook
Yo

following circular permutations. We observe that in both, the order of the circular arrangement
IS
f?!, ^2/ ^3/ <74. In Fig. 10.2 the order is anti-clockwise whereas in Fig. 10.3 the order is clockwise.
eB

Thus, the number of circular permutation of n things in which clockwise and anti-clockwise
arrangements give rise to different permutations is (n - 1)!. For example, the number of
permutations of 5 persons seated around a table is (5 -1)! = 4!. Because with respect to the table,
r
ad
ou

the clockwise and anti-clockwise arrangements are distinct.


"3
Y
Re
nd

«4 ^2 «4 ai
Fi

a a
1 1

Fig. 10.2 Fig. 10.3

If anti-clockwise and clockwise order of arrangements are not distinct e.g. arrangements of
beads in a necklace, arrangements of flowers in a garland etc., then the number of circular
permutations of n distinct items is ^ {(n -1)!).
ILLUSTRATIVE EXAMPLES

\\
In how many ways can 8 students be seated in a (i) line (ii) circle ?
SOLUTION (i) The number of ways in which 8 students can be seated in a line = ®Pg = 8! = 40320.
(ii) The number of ways in which 8 students can be seated in a circle = (8 -1)! = 7! = 5040.
PERMUTATIONS 10.45

EXAMFLli 2 (/) In how many ways can 5 persons be seated around a circular table ?
(ii) In hozo many of these arrangements will tzuo particular persons be next to each other ?
SOLUTION (i) Five persons can be seated around a circular table in (5 -1)! = 4! = 24 ways,
(ii) Considering two particular persons as one person, we have 4 persons in all. These 4 persons
can be seated around a circular table in (4 - 1)! = 3! ways. But two particular persons can be
arranged among themselves in 2! ways. Hence, the total number of arrangements = 3! x 2! = 12
EXAMPLE 3 1/20 persons zvere inzhtedfor a party, in hozo many zvays can they and the host be seated at a
circular table ? In hozv many of these zvays zvill tzvo particular persons be seated on either side of the host?
SOLUTION (i) Clearly, there are 21 persons, including the host, to be seated round a circular
table. These 21 persons can be seated round a circular table in (21 -1)! = 20! ways,
(ii) Let , P2 be two particular persons and H be the host. These two particular persons can be
seated on either side of the host in the following two ways : (i) Pj H P2 and (ii) P2 H P^.

w
Consider the two particular persons and the host as one person, we have 18 persons in all. These
19 persons can be seated round a circular table in (19 -1)1 = 18! ways. But two particular persons
can be seated on either side of the host in 2 ways. So, the number of ways of seating 21 persons at

Flo
a circular table with two particular persons on either side of the host = 18! x 2.
EXAMPLE ●} There are 20 persons among whom are tzvo brothers. Find the number of zvays in zvhich we

ee
can arrange therfi around a circle so that there is exactly one person betzveen the tzvo brothers.

Fr
SOLUTION Let and 82 be two brothers among 20 persons and let M be a person. Clearly,
person M can be chosen from 18 persons (excluding and 82) in 18 ways. Considering the two
brothers S) and 82 and person M as one person, we have 18 persons in all. These 18 persons can
for
ur
be arranged around a circle in (18 - 1)1 = 17! ways. But 6| and 82 can be arranged among
themselves in 2! ways (6^ M 82 and 82 M B|).
ks

Hence, the total number of ways = 18 x 17!x 2! = 2 x 181


Yo
oo

EXAMPLES In how many ways can a party of 4 men and 4 zvomen be seated at a circular table so that no
two women are adjacent ?
eB

SOLUTION The 4 men can be seated at the circular table such that there is a vacant seat between
every pair of men in (4 - 1)1 = 3! ways. Now, 4 vacant seats can be occupied by 4 women in 4!
r

ways. Hence, the required number of seating arrangements = 3! x 4! = 144.


ou
ad

EX.AMPLE 6 A round table conference is to be held betzveen 20 delegates of 2 countries. In how many
zvays can they be seated if tzvo particular delegates are (i) aizvays together? (ii) never together ?
Y

SOLUTION (i) Let andD2 be two particular delegates. Considering andD2 as one
Re
nd

delegate, we have 19 delegates in all. These 19 delegates can be seated round a circular table
in (19-1)! = 18! ways. But two particular delegates can arrange among themselves in 2!
Fi

ways (D;[ D2 and D2 Df). Hence, the total number of ways = 18! x 2! = 2 (18!)
(ii) To find the number of ways in which two particular delegates never sit together, we subtract
the number of ways in which they sit together from the total number of seating arrangements of
20 persons. Clearly 20 persons can be seated round a circular table in (20 -1)! = 19! ways.
Hence, the required number of seating arrangements = 19! - 2 x 18! =17 (18!)
EX.AMPLE 7 There are 5 gentlemen and 4 ladies to dine at a round table. In hozv many ways can they seat
themselves so that no two ladies are together ?
SOLUTION Five gentlemen can be seated at a round table in (5-1)! =4! ways. Now, 5 places are
created in which 4 ladies can be arranged in ways.
Hence, the total number of ways in which no two ladies sit together = 4! x = 4! x 5! = 2880.
EXAMPLE 8 In hozv many ways can seven persons sit around a table so that all shall not have the same
neighbours in any tzuo arrangements?
10.46 APPLIED MATHEMATICS-XI

SOLUTION Clearly, 7 persons can sit at a round table in (7-1)! = 6! ways. But, in clockwise and
anticlockwise arrangements, each person will have the same neighbours.
1
So, the required number of ways = — (6!) = 360.
EXAMPLE 9
Three boys and three girls are to be seated around a table in a circle. Among them, the boy X
does not want am/ girl neighbour and the girl Y does not want any hoy neighbour. Hozu many such
arrangements are possible?
SOLUTION Let , B2 and X be three boys and and G2 and Y be three girls. Since the boy X
does not want any girl neighbour. Therefore boy X will have his neighbours as boys and B2 as
shown in Fig. 10.4. Similarly, girl Y has her neighbours as girls Gj and G2 as shown in Fig. 10.4.
But, the boys and 62 can be arranged among themselves in 2! ways and the girls G^ and G2
can be arranged among themselves in 2! ways.

w
B

Flo
ee
'2

Fr
X
Fig. 10.4

Hence, the required number of arrangements = 2! x 2! = 4. for


ur
EXAMPLE 10 Find the number of ways in -which 8 different flowers can be strung to form a garland so
that 4 particular flowers are never separated.
k s
Yo

SOLUTION Considering 4 particular flowers as one flower, we have five flowers which can be
oo

strung to form a garland in 4! ways. But 4 particular flowers can be arranged in 4! ways. Thus,
eB

the required number of ways = 4! x 4! =576.


EXAMPLE 11 Find the number of ways in zohich 10 different beads can be arranged to form a necklace.
Ten different beads can be arranged in circular form in (10 - 1)! = 9! ways. Since
r

SOLUTION
ou
ad

there is no distinction between the clockwise and anticlockwise arrangements. So, the required
1
number of arrangements = — (9!)
Y
Re
nd

EXERCISE 10.6
Fi

1. In how many ways 10 persons may be arranged in a (i) line (ii) circle ?
2. In how many ways 8 beads can be arranged to form a necklace ?
3. In how many ways can 7 persons form a ring ? In how many ways can 7 Indian and 7
Americans sit down at a round table, no two Americans being together ?
4. In how many ways can 11 members of a committee sit at a round table so tliat the secretary
and the joint secretary are always the neighbours of the president ?
5. A gentleman invites 6 of his friends to a party. In how many different arrangements they,
along with the wife of the gentleman can sit at a round table for a dinner if the host and his
wife always sit side by side?
6. In how many ways can a garland of 20 different flowers be made ?
/.
In how many ways can n men be seated around a round table when in no two ways a man
has the same neighbours ?
8. In how many ways can 6 Hindus and 6 Muslims sit around a round table so that two
Muslims may never sit together ?
PERMUTATIONS 10.47

ANSWERS

~(7!) 3. 720,3628800 4. 80640


1. (i) 10! (ii) 9! 2.

1
6. 1 8. 86400
^(19!)
5. 1440 7.

MULTIPLE CHOICE QUESTIONS (MCQs)

Mark the correct alternative in each of the foUozving:


1. The number of permutations of n different things taking rata time when 3 particular things
are to be included is

(a)
II -
3p‘r-3. (b)
II- 3
Pr (c) (d) r!''-^C r -3

w
2. The number of five-digit telephone numbers having at least one of their digits repeated is
(a) 90000 (b) 100000 (c) 30240 (d) 69760

Flo
3. The number of words that can be formed out of the letters of the word "ARTICLE" so that
vowels occupy even places is

ee
(a) 574 (b) 36 (c) 754 (d) 144

Fr
4. How many numbers greater than 10 lacs be formed from 2,3,0,3,4, 2,3 ?
(a) 420 (b) 360 (c) 400 (d) 300
for
ur
5. The number of different signals which can be given from 6 flags of different colours taking
one or more at a time, is
ks
(a) 1958 (b) 1956 (c) 16 (d) 64
Yo

6. The number of words from the letters of the word 'BHARAT' in which B and H will never
oo

come together, is
eB

(a) 360 (b) 240 (c) 120 (d) none of these


7. The number of six letter words that can be formed using the letters of the word "ASSIST" in
r
ou

which S's alternate with other letters is


ad

(a) 12 (b) 24 (c) 18 (d) none of these


Y

8. The number of arrangements of the word "DELHI" in which £ precedes / is


nd

(d) 59
Re

(a) 30 (b) 60 (c) 120


9. The number of ways in which the letters of the word 'CONSTANT' can be arranged
Fi

without changing the relative positions of the vowels and consonants is


(a) 360 (b) 256 (c) 444 (d) none of these

10. The number of ways to arrange the letters of the word CHEESE are
(a) 120 (b) 240 (c) 720 (d) 6
11. Number of all four digit numbers having different digits formed of the digits 1,2,3,4 and 5
and divisible by 4 is
(a) 24 (b) 30 (c) 125 (d) 100
12. If the letters of the word KRISNA are arranged in all possible ways and these words are
written out as in a dictionary, then the rank of the word KRISNA is
(a) 324 (b) 341 (c) 359 (d) none of these
13. If in a group of h distinct objects, the number of arrangements of 4 objects is 12 times the
number of arrangements of 2 objects, then the number of objects is
10.48 APPLIED MATHEMATICS-XI

(a) 10 (b) 8 (c) 6 (d) none of these


14. The number of ways in which 6 men can be arranged in a row so that three particular men
are consecutive, is
(a) 4!x 3! (b) 4! (c) 3!x 3! (d) none of these
15. A 5-digit number divisible by 3 is to be formed using the digits 0,1, 2, 3, 4 and 5 without
repetition. The total number of ways in which this can be done is
(a) 216 (b) 600 (c) 240 (d) 3125
16. The product of r consecutive positive integers is divisible by
(a) r! (b) r!+l (c) (r + l)! (d) none of these

17. If k^5p 1 - Pj^, then the values of k are


2

w
(a) 7 and 11 (b) 6 and 7 (c) 2 and 11 (d) 2 and 6
18. The number of arrangements of the letters of the word BHARAT taking 3 at a time is

F lo
(a) 72 (b) 120 (c) 14 (d) none of these.

ee
19. The number of words that can be made by re-arranging the letters of the word APURBA so
that vowels and consonants are alternate is

Fr
(a) 18 (b) 35 (c) 36 (d) none of these
for
20. The number of different ways in which 8 persons can stand in a row so that between two
ur
particular persons A and B there are always two persons, is
(a) 60x5! (b) 15 X 41x5! (c) 4!x5! (d) none of these
s
ook
Yo

21. The number of ways in which the letters of the word ARTICLE can be arranged so that even
places are always occupied by consonants is
eB

(a) 576 (b) ^C3x4! (c) 2x4! (d) none of these


22. In a room there are 12 bulbs of the same wattage, each having a separate switch. The
r
ou
ad

number of ways to light the room with different amounts of illumination is


(a) 12^-1 (b) 2
12
(C) 2^2 _1 (d) none of these
Y

23. There are four bus routes between A and B; and three bus routes between B and C. A man
Re
nd

can travel round trip in number of ways by bus from A to C via B. If he does not want to use
a bus route more than once, the number of ways he can make round trip, is
Fi

(a) 72 (b) 144 (c) 14 (d) 19


Ail the letters of the word 'EAMCET' are arranged in different possible ways. The number
of such arrangements in which two vowels are adjacent to each other, is
(a) 360 (b) 144 (c) 72 (d) 54
23. The number of possible outcomes when a coin is tossed 6 times, is
(a) 36 (b) 64 (c) 12 (d) 32
The number of different four digit numbers that can be formed with the digits 2,3,4,7 and
using each digit exactly once, is
(a) 120 (b) 96 (c) 24 (d) 100
The sum of the digits in unit place of all the numbers formed with the help of 3,4,5 and 6
taken all at a time, is
(a) 432 (b) 108 (c) 36 (d) 18
PERMUTATIONS 10.49

ANSWERS

1. (d) 2. (d) 3. (d) 4. (b) 5. (b) 6. (b) 7. (a) 8. (b) 9. (a)


10. (a) 11. (a) 12. (a) 13. (c) 14. (a) 15. (a) 16. (a) 17. (b) 18. (a)
19. (c) 20. (a) 21. (a) 22. (c) 23. (a) 24. (b) 25. (b) 26. (c) 27. (b)
FILL IN THE BLANKS TYPE QUESTIONS (FBQs)

1. The value of P, +r r-l


IS

2. If''P4:"/^=l:2,thenn = .
3. =1320, then r =

ow
4. The number of permutations of u distinct object, taken r at a time, when repetitions are not
allowed, is
5. The number of permutations of ii distinct objects, taken r at a time, when repetitions are
allowed, is

e
Fl
6. The number of ways 'm‘ men and 'n‘ women {m > n) can be seated in arow so that no two

re
women sit together is

F
7. In an examination there are three multiple choice questions and each question has four
ur
choice. The number of ways in which a student can fail to get all answers correct, is

or
8. The number of ways in which three letters can be posted in five letter boxes, is
sf
9. The number of six digit numbers, all digits of which are odd, is
k
10. The number of different words that can be made from the letters of the word
Yo
oo

INTERMEDIATE, such that two vowels never come together, is


eB

ANSWERS

1. "Pr 2. 6 3. 3 4. 5. ^ m!(m + l)! 7. 63


ur

(m-tt + 1)!
ad

'^6
Yo

6!
8. 5^ 9. 5^ 10. —X = 151200
2! 3!2!
d

VERY SHORT ANSWER QUESTIONS (VSAQs)


Re
in

Anszver each of the folloiohig questions in one ivord or one sentence or as per exact requirement of the
F

question:
1. In how many ways can 4 letters be posted in 5 letter boxes?
2. Write the number of 5 digit numbers that can be formed using digits 0,1 and 2.
3. In how many ways 4 women draw water from 4 taps, if no tap remains unused?
4. Write the total number of possible outcomes in a throw of 3 dice in which at least one of the
dice shows an even number.

5. Write the number of arrangements of the letters of the word BANANA in which tw'o N's
come together.
6. Write the number of ways in which 7 men and 7 women can sit on a round table such that
no two women sit together.
7. Write the number of words that can be formed out of the letters of the word 'COMMITTEE'.
10.50 APPLIED MATHEMATICS-Xl

8. Write the number of all possible words that can be formed using the letters of the word
'MATHEMATICS'.

9. Write the number of ways in which 6 men and 5 women can dine at a round table if no two
women sit together.
10. Write the number of ways in which 5 boys and 3 girls can be seated in a row so that each girl
is between 2 boys.
11. Write the remainder obtained when l!+2!+3! + ... + 200! is divided by 14.
12. Write the number of numbers that can be formed using all for digits 1,2,3,4.

ow
ANSWBRS

9! 11!
1. 5^ 2. 2x 3. 4! 4. 189 5. 20 6. 7!x 6! 7. 8.
(2!) 3 2! 212!

e
9. 6!x5! 10. 2880 11. 5 12. 24

re
Frl
F
ou
or
kfs
oo
Y
B
re
oYu
ad
d
in
Re
F
CHAPTER

COMBINATIONS

11.1 INTRODUCTION

In the previous chapter, we have studied arrangements of a certain number of objects by taking
some of them or all at a time. Most of the times we are not interested in arranging the objects, but
we are more concerned in selecting a number of objects from given number of objects. In other
words, we do not want to specify the ordering of selected objects. For example, a company may
want to select 3 persons out of 10 applicants, a student may want to choose three books from his

w
library at a time etc.
Suppose we want to select three persons out of 4 persons A, B, C and D. We may choose A, B, C

F lo
or A. B, D or A, C, D or 6, C, D. Note that we have not
and A, C, B separately here, because they represent
listed A. B, C; B, C A; C, A, B; B, C; C, B, A
the same selection A, B, C. But, they give rise
to different arrangements. It is evident from the above discussion that in a selection the order in

e
which objects are arranged is immaterial.
11.2 COMBINATIONS
Fre
for
COMBINATIONS Each of the dijferent selections made bij taking some or all of a number of objects,
irrespectwe of their arrangements is called a combination.
r
List the different combinations formed of three letters A, B, C taken two at a time.
You

ILLUSTRATION 1
oks

SOLUTION The different combinations formed of three letters A, B, C are: AB, AC, BC.
eBo

ILLUSTRATION 2 Write all combinations offour tetters A, B, C, D taken two at a time.


SOLUTION Various combinations of two letters out of four letters A, B, C, D are:
ad

AB, AC, AD, BC, BD, CD.


our

DIFFERENCE BETWEEN A PERMUTATION AND COMBINATION


(i) In a combination only selection is made whereas in a permutation not only a selection i
is
Re

made but also an arrangement in a definite order is considered,


dY

(ii) In a combination, the ordering of the selected objects is immaterial whereas in a


Fin

permutation, the ordering is essential. For example. A, B and B, A are same as combinations
but different as permutations.
(iii) Practically to find the permutations of n different items, taken r at a time,_ we first select r
items from n items and then arrange them. So, usually the number of permutations exceeds the
number of combinations,

(iv) Each combination corresponds to many permutations. For example, the six permutations
ABC, ACB, BCA, BAC, CBA and CAB correspond to the same combination ABC.
REMARK Generali}/ zve use the zvord 'arrangements' for permutations and the ivord selections foi
combinations.

NOTATION The number of all combinations ofn objects, taken r at a time is generallij denoted bg C(n, r)
or, or, "j.
Thus, "Cr or C (n, r) = Number of ways of selecting r objects from n objects.
11.2
APPLIED MATHEMATICS-XI

Clearly, is defined only when n and r are non-negative integers such that 0 < r < n.
THEOREM The number of all combinations ofn distinct objects, taken rata time is given bij
nl
Cr =
(n-r)lrl
PROpl Let the number of combinations of n distinct objects taken rata time be a:. Consider one
of these .v ways. There are r objects in this selection which can be arranged in r! ways. Thus, each
of the A- combinations gives irse to r! permutations. So, .r combinations will give rise to .v x (r!)

w
permutations. Consequently, the number of permutations oin things, taken rat a time is a: x (r!).
But, this number is also equal to ",P..
II
Ar(r!) = Pr
n
Pr n\ n\

e
X =
r! (» - r)!r! in-r)[

ro
re
n nl
Cr = Q.E.D.
(n -r)!r!

F
REMARK I Wc have,

Fl
n nl
Cr =
{n-r)\r\

Cr ^= n (»-l) (n-2)... (h - r + 1) (fi - r) (n - r -1}...


u 3.2.1

sr
n

{(«-r) (?i-r-l)... 3.2.1( (1.2.3....r|

ko
o
n _
Cr =
»(»-l) {n-2)...{n-r + l) of
1.2.3....r

Sometimes this form of'^Cr is also wry convenient to use.


o
Y
?i!
REMARK 2 We have, "C^ =
erB

Oj-r)!r!
Putting r = n, zue obtain
uY

II nl nl
CII = 1
(n ~n) I nl ?j!0! [V 0! = lj
Putting r=0, zue obtain
ad
do

I! nl nl
C0 = 1
(«-0)!0! nl
in

n n
Thus, C II Cq = 1.
Re

REMARK .1 We have,
F

II nl 1
II
R.I
Cr =
(n-r)lrl r!^(«-r)! r!

11.3 PROPERTIES OF "C, OR,C(;i,r)


In this section, we shall discuss some important properties of "C^ .
PROPERTY 1 ForO<r<n, zue have "Cr =
11
C
'~ii-r-

PROOF We have,
n nl nl
C„-r = II
C..I
{n -r)!(« -(« -r))! (n-r)lrl
REMARK I The use of this properti/ simpliifes the calculation of "Cr luhen r is large.
For example, if we zuant to calculate bi/ using this property, we get
20 20 20
c 19 C20 - 19 =
- Cl = 20.
COMBINATIONS 11.3

KFMARK 2 The above property can be restated as follows:


If X and y are non-negative integers such that x + \/ = n, then = ”C,y
This can also be stated as: "Cx = "Cy => x = y, orx + y = n

ILLUSTRATION ! If "Cj = "C4 , find the value ofn.


SOLUTION We know that: ''C.^ = "Qy o x + y-n or x=y.
Cj="C^ => ?i=7 + 4 = ll
n

ow
JI +
PROPERTY 2 Let n and r be non-negative integers such that r < n. Then, "Q + "Q _ i = *“r-

I’ROor We have.
II 7?! ;i!

^ (7i-r)!r! {77-(r-1))! (r -1)!

e
77! 77!

re
+
(77-r)!r! (77-r + l)!(r-1)!

F
77! 71!

(77-;●)!?● (r-l)!

Frl (77-r + l) (77-?-)!{7- -1)!


77!
ou1 1

sr
< —I-

(77-r)!(r-1)! r 77-r + l
71!

(77-r)!(7--l)![7- (77-r + l) I
77 - r + 1 + r
kfo
oo
77!(?7 + 1)
Y

(n-r)!(r-l)!r (77-r + l)
reB

_ (77 +1) 77! (77 + 1)1 (77 + 1)! »+ 1


Cr-
(77 -r + 1) (77 -r)!r (r -1)! (77 -r + l)!r! ((77 + 1) -r)!r!
uY

RLMARK 3 This property is known as Pascal's rule and it can also be proved by giving combinatorial
arguments.
ad

5 52-i
do

III LiS'iR.\TlON 2 Find the value of the expression + 2 ^C3-


; = i
in

SOLUTION We have.
Re

5
47 52- /
C4 + ,2
F

/ = !
47
C3 + "^C3 + 48c3 + C3
47 50
C4 + C3 +
C3 + '"^C4 + %3 + C3 + 50C3 + 5^C3
47 49

= (4^C3 + ‘^^C4) + 48C3 + C3+5‘’C3+5^C3


49

48 r' 17+ 1
_i_ 48^ , 49^ , 50^ , 51(^ [■■■ "Q^i + "C,= Q]
C4+ C3+ C3+ C3+ L3
11+ 1
,48^ 48^ \ , 49^ , , 51/— [V +X = Cr]
= ( C3 + 04)+ C3 + C3 + C3

C4 + "^C3+^°C3 + ^'C3
49

17+ 1
= {“^C3 + 4^C4) + 50c^+51c^ [V ”C,_| + ”C,= Q]

C4+5OC3 + 5IC4
50

77+ 1
= (50C3+50C4)+5'‘C3 [V "C,_i + "Q = Q]
71+1
- ^'C4+5’C3
52
C4 [●-● ”C, -1 + »C,= c,.)
11.4 APPLIED MATHEMATICS-Xl

» -1
PROPERTY 3 Let n and r be non-negative integers such that 1 < r < n. Then, "Q = - ● Q-l-
r

PROOF We have.
II ?i!
Cr =
(n -r)!r!
» n («-!)! n («-!)! n »-i
Q = Q-i
K«-l)-(r-l)}!r(r-l)! r {(„-l)-(r-l))!(r-l)! r

REMARK 4 This properti/ is very useful to find the value of”C,..

w
10
For example, ^^03 = 3 X VL-j^32 10 9 10 9 87^
= — X — X C 1 - — X — X — C0
3 2 1
10 x-x-xl
= —
3
9 8 ^ =120
2 1
ion

REMARK 5 By using the above property, zue obtain that


II n n~l n-2 n{r-l) n-(r-l)
Q =

o
e
r r-1 r-2 2 1

re
9 8 7 6
For example, = — X — X X - = 126.

Frl
4 3 2 1

F
II-1
PROPERTY 4 U l<r<n, then n Cr - {n - r + 1) "Cr _ 1.
ou
PROOF We have,

sor
II- 1 (»-!)!
n.
Q-i n ■

{(«-l)-(r-l)}!(r-l)!
kf
II-1 n!
Q-i
oo
n.

(»-r)!(r~l)!
Y

II-1 (»-r + l)-u!


Q-i =
B

n.

(n-r + 1) Oi-r)!(r-l)!
nl
re

II-1
n.
Q_i = (n-r + l)-
oY

(n-r + l)!(r-l)!
u
ad

II-1 n\
n. = (n-r + \)
{n-(r-l)}!(r-l)!
d

II- ]
n.
C,_-i = („-r + l)”C,
in

-1
Re

II
PROPERTY 5
Qr “ "Cy ^ X ~y or, x + y - n.
F

PROOF We have,
II
C
'“I - ”r y
II
C = ”C II
c r- ”c ="c 1
y II-y [. ‘“II-yJ
X = y or .r n-y
X = y or x + y = n.

REMARK 6 // "C.^ = "Cy and x y, then a: + y = n.


ILLUSTRATION 3 If "Cjs = "Cg , find the value of''€21-
SOLUTION We have.
II
C 15 = "Cg => n = (15 + 8) = 23 rC,="Cy ^ x + y=n]
II 23 23
C21 - C 21 C 23-21
[■: % = "C II- rl
11.5
COMBINATIONS

n M-1
23
C2 =
23
— X 22 ^ 2V V »Q=-x — -2
r r
2 1

23 22
— X — xl= 23x 11 = 253 [V "Co =1]
2 1
10
ILLUSTRATION 4 //^°Cy = CY + 4 ' . find the value ofx.
SOLUTION We have, = 10c x+ 4
=> X + .Y + 4 = 10 => 2y = 6 => y = 3.

PROPERTY 6 Ifn is an even natural niunbcr, then the greatest of the values
C,,"C2 "C„is"C„/2
n n
C0 '

!fn is an odd natural number, then the greatest of the values


Co ' "C-i , ”C2 ,'C„ is C„_i/2 “ C„+;i/2
n

ILLUSTRATIVE EXAMPLES

w
F lo
52
Cg (ii) ’^°C98 (Hi)
10
EXAMPLE 1 Evaluate the folloiuing: (i) C52
SOLUTION (i) lOc 8 10 C 10-8 [V "Q ="C„_,]
^ 10 9 8r

e
10 ● ● "c = Q-1
C2 - — X Y X Cq
Fre
'-r
r

10 9 , [V "Co =1)
X — X 1
for
2 1
= 45 Iv "C. ="C„_,1
r
You

11
100 "c, = C
oks

100 ^n-r
(ii) C98 - c 100 - 98
eBo

100 99 ^ 98rL 0
n -
100
X — X
C2 = 2 1
r

100 X
99 X
,1 [●●● "Co =11
ad
our

2 1
= 4950
52
C52 = 1 [V "c„=U
(iii)
Re
dY

EXAMPLE 2 If "Cg =''Cg , find "C2


SOLUTION If "Cy = "C,/ and y ^ y, then x + \j =n.
Fin

II
n = (8 + 6) = 14
n
Cs =
n H-i
14 ,, 14 13 Ur [●●● "C, = r Cr-ll
C2 - Y X X Cq
n
Now, Cl =
14 13 , [■■■ "Co=l]
2 1

EXAMPLE 3 If ' P, = 720 and "Q = 120, find r.


SOLUTION We know that
II
H Pr
Q =
r!
720
120 = "Q =120 and =720
r!
r! = 6 ^ r! = 3!=^> r = 3.
11.6
APPLIED MATHEMATICS-XI

EXAMPLE 4 If the ratio : "C3 is equal toll: l,find n.

SOLUTION We have,
2"C3:”C3 =11:1
2ii
C3 _ 11
n
1
C3
(2«)!
{2n-3)\3[ 11
nl 1
(«-3)!(3!)
{2n)\ X,,(»-3)! 11

ow
(2n-3)! ul 1

(2»)(2n-l)(2»- 2) {2}i - 3)1 X


(«-3)! 11

(2m-3)! m(m-1) (m-2) (m-3)! 1


{2n) (2m-1) (2m-2) II

M(M-l) (m-2) 1

e
4(2m-1) 11

re
=>
m-2
= _=>8m-4 = 11m-22=> 3m = 18=>m = 6
1

rFl
F
EXAMPLE 5 Prove that: 11 “
2” {1-3-5-...(2m-1)}

r
m!
ou
SOLUTION We have. fo
ks
2ji
Cn
2/i! (2m)!
(2/i-m)!m! mIm!
oo

^ (2m)(2m-1) (2m-2)...5-4-3-2-1
Y

m!m!
eB

^ {1-3-5-...(2m-1)1{2-4-6-...2m1
^m!
r
ou

^ {1-3-5-...(2m-1)}x2» {1-2-3-...m1
ad
Y

m!m!

{1-3-5-...(2m-1))x2"xm! ^ {1-3-5...(2m-1)1
d

mIm! Ml
Re
in

n + 2
EXAMPLE 6 If ^8 ● ” ^^4 = 57:16, find n.
F

SOLUTION We have,
n+2,- .11-2
Cs- P4 = 57 :16
;i+ 2
C8 _ 57
«-2
16
P4
(m + 2)I -
(m-6)I _ ^ X

8!(m-6)I (m-2)! ~ 16
(m+2) (m + 1)m (m-1) (m-2)I I 57
8! (M-2) I 16
57 19 X 3
(m + 2) (m + 1) M (m -1) = — X 8! = x8x7x6x5x4x 3x2x1
16 16

(m + 2)(m + 1)(m-1)m = 143640


(m-1)m(m + 1)(m + 2) = 19x 3x7x6x5x4x 3
COMBINATIONS 11.7

{n-l)n{n + l){}i + 2) = 19 x (3 x 7) x (6 x 3) x (4 x 5)
(n -1) n {n + 1) (ji + 2) - 18 x 19 x 20 x 21 => )i -1 = 18 => n = 19
EXAMPLE? // "Q _ 1 = 36, "Q =84 and "Q j =126, tlwnfind ’'€2-
SOLUTION We know that
II
Cr r + 1
II
cr + 1
n-r

It is given that = 84 and ”C,.+1 = 126


It
C, 84
II
126
Q+1
I- + 1 2
= — ^2n -5r = 3 ...(i)
n-r 3
II
r +l

w
Replacing r by (r - 1) in II
, we get
C r+ 1
n-r

F lo
II
Cr-1 r

It
Cr n-(r-l)

e
36 r
"C r-l = 36 and "Q = 84

Fre
84 ?i-r + l
r 3
= — ^ 3n -lOr = - 3
for ...(ii)
n-r+1 7
r
Solving (i) and (ii), we get r = 3.
You

3!
oks

^C2 = ^C2 = = 3.
(3-2)!2!
eBo

It
Cr n-r + 1
NOTE Students are advised to learn that It
as it is a very useful result.
Cr-l r
ad
our

EXAMPLE 8 If "Pr - +t Values of n and r.


SOLUTION We have,
II
p ="pV + 1.
'r
dY
Re

ii! n!
Fin

(n-r)! (II-r-1)1
1 1

(ii-r)(ii-i'-l)! (ii-r-l)l
n-r = 1
II It
and. Q = Cr-l
III ii!
=^>
(n-r)lrl (ii-r + l)!(r-l)!
nl nl

(n-r)lr(r-l)l (n-r +1) (n-r)l(r-1)1


1 1
=> => n-r + 1 = r => n -2r = -1 ...(ii)
r n-r + 1

Solving (i) and (ii), we obtain iz = 3 and r = 2.


11.8 APPLIED MATHEMATICS-XI

EXERCISE 11.1

1- Evaluate the following:


(i) '“C3 (ii) (iii) 35c 35 (iv)
II +
ti (v) I r = l

2. If “Cj2 = ”C5, find the value of n. 3. If"C4='t6,fmd^2c„. n

5. If24c^ = C2x+ 3'f^clA:.


24
4. If "Cio = "Ci2, find
6. If ●r+ 2 , find X. 7. If^^C 3r -
15
Cr + 3' find r.

8. If Sq -2^3 =2c2,findr. 9. If -1 = 11:5, find r.


n+2
10. If C8:""^P4=57:16,find h. 11. If 28c Ir . 24
■ C 2r-4 = 225:11, find r.

12. If"C 4 13. If2"C3:"C2=44:3, find«.


n
' C5 and "Cg are in A.P., then find n.

w
14. If^^Q =^^C,^2^find"C4. 15. Ifa = "t 2’ then find the value of *^C2-

16. For all positive integers n, show that 2"C„ +



F lo In
cn

:^'C„ =[1-3-5...(4?i-1)]:[1-3-5...(2«-1)]2.
-
1 Til+2
Qi + l)-

ree
17. Prove that: C 2n ●

F
5 -1C
18- Evaluate 2®Cc + ^S 25-r
C4.
for
r = 2

19. Let r and n be positive integers such that 1 < r < n. Then prove the following:
r
ti
Cr _« -r + 1
You

(i) (ii) n -{n-r + 1) "Q _ j


oks

II
r
eBo

II
Q II
71+2
(iii) II-1 (iv) ”Q+2"C,-1 + "Q_2 = Q-
Q-i '●
our
ad

ANSWERS
1. (i) 364 (ii) 66 (iii) 1 (iv) {n + 1) (v) 31 2. 17 3. 66
4. 23 5. 7 6. 8 7. 3 8. 3,5 9. 5 10. 19 11. 7
(m + l)(7H) (m-1) (w-2)
dY
Re

12. 14,7 13. 6 14. 35 15. 18. 42504


8
Fin

11.4 PRACTICAL PROBLEMS ON COMBINATIONS

In this section, we intend to discuss some problems in real life where the formula for "Q and its
meaning can be applied.
ILLUSTRATIVE EXAMPLES

EXAMPLE 1
From a class of 32 students, 4 are to be chosen for a competition. In how many ways can this
be done ?
32
SOLUTION Out of 32 students, 4 students can be selected in C4 ways.
32!
Required number of ways ^^C4 = 2814!
EXA.MPLE2
Three gentlemen and three ladies are candidates for two vacancies. A voter has to vote for
tzvo candidates. In how many loays can one cast his vote ?
11.9
COMBINATIONS

SOLUTION Clearly, there are 6 candidates and a voter has to vote for any two of them. So, the
required number of ways is the number of ways of selecting 2 out of 6 i.e.
6!
Hence, the required number of ways = ^€2 = 2!4! = 15.

EXAMPLE 3 If fhere are 11 persons in a parti/, and if each two of them shake hands with each other, hoiu
many handshakes happen in the party?
shake hands, it is counted as one handshake,

ow
SOLUTION It is to note here that, when two persons
not two. So, this is a problem on combinations.
The total number of handshakes is same as the number of ways of selecting 2 persons among 12
12!
persons i.e. ^^€2 = lOIx = 66.

e
2!

re
EXAMPLE 4 /I question paper has tzoo parts, Part A and Part B, each containing 10 questions. If a
student has to choose 8from Part A and 5from Part B, in how many zuays can he choose the questions ?

Flr
SOLUTION There are 10 questions in Part A out of which 8 questions can be chosen in Cg

F
10
ways. Similarly, 5 questions can be chosen from part B containing 10 questions in C5 ways.
ou
Hence, the total number of ways of selecting 8 questions from part A and 5 from part B

sr
10 10! 10!
Cgx“C5 = 8!2!
fo
= 11340.
51x5!

EXAMPLE 5 In hozo many zuays a committee of 5 members can be selected from 6 men and 5 women,
k
oo
consisting of 3 men and 2 zuomen ?
Y

SOLUTION Three men out of 6 men can be selected in ways. Two women out of 5 women
reB

can be selected in ^C2 ways. Therefore, by the fundamental principle of counting, 3 men out of 6
uY

men and 2 women out of 5 women can be selected in


6x5x4 5x4
'C3X=C2 = X
= 200 ways.
3x2x1 2x1^
ad
do

EXAMPLE 6 hi hozu many ways can a cricket eleven be chosen out of a batch of 15 players if
in

(i) there is no restriction on the selection? (ii) a particular player is alzvays chosen?
Re

(Hi) a particular player is never chosen ?


F

SOLUTION (i) The total number of ways of selecting 11 players out of 15 is


15 15x 14x 13x 12
15c 11 - 15c 15 - 11-
= 1365
4x 3x 2x 1

(ii) If a particular player is always chosen. This means that 10 players are selected out of the
remaining 14 players.
14 14 14
Required number of ways = c 10 -
c 14- 10 - €4=1001

(iii) If a particular player is never chosen. This means that 11 players are selected out of the
remaining 14 players.
14
Required number of ways = = 14c 14-11 - C3 = 364

EXAMPLE 7 A committee of 12 is to be formed from 9 zuomen and 8 men. In hozu many zuays this can be
done if at least five zuomen have to be included in a committee ? In hozu many of these committees (i) the
zuomen are in majority (ii) the men are in majority ?
11.10 APPLIED MATHEMATICS-XI

SOLUTION There are 9 women and 8 men. A committee of 12, consisting of at least 5 women, can
be formed by choosing:
(i) 5 women and 7 men (ii) 6 women and 6 men (iii) 7 women and 5 men

(iv) 8 women and 4 men (v) 9 women and 3 men


Total number of ways of forming the committee
= '’Cs X 8c, + X 8Cj + % X % + X 8C4 + X 8C3
= 126 X 8 + 84 X 28 + 36 X 56 + 9 X 70 + 1 X 56 = 6062
Clearly, women are in majority in (iii), (iv) and (v) cases as discussed above.
So, total number of committees in which women are in majority

= % X ®Cg + X ^C4 -h X = 36 X 56 + 9 X 70 + 1 X 56 - 2702


Clearly, men are in majority in only (i) case as discussed above.

w
So, total number of committees in which men are in majority = ^Cg C7 =126x 8=1008.

F lo
EXAMPLE 8 A committee of three persons is to be constituted from a group of 2 men and 3 zoomen. In
hozv many ways can this be done? Hozv many of these committees zoould consist of 1 man andlioomen?
SOLUTION There are 5 persons (2 men and 3 women). In order consitute a committee of 3

ee
Fr
persons we need to select three persons out of given 5 persons. This can be done in ^Cg ways.
5!
So, the committee can be formed in ^Cg = 3!2! = 10 ways.
for
r
Now, 1 man can be selected from 2 men in j ways and 2 women can be selected from 3 women
You
s

in C2 ways.
ook

Therefore, required number of committees is ^C^ x ^C2 = 2 x


eB

3=6
EXAMPLE 9
What is the number of zuays of choosing 4 cards from a pack of 52 playing cards? In how
many of these
our
ad

(i) four cards are of the same suit? (ii) four cards belong to four different suits?
(iii) four cards are face cards? (iv) tzoo are red cards and tzvo are black cards?
dY

(v) cards are of the same colour?


Re

SOLUTION Four cards can be chosen from 52 playing cards in ^^C4 ways.
Fin

52 52! 49 X 50 X 51 X 52
Now, C4 = = 270725
4814! 2x3x4

Hence, required number of ways = 270725


(f)
13
There are four suits (diamond, spade, club
13
and heart) of 13 cards each. Therefore, there are
C4 ways of choosing 4 diamond cards. C4 ways of choosing 4 club cards, ^^C4 ways of
choosing 4 spade cards and ways of choosing heart cards.
13 13!
Required number of ways = C4 + ^^C4 + ^^C4 + ^^C4 = 4 X ^^C4 = 4 X 914!
= 2860

(ii) There are 13 cards in each suit. Four cards drawn belong to four different suits means one

card is drawn from each suit. Out of 13 diamondcards one card can be drawn in j ways.
13 13
Similarly, there are C| ways of choosing one club card, Cj ways of choosing one spade card
13
and C4 ways of choosing one heart card.
13 13
.-. Number of ways of selecting one card from each suit = Ci X X 1
X Cl =13^
COMBINATIONS 11.11

19
(iii) There are 12 face cards out of which 4 cards can be chosen in C4 ways.
19 12!
Required number of ways = C4 = 418! = 495

26
(iv) There are 26 red cards and 26 black cards. Therefore, 2 red cards can be chosen in C2 ways
26
and 2 black cards can be chosen in C2 ways. Hence, 2 red and 2 black cards can be chosen in
26!
>2
26
C2 X 26c, = 24! 2!
= (325)2 105625 ways.

(v) Out of 26 red cards, 4 red cards can be chosen in 26C4 ways. Similarly, 4 black cards can be
chosen in 26C4 ways.
26!
Hence, 4 red or 4 black cards can be chosen in + 26C4 = 2x =2 4122!
= 29900 ways.

w
EXAMPLE 10 Out of 5 men and 2 women, a committee of 3 is to be formed. In how many xonys can it be
formed if at least one woman is to be included ?
SOLUTION The committee can be formed in the following ways:

Flo
(i) By selecting 2 men and 1 woman (ii) By selecting 1 man and 2 women

e
5 ^
Now, 2 men out of 5 men and 1 woman out of 2 woman can be chosen in C2 x ways.

re
5 2
And, 1 man out of 5 men and 2 women out of 2 women can be chosen in x C2 ways.

F
Total number of ways of forming the committee = ^C2 x x 2C2 = 20 + 5 = 25.
ur
r
EXAMPLE 11
fo
In hoxo many ways can a cricket team be selected from a group of 25 players containing 10
bafsynen, 8 bowlers, 5 all-rounders and 2 wicket keepers ? Assume that the team of 11 players requires 5
ks
batsmen, 3 all-rounder, 2 bozolers and 1 xvicket keeper.
Yo
oo

SOLUTION The selection of team is divided into four phases:


(i) Selection of 5 batsmen out of 10. This can be done in ^^€5 ways,
B

(ii) Selection of 3 all-rounders out of 5. This can be done in ^€3 ways.


re

(iii) Selection of 2 bowlers out of 8. This can be done in °C2 ways,


u
ad

(iv) Selection of one wicket keeper out of 2. This can be done in ways.
Yo

The selection of team is completed by completing all the four phases.


C5 X 5c3 X «C2 2Ci =141120 ways.
10
The team can be selected in
d
Re
in

EXAMPLE 12 A committee of 5 is to be formed out of 6 gents and 4 ladies. In how many zuays this can be
done, lohen
F

(i) at least tzuo ladies are included? (ii) at most tzuo ladies are included ?
SOLUTION (i) A committee of 5 persons, consisting of at least two ladies, can be formed in the
following ways:
I Selecting 2 ladies out of 4 and 3 gents out of 6. This can be done in x ^03 ways.
4 6
II Selecting 3 ladies out of 4 and 2 gents out of 6. This can be done in C3 x C2 ways.
Ill Selecting 4 ladies out of 4 and 1 gent out of 6. This can be done in ‘^C4 x ways.
Since the committee is formed in each case. Therefore, by the fundamental principle of addition.
The total number of ways of forming the committee = ‘^€2 x + ^03 x ^€2 + x
= 120 + 60 + 6=186

(ii) A committee of 5 persons, consisting of at most two ladies, can be constituted in the
following ways:
11.12 APPLIED MATHEMATICS-XI

I Selecting 5 gents only out of 6. This can be done in ways.


II Selecting 4 gents only out of 6 and one lady out of 4. This can be done in x ways.
4/-
Ill Selecting 3 gents only out of 6 and two ladies out of 4. This can be done is C2 ways.
Since the committee is formed in each case. So, the total number of ways of forming the
committee = % + ^€4 x x ^^02 = 6 + 60 + 120 = 186.
EXAMPLE 13 A box contains 5 different red and 6 different white balls. In how many ways can 6 balls be
selected so that there are at least two balls of each colour?
SOLUTION The selection of 6 balls, consisting of at least two balls of each colour from 5 red and
6 white balls, can be made in the following ways :
(i) By selecting 2 red balls out of 5 and 4 white balls out of 6. This can be done in

ow
^C2 X ways,
(ii) By selecting 3 red balls out of 5 and 3 white balls out of 6. This can be done in
^C3 X ways,
(iii) By selecting 4 red balls out of 5 and 2 white balls out of 6. This can be done in

e
^C4 X ^C2 ways.

re
Fl
Since the selection of 6 balls can be completed in any one of the above ways.

F
Hence, by the fundamental principle of addition, the total number of ways to select the balls
ur
= ^C2 X ^C4 +^C3 X ^C3 +^C4 X ^C2 = 10 x 15 + 10 x 20 + 5 x 15 = 425.

r
fo
EXAMPLE 14 For the post of 5 teachers, there are 23 applicants, 2 posts are reserved for SC candidates
ks
and there are 7 SC candidates among the applicants. In hozv many ways can the selection be made ?
Yo
SOLUTION Clearly, there are 7 SC candidates and 16 other candidates. We have to select 2 out
oo

This can be done in ^C2


16
of 7 SC candidates and 3 out of 16 other candidates. C3 ways.
eB

The number of ways of making the selection = '^€2 x ^^C3 = 11760.


EXAMPLE 15 How many triangles can be formed by joining the vertices of a hexagon ?
ur

SOLUTION There are 6 vertices of a hexagon. One triangle is formed by selecting a group of 3
ad

vertices from given 6 vertices. Tliis can be done in ^C3 ways.


Yo

6!
Number of triangles = = 20.
d

3! 3!
Re
in

EXAMPLE 16 How many diagonals are there in a polygon zvith n sides ?


SOLUTION A polygon of n sides has n vertices. By joining any two vertices of a polygon, we
F

obtain either a side or a diagonal of the polygon. Number of line segments obtained by joining
the vertices of an n sided polygon taken two at a time
n(n-l)
= Number of ways of selecting 2 out of n = “C2 = 2

Out of these lines, n lines are the sides of the polygon.


Number of diagonals of the polygon = ^ -n
n {n ~ 3)
2

EXAMPLE 17 A polygon has 44 diagonals. Find the number of its sides.


SOLUTION Let there be n sides of the polygon. We know that the number of diagonals of
sided polygon is
11.13
COMBINATIONS

11 {n - 3) (●.● n > 0)
= 44=> - 3n -88 = 0=> (m -11) (« 4- 8) =0 => n =11
2

Hence, there are 11 sides of the polygon.


EXAMPLE 18 How many chords can be drawn through 21 points on a circle?
SOLUTION A chord is obtained by joining any two points on a circle. Therefore, total number of
chords drawn through 21 points is same as the number of ways of selecting 2 points out of 21
points. This can be done in C2 ways.
_ _

21 21!
Hence, total number of chords = C2 = = 21x10=210.
1912!

EX.AMPLE 19 A person xvishes to make up as many different parties as he can out of his 20 friends such

w
that each party consists of the same number of persons. How many friends should he invite ? 20
SOLUTION Suppose he invites r friends at a time. Then the total number of parties is Q.

o
We have to find the maximum value of which is for r = 10, because 'Q is maximum for

e
r =n/l, when n is even.

re
rFl
Hence, he should invite 10 friends at a time in order to form the maximum number of parties.

F
EXAMPLE 20 Ifm parallel lines in plane are intersected by a family ofn parallel lines. Find the
number of parallelograms formed.
SOLUTION A parallelogram is formed by choosing two straight lines from the set of m parallel

r
ou
lines and two straight lines from the set of n parallel lines. sfo
Two straight lines from the set of m parallel lines can be chosen in ^^€2 ways and two straight
k
lines from the set of n parallel lines can be chosen in ”C2 ways.
oo

Hence, the number of parallelograms formed = C2x"C2


m
Y

m {m -1) n {n -1) mn {m -1) (n -1)


eB

2 2 4

EXAMPLE 21 There are 10 points in a plane, no three of which are in the same straight line, excepting 4
points, which are collinear. Find the (i) number of straight lines obtained from the pairs of these points; (ii)
r
You

number of triangles that can be formed with the vertices as these points.
ad

10
SOLUTION (i) Number of straight lines formed joining the 10 points, taking 2 at a time = C2
10!
= 45.
d

218!
4!
Re
in

= 6
Number of straight lines formed by joining the four points, taking 2 at a time - C2 2! 2!
F

But, 4 collinear points, when joined pairwise give only one line.
Required number of straight lines = 45 - 6 + 1 = 40. 10 10!
C3 = = 120.
(ii) Number of triangles formed by joining the points, taking 3 at a time = 317!

Number of triangles formed by joining the 4 points, taken 3 at a time = "^C2=^Ci~ 4.


But, 4 collinear points cannot form a triangle when taken 3 at a time.
So, Required number of triangles = 120 - 4 =116.
EXAMPLE 22 In a plane there are 37 straight lines, of which 13 pass through the point A and 11 pass
through the point B. Besides, no three lines pass through one point, no line passes through both points A
and B, and no two are parallel. Find the number of points of intersection of the straight lines.
37
SOLUTION The number of points of intersection of 37 straight lines is C2. But 13 straight lines
out of the given 37 straight lines pass through the same point A. Therefore instead of getting
C2 points, we get merely one point A. Similarly, 11 straight lines out of the given 37 straight
13
11.14 APPLIED MATHEMATICS-XI

II
lines intersect at point B. Therefore instead of getting C2 points, we get only one point B.
37 13
Hence, the number of intersection points of the lines is C2- C2-^^C2 + 2=535.
EXAMPLE 23 From a class of 25 students. Ware to be chosen for an excursion party. There are 3 students
zvho decide that either all of them will join or none of them zuill join. In how many ways can they be
chosen ?

SOLUTION We have the following possibilities:


(i) Three particular students join the excursion party: In this case, we have to choose 7 students
22
from the remaining 22 students. This can be done in Cy ways,
(ii) Three particular students do not join the excursion party: In this case, we have to choose 10
22
students from the remaining 22 students. This can be done in CjQ ways.
Hence, the required number of ways = ^'^Cy +
22
Cio =817190.

w
EXAMPLE 24
4 boy has 3 library tickets and 8 books of his interest in the library. Of these 8, he does not
loant to borrozv Chemistry Part II, unless Chemistry Part I is also borrozved. In how many zuays can he

F lo
choose the three books to be borrozved ?
SOLUTION We have the following two possibilities :

ee
(i) When Chemistry part I is borrozved: In this case the boy may borrow Chemistry Part II. So, he

Fr
has to select now two books out of the remaining 7 books of his interest. This can be done in ^C2
ways. for
(ii) When Chemistry part I is not borrowed : In this case the boy does not want to borrow
ur
Chemistry Part II. So, he has to select three books from the remaining 6 books. This can be done
in ^€3 ways.
s
ook
Yo

Hence, the required number of ways = ^C2 + = 21 + 20 = 41.


eB

EXAMPLE 25 In hozv many zvays can 7 plus (+) signs and 5 minus (-) signs be arranged in a row so that
no two minus signs are together ?
SOLUTION The plus signs can be arranged in only one way, because all are identical, as shown
r
ad
ou

below:
Y

A blank box in the above arrangement shows available space for the minus signs. Since there are
Re
nd

7 plus signs, the number of blank boxes is therefore 8. The five minus signs are now to be
arranged in the 8 boxes so that no two of them are together. Now, 5 boxes out of 8 can be chosen
Fi

in C5 ways. Since all minus signs are identical, so 5 minus signs can be arranged in 5 chosen
boxes in only one way. Hence, the munber of possible arrangements = 1 x x 1 = 56.
EXAMPLE 26 In how many zvays can 21 identical books on English and 19 identical books on Hindi be
placed in a rozv on a shelf so that tzvo books on Hindi may not be together?
SOLUTION In order that no two books on Hindi are together, we must first arrange all books in
English in a row. Since all English books are identical, so they can be arranged in a row in only
one way as shown below:
x£x£x£x£x...x£x£

Here E denotes the position of an English book and x that of a Hindi book.
Since there are 21 books on English, the number places mark x are therefore 22. Now, 19 books
on Hindi are to be arranged in these 22 places so that no two of them are together. Out of 22
places 19 places for Hindi books can be chosen in 22
Ci9 ways. Since all books on Hindi are
COMBINATIONS 11.15

identical, so 19 books on Hindi can be arranged in 19 chosen places in only one way. Hence, the
22
required number of ways =1 x Ci9 X 1 =1540.
EXERCISE 11.2

1.
From a group of 15 cricket players, a team of 11 players is to be chosen. In how many ways
can this be done?
2.
How many different boat parties of 8, consisting of 5 boys and 3 girls, can be made from 25
boys and 10 girls?
3.
In how many ways can a student choose 5 courses out of 9 courses if 2 courses are
compulsory for every student?
4.
In how many ways can a football team of 11 players be selected from 16 players? How

w
many of these will (i) include 2 particular players? (ii) exclude 2 particular players?
5.
There are 10 professors and 20 students out of whom a committee of 2 professors and 3
students is to be formed. Find the number of ways in which this can be done. Further find in

Flo
how many of these committees:
(i) a particular professor is included, (ii) a particular student is included.

e
(iii) a particular student is excluded.

re
6.
How many different products can be obtained by multiplying two or more of the numbers

F
3,5,7,11 (without repetition)?
From a class of 12 boys and 10 girls, 10 students are to be chosen for a competition; at least
ur
r
7.
fo
including 4 boys and 4 girls. The 2 girls who won the prizes last year should be included. In
how many ways can the selection be made?
ks
8.
How many different selections of 4 books can be made from 10 different books, if
Yo
oo

(i) there is no restriction; (ii) two particular books are always selected;
(iii) two particular books are never selected?
B

9. From 4 officers and 8 jawans in how many ways can 6 be chosen (i) to include exactly one
re

officer (ii) to include at least one officer?


10. A sports team of 11 students is to be constituted, choosing at least 5 from class XI and at
u
ad

least 5 from class XII. If there are 20 students in each of these classes, in how many ways can
Yo

the teams be constituted?


11.
A student has to answer 10 questions, choosing at least 4 from each of part A and part B. If
d
Re

there are 6 questions in part A and 7 in part B, in how many ways can the student choose 10
in

questions?
F

12. In an examination, a student has to answer 4 questions out of 5 questions; questions 1 and 2
are however compulsory. Determine the number of ways in which the student can make
the choice.

13. A candidate is required to answer 7 questions out of 12 questions which are divided into
two groups, each containing 6 questions. He is not permitted to attempt more than 5
questions from either group. In how many ways can he choose the 7 questions?
14. There are 10 points in a plane of which 4 are collinear. How many different straight lines
can be drawn by joining these points.
15. Find the number of diagonals of (i) a hexagon (ii) a polygon of 16 sides.
16. How many triangles can be obtained by joining 12 points, five of which are collinear ?
17.
In how many ways can a committee of 5 persons be formed out of 6 men and 4 women
when at least one woman has to be necessarily selected ?
IS. In a village, there are 87 families of which 52 families have at most 2 children. In a rural
development programme, 20 families are to be helped chosen for assistance, of which at
least 18 families must have at most 2 children. In how many ways can the choice be made ?
11.16 APPLIED MATHEMATICS-XI

-)9. A group consists of 4 girls and 7 boys. In how many ways can a tern of 5 members be
selected if the team has (i) no girl? (ii) at least one boy and one girl? (iii) at least 3 girls?
:n. A committee of 3 persons is to be constituted from a group of 2 men and 3 women. In how
many ways can this be done? How many of these committees would consist of 1 man and 2
women?

-●1 Find the number of (i) diagonals (ii) triangles formed in a decagon.
22. Determine the number of 5 cards combinations out of a deck of 52 cards if at least one of the
5 cards has to be a king ?
23. We wish to select 6 persons from 8, but if the person A is chosen, then B must be chosen. In
how many ways can the selection be made ?
24. In how many ways can a team of 3 boys and 3 girls be selected from 5 boys and 4 girls?
25. Find the number of ways of selecting 9 balls from 6 red balls, 5 white balls and 5 blue balls if
each selection consists of 3 balls of each colour.
.O . Determine the number of 5 cards combinations out of a deck of 52 cards if there is exactly

w
one ace in each combination.

3 red balls can be selected. F lo


27. In how many ways can one select a cricket team of eleven from 17 players in which only
5 persons can bowl if each cricket team of 11 must include exactly 4 bowlers?
28. A bag contains 5 black and 6 red balls. Determine the number of ways in which 2 black and
for F
ree
20. In how many ways can a student choose a programme of 5 courses if 9 courses are available
and 2 specific courses are compulsory for every student?
.30. A committee of 7 has to be formed from 9 boys and 4 girls. In how many ways can this be
done when the committee consits of:
Your

(i) exactly 3 girls? (ii) at least 3 girls? (iii) at most 3 girls?


ks
eBoo

31. In an examination, a question paper consists of 12 questions divided into two parts i.e., Part
I and Part II, containing 5 and 7 questions, respectively. A student is required to attempt 8
questions in all, selecting at least 3 from each part. In how many ways can a student select
ad

the questions?
our

32. A parallelogram is cut by two sets of m lines parallel to its sides. Find the number of
parallelograms thus formed.
33. Out of 18 points in a plane, no three are in the same straight line except five points which are
Re
Y

collinear. How many (i) straight lines (ii) triangles can be formed by joining them ?
Find

ANSWERS
1. 1365 2. 6375600 3. 35 4. 4368 (i) 2002 (ii) 364
5. 51300 (i) 10260 (ii) 7695 (iii) 43605 6. H 7. 104874
8. (i)210 (ii) 28 (iii) 70 9. (i)224 (ii) 896 lo.
11. 266 12. 3 13. 780 14. 40 15. (i) 9 (ii) 104 le. 210 17. 246
52 35
18. 19 20 C0 19. (i) 21(ii) 441(iii) 91
20. 10, 6 21. (i) 35 (ii) 120 22. 886656 23. 22 24. 40 25. 2000
26. 778320 27. 3960 28. 200
in + 2
29. 35 30. (i)504 (ii) 588 (iii) 1630 31. 420 32. (
33. (i) 144 (ii) 806
COMBINATIONS 11.17

HINTS TO SELECTED PROBLEMS

2- Required no. of boat parties = X ^^03.


3- Since 2 courses are compulsory. So, the student is to choose 3 courses out of the remaining 7
courses. This can be done in ^03 ways.
16
C 11-
We have to select 11 players out of 16. So, required number of ways =
(i) Since 2 particular players are always included, so, we have to select 9 players out of
the remaining 14 players. This can be done in ways,
(ii) Since 2 particular players are excluded from every selection, so, we have to select 11
players from the remaining 14 players. This can be done in ways.
6- Total number of products = Number of ways of selecting 2 or 3 or all out of 4 numbers
3,5, 7,11

ow
= + “^C3 + = 6 + 4 +1 =11.
7. Since two girls who won the prizes last year are to be included in every selection. So, we
have to select 8 students out of 12 boys and 8 girls, choosing at least 4 boys and at least two
12 8
girls. This can be done in x ^€2 + ^ ^^3 + C4 X °C4 =104874 ways.

e
9. (i) Required number of ways =

Flx ®C5

re
(ii) Required number of ways = Total no. of ways - No. of ways of selecting no officer

F
12
^6 ” ^6-
ur
r
20
10. Required number of ways = ^'^C5 x x C5. fo
11- The various possibilities are : (i) 4 from part A and 6 from part B (ii) 5 from part A and 5
ks
from part B (iii) 6 from part A and 4 from part B.
Yo
oo

So, the required number of ways = x x ^05 + x = 266.


B

12.
Required number of ways = €2-
13. Required number of ways = ^€5 x ^€2 + x ^03 + + ^C2
re

=780.

14. Number of straight lines = ^^€2 - ’C2 +1-


4
u
ad
Yo

16. Number of triangles = C3-"C3-


12

18. 52 families have at most 2 children, while 35 families have more than 2 children, The
d

selection of 20 families of which at least 18 families must have at most 2 children can be
Re
in

made as under:

(i) 18 families out of 52 and 2 families out of 35


F

or. (ii) 19 families out of 52 and 1 family out of 35


or, (iii) 20 families out of 52.
19. (i) From a group of 4 girls and 7 boys, a team of 5 consisting of no girls can be chosen
in ^ C5 = 21 ways,
(ii) A team of 5 consisting of at least one boy and one girl can be chosen in
X ^C4 + ^C2 X ^C3 + X ^€2 + ^C4 X = 441 ways,
(iii) A team of 5 consisting of at least 3 girls can be chosen in
“^C3 X + ^C4 X = 91 ways.
21- A committee of 3 persons out of 2 men and 3 women can be constituted in ^C3 = 10 ways.
2 3
A committee of 1 man and 2 women can be constituted in x C2 = 6 ways.
11.18 APPLIED MATHEMATICS-XI

22. Required number of combinations = Total number of 5 card combinations


- Number of 5 car combinations having no king.
52
Cg - ^Cg = 886656.
24. Number of ways of selecting team = x ^Cg = 40.
25. Number of ways of selecting 9 balls = ^Cg x ^Cg x ^Cg = 2000.
26. Out of 4 aces one ace can be selected in ways and from the remaining 48 cards, four
48
cards can be selected in
C4 ways. So, number of 5 cards combinations consisting of
exactly one ace = x '^^C4 =778320.
27. Required number of ways = ^04 x C7-
12

28. Out of 5 black and 6 red balls, 2 black and 3 red balls can be chosen in x ^Cg = 200 ways.
29. Required number of ways = Number of ways of selecting 3 courses out of 7 courses.
n

- Cg ways = 35.

w
30. (i) A committee consisting of 3 girls and 4 boys can be formed in x ^Cg =504 ways,

F lo
(ii) A committee consisting of at least 3 girls can
= 588 ways,
(iii) A committee of at most 3 girls can be formed in
be formed in x %g + ^Cg x

e
Fre
^Cy X ^Cq + X ^C-\ + ^Cg X ^C2 + ^C4 X ^Cg = 1632 ways.
31. At least 3 questions can be selected in the following ways:
Part I Part II
for
3 5
4
r
4
You

5 3
oks

So, required number of ways = ^Cg x ^Cg + x + ^Cg x ^Cg = 420.


eBo

32. Each set of parallel lines consists of {m + 2) lines and each parallelogram is formed by
choosing two lines from the first set and two straight lines from the second set.
ni + 2 ;h + 2
Hence, the total number of parallelograms = CgX ^2-
ad
our

11.5 MIXED PROBLEMS ON PERMUTATIONS AND COMBINATIONS


In this section, we intend to discuss some practical problems where both permutations and
dY
Re

combinations are used as is illustrated in the following examples.


Fin

ILLUSTRATIVE EXAMPLES

EXAMPLE 1 Out of 7 consonants and 4 vozuels, how many zuords of 3 consonants and 2 vowels can be
formed ?
SOLUTION Three consonants out of 7 and 2 vowels out of 4 can be chosen in ^Cg x ways.
Thus, there are ^Cg x groups each containing 3 consonants and 2 vowels. Since each group
contains 5 letters, which can be arranged among themselves in 5! ways.
Hence, the required number of words = (^Cg x '^Cg) x 5! = 25200.
EXAMPLE 2 Hoiv many four-letter words can beformed using the letters of the zvord 'FAILURE', so that
(i) F is included in each zvord ? (ii) F is not included in any word ?
SOLUTION There are 7 letters in the word 'FAILURE'.
COMBINATIONS 11.19

(i) To include F in every 4 letter word, we first select four letters from the 7 letters of the word
'FAILURE' such that F is included in every selection. This can be done by selecting three letters
from the remaining 6 letters i.e. A, I, L, U, R, E in ways. Now, there are 4 letters in each of
^C3 selections. Consider one of these selections. This selection contains 4 letters which can

rw
be arranged in 4! ways. Thus, each of selections provides 4! words.
Hence, the total number of words = ^€3 x 4! = 480.
(ii) If F is not to be included in any word, then we first select 4 letters from the remaining 6

e
letters. This can be done in ^€4 ways. Now, every selection has 4 letters which can be arranged

e
in a row in 4! ways.

ulo
Hence, the total number of words = ^€4 x 4! = 360.

r
F
EXAMPLES
Hozu many zvords zuith or zvitUoiit meaning, can be formed using all the letters of the zvord
EQUATlOhJ at a time so that vozvels and consonants occur together?

oF
SOLUTION There are 5 vowels and 3 consonants in the word EQUATION. All vowels can be
put together in 5! ways and all consonants can be put together in 3! ways. Considering all vowels

rs
as one letter and all consonants as at letter, vowels and consonants can be arranged in 2! ways.

k
Therefore, vowels and consonants can be put together in 5! x 3! x 2! ways i.e. 1440 ways.

o
EXAMPLE 4
Hozv many five-letter zvords containing 3 vozvels and 2 consonants can be formed using the
of
letters of the zvord 'EQUATION’ so that the tzvo consonants occur together ?
o
SOLUTION There are 5 vowels and 3 consonants in the word 'EQUATION'. Three vowels out
Y
of 5 and 2 consonants out of 3 can be chosen in ^03 x ^€2 ways. So, there are ^C3 x ^€2 groups
rYB

each containing 3 consonants and two vowels. Now, each group contains 5 letters which are to
be arranged in such a way that 2 consonants occur together. Considering 2 consonants as one
ue

letter, we have 4 letters which can be arranged in 4! ways. But two consonants can be put
together in 2! ways. Therefore, 5 letters in each group can be arranged in 4!x 2!ways.
d

Hence, the required number of words = (^€3 ^C2)x 4!x 21 = 1440.


o
ad

EXAMPLES Hozv many zvords zvitli or zintliout meaning, each 2 of vozvels and 3 consonants can be
in

formed from the letters of the zvord DAUGHTER?


SOLUTION There are 3 vowels and 5 consonants in the word DAUGHTER out of which 2
Re

vowels and 3 consonants can be chosen in ^C2 x ^03 ways. These selected five letters can now
F

be arranged in 5! ways.
Hence, required number of words = ^C2 x ^03 x 5! = 3 x 10 x 120 = 3600
EXA.MPLE6 The Ejiglish alphabet has 5 vozvels and 21 consonants. Hozv many zvords zvith tzvo different
z^oivels and 2 different consonants can be formed from the alp’habet ?
SOLUTION Out of 5 vowels and 21 consonants, 2 vowels and 2 consonants can be chosen in
^C2 X ^^C2 ways. These selected 4 letters can now be arranged in 4! ways. Therefore, by the
fundamental principle of counting, required number of words is
^C2 X 2^C2 X 4! =10 X 210 X 24 =50400.
EXAMPLE? In hozv many zvays can 5 girls and 3 boys be seated in a rozv so that no tzvo boys are
together?
SOLUTION Since boys are to be separated. Therefore, let us first seat 5 girls. This can be done in
5! ways. For each such arrangement, three boys can be seated only at the cross marked places.
xGxGxGxGxGx
11.20 APPLIED MATHEMATICS-XI

There are 6 crossed marked places and three boys can be seated in x 3!ways. Hence, by the
fundamental principle of counting, the total number of ways is 5! x 3! = 14400.
LXAMPLE 8 Wozy many words can be formed by taking 4 letters at a time out of the letters of the word
‘MATHEMATICS'.

SOLUTION There are 11 letters viz. MM, AA, TT, H, E, I, C, S. All these letters are not distinct, so
we cannot use We can choose 4 letters from the following ways:

(i) All the four distinct letters: There are 8 distinct letters viz. M, A, T, H, E, I, C, S out of which 4
can be chosen in ^€4 ways. So, the total number of groups of 4 letters = ^04. Each such group
has 4 letters which can be arranged in 4! ways.
Hence, the total number of words = ®C4 x 4! = ^p4 =1680.

w
(ii) Two distinct and two alike letters: There are 3 pairs of alike letters viz MM, AA, TT, out of
which one pair can be chosen in ways. Now we have to choose two letters out of the

F lo
remaining 7 different types of letters which can be done in ^ C2 ways. So, the total number of
groups of 4 letters in which two are different and 2 arealike is x ^€2-Each such group has 4

ee
4!
letters of which 2 are alike and remaining two distinct and they can be arranged in ways.

Fr
2!
4!
Hence, the total number of words in which two letters are alike =
for x '^€2 x 2!
= 756.

(hi) Two alike of one kind and two alike of other kind: There are 3 pairs of 2 alike letters out of
ur
which 2 pairs can be chosen in ^€2 ways. So, there are ^€2 groups of 4 letters each. In each
s

group there are 4 letters of which 2 are alike of one kind and two alike of other kind. These 4
ook
Yo

4!
letters can be arranged in ways. Hence, the total number of words in which two letters are
2!2!
eB

4!
alike of one kind and two alike of other kind = ^€2 x 2!2!
= 18.
our
ad

From (i), (ii) and (iii) the total number of 4 letter words = 1680 + 756 +18 = 2454.
EXAMPLE 9 Eighteen guests have to be seated, half on each side of a long table. Four particular guests
desire to sit on one particular side and three others on the other side. Determine the number of ways in
Y

which the seating arrangement can be made.


Re
nd

SOLUTION Since four particular guests want to sit on a particular side A (say) and three others
on the other side B (say). So, we are left with 11 guests out of wliich we choose 5 for side A in
Fi

C5 ways and the remaining 6 for side B in °Cg ways. Hence, the number of selections for the
two sides is ^^05 x
Now 9 persons on each side of the table can be arranged among themselves in 9! ways.
11!
Hence, the total number of arrangements = x x 9!x 9! = X 9!x 9!
6!5!

EXAMPLE 10 Hozu many four-letter zvords can beformed using the letter of the zuord 'INEFFECTIVE' ?
SOLUTION There are 11 letters in the word 'INEFFECTIVE', viz. EEE, FF, II, C, T, N, V.
The four-letter words may consist of:
(i) 3 alike letters and 1 distinct letter
(ii) 2 alike letters of one kind and 2 alike letters of the second kind
(iii) 2 alike letters and 2 distinct letters
(iv) all different letters
eoMDlKA.TlOMS 11.21

Now we, shall discuss these four cases one by one:


(i) 3 alike letters and 1 distinct letter: There is one set of three alike letters viz. EEE. So, three alike
letters can be selected in one way. Out of the 6 different letters F, I, T, N, V, C one letter can be
selected in ways. Thus, three alike and one different letter can be selected in 1 x
ways. So, there are groups each of which contains 3 alike letters and one different letter.
4!
These 4 letters can be arranged in 3!1!
ways.

Hence, the total number of words consisting of three alike and one distinct letters
4!
= ^Cix 3!1!
= X 4 = 24.

(ii) 2 alike letters of one kind and 2 alike letters ofsecond kind: There are three sets of two alike letters
selected in ^€2 ways. So, there are ^€2 groups

low
viz EE, FF, II. Out of these three sets two can be
each of which contains 4 letters out of which 2 are alike of one type and two are alike of second
4!
type. Now, 4 letters in each group can be arranged in 2! 2!
ways.

Hence, the total number of words consisting of two alike letters of one type and 2 alike letters of

ee
second type = ^€2 x
4!
= 18.

F
Fr
2! 2!

(iii) 2 alike and 2 different letters: Out of 3 sets of two alike letters one set can be chosen in
for
ur
ways. Now, from the remaining 6 distinct letters, 2 letters can be chosen in ^€2 ways. Thus, 2
^ 3 6
alike letters and 2 distinct letters can be selected in x ”C2) ways. So, there are( Cj x C2)
ks

4!
Yo

groups of 4 letters each. Now, letters of each group can be arranged among themselves in —
oo
eB

ways.
Hence, the total number of words consisting of two alike letters and 2 distinct
= ^Cjx ^C2x —=540.
r

2!
ou
ad

(iv) All different letters: There are 7 distinct letters E, F, 1, T, N, V, C out of which 4 can be selected
Y

in ^ C4 ways. So, there are ^ C4 groups of 4 letters each. The letters in each of ^ C4 groups can be
arranged in 4! ways.
Re
nd

So, the total number of 4 letter words in which all letters are distinct = C4 x 4! = 840.
Fi

Hence, the total number of 4-letter words = 24 +18 + 540 + 840 =1422.
EXAMPLE 11 In }jqio many zvays can the letters of the word PERMUTATIONS be arranged if there are
always 4 letters betzveen P and S?
SOLUTION There 12 letters in the given word of which 2 are T's. There can be 4 letters between
P and S in one of the following ways:
(i) There are 2T's and 2 other letters from the remaining 8 letters (excluding 2T's and P
and S).
(ii) One T and 3 other letters from the remaining 8 letters,
(iii) There is no T and 4 other letters.
Let us now find the number of words in each case.
11.22
APPLIED MATHEMATICS-XI
(i) In the first case, 2 letters can be chosen from remaining 8 letters in wa^rs. Now, 2T'c and 3

other letters can be arranged between P and S in ways. Also, P and S can interchange their
positions. So, 2T's and 2 other letters can be arranged between P and S in ^€2 x ^ x 2! ways.
Considering these six letters as one letter and the remaining 6 letters can be arranged in 7! ways.
Total number of words, in this case =®C2X—x2!x7!

w
2!

(ii) In this case, 3 letters can be chosen from the remaining 8 letters in ®C3 ways. Now, one T and
3 other letters from the remaining 8 letters can be arranged between P and S in 4! ways. Also, P
and S can interchange their positions. So, one T and 3 other letters can be arranged between

e
P and S in C3 x 4!x 2! ways. Considering these six letters as one letter and the remaining 6

re
o
letters can be arranged in 7! ways.

r
8
Total number of words formed = C3 X 4!x 2!x7!

F
(iii) In this case, 4 letters other than 2T's can be chosen from the remaining 8 letters in ^C4 ways.
oF
ul
These 4 letters can be arranged between P and S in 4! ways. Also, P and S can interchange their
positions in 2! ways. Thus, 4 letters between P and S can be arranged in x 4!x 2! ways.

sr
Taking these 6 letters as one letter with the remaining 6 letters (including 2T's), we have 7 letters

ko
7!
which can be arranged in 2! ways. of
7!
Number of words formed = ®C4 x 4! x 2! X —
o
2!
Y

Hence, total number of words = x — x 2!x 7! + ®C-j x 4! x 2! x 7!+® C,. x 4'x 2'x —
B

2! ^ ■ 2!
Y

= 25401600
er

EXERCISE 11.3
u

1.
How many different words, each containing 2 vowels and 3 consonants can be formed with
5 vowels and 17 consonants?
d
o
ad

2. There are 10 persons named P^, P2, P3 ..., Pjq. Out of 10 persons, 5 persons are to be
in

arranged in a line such that is each arrangement P^ must occur whereas P4 and do not
occur. Find the number of such possible arrangements.
Re

3. How many words, with or without meaning can be formed from the letters of the word
F

'MONDAY', assuming that no letter is repeated, if (i) 4 letters are used at a time (ii) all
letters are used at a time (iii) all letters are used but first letter is a vowel?
4. Find the number of permutations of n distinct things taken r together, in which 3 particular
things must occur together.
5. How many words each of 3 vowels and 2 consonants canbe formed from the letters of the
word INVOLUTE?
6.
Find the number of permutations of n different things taken r at a time such that two
specified things occur together?
7. Find the number of ways in which: (a) a selection (b) an arrangement, of four letters can be
made from the letters of the word 'PROPORTION'.
8. How many words can be formed by taking 4 letters at a time from the letters of the word
'MORADABAD'?
9.
A business man hosts a dinner to 21 guests. He is having 2 round tables which can
accommodate 15 and 6 persons each. In how many ways can he arrange the guests?
11.23
COMBINATIONS

10. Find the number of combinations and permutations of 4 letters taken from the word
'EXAMINATION'.
11. A tea party is arranged for 16 persons along two sides of a long table with 8 chairs on each
side. Four persons wish to sit on one particular side and two on the other side. In how many
ways can they be seated?
ANSWERS
11-3
1. 816000 2. ^C4x5! 3. (i) 360 (ii) 720 (iii) 240 4. C,_3(r-2)!3!
II-2
5.2880 6. 2{r-l) Pr-2 7. (a) 53 (b) 758 8. 626
10
9. X 14! X 5! 10. 2454 11.

_HINTS TO SELECTED PROBLEMS


1. 2 vowels out of 5 and 3 consonants out of 17 can be chosen in ^€2 ^ C3 ways.

w
Now, 5 letters in each selection can be arranged in 5! ways.
17
So, total number of words = ^€2 x C3 X 5! = 816000

Flo
3. (i) Total number of 4 letter words formed from the letters of the word 'MONDAY"
= ^04x41 = 360.

ee
(ii) Total number of words formed by using all letters of the word 'MONDAY

Fr
= 61=720

(iii) There are two vowels A and O. So, first place can be filled in 2 ways and the remaining
for
ur
5 places can be filled in 5! ways.
So, total number of words beginning with a vowel = 2x5! = 240.
s
5. Required number of words = ^C3 x "^€2 x 5!
k
Yo
oo

11-2
6. Out of {n - 2) remaining things select (r - 2) things in Q _ 2 ways. Consider two
eB

specified things as one and mix it with (r - 2) selected things. Now we have (r -1) things
which can be arranged in (r — 1)! ways, but two specified things can be put together in 2!
ways. Hence, required number of ways = ” " ^ C,_2x(r-l)!x2L
r
ou
ad

9. Total number of ways = x x 14! x 5!


Y

11. 4 persons wish to sit on side A(say) and two on the other
10
side B(say). So, 10 persons are left,
out of which 4 persons for side A can be selected in C4 ways and 6 persons for side B from
Re
nd

the remaining 6 persons in ways. Hence, the number of selections for two sides
Fi

Now, 8 persons on each side can be arranged amongst themselves in 8! ways.


10
C4 X
Hence, the total number of seating arrangements = x x 8! x 8!
MULTIPLE CHOICE QUESTIONS (MCQs)
Mark the correct alternative in each of the follozving:
1. If _4o ' then is equal to
(a) 4896 (b) 816 (c) 1632 (d) none of these

2. If + 4 , then ''C3 is equal to


(a) 54 (b) 56 (c) 58 (d) none of these

3. If ^^C3r = + 3, then r is equal to


(a) 5 (b) 4 (c) 3 (d) 2
11.24 APPLIED MATHEMATICS-Xl

4. If^°C r +1 -
20
Q _■], then r is equal to
(a) 10 (b) 11 (c) 19 (d) 12
3- If C (n, 12) = C {n, 8), then C (22, n) is equal to
(a) 231 (b) 210 (c) 252 (d) 303
6. If'”Ci=”C2,then
(a) 2m=n (b) 2}}i = n{n + 1) (c) 2 m = n(» -1) (d) ln = m {m -1)
7. If"Ci2="C8,then;i =
(a) 20 (b) 12 (c) 6 (d) 30
» + i
8. lf"C, = C;f, then X =
(a) r (b) r-1 (c) n (d) r + 1

w
9,
, then <7 =

Flo
(a) 2 (b) 3 (c) 4 (d) none of these
10.
+ ^C2 + ^C3 + ^€4 + ^C5 is equal to

ee
(a) 30 (b) 31 (c) 32 (d) 33

Fr
11- Total number of words formed by 2 vowels and 3 consonants taken from 4 vowels and 5
consonants is equal to for
ur
(a) 60 (b) 120 (c) 7200 (d) none of these
12. There
are 12 points in a plane. The number of the straight lines joining any two of them
s
when 3 of them are collinear, is
k
Yo
oo

(a) 62 (b) 63 (c) 64 (d) 65


eB

13. Hiree persons enter a railway compartment. If there are 5 seats vacant, in how many
ways can they take these seats ?
(a) 60
r

(b) 20 (c) 15 (d) 125


ou
ad

In how many ways can a committee of 5 be made out of 6 men and 4 women containing at
Y

least one women ?


(a) 246 (b) 222 (c) 186 (d) none of these
Re
nd

13. There are 10 points in a plane and 4 of them are collinear. The number of straight lines
Fi

joining any two of them is


(a) 45 (b) 40 (c) 39 (d) 38
1^- There are 13 players of cricket, out of which 4 are bowlers. In how many ways a team of
eleven be selected from them so as to include at least two bowlers ?
(a) 72 (b) 78 (c) 42 (d) none of these

17. If Cq + C] + C2 + ... + C„ = 256, then '^'^€2 is equal to


(a) 56 (b) 120 (c) 28 (d) 91
1^’ The number of ways in which a host lady can invite for a party of 8 out of 12 people of
whom two do not want to attend the party together is
(a) 2x ”C7 +
10
Cs (b) + ”C7
(c) (d) none of these
11.25
COMBINATIONS

19. Given 11 points, of which 5 lie on one circle, other than these 5, no 4 lie on one circle. Then the
number of circles that can be drawn so that each contains at least 3 of the given points is
(a) 216 (b) 156 (c) 172 (d) none of these

20 How many different committees of 5 can be formed from 6 men and 4 women on which
exact 3 men and 2 women serve ?
(a) 6 (b) 20 (c) 60 (d) 120

21.
If 3r + 1 , then the value of r is

(a) 12 (b) 8 (c) 6 (d) 10 (e) 14

22. The number of diagonals that can be drawn by joining the vertices of an octagon is
(a) 20 (b) 28 (c) 8 (d) 16

w
23.
The value of 0 + ^Cil + f^q +^C2 +... + is

F lo
(a) f -1 (b) 2® - 2 (c) 2^* -1 (d) 2'

24. Among 14 players, 5 are bowlers. In how many ways a team of 11 may be formed with at

ee
Fr
least 4 bowlers?

(a) 265 (b) 263 (c) 264 (d) 275

25 A lady gives a dinner party for six guests. The number of ways in which they may befor
ur
selected from among ten friends if two of the friends will not attend the party together is
(a) 112 (b) 140 (c) 164 (d) none of these
s
ook
Yo

n+ 1
26.
If C3 = 2 ● "C2 / then» =
(a) 3 (b) 4 (c)5 (d) 6
eB

27. The number of parallelograms that can be formed from a set of four parallel lines
intersecting another set of three parallel lines is
r

(d) 18
ou
ad

(a) 6 (b) 9 (c) 12


28 The number of ways in which a committee consisting of 3 men and 2 women, can be chosen
Y

from 7 men and 5 women, is


(a) 45 (b) 350 (c) 4200 (d) 230
Re
nd

29. The number of signals that can be sent by 6 flags of different colours taking one or more at a
Fi

time is

(a) 63 (b) 1956 (c) 720 (d) 21


30. The straight lines I2 and 12, are parallel and lie in the same plane. A total number of
m

points are taken on n points on I2, k points on The maximum number of triangles
formed with vertices at these points are
m+n+k »i+n+t m

(a) ^3 (b) ^3
(c) '"C3 + "€3+^03 (d)

31 The number of committees of five persons with a chairperson that can be formed from 12
persons, is
(a) (b)
12
C4 (c) 12 X
11
C4 (d) ''C4
32. Total number of words formed by 2 vowels and 3 consonants taken from 4 vowels and 5
consonants is equal to
(a) 60 (b) 120 (c) 7200 (d) 720
11.26 APPLIED MATHEMATICS-XI

33. A five digit number divisible by 3 is to be formed using the numbers 0, 1, 2, 3, 4 and 5
without repetitions. The total number of ways this can be done is
(a) 216 (b) 600 (c) 240 (d) 3125
34.
Everybody in a room shakes hands with everybody else. The total number of hand shakes
is 66. The total number of persons in the room is
(a) 11 (b) 12 (c) 13 (d) 14
33. The number of triangles that are formed by choosing the vertices from a set of 12 points,
seven of which lie on the same line is
(a) 105 (b) 15 (c) 175 (d) 185
36. Given 5 different green dyes, four different blue dyes and three different red dyes, the
number of combinations of dyes which can be chosen taking at least one green and one blue
dye is

w
(a) 3600 (b) 3720 (c) 3800 (d) 3600
3. ● The total number of 9 digit numbers which have all different digits is

F lo
(a) 10! (b) 9! (c) 9x9! (d) 10x10!
38. The number of parallelograms that can be formed from a set of four parallel lines

ee
intersecting another set of three parallel lines is

Fr
(a) 6 (b) 18 (c) 12 (d) 9
- 9. The number of 5-digit telephone numbers having at least one of their digits repeated is
(a) 90,000 (b) 10,000 (c) 30240 for (d) 69760
ur
40.
The number of ways in which we can choose a committee from four men and six women so
that the committee includes at least two man and exactly twice as many women as men, is
ks

(a) 94 (b) 126 (c) 128 (d) none of these


Yo
oo

41. The number of ways in which a team of eleven players can be selected from 22 players
always including 2 of them and excluding 4 of them,
eB

is
16
(a) 11 (b) Cs (c) (d)
20
C9
r

ANSWERS
ou
ad

(b) 2- (b) 3- (c) 4. (a) 5. (a) 6- (c) 7. (a) 8. (d)


9. (b) 10. (b) 11. (c) 12. (c) 13. (a) 14. (a) 13. (b) 16. (b)
Y

17. (b) 18. (c) 19. (b) 20. (d) 21. (a) 22. (a) 23. (b) 24. (c)
Re
nd

25. (b) 26. (c) 27. (d) 28. (b) 29. (b) 30. (b) 31. (c) 32. (c)
33. (a) 34. (b) 35. (d) 36. (b) 37. (c) 38. (b) 39. (d) 40. (a)
Fi

41- (c)
FILL IN THE BLANKS TYPE QUESTIONS (FBQs)
1. If "P, = 840 and "Q = 35, then r =
15 C
^9 “ 13^
15
2. The value of ^6 C7 is

3. The value "P^ h- is

4. If u is even, then "Q is maximum when


n~2
5- If2x ''C5 =9x C5, then n =

6. If"Q-i=36, "Q=84and"C r+l = 126, then r -


18
Ci5+2('*Ci6) + Cjg +1 = "C3 , then n =
17
7. If
COMBINATIONS 11.27

8. If''Ci2=''C6,then”C2=
9. If , then r =

10. If "P4 =24.''C5, then the value of» is


11, The value of "Q +2 "C^_i +”Q_2/
12. A box contain 2 white balls, 3 black balls and 4 red balls. The number of ways three balls be
drawn from the box if at least one black ball is to be included in the draw is

13. Three balls are drawn from a bag containing 5 red, 4 white and 3 black balls. The number of
ways in which this can be done if at least 2 are red is
14. The total number of ways in which six and four signs can be arranged in aline such
that no two signs occur together is

w
15. A committee of 6 is to be chosen from 10 men and 7 women so as to contain at least 3 men
and 2 women. The number of different ways this can be done, if two particular women

F lo
refuse to serve on the same committee is

16. The number of committees of five persons with a chair person can be selected from 12

ee
persons, is

Fr
17. The number of automobile license plates that can be made if each plate contains two
different letters of English alphabet followed by tliree distinct digits, is
for
18. The number of permutations of n distinct objects taken r at a time in which three particular
ur
objects occurs together is
s
19. Out of 10 persons P], P2^--->P\0' ^ persons are to be arranged in a line such that in each
ok
Yo

arrangement P^ must occur whereas P4 and do not occur. The number of such
o

arrangements is
eB

ANSWERS
n
7. 20 8. 153
r

1. 4 2. 0 3. r! 4. r = 5. 10 6. 3
ad
ou

2
h+2
9. 36 10. 9 11. Cr 12. 64 13. 80 14. 35 15. 7800 16. 3960
Y

17. 4,6800 18.


h-3
C,_3(r-2)!3! 19. ^C4x5!
Re
nd

VERY SHORT ANSWER QUESTIONS ('V'SAQs;


Fi

Answer each of the following questions in one word or one sentence or as per exact requirement of the
question:
HI
n + r
1. Write S Cf in the simplified form.
r = 0

35
2. If h+7
C 4/1 - 2 ' then write the values of n.

3. Write the number of diagonals of an n-sided polygon.


4. Write the expression +1+ _ 1 + 2 x "Q in the simplest form.
6
56-r 50
5. Write the value of S ^3 + C4.
r = 1

6. There are 3 letters and 3 directed envelopes. Write the number of ways in which no letter is
put in the correct envelope.
7. Write the maximum number of points of intersection of 8 straight lines in a plane.
11.28 APPLIED MATHEMATICS-XI

8. Write the number of parallelograms that can be formed from a set of four parallel lines
intersecting another set of three parallel lines.
9. Write the number of ways in which 5 red and 4 white balls can be drawn from a bag
containing 10 red and 8 white balls.
10. Write the number of ways in which 12 boys may be divided into three groups of 4 boys
each.

11- Write the total number of words formed by 2 vowels and 3 consonants taken from 4 vowels
and 5 consonants.

ANSWERS
JI + H/ + 1 n {n - 3) n+2
1.
1 2. 3,6
2
4.
Q + 1 5. 56c4 6. 2
12!
7. 28 8. 18 9. lOCg X ^C4 10. 11.
‘*C2x5C3x5!
(4!)^ 3!

w
F lo for F
ree
r
You
oks
eBo
ad
our
Re
dY
Fin
CHAPTER 12

STRAIGHT LINES

12.1 DEFINITION OF A STRAIGHT LINE


A straight line is a cwve such that every point on the line segment joining any two points on it lies on it.
THEOREM Everyfirst degree equation in x, y repmsents a straight line.
l’RtX)F Let ax + by + c = 0 be a first degree equation in .v, y where a, b, c are constants.
Let P yfi and Q (.t2,1/2) be any two points on the curve represented by ax + by + c = 0. Then,
axi + byi + c = 0 and ax2 + by2 + c = 0

w
Let R be any point on the line segment joining P and Q. Suppose R divides PQ in the ratio X: 1.
Then, the coordinates of R are

of X.
Now,

a
X. X2 "F -V-j + b
^ y2 + j/i
X + 1

+ c
F lo
'X%2+ -Vi Xy2+yC . In order to prove that the curve
A. + 1

represented by <7.v + by + c = 0 is a straight line, it is sufficient to show that R lies on it for all values
for F
ree
X {ax2 + bp2 + c) + (t7.Yi + byi + c)
X + l X+\

= + 0 = 0 [Using (i)]
Your

^XX2+ -Yi y2 + j/l


ks

R lies on the curve represented by ax + by + c = 0.


eBoo

^ X, +1 X. + 1
Thus, every point on the line segment joining P and Q lies on ax + by + c = 0.
Hence, + by + c = 0 represents a straight line.
ad
our

Q.E.D.

NOTE When we say that a first degree equation in x, y i.e., ax + by + c = 0 represents a line, it means that
all points (x, y) satisfying ax + by + c= 0 lie along a tine. Thus, a line is also defined as the locus of a point
Re

satisfying the condition ax + by + c = 0 lohere a, b, c are constants.


Y

It follows from the above discussion that ax + by + c = 0 is the general equation of a line.
Find

It should be noted that there are only two unknowns in the equation of a straight line because
equation of every straight line can be put in the form + by +1 = 0 where a, b are two unknowns.
Note that x, y are not unknowns. In fact these are the coordinates of any point on the line and are
known as the current coordinates. Thus, to determine a line we will need two conditions to
determine the two unknowns. In the further discussion on straight line you will find that
whenever it will be asked to find a straight line there will always be two conditions cormecting
the two unknowns.

12.2 SLOPE (GRADIENT) OF A LINE


The trigonometrical tangent of the angle that a line makes with the positive direction of the x-axis in
anticlockwise sense is called the slope or gradient of the line.
The slope of a line is generally denoted by m. Thus, in = tan 0.
Since a line parallel to r-axis makes an angle of 0°with r-axis, therefore its slope is tan 0°= 0. A
line parallel to y-axis i.e., perpendicular to x-axis makes an angle of 90° with .r-axis, so its slope is
12.2 APPLIED MATHEMATICS-XI

tan ji/2 = CO. Also, the slope of a line equally inclined with axes is 1 or -1 as it makes 45® or 135°
angle with x-axis.

y y

B B

0
X’ 0 X X' X

w
A o o

Y' r

Flo
(i) (ii)
Fig. 12.1

ee
Fr
The angle of inclination of a line with the positive direction of x-axis in anticiockivise sense
always lies between 0°and 180°.
ILLUSTRATION! for
Find the slope of a line whose inclination to the positive direction of x-axis in
ur
anticlockwise sense is (i) 60° (ii) 0°(iii) 150° (iv) 120°.
SOLUTION (i) Slope = tan 60° = VI.
k s
(ii) Slope = tan 0° = 0.
Yo
oo

1
(iii) Slope = tan 150° = tan (90° + 60°) = - cot 60° =
eB

VI-
(iv) Slope = tan 120°= tan (90°+30) = - cot 30° = - VI.
r

ILLUSTRATION 2 What can be said regarding a line if its slope is (i) positive (ii) zero (iii) negative?
ou
ad

SOLUTION Let Obe the angle of inclination of the given line with the positive direction of x-axis
Y

in anticlockwise sense, Then, its slope is given by rn = tan 0.


(i) If the slope of the line is positive, then
Re
nd

= tan 0 > 0 =?■ 0 lies between 0° and 90° ^ 0 is an acute angle.


Fi

Thus, a line of positive slope makes an acute angle with the positive direction of .x-axis.
(ii) If the slope of the line is zero, then
m = tan 0 = 0 => 0 = 0
=> either the line is x-axis or it is parallel tox-axis.
Thus, a line of zero slope is parallel or coincident to x-axis.
(iii) If the slope of the line is negative, then
m = tan 0 < 0 => 0 lies between 90° and 180° => 0 is an obtuse angle.
Thus, a line of negative slope makes an obtuse angle with the positive direction of x-axis
in anticlockwise direction.

12.2.1 SLOPE OF A LINE IN TERMS OF COORDINATES OF ANY TWO POINTS ON IT


Leti^ (xj, yj) andQ(x2, y2) be two points on a line making an angle 0 with the positive direction
of.x-axis. Draw PL, QM perpendiculars on x-axis and PN1 on QM. Then,
PN = LM=OM-OL= X2 - x^ and, QN =QM -NM =QM - PL = 1/2 “)/i
STRAIGHT LINES 12.3

In A PQN, we have
tan 0 =
QN_i/2-yi
PN ^'2 -
y

w
e
e
or
X'

r
o

F
oF
ul
r

Fig. 12.2

s
r
Thus, if t/j) and (.\'2, x/2) are coordinates of any two points on a line, then its slope is

ko
y2-yi Difference of ordinates
m

X2-.V1 Difference of abscissae of


o
ILLUSTRATION 3
Find the slope of a line which passes through points (3, 2) and (- I, 5).
Y
SOLUTION We know that the slope of a line passing through two points (.Vi, y^) and (.V2, y2) is
rB

y2-yi
eY

given by m = . Here, the line passes through (3,2) and (-1,5).


^2-^1
5-2 _ 3
So, its slope is given by m =
u

4
d

12.3 ANGLE BETWEEN TWO LINES


o
ad

nij -
THEOREM
The angle 0 between the lines having slopes and 1112 is given by tan 0 = ±
in

PROOF
Let nil slopes of two given lines AB and CD which intersect at a point P
Re

and make angles 0^ and 02 respectively with the positive direction of .T-axis. Then,
F

nil = tan 0^ and »»2 = ^an 02-


y

X'

r
12.4 APPLIED MATHEMATICS-XI

Let ZAPC = 0 be the angle between the given lines. Then,


O2 = G + G-j [See Fig. 12.3]
G — 02 ~ 0j
tan 0 = tan (02 - 0^)
tan 0 =
tan 02 - tan 0j tan A - tan B
Using :tan{A-B) =
1 + tan 02 tan 0^ 1 + tan A tan B

tan 0 =
m2 -m-^

ow
1 + ?»2
Since ZAPD = ti - 0 is also the angle between AB and CD. Therefore,
m2 -
tan ZAPD = tan (ti-0) = - tan 0 = — [Using (i)l ...(ii)
1 + m2

e
re
From (i) and (ii), we find that the angles between two lines of slopes m^ and m2 are given by
tan 0 = ±
m2 - mj -1 m2 - m-[
^ 0 = tan ±

Frl
F
1 + m-i m2 1 + nil ’^^2

m2 -nil
ou
The acute angle between the lines is given by tan 0 =
1 + ni2

sor
Q.E.D.
ILLUSTRATION
kf
If /4 (-2,1), 6 (2, 3) and C(-2,-4) are three points, find the angle between
BA and BC.
oo

SOLUTION Let nii and m2 be the slopes of BA and BC respectively. Then,


Y

3-1
- -4-3 _ 7
2
i, and
B

nil =
2-(-2) 4 -2-2 ” 4
re

Let 0 be the acute angle between BA and BC. Then,


oY

7 _1
u

10
- nil 4 2 8
ad

tan 0 = = - ^ 0 = tan
1 + nil 11 7- 1 15 3 3J
d

+ X
4 2 8
in

CONDITION OF PARALLELISM OF LINES If two lines of slopes m^ and m2 are parallel, then the
Re

angle 0 between them is of 0°.


F

tan 0 = tan 0° = 0

m2 - nil m2 - nil
= 0 Using : tan 0 = ±
1 + nil ^”2 1 + nil
ni2 = nil
Thus, when two lines are parallel, their slopes are equal.
CONDITION OF PERPENDICULARITY OF TWO LINES If two lines of slopes mj and 1112 are perpen
dicular, then the angle 0 between them is of 90°.
From Fig. 12.3, we have
©2 =0+0]
02 =90°+ 0] [v 0 = 90°]
tan ©2 = tan (9O°+0])
=>
tan 02 = -cot 0]
STRAIGHT LINES 12.5

tan 0| tan 62 =-l ^ ??J|W2 =-l


Thus, zvhen two lines are perpendicular, the product of their slopes is-1. Ifm is the slope of aline, then the
slope of a line perpendicular to it is - (1/m).

ILLUSTRATIVE EXAMPLES

EXAMPLE 1 Determine x so that the line passing through (3, 4) and (x, 5) makes 135° angle with the
positive direction ofx-axis.
SOLUTION Since the line passing through (3, 4) and (.y, 5) makes an angle of 135° with Y-axis.
Therefore, its slope is tan 135° = -1. But, the slope of the line is also equal to
5-4 yi-'A
Using: m =
Y - 3 Y2-Y1

ow
5-4
-1 = => -y+3=1 => y = 2.
Y - 3

EXAMPLE 2 Find the angle betzoeen the lines joining the points (0, 0), (2, 3) and thepoints(2,-2),
(3, 5).

e
SOLUTION Let 0 be the angle between the given lines.

Fl
re
We have.
3-0 _ 3

F
nj] = Slope of the line joining (0, 0) and (2, 3) = 2-0 ~ 2
ur
5 + 2

or
= 7
= Slope of the line joining (2, - 2) and (3,5) = 3-2
sf
tan 0 = ±
m2 - Wlj ^ 7-3/2 = ± ^9 = tan
-1
+ — .
k
23/2 23 23
1+7 (3/2)
Yo
1 + m-^ m2
oo

EXAMPLE 3 Let A {6, 4) and B (2,12) be tzuo given points. Find the slope of a line perpendicular to AB.
eB

SOLUTION Let m be the slope of AB. Then,


12-4 8
m = - 2
2-6 -4
ur

1
ad

So, the slope of a line perpendicular to AB is


Yo

m l

EXAMPLE 4 Dcterznine x so that 2 is the slope of the line through (2,5) and (x. 3).
3-5
d

SOLUTION The slope of the line through (2, 5) and (y, 3) is . But, the slope of the line is
Re

y-2
in

given as 2.
F

3-5
= 2=> 2y-4=-2=> y=1
y-2

EXAMPLE 5 Wmt is the value ofyso that the line through (3, y) and (2,7) is parallel to the line through
(-1,4) and (0, 6)?
SOLUTION Let A (3, y), B (2, 7), C (-1,4) and D (0, 6) be the given points. Then,
7-y
= Slope of the line AB = 2-3 = y-7
6-4
= 2
and. m2 = Slope of the line CD =
O-(-l)
Since AB and CD are parallel.
nil =ni2^ y -7 =2^ y = 9.
12.6 APPLIED MATHEMATICS-XI

EXAMPLE 6 Without using Pythagoras theorem, shoiv that A (4, 4), B (3, 5) and C (-1, -1) are the
vertices of a right-angled triangle.
SOLUTION In A/4BC, we have,
4-5 4-(-l)
= Slope of AB = = -l and, ni2 - Slope of .AC = = 1
4-3 4-(-l)
Clearly, ni2 = -1. This shows that AB is perpendicular to AC i.e. ZCAB = n/2.
Hence, the given points are the vertices of a right-angled triangle.
EXAMPLE 7 A quadrilateral has the vertices at the points (-4, 2), (2, 6), (8,5) and (9, -7). Shoiv that
the mid-points of the sides of this quadrilateral are the vertices of a parallelogram.
SOLUTION Let A (-4, 2), 6 (2, 6), C (8,5) and D (9,-7) be the vertices of the given
quadrilateral. Let P, Q, R and S be the mid-points of AB, BC, CD and DA respectively. Then, the
coordinates of P, Q, R and S are P (-1, 4), Q (5,11/2), R(17/2,-1) and S(5/2,-5/2)

low
respectively. In order to prove that PQRS is a parallelogram, it is sufficient to show that PQ is
parallel to RS and PQ = RS. Let m-^ and m2 be the slope of PQ and RS respectively. Then,
11/2-4 1 5/2 + 1 1
mi = = — and, =
5-(-l) 512-17/2 4

ee
Clearly, nii = Therefore, PQ is parallel to PS.
Now,

F
Fr
PQ = (5 + 1)2, —'
— ^ PS
and, DC = J 17f + f-^ + i
I 2 2 III2 2 J
for 2 2
ur
PQ = PS
Thus, PQ 11 PS and PQ = PS. Hence, PQRS is a parallelogram.
ks

EXAMPLE 8 Prove that A (4, 3), B (6, 4), C (5, 6) and D {3,5) are the angular points of a square.
Yo
oo

SOLUTION Clearly,
eB

AB = ^4)2+(4-3)2 V5, BC = 7(6-4)2+(5-4)2


CD = 7(5-6)2+(3-5)2= Vs and, DA = 7(5 - 3)2 + (3 - 4)2 = VS
r
ou
ad

AB = BC = CD = DA.
4-3 1 6-4
Now, ni] = Slope of AB =
Y

6-4 2 '
ni2 = Slope of BC = = -2
5-6

5-6 _ 1
nd
Re

and. /»3 = Slope of CD =


3-5 " 2
Fi

Clearly, ;»2 = (1/2) (- 2) = -1 and = m3. Therefore, AB is perpendicular to BC and it is


parallel to CD. Thus, AB = BC = CA = AD, AS 1 BC and AB is parallel to CD. Hence, ABCD
is a square.

EXAMPLE 9 if the angle between tzvo lines is ^ and slope of one of the line-,find the slope of the other
line.

SOLUTION We know that the acute angle 8 between two lines with slopes and m2 is given by
tan 0 = ni2 -wq
...(i)
1 + m2
1
Let mj = - and m2 = m = slope of the other line. It is given that ®

Substituting m^ = m2 = m and 0 = — in (i), we obtain


4
STRAIGHT LINES 12.7

1
m ~
71
tan - 1
4 1
1 + 777 X
2
2777 -1
1 =
2 + 777
2 777 -1 , 1
± 1 => Im - 1 = 777 + 2 or, 2777 - 1 = - (777 + 2) => 777 = 3 or, 777 = —
777 +2 3

1
Hence, the slope of the other line is 3 or, -
tXAMPLE 10 If the points P {h, k), Q {x^, xji) and R (^2,1/2) ^ that:
{h - x-^) iij2 - xji) = (fc - i/i) {X2 - x{}
SOLUTION It is given that the points P {h, k), Q (.Vi, pi) and R (.Y2, 1/2) collinear.

w
Slope of PQ = Slope of QK

F lo
y2-yi
=> (fc - yi) {^2 - ^'i) = 0^ - ^i) (y2 - yi)
/7-.Y1 Y2-.Y1

EXAMPLE 11 If points (a, 0), (0, b) and (x, p) are collinear, using the concept of slope, prove that

e
Fre
=1.
a b

Let A {a, 0), B (0,1?) and P (.v, 1/) be the given collinear points. Then,
SOLUTION
for
Slope of AB = Slope of BP
r
b-0 y-b
You
oks

0-77 .Y - 0

-b y-b
eBo

a X

-bx - ^71/ ~ab => bx + ay = ab => — + y— =1 [On dividing both sides by ab]
our
ad

77 b
EXERCISE 12.1

1. Find the slopes of the lines which make the following angles with the positive direction of
-Y-axis:
dY
Re

2 n 3 n
(ii)- (iii) -
4 (iv)f
Fin

2. Find the slope of a line passing through the following points;


(i) (- 3, 2) and (1,4) (ii) (at], 2 at]) and (at^, 2 77^2) (iii) (3, - 5), and (1, 2)
3. State whether the two lines in each of the following are parallel, perpendicular or neither:
(i) Through (5, 6) and (2, 3); through (9, -2) and (6, -5)
(ii) Through (9, 5) and (-1,1); through (3, -5) and (8, -3)
(iii) Through (6, 3) and (1,1); through (-2, 5) and (2, -5)
(iv) Through (3,15) and (16, 6); through (-5, 3) and (8, 2).
4. Find the slope of a line (i) which bisects the first quadrant angle (ii) which makes an angle
of 30° with the positive direction of i/-axis measured anticlockwise.
5- Using the method of slope, show that the following points are collinear:
(i) A (4, 8), B (5,12), C (9,28) (ii) A (16, - 18), B (3, - 6), C (-10, 6)
12.8 APPLIED MATHEMATICS-XI

6. What is the value of y so that the line through (3, y) and (2,7) is parallel to the line through
(-1,4) and (0,6)?
7. What can be said regarding a line if its slope is
(i) zero (ii) positive (iii) negative?
8. Show that the line joining(2, - 3) and (-5, l)isparallel to the line joining (7,-1) and (0,3).
9. Show that the line joining (2, - 5) and (- 2,5) is perpendicular to the line joining (6,3) and
(1.1)-
10. Without using Pythagoras theorem, show that the points A (0, 4), B (1, 2) and C (3, 3) are
the vertices of a right angled triangle.

ow
11. Prove that the points (-4, -1), (-2, -4), (4, 0) and (2,3) are the vertices of a rectangle.

12. If three points A (It, 0), P (a, b) and B (0, k) lie on a line, show that: - + - = 1.
h k

13. The slope of a line is double of the slope of another line. If tangents of the angle between

e
Fl
re
them is , find the slopes of the other line.

F
14. Withoutusing the distance formula, show thatpoints(-2, -1), (4, 0), (3, 3)and(- 3, 2) are
ur
the vertices of a parallelogram.
or
15. Find the angle between the X-axis and the line joining the points (3, -1) and (4, - 2).
sf
16. Line through the points (- 2, 6) and (4,8) is perpendicular to the line through the points
k
(8,12) and {x, 24). Find the value of .t.
Yo
oo

17. Find the value of a: for which the points (x, -1), (2,1) and (4,5) are collinear.
18. Find the angle between X-axis and the line joining the points (3,-1) and (4, - 2).
B

19. By using the concept of slope, show that the points (- 2, -1), (4,0), (3,3) and (- 3,2) are the
re

vertices of a parallelogram.
u

ANSWERS
ad

l.(i) -1 (ii) -V3 ”(iv) V3


Yo

(iii) -1

2.(i)^
2
(ii)
f2 +fi (iiO ~l
d
Re

3. (i) parallel (ii) parallel (iii) perpendicular (iv) neither.


in

4. (i) 1 (ii) -V3 6. 9


F

7. (i) The line is either .v-axis or it is parallel to x-axis.


(ii) The line makes an acute angle with positive direction of .v-axis.
(iii) The line makes an obtuse angle with the positive direction of v-axis.
13. 4
3tu
15. 135“ 16. 4 17. 1

HINTS TO SELECTED PROBLEMS


12. It is given that points A {h, 0), B (0, k) andP {a, b) are collinear.Therefore,
Slope of PA = Slope of PB
b-0 b-k
a-h a-0

ab = {a - h) (b - k) ^ ab = ab -ak - bh + hk => hk - ok + bh ~ =1
h k
STRAIGHT LINES 12.9

13- Let m be the slop of first line. Then the slope of the second line is 2m. Let 0 be the angle
between the lines. Tlien,
2m - m
tan 0 -
1 + 2m^
1 m
— =± ^
3 1 + liir

hi? ± 3m +1 = 0

2m^ + 3m + 1 = 0 or, 2n? - 3m +1 = 0

ow
{2m + 1) {m + 1) =0or, (2f» -1) (?» -1) = 0 => m = ± ±L
2

15. Let 0 be the angle between the line joining the points (3,-1) and (4, - 2) and A:-axis. Then,
-2 + 1
tan 0 = -1^ 0 = 135®.
tan 0 = Slope of the line =^> tan 0 = 4-3

e
re
16. It is given that the line through the points A (- 2, 6) and B (4,8) is perpendicular to the line

rFl
through the points C (8,12) and D {x, 24). Therefore,

F
Slope of AB X Slope of CD = -1
8-6 24-12 1 12
= -lr^-4=.Y-8=> -V=4

r
= -1 => -X
3 x-8
ou
4 + 2 a: - 8

1
fo
It is given that points A (.r, -1), B (2,1) and C (4,5) are collinear.
ks
Slope of AB = Slope of BC
oo

1+1_5-l 2
= 2^2-x=l =>x=l.
Y

2-x'4-2 2-x
eB

’ 3. Let the required angle be 0. Then,


-2-(-l) = 1
ur

7U
=> 0=-
tan 0 = Slope of the line ^ tan 0 = 4-3 4
ad
Yo

19. Given points are A (- 2, — 1), B (4, 0), C (3, 3) and D (- 3, 2). Therefore,
2-3 1
d

0 + 1 _1
= Slope of AB = 1112 = Slope of CD =
4 + 2”6'
Re

-3-2 6
in

3-0 2 + 1
F

= -3
m3 = Slope of BC = = - 3 and, ii^ = Slope of AD - 3 + 2
3-4

Clearly,m-i =m2 andm3 =??i4. Therefore, AB||CD and BC | AD.


Hence, ABCD is a parallelogram.
12.4 INTERCEPTS OF A LINE ON THE AXES

If a straight line cuts x-axis at A and the y-axis at B, then OA and OB are knoivn as the intercepts of the
line on x-axis and y-axis respectively.
The intercepts are positive or negative according as the line meets with positive or negative
directions of the coordinate axes.

In Fig. 12.4, OA = x-intercept, OB = y-intercept.


OA is positive or negative according as A lies on OX or OX' respectively. Similarly, OB i.
is

positive or negative according as B lies on OY or OY' respectively.


12.10 APPLIED MATHEMATICS-XI

i
a.

ow
X' X
o
—X- intercept

e
Y'

re
Fig. 12.4

Flr
F
12.5 EQUATIONS OF LINES PARALLEL TO THE COORDINATE AXES

12.5.1 EQUATION OF A LINE PARALLEL TO x-AXIS


ou
Let AB be a straight line parallel to .v-axis at a distance b from it. Then, clearly the ordinate of each

sr
point on AB is b. Thus, AB can be considered as the locus of a point at a distance b from x-axis.

fo
Thus, if P (.V, y) is any point on AB, then y =!?. (See Fig. 12.5).
y
k
oo
t/ = b
Y

T
reB

b
uY

X' X
1
o
ad
do
in
Re

r'
F

Fig. 12.5
Hejice, the equation of a line parallel to x-axis at a distance bfrom it is y = b.
Since x-axis is a parallel to itself at a distance 0 from it, therefore the equation of x-axis is y = 0.
If a line is parallel to x-axis at a distance b and below x-axis, then its equation isy ~-b.
12.5.2 EQUATION OF A LINE PARALLEL TO 1/-AXIS
Let AB be a line parallel to y-axis and at a distance a from it. Then, the abscissa of every point on

AB is a. So it can be treated as the locus of a point a distance a from y-axis. Thus, if P (x, y) is any
point on AB, then x = ^7. (See Fig. 12.6)
Hence, the equation of a line parallel to y-axis at a distance a from it, is x = a.
Since y-axis is parallel to itself at a distance 0 from it, therefore the equation of y-axis is x = 0.
If a line is parallel to y-axis at a distance a and to the left of y-axis, then its equation is x = -a.
STRAIGHT LINES 12.11

P(x\ y)

X' X

x = a

w
r

Fig. 12.6

Flo
ILLUSTRATIVE EXAMPLES

EXAMPLE 1
Write down the equations of the follozoing lines:

ee
(i) x-axis (ii) if-axis

Fr
(iii) A line parallel to x-axis at a distance of 3 units below x-axis.
(iv) A line parallel to p-axis at a distance of 5 units on the left hand side of it.
for
ur
SOLUTION (i) y = 0 (ii) x = 0 (iii) y = -3 (iv) x = -5.
EXAMPLE 2 Find the equation of a line ivhich is parallel to x-axis and passes through (3, -5).
ks

SOLUTION The equation of a line parallel to .v-axis is i/= b. Since it passes through (3, - 5).
Yo
oo

So, -5 = b ^ b = -5. Hence, the equation of the required line is y = - 5.


eB

ALITHR Since y-coordinate of every point on a line parallel to x-axis is always same, it follows
that the equation of the required line is y = - 5.
r

EXAMPLE 3 Find the equation of a line which is parallel to i/-axis and passes through (-4, 3).
ou
ad

SOLUTION The equation of a line parallel to y-axis is .v = rt. Since, it passes through (-4,3).
So-4 = a ^ a = -4. Hence, the equation of the required line is x = - 4.
Y

AI.ITER Since the abscissa of every point on a line parallel to \/-axis is always same. So, the
Re
nd

equation of the required line is .x = - 4.


Fi

EXAMPLE 4 Find the equation of a line zohich is equidistant from the lines x = - 4 and x = 8.
SOLUTION Since the given lines are both parallel to y-axis and the required line is equidistant
from these lines, so it is also parallel to y-axis and its distance from y-axis is 4 + 8) = 2 units.

Hence, its equation is x = 2.


EXERCISE 12.2

1. Find the equation of the line parallel to x-axis and passing through (3, - 5).
2. Find the equation of the line perpendicular to .x-axis and having intercept - 2 on x-axis.
3. Find the equation of the line parallel to x-axis and having intercept - 2 on y-axis.
4. Draw the lines x = - 3, x = 2, y = - 2, y = 3 and write the coordinates of the vertices of the
square so formed.
12.12 APPLIED MATHEMATICS-XI

5. Find the equations of the straight lines which pass through (4, 3) and are respectively
parallel and perpendicular to the .v-axis.
6. Find the equation of a line which is equidistant from the lines x=-1 and x = 6.
7. Find the equation of a line equidistant from the lines y = 10 and y --1.
ANSWERS

1. y = -5 2. y = -2 3. y=-2 4. (2, 3), (-3, 3), (-3,-2), (2,-2)


5. y = 3, -Y = 4 6. Y = 2 7. y = 4
_HINTS TO SELECTED PROBLEMS
6. Since the given lines are parallel to y-axis and the required line is equidistant from the given

w
lines. Therefore, it is parallel to y-axis at a distance — (- 2 + 6) = 2 units from it. So its
equation is y = 2.

Flo
12.6 DIFFERENT FORMS OF THE EQUATION OF A STRAIGHT LINE

In section 12.1, we have seen that a first degree equation in x, y represents a straight line. The

e
re
equation of a straight line can be written in different forms depending on the data given. In this
section, we shall learn about these forms.

F
12.6.1 SLOPE INTERCEPT FORM OF A LINE
ur
r
THEOREM The equation of a line ivith slope m and making an intercept c on y-axis is y = mx + c.
fo
PROOF Let the given line intersects y-axis at Q and makes an angle 0 with Y-axis. Then,
ks
m = tan 0. Let P (y, y) be any point on the line. Draw PL perpendicular to .r-axis and QM1 PL.
Yo

Clearly, Z MQP - 9, QM = OL = y
oo

y
B
re

P(x,y)
u
ad
Yo

.v-f
d

Q. M
Re
in

Q
F

X' X

Fig. 12.7

and. PM = PL -ML = PL-OQ = y-c.


From triangle PMQ, we have
PM y-<:
tan 0 =
QM X

m
X

y = mx + c, which is the required equation of the line. Q.E.D.


STRAIGHT LiNES 12.13

ri-:mark 1
If the line passes through the origin, then 0 = mO + c ^ c = 0. Therefore, the equation of a
line passing through the origin is y = mx, where tn is the slope of the line.
KI-MAkK2
If the line is parallel to x-axis, then m = 0, therefore the equation of a line parallel to x-axis is
1/ = C.

ILLUSTRATIVE EXAMPLES

EXAMI’LE 1 Fhui the equation of a litre zuith slope-1 and cutting off an intercept of 4 units on negative
direction ofy-axis.
SOLUTION Here, ni = -l and c = -4
Substituting these values in y = mx + c, we obtain that the equation of the line is
y = - X - 4 or, .v +y + 4 = 0

ow
EXAMPLE 2 Find the equation of a straight line which cuts off an intercept of 5 units on negative
direction ofy-axis and makes an angle of 120° zvith the positive direction ofx-nxis.
SOLUTION Here, m - tan 120® = tan (90 + 30°) = - cot 30°= - V3 and c = -5.
Substituting these values in y = mx + c, we obtain that the equation of the line is
y=-V3x-5 or, yfS X + y + 5 = 0

e
re
EXAMPLE 3
inclined to the axes.

Fl
Find the equation of a straight tine cutting off an intercept -Ifrom y-axis and being equally

F
SOLUTION Since the required line is equally inclined with the coordinate axes, therefore it
ur
makes either an angle of 45° or 135° with the x-axis.

r
So, its slope is either m = tan 45° or, J7J = tan 135° i.e. m = l or,-1. It is given that c = -1.
fo
Substituting these values in y = mx + c, we obtain that the equations of the lines are y = x-l and
ks
y =-.Y-l.
Yo
EXAMPLE 4 Find the equation of a straight line which cuts off an intercept of length 3 on y-axis and is
oo

parallel to the line joining the points {3, -2) and (1, 4).
SOLUTION Let m be the slope of the required line. Since the required line is parallel to the line
eB

joining the points A (3, -2) and B (1,4).


4-(-2)
m = Slope of the line AB = = -3
ur

1-3
ad

It is given that c = 3. Substituting these values in y =mx + c, we obtain that the equation of the
Yo

required line is y = - 3.v + 3 or, 3.Y + y-3 = 0.


EXAMPLE 5 Find the equation of a line that has y-intercept 4 and is perpendicular to the line joining
d

(2,-3) and (4, 2).


Re
in

SOLUTION Let m be the slope of the required line. Since the required line is perpendicular to
F

the line joining A (2, -3) and B (4,2).


2+3
X Slope of AB = -l=>??7x 4-2 5

The required line cuts off an intercept of length 4 on y-axis. So, c = 4


Substituting these values in y = mx + c, we obtain that the equation of the required line is
y = - -.v + 4 or, 2.r+5i/-20 = 0.
5
EXAMPLE 6 Find the equation of the straight line which makes angle of 15° with the positive direction of
x-axis and which cuts an intercept of length 4 on the negatwe direction of Y-axis.
SOLUTION Let m be the slope of the line. Then,

777 = tan 15° = tan (45°-30°) =


tan 45° - tan 30
VI , V3-1
1 + tan 45° tan 30°
1. ^ V3 +1
12.14 APPLIED MATHEMATICS-XI

4-2V3
m = 2-yf3
(V3+1) (V3-1) 2

It is given that the line cuts an intercept of length 4 on the negative direction of y-axis.
c = -4

Substituting these values in y = mx + c, we get


1/ = (2-V3) :r - 4 as the equation of the required line.
EXERCISE 12.3
1. Find the equation of a line making an angle of 150° with the x-axis and cutting off an
intercept 2 from y-axis.
2. Find the equation of a straight line:
(i) with slope 2 and y-intercept 3; (ii) with slope -1/3 and y-intercept - 4.
(iii) with slope -2 and intersecting the x-axis at a distance of 3 units to the left of origin.

w
3.
Find the equations of the bisectors of the angles between the coordinate axes.

F lo
4. Find the equation of a line which makes an angle of tan (3) with thex-axis and cuts off an
intercept of 4 units on negative direction of y-axis.
5.
Find the equation of a line that has y-intercept-4 and is parallel to the line joining (2,-5) and

ee
(1,2).

Fr
6.
Find the equation of a line which is perpendicular to the line joining (4,2) and (3,5) and
for
cuts off an intercept of length 3 on y-axis.
7. Find the equation of the perpendicular to the line segment joining (4,3) and (-1,1) if it
r
cuts off an intercept -3 from y-axis.
You

Find the equation of the straight line intersecting y-axis at a distance of 2 units above the
s

8.
ook

origin and making an angle of 30° with the positive direction of the x-axis.
eB

ANSWERS

1. X + aAS y = 2 ^f3
our

2. (i) y = 2 X + 3 (ii) X + 3y + 12 = 0 (iii) 2x + y + 6 = 0


ad

3. x±y = 0 4. y = 3x-4 5. 7x + y + 4 = 0 6. X - 3y + 9 = 0
7. 5x + 2y + 6 = 0 8. X - yfSij + 2V3 = 0
dY
Re

12.6.2 POINT-SLOPE FORM OF A LINE


Fin

THEOREM The equntio7i of a line which passes through the point (X|, y^) and has the slope ‘m’ is
y-yi=m(x-xi).
I'KOOl- Let the line pass through the point Q(xj, i/j) and let P (x, y) be any point on the line.
Then,
y-yi
Slope of the line is =
X-Xj

But, the slope of the line is ni.


y-j/i
=

y-yi =w(x-xi)
X-Xi

Hence, y - yi = m (x - x^) is the required equation of the line. Q.E.D.

ILLUSTRATIVE EXAMPLES

EXAMPLE 1
Find the equation of a line passing through (2,-3) and inclined at an angle of 135° with the
positive direction of x-axis.
STRAIGHT LINES 12.15

SOLUTION Here, m = Slope of the line = tan 135° = tan (90° + 45°) = - cot 45° = -1.
yi ~ ~ ^
So, the equation of the line is ]/ ~ }/\ =fn (.v - Xi)
i.e. ly - (- 3) = -1 (.Y - 2) => y + 3 = - a: + 2 => x + y + 1 = 0.
EXAMPLE 2 Determine the equation of line through the point (- 4, - 3) and parallel to x-axis.
SOLUTION Here, = Slope = 0, X] =-4, - - 3.
So, the equation of the line is y - yj = m (x - x^)
or. y + 3 = 0 (x + 4) => y + 3 = 0.
EXAMPLES Find the equation of the line passing through (1, 2) and making angle of 30° with i/~axis.
SOLUTION The required line makes 30° with the positive direction of y-axis as shown in
Fig. 12.8. So, it makes 60° with the positive direction of x-axis. Therefore, its slope m is given by
m = tan 60°= -J3.
V

w
F lo
30>
(L 2)

e
Fre
60'

X' O X
for
r
You
oks
eBo

Fig. 12.8

Thus, the required line passes through (1, 2) and has J~3 as its slope.
ad
our

Hence, its equation is


y-2 = M'^(x-l) or ^/^3x-y^-2-^/3 =0
EXAMPLE 4 Find the equation of the perpendicular bisector of the line segment joining the points
Re
dY

A {2, 3) atidB{6,-5).
Fin

SOLUTION The slope of AB is given by


-5-3 y2~yi
m = 2 Using: w =
6-2 X2 - x^
1
Slope of a line perpendicular to AB =- — 2

Let P be the mid-point of AB. Then, the coordinates of P are \ , i.e. (4,-1).
2 2 )
1
Thus, the required line passes through P (4, -1) and has slope -. So its equation is
1
y + 1 = - (x - 4) [Using:y-yi=»j(x-xi)]

X - 2y - 6 = 0.
12.16 APPLIED MATHEMATICS-XI

1
l-XAMPLl; 5 Find the equation of the line for which tan 0 = —, where G is the inclination of the line and
3
(i) x-intercept equal to 4. (ii) y-intercept is -

SOLUTION (i) Clearly, the line passes through (4, 0) and has slope = .

So, the equation of the line is


1/-0 = i (,v-4) Putting = 4, yj = 0 and
1
— in y - i/j = in (a: - .Yj)

-T - 2y - 4 = 0
3^
/
1
(ii) The line passes through 0,-— and has slope = —.
V 2 J 2

So, it equation is
( 3 1 3 . 1 ■
y- -
^(Y-0) Putting = 0, 1/1 = --- and m= - \
- iri y-l/l =w(.v-Yi)

w
2J

F lo
2y + 3 = Y or, y - 2y - 3 = 0
LXAMPLL6 The perpendicular from the origin to a line meets it at the point {- 2, 9), find the equation of
the line.

SOLUTION We have,

e
Fre
Y
9-0 9
ni] = Slope of OP =
-2-0 2
for
Let m be the slope of the line AB. Then, B
Slope of AB X Slope of OP = -1
r
9
P{-2, 9)
You

mx — 1
oks

2
eBo

2
m
X' /A 0(0,0) X
9

The equation of AB is
ad
our

y-9 =
I lx-(-2)1 Y’

9y - 81 = 2y + 4 Fig. 12.9
Re

2Y-9y + 85 = 0
dY

[EXAMPLE?
_ 7 Fhid the equation of the line passing through the point (0,2) making an angle — with the
Fin

3
positive x-axis. Also, find the equation of line parallel to it and crossing the y-axis at a distance of 2 units
below the origin,
ly
SOLUTION The equation of the line passing through (0, 2) and making an angle — with the
3
positive Y-axis is
2n
y - 2 = tan -j- (y - 0) [Using: y-1/1 =»j(y-Yi)]
y - 2 = - yfSx ^ -fSx + y - 2 = 0
A line parallel to this line crosses y-axis at a distance of 2 units below the origin. So, it passes

through (0, - 2) and makes an angle ^ with the Y-axis. Hence, its equation is
2jt
y + 2 = tan — (y - 0) => y + 2 = - V^y ■v/3 Y + y + 2 = 0
STRAIGHT LINES 12.17

EXAMPLES Find the equation of the line passing through {- 3,5) and perpendicular to the line through
the points (2,5) and (- 3, 6).
6-5 _-l
SOLUTION The slope of the line passing through (2, 5) and (- 3, 6) is -3-2~ 5

Let in be the slope of the line perpendicular to the line passing through (2,5) and (-3, 6). Then,
m X —1 = -11 => m = 5r
5

The required line passes through (- 3,5) and has slope in = 5. So, its equation is
1/ - 5 = 5 (x + 3) or, 5-Y - y + 20 = 0
EXAMPLE 9 A line perpendicular to the line segment joining the points{l, 0) and (2, 3) divides it in the
ratio 1: n. Find the equation of the line.
3-0
= 3 and the coordinates of
SOLUTION The slope of the line joining A (1, 0) and B (2, 3) is

w
2-1

M + 2 3
the point dividing it in the ratio 1: n are — . The slope of the line perpendicular to the

F lo
n + 1 ' n+1)
1
line segment AB is - —.

ee
Fr
Hence, the equation of the required line is
3 1 n + 2
y- X - or, (n +1) X + 3 {n + 1) y = + 11
for
JI + 1 3 M + l
ur
EXAMPLE 10 Find the equation of a line which divides the join ofd, 0)and (3,0) in the ratio 2 :1 and
s

perpendicular to it.
ook
Yo

SOLUTION LetC be the point which divides the join oiA (1,0) and B (3, 0) in the ratio 2:1. Then,
the coordinates of C are
eB

2x 3 + 1x1 2x0+lx0 7
- , 0
2 + 1 2 + 1 3
our
ad

Since AB is along x-axis, therefore a line perpendicular to AB is parallel to y-axis. As it passes


through C (7/ 3, 0), therefore its equation is
Y
Re

X =— ^ 3x = 7
nd

3
Fi

Hence, the equation of the required line is 3 x = 7.


EXAMPLE 11 The vertices of a triangle are A (10, 4), B (- 4, 9) and C (- 2, -1). Find the equation of its
altitudes. Also, ifnd its orthocentre.
SOLUTION Let AD, BE and CF be three altitudes of A ABC.
Clearly, AD 1 BC, BE ±CA and CF 1 AB.
We have.
1-9
= -5
Slope of BC = -2 + 4

Slope of AD [v AD ± BC]

Since AD passes through A (10, 4). Therefore, equation of (-2, -1)


AD is
12.18 APPLIED MATHEMATICS-XI

1
y - 4 = 5- (.v-10) => a: - 5y + 10 = 0 ...(i)

Slope of AC =
4+1 _
10+2 ~ 12

SlopeofB£=- — [v BEIAC]
^ 5
Clearly, BE passes through B (- 4,9) and has slope -12/5. So, the equation of BE is
y-9=-^(.t
5
+ 4)=>12x + 5y + 3 = 0 ...(ii)
4-9
Slope of AB - = - — ^ Slope of CF = — [●●● CF 1 AB]

ow
10+4 14 ^ 5

Clearly, CF passes through C (- 2, - 1) and has slope So, the equation of CF is


14
y + 1 = — {x + 2) => 14 .Y - 5 y + 23 = 0

e
...(hi)

re
Thus, the altitudes of A y4BC are

Fl
F
x-5y+ 10 = 0,12x + 5y + 3 = 0 and, 14.y-5y + 23 = 0.
ur
The orthocentre of A ABC is the point of intersection of its altitudes.

r
Solving (i) and (ii) by cross-multiplication, we get fo
ks
X
Yo
-65 117 65
oo

Hence, the coordinates of the orthocentre are (-1, 9/5).


eB

nXAMPLE12
Find the equations of the altitudes of the triatigle xuhose vertices are A (7, -1), B (- 2, 8)
andC (1, 2).
ur

SOLUTION Let AD, BE and CF be three altitudes of triangle ABC. Let


ad

m2 and m^, be the


Yo

slopes of AD, BE and CF respectively. Then,


AD 1 BC => Slope of/ID X Slope of BC = -1
d

f 2-8
Re

1
in

=> ni] X
1 + 2
= -l=>„q = -
F

BE-LAC => Slope of BE X Slope of AC = -1


-1 -2
^ m2 X
7-1
= -1 => m2 =2

and, CF 1 AB => Slope of CF x Slope of AB = -1


-1-8
=> X = -l => =1.
7 + 2

Since AD passes through A (7, -1) and has slope


So, its equation is
y+1 " ^ ar-2y-9 = 0

Similarly, equation of BE is
STRAIGHT LINES 12.19

y-8 = 2{x + 2) => 2.r-y + 12 = 0


Equation of Cf is y-2 = 1 (.v-1) ^ .v-y + 1 = 0
EXAMPLE 13 The mid-points of the sides of a triangle are {2,1), {-5,7) and (-5,-5). Find the
equations of the sides of the triangle.
SOLUTION Let D(2,l),£(-5,7) and f(-5,-5) be the
mid-points of sides BC, CA and AB respectively of A ABC.
We know that the line joining the mid-points of two sides of a
triangle is parallel to the third two side.
DE\\AB, EF\\BC and DF\\AC
Slope of = Slope of DE
Slope of BC = Slope of £f and, Slope of AC = Slope of Df
Let nil, '”2 '”3 slopes of AB, BC and CA respectively.

w
Fig. 12.12
Then,
7 -1 -6
nil ~ Slope of AB = Slope of DE -

Flo
-5-2 7

7+5
ni2 = Slope of BC = Slope of EF = (Undefined)

ee
-5+5

Fr
1+5 6
j»3 = Slope of CA = Slope of DF = 2+5 7

Side AB passes through F (-5, -5) and has slope nq for


—-. So, its equation is
ur
=
7

y +5 =
■^(a: + 5)=> 6.v + 7y+65 = 0
ks
Yo

Side BC is parallel to Y-axis and passes through D (2,1). So, its equation is .t = k. As it passes
oo

through (2,1).
eB

2 = k.
Hence, equation of BC is .x = 2.
Side CA passes through £ (- 5,7) and has slope = -7 . So, its equation is
r
ou
ad

y-7 =
|(x + 5) 6.v-7y+79 = 0
Y

EXAMPLE 14 Find the equation of the perpendicular bisector of the line segment joining the points {1,1)
nd
Re

and (2, 3).


SOLUTION Let P be the mid-point of the line segment joining points A (1,1) and B (2, 3). Then,
Fi

f 3 \
the coordinates of P are — , 2 ,
V2
C
Let m be the slope of perpendicular bisector of AB.
Then,
m X Slope of AB = -1
3-1
m X = -1
2-1
-1
m
2
A (1,1) P(3/2,2) 6(2,3)
Clearly, perpendicular bisector of AB passes
through P — , 2 and has slope m = —. So, its
\2 J 2
equation is D

Fig. 12.13
12.20 APPLIED MATHEMATICS-XI

1 ( 3\
y-2 = .r — or, lx + 4i/ -11 = 0.
2j
i-XAMPLE 15 Show that the perpendicular draxvn from the point (4,1) on the line segment joining
(6,5) and (2, -1) divides it internail]/ in the ratio 8:5.
SOLUTION Suppose perpendicular drawn from P (4,1) on the line joining A (6,5) and B (2, -1)
meets AB at M. Let m be the slope of PM. Then,
PMIAB
P (4,1)
m X Slope of AB = -1
-1 -5
m X = -1
2-6
3
m X 1
2 X 1
2

w
m - -
A (6,5) M
3
2 Fig. 12.14
Clearly, PM passes through P (4,1) and has slope m = —.

Flo
3
So, its equation is

ee
y-1 =-—(.y-4) or, 2.r+3y-ll = 0
3

Fr
Suppose M divides line segment AB in the ratio X: 1. Then, coordinates of M are
'2X+6 -X + 5'
x + 1 ' ;^ + i for
ur
Since M lies on line PM whose equation is 2x + 3y -11 = 0

2 ^2X+ 6^1 . f-X + 5^


s
+ 3 -11 = 0
k
Yo

X, +1 X + 1
oo

8
4X + 12-3?.+ 15-ll?.-ll = 0 => -lOX+16 = 0 => X = -
eB

5
Hence, M divides AB internally in the ratio 8:5.
r
ou
ad

EXERCISE 12.4
Y

Find the equation of the straight line passing through the point (6,2) and having slope - 3.
Find the equation of the straight line passing through (- 2, 3) and inclined at an angle of 45°
■)
Re
nd

with the Y-axis.


Fi

3.
Find the equation of the line passing through (0,0) with slope m.
4.
Find the equation of the line passing through (2, 2^f3) and inclined with .v-axis at an angle
of 75°.

5.
Find the equation of the straight line which passes through the point (1,2) and makes such
3
an angle with the positive direction of Y-axis whose sine is —.

6.
Find the equation of the straight line passing through (3,-2) and making an angle of 60°
with the positive direction of y-axis.
7.
Find the lines through the point (0,2) making angles — and — with the -Y-axis. Also, find the
3 3
lines parallel to them cutting the y-axis at a distance of 2 imits below the origin.
8.
Find the equations of the straight lines which cut off an intercept 5 from the y-axis and are
equally inclined to the axes.
STRAIGHT LINES 12.21

9. Find the equation of the line which intercepts a length 2 on the positive direction of the
.v-axis and is inclined at an angle of 135° with the positive direction of i/-axis.
1(1. Find theequation of thestraightlinewhich divides the join of the points(2, 3)and(-5, 8) in
the ratio 3 : 4 and is also perpendicular to it.
11. Prove that the perpendicular drawn from the point (4, 1) on the join of (2, -1) and (6,5)
divides it in the ratio 5 : 8.

12. Find the equations to the altitudes of the triangle whose angular points are A (2, -2),
B(l,l) and C{-1,0).
13. Find the equation of the right bisector of the line segment joining the points (3, 4) and
(-1,2).

ow
14. Find the equation of the line passing through the point(- 3,5) and perpendicular to the line
joining (2,5) and (- 3, 6).
15. Find the equation of the right bisector of the line segment joining the points A (1, 0) and
6(2, 3).

e
re
Frl
ANSWERS

F
1. 3.y+i/-20 = 0 2. X - y + 5 = 0
3. y=mx 4. (2 +V3).v-y-4 = 0
ou
or
5. 3.r-4y + 5=0 6. X - a/3}/ - 3 - 2-/3 = 0
7. 43x - y + 2 = 0, 4Zx + y - 2 = 0, Vs.r + y + 2 = 0, -J>lx-y-2 - 0
kfs
8. y = A + 5 or A + y=5 9. ,\:-y-2 = 0
12. 2 -A + 1/ - 2 = 0, 3 -a - 2 y -1 = 0, -Y - 3 y + 1 = 0
oo

10. 49 A - 35 y +229=0
13. 2A + y=5 14. 5a - y + 20 = 0 15. A + 3y - 6 = 0
Y
B

H\NTS TO SELECTEDPROBLEMS
re

3. The equation of the line passing through (0, 0) and slope m is y - 0 - ni (a 0) or, y
= mx.
oYu

4. The equation of the required line is


ad

y - 2^/3 = tan 75°(a - 2)


tan 45° + tan 30°
d

>/3 + l
or. y-2V3 = >/3-1 (.^-2) V tan 75° = tan (45°+30°) =
1 - tan 45° tan 30°
in
Re

or. y - 2V3 = (2 + 73) (a - 2)


F

13. The right bisector of the segment joining A (3, 4) and 6 (-1,2) passes through the
mid-pointC (1, 3) of AB and is perpendicular to AB. ^

Let m be the slope of AS. Then, /I (3,4) C(l,3)


m - -1 Fig. 12.15
-1-3 2

So, ni] = Slope of a line perpendicular to AB = - 2.


Hence, the equation of the right bisector of AB is y - 3 = - 2 (a-1) or, 2a + y-5 = 0.
12.6.3 TWO-POINT FORM OF ALINE

THEOREM The equation of a line passing through two points (aj, y^) and (.Y2, y2) is
.v-yi = (a-Ai).
.Y2-.Y1
12.22 APPLIED MATHEMATICS-XI

EEQQF Let m be the slope of the line passing through (atj, y^) and (.^2, y2). Then,
m
_ Hi -y\
^2-^1

So, the equation of the line is


y - yi = m (x - x^) [Using point-slope form]
Substituting the value of m, we obtain
_ 1/2 -yi
y - yi =
Y2 -

This is the required equation of the line in two point form.


Q.T.D.

ILLUSTRATIVE EXAMPLES

ow
EXAMPLE 1
Find the equation of the line joining the points (-1, 3) and (4, - 2).
SOLUTION Here, the two points are(.Yi, y^) =(-l, 3) and (.Y2, y2> =(4, - 2).
So, the equation of the line in two-point form is
3-(-2)

e
y - 3 = (Y + l)=>y-3=-Y-l => x + y- 2 = 0.

Fl
re
-1 -4

F
EXAMPLE 2
Find the equation of the line joining the points {at-^^, 2 at-^) and {at2^, 2 at2).
xi=ati^, iji=2ati, X2=nt2^, \j2=2at2.
ur
SOLUTION Here,

So, the equation of the required line is or


sf
_ 2 at2 -2 at-^
]/ ~ 2 at^
at2~ - at-i^
k
Yo
oo

y - 2at^
B

=> y (f-i + t2) - 2 - 2 l2 = 2 .Y - 2 at^^


re

^ y (^1 ^2) ~ 2 X + 2 t2‘


u

EXAMPLES Find the equations of the medians of the triangle ABC whose vertices are A (2 5)
ad

B{-4,9)andC{-2,~l). ' '


Yo

SOLUTION Let D, £, F be the mid-points of BC, CA and AB respectively. Then, the coordinates
of these points are D (- 3, 4), £ (0, 2) and F (-1,7) respectively. The median AD passes through
d
Re

points A (2,5) and D (- 3, 4).


in

A{2,5)
So, equation of AD is
F

4-5
y - 5 = (y-2)
-3-2
1
=>
y - 5 = -(x-2)

-t - 5 y + 23 = 0
The median BE passes through points B (- 4, 9) and £ (0, 2).
So, equation of median BE is D{-3,4)

2-9 Fig. 12.16


(y-9) = (y + 4) => 7Y + 4y-8 = 0.
0 + 4

Similarly, the equation of the median CF is


7+1
(y + 1) = (y + 2) => 8 Y - y + 15 - 0.
-1 + 2
12.23
STRAIGHT LINES

EXAMPLE 4 In ivhnt ratio is the line joining the points (2,3) and (4,1) divides the segment joining the
points (1, 2) and (4, 3)?
SOLLITION The equation of the line joining the points (2,3) and (4,1) is
1-3
y-3 = (.r-2)=>y-3 = -x + 2=> :c + y-5 = 0 -(i)
4-2

Suppose the line joining (2,3) and (4,1) divides the segment joining (1,2) and (4,3) at point P iii

w
'4X+1 3X + 2'
the ratio X: 1. Then, the coordinates of P are
X + 1 X + 1

Clearly P lies on line (i).

e
4X+1 3X+2
+ 5=0 => X = 1.

ro
re
X + 1 X + l

Hence, the required ratio is X: 1 i.e., 1:1.

F
In lohat ratio, the line joining (-1,1) and (5,7) is divided by the line x + y = 4 ?

Fl
EXAMPLES

SOLUTION

u
Suppose the line x + y = 4 divides the join of
5X + 1 7X + r
(-1,1) and B (5,7) in the ratio X;l.
. It lies on X + y = 4.

sr
The coordinates of the point of division are X +1 ' X+1

ko
o
5X-1 7X + 1
+ = 4 => 5X-1 +7X+1 = 4(X + 1) => 12X = 4X+4 => 8X = 4
of => X = ~
X + 1 X + 1
o
Hence, the required ratio are 1:2.
Y

EXAMPLE 6 Prove that the points (5,1),(I,— V and (’31,4) are collinear. Also ,find the equation of the
erB

straight line on zvhich these points lie.


uY

SOLUTION Let the given points be A (5,1), B (1, -1) andC (11, 4). Then, the equaHon of the line
passing through A and B is
1 -1
ad
do

1/ - 1 (x-5) =>x-2y-3 = 0
1-5

Clearly, point C (11, 4) saUsfies the equation x - 2 y - 3 = 0. Hence, the given points lie on the
in

same straight line, whose equation is x - 2 y - 3 = 0.


Re

Find the equation of the internal bisector of angle BAC of the triangle ABC whose vertices
F

EXAMPLE?

A, B, C are (5, 2), (2, 3) and (6,5) respectively.


SOLUTION We have,

AB = J(5-2f+(2-3f = VTO
and. AC = ^(5-6)^+(2-5)- = Vio
AB-.AC = VlO :ViO - 1:1
The internal bisectorAD of Z BAC dividesBC in the ratio AB: AC
i.e. 1 :1. So, coordinatesof D are
(2^ 3+5 = (4, 4).
2 ' 2

4-2
Equation of AD is y - 2 = (x - 5) or, 2x + y -12 = 0
4-5
12.24
APPLIED MATHEMATICS-XI

EXERCISE 12.5
1. Find the equation of the straight lines passing through the following pair of points:
(i) (0,0) and (2,-2) (ii) (a, b) and {a + c sin a,b + c cos a)
(iii) (0, - a) and {b, 0) (iv) {a, b) and {a + b, a-b)
(v) {at-i,a/t^) and{at2, a/tj) (vi) {a cos a, a sin a) and {a cos P, a sin P)
2. Find the equations to the sides of the triangles the coordinates of whose angular points are
respectively: (i) (1,4), (2, - 3) and (-1, - 2)(ii) (0,1), (2, 0) and (-1, - 2).
3. Find the equations of the medians of a triangle, the coordinates of whose vertices are
(-1,6),(- 3,-9) and (5,-8).
4. Find the equations to the diagonals of the rectangle the equations of whose sides are
X = a, X = a', 1/ = b and \j =b'.
5. Find the equation of the side BC of the triangle ABC whose vertices are/1 (-1,-2), 6 (0,1)
and C (2, 0) respectively. Also, find the equation of the median through A (-1, - 2).

w
6. By using the concept of equation of a line, prove that the three points (- 2, - 2), (8, 2) and
(3, 0) are collinear.
7. Prove that the line y - x + 2 = 0 divides the join of points (3,-1) and (8, 9) in the ratio 2:3.

F lo
8. Find the equation to the straight line which bisects the distance between the points
(^7, b), {a',b') and also bisects the distance between the points(- a, b) and {a',~b').

ee
9. In what ratio is the line joining the points (2, 3) and (4, -5) divided by the line passing

Fr
through the points (6,8) and (- 3, - 2).
10. The vertices of a quadrilateral are A (-2, 6), B (1, 2), C (10, 4) and D(7,8). Find the
equations of its diagonals. for
ur
ANSWERS
1. (i)y=-x (ii) y-1) =cot a (x -«?) (iii) ax - by = nb
s
ook

(iv) (a -2 b) X - by + b^ + lab-a~ =0
Yo

(v) t-^t2y +x = a(t-i +t2)


g + p' ■ ^ct + p a -p
eB

(vi) X cos + ysm = flCOS


, 2 , 2

2. (i) x+3y + 7=0,y-3x=l,y + 7x=ll (ii) 2 X - 3 y = 4, y - 3 X = 1, x + 2 i/ = 2.


r
ad
ou

3. 29 X + 4 y + 5 = 0, 8 X -5 y - 21 = 0,13 X + 14 y + 47 = 0
4. y(a' -a) -x{b' -b)-a’b -ab',y{a' -a) + x {b' -b)=a' b' -ab
Y

5. X + 2y -2=0, Median: 5x - 4y - 3 = 0
Re

6. 2 ay-2 b'X =nb - a'b' 9. 5:97


nd

10. X + 6y - 34 = 0, X -y +1 = 0
Fi

HINTS TO SELECTED PROBLEMS


6. The equation of the line passing through points (- 2, - 2) and (8, 2) is
2 +2
y+2 = (x + 2) or, 2x -5y -6 = 0
8+2

Clearly, (3, 0) satisfies this equation which means that the line passing through (- 2, - 2)
and (8, 2) also passes through (3, 0). Hence, these points are collinear.
9. The equation of the line passing through (6, 8) and (- 3, - 2) is
8 +2
]/ + 2 = (x+3) or, 10x-9y + 12 = 0 ...(i)
6+3

Suppose this line divides the line segment joining (2,3) and (4, 5) in the ratio X: 1, then the
point of division
(4X+2 , -5X+ 3],lies on (i).
t X+1 X+l I
12.25
STRAIGHT LINES

4X+2 r-5?.+ 3A + 12 = 0
10 9
^ +1 X +1

40X + 20 + 45X-27+12X. + 12 = 0 97?. + 5 = 0 ^ = -^-


Hence, the required ratio is 5 :97 externally.
12.6.4 THE INTERCEPT FORM OF A LINE

The equation of a line which cuts off intercepts a and b respectively from the x and y
-axes IS
THEOREM

X y
- + ^ = .1.
a b

I’KOOF Let AB be the line which cuts off intercepts OA =fl and OB -b on the x and y axes
respectively. Let P {x, y) be any point on the line. Draw PL 1 OX. Then, 01 = x and PL = y.

w
y

\B
F lo
e
.X
y) Fre
M
for
y
X
r
X'
You
oks

O L
a
eBo

Fig. 12.18
ad
our

Clearly,
Area of A OAB = Area of A OP A + Area of A OPB
-OAOB = -2 OA-PL + \oB-PM
Re
dY

2 2
Fin

1
-ab = — ay + —
2 2 ^ 2
ab = ay + bx
y =1
x
+
a b

This is the equation of the line in the intercept form. Q.E.D.

ILLUSTRATIVE EXAMPLES

EXAMPLE 1 Find the equation of the line which cuts off an intercept 4 on the positive direction ofx-axis
and an intercept 3 on the negative direction of y-axis.
SOLUTION Here a = 4, b = — 3. So, the equation of the line is
12.26
APPLIED MATHEMATICS-XI

- + ft =1 or, ^4 + i
X

a -3
= l or, 3.v-4v=12.
^

EXAMPLE 2 Find the equation of the straight line which makes equal intercepts on the axes and passes
through the point (2,5). r t
SOLLITION Let the equation of the line be ~ + -y =1- Since it makes equal intercepts on the
a

coordinate axes, therefore a = b. So, the equation of the line IS


i
- + = 1 or, -A + 1/ = a
a a

This passes through the point (2, 3).


2 + 3 = => =5.

ow
Thus, the equation of the required line is r + y =5. [Putting <7 =5 in (i)]
EXAMPLE 3 Find the equation of the line which cuts off equal and positive intercepts from the axes and
pmsses through the point (a, p).

e
y
SOLUTION Let the equation of the line be —
+ ^ -1 which cuts off intercepts a and b with the

Fl
re
a

coordinate axes. It is given that a = b. Therefore, the equation of the line is

F
X y ,
— + = 1 => X + y = a
ur
a a

It is given that the line (i) passes through (a, P). or


sf
a + p = <7.
k
Yo

Putting the value of i7 in (i), we obtain the equation of the line as .Y + y = a + p.


oo

EXAMPLE 4 Find the equation of a straight line which passes through the point (4, 2) and lohose
B

intercept on y-axis is twice that on x-axis.


re

SOLUTION Let the equation of the line be


u
ad

a b ...(i)
Yo

It is given that its y-intercept is twice the .v-intercept.


d

b=2a
Re
in

Putting/? = 2(7 in (i), we get


F

X V
-a + ^
2a
= 1 or, 2.Y +y●' = 2a ...(ii)

It passes through the point (4, - 2). Therefore, putting ,y = 4, y = -2 in (ii), we get
8-2 =2^7 (7 = 3.

Substituting (7 = 3 in (ii), we get


2.V + y = 6 as the required equation of the line.
EXAMPLE 5 Find the equation of the straight line -whose intercepts on X-axis and Y-axis are respectively
twice mid thrice of those by the line 3x + 4i/ == 12. ^
SOLUTION The equation of the given line is 3x + 4y = 12. This can be written as - + ^ = 1.
4 3
Clearly, its intercepts on X and Y-axes are 4 and 3 respectively.
STRAIGHT LINES 12.27

A.'-intercept of the required line = 2x4 = 8 and, y-intercept of the required line =3x3 = 9

Hence, the equation of the required line is ^+ ^ =1 or, 9-Y + 8i/=72.


EXAMPLE 6 Find the equation of the line through (2, 3) so that the segment of the line intercepted
between the axes is bisected at this point.
X 1/
SOLUTION Let the equation of the line be -a + ^=1
b
which meets the x and y axes at
A {a, 0) and B (0, b) respectively. The coordinates of the mid-point of AB are (a/2, b/2). It is
given that the point (2,3) bisects AB.
a
= 2 and — = 3 ^ rt = 4 and b = 6.
2 2

Puting (7 = 4 and /;=6in — + ^=1, we obtain


-4 + ^6 = 1 or, 3x + 2i/=12

w
Hence, the equation of the required line is 3x + 2y = 12.

F lo
EXAMPLE 7 If the intercept of a line between the coordinate axes is divided by the point (-5, 4) in the
ratio 1:2, then find the equation of the line.

e
Fre
X V
SOLUTION Let the equation of the line be — + — = 1. It meets the coordinate axes at A {a, 0) and
a b
for
B (0, b). It is given that P (-5, 4) divides AB in the ratio 1:2.
r
You
oks
eBo
ad
our

X' X
Re
dY
Fin

Using section formula the coordinates of P are


'1x0+2x(7 1x1j + 2x0' 2a b
1 +2 1 +2 3 ' 3

-5=^,4 = L3
15
b=12.
3 2 '

Substituting the values of a and 1? in - + ^ = 1, we obtain


12.28 APPLIED MATHEMATICS-XI

2a:
+ -^=1 or, -8a: + 5i/ = 60 or, 8a:-5i/+60 = 0 as the equation of the line.
15 12

HXAMFLE 8 A Straight line cuts interceptsfroni the axes of coordinates the sum of whose reciprocals is a
constant. Show that it always passes through a fixed point.

SOLUTION Let the equation of the line be - + ^ = 1


Its intercepts on x and y axes are a and b respectively. It is given that

- + - = Constant =/c (say)


a b

1
—+ — = 1
ka kb

w
1 1

i+i = 1

F lo
a b

'11 X y
—, — satisfies the equation — + — = 1
k kJ a b

ee
Fr
fl
Hence, line (i) passes through the fixed point - , -
k k/ for
EX.AMPLE 9 Find the equation of the line which passes through the point (3, 4) and the sum of its
ur
intercepts on the axes is 14.
s

SOLUTION Let the equation of the line be — + ^ = 1


ook
Yo
eB

This passes through (3, 4).


3
+ i=l -(ii)
our
ad

a b

It is given that a + b =14


Y

b = 14-a
Re
nd

Putting b =14 - a in (ii), we get


Fi

3
— +
a
—-—
14- a
=1=> 3{\4-a) + 4a = a{14-a)=^ fl^-13fi+42=0=> {a ~7){a - €) = ^3^ a =7, 6

When a=7,b=14~a => Ij=14-7=7 and for <7 = 6, b=14-a =>l?=14-6 = 8.

Thus, we obtain

a=7,b=7 or, a = 6,b=8

Putting the values of a and b in (i), we obtain that the equations of the lines are

— + — =1 and — + — =1 or, x + y =7 and 4 x + 3 y = 24.


7 7 6 8

EXAMPLE 10 Find the equations of the lines which cut-off intercepts on the axes xvhose sum and product
are 1 and - 6 respectively.
STRAIGHT LINES 12.29

SOLUTION Let the equation of the line be


X 1/ -
- + ^ = 1
b

Clearly, it cuts off intercepts a and b on x and y-axes respectively. It is given that
a +b =\ and nb =-6

(a-b)'^ = {a+ b)^-4ab ^ {a-b)'^ =l -6 =25 =^a-b=±5


Solving a + b = I and n -b =5, we get: a = 3 and b = - 2
Solving a + b = 1 and a - b = -5, we get: a = -2 and b = 3.
Substituting these values in (i), we obtain the equations of the required line as
=1 and - —2 + .^3 = 1 or, 2.V - 3y - 6 = 0 and - 3a: + 2y - 6 = 0

w
3 2

1. Find the equation to the straight line


F lo EXERCISE 12.6

e
(i) cutting off intercepts 3 and 2 from the axes,
(ii) cutting off intercepts - 5 and 6 from the axes.
Fre
for
2. Find the equation of the straight line which passes through (1,-2) and cuts off equal
intercepts on the axes
r
3. Find the equation to the straight line which passes through the point (5, 6) and has
You
oks

intercepts on the axes


eBo

(i) equal in magnitude and both positive, (ii) equal in magnitude but opposite in sign.
4. For what values of a and b the intercepts cut off on the coordinate axes by the line
ad

nx + bi/ +8 = 0 are equal in length but opposite in signs to those cut off by the line
our

2.V - 3y + 6 = 0 on the axes.


■ Find the equation to the straight line which cuts off equal positive intercepts on the axes
and their product is 25.
Re
dY

(j. Find the equation of the line which passes through the point (- 4, 3) and the portion of the
Fin

line intercepted between the axes is divided internally in the ratio 5 : 3 by this point.
7. A straight line passes through the point (a, P) and this point bisects the portion of the line
y =1.
intercepted between the axes. Show that the equation of the straight line is —
2a
+
2P

8. Find the equation of the line which passes through the point (3, 4) and is such that the
portion of it intercepted between the axes is divided by the point in the ratio 2 : 3.
9. Point R (h, k) divides a line segment between the axes in the ratio 1:2. Find the equation of
the line.

10. Find the equation of the straight line which passes through the point (- 3, 8) and cuts off
positive intercepts on the coordinate axes whose sum is 7.
11. Find the equation to the straight line which passes through the point (-4,3) and is such that
the portion of it between the axes is divided by the point in the ratio 5 : 3.
12.30 APPLIED MATHEMATICS-XI

ANSWERS

1. (i) 2 .Y + 3 1/ =6 (ii) - 6 Y + 5 y = 30 2. Y + y = -l
3.{i) -t + y =11 (ii) Y-y = -l
4. a = 5. Y + 1/ =5 6. 9Y-20y + 96 =0 8. 2y + i/=10
3'
9. 2kx + hy = 2>hk 10. 4Y+ 3y =12 11. 9 Y - 20 y + 96 = 0
HINTS TO SELECTED PROBLEMS

ow
2. The equation of a line cutting off equal intercepts 'n' on the coordinate axes is
^ y 1
- + — = 1 or, Y + y = fl
a a
-(i)

If it passes through (1,-2), then 1-2 = rt => a = -\.


Substituting rt = 1 in (i), we get y + y = -1 as the equation of the line.

e
re
7. Let the equation of the line be
Y
y

Frl
— + =1 ...(i)

F
a b

This line cuts the coordinate axes at A {a, 0) and B (0, b). It is given that (a, p) bisects the
segment AB.
ou
r
Q + b
=> a=2 a, & =2P
so
a =
2 2
kf
Substituting these values in (i), we get
oo
Y 1/
+ =1
2a 2P
Y
eB

9. Let the equation of the line be


...(i)
a b
ur
oY

It cuts the axes at A {a, 0) and B (0, b).


ad

It is given that the point R {h, k) divides segment AB in the ratio 1 : 2.


2n + 0 0 + b
b =
d

and k = => a = — b = 3k
3 3 2 '
in

2y

+ ^ = 3 or, 2kx + hy = 3hk as the equation


Re

Substituting these values in (i), we obtain —


h
F

of the line.

12.6.5 NORMAL FORM OR PERPENDICULAR FORM OF A LINE

THEOREM The equation of the straight line upon which the length of the perpendicularfrom the origin is
p and this perpendicular makes an angle a zvith x-axis is x cos a + y sin a = p.
lltOOF Let the line AB be such that the length of the perpendicular OQ from the origin O to
the line be p and ZXOQ = a. Let P (y, y) be any point on the line. Draw PL 1 OX, LM 1 OQ
and PN1 LM. Then, OL = x and LP = y.
In A OLM, we have
OM
cos a
OL

OM = OL cos a = Y cos a.

In A PNL, we have
PN
sm a
PL
STRAIGHT LINES 12.31

Q 90-a
M
P(x,y)
P.-
N aiy
X' X
t
o L /I
X

low
y

Fig. 12.20

PN = PL sin a = y sin a

ee
MQ = PN = 1/ sin a

F
Fr
Now, y=OQ=OM+ MQ=.v cos a + y sin a
Hence, the equation of the required line is x cos a + y sin a = p.
for
ur
ILLUSTRATIVE EXAMPLES
ks

EXAMPLI-1 Find the equation of the line which is at a distance 3 from the origin and the perpendicular
Yo
oo

from the origin to the line makes an angle of 30° with the positive direction of the x-axis.
SOLUTION Here, p = 3, a = 30°.
eB

The equation of the line in the normal form is


X cos 30° + 1/ sin 30° = 3 => x — + ^ = 3 => >/3 .v + y = 6.
r
ou
ad

EXAMPLE 2 Find the equation of the straight line on which the length of the perpendicular from the
Y

origin is 4 units and the line makes an angle of 120° with positive direction of x-axis.
SOLUTION It is given that ZXAB = 120°. Therefore, ZAOP = 30°.
Re
nd

Thus, we have
Fi

B
p

120“
30°

0 A X
X'

Fig. 12.21
12.32 APPLIED MATHEMATICS-XI

p = 4 and a = 30°
So, the equation of the line is
A'cosa + i/sin a = p or, .Ycos30°+ysin 30°=4 or, ●v/3.v + y = 8
EXAMPl.r.3 The length of the perpendicular from the origin to a line is 7 and the line makes an angle of
150° with the positive direction ofy-axis. Find the equation of the line.
SOLUTION It is evident from the Figure 12.22 that the perpendicularOQ from the origin on the
line makes 30° angle with x -axis, Therefore, a = 30° It is given that OQ = 7. Therefore, p = 7.

150*

w
30‘

60°
F lo Q

e
X'
>\'30° X

Fre
o A
for
r
Y'
You
oks

Fig. 12.22

So, the equation of the required line is


eBo

.rcosa + ysin a=p or. A' cos 30° -i- y sin 30° = 7
yf3x + ^y = „7 => V3
nr A- + 1/
our
ad

= 14.
2 2

EX.4MPLL4
Find the equation of the straight line upon which the length of perpendicular from origi)i is
3V2 units and this perpendicular makes an angle of 75° with the positive direction ofx-axis.
dY
Re

SOLUTION Let OL be the perpendicular from the origin on tlie required line. It is given that
Fin

OL = 3V2 and ZXOL = 75° i.e. p = 3^2 and a = 75°.


STRAIGHT LINES 12.33

So, the equation of the line is


.rcosa + ysina =p or, .r cos75° + y sin 75® = 3^2 ...(i)

r>/3+i
or.
2>/2
A- +
2V2
y = 3V2 ■■■ cos75°=
2V2
and sin 75° =
2V2

or. -1) A + (Vs + 1) y = 12, which is the required equation.


EXAMPLE 6 A line forms a triangle of area 54a/3 square units with the coordinate axes. Find the
equation of the line if the perpendicular drawn from the origin to the line makes an angle of 60°iyff/i the
X-axis.

SOLLJTION Let AB be the given line and OL = y be the perpendicular drawn from the origin on
the line.

w
It is given that the perpendicular OL makes 60° angle with x-axis. Therefore, a = 60°.

F lo
ee
Fr
for
ur
s
ook
Yo
eB
r

Thus, the equation of the line AB is


ou
ad

X cos a + y sin a = p or, .v cos 60° + y sin 60° = p


V3y
Y

or. A + VSy = 2p or, — + =1


^ 2p 2p
Re
nd

2p
This, cuts the coordinates axes at A and B such that OA = 2p and OB - V3'
Fi

It is given that area of AOAB is 54-V3 sq. units.


-xOAxOB = 54V3
2
1 2p
=> -X 2px = 54V3 => = 81 => p = 9
2 ^ V3
Substituting p = 9 in (i), we get a + VSy = 18 as the equation of the required line.
EXERCISE 12.7

1. Find the equation of a line for which


(i) p = 5, a = 60° (ii) p = 4, a = 150° (iii) p = 8, a = 225° (iv) p = 8, a = 300°
2. Find the equation of the line on which the length of the perpendicular segment from the
origin to the line is 4 and the inclination of the perpendicular segment with the positive
direction of A-axis is 30°.
12.34 APPLIED MATHEMATICS-XI

3.
Find the equation of the line whose perpendicular distance from the origin is 4 units and
the angle which the normal makes with the positive direction of .Y-axis is 15®.
4.
Find the equation of the straight line at a distance of 3 units from the origin such that the
perpendicular from the origin to the line makes an angle a given by tan a = — with the
positive direction of Y-axis.
D.
Find the equation of the straight line on which the length of the perpendicular from the
origin is 2 and the perpendicular makes an angle a with .r-axis such that sin a = —.

ow
3
6.
Find the equation of the straight line upon which the length of the perpendicular from the
5
origin is 2 and the slope of this perpendicular is — .
12

e
ANSWERS

re
1. (i).v +V3 _i/=10 (ii) ~-^f3 X + y = 8 (iii) y + y + 8 v"2 (iv) x-yf3\j=16
2. ■'J3x + 1/ = 8 3. (V3+l)Y + (y3-l)y-8V2 4. 12Y+5y = 39

Frl
F
5. 2 V2 Y + y = 6 6. 12y + 5y ± 26 = 0
ou
12.7 TRANSFORMATION OF GENERAL EQUATION IN DIFFERENT STANDARD FORMS

sor
The general equation of a straight line is Ax ■+ By + C —0 which can be transformed to various
standard forms as discussed below, kf
(i) Transformation of Ax + By + C = 0 in the slope intercept form (y = mx + c):
oo
The equation of the line is
Y

Ax + By + C = 0 => By = - Ax -C => y = — B
A
X + '_C'i
B

V B
A C
This is of the form y =mx + c, where m = - and, c = -
re

B B
oY
u

Thus, for the straight line Ax + By + C =0, we have


ad

/I Coefficient of x C Constant term


m = Slope = and. Intercept on y-axis =
6
Coefficient of y
d

B Coefficient of y
__ Coefficient of x
in

note To determine the slope of a line by the formula ,we must first transfer all
Coefficient of y
Re

terms in the equation on one side.


F

(ii) Transformation of Ax + By + C =0 in intercept form f - + ^b = 1


\a
The equation of the line is
Ay By X
Ax + By + C = 0 ^ Ax + By = -C => + +
-c -c c^
A B

This is of the form — + = 1.


a b

Thus, for the straight line Ax +By + C = 0, we have


C Constant term C Constant term
Intercept on Y-axis = - , Intercept on y-axis = —
A Coefficient of y B Coefficient of y
NQIE As discussed above the intercepts made by a line with the coordinate axes can be determined by
reducing its equation to intercept form. We may also use thefollozoing method to determine the intercepts
on the coordinate axes:
STRAIGHT LINES 12.35

For intercept on x-nxis : Put y =0 in the equation of the line and find the value of x. Similarly to find
y-intercept, put x = 0 in the equation of the line and find the value ofy.
ILLUSTRATIVE EXAMPLES

EXAMPLE 1
Transform the equation of the line VS x + y - 8 = 0 to (i) slope intercept form and find its
slope and y-intercepf (ii) intercept form and find intercepts on the coordinate axes.
SOLUTION (i) We have,
-JSx + y- 8 = 0 => y = -J3x + 8, which is the slope intercept form of the given line.
Slope = -Vs, and i/-intercept = 8
(ii) We have.

ow
y
yfd X + y 8 = 0 =>
8/VS
+
8
= 1, which is the intercept form of the given line.

So, -V-intercept = and, y-intercept = 8


Vs

e
re
EXAMPLE 2 Find the values ofk forxvhich the line {k - 3) x -{4-k^) y + k^ -7k + 6 = Ois

rFl
F
(i) parallel to the x~axis. (ii) parallel to the y-axis. (iii) passing through the origin.
SOLUTION Let m be the slope of the line

r
{k-3)x-(4-k^)y + k^ -7k + 6 = 0
ou
-(i)
fo
ks
Then,
(k-3) k-3
oo

m
4-Jc^
Y
B

(i) If the line is parallel to .v-axis, then


k-3
re

Slope = 0 ^ = 0=>fc-S = 0=>Jc = S


4-k^
ou
Y
ad

(ii) If the line is parallel to y-axis, then


4-fc^
- = 0 => = 0 ^ 4-k^ = 0 => k = ±2
d

m k-3
in
Re

(iii) If the line passes through the origin, then (0,0) must satisfy the equation (i).
F

.-. (k - 3) X 0 -{4 -k^) X 0 + k^ -7k + 6 = 0 => (^ -1) (^ - 6) = 0 => cf=l,6.


EXERCISE 12.8

1. Reduce the equation Vs .r + y + 2 = 0 to:


(i) slope-intercept form and find slope and y-intercept;
(ii) intercept form and find intercept on the axes;
2. Put the equation -a + —
b
= 1 to the slope intercept form and find its slope and y-intercept.

3. Reduce the equation 3 .r - 2 y + 6 = 0 to the intercept form and find the .x and y intercepts.
ANSWERS

1. (i) Slope = - Vs, y-intercept = - 2 (ii) x-intercept = — y-intercept = - 2

2. Slope = --, y-intercept = b 3. x-intercepf =- 2, y-intercept = 3


a
12.36 APPLIED MATHEMATICS-XI

12.8 POINT OF INTERSECTION OF TWO LINES

Let the equations of two lines be


(?i X + y + = 0
and. 02 X + 1/ + C2 = 0 ...(ii)
Suppose these two lines intersect at a point P (xj, y^). Then, (X|, y^) satisfies each of the given
equations.
Xj + ly-j + C] — 0 and, 02 ^2 j/l ^2 ~ ®
Solving these two equations by cross-multiplication, we get
Vi 1 Cl 02 - C2 <7|
=> Xi = >yi =
b-^ C2 ~ ^2 ^1 ^2 ~ ^2 ^2 ~ ^2 rtj &2 “ ^2 (7i &2 - ^^2
Hence, the coordinates of the point of intersection of lines (i) and (ii) are:
1?1 C2 ~ ^2 *"1 ^2 ~^2 ^1
<?1 &2 “ ^2 ^2 ~ ^2 ^1

w
NOTH
To find the coordinates of the point of intersection oftzuo non-parallel lines, xve solve the given

F lo
equations simultaneously and the values ofx and \j so obtained determine the coordinates of the point of
intersection.

ILLUSTRATIVE EXAMPLES

e
EXAMPLE!
Find the coordinates of the point of intersection of the lines 2x-y + 3=0 and

Fre
X + 2 y - 4 = 0. for
SOLUTION Solving the equations 2x-y+3 = 0 and x +2y-4 = 0, simultaneously we obtain
1
X
y X
= X = 1 => X
2 11

4-6 3+8 4+1 -2 n 5 5 ' ^ 5


r
You

Hence, (- 2/5,11/5 is the required point of intersection.


oks

EXAMPLE 2 Find the area of the triangle formed by the lines y = x, y = 2x and y = 3x + 4.
eBo

SOLUTION The given equations are


y =x y = 2x ...(ii) and y = 3x + 4 ...(iii)
ad
our

Suppose the equations (i), (ii) and (iii) represent the sides AB, BC and CA respectively of a
triangle ABC.
Solving (i) and (ii), we get: x = 0 and y = 0. Thus, AB and BC intersect at B (0,0).
Re
dY

Solving (ii) and (iii), we obtain: x = - 4, y = - 8. Thus, BC and CA intersect at C (-4, -8).
Solving (iii) and (i), we get: x = - 2 and y = -2. So, CA and AB intersect at A (-2,- 2).
Fin

Thus, the coordinates of the vertices of the triangle ABC are: A (-2, -2), B (0, 0) and C(-4,-8).
-2 -2 1
Area of A ABC = - 0 0 1 = 4 sq. units.
2
-4 -8 1

EXAMPLES Find the equations of the medians of a triangle formed by the lines x+i/-6=0,
X - 3y - 2 = 0 and 5x - 3y + 2 = 0.
SOLUTION The given equations are :
x + y-6 = 0 ...(i) X - 3y - 2 = 0 ...(ii) and 5x - 3y + 2 = 0 ...(iii)
Suppose equations (i), (ii) and (iii) represent the sides, AB, BC and CA respectively of triangle
ABC.

Solving (i) and (ii), we get: x =5 andy =1. Thus, AB and BC intersect at B (5,1).
STRAIGHT LINES 12.37

Fig. 12.25

Solving (ii) and (iii), we get: x = -1 and y = -1. Thus, BC and CA intersect at C ( 1, 1).
Solving (i) and (iii), we get: x = 2 and i/ = 4. Thus, AB and CA intersect at A (2, 4).
Thus, the coordinates of the vertices A, B and C of triangle ABC are (2, 4), (5, 1) and (-1, -1)

w
respectively. Let D, £ and f be the mid-points of sides BC, CA and AB respecHvely. Then, the
coordinates of D, E and F are

F lo
2 + 5 4 + l'i_r7 5
\ /
r2-l 4-1 1 3^
D = (2,0);£ — and F
2 ' 2 2 ' 2 2 ' 2 '-2'2
respectively.

ee
The median AD passes through A (2, 4) and D(2, 0). So, its equation is

Fr
0-4
y-4 = (x-2) for
2-2
2-2
ur
y-2 = (1/ - 4) => -t - 2 = 0 => .Y = 2
0-4
s

The median BE passes through points 6(5,1) and £(1/2, 3/2). So, its equation isi
ook
Yo

3
-1
eB

(:^-5)
'-5
2
our
ad

1
=> y-l=-.l(x-5) => .r + 9y-14 = 0
The median C£ passes through points C(-l, -1) and £(7/2,5/2). So, its equation is
dY

5.1
Re

y + 1 = I + 1 (y + 1) => y +1 = ^ (y +1) => 7.Y - 9y - 2 = 0


Fin

Hence, the equations of the medians of the triangle are y = 2, .r + 9y -14 - 0 and7Y - 9y - 2 - 0.
EXAMPLE 4 Find the value of in for mhich the lines oty+(2m + 3) y+ i?r + 6 = 0 and
{2m +1) Y + {m -1) y + ?;? - 9 = 0 intersect at a point on y-axis.
SOLUTION The equations of the lines are
mx + {2m + 3) y + PJ + 6 = 0 ●●●(i)
{2m +1) Y + (m -1) y + /» - 9 = 0 ...(ii)

Solving these two equations by cross-multiplication, we obtain 1


Y y t

(2f?r+ 3) (?7J -9) -(m-1) {m + 6) (2m+l) {m + 6)-m (m-9) m(m-l)-(2?w+l) (2»;+ 3)


Y y 1

m^ - 20m - 21 m^ + 22m + 6 - 3 {nP' + 3m +1)


12.38 APPLIED MATHEMATICS-XI

m ^ - 20m - 21 + 22m + 6
.V aiidi/ =
- 3 (m^ + 3m +1) - 3 (m^ + 3m +1)
-20m-21 + 22m + 6
So, given lines intersect at the point
- 3 (m^ + 3m + 1) - 3 (m^ + 3m + 1)
If it lies on y-axis, then its .r-coordinate is zero.
- 20>» - 21
= 0 => m^-20m-21=0 => (m - 21) (m + 1) = 0 => m =-l, 21
- 3 (m^ + 3m + 1)
EXAMPLE 5 Find the area of the triangleformed by the lines y = mi x+ Cj,y =»Z2 x + C2 and .v = 0.
SOLUTION Lety = m-j x + c^, y = m2 x + C2 and x = 0 be the sides AB, BC and CA respectively of
a triangle /IBC. Solving y x + and y =m2 x + C2 as linear equations in x, y, we get
x=^^ ^ y = pq C2 “ ‘^2 *^1
m-i - m2 ni] - m2

w
So, the coordinates of B are ‘-2 “^1 ^2 “^”2 *'1
ni] - m2 nil ~^’^2

F lo
e
Fre
x = 0

qo, C2)
for
M B
r
'f?.
You
oks
eBo

A{Q,c^)
X' O
X
i-
ad
our

■>
^\\
Y'

Fig. 12.26

Solving y = m2 X + C2 and X = 0, we get: x = 0, y = C2. So, coordinates of C are(0, C2).


dY
Re

Similarly, by solving x = 0 and y = /;q x + , we get the coordinates of A as (0, c^).


Fin

0 1

0 ^2 1

.'. Area of A ABC =- ^2-q nil C2 -1«2 Cl


2 1
mi - m2 mi - m2

1 C2 -q
(ci - ^^2) = T
1 (q -^2)^
in magnitude.
2 [mi- ni2 2 m-] -m2
ALITER Given lines are
y = lUi X + Cl ... (i)
y='«2X+C2 - (ii)
X =0
... (iii)
Lines (i) and (ii) intersect line (iii) at A (0, Ci) and C (0, C2) respectively.
STRAIGHT LINES 12.39

^1 C2 -^2 ^1
Solving (i) and (ii), we obtain the coordinates of B as
m-i -/«2 - ni2

^2-^1
BM = :c-coordinate of B =
Wj -Ml2
Clearly, AC =| C2 -c-j | 1
1
AreaofA/\6C = A Base x Height = - (/\C x BM)
1 ^2 1 (C2 -
2 I ^^2 “ I
X

Wj -m2 2 I »i-| - lUj


EXERCISE 12.9

1. Find the point of intersection of the following pairs of lines :

w
(i) 2x-y+ 3 = 0 and x + y - 5 = 0 (ii) hx + ay - ab and ax + by = ab.
a a

(iii) y =nii x + — and y =ni2 x + —.

F lo
ni2

2. Find the coordinates of the vertices of a triangle, the equations of whose sides are:

ee
(i) ,T + y-4 = 0, 2.v-y + 3 = 0 and x-3y + 2 = 0

Fr
(ii) y + f2) = 2 X + 2 fl f2' ■/ (^2 + ^3) - 2 x + 2 n ^2 ^3 ¥ (h + ^i) = 2 -V + 2 nt^
3. Find the area of the triangle formed by the lines
(i) y = ntj .V + c-j, y = m2 x + C2 and x = 0
for
ur
(ii) y = 0, X = 2 and x + 2 y = 3.
(iii) x+i/-6 = 0,x-3y-2 = 0and5x-3y + 2 = 0
s
ok
Yo

4. Find the equations of the medians of a triangle, the equations of whose sides are:
o

3 X + 2 y + 6 = 0,2 X -5 y + 4 = 0 and x-3y-6 = 0


eB

5. Prove that the lines y = V3x + l,y=4 and y = - ^/3 x + 2 form an equilateral triangle.
6. Classify the following pairs of lines as coincident, parallel or intersecting:
r
ad
ou

(i) 2x + y- l= 0 and 3 x + 2 y + 5 = 0 (ii) x - y = 0 and 3 x - 3 y + 5 = 0


(iii) 3x + 2y-4=0 and 6x + 4y-8=0.
Y

7. Find the equation of the line joining the point (3, 5) to the point of intersection of the lines
Re

4 X + y -1 = 0 and 7 x-3y-35=0.
nd

8. Find the equation of the line passing through the point of intersection of the lines
Fi

4^x -7y - 3 = 0 and 2x - 3y +1 = 0 that has equal intercepts on the axes.


ANSWERS

f 2 13 ab lib
(iii)
a
1 h
1 11
(ii) , a
a + b'a + b ^ nh ni-i m2)
1 Ii) (_Z_ i' 5 3] (ii) {nti^, 2 at{), {at2^, 2 atj), («f 3^, 2 at2)
3j'l 5'sJ' 2'2,

3. (i)
(^1 -^2)^ (ii) 0 (iii) 12. sq. units
2 (mj - m2)
4. 41 X-112 y-70 = 0, 16 x-59 y-120 = 0 and 25x-53y + 50 = 0
6. (i) intersecting (ii) parallel (iii) coincident
7. 12x-y-31 = 0 8. X + y +13 = 0
12.40 APPLIED MATHEMATICS-XI

12.9 CONDITION OF CONCURRENCY OF THREE LINES

Three lines are said to be concurrent if they pass through a common point i.e. they meet at a
point.
Thus, if three lines are concurrent the point of intersection of two lines lies on the third line. Let
^1 '-'●’l + y + C-] = 0 ...(i) .V + ]/ ■*■ ^2 “ ^ + ^’3 i/ + ‘-'3 = 0 ...(hi)
be three concurrent lines. Then the point of intersection of (i) and (ii) must lie on the third.
The coordinates of the point of intersection of (i) and (ii) are:
h ^2 ~^2 ^1 Cj <72 ~ ^2 ^1
[See section 12.8 on page 12.36]
V ^2 ~ ^2 ^2 ~ ^2 ^1
This point must lies on line (iii).
/>! t'2 ~b2 C] + 1) C\ ^2 ~^2
a
+ C3 - 0
^ 1^ (7-] 1)2 - <?2

w
(7] b2 ~ ^2 J
^5

<?2 1)2 C2 = 0
F lo
^2 ~ h ^l) + ^3 (^1 ^2 - ^2 ^l) + <^3 (‘’l ^2 “ ^2 ^l) = 0
Cl

e
Fre
^3 ^’3

This is the required condition of concurrency of three lines.


for
ILLUSTRATIVE EXAMPLES
r
EXAMPLE 1 Prozv that the lines 3 .v + 1/ -14 = 0, .v - 2 y = 0 and 3x-8y+4 = 0 are concurrent.
You
oks

SOLUTION Given lines arc 3x + y -14 = 0, .v - 2y + 0 = 0 and 3 .v - 8 1/ + 4 = 0.


eBo

3 1-14

We have, 1 -2 0 = 3 (- 8 + 0) -1 (4 - 0) -14 (- 8 + 6) = - 24 - 4 + 28 = 0.
ad
our

3 4

So, the given lines are concurrent.


EXAMPLE 2
Show that the lines x - y - 6 = 0,4 x - 3 y - 20 = 0 and 6.r + 5 y + 8 = 0 are concurrent.
Re
dY

Also, find their common point of intersection.


SOLUTION The given lines are
Fin

.v-y-6-0 ●●●(i) 4.V - 3y - 20 = 0 ●●■(ii) 6.X' + 5y + 8 = 0 ...(hi)


Solving (i) and (ii) by cross-multiplication, we get
a:
.1/ 1
— => -V = 2 and 1/ = - 4.
20-18 -24 + 20 -3 + 4

Thus, the first two lines intersect at the point (2, - 4). Putting .y = 2 and y = - 4 in (iii), we get
6x2 + 5x-4 + 8 = 0

Thus, the point (2, - 4) lies on line (iii).


Hence, the given lines are concurrent and their common point of intersection is (2,-4).
EXAMPLE 3 Find the value of\ if the lines 3.v - 4y -13 = 0, 8.v - lly - 33 = 0 and 2x-3ij + X = 0
are concurrent.

SOLUTION The given lines are concurrent, if


12.41
STRAIGHT LINES

3 -4 -13

8 -11 -33 = 0
2 -3 X

X-7 = 0=J> X = -7
3 (-11 X-99) + 4(8?.+ 66) -13 (-24 + 22) = 0 =>
,\l irHR The given equations are
8.r - lly - 33 = 0 ...(ii) and, 2x - 3y + ?. = 0 ...(iii)
3x-4y-13 = 0 ...(i)
Solving equations (i) and (ii), we get x =11 and y =5. Thus, (11, 5) is the point of intersection of
lines (i) and (ii). The given lines will be concurrent if they pass through the common point i.e.
the point of intersection of any two lies on the third. Therefore, the point (11, 5) must lie on the

w
line (iii).
2x11-3x5 + ?^ = 0=>X = -7.

F lo
EXERCISE 12.10

ee
1. Prove that the following sets of three lines are concurrent:

Fr
(i) 15 X -18 y + 1 = 0,12 X + 10 y - 3 = 0 and 6 X + 66 y -11 = 0
(ii) 3x-5y-ll = 0,5x+ 3y-7 = 0andx + 2y = 0 for
ur
(iii) - +^=1,-+
b a b
^ =1 andy = .Y.a
s

2. For what value of ?.are the three lines 2x-5y + 3 = 0, 5x-9y + X = 0 and x-2y + l= 0
ok
Yo

concurrent?
o
eB

3. Find the conditions that the straight lines y =m^ x + Ci,y x+ C2 and \j=m^ x + c^ may
meet in a point.
r
ad
ou

ANSWERS

2. >. = 4 3. mi (C2 - C3) + m2 (C3 - Cl) + /»3 (Ci - C2) = 0


Y
Re
nd

12.10 LINES PARALLEL AND PERPENDICULAR TO A GIVEN LINE


Fi

LINE PARALLEL TO A GIVEN LINE


THEOREM 1 Prove that the equation of a line parallel to a given line ax + bij + c==0 isax+by+X = 0,
where is a constant.

PROOF Let m be the slope of the line ax + by + c = 0. Then,


Coeff. of X
a
m Using: m
b Coeff. of y

The required line is parallel to the given line. So, the slope of the required line is also m. Let Ci be the
y-intercept of the required line. Then, its equation IS

y = m X + Cl
a
y = --.v+c,

ax +by-bci =0
where X = -bci= constant. Q.E.D.
AX + by + X = 0,
12.42
APPLIED MATHEMATICS-XI

MUE To write a line parallel to a given line zve keep the expression containing x and y same and simply
replace the given constant by an unknown constant X. The value of X can be determined by some given
condition.

LINE PERPENDICULAR TO A GIVEN LINE

THEOREM 2 Prove that the equation of a line perpendicular to a given line ax +by + c = 0
is bx - ay + X = 0, where X is a constant.
PROOF Let be the slope of the given line and m2 be the slope of a line perpendicular to the
given line. Then, oij = As the lines are perpendicular.
1 b
nil =-l => /«2 = “
nil ^

w
Let C2 be the i/-intercept of the required line. Then, its equation IS
i
y = m2 X + C2
b

Flo
y = - X + C2
a

ee
bx -ay + ac2 =0

Fr
=>
bx - ay + X = 0, where A. = r?C2 = constant. Q.E.D.
To write a line perpendicular to a given line we may use the following algorithm.
for
ur
ALGORITHM

STLl’ 1
Interchange X and y.
s
STI-PII
If the coe^icients of x and y in the given equation are of the same sign make them of opposite
k
Yo

signs and if the coefficients are of opposite signs make them of the same sign.
oo

SIEPm Replace the given constant by a new constant X which is determined by a given condition.
eB

ILLUSTRATIVE EXAMPLES
r

EXAMPLE 1 Find the equation of the line which is parallel to 3x-2y + 5 = 0 and passes through the
ou
ad

point (5,-6).
SOLUTION The equation of any line parallel to the line
Y

3x - 2i/ + 5 = 0 is
3 .Y - 2 y + A. = 0 ...(i)
Re
nd

This passes through (5, - 6).


Fi

3x5-2x-6 + X = 0 => X = - 27.


Putting ?. = - 27 in (i), we obtain 3x - 2y 27 = 0 as the required equation.
Al
JXER The slope of the given line is 3/2. Tlierefore, the slope of the required line is also 3/2.
Since the required line passes through (5, -6), so its equation is
3
y+6=-(Y-5) or, 3x-2y-27 =0 [Using: y-yj =m(Y-.Vi)]
EXAMPLE 2 Find the equation of the straight line that passes through the point (3,4) and perpendicular
to the line 3Y+2y+5=0.
SOLUTION The equation of a line perpendicular to 3y + 2i/ + 5 = 0 is
2x - 3y + X = 0
This passes through the point (3,4).
3x2-3x4+?. = 0=>?t = 6
STRAIGHT LINES 12.43

Putting X = 6 in (i), we obtain 2x-3y + 6= 0 as the required equation.


ALITER The slope of the given line is -3/2. Since the required line is perpendicular to the given
line. So, the slope of the required line is 2/3. As it passes through (3,4). So, its equation is
y-4= —(.y-3) or, 2y - 3y + 6 = 0 [Using:y-i/i =w (.y-a:-,)].
3

EXAMPLES T/hrf the equatioji of the line perpendicular to x-71/+5 = 0 and having x- intercept 3.
SOLUTION The equation of a line perpendicular to .v -7 i/ + 5 = 0 is
7.T +y+ ?. = 0
Its Y-intercept is 3. This means that the line cuts .Y-axis at a distance of 3 imits from the origin.
Consequently, it passes through the point (3,0) on .Y-axis.
21 + 0 + X = 0 =>?. = -21

w
Putting X = - 21 in (i), we obtain 7y + y - 21 = 0 as the equation of the required line.
EXAMPLE 4 Find the coordinates of the foot of the perpendicular drawn from the point (1,- 2) on the line

Flo
y = 2 Y + 1.
P(l,-2)
SOLUTION Let M be the foot of the perpendicular drawn from

ee
P (1, - 2) on the line y=2x + 1. Then, M is the point of

Fr
intersection of y = 2 y +1 and a line passing through P (1, - 2)
and perpendicular to y = 2Y + l. The equation of a line
perpendicular to y = 2 y +1 or, 2 y - y + 1 = 0 is for
ur
Y + 2y + X = 0 ...(i)
This passes through P (1, - 2).
ks
A M B
Yo

l-4+X = 0=>X=3
oo

y = Zv + l
Putting X = 3 in (i), we get Fig. 12.27
B

Y+2y+3=0
re

Point M is the point of intersection of the lines


ou

2.Y-y + l = 0 and y + 2 y + 3 = 0.
ad

Solving these equations by cross-multiplication, we get


Y

=> Y = -l, y = -l.


nd
Re

-5 -5 5

Hence, the coordinates of the foot of the perpendicular are (- 1, - 1).


Fi

EXAMPLES Find the equation of a straight line parallel to2x + 3i/+lJ=0 and which is such that the
sum of its intercepts on the axes is 15.
SOLUTION The equation of a line parallel to 2 y + 3 y + 11 = 0 is
2y+ 3y + X = 0, Xis a constant ...(i)
To find Y-intercept of this line, we put y = 0 in its equation. Putting y = 0 in (i), we get
=> 2y + X = 0=>y--X/2
So, Y-intercept = - X / 2.
To find y-intercept of this line, we put y = 0 in its equation. Putting y = 0 in (i), we get
3i/+ X = 0=^ y= — X/3
So, y-intercept = -X/3.
It is given that the sum of the intercepts of the line (i) on the coordinate axes is 15.
5X
+ = 15 => - = 15 => X = -18
I 2 3j 6
12.44 APPLIED MATHEMATICS-XI

Putting k - -18 in (i), we get: 2x + 3y -18 = 0.


Hence, the equation of the required line is 2x + 3y -18 = 0.
EXAMPLE 6 Find the image of the point (- 8,12) zvith respect to the line mirror 4 .v + 7 y +13 = 0.
SOLUTION Let the image of the point P (- 8,12) in the line mirror /IBbeQ (a, P).Then, the line
segement PQ is perpendicularly bisected at R. So, the coordinates of R are
g-8 0 + 12^
2 '~2 P(-8,12)

As it lies on 4 .v + 7 y + 13 = 0.
2a -16 + + 13 = o=> 4a + 7B + 78 = 0 ...(i)
2
4x + 7\f + 13 == 0
The line segment PQ is perpendicular to AB. R

(Slope of AB) X (Slope of PQ) = - 1

w
=>
4 X
P-12 = -1

F lo
7 a + 8
Q(a,P)
7a-4p + 104 = 0 ●●●(ii) Fig. 12.28
Solving (i) and (ii), we get: a = -16, p = - 2.

e
Fre
Hence, the image of (- 8,12) in the line mirror 4 x + 7 y + 13 = 0 is (- 16, - 2).
EXERCISE 12.11
for
1. Find the equation of a line passing through the point (2, 3) and parallel to the line
r
3A.'-4y + 5=0.
You

2. Find the equation of a line passing through (3, - 2) and perpendicular to the line
s
ook

AT-3y + 5=0
eB

3. Find the equation of the perpendicular bisector of the line joining the points (1,3) and (3,1).
4. Find the equations of the altitudes of a A ABC whose verticesare A (1,4), B (- 3, 2) and
C(-5,-3).
our
ad

5. Find the equation of a line which is perpendicular to the line ^/^ x -1/ + 5 = 0 and which
cuts off an intercept of 4 units with the negative direction of y-axis.
6. If the image of the point (2,1) with respect to a line mirror is (5,2), find the equation of the
dY
Re

mirror.
Fin

7. Find the equation of the straight line perpendicular to 2 .t - 3 y = 5 and cutting off an
intercept 1 on the positive direction of the x-axis.
8. Find the equation of the straight line perpendicular to 5 x - 2 y = 8 and which passes
through the mid-point of the line segment joining (2,3) and (4,5).
9. Find the equation of the straight line which has y-intercept equal to 4/3 and is
perpendicular to 3 x - 4 y 11 =0.
10. Find the coordinates of the foot of the perpendicular from the point (-1, 3) to the line
3x-4y-16=0.
11. Find the image of the point (2,1) with respect to the line mirror x -f i/ -5 = 0.
12. If the image of the point (2,1) with respect to the line mirror be (5,2), find the equation of
the mirror.

13. Find the equation to the straight line parallel to3x-4y + 6 = 0 and passing through the
middle point of the join of points (2, 3) and ( 4, - 1).
STRAIGHT LINES 12.45

14. Prove that the lines 2x-3i/ + l=0, .v+i/ = 3, 2.r-3i/=2 and a: + y = 4 form a
parallelogram.
X

15. Find the equation of a line drawn perpendicular to the line - + ^ = 1 through the point
where it meets the y-axis.
16. The perpendicular from the origin to the line y = nix + c meets it at the point (-1, 2). Find
the values of m and c.

17. Find the equation of the right bisector of the line segment joining the points (3, 4) and
(-1,2).
18. Find the image of the point (3,8) with respect to the line x+ 3\j =7 assuming the line to be a
plane mirror.

w
ANSWERS

1. 3x-4y + 6 = 0 2. 3.r + y-7=0 3. y = x


4. 2x + 5y-22 = 0,6.r + 7y + 4 = 0,2x + y + 13 = 0

Flo
5. X + V3y + 4V3=0 6. 3:r + y-12 = 0

e
8. 2x + 5y-26 = 0 9.4.r+3y-4 = 0

re
7. 3.r + 2y-3 = 0
[68 49 ^

F
10. 11. (4,3) 12. 3x + y=12 13. 3.r-4y=5
25 ' 25 J
ur
r
15. 2x-3y + 18 = 0 16. m =1/2, c =512 17. 2A- + y-5=0 fo 18. (-1,-4)

HINTS TO SELECTED PROBLEMS


ks
Yo

10. Let M (a, P) be the foot of perpendicular from P (-1, 3) on the line 3a: - 4y -16 =0.Tlien,
oo
B

a + 1 4
re

=> 4a + 3P -5 =0 ●●●(i)
Point M (a, p) lies on 3.v - 4y -16 = 0.
u
ad

3a-4p-16=0 ...(ii)
Yo

Solving (i) and (ii), we get


49
d

a = — and p = - 3x - 4y -16 = 0 M(a, p)


Re

25 25
in

Fig. 12.29
Hence, the coordinates of M are (68/25, - 49/25).
F

15. The line -4 + ^6 = 1 cuts y-axis at (0,6) and has slope is —.


2
2
Hence, equation of the required line is y - 6 = —3 (x - 0) or, 2x - 3y +18 = 0

16. Clearly, (-1,2) lies on y = mx + c.


2 = -m + c ●●●(i)
The line joining the origin to (-1,2) is perpendicular to y = mx + c.
2-0
X in = -1 => m =-
-1 -0 2

1 ● . 5
Putting m - - m (i), we get c = -.
12.46 APPLIED MATHEMATICS-Xl

18. Let the image of the point P (3, 8) in the line mirror ,v + 3y = 7 be Q (a P). Then, PQ is
perpendicularly bisected at M. The coordinates of M are ^ ^ ^ ^ Since M lies on \ 2 ' 2
x+ 3y = 7.

P + 8-| ^ ^
a + 3
+ 3 P(3,8)
2 2 J
a + 3p + 13 = 0
Line segment PQ is perpendicular to a: + 3y =7.
P-8 1 M .V + 3y = 7
X = -l
a - 3 3

w
3a-9 = P-8
, Q(a.P)
3a-p-l=0 ...(ii) Fig. 12.30

o
Solving (i) and (ii), we get a = -1 and p = - 4.

e
re
Hence, the coordinates of Q are (-1, - 4).
12.11 ANGLE BETWEEN TWO STRAIGHT LINES WHEN THEIR EQUATIONS

rFl
F
ARE GIVEN

THEOREM Prove tJuit the acute angle 0 between the lines a-^x + b-^y + c-^=0 and a2X+b2y+C2=0
is given by

or
ou
tan 9 =
ksf
i?! ^2 “t ^1 ^2
oo

PROOF Let m-i and ni2 be the slopes of the lines flj a: + 1;] y + Cj = 0 and -T + 1^2 3/ + ^2 ~ 0-
Then,
Y
B

= -
and ?«2 = - —
h
re

- m2
oYu

Now, tan 0 =
ad

1 + m-i nh

^ ^2
d

h h ^2 ^1 — ^2 — 1 ^2 ~ ^2
in

tan 0 =
Re

■\ /
=> tan 0 = tan

1 + .^1 ^2 fll ^2 b-^ b2 (?! (?2 "F 1^]


F

h)[ h
Q.E.D.

CONDITION FOR THE LINES TO BE PARALLEL Ifthelmesa-^ X + bi y + Ci = 0anda2 A■ + &2 y + ^2 = 0


are parallel, then

mi = m2^-^ =
fc2 ^2 ^2
CONDITION FOR THE LINES TO BE PERPENDICULAR If the lines a^X + b^y + q =0 and
^2 A + &2 y + ^2 = 0 are perpendicular, then

/111 m2 — — —1 <?! <?2 "F 3?i b2 — 0


hi &2
It follows from the above discussion that the lines fli a + h] y + Ci = 0 and a2 x + b2y + C2=0 are:
12.47
STRAIGHT LINES

(i) Coincident, if Hi - h. - Si (ii) Parallel, if


^2 bi '^2 ^2 ^2 ^2

(iii) Intersecting, if ‘Si^h (iv) Perpendicular, if <7j ^2 + ^’2 ~


<?2 1^2
To find the acute angle between two lines when their slopes are given, we may use the following
algoritlun.
ALGORITHM

STHP 1 Obtain the equations of the lines.


Coeff. of X
STLPII Obtain the slopes lUi and m2 of two lines by using the formula: Slope = -
Coejf. of y

w
lUi -W2
STEP 111 Use the formula : tan 0 = to find the acute angle 0 between the lines.
1 + Wj m2

F lo
ILLUSTRATIVE EXAMPLES

ee
EXAMPLE 1 Find the angles between the pairs of straight lines

Fr
(i) X -j3y-5 = 0 and V3 x + y - 7 =0 (ii) y =(2 - V^) x + 5 and y =(2 + \f3) x-7.
SOLUTION (i) The equations of two straight lines are: for
X -^/3y -5=0 ...(i) and a/3 x + y - 7 = 0 ...(ii)
ur
Let and m2 be the slopes of these two lines. Then,
s
ook

-1
Yo

m^ = and ni2 = = -V3


-^/3 a/3 1
eB

We observe that ni] m2 = -1. Thus, the two lines are at right angle,
(ii) Let Wj and?»2 slopes of the straight linesy = (2--v/3) X+ 5 and y =(2 + a/3) x-7
our
ad

respectively. Then, = 2 - a/3 and /«2 = 2 + a/3.


Let Obe the angle between the lines. Then,
2/3
Y

tan 0 =
lUi - m2 (2-V3)-(2 + a/3) = a/3
Re

1+ 7??2 1 + (2 - -fS) (2 + a/3) 1+4-3


nd

^ 0 = 7T/3
Fi

Thus, the acute angle between the lines is of 60°.


EXAMPLE 2 Find the tangent of the angle between the lines whose intercepts on the axes are respectively
a, - b and b, - a.
SOLUTION The line which cuts off intercepts a and -b on the coordinate axes passes through
points A {a, 0) and B (0, - b).
-&-0 b
Slope of line AB = nii = Q-a a

The line which cuts off intercepts b and - rt on the coordinate axes passes through points C {b, 0)
and D (0,-<7).
-rt-0 a

Slope of line CD = m2 = 0-b b


12.48 APPLIED MATHEMATICS-XI

Let 9 be the angle between AB and CD. Then,


b a

tan 0 = ± m-i - ni2 __ ±


a
±
1 + ?«2 b a lab
1 + ●
a b

EXAMPLE 3 Find the value ofk if the straight line 2 x + 3 }/ + 4 + k(6x-y + 12) = 0is perpendicular to
the line? x + 5y-4 = 0.
SOLUTION The two lines are

X (2 + 6 k] -i- y {3-k) + 4 + I2k = 0 ...(i) and 7x+5y-4=0 ...(ii)


Let and n»2 be the slopes of (i) and (ii) respectively. Then,
2 + 6k 7
nil = -

w
If lines (i) and (ii) are perpendicular. Then,

Flo
nil '”2 ~ 1 ^ 2 + 6k'\( 7" 29

3-k Jl 5,
= -l =i> 14 + 42fc=-15+51c => k=-
37

ee
EXAMPLE 4 A line passing through the points (a, 2 a) and (- 2, 3) is perpendicular to the line

Fr
4.t+3j/ + 5 = 0,fmd the value of a.
2a~3
SOLUTION Let nil be the slope of the line joining A {a, 2a) and B (- 2,3). Then, for =
ur
a+2

4
s
Let ni2 be the slope of the line 4x+3i/+5 = 0. Then,
k
Yo
oo

Since given lines are perpendicular. Therefore,


eB

2a-3 4
nil = -1 => X
= -l=>8fl-12 = 3rt+ 6=> fl=18/5.
a + 2 3
r

EXAMPLES
Classify the follozving pairs of lines as coincident, parallel or intersecting:
ou
ad

(i) AT + 2y-3 = 0 and - 3x-6i/+9=0 (ii) x + 2\j + \ = 0 and 2x + 4y+3=0


Y

(iii) 3x-2y + 5= 0 and 2 .v + y - 9 = 0


Re
nd

SOLUTION (i) The given lines are rtj x + y + C;i = 0 and «2 x + &2 1/ + ^^2 = where
ai = l,bi=2, Ci = - 3,02 = - 3,b2 = - 6 and C2 - 9.
Fi

Clearly, ^ ^ h = ^ 1
—. So, the given lines are coincident.
^2 f*2 ^2 3

(ii) The given lines are aix + biy+ci=0 and a2X + b2}/ + C2= 0, where
ai = 1, = 2, = 1, <72 = 2, b2 = 4, and C2 = 3.
Clearly, -k = ^ ^ £i. go, the given lines are parallel.
02 h ^2
(iii) The given lines are nj x + y + C] = 0 and <?2 x + y + ^2 = ^, where
Oi = 3,&j =- 2, C| = 5, <?2 = 2,1?2 = 1/ ‘ind C2 = - 9.
Clearly, — ^ So, the given lines are intersecting.
02 h
12.49
STRAIGHT LINES

EXERCISE 12.12

1. Find the angles between each of the following pairs of straight lines:
(i) 3 -V + y + 12 = 0 and x + 2y -1 = 0 (ii) 3A'-y + 5 = 0 and .t - 3 y + 1 = 0
(iii) 3j: + 4y-7=0 and 4.v-3y + 5= 0 (iv) x-4y = 3 and 6.v-y=ll
(v) (m^ - mu) y = {mn + n^) x + and {mu + m^) y = {mu - u^) x + m^.
2. Find the acute angle 0 between the lines 2.r-y+3=0 and .r + y + 2 0.
3. Find the angle 0 between the line joining the points (2,0), (0, 3) and the line .t + y == 1.
4. Find the angle between the lines x = ci and by + c = 0.
5. Find the tangent of the angle between the lines which have intercepts 3,4 and 1,8 on the

ow
axes respectively.
6. Show that the line a^x + «y +1 = 0 is perpendicular to the line x - ay =1 toi all non-zero real
values of a.
X U X y . lab
7. Show that the tangent of an angle between the lines -a + f = 1 and -a --b = 1 is

e
° ° b a

re
Fl ANSWERS

F
(a4 m 1 n 1\
ur (13'\
^ -3 J (iii) 90° (iv) tan
-1 -1 -1
1. (i) 45° (ii) tan (v) tan

r
4 4
10 m - n

1
fo 5.^
ks
2. tan 0 = 3 3. tan 0 = - 4. 90°
5 7
Yo
oo

12.12 DISTANCE OF A POINT FROM A LINE


eB

THEOREM Prove that the length of the perpendicular from a point (xi^ y{) to a line ax + by + c=0is

a X'^ + b yi + c
ur
ad
Yo

PRCX)F The line ax + by + c = Q meets .r-axis at y = 0. Therefore, putting y = 0 in ax + cfy + c = 0,


Thus, the coordinates of the point A where the line ax + by + c = 0 meets x-axis are
d

we get X = - -.
Re

a
in

{-c/a, 0). Similarly, the coordinates of B where the line cuts y-axis are {0,-c/b).
F

Let P (xj, yj) be the point. Draw PN 1 AB.


Now,

f-)/i'l + o(yi-0)
c\ c (
Area of A PAB = — -
0 + -
2 H b) a \ V

_ 1
= (rtxi+/jyi + c)^^
+
cyi
2 b a ab

Also,
1 1
Area of APAB = - AB x PN = —, , +
^xPN — Ja^ + b^ X PN
= 2ab^
2 2\a^ b^
12.50 APPLIED MATHEMATICS-XI

From (i) and (ii), we get

w
COROLLARY
{ax^ + byi + c) —

PN =
2

I nxi + bi/i + c\
F lo
^ +b^ X PN

for F
ree
The length of the perpendicular from the origin to the line a.v + /n/ + c = 0 is
Q.E.D.

kl

We may use the following algorithm for finding the length of the perpendicular from a point
{.T|, i/]) to the line ax + bi/ + c = 0.
Your
oks

ALGORITHM
eBo

STEP I
Write the equation of the line in the form ax + by + c = 0.
STEP 11
Substitute the coordinates atj and of the point in place of x and \j respectively in the
expression.
ad
our

Divide the result obtained in step II by the square root of the sum of the squares of the
coefficients of x and y.
STEP IV
Take the modulus of the expression obtained in step III.
Re
dY

ILLUSTRATIVE EXAMPLES
Fin

EXAMPLE 1 Find the distance between the line 12x -5y+ 9-0 and the point (2, I).

SOLUTION Required distance = 12x 2-5x1 + 9 _ |24 -5 + 9j _ 28


13 13

Ifp is the length of the perpendicularfrom the origin to the line —a + — ^ = 1, then prove that
EXAMPLE 2

1 1 1
(ii) =0, if 1?^ are in A.P.
a' b^

SOLUTION (i) The equation given line is + fly 0 ...(i)


It is given that
p = Length of the perpendicular from the origin to line (i)
STRAIGHT LINES 12.51

_ |b(0) + fl(0)-/7b| ab
^ ~ I,.2 +. n.2
b^ 1 1 1 1
+
2
a b^
(ii) If (7^, p^,b^ are in A.P, then 2p~ +b'^.
Now, 1 J_ J_ [from (i)]

p'^{a'^ + b'^) = ah~


(a^+b^)

ow
2
{(t'^+b^) =ah'^ [Using =a^ +b^]
{a^ +b^f = lah'^ ^a^+b"^ =0.
EXAMPLES What are the points on x-axis zvhose perpendicular distance from the line 4x + 3y =12

e
is 4?

re
SOLUTION

rFl
Let the required point be P (a, 0). Then, Length of the perpendicular from P (a, 0)

F
on 4x + 3y -12 = 0 is 4.
4a + 3 X 0 “12
= 4

r
ou
4a-12
fo
ks
= 4^ I 4a-121 “ 20^|a-3| = 5=> a-3 = ±5^ a = 8,-2
5
oo

Hence, the required points are (8,0) and (- 2, 0).


Y

EXAMPLE 4 Find the points on y-axis whose perpendicular distance from the line 4x - 3y -12 = 0/s3.
eB

SOLUTION Let te required point be P (0, a). It is given that the length of the perpendicular
from P 0, a) on 4.v - 3y -12 = 0 is 3.
r

4x0-3a-12
ou

= 3
ad
Y

V4^+('3)2
3a+ 121 =15=i>|a + 4| = 15 => a + 4 = ±5=> a =1,-9
d

Hence, the required points are (0,1) and (0, - 9).


Re
in

EXAMPLE 5 Find the equation of the straight line which cuts off intercept on X-axis which is twice that
F

on Y-axis and is at a unit distance from the origin.


SOLUTION Let the equation of the straight line be -a + ^ = 1. It is given that a = 2b.

Putting a = 2b in the above equation, we get


x + 2xj-2b =0 -ii)
This line is at a unit distance from the origin.
0 + 2 X 0 - 2t m V5
= 1 => = 1 ^ |21j| = ffs ^ 2b = ±-j5=>b=± 2
V5

Substituting the value of in (i), we obtain .r + 2i/ ± \/S = 0 as the equations of the required
line.

EXAMPLE 6 Tlze equation of the base of an equilateral triangle is x + y - 2 = 0 and the opposite I’ertex
has coordinates (2, -1). Find the area of the triangle.
12.52 APPLIED MATHEMATICS-XI

SOLUTION Let p be the altitude of the given triangle and 'a' be


the length of each side. Then,
p = Length of perpendicular from (2,-1) on a: + y - 2 = 0
2-1 -2 1
=> p =

In dABD, we have
2 1 2
sin 60® = => a- a = X +y-2=0
a 1 a V3 V3"V2 3
Fig. 12.32

2^ 1
Area of the triangle = sq. units
4 3 2V3

w
LXAMPLE 7 Find all points onx + y = 4 that lie at a unit distance from the line 4x + 3y -10 = 0.

F lo
SOLUTION Note that the coordinates of an arbitrary point on x + y = 4 can be obtained by
putting X = t {or ]/ = t )and then obtaining y (or a:) from the equation of the line, where f is a
parameter. Putting a: = f in the equation a: + y = 4 of the given line, we obtain y = 4 - f.

ee
Fr
So, coordinates of an arbitrary point on the given line are P (f, 4 - /).
Let P (f, 4 -1) be the required point. Then, distance of P from the line 4x + 3y -10 = 0 is unity.
for
ur
4f+3(4-/)-10
= 1^ |f + 2|=5^f + 2 =±5=>f= —7 or, f = 3
^4^ + 3^
s
ook
Yo

Hence, required points are (- 7,11) and (3,1).


eB

EXAMPLES
Find the equations of lines passing through the point (1, 0) and at a distance —^from the
our
ad

origin.

SOLUTION Let m be the slope of a line passing through (1, 0). Then, its equation is
Y

y-0=w(A:-l) or, mx -y -m = 0...{i)


Re

43
nd

It is given that line (i) is at a distance from the origin.


Fi

mx 0-0-m 43
2

|m| 43
^ 4m^ = 3m^ + 3 ^ = 3 => m = + 4S
41^ 2
Substituting the values of rn in (i), we obtain
43x - y + 43 = 0 and 43x + y + 43 = 0 as the equations of the line.
EXERCISE 12.13

1. Find the distance of the point (4,5) from the straight line 3a: - 5y + 7 =0.
2. Show that the perpendiculars let fall from any point on the straight line 2A: + lly-5=0
upon the two straight lines 24 x + 7 y = 20 and 4x-3y-2=0 are equal to each other.
STRAIGHT LINES 12.53

3. Find the distance of the point of intersection of the lines 2x + 3 y = 21 and 3a:-4y + ll = 0
from the line 8x + 6y + 5 = 0.
4. Find the length of the perpendicular from the point {4, - 7) to the line joining the origin and
the point of intersection of the lines 2x - 3y +14 = 0 and 5x+4y-7 =0.
X 1/
5.
5 are the points on X-axis whose perpendicular distance from the straight line - + ^ = 1
is fl ?
6. Find the perpendicular distance from the origin of the perpendicular from the point (1,2)
upon the straight line x - V3 y + 4 = 0.
7. Find the distance of the point (1,2) from the straight line with slope 5 and passing through
the point of intersection of x + 2 y = 5 and x - 3 y = 7.
X V
8. What are the points on y-axis whose distance from the line ^ " 1 is 4 units?
9. In the triangle ABC with vertices A (2, 3), B (4, -1) and C (1, 2), find the equation and the

w
length of the altitude from the vertex A.
ANSWERS

1.
6
3.®
10

F lo
4. 1 5. -b (!> ± Ja^ +b^ ), 0 ■

e
132

Fre
6. -(2 + ^/3) 8. (0, 32/3), (0,-8/3)
2

9. x-y + l=0,V2
for
12.13 DISTANCE BETWEEN PARALLEL LINES
r
If two lines are parallel, then they have the same distance between them throughout. Therefore
You
oks

to find the distance between two parallel lines choose an arbitrary point on one of them and find
eBo

the length of the perpendicular on the other. In order to choose a point on a line, we give an
arbitrary value to x or y and find the value of the other variable.
We may use the following algorithm to find the distance between two parallel lines.
ad
our

ALGORITHM

Let the two parallel lines be ax + bi/ + Cj = 0 and ax + by + C2 = 0. To find the distance between
these two lines we proceed as follows:
Re
dY

STBM Choose a point.on any one of the tiuo lines by giving a particular value to x or y of your choice.
Fin

sTEr n Find the length of the perpendicular from the chosen pomt in step I to the other line.
STEP 111 The length obtained in step II is the required distance between the parallel Iwes.
THEOREM Prove that the distance between two parallel Imes ax + by + Cl = 0andax + by + C2 = 0
ki -C2I
is given by

PR(K 'F Given lines are

ax + by + Cl =0 ...(i)
ax + by + C2 =0 ...(ii)

Let P {h, k) be a point on the line ax + by + = 0. Then,


ah + bk + Cl - 0 ...(iii)

Clearly, distance ‘d' between parallel lines (i) and (ii) is equal to the length of perpendicular from
P on line (ii).
12.54 APPLIED MATHEMATICS-XI

X' X

w
d = PM

, ah + bk + c-y
F lo
e
Fre
a = —, ^ [From (iii): ah +bk = -c'^]
for
d =
\c\-c2\
Q.E.D.
^ja^+b^
r
You
oks

ILLUSTRATIVE EXAMPLES
eBo

EXAMPLE 1 Find the distance betioeen the parallel lines 3.v - 4y + 9 = 0 and 6x - 8y -15 = 0.
SOLUTION Putting y = 0 in 3.r -4y + 9 = 0, we get x =-3. Thus, (- 3, 0) is a point on the line
3x - 4y + 9.
our
ad

Length of the perpendicular from (- 3,0) to 6x - 8y -15 = 0 is given by


-3x6-8x0-15 33
d =

P + (-sf 10
dY
Re

33
Hence, the distance between the given lines is units.
Fin

10

ALITF.R Given lines are


15
3a: - 4y + 9 = 0 and,
6a:-8y-15 = 0 or, 3A:-4y-'^ = 0 ...(h)

-15 15
9- 9 +
2 J
Required distance =
5 '10

EXAMPLE 2 Find the equations of lines parallel to 3 x - 4: y -5 = 0 at a unit distance from it.
SOLUTION Equation of any line parallel to 3 a: - 4 y - 5 = 0 is
3a: - 4y + X = 0
Putting a: = -1 in 3a; -4 y-5 = 0, we get y =-2. Therefore, (-1,-2) is a point on 3A:-4y-5 = 0.
STRAIGHT LINES 12.55

Since the distance between the two lines is one unit. Therefore, the length of the perpendicular
from (-1, - 2) to 3 -V - 4 y + X. = 0 is one unit.
i.e.
[ 3x -1 -4x -2 + X|

15 + X.l = 1 => 5 + X.|=5=> 5 + X = ±5 =^> X. = 0 or -10.


5

Substituting the values of X in (i), we get


3x-4y = 0 and,
3a: - 4y-10 = 0 as the equations of the required lines.
AUTI-R Let the equation of a line parallel to 3a: - 4y-5 = Obe
3a: - 4y + X. = 0 ...(i)
It is given that the distance between the line 3a: - 4y - 5 = 0 and line (i) is 1 unit.

w
X +5
= 1 => = 1=>|X + 5| = 5=> X + 5 = ±5 => X = 0,-10.
5
3^+(-4)^

F lo
Substituting the values of X in (i), we get 3.v - 4y = 0 and 3.\: - 4y -10 = 0 as the equations of
required lines.

ee
liXAMPLE 3 Two sides of a scjuare lie on the lines a: + y = 1 and x + y + 2 = 0. What is its area?

Fr
SOLUTION clearly, the length of the side of the square is equal to the distance between the
for
parallel lines
and .V + y + 2 = 0 ...(ii)
ur
.V + y - 1 = 0
Putting a: = 0 in (i), we get y -1. So (0,1) is a point on line (i).
s
ook

Distance between the parallel lines


Yo

0 + l+2(_J^
eB

= {Length of the perpendicular from (0,1) to .t + y + 2 = 0} = V2 ■

3 f
our

3 9
ad

^
Thus, the length of the side of the square is sl2 and hence its area is — square units
V2
\Lli. R
The equations of parallel sides of the square are a: + y -1 = 0 and a: + y + 2 = 0.
Y

2-(-l) 3
Re

Length of the side of the square = Distance between parallel side =


nd

42

( ^f
Fi

9
Hence, Area of the square =(Side) = — sq. units.

EXAMPLE 4
Prove that the line 5A:-2y-l=0 is mid-parallel to the lines 5.r-2y-9=0 and
5a- - 2y + 7 = 0.
SOLUTION Clearly, the slope of each of the given lines is same equal to 5/2. Hence, the line
5.V - 2y -1 = 0 is parallel to each of the given lines.
In order to prove that the line 5.v - 2y -1 = 0 is mid-parallel to the given lines it is sufficient to
show that the line 5a: - 2y -1 = 0 is equidistant from the given lines.
Puttingy = 0in5.v -2y -1 = 0, we get a: = 1/5. So, the coordinates of a point on5.A: - 2y -1 = 0are
(1/5, 0).
The distance frj between the lines 5.v - 2y -1 =0 and 5a: - 2y - 9 = 0 is given by
d-^ = Length of the perpendicular from (1 / 5, 0) to 5a: - 2y - 9 = 0
12.56 APPLIED MATHEMATICS-Xl

5x(l/5)-2x0-9 8
d,=
j5^+(-2f
The distance £#2 between the lines 5x -2y-l =0 and 5x-2i/ + 7 = 0 is given by
^2 = Length of the perpendicular (1/5, 0) to5A:-2y + 7=0
5x (1/5)- 2x0 + 7 8
^2 -
>/5^ + (-2)^
Clearly, = ^2-Consequently the lineSx - 2i/ -1 = 0 is equidistant from the lines 5;r - 2y - 9 = 0
and 5a: - 2y + 7 =0. Hence, the result follows.

w
EXERCISE 12.14

1. Determine the distance between the following pair of parallel lines :

F lo
(i) 4x - 3y - 9 = 0 and 4.x - 3y - 24 = 0
(ii) 8x + 15y - 34 = 0 and 8x + 15y + 31 = 0

ee
(iii) y=mx + c and y=7nx + d

Fr
(iv) 4x+3y-ll=0 and 8x+6y=15
for
2. The equations of two sides of a square are 5x - 12y - 65 = 0 and 5x - 12y + 26 = 0. Find the
ur
area of the square.
3. Find the equation of two straight lines which are parallel to x + 7y + 2 = 0 and at unit
s
ook

distance from the point (1, -1).


Yo

4. Prove that the lines 2 x + 3 y =19 and 2 x + 3 y + 7 = 0 are equidistant from the line
eB

2 X + 3 y = 6.
r

ANSWERS
ou
ad

65 c-d
1. (i) 3 rmits (ii) units (iii) (iv) ^10 units
Y

^1 + n?
17
Re
nd

2. 49 sq. units 3. x + 7y + 6±5V2=0


Fi

MULTIPLE CHOICE QUESTIONS (MCQs)


Mark the correct alternative in each of the folloioing:
1. The point which divides the join of (1, 2) and (3, 4) externally in the ratio 1 :1
(a) lies in the III quadrant (b) lies in the II quadrant
(c) lies in the I quadrant (d) cannot be found

2. A line passes through the point (2, 2) and is perpendicular to the line 3x + y = 3.
Its y-intercept is
(a) 1/3 (b) 2/3 (c) 1 (d) 4/3

3. If the lines ax + 12y + 1 = 0,bx + 13y + 1=0 and cx + 14y + 1=0 are concurrent, then
a, b, c are in
(a) H.P. (b) G.P. (c) A.P. (d) none of these
STRAIGHT LINES 12.57

4. The number of real values of X for which the lines o:-2y+3 = 0, X,^: + 3y + l- 0 and
4;c - X1/ + 2 = 0 are concurrent is
(a) 0 (b) 1 (c)2 (d) Infinite

5. The equations of the sides AB, BC and CA of A ABC are y — x = 2, x + 2y -1 and


3:r + y + 5 = 0 respectively. The equation of the altitude through B is
(a) :r - 3y +1 = 0 (b) x - 3y + 4 = 0 (c) 3x -y + 2 = 0 (d) none of these
6. Area of the triangle formed by the points ((« + 3) {a + 4), a + 3), ((a + 2) {a+ 3), {a + 2))
and((fl + l) (fl + 2), (fl +1)) is
(a) 25fl^ (b) (c) 24fl2 (d) none of these

7. The line segment joining the points (- 3, - 4) and (1, - 2) is divided by y-axis in the ratio
(a) 1:3 (b) 2:3 (c)3:l (d) 3:2

w
8. The area of a triangle with vertices at (- 4, -1), (1,2) and (4, - 3) is
(a) 17 (b) 16 (c) 15 (d) none of these

F lo
9. The line segment joining the points (1,2) and (- 2,1) is divided by the line 3x + 4y=7in
the ratio

e
(a) 3:4 (b) 4:3 (c)9:4 (d) 4:9

Fre
10. If the point (5,2) bisects the intercept of a line between the axes, then its equation is
for
(a)5x + 2y = 20 (b)2x + 5y = 20 (c)5x-2y = 20 (d)2x-5y = 20
11. A (6, 3), B (- 3,5), C (4, - 2) and D (x, 3x) are four points. If A DBC : A ABC =1:2, then x
r
You

is equal to
s
ook

(a) 11/8 (b) 8/11 (c) 3 (d) none of these

12. If p be the length of the perpendicular from the origin on the line x/a + y/b = 1, then
eB

(a) p'^=a^+b^ (b) + (c) = -i- + (d) none of these


b^ ^2
our
ad

13. The equation of the line passing through (1, 5) and perpendicular to the line
3x-5y + 7= 0is
dY
Re

(a) 5x+3y-20 = 0(b) 3x-5y + 7=0 (c)3x-5y + 6 = 0 (d)5x+3y + 7=0


Fin

14. The figure formed by the lines ax ± &y ± c = 0 is


(a) a rectangle (b) a square (c) a rhombus (d) none of these

15. The inclination of the straight line passing through the point (- 3, 6) and the mid-point of
the line joining the point (4, - 5) and (- 2, 9) is
(a) 7i/4 (b) 7i/6 (C) 71/3 (d) 3ti/4

16. Distance between the lines 5x + 3y - 7 = 0 and 15x + 9y +14 = 0 is


35 35 35
(a) (c) (d)
3VM 2VM

17. The angle between the lines 2x - y + 3 = 0 and x + 2y + 3 = 0 is


(a) 90° (b) 60° (c) 45° (d) 30°

18. The value of X,for which the lines 3x + 4y = 5,5x + 4y = 4 and A, x + 4y = 6 meet at a point is
(a) 2 (b) 1 (c)4 (d) 3
THE STRAIGHT LINES

19. Three vertices of a parallelogram taken in order are(-l, - 6), (2, -5) and (7, 2). Tl-ie fourth
vertex is

(a) (1,4) (b) (4,1) (c) (1,1) (d) (4, 4)


20. The centroid of a triangle is (2, 7) and two of its vertices are ( 4, 8) and (- 2, 6). The third
vertex is

(a) (0, 0) (b) (4,7) (c) (7, 4) (d) (7,7)


21. If the lines .r + q = 0, i/ - 2 = Oand 3x + 2i/ + 5 = 0 are concurrent, then the value of will be
(a) 1 (b) 2 (c) 3 (d) 5

22. The medians AD and BE of a triangle with vertices A (0, b), B (0, 0) and C {o, 0) are

perpendicular to each other, if

w
b

(b) b = I (c) ab = 1 (d) a = ±^/2b

23. The equation of the line with slope

Flo
3/2 and which is concurrent with the lines
4a' + 3y - 7 = 0 and 8.v + 5y -1 = 0 is
(a) 3a + 2y - 63 = 0

ee
(b) 3a + 2y - 2 = 0
(c) 2y - 3a - 2 = 0 (d) none of these

Fr
24. The ratio in which the line 3.a + 4i/ + 2 = 0 divides the distance between the lines
3a + 4y+ 5 = 0 and 3a + 4i/-5 = 0 is for
ur
(a) 1 : 2 (b) 3:7 (c) 2 : 3 (d) 2:5
25. The coordinates of the foot of the perpendicular from the point (2, 3) on the line
ks
A + y -11 =0 are
Yo

(a) (-6,5) (b) (5, 6)


oo

(c) (-5, 6) (d) (6,5)


26. The reflection of the point (4, -13) about the line 5a + y + 6 = 0 is
eB

(a) (-1,-14) (b)(3,4) (c) (0, 0) (d)(l,2)


27. The inclination of the line a - y + 3 = 0 with positive direction of .v-axis, is
r

(a) 45° (b) 135° (c) -45° (d) -135°


ou
ad

28. The two lines a-^x + b-[i/ = c-j and (i2^ + b2}/ = C2 are perpendicular if
{a) a-in2+b-[b2=0 (b) rtj (c) (^\bi + ~ ^ (d) ^1^2 ^2^1 ~ ®
Y

29. The coordinates of the foot of the perpendicular from the point (2, 3) on the line
A + y-11 =0are
nd
Re

(a) (-6,5) (b) (5, 6) (c) (-5, 6) (d) (6,5)


Fi

30. The coordinates of the image of the point (2,3) in the line mirror A + y-11 =0are
(a) (5,6) (b) (9,8) (c) (8,9) (d) (-8, -9)
31. The intercept cut-off by a line from y-axis is twice than from A-axis and the line passes
through the point (1,2). The equation of the line is
(a) 2A + y = 4 (b) 2.A + y + 4 = 0 (c) 2.v-y = 4 (d) 2.v-y + 4 = 0
32. A line passes through the point P (1, 2) such that its intercept between the axes is bisected at
P. The equation of the line is
(a) A + 2y=5 (b) A-y + 1 =0 (c) A + y - 3 = 0 (d) 2A + y-4 = 0
33. The reflection of the point (4, -13) about the line 5a + y + 6 = 0 is
(a) (-1,-14) (b) (3,4) (c) (0, 0) (d) (1, 2)
34. One vertex of the equilateral triangle with centroid at the origin and one side as a + y - 2 = 0
is

(a) (-1, -1) (b) (2, 2) (c) (-2,-2) (d) (2,-2)


12.59
STRAIGHT LINES

35. The coordinates of the foot of perpendicular from the point (2, 3) on the line y - 3.y + 4 are
given by
( 10 ^2
(a) (b) (c) 10 (d)
.10' 10 . lO' 10 . l3
37' 3)
36. If the coordinates of the middle point of the portion of a line intercepted between the
coordinate axes are (3, 2), then the equation of the line will be
(a) 2.v+3y=12 (b) 3y+2i/ = 12 (c) 4.Y-3y = 6 (d) 5y-2i/=10
37. Equations of diagonals of the square formed by the lines .y = 0, y = 0, y = 1 and y = 1 are
(a) y = Y, y = Y +1 (b) y = y, y + y = 2 (c) 2y = y, y + y = — (d) y = 2y, y = 2y +1
3S For specifying a straight line, how many geometrical parameters should be known?
(a) 1 (b) 2 (c) 3 (d) 4
39. The tangent of the angle between the lines whose intercepts on the axes are a, -b and b,
-a

w
respectively, is
b^-a^
(d) none of these

F lo
(a) (b)
nb 2 2nb

40. If the line-n + ^b =1 passes through thepoints (2, - 3) and (4, -5), thenfr?, b)

ee
Fr
(a) (1,1) (b) (-1,1) (c) (1,-1) (d) (-1,-1)
41 The distance between the lines y = mx + and y = /»y + C2, is
for
^1-^2 C2-C1
(a) (b) (c) (d) IC1-C2I
Vl +in^ ^h +m^
ur
42. The distance of the point of intersection of the lines 2y - 3y + 5 = 0 are 3y + 4y = 0 from the
s
ook

line 5y - 2y = 0, is
Yo

130 13 130
(a) (b) (0- (d) none of these
eB

17V^
43. Slopes of lines which cuts off intercepts of equal lengths on the coordinate axes
are
our

(b) ±4? (d) ±V3


ad

(a) 0 (c) ±1
V3
3
44 . A line cutting off intercept - 3 from the y-axis and the tangent of angle to the Y-axis is -, its
Y
Re

equation is
nd

(a) 5y-3.Y + 15=0 (b) 3y-5.Y + 15 = 0 (c) 5y-3y-15 = 0 (d) none of these
Fi

45. The equation of the straight line passing through the points (3, 2) and perpendicular to the
line y = y is
(a) Y-y =5 (b) Y + y = 5 (c) Y + y = 1 (d) Y-y=l

ANSWERS

1. (d) 2. (d) 3. (c) 4. (a) 5. (b) 6. (d) 7. (c) 8. (a)


9. (d) 10. (b) n. (a) 12. (c) 13. (a) 14. (c) 15. (d) 16. (c)
17. (a) 18. (b) 19. (b) 20. (b) 21. (c) 22. (d) 23. (b) 24. (b)
25. (b) 26. (a) 27. (a) 28. (a) 29. (b) 30. (c) 31. (a) 32. (d)
33. (a) 34. (b) 35. (b) 36. (a) 37. (a) 38. (b) 39. (c) 40. (d)
41. (b) 42. (a) 43. (c) 44. (a) 45. (b)
12.60 APPLIED MATHEMATICS-XI

FILL IN THE BLANKS TYPES QUESTIONS (FBQs)


1. If 2a+3b + 4c= 0, then the straight lines ax + bij + c=0 will always pass through the point

2. If a, b, c are in A.P., then the straight lines ax+by+c = 0 will always pass through the
point
3. The line which cuts off equal intercepts from the axes and pass through the point (1,-2)
is

4. Equations of the lines through the point (3, 2) and making an angle of 45° with line
x-2y~3 =0are
5. If the lines fl.v + 2i/ +1 = 0, + y +1 = 0 and c.r + 4i/ +1 =0 are concurrent, then a, b, c are in

6. If the line (2.v + 3y + 4) + X (6.v -y +12) = 0 is perpendicular to the line 7x + y -4 = 0, then


X =.

w
/. The equation of a straight line passing through (-3, 2) and cutting intercepts from the axes

whose sum is zero, is

2.v+2y =llis
F lo
S. The number of points on the line Ar + y = 4 which are at a unit distance from the line

9. If the lines 2-.c - 3y + /: = 0, 3;r - 4y -13 = Oand 8:c - lly - 33 = 0 are concurrent, then k =

ree
10. If 2:r + y-5=0 and 4.T + 2y-15=0 are two parallel sides of a square, then its
1
is
for F
1 - If the lines 3.v - 4y + 4 = 0 and 6.v - 8y -7 =0 are two parallel tangents to a circle, then the
area

length of its diameter is


Your
ks

ANSWERS
eBoo

1 (- 2. (1,-2) 3- AT + y+1 =0
2' 4
17
4. 3.r-y-7 =0, .v+3y-9 =0 5. AP 6. -
ad
our

41

7. x-y + 5 = 0 8. Infinite -7 10. 5- sq. units


4 ^
3
11. -
Re

2
Y
Find

—- VERY SHORT ANSWER QUESTIONS (VSAQs)


Answer each of the following cjiiestions in one word or one sentence or as per exact recjuirement of the
question:

1- Write an equation representing a pair of lines through the point {a, b) and parallel to the
coordinate axes.

2. If the centroid of a triangle formed by the points (0, 0), (cos 0, sin 0) and (sin 0, - cos 9 lies
on the line y = 2.t, then write the value of tan 0.
3. Write the distance between the lines 4.r + 3y-11 = 0 and 8.r + 6y-15 = 0.
4. If the lines x + ay + a = 0, bx + y + b = 0 and cx + cy +1 = 0 are concurrent, then write
the value of 2abc -ab - be - ca.

5. Write the coordinates of the image of the point (3, 8) in the line x + 3y -7 = 0.
12.61
STRAIGHT LINES

6. Write the integral values of m for which the Ar-coordinate of the point of intersection of the
lines y = mx + \ and 2>x + Ay = 9 is an integer.
7. lia ^ b ^ c, write the condition for which the equations{h — c) x + {c - a)y + {a ~b) — 0and
(b^ -c^)a: + {c^ -a^)y + {a^ -1?^) = 0 represent the same line.
8. If care in G.P. write the area of the triangle formed by the line i7:c + bi/ + c = 0 with the
coordinates axes.
9. If a, b, c are in A.P., then the line ax + h/ + c = 0 passes through a fixed point. Write the
coordinates of that point.
10. Write the equation of the line passing through the point (1, -2) and cutting off equal
intercepts from the axes.

w
ANSWER

2.-3 3. 7/10 units 4. -1


1. (A'-n)(y-ii) = 0

Flo
1
6. -1,-2 7. a+ b + c = 0 8. — Sq. units
5. (-1,-4) 2 ^

ee
9. (1,-2) 10. AT + y+ 1 =0

Fr
for
ur
k s
Yo
oo
eB
r
ou
ad
Y
Re
nd
Fi
CHAPTER

CIRCLE

13.1 DEFINITION

A circle is defined as the locus of a point which moves in a plane such that its distance from a fixed point in
that plane is always constant.
The fixed point is called the centre of the circle and the constant distance is called the radius of the
circle.

w
In Fig. 13.1, P is the moving point, C is the fixed point and CP is equal to the radius.

Plane
F lo
(Fixed point)

Fig. 13.1
(Moving
point)
for F
ree
EQUATION OF A CIRCLE By the equation of a circle is meant the equation of the circumference; it
r
is a relation between the coordinates v, y of the moving point P, involving some constants
You
oks

depending upon the position of the centre and the length of the radius. In set theoretical
notations it is the set of all points lying on the circumference of the circle.
eBo

13.2 STANDARD EQUATION OF A CIRCLE


In this section, we will find the equation of any circle whose centre and radius are given.
ad
our

Let C be the centre of the circle and its coordinates be (/i, k). Let the radius of the circle be a and let
P (.V, y) be any point on the circumference. Then,
CP =a
Re
dY

=> CP^=a^
=> {x-h)'^+{y-kf =a‘^
Fin

This is the relation between the coordinates of any point on the circumference and hence it is the
required equation of the circle having centre at (h, k) and radius equal to a.
note 1 The above equation is known as the central form of the equation of a circle.
13.2 APPLIED MATHEMATICS-XI

\0IE2 If the centre of the circle is at the origin and radius is a, then from the above form the equation of
the circle is = a^.
ILLUSTRATION 1 Find the equation of a circle whose centre is (2, -3J and radius 5.
SOLUTION The equation of the required circle is
(x-2)^ + (y+3)^ =5^ or, + i/^-4.v+6y-12 =0.
ILLUSTRATION 2 Find the equation of a circle whose radius is 6 and the centre is at the origin.
SOLUTION The equation of the required circle is
= 6^ or, -Y^ + y^ = 36.
13.3 SOME PARTICULAR CASES

The equation of a circle with centre at {h, k) and radius equal to a, is


{x-h)^ =0^

w
-(i)
(i) When the centre of the circle coincides with the origin (Fig. 13.3).

Flo
y,

ee
Fr
a

X' for X
ur
a
O
ks
Yo
oo
eB

Fig. 13.3
r
ou
ad

In this case, h -k = 0. Putting h =0, k = 0in equation (i), we obtain y^ + y^ = a^ as the equation
Y

of the circle having centre at the origin and radius equal to 'a'.
(ii) When the circle passes through the origin (Fig. 13.4):
Re
nd

Let O be the origin and C {h, k) be the centre of the circle. Draw CM 1 OX.
Fi

Using Pythagoras Theorem in A OCM, we obtain


OC^ =OM^+CM^
a^ =h^ + k^
Y

a
k

X' O h M X

Y'

Fig. 13.4
CIRCLE 13.3

The equation of the circle (i) then becomes


(x-h)'^ + {y-k)^ = or, - 2hx - 2 ky = 0.
(iii) When the circle touches x-axis (Fig. 13.5):
Let C (h, k) be the centre of the circle. Since the circle touches the A:-axis. Therefore, a = k
Hence, the equation of the circle is
Y

C(K

w
k

F lo
O M X
X'

ee
Y
Fig. 13.5

Fr
(x-h)'^ + iy-a)'^=a^ or, x“^ + y^ -2hx-2ay + h^ =0 for
ur
(iv) When the circle touches y-axis (Fig. 13.6):
Let C (K k) be the centre of the circle. Since the circle touches the y-axis. Therefore, h = a
s
ok
Yo

Hence, the equation of the circle is


o

(x - a)^ + (y - k)^ = or, x^ + y^ - 2ax-2ky + k^ = 0.


eB

Y
r
ad
ou
Y

C(h,k)
Re
nd

M -♦

a
Fi

O X
O X X'
X'

r r

Fig. 13.6 Fig. 13.7

(v) When the circle touches both the axes (Fig. 13.7):
In this case we have, h=k= a

Hence, the equation of the circle is


(x - a)^ + (y - a)'^ = or, x^ +y^ -2ax-2ay + a^ = 0.
13.4 APPLIED MATHEMATICS-XI

(vi) When the circle passes through the origin and centre lies on x-axis (Fig. 13.8):
In this case, we have k=0 and h = a.
Hence, the equation of the circle is
(.Y-fi)^ +(y-0)^ = a ^ or, .Y^ + - lax = 0.
y,

X'

o X

w
y'

Fig. 13.8

F lo
(vii) When the circle passes through the origin and centre lies on y-axis (Fig. 13.9):
In this case, we have h = 0 and k=a.

ee
Hence, the equation of the circle is

Fr
(x-0)^ + iy-a)^ a^ or, x^ + y^ - lay = 0.
Y for
ur
s
ook
Yo

c (0,«)
eB
our
ad

X' o X

r
Y

Fig. 13.9
Re
nd

IMPORTANT POINTS TO REMEMBER


Fi

(i) When a circle touches A-axis, then its radius is equal to the absolute value of the
y-coordinates of the centre.
(ii) When a circle touches y-axis, the .Y-coordinates of its centre, in magnitude, is equal to the
radius.

(iii) When a circle touches x-axis at the origin, then its centre lies on y-axis and absolute value
of y-coordinates of the centre is equal to the radius.
(iv) When a circle touches y-axis at the origin, then its centre lies on .Y-axis at a distance equal
to the radius of the circle.
(V) When a circle touches both the axis, then the coordinates of its centre are (± a, ± a), where
a is the radius of the circle.
(Vi) When a circle touches a line, then length of the perpendicular from its centre on the given
line is equal to the radius of the circle.
CIRCLE 13.5

ILLUSTRATIVE EXAMPLES

Type I ON FINDING THE EQUATION OF A CIRCLE WHEN ITS CENTRE AND RADIUS ARE KNOWN
EXAMPLE 1 Find the equation of the circle whose centre is (2, - 3) and radius is 8.
SOLUTION The equation of the circle is
{x-2f +
.2
[Using:(A--/0^+(y-fc)^=fl^l
=> (-Y - 2)^ + (i/+ 3)^ = 8^ or, + 1/^-4 X + 6 y-51 = 0.
EXAMPLE 2
Find the equation of the circle u’hich passes through the point of intersection of the lines
3x-2 \j-1 = 0 and 4 x + y - 27 = 0 and whose centre is (2, - 3).
SOLUTION Let P be the point of intersection of the lines AB and LM whose equations are
respectively

w
3 -A - 2 y - 1 = 0 ...(i) and 4.r + y - 27 = 0 ...(ii)

Solving (i) and (ii), we get x = 5, y = 7. So, coordinates of P are

Flo
B
3.Y-2y-l =0
(5, 7). Let C ( 2, - 3) be the centre of the circle. Since the circle 4.V + y - 27 = 0
M
passes through P.

ee
P

Fr
CP = Radius
L

■J(5-2f+(7 + 3f = Radius
for
ur
Radius = Vl09.
k s
Yo

Thus, the required circle has its centre at C (2, - 3)


oo
eB

and, radius - Vl09. So, its equation is

(x - 2)^ + (y + 3)^ = (VlW)^ or, x^ + y^ - 4 X + 6 y - 96 = 0


r
ou
ad

EXAMPLE 3 Find the equation of the circle having centre at (3,-4) and touching the line
Y

5x + 12y-12 = 0.
SOLUTION Let C(3,-4) be the centre of the circle. If the line 5x + 12y-12 = 0 touches the
nd
Re

required circle at R.Then, CP is perpendicular to the line and is equal to the radius of the circle.
Radius = CP = Lengtlr of perpendicular from C(3,-4) on the line5x + 12y-12 = 0
Fi

iP

5a-+12i/-12=0
13.6 APPLIED MATHEMATICS-Xl

5x 3 + 12X-4-12 45
=> Radius =
Vs^ + 12^ 13

45
Thus, the required circle has its centre atC (3, -4) and radius = —.
13

45'!^
Hence, its equation is (x - 3)^ + (y + 4)^ = 13

EXAMPLE 4 If the equations of the two diameters of n circle are x - \j = 5 and 2.v + y = Aand the radius of
the circle is 5, ifnd the equation of the circle.
SOLUTION Let the diameters of the circle be AB and LM whose equations are respectively

X - y = 5 ... 1
2x + y = 4 ...(ii)
Solving (i) and (ii), we get : x = 3 and y~-2.

w
Since the point of intersection of any two diameters of a circle is its centre. Therefore,
coordinates of the centre of the required circle are (3, - 2) and its radius is 5 (given).
Hence, its equation is

Flo
(x “ 3)^ + (y + 2)^ =5^ or, - 6 x + 4 y-12 = 0

ee
EXAMPLE 5 Bind the equation of a circle whose diameters are 2x - 3y + 12 = 0 and x + 4y - 5 = 0 and

Fr
area is 154 square units.
SOLUTION The centre is the point of intersection of diameters. Solving 2x - 3y + 12 = 0 and
X + 4y - 5 = 0, we get x = - 3 and y = 2. So, the coordinates of centre are (-3, 2). Let r be the
for
ur
radius of the circle. Then,
Area = 154 => nr^ = 154 => -—xr^= 154 z=> r = 7
ks
7
Yo

Hence, the equation of the required circle is (x + 3)^ + (y - 2)^ = 49.


oo

EXAMPLE 6 Find the equation of a circle of radius 5 whose centre lies on x-axis and passes through the
eB

point (2,3).
SOLUTION Let the coordinates of the centre of the required circle be C {a, 0). Since it passes
r

through P (2, 3).


ou
ad

CP = radius
CP = 5
Y

^2)2+(0-3)2 = 5
nd
Re

(f?-2)2 + 9 = 25=>fl-2=±4 => a = 6 or, a -2


Fi

y,

P(2,3)

o
X' C(a, 0) X

y”

Fig. 13.12
CIRCLE 13.7

Thus, the coordinates of the centre are (6,0) or (-2,0). Hence, the equations of the required circle
are

(Y-6)2+(I/-0)2 5^ and (.y + 2)^ + (i/- 0)^ = 5^


=> Y2 + y2_i2j- + ll = 0 and Y 2 + 1/^ + 4y - 21 = 0
Ti/pe 7/ ON FINDING THE CENTRE AND RADIUS OF A GIVEN CIRCLE
EXAMPLE 7 Find the centre and radius of each of the following circles:
(i) x2+(y + 2)2 = 9 (ii) j2 + y2 - 4y + 6y = 12
(hi) (y + 1)2 +(y-l)2 = 4 (iv) Y^ + y2 + _ 4y + 4 = o.
SOLUTION (i) We have,
+ +(y+2)=^ = 9 => (,T-0)2 + {y-(-2)f = 3^
Comparing this equation with (y - ar {}/ ~ i’) 2, we find that the given circle has its centre at

w
= r

(0, -2) and radius 3.


(ii) We have, y^ + y^ - 4y + 6y =12

Flo
=> (y2 -4y) + (y2 + 6y) = 12
(y^ - 4y + 4) + (y2 + 6y + 9) =12 + 4 + 9

ee
=>

(Y-2)2+(y+3)2 =52 => (Y-2)2 + {y-(-3)]2 = s2

Fr
Comparing this equation with (y -a) +{y -b) = 1-2, we find that the given circle has its centre
2 2

at (2, -3) and radius 5. for


ur
(hi) We have, (y +1)2 + (y -1)2 = 4
|,V-(-l)l2+(y-l)2 = 2^
ks
Yo

Clearly, the given circle has its centre at (-1,1) and radius 2.
oo

(iv) We have, y-2 + y^ + 6y - 4y + 4 = 0


eB

^ (y2 + 6y) + (y2 - 4y) = - 4


=> (y2 + 6y + 9) + (y2 - 4y + 4) = - 4 + 9 + 4
r

=> (Y+s2)+(y-2)2 = 32 {,v-(-3)}2+(y-2)2 = 32.


ou
ad
Y

Clearly, this circle has its centre at (-3,2) and radius 3.


Type III ON FINDING THE EQUATION OF A CIRCLE SATISFYING SOME GIVEN GEOMETRICAL
nd
Re

CONDITIONS

EXAMPLE 8 Find the equation of the circle which touches:


Fi

(i) the x-axis and whose centre is (3, 4) (ii) the x-axis at the origin and ivhose radius is 5
(Hi) both the axes and whose radius is 5 (iv) the lines x = 0, y = 0 and x~a.

Y.

0 P X

Fig. 13.13
13.8 APPLIED MATHEMATICS-XI

SOLUTION (i) Clearly, radius=CP = 4 (Fig. 13.13) and the coordinates of the centre are (3,4).
Hence, the equation of the required circle is
(x - 3)^ + (y -4)^ =4^ or, +y^ -6x-8y + 9=0
(ii) Since the circle touches the x-axis at the origin and has radius 5. So, the coordinates of the
centre are (0,5) as shown in Fig. 13.14. Hence, the equation of the circle is
(a:-0)^+(y-5)^ =5^or, x^+y^-10y = 0.
Yi

C(0,5)

w
F lo
O,

Fig. 13.14
X

e
Fre
(iii) The circle touches both the axes and has radius 5. So, the coordinates of the centre are (5,5)
and radius = 5 as shown in Fig. 13.15. So, the equation of the required circle is
for
(a: -5)^ + (y -5)^ = 5^ or, + y^ -lOx -lOy + 25 =0.
Since, the circle may lie in any one of the four quadrants. So, there are four such circles. The
r
equations of these circles are given by + y^ ± lOx ± lOy + 25
You

= 0.
oks
eBo

Yi
ad
our

x = 0

x = a
dY
Re

an
c(an,a/2)
Fin

5 C(5,

5 X' O y=o X
a

O, X Y'

Fig. 13.15 Fig. 13.16

(iv) The circle touches the coordinate axes and the line x = a as shown in Fig. 13.16. So, the
centre of the required circle is at {a/2, a/2) and radius = fl/2.
Hence, its equation is {x-a/2)^ + (y-a/2)^ ={a/2)^.
There may be two such circles, one lying above :c-axis and other below x-axis. The circle lying
below x-axis has its centre at {a/2, -a/2) and radius a/2. The equation of this circles is
CIRCLE 13.9

a ( a n2 a
X — + 1/ +—
2j r 2 2j
^2 a
Hence, tlie equations of the circles are V .t —2 + V !/ +”
/ 2 u
EXAMPLL9 Find the equations of the circles which passes through two points on the x-axis zvhich are at
distances 4 from the origin and whose radius is 5.

ow
5
C(0, 3)

e
X' X

re
A'(-4,0)

rFl
O A(4, 0)

F
C'(0, -3)

r
ou
fo
ks
r
oo

Fig. 13.17
Y
eB

SOLUTION As is evident from Fig. 13.17 there are two circles which pass through two points
A and A' on y-axis which are at a distance 4 from the origin. The centres of these circles lie on
r

y-axis.
ou

Applying Pythagoras Theorem in A OAC, we get


ad
Y

AC^ - OA^+OC^ => 5^ = 4^+OC^ OC = 3.


So, the coordinates of the centres of the required circles are C (0, 3) and C (0, - 3).
d

Hence, the equations of the required circles are


Re
in

(x - 0)^ + (y + 3)^ =5^ or, x^ + y^ + 6y -16 = 0.


F

EXAMPLE 10 Find the equation of the circle which passes through the origin and cuts off intercepts 3
and 4 from the positive parts of the axes respectively. y,
SOLUTION Let the circle cuts off intercepts OA and OB from
OX and OY respectively. It is given that OA - 3 and OB - 4.
OL=-and, CL =2
2 B

In A OLC, we have
OC^ = OL^ + LC^ 4

OC^ = 3^ + 2^
2)
O X
oc = 5
2
13.10 APPLIED MATHEMATICS-Xl

Thus, the required circle has its centre at (3/2,2) and radius 5/2.
/
? /5'\^
Hence, its equation is .r - — + (y-2) = ^2§ ■
EXAMPLE 11 Find the equation of a circle which touches \j-axis at a distance of 4 units from the origin
and cuts an intercept of 6 units along the positive direction ofx-axis.
SOLUTION The given circle touches y-axis at L(0, 4) and cuts an intercept AB = 6 along the
positive direction of x-axis. As shown in Fig. 13.19, there are two such circles.
y

L
5 C(5,

A 5,
X' 4 X

w
O 3 !M

L'

F lo
e
Fre
r

Fig. 13.19
for
In A CAM, we have
CA^ = CM^ + AM^
r
You

CA^ =4^ + 3^
oks
eBo

=> CA = 5

Also, CL = CA = 5.

Thus, the coordinates of the centres are (5,4) or (5, - 4) and radius = 5.
ad
our

Hence, the equations of the required circles are


{.r-5)^+(i/ +4)^ = 5 => -lOx + 8y + 16 = 0.
Re
dY

EXERCISE 13.1
Fin

1. Find the equation of the circle with:


(i) Centre (-2,3) and radius 4. (ii) Centre (a, b) and radius ^a^ + b^.
(iii) Centre (0, -1) and radius 1. (iv) Centre {a cos a, a sin a) and radius a.
(v) Centre (a, a) and radius a.
2. Find the centre and radius of each of the following circles:
(i) (Y-l)2+y2 = 4 (ii) (Y+5)2+(y + l)2 ^ 9
(iii) Y^ + y^ - 4y + 6y = 5 (iv) Y^ + y^ - Y + 2y - 3 = 0.
3. Find the equation of the circle whose centre is (1, 2) and which passes through the point
(4,6).
4. Find the equation of the circle passing through the point of intersection of the lines
Y+ 3y = 0 and 2Y-7y = 0 and whose centre is the point of intersection of the lines
Y + y + 1 = 0 and y - 2y + 4 = 0.
13.11
CIRCLE

5. Find the equation of the circle whose centre lies on the positive direction of y-axis at a
distance 6 from the origin and whose radius is 4.
6. If the equations of two diameters of a circle are lx + y = 6 and 3a: + 2y = 4 and the radius is
10, find the equation of the circle.
7. Find the equation of a circle
(i) which touches both the axes at a distance of 6 units from the origin,
(ii) which touches .t-axis at a distance 5 from the origin and radius 6 units
(iii) which touches both the axes and passes through the point (2,1).
(iv) passing through the origin, radius 17 and ordinate of the centre is -15.
8. Find the equation of the circle which has its centre at the point (3,4) and touches the straight
line 5a: + 12y -1 = 0.

ow
9. Find the equation of the circle which touches the axes and whose centre lies on x-2y = 3.
1(1. A circle whose centre is the point of intersection of the lines 2.v-3y + 4 = 0 and
3.r + 4y - 5 = 0 passes through the origin. Find its equation,
n. Find the equations of the circles touching y-axis at (0,3) and making an intercept of 8 imits

e
on the A-'-axis.

re
rFl
12. Find the equations of the circles passing through two points on
y-axis at distances 3 from

F
the origin and having radius 5.
13. If the lines 2a: - 3y = 5 and 3a: - 4y = 7 are the diameters of a circle of area 154 square units,

r
then obtain the equation of the circle.
fo
ou
14. If the line y = VSa: -i- k touches the circle + y^ = 16, then find the value of k.
ks
15. Find the equation of the circle having (1,-2) as its centre and passing through the
oo

intersection of the lines 3.v-i- y =14 and 2A: + 5y =18.


16. If the lines 3a: - 4y + 4 = 0 and 6.t - 8y -7 = 0 are tangents to a circle, then find the radius of
Y
eB

the circle.
ANSWERS
ur

1. (i) (x + 2)2+(y-3)2=16 (ii) + y^ - 2^x - Ihy = 0 (iii) a:^ + y^ + 2y = 0


ad
Yo

(iv) x^ + x/ {la cos a). X - (2i7 sin a). y = 0 (v) + y^ - - lay = 0


Vl7
d

1
2. (0 (1,0); 2 (ii) (-5,-1); 3 (iii) (2, -3);3^[l (iv) -,-l —
^ ^2 ) 2
Re
in

3. x^ + y^ - 2x - 4y - 20 = 0 4. x^ + y^ + 4x - 2y = 0 5. x^+y'^ - 12y 20 = 0
F

6. x^ + y^ - 16x + 20y + 64 = 0 7. (i) + 1/2 _i2x-12y+36=0


(ii) x^+y^-10x-12y + 25=0 (iii) .x^ + y^ - 2x - 2y + 1 = 0, x^ + y^-lOx - lOy + 25 = 0
(iv) x^ + y^ ± 16x -1- 30y = 0 8. 169 (x^ + y^ - 6x - 8y) + 381 = 0
9. x^ + y^ + 6x + 6y + 9 = 0 or x^ + y^ - 2x + 2y + 1 = 0
1 22 485
10. X-H — + V
17 r 17 289

11. x^ + y^±10x-6y + 9=0 12. x^+y2±8x-9=0


13. x^+y^-2x + 2y-47=0 14. jt=±8

15. x^-hy^-2x + 4y-20=0 16. 3/2


13.12 APPLIED MATHEMATICS-XI

13.4 GENERAL EQUATION OF A CIRCLE

THEOREM Prove that fheeqiiafioux~ + xr + 2gx + 2jy + c = 0 always represents a circle zuhose centre is
(- gr -f) and radius = -^jg^ + ~ c.
I^RCXIF The given equation is x^+y~+ 2 gx + 2fy + c~0 ...(i)
^ {x^ + 2gx + g^)+{i/ + 2fy+f^) = g^+f^ - c

w
2

(x + gf + {y+f)^= ■ -C ■

e
ro
re
This is of the form (x - li)^ + (y - k)^ = a
which represents a circle having centre at {h, k) and
radius equal to a.

F
Fl
Hence, the given equation (i) represents a circle whose centre is at
i-g'-f) i-e-
1

u 1
— Coefficient of at, — Coefficient of i/
2 2
^

sr
\n- Coeff. of .T ^ + (\
Radius = -^g^ + / ^
\'y

ko
and, - Coeff. of y ^
o
- Constant term
I/I2 > 2
of ,

Q.E.D.

a circle of radius ^g^ + f ^


o
NOTE I The equation x~ + y~+ 2 gx+2fy+c=0 represents
Y
- c.

y y
erB

+/‘ - > h, then the radius of the circle is real and hence the circle is also real.
Ifg-^f- - c - 0, then the radius of the circle is zero. Such a circle is knozun as a point circle.
uY

ifg^ + f 2. c < 0, then the radius +f ^ c of the circle is imaginary but the centre is real. Such a
circle is called an imaginary circle as it is not possible to drazv such a circle.
ad
do

NOTE 2 Special features of the general equation x^ + y^ + 2 gx + 2 fy + c = 0 of the circle are:


(i) it is quadratic in both x a)id y.
in

(ii) Coefficient ofx^ = Coefficient ofy^.


Re

(Hi) there is no term containing xy i.e., the coefficient of xy is zero,


F

(iv) it contains three arbitrary constants viz. g,fand c.


NOTE 3 The equation ax^ + m/^ + 2 gx-^ 2 fy c= 0,a 0 also represents a circle. This equation can
also be written as
^g 2/
— .V + -
y + - = 0.
a a a

The coordinates of the centre of the circle are (-gla, -f/a) and, radius = Ig^ +
/“ c
V a^ a
2

NOTE 4 On comparing the general equation x^ + y^ + 2 gx + 2 fy + c = 0 of a circle with the general


equation of second degree ax^ + 2 hxy + by^ -\-2gx + 2fy + c = 0, zvefind that it represents a circle ifa =b
i.e., coefficient ofx^ = coefficient ofy^ and h = 0 i.e., coefficient of xy = 0.
NOTH While solving problems it is advisable to keep the coefficient ofx^ and \/ unih/.
CIRCLE 13.13

ILLUSTRATIVE EXAMPLES

Ti/p)e I ON FINDING THE CENTRE AND RADIUS OF A CIRCLE WHEN ITS EQUATION IS GIVEN
RESULT The coordimtes of the centre of the circle x~ + + 2gx + 2fy + c = 0are
f 1
- Coefficient of .r, - — Coefficient of i/
2 2

fl n
and, Radius Coefficient of .v — Coefficient of i/ - Constant term
vU 2 ■^)
EXAMPlTi 1 Fmd the centre and radius of the circle x^ + y^ - 6.v + 4i/ -12=0.
SOLUTION The coordinates of the centre of the circle x^ + \r - 6.v + 4y -12 = 0 are
f 1 1. 1. 1X
— Coeff. of X, — Coeff. of i/ i.e. — x-6, — x4 = (3,-2)
2 2 ■ J I 2 2

w
^ 6)^
and, Radius =
^ 1)
+
-(-12) = ^9 + 4 + 12 = 5.
EXAMIM.I- 2

F lo
Find the centre and radius of the circle given by the equation
2,v^+2y^ + 3x+4y + -8 = 0.

e
Fre
0 'y
SOLUTION In the given equahon the coefficients of .v and y“ are not unity. So, we re-write the
for
equation to make the coefficients of .v and y“ unity.

We have, 2.x^ + 2y^ + 3x + 4y + - = 0 => + ^y^ + —2 X + 2i/-16


+— = 0.
r
You
oks

So, the coordinates of the centre are (- 3/4, -1) and, Radius = jf-f— = 1.
eBo

KUJ 16
Type II ON FINDING THE EQUATION OF A CIRCLE SATISFYING GIVEN CONDITIONS
EXAMPLE 3 Find thee^quation of the circle xvhose centre is at the point (4,5) and which passes through
our
ad

the centre of the circle x^ + y^ - 6x + 4y -12 = 0.


SOLUTION The coordinates of the centre of the circle x^ + i/^ - 6x + 4i/ -12 = 0 are (3, - 2).
Therefore, the required circle passes through the point (3, - 2) and has its centre at the point
dY
Re

C (4,5). So, its radius is equal to


C Cl = ^(4-3)2+(5+ 2)- = J50
Fin

Cj (3, -2)j x^y^6x+4ijA2=0

Hence, the equation of the required circle is


(x - 4)2 + {y -5)2 = (V^)2 or, x2 + _ g_^ _ _9 ^q
13.14 APPLIED MATHEMATICS-XI

7 9
EXAMl’l.t4 Find the equation of the circle concentric with the circle 2x + Ixj + 8.v + lOy ~ 39 = 0
and having its area equal to 16 n square units.
SOLUTION The equation of the given circle is
2x^ + 2y^ + 8.V + lOy - 39 ^ 0 => + y^ + 4.v + 5y - 39/2 = 0.
The coordinates of its centre are (- 2, -5/2). The required circle is concentric with the above
circle, therefore the coordinates its centre are (- 2, -5/2).
Let r be the radius of the required circle. Then, its area is Tir . But, it is given that its area
isl6jrsq. units.
Tcr 2 = 16 n => r = 4

ow
Hence, the equation of the required circle is
{X + 2f + (y + 5/2)^ = 4^ or, 4x^ + 4y^ + 16r + 20y - 23 = 0.
Type in ON FINDING THE EQUATION OF A CIRCLE PASSING THROUGH THREE GIVEN POINTS
EXAMPLE 5 Find the equation of the circle that passes through the points (1, 0), (-1, 0) and (0,1).

e
re
SOLUTION Let the required circle be + y^ + 2^.v + 2/y + c = 0

rFl
It passes through (1, 0), (-1, 0) and (0, 1). Therefore, on substituting the coordinates of three

F
points successively in equation (i), we get
1 + 2^ + c = 0 ...(ii), 1 - 2^ + c = 0 ...(iii), 1 + 2/ + c = 0 -(iv)

r
ou
Subtracting (iii) from (ii), we get
4g = 0 => g = 0
k sfo
Putting ^ = 0 in (ii), we obtain c = -1.
oo

Now, putting c =-l in (iv), we get / = 0.


Y

Substituting the values of g, f and c in equation (i), we obtain the equation of the required circle
B

as x^ +y^ =1.
re

EXAMPLE 6 Find the equation of the circle which passes through the points (5, - 8), (2, - 9) and (2, V.
Find also the coordinates of its centre and radius.
ou
Y
ad

SOLUTION Let the required circle be


x^ + y^ + 2^'X + 2/y + c = 0 -(i)
d

It passes through the points (5, - 8), (2, - 9) and (2,1). Therefore,
in
Re

89 + lOg - 16/ + c = 0 ...(ii)


85 + 4g - 18/ + c = 0 ...(iii)
F

5 + 4g+2/ + c= 0 ...(iv)
Subtracting (iii) from (ii), we obtain
4 + 6^+2/ = 0^ 2+3^ + / = 0 -(V)
Subtracting (iv) from (iii), we get
80 + 0^-20/ = 0=>/-4
Putting/ = 4 in (v), we get g = -2. Putting/ = 4, g = -2 in (iv), we get
5-8 + 8 + c = 0 ^c=-5

Substituting the values of g,f and c in equation (i), we obtain the equation of the required
circle as
X
^ + y^ - 4x + 8y - 5 = 0.
The coordinates of the centre are (- g, -/) i.e (2, - 4).
and, Radius f4 + \6 + 5 = 5.
CIRCLE 13.15

X 1/
EXAMPLE 7 The straight line — + — = 1 cuts the coordinate axes at A and B. Find the equation of the
a b

circle passing through O (0, 0), A and B.

rw
SOLUTION The Straight line - + -
^ = 1 cuts the coordinate axes at A {a, 0) and B (0, b).
a b

Let -Y^ + + 2fy + c = 0


be the circle passing through O, A and B. Then,
0+ +c = 0 ...(h)
^ + 2ga + c = 0

e
a ...(hi)

h^ + 2fb + c = 0 ...(iv)

luo
r
Solving (ii), (iii) and (iv), we obtain

F
^ f _ ^ and c = 0.
g = -
2'" ~ ~2

oF
Substituting these values in (i), we obtain the equation of the required circle as

rs
-Y^ + y^ - ax - by = 0

k
X u

ALITHK The line represented by the equation — + ^ = 1 meets the coordinate axes at A (a, 0) and

o
o
B(0,1;). Clearly, ZAOB = 90°. So, AB is a diameter of the circle such that
AB=^(a-0f+{Q-bf f
o
Y
Y
rB
ue
od
ad
in
Re
F

Radius = -● AB =
2

The centre C of the circle is the mid-point of AB and so its coordinates are
a +0 a b

2 2 2'2j'
Hence, the equation of the circle is
a r
-Y — + 1/ — or, .Y^ + y^ -ax-by = 0
2) I.-" 2 U
13.16 APPLIED MATHEMATICS-XI

TypcIV ON CONCYCLIC POINTS


EXAMPLE 8 SJtow that the points (9,1), (7, 9) (- 2,12) and (6,10) are concyclic.
SOLUTION Let the equation of the circle passing through (9,1), (7, 9) and (-1,12) be
-v“ + + 2 + 2 + c = 0 ...(i)

Then, 82 + 18g + 2/ + c = 0 ...(ii)


130 + 14^ + 18/ + c = 0 ...(iii)
148 - 4^ + 24/ + c = 0 -(iv)
Subtracting (ii) from (iii), we get

w
48 - 4^ + 16/ = 0 => 12 - g + 4/ = 0 ...(V)
Subtracting (iii) from (iv), we get
18-18g+6/ = 0=>3-3^ + / = 0 ...(Vi)

o
Solving (v) and (vi) as simultaneous linear equations in g and / we get: /=-3,g = 0.

e
re
Putting / = - 3, g = 0 in (ii), we get
82 + 0 - 6 + f = 0 =>

rFl
c = -76

F
2 2
Substituting the values of g,/and c in (i), we get a: + y - 6y 76 = 0 as the equation of
the circle passing through points (9,1), (7, 9) and (- 2,12).

or
ou
Clearly, point (6,10) satisfies this equation. Hence, the given points are concyclic.
ksf
type V ON FINDING THE EQUATION OF A CIRCLE SATISFYING THREE GIVEN CONDITIONS
EXAMPLES find the equation of the circle which passes through the points (1,-2) and (4,-3) and
oo

has its centre on the line 3 .t + 4 y = 7.


Y

SOLUTION Let the equation of the required circle be


B

.v“ + y^ + 2 g.r + 2 // + c = 0 ...(i)


re

It passes through (1, - 2) and (4, - 3).


oYu

5 + 2g-4/ + c = 0 ...(h)
ad

and. 25 + 8g-6/+c = 0 ...(hi)


The centre of (i) lies on 3 .v + 4 y = 7.
d

-3g - 4/ = 7 ...(iv)
in
Re

Subtracting (h) from (iii), we get


●●●(V)
F

20 + 6g-2/ = 0=>10+ 3g-/ = 0


Solving (iv) and (v) as simultaneous equations, we get
47
g = - and / = —
15 ●' 5
Substituting the values of g and/in (ii), we get
94 12
+ C = n0=> C = 55 :
11
5 -
15 5 15 3

Substituting the values of g,/and c in (i) we obtain the required equation of the circle as
94 6 11
= 0 or, 15(x^+y^) - 94 x + 18 y + 33 = 0
2
X
+ y- — X + - 1/ + —
15 5 ' 3
EXAMPLE 10 Find the equation of the circle circumscribing the triangle formed by the Ihies x + y = 6,
2 X + y = 4 and x + 2 y =5.
SOLUTION Let the equations of sides AB, BC and CA of A ABC are respectively
X +y=6 2x + y = 4 ...(h) and x + 2y =5 ...(hi)
CIRCLE 13.17

Solving (i) and (iii), (i) and (ii); (ii) and (iii) we get the coordinates of B and C. The coordinates
A, B and C are (7, - 1), {- 2, 8) and (1, 2) respectively.
Let the equation of the circumcircle of A ABC be
.v“ + 1/^ + 2 ^.Y + 2 ^ + c = 0 ...(iv)

It passes through the points A (7, -1), B (- 2,8) and C (1,2). Therefore,
50 + Ug - 2/ + c = 0 ..●(V)
68 - 4g + 16/ + c = 0 ...(vi)
5 + 2^‘^ + 4/ + c = 0 ...(vii)
Subtracting (v) from (vi), we get
18 - 18 g + 18/ = 0 ^ 1-g+f = 0 ...(viii)
Subtracting (v) from (vii), we get: -45-12^+6/ = 0 ...(ix)
Solving (viii) and (ix), weget: g =-17/2, f = -19/2.

w
Putting the values of ^ and fm (v), we get c = 50.

I.
A
- 17 A- - 19 y + 50 = 0

Find the coordinates of the centre and radius of

(i) A^ + y^ + 6a - 8y - 24 = 0
F lo
Substituting the values of g, f and c in (iv), the equation of the required circumcircle is
^

ree
EXERCISE 13.2

each of the following circles :

F
(ii) 2a^ + 2y^ - 3a + 5y = 7
for
(iii) -^(a^ + y^) + a cos 6 + y siiY 0 - 4 = 0 (iv) A^ + y^ -nx -by = 0
r
2. Find the equation of the circle passing through the points:
You
oks

(i) (5,7), (8,1) and (1,3) (ii) (1, 2),(3, -4) and(5, -6)
eBo

(iii) (5,-8), (-2, 9) and (2,1) (iv) (0, 0),(-2,1) and(- 3, 2)


3. Find the equation of the circle which passes through (3, -2), (-2,0) and has its centre on the
line 2a - y = 3.
ad
our

4. Find the equation of the circle which passes through the points (3,7), (5,5) and has its centre
on the line a - 4y = 1.
5. Show that the points (3, - 2), (1, 0), (-1, - 2) and (1,-4) are concyclic.
Re
dY

6. Show that the points (5, 5), (6, 4), (-2, 4) and (7,1) all lie on a circle, and find its equation,
centre and radius.
Fin

7. Find the equation of the circle which circumscribes the triangle formed by the lines
(i) A + y + 3 = 0, A-y + 1 = 0 and a = 3
(ii) 2a + y - 3 = 0, A + y-1 = 0 and 3.\'+2y-5 = 0
(iii) A + y = 2, 3a - 4y = 6 and A - y = O.
(iv) y = A + 2, 3y = 4a and 2y = 3a.
8. Prove that the centres of the three circles x + y - 4a - 6v ●“12 = 0,
A“ + y^ + 2a + 4y -10=0 and x^ + - 10a - 16y -1=0 are collinear.
9 ^ 7 9 9 9
9. Prove that the radii of the circles a + y =1, .y + y - 2a - 6y - 6 = O and a + y - 4a
- 12y - 9 = O are in A.P.
10. Find the equation of the circle which passes through the origin and cuts off chords of
lengths 4 and 6 on the positive side of the .Y-axis and y-axis respectively.
11. Find the equation of the circle concentric with the circle a^ + y^ - 6a -t 12y + 15 = 0 and
double of its area.
13.18 APPLIED MATHEMATICS-Xl

12. Find the equation to the circle which passes through the points (1,1) (2,2) and whose radius
is 1. Show that there are two such circles.

13. Find the equation of the circle concentric with x + y - 4.v - 6y - 3 = 0 and which touches
the y-axis.
14. If a circle passes through the point (0, 0),{a, 0),(0, b), then find the coordinates of its centre.
15. Find the equation of the circle which passes through the points (2, 3) and (4,5) and the
centre lies on the straight line y -4.v +3=0.
ANSWERS

.... 3 5 3 VTo
1. (i)(-3,4);7 ” 7'"7
U 4j
4 ' 4
'a y
(iii) (- cos 0, - sin 0); 3 (iv)

w
.2'2,
2. (i) 3 (x~ + \/) - 29x - 19y + 56 = 0 (ii) + y^ - 22.V - 4y + 25 = 0

Flo
(iii) x^ + y^ + 116.Y + 48y - 285 = 0 (iv) .Y“+y^-3.T-lly = 0

ee
3. + y^ + 3.Y + 12y + 2 = 0 4. + y^ + 6.V + 2y - 90 = 0

Fr
6. + y^ - 4.r - 2y - 20 = 0; (2,1), 5
7. (i) .Y^+y^-6.v + 2y-15=0 (ii) x^ + y^-13x-5y + 16 = 0
for
ur
(iii) + i/+ 4x + 6y -12 = 0 (iv) x^ + y^ - 46x + 22y = 0
10. x^ + y^ - 4x - 6y = 0 11. x^ +y^ -6x + 12y-15=0
ks
Yo

12. x^ + y“ - 4x - 2y + 4 = 0, x^ + y^ - 2x - 4y + 4 = 0
oo

a b\
13. x^ + y^ - 4x - 6y + 9 = 0 15. x^+y^-4x-10y + 25 = 0
eB

14.
2' 2

13.5 DIAMETER FORM OF A CIRCLE


r
ou
ad

THEOREM The equation of the circle drawn on the straight line joining tzvo given points (X|, and
(X2,1/2) (Js diameter is (x - x{) (x - X2) + (y - yO (y - y2> = 0.
Y

OROOl Let A and B be the extremities of the diameter AB having coordinates (.x^, yf) and
nd
Re

(X2,1/2) respectively. LetP (.x, y) be any point on the circle. Join point P to points A and B.Then,
y - j/i y-yi
= Slope of the line AP = and, /»2 = Slope of the line BP =
Fi

x-x-i X-X2
Y

B(x2, 1/2)

X' O X

Fig. 13.22
CIRCLE 13.19

The angle subtended at the point P in the semi-circle APB is a right angle.
J7J] »Z2 = -1
=>
■y-yi ■■ y ~.'/2 = -1
.r - Xi X - X2
(y - yi) (y - yz) = - (^ - '-^i) - -^2)
{.Y - Y-i) (y - Y2) + (y -1/^) (j/ - ^2) ^ 0
This is the required equation of the circle having (y-|, i/|) and (y'2, ^2) coordinates of the
end points of a diameter. Q.E.D.

RlAlARK J If the coorciiimtes of the end points of a diameter of a circle arc given, we can also ifnd the
equation of the circle by finding the coordinates of the centre and radius. The centre is the mid-point of the
diameter and radius is half of the length of the diameter.

w
KHMARK2 Equation (i) can also be written as
x^ +1/-X (xj + Y2) -1/ (i/i +1/2) + ^T^2 + yi}/2 = ^
x^ +y^ -X {Sum of the abscissae) - y (Sum of the ordinates) + Product of the abscissae

Flo
or.

+ Product of the ordinates = 0.

ee
ILLUSTRATIVE EXAMPLES

Fr
FX AMI’LI: I Find the equation of the circle, the coordinates of the end points ofzvhose diameter are (-1,2)
and (4, -3).
for
SOLUTION We know that the equation of the circle described on the line segment joining
ur
(xi, y|) and {x2,1/2) a diameter is (y - xf (x - Y2) + (1/ - i/^) (1/ - 1J2) = 0.
Here, xq = -1, .T2 - 4, t/i = 2 and P2 = - 3.
ks

So, the equation of the required circle is


Yo
oo

(x + 1) (x - 4) + (y - 2) (1/ + 3) = 0 or, x“ + y^ - 3x + 1/ -10 = 0.


eB

EXAMPLE 2
Find the equation of the circle drawn on the intercept made by the line 2x + 3y = 6 between
the coordinate a.xes as diameter.
r

SOLUTION The line 2x + 3y = 6 meets x and y-axes at A (3, 0) and B (0, 2) respectively. Taking
ou
ad

AB as a diameter, the equation of the required circle is


[Using: (x - xf (x - X2) + (y - 1/1) (y - 1/2) = 0]
Y

(.^_3)(x-0)+(y-0)(y-2) = 0
or,x^ + y^ - 3x - 2y = 0
nd
Re

EXAMi'LES Find the equations of the circles drawn on the diagonals of the rectangle as its diameter
Fi

whose sides arc x = 6, x = - 3, y = 3 and y = -1.

D(-3,3) >/ = 3 C {6, 3)

.t = -3
-V = 6

X' O X

A (-3,-1) y = -i

y-

Fig. 13.23
13.20 APPLIED MATHEMATICS-XI

SOLUTION Let the sides AB, BC,CD and DA of the rectangle ABCD be represented by the
equations y = -1, a' = 6, y = 3 and x = - 3 respectively. Then, the coordinates of the vertices are
A {- 3,1), B (6, -1), C (6, 3) and D {- 3, 3).
The equation of the circle with diagonal AC as diameter is
(x + 3) (x - 6) + (y + 1) (y - 3) ^ 0 or, x^ + y^ - 3x - 2y ~ 21 = 0
The equation of the circle with diagonal BD as diameter is
(x - 6) (x + 3) + (y + 1) (y - 3) = 0 or, x~ + y^ - 3x - 2y - 21 = 0.
EXERCISE 13.3

ow
1. Find the equation of the circle, the end points of whose diameter are (2, - 3) and (- 2, 4).
Find its centre and radius.

e
2. Find the equation of the circle the end points of whose diameter are the centres of the
circles x^ + y^ + 6x- 14y - 1 = 0andx“ + y“ - 4x+ lOy

re
- 2 = 0.

3. The sides of a square are x = 6, x = 9, y = 3 and y = 6. Find the equation of a circle drawn

Frl
F
on the diagonal of the square as its diameter.
4. Find the equation of the circle circumscribing the rectangle whose sides are x - 3y = 4,
ou
3x + y = 22, X - 3y = 14 and 3x + y = 62.

sor
5. Find the equahon of the circle passing through the origin and the points where the line
3x + 4y = 12 meets the axes of coordinates. kf
6. Find the equation of the circle which passes through the origin and cuts off intercepts a and
oo
b respectively from x and y-axes.
Y

7. Find the equation of the circle whose diameter is the line segment joining (-4, 3) and
B

(12, -1). Find also the intercept made by it on y-axis.


ANSWERS
re
oY

1. x^ + y^ -y-16 =0; oi2j' 2. x^ + y^ + X - 2y - 41 = 0


u

2
ad

3. x^+y^-15x-9y + 72 = 0 4. x^ + y^ - 27x - 3y + 142 = 0


d

5. x^ + xp 4x - 3y = 0 6. yp- + y^ ±ax±by = Q
in

7 7
7. X +y 8x-2y-51 =0,4Vl3
Re

MULTIPLE CHOICE QUESTIONS (MCQs)


F

Mark the correct alternatives in each of the following:


1. The equation of a circle with radius 5 and touching both the coordinate axes is
(a) x^+y2±10x±10y + 5 = 0 (b) x^ + y^ ± 10 X ± 10 y = 0
(c) x^ + y^ ± 10 X ± 10 y + 25 = 0 (d) x^ + y^ ± 10 X ± 10 y + 51 = 0
2. Tlie equation of the circle passing through the origin which cuts off intercept of length 6
and 8 from the axes is

(a) x^ + y^ -12 X -16 y = 0 (b) x“ + ip + 12 X + 16 y = 0


(c) x^ + y^ + 6 X + 8 y = 0 (d) x^ + y^ - 6 X - 8 y = 0
3. The equation of the circle concentric with x + - 3x + 4y - c = 0 and passing through
(-1,-2) is
(a) x^ + y^ - 3 X + 4 y -1 = 0 (b) x^ + ip 3x+4y=0
(c) x^ + y^ - 3 X + 4 y + 2 = 0 (d) none of these
CIRCLE 13.21

0 'j 'y 'y


4. If the circles .v + i/ =a and .v + i/“ - 6 x - 8 y + 9 = 0, touch externally, then a =
(a) 1 (b) -1 (c) 21 (d) 16
5. If (x, 3) and (3,5) are the extremitiesof a diameterof a circle with centre at (2, y), then the
values of x and y are
(a) (3,1) (b) x = 4,ys=l (c) X = 8, y = 2 (d) none of these
6. If (- 3, 2) lies on the circle x^ + y^ + 2 ^x + 2 Jy + c = 0 which is concentric with the circle
'y 1

x~ + }/~ + 6 X + 8 y - 5 = 0, then c -
(a) 11 (b) -11 (c) 24 (d) none of these
9 9
7. Equation of the diameter of the circle x + y 2 X + 4 y = 0 which passes through the
origin IS
(a) X + 2 y = 0 (b) X - 2 y = 0 (c) 2 X + y = 0 (d) 2x-y = 0

w
8. Equation of the circle through origin which cuts intercepts of length a and b on axes is
(a) x^ + y^ + ax + Ziy = 0 (b) x^ + y^ - ax - by = 0

Flo
(c) x^ + y^ + hx + ay = [) (d) none of these
9. The equation of the circle in the first quadrant touching each coordinate axes at a distance of

ee
one unit from the origin is

Fr
(a) x^+y^-2x-2y + l =0 (b) x^+y^-2x-2y-l =0
(c) x^+y^-2x-2y-0 (d) x^+y^-2x + 2y-l =0
for
10. The equation of the circle having centre (1,-2) and passing through the point of
ur
intersection of the lines 3x + y = 14 and 3x + 5y = 18 is
(a) x^ + y^-2x + 4y-20=0 (b) x^+y^-2x-4y-20=0
k s
Yo

(c) x^ + y^ + 2x-4y-20=0 (d) x^+y^+2x + 4y-20 = 0


oo

11. Equation of a circle which passes through (3,6) and touches the axes is
eB

(a) x^ + y^ + 6x + 6y + 3 = 0 (b) x^ + y^ 6x-6y-9 =0


(c) x^ +1/^ - 6x - 6y + 9 = 0 (d) None of these
r
ou

12. The equation of a circle with origin as centre and passing through the vertices of an
ad

equilateral triangle whose median is of length 3a is


Y

(a) x^ + y^ = 9i7^ (b) .Y^+y^ =16f?^ (c)x^+y^=4rt^ (d)x^+y^=rt^


13. The equation of the circle with centre on the y-axis and passing through the origin and the
Re
nd

point (2, 3) is
(a) .Y^ + y^ +13y =0 (b) 3x^ + 3y^+13x+3 = 0
Fi

(c) 6x^ + 6y^ -13x = 0 (d) ,Y^+y^+13x+3=0


ANSWERS

1. (c) 2. (d) 3. (b) 4. (a) 5. (a) 6. (b) 7. (c) 8. (b)


9. (a) 10. (a) 11. (c) 12. (c) 13. (c)

FILL IN THE BLANKS TYPE QUESTIONS (FBQs)

1. The equation of the circle which passes through the point (4,5) and has its centre at (2,2) is

2. The equation of the circle having centre at (3, -4) and touching the line 5x + 12y-12 = 0
is.
13.22 APPLIED MATHEMATICS-XI

3. The equation of the circle circumscribing the triangle whose sides are the lines
y = .r + 2, 3y = 4.v and 2y = 3x, is
4. The area of the circle passing through the point (4,6) and having centre at (1,2) is
5. If the coordinates of one end of a diameter of the circle x +y - 4.r - 6y +11 = 0 are (3, 4),
then the coordinates of the other end are

6- If the line y =^/3x + k touches the circle .v“ + = 16, then k =


7. The equation of the circle concentric with the circle X +y -6x + 12y + 15 = 0 and double
its area is

8. The equation of the circle which touches x-axis and has its centre at (1,2) is
9. If a circle passes through (0,0), {a, 0) and (0, b), then the coordinates of its centre are

w
y y

10. If the line x + Ihy + 7 = 0 is a diameter of the circle x + y - 6x + 2y = 0, then b =


y y

II Ifthecirclex +y -fcx-12y + 4=0touchesx-axis,thenk=

F lo
9 7 9 9
12. The value of k for which the centres of the circles x +y =l,x +y +6x-2y = l and
x^ + y^ - 6kx + 4y -1 =0 are collinear, is

ee
Fr
ANSWERS

45 f
1. (.v-2)^+(y-2)2=13 2. (Y-3)^+(y+4)2 = for
ur
13

.Y^ + i/^-46.Y + 221/=0 4. 257u sq. units 5. (1,2) 6. ± 8


ks
a b'
7. Y^ + _v^-6Y + 12y-15=0 S. x^+y“-2x-4y + l=0
Yo

9.
oo

2'2,
10. 5 11. ± 4 12. 2
eB

VERY SHORT ANSWER QUESTIONS


r

Aitsxvcr each of the following questions in one word or one sentence or as per exact requirement of the
ou
ad

question:
Y

1. Write the length of the intercept made by the circle x + y + 2x - 4y - 5 =0 on y-axis.


2. Write the coordinates of the centre of the circle passing through (0, 0), (4,0) and (0, - 6).
nd
Re

3. Write the area of the circle passing through (- 2, 6) and having its centre at (1, 2).
Fi

4. If the abscissae and ordinates of two points P and Q are roots of the equations
X +2ax-b - Oandx +2px-q = 0 respectively, then write the equation of the circle
with PQ as diameter.
9 9
5. Write the equation of the imit circle concentric with x + y - 8x + 4y -8=0.
9 9

6. If the radius of the circle x + y + rtx + (1 - rt) y + 5 = 0 does not exceed 5, write the
number of integral values a.
7. Write the equation of the circle passing through (3, 4) and touching y-axis at the origin.
8. Write the coordinates of the centre of the circle inscribed in the square formed by the lines
x=2, x = 6,y=5 and y = 9.
ANSWERS

1. 6 units 2. (2, - 3) 3. lOnsq. units


4. x^ -I- y^ + 2i?x -i- 2py -b^ -q^ = 0 5. x^ + y^ - 8x + 4y -I-19 = 0 6. 16

7. 3(x^+y^)-25x = 0 9. (4,7)
CHAPTER 14
PARABOLA

14.1 THE PARABOLA

ANALYTICAL DEFINITION A parabola is the locus of a point (or curve generated by a point) which moves
in a plane such that its distance from a fixed point in the plane is ahvays equal to its distance from a fixed
straight line in the same plane.

w
Z
P
The fixed point is called the focus and the fixed straight line is called the M
directrix of the parabola. The line through the focus and perpendicular to

F lo
the directrix is the axis of the parabola. The point on the axis midway S
between the focus and directrix is called the vertex of the parabola. Focus

ee
X

Let S be the focus, ZZ' be the directrix and let P be any point on the ■£
u

Fr
<y

parabola. Then, by definition u

SP = PM for Z'
ur
where PM is the length of the perpendicular from P on the directrix ZZ'. Fig. 14.1
s
ook

Find the equation of the parabola whose focus is (- 3,2) and the directrix is x + y= 4.
Yo

1 LI.USTRATION 1
SOLUTION Let P (.Y, y) be any point on the parabola whose focus is S (- 3,2) and the directrix
eB

.V + y “ 4 = 0. Draw PM perpendicular to x + y - 4 = 0. Tlien,


Z
SP = PM [By definition]
P{x,y)
SP^ = PM^
r

M
ad
ou

2
.Y + y - 4
(.Y+3)2+(y-2)2 = o
S(-3,2)
Y

JTTT
II
-t
I
Re
nd

2 .Y^ + y^ + 6 Y - 4 y + 13 = x^ + y^ + 16 + 2xy - 8.y - 8y


Fi

x^ + y2 - 2 .vy + 20 .y + 10 = 0 Z’

Fig. 14.2
Thus, the required equation of the parabola is y" + y - 2 Yy + 20 y +10 = 0.
ILLUSTRATION 2 Find the equation of the parabola zuhose focus is (- 3,0) and the directrix is y + 5 = 0.
SOLUTION Let P (y, y) be any point on the parabola having its focus at S (- 3,0) and directrix as
the line y + 5 = 0. Then,
SP = PM, where PM is the length of the perpendicular from P on the directrix
SP^ =PM^
2
Y + Oy + 5
(j-+3)2+(y-0)^ "= VT + 0~
14.2 APPLIED MATHEMATICS-XI

M ■fix, If)
X' S(-3,0) O X

r
j- + 5 = 0

Fig. 14.3

w
y 4x + 16, which is the required equation of the parabola.

F lo
14.2 EQUATION OF THE PARABOLA IN ITS STANDARD FORM

Let S be the focus, Z Z' be the directrix. Draw SK perpendicular from S on the directrix and

ee
bisect SK at A. Then,

Fr
AS = AK

Distance of A from the focus = Distance of A from the directrix

A lies on the parabola


for [By def.]
ur
=>

Let SK = 2a. Then, AS^AK = a.


Let us choose A as the origin, AS as x-axis and ,4 Y a line perpendicular to AS as y-axis. Then, the
s
ook
Yo

coordinates of S are (a, 0) and the equation of the directrix ZZ' is .r = - a.


eB

Z y,

M
r

x = -a
ou
ad

K /I \ Focus X
Y

X' N S{a, 0)
Re
nd

X
U

a
Fi

Z' r = 4rt.v

Fig. 14.4

Let P (.r, y) be any point on the parabola. Join SP and draw PM and PN perpendiculars on the
directrix Z Z' and X-axis. Then,
PM = NK = AN + AK = X + a.

Since P lies on the parabola. Therefore,


SP = PM [By definition of parabola]
SP^ = PM^
(x-i7)^+(y-0)^ =(x + f?)^
i/ = 4 nx
PARABOLA 14.3

This is the equation of the parabola in its standard form.


NtTiH The parabola has two real foci situated on its axis one of which is the focus S and the other lies at
infiniti/. The corresponding directrix is also at infinity.

14.2.1 TRACING OF THE PARABOLA if = 4 ax, a>0


The given equation can be written as y = + 2 -Jc^.
We observe the following:
(i) Symmetry: For every positive value of x, there are two equal and opposite values of y.
(ii) Region: For every negative value of x, the value of y is imaginary. Therefore, no part of the
curve lies to the left of y-axis
(iii) Origin: The curve passes through the origin and the tangent at the origin is .t = 0 i.e.,

w
y-axis
(iv) Intersection zuilh the axes: The curve meets the coordinate axes only at the origin,

Flo
(v) Portion Occupied: As x co, y oo. Therefore the curve extends to infinity to the right of
axis of y.

ee
With the help of the above facts and by joining some convenient points on the parabola the

Fr
general shape of the parabola y^ = 4 ax is as shown in
Fig. 14.4.
14.2.2 VARIOUS RESULTS RELATED TO THE PARABOLA
for
ur
As discussed in section 14.1, the focus of the parabola jr = 4rtxis at (a, 0) and the directrix isx = -a.
The axis is a line passing through the focus and perpendicular to the directrix. In Fig. 14.5 x-axis
s
i.e., y = 0 is the axis of the parabola i/^ = 4 ax. The axis meets the curve ir = 4 rtx at A, the origin. So,
k
Yo

the coordinates of the vertex are (0,0). Clearly, the vertex A is the midway between the focus and
oo

the directrix i.e., the vertex is equidistant from the focus and the directrix.
eB

DOUBLE ORDINATE Let P be any point on the parabola y~ = 4 ax. A chord passing through P
perpendicular to the axis of the parabola is called the double ordinate through the point P.
r
ou

In Fig. 14.5, PP' is the double ordinate of point P.


ad

LATUS-RECTUM A double ordinate through the focus is called the latusrectum i.e. the latusrectum of a
Y

parabola is a chord passing through the focus perpendicular to the axis.


Re
nd

In Fig. 14.5, LSU is the latusrectum of the parabola y = 4 ax. By the symmetry of the curve
SL = SL' = X (say). So, the coordinates of L are {a, X). Since L lies on y^ = 4 nx. Therefore,
Fi

Z Y

L P(x, 1/)

X' K A X

S{a, 0)

x = -«

L'

P'
y- = 4ax
Z' Y'

Fig. 14.5
14.4 APPLIED MATHEMATICS-XI

=> k = 2a => LL'=2l^4:n.

Latusrectum = 4 fl.

The coordinates of Land L' ,end points of the latusrectum, are {a, 2 a) and {a,~2a) respectively.
FOCAL DISTANCE OF ANY POINT The distance ofP (x, \j) from the focus S is called the focal distance of
the point P.

Clearly, SP = -J(x -a)^ + (y - 0)“


SP = yj(x~a)^+y^
2
SP = + 4 rt.Y [●.● P (.Y, 1/) lies on the parabola y = 4 fl.vl

w
SP = = Y + rt =n +Y
['.● Y > 0, >0 Y + rt>0]

Flo
Hence, + y is the focal distance of any point P (y, y) on the parabola y = 4^y.
FOCAL CHORD A chord of the parabola is a focal chord, if it passes through the focus.

ee
Fr
14.2.3 SOME OTHER STANDARD FORMS OF PARABOLA

Proceeding as in section 14.1, we find that there are three other standard forms of parabola viz.
y^ = - 4 ax, x^ = 4 a\/ and y^ = - 4 ay depending upon the choiceof the axes. Thus, in all there are
for
ur
four standard forms. The shapes of the curves in these four standard forms and their
corresponding results are as follows:
k s
Yo

y- = 4 ax y2 = _ 4 ax Y^ = 4 ^^y x^ = -4 ay
oo

Coordinates of vertex (0,0) (0,0) (0,0) (0,0)


eB

Coordinates of focus (/^,0) 0) (0,^) (0,-^7)


r

Equation of the directrix x = -a X = a


y = -a y = a
ou
ad

Equation of the axis y = 0 y =0 Y = 0 Y = 0


Y

Length of the Latusrectum 4fl 4a 4a 4a


Re
nd

Focal distance of a point P (y, y) a + X a - X ^ + y a -y


Fi

Z y, z

X’ K A (0. 0) X K X
S(n, 0)

x=-a y2 = X = a

Z' r r Z'

Fig. 14.6 Fig. 14.7


PARABOLA 14.5

Y Y
Z' K y = a Z
.t2 = 4(71/
A(0, 0)
X' X

S(0, a)
S(0,-a)

X' X

A(0, 0)
.x2 = - 4iii/
Z' K y = -a 2
Y' r

Fig. 14.8 Fig. 14.9

w
RFMARK If the vertex of the parabola is at the point A (h, k) ami its latusrectum is of length 4a, then its
equation is
(i) (y -~k)^ =4a{x-h)oT,{y -k)^ =-4i? (x -/;) according as its axis is parallel to OX orOX'.

Flo
(ii) (.r - h)^ ~4a (y ~k) or, (x -h)^ =-4a (y -/:) according as its axis is parallel to OY orOY'.

ee
ILLUSTRATIVE EXAMPLES

Fr
Ti/pe 1 ON FINDING THE EQUATION OF A PARABOLA WHEN ITS FOCUS AND DIRECTRIX ARE GIVEN
EXAMPLE 1 Find the equation of the parabola whose focus is the point (0, 0) and whose directrix is the
for
ur
straight tine 3x - 4y + 2 = 0.
SOLUTION Let P(.v, y) be any point on the parabola whose focus is S(0, 0)
P{x, y)
s
and the directrix 3x - 4y + 2 = 0. Draw PM perpendicular from P on the M

\
ok
Yo

directrix. Then, by definition


SP = PM
o

o S(0, 0)
SP^ = pm'^
eB

2 +

3.V - 4y + 2
(,r-0)2+(y-0)2
r

I
ou
ad

{3x-4\j + 2f
Y

.Y
25 Fig. 14.10
nd
Re

25(.T^ + y^) = (3.v-4y + 2)^


25-V- + 25\/ = 9x^ + 16y^ + 4 - 24.vy + 12.v - 16y
Fi

16.v^ + 9y^ + 24.\-y - 12.v + 16y -4 = 0


This is the required equation of the parabola.
EXAMPLE 2 Find the equation of the parabola whose focus is at (-1, -2) and
the directrix the line .y - 2y + 3 = 0.
SOLUTION Let P(.v, y) be any point on the parabola whose focus is S(-l, P(.v,y)
M
-2) and the directrix .y - 2y + 3 = 0. Draw PM perpendicular from P{x, y)
on the directrix y - 2y + 3 = 0. Then, by definition \ S(-l,-2)
SP = PM o

SP^ = PM^
+
2
(N
Y - 2y + 3
(,v + l)2+(y+2)2
I

#T4
Fig. 14.11
14.6 APPLIED MATHEMATICS-X!

5 J (.r + 1)2 + (y + 2)2 I = {x-ly^. 3)2


5 (,v2 + 1/2 + 2x + 4i/ + 5) = (.y2 + 4y2 + 9 _ 4.V1/ + 6.y -\ly)
=> 4x2 + y2 22i/ + 16 = 0
This is the equation of the required parabola.
7'i/pt’ II ON FINDING THE FOCUS, DIRECTRIX. LATUS-RECTUM, AXIS ETC. FOR A GIVEN PARABOLA
IN ONE OF THE STANDARD FORMS

EXAMPLE 3 for the foIlouHiig parabolas find the coordinates of the foci, the equations of the directrices
and the lengths of the latus-rectum:
(i) 1/2 =8x (ii) x2 = 6y (iii) 1/2 12x (iv) x2=-16i/
SOLUTION (i) The given parabola y2 = 8^ is of the form i/2 = 4nx, where 4a = 8 i.e. a = 2.
The coordinates of the focus are (a, 0) i.e. (2, 0) and the equahon of the directrix is x = - rt

w
i.e. X = - 2.

F lo
Length of the latus-rectum = 4i? = 8.
(ii) The given parabola x = 6y is of the form x = 4ay, where 4a = 6 i.e. a = 3/2.
Clearly, the coordinates of the focus are (0, a) = (0,3/2) and the equation of the directrix is 1/ = -17

e
i.e. y = -3/2.

Fre
Length of the latus-rectum = 4a = 6. for
(iii) The given parabola y = - 12x is of the form y = - 4ax, where 4a = 12 i.e. a = 8.
Clearly, the coordinates of the focus are {-a, 0) = (-3, 0) and the equation of the directrix is x = «
r
i.e. X = 3.
You
oks

Length of the latus-rectum = 4rt = 12.


eBo

(iv) The given parabola is of the form x = - 4ay, where 4a = 16 i.e. a = 4.


Clearly, the coordinates of its focus are (0, -a) = (0, - 4) and the equation of the directrix is y =
i.e.y = 4.
our
ad

Length of the latus-rectum =4a = 16.

EXERCISE 14.1
1. Find the equation of the parabola whose:
dY
Re

(i) focus is (3,0) and the directrix is 3 x + 4 y = 1


Fin

(ii) focus is (1,1) and the directrix is x + y +1 = 0


(iii) focus is (0,0) and the directrix 2x-y-l = 0
(iv) focus is (2, 3) and the directrix x - 4 y + 3 = 0.
2. Find the equation of the parabola whose focus is the point (2, 3) and directrix is the line
X - 4 y + 3 = 0. Also, find the length of its latus-rectum.
3. Find the vertex, focus, axis, directrix and latus-rectum of the following parabolas
(i) y^=8.v (ii) 4 x2 + y = 0(iii) y2=-24x (iv) 2x2 ^ 3 y
4. Find the area of the triangle formed by the lines joining the vertex of the parabola x2 = 12i/
to the ends of its latus-rectum.

5. At what point of the parabola x2=9i/ is the abscissa three times that of ordinate ?
6. Find the equation of a parabola with vertex at the origin, the axis along x-axis and passing
through (2, 3).
14.7
PARABOLA

ANSWERS

1. (i) 16 +9 - 24 xy -144 .r + 8 y + 224 = 0


(ii) .v^ + y^ - 2 xy - 6 .V - 6 y + 3 = 0
(iii) x^ + 4 y^ + 4 xy + 4 X - 2 y -1 = 0
(iv) 16 x^ + y^ + 8 xy - 74 x - 78 y + 212 = 0.
14
2. 16 x-2 + y^ + 8 xy -74 x -78 y + 212 = 0, L.R. = Vl7'
directrix LR.
3. vertex focus axis

x = -2 8
(i) (0, 0) (2,0) y =0
(0, -1/16) y=l/16 1/4
(ii) (0,0) X

w
X = 6 24
(iii) (0,0) (-6, 0) y =0
3 3
3^ x =0
0, J/ = -

F lo
(iv) (0,0) 8 2

4. 18 sq 5. (3,1) 6. 2y^=9x

ee
HINTS TO SELECTED PROBLEMS

Fr
4. Required Area = (LL'xOS) = — xl2x 3-18sq. units for
ur
●2= I2y
oks

y
Yo
o

(0,3)
eB

L'i-6,3) V JUe,3)
S
our
ad

O X
X'
Y

Fig. 14.12
Re
nd

MULTIPLE CHOICE QUESTIONS (MCQs)


Fi

Mark the correct alternative in each of the folloiving:


The equation of the parabola whose focus is (1, -1) and the directrix is x 4- y + 7 — 0 is
(a) x^ + y“ - 2xy - 18x -10 y = 0 (b) x^ - 18x - lOy - 45 = 0
(c) x^ + y^ - i8x - lOy - 45 = 0 (d) x^ + y^ - 2.xy -18x - lOy - 45 = 0
2. The area of the triangle formed by the lines joining the vertex of the parabola x = 12y to the
ends of its latusrectum is
(a) 12 sq. units (b) 16 sq. units (c) 18 sq. units (d) 24 sq. imits
3. The equations of the lines joining the vertex of the parabola y^ = 6x to the points on it which
have abscissa 24 are

(a) y ± 2x = 0 (b) 2y±x = 0 (c) x±2y = 0 (d) 2x±y = 0


4, The focus of the parabola is (0, - 3) and directrix is y = 3, then its equation is
(a) x^ =-12y (b) =12y (c) y^ =-12x (d) i/=l2x
14.8
APPLIED MATHEMATICS-XI

5. If the vertex of the parabola is the point (-3, 0) and the directrix is the line x + 5 = 0, then its
equationis
(a) 1/^ =8{y+3) (b) .v2=8(y+3) ic) i/^=-S{x+3) (d)i/=8{x + 5)
6. If the parabola = 4ax passes through the point (3, 2), then the length of its latusrectum IS
i

(a, I
1
(d) 4

ANSWERS

1- (d) 2. (c) 3. (b) 4. (a) 5. (a) 6. (b)

FILL IN THE BLANKS TYPE QUESTIONS (FBQs)

ow
1. The coordinates of the points on the parabola i/^ = 8.t whose focal distance is 4, are
2. If the points (0, 4) and (0, 2) are respectively the vertex and focus of a parabola, then the
length of its latusrectum, is
●. If the vertex of a parabola is at the origin and directrix is .v + 5 =0, then its latusrectum is

e
4 The equation of the parabola with focus (3,0) and tlie directrix .v + 3 = 0, is

re
rFl
5. If a double ordinate of the parabola = 4ax is a length 8fl, then the angle between the lines

F
joining the vertex of the parabola to the ends of this double ordinate, is
6. The coordinates of the end-points of the latusrectum of the parabola .y^ + 8y = 0, are

r
fo
ou
The coordinates of the point on the parabola y- =18.y whose ordinate is three times the
abscissa, are
ks
8. The equation of the parabola whose focus is the point (2, 3) and directrix is the line
oo

-Y - 4y -I- 3 = 0, is
The equation of the parabola having focus at (-1,-2) and the directrix .y-21/+ 3 = 0, is
Y
eB

ANSWERS
ur

4. y^=12.Y
ad

1, (2, 4), (2,-4) 2. 8 3. 20


Yo

5. 90' 6. (-4,-2) and (4,-2) 7. (2,6)


8. 16x^ + y^ + 8ry -74Y-78y ^ 212 = 0 y. 4.y^ -i- 4.Yy + y^ + 4.y + 32y -i-16 = 0
d
Re
in

_ VERY SHORT ANSWER QUESTIONS (VSAQs)


F

Anszver each of the folloxving questions in one word or one sentence or ns per exact requirement of the
questioiz:
1. Write the axis of symmetry of the parabola y^ = Y.

2. Write the equation of the parabola with focus (0, 0) and directrix y + y - 4 = 0.
3. Write the length of the chord of the parabola y^ = 4ax which passes tlirough the vertex and
is inclined to the axis at
4

4. If the parabola y^^4nx passes through the point (3,2), then find the length of its
latusrectum.

ANSWERS

1. .Y-axis
2. x^ + \/- 2x\f + 8y 8y -16 = 0 3. 4f2a 4. 4/3
CHAPTER 15
FUNCTIONS

15.1 INTRODUCTION

In this chapter, we shall study about one of the most important concepts in mathematics known
as a function. Functions form one of the most important building blocks of Mathematics. The
word "Function" is derived from a Latin word meaning operation and the words mapping and
map are synonimus to it. Functions plsy 3 very important role in differential and integral

w
calculus which will be studied in XII class. In this chapter, we shall introduce the concept of a

15.2 FUNCTION AS A SPECIAL KIND OF RELATION


DEFINITION
F lo
function as a correspondence between two sets. We shall also study function as a relation from
one set to the other set.

Let Amid B be tzuo non-empt\/sets. A relation ffrom A


a function (or a mapping or a map) from A to 6, if
(i) for each a eA there exists b € B such that (a, b) e f ree
to B, i.e., a subset o/A x B, is called
for F
(ii) (a,b) ef and(a, c) sf => b = c.
r
Thus, a non-void subset /"of A x B is a function from A to B if each element of A appears in some
You
oks

ordered pair in/and no two ordered pairs in/have the same first element.
eBo

If (a, b) e/, then b is called the image of a under/.


ILLUSTRATION 1 Let A = ( 1,2,3 },B = { 2,3,4} and fy /2 and f^ be three subsets of A xBas given
belouK
ad
our

/l={{l,2),(2, 3), (3, 4)1,/2=i(l,2), (1,3), (2, 3), (3, 4)1,/3={(1, 3), (2, 4)}.
Then, fi is a function from AtoB but f2 andf^ are not functionsfrom A to B. fo is not a function from A to
B, because 1 e A has two images 2 and 3 in B and f-^ is not a function from AtoB because 3 e A has no
Re

image in B.
dY

If a function f \s expressed as the set of ordered pairs, the domain of / is the set of all first
Fin

components of members of/and the range of/is the set of second components of members of/
i.e. Domain of / = (a : {a, b) e /1, and Range of f = {b :(a,b) e f\
ILLUSTRATION 2 If .V, 1/ 6 (1, 2, 3, 41, then which of the following are functions in the given set ?
(a) /i-{(.v, y):y = .v + ll (b) /2 - |(.T, i/): .V + y > 4}
(c) /3 = {(.t, y): y < x} (d) /4={(A-,y):.v + y=51
Also, in case of a function give its range.
SOLUTION If we express/i, /2 / /a and f^ as sets of ordered pairs, then we have
/^ = 1(1, 2), (2, 3), (3, 4)1,
/2 = {(1, 4), (4,1), (2, 3), (3, 2), (2, 4), (4, 2), (3, 4), (4, 3)1,
/3 = 1(2,1), (3,1), (4,1), (3, 2), (4, 2), (4, 3)] and /j =|(1,4),(2, 3), (3, 2),(4,1)|.
(a) We have,/| =((1, 2), (2, 3), (3, 4)|.
We observe that an element 4 of the given set has not appeared in first place of any ordered pair
of fy So, fi is not a function from the given set to itself.
15.2
APPLIED MATHEMATICS-Xl

(b) We have, /2 ={(1, 4), (4,1), (2, 3), (3, 2), (2, 4), (4, 2), (3, 4), (4, 3)1-
We observe that 2,3,4 have appeared more than once as first components of the ordered pairs in
/2- So, /2 is not a function,
(c) We have, /g =1(2,1), (3,1), (4,1), (3, 2), (4, 2), (4, 3)].
We observe that 3 and 4 have appeared more than once as first components of the ordered pairs
in /3. So,/3 is not a function,
(d) We have, =|(1,4),{2, 3), (3, 2), (4,1)).
We observe that each element of the given set has appeared as first components in one and only
one ordered pair of/4. So,/j is a function in the given set. In this case. Range of/ = {l, 2, 3, 4).
ILI.USTRATION 3 Let f bc a relation on the set N of natural numbers defined by f = {{n, 3n): n e N]. Is
f a function from N to N. If so, find the range off.

w
SOLUTION Since for each n e N, there exists a unique 3n e Nsuch that {n, 3n) e f. Therefore,/ is a
function from N to N.

F lo
Clearly, Range of / = (/ (n):« e W} ^ 13;i; n e N\.
ILLUSTR.ATION 4 Let f be a subset of ZxZ defined by f = {{ab, (a + b)):a,b eZ\. Is f a function
from Z into 2. Justify your answers.

ee
Fr
SOLUTION We observe that:

1x6 = 6 and 2x3 = 6

=> (1 X 6,1+6) e/and(2x 3,2+3) e/


for
ur
=> (6,7)e/ and (6,5) e/
s

So, / is not a function from Z to Z.


ook
Yo

15.3 FUNCTION AS A CORRESPONDENCE


eB

DEFINITION Let A and B be two non-empty sets. Then a function 'f from set A to set B is a rule or
method or correspondence which associates elements of set A to elements of set B such that:
our

(i) all elements of set A are associated to elements in set B.


ad

(ii) an element of set A is associated to a unique element in set B.


In other words, a function f 'from a set A to a set B associates each element of set A to a unique element of
Y

set B.
Re
nd

Terms such as "map” (or "mapping”), "correspondence" are used as synonyms for "function".
If/is a function from a set /I to a set B, then we write /A B or A B, which is read as/is a
Fi

function from Ato Borf maps A to B.


If an element <7 € yl is associated to an element b & B, then b is called 'the f-image of a' or 'image of a
under f or 'the value of the function f at a'. Also, a is called the pre-image of b under the function/
We write it as: b = f{a)
ILLUSTRATION Let A = U, 2, 3, 41 and B = {a, b, c, d, e} be two sets and let /, /2, f^ and f^ be rules
associating elements (A to elements of) B as shown in the following figures:

2
3
4

Fig. 15.1
FUNCTIONS 15.3

A h B A /a B

1
b 1 b
2—
c
3 3
— fr
4 4

Fig. 15.3 Fig. 15.4

We observe that/^ is not a function from set A to set B, because there is an element Be A which is
not associated to any element of B.
Also,is not a function from A to S because an element 4 e A is associated to two elements c
and e in B. But,/3 and/4 functions from A to B, because under and each element in A is
associated to a unique element in B.
15.3.1 DESCRIPTION OF A FUNCTION

Let/: A -> Bbe a function such that the set A consists of a finite number of elements. Then,/(x)
be described by listing the values which it attains at different points of its domain. For example,
if A = {-1,1,2,3} and B is the set of real numbers, then a function /: A -» B can be described as

w
/(-I) = 3,/■(!) =0,/(2) = 3/2 and/’(3) = 0. Incase, A is an infinite set, then/cannot be described
by listing the images at points in its domain. In such cases functions are generally described by

F lo
r a

formula. For example,/: Z Z given by f{x) = x^ +1 or/: B R given by / (x) = etc.


15.3.2 DOMAIN, CO-DOMAIN AND RANGE OF A FUNCTION

e
Let /: A -> B. Then, the set A is known as the domain of / and the set 6 is known as the

Fre
domain off. The set of all/-images of elements of A is known as the range of/or image set of A
co-

under/and is denoted by / (A).


for
Thus, /(A) = i/(x):xeAl = Range of /
Clearly, / (A) c B.
r
LetA = i -2, -1, 0,1,2 1 and B = 10,1,2, 3, 4, 5, 6 }. Consida- a rule f (x) = x^.
You

ILLUSTRATION 1
oks

Under this rule, zve obtain /(-2) =(-2)^ =4, /(-I) =(-l)^ =1, /(0)=0^=0, /{1)=1“=1 and
eBo

/ (2) = 2^ = 4. We obseroe that each element of A is associated to a unique element of B. So, f: A B


given bi/f (.v) =x^ isafimction. Clearly, domain (f) = A = I -2,-1,0,1,2 } and range (f)= f 0,1,4 /.
ad
our

ILLUSTRATION 2 Consider a rulcf (.v) = 2x - 3 associating elements ofN (set of natural numbers) to
elements of N.This rule does not definea fimction from N to itself,because} (1) = 2x1-3 = -1 iNi.e.
1 eN (domain) is not associated to any element of hi (co-domain).
Re

be given byf(x) =x^ -2x- 3. Find:


dY

ILLUSTRATIONS Let A = (-2,-1, 0,1, 2} and /; A ^ Z


(i) the range off (ii) pre-images of 6, -3 and 5.
Fin

SOLUTION (i) We have,/{x) =.v^-2x-3.


/ (-2) = i-lf - 2 (-2) -3=5 / (-1) = (-1)^ - 2 (-1) - 3 = 0,/ (0) = - 3,
/(l)=l^-2xl-3 = -4and/(2)=2--2x2-3=-3.
So, range (/) = (/(-2), /(-I), /(O), /(I), /(2)1 = |0,5, - 3, - 4[
(ii) Let X be a pre-image of 6. Then,
f(x)=6 => x^ -2x-3=6 => -v^ -2x-9 = 0 ^ x=l±/T0
Since x = 1 ± -JTO g A. So, there is no pre-image of 6.
Let X be a pre-image of - 3. Then,
/(a-)=-3=> x^-2x-3=-3 => x^-2x = 0=> x = 0,2.
Clearly, 0, 2 e A. So, 0 and 2 are pre-images of - 3.
15.4 APPLIED MATHEMATICS-XI

Let X be a pre-image of 5. Then,


f{x)=5=> -2x-3=5 => x^ -2x~8 = 0 => (.r-4) (x + 2) =0 => .v = 4, -2.
Since, - 2 sA bet 4 g ^4. So, -2 is a pre-image of 5.
15.4 EQUAL FUNCTIONS

DEFINITION Tu’o functions f and g are said to be equal iff


(i) domain off= domain of g (ii) co-domain off = co-domain of g,
and (iii) / (.v) = g (.v) for every x belonging to their common domain.
If two functions/and g are equal, then we write f =g.
ILLUSTRATION 1
Let A = 11, 2 }, B = {3, 6} and f :A ^ B given by f (.y) = + 2 and g:A^B
given by ^(.y) = 3.y. Then, xve observe that fand g have the same domain and co-domain. Also we have,
/ (1) = 3 = ^1) and / (2) = 6 =g{2). Herxce, f=g. .2
X 4
ILLUSTRATION 2 let f: K-\2\ ^ R be defined by f (x) = - and g: R R be defined by
x-2

w
g (x) = X + 2, Find whether f = gor not.
x^-4
SOLUTION We have, /(x) =

=>
fix) =
(x-2) (x+2)
x-2

fix) =gix) for all X


Thus, / (.y)
x-2

2.
, x ^ 2.

= X + 2 for all x ^ 2.
F lo for
F ree
(.y) for all x e R - (2). But, / (x) and g (x) have different domains.
Infact, domain of / = R -{2} and domain of g = R. Therefore, / g.
r
ILLUSTRATION 3 Let f: Z ^ Z andg: Z ^ Z be functions defined by f = [{n, 11^) in ^ Z\ and,
You
oks

g = {(?/, 1171^) :;i eZ). Show that: f =g.


eBo

SOLUTION Clearly,
Domain of/= Domain of g = Z and. Co-domain of/= Co-domain of g = Z.
ad
our

We have, / (j?) =n^ and g (77) = \n \ ^ n


2
[V I 771
2

f (”) = g (”) fOL ‘ill » e Z.


Re

Hence, / = g.
dY
Fin

ILLUSTRATIVE EXAMPLES

I-XAMPLE ]
Express the following functions as sets of ordered pairs and determine their ranges
(i) f:A^R,f{x)=x^+ 1, zvhere 4-1-1, 0, 2, 4}.
(ii) g: A N, g (x) = 2x, where A={x:xeN,x< lO}.
SOLUTION (i) We have,/(x) =x^+1.
/{-l)-{-l)2+l=2,/(0)=02+l=l,/(2) = 2^+l=5and/(4)=4^-H=17
So, / = I (X, / (X)): X e 41 = {(-1, 2), (0,1), (2,5), (4,17)1.
Hence, Range of (/) = (2,1,5,17|
(ii) Wehave,g(x) =2x and 4 = |1, 2, 3, 10}. Therefore,
g(l)-2xl=2. g(2) =2x2 = 4, g(3)-2x 3=6, g(4)=2x4=8,g(5)=2x5=10.
FUNCTIONS 15.5

g(6)=2x6=12, 5(7)=2x7=14, g(8) = 2 x 8 = 16, g(9) = 2 x 9 =18


and, g(10) = 2 X 10 = 20.
g = [ix,s{x)-.xeA] = {(1,2), (2, 4), (3, 6) (10,20)1.
Hence, Range of g =g(-4) =|g(.v): .t e A) = |2, 4, 6, 8,10,12,14,16,18, 20).
HXAMl’LE 2 Find the domain for which lhefunctionsf{x) =2x~ -landg{x)=l - 3x are equal.
SOLUTION The values of x for which f{x) and g{x) are equal are given by
f{x)=g{x)
=> 2x^-1 =1-3.y 2x^ + 3.T-2=0=>(.v + 2)(2x-l)=0=> x = -2,1/2.
Thus,/ (.x) and g(.v) are equal on the set {-2,1/2}.
EXAMPLES Is g = ((l,l),(2, 3),(3,5),(4,7)1 a function? if this is described bij the formula,
g{x) = CLX + p, then what values should be assigned to a and p?
SOLUTION Since no two ordered pairs in g have the same first component. So, g is a function

w
such that g(l) = 1, g{2) = 3, g( 3) = 5 and g(4) = 7.
It is given that g(x) = ax + p.

F lo
g(l) = 1 and g(2) = 3=> a+p = l and 2 a + p = 3 => a = 2, p = -1.
.2
HXAM1'LE4 Given /\ = 1-1, 0, 2, 5, 6, IV, B = (-2, -I, 0, 18, 28, 108} and f{x) =x -x-2. Is

ee
f{A) = B?Findf (A).

Fr
SOLUTION We have, /(.v) = x^ = x - 2.
/(-1)=H)2-(-1)-2 = 0, /(0)=0^-0-2= -2, /(2)=2^-2-2 = 0,
for
/ (5) =5^-5-2 =18, /(6)=62-6-2=28 and / (11) =11^-11 - 2 =108.
ur
Hence, / (/I) = (/ (,v): ,v e A) = |/ (-1), / (0), / (2), / (5), / (6), / (11)1 = |0, - 2,18, 28,108)
s
ook

1 ef(A). So, f {A) ^ B.


Yo

We observe that -1 € 6, but

EXAMPLE 5 Letf:R-^Rbe given b\/ f (x) = x^ + 3. Find (i) {x: / (x) = 281 (h') the pre-images of 39
eB

and 2 underf.
SOLUTION (i) We have, /(.x) = x^ + 3
our
ad

/ (x) = 28 => x^ + 3 = 28 => x^ = 25 ^ x = ± 5


Hence, [x :/(x) =281 = {-5, 51-
Y

(ii) Let X be a pre-image of 39. Then,


Re

/ (x) = 39 => x^ + 3 = 39 x^ = 36 ^ x = ± 6
nd
Fi

So, pre-images of 39 are - 6 and 6.


Let X be a pre-image of 2. Then,
/(x)=2=> x^ + 3=2=> .v^=-l
We find that no real value of x satisfies the equation x^ = -1. Therefore, 2 does not have any
pre-image under/.
fy

EXAMPLE 6 Let f: R ^ R be a function given by f (x) = x + 1. Find:


(i) / " ^ 1-51 (ii) / ■ ^ 1261 (iii) / ” ^ |10, 371
-1
SOLUTION Recall that if / : A ->■ B is a fimction and y e B. Then, / |i/l ={xeA:f (x) = i/l- In
other words,/ “ ^ |i/l is the set of pre-images of y.
(i) Let/“ ^ (-5) =x. Then,
f (x) = - 5 => x^ + 1 = 5 => x^ = - 6
15.6 APPLIED MATHEMATICS-XI

Clearly, this equation is not solvable in R. Therefore, there is no pre-image of-5. So,/ ^{-5} =^.
-1
(ii) Let/ (26) =a:. Then,
/ (.t) = 26 => +1 - 26 => .V = ± 5
So, pre-image of 26 are -5 and 5.
/-M26}=(-5,5}.
(hi) Let/”^(10)=.v.Then,
/ (x) = 10 + 1 - 10 => = 9 =:!* X = ± 3
So, pre-image of 10 are -3 and 3.
Let/"^(37) =x. Then,
fix) = 37 => x^ +1 = 37 => x^ = 36 => X = ± 6

w
So, pre-images of 37 are -6 and 6.
Hence, / " MlO, 37) -1 3, - 3, 6,-6|.

F lo
l;XAMPLE7 Let / - {(1,1), (2, 3), (0, -1), (-1,-3)) be a function described by the formula
f (x) = ax + b for some integers a, b. Determine a, b.
SOLUTION Clearly, / (1) = 1, / (2) = 3, / (0) = -1 and / (-1) = - 3.

ee
Fr
It is given that/ (x) = ax + b. Therefore,
/ (1) =1 and /(2) = 3 => a + b =1 and 2a + b = 3 a = 2, b = -1.
for
Substituting the values of r? and b in/(x) =ax + b, we get/(x) = 2x -1.
Clearly,/(0) =-l and/ (-1) =- 3 are true.
ur
Hence, a = 2 and i; = -1.
s

EXAMPLE 8 Iff:R->Rbe deifned as follows:


ook
Yo

/(.'■)={ -1,
h ifxeQ
ifxgQ.
eB

Find (i) /(l/2),/(rr),/(V2) (//) Range off (Hi) pre-images of 1 and -1.
SOLUTION (i) It is evident from the definition of / that at every rational point the function
our
ad

attains value 1 and at every irrational point attains value -1.


1 fl
2 ^
f -
^ 2 = 1, TugQ => f(n)=-l and 42 => /(V2) =-l.
Y
Re

(ii) Range of / = {/ (x): x e R }.


nd

Also, by definition / (x) attains values 1 or -1 according as x is rational or irrational and a real
Fi

number is either rational or irrational. Thus, all rational numbers have image 1 and all irrational
numbers have image -1. Hence, Range of / = {1, -1).
(iii) Since/ (.x) = 1 for all x e Q. Therefore, pre-images of 1 are rational numbers i.e/ -1
(i)=Q.
Also, -1 is the image of every real number which is not rational.
/ ^ (-1) =R - Q = Set of irrational numbers.
EXAMPLE 9 Let f \R ^ Rbe such that f (x) ^ 2^. Determine:
(i) Range off (ii) |x:/(.y) =1) (iii) ivhether f (x + y) =f (x) f (y) holds.
SOLUTION (i) Since 2'^ is positive for every x e R. So, / (x) = 2^ is a positive real number for
every x e R. Moreover, for every positive real number x, there exist log2 x e R such that
/(log2 X) = 2’*^® 2 ^ = .V
Hence, we conclude that the range of/is the set of all positive real numbers.
15.7
FUNCTIONS

(ii) /(.y)=1=> 2-^=1 => 2^'=2° => x = 0.


[x:f{x)=l}=\0l
(iii) We have, f{x) ^ 2^.
f {X + y) = 2^ ^ X 2^ =/ (X) f (If)
Hence, / (,v + y) = / (.t) /(y) holds for all y, y e R.
EXAMl’I.F 10 Let A be the set of two positive integers. Let f :A -> Z'*’ fscf of positive integers) be
defined by
f (n) = p, where p is the highest prime factor ofn.
If range off =13). Find set A. Is A iiniquehj determined?
SOLUTION It is given that the set A consists of two positive integers. So, let/4 = {n, jh). Since range

w
of/ = {3).
/ (”) = 3 / ('^0 = 3
=> Highest prime factors of n and ?n both are equal to 3.

F lo
=> (?i=3and»i = 6)or(» = 3andw = 9)or(« = 3andm=12) or (;i = 6 and m =12) etc.
=> 2l={3,6|, or 21=13, 9), or A =13,121, or 2\=|6,12} etc.

ee
Clearly, A is not uniquely determined.

Fr
EXAMPLE 11 LelA^N and f:A^Abe defined bij : f (n) = the highest prime factor ofn.
If range off is A. Determine A. Is A uniquely determined? for
ur
SOLUTION For any n e A, we have
/■ (n) = Highest prime factor of »
ks

=> /(n) takes prime values only


Yo
oo

=> Range of f consists of prime numbers only


But, it is given that range of / is A. Therefore, set A consists of prime numbers only.
eB

Hence, A = Set of some prime numbers. Clearly, A is not uniquely determined.


r

EXERCISE 15.1
ou
ad

I. Define a function as a set of ordered pairs.


Y

2. Define a fimction as a correspondence between two sets.


3. What is the fundamental difference between a relation and a function ? Is every relation a
nd
Re

fimction ?

4. Let A =l-2, -1, 0,1, 2| and /: A Z be a function defined by/(.v) = .v^ -2.v - 3. Find:
Fi

(i) range of/i.e./ (A) (ii) pre-images of 6, -3 and 5.


5. If a function /: R -> R be defined by
3.r - 2 , .V < 0
1 , .Y = 0
4y + 1 , A- > 0

Find: /(l),/(-l),/(0),/(2).
6. A function / : R -> R is defined by / (y) = y^. Determine
(i) range of/ (ii) {y:/(y)=4} (iii) ly:/(y)=-l}-
7. Let/: R'^ ^ R, where R"^ is the set of all positive real numbers, be such that/ (y) = log^ y.
Determine

(i) the image set of the domain of/ (ii) 1y:/(.v)=-2)


(iii) whether / (Yy) =/ (y) + / (y) holds.
8. Write the following relations as sets of ordered pairs and find which of them are functions:
15.8 APPLIED MATHEMATICS-XI

(i) {(.V, y):y = 3.v, xe{l, 2, 3|, y e{3, 6, 9,12}}.


(ii) |(.v, y): y > -r + 1, :r = 1, 2 and y = 2, 4, 6} (iii) |(.v, y): -v + y = 3, .r, y e (0,1, 2, 3))
9. Let/:R R and ^: C -> C be two functions defined as / (a:) = a:^ and g (a:) = Are they
equal functions?
If/, g, h are three functions defined from i? to R as follows:
(i) f{x)=x^ (ii) g (.v) = sin a: (iii) h (at) = + 1
Find the range of each function.
II. LetX = (l,2, 3,4} and Y = {1,5, 9,11,15,16}
Determine which of the following sets are functions from X to Y
(i) /i= {(1,1), (2,11), (3,1), (4,15)} (ii) /2= {(1,1), (2, 7), (3, 5)}
(iii) /3 = 1(1, 5), (2, 9) (3,1), (4, 5), (2,11)}.
12. Let A = (12,13,14,15,16,17} and /: A Z be a function given by

w
/ (at) = highest prime factor of a:.
Find range of/.
13. If/:K -)■

F lo
be defined by f (x) =x^ + 1, then find /“^ {17} and/“^ (- 3}.
14. Let A = {y, (j, r, s} and B = {1, 2, 3}. Which of the following relations from A to B is not a
function?

ee
Fr
(i) K3={(y,l),(^j,2),(r,l),(s,2)} (ii) R2=((B,l),(y, 1), (r,l),(s,l))
(iii) Rg = {(y, 1), {q, 2), (y, 2), (s, 3)} (iv) R4 = {(y, 2), (g, 3), (r, 2), (s, 2)}.
for
15. Let A
= {9,10,11,12,13} and let /: A—> Nbe defined by f (??) = the highest prime factor
of n. Find the range of /.
ur
16. The function/ is defined by /(.v) = -v^,0<.v<3
s

3a:, 3<.y<10
ook
Yo

The relation^is defined by <^j(.v) = y^,0<y<2


eB

3.r, 2 <Y<10

Show that / is a function and g is not a function.


our
ad

17, If/(Y) = Y^, find^^iHlz^.


(1.1)-1

18. Express the function f:X^R given by /(a*)=a'^+1 as set of ordered pairs, where
Y
Re

X=|-l, 0, 3, 9,7}.
nd
Fi

ANSWERS

4. (i)/(A) =1-4,-3,0,5} (ii) 4), {0, 2},-2. 5./(I) =5,/(-I) =-5,/(O) =1,/(2) =9
6. (i) R'^ (set of all real numbers greater than or equal to zero) (ii) {-2, 2} (iii) (})
7. (i) R (ii) (iii) Yes
8. (i) {(1, 3), (2, 6), (3, 9)}; Function (ii) {(1, 4), (1, 6), (2, 4), (2, 6)1; Not a function
(iii) {(0, 3), (1, 2), (2,1), (3, 0)}; Function.
9. No, Since domain of / domain of g.
10.
(i) R"" ={YeR|.Y>0| (ii) {A-eR:-l <.T<1} (hi) {.t e R : y >1}.

ri. (i) 12. (3,13,7,5,2,17} 13. (17)= (-4, 4},/-I (-3)=4>


14. (iii) 15. Range(/)=|3,5,11,13|. 17. 2.1

18. / = K-1, 0), (0,1), (3, 28), (9,730), (7, 344)}


15.9
FUNCTIONS

HINTS TO SELECTED PROBLEMS

function because (2, 9) and


11. (iii) /3 = {(1,5), (2,9), (3,1), (4,5), (2,11)} is not a

(2,11) e/3, which means that 2 is related to two elements in \/.


12. Clearly, / (12) = highest prime factor of 12 = 3. Similarly, / (13) = 13, / (14) =7, / (15) =5,
/ (16) =2 and/(17) =17. Hence, range (/) ={3,13,7, 5, 2,17}.
15. We have,
/I ={9,10,11,12,13} and/: A ->Nis defined by/ (m) = the highest prime factor ofn.
/ (9) = 3, / (10) =5, / (11) = 11, / (12) = 3 and/ (13) = 13

ow
Hence, range (/) = 1/(9),/(lO),/(ll), /(12), /(13)} = {3,5,11,13}
v2 0 < :v < 3 associates all numbers in [0,10] to numbers in R
16. We observe that /(.r) = Sx', 3<-Y<10
and no number in [0,10] is associated to two or more numbers. Hence,/ is a function. But,^?

e
is not a function because 2 is associated to two distinct elements viz. 4 and 6.

re
17. We have,/(.r) =

Flr
/(1-1)-/(1) _ (1-1)^-'^^ (11 + 1) (1.1-1)

F
= 21
(1.1)-1 (1.1)-1 (1.1-1)
15.5 REAL FUNCTIONS
ou
and co-domain both as subsets of the

sr
In this section, we will discuss functions having domain
set R of all real numbers. Such functions are called real functions or real valued functions of the
real variable as defined below.
fo
k
REAL VALUED FUNCTION A function f:A^ B is called a real valued function, ifB is a subset of R (set
oo
of all real numbers).
Y
If A and B both are subsets of R, then / is called a real function.
reB

In section 15.3.1, we have discussed the description of a function. Generally, domain and
co-domain both are infinite subsets of R in case of real functions of real variable. Therefore, a real
function is generally described by some general formula. In other words, images of various
uY

elements in the domain of a real function are provided by some general formula. For example,
.Y-1
/; K ^ R given by / (.y) = .y^ + .y +1 or, /: 71 -+ B given by / (.y) = .Y- etc. In practice, real
ad
do

- 4

functions are described by giving the general expressions or formulae describing them without
mentioning their domains and co-domains. Following are some examples of real functions.
in
Re

ILLUSTRATIVE EXAMPLES
F

EXAMPLF. 1 //fix) = S.v** - 5.v^ + 9, find f {x -1).


SOLUTION Wehave, / (.y) = -5.y^ + 9. Replacing xby (.y-1), we obtain
/(.Y-1) = 3(.Y-l)'^-5(.Y-l)^ + 9 = 3.y‘^-12.y^+13y^-2.y + 7
1 7 ■? n
EXAMPLE 2 If fix) = .Y + -.Y , prove that [/(.v)]'^ = fix^) + 3/ \X- J .
SOLUTION Wehave,
1
/(x)=.Y + Y
1 iv^
+ ^ and [fix)]^ X +
Xj
IV^
Now, Ifix)]^ = x + ~
X
15.10 APPLIED MATHEMATICS-XI

^ 1 f I']
=^X ^ + -.+ 3 x + ~
\ xj

=>
[/(a-)]^=/(x3)+3/W V f{x^)=x^ + ~ and /(.v) = A- + -
1

.Y

|/(x)]'’= /(x3) + 3/fi-


n 1 f I
.Y
v/(y) =Y + _{=>f{x)=f -
A V -Y

1 / A 2x +1 3
EXAMPLE 3 Iff (x) = — , X ^ - —2 , then show that j I f (x) = provided that .y .
2x +1 2x+ 3'
1
SOLUTION We have, / (x) =
2x +1

f \ ( I 1 1 2a- +1 2x +1
f /w =/ 2
2x +1 1 2 + 2x + 1 2x + 3
2 + 1 + 1
2x + l 2x +1

w
2x +1 f \ o
Clearly,/ / (x) = isrealfor2x+3?i0i.e./ / (x) isdefined for2x + 3 ^ Oi.e.x

F lo
2x + 3 2
2x +1 3
Hence, / / (.y) = provided that x .
j 2x + 3 '

e
Fre
x-1 f \ I
EXAMPLE 4 Iff{x) = , A- -1, then sho2v that f / (x) ~, provided that x 0.
A* + 1 X
for
SOLUTION We have,
x-1
r
fix) =
You

x + 1
oks

x-1
eBo

/ /w =/
^x + 1
x-1
-1
x + 1
our
ad

x-1 .
/ /(.V) = x-1 Replacing x by m
the formula for / (x)
+ 1 x + 1
x + 1

x-1 -x-1 -2 1
dY
Re

/ /w =
V J X -1 + X + 1 2x X
Fin

1 / A
Since
— is meaningful for x ^ 0. Hence, / / (x) = -X , provided that x ^ 0.

x^ -16
, X -4
EXAMPLES
Let fbe defined by f (x) = x - 4 and g be defined by ^(x) = ■ x+ 4
X ,x = -4

Find X such that f{x)=g (x) for all x.


SOLUTION We have,
/ (-'■) = g (t) for all X € R
/(-4) =g{-4)
-4-4 = X
X = -8.
[v /(x)=x-4 .■,/(-4)-4=-8]
x-1
3/(x) + l
EXAMPLE 6 Iff is a real function defined by f (x) = , then prove that: f (2a-) = -
x + 1
/(x) + 3 ■
15.11
FUNCTIONS

x-1
SOLUTION We have, / (a*) = A +1

fix) _x-l
1 x +\
f (.V) + 1 _ .V - 1 + -Y + 1 [Applying componendo and dividendo]
f{x)-l A -1 - A -1

/(●v) + l_
/(-v)-l

1-/W
A-l
Again, /(a) = A +1

w
2a-1
/(2a) =
2a+ 1

/(2-v) =
V-f(x)\
-1

F lo [Using (i)]

ee
J/(●'-)+ 1 ^ +1

Fr
ji-zw for
2/(a) + 2-1+/(a)
/(2.a) =
2/(a) + 2 + 1-/(a)
ur
3/(A)-fl
oks

/(2-a) =
/(A) +3
Yo
o
eB

EXERCISE 15.2

1. If/(a) = a^ - 3a + 4, then find the values of a satisfying the equation /(a) =/(2a +1).
our
ad

2. If/(a) =(a (a find /(rt + 10-


ax -b
3. Ify=/(A) = show that A = /(i/).
Y

1?A - a'
Re
nd

4. If/(A)=- ~—,show thatflf {/(a)11=.y.


Fi

-A

A + 1
5. If/(A) = - , show that / [/ (a)] = a.
A -

A“, when A < 0


6. If fix) = A , when 0 < A < 1
- , when A > 1
A

Find: (i)/(1/2) (ii)/(-2) (iii)/{!) (iv)/(V3) and {v) f(^).


1 (1
7. If/(.A)=A^ show that/(.v)+/ Uv- = 0.

8. If/(.v) =—^^,
1 + A^
show that/(tan 0)=sin 20.
15.12
APPLIED MATHEMATICS-Xl

.r-1
9. If/(.v) = then show that
x + \'
f U 1
(i) / - =-f{x) (ii) / -
X
Xj fix)
ANSWERS
- I ■>
1. .T = -l, 2/3 2. n~ b~

6- (i) \

ow
(ii) 4. (iii) 1 (iv) 4
V3
(v) does not exist 11.
1 n
- -bx
5

a~ -b^ I ^ a + b

HINTS TO SELECTED PROBLEMS


6. We have.

e
re
.v^ , when .v < 0
fix) = -Y , when 0 < Y < 1

Flr
1

F
- , when Y > 1
X

/n 1
ou [Using /(y)=y, 0<Y<1]

sr
1
(ii) f(-2) = (-2)2 = 4 (iii)/(I) ^
fo
= 1

(iv) /(V3) =
1
1

(iv)/(-^) ^ (-73)2 ^ 3
k
oo
73
Y

15.6 DOMAIN OF REAL FUNCTIONS


reB

Mathematically to define a function one has to provide its domain, co-domain and the images of
elements in its domain either by giving a general formula or by listing them one by one. As the
uY

domain and co-domain of real functions are subsets of R. Therefore, conventionally, real
functions are described by providing the general formula for finding the images of elements in
its domain. In such cases, the domain of the real function / (y) is the set of all those real numbers
ad
do

for which the expression for / (y) or the formula for f{x) assumes real values only. In
otherwords, the domain of / (.v) is the set of all those real numbers for which / (y) is meaningful.
in

3y - 2
For example, a real function / (y) described by the general formula f (y) =
Re

assumes real
.2
Y^ -1
F

3v — 2
values for all y s R except for y = ± 1, because denominator of — becomes zero for y = ± 1.
y2-1
So, domain off (x) is the set of all real numbers other than -1 and 1 i.e. domain if)=R -(-1,1).
Following examples will illustrate the procedure for finding the domain of a real function of a
real variable.

ILLUSTRATIVE EXAMPLES

EXAMPLE 1
Fimi the liomaiii of each of the foUozuing real valued functions:
(i) fix) = ^
Y + 2
(u)/W=^’“'
y-3

2y-3 y2 + 3y + 5
(iii)/(.v) = (iv)/(Y) =
Y 2 - 3y + 2 y2 -5y + 4
15.13
FUNCTIONS

SOLUTION (i) We have, / {x) = —


X +2

Clearly, / (x) assumes real values for all real values of ^ except for the values of x satisfying
x + 2=0i.e..v=-2. Hence, Domain(/) = R'-{-2l.
x-1
(ii) We have, / (.v) = X - 3
x-L1 .
We observe that / (x) is a rational function of x as -—-
is a rational expression. Clearly, / (x)

assumes real values for all x except for the values of x for which x — 3 = 0 i.e. x = 3.
Hence,Domain{f)=R -13}.
2x-3
(iii) We have, / (x) =
X ^-3x + 2
2x-3
Clearly, / (x) is a rational function of x as is a rational expression.

w
X 3x + 2

F lo
We observe that / (x) assumes real values for all x except for
x'^ -3x +2 = 0 i.e. x = 1, 2. Hence, Domain (/) = R - {1, 2).
x^ + 3x + 5
all those values of x for which

e
(iv) We have, / (x) =

Fre
.2
x^ -5x + 4

x^ + 3x + 5
for
Clearly,/(x) is a rational function of x as is a rational expression in x. We observe
X 5x + 4
r
that/ (x) assumes real values for all x except for all those values of x for which x - 5x + 4 = 0 i.e.
You
oks

X = 1,4. Hence, Domain (/) = R-{1,4}.


eBo

EXAMPLE 2 Find the domain of each of the following functions:


(iii) /(.V) = ^4-x2
(i) fix) =
(ii)/(x)=^
ad
our

SOLUTION (i) We have, /{x) = fx -2


Clearly,/(x) assumes real values for allxsatisfyingx-2 >0 => x>2 => xe[2, co).
Hence, Domain (/) = [2, co).
Re
dY

(ii) We have, / (x) - , ^ —


Fin

-X

_ .t > 0 ^ 1 > X => X < 1 => X € (- CO, 1).


Clearly, / (x) assumes real values for all x satisfying 1
Hence, Domain (/) =(-co, 1).
(iii) We have, /(x) =-^4 - x^
Clearly, / (x) assumes real values for all x satisfying
4 - x^ > 0 =^> - (x^ 4) > 0 => x^ - 4 < 0 => (x - 2) (X + 2) < 0 x e [- 2, 2].
Hence, Domain (/) ~ [-2, 2].
1
EXAMPLES Find the domain of the function/(x) defined by f{x) =./4 -X +
2-1
X

SOLUTION Clearly, / (x) is defined for all x satisfying


4 - X > 0 and x^ -1 > 0
15.14
APPLIED MATHEMATICS-XI

=> ;ic:-4<0 and (x -1) (:i-+ 1) > 0


x<4 and (x<-l or :r>l)
=> xe(-co,-1)u(1,4].
Hence, Domain(/) = (-co,-1) u(l, 4].
15.7 RANGE OF REAL FUNCTIONS
The range of a real function of a real variable is the set of all real values taken by / (x) at points in
i".
its domain. In order to find the range of a real function / (x), we may use the following
algorithm.
ALGORITHM

STEP 1 Puty=f{x).
STEP II
Solve the equation y =/ (x) for x in terms ofy. Let x = (|)(i/).
STEPIII
Find the values ofy for which the values ofx, obtained from x = ^ (i/), are real and in the domain

w
Off
STEP IV
The set of values ofy obtained in step III is the range off

F lo
Following examples will illustrate the above algorithm.
ILLUSTRATIVE EXAMPLES

e
Fre
Y — 9
EXAMPLE 1
Find the domain and range of the function f (x) given by f (x) = 3 -X
.

x-2
for
SOLUTION We have,/(x) =
3 -X
r
You

Domain off Clearly, / (x) is defined for all x satisfying


s

3 -x 0 i.e. x 3.
ook

Hence, Domain {f)=R- {3}.


eB

Range off Let y =/(x). Then,


x-2
3y + 2
y =
3-x
=> 3y - xy = X - 2 ^ x (y +1) = 3y + 2 => x =
our

y + 1
ad

Clearly, x assumes real values for all y except y + 1 = 0 i.e. y 1.

Hence, Range (/) = K-l-l).


dY
Re

EXAMPLE 2 Find the range of each of the followhig functions:


1
(i)/(-x) = (ii) / (X) = ^16-x^ (iii) f(x) = (iv) /(X) =
Fin

1 + X^ 2 - x^

SOLUTION (i) We have, / (x) = ^


fx-5
Clearly,/(x) takes real values for all X satisfying X-5 >0 => x>5 =i> X€(5, co).
Domain (/) = (5 /

For any x > 5, we have


X -5 >0 Jx -5 > 0 ^ > 0 => / (x) > 0.
fx-5
Thus, / (x) takes all real values greater than zero. Hence, Range (/) = (0, co).
(ii) We have,/ (x) ^-Jl6-x^
We observe that / (x) is defined for all x satisfying
FUNCTIONS 15.15

16-x^ >0=> x^-16<0=> (x-4)(x + 4)<0 => -4<x<4=> xe [-4,4]


Domain (f) = [- 4, 4].
Let v=/(3c). Then,
y = -yjl6-x^ => =16-x^ => Tp- =16-y^ => x = ^Jl6-}/^
Clearly, x will take real values, if

rw
16-y^>0=> y^-16<0=> (y-4)(y + 4)<0=> -4<y<4=> ye[-4,4]
Also, y = yjl6 - x^ >0 for all ;e e [- 4,4]. Therefore, y e [0, 4] for all x e [- 4,4].
Hence, Range (/) = [0,4]

e
e
(iii) We have, / (x) = —
l + x'^

lo
r
We observe that/(jc) takes real values for all xeR. Hence, domain (/) = R.

F
Let y =/ (at).Then,

u
l±-Jl-4y^
y=f{x)^ y =
l + x^
=>
oF
y-x + y = 0=> X =
2y

rs
Clearly, x will assume real values, if

k
1 -4y^ >0 andy^O

o
=> 4y^ -1 <0 and y^O
of
o
- — <0 and y^O
Y
IV i\
rYB

- <0 and y^O


y + TT
y-2 y V 2J ^
1 1
— < V ^ — and V 9^ 0
ue

2^2 ^
ye[-l/2,l/2]-{0}
od

Also, y = 0 for a: = 0.
ad

Hence, Range (/)=[-1/2,1/2]


in

(iv) We have,/(a:) = —
2 - a:
Re
F

For / (a:) to be real, we must have


2-A:^5tO => X 9^ ±^/2
Domain (f) = R V2, V2|
Let/(a:) =y.Then,
y = f(x)^y = 2y-x^y = 3=> x^y = 2y-3=> x = ±,
2 - a:'^ V y
We observe that x will take real values other than - V2 and ^/2, if
2y-3 [See Fig. 15.5]
> 0 => y 6 (- 00 ,0) u [3/2, oo)
y

Hence, range (/) = (- oo ,0) u [3/2, oo).


-00 0 3 00

2
2v- 3
Fig. 15.5 Signs of —
15.16 APPLIED MATHEMATICS-XI

2-9
EXAMPLE 3 Find the domain and range of the function f (x) = -
x-3

2-9
SOLUTION We have, f(x)^-
^ x-3
Domain off Clearly,/ (.y) is not defined for y - 3 = 0 i.e. y = 3. Therefore, Domain (/) = R - {3}.
Range off Let f{x)=y. Then,

f(x)=y^
y2 -9 ^
[●/ Y ^ 3]

It follows from the above relation that y takes all real values except 6 when x takes values in the
set i? - {3}. Therefore, Range (/) = R-{6}.

ow
4-y
EXAMPLE 4 Find the domain and range of the real valued function f (y) given by f{x) =
y-4

4-y
SOLUTION We have,/(Y) =
y-4

e
re
rFl
Domain of f: We observe that / (y) is defined for all y except at y = 4. At y = 4, / (y) takes the

F
indeterminate form -0 . Therefore, Domain (/) = R - {4).

Range off: For any x e Domain (/) i.e. for any y ^ 4, we have

r
4-y -(y-4) fo
ou
fix) = = -1.
y-4 y-4
ks
Range (/)={-1}.
oo

y2
EXAMPLES letf=\ Y, K ● be a function from R intoR. Determine the range of f.
Y

: X e
eB

1 + y2
y2
SOLUTION We have, / (y) =
ur

y2+1
ad

Domainoff Clearly,/(y) is defined for all y e R asY2 +1 Ofor any y e R. So, Domain(/) = R.
Yo

Range off: Let/(Y) = y. Then,


d

= y
Re

2
in

y2 + 1
= y=> x^ x2y + y => Y2(l-y) = y=> y2 = ±
y

p-y
F

Clearly, x will take real values, if


y > 0
+ +
i-y
0 1
y-0
- 00 00

< 0
y
y-i Fig. 15.6 Signs of
1-y
0<y<l

y e [0,1) [See Fig. 15.6]


Hence, range (/) = [0,1).
r/
1
EXAMPLE 6 Find the domain and range of thefinction f = ■ Y: : Y e R, Y ± 1 ●.
1-y2

SOLUTION We have, / (y) - —


1 -y2
FUNCTIONS 15.17

Domnin off: Clearly, / (a*) is defined for all x eR except for which ,v^ -1 9^ 0 i.e. x - ± 1-
Hence, Domain of / =
Range off: Let/(a) = y. Then, ^
t/-l
= y => 1 -.v'^ =— => X 2 ^2 _i
1 2 1 => A- = ±
f{x)=y => -
-a-2 y y y Vy-0
Clearly, .v will take real values, if 1-

y-1 >0
— 00 0 I CO

y-0 J/-1
Fig. 15.7 Signs of
y-0

ow
=> y < 0 or y > 1
y e(-x, 0) L.' [1, oo) [See Fig. 15.7]
Hence, range (/) = (- co, 0) u [1, <»).
EXERCISE 15.3

e
1. Find the domain of each of the following real valued functions of real variable:

re
1 3.Y-2
(i)/(-v) = -X (ii) /(.V) =

rFl
x-7
(iii) /W = Y + 1

F
2y + 1 x^ + 2x +1
(iv)/(.A) = -9 (v) fix) = ^

r
.2
- 8y + 12
ou
Y

fo
2. Find the domain of each of the following real valued functions of real variable:
ks
1
(i) fix) = (ii)/W =
oo
Y

y-2
B

(iv)/W =
p-x
re

3. Find the domain and range of each of the following real valued functions:
rtY + b AY -b
ou
Y

(i) fix) = (ii) fix) =


ad

bx - a cx-d

(iii) fix) = (iv) fix) =


d

y-2
(v)/(.v) = (vi) fix) - |y-1|
in
Re

2-y

(viii) /(y)=-J9-y^
F

(vii)/(Y)=-|Yi
1
(ix) fix) = (x) /(y)-Vy^-16
Vi6-y^"
ANSWERS

1. Domain 2. Domain Range


(i) R-101 (i) [2, «) [0, co)
(ii) K-(7) (ii) (-CO,-1) u(l, co) (-CO,-l]u(0, oo)
(hi) R-l-H (hi) [-3,3] [0, 3]
(iv) R-1-3, 3} (iv) [2, 3) [0, co)
(v) R-{2,61
3. Domain Range Domain Range
a a a
R - (ii) R - - R -
(i)K- ;; b c c
15.18 APPLIED MATHEMATICS-XI

(iii) [1, co) [0, oo) (iv) [3, co) [0, oo)
(v) R -12) 1-11 (Vi) R [0, co)
(vii) R -CO, 0] (viii) [- 3, 3] [0, 3]
1
(ix) (-4, 4) 00
(x) (-00, -4) u[4, co) [0, oo)
4'

HINTS TO SELECTED PROBLEMS

+ 2X + 1
1- (v) f(x) = is defined for all a: satisfying
x^ - 8x + 12 (X- 6) (x - 2)
(at - 6) (a: - 2) 0 i.e. a: 5^2, 6.
Domain (f) = R - {2, 6}
3. (iii) / (at) = yJx-1 is defined for all a: satisfying a: -1 > 0 i.e. a: > 1. So, domain (/) = [1, oo).

w
Let y = ^-1. Clearly, y > 0 for all a: e [1, oo ). So, range (/) = [0, oo).
(vi) We have,/(a:) =| at -1 |. Clearly,/ (a^) is defined for all a: 6 R. So, domain (/) =R.

F lo
Also, / (a:) = I a: -11 > 0 for all a: e K. So, range (/) = [0, co).
(vii) We have / (a:) = -1 a: |

ee
We observe that / (a:) is defined for all x sR. So, domain (/) = R.

Fr
Also,I a:I > 0 for ail a: e R => -| a:| < 0 for all a: g K => / (a:) < 0 for all a: e K.
So, range (/)=(- co, 0]^ for
(viii) We have,/(a:) = -^9 - .v^ . Clearly,/(a:) takes real values if
ur
9 - a:^ > 0 => a:^ - 9 < 0 => (a: - 3) (x + 3) < 0 => x e [- 3, 3]
s
ook
Yo

Domain (/)=[-3, 3]
Also, / (x) = -^9- x^ > 0 for all x g [- 3, 3].
eB

Let y = ^9-x^. Then, y^ = 9 - x^ => x^ + = 9 => X =

Clearly, x g K if y g [- 3, 3]. But, y > 0. Therefore, y g [0, 3].


r
ad
ou

Hence, range (/) = [0, 3]


15.8 SOME STANDARD REAL FUNCTIONS AND THEIR GRAPHS
Y

In this section, we shall discuss some standard real functions which frequently occur in the
Re
nd

study of calculus.
Fi

CONSTANT FUNCTION Ifk is a fixed real number, then a function f (x) given by f {x)=k for all x e K
is called a constant function.

y,

fix)=k
k

X' O X

Y'

Fig. 15.8 Constant function


15.19
FUNCTIONS

Sometimes we also call it the constant function k.


We observe that the domain of the constant function/ (x) =kis the set f? of all real numbers and
range of / is the singleton set {k}.
The graph of a constant function/ (x) = cf is a straight line parallel to :r-axis (See Fig. 15.8) which
is above or below x-axis according as k is positive or negative. If ^: = 0, then the straight line is
coincident to :r-axis.

IDENTITY FUNCTION The function that associates each real number to itself is called the identity
function and is usually denoted by 1.
Thus, the function I :R ->R defined by I (x) for all xsR is called the identity function.
Clearly, the domain and range of the identity function are both equal to R.
The graph of the identity function is a straight line passing through the origin and inclined at an
angle of 45® with X-axis.
y

w
F lo
ee
X' o X

Fr
for
ur
y
s
Fig. 15.9 Identity function
ook
Yo

MODULUS FUNCTION The function fix) defined by /(x)=|3:|=|


X , zvhen :r > 0 is called the
eB

- X , when a: < 0

modulus function.
r

It is also called the absolute value function.


ad
ou

We observe that the domain of the modulus function is the set R of all real numbers and the
range is the set of all non-negative real numbers i.e. = (x e R : x > 0).
Y

The graph of the modulus function is as shown in Fig. 15.10. for x > 0, the graph coincides
Re
nd

with the graph of the identity function i.e. the line y = x and for x < 0, it is coincident to the line
Fi

y = -x.
y

X' o X

y’

Fig. 15,10 Modulus function


15.20 APPLIED MATHEMATICS-XI

PROPERTIES OF MODULUS FUNCTION The modulus function has the following properties:
(i) For any real number x, V? = [ ^ | ■
K
COS X , 0<x<-
For example, ^cos^ x = | cos x| = ■ 2
K
-COS X , — <X <Tl
2

(ii) If a, b are positive real numbers, then


o A:|<fl -a<x <a
x^>a^ o |T|>fl <=> x<-aor,x>a
2 2 II
X <a o\x\<a<^-a<x<a
2 2
;r >a^ <=>
A:|>rt o x <-a or, X >a
a^<x^<b^ a<\x\<b o x ^[-b, - a]u[a,b]

w
<x^ <b^ ^ a<\x\<b <=> x ^{-b, - a) <j{a,b)

F lo
(iii) For real numbers x and y, we have
|x + y| = X +1 y I O’ (x > 0 and y > 0) or, (x < 0 and y < 0)
|x-y| =[:^|-|y| (x>0,y>0 and| xj >| y |) or, (x < 0 , y < 0 and| x| >| y |)

ee
^±yl ^1 ^1 +1 y

Fr
|x±yi> |x| -|y|
GREATEST INTEGER FUNCTION (FLOOR FUNCTION) For any real number x, we use the symbol [x]
for
ur
or, [_xj to denote the greatest integer less than or equal to x.
ks

For example, [2.75] - 2, [3] = 3, [0.74] = 0,[-7.45] = 8 etc.


Yo

The function f :R -)■ R defined by f (x) = [x] for all x sR is called the greatest integer function or
oo

the floor function.


eB

It is also called a step function.


Clearly, domain of the greatest integer function is the set of all real numbers and the range is
r

the set Z of all integers as it attains only integer values.


ou
ad

The graph of the greatest integer function is shown in Fig 15.11.


Y

n
Re
nd

3
Fi

2 ■o

1 o

o
X' -3 -2-1 1 2 3 X
-1

o -2

o -3

Fig, 15.11 Greatest integer function

PROPERTIES OF GREATEST INTEGER FUNCTION Ifn is an integer and x is a real number between n
and n + 1, then
FUNCTIONS 15.21

(i) [-«] = -[>i] (ii) [x + cf] = I-v] + k for any integer k.


-1 , if X g Z
(iii) \-x] = -[x]-l (iv) M + [-x] = 0 , ifxeZ

2[.v] + 1 , ifATgZ (vi) [a:]>fc => X >k, wherek eZ


(v) M-I-a:] = 2[a-1 / if a; e Z

(vii) [.r] <k ^ X <k + I, wherefc eZ (viii) [a] >k => .Y >^: +1, were k eZ
(ix) [.v] <k => x<k, where^:eZ (x) [a + y] = [a] + [y + A - [a]] for all x,yeR

ow
1 2 n-l
(xi) [a] + A + - + A + — + ... + A + = [«a] ,neN.
n n n

SMALLEST INTEGER FUNCTION (CEILING FUNCTION) For any real number a, we use the symbol
[a] to denote the smallest integer greater than or equal to x.

e
re
For example, [4.71 [-7.21 =-7, [0-^51 =1 etc.

Frl
The function f:R^R defined by f (a) = [a1 for all xeR is called the smallest integer function or the

F
ceilingfunction.
It is also a step function.
ou
r
We observe that the domain of the smallest integer function is the set R of all real numbers and
its range is the set Z of all integers. so
kf
Y
oo
Y
eB
ur
oY

O
ad

X' X
d
in
Re
F

Y'

Fig. 15.12 Smallest integer function

The graph of the smallest integer function is as shown in Fig. 15.12.


PROPERTIES OF SMALLEST INTEGER FUNCTION Following are some properties of smallest
integer function:
(i) -71 =-[7j1, where fieZ
(ii) [-a1=-[a1 + 1, whereAeK-Z
(iii) [a + /i1 = [a1 + ;j , where a e 1^ - Z and h £ Z
. r -I r -I f1 / if A g Z
(IV) M + r-*l= |o , ifjeZ
(V) N-r-i= mi
15.22 APPLIED MATHEMATICS-XI

FRACTIONAL PART FUNCTION For any real number x, we use the symbol {.r} to denote the
fractional part or decimal part of a:.
For example, [3.45] =0.45, [-2.75] = 0.25, [-055] = 0.45, [3] =0,[-7] = 0 etc.
The function f :R ^ R deifned bi/ f (.t) = [.x^ for all x e Ris called the fractional part function.
We observe that the domain of the fractional part function is the set R of all real numbers and the
range is the set [0,1).
It is evident from the definition that f{x) = W = .r - [.r] for all x eR
Y

w
(0,1)

X’ -2 -1

F loO 1 2 3 X

ee
Fr
y for
Fig. 15.13 Fractional pari function
ur
The graph of the fractional part function is as shown in Fig. 15.13.
s
ook
Yo

SIGNUM FUNCTION Tiiefunctionf defined by


Y
1 , Y>0
eB

, X 0
fix) = X or, / (y) = -^ 0 , Y = 0
0 , -Y = 0 -1 , Y<0
our

is called the signum function.


ad

The domain of the signum function is the set R of all real numbers and the range is the set
{-1,0,1}
Y

Y
Re
nd
Fi

(0,1)

X' O X

(0,-1)

Y'

Fig. 15.14 Signum function

The graph of the signum function is as shown in Fig. 15.14.


15.23
FUNCTIONS

EXPONENTIAL FUNCTION If a is a positive real number other than unity, then a function that associates
each X eRtoa^ is called the exponential function.
In other words, a function f: R —> R defined by / (x) =a^, where d > 0 and n 1 is called the
exponential function.
We observe that the domain of an exponential function is R the set of all real numbers and the
range is the set (0, oo) as it attains only positive values.
As <7 > 0 and a^l. So, we have the following cases:
CASE 1 When a>\

We observe that the values of y =/ (x) = a^ increase as the values of x increase.


y

y = a^,a>\

w
F lo
(0,1)

X’ O X

ee
Fr
for
r
r
Fig. 15.15 Exponential Junction/(jr) = / forfl>]
You
s
ook

< 1 for X < 0


Also, /(x) = a^ = 1 for X = 0
eB

> 1 for X > 0.

Thus, the graph of / (x) =a^ fora > I as shown in Fig. 15.15.
our
ad

We also observe that:


dY
Re
Fin

y = 2"-
y=y
¥

X' o X

Fig. 15.16 Graphs of exponential functions

2^ < 3^ < 4^ <... for all X > 1


15.24
APPLIED MATHEMATICS-XI

2"^=3^=4^=...=lforx = 0
2Ar>3^ >4^ >_ forallA:<l

y = fl’’, 0 < a < 1 y,

(0,1)

X' O -X

w
Y'

F lo
Fig. 15.17Graphofexponentialfunction f{x) = a^' for0<a<l

e
Fre
for
r
You
o ks
eBo

X' O X
our
ad

Fig. 15.18
dY

So, the graphs of / (.v) = 2^, f (a:) = 3^, / (.r) = 4^ etc. are as shown in Fig. 15.16.
Re

CASE 11 When 0 < <7 < 1


Fin

In this case, the values of y =/ (;t) = decrease with the increase in x and y > 0 for all x eR.
Also,
>1 for a: < 0
y = f (x) = i= 1 for a: = 0
<1 for a: > 0

Thus, the graph of / (a) = for 0 < < 1 is as shown in Fig. 15.17.
The graphs of / (a:) = <7^', 0 < a < 1 for different values of a are shown in Fig. 15.18.
REMARK Wehave,l<e < 3. Therefore, graph off (x) =e^ is identical to that off {x) = for a > 1 and
the graph off(x) =e~^ is identical to that of f{x) = a^ for 0<a<l.
LOGARITHMIC FUNCTION If a >Qand a then the function deifned by f (Ar) = loga x, x >Oiscalled
the logarithmicfiinction.
Previously we have learnt that the logarithmic function and the exponential function are inverse
functions i.e. log^, .r = y <=> x = a^.
FUNCTIONS 15.25

We observe that the domain of the logarithmic function is the set of all non-negative real
numbers i.e. (0, oo) and the range is the set R of all real numbers.
As <7 > 0 and <7 1. So, we have the following cases.
y
/(A.) = logj.Y, n>l

X' 0/ (1,0) X

w
r

F lo
Fig. 15.19 Logarithmic function/(.t) = log,,Ar for (?>!

CASE 1 When a>\

ee
In this case, we have

Fr
< 0 for 0 < a: < 1
y = logrt 0 for.\-=l for
> 0 for .V > 1
r
Also, the values of y increase with the increase in x.
You
s

So, the graph of y = log„ a: is as shown in Fig. 15.19.


ook

CASE II When 0<a<l


eB

In this case, we have


> 0 for 0 < a: < 1
y = logrt ^ = 0 for a: = 1
our
ad

< 0 for a: > 1

Also, the values of y decrease with the increase in x.


dY

y.
Re
Fin

(1,0)
X' o X

/(j)=log„x,0<fl<l
y'

Fig. 15.20 Logarithmic function f{x) = log„Ar,0<(7<l

So, the graph of y = log^ a: is as shown in Fig. 15.20.


15.26 APPLIED MATHEMATICS-XI

PROPERTIES OF LOGARITHMIC FUNCTION Following are some useful properties of logarithmic


function:
(i) logfl 1=0, where a>Q,a^\
(ii) log^ a = \, where a >0, a
(iii) logfl (xy) = logrt I ;c I + logrt \y\, where a>0,a=^l and xy > 0
^x' X
(iv) ●og, = *og(71 I “ logfl \y\> where a>0,a*-[ and — > 0
y y
(V) logfl (x'*) = n logfl I XI, where a>0,a^l and x" > 0

(vi) log H x"' = — logfl X, where a>0,a=^l and x > 0

logo -T
(vii) = y , where x>0, y>0, rt>0,
(viii) If fl > 1, then the values of / (x) = log^ x increase with the increase in x.

w
i.e x<y o log^^<log„y
[ < 0 for 0 < X < 1
Also, loga X = 0 for X = 1

F lo
> 0 for X > 1.

(ix) If 0 < fl < 1, then the values of / (x) = log^ x decrease with the increase in x.

ee
i.e x<y « logaX>logay

Fr
f> 0 for 0 <x <1
Also, loga X ■ = 0 for X = 1 for
<0 for X > 1
ur
1
(x) loga X = for <7 > 0, fl 1 and x > 0, x 1.
s
ook
Yo

Rl'MARK Functions f{x)=\ogi, x andg(x) =fl^ are inverse of each other. So, their graphs are mirror
eB

images of each other in the line mirror y = x.


RECIPROCAL FUNCTION The function that associates a
real number x to its reciprocal 1/x is
r
ad
ou

called the reciprocal function. Since 1/x is not defined for x = 0. So, we define the reciprocal
function as follows:
Y

Y
Re
nd
Fi

1
i/ = x
X'

o
X

Fig. 15.21 Reciprocal function


FUNCTIONS 15.27

1
DEFINITION The function / :R-{01->R defined by f (x) =-is called the reciprocal function.

Clearly, domain of the reciprocal function is R - {0} and its range is also R - {0}.
We observe that the sign of —X is same as that of x and —X decreases with the increase in x.

So, the graph otf(x) = —is as shown in Fig. 15.21.

SQUARE ROOT FUNCTION The fimction that associates a real number x to + ^fx is called the
square root function. Since-Jx is real for a: > 0. So, we defined the square root function as follows:
y,

1/2

w
F lo
ee
X' O X

Fr
for
ur
s
r
ook
Yo

Fig. 15.22 Square root function


eB

DEFINITION The function f\K^ R defined by f {x)-+^fx is called the square root function.
Clearly, domain of the square root function is R"*" i.e. [0, oo) and its range is also [0, co).
r
ad
ou

We observe that the values of / (x) = + increase with the increase in x. So, the graph of
/ (x) =+ Vx is as shown in Fig. 15.22.
Y

SQUARE FUNCTION The function that associates a real number x to its square i.e. x^ is called the
Re
nd

square function. Since x^ is defined for all x e R. So, we define the square function as follows:
Fi

= .r2

X' o X

Fig. 15.23 Square function


15.28 APPLIED MATHEMATICS-X!

DEFINITION The function f:R-^R defined by f (x) = is called the square function.
Clearly, domain of the square function is R and its range is the set of all non-negative real
numbers i.e. [0, co). The graph of / (x) = is parabola as shown in Fig. 15.23.
CUBE FUNCTION The function that associate a real number .y to its cube is called the cube
function. We observe that x^ is meaningful for all x eR. So, we define the cube function as
follows:
Y

y = AT^,

low
X' O X

ee
F
Fr
Y'

Fig. 15.24 Cube function

DEFINITION The function f :R -> R defined by f (.t) = for


is called the cube function.
ur
We observe that the sign ofY^ is same as that of y and the values of y^ increase with the increase
in Y. So, the graph of /(y) = y^ is as shown in Fig. 15.24. Clearly, the graph is symmetrical in
k s
Yo

opposite quadrants.
oo
eB

CUBE ROOT FUNCTION The function that associates a real number x to its cube root
called the cube root function. Clearly, x^^^ is defined for all y e R. So, we define the cube root
function as follows:
r
ou
ad

DEFINITION The fimction f :R -> R defined by f (y) = x^^ ^ is called the cube root function.
Clearly, domain and range of the cube root function are both equal to R.
Y

Also, the sign of y^^ ^ is same as that of y and x^^ ^ increase with the increase in y. So, the graph of
Re
nd

/ (y) = Y^^ ^ is as shown in Fig. 15.25.


Fi

X' o X

y-i

Fig. 15.25 Cube root function


FUNCTIONS 15.29

REMARK I A function f:R^Ris said to be a polynomial function if f (:t) is n polynomial in x. For


example, f (.i-) =x'^ - x + A,g{x)=x^ + 3x^ + fl x -1 etc are polynomial functions.
Pix)
REMARK 2 A function of the form f (;<;) = where p (a:) and q (a:) are polynomials and q (a:) ^ 0, is
qix)'
p{x)
called a rational function. The domain of a rational function f (a;) = is the set of all real numbers,
q{x)
except points where q (a:) = 0.
15.9 OPERATIONS ON REAL FUNCTIONS
In this section, we shall introduce various operations, namely addition, subtraction,
multiplication, division etc. on real functions.
ADDITION Let f:Di ^ R and g:D2 ^ R be two real functions. Then, their sum f + g is defined as
that function from Dj n D2 to R which associates each a: e n D2 to the number f (a:) + g (a:).

w
In other words, if /: Dj -> R and ^: D2 -> R are two real functions, then their sum / + ^ is a
function from Dj n D2 to R such that

F lo
if + g){x)=f{x)+g{x) iora\lx^D^nD2-
PRODUCT Let f:Di^R and g:D2^R be tzoo real functions. Then, their product (or pointzvise
multiplication) f g is a function from Dj r\D2toR and is deifned as

ee
if g) (x) = fix) g (x) for all a: e Dj n D2

Fr
DIFFERENCE (SUBTRACTION) Let f-.D^-^R and g:D2^R be tzvo real functions. Then the
difference ofgfrom f is denoted by f -g and is defined as for
if-g)ix) =fix)-g(x) forallx^D^nD2
ur
QUOTIENT Letf-.Di -^Randg:D2 Rbetzvorealfunctions.Then the quotient off by g is denoted
s

by — and it is a function from D;j n D2 - (x: g (x) = 0) to R defined by


ook
Yo

g
eB

fix)
for all X £ Dj n D2 - {x: g (x) = 0)
our

MULTIPLICATION OF A FUNCTION BY A SCALAR Let f: D -> R ba a real finclion and a be a scalar


ad

(real number). Then the product a f is a function from D to R and is defined as


(a/)(x) = a/{x) forallxeD.
Y

1 .
RECIPROCAL OF A FUNCTION ///:D ^ R is real function, then its reciprocal function - is a
Re

f
nd

(l'] 1
Fi

function from D ~{x:f (x) = 0) to R and is defined as — (x) =


f fix)
REMARK 1 The sum, difference product and quotient are defined for real functions only on their
common domain. These operations do not make any sense for general functions ezjen if their domains are
same, because the sum, difference, product and quotient may or may not be meaningful for the elements in
their common domain.
REMARK 2
For any real function /: D n R and n e N, we define
(///..../)(x)=/(x)/(x).../(x) = {fix)f forallxsD
ji-times H-times

ILLUSTRATIVE EXAMPLES

EXAMPLE 1 Find the sum and difference of the identity function and the modulus function.
SOLUTION We know that /: R ^ R defined by /(x) = x is the identity function and g:R->R
defined by g(.x)=|x] is the modulus function. Clearly, f and g have the same domain.
15.30 APPLIED MATHEMATICS-XI

Therefore, / + g:R -> R and f -g:R -> R.


Now,
(/ + g) (x) = fix)+g ix)
if + g) (x) = + [ -VI
x + X , if X > 0
if + g)(x) = X - X , if ;r < 0

2x , if a:>0
{f + g){x) = 0 , ifx<0

and. (f-g){x) = f{x)~g{x)


{/'■g){x) = a:-|:r|
x-x , if .r>0
if-g)ix) = x-(-a:) , if a: < 0

(f-g)(x) = I
0 , if a: > 0

w
2x , if a: < 0

Thus, f + g- R

if+ g)(x) = j
R and f -g: R -> Rare defined as
2x ,
0 ,
if AT > 0
if X < 0
F lo
and, (f~g)(x) = I 2x ,
0 , if X > 0
ifx<0

EXAMPLE 2 What are the sum and difference of the identityfunction and the reciprocalfunction ?

e
Fre
SOLUTION Let / and g denote respectively the identity function and the reciprocal function.
for
Then, f :R R and ^: i? - (0) -> R such that / (x) = x for all x e R and, (x) = — for all
X

xeR-{0).
r
The domains of / and g are R and R - {0} respectively. Also, we have R nR - \0] = R - {0}.
You
oks

Therefore, f + g: R - {0}R and f - g: R - {0} ^ R are given by


eBo

if + g)ix) =f{x) + g(x) =x + -and,


X
(/-g)(x) =/(x)-g(x) =x--
X

EXAMPLE 3 Let f: [2, co) K and g:[- 2, <x>) ^ R be txoo real functions defined by f (x) = /x-2
ad
our

andg (x) = /x + 2. Find f + g and f -g.


SOLUTION Let D-^ = [2, oo) and D2 = [- 2, oj). Then, Dj n D2 = [2, co).
Thus, / + ^: [2, 00) ^ R and / - g: [2, co) —> R are given by
Re

(/ + g) (^) =f{x) + gix) = /x-2 + fx+2 for all x e [2, co)


dY

and. if-g) ix) =f (x) - g (x) = /x-2 - /x + 2 for all x g [2, co).
Fin

EXAMPLE 4 Find the product of the identity function and the modulus function.
SOLUTION Let / and g denote respectively the identity function and modulus function. Then,
/: R R such tliat /(x) = x for all x and, g:R->R such that g (x) = | x | for all x.
Clearly, / and g have the same domain. Therefore, the product / g’ is a function from R to itself
and is given by
ifg)ix) = fix)g{x) = xlx|=-
x^ , if X > 0
if X < 0

EXAMPLE 5 Find the quotient of the identity function by the modulus function.
SOLUTION Let / and g denote respectively the identity function and the modulus function.
Then, /: R R is defined as / (x) = x and, g:R R is defined as (x) = | x |.
Clearly, / and g have the same domain.
And, g (x) = 0 => I XI = 0 => X = 0.
15.31
FUNCTIONS

/ {01 R and is defined as


Therefore, the quotient oi f by g i.e. — is a function from R
8

X > 0
fix) X
a:
gix) 1X a: <0
-.t

EXAMPLE 6 Find the product of the identity function and the reciprocal function.
SOLUTION Let / and g denote respectively the identity function and the reciprocal function.
for
Then, f:R^Ris defined as/ (x) = x for all x e R and, g: R - {0) R is defined as g (x) = - l _

ow
all X e R - {0|.
Now, Domain (/) n Domain (g) = R n R - {0} = R - {0}
Therefore, the product^ is a function from R - {0} to R and is defined as
1
{fg)ix)=f{x)g{x) = xx- = 1 for allx e R -{0)

e
re
Thus, : R - {0} -> R is given by (/g) (x) =1 for all x g R -{0}.
EXAMPLE?

Frl
Find the quotient of the identity function by the reciprocal function.

F
SOLUTION Let / and g denote respectively the identity function and the reciprocal function.
Then,/ : R -> R is defined as / (x) = x for all xeR and, g: R -101 - R is defined as g (x) = - for
ou
or
allxGR-{0}
kfs
Now, Domain if) n Domain (g) = R n R - {0} = R - (01
And, g (.t) 0 for any x e R - {0}
oo

/:
- IS a function from R -10) R and is given by
Y
B

L {x) =
g(x) 1/x
re

g
( c\
-: R-{0} ^ R is givenby — (x)=x^ for all X e R -|0).
oYu

Hence,
ad

g g
X

EXAMPLE 8 Let cbe a non-zero real number and f:R-^R be a function deifned by fix) = ^
d

/1 \
in

for all X e R. Find (i) c/ (ii)c^/ (iii) - /●


Re

V /
F

^1 \
SOLUTION Clearly, cf,c^f and \C- ) / are functions from R to itself.
Now,

(i) icf) (x) = cfix) =cxf = X for all X G R

(ii) ic'^f) (x) = C^fix) = X - = cx for all x e R


fl^ 1
(iii) - / (T)= - fix)=--x —C =Ar forallxGR.
vV c > \c J c

EX.AMPLE‘) Let f and g be tioo real fiinctions defined by fix) -— <mdg(x) =(x + 4)^.
X + 4
1 1
/
Find the following: (i) / + g (ii)/-g (iii)/g (iv) - (v) 2/ (vi) — (vii) —
g / g
15.32 APPLIED MATHEMATICS-XI

1
SOLUTION We observe that/(;c) = is defined for all x - 4. So, domain (/) = R-{- 4).
X + 4

Clearly, ^ (jr) = (:r + 4)^ is defined for all a: s R. So, domain (5) = R.
Domain (/) n Domain (^) = K - {- 4).
(i) / + g: K - {- 41 -> K is given by
+ (a: + 4) ^ = (.r +4)^ + 1
1
(/ + {^) =/ (.v) + g (.v) = a: + 4 a: + 4

(ii) f -g: R -{-4} —> R is defined as


(/-^)(^) = f{x)-g{x) =
1
-(at + 4)^ l-(x + 4f
A-+ 4 A + 4
(iii) ^: R - 4) -> R is given by
(/^)W=/WgW= A +^ 4 (a + 4)^ =(a + 4)^
g(a) = 0 => (a + 4)^ = 0 => A = -4.

w
(iv)

Domain
/

F lo
= Domain (/) n Domain (g) - {a : g (a) = 0) = R - {- 4)
§

Thus, /. fix) 1
- : R - {- 4} -> R is given by (X) =

ee
s g g{x) (a+ 4)^

Fr
(V) If; R -(-4) -> R is given by
(2/)(a) = 2(/(a))= A +^ 4 for all A e R -(-4}.
for
r
(vi) We observe that/ (a) 9^ 0 for any a e R - {- 4}.
Therefore, ^ : R -1- 4) -> R is given by
You
s
ook

1 1 1
T (X) = = (a + 4)
eB

J/ fix) 1/(a + 4)

(vii) We observe that g (a) = (a + 4) ^ = 0 for a = - 4. Therefore, - : R - {- 4) ^ R is given by


our
ad

8
ll, « 1 1
- (^) = —
.8 gix) (a + 4)^
dY
Re

Let f and g be real functions defined by f (a) = .^a + 2 and g (a) = ^-x^. Then, find
EXAMPI.n 10
Fin

each of the foUozving functions:


f
(i) / + g (ii) f~g (hi) fg (iv) — (v) ff (vi) gg
8

SOLUTION We have, / (a) = fx + 2 and g (a) =


Clearly, / (a) is defined for all a satisfying
a + 2>0=> a>-2=^ ae[-2, 00)
Domain (/)=[- 2, 00)
We observe that g (a) is defined for all a satisfying
4 - A^ > 0 A^ - 4 < 0 => (A - 2) (a + 2) < 0 => A e [- 2, 2]
Domain (g) = [- 2, 2]
Now,
Domain (/) n Domain (g) = [- 2,00) n [- 2, 2] = [- 2, 2]
15.33
FUNCTIONS

(i) / + ^: [- 2, 2] ^ K is given by
(/ + = / (-V) + g {x) = + 2+
(ii) /- 2, 2] -> R is given by
(/-^)w = /w-sw =
(iii) /g: [- 2, 2] R is given by
(Jg) {x) - fix) g (X) = fx+2 X ^4-x^ ^{x + 2f{2-x) {X + 2) ^2^
(iv) We have, g (x) = ^4 - x^
g(x) = 0 => 4-x^ = 0 => X = ±2.
/
= h2,2]-(-2,21 = (-2,2)

w
So, Domain

/.
—: 2, 2) -> R is given by

If /(.V) ^ 1

F lo
ee
- (a:) =
g

Fr
(v) Since domain (/)= [-2, co). Therefore,
ijf)ix)=f{x)fix) = [fix)f = (V^)^ = ●V + 2forallxG[-2,co) for
ur
(vi) Since domain (g) =[-2, 2). Therefore,

W(A-) = g(A:)g(A-) = [g(-Y)f = [-^4


s
= 4 - x^ for all x € [- 2, 2]
ook

-X
Yo

Let f be the exponential function and g be the logaritlnnic function. Find:


eB

EXAMPLE 11

(i) (/ + 5){1) (ii) ifg)i^) (iii) (3/)(l) (iv) (5g)(l)


SOLUTION We have,
r

/ : R ^ R given by /(x) = e^ and, g:R^ R given by g (x) = log^ x.


ou
ad

(i) Since Domain (/) n Domain (^) = R n R = R Therefore,


Y

f + g-.R'*' ^ R is given by
Re
nd

if + g)ix) = fix) + gix) = e^ + loge A- for all x e R


Fi

if + g) (1) = + ioge 1 - c + 0 = e.
(ii) Domain (/) n Domain (g) = R n R = R'^. Therefore,/g:R^ -^R is given by
ifg) (a) = / (a) ^ (x) = . log, X.
(/g)(l)= e^xlog,! = ex 0=0
(iii) Clearly, (3/)(x) = 3(/(x)) = 3^^^
(3/)(l) = Sc’ = 3e
(iv) Clearly, (5g) (x) = 5 (^ (x)) = 5 log, x
(5^)(1) = 51og,l = 5x0 = 0.
EXAMPLE 12 Find the domain of each of the following functions given by
1 1
(i) fix) = (ii) fix) =
-JxT\\T
1 1
(iii) fix) = (iv) fix) =
fx-[x] ■fx + [a]
15.34 APPLIED MATHEMATICS-XI

1
SOLUTION (i) We have, / (x) =
■Jx-\x\
X , ifx>0
We know that \ x\ = -X , if a: < 0

a:-:»: = 0 , ifa:>0
■^”1^1 = X + x = 2x , if x < 0
at-|a:|<0 forall:»:
1
does not take real values for any x e R
/x-\x\
f (a:) is not defined for any x eR.
Hence, Domain (/) = c|i
1
(ii) We have,/(a:) =

low
V^ + l-v|
X , ifa:>0
We know that | x -X , if a:<0

a: + a: ^ j X+X , ifA.*>0
a: - a: , if .a: < 0

ee
2x , ifA:>0
a: + | a:| =
F
Fr
0 , if a: < 0
1
Thus, / (a;) = assumes real values, if
+ X
for
ur
a: +1 a:| > 0
x>0
s
[Using (i)]
k
Yo

X 6(0, x).
oo

Hence, Domain (/) = (0, x).


eB

1
(iii) We have, / (x) =

We know that 0 < x - [x] < 1 for all xeR. Also, x - [x] = 0 for x e Z.
r
ou
ad

Thus, / (x) = , \ , is defined, if


^Jx-[x]
Y

X - [x] > 0 ^ xeR-Z [●.■ X - [x] = 0 for X 6 Z and 0 < x - [x] < 1 for X 6 R-Z]
nd
Re

Hence, Domain {f)=R -Z.


1
(iv) We have, / (x)
Fi

yjx + [X]
We know that

X + [x] > 0 for all X > 0


X + [x] - 0 for X = 0
X + [x] < 0 for all X < 0

1
Also,/(x) = is defined for all x satisfying x + [x] > 0. Therefore, from (i), we obtain
V^+ [x]
that. Domain (/) = (0, co).
EXAMPLE 1.^
Find the domain of definition of the function f (x) given by
fix) = log4]log5 log3(18x-x^-77
SOLUTION We have.
FUNCTIONS 15.35

/(a:) = log4 hogs log3(18x-.r^-77) ●


Since log„ x is defined for all x > 0. Therefore, / (.r) is defined if
logs -77)1 > 0 and 18.r -77 > 0
=> logs (18.t-x^-77) >5^’ and -18x + 77 < 0
=> log3(18.t-A:^-77) >1 and (;c-11) (.v-7) <0
=> 183:-a'^-77>3^ and7cr<ll
^ 18x - - 80 > 0 and 7 < a: < 11

=> .v^-18x + 80<0 and 7<a:<11


=> {.V -10) (a: - 8) < 0 and 7 < a: < 11

ow
=> 8 < a: < 10 and 7<a:<11=> 8<a:<10=> a:s(8, 10).
Hence, the domain of / (a:) is (8,10).
1
EXAMPLE 14 Find the domain of definition of thefunction f(x) given by f{x) = +
TTTI.
logic -^)

e
1

Fl
V^ + 2

re
SOLUTION We have,/(a:) =
logic (1-^)

F
1
Let§(.\) = and/i(A') = ^x + 2. Then, f{x)=g (a') + h (a:)
logic (1 -
ur
Now,
Domain (/) = Domain (g) n Domain (//)
g (a:) =
1
is defined for all x for which logic
f or 1® defined and
ks
logic (1 -
Yo

logic => 1-a:>0 andl - a: ^ \ => a: <1 and a: 0 => x e(-oo , 0) cj(0,1)
oo

Domain (g) = (- co, O) u (0,1).


h{x) = ^ + 2 is defined for all a: satisfying.
B

And,
X4-2>0=> a:>-2=> at6[-2, oo).
re

Domain {h) = [- 2, co).


Hence, Domain (/) = (- co, 0) u (0,1) n 1- 2, x) = [- 2, 0) u (0,1)
u
ad

EXAMPLE 15 Find the range of each of the following functions:


Yo

(i) f{x)=\x-3\
(ii) fix) = 1-1 -A-2
d
Re

a-41
in

(hi) fix) =
a-4
F

SOLUTION (i) We have, fix) = | a - 31


Clearly, / (.a) is defined for all a e R. Therefore, Domain (/) = R.
[ A - 31 > 0 for all A e R
0 <1 A - 3| <x for all A € R
=> 0</(a)<x for all a eR
=> / (a) e [0, X) for all A G R
Hence, Range if) = [0, x).
(ii) We have, /(a) =1-|a-2|.
We observe that / (a) is defined for all a g R. Therefore, Domain (/) = R.
0 < I A - 21 < X for all A € R
- X < -1 A - 21 < 0 for all A G R
=> -x<1-|a-2|<1 forallAGR
15.36 APPLIED MATHEMATICS-XI

< / {.t) < 1 for all .t 6 K


- CO

f{x)s{-ccA]
Hence, Range (/) =(-co, 1]
|a--4|
(iii) We have, / (.t) =
x-A

Clearly,/(;r) is defined for all a: € R except at a: = 4. Therefore, Domain (/) = R -14}


Now,
\x-A
fix) =
x-4

x-A
= 1 , ifx>4
x-A
fix) = ■ ix-A)
= -1 , if a: <4

w
a:-4

Hence, Range (/) = {-1,1}

F lo
EXAMPLE 16 Find the domain and rmige of each of the folhzving functions given hy
if) fix) = (ii)/(x) = l-lx-3

ee
Fr
SOLUTION (i) We have, / (.v) = ■ . ^
fx-[x]
for
ur
Domain off: We know tliat
0 < .V - [x] < 1 for all ,r e R
ks
And, ,T-[x] = 0 for.veZ.
Yo

0 < X - [x] < 1 for all X e R - Z


oo

1
eB

fix) = exists for all x s R - Z.


y[]^
Hence, Domain (/) = R - Z.
r
ou
ad

Range off We have,


0 < X - [x] < 1 for all x e R - Z
Y

=> 0 < fx - [x] < 1 for all X e R - Z


nd
Re

1
1 < < CO for allx sR - Z
fx - [x]
Fi

l</(x)<Qo for allx gR-Z


Range (/) =(l,co).
(ii) We have,/(x) = l-|x-3|
Clearly,/(x) is defined for all X G R. Therefore, Domain (/) = R.
Range off: For any x g R, we have
I x-3| >0 -lx-3| <0 1 -[x-3| <1 => /(x) <1 => / (x) g(-co,1]
Hence, Range (/) = (-oo, 11.
EXERCISE 15.4
1 f
1. Find/ + ^, f-g, cf (c gR, 0), fg and — in each of the following:
' / %
(i) /(x) =x^ +1 andg-(x) =x + 1 (ii) / (x) = ^x-1 and g (x) = fx^\.
FUNCTIONS 15.37

2. Let f{x)=2x + 5 and g{x)=x^ + x. Describe (i)/ + ^ (iv)//g. Find the


domain in each case.

=j
-1 , - 2 < -r < 0
3. If/(x) be defined on [- 2,2] and is given by f{x) x-l , 0cr<2

and g(x) =/( I j: I) +1 f{x) |. Find g(x).


4. Let/, g be two real functions defined by / (;r) = /x + 1 and g (x) = -^9 - x'^. Then, describe
each of the following functions:
(i) / + g (ii) g-/ (iii) / g (iv) f/g
(V)
g
(vi) 2/-^/5 g (vii)/2+7/ (viii) ^
/
5. If / (x) = log^ (1 - x) and g (x) = [x], then determine each of the following functions:
/ g
(i) / + g (ii) /g (iii) (iv)
g /

w
Also, find (/ + g)(-l),(/g)(0). ^/Vii fiVr
gJUJ'UJUJ-
6. If/,g, /i are real functions defined by/ (.v) =^x

F lo + 1, g{x) = —and/i (.t) =2x^ - 3, then find

ee
the values of (2 / + g - /i) (1) and{2f + g-h) (0).

Fr
I 1 - X , -t < 0
7. The function / is defined by/ (:r) = -11 , x = 0. Draw the graph of / (a:),
[a: + 1 , X > 0
for
/
be defined, respectively by/ (.r) = .v + l,g(A:) = 2x-3.Find/+g//-gand
ur
8. Let/, g:R g
oks

9. Let /:[0, oo)->K and g:R^R be defined by fix)=^^x and g(x)=.v. Find
Yo

f + gJ-SJi aiid {■
o
eB

10. Let/(a:) = .t^ andg(.v) = 2.v +1 be two real functions. Find(/ + g) (.v), if -g) {x),{fg) (a:) and
(f (X).
our
ad

ANSWERS
Y
Re

I- (i) / + g: R given by (/+ g) (a:) = a:^ + X + 2


nd

f-g:R^R given by (f-g){x) = x^ - x


Fi

cf :R ^R given by (c/) (x) = c (.r^ + 1)


fg:R^R given by (fg) (x)={x + 1)^ (x^ - x + 1)
1 h'l 1
-: R - {-1}-> R given by - (x) = x^ +1
/ /
f (f] 2
-:R-{-l}^R given by - (x)=.x^ + x + l
g g

(ii) / ± g: [1, co) R defined by (/ + g) (x) = Jx-1 ± ^x + 1


c/ : [1, °o) “> R defined by (c/ ) (x) = c Jx -1
fg:[l, <x>) R defined by (^) (x) = yx^ -1
15.38 APPLIED MATHEMATICS-XI

1 (1 1
: (1, oo) —> R defined by - (x) =
/ /
/. / -r-1
; [1, oo) R defined by
Sj p' + l
2. (0 (/ S) M =x^ + 3x +5; dom (f+g)=R (ii) (/ -g)(x) =5 + x-x^; dom (f-g)=R
2x + 5 7'
(iii) (fgXx) = 2.1-3 ^ 7^2 ^ 5^. ^^ (iv) (.r)=^ ,dom ^ =K-|0,1)
RJ X + X R
-2<x<0
3- 0, 0 <>: <1

ow
[2(a--D,
CO / + : [“ 1 / 3] -)■ R defined hy {f + g) (x) = p +
1 + -^9 -
(iO R 1< 3] -> R defined by -/) (x) = -^9 - x^ - p + 1

e
(iii) fg: 1, 3] -> R defined by (fg) (x) = ^9 + 9x-x^ -x^

re
f

Frl
( x + 1

F
(iv) [-1, 3] ^ R defined by — (x) =
<? R 9-x^
9-X-2
/ \
ou
or
(v) '^ :(-l, 3]-> R defined by \f)
— (x) =
R
x + 1
kfs
(vi) 2/ - V5 3] -> R defined by (2/ - ^/5 r) (x) = 2p + l --^45 -5x^
oo

(vii) +7 /: [-1, x) ^ R defined by (/^ + 7/) (x) ^ x + 1 + 7 p + 1


Y

5 Ts'l 5
B

(viii) —: 3, 3)-> R defined by - (x) = ■ ,


R R
p-x^
re

3- (0 /+ r:(-oo, 1) ^ R defined by (/ + r) (x) = log^ (1 -x) + [x]


oYu

(iO fg: (- CO, 1) R defined by (/ g) (x) = [x] log^ (1 - x)


ad

/. 7^ loge (1--V)
(iii) —: (- X, 0) ^ R defined by — (x) =
d

R kR [x]
f
in

(iv) /^:(-X, 0) u (0,1)-> R defined by — (x) = loge I-v](1 - -v)


Re

7
F

(/ + R)(-1) = Iog^2-l and, (/r) (0) = 0, -V-' does not exist —V-' = 0
,R,v2J 7
6. 2 (V2 +1), 0, does not exist.
8. / + R: R -> R defined by (/ + r) (x) = 3x - 2; / - r : R -> R defined by (/ - r) (x) = - x + 4
f :R
.
I —I -> R defined by —
x + 1
- (.t) =
1-2J [r 2x-3

/ + R :[0/ ^ defined by (/ + r){x) = Vx + x; /-r : [0,x) -> defined by {f -g){x)=4x -X

fg: [0, x) R defined by {fg) (x) = x^^^; ^ : (0, x) -> R defined by ^ (x) = ^
R

lO- if + g) (^) ={x + 1)^. (/ -r) (a:) = x^ -2x -1, {fg) (x) = 2x3 ^ p{/I^ x2
R 2x + l
FUNCTIONS 15.39

HINTS TO SELECTED PROBLEMS

7. We have.

l-x, a: < 0
fix) = 1, x =0
x + 1. X > 0

Let/(x) =y.Then,
y = 1 - X or X + _v = 1 for x < 0

w
y = 1 for X = 1
and y = x +1 or - x + y = 1 for x > 0.
To draw the graph of /(x), draw the line x + y = 1 and take its that part for which x < 0 i.e.
take that portion of the line which lies on the left side of y-axis.

e
For X > 0, draw the line -x + y =1 and take its that portion which lies on the right side

ro
re
of y-axis. For x = 0, mark the point (0,1).
So, the graph of / (.x) is as shown in Fig. 15.26.

F
Fl
y

u N

sr
ko
-f-
o
(0,1) of
0 X
X' (-L0) (LO)
o
Y
erB
uY

Fig. 15.26

8. /: R R and ^: R K are given by /(x) = x + 1 and g (.x) = 2x - 3.


ad

Clearly, D (/) = R and D (g) = R. Therefore,


do

(i) D{f + g)=D{f)nD{g)=R and, (/+ r) (x) =/(x) + g (x) =x + 1 -t 2x - 3 = 3x - 2


in

(u) D(/-g)=D(/)nD(R)=R and,(/-r) (x) =/(x)-r(x) -x4-1-2x+3 = -x + 4


(iii) D{fg)=D{f)nD{g)=R and, (/r) (x) =/(x) r (.x) ={x +1) (2x - 3) = 2x^-x-3
Re
F

/ f 3l f / (x) X 4- 1
(iv) D
u D{f)nD(g)-{x:g{x)=0} = R-|/ and, =—=—
9. It is given that /: [0, °o) —> R and r : R —4 R are such that/ (x) = -fx and g (x) = X.

D (/ + R) = [0, co) o R = [0, co)


So,/4-r:[0, co)R isgivenby(/4-R)(x) =/(x)4-r(x) = Vx 4-x
D (/ - r) = D (/) n D (r) = [0, oo) n R = [0, «j)
So,/-r:[0, oo)->R isgivenby{/-r) (x) =/(x)-r(x) = Vx-x
D(/r) = D{f)nD{g) = [0,co)nR = [0, x)
So,/r:I0, x)->Risgivenby (/r) (x) =/(x) R (x) = Vx x=x^^^
/
D D if) nD ig) - {x: g (x) = 0} = (0, x)
15.40 APPLIED MATHEMATICS-XI

f. fix) xr 1
So, — : (0, c») K is given by ix) =
s Six) X

10. We have,/(.y) = Y-and^(Y) = 2y + 1. Clearly, O (/)= K and D (g) = R.


D{f±g) = D(f)nDig) = Rr^R = R
D(fg) = D if) Dig) = RnR = R
f
= D if) nD ig) - {x: g ix) =0} = R oR - j-i|
1
D = R -
g 2

Thus,

f + g:R~^ R is given by (/ + g) (y) = fix)+g (y) = x^ + 2y + 1


/-^:R^-Risgivenby(/-^)(Y) =fix)-gix) =x^-2x-l
ifg):R^Ris given by ifg) (y) = / (y) ^ (y) = y (2y + 1)
:R-
ri] fix) X
s — > ^ R is given by — ix) =

w
S) [g 2y +1

Mark the correct alternative in each of the following:


F lo MULTIPLE CHOICE QUESTIONS (MCQs)

e
1. Let .A ={1,2, 3 },B = (2, 3, 4), then which of the following is a function from A to 6 ?
(a) {(1,2), (1,3), (2, 3), (3, 3)1 (b) {(1,3), (2,4)1
Fre
for
(c) {(1,3), (2, 2), (3, 3)1 (d) {(1,2), (2,3), (3, 2), (3,4)}.
2. If /: Q ^ Q is defined as / (y) = x^, then / (9) is equal to
r
(a) 3 (b) -3 (c) {-3,3} id) (j)
You
oks

3. Which one of the following is not a function?


eBo

(a) {(Y,y):Y,y€R,Y^=i/} (b) {(y, y):Y,y6R,y^=Y}


(c) {(y, y):Y,y sR, Y=y^l (d) ((y, y):Y,y eR,y=Y^l
ad
our

y2
4. If/ (y) = cos (log y), then/(Y^) /(y^) - ^ / + fix^ y^) I has the value
(a) -2 (b) -1 (c) 1/2 (d) none of these
Re
dY

1
5. The domain of the function/defined by fix) = IS
Fin

(a) Rq (b) R^ (C) R' (d) none of these


6. Let/(y) =1 Y-11. Then,
i^) fix^)=lfix)f (b) /(Y + y) =/(Y)/(y)
(c)/(U|)=|/(a:)| (d) none of these
1 jri
7. If/(Y)=Y^ then fix) + f is equal to
y"* \xj

(a) 2y3 (b)4


y"*
(c) 0 id) 1

8. Which of the following are fimctions?


(a) |(Y, y): y2 = y, y, y g R) (b) {(Y,y):y =|y|, Y,yeR}
(c) {(y, y):Y^ + y^ = 1,y, yeR} (d) ((y, y): y^ - y^ =1, y, y g R}
15.41
FUNCTIONS

il+x] 3x +
9. If/(x) = log and g (x) = then / (g(j:)) is equal to
1 -X 1 + 3x

(a) /(3.V) (b) {/(x)}3 (c) 3/(x) (d) -/(X)


10. If A = {1, 2, 31, B = 1-v, xj\, then the number of fimctions that can be defined from A into B is
(a) 12 (b) 8 (c) 6 (d) 3
1 + x 2x
, then / j is equal to
11. If/(x)=log|^- -X 1 + X

(a) t/(xf (b) {/(x)l 3 (c) 2/ (.x) (d) 3/(.x)

ow
12. The domain and range of the function/given by /(x) =2 -|x-5l, is
(a) Domain , Range = (-w, 1] (b) Domain = R, Range = (- oo, 2]
(c) Domain =R, Range =(-00, 2) (d) Domain = R , Range = (- 2]
■v^+2x + l

e
13. The domain of the function/given by /(x) =

Fl
re
x^-x-6

F
(a) R-l-2, 3) (b) R-{-3,21 (c) R-[-2, 3] (d) R -(-2, 3)
14. The domain and range of real function/defined by /(x) = Vx-1 is given by
ur
(a) Domain = (l,co), Range = (0, oo)
or
(b) Domain = [l,oo), Range = (0, oo)
(d) Domain = [l,oo). Range = [0, oo)
sf
(c) Domain =[l,oo). Range = [0, x)
15. Let / : R -> Rbe defined by / (x) = 2 x + | x|.Then/ (2 .v) +/(-x) -/(x) =
k
Yo

(b) 2ixl (c) - 2x (d) -2|xl


oo

(a) 2x
x^ - X .
eB

16. The range of the function / (.v) = IS

x^ + 2x
(a) R (b) R-{1) (c) R-{-1/2,1} (d) none of these
ur

x +1 .
a real function, then / (/ (/ (2)) is
ad

17. If X 1 and / (x) = is


Yo

x-1

(a) 1 (b) 2 (c) 3 (d) 4


d

4-x
18. The domain and range of the real function of defined by /(x) - x-4 is given by
Re
in

(a) Domain = R, Range = 1-1,1} (b) Domain = R -(!}, Range = R


F

(c) Domain = R -14), Range = 1-1} (d) Domain = R -1-4), Range = |-1,11
19. Let /(x) = X, g(x) = -X and h (x) = / (x) g (x). Then, h (x) = 1 for
(a) X e R (b) xeQ (c) X eR - Q (d) X € R, X 0
20. If/(.x) =ax + b, where a and b are integers,/(-I) =-5 and /(.x) = 3, then a and b are equal
(a) fl=-3,t=-l {b)a = 2,h=-3 (c) a = 0,b=2 (d) = 2, b = 3

21 ThefuncHon/:R ^ R is defined by / (x) = cos^ x + sinx. Then,/(R) =


(a) (3/4,1) (b) (3/4,1] (c) [3/4,1] (d) (3/4,1)
— for X e A.
22. Let A = {xeR : x 0, - 4 <x < 4j and f:A R be defined by f{x) = '-^
Then A is

(a) 11,-1} (b) lx:0<x<4} (c) 11} (d) lx:-4<x<0}


15.42 APPLIED MATHEMATICS-XI

23. If /: R -> R and ^: R R are defined by f {x) = 2x + 3 and g (x) =x~ + 7, then the values
of X such that g (/ {x)) = 8 are

(a) 1, 2 (b) -1,2 (c) -h-2 (d) 1,-2

24. If/: [- 2, 2] R is defined byf (x) = |


-l,for-2<A:<0
, then
a: -1 , for 0 < a: < 2

{x g[- 2, 2]:x < Oand/( I a:|) =a'} =


(a) {-1} (b) 10) (c) {-1/2} (d) (j)
10 +a: 200 a:
25. If = e (-10,10) and f {x)=k f , then k =
10-x' m + x^
(a) 0.5 (b) 0.6 (c) 0.7 (d) 0.8
26. Domain of/(:r) = -X ^, <7 > 0 is

w
(a) (-a, a) (b) [-a, a] (c) [0, a\ (d) {-a, 0]
1
27. The domain of the function /(a:) = V4-a: + , IS equal to

F lo
V-v^ -1
(a) {-CO, -1) u(l, 4) (b) (-CO,-l]w{l,4]

ee
(c) (-CO,-l)u[l,4] (d) (-CO,-l)vj[l,4)

Fr
2
28. If [a-] -5 [a:] + 6 = 0, where [●] denotes the greatest integer function, then
(a) a: e [3, 4] (b) xe{2, 3] (c) a: € [2, 3] for (d) a: 6 [2, 4)
ur
29. The range of the function / (a:) = | a: -1 | is
(a) (-co,0) (b) [0,oo) (c) (0, co) (d) R
s
a: + 2
30. The range of the function/ (a:) =
ook

, a: - 2 is
Yo

|a + 2|
(a) [-1,1} (b) {-1,0,1}
eB

(c) {1} (d) (0,co)


31.
The domain of the function / (a:) = ■yj2-2x -x^ is
r

(a) [-V3,^/3] (b) [-1-/3,-! +/3]


ou
ad

(c) [-2,2] id) [-2-V3, -2 + ^^]


Y

a: + 3
32. The domain of definition of / (a:) = IS

(2-a:) (a-5)
Re
nd

(a) (-<»,-3] u (2,5) (b) (-^,-3)u(2,5)


Fi

(c) (-«,,-3]u[2,5] (d) none of these


ix + l)ix-3) .
33. The domain of the function / (a:) = IS
a:-2

(a) [-1,2)u[3,oo) (b) (-l,2)u[3,00)


(c) [-l,2]u[3, co) (d) none of these
34. The domain of definition of the function/ (a:) = yJx-1 + /3 - .v is
(a) [1, co) (b) (- GO, 3) (c) (1, 3) (d) [1, 3]
a:-2 1 -a: .
35. The domain of definition of the function / (a:) = + is
a: + 2 1 +a:

(a) (-CO,-2]u[2, x) (b) [-1,1]


(c) 4) (d) none of these
FUNCTIONS 15.43

36. The domain of definition of the function f{x) = log | a: | is


(a) R (b) (-«=,0) (c) (0, co) (d) R-{0|

37. The domain of definition of / (x) = ^4.y - i


(a) R - [0, 4] (b) R-(0,4) (c) (0, 4) (d) [0, 4]
38. The domain of definition of / (x) = ^x - 3 - 2^x -T -3+2 -4 is
(a) [4, 00) (b) (-CO, 4] (c) (4, co) (d) (- 00, 4)

39. The domain of the function/(x) = -X ^ - 6 is

(a) (-3,-2)u(2, 3) (b) I- 3,-2)u[2, 3)


(c) [-3,-2]u[2, 3] (d) none of these

w
40. The range of the function / (^) = is
■^1

Flo
(a) K-{0) (b) K-1-1,1) (c) 1-1,1) (d) none of these

e
ANSWERS

re
1. (c) 2. (c) 3. (b) 4. (d) 5. (d) 6. (d) 7. (c) 8. (b)

F
9. (C) in. (b) 11. (c) 12. (b) 13. (a) 14. (c) 15. (b) 16. (c)
ur
■IS. (c) 19. (d) 20. (b) 21. (c) 22. (a) 23. (c) 24. (c)

r
17. (c)
25. (a) 26. (b) 27. (a) 28. (d) 29. (b)
fo
30. (a) 31. (b) 32. (a)
33. (a) 34. (d) 35. (c) 36. (d) 37. (d) 38. (a) 39. (c) 40. (c)
ks
Yo

FILL IN THE BLANKS TYPE QUESTIONS (FBQs)


oo

1. Let A and B be any two sets such that n(A)=p and n{B)=cj, then the total number of
B

functions from A to B is equal to


re

2. If/(a)=-^=-,then/(i/)=
u

^-1 y
ad
Yo

ax + b
3- ify=/W = then/(y) =
cx-d'
d

1
Re

4. The domain of the function f{x) = IS


in

^/N -X
F

5. The range of the function f{x) = [a:] - x is


x + 2 .
6. The range of the function/(:c) = x +2
IS

7. The range of the function f{x) = log„ .r, <7 > 0 is


8. Let / and g be two functions given by / = {(2, 4), (5, 6), (8,-1), (10, - 3)} and
g = {(2,5), (7,1) (8,4), (10,13), (11, -5)1. Then, domain of / + g is
9. Let / and g be two real functions given by / =!(10,1), (2, 0), (3, -4), (4, 2), (5,1)) and
g ={(1, 0), (2, 2), (3, -1), (4, 4), (5, 3)1. Then the domain/g is given by
10. The domain for which the functions/(a:) = 3x^ -landg(A:) = 3 + a: are equal is
a:^+1 .
11 The domain of the function f{x) - IS
x'^-3x+2
15.44 APPLIED MATHEMATICS-XI

x-1 /I ^
12. If/(.r) = then / - + f{x) is equal to
x + l'
x-l n
13. If/(A-) = then/(.f)/ - is equal to
x + l' X

14. If f{x) = [.\-] -5 M + 6, then the set of values of .y satisfying f{x) = 0 is


1
15. The domain of the function f{x) = V9 -.y + is equal to
V? 16
2-.Y

w
16. The domain and range of the function/(y) = are .and. respectively.
x-2
1
17. The domain of the function/(y) = IS

V[.vp-3[.v]+2

o
e
X-i\ IS
.
18. The range of the function/(Y) =

re
y-4

Frl
19. The domain of the function f{x) = y + [.y] is

F
20. The range of the function f{x) = Vl -x^ is
ou
sor
21. Tlie domain of the function/(y) = ^ ^ j-i
IS

«=1

I-^1-2.IS
kf
22. The domain of the function/(Y) =
y|-3
oo

23. If /(2y+ 3) ^4y^ +12.V + 15, then the value of /(3y + 2) is


Y
B

24. The number of elements of an identity function defined on a set containing four elements
IS
re
oY

ANSWERS
u

1. /
ad

2.1-y 3. Y 4. (-CO, 0) 5. (-1, 0] 6. 1-1,1)


d

7. R 8. {2, 8,10) 9. 12,3,4,5] 10. -1,^3 11. K-11, 2} 12. 0


in
Re

13.-1 14. [2, 4) 15. (-oo,-4)u(4, 9] 16. 1^-12), 1-11


F

17. (-00, l)u[3, oo) 18. 1-1,1} 19. R 20. [0,1]

21. R- i,l,^,2,^,3,^,4,^,5
2 2 2 2 2
22. R-[-3,3] 23. 9y^-12.y+24 24.4

VERY SHORT ANSWER QUESTIONS (VSAQs)


Answer each of the following questions in one word or one sentence or as per exact requirement of the
question:
1. Write the range of the real function/(y) = |y(.
f 1 ") 1
2. If / is a real fimction satisfying / y + — + for all Y e K -10), then write the
X) x^
expression for / (y).
3. Find the set of values of y for which the functions/(Y) = 3y^ -1 and g(.Y) = 3 +Yare equal.
FUNCTIONS 15.45

4. Let / and g be two real functions given by


/ = {(0,1), (2, 0), (3-4), {4, 2), (5,1)1 and g = [(1, 0), (2, 2), (3, -1), (4, 4), (5, 3)}.
Find the domain of fg.
5. Let / and g be two functions given by
/ = {(2, 4), (5, 6), (8, -1), (10, - 3)1 and g = |(2,5), (7,1), (8, 4), (10,13), (11, -5)).
Find the domain of / + g.
6. Write the range of the function/(a:) = a: e R.
ctx
7. Let/ (a:) =
A- + 1
, a: -1. Then write the value of a satisfying/ (/ (a:)) = x for all x*-l.
1 1
8. If/(a:) = 1—, then write the value of/ /
X yxjj
x-2

w
9, Write the domain and range of the function f{x) =
2-a-

11. lff,g,h are

F lo
10. If / (a:) =4a: -x^, x .eR, then write the value of/(fl +1) -/ («-!)●
real functions given by / (.v) = x^, g (a:) = tan x and, h (.v) = log^, x, then

e
Fre
( In'
write the value of (hogof) J- .
V ' ^, for
12. Write the domain and range of function/(a:) given by/(a‘) =-^
r
You

13. Write the domain and range of / (a:) = yjx - [.v].


oks

11. Write the domain and range of function/(x) given by / (a:) = ^[x]
eBo

15. Let/I and 6 be two sets such that n( A) =pandn{B) =£/, write the number of functions from A
to B.
our
ad

ANSWERS

1. [0, co) 2. / (at) = x^ - 2, where I a:! > 2 3. {-1,4/3}


dY
Re

4. {2,3,4,51 5. {2, 8,10} 6. [l,e) 7. a=-l


Fin

x
8. 9. D(/) = R-(2),R(/)={-ll 10, 4(2-rt)
x-1

11. 0 il D (/)=({) = Rif) 13. D(/)=R,R(/)=[0,1)


14. D(/)=2,R(/)=(01 15. qP
CHAPTER 16
LIMITS

16.1 INFORMAL APPROACH TO LIMIT

ow
X 2-4
Consider the function J{x) = ~
x-2

0
Clearly, this function is defined for all x except at a: = 2 as it assumes the form — (known as an

e
indeterminate form) at a: = 2. However, if a- 2, then

re
fix) =
(A-2) (A+ 2) = a + 2

rFl
F
x-2

The following table exhibits the values of/(a) at points which are close to 2 on its two sides viz.

r
left and right on the real line.
ou
fo
ks
*■

X 1.4 1.5 1.6 1.7 1.8 1.9 1.99 2 2.01 2.1 2.2 2.3 2.4 2.5 2.6
oo

3.4 3.5 3.6 3.7 3.8 3.9 3.99 0 4.01 4.1 4.2 4.3 4.4 4.5 4.6
fix)
Y
B

0
y
re

x2-4
fix) =
ou
Y

x-2
ad

(0, 4)
d

(0, 2)
in
Re
F

X' (-2,0) O (2,0) X

Y'

2-4
Fig. 16.1 Graph of/(ar) = -.x-2

The graph of this function is shown in Fig. 16.1.


It is evident from the above table and the graph of /(a) that as a increases and comes closer to 2
from left hand side of 2, the values of /(a) increase and come closer to 4. This is interpreted as:
When X approaches to 2 from its left hand side, the function f(x) taids to the limit 4.
It we use the notation 'a 2~' to denote 'a tends to 2 from left hand side', the above statement
can be restated as:
16.2 APPLIED MATHEMATICS-XI

as -> 2 , f{x) 4
or. lim f{x) = 4
x->2~

or. Left hand limit off{x) at x = 2is A.


Thus, lim f{x) = 4 means that as x tends to 2 from left hand side, the values of f{x) are
x->2~

tending to 4.

w
From the above table as well as the graph of f{x), shown in Fig. 16.1, we observe that as x
decreases and comes closer to 2 from right hand side, the values of/(:c) decrease and come closer
to 4. This is interpreted as:
When X approaches to 2from its right hand side, the function frx) tends to the limit 4.

e
Using the notation 'x->2'^' to denote 'x tends to 2 from right hand side', the above statement

ro
re
can be re-stated as:
as
X2^, f(x) 4

F
or. lim f{x) = 4

or. Right hand limit offrx) at x = 2 is 4.

Fl
u
sr
Thus, lim f(x) = 4 means that as x tends to 2 from right hand side, the values offrx) are tending
to 4. x-^2'*'

ko
o
of X 2-4
It follows from the above discussion that for the function f{x) given hyf{x) = —x-2
o
Y
(i) lim f(x) = 4 (ii) lim f{x) = 4
erB

x->2 x-^2'*'

(iii) x->2~
lim_ f{x)= x-^2'^
lim f (x) (iv) ./(2) does not exist i.e. f{x) is not defined at x = 2.
uY

|x-3|
Now, consider the function f{x) =
;c-3
ad
do

0
This function is defined for all x except x = 3, as it assumes the form—(an indeterminate form) at
in

x = 3. The graph of f(x) is shown in Fig. 16.2.


Re

Y
i
F

\x-3\
x-3
(3,1)6
(3,0)
X' O X

(0,-1)
4 (3.-1)

Y' t
|x-3|
Fig. 16.2 Graph off(x) = x-3

The following table shows the values of frx) at points which are close to 3 and are on its two
sides.
LIMITS 16.3

2.3 2.4 2.5 2.6 2.7 2.8 2.9 2.99 3 3.01 3.1 3.2 3.3 3.4 3.5 3.6
0

fix) -1 -1 -1 -1 -1 -1 -1 -1 0 1 1 1 1 1 1 1

It is evident from the table and the graph oif(x) that as jc 3 from its left hand side the values
of f{x) are everywhere -1.
i.e. lim f{x) = -1 or. Left hand limit (LHL) of f{x) at x = 3 is -1.

We also observe that at every point on the right hand side of 3, the function assumes value 1.
lim fix) = 1

ow
^ >3*^
Let us now consider the function/(jc) = —, x 4. Here also the function is imdefined at x = 4

as/(4) assumes the form In this case it is evident from the graph shown in Fig. 16.3 that as x

e
approaches to 4 from the left hand side, fix) decreases to - 00.

re
rFl Y

F
r
ou
fo
ks
oo

X' O (4,0) X
Y
eB
ur
ad

r
Yo

Fig. 16.3 Graph oifix) =


d
Re
in

i.e. lim fix) = — 00

x-^4~
F

Also, we observe that fix) increases to + oo as approaches to 4 from the right


i.e. Ihii fix) = + 00

So, we say that lim fix) and lim fix) both do not exist.
x-^4~ x-^A"^
It follows from the above discussion that as we can approach to a given number 'a' (say) on the
real line either from its left hand side by increasing numbers which are less than "a' or from right
hand side by decreasing numbers which are greater than 'a'. So, there are two types of limits viz.
(i) left hand limit and, (ii) right hand limit. We also observe that for some functions at a given
point 'a' (say) left hand and right hand limits are equal whereas for some fimctions these two
limits are not equal and even sometimes either left hand limit or right hand limit or both do not
exist.
If lim fix)= lim /(jc)i.e. (LHLatx = a) = (RHLatx = fl),thenwesay that lim /(x)exists.
x^a

Otherwise, lim /(x) does not exist.


x-^a
16.4 APPLIED MATHEMATICS-XI

16.2 EVALUATION OF LEFT HAND AND RIGHT HAND LIMITS

In the previous sections, we have learnt that a real number is the left hand limit of function
f {x) at X = a if the values of / (a:) can be made as close as desired to the number at points
closed to fl and on the left of In such a case, we write lim f{x) = /j. Also, a real number/2 is
x-*a

the irght hand limit of / (x) at a: = a i.e. lim +


f (x) = /2, if the values of/(a:) can be made as
X -> ff

close as desired to the number I2 at points close to 'a' on the irght of 'a'.
In this section, we shall discuss methods of evaluation of left hand and right hand limits of a
function at a given point.
As discussed earlier that statement a —> a" means that a: is tending to a from the left hand side i.e.
a: is a number less than a but very very close to a. Therefore, a: is equivalent to x = a-h
where h>0 such that h -*-0.

w
Similarly, x a'*' is equivalent tox=a + h where ft -> 0. Thus, we have the following algorithms

F lo
for finding left hand and right hand limits at a: = fl.
ALGORITHM

ee
': .rp I
Write lim f{x)

Fr
x-ya

Mi r
Put x = a-h and replace x a by ft 0 fo obtain lim = f{x) = lim f{a ~ ft).
for
X -» (7
r
MHP 111
Simplijy hlim
^0
f{a -h)by using the formula for the given function,
You
s
ook

srEP r
The value obtain in step III is the LHL offix) at x = a.
eB

li.LUSTRAlTON 1 Evaluate the left hand limit of the function


|a:-4|
, x^ 4
fix)=\ x-4 atx = 4.
our
ad

0 , a: = 4

SOLUTION We have,
(LHL of/(a) at a = 4)
dY
Re

= lim /(a) (Step I)


Fin

x^4“

= lim /(4-ft) (Step II)


h ->0

lim
|4-ft-4| lim
l-ftl lim
ft
lim -1 = -1. (Step in)
/i->0 4 -ft -4 h^O -ft h -ft h^O

To evaluate RHL of/(a) at a = a i.e. lim


+
/(a) we use the following algorithm.
X a
ALGORITHM

M I'P I
Write lim + /(a)
X a

STEP 11 Put X = a + h and replace X -^a"^ by ft -^0 to obtain lim /(a) = lim f(a+h).
X —> (7 /i -> 0

sri:i' 111
Simplify /i lim
-> 0
f{a + ft) by using the formula for the given function.
The value obtained in step III is the RHL affix) at x = a.
LIMITS 16.5

ll.LUSTRATlON 2 Evaluate the right hand limit of the function


|jr-4l
, A
/(:»^)=i x-A atx = 4.
0 , x=4

SOLUTION We have,
(RHL of/(:c) at X = 4)
= lim + f{x) (Step I)
X 4

lim /(4 + h) (Step II)


h -*0

ow
\4 + h-A\ h\ h
lim lim lim lim 1=1 (Step III)
h ^0 4 + /I-4 h ^0 h /i -> 0 /i 0

e
ILLUSTRATIVE EXAMPLES

re
EXAMPLE 1 Evaluate the left hand and right hand limits of the function defined by
\+x^, if 0<x<l at X = 1.

Frl
F
fix) = 2-x, if X>1
ou
or
Also, show that lim f{x) does not exist.
X->1 kfs
SOLUTION (LHL off{x) at X = 1)
oo

y
Y
B
re
oYu
ad

X'
o X
d
in
Re
F

r
1
Fig.16.4 Graph of/(x) = -
X

lim fix) = h lim


-vO
fil-h)= /t-^0
lim l+fl-Zi)^ = h lim
^0
2-2/i + h^ =2.
X -»!'

and, (RHL of fix) at X = 1)


= lim fix) = lim f {1 + h) = li lim
->0
2-(1 + 10= h -^0
l-h=l.
X -* 1
+ h -*0

Clearly, lim /(x) ^ lim f{x).


X -> 1" X ->1-^

Hence, lim /(x) does not exist.


X ->1
16.6 APPLIED MATHEMATICS-XI

x-|x|
, # 0
EXAMPLE 2 If fix) = - show that lim f{x) does not exist.
"2, x =0 X -> 0

SOLUTION We have,
(LHL of fix) at X = 0)
= lim fix)

rw
X -» 0"

e
e
lo
r
F
u
X'

oF o X

rs
ko
y

Fig. 16.5 Graph of/(x) = of


o
X
Y
-h-\-h\ -h-h -2h
B

lim /(0-/i) = lim lim lim lim 2 = 2.


rY

h ^0 h ^0 i-h) /? -> 0 -h h -i'O -h h 0

(RHL of fix) at a: = 0)
ue

= lim + fix)
X 0

h-\h\ h-h
od

0
lim /(O + h) = lim lim lim lim 0 = 0.
ad

X 0 h 0 h /i 0 h h -^0 h /i ^ 0
in

Clearly, lim fix) *■ lim fix).


X -)● 0 X ^ 0^

Hence, lim /(x) does not exist.


Re

X -> 0
F

5a: - 4, 0 < X <1


EXAMPLE 3 If fix) = 4a:^ - 3a: 1 < x < 2' exists.
SOLUTION We have.
(LHL of/(a:) atx=l)
lim fix) - lim /(l-/i)= lim 5(l-/z)-4 = lim 1-5/j = 1.
x->l h^Q h^O h^O

(RHL of/(A:) atA: = l)


= lim fix)
x-> l'*’

= h^O
lim fil+h)= h^O
lim 4 (1 + - 3 (1 +/i) = 4(1)^ - 3 (1) = 1

Clearly, lim fix) = lim +


f{x).
x->l x-*l

Hence, lim fix) exists and is equal to 1.


x^l
LIMITS 16.7

nXAMPLL -I Discitss the existence of each of thej., limits:


1
(i) lim
x-»0 X
(ii) X lim 0 p
1 -t I
1
SOLUTION (i) The graph of/(x) = - is as shown in Fig. 16.4. We observe that as x approaches to
0 from the LHS i.e. a: is negative and very close to zero, then the values oil/x are negative and
very large in magnitude.
1
lim > -00.
X

ow
Similarly, when x approaches to 0 from the right i.e. x is positive and very close to 0, then the
values of —X are very large and positive.
1
lim ><x>.

e
re
1
Thus we have. lim — ^ lim i. Hence, X lim
-> 0 .T
does not exist.
x->0“ ^ X ^ 0+ .r

F
(ii) The graph of/(x) =
1

Frl
is shown in Fig. 16.5. We observe that as x approaches to 0 from
1^
ou
osr
LHS i.e. X is negative and close to 0, then | x | is dose to zero and is positive. Consequently, - - IS
i
large and positive.
kf
oo
1
lim -» 00

x-» 0 ^1
Y

and close to 0, then| x [ is close to zero and is


B

Also, if X approaches to 0 from RHS i.e. x is positive


positive.
re
Y

Consequently, — is large and positive.


u
ad

1
do

lim -> 00
+ X
x^ 0
in

Thus, we have
Re

1 1
lim lim
F

X + X
x-> 0 X ^ 0

Hence, lim — exists and it tends to infinity.


X ^ 0 1X
cos X, if X > 0
EXAMPLES Letf{x) = X + cf , if X < O’
Find the value of constant k, given that X lim
-> 0
/(x) exists.

SOLUTION It is given that


lim /(x) exists
x^O

lim fix) - lim /(x)


x->0 x-> O'^

lim X + k= lim cosx [Using definition of/(x)]


x->0 x->0

0 + /c = cos0=>/c=l
16.8 APPLIED MATHEMATICS-XI

Let f (x) be afunction defined b\/ /(;c) = |


EXAMPLE h 4iX-5 ,iix<2
x-X , ii X >2

FindX, if lim f{x) exists.


x->2

SOLUTION We have.

/w = {
4x-5 , if X<2
x-X , if X>2

lim /(x) = lim f{2-h) = lim 4(2-/i)-5 = lim 3-4h = 3


h-^0 h-^0
and.
lim + /(x) = lim f{2 + h) = lim 2 + h-X =2-X
x^2 h-*0 h-^0

w
If lim / (x) exists, then
lim /(x) = lim
+
f{x)^3=2-X=>X = -l.
X ^ 2 X ^ 2

Flo
mx^ + w, X < 0

e
example: Uf(x) =

re
«x + m,0<x<l
rx

F
nx + m, X > 1

For what values of integers m and n does the limits lim / (x) and lim / (x) exist.
ur
SOLUTION It is given that x->0
r x-»l
fo
lim / (x) and lim / (x) both exist
ks
x->0 x-^1
Yo

o lim / (x) = lim / (x) and, lim / (x) = lim / (x)


oo

x->0 x^O'*’ x^l" x-^1'*'


eB

<t^>
lim f{0-h) = lim /(0 + /i)and, lim f{l-h) = lim f (1 + h)
h -* 0 h 0 /j -> 0 h -> 0

lim m{-h) +n= lim n{h) + mand, lim n (1 - /i) + m = lim n{\ +h)^ + m
ur

<=>
/i -> 0 h ->■ 0 /i -> 0 /t 0
ad
Yo

<=> n - m, and n + m = « + m
o m = n

Hence, X lim 0 /(x)and lim / (x) both sides for« = m.


d

X ^ 1
Re
in

|x|+l , x<0
F

EXAMPLE 8 If f{x) = \ Q , X = 0. For what value (s) of'a'does lim f{x) exist?
x| -1 , X >0 X -* a

SOLUTION We have.
x| +1 , X <0
fix) = 0 , x =0
x|-1 , X > 0
-x + 1 , X <0
X, X > 0
fix) = 0 , X = 0 X
x-1 , x>0 -X,X <0

Clearly, lim / (x) exists for all a 0. So, let us see whether lim /(x) exist or not.
X-> fl x^ 0

We observe that
lim fix) = lim /(0-/i) = lim -{-h) +1 = 1
x-> 0 x-^ 0 h-> 0
LIMITS 16.9

and. lim f (x) = lim f{0 + h) = /i lim


^ 0
h-1 =-1
1^0
+ h-*0

lim fix) # lim +


f{x)
X -» 0 X ^ 0

So, lim / (x) does not exist. Hence, X lim / (x) exists for all a^Q.
X ^ 0 -* a

a + bx , X <1
UXAMPLE9 Suppose f (x) = -| 4 ,x = I

w
b -ax, X >1

and, if lim f {x) = f (1). What are possible values of a and b?


x^l

e
SOLUTION We have,
lim fix) =/(l)

ro
re
x^l

C5> lim / (x) = lim / (x) = / (1)

F
x->r x-^l'^

lim fix)=f (1) and, lim

Fl
/ (x) = / (1)

u
<=>
x-^r x-^i"^
lim / (1 - h) = 4 and, /i-»0
lim / (1 + /i) =4

sr
<=>
h-^0

ko
o
lim <a + bil - h) = 4 and, lim b - a il + h)\ =4
<=>
h^O /i->0
of
O a+ b = A and, b - a = 4:
o
Y
o a = 0, b = 4
erB

EXAMPLE 10 Find the left hand and right hand limits of the greatest integer function fix) = [x] =
greatest integer less than or equal to x,atx = k, where k is an integer. Also, show that lim fix) does not
uY

exist.
SOLUTION We have,
(LHLatx = cf) = lim Hx) = lim fik-h) = lim [k-h]
h-*0 /j-»0
ad
do

X -*k

h^O
lim k - I = cf -1 [v k-l<k-h<k .. [k-h] = k-1]
in

(RHLatx = /c)= lim fix) = Ii lim 0 fik + h) = h lim 0 [k + h]


Re

+
X -* k
F

/t
lim
0
k = k [/ k <k + h <k +1 .'. [A: + h] = k]

Clearly, lim fix) ^ lim


+
fix).
X -* k X k

Hence, lim /(x) does not exist.


X —> k

EXAMPLE 11 Prove that lim [x] = [fl] for all aeR. where [.] denotes the greatest integer
X -> fl'*’
function.
SOLUTION Since aeR. Therefore, there exists an integer k such that k<a<k + 1.
Now, lim [x] = lim [a + h] = k [●.● k<a<k + l k<a + h<k + l ^[a + h] = k]
h^O

= W [●.● k<a<k + l => [fl] = k]


16.10 APPLIED MATHEMATICS-XI

^1/at -1 ^
tXAMI’LF. 12 Show that lim — does not exist.
e
l/.t + 1

SOLUTION Let/(>;) = . Then,

(LHL of fix) atx = 0)


e-^/>> -1
lim fix) = Urn fiO-h) = lim
j: ^ 0 /i-»0 0 e~^' ’ +1
1
-1
0-1 1
= lim 1 ●●● h 0 ^ > 00 ^ e Wh ->oo => ^0

w
1
h^O
+ 1
0+1 h el/h
1 \/h
V e

and. (RHL of/(.v) at a: = 0)

Flo
,1//<_1
lim + fix) = lim / (0 + /i) = lim

e
h->0

re
a: -> 0 0 + 1

F
1
1-
lim ^ _ 1-0 = 1
ur
r
1
/t->0 1 + 0
1 + e 1/^'
fo
ks
Clearly, a: lim^ fix) lim fix). Hence, x->0
lim /(x) does not exist.
Yo
+
0 x->0
oo

LXAMI’LF13 Iff is an odd function and if lim fix) exists. Prove that this limit must be zero.
eB

SOLUTION It is given that ^ ^°


lim fix) exists
x->0
ur
ad

lim fix) = lim + fix)


Yo

0 0

lim /(0-/j)= lim /{O + /2)


d

h^O h~>0
Re
in

lim fi~h) = lim /(/?)


h^O ;i-> 0
F

- lim fih) = lim /(/i) [v fix) is odd ■■■ fi-h)=-fih)]


/,-^o ;j^o

2 lim fih) = 0 => lim fih) = 0 => lim fix) = 0


h ^ 0 /i->0 x^O

EXAMPLE 14 Iff is an even function, then prove that lim / (x) = lim + fix).
x^ 0 x^O
SOLUTION Clearly,
lim fix) = lim /(0-/i)= lim fi~h)
x->0 h-*0 /i -> 0

lim fih) [v / is even fi~h) = fih)]


h 0

lim_ fiO + h) =
h -*0
lim + fix).
X 0
LIMITS 16.11

EXERCISE 16.1

X
1. Show that lim — does not exist.
0 |a:|
2x + 3, x<2
2. Find k so that lim f{x) may exist, where f{x) = X +k , x>2-
x->2

3. Show that lim — does not exist.


x->0 X

3x
a:# 0
4. Let/(a:) be a function defined by/(x) = -\x\ + 2x '

w
x =0 ●
0

Show that lim /(a:) does not exist.


a: ^ 0

o
- 1 ^ £/ \ [x + 1, ifAT>0

e
.. Let/(AT) = Prove that lim f{x) does not exist.

re
0

6. Let/{a:) = I a: -4,
a:+ 5, if a:>0
if a: < O’ Prove that lim

Frl
/ (x) does not exist.

F
4, if X > 3
7. Find lim / (a:), where / (a:) =
ou
X 4-1, if X < 3

r
x^3

so
2x + 3 , x<0
S. If/(x) = 3(a: + 1) , a:>0- Find x-»0
lim / (x) and lim / (x).
x->l
kf
oo

9. Find Urn /(x),if/(x) =


Y

x->l
B

10. Evaluate x->0


lim / (x), where/(x) =-^ x
l^,x ^ 0
re

0 ,x = 0
oY
u

X - [x] , x<2
ad

11. Evaluate lim /(x) (if it exists), where/(x) = < 4 , x = 2.


x->2 3x-5 , x>2
d

12. Evaluate the following one sided limits:


in
Re

X - 3 x-3 1
(i) lim + (ii) lim (iii) lim
x^ -4 x^ -4 + 3x
F

x-»2 x->2“ x->0

2x 2 x^ - 3x + 2
(iv) lim (v) lim + (vi) lim
X + 8 x->0 xl/5 x->0 x^ - 2.t^
x^-l 1
(vii) lim (viii) lim
x->-2
+ 2x + 4 x-1

-1/x does not exist.


13. Show that lim e
x^O

14. Find:
(iii) lim [.x] (iv) lim [x].
(i) lim [x]
x->2 (ii) lim^lx] X 1 x->-5/2
2

15. Prove that lim + [x] = [a] for alU e R. Also, prove that lim [x] = 0.
X -> a x-»r
16.12 APPLIED MATHEMATICS-XI

X
l('. Show that lim — ^ lim
[x] X -» 2^ [x]
X a:
17. Find lim —. Is it equal to lim
x-> 3
+
[X] x^3~

ANSWERS

k = 5 4 8. 3, Does not exist 9. Does not exist 10. Does not exist

' 12. (i) -00 (ii)oo (iii) 00 (iv) -cc (V) GO


(vi) - 00 (vii) 00 (viii) oo

ow
14. (i) Does not exist (ii) 2 (iii) Does not exist (iv) -3
1, No

HINTS TO SELECTED PROBLEMS

e
X X
X = 1- ^1

re
lim lim = -1 and, lim — lim — =1
X ^ 0" I ^ x->0 - X + X
0+ ^

Flr
F
lim ^ lim —
X-.0- U x->0+ 1^1
X
Hence, lim ;— does not exist.
ou
sr
X->0 X

fo
3. We have.

lim f{x) = lim /(O -h) = lim


1
= -00 and. k
lim f{x) = lim /(O + h) = lim
1
— =00
oo
x^O h^O h^Q -h x-).o+ h^O h
Y

Clearly, lim /(x) lim /(x). Hence, lim/(x) does not exist.
reB

+
x->0" x->0 x-»0

8. We have.
uY

/w={ 3(x + l)
2x+ 3, x<0
x>0
ad
do

lim /(x) = lim (2x+3) = 2 x 0 + 3 = 3


x->0 X ^ 0
in

and. lim /(x)= lim 3 (x +1) = 3 (0 + 1) = 3


Re

x^O"^ x->0'*’
F

So, lim /(x) exists and is equal to 3.


x^O

lim /(x) = lim 2x+3=2xl + 3=5


x-> r x^r

lim /(x) = lim 3(x + l) = 3 (1+1) = 6.


X -► I'*’ X -> C

lim /(x) lim /(x)


x->r

Hence, lim /(x) does not exist.


x-^l
LIMITS 16.13

We have.
x<l
f(x) = x>l

lim
lim f{x)= lim x^-l=l^-l=0 and, lim f{x) = ,r->l -x^-l=-l-l^-2
+
->r x-^^

lim f{x) ^ lim + f{x).


x-^r X-¥l

Hence, lim / (:t) does not exist.


x-»l

We have.

low
X 0

m=\ X
0 , x =0

lim l^ = — X x
lim — = -1 and, lim f{x) - lim lim -=1.
lim f{x) =

ee
x->0
x->0 x-^0- ^ x->0 rFX x->0^

Fr
lim f{x) * lim f{x).
x^0“ x^O'*’

Hence, lim f{x) does not exist.


for
u
x^O
ks
Yo

13. We have.
o

Vh
lim f{x)= lim f{0-h)= lim e
Bo

= 00

x->0 h-^0

1
re

-1//1
and. lim + f{x) = lim /(O + h) = hlim e = lim —T71- = 0
x^O h^O ->0
ou
ad

16.3 DIFFERENCE BETWEEN THE VALUE OF A FUNCTION AT A POINT AND THE


Y

LIMIT AT THAT POINT

Let/(x) be a function and let be a point. Then, we have the following possibilities:
nd

(i) lim f{x) exists butf{a) (the value off{x) at x = a) does not exist:
Re

X 2-9
X a
Fi

Consider the function f(x) defined by f{x) = -X - 3

Clearly, this function is not defined at x = 3 i.e./(3) does not exist, because it attains the
form—.But, it can be easily seen that lim /(x) = lim /(x)=6.So, lim /(x)exists.
0 x-»3" x->3-^ ^-^3

Thus, the lim /(x) exists but/(3) does not exist.


x^3

(ii) The value f{a) exists but lim /(x) does not exist:
X —»

In example 4 on page 16.6, we have seen that X lim


-*k
/(x) does not exist but f{k) = k exists.

(iii) lim f{x) and f(a) both exist but are unequal:
X -* a

Consider the function/(x) defined by


16.14 APPLIED MATHEMATICS-XI

X x^l
m = x-2

It can be easily seen that lim /(x}=4= lim f{x).


+
X X 2

So, lim f{x) exists and is equal to 4. Also, the value/(2) exists and is equal to 3.
x-*2

Thus, lim f{x) and f(2) both exist but are unequal.
x^2

(iv) lim f{x) and f{a) both exist and are equal
x->a

Consider the function/(a;) defined by


-4 , x^2
/w = a:-2

w
4 , x = 2

16.4 THE ALGEBRA OF LIMITS


,t->2

F lo
For this function, it can be easily seen that lim /(x) and/(2) both exist and are equal to 4.

Let/and g be two real functions with common domain D. In the chapter on functions, we have
defined four new function / ± g,fg,f/g on domain D by setting
(J±g) {x) = fix) ± g (.v),
ree
for F
(fg) (x) = fix) g(x)
if/g) (^') = f{x)/g (x), if g(.t) ^ 0 for any x eD.
r
You

Following are some results concerning the limits of these functions.


oks
eBo

Let X lim
-> a
/(.t)=/and lim g(A:)=m. X -> a

If / and m exist, then


ad
our

(i) X
lim (/±g) (x) = lim f(x) ± lim g(x) =!±m
X a

(ii) lim (/g)(.r)= lim f{x)- lim g{x) = lm


Re
dY

X Iftrf fix)
X a X -> a

X -*a /
(iii) lim = —, provided m ^ 0.
Fin

X a g lim g(.v) m
X -*a

(iv) lim kfix)=^k lim fix), where k is constant


X -> a X —*a

(v) lim |/(a:)| =1 lim /(a-)| =[ /[


X —> rt

(vi) lim \ fix) ■


X ->lT
= /

(vii) If/{A.-) (.v) for every a: in the deleted neighbourhood of a, then lim fix) < lim gix)
X X fl

(viii) If fix) < g (at) < h (a:) for every x in the deleted neighbourhood of a and
lim fix)=l = lim
X —> 17
/z(a:), then lim gix)=l.
X ->a X 17
LIMITS 16.15

This result is often stated as Sandwich Theorem.

(ix) If lim f{x) = +00 or-CO , then lim — = 0.


X a x-*a m
16.5 INDETERMINATE FORMS AND EVALUATION OF LIMITS

Uptill now we have been discussing left hand and right hand limits and the existence of limits.
In what follows, we will be assuming that the limit of a function at a given point exists. In the
previous section, we have stated that
lim fix)
fix)

ow
X a
lim provided that lim g (x) ^ 0.
X -* a gix) lim g(x)' X ^ a

X -* a

fix) takes
An interesting" situation now arises. If lim / (x) = lim g (x) = 0, then lim
X a- X ^ a gix)

e
X a

0 fix):IS

re
the form — , which is undefined or meaningless. But, this does not imply that lim

Frl
F
meaningless or it does not exist. In fact, in many cases this limit exists and has a fimte value. The
determination of limit in such a case is traditionally referred to as the evaluation of the
ou
indeterminate form -, though literally speaking nothing is indeterminate involved here.

sor
0 '
0 0
Sometimes — is referred to as undetermined form or illusory form. In addition to — there are six
0 kf 0
other indeterminate forms, namely, — , 0 x x, co - oo, 0^, and I*". Among all these seven
oo
X
Y

indeterminate forms — is the fundamental one because all the remaining six forms can easily be
B

reduced to this form. In this chapter, we shall study how to evaluate a limit which belongs to one
re

of following in determinate forms:


oY

0
u

, 0 X X and x - x.
0
ad

To facilitate the job of evaluation of limits we categorize problems on limits in the following
d

categories:
in

(i) Algebraic Limits. (ii) Non-algebraic Limits.


Re

If a problem on limits does not involve trigonometric, inverse trigonometric, exponential and
logarithmic function, then it is a problem on algebraic limits, otherwise, it is a problem on
F

non-algebraic limits.
For example.
X 3-1 + X -
yi -a: x^ -32
(i) lim (ii) lim (iii) lim
1 x-1 0 X X 2 x-2

X^ + X + 1
(iv) lim etc. are problems on algebraic limits.
X —> CO 2x^+5

Following are some examples of non-algebraic limits:


sm X
-1 3 sin” ^ 2x sm X - cos X

(i) lim - (ii) lim (iii) lim


X ^ 0 X X ^ 0 sm X X -> n/i 4X - 71

2-tx_3x
(iv) lim —
X -+ 0 X
16.16 APPLIED MATHEMATICS-XI

16.6 EVALUATION OF ALGEBRAIC LIMITS

In order to evaluate algebraic limits we have the following methods,


(i) Direct substitution method,
(ii) Factorisation method,
(hi) Rationalisation method,

w
(iv) By using some standard limits,
(v) Method of evaluation of algebraic limits at infinity.
We shall now discuss these methods with suitable illustrations in the following sub- sections.

e
e
●6.6,: DIRECT SUBSTITUTION METHOD

o
r
Consider the following limits:

r
0(a:)

F
(i) Urn fix) (h) lim
.r -> (7 X —>a Vj/(X)

oF
ul
If / (fl) and exist and are fixed real numbers, then we say that
V(«)

sr
ko
lim f (x) = / (a) and lim
x->a x-*a y (iJ) y (a)
of
In other words, if the direct substitution of the point, to which the variable tends to, we obtain a
o
fixed real number, then the number obtained is the limit of the function. In fact, if the point to
Y

which the variable tends to is a point in the domain of the function, then the value of the function
rB

at that point is its limit.


eY

Following examples will illustrate the above method.


u

ILLUSTRATIVE EXAMPLES
d
o
ad

l.XAMPI.i Evaluate: lim 3x^ + 4x + 5.


x-*\
in

SOLUTION lim 3x^ + 4x + 5 = 3(l)^ + 4(l)+5=12.


Re

x^l
F

x^-4
Evaluate: lim
x^2 x+ 3

SOLUTION Using direct substitution method, we obtain

lim - 4 4-4 0 ^
x-v2 -t+3 2+3 5

Evaluate lim
.^1 + X + —X
1 + .t

SOLUTION Using direct substitution method, we obtain

lim + X + .^1 — X + 0 + .^1 — 0 1 + 1


= 2.
x^O 1 + x 1 + 0 1
16.17
LIMITS

f-XERCISE 16.2

Evaluate the following limits:


lim
+1
lim
2x^ + 3x + 4
lim
flx+~3
x + l X 0 x^ + 3x +2 X-¥ Z AT + 3

^x+8 Vx + Vrt
lim . lim lim
x-»l x->a X + a x^l l + x2
x2/3_9 x^-3x + l X 2-9
lim lim lim
x->0 x-27 x^-l x-1 X -> 3 X + 2
ax + b
111 lim ,di^O
0 CX + d

ow
ANSWERS

1 1 1 1 -3
2 . 3 0
1. 1
2 ■af 2 3 2

e
re
16.6.2 PACTORIZATION METHOD

Fl
fix)

F
Consider the limit: lim
x->a gix)
ur
fix) 0

r
If by substituting x = a, reduces to the form then (x -a) is a factor of / (x) and g (x)
gix) '
both. So, we first factorize/ (x) and g (x) and then
o' fo
cancel out the common factor to evaluate the
ks
Yo
limit.
oo

Following algorithm may be used to evaluate the limit by factorization method.


eB

ALGORlTHr.:

fix)
Obtain the problem, say, Xlim where lim fix) =0 and lim g (x) - 0.
gix)'
ur

X -> a x-*a
-> fl
ad

Factorize / (x) and g (.x).


Yo

Cancel out the common factor(s) of fix) and ^(x).


Use direct substitution method to obtain the limit.
d
Re

Some useful results to remember:


in

(i) (j2 _ (fl + y) (ii) a^-b^ = (a- b) + ab-i-


F

(iii) = (fl + b) (iv) a^~b^ = {d^-


- i?b + i?2) + b^) = {a + b){a-b) {a^ + b^)
(v) If/(a) = 0, then x - a is a factor of fix).
Following examples illustrate the above algorithm.
ILLUSTRATIVE EXAMPLES

;2 - 5x + 6
X
e:- V- -1 Evaluate: lim
x^-2 X 2-4 ■
X 2 -5x+ 6 0
SOLUTION When x = 2 the expression assumes the indeterminate form —.
0
x2 -4
Therefore, (x - 2) is a conunon factor in numerator and denominator. Factorising the numerator
and denominator, we obtain
16.18 APPLIED MATHEMATICS-XI

^-5x+6 0^
lim - form —
x-*2 0

(x-2){x-3)
lim
x-3 2-3 1
lim
x^l{x + 2){x-l) i^2x+2 2+2 4

x^-l
l-XAMPLl:- Evaluate: lim
x-^i X-1

3-1 0
SOLUTION When x = 1 the expression — assumes the indeterminate form—.Therefore,
x-1 0

(x -1) is a common factor in numerator and denominator. Factorising the numerator and
denominator, we obtain

w
3-1 0^
lim - form —
x-*l x-1 0

(x-1) (x^ + x + 1) = lim x^ + X + 1 = 1^ +1 + 1

Flo
lim = 3.
x^\ (x-1) X->1

e
re
x3-3x^+4
vipi Evaluate: lim
x^ - 8x^ +16

F
x-»2

3-3x^ + 4
ur
0

r
SOLUTION When x = 2, the expression ~ assumes the indeterminate form -.
x^ -8x^ +16 fo 0

Therefore, (x - 2) is a factor common to numerator and denominator. Factorising the numerator


ks
and denominator, we obtain
Yo

x3 -3x^+4
oo

lim
x-*2 x^ -8x^ +16
B

(x - 2) (x^ - X - 2)
re

lim
x->2
(x^ - 4f
u

(x-2) (x-2)(x + l) x +1 2 +1 3
ad

lim
lim ^=: T= —
Yo

x~^2
(x-2)^(x + 2)3 X-+2 (x + 2)^ (2 + 2)^ 16
x3 -6x^ + llx -6
d

Evaluate: lim
Re

x->2 x^ - 6x + 8
in

3 -6x^ +llx-6
F

0
SOLUTION When x = 2, the expression — assumes the form —. Therefore,
x^ - 6x + 8 0

(x - 2) is a factor common to numerator and denominator. Factorising the numerator and


denominator, we get
x3 -6x^ + llx - 6
lim
x-*2 x^ - 6x + 8
lim
(x-1) (x-2) (x-3) (x-1) (x-3) (2-1) (2-3) 1
lim
x-*2 (x-2) (x-4) x->2 (x-4) (2-4) 2
o„3 _ -t
I.XAMI’l ● Evaluate: lim
x^l/2 16x^-l
8x3 0 n
SOLUTION When X =1/2, the expression assumes the form —. Therefore, x — or.
16x“^ -1 0 2J
LIMITS 16.19

2x-1 is a factor common to numerator and denominator. Factorising the numerator and
denominator, we obtain
8x^ -1 0
- form
lim
0
X ^1/2 lex"^-1

■.^/2 (4X^)2-l2
(2.y-1)(4.\-2 + 2a- + 1) fQ
— form
= lim = T 0
y-j-1/2 (4y^ +1)(4y2 -1)
. (2y-1)(4y2 + 2y + 1) 4^2 + 2y + 1 3
lim
x^l/2 (4y^+1)(2y-1)(2y + 1) x^l/2 (4y^ +1) (2y +1) 4

w
2 1
EXAMPI-Lfe Evaluate: lim +
x->1 Y-1

Flo
SOLUTION We have,
2 1 2 1

ee
lim 4- lim
y^i[1-y2 X-1^ X -4 1 1 - 1 - -I'

Fr
2 1
When X = 1, the expression - assumes the form oo - co, So, we need some
1-y2 1-x for
ur
simplification to express it in the form —.
0
Taking LCM, we get
k s
2 1
Yo

lim (oo - CO form)


oo

A- -> 11 1 y-2 1-X


eB

0
2 -(1 + X) form
lim
0
x-»l l-.x2
r

1 -Y 1 1
ou
ad

lim lim
A->1 1 -Y^ X^l1 + Y 2
Y

1 Y
rxAMPLL" Evaluate: lim
x^l Y^ + Y - 2 3-1 ■
Re
nd

1 Y
assumes the indeterminate form
Fi

SOLUTION When y = 1, the expression


y-2 + Y - 2 Y 3-1
0
CO -
CO. So, we need simplification to reduce the expression in the indeterminate form —.
1 Y
lim (co - CO form)
X —)● 1 y2 + Y - 2 Y 3-1
1 Y
lim (oo - CO form)
1 I (y + 2) (y -1) (y -1) (y'2 + y +1)
,lim <f (y^ + Y + 1) - Y= (Y + 2) 0
— form
0
X ^ 1 [ (y + 2) (y -1) (y^ + y +1)
-(A-1) -1 1
lim = lim =
9
1 (y + 2) (y -1) (y2 + Y + 1) A-i- 1 (y + 2) (y^ + Y + 1)
16.20 APPLIED MATHEMATICS-XI

-4
I XAMl’Lf-8 Evaluate: lim
■J2 x'^ + 3xj2-s'
-4 0
SOLUTION Whenx =
■Jl, the expression —j + 3x V2-8
assumes the indeterminate form —.So,
0

(x - V2) is a factor of numerator and denominator. Factorising the numerator and denominator,
we get
-4

ow
lim form
0^
V2 x^ + 3x42-8 0

lim
(x^-2) (x^ + 2) form —
0^
(a: + 4V2) (x - 42) 0

e
{x-42) {x + 42) {x^+2) 0^

re
lim form —
4i {x + 4V2) {x - 42) 0

Flr
{x + 42) + 2) (2V2) (2 + 2) 8

F
lim
V2 {x + a42) 542 5
ou EXERCISE 16.3

sr
Evaluate the following limits:
lim
2x^ +9x-5
lim
-4x + 3
fo
k
oo
i->-5 x +5 :r->3 x^-2x-3
^ -81
Y
x
lim - lim
reB

x-^3 x^-9 x-»2 x^-4


8:r^ + l ^-7x + 12
uY

lim f>- lim -


x^-1/2 2x + l a:->4 x^ -3x-4

x^-9x + 20
ad

-16
do

lim lim
X -+2 x-2 X -»5 x^ -6x +5
in

x^ +1 x^ -125
lim lim
Re

X -1 x + 1 X-+5 a:^-7a: + 10
F

X^ -2 x^ -3
lim 12. lim
V2 x^ +42x-4 43 x^ + 343x-12
-9 X 4
lim 1" lim
43 x^ + 443x-15 x->2 x-2 x^ -2x
1 X 1 2
lim lim
x^l a:^ +X-2 x^ -1 ar^3l.A:-3 :t‘^-4a:+3
1 2 4x-l
1' lim 18. lim —j=
x-»2 x-2 x^ -2x ar^l/4 2VX-1

x^ -16 (a + x)^ -a
2
F’ lim 20. lim
a:-»4 x->0 X
LIMITS 16.21

ANSWERS

1 1 1
-11 3. 18 4. 3 3 6. /. 32
2 5 4

2 2 1 1
0. 3 10. 25 12. - 2 14. 2 lb.
3 5 9 2

17. - 18. 2 10. 32 , 2a


2

16.6.3 RATIONALISATION METHOD

This is particularly used when either the numerator or denominator or both involve expression
consisting of square roots and substituting the value of x the rational expression takes the form
0 00

low
— etc.
0' CO

Following examples illustrate the procedure.


ILLUSTRATIVE EXAMPLES

ee
■^2 + X — -J2
l.XAMPLE 1 Evaluate: lim

F
Fr
x->0 X

0
SOLUTION When x = 0, the expression takes the form for
ur
X 0

Rationalising the numerator, we get


s
V2T^-V2 (V2T7-V2) (V2TI + V2) 0^
k
Yo

lim lim form -


oo

x^O X x^O + X + V2) 0


eB

2 + X-2 O'!
lim form —
x->0 X
(V^ + V2) 0
r

1 1
ou
ad

lim
x->0 + X + V2 2-Jl'
Y

X
liXAMPLL ? Evaluate: lim
b -^a-x
Re
nd

X 0
takes the form -.
Fi

SOLUTION When X = 0, the expression


+ X -
yja^ 0

Rationalising the denominator, we get

lim
X
lim
X
X —^
iy[a + X +
yja-x) 0\
form -
0 ^a + x -^a-x 0 Qa + x ~yja-x) X + ^a-x) 0

lim
X Qa + x + ^a-x) 0^
form —
0
(fl + x- fl + x)

lim
{■Ja + X + ^a -x) 2 Vfl
= Vfl
x^O 2 2

2
a + X
EX.XMPl_ ; Evaluate: lim
x->0 x2
16.22 APPLIED MATHEMATICS-XI

+ x~ -x^ 0
SOLUTION When x = 0, the expression takes the form—.
0

Rationalising the numerator, we get

lim = lim
+ AT
^ -V?-^) (V?
a-+ + ^|a^-x^) form
0

X -^0 0

2 2 2 2
lim
a + X -a +X 0^
form
x->0 + X
0

2 2 1
lim

i^ja'^ + x^ +^a^-x^)
x->0 2 2 a
+ a

x^ -16
L'XAMPLE 4 Evaluate: lim

w
X —> 4
^+9-5 x^ -16 0
SOLUTION When x = 4, the expression assumes the form —.

F lo
Rationalising the denominator, we get -<jx^ +9 -5 0

x^ -16 (:r^-16) i^jx^+9+5)

ee
0
lim lim form —

(^+9-5) i^x^ + 9+5)

Fr
x^4
+ 9 -5 x-»4 0

lim {x^ -16) {^jx^ +9+5) for form —


0^
ur
X ^4
+ 9 - 25) 0

lim
yjx^ + 9 + 5 = V16 + 9 +5 =5+5=10
s
X -> 4
ok

y V.
Yo

+ 2x --JTx
o

E\.AMPLE5 Evaluate: lim


eB

x-*a
yj3a + X -2-Jx
+ 2.x - VsT 0
SOLUTION When x = a, the expression assumes the form —.
^3a + X - 2yfx
r

0
ou
ad

Rationalising the numerator, we get


yja + 2x --JSx Qa + 2.T - ^f3x) Qa + 2x + ^/^) (^[3a 2^) + X + 0^
Y

lim lim form —


X-»rt
yj3a + X - 2-Jx X
2V?) (V^ + X + 2Vx) (^rr^+vsx)
+ X - 0
Re
nd

lim {a + 2x- 3x) QSa + x + 2-Jx


Fi

X-»fl
(3i? + :c - 4;r) Qa + 2x + 4^)
lim
yj3a + X + 2 Vx form —
0
X -> a 0
3 ■ yja + 2x + ●

yj 3a + a + 2 -Ja 1 4 -Ja ^ 2

3 {yja + 2a + V 3a) 3"" ITsfl " sTf


EV.XMPl.E 6 Evaluate: lim 3-ys + X

x^4
1-#^'
SOLUTION When x = 4, the expression
3-# + X assumes the form —.
0

1-^/^ 0
LIMITS 16.23

Rationalising the numerator and denominator, we get

lim
3-^5 + X
lim
{3 - (3 + ^5 + x) (1 + ^5^) form —
0

X
4 1 - ^5-x x—>4 (1 -^5-x){l +^5-x) (3-\-^5 + x) 0

lim
(9-5-x)( 1+^5^ form
x->4
(1 -5 + j:) 3 + ^5 + x 0

lim
(x-4)(l+^/^) 0
form —

ow
x-*i
(A:-4)(3 + ,/rr7) 0

(1 + 1) 1
lim
r->4
(3 + ^/^) (3+3) 3

e
X^ -4

re
example: Evaluate: lim
x^2
^3x - 2 -^x + 2

Flr
x^ -4 0

F
SOLUTION When X = 2, the expression assumes the form -.
0

Rationalising the denominator, we get


ou
sr
lim
x^ -4
lim
(a:-2)(x+2) (V3F^ + V^) form
0

fo
- V^T+I) (43^ + 47+2) 0
x^2
^3x - 2 -^x + 2 x-*2

jx-2) ix + 2){437^ + 4^)


k
oo
lim form
0
(3X-2-X-2)
Y

(x-2){x + 2) (43^+4^)
reB

0^
lim form —
x^2 2(x-2) 0
uY

lim
{x + 2) (43^ + 4^) (2 + 2) (2 + 2)
x^2 2 2
ad
do

EXAMPLES Evaluate: lim


{2x-3) (Vx-1)
x->l 2:c^ + a: - 3
in

. (2a:-3)(V^-1) 0
Re

SOLUTION When .r = 1, the expression takes the form —.


2x^ + a: - 3 0
F

Rationalising {Jx -1) in the numerator, we get


lim
(2a:-3) (VI-1) lim
(2x-3)(Vx-1) (Vx+1) form
0

X->1 2x^ + X - 3 x->l (VI+ 1) (2x+ 3) (a:-1) 0

(2x-3)(x-1) 0^
lim form —
1(VI +1)(2x+ 3) (X“1) 0

2x-3 1
lim
1 (VI +1) (2x + 3) 10

EXERCISE 16.4

Evaluate the following limits:

1. lim
^1 + x + x'^ -1 iZ. lim
2x

x-»0 X
VI + X -
4a^
16.24 APPLIED MATHEMA7ICS-XI

3. lim
~a
4. lim
■+ X -
yi -y
2x

lim
V3^-l 6. lim
x-3

x-^2 2-x x^3


^x-2-^4:-x
lim
X
8, lim ^X~~4: -

w
ar-^0 + X - - X x^l x~l

lim
x-1
lim .JxTS-yfS
^x'^ + 3-2 x^-9

e
^5x -4 - Vx

re
Ti. lim 12. lim

ro
X

^jx^ +1 - Vs

F
x-2
Um 14. Um
x-»2 -Jx -y/2
X-2
oF
lim 4-V9
1-#
+ X

ul 16. lim ^+ X -4a

sr
x->7 - X x^O
X + ax

ko
lim ,
X -5
. lim
^5x-4 - Vx
x->5 ^6x-5-^4x + 5
of x^-l
yj1 + Ax -^5 + 2x .^3 + X -^5-x
o
Y
19. lim lim
X-2 x2-l
B

-^1 + X^ --^1-X^
Y

+ X + X
. lim 22. lim
er

x^O X x^O 2x^


u

2: . lim ^3 + X -
7' lim
(2x-3) (V^-1)
d

2-1
o

3x2 + 2x-6
ad

x->l X 1

x^~^ ^x + h - Vx ,
in

lim ' . lim X #0


x-> 1 Vx -1 h^O h
Re
F

ANSWERS
1 1 . 1
1. - 2 ^/fl 3. — 4. - 6. 1 7. 1 8. 2
2 2a 2 2
1 2
9. 2 10. 11. 1 12. - 13 14. 2V2 15. -- 16.
I2V6 2 " V5 4 2a4a
2 1 1 1
5 18. - 19. 2H. - 0 22. -
3 3 4 4
1 1 1
23. -
I
2‘, 3
4 18 " ■ 2V^

16.6.4 EVALUATION OF ALGEBRAIC LIMITS BY USING SOME STANDARD LIMITS

Following theorem will be used to evaluate some algebraic limits.


V'-fl" H - 1
THEOREM If« 6 Q, then lim = ?ia
X-M7 x -a
LIMITS 16.25

We have,

lim lim
+
V x-*a
Urn fix) exists x-*a
lim fix) = lim fix)
X ->a x-a X -a

ia + h)" - a
n

lim
h^O a + h -a

fl” 1 +
h1‘
-u
a

lim

ow
/i-»0 h

h «(«-!) + ... -1
● 1 + « - +
2
a 2! a n(n-l) X 7
= fl" lim — ■: (1 + x)" = l+nx + 2!
+...

e
h^O h

re
= d' lim
n
- + njn-l) h ^
2!

Frl
a a

F
Following examples will illustrate the use of the above result in evaluating algebraic limits.
ou
sor
ILLUSTRATIVE EXAMPLES

-1024 kf
EXAMPLE j Evaluate: lim
x->2 x-2
oo

-1024 0
Y

SOLUTION When x = 2, the expression assumes the form - .


0
B

x-2

10 10
x^^ -1024 -2
= 10(2^° “b =5120
re

Now, lim = lim -


oY

x->2 ;c-2 x^2 X-2


u

x^°-1024
ad

EXAMPLES Evaluate: lim


2 x^-32 '
d

;».10 _;^Q24
in

0
s the indeterminate form —.
Re

SOLUTION When x=2, the expression assume


0
- 32
F

x^^ -1024 0
form —
Now, lim 0
x^2 / -32
^10_2l0 0
form -
lim
0
2 x5-2^

^10-210
-t-2 ^10-210 x^-25 10-1
^ 5-2
5-1
= 64.
lim lim -r lim = 10-2
X -* 2 a^-2^ x^2 x-2 x-*2 x-2

X-2

x^^^-27
EXAMPLE 3 Evaluate: lim
x-*9 x-9
16.26 APPLIED MATHEMATICS-XI

3/2 -27
X 0
SOLUTION When 'X = 9, the expression assumes the form
x-9 0

;f3/2 _27 ^3/2_g3/2 3


3/2-1 = ^(3) = ^2
9
Now, lim lim (9)
x-^9 x-9 x^9 x-9 2 2

X Vx -a^fa
rXAMPLl;-i Evaluate: lim
x-*a x-a

SOLUTION We have.
X Vx -a-yfa x3/2_^3/2
lim lim = 3«3/2-1 ^3^
ar->(7 X - fl X -*a x-rt 2 2
m
-fl"'
LXAMPLE =. Evaluate: lim —
ft n
x-^a x -a

w
SOLUTION We have.
x'
H m
- a x'"-a"' x-a Ji

F lo
X
lim lim lim
n n
x-*a X -*a X-a X -a x-¥a X-a x-a

yT-d^ JH-1
^ nd'-^ = '^d^-r

e
lim ^ lim ma

Fre
x-^a x-a x-*a x-a n

x-2
EXAMPLE i> Evaluate: lim
2 ^
for
r
SOLUTION We have,
You

x-2 1 1 1
s

lim
= 3(2^/^)
ook

^1/3 _2l/3 ^1/3 _2l/3


x-*2
lim
1
2I/3-I ^x(2-2/3)
eB

x-^2 x~2~ ' 3l 3

EXAMPLE 7 Evaluate: lim


(l-x)"-l
our
ad

i->0 X

SOLUTION We have,
dY

(l-x)"-l (l-x)"-l
Re

lim = - lim
x-»0 X ar -»● 0 (1 - x) -1
Fin

/-I
«

= - lim , where y = 1 - x. [v X -> 0 =5> y ^ 1]


y->l y-1
= =
-n.

5/3 5/3
EXAMPLES Evaluate: lim
(x + 2) -{a+ 2)
x->a x-a

SOLUTION We have.
5/3 5/3
lim
(^ + 2) -io + 2)
x ->●(? x-a

5/3 5/3
lim
(X + 2) -(«+2)
x-*a (x + 2)-(a + 2)
LIMITS 16.27

lim , where x + 2 = y and <7 + 2 = b.


y y-h
5
^5/3-1 ^ 5j^2/3 ^
3 3 3

X 4-1 x^-k^
EXAMPLE 9 Find the value of k, if Urn - = lim
X x-l X -* k X

w
SOLUTION We have,

x4 -1 x4-l4 4-1
lim lim = 4(1) = 4
X->1 x-1 ar->l x-1

e
3-P ;,3_fc3 ^2_fc2

ro
X-k

re
and, lim lim - = lim
X X-k X -*k x-k x-k

x^-k^ x^-k^

F
3-1 2-1 3
lim -r lim = Sit - Ik = -k

Fl
x-^k x-k x-k 2

lim -
4-1
lim
x^-k^

u
sr
X ->1 x-1 x~^k x^-k^

ko
o
3fc
=> 4 = k = — of
2 3
x"-2"
EXAMPLE 10 If lim = 80 and neN, find n.
o
Y
x^2 x-2
erB

SOLUTION We have,
uY

y -2
n
n- 1 «-l 5-1
lim = 80 => ?i -2 = 80 => n-2 = 5-2 => «=5.
x->2 X -2
9 9
x’ + a
ad
do

EXAMPLE 11 If lim = 9, find the real values of a.


X + a
in

SOLUTION We have,
9 9
x^ + fl
Re

lim = 9
F

x—>-a X + a

=> lim
x^-{-af = 9 => 9 (- 77)
9-1 8
= 9 => 9T7° = 9 => 77
8
= 1 => 77 = +1
X - 77 X-(-77)

EXAMPLEir. Evaluate: lim


(x + x^ + x^ + ■■■ + x”) -n
X ^ 1 x-1

SOLUTION We have.

lim
(x + X^ +X^ + ... + x") -77 0
form —
x-»l x-1 0

lim
(x-l)+(x2-l)+(x^-l) + ... + (y’-l)
x^l x-1
n
x-1 x‘^-1
lim + lim + lim +... + lim
x^lX-1 x^l x-1 X->1 x-1 X -T’l X-1
16.28 APPLIED MATHEMATICS-XI

2-1 3-1 H-1 n {n +1)


= 1 + 2(1) + 3(1) + ... + «(!) = 1+ 2+ 3+ ...+« =
2

EXERCISE 16.5

Evaluate the following limits:


lim
ix + 2f^^-{a + 2) 5/2 lim
ix + 2)
3/2
-(fl + 2)
3/2

X -^a X - a X a x-a

^2/7 -a
2/7
3. lim 4. lim
0(l+:c)2-i x^a x-a

;c^/7_,5/7 Sx^ + 1
5. lim 6. lim
in in
x^ a X a x-»-l/2 2x + l

w
1/3 1/3
7. lim
+ 3) (^ -3) -64
8. lim
x->27 x-27 x->4 -16

Flo
15
9. lim —
-1 x^ +1
‘H. lim
10
X->1 X -1 -r ^ - 1 x + 1

ee
^2/3 2/3 ti

Fr
n
-a
n. lim - If lim = 108, find the value of n.
X ->(? -a
3/4 x-> 3 X-3

x^ 9
for
ur
-a
If lim = 9, find all possible values of a.
X ->a x-a
s
ok

' . If lim
= 405, find all possible values of a.
Yo

x—*a x-a
o

x^ 9
eB

-a
■ '■ If lim = lim (4 + x), find all possible values of <7.
x-> a x-a x^5

x^ -a
3
-1
r

16. If lim lim find all possible values of a.


ou
ad

x a x-a x —^ 1 X —1
Y

_,_ANSWERS
5 , 3
1. -(fl+2)
3/2
(« + 2)
1/2
3. 3 4. 2 i7 -5/7
— 5,^«3/7
nd
Re

2 2 7 2
2 3 8
6. 3 - 1/12
Fi

7. - 8. 6 9. ~ 10. 3 11. -i7 12. 4


9 2 9
2
1'.^. 1,-1 14. fl = 3,-3 15. 1,-1 16. ±
^/3
HINTS TO SELECTED PROBLEM

X 15 _|15
xl5_i x-1 15(1) 15-1 ^3
9. lim lim
10 10 10
x^ 1 X -1 x-> lx -1 10(1) 10-1 2
x-1

16.6.5 METHOD OF EVALUATION OF ALGEBRAIC LIMITS AT INFINITY


1 1
Consider the functions / (x) = — and g (x) = —Graphs of these functions are shown in Figures
X x^
16.6 and 16.7.
LIMITS 16.29

X'
O X O X
X'

ow
Y' r
1
Fig. 16.6 Graph of/(x) = - Fig. 16.7 Graph of f{x) =
X

e
re
1 1
decrease
We observe from the graphs that as x increases, the values of f{x) = - and ^(x) = —

Frl
F
1 1
rapidly and when x is indefinitely large —X and x^ are indefinitely small i.e. very close to zero. In
ou
such cases, we write

or
lim
X-¥+ 00 X
- = 0 and
:c
lim ^ =0.
-> + 00 X
kfs
We also observe from the graphs of these two functions that as x decreases and is very small
oo

negative real number, then also the values of —X and X approach to zero. So, we wnte
Y
eB

lim — = 0 and
X -> -00 X X ^
lim \ ~ X
ur

It follows from the above discussion that:


oY

(i) lim c =c (ii) lim c = c


ad

a: -> 00 X -> -00


d

(iii) lim = 0,n>0 (iv) lim — = 0, n e N


x^ oo jc” X -¥ -00
in
Re

From the graphs of real functions, we obtain the following useful results:
(ii) lim
F

(i) lim X ->■ + 00 X -> -00


X -^ + 00 X -00

(iii) lim x2 ^ + 00 (iv) lim x2 -> +00 and SO on.


X -> + 00 X -► -00

(v) lim 00 or, lim e ^ 00


X ^ 00 X -00

(vi) lim e” ^ 0 or, lim


X -> 00 X^ - 00

(vii) lim 0, if I I < 1 (viii) lim —> oo, if a > 1


X 00 X -> 00

(ix) lim logfl X - 00 and lim log^ x oo,wherea> 1


X ^ 0+ X -> 00

(x) lim loga x 00


and lim log^ x ^ - oo, if 0 < fl <1.
x^0+ X -> 00
16.30 APPLIED MATHEMATICS-XI

We use these results to evaluate limits at infinity. Following algorithm may be used to evaluate
algebraic limits at infinity.
ALGORITHM

fix)
Write down the given expression in the form of a rational function, i.e. if it is not so.
g(x) '
Ifk is the highest power ofx in numerator and denominator both, then divide each term in
numerator and denominator bi/ x^.
Use the results lim — = 0and lim c = c, where «>0.
X -> CO x” a: oo

ow
Following examples will illustrate the above algorithm.
ILLUSTRATIVE EXAMPLES

e
2
ax +bx + c

re
EXAMPLII1 Evaluate: lim —= .
.T ^ ® dx^ + f

Frl
F
SOLUTION Here the expression assumes the form —. We notice that the highest power of x in
CO
ou
or
both the numerator and denominator is 2. So we divide each term in both the numerator and
denominator by x^.
kfs
b c
a +
ax^ + bx + c /7 + 0 + 0
oo
a
lim Um
® dx^ + ex + f / rf + 0 + 0 d
rf + -X + ^2
Y

x^
B

5.V - 6
re

win I Evaluate: lim


X->00
^4? + 9
oYu
ad

SOLUTION We have,
5.V-6 5 -b/x
lim lim
[Dividing each term in and D’’ by x]
d

.T CO
1/4? + 9 .r 00
^4 + 9/.v^
in
Re

5-0 _ 5
74 + 0 “ 2
F

EX.Wll’LE 3 Evaluate: lim ■1^ -1 +


^2x^-1
X -> CO 4.T+ 3

SOLUTION Dividing each term in the numerator and denominator by .x, we get

lim
-1 +
^l2x^-l _ lim +72-1/-V^ V3+V2
X ^ 00 4.V + 3 X->GO 4+ 3/x 4

i;XAViri.H4 Evaluate: lim


X->co
X
x + c - Vx).
SOLUTION The given expression is of the form 00 - oo. So we first write it in the rational form
fix)
. So that it reduces to either - form or — form.
gix) 0 00
LIMITS 16.31

^/x● ^X + C - yfx ■ ■ ^X + C +-Jx ■


sr
lim -Jx ■ -Jx + c - Vx ● =
—> 00 X
lim
-> «

● ^X + C + ^/x

lim
yfx (x + c-x)
X -> <x>
yjx+ C + yfx
c -Jx form —
cc
lim
X —» oo
yJx + C + yfx ccj

C
lim [Diving and D’^ by yfx]

w
AT -> 00
1 + - +1
V X

c c

lo
TTTo+i 2

e
●jx^ + X + l - yjx"^ + 1

re
nxAMPLE 5 Evaluate: lim
a:-»oo V

rF
F
SOLUTION Here the expression assumes the form 00 _ 00 as X -> 00. So, we first reduce it to the
fix)

r
rational form
gix) fo
u
● yjx^ + a: + 1
ks
-
Yo
lim
■^x^ + a: + 1 -yj^+ 1 lim 4
oo

. ^x'^ + .T + 1 + -Jx^ +1
x->«> x-*«>
B

.v^ + X + 1 - -1
e

lim
ur

X->oo
^x^ + a: + 1 + yjx'^ + 1
ad
Yo

X
lim
X -» oo
yjx^ + a: + 1 + ^x'^ + 1
d

1
Re

[Dividing and D'^ by x]


in

lim
1
2 AV ^ ^ .X 2
F

V ^ X^
.
1 1

1+1 2

1 + 2+ 3+...+M
[●.XAVITLE f' Evaluate: lim 2
« ->CO n

SOLUTION We have.
1 + 2+3 + ...+n 1 n{n + \) n{n + l)'
lim lim 1 + 2 +... + n —
n —> 00
2
n -» 00 n 2 2
n

lim
« -> 00
Ifi
2l + iui
nj 2
n!
EXAMPLE? Evaluate: lim
n ^ CO (n + l)!-«!
16.32 APPLIED MATHEMATICS-XI

SOLUTION We have,
«! ill 1
lim lim lim lim 1 = 0
O —> CO (« + !)!-??! H CO {n +1) n! -n! H —> CO tt + 1-1 n ^ as n

ax + b
NMPU Let fix) = lim fix) =2 and lim fix) =1 prove thatf(-2) = 0.
x +\ ' x-^0 X-*CO

SOLUTION We have.

w
ax + b b
lim fix) = 2 => lim = 2 => - = 2=>b=2
x->0 x->0 a: +1 1

It is also given that


b

e
a + -
ax + b fl + 0
lim /(x)=l => lim ^=1 =>

ro
= 1 => lim = 1 => a=l.

re
a: oo X -* as X + 1 .r —* as 1 1 + 0
1 +
a:

F
ax + b
Substituting the values of a and b in fix) = we obtain

Fl
x+l '

fix) =
x + 2
fi-2) =
-2 + 2

u = 0.

sr
x + l -2 + 1

ko
o
Evaluate: lim
ix + + ix + 2)^° +.... + (x + 100) 10 of
X -> as x^^ +10 10
o
SOLUTION We have.
Y
erB

lim
(x + l)^° +(x + 2) 10 + .... + (x + 100)
10

X —» 00
x'O+iQlO
uY

10
2x10 10
1.1 + 1 + + ... + 1 +
100

lim xj a: J
ad
do

10
X -> 00
rioA
1 +
\ X J
in

1 + 1 +... +1 (100 - times) 100


Re

= 100
1+0 1
F

EXERCISE 16.6
Evaluate the following limits:
lim
(3x-l) (4x-2) 3x^ -4.v^ + 6x-l
● 9
X -> CO (x + S) (x-1) 2X-’ + x^ -5x + 7

5x^ -6
3. lim 4. lim yj^ + cx-x
X -> «
yj9 + 4x^ X -> as

5. lim ^x + 1 - 4x
X ^ CO
6.
X^co
lim -^x^Ttx -X
2
X n
7. lim 8. lim
X -*as
yj4x^ +1-1 n-^as l + 2+ 3 + ... + n
LIMITS 16.33

3a: ^ + 4a: ^ +a^ +


9. lim _ 10. lim
x-^<x> 5x ^ + 6x X—>00
yjx^ + c^ + ^jx^~+
(n +2)!+ (?? + !)!
11. lim 12. lim X ● Jx^ + 1 - Jx^ -1 ●
?l -> CO (n + 2)!-(fi + l)! j: —> CO

1^ + 2^ +...
13. a:->oo
lim \JxTT-^fx}^ JjTPi 11.
n
lim
CO n
3

1 2 3 n-1 1^ + 2^ +...+n^
15. lim - - + -^ + ^ +... + 16. lim
«-»co
2 2 2 4
n n n H ->00 n

+2^ +.. . + n
3
17. lim 18. lim Jx ■ yjx + 1 -yfx ■

w
n^co
(n-lf X—*<X)

1 1 1 1
19. lim

Flo
- + ^ + ^ + ...+
n —>« 3 3^ 3^ 3"

a:^ + 7x^ +A6x + a

ee
20. lim , where a is a non-zero real number.
a:^ + 6

Fr
X“> oo

21. fix) = lim /{A:)=land lim /(x) =1, then prove that/(-2) =/(2) =1.
.r'^+lx^O x^oo for
ur
22. Show that lim {Jx^ + x + l~x) ^ lim {Jx^ +1 - x)
s
X -»co ~ V
k
Yo
oo

ANSWERS
eB

3 5 c 7 1 3
1. 12 2. - 3. - 4. - 5. 0 6. r j. 8. 2
2 2 2 2 2 5
1 1 1 1 1 1
r

10. 1 ’■ 1 12. 1 13. - 14. - 15. — 16. - 17. - 18. -


ou
ad

2 3 2 4 4 2
1
20. 1
Y

19. -
2
Re
nd

16.7 EVALUATION OF EXPONENTIAL AND LOGARITHMIC LIMITS

Sometimes, following expansions are useful in evaluating limits. Students are advised to learn
Fi

these expansions.
1. (1 + x)" = 1 + nx + njn-1) ^2 ^ »(»-l) jn-2) 3 X +...
2! 3!

, . . x^ x^ x'^ x^
2 3 4 5

3, log^(l -x) = -X- —


2 3 4 5
2 3 4
r . X X"^ X
4. e-^ =l+x + — +
2! 3!'^4!'"
x2 X
5. = 1 -x +
2! 3! 4!‘“
16.34 APPLIED MATHEMATICS-XI

6. = 1 + X (loge rt) + ^
2!
(log,, +...

7, sin X = X -
3! 5! 7 !

8, cos X = 1 -
2! 4! 6!

x3
9. tan X = X + +...
3 15

ow
-1 loge (1 + X)
THEOREM Prove that: (i) lim = log,, a. (ii) lim = 1.
x^O X X 0 X

l‘RQQF (i) Using expansion of , we obtain


.2

e
Y 0
1 + X logg + '2! (logf fl) +...-1

re
lim
-1

Fl
lim

F
x^O X x^O X
ur
n^' -I X 2

r
lim = x-»0
lim ■^Iog,.fl + — (log,, fl) +...> = log,, rt.
x->0 X 2! fo
ks
(ii) Using expansion of log (1 + x), we obtain
Yo
X
oo

x2
X -

lim
log (1 + X) lim 2 3 = Hm 1 - — + ... = 1.
eB

x->0 X x-»0 X x-»0 2 3

COROLLARY Puttinga=e in (i) we get


ur

-1
lim = log,. ^=1.
ad
Yo

x->0 X

ILLUSTRATIVE EXAMPLES
d
Re
in

EXAMPLE 1 Evaluate the following limits:


n^-b^ 2^ -1
F

(i) lim (ii) lim


x-)-0 X
b yjlTx -1
SOLUTION (i) We have,
a^ -b '^ {a^-l)-{b^-D -1 b’^ -1 fa\
lim = lim = lim - lim
x^O
= log fl - log b = log - .
x-»0 X x^O X x-^0 X X u J

a
b^ ■ -U
-b^ 1.0 1 1
A LITER lim lim = xlog r =log 7
x-*0 X x-»0 X bj bJ

(ii) We have.
2^ -1
lim
0 ^1 + x-l
LIMITS 16.35

- lim
2^-1 Ql+x+l) lim
2^ -1
X ■ + X +1 ●
x-0 + X - x-0 X

2^-1
= lim X lim (jm + l)=(log2)2 = 21og2.
a--»0 -V ar^O

EXAMPLE 2 Evaluate the following limits:


+ 3“^ -2 32- -2
3x
32- -2
3a:

(i) lim (ii) lim (iii) lim 3a:


1^0 ,t2 ;r->0 X x^O 2 -1

ow
SOLUTION (i) We have,
3-1^ + 3- ^ - 2
lim
ar ^ 0

e
0

re
32-^- -2x 3^+1 3"^ -1 1 ri
lim lim X
= (log 3)2 X = (log 3)2
x^O 3^xx2

Frl a--»-0 X 3^ 3;

F
(ii) We have.
ou
3a:

or
32- -2
lim
x->0 X
kfs
32a -1 23a -1
oo
lim ■
a^O X X
y V
Y
B

f o2a f^3x
3 -1 2 1 1 (9
- lim X 2 - lim -x3 =21og3-31og2 = log9-log8=log V -o y
3x
re

a^O 2.r a->0


oYu

(iii) We have.
ad

32- -1
lim
3a
d

a^O 2 -1
in

^ -,2a
32-
Re

3 -1 -1
X 2.a lim
2a 2 a-»0 2.V
2 f log 3 'j _ log 3^ _ log 9
F

lim
3 I log 2 J log 2^ logs
— X

a^O f 2^^ -1 3
lim
23a -1
X 3a
3a 3a

EXAMPLE 3 Evaluate the follozving limits:


-1 e^ -e-^
(i) lim (ii) lim —
a->0 a a->0 a

a-1 e^+e-^-2
(iii) lim (iv) lim 2
a 1 loge a a ^ 0 a

SOLUTION (i) Putting -a = y, we obtain


-1 y-1 y_i
= lim - lim - = -l
a^O a y^o -y y->0 IJ
16.36 APPLIED MATHEMATICS-XI

(ii) We have,
{f X \ ( \ f
-1 e~^ -1 e^-1 -1
lim - lim - lim + lim - =1+1=2
X x->0 X X x->0 AT .Y 0 X
y \ y V
f

(iii) We have,
x-1 1 + h-l h 1
lim , lim = lim 1=1
1 log^ X h^O lo^g{l+h) h^O loge(l+/i) lim log (1 + h) 1
h^O

x-1 x-l 1 1
\'iIER lim lim = lim -=1
Y 1 lOgj; X Y-»l loge {1 +(a:-1)} 1 logf (1+ (x-l)} 1

ow
x-l

(iv) We have.

lim
x2

e
re
= lim
e^^ + l-2e^

Fl = lin.fl!
-1
V
= lim ( — -1
n2

X Urn (l^)x(?‘^=l.

F
Y->0 x^ Y->0 X Y->0 X Y->0
ur
r
LXAMPLC 1
Evaluate the following limits:
(i) lim log (5 + x)-log(5-x) (ii) lim
fo
log X-log 5
ks
Y-»0 X Y ^5 X -5
Yo
SOLUTION (i) We have.
oo

log (5 + x) - log (5 - x)
eB

lim
Y-> 0 X

-logisfl-^5)
ur

log j 5 1 + 5j
ad

lim
Yo

Y->0 X
d

X
log 5 + log 1 + log 5 + log 1 +
Re

5)
in

lim
Y->0 X
F

f ( X
log 1 + - - log 1 - -
lim 5y 5
Y-»0 X

( x\ ( X
1 log 1 + ^
5J - lim
log 1-^5
lim - X 1 i 1 _ 2
-x/5
X
Y^O 5 x/5 Y^O (-5) 5 ^5 " 5
(5 + h
\
h^
log log 1 +
(ii) lim
log X - log 5 lim log (5 + /?) - log 5 = lim I 5 5 1 1
- = lim ^x-
x-*5 X -5 h->0 h /i^O h h-^0 h 5 5
5
LIMITS 16.37

a:
log log 1 + -1

--! I'tHR lim


log X - log 5 lim li - lim
5
X-5 a:-5 .v^5 X -5

x-S'i
log 1 +
lim 5 J 1 1
X — = 1 X — =
1

fx-5''

w
x->5 5 5 5
5 .

LXAMPLE5 Evaluate: lim


logx~l
x->e x-e

e
SOLUTION We have,

e
log a: -1 log {e + h) -1 log {e + h) - log e

or
lim lim = lim [●.● log e = 1]

r
x-^c x-e h^O e + h-e h->Q h

F
h^ h
log 1 + log 1 +
eJ 1
lim

oF— = lim

ul
X -

h->0 h h^O h e

rs
1 1 1
=1X-=- lim
log(l + ^) = 1

ko
e e x-*0 x

fx\

■ALITER lim
log a: -1 lim
log X - log e lim
log - of
o
Y
x^e X-e x-*e X-e x->e x-e
B

X x-e']
log 1 + -1 log 14-
Y

e e 1 1
lim lim Z=ixl
er

x-*e x-e x-*e x-e e e


e

K e )
u

EXERCISE 16.7
od
ad

Evaluate the following limits:


5^-1
in

]. lim , 2. lim
log (1 -H a:)
x^O + X-2 3^-1
Re

a^+a-^-2
F

3. lim 4. lim
AT->0 x^ x-»o

a^ + h^-2 9^ - 2.6^ + 4-’'


5. lim 6. lim
x x^O x^
8--^-4^-2^+1 ^X _^HAT
7. lim 8. lm\
2
x-^0 x X

a^ + -3 x-2
9. lim 10. lim
A-->0 a:
x^l logfl(Ar-l)
S-'" + 3^ 2^ - 3
11. lim 12. lim (a^/''-1}a:
AT X —> 00

log X - log a ^X ^^X _^X _ ^x


13. lim 14. lim
x^a X-a x-^0 X
16.38 APPLIED MATHEMATICS-XI

15. lim
log (a + x) - log {a - x) 16. lim
log {2 + x) + log 05
X .T->0 X

log (fl + a:) - log a log(3 + A:)-log(3-x)


17. lim 18. lim
X x-^0 a:

8^ - 2^
19. lim 20. lim
A--»0 X a: -> 0 log (1 + x)
x+2
.5 -.2
21. lim — 22. lim -
a:->5 a:-5 .T^O ,T

ANSWERS

1 m log a
1. 4 log 5 2. 3. (loge nf 4.

w
loge 3 n log b
r,log-3f a

5. log(nlj) 6. 7. (log 4) (log 2)


“«i?j

Flo
V ^ J

9. log {abc) 10. logfl n. log 30 12. logfl

e
ah')

re
15.2
1 1
13.
14. log [cd
^ 16.
2

rF
a a

1
17. -
-f 19. log 4 20. -
2
ur
a

21. 22.
fo
MULTIPLE CHOICE QUESTIONS (MCQs)
ks
Yo

Mark the correct alternative in each of the following:


oo

l^+2^ + 3^+.. . + n 2
1. lim is equal to
B

H —> 00
3
n
re

(a) 1 (b) 1/2 (c) 1/3 (d) 0


u

1 2
ad

n
2. lim + +... +
is equal to
Yo

H 00 l-«2
(a) 0 (b) -1/2 (c) 1/2 (d) none of these
nd
Re

,. x - 3 . ,
3. lim , is equal to
x^3|a:-3| ^
Fi

(a) 1 (b) -1 (c) 0 (d) does not exist

4. lim is equal to
x-*a X - a

1- 1
(a) (b) nd (c) na (d) 1
-1
5. lim is equal to
.r oo 2x + l

(a) 1 (b) 0 (c) -1 (d) 1/2


X + x'^ + x^ +... + a:" -n n

6. If lim = 5050, then « equals


x-*l a: -1

(a) 10 (b) 100 (c) 150 (d) none of these


LIMITS 16.39

+ (1 + ,
7. The value of lim , IS
I ^ 00

(a) -1 (b) 1 (c) 2 (d) none of these

lim VTT7-1 is equal to


8. x^O X

1
(b) 2 (C) 0 (d) 1
(a)^
'P
2
- <7X + a: + aa: + .Y
9. The value of lim ●/ IS
.r^O + X -

(a) a (b) (c) - a (d) --Ja


n\
10. The value of lim IS

w
X-¥co (n+ !)!-»!'
(a) 0 (b) 1 (c) -1 (d) none of these
n. The value of

(a) 0
n->x
lim
(j7 + 2)! + (;i +1)! .
(n+2)!-{f7 + l)!
(b) -1
, IS

F lo
(c) 1 (d) none of these

e
Fre
12. The value of lim
(y + 1)^°+(a: + 2)
10
+ ...+(^ + 100)^*^ / IS.
X -> oo
for
10
(a) 10 (b) 100 (c) 10 (d) none of these
r
13. The value of lim
[l + 2+ 3 + .,.+?i fi
You

/ *s
‘ ti + 2
ks

H -> 00 2
o
eBo

(a) 1/2 (b) 1 (c) -1 (d) -1/2


14. lim [.Y -1], where [●] is the greatest integer function, is equal to
Y->1

(a) 1 (b) 2 (c) 0 (d) does not exist


our
ad

1-vl
15. lim —is equal to
Y-^0 -Y

(a) 1 (b) -1 (c) 0 (d) does not exist


dY
Re

16. The value of lim


(1+Y)"-1 is
^
Fin

X —> 0 X

(a) n (b) 1 (c) -n (d) 0


jn
1 .
17. The value of lim - IS
1 y"-1
2
ni in m
(a) 1 (b)
n
(c) —
n
(d) ^n

18. The value of lim


(V^-1)(2.t-3) / ,IS
x-^1 2y^ + Y - 3

(a) (b) (c) 1 (d) none of these


10 10

ANSWERS

1. (c) 2. (b) 3. (d) 4. (b) 5. (d) 6. (b) 7. (b) 8. (a)


16.40 APPLIED MATHEMATICS-X!

9. (d) 10. (a) 11. (c) 12. (b) 11. (d) 14. (d) 15. (d) 16. (a)
17. (b) 18. (b)
FILL IN THE BLANKS TYPES QUESTIONS (FBQs)
1. lim ~=,
3- M

2. lim is equal to
1+2+ 3+...+n
3. lim
H -> 00 K^+100

ow
4. lim is equal to
x^l x-1

5. Let f{x) = x^-l,0<x<2 , the quadratic equation whose roots are lim f(x) and
2x+3,2<x<3 x->2~

e
lim + fix), is

re
x^2
X>1

rFl
F
<-■ If/W= ^2' X<1 , then lim f{x) =

4?^ -X

r
7. lim is equal to
ou
X->oo V
fo
ks
3^^^ - 3 .
8. The value of lim IS
x-*2 3^-9
oo

ANSWERS
Y
eB

m 1
1. 1 2. — 3. - 4. 1 5. a:^-12;c+35=0 6. 1 7. 0 8. -
2 6
r
ou

VERY SHORT ANSWER QUESTIONS (VSAQs)


ad
Y

Answer each of the following questions in one word or one sentence or as per exact requirement of the
question:
d

1- Write the value of lim [:r].


Re
in

2. Write the value of lim [a:].


F

x^O'^

1 Write the value of lim x-[x].


4. Write the value of ’'lim^ ^ ^ ^ ^
x^2 x-2

5. Write the value of lim (3a + -^9x^ -x).


A —> - oo

6. Write the value of lim


«! + (n + l)!
n-*oo (« + !)! + («+ 2)!
1 + 2+ 3 + ...+n
7. Write the value of lim
n —> 00
2
n

ANSWERS

1
1. -1 2. 1 3. 1 4. Does not exist 3. 6. 0 7.1
6 2
CHAPTER LZ
CONTINUITY

17.1 INTUTIVE NOTION OF CONTINUITY

Intutively a function is continuous in its domain if its graph is a curve without breaks or jumps

low
throughout its domain and a function is continuous at a point in its domain if its graph does not
have breaks or jumps in the immediate neighbourhood of the point. Consider the graph of a
function / (x) shown in Fig. 17.1. It is evident from the graph that/(a:) is not defined atx=a.
Consequently, there is hole in the curve y =/ (x) and so / (a:) is not continuous at a: = fl. We also

ee
obser\'e that L=R i.e. lim / (a:) = lim + / (a:) and so lim / (a:) exists. Thus, the continuity

F
Fr
x—*a x-*a x-*a

of / (a:) at a: = fl is destroyed, if lim / (a:) exists but/ (x) is not defined at a: = fl.
x-*a

for
ur
Y
k s
Yo
oo

T
eB

i ,i _ 1 I
r
ou

i
ad

o a X
Y

HOLE :f(a) is not defined, lim/x) exists


X-* a
nd

Fig. 17,1
Re

Let us now consider the function / whose graph is shown in Fig. 17.2. Clearly, L= R i.e.
Fi

lim / (a:) = lim / (x). Consequently lim f{x) exists. But, there is hole in the curve because
x-^a~ x-^a* x->a
Y
1

m
I

.......
I
I

I
I
L I I

+
- o -Xi
a
I

HOLE : fia) is defined, lim f{x) exists and not equal to f{a)
X->fl

Fig. 17.2
17.2 APPLIED MATHEMATICS-XI

lim f{x) is not equal to/(fl).So,/(a:) becomes discontinuousatA: = flif, lim /(x) exists but it is
x-¥a x—>a

not equal to the value of / at a: = rt.

In Fig. 17.3, we observe that L R i.e. lim /(a:)^* lim + f{x).So, lim /(a:) does not exist.
x-*a a: —

Also, f (x) is not continuous at x =a. Thus, the continuity of / at x = fl is also destroyed if
lim / (x) does not exist. This happens due to the jump in the values of / (x) as x crosses' a'.
●V -> n

1 ■
y

w
y-m
R

F lo
L

ee
0 X

Fr
a

JUMP: Iim/(x) is not the same as lim /{.r)


X-* a' x—> a"*"
for
Fig. 17.3
ur
The continuity of a function / is also destroyed if either of the two limits lim /(x) and
s
ok
Yo

X -> a
o

lim / (x) or both tend to + oo or - co and / («) is finite as is evident from in Fig. 17.4.
eB

x-*a
r

Y
ad
ou

Yi
Y

y=Ax)
Re
nd

.TJ
Fi

O X'
O a
a X

f{x) is defined atx = a, lim f{x) = +<x> and


f{x) is defined at x = a, lim f{x) = + oo lim f{x) = + 00 X-* a
x-> a
a:-> a*
Fig. 17.4

It follows from the above discussion that a function/(x) can be continuous at a point x =a iff

(i) f{a) is defined. (ii) lim /(.x) exists and. (iii) lim /(x) =/{a).
x-*a

This is also evident from Fig. 17.5


Thus, we define continuity of a function at a point as follows.
CONTINUITY 17.3

y y=Ax)

-M

L = R
i.

O X'

^ f(a) exists and is equal to f{a)


Function f(x) is continuous at a point x = a
Fig. 17.5
17.2 CONTINUITY AT A POINT

w
DEFINITION lim fix) =/(«)●
A function f{x) is said to be continuous at a point x = a of its domain, iff x->a
Thus,
{f{x) is continuous at x = <j) c=> lim / (.r) =f (a) o
X -> <7

F lo
If fix) is not continuous at a point x = a, then it is said to be discontinuous at a: = a
lim
x ->a
/ (a:) = lim
X->fl
fix)=f{a)

e
Fre
If lim / (x) = lim f{x) ^ f(a) , then the discontinuity IS
i known as the removable
X X fl'*’ for
discontinuity, because /(a:) can be made continuous by redefining it at point a: = fl in such a way
that/(fl) = lim f{x) .
r
x^a
You
oks

If lim /(a:)?^ lim f {x), then f{x) is said to have a discontinuity offirst kind.
eBo

X ^ a X -> a

A function f{x) is said to have a discontinuity of the second kind at x = <7 iff
lim / (x) or, lim /(x) or, both do not exist.
our
ad

x-*a~ X —*a

A function / (x) is said to be left continuous or continuous from the left at x = fl, iff
(i) lim / (x) exists and. (ii) lim fix) =fia)
dY
Re

X->fl X->fl

A function / (x) is said to be right continuous or continuous from the right at x = a, iff
Fin

(i) lim / (x) exists and. (ii) lim / (x) = / (rt)


x-^a^ x^a'*'
It follows from the above definitions that
/ (x) is continuous at x = fl iff it is both left as well as right continuous at x = a.
REMARK A function f ix) fails to be continuous at X = a for any of the following reasons.
(i) lim /(x) exists but it is not equal to f ia). (ii) lim / (x) does not exist.
x-^a X -»a

This happens if either lim fix) does not exist or, lim f ix) does not exist or both \im ^ f ix) and
X—>a
x^a

lim / (x) exist but are not equal.


x-*a

(iii) fis not defined at x = a i.e. f id) does not exist.


17.4 APPLIED MATHEMATICS-Xl

17.3 ALGEBRA OF CONTINUOUS FUNCTIONS

Regarding the continuity of the sum, difference, product and quotient of functions, we have the
following theorems.
I HEOREM 1
Let f and g be two real functions, continuous atx = a. Let a be a real number. Then,
(i) f + g is continuous at X = a. (ii) f -gis continuous at x = a.
(iii) a / is continuous atx = a. (iv) fg is continuous atx = a.
(v) f^ is continuous atx = a, provided that f (a) ^ 0.
f
(Vi) is continuous atx=a, provided that g (a) ^ 0.
S

iMU)pi Left as an exercise.

THEOREM 2 Let f and g be real functions such that fog is defined. Ifg is continuous atx = a and f is

w
continuous at g (a), show that fog is continuous at X = a.
i’ROOF Left as an exercise.

F lo
ILLUSTRATIVE EXAMPLES

ee
Tifpc I ON TESTING CONTINUITY OF A FUNCTION AT A POINT WHEN THE FUNCTION HAS SAME

Fr
DEFINITION ON BOTH SIDES OF THE GIVEN POINT

for -i ; a: 0
EXAMPLE I
Test the continuity of the following function at the origin: f {x) = { x
ur
^ ; a:=0
SOLUTION We observe that:
s
ok
Yo

(LHLatx = 0) = lim f {x) - /j->0


lim /(0-/i) = lim f{~h)
x-»0
o
eB

lim
-/ll lim
h
lim -1 = -1
r

h->0 -h h^O -h h-*0


ou
ad

and. (RHLatx = 0) = lim / (x) - lim / (0 + h) - lim / (/i)


Y

/i->0 ?i->0
Re
nd

h
Fi

lim lim lim 1=1


h^O h h->0 h /i-»0

Thus, we have lim / (x) lim + / (x).


x~*0

Hence,/(x) is not continuous at the origin.


ALITER We have.
^=1 ;x>0
X

—t ; x^Q -X X, x>0
fix) = { X - r, or, /(X) = = -1 ; X <0 X
^ ; x =0 X -X, x<0
1 ; X =0
CONTINUITY 17.5

(LHLatx = 0) = lim /(x) = lim -1= -1 fix) = -1 for a: < 0 and x -> 0
x^O x->0 means that x < 0 such that x 0

(RHLatx = 0) = lim fix) = lim 1=1 /(x) =1 for X > 0 andx 0


x^O* ar->0 means that x > 0 such that x 0

lim fix) lim + fix).

w
x-)-0 x-^'O

Hence, fix) is not continuous at the origin.


^ -1 , when X 0
LXAMI’Lh;
Show that the function fix) given by fix) = ■ i is disco7itmuous

e
0 / when X = 0

re
o
flf X = 0.

r
SOLUTION We observe that:

F
{LHLatx = 0)= lim /(x) = lim /(0-/i)=lim fi-h)
oF
ul
x-^0 h^Q h^O

g-l//^_l 0-1 1

sr
— lim —,,,—^ = lim
1
= -l lim
1//J
= 0
ft_>0 1 h-*0
+ 1
0 + 1 h-*0 e

o
k
el/h
and. (RHLatx = 0)= lim fix) = lim fiO + h) = lim fih)
h->0 h->0
of
o
Y
gl//«_l l/h
1 -1/e 1-0
lim lim = 1
rB

e 1/A + 1
1/A 1 + 0
A ^0 A^O 1 + 1/e
eY

(LHLatx = 0) 7^ (RHLatx = 0)
So, / (x) is not continuous at x = 0 and has a discontinuity of first kind at x = 0.
u

Iiipr 1! ON TESTING CONTINUITY OF A FUNCTION AT A POINT WHEN THE FUNCTION HAS DIFFERENT
d
o
ad

DEFINITIONS ON BOTH SIDES OF THE GIVEN POINT

Let a function fix) be defined as


in

<l)(x) ; if x<fl

/w={
(j)(x) ; if X < rt (j){x) ; if x<fl
or, /(x)=«^ k
Re

or, fix) = ; if X =
vj/ (x) ; if X > y (x) ; if X > i2
F

v|/ (x) ; if X > fl

To test the continuity of such functions at x = a, we have to find left hand and right hand limits of
fix) at x=fl. For finding these two limits one can use the method which we have used in
previous examples or we can use the following method:
■: X ^ a <=> X < and x
{LHLatx = a)= lim fix)= lim (j)(x) lim /(x) = lim (j)(x) [v/(x) = <l»(x) for x <a]
x-*a
X -*a x-*a
x-*a

Now, lim (j)(x) can be calculated by various methods of evaluating limits as discussed in the
X -> fl

chapter on limits.
Similarly, we have
+
X -> a <=>x>a&x->fl
(RHLatx=a)= lim + /(x)= lim \}/(x) lim / (x) = lim \j/ (x) [: fix) = y (x) for x > a]
X ->a X-¥tt
17.6 APPLIED MATHEMATICS-XI

Now, lim V|/(a:) can be calculated by various methods of evaluating limits.

EXAMPLE 3 Discuss the continuity of the functionf{x) at x =1/2, zvhere


1/2-x ; 0<x<l/2
m = 1 ; x = l/2
312-X ; l/2<x<l

SOLUTION We observe that:

(LHLatY=l/2) lim f{x)= lim (1/2-y) — - .Y for 0 < Y < -


y^1/2“ .x^l/2 2

= 1/2-1/2 = 0 [Using direct substitution method]

w
and. (RHLatY=l/2) = lim /(y) = lim_(3/2-Y) - Y for - <Y<1
2
,y->1/2^ y->1/2

= 3/2-1/2 = 1 [Using direct substitution method]

Flo
Clearly, lim /(y) lim /(y)
x->l/2" y^1/2

e
re
Hence,/(y) is not continuous at y =1/2. Clearly,/(Y) has discontinuity of first kind at y =1/2.

F
2-y, y<2
EXAMPLE 4 Discuss the continuity of the function f(x) given by f{x) = 2 + y, y>2
at X = 2.
ur
r
SOLUTION We observe that:
fo
(LHLatY = 2) = lim /(y) = lim (2-y) [v /(Y)=2-YforY<2)
ks
y->2“ x^2
Yo
oo

= 2-2 = 0

and. (RHLatY=2) = lim /(y) = lim (2 + y) [●●● /(y) = 2 + y = for y > 2]


B

y^2'^ ^^2
re

=2+2=4

lim /(.y) ^ lim + /(y).


u
ad

y^2" x^2
Yo

Hence, f{x) is not continuous at y = 2.


f 5y-4 , wheiz 0 < Y < 1 .
EXAMPLES Show thatf{x) = ■ ^^3
is continuous atx = \.
d

- 3y / when 1 <x <2


Re
in

SOLUTION We have,
F

(LHLatY=l) = lim f{x) = lim 5y-4 [v /(y) =5y - 4, when Y <1]


X -> r x^l

= 5x1-4 =1

(RHLat Y=l) = lim fix) = x-»l


+
lim 4y^ - 3y [v /(y)=4y3-3y,y>1]
x-^1

= 4(1)3 3(1) = 1
and, /(I) =5x1-4 =1 fix) =5y - 4, where y < 1]
lim fix) = /(I) = lim+ fix) .
X-+1 x^l

So, fix) is continuous at y = 1.


EXAMPLE 6 Show that the function f ix) = 2x-\x\is contmuous at x =0.
SOLUTION We have.
2y - Y , if Y > 0
fix) = 2y-|y| =
2y -(-y) , if Y < 0
CONTINUITY 17.7

X , if a: > 0
/w = 3x , if X <0
Now,
(LHL at a: = 0) = lim / (x) = lim 3.t = 3 x 0 = 0
x->0 x->0

(RHL at X = 0) = lim / (x) = lim X = 0


+

and. /(O) = 0
lim / (x) = lim / (x) = / (0).
x->0

So, / (x) is continuous at x = 0.


EXAMPLE? Discuss the continuitij of the function of given by / (x) =| x -11 +1 x - 21 at x = land
x = 2.

SOLUTION We have,

w
/(X) =lx-l|+|x-2|
-<x-l)-(x-2) , if x<l
^ /(x) = ] (x-l)-(x-2) , ifl<x<2
[ (x-l)+(x-2) , if x>2
F lo
ee
- 2x + 3 , if X < 1

Fr
fix) = 1 , if 1 < X < 2
2x-3 , x>2

Continuit]/ at x = 1:
for
ur
lim fix) = lim (-2x+3)=-2xl + 3=1, lim /(x) = lim 1 =1 and,/(I) =1.
s

->r ,T-»i
ook
Yo

linA f{x)=f (1) = lim + / (x)


eB

x-*r X->1

So,/(x) is continuous at X = 1.
our
ad

Continuity at x = 2:
lim /(x) = lim 1=1, lim +
/(x) = x^2
lim (2x-3) = 2x2-3 =1
x->2~ x^2" x-»2
Y
Re

and. /(2) = 2x 2-3 = 1.


nd

lim fix) = lim fix) =/(2)


Fi

X-»2" x^l^

So, / (x) is continuous at X = 2.


Type 111 ON FINDING THE VALUE(S) OF A CONSTANT GIVEN IN THE DEFINITION OF A FUNCTION WHEN
IT IS CONTINUOUS AT AN INDICATED POINT

A function/(x) is continuous at a point X = <7 iff lim /(x)= lim /(x) =/(«).
X > X > fl"*"
But, lim / (x) = lim / (x) => lim fix) exists.
x-*a x^a-^

Thus,/(x) is continuous at X = iff lim /(x)=/(fl) .

We will use this result in finding unknown quantity in the definition of a function when it is
given to be continuous at a given point.
17.8 APPLIED MATHEMATICS-XI

r.XA.viP! !● Determine the value of kfor which the following function is continuous at x = 3.
x^-9 , x^3
fix) = \ x-3

SOLUTION Since f{x) is continuous at x = 3.


lim f(x) = f{3)
X ^ 3

lim f{x) = k [vf{3)=k]


x^3

x^-9 (X -3){x+ 3)
lim = ^ lim = k => lim {x+ 3) = k => 6 = k
X 3 X — 3 x—> 3 X-3 x-^3

w
Thus, f(x) is continuous at x = 3, if A: = 6.
i:\AMPi I Find the value of the constant Xso that the function given below is continuous at x = -1.

F lo
x^ -2x-3 ,
fix) = x + 1

ee
X x = -l

Fr
SOLUTION Since f{x) is continuous at x = - 1.
x^-l
lim fix) =/(-!)
for
ur
x^-2x-3
Urn = X
[■■■ /(-1)=A]
oks

x-*-l x + 1
Yo

lim
(x-3)(x + l)
= ^ =^> lim (x-3)=X=>-4 = X
o
eB

x-*-l x + 1 x-^-1

So, fix) is continuous at x = -1, if X = - 4.


2x -1 , X <2
our

liXAMPl.Elo Findthevalueof'a'ifthefunctionf(x)definedbyfix) , X = 2 is continuous


ad

= ■ a

atx = 2. x + 1 , x>2
SOLUTION We observe that:
Y

(LHLatx = 2) = lim fix) = lim (2x-1) = 2 x 2 -1 = 3,


Re

x^2~ x->2
nd

(RHL at X = 2) = lim + fix) = lim (x + 1) =2 + 1 = 3,


Fi

x-*2 x^l

and. /(2) = «
Since/(x) is continuous at x = 2. Therefore,
lim fix) = lim fix) = /(2) =>3 = 3 a => a = 3

Thus,/(x)is continuous at x = 2, if a = 3.
log (1 + <7x) - log (1 - l>x) , if X 0
lAAMPLL J !
If the function fix) defined b}/ fix) = ■ X

is continuous at x = 0,find k. k , if X = 0
SOLUTION Since /(x) is continuous at x = 0.
lim fix) = /(O)
17.9
CONTINUITY

=> lim
log (1 + flx-) - log (1 -bx) [v/(0)=/c]
a:-»0 X

lim
(log(l+flj:) log(l
x->0

lim
log (1 + ax) - lim
log(l-b3:)
a: 0 X ar-*0 X

a lim
log (1 ax)
-{-b) lim log (1 -bx) ^ ^
x-»0 ax 0 {-b):c
log (1 + x) =1
^(1)-(_{,)(!) = k Using: lim
X ^ 0 X

ow
a +b = k

Thus, f{x) is continuous atx = 0,if k=a + b.


Sax + b , if a: > 1
, if x-I is continuous at X = l,find

e
11
liXAMPLt 12 If the function f {x) given by f {x) - ●

Fl
re
5ax - 2b , if .t < 1

F
the values of a and b.
ur
SOLUTION We observe that:

(LHLatx=l)= lim f (x) = lim {5ax-2b)=5a-2b or


sf
x-»r x->l
k
Yo
(RHLata: = l) = lim f{x)= lim {Sax + b)=3a + b
oo

+
x-»l

/(I) = 11.
B

and.
Since/(x) is continuous at X = 1.
re

lim f{x)= lim +


/ (x) = f (1)
u

x->r X ^ 1
ad
Yo

5a-2b - Sa + b = 5a-2b =11 and 3a + b=n=>a = 3 and b = 2


LXAMPLE 13 Prove that the greatest integer function [x] is continuous at all points except at integer
d

points.
Re
in

SOLUTION Let f{x) - [x] be the greatest integer function and let k be any integer. Then,
F

k -1 , iffc-l<a:<fc
fix) = [x] = [By definition of [x]]
k , ifk <x <k + 1

Now,

(LHLatx = A:) = lim f{x) = h-*0


lim fik-h) = h-*0
lim [k-h]
x-*k

lim (/c -1) = ^: -1 [v k -1 <k -h <k [k ~h] = k - \]


h-^0

and.
(RHLata: = cf) = lim fix) = /j->0
+
lim fik + h) = h^O
lim [k + h]
x-^k

lim k = k [.■ k<k + h<k + l .-.[k + h] = k]


17.10 APPLIED MATHEMATICS-XI

lim f{x) lim f{x) .


I —»/c

So, f{x) is not continuous atx = k.


Since kisan arbitrary integer. Therefore,/(:r) is not continuous at integer points.
Let a be any real number other than an integer. Then, there exists an integer k such that
k - I <a <k.

Now,

(LHLatA,'=fl) = lim f{x) = lim f{a~h) = lim [a-h]


X -^a /i-»0 h^Q

lim k -1 - k -1 [●.● k -1 <a - h <k -li] = /c -1]

(RHLatX'=rt) - lim f{x) = lim f{a + h)

w
h-^0

k -1 <a + h <k

F lo
= lim [a + h] = lim {k -1) =k - I
/i->0 [a + h] = k-l
and. f{a) =[a] =k-l [v k-l<a<k [fl]=fc-l]

ee
Fr
Thus, lim f(x) = lim f{x) = f{a)
+
.T - X a

So, f{x) is continuous at x = a. Since a is an arbitrary real number, other than an integer.
for
ur
Therefore, f{x) is continuous at all real points except integer points.
s

EXERCISE 17.1
ook
Yo

1. Test the continuity of the following function at the origin:


eB

...
f{x) = \ \x'
^
1 , x =0
our
ad

x^ - x-6
; if 3
2. A function f{x) is defined as f{x) = ■ x-3
Y

5 ; if a: = 3
Re
nd

Show that/(x) is continuous atx = 3.


Fi

x^ - 9
; if a: 3
3. A function f{x) is defined as f{x) = ● _3
6 ; if a: = 3

Show that /(at) is continuous at a: = 3.


(x2-l ; for a: # 1
4. If/(x) = a:-1 . Find whether/(a:) is continuous at a: = 1.
2 ; for a: = 1

a:-|;c1
, when a: 0
5. Show that/(x) =■ 2 is discontinuous at x = 0.
2 , when x = 0
17.11
CONTINUITY

x-a
, when a
is discontinuous atx = n.
6. Show that f{x) = ■ x-a

1 , when x = a

7. Discuss the continuity of the following functions at the indicated point(s):


-1
, if 0 at .r = 0
(i) = log (1 + 2.x)
7 , ifx = 0

.n
1 -X
X9^1
(ii) fix) = neN at x = 1

w
x=l

, for X 7^ 1

Flo
at X = 1
(iii) /(.V) = x-1
2 , for X = 1

e
re
2|xl + x^

F
at x = 0
(iv) fix) = X
0 , X =0
ur
r
8. Show that /(x) = jl2-x
+ x^ , if 0<x<l is discontinuous at x =
fo 1.
, if X >1
ks
Yo

3.V-2 , x<0
at x = 0.
oo

9. Examine the continuity of the function /(x) = x + 1 , X >0


B

Also sketch the graph of this function.


re

X x>0

10. Discuss the continuity of the function /(.v) X = 0 at the point x = 0.


u
ad

x<0
Yo

X , 0<x<l/2
n. Discuss the continuity of the function fix) ● 1/2 X = 1/2 at the point X =1/2.
d

1 -X , l/2<x<l
Re
in

2x -1 x<0
F

atx = 0.
12. Discuss the continuity of fix) = )2x + l x>0

x^-l
XT^l
continuous at x = 1 ?
13. For what value of k is the function fix) - x-1
k x=l

x^ - 3x + 2
, if x^l
14. Determine the value of the constant k so that the function/(x) - x-1
k , if x=l

is continuous at x = 1
15. Determine the value of the constant k so that the function
kx^ , if x<2
is continuous at x = 2.
fix) = if x>2
17.12
APPLIED MATHEMATICS-XI

i
ax+ 5 , x<l
i ● Find the values of a so that the function/(x) = x-\ , if x>l
is continuous dAx = 2.

I”. Prove that the function f{x) = \x\ + lx^ remains discontinuous at x = 0,
k , a: = 0

regardless the choice of k.


x-4
+ a , if x<4
\x-4\
1». Jff{x) = . a +b , iS X = 4 is continuous at ;c=4, find
x-4

w
+b
|a:-4| , if :r>4

2^+2 -16

Flo
45-16 is continuous at jc = 2, findJt.

e
k , ifx = 2

re
20. In each of the following, find the value of the constant k so that the given function is

F
continuous at the indicated point:
fcc + l , if X < 5
ur
r
(i) fix) = at a: =5
3x-5 , if X > 5 fo
x^-25
ks
,Xi^5
Yo
(ii) f .{x) = x-5 at x=5
oo

k , x=5
B

(iii) f{x) =
kx^ ,x>l at x=l
re

4 , x<l

k(x^ + 2), if X <0


u
ad

(iv) f{x) = at a: = 0.
Yo

3x + l , if a:>0

x^ +x^ -16a:+ 20
2
d

(V) fix) = (x~2f


Re

at a: = 2.
in

it ,x = 2
F

21 Find the values of a and b so that the function / given by


1 / if a: < 3
f (x) = ● ax+ b , if 3 < a: <5 is continuous at a: = 3andx = 5.
7 , if:c>5

if 0<a:<1
22. If/(:c) = 2'
. Show that / is continuous at a: = 1.
2x^-3x + - ifl<x^2
2'
23.
Discuss the continuity of the / (jt) at the indicated points:
(i) fix) = \x\+\x-l\ atx = 0,1.

(ii) /(x) = I a:-1|+|x + 1| atx = -l,l.


CONTINUITY 17.13

X -\ X , X*0
24. Prove that/(a:) = X is discontinuous at a: = 0.
2 , x =0

2x^ + k , a X > 0
2.x If/(A:) = , then what should be the value of k so that / (x) is
- 2x^ + k , a X < 0
continuous at X = 0.

For what value of X is the function / (x) =


k{x^^2x) , if X < 0
continuous at x = 0?
4x + l , if X > 0

What about continuity at x = ± 1 ?


27. For what value of k is the following function continuous at x = 2?
2x + 1 ; if X < 2
fix) = k ; x =2

w
3x -1 ; X > 2

28. Find the relationship between 'a' and 'b' so that the function'/' defined by
flx +1 , if X < 3
f{x) = bx + 3 , if X > 3

F lo
is continuous at x = 3.

e
ANSWERS

Fre
1. Discontinuous 4. Continuous for
7. (i) Discontinuous (ii) Discontinuous (iii) Discontinuous (iv) Discontinuous
9. Discontinuous 10. Discontinuous Continuous
r
12. Discontinuous 13. 2 14.-1
You
oks

15. 3/4 16. -2 18.fl = l,b = -l 19. 1/2


eBo

20. (i) Jc=- (ii) cf=10 (iii) cf=4 (iv)fc=l/2 (v)fc=7 2!. fl = 3,b =-8
5

23. (i) Continuous, (ii) Continuous A: is any real number.


ad
our

26. There is no value of X for which it is continuous at x = 0; Continuous.


27. k=5 2R. 3a-3b =2
Re
dY

17.4 CONTINUITY ON AN INTERVAL

CONTINUITY ON AN OPEN INTERVAL A function f (x) is said to be continuous on an open interval (a, b)
Fin

iff it is continuous at every point on the interval (a, b).


CONTINUITY ON A CLOSED INTERVAL A function fix) is said to be continuous on a closed interval [a, b]
iff
(i) f is continuous on the open interval (a, b) (ii) lim f{x)=f{a) and, (iii) lim^/(x) =/(b).
X ^ fl'*' x-^b

In other words, fix) is continuous on [a, b] iff it is continuous on (a, b) and it is continuous at a from the
right and at b from the left.
CONTINUOUS FUNCTION A function f(x) is said to be continuous, if it is continuous at each point of its
domain.

EVERYWHERE CONTINUOUS FUNCTION A function ffx) is said to be everywhere continuous if it is


continuous on the entire real line (- <x>, co).
17.14 APPLIED MATHEMATICS-XI

17.5 PROPERTIES OF CONTINUOUS FUNCTIONS


In this section, we shall learn some properties of continuous functions and prove the continuity
of some standard real functions in their domains.

THEOREM 1 Iff and g are two continuous functions on their common domain D, then
(i) f + g is continuous on D
(ii) f - g is continuous on D
(iii) fg is contmuous on D
(iv) a / is continuous on D, where a is any real number.
(v) — is continuous on D-{.r:g(T) 0)
g
1 .
(vi) — is continuous on D ~{x:f (x) ^ 0)
/

w
PROOF Left as an exercise.

THEOREM 2 The composition of two continuous functions is a continuous function.

Flo
PROOF Left as an exercise.

THEOREM 3 Iff is continuous on its do?nain D, then \f\is also continuous on D.

ee
PROOF Left as an exercise.

Fr
REMARK The converse of the above theorem may not be true. For example, consider the function
1 , if X sZ
fix) = for
ur
if X sR -Z

Let a be an arbitrary integer. Then,


ks
Yo

lim / (t) = lim / (<? - h) - lim -1 = -1 h >0, a - h ^Z ash is very small]


oo

h^O h^O
B

lim f{x) = lim f{a + h) = lim -1 = -1


/j-^0
re

and. f(a) = 1.
ou
ad

lim / (x) = lim / (x) ^ f {a)


X -*a
Y

x-*a

So,/is discontinuous at X = a.
nd
Re

Now, [/|(x) = |/(x) I = 1 for all xeR. So, |/| is a constant function and hence , it is
everywhere continuous.
Fi

THEOREM 4 A constant fimction is everywhere continuous.


PROOF Left as an exercise.

REMARK 1 It is evident from the graph of a constant function that is everyzohere continuous.
THEOREM 5 The identity function is everywhere continuous.
PROOF Left as an exercise.

REMARK 2 The above fact can be easily observed from the graph of the identity function.
THEOREM 6 A polynomial function is everywhere continuous.
PROOF Left as an exercise.

A simple consequence of the above theorem is the following:


COROLLARY Every rational function is continuous at every point in its domain.
THEOREM 7 The modulus function is everywhere continuous.
PROOF Left as an exercise.
17.15
CONTINUITY

THEOREM 8 The exponential function , a>0 is everywhere continuous.


PROOF Left as an exercise.

Since c is an arbitrary real number. Hence, / (.v) = is everywhere continuous. Q.E.D.

COROLLARY is everywhere continuous.


THEOREM 9 The logarithmic function is continuous in its domain.
PROOF Left as an exercise.

ILLUSTRATIVE EXAMPLES

Type 1 ON TESTING THE CONTINUITY OF A FUNCTION IN ITS DOMAIN


.t-4 -V 4

w
EXAMPLE 1 If a function f is defined asf{x) =

Flo
Show that f is everywhere co7ttinuous except at x = 4.

e
SOLUTION We have.

re
x-4\ x?i4

F
f{x) = ■ X - 4 '
ur
0 , X =4

r
-ix-4)
fo
= -1 ; .Y < 4
ks
y-4
Yo

y-4 ~{x-4), y<4


oo

=> f{x) = =1 ; y>4 V 1y-41 = y-4, y>4


y-4
eB

0 ; y=4
ur

When Y <4, we have /(.t) =-l, which, being a constant function, is continuous at each point
ad
Yo

Y < 4.

Also, when y > 4, we have f{x) = 1, which, being a constant function, is continuous at each point
d
Re

Y > 4.
in

Let us consider the point y = 4.


F

We have.

lim fix) = lim -1 = -1, lim fix) = y->4


+
lim 1 = 1 and, /(4) = 0.
y->4 X->4 x-*4

lim fix) lim + fix)


Y-»4 X-¥i

So, fix) is not continuous at y = 4.


Hence, fix) is everywhere continuous, except at y = 4.
Y
, if Y 0
EXAMPLE 2 Discuss the continuity of the function fix) = ■ x

0 , ifY = 0
17.16 APPLIED MATHEMATICS-XI

SOLUTION We have,
-X
= -l , if X < 0
riM X

, if 0 a:, if ;c> 0
f(x)=\ X => fix) = ■ ^=1 , ifx>0 1^
a: -X, if a: < 0
0 , if =0
0 / ifa: = 0

We observe that f{x) is a constant function for all a: < 0 as well as for a: > 0. So, it is continuous for
all a: > 0 and for all a: < 0.

Consider the point a: = 0. At a: = 0, we have

w
(LHLata: = 0) = lim f(x) = lim -1 = -1 and, (RHL at a: = 0) = lim f{x)= lim 1=1
a-»0 1^0

(LHLata: = 0) (RHL at a: = 0)

Flo
So, f{x) is not continuous at a: = 0. Hence, f{x) is continuous at each point, except at x = 0.

e
re
2x-l , ifx<0
"Li. -● Discuss the continuity of the function f(x) given byf{x) = 2x + l , ifx>0

F
SOLUTION When X < 0, we have/(x) = 2x-l.
ur
r
Clearly, /(x) is a polynomial function for x < 0. So, /(x) is continuous for all x < 0.
fo
When X > 0, we have /(x) = 2x +1.
ks
Clearly, /(x) is a polynomial function for x > 0. So, it is continuous for all x > 0.
Yo
oo

Let us now consider the point x = 0. At x = 0, we have


B

lim fix) = lim (2x-l) = -1 and. lim /(x) = lim (2x + l) = 1.


x->0 x->0 X-»0
re

lim fix) * lim /(x)


u
ad

x->0 x^O'
Yo

So,/(x) is not continuous at X = 0. Hence,/(x) is everywhere continuous except at x = 0.


EXAMPLE
Shozv that the functio?i f defined byf(x)= l—x + \x\\is everywhere continuous.
d
Re
in

SOLUTION Letg(x) = l-x + |x| and/i(x) = | x| be two functions defined on R. Then,


F

(hog)ix) = h (^(x)) = (1 -X + 1 x|) = |l -x + l X = fix) for all X € R.


Since (1 - x), being a polynomial function and | x | being a modulus function are continuous on R.
Therefore, g(x) = 1 - x +1 x | is everywhere continuous.
Also, hix) =1 XI is everywhere continuous. Hence, / = hog is everywhere continuous.
MTTER We have.
|1 -x-x|, if X < 0
fix) = |l -x + |x|| = |l-x + x|, ifx>0
f(y^_j(^~'^x) ,iix<0
\ 1 ,ifx>0
For X < 0, we have fix) = 1 - 2x.
Clearly, fix) is a polynomial function. So, fix) is continuous for all x < 0.
For X > 0, fix) = 1, being a constant function, is continuous.
CONTINUITY 17.17

So, X = 0 is the only point of possible discontinuity.


Now,
lim f{x)= lim (l-2x)=l and lim + fix) = 1^0
lim 1=1
x->0 x->0

Thus, lim /(x)=/(0)= lim +


f{x)
x->0 AT->0

So,/(x) is continuous at x = 0.
Hence,/(x) is everywhere discontinuous.
1

ow
UXAMl’LtS Given fix) = . Find the points of discontinuity of the composite function f (/(x)).
x-1
1
SOLUTION We find that lim /(x) = lim /(l-li)= Um - 00

x^r /i-»o h^o-h

e
1
and. lim fix) = lim / (1 + h) = /i lim — ->co

re
/i->0 -> 0 h

lim fix) lim fix)

F
Frl
+
X -*l 1

So, fix) is discontinuous at x =1. Also,/(x) is not defined at x =1.


So, for X 1.
ou x-1

osr
1 1
/(/(^)) =/ x-1
1 -1 2-x
x-1 kf
x-1
Letg(x) =/(/(x)) = .Then,
oo
2-x
2-h-l 1
Y

= lim 1 CO
lim gW = .ltag(2-10=,lta
B

x-f2~ /i-»0 0\h

and. lim + v(x) = lim v(2 + li)= lim ——= lim


re

^ /,^o2-(2 + ;o
uY

x^l h^Q h)

lim six) # lim g(x) or, lim f f ix) ^ lim / / (x)


ad
do

x-»2“ x-»2‘*' x^2~ \ ) x^l^ \


So, fifix)) is discontinuous at x = 2.
in

Hence, fifix)) is discontinuous at x = 1 and x = 2.


Re

Tlipe II ON FINDING THE VALUERS) OF A CONSTANT GIVEN IN THE DEFINITION OF A FUNCTION Y^HEN
F

IT IS CONTINUOUS ON ITS DOMAIN

kx^ , ifx<2 .
EXAMPLE i> Determine the value of the constant k so that the function fix) = ■ ^ , ifx>2
ts

continuous.
SOLUTION When x < 2, we have
fix) = k x^, which being a polynomial function is continuous at each x < 2.
When X > 2, we have
fix) = 3, which being a constant function is continuous at each x > 2.
Let us now consider the point x = 2. At x = 2, we have
(LHLatx = 2)= lim /(x)= x^2
lim cfx^ = 4fc [●.● fix) = kx^ for X < 2]
.t^2"

(RHLatx = 2)= lim + /(x)= lim 3 = 3 [v /(x) = 3 for X > 2]


x->2 x->2
17.18 APPLIED MATHEMATICS-XI

and. /(2) = k{2f = 4k.


As f{x) is continuous in its domain. Therefore, it is also continuous at a- = 2. Consequently, we
have

lim /(x) = /(2) = lim /(x) => 4k = 3^k= —


x^2~ x^2
+ 4

1 , if X < 3
EXAMPLE? // /(x) =^rtx + l; , if 3 < X <5 . Determine the values of a and b so that fix) is
7 , if 5 <x
continuous.

ow
SOLUTION The given function is a constant function for all x < 3 as well as for all x > 5. So, it is
continuous for all x<3 as well as for all x>5. We know that a polynomial function is
continuous. So, the given fimction is continuous for all x 6 (3,5). Thus,/(x) is continuous at each
X e R except possibly at x = 3 and x =5.
At X = 3, we have

e
re
lim /(x) = lim 1=1, lim + /(x) = :t lim ax + b = 3a + b and,/(3)=1
.V -4 3 X —>3 X -> 3

rFl -> 3

F
For /(x) be continuous at x = 3, we must have
lim fix) = lim + /(x) = /(3)

r
.r-> 3" X -*^3
ou
1 = 3a+ b fo ...(i)
ks
At X = 5, we have
lim /(x) = lim ax + b - 5a+ b, lim fix) = x->5
Urn 7 = 7 and, /(5) = 7
oo

x^5~ x->5
Y
eB

For fix) to be continuous at x =5, we must have


lim fix) - lim fix) ~ /(5)
,t^5 X 5+
r
ou

5a+ b =7 ...(ii)
ad
Y

Solving (i) and (ii), we get: a = 3, b = -8.


EXERCISE 17.2
d

^ , X 0
Re
in

1. Discuss the continuity of the function fix) = x

, X = 0
F

2. Find the points of discontinuity, if any, of the following functions:


x''-x^ + 2x-2 . iix^l
(i) M =
4 , ifx=l

x^ -16 , if X 9^ 2
(ii) fix) = x-2
16 . ifx = 2

x-3 , ifx>l
(iii) fix) = x2 3x 13
4 2 "^4 , if X < 1
17.19
CONTINUITY

, if X < - 3
lx| + 3
(iv) fix) =\-2x , if - 3 < X < 3
6x + 2 , if X > 3

-1 if x<l
(V) fix) =
if X >1

2x if X <0
if 0<x<l
(Vi) /(X) = 0
4x if X >1

ow
-2 if X < -1

(viii) fix) = \ 2x if -l<x<l


2 if x>l

3. In the following, determine the value(s) of constant(s) involved in the definition so that the

e
given function is continuous:

re
'kx + 5, ifx<2
(i) fix) - X -1, if X > 2

F
, ifx<3

Frl
2

(ii) fix) = lax + b , if3<x<5


9 , ifx>5
ou
sr
^1 + px-^l-px
(iii) fix) = 2x + l
X
,

,
if -1 < X < 0

if 0 < X < 1
kfo
oo
x-2
Y

5 , if x<2
reB

(iv) fix) = lax + b / if 2<x<10


21 , if x>10
uY

x2 , if 0 <x <1
a
ad

, ifl<x<V2
do

a
4. The function fix) =
-4b
, if -J2- < X < 00
in

x2
Re

is continuous on [0, oo). Find the most suitable values of a and b.


F

x^ + ax +b , 0 < X < 2
5. The function/(x) is defined by/(x) = < 3x + 2 ,2<x<4
2flx + 51? ,4<x<8

If / is continuous on [0,8], find the values of a and b.


2x-l , if x<2
6. Discuss the continuity of the function /(.x)=- ^ , ifx>2‘
2

7. Show that the fimction g (x) = x - [x] is discontinuous at all integral points. Here [x] denotes
the greatest integer function.
8. Find all the points of discontinuity of / defined by / (x) = | x j — | x + 1 j.
1
9. Given the function fix) = . Find the points of discontinuity of the function/(/(x)).
x+2
17.20
APPLIED MATHEMATICS-XI

1
10. Find all point of discontinuity of the function /(f) = , where f -
r+t-2 x-1

ANSWERS
1. Discontinuous at a: = 0
^ (i) x=l (ii) AT = 2 (iii) Nowhere discontinuous (iv) Discontinuous at a: = 3
(v) Discontinuous at X = 1 (vi) Discontinuous at x=l (vii) Everywhere continuous
. (i) k = -2 (ii) a=7/2,b=~17/2 (iii) p=-l/2 (iv) a = 2, b = 1
4. a=-l,b=lov a = l,b=l±y/2 3. a = 3,b = -2
6. Everywhere continuous. 8. No. point of discontinuity
9. Discontinuous at x = -2 and x = -5/2
IP. DiscontinuousX =1/2,1, 2.

MULTIPLE CHOICE QUESTIONS (MCQs)

w
4-x^
1. The function / (x) =
4x -x^

Flo
(a) discontinuous at only one point (b) discontinuous exactly at two points

ee
(c) discontinuous exactly at three points (d) none of these

Fr
|x^ -x( ,x?i0,l
X^ -X
2. If / (x) defined by / (x) = for
ur
1 , X=0 then / (x) is continuous for all
1 ,x = l
k s
Yo
oo

(a) X (b) X except at x = 0


eB

(c) X except at x = 1 (d) X except at x = 0 and x = 1.


f x-4 X <4
+ (t,
r

x-4
ou
ad

3. Let / (x) = a+b , X = 4. Then, / (x) is continuous at x = 4 when


x-4
Y

+ &,
x-4 x>4
Re
nd

(a) a = 0,b=0 (b) a=l,b =1 (c) a = -l,b = l (d) a = l,b=~l


Fi

x"* -5 x^ + 4
(x-l)(x-2)i x^l,2
4. Let / (x) = 6 x=l . Then, / (x) is continuous on the set
12 x =2

(a) R (b) R-{1] (c) R-{2) (d) i?-{l,2}


1/8
2-(256-7 x)
5. The value of /(O) so
that the function / (x) = , X 0 is continuous
(5x+ 32)^/^ -2
everywhere, is given by
(a) -1 (b) 1 (c) 26 (d) none of these
CONTINUITY 17.21

^1+pX-^l-px , -l^x<0
X
6. f(x) = 2x + l
is continuous in the interval [- 1, 1], then p is
0<a:<1
x-2

equal to
(a)-l (b) -1/2 (c) 1/2 (d) 1

2
X a , 0<x<l
7. The function /(x) = a
/ 1 < X < V2 ^ continuous for 0 < x < oo, then the most
2b^-Ab
■/ V2<x<oo
x2
suitable values of a and b are
(a) a = l,b=-l (b) fl = -l,&=l + V2

w
(c) a = -l,b=l (d) none of these
5x-4 , 0<x<l

F lo
8. The value of b for which the function / (x) = is continuous at
4x^ + 3bx , 1 < X < 2
every point of its domain, is

e
(a) -1 (b) 0 (c) 13/3 (d) 1

Fre
x^ + x^ -16X + 20
9. The function / (x) = is not defined for x = 2. In order to make / (x)
for
x-2

continuous at x = 2, / (2) should be defined as


r
(a) 0 (b) 1 (c)2 (d) 3
You

flx^+b , 0<x<l
oks

10. If fix) = 4 , x =l , then the value of (a,b) for which /(x) cannot be
eBo

x+ 3 , 1 <x<2

continuous at x = 1, is
(b) (3,1) (c) (4,0) (d) (5,2)
our

(a) (2,2)
ad

2Vx , 0<x<l
11- The points of discontinuity of the function / (x) 4-2x, l<x<^is(are)
dY

=
Re

2x-7 , I
-<x<4
Fin

(a) X = 1, X = I (b) X = I (c)x = 1,4 (d) X = 0, 4

1(2x2+ 3) , X <1

12. The points of discontinuity of the function fix) = 6-5x , 1 < X < 3 is (are)
x-3
, x>3

(a) X = 1 (b) X = 3 (c) X = 1, 3 (d) none of these


5x-4 ,if 0<x<l
13- The value of a for which the function / (x) = is continuous at
4x2 ^ , if 1 < X < 2
every point of its domain, is
(a) 13/3 (b) 1 (c)0 (d) -1
17.22 APPLIED MATHEMATICS-XI

14. lff{x) =2x and^(x) + 1, then which of the following can be a discontinuous function
six)
ia)f{x) + g{x) (h) f{x)-g{x) {c)f(x)gix) (d)
fix)
1
15. The number of points at which the function f{x) = is not continuous is
x-[x]
(a) 1 (b) 2 (c) 3 (d) none of these
16. The function f{x) =[a:] is continuous at
(a) 4 (b) -2 (c) 1 id) 1.5

ANSWERS

1. (c) 2. (d) 3. (d) 4. (d) 5. (d) 6. (b) 7. (c) 8. (a) 9. (a)

w
10. (d) n. (b) 12. fb) 13. (d) 14. (d) 15. (d) 16. (d)

F lo
FILL IN THE BLANKS TYPE QUESTIONS (FBQs)

, x^ a
1. lff(x)=\

ee
x-a is continuous atx=a, then b =

Fr
b , X =a

ax ^-b,0<x<l
for
2. If the function/(a:) = ● 2 ,x=l is continuous at a: = 1, then a-b =
r
-V +1 , 1 < .r < 2
You
s
ook

x +k , X <3
3. If fix) = ■ 4 , .r = 3 is continuous at a: = 3, then k =
eB

3a:-5, a: > 3

at-4
our
ad

+ <7,a:<4

4. Let/(A:)=- a+b , X - 4. Then fix) is continuous at a: = 4 when a + b =


x-4
dY

+ b , x>4
Re

|a:-4|
Fin

X ^ -9
5. If/W=-
—f-,x^ 3 is continuous at a: = 3, then k =
-v-3
2x + k ,x = 3

x^-1
,x^l
6. If the function/(a:) = ■ a:-1 is given to be continuous at .r = l, then thevalueoffc
k ,.T=1
is

7. The set of points where fix) = x -[a:] is discontinuous is


1
8. The set of points at which the function fix) = is not continuous, is
log I a:

9. If/(x)=|
i7a: +1, if X > 1
x + 2, if x<l is continuous, then 'a' should be equal to
CONTINUITY 17.23

10. If f{x) is continuous atx = n and lim f{x) = lim^ f{x) - k, then k is equal to
x-¥a

11. The set of points of discontinuity of/(.v) =[.t], is


12. The set of points of discontinuity of f{x) = —, is
x-[x]
ANSWERS

1. 3a^ 2. 2 3. 1 4. 0 5. 0 6. 2 7. Z

8. {-1,0,1} 9. 2 10./(fl) 11. z 12. Z

VERY SHORT ANSWER QUESTIONS (VSAQs)

Answer each of the following questions in one word or one sentence or as per exact requirement of the

w
questions.
1. Define continuity of a function at a point.

F lo
2. What happens to a function/(.v) at = rt, if lim f{x)=f{a) ?
X—>(7

ee
X

3. Find/(0), so that/(a:) = becomes continuous at = 0.


1- y/T^

Fr
x^ -16
4. lff(x) =
, if AT 4
is continuous at ;c = 4, find k. for
ur
x-4

fc,if.t=4
ks

5.Y-4 0<.t<l
Yo

5. Write the value of b for which / (.v) = is continuous at at = 1.


[4a:^ + 3bx 1 <x <2
oo
eB

kx
x<0
6. Determine the value of constant 'k' so that the function / (a:) = h a: | ' is continuous at

3 ,.x>0
r
ou
ad

x = 0.
Y

+ 3a:-10
,x^2
7. Find the value of k for which the function / (.v) = .r-2 is continuous at a: = 2.
Re
nd

k , x^
Fi

ANSWERS

4.8 5. -1 6. -3 7. 7
2. / (x) becomes continuous at a: 3.2
ir Hi 1

CLASS XI (Code 241) VOLUME-2

Based on the syllabus of Applied Mathematics (Code 241) prescribed

ow
by CBSE for Class XI under 10+2 Pattern of Senior School Certificate Examination

e
re
rFl
F
r
fo
ou
ks
Dr. R.D. SHARMA
oo

B.Sc. (Hons) (GoldMedalist),M.Sc. (GoldMedalist), Pli.D.


Y
B

Professor and Head of Department of Sciences and Humanities


Delhi Skill and Entrepreneurship University (DSEU)
e

Aryabhatt DSEU Ashok Vihar Campus, Delhi


ur
ad
Yo
d
Re
in
F

DHANPAT RAI PUBLICATIONS (P) LTD.


22, ANSARI ROAD, DARYAGANJ, NEW DELHI-110002
E-Mail: lsh.dhanpat@gman.com
Combined Price : ? 745.00 for Vol. 1 & 2
(Not to be sold separately)

To prevent a pirated book being sold to you, always ask the bookseller
to put his stamp on the first page of this book.

ISBN: 978-81-944765-7-3

w
F lo
First Edition : 2020
Reprints 2021, 2022

ee
This Reprint 2023

Fr
for
Published by Ish Kapur, for Dhanpat Rai Publications (P) Ltd.
ur
s
ook
Yo

EVERY GENUINE COPY OFTHIS BOOK HAS A HSHF


eB

The title of the book has been Laminated by a High Security Holographic Film (HSHF) to prevent Piracy.
Iinportatit Note: Buying & Selling pirated books which do not carry HSHF is illegal and a criminal offence
under the Indian Penal Code. This also deprives the author and the publisher of their due earnings for
our
ad

providing you the knowledge through this book. Pirated books are also sold as old editions.
Y

© Author
Re

This book shall not, by way of trade or otherwise be lent, resold, hired out, or otherwise circulated without the
nd

publisher's prior written consent in any form of binding or cover other than that in which it is published. No
Fi

part of this book may be reproduced or copied in any form or by any means [graphic, electronic or mechanical,
including photocopying, recording, taping, or information retrieval system] or reproduced on any disc, tape,
perforatedmedia or otherinformationstorage device,etc., without the written permission of the author and
publisher. Breach of this condition is liable for legal action.
DISCLAIMER:

Every effort has been made to avoid errors or omissions in this publication. In spite of this, some errors might
have crept in. Any mistake, erroror discrepancynoted may be brought to our notice which shall be taken care
of in the next edition. It is notified that neither the publisher nor the authors or seller will be responsible for any
damage or loss of action to any one, of any kind, in any manner, therefrom.
For binding mistakes, misprints or for missing pages, etc., the publisher’s liability is limited to replacement
within one month of purchase by similar edition. All expenses in this connection are to be borne by the
purchaser.
All disputes are subject to Delhi jurisdiction only.
Dear Teachers & Students

w
CBSE has introduced Applied Mathematics (Code 241) as an academic elective at Senior
Secondary Level from Academic Session 2020-21 and onwards. This book of Applied
Mathematics for Class XI has been written strictly as per the latest syllabus prescribed by
CBSE.

e
e
Entire text has been divided into 30 chapters which have been divided into two volumes

o
r
for convenient handling. Volume-I consists of chapters 1-17 and Volume-II consists of

r
chapters 18-30. In each chapter, all concepts have been discussed in detail in a lucid manner

F
and have been explained with suitable illustrations and illustrative examples.

oF
ul
Some unique features of the book are:
— Detailed theory with Illustrations

sr
— Algorithmic approach

ko
— Large number of graded Illustrative Examples of
— Large number of unsolved Exercises
o
Y
— Large number of Multiple Choice Questions (MCQs), Very Short Answer Questions
rB

(VSAQs) and Fill in the Blanks Questions (FBQs).


eY

I avail this opportunity to convey my sincere thanks to Sh. Ish Kapur of Dhanpat Rai
Publications (P) Ltd for his painstaking and sincere efforts in bringing out two volumes of
u

this book in such an excellent form.


d
o
ad

Although every effort has been made to keep this book error free, inspite of this some printing
errors might have crept in. I would be extremely thankful if the same are brought to my notice
in

through email to rdsmath.feedback@gmail.com or by post.


Re
F

With my Best Wishes


Dr. R.D. SHARMA
APPLIED MATHEMATICS - XI (Vol. II) CONTENTS
Preface to the First Edition
(in)

18. DERIVATIVES
18.1-18.50

w
19. TANGENTS
19.1-19.22

20. PROBABILITY
20.1-20.157

o
e
re
21. DATA REPRESENTATIONS AND VISUALIZATION 21.1-21.31

Fr l
F
22. DATA INTERPRETATION-I (Central Tendency) 22.1-22.80

or
ou
23. DATA INTERPRETATION-II (Dispersion) 23.1-23.45
kfs
24. DATA INTERPRETATION-III (Skewness and Kurtosis)
oo

24.1-24.44
Y

25. CORRELATION ANALYSIS


B

25.1-25.27
re

26. INTEREST AND PRESENT VALUE 26.1-26.34


oYu
ad

27, ANNUITY
27.1-27.15
d
in

28. TAXATION AND BILLS


Re

28.1-28.20
F

29. MATHEMATICAL REASONING 29.1-29.43

30. LOGICAL REASONING


30.1-30.23

LOGARITHMIC AND OTHER TABLES T.I-T.34


APPLIED MATHEMATICS-XI (Vol. 2) CONTENTS IN DETAIL

Preface to the First Edition m

18. DERIVATIVES 18.1-18.50

18.1 Derivative at a Point 18.1

18.2 Derivative of a Function 18.4

18.6
18.3 Differentiation from first Principles
18.4 Fundamental rules for Differentiation 18.14

w
18.5 Differentiation of a function of a Function 18.25

F lo
18.32
18.6 Relation between dy/dx and dx/dy
18,33
18.7 Differentiation of Implicit Functions
18.36
18.8 Logarithmic Differentiation

ee
Fr
18.9 Differentiation of Parametric Functions 18.46

18.47
18.10 Differentiation of a Function with respect to another Function
for
19.1-19.22
ur
19. TANGENTS

19.1
19.1 Slopes of Tangent and Normal
s
ook
Yo

19.7
19.2 Equations of Tangent and Normal
eB

20. PROBABILITY 20.1-20.157

20.1 Introduction 20.1


our
ad

20.1
20.2 Random Experiments
20.2
20.3 Sample Spaces
dY

20.4 Event 20.8


Re

20.9
20.5 Algebra of Events
Fin

20.11
20.6 Types of Events
20.18
20.7 Axiomatic approach to Probability
20.38
20.8 Addition theorems on Probability
20.58
20.9 Conditional Probability
20.66
20.10 Multiplication theorems on Probability
20.11 More on conditional Probability 20.71

20.84
20.12 Independent Events
20.103
20.13 More on theorems of Probability
20.117
20.14 The law of Total Probability

(V)
(Vi) Contents

20.15 Baye's Theorem 20.126

21. DATA REPRESENTATIONS AND VISUALIZATION 21.1-21.31

21.1 Statistics 21.1

21.2 Statistical Data 21.1

21.3 Classification of Data 21.3

21.4 Data Representation 21.4

21.5 Visualization of Data 21.9

21.6 Graphical Representation of Frequency Distributions 21.13

w
22. DATA INTERPRETATION-I (Central Tendency) 22.1-22.80

22.1 Introduction 22.1

o
22.2 Measures of Central Tendency 22.1

e
re
22.3 Arithmetic Mean of individual observations or ungrouped data 22.1

rFl
22.4 Arithmetic Mean of grouped data or discrete frequency distribution

F
22.9

22.5 Median 22.31

r
22.6 Quartiles 22.44
ou
22.7 Deciles fo 22.50
ks
22.8 Percentiles 22.54
oo

22.9 Percentile Rank 22.58


Y

22.10 Mode 22.62


eB

23. DATA INTERPRETATION-II (Dispersion) 23.1-23.45


r

23.1 Introduction 23.1


ou
Y
ad

23.2 Measures of Dispersion 23.2

23.3 Range 23.2


d

23.4 Mean Deviation 23.3


in
Re

23.5 Variance and Standard Deviation 23.13


F

23.6 Analysis of frequency Distributions 23.33

24. DATA INTERPRETATION - III (Skewness and Kurtosis) 24.1-24.44

24.1 Introduction 24.1

24.2 Skewness 24.2

24.3 Measures of Skewness 24.3

24.4 Absolute measures of Skewness 24.4

24.5 Relative measures of Skewness .. 24.4

24.6 Moments 24.29

24.7 Kurtosis 24.35


Contents (vii)

25. CORRELATION ANALYSIS 25.1-25.27

25.1
25.1 Bivariate distribution
25.1
25.2 Covariance
25.7
25.3 Correlation Analysis
25.17
25.4 Spearman's rank Correlation

26. INTEREST AND PRESENT VALUE 26.1-26.34

26.1
26.1 Introduction

ow
26.1
26.2 Some useful Terms
26.2
26.3 Simple Interest
26.9
26.4 Compound Interest

e
27. ANNUITY 27.1-27.15

re
rFl
27.1
27.1 Introduction

F
27.1
27.2 Annuity
27.2
27.3 Types of Annuities

or
ou
27.3
27.4 Amount of an ordinary Annuity
ksf
28. TAXATION AND BILLS 28.1-28.20
oo

28.1
28.1 Introduction
Y

28.1
B

28.2 Goods and Services Tax (GST)


28.14
28.3 Income Tax
re

28.18
28.4 Bills
oYu
ad

29. MATHEMATICAL REASONING.., 29.1-29.43


d

29.1 Introduction 29.1


in

29.1
Re

29.2 Mathematical Logic


29.2
29.3 Statements or Propositions
F

29.2
29.4 Open Statements
29.5
29.5 Use of Venn Diagrams in checking truth and falsity of Statements
29.7
29.6 Compound Statements
29.8
29.7 Basic logical connectives or logical operators
29.8
29.8 Truth Tables
29.9
29.9 Basic logical connectives
Conditional and Biconditional Statements 29.15
29.10

29.11 Construction of Truth Tables 29.18

29.23
29.12 Logic Equivalence
(vHi) Contents

29.13 Negation of Compound Statements 29.25

29.14 Tautologies and Contradictions 29.35

29.15 Algebra of Statements 29.38

30. LOGICAL REASONING 30.1-30.23

30.1 Introduction 30.1

30.2 Logical Reasoning 30.1

30.3 Coding - Decoding 30.14

30.4 Odd Man Out or Odd One Out 30.19

w
30.5 Blood Relations 30.21

LOGARITHMIC AND OTHER TABLES T.1-T.34

o
e
re
rFl
F
r
ou
fo
ks
oo
Y
r eB
ou
Y
ad
d
Re
in
F
iCHAPTER 18
DERIVATIVES

18.1 DERIVATIVE AT A POINT

ow
DEFINITION Let f (x) be n real valued fimctiou defined on an open interval {a, b) and let c e {a, b). Then.
/ (x) is said to be dijferentiable or derivable at x = c, iff
f{x)-f{c)
lim ■ exists finitely.
X->L- .V -c

e
of / (x) at .v = t: and is denoted by / ' (c) or

re
This limit is called the derivative or differentiation

ItM
rFl
F
Df (c) or
x = c

r
That is.
ou
f{x)-f{c)
/'(c) = .v->c
lim , provided that the limit exists. fo
ks
x-c

Throughout this chapter it will be assumed that a given function / (x) is differentiable at every
oo

point in its domain i.e. lim


fix) -f{c)_ for all c in its domain.
Y
B

x-*c x-c

/(●v)-/(c) _ lim
fix)-fjc)
re

/'(c) = lim + x-c X - c


ou

fic + h)-fjc) or, /'(c) = lim fjc-h) -fjc)


Y
ad

f'(c) = lim h -h /i ->0

Find the derivative of f (.x) = /c of x = 0 and x - 5.


d

ILLUSTR.^TION 1
in
Re

SOLUTION By definition
fi0 + h)-f{0) _ k-k 0
lim lim — = 0
/' (0) = lim ^
F

b-^0 h /i->0 h It ^0 h
k-k 0
/ (5+/!)-/(5) _ lim lim = 0
and, /' (5) = lim h /i -»0 h h-^0 h
h^O

ILLUSTRATION 2 Let f be a real valued function defined byf (x) = x^ + 1. Find f ' (2).
SOLUTION We have, /(x) = X 2+1
f{2 + h)-f{2)
/'(2)= /i lim^
->0 h

{(2 + h}^ + 11 - \2^ +11


/'(2) = /i lim
-)-0 h

(/f2 + Ah +5) -5 =_ /j2 + Ah


lim = lim h + A = A
f'(2) = lim h /i ->0 h It ->0
/i -»0
18.2
APPLIED MATHEMATiCS-XI

ILLUSTRATION 3 Iff is a real valued function defined by f {x) y"p + 4.V + 3, then find f' (1) and
SOLUTION We have, / (.r) = + 4x + 3
/'(I) = h->0
lim fJllJltlfS}!
b

/'(I) = Urn \{l + hf + 4(l + h) + 3\~{l^ + 4x1 + 3)


h

/' (1) = lim


{b^ + 6/7 + 8) - 8 lim
+ 6h
h^O h
lim h + 6 =6
h^O h h->0

ow
and. /'(3) = lim /(3 + /7)-/(3)
h

/'(3) = lim ((3 + /7)^ + 4(3 + /7)+31-{3^ + 4x3+3)


h-*0 h

e
re
/' (3) = lim

Fl
jb'^ + 10/? + 24) - 24 lim
I? +10/7
lim /? + 10= 10

F
/7-+0 b /j->0 h h-*0

ILLUSTRATION 4 Find the derivative off (x) = 2x^ + 3x -5 rtf x = - 1. Also, prove that
ur
r
/'(0)+3/'(-l) = 0.
SOLUTION Let us first find the derivatives of / (x)
fo
at x = 0 and x = -1.
ks
By definition
Yo

/' (0) - lim /(0 + /7)-/(0)


oo

h-^0 b
eB

/ ' (0) = lim f(h) -/(O)


h^O h
ur

/' (0) - lim (2/7^ + 3/7-5)-{2x(0)^ + 3 X (0) -5)


ad

h-*0 h
Yo

/'(O) = Urn (2/7^+ 3/7-5) -(-5)


d

h~^0 b
Re
in

2h^ + 3/7
/ ’ (0) = lim = lim (2/7 + 3) = 2 X 0 + 3 = 3
/j-»0 h
F

/i->0

and. /'(-I) = lim /(-l+/7)-/(-l)


h^O h

/'(-I) = lim
{2 (-1+/7)2 +
3(-l +/7) -5} -(2(-l)^ + 3(~1) -5}
ij-^0 b

2h^-h
/' (-1) = lim = lim (2/7-1) = 2x0-1 = -1
h^Q b h^O

/'(0) + 3/'(-l) = 3+3X-1 = 3-3 = 0

18.1.1 PHYSICAL INTEGPRETATION OF DERIVATIVE AT A POINT


Let a particle be moving in a straight line OX starting from point O towards point X as shown in
Clearly, the position of the particle at any time t depends upon the time elapsed. In other words.
18.3
DERIVATIVES

t = la t = t,, + h
r Q X
o

Fig, 18.1
function / of time f taken by
the distance of the particle from O depends upon fhe time i.e. it is a
the particle. further time h i.e. at time
Let at any time f„ i.e. at time f =(o , the particle be at P and after a
t =fo + li, it is atQ.
OP = f (fo) OQ = f (to + /')
Distance travelled in time h = PQ= OQ - OP = / (tg +
PQ /'(fn + ^0-/(^o)
Clearly, Average speed of the parHcle during the journey from P to Q = h h

As h -> 0, we observe that Q P.


Urn
/(fo + /0-/(*o) = /'(fo)
(Instantaneous speed at time f = fo) h^O h

w
Thus, if f (f) gives the distance of a moving particle at time t, then f' (fo) i.e. the derivative off at t fy

F lo
represents the instantaneous speed of the particle at time t - fo or. at the pointin P.a straight line in t seconds
ILLUSTRATION The distance f (f) in metres moved by a particle travelling
is given by f (f) = f ^ + 3f + 4. Find the speed of the particle at the end of 2 seconds.

e
Fre
SOLUTION We have, / (f) = f^ + 3f + 4.
.e. the derivative of / (f) at f - 2.
for
The speed of the particle at the end of 2 seconds is given by /' (2) i.e
Now,
r
/ (2+ /»)-/(2)
You

/'(2) = ii->0
lim
oks

h
eBo

{(2 + hf + 3 (2 + /)) + 4} - 12^ + 3 X 2 + 4l


/'(2) = /i->0
lim h

+7h
(/i^+7/j+14)-14^
ad
our

lim — lim h + 7 =7
/'(2) = Urn0 h /i->0 h h->0

Hence, the speed of the particle at the end of 2 seconds is 7 m/sec.


Re
dY

18.1.2 GEOMETRICAL INTERPETATION OF DERIVATIVE AT A POINT


Let/(x) be a differentiable function. Consider the curve,/ = / (a). Let P {c fiO) be a point on the
Fin

curve y =f (.t) as shown in Fig. 18.2 and let Q (c + }j^ f [c + h)) be a neighbouring point on the
curve y =f (x). Then,
QN _ / (c + h) -f{c)
Slope of chord PQ = tan Z QPN PN h

Taking limit as Q P i-e. h 0, we obtain


f{c + h)-f{c) ...(i)
lim (Slope of chord PQ) = h-*Q
lim h
Q-^P

AsQ -» P chord PQ tends to the tangent to y = / (v) at point P. Therefore, from (i), we get
lim
/(c + /0-/(c)
Slope of the tangent at P = h-^0 h

Slope of the tangent at P =/' (c) i.e,


i tan \\i =f (c).
18.4
APPLIED MATHEMATICS-XI

Q(c + h,f{c + h))

,y.
N
h

T O ■^x
L M

Fig. 18.2

where y is the inclination of the tangent to the

ow
ru, f, . ■ , r point(c,/(c)) with the A--axis.
It / (T "^ ciin’e y=f (.v) at
illustratiox
Find the slope of the tangent to the ciirzv y = x^ at (-1/2,1/4)

e
SOLUTION Let/(i) = .v2.Then,y=/(.v)isthegiven curve. Clearly, slope of the tangent to the

Fl
re
- IVIS equal to/' -
r n.

F
curve at -
I 2'4. \ 2) i.e. the derivative of / (.y) at y = -1 /2.
ur
1

Now, /' / - 1' lim or


sf
V 2J /j -»0 h
k
Yo
/ 1 \2
f
oo

+ //
f ’( I'l - lim V 2 2j
B

2 J /f -> 0 h
re

n 1
~h + h^ -
1 '1 U h^-h
/ lim i_4
u

lim lim h -1 = -1
I 2)
ad

h^O h h^O h h -*Q


Yo

Hence, slope of the tangent to the curve y = y^ at pointf - -2' 41 to ~ 1- This means that
d
Re

/i n
the tangent to the curve at point
in

2'4 makesl35°angle with the positive direction of X-axis


F

18.2 DERIVATIVE OF A FUNCTION

LetT(T)be '"'T'm domain


7“
of any point ar m the domain is the value of the derivative of / at v i.e. / ’ (,v) or ± (f (x)). This
d.Y
function is called the derivative
or differentiation of/ (y) and is given by
/ ' (x) = lim /(Y + /J)-/(Y) or. /-(/(x))= lim +
/j ->0 h dx h~*0 h
DERIVATIVES 18.5

The process of finding the derivative of a function by using the above formula is known as the
differentiation or derivative from the first principles.
18.2.1 DERIVATIVE AS A RATE MEASURER

Let / (;r) be a function of j and let i/ = / (x). Clearly, the value of i/ depends upon the value of v
and it changes with a change in the value of x. So, .v is called the indL’pcndeiU vnriiiblc and y the
dependent variable. Let A.V be a small change (positive or negative) in x and let Ay be the
corresponding change in i/ =/ (x). Then, the value of x changes from .v to .v + A.v and the value of
the / (x) changes from / (x) to / (x + Ax). So, change in the value of f is
/ (x + Ax) - / (x) or. Ay - / (x + Ax) - / (x) ...(i)

ow
Thus, we observe that due to change Ax in x, there is change Ay in y. Therefore, due to one unit
change in x, change in i/ is equal to Tliis is known as the average rate of charge of y with
respect to .x.

e
As Ax ► 0, we observe that Ay also tends to zero.

re
A)/
Instantaneous rate of change of y with respect to x = Urn
^

Frl ^ A.V 0 Ax

F
If we use the phrase rate of change instead of instantaneous rate of change, we have
ou
or
Rate of change in y with respect to x = lim
^ ‘ A.Y-^0 Ax kfs
lim
/(X+ A.v) -fix) [Using (i)l
Ax->0 A.V
oo

T(/(,v» [Using def. of derivative]


Y

dx
B

-
re

dx
oYu

Thus, — or. — (/(x)) measures the rate of change of y = / (x) with respect to x.
ad

dx dx

i.e.. lim
d

dx Ax 0 Ax
in

TJje meaning of the term "rate of change ofy with respect to x” is that if x is increased by an
Re

REM.\Rk I
dy
additional unit the change in y is given by . For example, the rate of change of displacement of a particle
F

dx

is defined as its velocity, so if we say that a particle is moz’ing zuith the velocity v km/hr then it means that
zvhen time is increased by one hour the displacement changes by v km.
Ay /(x + Ax)-/(x) zohich are exactly
REMARK 2 Some authors also define — as lim or, lim
' dx A.r^0 Ax Ax^O Ax

identical to the definition given in this book.


REMARK ' We have seen that — or, — (f(x)) is the derivative or differentiation of y =f (x). Also ' dx
dx dx

or , dx to x. So, zve can say that the derivatwe of a


— (/(x)) measures the rate of change of y zoith respect
function y =/ (x) is same as the rate of change of f (x) zoith respect to x. Consequently, phrases such as
''dijferentiation of a function f (x)" and ''differentiation ofa function f (x) zoith respect to x" convei/ the same
meaning and are used invariably.
18.6
MATHEMATICS-X!

18.3 DIFFERENTIATION FROM FIRST PRINCIPLES


In the previous sections, we have learnt that the derivative of a function f (a:) is given by
fix) = !im
f{x + h)-f{x)
h-*0 h

The process of finding the derivative of a function by using the above definition is called the
differentiation from first principles or by ab-initio method or, by delta method.
In this section, we will find the derivatives of some standard functions viz. .y” / c
.V
, /?' and log -Y
by first principles. Following results will be very helpful in finding the same.

-c/’ /i-l
(i) lim - na
(ii) lim - = log^, a,a>0,a^i

low
x^a X -a x^O X

(iii) lim = 1 log^ (1 + .y)


.Y
(iv) lim =1
-t^O .Y

THEOREM 1 If f (.y) = y" , where n s R, then, the differentiatio n of with respect to x is nx


ji-1

e
re
n-\
he.
dx
nx

rF
F
rROOF Let/ (y) = x". Then, f{x + h) = (y + h)
H

r
f(/M) = lim fix + h) -fix)
fo
u
dx /i->0 h
ks
(Y + Zi/'-Y"'
f (/(.v» = lim (x + hf-x^'
Yo
lim
flY h^O h h-*o (y + h) - x
oo

z"-y"
B

=
lim , wherez=x + h andz -> y as Z» ^ 0
dx 2 ^ Y z - Y
re

II
-d'
nx
Using: lim —
u
ad

X-* a x - a
Yo

it-1
Hence, = nx
Q.E.D.
dx
d

n.LUSTRATlONS Using the above formula, we obtain


Re
in

(i)
F

dx

.... d M > .-3-1 -3


-3y
^
dx I, dx Y^

(iii) „l/2-l _ 1
dx dx 2 2^
- 3
1
(iv) —
dx[-Jx
= A(.,-V2)_i
dx 2
^.-1/2-1^ 1
2
Y 2

(v) ;^(-y)=^(y^)=1xy^“^
dx dx
.0
= 1 X Y^ =1

,(vi) d n
dx YJ
4-
dx
(.V-')= -IXY .-1-1 1

y-2
THEOREM 2 The differentiation ofe^ zvith respect to x is e^. Q.E.D.
18.7
DERIVATIVES

d
i.e. ^
dx
ie^) =

PROOF Let f{x) = e^'. Then, /(x + h) =e^'^'‘


/(X + h) -fix)
^(/(x))= lim
dx h -> 0 h

d
e‘^-1
= lim
ifix)) = /ilim h /;^0 h h-^0 h
dx -> 0

ow
e^' -1
^ (/(!■)) = ●.● lim = 1
e'^ lini = X 1 =
h
h h^O
dx /i->0

d Q.E.D.
Hence,
dx

e
The differentiation of a''^' (/? > 0, r 5^ 1) with respect to x is log^ a.

re
THEOREM 3

d
^ logc n

Flr
i.e.

F
dx
-V + h
PROOF Let/{x) = . Then, f{x + h) = a

fm) =
fix ^h)-fix)
ou
sr
flx /i -> 0 h

fo
X + h h
-a^ a^' a ' -
^(/(x)) = Um
a
lim
dx h 0 h /i->0 h
k
oo
h
a" -1 a^ -1
^(/(x)) = lim = log,, a
lim
= logt- a .Y^O X
Y
h
reB

Hence, ^
dx
{«■■'■) = a^ logc a Q.E.D.
uY

1LLUSTRATIONS the above formula, we get

(i) ^(5"'')=5''
dx logeS
ad
do

(ii) A
dx
(10") =10' log,, 10.
in

(iii) A. =A ((f>2)--^) = {e^y^ log ■ 2 log e = 2e


2y
Re

dx dx
F

1
Q.E.D.
THEOREM 4 The differentiation o/log^ x, x > 0 is —.
X

1
I.e. 4-
dx
(log^ x) = X

PROOF Let f(x) = log,.-V. Then,/(x +/z) - logp (x +/j)


' /(x + /z)-/(x)
4-
dx
ifix)) = lim
h-*0 h

d log,. (X + h) - log,. X
^ifix))
dx = h^O
.lim h

loge (1 + h/x) =_ loge (1 + h/x) f


fim) =
dx
lim
/j->0 h
lim
h-*0 h/x X

1 log,. (1 + x)
^(/W) = lim =1
Y^O X
dx x
18.8
APPLIED MATHEMATICS-XI

Hence, 4~
dx
(loge x)=- X Q.E.D.

1
THEOREM 5
The dijferentiation o/log,, x{a> 0,n ^1) with respect to x is
a: log,, a
^ n X 1
i.e.
— (log^ X) = —
dx X logj, n
PRO^ Let fix) = log,, .V. Then, f(x + h) = log„ (.v + h)
to fix + h) -fix)
dx /, ^ 0 h

x + h
log a

ow
X
= lim
dx h~^0 h h^O h

T(m
dx
= lim iQgfl (1 + /? / -V)
h
lim
log,. (1 + h/x)
V iog„;.=
/j ^ 0 (log,, a) ■ h log,, a

e
Fl
re
\
I f 1
log 1 +
-^(/(-v)) = log,. (1 + h/x)
1 1

F
lim lim
.V
dx =1
log,, a h^o xih/x) X log,, a h-*0 h
ur
or
X

Hence,
^(log„.T)=—
sf
Q.E.D.
dx X log„ a e
k
Yo
1 l.LUSTRATIONS We have,
oo

(i) ^(log3.v) = X log,


1 .... d 1

= £(log5.v) = -V log, 5
1
(ii) —
B

dx
3 dx[ log.,. 5
The above results can be summarized as under:
re

d -1
(i) i~ix”)=a.^’ (ii)
u

(iii) ~(n^') = /I-'" log,, a.


ad

dx dx dx
Yo

1
(iv) dx
(log,, X) = - (v) ~ (log„ x) =
dx
X
X log,a
d
Re
in

ILLUSTRATIVE EXAMPLES
F

EXAMPLE 1
1 Find the derivatives of the following functions from first principles:
(i) a: ^ - 27 (ii) (A- -1) (A - 2) (iii) A.
A

SOLLTTION (i) Let/(.v) = x^ -27. Then,/(a + h) = (a + li)^ - 27


T (/(.V)) =
dx
lim
/f->0
fix^h)-fix)
h

ix + h)^ -27 ■ -f.v^ -27


=> lim
xT = h->0 h

~ ifix)) = lim (.v^ + 3a^ h + 3.r/(^ + h^ -27) -(a^ -27)


dx /r -> 0 h
18.9
DERIVATIVES

3x^h + 3xl? + h^
lim
ilx h^O h

=>
dx
lim (3y^ + 3.V/J + h^) = 3y^ + 3y x 0 + 0 = 3y^
- (/(.y)) = /j->U
(ii) Let f (y) = (:t -1) {x - 2). Then, / (y + h) = (y + h-l) {x + h-2)
f{x + h)-f{x)
i (/{.V)) =
dx
lim
h^O h

lim
(Y + h-l) (y + /?-2)-(y-1) (y-2)
X
dx (fM) =

ow
=5>
/i-»0 h

{(Y -1) + h\ {{X -2) + h] -{X -1) (y-2)


f (/(■'■)) =
dx
lim
/i->0 h

(y -1) (y - 2) + h (y -1) + /i (y -2) + -{x -1) (y - 2)


^(/(y))= lim

e
dx /i 0 h

re
h{x-\) + h{x-2) + h^ -_
=> 4- if (●■^■)) = lim lim {(y -1) + (-f - 2)1 + h ~ 2x - 3

Frl
F
dx h /(->0

1 1
(iii) Let /(y) = —.Then,/(Y +//) =
X
ou (y + hf

sor
d f{x + h)-f{x)
if (-^')) = lim h
dx h -> 0 kf
1 1
oo
ix + hf Y^
4~ if (-V)) = lim
Y

dx h
B

Y^-(Y + /I)^
^ aw) =
dx
lim
li~*0
=
hx^{x + hr
y
re
oY

(/(●'■)) = h,lim 0 hx^


-2hx-h^ - 2y - h - 2x - 0 -2
u

4-
dx
=
(Y + hy
-
h
lim
0 y-2 (y + bf Y^ X Y^ Y^
ad
d

EXAMIML 2 Differentiate tbefoUoiving functions with respect to x from first principles:


1
1
in

(i) Vy (ii) yjax + b (iii) (iv)


ax + b
Re

SOLUTION (i) Let/(Y) = Vy. Tlien, / (y +/)) = ^Jx+iT


F

f{x + h)-f{x)
4~
dx
(fix)) = lim /j^O h

yjx +”/i - %/y


4-
dx
(/(.V)) = lim
h ->0 h

yjx + /r + Vy ^y + /i + ^/Y
4- (/(●'■)) =
/ V
lim
dx h ■-» 0
h ^x +iJ +4x
h 1 1 1
4(/m) =
dx
lim
//->0
lim
h->0
^x + h + Vy Vy + Vy 2-Jx
h yjx + h + Y
18.10
APPLIED MATHEMATICS-XI

d . f—. 1
T- (V^) = T-
dx 2^

(ii) Let/(a:) = yjax+ b.Then,f{x + h) = ^(x + h)+b = ^{ax + b) + ah


(J{x)) = lim
f{x + h)-f{x)
dx /, _> 0 h

~ (fix)) = lim ^{ax + b) + ah - ^ax + b


dx h^o h

if {x)) = lim
Max + b)+ah - fax + b M ax + b) + ah+ ^ax + b
dx /,->o h "" ^
^jiax + b) + ah + ^ax + b

w
(fix)) = lim
{ax + b) + ah-(ax + b)

h ^l^iax +b) + ah + ^ax + b >


dx h-^0

o
ah

e
-r (/W) = ^
dx h^o

re
h
< ^[iax + b) + ah + ^ax + b
rFl
F
4- (fix)) = lim a a
dx
0 ^{ax + b) + ah + ^ax + b yjax + b + ^ax + b
r
ou
Hence,
|(V^) = 2 yjax + b
a fo
ks
(iii) Let/(AT) = ^.Then,f{x + h) =
1
oo

x+ h
Y
eB

4-
dx
(fix)) = h^O
lim fix + h)-fjx)
h
r

1 1
You
ad

/
dx (/W) = h^o
lim
x + h X
h

/
dx (/W) = /,,lim
d

x + h-x x-jx + h) -1 1
lim lim
Re

-> 0
in

h-^0 hx (a: + h) h-^0 x{x + h)


h j yjx + h + 4x
F

^
Hence, — f-1 1
dx I,a:.
1 1 1
(iv) Let / (a:) = .Then,f{x + h) =
ax+ b
a(x + h)+b (ax + b)+ah

f
dx (fix)) = /, lim
-> 0
f(x + h) -/ (a:)
h

1 1

4-
dx (fix)) = h^o
lim (ax + b) +ah h
ax+ b

4-
dx
(fix)) = lim [ax + b\- {(gA: + b) + ah}
h-^0 h(ax + b) {(ax + b) + ah]
18.11
DERIVATIVES

-all
lim
X
ax
(/(x» = /, _»0 /j + b) {(fl.x + b) + ah\
-a a

4-
dx
(/(■'●» = lim
/i ^ 0 {ax + b) {(<7X +1) + ali\ (rtx + b)^
1 -a

Hence, -r
dx ax + b {ax + b)^
EXAMPLES Differentiate the foUoioing functions zuith respect to x from first principles:
■? /● V 2x + 3 (v) X
- 3/2
(iv)
(i) V^+ 3 (ii) ^4-.t (iii) ax^ + X 3x + 2

SOLUTION (i) Let/{x) = ^2x + 3. Then, / (x + h) = .^2{x + /i) + 3


/(x + /Q-/(x)
~{f{x)) = lim h
dx /i->0

j2{x+h) + 3-px+3
-^{f{x))
dx

4-(nx))
dx
= lim

=
h^O

lim
h-^0
h

. p [X +h)+3- yj2x + 3 F low ■ ●

h ■ p (x + h) +~3 + px + 3 ■
p{x + h) + 3+^2x+3 ■
for Fre
\
(2x + 2/> + 3 - 2x - 3)
f(/(-r))
dx
= lim
/r->0 h
X -p-

. px +2h+3 + px+ 3 ■
eBo ks
Your

2/j 1
4(m) =
dx
lim —X lim
^0 h h^O
. px + 2h +3+ px + 3" ■
ad
our

1 2 1
?-(/(■'●» =
dx
2x
pJT3 + px + 3 2{pIT3) px + 3
Re

(ii) Let /(x) =p- X. Tlien,/ (x + h) = p-{x + h)


f{x+h)-f{x)
Find Y

lim
dx h-^0 h

^i-{x + h)-^4^
4 (fix)} =
dx
lim
h^Q h

p-{x + h) - p-x't ●( p - (X + ii)+ y[^x ).


dx
^ (/(!●)) = lim
/i->0
h p-{x + ii) + 4^x )
4 - (X + h)-{A-x)
■^{m)
dx
= lim
/i->0
b J ^4-(x+”7i) + p^ ■
18.12
APPLIED MATHEMATICS-XI

-h -1
= lim
dx /,-»0
27^
h- yl4-x-h + ^4-x
(iii) Let f{x) =ax^ + ~.
^
Then, f{x + h) = a{x + hf + -^
x + h

~(f(x)) = lim fix + h) -f(x)


dx /i^o h

2 b
a{x + h)^ + — ax^ + -

^ifix)) = h^o
lim x+h f X

dx

ow
h

1
a {(x + h)^ ~x^\ + b ■ —
fm) = h-^o
lim x+h X j
dx h

e
Fl
re
x-x-h
a(2hx + h^) + b -
^(/W) = h^o
lim x{x + h) J

F
dx h
ur
4-<j.ix))
dx
= /j->0
lim ■ a {2hx + h^)
h
b{-h)
hx (x + h) j or
sf
-^(f(x)) = //^o[
lim < a{2x + h)x(x + h)
k
Yo
oo

dx
eB

2x+3
2(x + h) + 3 2x+3 + 2h
ur

(iv) Let fix) = .Then,fix + h) =


3x + 2
3(x + h) + 2 3x + 2+3h
ad
Yo

d
— (/(:c)) = lim f(x + h)-fix)
dx h^o h
d

2x+3 + 2h 2x+ 3
Re
in

f(/(x)) = /,Urn
3x + 2+3h 3x + 2
F

dx -> 0 h

4-(fix)) = lim (2x + 3 + 2h) j3x + 2) -j2x + 3) j3x + 2+ 3h)


dx //->0
/T(3x + 2) (3x + 2 + 3h)
4~(f(x))
dx =. 1,^0
lim j2x + 3) (3.t + 2) + 2hj3x + 2) -j2x + 3) (3x + 2) - 3h j2x + 3)
hi3x + 2)i3x + 2+ 3h)
4-m)
dx
= lim h (6x + 4 - 6x - 9)
/;^0/i(3x + 2)(3x + 2+ 3/z)
-^ifix))
dx
= lim -5 5

h-^0i3x + 2)i3x + 2+3h) (3x + 2)2‘


(v) Let fix) = X + h)=ix + h)~
-^if(x))
dx = h^o
lim fix + h) -fix) h
DERIVATIVES 18.13

(,..,/,)-3/2_^-3/2
~(/W)
cix
= lim
h
/i-^O

d
= lim
(,. + /0-3/2_.,-3/2
dx /i^O {x + h)-x

2-3/2 _ ^-3/2
lim , where z=x + h and z -> .v as h 0
dx z -> .Y z-x

.11 II

^(/(.V))= (-3/2),v-V2-’
-a 1-1
Using: lim - = nd
dx x^a X -a

-5/2

1 J'{x)
EXAMi’iEJ Using first principles, proi>e that —
dx\f{x)\

w
F lo
1 1
SOLUTION Let([>(A;) = . Then, (j) (.v + //)
/(A-) / {X + /i)
(|)(.T + //)-(f)(.Y)
im- lim

e
h

Fre
h->0

1 1 for
lim
/(●V + /Q /(.V)
h-*0 h
r
/(.v)-/(.v + /Q
You

— {<j>(.v)} = lim
oks

=>

dx h^o hf{x)f{x + h)
eBo

d fix)-fix+ h) 1
— {4>(.v)}’ = h-^0
dx '■
lim h
X lim
h-^O fix) fix+ h)
fix + h)-fix)
^ {<i>(A-)} = - lim
our

1
ad

X lim
h^Q h fix) fix+ h)
d 1 fix) is differentiable
— {'I'(t)} = -f'{x)x fix) is continuous => lim fix + h)= fix)
dY
Re

fix) fix) h-^{)


Fin

d fix)
\fix)f
EXERCISE W.1

Differentiate each of the following from first principles:


1
1. (i) -
X
(ii) 4 73
X

+ 1 .Y .-2-1 A- + 1
(iv) (V) (Vi)
X X x+2

x + 2 .11
1
(vii) (viii) k x (ix)
3y + 5

(x) y2 + A- + 3 (xi) (,t + 2)3 (xii) -Y^ + 4y2 + 3x+2


18.14 APPLIED MATHEMATICS-XI

2.V + 3
(xiii) (.Y^ +l)(x-5) (xiv) -^2? + 1 (XV)
y-2
3.V rt.Y + b
2. (ii) e (iii)
(iv) xe^ (V) -Y (Vi) (-Y)-l
ANSWERS

1
1. (0-2.T-2 (ii) -1.1- 3/2
(iii) - 3y
-4
(iv) l-~
Y^
1 1
(v): + ~ (Vi) (vii) (viii) nkx’’ ^
X (y + 2)2 (3y + 5)^
1
(ix) 3/2
(x) 2y + 1 (xi) 3(y + 2)2 (xii) 3y^ + 8y + 3
2(3-y)
2y -7
(xiii) 3y^ -10y + 1 (xiv) (XV)
(1-2)2

w
- X 3x ax + b
2. (i)-c’ (ii) 3e (iii) a e (iv) (Y + 1) r'-

F lo
(v) -1 (vi)
Y

ee
HINTS TO SELECTED PROBLEMS

Fr
x^ +1 1
1. (iv) Let/(Y) = = Y +
—. Then, f{x + h) = (y + h) + for
X X Y + h

f{x + h)-f{x)
ur
/'(y)= lim
/i -»0 h
s
ook
Yo

■ {x + ll) - X ■+ ■ X +^ h
1 1 1
(y + h) + X +
x + h X
eB

X
/'(Y)= lim lim
/i->0 h /i^O h

x-h h
ht + ^
our

h -
ad

X (y + h) X (y + h) 1 1
/’ (y) = lim lim = lim 1 - = 1 -
h->0 h h->0 h h-^0 Y (y + h) y"
Y

2. (v) Let f{x) = ~x.Then,f{x + h)=-{x + h)


Re

/(Y + /0-/(Y) = lim - (y + h) + Y -h


nd

f ’ (y) - lim = lim — 1.


h~>0 h /j->0 h h^O h
Fi

1 1
(vi) Let / (y) =—. Then, / (y +/i) =
X Y + h

/(y + /0-/(y)
/’(y) = lim
/i-»0 h
1 1
+
Y + /l Y y + (y + /j) h 1 1
/'(Y)= lim = lim = lim = lim
h^Q h h 0 y(y + /0 h /i^O/?y(y + /0 h 0 Y (y + h) X-

18.4 FUNDAMENTAL RULES FOR DIFFERENTIATION

In the previous section, we have used the definition of derivative to find derivatives. This
section is mainly devoted to develop several rules that allow us to find derivatives without
using definition directly.
DERIVATIVES 18.15

THEOREM 1 Differentiation of a constant function is zero i.e., 4-w=o.


dx

PROOF Let f (.t) = c be a constant function. Then,

!(/(,))= ■ ita "



dx
+ h’ /;->0 /J
=
d
Hence, — (c) = 0, u'here c is a constant,
dx

REMARK Geometricalli/ the graph of a constant function is a straight line parallel is x-axis. So, tangent
at eiyery point is parallel to x-axis. Consequently, the slope ofthe tangent at every point is zero, i.e. ^

w
THEOREM 2 Let f (x) be a differentiable function and let c be a constant. Then, c f (x) is also
differentiable such that f |c/(.V)l = cf{f(x)).
dx dx

o
i.e. the derivative of a constant times a function is the constant times the derivative of the function.

e
re
PROOF Since / (.r) is differentiable. Therefore,
f{x + h)-f{x)
rFl ^if(x)).

F
lim exists finitely and is equal to dx
h^O h

r
...(i)
ou
I.e.
dx h^O h fo
ks
Let g{x)=c f{x).Then,
gjx + h)-g{.x)
-^{g{x))= lim
oo

dx /i-»0 h
Y
eB

cf{x + h)-cf{x)
^(g(.v))
dx = lim /i->0 h
r

-^igix)) =
ou

c lim [Using (i)l


Y
ad

dx h-^0 h dx

Hence, g {.t) = c f (x) is differentiable such that ^ / (^')) = "p (/


d
Re
in

ILLUSTRATIVE EXAMPLES
F

EXAMPLE 1 Differentiate the following functions with respect to x:


(i)log,.r'.(ii)e=*'°8-’^(iii)2'“»2^ (iv)5(2^'“S2-'^) (v) 5 r'‘ (Vi) 9(3')
SOLUTION (i) We know that log_YX =1.
^(1) = 0.
d
— (log.^ x) = dx
dx

(ii) We know that e^°^ ^ =k.


d d
dx
(^,3108^) dx dx

(hi) We know that a = n

(2'°S21-) = T(j) = 1.
d

dx dx
18.16 APPLIED MATHEMATICS-XI

(iv) Clearly,

dx
= 5 rf.v_^(23>og2-Vj 5i_(2l“82^^)
dx
=5—(.t3)
dx

= 5(3.y“) = 15

(v) Clearly,
-^(5
dx
£.-'■) = 5-^(1-^')
dx
=5e^
(vi) Clearly,

^(9.3') = 9^(3'') ^ 9 (3^ log,, 3).


dx dx

THEOREM 3 [f f{x) and g{x) are differentiable functions, then show that f{x) ± ^(.r) are also
differentiable such that

w
F lo
i.e. the derivative of the sum or difference of two functions is the sum or difference of their derivatives.
,'IY I. M
Since / (y) and g (y) both are differentiable functions. Therefore,

T(/(,v))= ita l(jL"';>-/W,andA(g(,)). ot both exist.

e
dx h-,0 h frY * h-,0 h
Now,
Fre
for
^\f(x)+g{x} ■ =
dx
lim
h^O
[f (y + h)±g (x + h)l - {/ (y) + g (y)}
h
r
You

\f (Y + //) -/ (y)1 ± (Y + h)-g (,Y)|


oks

lim
/?->0 h
eBo

lim
fix + h)-f{x) ± lim g{x + h)~g{x)
h->0 h ;»->o h
ad
our

[Using (i)]

Hence, / (,v) ± g (x) is differentiable and f ^ (x) ± g (,v)J


dY
Re

The above result can be extended to a ifnite number of differentiablefunctions. Thus, we have
Fin

£{/i(y)±/2(y)+....±/,(y)} =
d
±... ±
dx

ILLUSTRATIVE EXAMPLES
tiXAMI’I.E 1
Differentiate thefolloxuing functions with respect to x.
ax^ + bx + c
(i) Y^ + ^
1
(ii) (iii) Vy +
Y VI VI
SOLUTION (i) Clearly,
d \ 2 ■ 1
dx [ j2

^ (y-2 + .V 2) = a (,v2) + ± ^ 2x + (- 2) Y-^


2
= 2y
dx dx dx y-3
18.17
DERIVATIVES

(ii) Clearly,
d nx^ + bx + c
dx

.2
d ax bx c

Tx\7I^ Jx~^ 4^'


dx\
d
d
,3/2k A U,-V2 .-1/2

ow
● + c.v
— < ax
dx dx dx

d
a — d K.3/2L^AJ,1/2 ^ + c — <x
dx
-1/2

dx dx

e
re
( 1
^.-3/2^1 ^
a

rFl + c
I 2 2
+ —;
2 2

F
2 U

(iii) We have.

r
1 ^2
ou
d
dx fo
ks
d
.V + 1+2 = ±{x)+—{x-'^) + -^{2)= l+(-l)x 2+0=1-^X
oo

dx X dx dx dx
Y

^2! + — + ..., siioio that ^ =y.


B

EXAMPLL 2 + -
1 ! 3! dx
re

SOLUTION We have,
ou

,v2 .r^
Y

y=l.^
ad

+ — + +...
1! 2! 3!

d ^ d -T^ 1
d

dx
= ^(1) +-
dx"' dx[.v }
+ —
dx 2!
+ —
dx 3!
+...
in
Re
F

or.
dx dx Wdx 2! dx 3! dx

or.
dx
= 0 + 1 + -(2.v-)+
2!
i-(3.v^)+...
3!

x^
or.
dx 2!

dy
or. -r =y-
dx

A1 1TI:k We have,

y = e^ 'll ^ = e* = y.
dx dx
n
X^
●)
y' dy
x~
- ^y + ^
n\ = 0.
EXAMPLE 3 ^Ay=l + ^■*-^ +J| + ... + —,I
show that -fi
n dx
18.18
APPLIED MATHEMATICS-X!

SOLUTION We have.
2 3 4 .n
X
y = + + +^+. + + —
X

1! 2! 3! 4! Ill

+ —
d ]+ d

a-3 1 + ... +
d X
It

».v dx dx 11 dx 2! dx 3! dx nl

dx dx 1! dx 21 "IT I I (-0
di/ I
dx
= 0 + -
1 !
+
i(2,T)
2!
+ i(3A-2)+..
3!
. + — (nx
1 n-l

2 n-l
dy

_
= 1 + —H
-A A A
!■...+
dx 1! 2! {»-!)!

w
a2 .V'-' .11
X X
It

+ + ... + + —
dx 1! 2! in-1)1 nl

F lo
n\

‘^1/ A-”
-f- = y r
dx nl

e
Fre
.11
X
=>
-y + — = 0.
dx for
UXAMPLE 4 Differentinte Uw following functions with respect to a-
r
(i) (x^ - 3a + 2) (a + 2) (ii) -v^
1
You
s

A-‘ J
ook

SOLUTION (i) Clearly,


eB

dx
■ {x^ - 3a + 2) (a + 2)
our
ad

d , 1 7
dx
(a‘^-a^-4a+4)
dY
Re

- 4 ^ (a) + ^ (4) = 3 A^ - 2a - 4 + 0 = 3a^ - 2v - 4


Fin

dx dx dx dx
(ii) Clearly,
d j 2 1 V’
— < X
dx

d
a" + 3.v^ + 4
1
+
dx A^ X^

= 6a^ + 6a+ 3(-2)a"''^+{-6)a-^ =6a^ + 6a—~ 6

a3 a^
18.19
DERIVATIVES

/'(I)
EXAM I’LE 5 If f{x) = a .v", pwvc that a = n

SOLUTION We have, /(x)=ax"


Differentiating both sides with respect to x, we obtain
l(/W)=^(ay-)
/'W = a
dx
ji-l
fix) = a n X

Putting X = 1 on both sides, we get

ow
/'(I)
/ ’(1) = a M => a n

EXAMPLE 6 Iffix) =x" and iff'(I) =10,/)W the value ofn.


We have, /(x) =x".

e
SOLUTION

Fl
re
Differentiating both sides with respect to x, we get/'W^nx" \

F
Putting X = 1, we get [v /'(I) =101
ur
r
/'(I) =n => 10 = n
CXAMI'1.E 7 Iff(x) = mx + c md f(0)=f'(0)=l. WImt is fail
fo
ks
SOLUTION We have,
Yo

fix) =mx + c
oo

Differentiating with respect to we get ...(ii)


B

/'(x)=m-l + 0 =>f'ix)=ni
re

Putting X = 0 in (i) and (ii), we get


/(O) =c and/'(0)
u

[V /(0)=/'(0)=l]
ad

1 = c and 1 = m
Yo

Putting the values of m and c in fix) = mx + c, we get fix) = x + 1.


[Puttingx = 2in/(x)=x + l]
/(2) = 2 + l=3.
d
Re
in

EXERCISE 18.2
F

Differentiate the following functions zvith respect to x; (1 18) fllogrt


,a log X
1. 3-"+x^ + 3^ 2. y-2V^ + ^ 3. e X log a + e + e

1 ^
6. X + -
4, (2x^ +l)(3x + 2) 5. log 3 X + 3 log^. X X / \

2x^ + 3x + 4 (x^+l)(x-2)
7. +^ 8. 9. =7
X x^
V ●'JX J
10. (7(1 x” + (J] x”'^ + 02 X-""^ + ... + x + a„.
n.
(x + 5) (2x^-1) 12. log
1
+ 5x" - 3a^' + V +
X

+ x^ -1 at X = 1.
13. Find the slope of the tangent to the curve / (x) = 2x^
18.20
APPLIED MATHEMATICS-XI

du f y
14. = + ^ prove that 2.ti/-^= —
a

a
' dx U .V
13. Find the rate at which the function/ (a) = .T
4 3 2
-2x + 3.V +>: +5 changes with respect to A.
2a-9
1ft. Ifi/=_
● 3
^/
7
+ 6,v3 - X, find
d\j atx=l.
dx

17. If for/ (X) = X a-2 + m X + 12, /' (4) =15 and/' (2) = 11, then find and p .
^.100 _^.99 x2
18. For the function /(x) = +...+—
+ X +1. Prove that/' (1) = 100 /' (0).
100 99 2

ANSWERS

ow
I- 3-Mog3+3x2 2. x^ -X -10 x""^ 3. ^7'^ log n + ^7x'’ ^
4. 18 x^ + 8 X + 3 5. 1 ^ 3
X log 3 X
ft. - x^^2 +
12 A--1/2 _ i2 3/2_3 _^_5/2 7. i2 ' 3 ^^_5/2 + -x“ 3/2

e
2 ' 2 2 2

re
Fl
2
8. 2-^
‘J. 2x-2-\ + x3
4
x2

F
x~

10. ;//7q y' 1 +(/7-l) ji~2


ur
a^x 11. 4X + 10 + —

r
x-2
12.
2x
+ - 3(1^ log (1 + ~
3
X'
1/3
^,y“7/4 fo
ks
2
13. 16 15. 4x''^-6.x^ + 6.V + 1
Yo
16. 18
17. A = l,p =7
oo

18.4.1 PRODUCT RULE FOR DIFFERENTIATION


eB

THEOREM 1 Iff(x) mid g (x) are’ two differentiable functions, show that f(x) g(x)
such that is also differentiable

flf(^)g(x)]’=f(x)flg(x)}+g(x)
ur

d
ad

dx
Yo

i.c. Derivative of the product of two functions


- [(First f„nction) x (Dvrivative of 2nd function) + (Second function) x (Derivative offirst function)]
d

I'KOOI Since/ (j) and g (x) are differentiable functions. Therefore,


Re
in

£(/W) = /ta and/te(.v)) = lim


F

h dx h^O h
...(i)

Let <l>(.x) =/(A-)g(x).Then,

^(<j>(x)) = lim
h-*0
(|>(x + /0-(J)(x)
h

=
lim fix + }i)g{x + h)-f{x)g(x)
dx It^O h

dx
!^(4>(x)) = /i->0
lim / (.V + h) g(x + h)-f(x + h) g (X) c-f{x + h) g (.r) - f (X) g (.t)
h

[Adding and subtracting/ (x + h) g (x) in numerator]


~{^(x))
dx
= /, lim
-> 0
/ (-X + /Q -c? (-X)) + R (x) {f(x + h)-f (,v)|
h
DERIVATIVES 18.21

f{x + h){g{x + h)-g{x)} g {x){f {x + h) -/ (.v)}


f (♦(.V)) =
ax
lim
/i->0 h
+ lim
h^O h

/ (.X- + h) -f (a-)

u
g{x + h) -g{x)
lim f (x + h) ● lim + g (x) ● lim h
dx h->0 ' h^O b //->0

4- (4. (.V» = / (A) 4- t? (All + g{x) 4-[f (A)|

F
dx

o
dx dx
/(x) is differentiable.
It is continuous, and hence lim /(x + h) = f{x)

s
h-> 0

4-dx if (■'●) s (^-)I = / (A) dx et (A)I +gix)4-1/ (A)|.

o
Hence, =/ S (^) differentiable and

k
F
dx

K]'MARK The above result may also be expressed as

r
l
(/?)' = /' ^ + /^ or, ifg)' ={fg)

Y
o
f S

o
It can be generalized for the derivative of the product of more than two functions as given below

o
Y
f
if8i^y=w
f 8
B
THEOREM 2 (Generalization of the product rule) Letf{x), g {x),h (.v) be three differentiable functions.
Then,
r
d

£ {/ (-V) ^ (-V) h (x)}


= I ^ (/ {■ '■) I 8 {^) (x) + f (-V) ^(^(■v)) I /j(.v)+/(-v)^(-v)|
u

d
^(h{x))
e

dx
n

iMiOO) We have,
o

f{f{x)g{x)b{x)} ^[1/(.V)?(A)W/(A)|
i

dx
ad

= {/'(A) <? (A)} £ ('> (All + h (A) £ {f (A) ^ (A)}


F

{f (x) g (x)} ^ {b (x)} + h (x) / (.V) j- \g (x)l + g (x) ^ 1/ (.v)j


Re

ILLUSTRATIVE EXAMPLES

HXAMm: 1 Differentiate thefolhiuingfunctions with respect to x:


(i) (x + log x) (ii) .v” log,, X t’,.v
SOLUTION (i) We have,
4-
dx
|f^' (x + log x)l
C-'^' 4~
dx
(X + log x) + (X + log x) 4~
dx
(f’' )
=
+ (x + log .y) r''
18.22 APPLIED MATHEMATICS-X!

1 1
=
1 + -
X
+ {x + log x) 1 + + X + log .t
X

(ii) We have.

^ f » 1

||:(.v")|(log„.v)."+:t”| 4- (log,I ●'■) 1 dx


loga -V dx

= « .y" ^ logrt .r +X ji
+ x" (log,, x)
Y logc a
1
= y" ^ ■ n log,, Y + + Ylog„Y ●

w
log,, a
EXAMPLE 2
Differentiate die following functions ivith respect to x:

F lo
(i) Y^ e-^+(Y^+l)(Y-l) (ii) Y^log2.Y + Y^ 2'^

SOLUTION (i) +(y^+1)(y-1)|

ee
Fr
= dx e^) + -^{x^
dx + l) (Y-1) for
ur
(Y-1)
s

= {e^ X 3y^ + Y+ {(y -1) 2y + (y^ +1) x 1)


ook
Yo

= (3y^ e-^+ x^e^)+(2x^ 2y +Y“ +1)


eB

= (y^ + 3y^) e-"+(3Y^-2Y + l)


d
(ii) (x^log2Y+y^2^‘)
r
ad
ou

dx
d
(x^ log2X) + ^(x3 2-')
Y

dx dx

£(X^) log2X + x’T(log2X)|+| -^(y^)-2"+y^—(2^)


Re
nd

dx dx
Fi

y2
= 2Ylog2Y + + 3y- 2-'' + Y^2Mog,.2
Y log, 2

log, 2 + 3Y^2-'' + Y^2'''log,2


= 2y logo Y + —-—
^
EXERCISE 18.3

Differentiate the following functions luith respect to x: (1-22)


1. x^e^ 2. x^ e'^ log Y 3. Y"log„Y
4. Y''’ + x^ log Y 5. log^2 6. y^(3-6y^^)
7. y“‘^(3-4y“^) 8. y“ ^ (5 + 3.y) 9. (flY + b) /(cx + d)
10. (ax + bf'(ex + d)
III
18.23
DERIVATIVES

11 . Differentiate each of the following functions by tlie product rule and the other method and
verify that answer from both the methods is the same,
(i) {3x^ + 2f (ii) {x + 2){x + 3)

ANSWERS

II-1
1
1. x^ {3 + x) 2. xe-’^ (1 + X log X + 2 log x) 3. X n log,, X +
log a

6. 15x^ + 24x ^

ow
4. x'^ (5e^ + xe"'^' + x + 6x log x) 5. 0

nd - be
7. -12x"^ + 36x
10
8. -15x"'^ -6x ^ Q.
{cx + d)^

e
^|mc {ax +1?) + na (cx + d)]
>1-1 i» -
10. {ax + b) {cx + d)

re
18.4.2 QUOTIENT RULE FOR DIFFERENTIATION
fix) ● ,

F
Frl
—— IS also
THEOREM If fix) and g{x) are two differentiable functions and g (x) ^ 0, then show that g{x)

differentiable and
ou
osr
d
gix) [f{x)]-f{x)~ffg{x)}
±\ m dx
kf
dx \ g{x)
oo
PROOi: Since / (x) and g (x) are differentiable functions. Therefore,
Y

d f{x + h)-f{x) and, —(g(x))


d g{x + h)-g{x)
B

= lim ...(i)
— (/(x))= lim h dx li -> 0 h
dx h^O
re

fix) .Then,
uY

Let(|){x) =
gix)
ad
do

dx

(^{x + h)-i?{x)
in

lim
Re

h 0 h
F

lim
g{x + h) g{x)
h 0 h

- lim
f{x + h)g{x)-f{x)g{x + h)
g ix) g {.X + h) h
= lim
f (X + h) g (X) - / (x) g (x) + / (x) ^ (x) - / (x) g (X + h)
h^O bg{x)g{x + h)
[On subtracting and adding / (x) g (x) in numerator]
lim
g (X) \f{x + h)-f (x)j -f{x){g{x + h) - g (x)]
h^O hg (x) g (x + h)
1
f{x + h)-f{x) g{x + h)-g{x) lim ^ —
lim g(x) - lim / (x) X

h-^0 h /i^o h h-^0 g{x)g{x + h)_


18.24
APPLIED MATHEMATICS-X!

/ (.V + h) -f (x) g{x + h)-g{x) 1


= ^(.y) liin -/(y) lim X lim
/j->0 h h-*0 h
h~^Q g{x)g{x + h)

1
g (y) is differentiable.
It is continuous, and hence
fe(.v)? lim g{x + h)=g{x)
/i->0

^(●v)

few?

Hence, is differentiable and — ■ ax {fix)}-f{x)^dx ef(.r)l


g- ( v) ' dx I g{x) Ig ix)f
\ 1 \ KK It is adi'isnblc to reineinbcr this result in thefollozving fiorm:
d\N']
— < llD^) , zvhere N'" = Numerator, = Denominator.

EX.4M1’LI: 1
(i) ^
dx

2y + 3

Y^ -5
o'"

F low
ILLUSTRATIVE EXAMPLES

Differentiate the follozving functions luith respect to x:


(ii)
Y + 3

y2 4-1
for Fre
SOLLfTlON (i) Using quotient rule, we have
d lx 4- 3 (y^ -5) dx^ (2y4- 3) -(2y+ 3) dxd (x^ -5)
Your
eBo ks

dx X ^ -5 (x^ -5f
(Y^-5)(2)-(2Y-f3)(2Y) - 2 (y^ + 3y 4- 5)
ad

ix^ -5)2 (y2-5)2


our

(ii)
d x+3 'I = (x^+1) dx^ (y+3)-(y+3) dxf (y24-1)
Re

dx ,v2 4-1 (.v2 4-1)2


_ (.y2 4-1)1-(y+3) (2y)
Find Y

- Y 2 - 6 Y 4- 1 1 - 6 y-y2
(y2 + 1)2 (y2 4-1)2 (y2 + 1)2
EXERCISE 18.4
Differentiate the follozving functions zvith respect to x:
1. ^ X ^ 3.
X + e^'
Y + 1
■ y2-.1 1 + log Y
4.
1 y-2 - a: 4-1
3.
6.
^7Y 2 4- 1?Y 4- C 1 4- y2 y-2 4- y + 1
sla + fx 1 + 3-'^
8. i + iog
■Ja -yfx 1 - 3 '' 1-logY
18.25
DERIVATIVES

10.
px^ + qx + r n.
ax + b
12.
ax + b px^ + qx + r “ 4- bx + c

ANSWERS

1.
x^ + 2x -1 , 2(l+.v-.v^) ^ -V log X ● (1 + " L’^ (1 - -V)
(A- + l)^ (1 + x'^f x{l+ log xf
-{lax + b) „ e-^(l-xf 6.
2{X^-1) 7.
4. 3.

{ax~ + bx + c)^ (l+.v2)2 (x^+x + l)- 2

2 ● 3-^' log 3 2 apx^ + Ihpx + bq -ar

w
8. 9. 10.
(1 - 3-'')- .Y (1 - log -Y)^ (fl.v + b)^

- apx^ - 2bpx + ar - bq -{2ax +b)

Flo
11, 12.
(px^ + qx + r)^ {ax~ + bx + c)^

e
re
HINTS TO SELECTED PROBLEMS

F
d (px^ + (y.Y + r \ {ax + b) ^ (px^ + qx + r) - {px^ + qx + r) ^^ {ax +1»)
ur
r
dx
10. fo
dx ax + b (rtY + b)~
ks
{ax + b) (2 px + q)-a{p x^ + qx + r) _ ap x^ + 2b px + bq-ar
Yo

{ax + i?)^
oo

((7.Y + b)^
eB

29.
d ax + b {px~ + qx + f) ^ +b) - {ax + b) {px^ + qx + r)
{px^ + qx + r)-
ur

dx
px^ + qx + r
ad
Yo

a {px^ + qx + r) - (f7.Y + b) (2 px + q) _ - apx^ - 2 bpx + ar - bq


{px^ + qx + r)^ {f)x^ + qx + rf
d
Re
in

18.5 DIFFERENTIATION OF A FUNCTION OF A FUNCTION


F

In this section, we will study about the differentiation of composition of two or more functions.
THEOREM (Chain Rule) Iff(x) ami g(x) are differentiablefimctkms, then fog isatsodifferentinbleand
ifos)'{x) = f'{g{x))g'{x)
d

£j(/0.)W}
d
or. {fog) (-V) gix) ■
^ g (x) dx

Vi^i X '1 Since/ (-Y) and g (.y) arc differentiable functions. Therefore,
d /(■y + /Q-/(.y)
{fix)) = lim 0 h
dx
...(i)
g{x + h)-g{x)
-^(sW)
dx
= h
lim
0 h
18.26
APPLIED MATHEMATICS-XI

Now, lim fog {X + h) - fog jx)


h->’0 h

lim f{gix + h)} -f{g{x)\


h 0 h

^ I (fog) (x)
dx
lim f\g(x + h)] -f\g (.y)! _ g {X + h)-g{x)
/i^O g(x + h)-gix) h

lim f{gix + h)}-f\g{x)} X Urn S(x + h)-g{x)


h-^0
g{x + h)-g{x) i;->0 h

^(fog) (a-)|
d
lim f \g (x + h)\ - f {g (x)} X lim g(x + h)-g{x)
dx
g{x+h)^g(x) g(x + h)-g{x) /?->0 h

w
●●● g (a-) is differentiable, g (a) is continousand hence lim g{x + h) =g (a)
h-*0

£{(/»?)«} ^{w(.v)}x£{jw}
Flo
ee
Q.L.D.

Fr
The above rule can also be restated as follows:
If z=f{y) and y=g{x), then for
ur
dx dy dx
OR
s
Derivative of z zuith respect tox = (Derivative of z ivith respect toy) x (Derivative ofy with respect to x)
k
Yo
oo

This chain rule can be extended further.


eB

Derivative ofz with respect to x = (Derivative ofz with respect to u) x ( Derivative ofu ivith
respect to v) x (Derivative ofv ivith respect to a)
r
ou
ad

ILLUSTRATIVE EXAMPLES
nXAMPLE 1
Differentiate the following functions with respect to x:
Y

1
(i) (a^ 4- a + 1)^ (ii) Ja^ + A +1 (iii) , _
Re
nd

^2 _ _^.2
SOLUTION (i) Lety = (a^+a + I)*^. Putting a^ + a + 1 = k, we get
Fi

1/ = and u ■ = A^ + A +1
dy ,3 , du = 2a + 1
du dx

Now, dy _ dy du
dx du dx

=>
dx
= 4 u^ (2a + 1) = 4 (a^ + a + 1)^ (2a + 1).
We have,
d
dx
l(.v^ + i-+ 1)-*1 = {(a^ + a + l)"^! X (a^ + a + 1) =4 (a^ + a + 1) ^(2a + 1)
d (a^ + a + 1)
(ii) Let y = + a + 1. Putting a^ + a + 1 = li, we get
18.27
DERIVATIVES

y = -Jii and u = + .v +1
1
-1 1 du
du 1 , and — = 2.V +1
du 2 dx
i4ii

Now,
d\/ _ dy dll
dx du dx
1
^ = JL- (2-r + l) = X x(2.v + l) [■: u = x^ + x + 1]
dx 2
2 -J.Y^ + X + 1
We have.
d
d
+ x + \ {x^ + x + )V^^'^ X

dx
(.v^ +X + 1)
dx d {x^ + .\; + 1)
1/2

w
- (j- + .t + 1) (2a- +1)
2
1
. Putting u=a^ x^, we get

F lo
(iii) Leti/ =

1
and u = a^-x^

ee
= u
y =

Fr
du
rfy ^ and — = - 2.a.
du 2 dx for
dy _ dy ^ ^
ur
Now,
dx du dx
s

1
[v u = a^ - x^]
a
ook

(-2.v)=- X
(-2a) =
Yo

X
3/2
dx 2 2u
eB

d d
M lllJl (.2 -a2)-l/2 X — a 2 -,v2
dx d{a^-x^) dx
our

x-3/2
ad

1 ( X
(0-2a) =
2 («2-x2)3/2
EXAMPLE 2 Differentiate thefolloioingfunctions with respect to x:
Y
Re

X
nd

(i) (ii) logy (logy X) (iii) log.v 2


Fi

,.v
SOLUTION (i) Leti/ = e‘ . Putting =», we get
y = L’" and u = (’■'

and = r'-
du dx

Now, f?y - ^ X —

dx du dx

^ = e">c e^ = X r" [v =
dx

^ (e^‘) - e*’ X
f . \
d ,x d
\im:K — e X

dx\ dx

(ii) Let 1/ = logy (logy a). Putting u = logy a, we get


1/ = logy u and u = logy a
18.28
APPLIED MATHEMATICS-XI

1 (ill 1
and
du II logj, 7 dx X log^, 7
Now,
dx du dx
di/ 1
X
1

dx u logj, 7 A- log,. 7
] 1
dx
log7 a; X log,. 7 X a: log,. 7 [●●● U = log7 x]
A- (log7 A-) (log,, 7)2

^{log7 (log7A')i = d (log? x)


d
-^(log7 X) =
\r ri I; 1 1
log? (log? X) X X
dx
(log? x) log,, 7 A log^ 7
1

A (log? A) (log,. 7)2

w
1
(iii) Lety = logY 2.Then, y =
log? A log/; « = 7^-r
log„
Putting ii = log? A, we get
y --u and u = log? a
F lo
e
Fre
iL ~ 1 du 1
dll -^and—
ir dx
=
X log, 2
for
Now, dxj _ dy dll _ 1 1 1 1
dx dll dx 2 A log, 2
X
[●●● II = log? a]
(log? a)2 A log, 2
r
II
You
oks

\n 11.:, Using chain rule, we obtain


eBo

^(log,2) = 4-
1 d 1
4- C«g2 =
1 1
X
dx
dx ^ log? A d (log? .y) ^ log? A dx
(log2 X log, 2
ad
our

l'XAMrLI;3 Ify = X + X
2
+ n
21J , tliL’ii prove that ^
dx
= »y
'a2+.2
I!

SOLUTION We have, i/ =
jx-f-^A2 + rt2 I
Re
dY

dy _ d
Fin

dx dx

^ - n-t ^
II X — A + a'2 +
dx dx

^y n
dx

^ -
|a + -^a2 I
d
dx
n
1 + -2 (a2 + '
-1/2 X
(a-2 + ^2)
dx

^ -
|a + -J?+ (t2 j
1
X n + X 2a ●
dx
2 + r?2
18.29
DERIVATIVES

-V
_

n X + ..2+ .2 u +
dx
n

n
n-1 x^ + + .V ny
^^y _ n X +
2 2
dx x~ + a

EXAMi’Lt 4 Differentiate the following functions with respect to x: ^

ow
(i) log;io-v +log;^. 10 + log;!-.i: +logio 10 (n) 5^
SOLUTION (i) Let 1/ = logio + *'^810 10. Then,
1 -1
+ 2.
1/ = logioA.- + - + 1+1 = logio^' +(logic

e
log 10 .V

re
Differentiating with respect to x, we get

Flr
iL = 1

F
+ (-1) (logic -'■) (l^Sio ''■) + 0
dx X logj, 10
=
1
-(logic
ou
X
1

sr
dx xlog^, 10 X log,, 10

fo
1 1
cfi ^ 1 X
=>

dx .V log,, 10 x{log|o--f)^ log,. 10 k


oo
1
dp _ 1
^l^gc-lO
dx X log,, 10 x(logio-Y-log,, 10)
Y
reB

1
dy ^ I 2 logf
dx X log,. 10 x(log,. x)
uY

-,x2
(ii) Let 1/=5^ + (3-x2)5.
ad
do

Differentiating with respect to x, we get


‘M = A(53-'-2) + i- (3 - x~)
2.5
in

dx dx dx
Re

53-^2 log,. 5 X — (3-J-^)+5(3-.v^)-’ ' 1^(3-v^) X


F

dx dx dx

dx
- .V
log,. 5 X (0 - 2x) +5(3- x^)*^ X (0 - 2x)
^ = -2xl5^"-^'^og,5 + 5(3-x¥| .
dx

, show that ^ ^ ~t7


+ n~
IXAMI’ll >
.1 A
- X
a~ + X

SOl.UTION We have.

+ X + X - X
^X
+ x^~ + yja^ -x^
y =
^+X V? + - v - X
^ ^+X
18.30 APPLIED MATHEMATICS-XI

+ a^ -x^ + 2-J
n + X 9
a~ + a
2 ,.2
-X
y =
{a^ + x^)-{a^ -x^) 2a-2

=>
y =
2n^ + 2^a-^ -x^ (7
2
+ .2.V-2 +
2x2 x2
9
.V

rfl/ 2 d , -2^ ci 4
-,V^ X-2
-^
dx
= — (.V + ,
dx dx

=>
dx = _ 2,2 ;,-3 + (_ 2) x-3 ^/7I7 4- (x-2) 1 (,^ - .v^)-V 2 , dx
(,4 - X
4

‘Jy - 2,2 2
V7^
1
_(-4x-’)
rfx .t3 ? 2x2 4
-X
4

dy 2,2 2 2x
-x^ -
x3 ^

w
rfx
- x^

fS/ - 2,2 -2^


rfx x^ x-3
+
X

F lo
e
iL - 2,2
4 4 4
, -X + X^

Fre
-2i
dx x^
x^y-x^
for
=> ^y 2,^ 2,2 ,
2

dx x^
r
X 4 4
, - X
You
oks

1 -X

1 + X ' prove that (1 - x2) + |y - o.


ify =
eBo

dx

-X
SOLUTION We have, y = -
ad
our

1 + x

Differentiating with respect to x, we get


xl/2-l
dy _ 1 f 1 ~ X
Re

d 1 -X
dY

dx 2 1 +X rfx 1 1 + X
Fin

d d
(1 + x) (l-x)-(l-x) (1 + X)
^ = i /l±f X dx dx_
dx 2 VI-X
(1 + x)2
ii - I iii„(i + -4(-i)-(i-x)(i)
X
dx 2 7 -X (l+xf
=> ^ - 1 1^ ' '^ -1 - X -1 + X
dx ~ 2 VT^ "" (1 + X)2
^ - 1+X
X
1
dx
1 -X (1 + ,v)2

(l-x2)^ =
1+x
dx
1 - X (1 + X)
^d-x")
18.31
DERIVATIVES

+ 1/ = 0
dx P + .X dx dx

1 + dy
lfy = , shoiu thnt
p-e^ dx ,2x

SOLUTION We have, \j =
P'P
1 -e^

Differentiating both sides with respect to x, we obtain.


-Y \
d l + c
dy _ I 1 + ^ X —

Ix ~ 2 dx 1 -

w
1
^/ = 1 X
dx dx

dx 2All + t’-^ (1-^V

^ _ 1
dx
1 - g"
2^1 + g '^ "
(1
(l-e¥
+ (1 +

F lo 1_ i-e^
2p+e^
X
2e^
Yn2
a-e-^)

e
Fre
1 - g-'‘ 1
= g-" X
for
t/.v l + g'^ (1-rV +£■'■
r
g-" g^
^y _
You
oks

cf.t (1-g-^; 'i-g2-^-


eBo

EXERCISE 18.5

Differentiate the folloioing functions u>ith respect to x (1-25):


,Y
our
ad

3‘ lOgY 3

3-''^ + 2y gY log Y
^ (7^ + X~^
dY
Re

l-x2 1 + .Y
log (:r+ ■/?+!)
Fin

f+ .v^ 1 -Y

-2y Y^ + Y + 1
g"''' log Y + g
log ^
y2 ,2y_,-2y Y -Y + 1

-j.V“ + 1 + -^Y^ -1
1/.

Jx^ +1 - p -1
.2 log |y + 2 + Y + 4y + 1

- 3y
3g log (1 + y)
-1
If y = log 7^ 7y + 1 -, show that iY
2^ 1

If y = 7-y + 1 + p -T, prove that T^*y = ly


dx 2 ^
18.32
APPLIED MATHEMATICS-XI

Ifi/ = — , prove that .t =(i-y)i/


x +2 ^ dx

1
, prove that ^ =
-Y-1
If 1/ = log Vj +
dx 2x (.V +1)

- Ify = X +
\= , prove that 2x^=4x
dx
- 1

e^' - e ^

w
2\. Ify =
c
.V
+ e
-.V ' prove that ^
dx
=1-
x-1
- If y - I) log (.Y -1) -(x + 1) log (x + 1), prove that=log

e
dx 1 +x

ro
re
If y = ^x^ , prove that 'i/ dx
x = 0

F
'■i. If y = yjn'^ -x^, prove that i/~ + X = 0

Fl
dx

If x:/ = 4, prove that x ^ + ■i/^ = 3y

u
sr
(fx

ko
AWSlVEffS
o
3x^ ■ log 2 ●■ 3‘
,.v
log 3 ● e
of 1

xlog, 3(log3 x)2


o
Y
.v2+2.v 2n^ X
● (3 log 3) (2x + 2) .S.
3-’^'°S-^'{log 3) (1 + logx)
erB
uY

2x 1 1
3/2
.JTT^H-X)
2 r 1 2 -8
ad
do

X
log X + — log X n.
2x
,-2,V)2
X X
in

2(x2-1) 1.^. 2x +
2x-3 1
i :
x"l + X^ + 1
Re

-\/x^ + 4x +1
F

- 3.V 1
. 3e
x + 1
-3 1og(x + l) ■

18.6 RELATION BETWEEN 'fy


dx
AND
dx dy
Let X and y be two variables connected by a relation of the form f{x, y) = 0. Let Ax be a small
change in x and let Ay be the corresponding change in y. Then,
lim Ay , dx Ax
and, — lim
dx A X -> 0 Ax A 0 Ay
Ay Ax
Now, -^x — =1
Ax Ay
lim
Ay Ax
= 1
Ax->0 Ax Ay
18.33
DERIVATIVES

Ax
lim Urn [●.● Ax ^ 0 <=> Ay -> 0]
^ V^0 A.V A 1/ ^ 0 Ay
du dx
-=^ X — =1
dx d\/

dy ^ 1

dx dx/dy
18.7 DIFFERENTIATION OF IMPLICIT FUNCTIONS
Up till now we have discussed derivatives of functions of the form y = / (.v). If the variables a and
yare connected by a relation of the form/ {x, y) = 0 and it is not possible or convenient to express
to be an implicit function of x. To find
m
a function x in the form y = (J) (.v), then y is said

w
y as dx

such a case, we differentiate both sides of the given relation with respect to .v, keeping in mind
d ^ dy
that the derivative of (|) (y) with respect to x is
dy dx

o
dy d . 2. , dy
4- './) =

e
For example cosi/--p, —
dx
(y ) = 2y—
dx

re
dx ■ dx

It should be noted that


A

rFl
d
— (sin v) = cos 1/ but — (sin y) = cos y ●
^ dy

F
dy ● ● dx dx
Similarly , A(y) = 3ywhereas/(y') = 3/|.
r
ou
fo
ks
ILLUSTRATIVE EXAMPLES
oo

EXAMPLE 1 Ifx^ + 2xy + y^ = 42, find ^


Y
eB

SOLUTION We have,
x^ + 2xy + y^ =42.
r

Differentiating both sides of this with respect to .x, we get


ou
Y
ad

dx (x^) + 2^(xy)
dx + ^{i/)
dx dx
(42)

lx + 1 x^ + y + 3 i/^ ^ =0
d


dx ' ‘ dx
Re
in

2x + 2y + ^ (2x + 3 i/“) = 0
F

dx

dx
[lx + 3 \/) = - 2 (x + y)
^ - 2(x + y)
=>
dx
(2x+ 3y^)
EXAMPLE 2
Ifx^ + y^ = 3rtxy, find ^
SOLUTION Differentiahng both sides of the given relation with respect to x, we get
^(x^)+4-{y^) = 3fl-^(x-y)
dx dx dx

3x^ + 3i/^ ^
dx
= 3rt ■ [ X ^dx + y
18.34
APPLIED MATHEMATICS-XI

(2>y^-3ax)^ = 3ay - 3x^ ax

3{i/ -ax)^ = 3{aij-x^) dx


2
dy _ ay -X
dx 2 ●
y -ax
/ \

rXAMPLLS //log(A-2 + y2)=2tan 1 y


, show that ^ = ^ ^
\x) dx x-y
SOLLITION
Differentiating both sides of the given relation with respect to x, we get

11'"«
d
= 2 — { tan 1 y

ow
dx yx)
1 1 d
X 2x y
-v^ + y^ dx
l+(y/xf ^

e
x‘‘^ -yx

re
2
1

2 ^(^ > + -(y)


^ / 2v d , 2. = T2x_a:

rFl 2
dx
1

F
dx 2
.r +y X

r
2
■ + 2y dy
ou
x^ -y ■
^^+y^ dx
.T^ + y^ dx fo
ks
^jx + y ^yrf.tj = 2 x-^-y
dx
^y
oo

dy dy
Y

X +y = X -y
eB

dx dx
^y
^ (y~x) = ~{x + y)
r

dy _ x + y
ou
ad
Y

dx x-y

nXAMPLC4
UX -^1 + y + y + X = 0 flnd a: y, prove that ^ = 1
d

dx
(-V + l)^‘
Re
in

SOLUTION We have,
F

x^l^ + y,JiT^ = 0
x.yrT7 = -y^r + X

A-2(l+y) = y2(l+;,) [On squaring both sides]


2 2 2 2
X -y = y x-x^y
(x + y)(x-y) = -xy{x-y)
x + y = - xy
(v x^y]
^ = -y-xy
y (1 + x) = - X

X
=>
y = -
1 + X
18.35
DERIVATIVES

dy _ _ (1 + a:) X 1 - .V (0 -i-1)
^
dx ■ (1 + xf
1
^ -
dx
(1 + -v)2
EXERCISE 18.6

Find ;
III each of the following (1-11):
dx
.2/3 , .2/3 .2/3
1. xy = ^ 2. i/^ - 3xi/ + 3.r^ y 3. .X = a

2 2

4. 4.V + 3i/ = log (4.x - 3y) 6. .x^ + \/ = 5 xy

ow
a b

7. {x + i/)^ = 2axy /
s. {x^ + i/y
9. = log -
y

e
K). If.XI/=1, prove that ^ + 1/^ =0.
Fl
re
F
11. If.xy^ =l,provethat 2 ^ + 1/ 3=0.
ur
^^4-1=0.
12
If .X ^1 + y + 1/ + -X = 0, prove that (1 + .x) dx
or
sf
y (x^y + x + y)
13. If .XI/ log (.X + y) = 1, prove that ^ = - .X {xi/ + X + y)
k
Yo
oo

14.
If y xjx^ + 1 = log -1^ - -X , show that (.x" + 1) ^ + .xy + 1 - 0.
B
re

dy dy
or, ~^ + e y-^=o
15. If e' + e^ = prove that dx
dx (e^-1)
u
ad
Yo

ANSWERS

1/3
7 ^= (.x + y)^ 3. -y
d

y
1. -
Re

2 2
dx X
in

X y -2.xy-.x
4
F

4 (1 - 4 X + 3y) 5. 4
2
3 (4 X - 3 y +1) «y y

ay-x-y 8.
4x(x^ + y^)-y (J y (xe^~^-l)
x-y
X + y -ax x-4y{.x^ +y^) X (ye -1)

HINTS TO SELECTED PROBLEMS

10. We have,
1

Ay = 1 => y ^ ^1 ‘'y =-i. Therefore, ^


dx + y2
^ +
2
dx X X

11. We have,
3

xy 2=1 =. x=\
dx 2
3
^'=-L
dx 2
=> 2^^
dx
+ y2=0
y dy y
18.36
APPLIED MATHEMATICS-XI

18.8 LOGARITHMIC DIFFERENTIATION


We have learnt about the derivatives of the functions of the form [f (.v)]", n /(.V) and ii \ where
/ (.v) is a function of .vandjiis a constant. In this section, we will be mainly discussing derivatives
of the functions of the form where /(.y) and g(Y) are functions of y. To find the
derivative of this type of functions we proceed as follows:
Lety = [/(Y}] . Taking logarithm of both the sides, we get
= .?w-iog {fix)}
Differentiating with respect to y, we get
= gix) X ^
d
fix) +\og{fix)\.-^(gix)

ow
V fix) dx dx

six) d
dx ' fix) dx - fix) +\og{fix)}-j~^gix) ■
Alternatively, we may write

e
Fl
re
lf = [fix)f^-'-''^ fog fi^))

F
Differentiating with respect to .v, we get
ur
dx
g(x) ■
1 d f

fix) dx or
-fix) +\og{fix)}~(gix) ■
sf
dx
k
Yo
dy
f (x) \+log {fix)} ~(gix)
oo

dx fix) dx dx \
B
re

ILLUSTRATIVE EXAMPLES
EXAMPLE 1
Differentiate the followingfunctions with respect to
u

x:
ad
Yo

(i) Y^ (ii) Y
SOLUTION (i) Lety = y"'. Then,
d
Re

^Y.logY 1 ^
in

y = r ^
[●●● a
On differentiating both sides with respect to y, we get
F

dy X \ogx d

dx
= c ^ —
dx
(y log y)

(log .V) [v s Xl

dy 1
dx
= y’ log Y + Y X
X)
^ -
dx
Y'^ (1 + log y)
(ii) Lety = y^. Then,
y = ^Jx log .T
18.37
DERIVATIVES

On differentiating both sides with respect to .y, we get


iL
dx
^ ,V.vlogI ±{4x\0,fx)
dx

dx
_

Y
4x
■ logY
) dx
4- + '1x4-
dx
(log x)
'i 1
logY +
dx 2^ X

+
1 ]
dx 2y[x yfx

ow
UXAMPLE 2 Differentiate the folloioing functions with respect to x:
X
: 2(x+3)
(i) Y^ (ii) (-r")" (iii) Y'^ e
SOLUTION (i) Let y = x . Then,

e
X
X ●lOgY

re
y = ^
On differentiating both sides with respect to y, we get

rFl
F
X

it = e X ●log I
-^{Y^log Y)
dx dx

or
" A (.' ‘°S 'log.v)
ou
X
Y
dx dY
ksf
log,vA(/>“S") + f X log X 4^
dx (log x)
X
dy _ Y
X X

dx dx
oo

X log X d
— (y log y) + e X log Y ^ 1 .
X
*/ = X
log Y. e
Y

dx X
B

dx
X 1 X 1
dy _
● (log Y) Y^
X
Y X - + log Y + Y X —Y 1
re

Y
dx X
oYu

X
ad

dy Y x^
-A
dx
= 1 Y
(1 + log x) log Y + y
d

dy _ X
x^ X
|(1 + logY) log Y + ^j
in
Re

dx

(ii) Let y = (Y^)''^.Then,


F

^X^ . log X
2
= Y^ 1/ =
X. X
1/ = Y
On differentiating both sides with respect to y, we get
it
dx
^ ''“S'A
dx
(^2 log.V)
^ ^ ^,x^.logx
dx
log Y 4-
dx
(x^)+-Y^ 4-
dx
(i«g
2

|(logx)2.x- + ,rAl|
logx = Y X 1]
it - X
dx
2
^y
Y'^ (2X log Y + .V)
dx
2
^
dx
= Y.V^ (21ogx + l).
18.38
APPLIED MATHEMATICS-XI

(iii) Let]/ = ;»:^^2(.v+3) . Then,


y = ^.rlog.v_^^2(.v+ 3) [●.● .r
A‘

y = ^jrlog.v+2(.v+ 3)
On differentiating with respect to a, we get
_ ^.YlogA+2(A+ 3) — d {.V log a: + 2 (.V + 3)}
dx dx
^y =
dx
3)
4-
dx
(x log x) + l4dx (-Y+3)
^.,^,2(.v.3) (1 +log A-+ 2) = {3 + log x)
dx

ow
log ,v
EXAMPLE 3 Dfiferentiate: (log x)"''' + X
u’it/i respect to x.
SOLUTION Let y = (log x)^ + x^°S jhen.
^log (log log(.v‘‘^S^) = ^.V log (log .Y)

e
y = + e + e log .Y. log .Y

re
Fl
On differentiating both sides with respect to x, we get

F
I= lxlog(logx)K.^^‘^^-^-)%£(logx)^
ur
r
^y
dx - (log X)'' ■ log (log x) X
fo
(x) +xx~ {log (log x)i ■ + x'^’S , 2 (log x) ~ (log x))
ks
Yo
^y 1 1
oo

— = (log x)’’^ \ log (log x) + X x-> + X


-log-Y
2(logx)
1

log X X X
eB

dy 1
log.Y J 2 log X
dx = (log x) ’^ ] log (log x) + log X > + X
ur
ad

I-:XAI^PLE4 7/x^ .Y - 1/
prove that ^ = logx
Yo

= e
dx

SOLUTION We have,
d
Re

x^ =
in

(, y Jog .V ^ ^.Y - y [., = (.‘“S '-''


F

y log X = X - y => y log x + y = x => y (1 + log x) = X => y = X

1 + log X
On differentiating both sides with respect to .x, we get
1
(1 + log x) X 1 - X 0 +
^ - X
logT
dx
(1 + log x)^ (1 + logx)^
XAMPLE5 lfx-'+/ = 2, find dx
OLUTION We have,
x-'^ + y^' =2
> £..v’oS-’^ + t,-^'Jogy = 2
18.39
DERIVATIVES

On differenUating both sides with respect to .v, we get


d d
,y log -V xlogi/
r = ^(2)
dx V dx I dx

^yiog-T — (1/ log -t) + e""


dx ‘
-f
dx
(x log y) = 0

1 1 r , 1 frV = 0
x
y
dx
log y + 1/ X — ■ + y ● 1 X log 1/ + X X — —

X X^
X
X log y = 0

ow
■ x^ log X + X y H^
dx
+Jy ^ + 'y' log y [ = 0
-1
rfy _ + log y .

e
A-- 1
dx
X-'' log X + X y

re
FXAMPLE6 ifx-' -y^, find —.

rFl
F
SOLUTION We have,

or
ou
X
x^=y
Taking log on both sides, we get
ksf
y log X = X log y
Differentiating both sides with respect to x, we get
oo

(log X) + \ogx4-dx (y) = (>°g y) + '°s y -.rnx (-'■)


Y

V
B

dx flX
1 , 1 fry
^ + (log y) 1
re

-X + log X X -y- = XX —
dx y dx
oYu

, dy X dy y
logx -^--y = logy-^
ad

dx y dx
^y
= logy-^
d

log X - -
dx
in
Re

f y log X - X xlogy-y
F

dx y
X

^-1 X log y - y
dx
X l^ylogx-x
dy
Ify = /+e●%x^■ + .v^/mrf^atx -
a.
EXAM PI E7

SOLUTION We have,
a

y = 4-+ x^ + X
a
+ X
=5> y =
X log X d , (I.
) + ^(x )
dx dv dx dx dx
0- 1
dy X log .Y
(x log x) + a X
- \oga + +e dx
dx
18.40
APPLIED MATHEMATICS-XI

rt-1
a
log /I + -f (1 + log a) + a X
-

dx

= a‘^ log a + e" + (i‘^ (1+ log a) +


r?-l

(dx ''x = a
a a
= e" + 2a" (1 + log a)
REMARK In order to find the derivative of a product ofa number offunctions or a quotient of a number

w
offunctions, we first take logarithm of both sides and then differentiate. The procedure is illustrated in the
following examples.

EXAMPLE 8 If \J =
^/i^(2.v-3) 1/2
d\j

e
, find -f-.
{x^ + 2)^^^ dx

e
SOLUTION Taking log of both sides, we get

or
r
logy =
i logd - .v^) + i Iog(2.v - 3) - I log(.v^ + 2).

F
On differentiating both sides with respect to v we get
oF
ul
1^ 1 1 2 1
(-2.v) + x2--x X 2a
y 2(1-.v^) 2(2.v-3) 3 .v^ + 2

rs
dy

k
1

o
A- 4a-
= y ● - +
dx
l-.v^ 2.T-3
3(.v^ + 2)
dy j/r/?(2,r-3) 1/2
of
o
.V 1 4.V
Y
+
dx
(.v^ + 2)2''3 I-a-2 2.V-3
3(a-2 + 2)
YB

3/2
(a + 4)
EXAMPLE 9 Find the derivative of
er

4/3 with respect to x.


(4x-3)
u

3/2
V A- (a- + 4)
d

SOLUTION Let 1/ =
o

4/3 ●
ad

(4A--3)
in

Taking log of both sides, we get


logy =
i log X + I log (X + 4) -1 log(4x - 3)
Re
F

On differentiating both sides with respect to A', we get


3 1 1
y dx 2x'"2x + 4rfx^'''^^^ s'" 4x - 3 dx
^^(4a--3)
^ _ 1 3 4

^ 2x ^ 2 (x + 4) 3 (4x - 3)
X 4
dx

3/2
^ _ yfx{x + 4) 1 3 16
dx 4/3
(4x-3) 2x 2(x + 4) 3(4x-3)

EXAMPLE 10 //A-"'y” = (x + y) , prove that ^ ^X


m + n

dx
SOLUTION We have,
m

■ y" = (x + y)
m + n
X

Taking log on both sides, we get


DERIVATIVES 18.41

m log .Y + n log 1/ = {m + n) log(.v + y)


Differentiating both sides with respect to .v, we get
1 1 till IV+ n d
inx — + nx —- = {X + y)
X 1/ dx X + y dx
IV V
— + — X dy _ m + n ^ fj/
-Y y dx x+y v ,

n m+n dy _ m + n _m

ow
X +1/ dx x+y x

nx + vy - ivy - ny dy _ m.Y + vx -iiix - my

y (X + y) dx (.Y + 1/) -Y

e
nx-my dy _ vx - my ^ ^ _ V

re
1/ (.Y + y) dx (x + y) x dx X

Frl
2

F
ax bx
EXAMPLE 11 Ify =
+ -—h 1, prove that
{X - a) {X-b) {x - c) {x-b){x-c) x - c
ou
b
^ = lX \

sor
a c
+
dx a-X b - X c -x

SOLUTION We have.
2
kf
bx
oo
y =
(7.Y
^+1
(.Y - a) (.Y - b) (.Y - c) (-Y - b) (-Y - c) x -c
Y
B

2
77.Y bx C + X -c
+
y =
(y - (?) (Y-b) (y - C) (Y-b) (Y - c) x-c
re
oY

2
ax bx X
u

+
y =
{x-a){x-b){x-c) {x-b){x-c) x-c
ad

2
ax bx + y(y -1?)
d

y ^
(Y - (?) (Y - b) (Y - c) ^ (Y - b) (Y - c)
in

2 2
Re

ax X
+
y =
(y - (?) (y - b) (y - c) (y - b) (y - c)
F

2 2
(7Y +Y (y-??)
y =
(Y-n) {x-b){x-c)
3
Y
y =
(y-(?)(y-1?) (y-c)
3
Y

logl/ = log-
(y-(?) (x-b) (y-c)

log y = 3 log Y - I log (y - (?) + log (y-b) + log (y - c)|


On differentiating with respect to .v, we get
1 1 1
lf!y = 1-^ -f

y X Y -a X -b X - c
18.42
APPLIED MATHEMATICS-XI

dy 1 1 1 1 1 1
-f
dx
= y +
X X - a X X -b X X -c

rw
y V

a b c
-T = y i -
dx .y(.y-<7) x{x-b) y(.v-c)

=> ^ = y a b
+
c

dx X a-X b ~x x~c

r
EXAMPLE 12
Prove that the derivative of an even function is an oddfunction and that of an odd function

o
is an even function.

lu
F
SOLUTION Let / (y) be an even function. Then,
/(-^) =/(.v)

F
I (/(-.v)i =

s
=>

o
kr
dx
= fix)

oo
=/'(-v)
f'(-x) = -fix)
of
Y
Y
f' (y) is an odd function.
Let / (y) be an odd function. Then,
B

/(-Y) = -fix)
r
de
u

f’(-x)ffx)
dx
= /'(.V)
o
n

-f'i-x) = - fix)
ad

/'(--V) = fix)
i

f '(.y) is an even function.


F
Re

EXERCISE 18.7

Differentiate the following functions with respect to x: (1-18)


.l/.v
1. Y
2. (log.Y)-^' 3. e
,.r log X logY
4. (log y)

ini) +(x-3)^^
1
o.
0. (i) (y'‘‘) fx (ii) Y + — + Y^
Xj
dy
Find-^, (7-11) ivhen
dx

7. y = + lO'^' + Y^ 8. 1/ = y" + f + Y'^ + n


II

{x^-l)^{2x-l)
9. y = 10. y = y'^ + Y^^^
P~ 3) (ix-l)
11. y = ,v‘"s ' + (iog
V
18.43
DERIVATIVES

12. =(.Y+i/)^^’,provethat^=^ ax X

16 9 2 17
13. ifY 1/ = (.r + i/) , prove that y dx
= 2y

-'v^ (1 + log Y) + 1/''^ ■ log 1/


14. If Y'^ + ^1, prove that —
ax
= — <

(.v-1)
●}/

_ .. u X . . fry y (y +log y)
15. If.v-^-y =1, prove that
Y (y log Y + .y)

ow
X + !/
16. If.v-*' + y'^=(.v + y) , find —
dx

17. If.v”* i/' =1, prove that ii-


my
dx ux

e
dy (1 + log y)^
18. If y'=£?●*' prove that

re
dx logy
dy

Frl
F
19. Ife'^ + prove that dx

20. If C-^ = y^, prove that ^


ou
sor
dx logy-1
rfy 1 - -Y
21. If ^ - -V = 0, prove that dx X
kf
oo
22.lixy=e^
^
find dx
Y

23. If y" + -y'' + Y-''


B

= n
dx
X

dy
re

x-y
24. If(Y-y)e = prove that 1/— + Y = 2i/.
oY

■ dx
u

25. IfY=f''^'^^, prove that ^ ^


ad

(7Y YlogY
d

.V
26. If.rl'-y = a
in

dx
Re

ANSWERS
F

1
l/.V 1-log.Y
1. Y
Y
2 2. (log y)^ \ log (log y) + log Y
● 3. Y’’^ (1 + log y)

lO-^'
4. (log -Y)
log -V 1 + log (log -Y) 5. 10 10' (log,, 10)^
Y

2y + 1
6.(1) Y-^/^ 2y
+ log Y

Y^-1 1log f Y + -1
1 +
1 _ log Y _
(ii) Y+ ■ — + ^■ + Y 2 2
Y Y
Y +1 Y Y

f 2 2
(Hi) y'‘ < + 2y log Y ^ + (y - 3) < + 2y log (y - 3) ■
Y
18.44
APPLIED MATHEMATICS-Xl

7. + 10"' log 10 + log (e.v)


H-1
8. nx + log n + log (t’.v)

„ (x^-1)^2,t-1) f L 6x
+
2 1 2

■ p~3)(4x-l) |;,2 -1 2.V-1 2(.v-3) 4a:-1

1/.T 1 - log -V
10. (1 + log .r) + .T 2
A-

log A- J 2 log X

ow
1
n. A
A
+
(log a:)-' \ log (log x) + log A
(A- + .V)
(-■^ + y) {1 + log (A + y)} - i/x^ ^ ~ / log 1/
-v-l
A^ log A + Ay (x + y)
x+ y
{1 +log(A+l/)}

e
re
22.
y(^'-i)
23. ./logy+y^-’^"^ +-v''(i + iogA)
A(y + 1) -V-]
+ A-'^logA

Flr
^■y

F
26.
y^^ogy-y^^ ^
A^ log A-Ay
-v-l
ou
sr
fo
HINTS TO SELECTED PROBLEMS
Lety = (log a)
log A
. Then, k
oo
log |(IogA)*°8-’^|
Y
y = (>
reB

(log A) log (log A)


y = <■

^ ^ ^,0<’g-1^} log (log A)


j(logA) log (log a) j
uY

dx dx

dij log A
ad

-f- = (log
do

dx
in

1 1
dx + log A X X — ●
Re

X
log A A
F

^y
dx
= (log a)
log A log (log a) ^ 1 = (log a)
log A 1 + log (log a)
X A A

if .1+ 1/a
(ii) Lety = A + — + A . Then,
A

y = ^,Al0g(A+ 1/a) + e (1+ 1/A) log A

dy ,A log (a + 1/a) d f 1'l (1 + l/A)logA d_ 1


^ = t’ frlog A + - + e
1 + — log A ●
dx X dx X

\A
=> ^ = A + -
1
log A + - +
1 A
d (A 1) 1 + 1/a 1 ( 1 1 1
dx
+ — + A
■ - — log A + 1 +
X X
X + ^ dx , A
X X J X
A
DERIVATIVES 18.45

=> f ^
x + —
1
log x + -
( X
+ X
^+l/x 1 + x-logx
2
dx X) l xj X ; X

1 if 1 I ^ 2-1 1/x-l
x + - log x + — +- + a: [1 + x-log x}
dx X) V X^ x2+l

6. (iii) Lett/ = x + {x- . Then,


2
kx^ - 3) log(x-3)^
y = glog^ + e

^ ^(x2-3)log.x + e
x^\o^(x-3)
y =

ow
dy _ d ^(.v2-3)logArl d ^x^ log(x- 3)
dx dx I dx

^ = ^(x^-3)logx dxd_ ^2 log (I-3) 1

e
dx

Fl
re
2
2-3 <2x\ogx
o , ^^-3 + (1-3)*,2

F
dy ^
dx
+
X 2x log (j; - 3) + ^ ^
ur
r
11. We have.
x‘°S^+(logx)^
fo
ks
y =
Yo
^log(.v^°S^) ^^log(logxf
oo

y =

^logxlogx ^^xlog(logx)
eB

y =

g(log + e
X log (log x)
y =
ur

^ A(iogx)2 + e*'“8<'“8*>^(;.log(log*)|
ad

dx dx dx
Yo

-1 + (log xf < log (log x) + -^ xll


2 log X X -
d

dx xj log X X
Re
in

fJu r 1
-/ = —-— log X + (log x)' log (log X) + ●
F

dx X

22. We have,

xy = ex-y log (xy) = log {e^ ^) => log X + log y = x - y


Differentiating with respect to x, we get
1 1 fry 1 fry dy 1 +l
_ = 1 — => 1 ^ ^ y(y-i)
X y dx dx dx [y X dx X (y +1)
23. We have.
y* + :c!' + x* = /
jiogy* log x^ logx^ ^
+ e + e a
h

g^iogy + e
y log + e
^logx _ a
b
18.46 APPLIED MATHEMATICS-XI

Differentiating both sides with respect to .v, we get


A
dx
(«■'■ l°8 i') + ±
dx
(e-V '°8 ■'■) + ±
dx
(e-'- d , Ik
dx
)

log \f d log ^ ^ ^j.r log .T —d {x log .y) = 0


dx ' dx ' dx

X , -V dll
y logi/ + --^
1/ dx
+ X
.}/
^
dx
log x + ^X + ,Y'^ (1 + log x) -- 0
dy
(●VI/-'' log x) ^ - (y-' log y + 1/ x'^ ^ + x''' (1 + log x))
.1/
+ X
dx

!h - ly^ log y + y x-*^ ^ + x'^ (1 -f log x)}


dx ●V- 1
^y + X-' log X

w
18.9 DIFFERENTIATION OF PARAMETRIC FUNCTIONS

F lo
Sometimes x and y are given as functions of a single variable e.g. x = (])(/), i/ = (/) are two
functions of a single variable. In such a case x and i/are called parametric functions or parametric

ee
equations and t is called the parameter. To find — in case of parametric functions, we first obtain
dx

Fr
the relationship betweenxandy by eliminating the parameter f and then we differentiate it with
respect to x. But, it is not always convenient to eliminate the parameter. Therefore dy can also for be
' dx
ur
obtained by the following formula
dy __ dy/dt
ks

dx dx/dt
Yo
oo

To prove it, let A x and A y be the changes in x and y respectively corresponding to a small
eB

change A Mn f. Then,
lim
Ay dy
Ay _ Ay/At Ay 0 At
r

Af
lim
ou
ad

Ax Ax/At dx A Ax Ax dx
lim
A/ ->0 At dt
Y

ILLUSTRATIVE EXAMPLES
Re
nd

LXAMI’LEl Ify=nt^nndy = 2at, find^.


Fi

dx

SOLUTION We have, y = and y = 2at


=> ^^
dx
= 2«(and/!'=2a
dt

^ _ dy/dt __ 2a _1
dx dx/dt 2nf t

LX.-\MI’LE2 Ifn =n 111'2 ‘^ndy = 2t


find
dy
1-r dx

14-/2 2t
SOLUTION We have, x=a and y =
1-/2 1-/2
DERIVATIVES 18.47

dx
= <7 ■
(l-f^)x2f-(l+f^) (-20 ^dy
^ and — =
(l-r)x2-2fx-2^

2.2 df 2v2
df (i-n
(1-r)

tf.Y Aat
and^^ = . (i + r)
dt dt

d\j _ dy!dt _ l+l'^


dx dx/dt 2 at

EXERCISE 18.8

Find , zuhen
dx

ow
t -I
e +e ^ . e -e
1. Y = and V =
2 -^ 2
3 at 3at^
2. x = and y =
777

e
Fl
re
1
^0 and y -
0 -0
0 + 0-

F
3. X =e e
0
ur
2t

r
4. A- = and y -
1 +7 1 +t^
o
sf
1 -7 2t
k
Yo
5. x = and y =
1 +7
oo

l^l ( 1
eB

.u i.
6. If Y = rt t + - andy~a t - , prove that — =-
t) dx y
ur

1
lY t +
^ , find
^y
ad

7. IfY= t + ,y = a
Yo

t dx

ANSWERS
d

2t -2o(0^-o^ + e + i)
Re

1. ^ 3. (?
in

●>

y 1-7 (0^ + 0^ +0-1)


F

1
t +

4. -
x _ 7-1
D. /.
a
t

N/J- 1
y 2t 1
a t +
t

18.10 DIFFERENTIATION OF A FUNCTION WITH RESPECT TO ANOTHER FUNCTION


So far we have discussed derivative of one variable, say, y with respect to other variable, say, y.
In this section, we will discuss derivative of a function with respect to another function.
Let u=f (y) and v=g (x) be two functions of y. Then, to find the derivative of / (y) with respect to
du
g (y) i.e., to find —
dv
we use the following formula
du _ du/dx
dv dv/dx
18.48 APPLIED MATHEMATICS-XI

Thus, to find the derivative of f{x) with respect to g(.v), we first differentiate both with respect
to .rand then divide the derivative of f (x) with respect to ,v by the derivative of g (.v) with
respect to a:.
Following examples will illustrate the procedure.

ILLUSTRATIVE EXAMPLES

CX.-\MPLE 1 Differentiate with respect to x log .y.


SOLUTION Let II = x^' and v = .v log x. Then,
-V
u = X
and V = X log .y
dll 1
rf.Y
X
~ (-V log -V) and ~ = -Y X — + 1 X log X
dx lix

w
dll
dx
X
^ {1 + log .y) and —
dx
= 1 + log .y

Flo
du _ dti/dx _ (1 + log .y) X
X

dv dv/dx (1 + log x)

ee
Fr
ALlTER We have,
.V z>
It =x
=> log U = X log x=v => u e

for
ur
du d , r;. V dll X
= —(t’ ) = C X .
dv dv dv
s
EXERCISE 18.9
k
Yo

1. Differentiate x“ with respect to x .


oo

●) 3
2. Differentiate log (1 + x“) with respect to e'^
eB

3. Differentiate (log x)'^ with respect to log x.


r
ou

ANSWERS
ad

3
~ -V
2 2e
Y

1. -V-l
2.
3. -Y(logx) {1 + log X ● log (log .Y)}
3x
3x (1 + .Y^)
Re
nd

MULTIPLE CHOICE QUESTIONS (MCQs)


Fi

Mark the correct alternative in each of the following:


x-4
1. If/(x) = , then /' (1) is
2^

a,) I (c) 1 (d) 0


2 3
X X
2. Ifi/=1+- H h h..., then
1! 2! 3! dx

(a) y + 1 (b) y-1 (c) y (d)


3. lf/(x)=l-x + .Y^ X + ... - X + X ^, then /' (1) equals
3 <JQ 1 nn

(a) 150 (b) -50 (c) -150 (d) 50


18.49
DERIVATIVES

1
2
X
4. If i/ = , then
_

1 dx

4.y 4y l-x^ 4.Y


(a) - (b) - (c) (d)
.y 2-1 4.V .y 2-1

5. Ifi/=V^ + ^ , then —
dx
at.Y=l is
1
(a) 1 (c) (d) 0
^/2

ow
100 99
6. If/(.Y)=.Y + .Y +...+ .Y +1, then /'(I) is equal to
(a) 5050 (b) 5049 (c) 5051 (d) 50051
2 100
A'
, then/'(-I) is equal to
7. If/(.v)=l+.Y + y+...+ 100

e
(b) 100

Fl (c) 50 (d) 0

re
(a)
100

F
ANSWERS
ur
r
1. (a) 2. (c) 3. (d) 4. (a) 5. (d) 6. (a) 7. (d)
fo
_FILL IN THE BLANKS TYPE QUESTIONS (FBQs)
ks
2 3
Yo
A' .Y
,then^' =
oo

1. Ift/=1+ —+ + +
■' 1! 2! 3! dx
3 4 5
B

2. If y =1 --ll"^
.Y A
H
A' A
h ,then^^J' =
2! 3! 4! 5! dx
re

1
3. Let/(A-)=.Y-[A-], A-ey?.Then/’ w/
-
u
ad
Yo

4. If/(A') =A' + | -y|, then /'(I) =


5. lt‘/(A-) ^ A-+I 'Al, then/'(-I) =
d
Re

6. If f{x) = -A1 AI, then /' (- 2) =


in

\2
d 1
F

7. A +
dx

8. f-(logl -A|) =
rfA
, A ^ 0.

9. lf/(A-) =mA + c,/(0) = /'(O) =1, then /(2) =


d 1
10.
dx[\o^:,e ^
d 1
11. —
dx
log., 10
d m
12.
T 'I
dx A
18.50
APPLIED MATHEMATICS-Xi

ANSWERS

1.1+i: +
.V
— + —+... 2. — 1 1 1- 3. 1 4. 2
1! 2! 3! 1! 2! 3!

5. 0 1 1
6. 4 8. 9. 3
X' X
1 1
iO. 11. 12. 0
-V log^lO

VERY SHORT ANSWER QUESTIONS ('l/S>40s;


Answer each of the follounng questiotis in one word or one sentence or as per exact requirement of the
question:
1. Write the value of lim fiff-fic)

w
.T-»C x-c

2. Write the value of lim xfia)-a f (x)

F lo
X —>a X -a

3. If :r < 2, then write the value of — - 4.V + 4).


dx ^

e
Fre
L If/(.Y) = log 2 write the value of/'{.v).
5. Write the value of — (y|y|).
dx
L...
for
|(y+|y|)|.v||
r
6. Write the value of —
dx
You
oks

7. \if{x) = IYj+|.Y-1|, write the value of —


dx
(/(.r)).
o
eB

8. Write the value of the derivative of/ (.v) = |.y-1| + |.v - 3( at y = 2.


9. If/(.T) = 1'-^
write-^(/(y)).
dx
our
ad

10. Write the value of —


dx
(log | y).
dY
Re

11. If/(1) - !,/'(!) = 2, then write the value of lim


Y -1
Fin

12. Write the derivative of/(y) = 3|2 + .\-|atY = - 3.


13. lf|Yj <1 and 1/= 1 + ;e + Y^ + Y 3■ + .. then write the value of—.
dx

ANSWERS
!● /'(C) 2. f(a) -a f (a) 3. -1 4. 0

2y, y > 0 2 , Y >1


0, Y < 0
5. ■ 6. ● 7.
- 2y, y < 0 0,0 < Y < 1
4y, y > 0 dx
- 2 , Y <1
1 ,x>0
8. 0 9.
^(/W)
dx
=■ -1, Y<0
10.
1
X
, Y 0

1
11. 2 12. -3 13.

(1-.V)'
CHAPTER 12
TANGENTS

ow
19.1 SLOPES OF TANGENT AND NORMAL
Slolpe of the tangent: Let y = f(x) be a continuous curve, and let P {.Ti, i/i) be a point on it. Then,
as discussed in section 18.1 is the slope of the tangent to the curve y =/(-v) at point P
'[dx J?

e
re
y

F
Frl
ou
sr
kfo
oo
Y
reB
uY

i.e.
dx
= tan y = Slope of the tangent at P, where is the angle which the tangent at
P(.vi, i/i) makes with the positive direction of .r-axis.
ad
do

If the tangent at P is parallel to .r-axis, then


(dy
in

= 0
y = 0 => tan ij/ = 0 => Slope = 0 => dx
Re

If the tangent at P is perpendicular to AT-axis, or parallel to i/-axis then


F

. \
jt dx
= 0 => = 0
y = - => cot V|» == 0 ^ —
2 tan y dy /P
Slope of the normal: The normal to a curve at P (.r'l, i/i) is a line perpendicular to the tangent at P
and passing through P.
1 1 dx^
Slope of the normal at T = - dy dy /P
Slope of the tangent at P
dx jp
ILLUSTRATIVEEXAMPLES

7‘i//ir / ON FINDING THE SLOPES OF THE TANGENT AND THE NORMAL AT A GIVEN POINT
EXA.MI’LL 1 Find the slopes of the tangent and the normal to the curve + 3y + i/^ ^5 at (1,1).
SOLUTION The equation of the curve is .r“ + 3y + y =5.
19.2
APPLIED MATHEMATICS-XI

Differenhating with respect to .v, we get


2.v+3^
dx
+ 2y^
dx
=0
^ - 2.V
f^ 2 2
dx ly + 3 dx 2+3 5
^ L1)

Slope of the tangent at (1,1) = ^


rf.v
2

5
All)
-1 -1 5
and. Slope of the normal at {1,1) =
-2 2
[dx A 1,1) 5

EXAMPLE 2
Shozo that the tangents to the cun>ey = 2.v^ — 3 at the points zuhere x = land x = -2 are
parallel.

w
SOLUTION The equation of the curve is y = 2.y^ - 3.
Differentiating with respect to r, we get
^ = 6.v^
dx

F lo
ee
Now, nil - (Slope of the tangent at .y = 2) = = 6x(2)^=24

Fr
^dx Ax =2

nh - (Slope of the tangent at .y = - 2) = dy)


and.
dx
for
= 6 (-2)2 =24.
>x = -2
ur
Clearly, nii =iH2-
s

Thus, the tangents to the given curve at the points where .y = 2 and x = - 2 are parallel.
ook
Yo

EXAMPLE 3 Prove that the tangents to the curve y = x^ - 5x+6 at the points (2, 0) and (3, 0) are at
eB

right angles.
SOLUTION The equation of the curve is i/ = x^-Sx + h. ...(i)
our

Differentiating with respect to x, we get


ad

= 2.V-5
dx
Y

Now, nil = Slope of the tangent at (2, Q) = ^


Re

=2x2-5=-!
nd

dx
A 2,0)
Fi

and, ni2 = Slope of the tangent at (3, 0) = — = 2x 3-5=1


I dx ^(3,0)
Clearly, /??-, iih =-1x1 = -1.
Thus, the tangents to the given curve at (2, 0) and (3, 0) are at right angles.
EXAMPLE 4 The slope of the curve Ix-p = ax^ -\-bat{\,-1) is - 1. Find a, h.
SOLUTION The equation of the curve is
2y2 = nx^ + b ...(i)
Differentiating with respect to x, we get
4y^
dx
= 2.Y
dy rtX
=>
-a

dx 2y dx
(L-l) 2
19.3
TANGENTS

It is given that the slope of the tangent at (1, --1) is -1. Therefore,
a
= -1 => n = 2
2

Since the point (1, - 1) lies on (i). Therefore,


2{-l)^ =n{}f +b^a + b^2
Putting rt = 2 in + i’ = 2, we obtain = 0.
Hence, a = 2 and b =0.
T}/pc 11 ON FINDING THE POINT(S) ON A GIVEN CURVE AT WHICH TANGENT(S) IS (ARE) PARALLEL OR
PERPENDICULAR TO A GIVEN LINE

EXAMPLES find the points on the curve y=x^~ 2x^ - xnt which the tangent lines are parallel to the
line y = 3.v - 2.
SOLUTION Let P [x^, i/^) be the required point. The given curve IS

w
1/ = a: 3-2.t2‘ - X (i)

^ = 3.v^-4.v-l

Flo
dx

= 3.T,1 ^ - 4.Vi -1

ee
dx

Fr
Since the tangent at (.Yj, y^) is parallel to the line y = 3y - 2.
Slope of the tangent at (y^ , y^) = Slope of the line y = 3y - 2 for
ur
= 3
Jx j
k s

3y^^ - 4yi -1 = 3 => 3y^^ - 4y^ 4 = 0 => (.Vi -2) (3y, + 2) = 0 => y^ =
Yo
oo
eB

Since (.Yi, yi) lies on curve (i). Therefore, y^ = Y


1 -.Yi-

Now, = 2 => i/i = 2^-2(2)2-2 ^ _2.


2^2 2 -14
r

2
ou
ad

and. - -2 +
3 " ■''' = 3 3J 3 27

-U'l
Y

J-2
Thus, required points are (2, - 2) and V —,
3
Re
nd

FXAMI'I Eh Find the point on the curvey = 2x^ - 6x - 4 ativhich the tangent is parallel to thex-axis.
Fi

SOLUTION Let the required point be P (y^, yi). The given curve IS

y = 2y2 - 6y - 4
^ = 4.V-6 = 4yi - 6
dx {dx

Since the tangent at (Y|, y^) is parallel to Y-axis. Therefore,


(dy
dx)
= 0 => 4.v^ - 6 =0^ =I
Since (Y|, y^) lies on curve (i). Therefore,
yi = 2.v^^-6.v^ -4
= 3/2 ^ 1/1 = 20/2)2-6(3/2)-4 = -y
So, the required point is (3/2, -17/2).
19.4
APPLIED MATHEMATICS-XI

IIXAMPLC7 At what points on the curve +y^ -2a'-4i/ + 1 =0, the tangents are pyaralle! to the
l/-axis?

t’e the required point. As P(.r^,y|) lies on the curve

-v^+ j/^-2a-4i/ + 1 =0.


■^l^+y]^-2.Y]-4i/i+l=0
If the tangent to the given curve at P is parallel to i/-axis, then
= 0

Tlie equation of the curve is


-v“ + y“ -2.Y-4I/+ 1 =0
Differentiating both sides with respect to y, we obtain
2.v^
ciy
+ 2i/-2—-4
" liy
=0

w
2^(.t-1) = 2(2-,/) => dx] _2-yi

But
dx
ny

2-yi
Ai-l
= 0. Therefore,

= 0=> 2-y^ = 0^ j/j = 2

Putting y^ = 2 in (i), we obtain


ciy x-1

F lo Al-1

for F
ree
+ 4 - 2xi - 8 + 1 = 0 _ 2.vi - 3 - 0 (xq - 3) {xq +1) => xq = -1, 3
Hence, the coordinates of required points (-1, 2) and (3, 2).
Your
ks

EXAMPLES Find the points on the curve y =.v^ at which the slope of the tangent is equal to the
o
eBo

y-coordinate of the point.


SOLUTION
Let the required point on the curvey =x^ be P (.X| , i/j).
We have.
ad
our

y = x-3 ^ ± . 3x-2 ^ i?y o


= 3.x,1
2
dx dx
A-VI ,yi)
It is given that:
Re
Y

Slope of the tangent at P (x^ , y^) = Ordinate of P (xi , y^)


Find

= yi
dx

3
3 3
(^1 ' yi) I>C!S on y = X
X
1 1

^ (Xi - 3) = 0 => x'i = 0, X| = 3


Since (x, , y^) lies on y = x^. Therefore,
3
yi X
1

x-i =0 => 1/] =0 and, X| = 3 => y^ = 3^ = 27


Hence, required points are (0, 0) and (3, 27).
2
EXAMPLE 9 Find points on the curvi ^
9 16 the tangents are parallel to the
(i) x-axis (ii) y-nxis.
19.5
TANGENTS

2 2

SOLUTION Let P (j-'i, y|) be a point on the curve 16


2 2
h. h = 1
9 16

The equation of the curve is


X

9 16

Differentiating both sides with respect to .y, we get


16 Yi
2.T 2y ^ ^ (J ^ ^ Ifa ^
9 16 dx dx 9\j ,i/i) 9yi

(i) [fthc Imigcnt at P(yj, y^) is parallel to x-a\is, then


d]kx\ .in) 16 Yj
9yi
= 0 => 16 .Y| = 0 => Xi = 0

Putting X] = 0 in (i), we get y^ ^ -16, which is impossible as y^ is real. Hence, there is no point
on the curve where tangent is parallel to Y-axis,
(ii) Ifthe tangent at P (y-, , y;^) is parallel to i/-axis, then
dx

Putting yi = 0 in (i), we get Yj = ± 3 .


= 0 =>
9yi
16 Yj
F low
= 0 => y^ = 0.
for F
re
Hence, required points are (3, 0) and (- 3,0).
tXAMI’tl lU Find a point on the curve y = (y - 3)^, where the tangent is parallel to the line joining
Your
s

(4, l)rt)itf (3, 0).


eBo k

SOLUTION Let the required point be P(-V^, y{)- The equation of the given
curve is

y = (y-3)“
ad
our

= 2(.v-3) => = 2(.Vi-3)


dx dxj (X-l.l/l)

Since the tangent at P is parallel to the line joining (4,1) and (3,0). Therefore,
Re

Slope of the tangent at P = Slope of the line joining (4,1) and (3,0)
Y

0-1
Find

=> 2(.y-i-3) = 1 => -V;i = 7/2


dx 3-4
Ayi,i/i)
Since the point P(Yi, \Ji) lies on (i). Therefore,
y\ = (a-i-3)2
7 7
= - - 3
2 U 4

Thus, the required point is (7/2,1/4 ).


F.XAMIM.F. II Find the point on the curve y =y^ -11y + 5 nf which the tangent has the equation
y = Y-n. .3
-llY + 5.
SOLUTION Let the required point be P(y^, i/i). Since (Y|, y^) lies on y - y
...(i)
yi Y
1 ●■-ll'Yi+5
19.6
APPLIED MATHEMATICS-XI

Now, y = -ll.v + 5

= 3.1-^ -11 = 3.v,^-ll


lix tix 1

Since the line y = .v -11 is tangent at the point (a'i, y-,). Therefore,
Slope of the tangent at (.v^, j/j) = (Slope of the liney = .v -11).
‘h
I lix = (Slope of tile line .Y - 1/-11 = 0)

-1
3y,--11 = Coefficient of .y
1
-1 Slope =! -
Coefficient of y

ow
-53y, 2
1
= 12 Y 1 = ±2

Now, Yi - 2 yi = 2^'*-22+5 - -9 [Using (i)]


Yi = -2 => y = (-2)''*-ll(-2)+5 = 19 [Using (i)]

e
So, two points are (2, - 9) and (- 2,19). Of these two points, (-2,19) does not lie on i/ = Y

re
Fl
-11.
Therefore, the required point is (2, - 9).

F
EXERCISE 19.1
ur
1. Find the slopes of the tangent and the normal to the following curves at the indicated

r
points: fo
(i) y=VT at Y = 4
(ii) y = Vy at y = 9
ks
Yo
(hi) y=Y^-Yat.Y = 2 (iv) .v^ + 3y+ y^ =5 at (1,1)
oo

(v) Yy = 6at(l,6)
eB

2. Find the values of a and b if the slope of the tangent to the curve yi/ + i?Y + bi/ = 2at(l, 1) is 2.
3. Ifthe tangent to thecurvey = Y'^’ +/TY + Z7 at (1,-6) is parallel to the line y-i/ + 5 = 0, find
a and b.
ur

4. Find a point on the curve y = y ^ - 3y where the tangent is parallel to the chord joining
ad
Yo

(1,-2) and (2,2).


5. Find the points on the curve y = y^ -2y^-2y at which the tangent lines arc parallel to the
d

liney =2y-3.
Re
in

6. Find the points on the curve y^ = 2x^ at which the slope of the tangent is 3.
F

7. Find the points on the curve .vy + 4 = 0 at which the tangents are inclined at an an<^le of 45°
with the .Y-axis. ”

8. Find the point on the curve y = y“ where the slope of the tangent is equal to the Y-coordinate
of the point.
9. At what points on the circle y“ +^y^ -!^Y-4y+ 1 =0, the tangent is parallel to Y-axis.
10. Find the points on the curve ^ ^ 1 at which the tangents are parallel to the
(i) .Y-axis (ii) y-axis. 4 25
11. Find the points on the curve y^ + y^ - 2y - 3 = 0 at which the tangents are parallel to the
(i) -Y-axis. (ii) y-axis
12. Find the points on the curve —- +
= 1 at which the tangents are (i) parallel to .Y-axis
(ii) parallel to y-axis. ^ 16
13. Show that the tangents to the curve y - 7.v^ + 11 at the points y = 2 and y = - 2 are parallel.
19.7
TANGENTS

14. Find the points on the curve i/ - where the slope of the tangent is equal to j-coordinate
of the point.
ANSWERS

1.
Slope of the Slope of the Slope of the Slope of the
normal tangent iwrwal
tangent
3 -1/3 (iv) -2/5 5/2
(i)
1/6 -6 (V) -6 1/6
(ii)
(iii) 11 -1/11

w
7-2 7
3. a=-2,b = -5 4. ±
2. a = 5,b=-A
3'*^ 3 V3 ,
5. (2,-4); (-2/3, 4/27) 6. (2,4) 7. (2, - 2) and (- 2,2)

o
8. (0, 0) 9. (1,0), (1,4)

e
10. (i) (0,5), (0,-5) (ii) (2, 0), (-2, 0)

re
11. (i) (1,±2)
12. (i) (0,4), (0,-4)
(ii)
(ii)
(-1,0), (3,0)
(3, 0), (- 3, 0)

rFl 14.(0, 0), (1/3,1/27)

F
19.2 EQUATIONS OF TANGENT AND NORMAL

r
ou
We know that the equation of a line passing through a point (.\q, i/i) and having slope m is
fo
ks
i/-yi = m{x-xi)
As discussed in article 19.2 that the slopes of the tangent and the normal to the curve y = f{x) at a
oo

respectively. Tlierefore, the equation of the tangent at


Y

and -
point P(.\q, y^) are
eB

dx
dx Jv
r

P(.\q, y^) to the curve y = /(.v) is


ou
Y
ad

...(i)
.v-yi = (-V--M)
\dx
1
Since the normal at P(aq, y j) passes through P and has slope - ^dy —. Therefore, the equation of
d
Re
in

the normal at P(.v-,, y^) to the curve y = f{x) is dx )p


K- (-V - .vi) -(ii)
F

y-yi = -
^y
dx jp
dy then the tangejit at (.r^, y^) is parallel to y-axis and its equation is x = x-^.
REMARK 1 If -f-
dx
= co.
^P

dy = 0, then the normal at (.y^ , y^) is parallel to y-axis and its equation is x - x^.
REMARK 2 If
dx Jp
In order to find the equations of tangent and normal to a given curve at a given point, we may
use the following algorithm.
ALGORITHM

STEP I Find -^from


dx
the given equation y = f{x).
19.8
APPLIED MATHEMATICS-XI

STEP II Find the value given point P{x\, i/j).

STEP 111 If dx is a non-zero finite number, then obtain the equations of tangent and

normal at {x^.y-f) by using the formulae _r^y


y-yi =
dx
(a: - .Vj) and
1
y-yi =- (-Y - A-|) respectively. Otherwise go to step IV.
dx
-'ixi.n)

ow
rfy
STEP IV If 0, then the equations of the tangent and normal at (yj, y^) arey~y^=0 and
dx

Y- = 0 respectively. If ^ = ± CO, then the equations of the tangent and normal at


dx j

e
(Y| , i/|) arex-Xi~Q and y - yi = 0 respectively.

re
rFl
F
ILLUSTRATIVE EXAMPLES

r
ou
I'ype 1 ON FINDING THE EQUATIONS OF TANGENT AND NORMAL TO A CURVE AT A POINT
EXAMPLE 1
Fmd the equation of the tangent to the curve y ~-5x^ fo + 6x+ 7 at the point
ks
(1/2, 35/4).
oo

SOLUTION The equation of the given curve is


dy
Y

y = -5y^ + 6y + 7 -10y+6 =>


10
B

—^ + 6 = 1
dx dx 2
(1/2,35/4)
re

The required equation of the tangent at (1/2,35/4) is


35 ^y 35 n
ou

33
Y

X — = 1y-- => u =
ad

Y +
dx
7(1/2, 35/4) 2J 2 ^ 4

EXAMPLE 2
Find the equations of the tangent and normal to the parabola y^ =4ax at the point
d

(at^, 2al).
in
Re

SOLUTION The equation of the given curve is


F

y^ = 4ax ,..(i)
Differentiating (i) with respect to y, we get
2y ^
dx
= 4a => ^ _ 2rt ^ / fry' _ I
dx y 2at ~ t

So, the equation of the tangent at (at^, 2at) i IS

y-2at = dy] (x-at^)


■J
y-2at = -{x-at^)
t
=> ty = x + at^
The equation of the normal at {af^, 2at) i IS
19.9
TANGENTS

1
ij-2nt = - (x-at-)

y-2at=- l(x-rt^^) => \j-2at = => }/ + tx = 2at + nt^


t

EXAMPLE 3 Find the equations of the tangent and the normal to 16x^ + 9i/^ =144 at (.V|, i/^) zohere
.V;i = 2 and > 0.
SOLUTION The equation of the given curve is
-.(i)
16.Y^+9y^=144
Since (.Yj, lies on (i). Therefore,
r, T 9 2 80 4 vs
[●●● J/i > 0]
16-y
^2 + 9y^ = 144 => 16(2)^ +9y/= 144 =>i/j =—=>i/a=^

w
fo
So, coordinates of the given point are 2, .

Now, 16.y^ + 9y^ = 144

F lo [Differentiating with respect to x]

ee
32y + 18v— = 0
dx

Fr
16x 2
d\j _ -16 y
dx 9y dx JL4V5 9x
4V5 3VS for
3
3
ur
4V5^ IS.
The equation of the tangent at 2, —
s

I ^
ook
Yo

4VS 4 Vs 8
(y - 2) => 8y + 3 VSy - 36 = 0
eB

rfY 3>/S
' 3

4^/5Y
our
ad

The equation of the normal at 2, — IS


V ^
4VS 1
dY

(-■v-2)
Re

^y
dx
Fin

4VS -1 4VS 3 Vs
-8
(y - 2) => y 8
(Y - 2) => 9 Vs Y - 24y + 14 VS = 0.
3

3 Vs
- x/a
+ ^ = 1 touches the curve y = be at the point xohcre it crosses the
X
EXAMPLE4 Show that the line —
a b
y-rtYiS.
SOLUTION The equation of the given curve is
-x/(i ...d)
y = be
It crosses y-axis at the point, where y = 0. Putting y = 0 in (i), we get: y =be -b
So, the point of contact is (0, b).
Differentiating (i) with respect to y, we get
19.10
APPLIED MATHEMATICS-XI

^=be x/a d ( x\
^ ^ = ~h
,-x/a ^ dy
dx dx dx
a (j . dx A0,b) a a

The equation of the tangent at (0, b) is


dy
(x-n)
dx
Ao,b)

y - b - —a (a: - 0) rti/ - nb = -b x b X + m/ = ab =i> — + ^ = l .


n b
y
— = 1 touches the curve u = b e -x/a
Hence, — + f
a b at the point where it crosses the axis of i/.
x-7
EXAMPLES
Find the equations of the tangent and the normal to the curve y ~ at the
(x-2) {x-3)
point, where it cuts x-axis.
SOLUTION Tlie equation of the given curve is
y (a: - 2) (x - 3) - a: + 7 = 0

w
...(i)
This cuts the x-axis at the point, where y = 0. Putting y = 0 in (i), we get
-X + 7=0:=>X=7

So, the point of contact is (7,0).


Differentiating (i) with respect to x, we get
^(x-2)(x-3) + i/(2x-5)-l = 0
dx

Putting X = 7 and y = 0 in (ii), we get


F lo for F
ree
...(ii)

dy dy 1
dx
(7-2)(7-3)-l = 0
dx
r
^(7,0) 7(7,0)
20
You
oks

So, the equation of the tangent at (7, 0) is


eBo

dy
j/-o= ^
dx
7(7,0) (x-7) y-0 = -^(-v-7) => X-201/-7 = 0
The equation of the normal at (7,0) is
ad
our

1
y-0 = (x-7) => y-0 = -20 (x-7) => 20x-f-y-140 = 0
dy
dx
7(7,0)
Re
dY

EXAMPLE 6 Find the equation of the tangent to the curve y = (x*^ -1) (x - 2) at the points zvhere the
Fin

curve cuts the x-axis.

SOLUTION The equation of the curve is


y = (.v3-1)(.v-2)
It cuts x-axis at y = 0. So, putting y = 0 in (i), we get
(x^-l)(x-2) = 0
(x -1) (x - 2) (x^ + X + 1) =0
X -1 = 0 , X - 2 = 0
I-.- x^ + X +1 0]
X = 1, 2.
Tluis, the points of intersection of curve (i) with x-axis are (1, 0) and (2, 0).
Now, y = (x^ -l)(x-2)
19.11
TANGENTS

^
dx
= 3x^ {X - 2) + (.v"'’ -1) rf.Y
= - 3 and.
-*(2,0)
= 7.

Auo)

The equations of the tangents at(l, 0) and (2,0) are respectively


dy (.v-2)
i/-0 = {x -1) and y - 0 = dx
A2.0)

y-0 = -3(.r-l) and y-0 =7(y-2)


y+ 3y-3 = 0 and 7.v-y 14 = 0.
Type II ON FINDING TANGENT AND NORMAL PARALLEL OR PERPENDICULAR TO A GIVEN LINE
EXAMPLE? Find the equation of the tangent line to the curve y = px - 3 - 2 zohich is parallel to th
line Ax - 2y +3=0.

w
SOLUTION Let the point of contact of the tangent line parallel to the given line be P{xp y|).
The equation of the curve is y = px - 3 - 2.
Differentiating both sides with respect to .v, we get
dy _ 5 tfy

F lo 5

ee
=> —
dx
2^/57^ 2751^3
dx

Fr
(VI, 1/1)

Since the tangent at (Y|, yj) is parallel to the line 4.v - 2y +3=0. Therefore, for
= (Slope of the line 4.v - 2y +3-0)
ur
I dx)
s

-4
73
5
ook

=> 4 px-i - 3 = 5 => 16(5.r^-3) = 25 => -Vi = —


Yo

2 pxi ^ -2
eB

Since (yj, y^) lies on y = px - 3 - 2. Therefore,


3 73
- 5R X 73 3- -2 = - ●●●
yi = pxi - 3 - 2 => yi =
our
ad

4 80
\ 80
f 73 3
So, the coordinates of the point of contact are V —,
oU 4J
dY

Hence, the required equation of the tangent is


Re

3^ dy
(x-x-i)
Fin

y- -
V 4J [ dx

3^ 73 \ . .

= 2
= 2 Y
dx
4) 80

80.Y-40y-103 = 0
2
EXAMPLES Find the equation of tangent line toy = 2y + 7 zohich is parallel to the line 4y - y + 3 - 0.
SOLUTION Let the point of contact of the required tangent line be (y^ , y-j).
The equation of the given curve is
y = 2y^ + 7
Differentiating both sides with respect to y, we get
= 4y => (^y'\ = 4yi
dx .dx A-VI-1/1)
19.12
APPLIED MATHEMATICS-XI

Since the line 4.y - y + 3 = 0 is parallel to the tangent at (.Yi, 1/2).


Slope of the tangent at(Yi, yj) = {Slope of the line 4y -y + 3 =0)
-4
=> Coefficient of y
dx -1 V Slope = -
Coefficient of y
4y| = 4 ^ A'2 =1
Now, (Y|, yi) lies on y = 2y^ + 7.
yi=2Yj^+7^y2=2 + 7=9 [V .Yi=l]
So, the coordinates of the point of contact are (1,9).
Hence, the required equation of the tangent line is
y-9 = 4(y-1) => 4Y-J/ + 5 = 0
EXAMPLE 9 Find the equntion(s) oftnngent(s) to the curve ij = +2x + 6 which is fU’rpendiciilar to
the line x + 14y + 4=0.

w
SOLUTION Let the coordinates of the point of contact be (yj, y2). As it lies on y = y^ + 2y + 6
yi = Tj ^ + 2.V2 + 6
The equation of the curve is 1/ = y^ + 2.v + 6.
Differentiating both sides with respect to v, we get
F lo
ee
^ = 3Y^ + 2^f^^l = 3y^+2

Fr
dx dx 1 ...(h)
for
Since the tangent at (yj, i/-^) is perpendicular to the line y + 14y + 4 = 0. Therefore,
Slope of the tangent at {x-^, y{) x Slope of the line = -1
r
1
You

X = -l
s

. dx 14
ook

-1 ^ 3Yj^ + 2
eB

= 14 => Yi1 = ± 2

Now, Y2 = 2 j/2 = 2^+2x2 + 6 = 18 [Using (i)]


our
ad

^1 = -2 ^ yi = (-2)3+2 (-2)+ 6 = -6 [Using (i)]


So, the coordinates of the points of contact are (2,18) and (- 2, - 6).
From (ii), we obtain
dY

rfy') = 3 (2)^ + 2 =14 and. dy)


Re

dx) (2,18) = 3(-2)2+2=14


Irf.v 2(-2,-6)
Fin

The equation of the tangent at (2,18) is


y-18= ^
^y]
dx (●Y-2) => y-18 = 14(y-2) => 14Y-y-10 = 0
^(2,18)

The equation of the tangent at (- 2, - 6) is


dy
y-(-6)= {dx
^ (y-(-2)) y-(-6) = 14(.y-(_2)) => 14Y-y + 22 = 0
^{-2,-6)

I ype III ON FINDING TANGENT OR NORMAL PASSING THROUGH A GIVEN POINT


EXAMPLE 10
Find the coordiiiatesofthepomtson thecurvey=x^ + 3x + 4, the tangents at which pass
through the origin.
SOLUTION Let P (y2 , y-i) be a point on the given curve such that the tangent at P passes
through the origin. Since P (yi , 1/2) lies on y = y^ + 3y + 4.
19.13
TANGENTS

-(i)
yi = x-^^ + 3x-i+4
The equation of the curve is
1/ = .v^ + 3x + 4
Differentiating with respect to .v, we get
dy
= 2.V + 3 => = 2xi + 3.
dx dx)p
The equation of the tangent at P (.V| , i/|) is
j/-yi =
dx Jp
(.r-.\q) or, = (2.Vi+ 3)(.v-.Yj)

It passes through the origin i.e. (0, 0).


0-i/l = (2y^+ 3)(0-Yt) => yi = 2.Y,1 ^ + 3y 1
...(ii)

w
Subtracting (ii) from (i), we get
-^1 2 + 4 = 0 ^ .Yi = ± 2.
From (ii),

F lo
= 2=> i/| = 4 + 6 + 4= 14 and, x^ = - 2 => }/i =4-6 + 4 - 2

ee
●^1

Fr
Hence, the required points are (2,14) and (- 2, 2.).
7'i/pc iV MISCELLANEOUS EXAMPLES for
EXAMPLI-; n For the curve y = 4.t^ - 2x^ find all points at which the tangent passes through the origin.
ur
3 5
SOLUTION Let(Aq, i/i)be the required point on i/ =4.v -2.y . Then,
s
-(i)
ook

,1 3 2.r,^
Yo

.Vl = 4.v^ 1
3 5
The equation of the given curve is y = 4.y - 2.\ .
eB

Differentiating with respect to .v, we get


^ = 12x2 4
-lO.v'* => = 12a-^^ -10-y/
r

dx
ou
ad

dx

So, the equation of the tangent at(A:-|, y{) is


Y

y-v\ =
dy
(A- - ,vi) y -yi = (12a^^ -lO.v^'^) {x-x{)
Re

dx
nd

(XI,yi)

This passes through the origin. Therefore,


Fi

O-I/I = (12.Yj^-10.Y^^)(0-.vi) yi = 12.y,^


1
-lO.v,'’
1
...(ii)

Subtracting (ii) from (i), we get


0 =
-8aj^ + 8a.|^ => 8,V|^(a^^-1) = 0 => -Yi =0 or, x-i = ±1
[Using (ii)]
When A| = 0 => yt = (^
[Using (ii)]
When ,V| = 1 => y-j =12—10-2
[Using (ii)]
When .Y| = -1 = -12 + 10 = -2
Hence, the required points are (0,0), (1, 2) and (-1, - 2).
EXERCISE 19.2

1. Find the equation of the tangent to the curve Va + ^y = a, at the point (fl^/4, a~/A).
2. Find the equation of the normal to y = 2a^ - a^ + 3 at (1, 4).
19.14
APPLIED MATHEMATICS-XI

3. Find the equations of the tangent and the normal to the following curves at the indicated
points:

(i) 1/ = -bx^ + 13x^ - lOx + 5 at (0,5)


(ii) y = x‘^ - 6x^ + \3x^ - IO.y + 5 at a = 1
(iii) y = x^ at (0, 0) (iv) y = 2x^ - 3.V -1 at (1, - 2)
.3
at (2,-2) (vi) y = A + 4.V + 1 at X = 3
4 - .r

ow
+^
h~
= 1 at (<7 cos 0, h sin 0) (viii) ^ -y = 1 at (a sec 0, b tan 0)
b^

(ix) y^ = 4n x at 2a/m) (x) c^(.v^+y2)=.Y^y^at cos 0 ' sin 0

e
.2
1/^

re
(xi) A-y = at (cf, c/t) (xii) ^ + ^ = ] at(Aq,y-,)

Frl
F
(xiii)
x^ y2 2/3 2/3
^ = 1 at(Ao,yo) (xiv) A + y = 2 at (1,1)
ou
or
(xv) = 4y at(2,l) (xvi) y^ =4A-at(l, 2)
kfs
(xvii) 4a-2 + 9y^ = 36 at (3 cos 6, 2 sin 0)
oo

(xviii) y^=4rtA-at(Aq,y]) (xix) ^ ^ = 1 at {4la, b)


Y

b^
B

4. The equation of the tangent at (2,3) on the cur\^e \/ = ax


^ + b is y = 4x - 5. Find the values of
a and b.
re

■3. Find the equation of the tangent line to the curve y = .t^ + 4x -16 which is parallel to the
oYu

line 3x - y + 1 = 0. ^
ad

6. Determine the equation(s) of tangent(s) line to the curve y = 4.v^-3A- + 5 which are

perpendicular to the line 9y + x + 3 = 0.


d

7. Find the equation of the tangent line to thecurvey .2 - 2x + 7 which is


in

= x
Re

(i) parallel to the line 2x - y + 9 = 0 (ii) perpendicular to the lino 5y - 15x = 13.
F

8. Find ^the equations of all lines having slope 2 and that are tangent to the curve
y = X 3.
X -3 '

9. yFind
=
the^ equations of all lines of slope zero and that are tangent to the curve

x^ - 2x + 3 '
10. Find the equation of the tangent to the curve y = px - 2 which is parallel to the line
4x - 2y + 5 = 0.
n. Find the equation of the tangent to the curve x^ + 3y-3 = 0, which is parallel to the line
y = 4x-5.
12. At what points will be tangents to the curve y=2x^ -15x^ + 36x - 21 be parallel to x-axis?
Also, find the equations of the tangents to the curve at these points.
19.15
TANGENTS

13. Find the equation of the tangents to the curve 3x^ - i/^ = 8, which passes through the point
(4/3, 0).
ANSWERS

1. x + y = a^/2 2. x + 4i/=17
Normal
3. Tangent
(i) y + 10.T-5 = 0 X - lOy + 50 = 0
(ii) 2.V - y + 1 = 0 .r+ 2y-7 =0
x =0
(iii) y = 0
(iv) -y - 3 =0 X + y +1 = 0
(V) 2.v + y-2 = 0 .V - 2y - 6 = 0
(vi) 10.r-y-8=0 + lOy - 223 = 0

low
X y
(vii) - cos 9 + — sin 0=1 ax sec 0 - In/ cosec 0 = a~ -b
a b

(viii) - sec 0 - - tan 0=1 ax cos 6 + fjy cot 0 =


a b

(ix) m^ X -m y + a = 0 ^ X + m^ y - 2(7 -(7 = 0

ee
m

F A- sin ^ 0 - y cos^ 0 + 2c cot 20 = 0

Fr
(x) .vcos^ 0 + y sin^ 0 = c
4
(xi) A + y = 2cf .Yf^ -h/ = ct -c
for
ur
yy\ _i
a^ X b^y = (7
2
(Xll) +
iP- y\
ks
a

2 a b^y = .2 4-^2
Yo

(7
oo

+
(xiii) 2
b^ ^0 yo
eB

(7

(xiv) A + y - 2 = 0 y -A =0
(xv) A - y -1 =0 A +y - 3 =0
r
ou

A + y -3 =0
ad

(xvi) A - y + 1 = 0
(xvii) 2a cos 0 + 3y sin 0 = 6 3a sin 0 - 2y cos 0 -5 sin 0 cos 0 = 0
Y

-]/l
(xviii) yy^ =2(7(a + a^) y-yi = (a-A|)
2a
Re
nd

V2a y =1
ax
+ lnj = a'^+b^
(xix)
Fi

a b V2
4. a = 2,b =-7 5. 12a - 4y - 65 = 0
6. 9a - 1/ - 3 = 0, 9a - y + 13 = 0
7. (i) 2a - y + 3 = 0 (ii) 12a + 36y - 227 = 0
8. There is no tangent to the curve that has slope 2.
10. 48a - 24y = 23 11. 4a - y + 13 = 0
9. y=l/2
12. (2, 7), (3, 6) 13. y = 3A-4
HINTS TO SELECTED PROBLEMS

3. (i) The equation of the curve is y = x'^ bx^ +13.a2 -10a+ 5.

^
dx
= 4a-2 - 3hv2 + 26a -10 => f ^
dx
= -10
(0,5)
19.16 APPLIED MATHEMATICS-XI

The equation of tangent at (0,5) is

y-5 = (x-0) => y-5 = -10(a:-0) => lOx + y-5 = 0


^dx A0,5)

The equation of the normal at (0,5) is


1
y-5 = -
(x-0) => y-5=^(x-0) => x-lOy+ 50 = 0
dx

ow
(ii) The equation of the curve is y = x'^ - 6x^ + 13x^ - lOx + 5 ...(i)
^
dx
=4x^-18x^ + 26x-10=>f^ = 4-18 + 26-10 = 2
ydx yx=i

e
Putting X = 1 in (i), we get y = 3.

re
The equation of the tangent at (1,3) is
y-3 = (x-l)=>y-3=2(x-l)=>2x-y + l = 0

F
''X=l

The equation of the normal at (1, 3) is


Frl
ou
^ (:t-l)=>y-3=-i(x-l)=>x + 2y-7 = 0
osr
y-3 = -

^dx yx = i kf
oo
(iii) The equation of the curve is y = x^.
Differentiating with respect to x, we get
Y
B

^
dx
= 2x ^ dij] = 0
^dx\o^O)
re
uY

So, the tangent at (0,0) is parallel to x-axis and hence the normal there at is parallel to y-axis.
So, their equations are y = 0 and x = 0 respectively.
ad

^2 2
do

(xiii) The equation of the curve is ^ ^ = 1


...(i)
Point P(xq, yg) lies on (i). Therefore,
in
Re

= 1
F

...(ii)

Differentiating (i) with respect to x, we get


2x 2y dy
= 0 => ^ = etl - ^0
dx
■ ~ a^y
dx
^dxjp
The equation of the tangent at P (xg, yg) ISi
y-yo=(fl ix-xo) yp

=> y-vo = (x-Xg)

^ yyp-yo^ ^ xxp-x^^ ^ _ V _ xxg yyg _ = 1


[Using (ii)]
b^ a
2
.2 b^^~^ b^ "
The equation of the normal at P (xg, yg) is
19.17
TANGENTS

]/-yo=-
(dy^
dx JP

=> y-yo = - (X-Xq) -(y-yo) = ^


yo ^0 ^0 ^0
Xq
(xiv) We have,
1/3
x2/3 + y 2/3=1^ 2^_1/3^23
3
i/3^^0^
dx
^
rfx
= _y;cV3 ^ (dy
[dx ATI)
= -1

The equation of the tangent at (1,1) is


(dy) (x-i) => y-l = -1(^-1) =i>x + y-2 = 0
y-i= ^ dx
ATI)

The equation of the normal at (1,1) is

w
y-i=-
1
(x-l)=> y-1 = i(x-l) => X = y
'df
v^^/(T 1)
(xvi) The equation of the curve is \/ = 4x.
F lo
e
Fre
Differentiating with respect to x, we get
_ dy
2v— =4=>-t^=-=>
. dy 2 f—
dy^ 2
- = 1 for
^ dx
^ dx y dx
AT 2) ^
The equation of the tangent at (1,2) is
r
You

{x-l)=>y-2 = (x-l) => x-y + 1 = 0


s

y-2 =
ook

\dx\i^2)
eB

The equation of the normal at (1,2) is


(x-1) => y-2 = -^(^-1) => x + y-3 = 0
our
ad

v^^/(T 2)
7. The equation of the curve is y = x - 2x + 7.
dY
Re

Differentiating with respect to x, we get


^ = 2X-2
Fin

dx

(i) Let P (*1, y,) be a point on y = - 2a: + 7 such that tangent at P is parallel to the line
2x-y + 9 = 0. Then,
= 2=>2xi-2 = 2=> Xi = 2
^dx Jp
Since P (xj, yi) lies on y = x^ - 2x + 7. Therefore,
yi = x^^-2xj+7 => yi =4-4 + 7 = 7

Hence, required point is (2,7).


The equation of the tangent at (2,7) is
_ fdy^
y-7 = / (x-2) => y-7 = 2(x-2) => 2x-y+3 = 0
19.18
APPLIED MATHEMATICS-XI

(ii) If the tangent at P(.Yj,i/i) is perpendicular to the line 5y-15.Y = 13. Then,
X 3 = -1 => (2.Vi -2)x 3 = -]1 => Ai 5
= -
dx Jp ' 6
Since (a'i , i/j) lies on y = - 2.y + 7.
^ -2X-1+7 =i> ij^‘ = ^-^ +7
217
■■■ yi =
36 3 36
^5 217 V
The equation of the tangent at P - , IS
I 6 36
217 1 5^
y-
36
A ●● (^y) 1
3 6
[dx 3

or, 12a + 36y -227 =0

w
1
8. Let (Aq, ij{) be the point of contact of a line of slope 2 which touches the curve y =
A ^ 3. A - 3 '

Flo
1
Now, y = ^ it 1
dy] 1
A - 3 dx
(■v-3)2 (,Vi - 3f

e
re
But, = 2.

F
dx

1
ur
r
= 2
(Aq - 3)2 fo
=> 2 (aj - 3)2 = -1, which is not possible as LHS is positive and RHS is negative.
ks
Yo

Hence, there is no tangent line of slope 2 to the given


oo

curve,

9. Let P(Aq, i/i) be the point of contact of a


line of slope zero which touches the curve
B

1
y = .2 at point P.
A^ - 2a + 3
re

1
The equation of the curve is y =
u
ad

2 - 2a + 3 ’
Yo

Differentiating with respect to a, we get


dy _ -2(a-1)
d

-2(Aq-l)
Re

dx
(A-2-2A+ 3)2 ^ [dx
in

(V^-2Aq + 3)2
F

^y
It is given that -2- = 0.
dx

-2(AtD
= 0 => Ai1 = 1
(a^2_2.v^ + 3)2
1
Since (A|, y■^) lies on y = 2
A“ - 2a + 3
1 1 1
yi = 1/1 =
^ - 2ai + 3 1-2+3 2

Hence, the equation of the tangent is y - i = 0 (a - 1) or, y = - .

10. Let (a^ , yj) be the point of contact of tangent to the curve y = JSx - 2 which is parallel to
the line 4a - 2y + 5 = 0. Then,
19.19
TANGENTS

= (Slope of the line 4x - 2y + 5 = 0)


dx
A-vi-i/i)
^y = 2
dx
3
3 ^y ...(ii)
Now, 1/ =
2,[3^ ^ dx

From (i) and (ii), we get


3 41
= 2 => 9 = 16(3.vi 2) X‘i —
48

Since {xi, yi) lies on i/ = ./3.\-- 2. Therefore,


y\ = V^'^i l^_2 = 1

w
V16 4
, . T41 3^
So, the point of contact is — / — ●
^ Us 4^

Flo
. Ml 3^ IS

The equation of tangent at 48
—4

ee
3 4n
or, 48.V - 24y = 23

Fr
= 2 .r
^"4 48 J
MULTIPLE CHOICE QUESTIONS (MCQs)
for
ur
Mark the correct alternative in each of the following:
The point on the curve i/ - -v where tangent makes 45'’ angle with .Y-axis i
s
(d) (1,1)
(a)(l/2,l/4) (b) (1/4,1/2) (c)(4,2)
k
Yo

2. If the tangent to the curve x^at‘^,ij = 2at is perpendicular to .v-axis, then its point of
oo
eB

contact is
(a) {a, a) (b) (0, a) (c) (0, 0) (d) (rt, 0)

3 The point on the curve y = y^ - 3y + 2 where tangent is perpendicular to y = y is


r

(d)(2,-2)
ou

(c) (-1,6)
ad

(a) (0, 2) (b) (1,0)


4. The point on the curve y^ = y where tangent makes 45” angle with .r-axis is
Y

(a) (1/2,1/4) (b) (1/4,1/2) (c) (4, 2) (d) (1,1)


5. The point on the curve y = 12y - .y^ where the slope of the tangent is zero will be
Re
nd

(a) (0,0) (b) (2,16) (c) (3,9) (d) (6,36)


Fi

.2
3y + 2 meets Y-axis are
6. The equation of tangent at those points where the curve y = y
(a) Y-y + 2 = 0 = Y-y-l (b) Y + y-l=0=Y-y-2
(c) Y - y -1 = 0 = Y - y (d) Y-y = 0=Y + y
7. The slope of the tangent to the curve Y = f^ + 3f - 8,y = 2t^ -2f -5at point (2, -1) is
(d) 7/6
(a) 22/7 (b) 6/7 (c)-6 ^ ^ - 2y - 3 = 0 is zero
8 At what points the slope of the tangent to the curve y h- y
(a) (3,0),(-1,0) (b) (3,0),(1,2) (c) (-1,0),(1,2) (d) (1,2),(1,-2)
9. If the line y = y touches the curve y = x~ + !?y + c at a point (1,1) then
(b) l?=-l,c=l {c) b =2, c = 1 (d) l7=-2,C=l
(a) b^l,c = 2 3
-1 at Y =1 is
10. The slope of the tangent to the curve y = 3f“ +1, y = t
(a) 1/2 (b) 0 (C) -2 (d) CO
19.20
APPLIED MATHEMATICS-XI

11. The point on the curve y = 6.y - at which the tangent to the curve is inclined at jt/4 to the
line -V + y = 0 is
(a) (- 3’,-27) (b) (3,9) (c) 7/2,35/4) (ci) (0, 0)
12. Any tangent to the curve y = 2x^ + 3x + 5
(a) is parallel to .x-axis (b) is parallel to y-axis
(c) makes an acute angle with x-axis (d) makes an obtuse angle with x-axis
13. The slope of the tangent to the curve .v = + 3f - 8, y = 2f^ - 2t -5 at the point (2, -1) is
(a) 22/7 (b) 6/7 (c) 7/6 (d) -6/7
14. The line y = wx +1 is a tangent to the curve y^ = 4x, if the value of m is

ow
(a) 1 (b) 2 (c)3 (d) 1/2
15. n-ie normal at the point (1,1) on the curve 2y + x^ = 3 is
(a) x + y = 0 (b) x-y = 0 (c) X + y + 1 = 0 (d) x-y=l

e
16. The tangent to the curve y = at the point (0,1) meets .x-axis at ;

re
(a) (0,1) (b)
1

rFl
, 0 (c) (2,0) (d) (0,2)

F
2

17. The equation of tangent to the curve y (1 +x^) = 2 -x, where it crosses .x-axis i;

r
ou
(a) .x+5y = 2 (b) x-5y = 2 (c) 5x-y = 2fo (d) 5x + y = 2
ks
18. The points at which the tangents to the curve y = x 3
-12x +18 are parallel to .x-axis are
(a) (2,-2) (-2,-34) (b) (2, 34) (-2, 0)
oo

(c) (0, 34) (-2,0) (d) (2, 2) (-2, 34)


Y
B

19. The curve y=x^/^ has at (0,0)


re

(a) a vertical tangent (b) a horizontal tangent


(c) an oblique tangent (d) no tangent
ou
Y
ad

ANSWERS
1. (b) 2. (c) 3. (b) 4. (b) 5. (d) 6. (b) 7. (b)
d

8. (d)
9. (b) 10. (b) 11. (b) 12. (c) 13. (b) 14. (a) 15. (b) 16. (b)
in
Re

17. (a) 18. (d) 19. (a)


F

FILL IN THE BLANKS TYPE QUESTIONS (FBQs)


1. The point on the curve i/^ X, the tangent at which makes an angle of 45° with x-axis is
2. The value of a for which y - x^ + ax + 25 touches tlie axis of .x are

3. The points on the curve y = 12x -x^ at which the gradient is zero are
4. yie coordin.ites of a point on the curve y = ,v log, x at which the normal is parallel to the line
5. The coordinates of the point on
the curve y - 2 -1- V4x +1 where tangent has slope - are

6. The slope of the tangent to the curve X


= 3f^+l,y=f^-latx=lis
7. The angle of intersection of the curves y = x^ and x = i/^ at (0, 0), is ....
8. The slope of the tangent to the curve y = b where it crosses y-axis i IS
19.21
TANGENTS

9. The tangent to the curve y = at (0,1) cuts x-axis at the point


10. The slope of the normal to the curve i/^ - xy - 8 = 0 at the point (0,2) is equal to
11. The equation of the tangent to the curve y = x + , that is parallel to x-axis, is
12 The coordinates of the point on the curve y = x^ - 3x + 2 where the tangent is perpendicular
tothelinex-y =0are
13. If slope of tangent to curve y=x^ at a point is equal to ordinate of pomt, then the point i_
IS

ANSWERS

1.
1 1
2. ±10 3. (-2,-16), (2,16) 4. (t’-^-2c-2) 5. (6, 7)
4' 2
b
71
8. - 9. 10. -6 11. 1/ = 3
6. 0
2 a

w
12. (1,0) 13. (3, 27)

1
F lo
VERY SHORT ANSWER QUESTIONS (VSAQs)
Ansioer each of the foUoiving questions in one word or one sentence
question:
’ or as

for F
2. Find the slope of the tangent to the curve x = (2 + 3f -8, y = 2t^ -2(-5atf =2.
ree
. Find the point on the curve y = x^ - 2x + 3, where the tangent is paraUel to x-axis.
per exact recjiiiremeut of the

x-axis, then write the


3. If the tangent line at a point (x, y) on the curve y =/ (x) is parallel to
Your

value of —.
dx
oks

value of — , if the normal to the curve y =/ (x) at (x, y) is parallel to y-axis.


eBo

4. Write the
dx

5. If the tangent to a curve at a pointfx, y) is equally inclined to the coordinate axes, then write
ad
our

the value of—.


dx

6. If the tangent line at a point (x, y) on the curve y = / (x) is parallel to y-axis, find the value of
dx
Re
dY

^y
1
Fin

,y=f.
7. Find the slope of the tangent at the point ‘f on the curve x t

. Write the coordinates of the point on the curve / =rr where the tangent line makes
an
8

angle - with .x-axis.


^ 4
9. Find the coordinates of the point on the curve =3 - 4i- where tangent is parallel to the
line 2x + y 2 = 0.

in. Write the equation of the tangent to the curve y =;


— X -t- 2 at the point where it crosses the
y-axis.
= 2x^ - X + 1 is
n. Write the coordinates of the point at which the tangent to the curve y =
parallel to the line y = 3x + 9.
19.22
APPLIED MATHEMATICS-XI

ANSWERS
6
1. (1.2) 2. - 3. 0 4. 0 5. ±1 6. 0 7.^t^
7

8. 1 i'' 9. 1,1 10. -V + t/ - 2 = 0 11. (1. 2)


U ' 2 (2

ow
e
Fl
re
F
ur
for
ks
Yo
oo
B
u re
ad
Yo
d
Re
in
F
CHAPTER i2Q
PROBABILITY

20.1 INTRODUCTION
In earlier classes, we have learnt about two approaches to the theory of probability, namely,
(i) Statistical approach and (ii) Classical approach. The statistical approach has been discussed
in class IX. It is also known as repeated experiments and observed frequency approach. In this
approach, we have defined the probability of an event as the ratio of observed frequency to the

w
total frequency. Tlae classical approach has been discussed in class X. In this approach, we define

F lo
the probability of occurrence of an event as the ratio of favourable number of outcomes to the
total number of equally likely outcomes. Tliese equally likely outcomes are also known as
elementary events associated to the experiment. Both the theories have some serious
deficiencies and limitations. For instance, these approaches cannot bo applied to the

e
Fre
experiments which have large number of outcomes. The classical definition of probability
cannot be applied whenever it is not possible to make a simple enumeration of cases which can
for
be considered equally likely. For instance, how does it apply to probability of rain? What are the
possible outcomes? We might think that there are two cases 'rain' and 'no rain'. But at any given
locality it will not usually be agreed that they are equally like. The classical approach also fails to
r
You

questions like "what is the probability that a male will die before the age of 60 , what is
oks

answer

the probability tlaat a bulb will burn in less than 2000 hours? etc. In fact, the classical definition is
eBo

difficult to apply as soon as we deviate from the experiments pertaining to coins, dice, cards and
other simple games of chance.
mathematical point of view because an actual
The statistical definition has difficulties from a
ad
our

limiting number may not really exist. For this reason, modem probability theory has been
developed axiomatically. Tins theory of probability was developed by A.N. Kolmogrov
(1903-1987) a Russian Mathematician in 1933. He laid down certain axioms to interpret
probability, in his book 'Foundation of Probability' published in 1933. The axiomatic definition
Re
dY

of probability includes 'both' the classical and statistical approaches as particular cases and
of each of them. In order to understand this approach we must know
Fin

overcomes the deficiencies


.

about some basic terms viz. random experiment, elementary events, sample space, compound
events etc. So, let us begin with the term random experiment as discussed in the following
section.

20.2 RANDOM EXPERIMENTS


The word experiment means an operation which can produce some well-defined outcome(s).
There are two types of experiments viz. (i) Deterministic experiments and (ii) Random or
Probability experiments.
DETERMINISTIC EXPERIMENTS In our day-to-day life, we perform many activities/experiments
which have a fixed outcome or result no matter any number of times they are repeated. Such
experiments are known as deterministic experiments. For example, from the set of all triangles
in a plane if a triangle is choosen, then even without knowing the three angles, we can definitely
say that the sum of the measures of the angles is 180° In fact, when experiments in science and
engineering are repeated under identical conditions, we get the same result every time.
20.2
APPLIED MATHEMATICS-XI

RANDOM OR PROBABILISTIC EXPERIMENTS If an experiment, when repeated under identical


conditions, do not produce the same outcome every time but the outcome in a trial is one of the
several possible outcomes then such an experiment is known as a probabilistic experiment or a
random experiment. In other words, an experiment whose outcomes cannot be predicted or
determined in advance is called a random experiment.
For example, in tossing of a coin one is not sure if a head or a tail will be obtained so it is a
random experiment. Similarly, rolling an unbiased die and drawing a card from a well shuffled
pack of playing cards are examples of random experiments.
20.3 SAMPLE SPACES

In the previous section, we have learnt about random experiments. Throughout this chapter the
term experiment will mean random experiment. Associated to every random experiment there
are two basic terms viz. outcomes (or elementary events) and sample space. In this section, we
will discuss about these two for different random experiments.

w
ELEMENTARY EVENT If n random experiment is performed, then each of its outcomes is known as an
elementary event.

F lo
In other words, outcomes of a random experiment are known as elementary events associated to
it. Elementary events are also known as simple events.
SAMPLE SPACE The set of all possible outcomes of a random experiment is called the sample space

ee
associated with it and it is generally denoted by S.

Fr
If £i / £2 / £3 / are the possible outcomes (or elementary events) of a random experiment,
then S = {£j , £2 ,..., ^j) is the sample space associated to it. for
iLLUSTRATIO.Ni 1 Consider the random experiment of tossing of a coin. The possible outcomes of this
ur
experiment are H and T. Thus, five define
£] = Getting head (H) on the upper face and, £3 = Getting tail (T) on the upper face.
s
ook
Yo

Then, £-j and £2 are elementary events associated to the random experiment of tossing of a coin.
The sample space associated to this experiment is given by S = {£^ , £2).
eB

£| and £2 are generally denoted by H and T respectively. Tluis, we have S = {H ,T\.


ILLUSTRATION 2
Consider the experiment of throwing a die. Let the six faces of a die be marked as
our
ad

1, 2, 3, 4,5 and 6. f the die is thrown, then any one of the six faces may come upward. So, there are six
possible outcomes of this experiment, namely, 1, 2, 3, 4, 5, 6. Thus, five define
Ej = Getting a face marked with number i, where / = 1, 2, 3, 4,5, 6
Y

Then, £^ , £2 ,..., £g are six elementary events associated to this experiment. The sample space
Re
nd

associated to this experiment is S = [Ei ,£2 ,...£6).


In this experiment, elementary even Ej is denoted by i, where / = 1, 2,..., 6. Thus, we have
Fi

S= {1,2, 3,4,5,61.
ILI.USTR.ATIO.N 3 Consider the experiment of tossing two coins together or a coin tivice. In this
experiment the possible outcomes are:
Head on first and Head on second.
Head on first and Tail on second.
Tail on first and Head on second.
Tail on first and Tail on second.
If we define
HH = Getting head on both coins,
HT = Getting head on first and tail on second,
TH = Getting tail on first and head on second,
TT = Getting tail on both coins.
20.3
PROBABILITY

Then, HH, HT, TH and TT are elementary events associated to the random experiment of
tossing of two coins. The sample space associated to this experiment is given by
S =

Similarly, the sample space associated to the random experiment of tossing three coins
simultaneously or tossing a coin three times is given by
S = {HHH, HHT, HTH, THH, TTH, HTT, THT, TTT]
ILLUSTRATION 4 Consider the random experiment in zuhich two dice are tossed together or a die is
tossed twice. Ifzve define
Ejj = Getting number i on the upper face of first die and number j on the upper face
of second die.
where/ = 1, 2,..., 6 and j = 1, 2,..., 6.
Then, Ej- are elementary events associated to this experiment and are generally denoted by (/, j).
Thus,(1,1), (1,2), ...,(1,6), (2,1) ..., (2,6), (3,1),..., (3,6), (4,1)..., (4,6), (5,1), ..., (5,6) and (6,1),

w
..., (6,6) are 36 elementary events associated to the random experiment of tossing two dice and
the sample space associated to it is given by

ILLUSTRATION 5

F lo
S = [(l,l),..., (1,6), (2,1), ...,(2, 6), (3,1),...,(3, 6),...,(6,1), ...,(6, 6)).
Let there be a bag containing 3 white and 2 black balls. Let the zohite balls be denoted

e
Inj Wi , W2 W3 and black balls be denoted by , B2. If we draw fzoo balls from the bag, then there are Co

Fre
elementary events associated to this experiment. These elementary events for are:

Wi , W2 , W3 , B2 , B2 W2 , B2 W3 , W2 , W3 , W2 W3 , and B^ Bj. The set of


all these elementary events is the sample space associated to the experiment.
r
ILLUSTRATION 6 A coin is tossed. If it shozos head, zue drazv a ball from a bag consisting of 3 red and 4
You
ks

black balls; if it shows a tail, we throiu a die. Ifzve denote three red balls asR^, R2 and R^ andfour black
o

balls as B^ , B2 , B3 and B4. Then the elementary events associated to this experiment are :
eBo

HR3 , HR2 , HR3 , HB4 , HB2 , HB3 , HB4,7 1 , T 2 , r 3 , T 4 , T 5 and T6.


The set of these elementary events is the sample space associated to the given random
our
ad

experiment.
RLMARK1 Elementary events associated to a random experiment are also knozvn as indecomposable
events.
dY
Re

ILLUSTRATIVE EXAMPLES
Fin

EXAMPLE 1 From a group of 2 boys and 3 girls, tzvo children are selected. Find the sample space
associated to this random experiment.
SOLUTION Let the two boys be taken as Bj and 63 and the three girls be taken asG;],G2 andG3.
Clearly, there are 5 children, out of which two children can be chosen in ^C2 ways. So, there are
^C2 = 10 elementary events associated to this experiments and are given by
Bi Bj , B| G3 , Bi G2 , Bi G3, B2 Gi, B2 G2 , B2 G3 , Gi G2 , G^ G3 andG2 G3
Consequently, the sample space S associated to this random experiment is given by
S ={B^ B2,Gi , Bi G2 , Bj G3 , B2 Gi , B2 G2 , B2 G3 , Gi G2 , G^ G3 , G2 G3I.
EXAMPLE 2 A coin IS tosscd. If it shozvs head, zve drazv a ball from a bag consisting of 3 red and 4 black
balls; if it shozvs tail, zve throzv a die. What is the sample associated to this experimezit?
SOLUTION Let the three red balls be taken as R^ , R2 / B3 and four black balls be taken as
B-j , B2 , B3 and B4.
20.4
APPLIED MATHEMATICS-XI

If the coin shows head, we draw a ball which can be any one of the 7 balls. So, possible outcomes
are
(H, K|), (H , R2), (H , R3), (H , Bi), (H , B2), (H , B3), {H , B4).
If the coin shows tail, then we through a die which may produce any one of the six numbers on
its upper face. In this case, possible outcomes are (T, 1), (T, 2), (7, 3), (T, 4), (T, 5), (T, 6).
Thus, all elementary events associated to the experiment are:
(H, Ri), (H, R2), (H, R3), (H, Bi), (H, B2), (H, S3}, (H, 64), (T, 1), (T, 2), (7, 3), (7, 4),
(T,5),(7, 6).
Consequently, the sample space S is given by

ow
S = \{H, Ri), {H , R2), {H , R3), (H , Bj), (H , B2), (H , S3), (H , B4 ), (7,1), (7, 2),
(7, 3), (7, 4), (7,5), (7, 6)}.
CXAMPLli 3 An experiment consists of rolling a die and then tossing a coin once if the number on the die
is even. If the number on the die is odd, the coin is tossed twice. Write the sample space for this experiment.

e
re
SOLUTION If the die is rolled and we get an even number (2 or 4 or 6) on its upper face, then we

Flr
toss a coin which may result in head (H) or tail (7). So the possible outcomes in this case are :

F
(2, H), (4, H), (6, H), (2, 7), (4, 7), (6, 7)
ou
If the die is rolled and we get an odd number {1 or 3 or 5) on its upper face, then the coin is tossed
twice which may result in one of the following ways: HH, HT, TH, 77. So, the possible

sr
outcomes, in this case, are

(1,77), (3,77), (5,77). fo


(1, HH), (3, HH), (5, HH), (1, HT), (3, HT), {5, HT), (1,7H), (3, TH), (5, TH),
k
oo
Thus, all elementary events associated to the experiment are:
Y

(2, H), (4, H), (6, H), (2, 7), (4, 7), (6, 7), (1, HH), (1, HT), (1, TH), (1, 77),
reB

(3, HH), (3, H7), (3, TH), (3, 77), (5, HH), (5, HT), (5, TH), (5, 77).
So, the sample space associated to the random experiment is
uY

S = {(2, H), (4, H), (6, H), (2, 7), (4, 7), (6, 7), (1, HH), (1, HT), (1, 7H), (1, 77),
(3, HH), (3, H7), (3, 7H), (3, 77), (5, HH), (5, HT), (5, 7H), (5, 77)}.
ad
do

EXAMPLE 4 The numbers 7, 2, 3 and 4 are written separate!]/ onfourslip^s of paper. The slips are then
put in a box and mixed thoroughly. A person drains two slips from the box, one after the other, without
in

replacement. Describe the sample space for the experiment.


Re

SOLUTION It is given that two slips are drawn from the box one after the other without repl ace-
F

ment.

If the slip drawn in first draw bears number 1, then the slip drawn in second draw may bear any
one of the remaining 3 numbers viz. 2,3 and 4. Possible outcomes in this case are (1,2), (1,3) and
(T4).
If the slip drawn in first draw bears number 2, then the slip drawn in second draw may bear any
one of the remaining three numbers viz. 1, 3 and 4.
Thus, possible outcomes, in this case, are (2,1), (2, 3) and (2,4).
Similarly, possible outcomes when the slip drawn in first draw bears number 3 and 4 are

respectively (3,1), (3,2), (3,4) and (4,1), (4, 2), (4,3).


Thus, all elementary events associated to the random experiment are (1,2) (13) (14) (2 1)
(2, 3), (2, 4), (3,1), (3, 2), (3, 4), (4,1), (4, 2) and (4, 3).
The set of all these elementary events is the required sample space.
EXAMPLE 5 A coin is tossed. If the result is a head, a die is thrown. If the die shoius up an even number,
the die is thrown again. What is the sample space for this experiment.
w
lo
e
re
rF
F
r
fo
u ks
Yo
'IVZ '^Id unioiis sv pjpicijp
oo

ciq UVD spSvfs snouva ]v s.mioopw n]qissO(j ■)ii,'nuu<'Kix3 3aoqv sq} hi SdSuis 33uif} 3uv 3U3i{± >iyviNi;iy

●aDuds 3|diuys pajinba.1 aij4 si s^usAa XjL’^uauiap asavji [p? jo 40s ai[x
B

(9 '9 'H)'(S'9 'h)


e

'(1^ '9 'H) '(e ‘9 'H) ‘iZ '9 'H) 'd '9 'H)
ur

(9 'f 'H) '(Sd'H)


ad
Yo

'if 'f 'H) '(£'t 'H) 'iZ ‘f 'H) 'd 'f 'H)
(9 'Z 'H) 'te '£ 'H) 'if 'Z 'H) '{£ 'Z 'H)
d

'iZ 'Z 'H) 'd d 'H) 'd 'H) '(£ 'H) 'd 'H)' 1
Re
in

9JB 4U9uiuadxa uaAiS aq4 04 p94t’posse s4uaA9 XjB4uauiai9 \\\2 'os


F

(9 '9 'H) '(S‘9 'H)


'if ‘9 'H) '{£ '9' H) '(Z '9 'H) '(l '9 'H)
(9 d'H)'ted 'H)
'if 'f 'H) '{£ 'f' H) '(Z d 'H) 'd d 'H)
(9 'Z'H)'fed‘H)
'd 'z 'H) '(e 'z' H) '(z 'z 'H) 'd 'z 'h)
:ajB S9mo34no a]qissod '9se9 S1144 u[ uibSb ua\oji|4 si 41 J9qiunu u9A9 ire dn SA\oqs 9ip 9i(4 41
i£'H)'i£ 'H) 'd 'H)
:ajB 9SB9 siq4 ui 5901094110 ajqissoj
●p9ddo4S SI 4U9uiu9dx9 aq4 joqiunu ppo ue dn sA\oqs aip aqi J1 puB UMOiq4 si aip b '(h) peoq
SI 911109400 aq4 ji J9A0 SI 4U9uiU9dx9 aqi (j,) 11B4 si 90109400 ai[4 p 'p9SS04 si ui09 v NOIimOS
Ainiavaodd
S‘0Z
20.6
APPLIED MATHEMATICS-XI

flXAMPLE 6 A coin is tossed repeatedly until a head comes for thefirst time. Describe the sample space.
SOLUTION In this experiment, a coin is tossed. If the outcome is head the experiment is over.
Otherwise, the coin is tossed again. In the second toss also if the outcome is head the experiment
is over. Otherwise, the coin is tossed again. In the third toss, if the outcome is head the
experiment is over, otherwise the coin is tossed again. This process continues indefinitely.
Possible outcomes in various tosses may be exhibited as follows:
Hence, the sample space S associated to this random experiment IS

w
Flo
ee
Fr
for
ur
S = {H,TH,TTH,TTTH,TTTTH,....\
k s
KLM.\RK 1
In the aboiye example, the sample space is an infinite set.
Yo
oo

Kr:M.'\RK2
Let us consider the random experiment of drawing two cards from a well shuffled pack of 52
eB

playing cards . There are =1326 elejnentarij events associated to this experiment. So, the sample
space consists of1326 elements. Clearly, it is not convenient to describe the sample space completely. In the
re
mainingpart of this chapter, we will describe the sample space associated to a given random experiment
r
ou
ad

only if it is convenient and does not contain large number of elementary evetits.
Y

EXERCISE 20.1
1.
A coin is tossed once. Write its sample space
Re
nd

2.
If a coin is tossed two times, describe the sample space associated to this experiment.
Fi

3.
If a coin is tossed three times (or three coins are
tossed together), then describe the sample
space for this experiment.
4.
Write the sample space for the experiment of tossing a coin four times.
5.
Two dice are thrown. Describe the sample space of this experiment.
6.
Wliat is the total number of elementary events associated to the random experiment of
throwing three dice together?
7. A coin is tossed and then a die is thrown. Describe the sample space for this experiment.
8.
A coin is tossed and then a die is rolled only in case a head is shown on the coin. Describe
the sample space for this experiment.
9. A coin is tossed twice. If the second throw results in a tail, a die is thrown. Describe the
sample space for this experiment.
10.
An experiment consists of tossing a coin and then tossing it second time if head occurs. If a
tail occurs on the first toss, then a die is tossed once. Find the sample space.
20.7
PROBABILITY

11. A coin is tossed. If it shows tail, we draw a ball from a box which contains 2 red 3 black balls;
if it shows head, we throw a die. Find the sample space of this experiment.
12. A coin is tossed repeatedly until a tail comes up for the first time. Write the sample space for
this experiment.
13. A box contains 1 red and 3 black balls. Two balls are drawn at random in succession
without replacement. Write the sample space for this experiment.
14. A pair of dice is rolled. If the outcome is a doublet, a coin is tossed. Determine the total
number of elementary events associated to this experiment.
15. A coin is tossed twice. If the second draw results ina head, a die is rolled. Write the sample
space for this experiment.
16. A bag contains 4 identical red balls and 3 identical black balls. The experiment consists of
drawing one ball, then putting it into the bag and again drawing a ball. What are the
possible outcomes of the experiment?
17. In a random sampling three items are selected from a lot. Each item is tested and classified
as defective (D) or non-defective (N). Write the sample space of this experiment.

w
18. An experiment consists of boy-girl composition of families with 2 children,
(i) What is the sample space if we are interested in knowing whether it is a boy or girl in
the order of their births?

19. There are three coloured dice of red, white and


F lo
(ii) What is the sample space if we are interested in the number of boys in a family?
black colour. These dice are placed in a bag.
One die is drawn at random from the bag and rolled, its colour and the number on its

e
uppermost face is noted. Describe the sample space for this experiment.
Fre
20. 2 boys and 2 girls are in room P and 1 boy 3 girls are in room Q. Write the sample space for
for
the experiment in which a room is selected and then a person.
21. A bag contains one white and one red ball. A ball is drawn from the bag. If the ball drawn is
white it is replaced in the bag and again a ball is drawn. Otherwise, a die is tossed. Write the
r
You
oks

sample space for this experiment.


22. A box contains 1 white and 3 identical black balls. Two balls are drawn at random in
eBo

succession without replacement. Write the sample space for this experiment.
23. An experiment consists of rolling a die and then tossing a coin once if the number on the die
. If the number on the die is odd, the coin is tossed twice. Write the sample space for
ad
our

is even

this experiment.
24. A die is thrown repeatedly until a six comes up. What is the sample space for this
experiment.
Re
dY

ANSWERS
Fin

1. S ={H,J]
2. S ={HH,HT,TH,TT)
3. S ={HHH,HHT,HTH,THH,HTT,THT,TTH,TTT}
4. S HHTH,HHTT,HTTH,TTHH,rHHT, HTHT,
THTH, TTTH, TTHT, THTT, HTTP, TTTT}
{(1,1), (1, 2), (1, 3), (1, 4), (1,5), (1, 6), (2,1), (2, 2), (2, 3), (2 4), (2,5) (2 6)
5. S = 3,1), (3, 2), (3, 3), (3, 4), (3, 5), (3, 6), (4,1), (4, 2), (4-3), (4 4) (4 5) (4 6)
5,1), (5, 2), (5, 3), (5, 4), (5,5), (5, 6), (6,1), (6, 2), (6, 3), (6, 4), (6,5), (6, 6)1
6. 216

7. S = {(H,1),{H, 2),(H, 3),(H, 4),(H,5), (H, 6),{T,1),(T, 2), (T, 3),(T, 4),(T,5), (T, 6)1
8. S = {T,(H,1),(H, 2),(H, 3),(H,4),(H,5),(H,6)1
9. S = {HH,TH,(HT,1),(HT, 2),{HT, 3),(HT, 4),(HT,5),(HT, 6), (TT,1),{TT, 2), (TT, 3),
(TT, 4), (TT, 5), (TT, 6)1
10. S = {(T, 1), (T, 2), (T, 3), (T, 4), (T, 5), (T, 6), (H, H), (H, 7)1
20.8 APPLIED MATHEMATICS-XI

11. S = {{T,RiUT,R2UT,B^UT,Bo),{T,B^UH,lUH,2),{H,3)AH,4)AH,5UH,6 )}
12. S = }
13. S =
HR, BjX (Rr B2), (R, B3), (Bi , R), (Sj , B2I (61 , B3), {B2 , Bi), {B^, 63), (63 , R),
(B3,B),(63,Bi),(63,B2)}
14. 42

15. [TT, HT, {TH, 1), {TH, 2), (TH, 3), (TH, 4), (TH, 5), (TH, 6), {HH, 1), (HH, 2), (HH, 3),
(HH, 4), (HH, 6)}
16. RR,RB,BR,BB
17. S = IDDD,DDN,DND,NDD,DNN,NDN,NND,NNN]
18. (i) S = {(Bj, 62), (B^ , G2), (G] , Bj), (G] , G2)) (ii) S = {0,1,2}
19. S = {(R,1),(R,2),(R, 3),{B,4),(R,5),(B,6),(B,1),(B,2),(B, 3),(6,4),(B,5),(B,6)
(W, 1), (W, 2), (W, 3), (W, 4), (W, 5), (W, 6)|

w
20. S
= {{P, Bi), (P, B2), (P, Cl), (P, G2), (Q, B3), (Q, G3), (Q, G4), (Q, G,)l
21. S = {(W, W),(W, R),(R,1),(R,2),(R, 3),(R,4),(R,5),(R,6)}
22. S = {WB, BW, BB]

Flo
23. S = 1(2, H), (2, P), (4, H), (4, T), (6, H), (6, T), (1, HH), (1, HP), (1, PH), (1, PP), (3, HH),

e
(3, HP), (3, PH), (3, PP), (5, HH), (5, HP), (5, PH), (5, PP)}

re
24. S = {6, (1, 6), (2, 6), (3, 6), (4, 6), (5, 6), (1,1, 6), (1, 2, 6), (1, 3, 6), (1, 4, 6), (1,5, 6),

F
(2,1,6), (2, 2, 6), (2, 3, 6), }
20.4 EVENT
ur
f or
In the previous section, we have learnt about sample spaces associated with several random
experiments. In this section, we will introduce an important term associated with a random
ks
experiment.
Yo
oo

EVENT
A subset of the sample space associated with a random experiment is called an event.
Consider the random experiment of throwing a die. The sample space associated with this
B

experiment is S = {1, 2, 3, 4, 5, 6}.Clearly, S has 2^ = 64 subsets.


re

Each one of these 64 subsets is an event associated with the random experiment of throwing a
die.
u
ad
Yo

For Example, =|2, 4, 6), B ={1, 3, 5}, C = {3, 4,5, 6], D - [1, 2, 6| etc. are events as they are
subsets of S.

These events A, B and C can also be described in words as follows:


d
Re

A = Getting an even number, B = Getting an odd number,


in

C = Getting a number greater than 2


F

However, there is no general description in words for the event D. Thus, we find that some
events associated with a random experiment may be described in words. However, it is not
possible for every event.
Consider the experiment of tossing three coins at a time. The sample space S associated with this
experiment is S = {HHH, HHT,THH, HTH ,TTH,THT, HTT,TTT}.Let
A - {HHP,HPH,PHH),B = {HHH, HHT, HTH,THH\
C - IHHH, HHP, HPH, PHH, PPH, HPP, PHP} and, D = {HHH,TTT, HTH]
Clearly, A, B, C and D, being subsets of S, are events associated with the random experiment of
tossing three coins (or tossing a coin three times). These events can also be described in words as
follows:

A - Getting two heads, B = Number of heads exceeds the number of tails,


C = Getting at least one head.
But, event D caimot be described in words.
20.9
PROBABILITY

REMARK Single dement subsets of sample space associated with a random experiment define
elementary events associated with the random experiment.
OCCURRENCE OF AN EVENT An event A associated to a random experiment is said to occur if any one
of the elementary events associated to it is an outcome.
Thus, if an elementary event £ is an outcome of a random experiment and A is an event such that
EeA, then we say that the event has occurred.
Consider the random experiment of throwing an imbiased die. Let A be an event of getting an
even number. Then, A = {2, 4, 6}. Suppose in a trial the outcome is 4. Since 4 e A, so we say that
the event A has occurred. In another trial, let the outcome be 3, since 3 g A, so we say that in this
trial the event A has not occurred.
Suppose a die is thrown and the outcome of the trial is 4. Then, we can say that each of the
following events have occurred:

w
(i) Getting a number greater than or equal to 2, represented by the set |2, 3, 4,5, 6}
(ii) Getting a number less than or equal to 5, represented by the set {1, 2, 3, 4,5}.
On the basis of the same outcome, we can also say that the following events have not occurred:

Flo
(i) Getting an odd number represented by the set (1,3, 5)

e
(ii) Getting a multiple of 3, represented by the set |3,6}.

re
Let us now consider the random experiment of throwing a pair of dice. If (2,6) is an outcome of a

F
trial, then we can say that each of the following events has occurred:
(i) Getting an even number on first die. (ii) Getting even numbers on both dice,
ur
r
(iii) Getting 8 as the sum of the numbers on two dice. fo
However, on the basis of the same outcome, one can also say that following events have not
ks
occurred;
Yo

(i) Getting a multiple of 3 on first die. (ii) Getting an odd number on first die.
oo

(iii) Getting a doublet.


B

20.5 ALGEBRA OF EVENTS


re

In this section, we shall see how new events can be constructed by combining two or more

events associated to a random experiment.


u
ad

Let A and Bhe two events associated to a random experiment with sample space S. We define
Yo

the event "A or B" which is said to occur if an elementary event favourable to either A or B or
both is an outcome. In other words, the event "A or B " occurs if either A or B or both occur i.e. at
d

least one of A and 6 occurs. Thus, "A or B " is represented by the subset A u B of the sample
Re
in

space S.
F

For example, in a single throw of a die consider the following events;


A = Getting an even number, B = Getting a multiple of 3.
These two events are described by the sets {2, 4, 6} and {3, 6} respectively.
Clearly,
A\jB - Getting a number which is eitlier even or a multiple of 3 or both = |2, 3, 4, 6).
Similarly, if A, Band C are three events associated to a random experiment, then A u B uC
denotes the occurrence of at least one of the three events.
The event “A and B" is said to occur if an elementary event favourable to both A and B is an
outcome. In other words, the event "A and B" occurs if A and B both occur. The event A and B is
denoted by A n B.
For example, in a single throw of a pair of dice if we define
A = Getting an even number on first-die
and. B = Getting 8 as the sum of the numbers on two dice.
20.10
APPLIED MATHEMATICS-Xi

Then,
Ar\B = Getting an even number on first die such that the sum of the numbers is 8
= 1(2, 6), (6, 2), (4, 4)1.
NEGATION OF AN EVENT Corresponding to every event associated to a random experiment, we define
an event " not A" ivhich is said to occur ivhen and only when A does not occur.

For example, in a single throw of a die if A denotes the event that the outcome is an odd number.
Then A ={1,3,5} and A does not occur if the outcome is any one of the outcomes 2,4,6. Thus, the
event "not A
is represented by the set A and is called thecomplementary event of A or
negation of A.
Sometimes the occurrence of one event implies the occurrence of other. For example, in a single
throw of a die if A denotes the event that the outcome is 2 or 4 and B denotes the event that the

ow
outcome is even. Then, A = {2, 4} and B = {2, 4, 6}. Clearly, the occurrence of A implies the
occurrence of B. For if 2 or 4 occurs, we say that the outcome is an even number.
Thus, if the occurrence of an event A implies the occurrence of event B, then we say that "A
implies B". Clearly, if A implies B, then we have A c 6.

e
Verbal description of the event

Fl Equivalent set theoretic notation

re
Not A A

F
A or 8 (at least one of A or 6) AuB
ur
or
A and B AnB
sf
A but not B AnB

Neither A nor B
k
AnB
Yo
oo

At least one of A, 6 or C AuBuC


B

Exactly one of A and B (A n B ) u(A n B)


re

All three of A, B and C A n B nC

Exactly two of A, B and C (AnBnC)u(AnB nC)u(A nBnC)


u
ad
Yo

In the above discussion and in the previous sections, we have seen that the events associated to a
random experiment are generally described verbally, and it is very important to have the ability
d

of conversion of verbal description to equivalent set theoretical notations. In the following table,
Re
in

we give verbal descriptions of some events and their equivalent set theoretic notations for ready
reference.
F

ILLUSTRATION
If A, B and C are three arbitrary events. Find the expression for the events noted below.
in the context of A, B and C.
(i) Only A occurs (ii) Both A and B, but not C occur
(iii) All the three events occur (iv) At least one occurs
(v) At least two occur (vi) One and no more occurs
(vii) Two and no more occur (viii) None occurs
(ix) Not more than two occur.

SOLUTION (i) AnB nC (ii) A nBnC (iii) A nBnC (iv) A u Bu C


(v) (AnB)u^BnQu(AnC)u(AnBnC)
(vi) (AnBnC)u(Aj^BnC)u(AnBnC)
(vii) (AnBnC) u {A nBnC) u(A nBnC)
(viii) AnBnC=A^BKjC
(ix) (AnBnC)u(AoBnC)u(AnBnC)u(An6nC)u(A nBnC)u(AnB nC).
20.11
PROBABILITY

20.6 TYPES OF EVENTS

Let there be ti elementary events associated with a random experiment. Then the corresponding
sample space has n elements and hence 2” subsets. Each subset of S is an event associated to the
random experiment and the sample space is the universal set of these events. These 2" events are
divided into different types on the basis of their nature of occurrence. In this section, we shall
learn about such types.
CERTAIN (OR SURE) EVENT An event associated with a random experiment is called a certain event if it
always occurs whenever the expicrimenl is performed.
For example, associated with the random experiment of rolling a die, the event A "Getting an
even number or an odd number" is a certain event. Clearly, this event is represented by the set
{1, 2, 3, 4,5, 6} which is the sample space of the experiment.
If S is the sample space associated with a random experiment. Then, S, being subset of itself,
defines an event. Also, every outcome of the experiment is an element of S, so the event
represented by S always occurs whenever we perform the experiment. Consequently, the event
represented by S is a certain event.

w
Thus, the sample space associated with a random experiment defines a certain event.
IMPOSSIBLE EVENT

never occurs whenever the experiment is pterformed.


Consider the experiment of rolling a die. Let A be
F lo
An event associated with a random experiment is called an impossible event if it

the event "The number turns up is divisible by

e
7". Clearly, none of the possible outcomes 1,2, 3,4, 5, 6 is divisible by 7. So, the event A cannot

Fre
occur at all. In other words, there is no outcome belonging to set representing event A.So, the set
A is the null set.
for
If S is the sample space associated with a random experiment, then the null (empty) set (j)is a
subset of S and no outcome of the experiment is a member of (j). So, the event represented by (jiis
r
an impossible event.
You
oks

COMPOUND EVENT An event associated with a random experiment is a compound event, if it is the
eBo

disjoint union of two or more elementary events.


In other words, an event having more than one sample point is called a compound event.
In fact, other than elementary events and impossible events associated with a random
ad
our

experiment, all events are compoimd events as they are obtained by combining two or more
elementary events.
For example, in a single throw of an ordinary die there are, 6 elementary events and the total
Re
dY

number of events is 2^ = 64. So, 2^ - (6 +1) = 57 is the total number of compound events.
ri;mark If there are n elementary events associated to a random experiment, then the sample space
Fin

associated to it has n elements and so there are 2" subsets of it. Out of these 2” subsets there are n single
element subsets. These single element subsets define n elementary events and the remaining 2" - (h + 1)
subsets (excluding nidi set) define compmind events. Some of these compound ei’ents can be described in
words whereas for others there may not be any general description.
If a pair of dice is thrown together, then there are 36 elementary events associated to this
experiment. The sample space associated to this experiment is:
S = 1(1,1), (1,2)...., (1, 6), (2,1), (2,2)...., (2,6), (6,1), (6,2),...., (6,6)1
If we define the event A as "Getting a doublet", i.e. A ={(1,1), (2, 2),...., (6,6)j
Clearly, it is a compound event obtained by combining 6 elementary events,
Similarly, the event B given by "Getting 8 as the sum can be written as
B = 1(2, 6), (6, 2), (4, 4), (3,5), (5, 3))
20.12 APPLIED MATHEMATICS-XI

It is also a compound event obtained by combining 5 elementary events.


MUTUALLY EXCLUSIVE EVENTS Tzuo or more events nssociated with a random experiment are said to
be mutualli/ exclusive or incompitable events if the occurrence of any one of them prez^ents the occurrence
of all others i.e., if no tzoo or more of them can occur simultaneousli/ in the same trial.
Clearly, elementary events associated with a random experiment are always mutually
exclusive, because elementary events are outcomes (results) of an experiment when it is

w
performed and at a time only one outcome is possible.
Consider the random experiment of rolling a die. Let A, B, Che three events associated with the
experiment as given below:
A = Getting an even number, B = Getting an odd number, C = Getting a multiple of 3.

e
These events in set theoretical notations are:/I = {2, 4, 6), B = {1, 3,5) and C = {3,6}.

re
o
Clearly,/! nB = (}),/! nC <j), B nC ^ <|) and/! nBnC = <{).

r
So, A and B are mutually exclusive events but A and C as well as B and C are not mutually
exclusive. However, A, B and C taken all the three together are mutually exclusive events.

F
In the experiment of throwing a pair of dice events A =Getting 8 as the sum and 6 =Getting
oF
ul
an

even number on first die are not mutually exclusive, because A n B = {(2, 6), (6, 2), (4, 4)}
Let two cards be drawn from a well-shuffled pack of 52 cards. Consider the following events:

sr
A = Getting both red cards, B = Getting both black cards.

ko
Clearly, A and 6 are mutually exclusive events because two cards drawn cannot be both red and
black at the same time.

EXHAUSTIVE EVENTS Tzvo or more events associated with


of
a random experiment are exhaustive if their
o
union is the sample space i.e. events Aj, A2,...., A„ associated zuith a random experiment with sample
Y
space Bare exhaustive if A| u A2 vj ....kj A = S.
rB

Thus, a set of events associated with a random experiment is an exhaustive set of events if one of
eY

them necessarily occurs whenever the experiment is performed.


It is evident from the above definition that all elementary events associated with a random
u

experiment form a set of exhaustive events.


d

Consider the experiment of drawing a card from a well shuffled deck of playing cards. Let A be
o
ad

the event " card is red", B be the event "card is black." Clearly, A and B are exhaustive events
in

because A u B = S.

In a single throw of an ordinary die, let us consider the following events:


Re
F

At = Getting an even number = {2, 4, 6}, A2 = Getting an odd number = |1, 3,5],
A3 = Getting a multiple of 3 = |3, 6|, A4 = Getting a number greater than 3 = |4,5, 6}
We observe that Aj u A2 =S.A\so,A-y u A2 u A3 UA4 =S.But,Ai w A3 S.So,A-y and A2 are
exhaustive events. Also, A^,A2,A3,A4 are exhaustive events but A^ and A3 are not
exhaustive events.

MUTUALLY EXCLUSIVE AND EXHAUSTIVE SYSTEM OF EVENTS Let S be the Sample space associated
with a random experiment. A set of events A-j , A2 ,..., A„ is said to form a set of mutually exclusive and
exhaustwe system of eluents if
(i) Aj u A2 ... u A, - S i.e. ei’ents Aj , A2 ,..., A„fonn an exhaustive set of ez’ents.
(ii) Aj r\Aj = for i ^ j i.e. events A^, A2,..., A„ are mutually excluswe.
Clearly, elementary events associated with a random experiment always form a system of
mutually exclusive and exhaustive events.
In a single throw of a die, the events A = Getting an even
number and, B = Getting an odd
number are mutually exclusive and exhaustive events.
20.13
PROBABILITY

Consider the experiment of drawing a card from a well-shuffled deck of 52 playing cards. Let
A-^ , Ai , Aj,, four events defined as follows:
Ai = Card drawn is spades, A2 = Card drawn is clubs,
A3 = Card drawn is hearts, A4 = Card drawn is diamonds
Since the card drawn is one of the four types of cards, so one of these events surely occurs
whenever the experiment is performed. Also, if one of these events occurs, the others cannot
occur. So, Ai, A2, A3 and A4 form a mutually exclusive and exhaustive system of events.
Suppose a die is thrown once. Let A be the event "Getting a number greater than 3”, B be the
event "Getting a number less than 5". Then, A - [4,5, 6] and B = 11,2,3,4). Clearly,
A u B = S and A n B = (4) <{x So, events A and 6 are exhaustive but not mutually exclusive.
FAVOURABLE ELEMENTARY EVENTS Let S be the sample space associated with a random experiment

w
and A be an event associated with the experiment. Then, elementary events belonging to A are knoum as
favourable elementary events to the event A.
Thus, an elementary event £ is favourable to an event A if the occurrence of E ensures the

Flo
happening or occurrence of event A.

e
In a single throw of an ordinary die, let A be the event "Getting a multiple of 3 . Clearly,

re
A = {3, 6}. So, there are two elementary events favourable to A.

F
Consider the random experiment of throwing a pair of dice. Let A be the event "Getting 8 as the
ur
sum". Then, A = 1(2, 6), (6, 2), (4, 4), (5, 3), (3,5)). Clearly, A occurs if any one of the
elementary events (2, 6), (6, 2}, (4, 4), (5, 3), and
these elementary events are favourable to event A.
f or
(3, 5) is an outcome of the experiment. So, all
ks
Consider a random experiment of drawing 4 cards from a well-shuffled deck of 52 playing
Yo
oo

C4 elementary events associated with this experiment as 4 cards can be drawn


52
cards. There are
26
B

out of 52 cards in
52
C4 ways. Let A be the event "Getting all red cards". There
are
C4

elementary events favourable to A, because 4 red cards can be chosen out of 26 red cards in 26C4
re

ways. In this case, it is not convenient to list all favourable elementary events.
u
ad
Yo

ILLUSTRATIVE EXAMPLES

An experiment consists of rolling die until a 2 appears,


d

EXAMPLE 1 Ih
Re

(i) How many elements of the sample space correspond to the event that the 2 appears on the k
in

roll of the die?


F

(ii) How many elements of the sample space correspond to the event that 2 appears not later than
the k^^‘ roil of the die?
SOLUTION (i) 2 appears on k^ roll of the die means that each one of the first (k -1) rolls have 5
outcomes (1, 3,4, 5, 6) and k^ roll results in 1 outcome i.c. 2. k-1
5x5x ...x5xl =5
Number of elements of the sample space in the event =
(fc-1) times
in the first roll or in second
(ii) 2 appears not later than roll means that 2 may appear
roll or in third roll,..., or in roll.
From (i), the number of elements of the sample space corresponding to the event that 2 appears
on the k^^ roll of the die is 5^
Hence, The number of elements of the sample space corresponding to the event that 2 appears
not later than the roll of the die
20.14
APPLIED MATHEMATICS-XI

=5 1-1+52-1+53-1 +...+ 5 k-l 5^-1


= 1+5 + 52 + ... + 5^“l
5-1 4

EXAMPLE 2 An experiment involves rolling a pair of dice and recording the numbers that come up.
Describe the following events.
A = the sum is greater than S,B = 2 occurs on either die., C = the sum is at least 7 and a multiple of 3.
Also, find A n B , B r\C and Ar^C.

ow
Are: (i) A and B mutiialh/ exclusive? (ii) B and C mutually exclusive?
(iii) A and C mutualli/ exclusive?
SOLUTION The sample space associated with the given random experiment is given by
S = {(1,1), (1, 2), (1, 3), (1, 4), (1,5), (1, 6), (2,1), (2, 2), (2, 3), (2, 4), (2,5), (2, 6)

e
(3,1), (3, 2), (3, 3), (3, 4), (3, 5), (3, 6), (4,1), (4, 2), (4, 3), (4, 4), (4,5), (4, 6)

re
(5,1), (5, 2), (5, 3), (5, 4), (5,5), (5, 6), (6,1), (6, 2), (6, 3), (6, 4), (6,5), (6, 6))

Flr
We have.

F
A = The sum is greater than 8
ou
^ = {(3, 6), (4,5), (5, 4), (6, 3), (4, 6), (6, 4), (5,5), (6,5), (5, 6), (6, 6))
6 = 2 occurs on either die

sr
fo
B = {(2,1), (2, 2), (2, 3), (2, 4), (2,5), (2, 6), (1, 2), (3, 2), (4, 2), (5, 2), (6, 2)|.
and. C = The sum is atleast 7 and a multiple of 3 k
C = The sum is 9 or, 12. = |(3, 6), (6, 3), (4, 5), (5, 4), (6, 6)).
oo
(i) Clearly, A nB = (Ji. So, A and 6 are mutually exclusive events,
Y

(ii) Clearly, 6 n C = (]>. So, 6 and C are mutually exclusive events,


reB

(iii) Clearly, AoC = {(3, 6), (6, 3), (4,5), (5, 4), (6, 6)) # <|). So, A and C are
not mutually
exclusive events.
uY

EXAMPLES From a group of 2 boys and 3 girls, two children are selected at random. Describe the events.
(i) A =both selected children are girls, (ii) B = the selected group consists of one boy and one girl.
ad
do

(iii) C = at least one boy is selected.


Which pair (s) of events is (arc) mutualh/ exclusive?
in

SOLUTION Let 6^, 62 be two boys and , G2 , G3 be three girls. Then, the sample space
Re

associated with the random experiment is


F

S - {B-j 62 , B| G] , 6| G2,6-] G3,62 G-^, B2G2,62 G3 , G| G2 , Gj G3 , G2 G3l


(i) We have.
A
Both selected children are girls = {Gj G2 / Gi G3 , G2 G3}
(ii) We have.

B = The selected group consists of one boy and one girl.


6
= {B^ Gi, 61 G2,6, G3,62 G| , 62 G2 , B2 G3}
(iii) We have.
C = At least one boy is selected = {B^ 62 , Bj G| , 6, G2 , B^ G3,62 Gj , 63 G2,63 G3}
Clearly, A n B = (j) and A nC = <|x So, A and B, A and C are
two pairs of mutually exclusive
events.

EXAMPLE 4 Tzvo dice are thrown and the sum of the numbers which come up on the dice is noted. Let us
consider the following events:
20.15
PROBABILITY

A = The sum is evai, B = The sum is multiple of 3, C = The sum is less thmi 4,
D = The sum is greater than 11
Which pairs of these events are mutually exclusive?
SOLUTION The sample space associated with the random experiment is given in example 1.
We have.
A = The sum is even
= The sum is either 2 or 4 or 6, or 8 or 10 or 12
= {(1,1), (2, 2), (1, 3), (1, 5), (2, 4), (2, 6), (3,1), (3, 3), (3, 5), (4, 2), (4, 4), (4, 6),
(5,1), (5, 3), (5,5), {6, 2), (6, 4), (6, 6))
6 = The sum is a multiple of 3
= The sum is either 3 or 6 or 9 or 12
- {(1, 2), (2,1), (1,5), (5,1), (2, 4), (4, 2), {3, 3), (3, 6), (6, 3), (4, 5), (5, 4), (6, 6)}
C = The sum is less than 4 = (The sum is 2 or 3 = {(1,1), (1, 2), (2,1)}

w
D = The sum is greater than 11 = The sum is 12 = 1(6, 6)}

Similarly, we observe that A r\C


F lo
We observe that AnS = 1(1,5), (2, 4), (3, 3), (4, 2), (5,1), (6, 6)1 ^ 4.. So, A and 6 are not
mutually exclusive events.
^, A r\D ^ ^, B r\C ^ ^, B r\D ^ ^ and, C D = (j).

e
Fre
Hence, C and D are mutually exclusive events.
EXERCISE 20.2
for
1, A coin is tossed. Find the total number of elementary events and also the total number
events associated with the random experiment.
r
2. List all events associated with the random experiment of tossing of two coins. How many of
You
oks

them are elementary events?


eBo

T. Three coins are tossed once. Describe the following events associated with this random
experiment:
A = Getting three heads, B = Getting two heads and one tail,
ad
our

C = Getting three tails, D = Getting a head on the first coin,


(i) Which pairs of events are mutually exclusive?
(ii) Which events are elementary events?
Re
dY

(iii) Which events are compound events?


Fin

4. In a single throw of a die describe the following events:


(i) A = Getting a number less than 7 (ii) B = Getting a number greater than 7
(iii) C = Getting a multiple of 3 (iv) D = Getting a number less than 4
(v) E = Getting an even number greater than 4 (vi) F = Getting a number not less than 3.
Also, find A<jB,Ar\B,Br^C,EnF,DnF and F.
5. Three coins are tossed. Describe
(i) two events A and 6 which are mutually exclusive,
(ii) three events A, B and C which are mutually exclusive and exhaustive,
(iii) two events A and B which are not mutually exclusive,
(iv) two events A and B which are mutually exclusive but not exhaustive.
6. A die is thrown twice. Each time the number appearing on it is recorded. Describe the
following events;
20.16
APPLIED MATHEMATICS-X)

(i) A = Both numbers are odd. (ii) B = Both numbers are even.
(hi) C = sum of the numbers is less than 6
Also,find/4 B , A n B, A uC , A nC.Which pairs of events are mutually exclusive?
7. Two dice are thrown. The events A, B, C, D, E and F are described as follows:
A = Getting an even number on the first die.
B = Getting an odd number on the first die.
C = Getting at most 5 as sum of the numbers on the tu'^o dice.
D = Getting the sum of the numbers on the dice greater than 5 but less than 10.
E = Getting at least 10 as the sum of the numbers on the dice.
F = Getting an odd number on one of the dice,
(i) Describe the following events: A and B,BorC ,B and C , A and E, A or F , A and F
(ii) State true or false:
(a) A and B are mutually exclusive,
(b) A and B are mutually exclusive and exhaustive events,

w
(c) A and C are mutually exclusive events,
(d) C andD are mutually exclusive and exhaustive events,

(f) A' and B' are mutually exclusive events,


F lo
(e) C,D and E are mutually exclusive and exhaustive events,

(g) A, B, F are mutually exclusive and exhaustive events.

e
8. The numbers 1, 2, 3 and 4 are written separately on four slips of paper. The slips are then

Fre
put in a box and mixed thoroughly. A person draws two slips from the box, one after the
other, without replacement. Describe the following events:
for
A = The number on the first slip is larger than the one on the second slip.
B = The number on the second slip is greater than 2
r
C = The sum of the numbers on the two slips is 6 or 7
You
oks

D =
The number on the second slips is twice that on the first slip.
eBo

Which pair(s) of events is (are) mutually exclusive?


9. A card is picked up from a deck of 52 playing cards,
(i) What is the sample space of the experiment?
ad
our

(ii) What is the event that the chosen card is black faced card?

ANSWERS
Re

1. 2, 4
dY

2. {HH}, |HT1, [TH], ITT), |HH, HT], {HH, TH\, [HH, TT\, {HT,TH), [HT, TJ], {TH 7T)
Fin

{HH, HT, TH], [HH, HT, TT\, [HH, TH, TT), \HT, TH, TT\, {HH, HT, TH, TT\ .4
3. A = {HHH\,B = [HHT,THH,HTH],C = \TTT\,D = [HHH, HHT, HTH, HTT]
(i) A,B; A,C; B, C; C,D (ii) A and C (iii) B and D
4. (i) A = {1,2,3,4,5,61 (ii) (f) (hi) C = {3, 6j
(iv) D = (1,2, 3} (V) £ = (6} (vi) F = (3,4,5,61
AkjB = (1, 2, 3, 4,5, 6|, A n B = (|), BnC = (]), Enf = {6),D nf = [3] , F = \1, 2]
5. (i) A = Getting at least two heads, B = Getting at least two tails,
(ii) A = Getting at most one head, B = Getting exactly two heads,
C = Getting exactly three heads,
(iii) A = Getting at most two tails, B = Getting exactly two heads
(iv) A = Getting exactly one head, B = Getting exactly two heads.
6. (i) A = l(l,l),(l,3),(l,5),(3,l),(3, 3),(3,5),(5,1),(5, 3),(5,5)j
(ii) B = 1(2, 2), (2, 4), (2, 6), (4, 2), (4, 4), (4, 6), (6, 2), (6, 4), (6, 6))
20.17
PROBABILITY

(iii) C = 1(1,1), (1, 2), (2,1), (1, 3), (3,1), (2, 2), (1, 4), (4,1), {2, 3), (3, 2)|
/IvjB = 1(1,1), (1, 3), (1,5), (3,1), (3, 3), (3, 5), (5,1), (5, 3), (5,5), (2, 2), (2, 4), (2, 6),
(4, 2), (4, 4), (4, 6), (6, 2), (6, 4), (6, 6)1
A =
AuC = [(1/1). a- 3)/ (1/ 5), (3,1), (3, 3), (3,5), (5,1), (5, 3), (5,5), (1, 2), (1, 4), (2,1),
(2, 2), (2, 3), (3, 2), (4,1)1
AnC = 1(1,1), (1, 3), (3,1)1 and SnC
A and B, B and C are pairs of mutually exclusive events.
7. (i) A n B = (|)
B u C = 1(1,1), (1, 2), (1, 3), (1, 4), (1,5), (1, 6), (2,1), (2, 2), (2, 3), (3,1), (3, 2)
(3, 3), (3, 4), (3,5), (3, 6), (4,1), (5,1), (5, 2), (5, 3), (5, 4), (5, 5), (5, 6)1

ow
B n C = 1(1,1), (1, 2), (1, 3), (1, 4) (3,1), (3, 2)1
An£ = 1(4, 6),(6, 4), (6,5), (6, 6)1
A n F = 1(1, 2), (1, 4), (1, 6), (2,1), (2, 2), (2, 3), (2, 4), (2,5), (2, 6), (3, 2), (3, 4), (3, 6),
(4,1), (4, 2), (4, 3), (4, 4), (4, 5), (4, 6), (5, 2), (5, 4), (5, 6), (6,1), (6, 2), (6, 3),

e
(6, 4), (6, 5), (6, 6)1

re
rFl
A n f = 1(2,1), (2, 3), (2,5), (4,1), (4, 3), (4,5), (6,1), (6, 3), (6,5)1
(ii) (a) True, (b) True (c) False (d) False (e) True (f) True (g) False

F
8. A = 1(2,1),(3,1),(3, 2),(4,1), (4, 2),(4, 3)1,6 = {(1, 3), (2, 3), (1, 4), (2, 4), (3, 4), (4, 3)1
C = 1(2, 4), (3, 4), (4, 2), (4, 3)1, D = |(1, 2), (2, 4)j

r
ou
9.
A and D form a pair of mutually exclusive events.
(i) Tlie sample space is the set of 52 cards,
fo
ks
(ii) Required event is the set of jack, king and queen of spades and clubs.
oo

HINTS TO SELECTED PROBLEMS


Y
eB

3. We have.
A^{HHH],B = {HHT, HTH, THH}, C = fFTT}, D = {HHH, HHT, HTH, HTT}
ur

(i) We observe that A n B = (j), A n C = (j>but A oD ^ ( )


ad

So, A, B and A, C are pair of simutally exclusive events.


Yo

Also, B n C = 4) but B n D (ji So, B, C is a pair of muHially exclusive events.


Finally, C n D = 4). So, C and D form a pair of mutally exclusive events,
d
Re

(ii) Clearly, HHH and TTT may be outcomes of the random experiment of tossing three
in

coins. So, A and C are elementary events,


F

(iii) We observe that events 6 and D are obtained by combining more than one elementary
events. So, 6 and D are compound events.
5 Tlie smple space associated to the random experiment of tossing three coins is
S = {HHH, HHT, THH, HTH, HTT, THT, TTH, TTT}
(i) Clearly, A = \HHT.THH, HHT} and B = (TTH, THT, HTT]
are mutually exclusive
events,

(ii) We observe that A = {HHH, TTT}, B = {HHT, HTH, THH} and C = {HTT, THT, TTH}
are exhaustive and mutually exclusive events. Because, AnB = ^-BnC -C nA and
A B'-jC = S.

(iii) We observe that the events A = {HHH, HHT, HTH, THH} and B = [HHT,THH,
HTH, HTT, THT, TTH, TTT} are not mutuually exclusive, because A n B 4).
(iv) Events A = [HHT, HTH, THH} B = {TTT, TTH, HTT, THT} are mutually exclusive but
not exhausive as A r\B = 4>but A u B 5^ S.
20.18
APPLIED MATHEMATICS-XI

20.7 AXIOMATIC APPROACH TO PROBABILITY


The axiomatic approach to probability is deduced from the mathematical concepts laid down in
the previous sections. It is based upon certain axioms. In this approach, for a given sample space
associated to a random experiment, the probability is considered as a funcHon which assigns a
non-negative real number P{A) to every event A. This non-negative real number is called the
probability of the event A.
PROBABILITY FUNCTION Let S ~ (zej, zv2, lo^,...., ze„) be the sample space associated to a random
experiment. Then a function P which assigns everi/ event A c S to a unique non-negative real number
P{A) is called the probabiliti/ function, if the following axioms hold:
Axiom 1: 0 < P {wj) < 1 for all eS
Axiom 2; P (S) =1 i.c., P (rcj) + P{w2) + .... + P K) = 1.
Axiom 3: For any event A cz S, P (A) = 5] P (wi^l the number P {w0 is called the probability of

w
elementary event wkeA

Consider the experiment 'tossing a coin'. The sample space associated to this random

F lo
experiment is S = If we assign the number i to each of the outcomes (elementary events)
1 1
H and T i.e. P (H) = - and P(T) =-. Then this assignment satisfies first

ee
two axioms i.e.

Fr
1
0 < P (H) <1, 0 < P (T) <1, and P (H) + P (T) +-
- = 1. So, P is the probability function on S
for
and we can say that the probability of getting head is - and the probability of getting tail is also
ur
1 1 3 1 3
-. If we assign the number - to H and - to T i.e. P (H) =
— and P (T) = —. This assignment also
s
2 4 4
ook
Yo

defines a probability function on S = {H, T}. However, if we take P (H) = - and P (P) =
eB

8
Then this assignment is not a probability function on S = {H, T}.
ILI.USTR.ATION let S =\Wi,W2W2,W^,W^,W(^] be sample space. Which of the follozuing
a
r

assignments of probability to each outcome are valid?


ou
ad

Outcomes or

Elementary events:
Y

7^2 t('3 W’4 ^5


1 1 1 1 1 1
Probabilities: (i)
Re
nd

6 6 6 6 6 6
(ii) 1 0 0
Fi

0 0 0
1 2 1 1 1 1
(iii)
8 3 3 3 4 4
1 1 1 1 1 3
(iv)
12 12 6 6 6 2
(V) 0.1 0.2 0.3 0.4 0.5 0.6

SOLUTION (i) We have, P (zvj) = - , / = 1,2, 3, 4,5, 6.


6

0 < P (ze,-) < 1 for alhe,-6 S


and. P (zo^) + P (Z02) + ...+P (Wf,) = i6 + l6 + i6 + l+
6
i6 + i=
6
i
Hence, the assignment of the probabilities is valid,
(ii) We have.
P{zv^) = 1,P{Z02) = P{W2) = P(rc4) = P (zcg) = P (zv^) = o
20.19
PROBABILITY

0 < P (u?j) < 1 for all u’i e S


Also, P + P (1V2) + P (zvs) + P (u>4) + P (W5) + P (w^) = 1

Hence, the assignment of the probabilities is valid.


(ill) We have.
1 1
P(u>5) = - -4 < 0 and P (te^) = -—<0.
d

So, axiom 1 is not satisfied. Hence, the assignment of the probabilities is not valid.
3
(iv) We have, P (zi>^) =

So, axiom 1 is not satisfied. Hence, the assignment of the probabilities is not valid,

ow
(v) We observe that axiom 1 is satisfied. But,
P (wi) + P(zu2) + P (H’3) + P (^t’4) + P (w’s) + ^ + 05 + 0.6 = 2.1 1.
So, axiom 2 is not satisfied. Hence, the assignment is not valid.
Let S = {tei, if 2 /.. ■ - zv„} be the sample space associated to a random experiment such that all

e
the outcomes (elementary events) rf^, rf2,tf3, -,zo„ are equally likely to occur i.e. the chance of

re
occurrence of each elementary event is same. i.e. P (if,) = P for all if, e S where 0 <p<l.

rFl
F
Using axiom 2, we have
(I 1
2 P (zf,0 = 1 => p + p +... + f = 1 => ?ip = 1 => p =

r
n
ou
/ = 1 (ii-times)

If A is an event such that m elementary events are favourable to A. Then,


fo
ks
P(2l) = Z PUOk)
oo

1 1 1 1
Y

P{A) = -n + Pn + -n + ...+
B

n
iH-times
re

m Favourable number of elementary events


P(A) = -
Total number of elementary events
ou

n
Y
ad

Thus, we have the following definition of probability of an event when all the elementary events
are equally likely to occur.
d

PROBABILITY OF AN EVENT If there arc n elementar}/ events associated ivith a random experiment and
in
Re

m of them are favourable to an event A, then the probability of happening or occurrence of A is denoted by
F

P (A) and is defined as the ratio —.


n

m
Thus, P (A) =
n

Clearly, 0 < fi < n. Therefore,


0 <-<l => 0<P(A)<1
n

f/P (A) =1, then A is called certain event and A is called an impossible event, ifP (A) =0.
The number of elementary events which will ensure the non-occurrence of A i.e. which ensure
the occurrence of A is (« -m). Therefore,
_ — n-m
P(A) =
n

m
P(A) = 1--
n
20.20
APPLIED MATHEMATICS-XI

=> P(A) = I ~P (A)


=> P (A) + P (A) == 1
The odds in favour of occurrence of the event A are defined by m: {n - m) i.e. P {A):P (A) and
the odds against the occurrence of A are defined by n - ni: m i.e. P (Z): P (A).
ILLUSTRATIVE EXAMPLES

Tifpe / PROBLEMS BASED ON CONSTRUCTION OF SAMPLE SPACE


EXAMPLE 1
Find the probability of getting a head in a toss of an unbiased coin.
SOLUTION The sample space associated with the random experiment is S = {H, T}.
Total number of elementary events = 2.
We observe that there are two elementary events viz. H, T associated to the given random
experiment. Out of these two elementary events only one is favourable i.e. H.
Favourable number of elementary events = 1

w
Hence, required probability .

EXAMl’LE 2
(ii) exactly one head
(iv) exactly one tail (v) no tails.
F lo
In a simultaneous toss of two coins, find the probability of getting:
(i) 2 heads (iii) exactly 2 tails

SOLUTION The sample space associated to the given random experiment is given by

e
Fre
S = {HH,HT,TH,TT}
Clearly, there are 4 elements in S.
for
Total number of elementary events = 4.
(i) There is only one elementary event i.e. HH favourable to the given event
r
So, required probability = ^ .
You
oks
eBo

(ii) We observe that exactly one head can be obtained in two ways: HT or. TH.
So, favourable number of elementary events = 2.
2 1
ad
our

Hence, required probability = - = — .

(iii) Exactly 2 tails can be obtained in one way i.e. TT. So, favourable number of elementary
events =1.
Re
dY

Hence, required probability = -.


4
Fin

(iv) Exactly one tail can be obtained in one of the following two ways: HT, TH
Favourable number of elementar)^ events = 2.
2 1
Hence, required probability = - = — .

(v) Tliere is only one elementary event viz. HH favourable to the event "getting no tails".
So, required probability = i .
EXAMPI.E ?● Three coins are tossed once. Find the probability of <?ettins:
(i) all heads (ii) at least two heads (iii) at most two heads
(iv) no heads (v) exactly one tail (^d) exactly 2 tails
(vii) a head on first coin.
SOLUTION Let S be the sample space associated with the random experiment of tossing three
coins. Then, S = \HHH, HHT, HTH,THH, HTT,THT,TTH,TTT\.
20.21
PROBABILITY

Clearly, there are 8 elements in S.


Total number of elementary events = 8.
(i) There is only one elementary event, namely HHH, favourable to the given event.
Required probability = - .

(ii) At least two heads can be obtained if we obtain one of the following elementary events
as an

outcome: HHH, HHT, HTH,THH


Favourable number of elementary events = 4.
1
Hence, required probability = - o 2

(iii) At most two heads can be obtained in any one of the following ways:

ow
HHT, THH, HTH, HTT, THT, TTH, TTT
7
Favourable number of elementary events = -. o

(iv) "Getting no heads" means "Getting all tails". So, there is only one elementary event viz.

e
TTT favourable to the given event.

re
Hence, required probability = — .

rFl
F
o

(V) Elementary events favourable to "Getting exactly one tail" are: HHT, THH, HTH.
Favourable number of elementary events = 3

r
ou
Hence, required probability = - .
3
fo
ks
o

(vi) Elementary events favourable to "Exactly 2 tails " are; HTT, THT, TTH.
oo

Favourable number of elementary' events = 3


3
Y

Hence, required probability = — .


eB

(vii) A head on first coin can be obtained in one of the following ways; HTT, HHH, HTH, HHT.
r

Favourable number of elementary events = 4.


ou

4 1
ad
Y

Hence, required probability = --8 2

Tzvo ilice are thrown siimiltiweoiish/. Find the probabiliti/ of getting:


d

EXAMPLE 4

(i) an even number as the sum (ii) the Sinn ns n prime number
Re
in

(iii) n total of at least 10 (iv) a doublet of even number


F

(v) a multifile of 2 on one dice and a multiple of 3 on the othei dice


(vi) same number on both dice (vii) a multiple of 3 as the sum
SOLUTION When bvo dice are thrown together the sample space S associated with the random
experiment is given by (3,6),
S={(1,1),(1,2) (1,6), (2,1), (2, 2) .,(2, 6), (3,1), (3, 2)
(4,1), (4, 2) (4, 6), (5,1), (5, 2) .,(5, 6), (6,1), (6, 2),..., (6,6)1
36.
Clearly, total number of elementary events =
(0 Let A be the event "getting an even number as the sum" i.e., 2,4,6,8,10,12 as the sum. Then,
A = 1(1,1), (1, 3), (3,1), (2, 2), (1, 5), (5,1), (3, 3), (2, 4), (4, 2), (3, 5), (5, 3), (4, 4),
(6, 2), (2, 6), (5, 5), (6, 4), (4, 6), (6, 6)}
18
Favourable number of elementary events =
18 1
So, required probability = — 2
20.22
APPLIED MATHEMATfCS-XI

(ii) Let be the event "getting the sum as a prime number., i.e., 2,3, 5, 7,11 as the sum. Then
^ = {(1,1), (1, 2), (2,1), (1, 4), (4,1), (2, 3), (3, 2), (1, 6), (6,1), (2, 5), (5, 2), (3, 4), {4, 3), ’
(6, 5), (5, 6)}
Favourable number of elementary events = 15
So, required probability = A
36 ”12'
(iii) Let A be the event "getting a total of at least 10' i.e., 10,11,12 as the sum. Then,
A
- 1(6, 4), (4, 6), (5, 5), (6, 5), (5, 6), (6, 6))
Favourable number of elementary events = 6
So, required probability = — = i.6

ow
36

(iv) Let 4 be the event "getting a doublet of even number". Then, ^ = |(2, 2), (4, 4), (6, 6))
Favourable number of elementaiy events = 3
1
So, required probability = —

e
re
rFl
(v) Let 4 be the event "getting a multiple of 2 on one die and a multiple of 3 on the other". Then,

F
A = ({2, 3), (2, 6), (4, 3), (4, 6), (6, 3), (6, 6), (3, 2), (3, 4), (3, 6), (6, 2), (6, 4))
Favourable number of elementary events = 11

r
So, required probability = ~.
ou
36 fo
ks
(vi) Let A be the event "getting the same number on both the dice". Then,
/I =
((1,1), (2, 2), (3,3), (4,4), (5, 5), (6, 6)1
oo

Favourable number of elementary events = 6


Y
eB

So, required probability = —


36
= 1.6
r

(vii) Let A be the event " getting a multiple of 3 as the sum" 3, 6, 9,12 as the sum. Then,
ou
ad
Y

A = {(1, 2), (2,1), (1, 5), (5,1), (2, 4), (4, 2), (3, 3), (3, 6), (6, 3), (5,4), (4, 5), (6, 6)}
Favourable number of elementary events = 12
d

1 9 1
So, required probability = —
Re
in

36 3

LXAMPLE5 A fair coin with 1 marked on one face and 6 on the other and a fair die are both tossed, find
F

the probabdihj that the sum of numbers that turn up is (i) 3 (ii) 12.
SOLUTION The sample space S associated to the given random experiment is given by
S = {(1,1), (1, 2), (1, 3),(1, 4),(1,5),(1, 6), (6,1), (6, 2), (6, 3),(6, 4), (6,5), (6, 6)1
Total number of elementary events =12.
(i) Let/I be the event that the sum of the number is 3. Then, A = {(1,2)}
Favourable number of elementary events = 1
1
Hence, required probability = P (A) =
12

(ii) Let B denote the event that the sum of the numbers is 12. Then, B = {(6,6)
Favourable number of elementary events = 1
1
Hence, required probability = 12
20.23
PROBABILITY

LXAMIM.H h find the probability that a leap year, selected at random, will contain 53 Sundays.
SOLUTION In a leap year there are 366 days.
366 days = 52 weeks and 2 days.
Thus, a leap year has always 52 Sundays. The remaining 2 days can be:
(i) Sunday and Monday, (ii) Monday and Tuesday, (iii) Tuesday and Wednesday,
(iv) Wednesday and Thursday, (v) Thursday and Friday, (vi) Friday and Saturday,
(vii) Saturday and Sunday.
If S is the sample space associated with this experiment, then S consists of the above seven
points.
Total number of elementary events = 7.
Let A be the event that a leap year has 53 Sundays. In order that a leap year, selected at random,
should contain 53 Sundays, one of the 'over' days must be a Sunday. This can be in any one of
the following two ways;
(ii) Saturday and Sunday

w
(i) Sunday and Monday
Favourable number of e!ementar>' events = 2.

F lo
2
Hence, required probability = —.
7

rXAMi’Ln 7 Three dice are thrown together. Find the probability of getting a total of at least 6.

e
Fre
SOLUTION Since
. one die can be thrown in six ways to obtain any one of the six numbers
marked on its six faces. Therefore, if three dice are thrown, the total number of elementary
for
events = 6x6x6 = 216.

Let A be the event of getting a total of at least 6. Then, /I denotes the event of getHng a total of
r
less than 6 i.e., 3, 4, 5.
You

(1, 3,1), (3,1,1), (1, 2, 2), (2,1, 2), (2, 2,1)}


oks

= 1(1,1,1), (1,1, 2), (1, 2,1), (2,1,1), (1,1, 3),


So, favourable number of elementary events = 10
eBo

10
F{A) =
216
10 10 _ 103
ad
our

=> l-P(A) = => P {A)


216 216 ~ 108
tXA.\H’LL8 A coin is tossed. If head comes up, a die is thrown but if tail comes up, the coin is tossed
again. Find the probability of obtaining:
dY
Re

(i) two tails (ii) head and number 6 (iii) head and an even number.
Fin

SOLUTION The sample space S associated with the given random experiment is
S = 1(H,1), (H, 2),(H, 3), (H, 4), (H,5),(H, 6), (T,H),(T,T)}
Clearly, it has 8 elements.
Total number of elementary events = 8
(i) If the outcome is (T, T), then we say that two tails are obtained.
Favourable number of elementary events = 1
1
Hence, required probability = — o

(ii) Head and the number 6 is obtain in only one way i.e. when the outcome is (H, 6)
Favourable number elementary events = 1

Hence, required probability = ^


20.24
APPLIED MATHEMATICS-Xf

(iii) Head and an even number can be obtained in any one of the following ways:
{H, 2), (H, 4),{H, 6).
Favourable number of elementary events = 3
Hence, required probability = —.
8

EXAMPLE 9 One urn contains two black balls (labelled B1 and B2) and one white ball. /I second urn
contains one black ball and iivo white balls (labelled W1 and W2). Supipose the following expieriment is
performed. One of the hoo urns is chosen at random. Next a ball is randomly chosen from the urn. Then a
second ball is chosen at random from the same urn without replacing the first ball.
(i) Write the sample space showing all pwssible outcomes,

ow
(ii) What is the pnvbability that two black balls are chosen?
(iii) What is the pirobability that two balls ofopypwsite colour are chosen?
SOLUTION (i) Let the contents of first urn be W, SI, S2, and that of second urn be B, Wl, W2.

e
When two balls arc drawn in succession from first urn, we
may get any one of the following

re
outcomes as an outcome:

WSl, WB2, SI, S2, SIW, B21V, S2B1

Frl
F
Similarly, when we draw two balls in succession from the second um, we may obtain any one of
the following as an outcome:
ou
or
6W1, BW2, WIB, Wl W2, W2W1, W2B kfs
Thus, the sample space Sis
S = iWBl, WB2, B1B2, 61W, B2W, B261, BWl, BW2, W1BW1W2, W2W1, W2B)
oo

(ii) We obtain two black balls if the outcome is one of the following outcomes: 61S2, B2B1.
Y

Favourable number of elementary events = 2


B

2 1
Hence, required probability = — = —.
re

(iii) Two balls of opposite colour can be drawn in any one of the following outcomes:
oYu
ad

WBl, WB2, SI W, B2W, BWl, BW2, WIB, W2B


Favourable number of elementary events = 8
d

8 2
Hence, required probability ^ .
in
Re
F

.Non-: Consider an experiment if drawing 2 cardsfrom a pack of 52 cards. The sample space associated
52
with this experiment consists of C2 = U26 pwints and therefore it is not easy to list all the elements of
the sample space. So, in future we will not be xvriting the sampde space associated with the given random
experiment.
EXAMPLE 10 On her vacations Veena visits four cities A, B, C and D in a random order. What is the
probability that she visits:
(i) A before B? (ii) A before B and B berfore C?
(iii) A first and B last? (iv) A either first or second?
(v) A just before B?
SOLUTION Veena can visit four cities A, B,C and D in any one of the following orders:
ABCD, ABDC, ACDB, ACBD, ADBC, ADCB, BACD, BADC, BCAD, BCDA, BDAC, BDCA.
CABD, CADB, CBDA. CBAD, CDAB. CDBA, DABC, DACB, DBCA, DBAC, DCAB, DCBA
20.25
PROBABILITY

Total number of arrangements (orders) in which Veena can visit four cities A, B, C
and D is 4! = 24.

(i) Out of these 24 ordered arrangements Veena can visit city /I before city B in the following
arrangements;
ABCD, ABDC, ACDB, ACBD, ADBC, ADCE, CABD, CADB, CDAB, DABC, DACB, DCAB
So, there are 12 ways in which Veena can visit city A before city B.
12 1

Required probability “ ”2
(ii) Veena can visit /A before B and B before C in any one of the following four ways:
ABCD,ABDC,DABC,ADBC
1

Required probability = ^ ^
(iii) Veena can visit city A first and city B last in any one of the following two ways:

w
ACDB, ADCB

F lo
2 1

Required probability -^2


(iv) Veena can visit city A first in 3! ways or city A second in 3! ways.

e
Fre
Number of ways in which Veena can visity city A either first or second = 3! + 3! = 12
, , .,. 12 1
for
Required probability = “2
(v) Taking AB together A,B,C,D can be arranged in 3! ways.
r
3! 1
You
oks

So, required probability = — ^


eBo

EXAMPLE 11 A die has Izvo faces each zvith mimber '1', three faces each loith number '2' and one face
with number ‘3'. If die is rolled once determine:
(i) P (2) (ii) P (1 or 3) (iii) P (not3)
ad
our

SOLUTION Total number of elementary events = 6.


(i) Out of six faces, 3 faces are marked with number 2.
3 1
Re
dY

P(2)=-=-
6 2

(ii) Two faces are marked with number 1 and one face with number 3. Therefore, a face marked
Fin

with 1 or 3 can be chosen in 3 ways.

P(lor3)=| ^ O 2
1
(iii) There is only one face marked with number 3. Therefore, P (3) = -.
1 5
Hence, P (not 3) = 1 - P (3) =1 - -6 =-6

Tl//)f // PROBLEMS BASED UPON COMBINATIONS OR SELECTIONS


EXAMPLE 12 An urn contains 9 red, 7 white and 4 Mack balls. Iftzuo balls are drazon at random, find the
probability that:
(i) both the balls are red, (ii) one ball is zuhite
(iii) the balls are of the same colour (iv) one is white and other red.
20.26
APPLIED MATHEMATiCS-XI

SOLUTION There are 20 balls in the bag out of which 2 balls can be drawn in ^^€2 ways.
So, total number of elementary events = ^^€2 = 190.
(i) There are 9 red balls out of which 2 balls can be drawn in 2 ways.
Favourable number of elementary events = ^C2 = 36.
So, required probability = ^ 18
190 95

(ii) There are 7 white balls out of which one white can be drawn in ^C-\ ways. One ball from the
remaining 13 balls can be drawn in ways. Therefore, one white and one other colour ball
can be drawn in x ways.
So, favourable number of elementary events = x
13
Ci.
1

Hence, required probability =


^Ci X ’^C,1 91
20
190
C2

w
(iii) Two balls drawn are of the same colour means that either both are red or both are white or
both are black. Out of 9 red balls two red balls can be drawn in ^C2 ways. Similarly, two white

F lo
balls can be drawn from 7 white balls in ^C2 ways and two black balls from 4 black balls in ^€2
ways. Therefore,
The number of ways of drawing 2 balls of the same colour = '^€2 + ^€2 + ^€2 = 36 + 21 + 6 = 63

e
Favourable number of elementary events = 63.

Fre
63
So, required probability = .
^ 190
for
(iv) Out of 7 white balls one white ball can be drawn in ways and out of 9 red balls one red
r
ball can be drawn in ways. Therefore,
You
s

One white and one red ball can be drawn in x ways.


ook

So, favourable number of elementary events = x = 63.


eB

So, required probability = 63/190 .


EXAMPLE 13 A box contains 10 red marbles, 20 blue marbles and 30 green marbles. Five marbles are
our
ad

drawn from the box, what is the probabiliUj that (i) all will be blue? (ii) at least one will be green ?

SOLUTION Out of 60 marbles, 5 marbles can be drawn in ^^05 ways.


dY
Re

Total number of elementary events =^^€5


Fin

(i) Out of 20 blue marbles, 5 blue marbles can be chosen in


5 ways.

20
Favourable number of ways = C5
20

Hence, required probability = 60 C5


C5
(ii) Clearly,
Required probability = 1 - Probability that no ball is green
30
= 1 - Probability that 5 balls drawn are red or blue. = 1 - C5
60
C5
I:XAMPLE14 In a lottery 10,000 tickets are sold and ten equal prizes are awarded. What is the
probabdity of not getting a prize if you buy (i) 1 ticket (ii) tzvo tickets (iii) 10 tickets.
20.27
PROBABILITY

10000
SOLUTION (i) Out of 10,000 tickets, one ticket can be chosen in C-^ =10000 ways.
There are 9990 tickets not containing a prize. Out of these 9990 tickets one can be chosen in
9990
C| ways.
9990 _ 999
Probability of not getting a prize = 10000 ~1000
10000
(ii) Out of 10,000 tickets, two tickets can be chosen in C2 ways. As there are 9990 tickets
not contain any prize, if they are chosen
without any prize. Therefore, two drawn tickets will9990
from the remaining 9990 tickets. This can be done in C2 ways.
9990

So, required probability = 10000


C2

w
10000
(iii) 10 tickets can be drawn out of 10,000 tickets in C|Q ways. There are 9990 tickets
9990
C 10 ways. So, 10 drawn
without any prize. Out of these tickets 10 tickets can be chosen in

Flo
tickets will not contain any prize, if they are chosen from the remaining 9990 tickets.

e
9990
c 10

re
Hence, required probability = 10000
c 10

F
tXAMTLE 15 Four cards are drnzvii nl random from a pack of 52 playing cards. Find the probability of
ur
r
getting fo
(i) all the four cards of the same suit (ii) all the four cards of the same number
(iii) one card from each suit (iv) two red cards and two black cards
ks
(vi) all face cards.
Yo
(v) all cards of the same colour
oo

SOLUTION Four cards can be drawn from a pack of 52 cards in ^^04 ways.
B

So, total number of elementary events =


diamond, each of 13 cards. All the four cards
re

(i) There are four suits viz. club, spade, heart and
are of the same suit means that either four cards drawn are club cards or spade cards or heart
cards or diamond cards. So, the total number of ways of getting all the four cards of the same suit
u
ad

isl3Q + 13c^ + %4 + %4 = 4(13C4)


Yo

4(^^C4) _ 198
So, required probability = 52 20825
d

C4
Re
in

(ii) Four cards drawn can be of the same number in any one of the following ways:
F

(1,1,1,1), (2, 2, 2, 2), (3, 3, 3, 3),..., (13,13,13,13)


Favourable number of elementary events = 13.
13 13
So, required probability = 52 270725
C4
(iii) There are four suits each of 13 cards. One card from each suit means that there is one
diamond card, one club card, one spade card and one heart card. There are 13 diamond cards,
out of which one can be selected in ^ Cj ways. Similarly, one club, one spade and one heart, each
can be selected in ways. 13
Cix ‘3C|X
13
The number of ways of selecting 4 cards, one from each suit = Cl

2197
So, required probability = 52 20825
C4
28
MATHEMATICS-Xl

(iv) There are 26 red cards and 26 black cards. Out of 26 red cards, 2 cards can be drawn in
ways. Similarly, 2 black cards can be drawn in ways. Therefore, 2 red and 2 black cards can
be drawn in ^^C2 x ^^C2 ways.
26 26
C2X ^2
So, required probability = 52
C4
(v) There are two coloursviz. red and black. Out of 26 red colour cards, 4 cards can be drawn in
"®C4 ways. 4 black cards can be drawn in ^^€4 ways. Therefore, 4 red or 4 black cards can be
drawn in ^^04 + ^^04 = 2 (^^04) ways. ,

ow
2(“CJ
So, required probability = 52
C4
(vi) There are 12 face cards (4 kings, 4 queens and 4 jacks). Out of these 12 face cards, 4 cards can
be selected in ^^04 ways. i

e
Favourable number of elementary events = ^^04

re
Frl
12

F
So, required probability = 52
C4
EXAMPLE 16 In a lottery of 50 tickets mwibered 1 to 50, tu’o tickets are drawn simultaneously. Find the
ou
or
probability that: kfs
(i) both the tickets drawn have prime numbers, (ii) none of the tickets drawn has prime number,
(iii) one ticket has prime number.
oo

SOLUTION Out of 50 tickets 2 tickets can be drawn in ^^^02 ways.


Y

So, total number of elementary events = ''’‘^02 = 1225


B

(i) There are 15 prime numbers between 1 and 50 viz. 2,3,5,7,11,13,17,19,23,29,31,37,41,43,


re

47. Out of these 15 prime numbers 2 numbers can be selected in ^^C2 ways.
Favourable number of elementary events = ^^€2 = 105
oYu
ad

So, required probability = 105 ^ 21


1225 ” 245
d

(ii) Number of non-primes from 1 to 50 = 50 -15 = 35. Out of these 35 numbers 2 can be selected
in

in ^^C2 ways.
Re

Favourable number of elementary events = ^^€2 = 595


F

595 17
So, required probability =
1225 35

(iii) Out of 15 primes from 1 to 50, one prime number can be selected in ways. Therefore,
15 -5C

one prime and one non-prime can be selected in Cl X ways.


15 35
Favourable number of elementary events = Cl1 X C^ =525

So, required probability = 525 ^ 3


1225 " 7
EXAMPLE 17 Four pcrsons are to be chosen at random from a group of 3 men, 2 women and 4 children.
Find the probability of selecting:
(i) 1 man, 1 woman and 2 children (ii) exactly 2 children (iii) 2 women
SOLUTION There are 9 persons viz. 3 men, 2 women and 4 children. Out of these 9 persons 4
persons can be selected in '^€4 = 126 ways.
Total number of elementary events = 126
20.29
PROBABILITY

(i) 1 man, 1 woman and 2 children can be selected in x x - 36 w%ys.


Favourable number of elementary events = 36
36 2
So, required probability = 126 7

; 2 children out of 4 children and 2 persons from 5 persons


(ii) Exactly 2 children means ^ ^
consisting of 3 men and 2 women. This can be done in ^€2 x C2 ways.
Favourable number of elementary events = '^€2 x ^€2 - 60
i 60 10
So, required probability = 126 21

(iii) Wc have to select 4 persons of which 2 are women and the remaiiring 2 are chosen from 7
persons consisting of 3 men and 4 children. This can be done in ^€2 x Cj ways.

w
Favourable number of elementary events = C2 x C2 = 21
21 _1

F lo
So, required probability = 126 “ 6
rXAMI'LEiS ^ box contnins 10 bulbs, 0/which just three are defective. If a random sample offive bulbs
is drazvn, find the probabilities that the sample contains:

ee
(i) exactly one defective bulb, (ii) exactly two defective bulbs, (iii) no defective bulbs.

Fr
SOLUTION Out of 10 bulbs 5 can be chosen in ways.
So, total number of elementary events = for
(i) There are 3 defective and 7 non-defective bulbs. The number of ways of selecting one
ur
defective bulb out of 3 and 4 non-defective out of 7 is x C4.
7
s

Favourable number of elementary events = 'C;i x C4


ook
Yo

^C|X^C4 _ 5
So, required probability =
eB

10 12
C5
(ii) The number of ways of selecting 2 defective bulbs out of 3 defecHve bulbs and 3
r

defective bulbs out of 7 non defective bulbs is C2 x C3 .


ad
ou

non-
3 7
Favourable number of elementary events = C2 x C3
Y

-"C; X ^C3 5
So, required probability =
Re

10 12
C5
nd

(iii) No defective bulbs means all non-defective bulbs. The number of ways of selecting all 5
Fi

non-defective bulbs out of 7 is ^€5. ^


Favourable number of elementary events = ^€5
1
So, required probability = 10 12
C5
EXAMPLE 19 Five marbles are drawn from a % which contains 7 blue marbles and 4 black marbles.
V\!hat is the probabilih/that: (i) all will be blue? (ii) 3 will be blue and 2 black?
SOLUTION There arc 7+4-11 marbles in the bag out of which 5 marbles can be drawn m
11
C5 ways.
Total number of elementary events = ’^€5.
(i) There are 7 blue marbles out of which 5 blue marbles can be drawn in C5 ways.
7
Favourable number of elementary events = C5
20.30
APPLIED MATHEMATICS-Xl

Hence, required probability 7! 5!6!_ 1


11
Cs 2\5i''1T\~72
(ii) Three blue out of 7 blue balls and 2 black out of 4 black balls can be drawn in x
ways.

Favourable number of elementary events = ^Cg x


Hence, required probability = ^CgX^C2
7! 4! 5!x6! 5
11
3!4l'' 212!'' H! “ IT

ow
Cs
EXAMP
LE 20 Find the probabilih/ that when a hand of 7 cards is dealt from a weU-shujfled deck of 52
cards. It contains: (i) all 4 kings (ii) exacth/3 kings (Hi) at least 3 kings.
SOLUTION Out of 52 cards from a deck of 52 playing cards, 7 cards can be drawn in 7 ways.

e
52
Total number of elementary events = C7

re
(i) There are 4 kings. Therefore, 4 kings out of 4 kings and 3 other cards from the remaining 48

Flr
cards can be chosen in '^€4 x '^^Cg ways.

F
Favourable number of elementary events ^C4X 48
C3
'C4X
ou 48 r

sr
^3 1
Hence, required probability = 52

fo
7735
C7
(ii) Three kings out of 4 kings and 4 other cards out of remaining 48 cards can be chosen i k in
oo
"^Cg X ways.
Y

Favourable number of elementary events = '^Cg x ^^04


reB

4 48
C3 C4_ 9
Hence, required probability = 52
uY

"C7 1547

(iii) When 7 cards are drawn from


deck of 52 playing cards, then getting at least 3 kings
a

means: gettmg 3 kings and 4 other cards or getting 4 kings and 3 other cards. This can be done in
ad
do

■^Cg X + ^C4 X "^^Cg ways.


in

Favourable number of elementary events -*03 X X «C3


Re

48
F

Lg X L4 + C4 > C3 46
ice, required probability = 52
C7 7735

Type // PROBLEMS BASED UPON PERMUTATIONS OR ARRANGEMENTS


EXAMPLE 21 If the letters of the zvord ALGORITHM are arranged at random in a row what is the
'bility that the letters GOR must remain together as a unit?
JTION There are 9 letters in the word ALGORITHM. These 9 letters can be arranged in a
in 9! ways. °
Total number of elementary events = 9!
■insidering GOR as one letter there are 7 letters which can be arranged in a row in 7! ways.
Favourable number of elementary events = 7!
7' 1
:e, required probability = ^ = —
20.31
PROBABILITY

EXAMPLE 22 2 If the letters of the word 'ATTRACTION' are written down at random, find the
probability that (i) all the Ts occur together (ii) no twoTs occur together.
SOLUTION The total number of arrangements of tlie letters of the word ATTRACTION is
10!

3!2!

(i) Considering three T's as one letter there are 8 letters consisting of two identical A's. These 8
letters can be arranged in 2!
ways.

2!
318! 1
Hence, required probability = 10! 10! 15
3!2!

w
7!
(ii) Other than 3 T's there are 7 letters which can be arranged in ^ ways. There are 8 places, 6

Flo
between the 7 letters and one on extreme left and the other on extreme right. To separate three
T's, we arrange them in these 8 places. This can be done in ways. Therefore,

e
re
7!
Number of ways in which no two T's are together = 2!

rF
7!
'C
C3 7
ur
2!
Hence, required probability = 10! 15
fo
3! 2!
ks

A ifve digit number is formed by the digits 1, 2, 3, 4, 5 without repetition. Find the
Yo

EXAMPLE 23
oo

probability that the number is divisible by 4.


Total number of five digit numbers formed by the digits 1,2, 3,4,5 is 5!.
B

SOLUTION
re

Total number of elementary events = 5! = 120.


We know that a number is divisible by 4 if the number formed by last two digits is divisible by 4.
u

Therefore last two digits can be 12,24, 32, 52 that is, last two digits can be filled in 4 ways. But
ad
Yo

corresponding to each of these ways there are 3! = 6 ways of filling the remaining three places.
Therefore the total number of five digit numbers formed by the digits 1,2,3,4,5 and divisible by
4 is 4 X 6 = 24
d
Re

Favourable number of elementary events = 24


in

24 ^ 1
F

So, required probability = 120 ~ 5

i i/pe III PROBLEMS BASED UPON COMBINATIONS OR SELECTIONS


EXAMPLE 24 Out of 9 Outstanding Students in a college, there are 4 boys and 5 girls. A team offour
students is to be selected for a quiz programme. Find the probability that hoo are boys and txvo are girls.
SOLUTION Out of 9 students 4 students can be selected in ^04 ways.
So, total number of elementary events = ^04.
There are 4 boys and 5 girls out of which 2 boys and 2 girls can be selected in “^C2 x C2 ways.
So, favourable munber of elementary events = “*02 x ^C2
^C2 X ^C2 10
Hence, required probability = 21
O4
20.32
APPLIED MATHEMATICS-XI

EXAMPLE 25
In (j lot of 12 Microioave ovens, there are 3 defective units. A person has ordered 4 ofth::: ese

units and since each is identicall}/ packed, the selection zvill be random. What is the probabilih/ that (i) all 4
units are good, (ii) exactly 3 units are good (Hi) at least 2 units are good.
SOLUTION Out of 12 Microwave ovens, 4 can be chosen in 4 ways.
12
Total number of elementary events = C4
(i) There are 9 good units out of which 4 can be chosen in 4 ways.

Favourable number of elementary events = ^04


Hence, required probability = 12
C4 55

(ii) Exactly 3 good units can be chosen in ^C3 X ways.


28
Required probability =

w
12
55
C4

F lo
(iii) Required probability = 1 - P ( At most one unit is good) = 1 ‘'Ci X 3C3 = 1-—= —
12
C4 55 "55

ee
Type IV MISCELLANEOUS PROBLEMS

Fr
EXA.MPLE 26 A card is drazonfrom an ordinary pack of 52 cards and a gambler bets that, it is a spade or
an ace. What are the odds against his zvinning this bet? for
SOLUTION Let A be the event of getting a spade or an ace from a pack of 52 cards. Then,
r
Total number of elementary events = = 52
You

Since there are 13 spade cards including an ace of spade and three aces other than an ace of
s
ook

spade.
Favourable number of elementary events = = 16
eB

So,P{A) = 16 ^ ^
52 13 ■
our
ad

Hence, odds againsti4 are P(A):P(A) = ~ — = 9-4


13 13
EXAMPLE 27
The odds in favour of an event are 3 ,● 5. Find the probability of occurrence of this event.
dY
Re

SOLUTION It is given that the odds in favour of an event are 3:5. Therefore,
Favourable number of elementary events = 3.v
Fin

Unfavourable number of elementary events = 5.r.


So, total number of elementary events = 3x + 5.v = 8.v.
Hence, probability of the occurrence of the event = i£ = l
8.V 8
Type V MIXED PROBLEMS ON PROBABILITY
EXAMPLE 28 A letter is chosen at random from the loord 'ASSASSINATION'. Find the probability that
letter is (i) a vozvel iii) a consonant.
SOLUTION There are 13 letters in the word 'ASSASSINATION' out of which there are 6 vowels
viz.. A, A, I, A, I, O and 7 consonants.
Total number of ways of selecting a word from 13 letters is = 13.
Number of ways of selecting a vowel out of 6 vowels = 6
Number of ways of selecting a consonant out of 7 consonants = 7.
20.33
PROBABILITY

7
— and, P (Selecting a consonant) = 13
P (Selecting a vowel) = 13

EXAMPLE 29 If the letters of the word ASSASSINATION are nrnmged at random. Find the probability
that
(ii) Tzvo Ts and two N's come together.
(i) Four S's come consecutively in the xvord.
(iii) All A's are not coming together. (iv) No tzvo A's are coming together.
SOLUTION There are 13 letters in the word ASSASSINATION out of which there are 3A's 4S's
ll's 2N's, one O and one T. These 13 letters can be arranged in a row i ways.
in
3!4!2!2!1!1!

2/'s, 2 N's, one O; one T and one letter


(i) Considering 4S's as one letter there are 10 letters (3A's, 10!
formed by 4S's). These 10 letters can be arranged in ways.

w
3!2!2!1!1!1!
10!

3!2!2!1!1!1! 4!xl0!_ 2

Flo
P (4S's come consecutively) = 13! 13! 143
3!4!2!2!1!1!

e
re
4!
(ii) Two i's and two N's can be put together in ways. Considering these 4 letters as one.

F
2! 2!
10!
there are 10 letters which can be arranged in a row in ways.
ur
r
3!4!
fo 10! 4!
Number of arrangements in which two I's and two N s come together 3!4! 2!2!
ks
Yo
10!
oo

3!2!2!
B

10!
re

3!2!2! 2
Hence, P (Two I's and two N's come together) = 13! 143
u
ad

3!4!2!2!
Yo

(iii) Considering all A’s as one letter, there are 11 letters which can be arranged in a row in
11!
ways.
d

4!2!2!
Re
in

11!
4!2!2! 1
F

P (All A's come together) = 13! 26


3!4!2!2!
25
Hence, P (All A's are not coming together) = 1 = 26

(iv) Other than 3 A’s there are 10 letters (4S's, 2J's IN's,
one O and one T). These 10 letters can be
arranged in a row in 4!2!2! ways. In each arrangement of these 10 letters there are 11 places
11
which can be filled by 3A's in C3 ways.
10!
Number of arrangements in which no two A s come together 4!2!2!
x^^C3
10! 11!
X

4!2!2! 8!3!
20.34
APPLIED MATHEMATICS-X)

10! 11!
4!2!2! 8! 3! 15
Hence, P (No two A's are coming together) 13! 26
3! 4! 2! 2!

l:XAiVii’i}; 30 If 4-digit numbers greater than or equal to 5000 are randomlyformedfrom the digits 0,1,
3,5 and 7, what is the probabilty offorming number divisible by 5 whe n

(i) the digits may be repeated (ii) the repetition of digits is not allowed.
SOLUTION (i) Total number of 4-digit numbers formed from the digits 0, 1, 3, 5 and 7 and
greater than or equal to 5000 is2x5x5x5 = 250.

TTTT
2 5 5 5

w
A number is divisible by 5, if units digit is 0 to 5. Therefore, number of 4 digit numbers formed
from the digits 0,1,3,5 and 7, divisible by 5 and greater than or equal to 5000 is 2 x 5 x 5 x 2 = 100

F lo
Probability of forming a number divisible by 5 250 100 ^2
5
(ii) If repetition of digits is not allowed, then the total number of 4 digit numbers formed from

ee
the digits 0,1, 3,5 and 7 is 2 x 4 x 3 x 2 = 48.

Fr
Now,

Number of 4 digit numbers divisible by 5 having 0 at one's place - 2x 3x2x1=12


for
Number of 4 digit numbers divisible by 5 having 5 at one's place = lx3x2xl=6
ur
Number of 4 digit numbers with distinct digits and divisible by 5 = 12 + 6 = 18
s
18 3
Hence, probability of forming a number divisible by5 = 48
ok
Yo
o
eB

EXERCISE 20.3
Which of the following cannot be valid assignment of probability for elementary events or
outcomes of sample spaces = (wi, ?t»2, a'3, W4, iCg, 11^):
r
ad
ou

Elementary events: W2 ZC3 «>4 ‘^6 ti>7


(i) 0.1 0.01 0.05 0.03 0.01 0.2 0.6
Y

1 1 1 1 1 1 1
(ii)
Re

7 7 7 7 7
nd

7 7
(iii) 0.7 0.6 0.5 0.4 0.3 0.2 0.1
Fi

1 2 3 4 5 6 15
(iv)
14 14 14 14 14 14 14

2. A die is thrown. Find the probability of getting:


(i) a prime number (ii) 2 or 4 (iii) a multiple of 2 or 3.
. In a simultaneous throw of a
pair of dice, find the probability of getting:
(i) 8 as the sum (ii) a doublet
(iii) a doublet of prime numbers (iv) a doublet of odd numbers
(v) a sum greater than 9 (vi) an even number on first
(vii) an even number on one and a multiple of 3 on the other
(viii) neither 9 nor 11 as the sum of the numbers on the faces
(ix) a sum less than 6 (x) a sum less than 7
(xi) a smn more than 7 (xii) neither a doublet nor a total of 10
(xiii) odd number on the first and 6 on the second
20.35
PROBABILITY

(xiv) a number greater than 4 on each die


(xv) a total of 9 or 11 (xvi) a total greater than 8.
4. In a single throw of three dice, find the probability of getting a total of 17 or 18.
5. Three coins are tossed together. Find the probability of getting:
(i) exactly two heads (ii) at least two heads
(iii) at least one head and one tail.
6. What is the probability that an ordinary year has 53 Sundays?
7. What is the probability that a leap year has 53 Sundays and 53 Mondays?
8. A bag contains 8 red and 5 white balls. Three balls are drawn at random. Find the
probability that:
(i) All the three balls are white, (ii) All the three balls are red.

w
(iii) One ball is red and two balls are white.
9. In a single throw of three dice, find the probability of getting the same number on all the

Flo
three dice.
10. Two unbiased dice are thrown. Find the probability that the total of the numbers on the dice

e
is greater than 10.

re
11. A card is drawn at random from a pack of 52 cards. Find the probability that the card drawn

F
is:

(i) a black king (ii) either a black card or a king


ur
r
(iii) black and a king fo
(iv) a jack, queen or a king
(vi) spade or an ace
(v) neither a heart nor a king
ks
(vii) neither an ace nor a king (viii) a diamond card
Yo

(ix) not a diamond card (x) a black card


oo

(xi) not an ace (xii) not a black card.


B

12. In shuffling a pack of 52 playing cards, four are accidently dropped; find the chance that the
re

missing cards should be one from each suit.


13. From a deck of 52 cards, four cards are drawn simultaneously , find the chance that they will
u
ad

be the four honours of the same suit.


Yo

14. Tickets numbered from 1 to 20 are mixed up together and then a ticket is drawn at random.
What is the probability that the ticket has a number which is a multiple of 3 or 7?
d
Re

15. A bag contains 6 red, 4 white and 8 blue balls. If three balls are drawn at random, find the
in

probability that one is red, one is white and one is blue.


F

16. A bag contains 7 white, 5 black and 4 red balls. If two balls are drawn at random, find the
probability that: (i) both the balls are white (ii) one ball is black and the other
red (iii) both the balls are of the same colour.
17. A bag contains 6 red, 4 white and 8 blue balls. If three balls are drawn at random, find the
probability that: (i) one is red and two are white (ii) two are blue and one is red (iii) one
is red.
18. Five cards are drawn from a pack of 52 cards. What is the chance that these 5 will contain:
(i) just one ace (ii) at least one ace?
19. The face cards are removed from a full pack. Out of the remaining 40 cards, 4 are drawn at
random. What is the probability that they belong to different suits?
20. There are four men and six women on the city councils. If one council member is selected
for a committee at random, how likely is that it is a women?
20.36
APPLIED MATHEMATICS-XI

21. A box contains 100 bulbs, 20 of which are defective. 10 bulbs are selected for inspection.
Find the probability that: (i) all 10 are defective (ii) all 10 are good (iii) at least one i_
defective (iv) none is defective
22. Find the probability that in a random arrangement of the letters of the word 'SOCIAL'
vowels come together.
23. Tlie letters of the word 'CLIFTON' are placed at random in a row. What is the chance that
tv\'o vowels come together?
24. The letters of the word 'FORTUNATES' are arranged at random in a row. What is the
chance that the two 'T' come together.
25. A committee of two persons is selected from two men and two women. What is the
probability that the committee will have (i) no man ? (ii) one man? (iii) two men?
26. If odds in favour of an event be 2 : 3, find the probability of occurrence of this event.
27. If odds against an event be 7 : 9, find the probability of non-occurrence of this event.
28. Two balls are drawn at random from a bag containing 2 white, 3 red, 5 green and 4 black

w
balls, one by one without, replacement. Find the probability
different colours.
that both the balls are of

F lo
29. Two unbiased dice are thrown. Find the probability that:
(i) neither a doublet nor a total of 8 will appear
(ii) the sum of the numbers obtained on the hvo dice is neither a multiple of 2 nor a

ee
multiple of 3

Fr
3t). A bag contains 8 red, 3 white and 9 blue balls. If three balls are drawn at random, determine
for
the probability that (i) all the three balls are blue balls (ii) all the balls arc of different
colours.
r
31. A bag contains 5 red, 6 white and 7 black balls. Two balls are drawn at random.What is the
You

probability that both balls arc red or both are black?


s
ook

32. If a letter is chosen at random from the English alphabet, find the probability that the letter
is (i) a vowel (ii) a consonant
eB

33. In a lottery, a person chooses six different numbers at random from 1 to 20, and if these six
numbers match with six numbers already fixed by the lottery committee, he wins the prize.
our
ad

What is the probability of winning the prize in the game?


34. 20 cards are numbered from 1 to 20. One card is drawn at random. What is the probability
that the number on the cards is: (i) a multiple of 4? (ii) not a multiple of 4? (iii) odd?
(iv) greater than 12? (v) divisible by 5? (vi) not a multiple of 6?
dY
Re

35. Two dice are thrown. Find the odds in favour of getting the sum
Fin

(i) 4 (ii) 5 (iii) Wliat are the odds against getting the sum 6?
36. What are the odds in favour of getting a spade if a card is drawn from a well-shuffled deck
of cards? What are the odds in favour of getting a king?
37. A box contains 10 red marbles, 20 blue marbles and 30 green marbles. 5 marbles are drawn
at random. From the box, what is the probability that: (i) all are blue? (ii) at least one is
green?
38. A box contains 6 red marbles numbered 1 through 6 and 4 white marbles numbered from 12
through 15. Find the probability that a marble drawn is (i) white (ii) white and odd
numbered (iii) even numbered (iv) red or even numbered.
39. A class consists of 10 boys and 8 girls. Three students are selected at random. What is the
probability that the selected group has (i) all boys? (ii) all girls? (iii) 1 boy and 2
girls? (iv) at least one girl? (v) at most one girl?
40. Five cards are drawn from a well-shuffled pack of 52 cards. Find the probability that all the
five cards are hearts.
20.37
PROBABILITY

41. A bag contains tickets numbered from 1 to 20. Two tickets are drawn. Find the probability
that (i) both the tickets have prime numbers on them (ii) on one there is a prime number and
on the other there is a multiple of 4.
42. An urn contains 7 white, 5 black and 3 red balls. Two balls are drawn at random. Find the
ball is red and the other is black (iii) one
probability that (i) both the balls are red (ii) one
ball is white.
ANSWERS

(iii) I 3- (i) ^ (ii) \


1 1
1. (i), (ii) X (i) 36
1 1 1 11
1
(iii) (iv) r (V)- (Vi) - (vii) —
36
(viii) j6
12 12
7 1

(x) ^ (xi) ^

w
(Xii) - (xiii) TT (xiv) -
(ix) 12 12
18
1
(XV)-
1
(xvi) A 4. —
1
5- (i) I8 (“>2 (iii) T

Flo
54 4
o

28 1 1
6.
1
7.
1
8. (i) A (ii) (iii) 10.

ee
143 143 143 36 12
7 7
4

Fr
11. (i)
1
(ii) (iii) ^
26
(v):^
13
(vi) —
13
26 13

(xii) ^
12
(vii) 7T
11
(viii) -
1
(ix)44 (x)i for
(xi)r
ur
4 13
13
2197 4 2
13. 14. - 15.
12.
s
270725 5 17
20825
k
Yo

33
17. (i) ^68 (ii)-^
oo

7 1 37
(iii) (iii)
16. (i) — 120 34 68
eB

40
20
1000 3 C 10
3243 18472
(ii) 19. 20. - 21. (i)
18. (i) 9139 5
100
C 10
10829 54145
r
ou
ad

80 80 80 2
C 10 C 10 C
10 22. - 23. -
(ii) (iii)l - (iv)
Y

100 100 100 5 7


c 10 C 10 C 10

7 13
2

l25.(i)l (ii)| (iii)i


Re
nd

24. 26. 27. — 28. 0.78 29. (i) 7-


16 18
5
Fi

21
18 31
32. (i) 126
(ii)l30. (i)^ (ii) -
95
31.
153
(ii) —
26
1

(iii) \ (iv)|
1 1
33. 34. (i) - (V)-
38760 4

17 36. (i) 1 : 3, 1 : 12
(Vi) — 35. (i) 1 :11 (ii) 1 : 8 (iii) 31: 5
20
20 40 1 1
37. (i)
C5X
60
<=0 _ 34
(ii)
4367
4484
38. (i) 25 (ii) ^ (iii) 2
11977
Cs
35 ^ 29 , , 10
(iv)-
4
39. (i) ^
34
(ii) 102
(iii)
102
(iv) —
34
(v) —
17

8
42. (i) ^ (ii)^7
13
(ii):^19
33 14
40. C5 ^ 41- (i) — (iii) Ti:
35 Id
52 66640 95
C5
20.38
APPLIED MATHEMATiCS-XI

__ HINTS TO SELECTED PROBLEMS


13 13 13
CiI X X Ci1 X
12. Required probability = Cl
52
C4
13. Four honours means king,4 queen, jack and ace.

So required probability = C4 + %52 +


C4
14. Total number of elementary events = = 20.

ow
A multiple of 3 or 7 can be obtained as follows: 3, 6, 9,12,15,18, 7,14.
So, favourable number of elementary events ’C-
18
1.5.
Total number of elementary events = C3-
Favourable number of elementary events = x x

e
20. Out of 4 men and 6 women one person can be chosen in 10 C| =

re
10 ways.
The number of ways of selecting 1 women out of 6 women = 6.

Flr
= 1

F
3
Required probability = —
10 5
ou
2.5. A commi ttee of two persons can be formed from two men and two women in ^C2 - 6 ways,

sr
(i) Number of committees having no man = ^C2 = 1

fo
1
Probability that a committee has

(ii) Number of committees having one


no man
6 k
oo
man = x - 4

Probability that the committee will have one man - 4^2


Y

6 3
reB

1
Probability that the committee has two men
6
33. Total number of ways of selecting six numbers from numbers 20
uY

1 to 20 = C6
Total number of elementry events = = 38760
Favourable number of elernentary events = 1
ad
do

1
Required probability =
38760
in

20.8 ADDITION THEOREMS ON PROBABILITY


Re

Uptill now we have been computing the probability of occurrence or non-occurrence of an event
by using favourable and total number of elementary events. But it is not always convenient to
F

compute favourable number of elementary events to a given event. In such cases, we express the
given event as the union of two or more events and the probability of the given event is
expressed in terms of the probabilities of these events. Theorems which express the probability
of an event in terms of the probabilities of those events whose union is the given event are
known as addition theorems on probability. In this section, we shall discuss addition theorems
tor two or more events.

THEOREM 1 (Addition Theomn for two events) If A and B are two events associated with a random
experiment, then P(AvjB)= P(A) + P(B) - P (A n B).
I'KtXii
Let S be the sample space associated with the given random experiment. Suppose the
random experiment results in n mutually exclusive ways. Then, S contains n elementary events,
Let m-[, i»2and m be the number ofelementary events favourable to A, Band A n B respectively.
Then,

P(/l)=a, n n
and P(AnB)J”. n
20.39
PROBABILITY

The number of elementary events favourable to A only is


nil ~● Similarly, the number of elementary events
favourable to B only is ni2 - ni. Since m elementary events are
favourable to both A and B. Therefore, the number of
elementary events favourable to A or 6 or both i.e. /\ u B is m-j
- in + ni2 - 111 + 111 = '”1 + 1112 ~ 111-
mj + ni2 - 111 nil ni2 _ 111
So, P{A^B) =
n 11 n n

P{AuB) = P{A) + P{B)-P{A n B) Fig. 20.3

Q.E.D.

COROLLARY If A niid B are mutually exclusive events, then P{AnB)=Q.


P{AuB) = P{A) + P{B)

w
This is the addition theorem for mutually exclusive events.
THEOREM 2 (Addition Theorem for three events) If A, B, C are three rvents associated with a random
experiment, tlum

F lo
P{AuB uC)=PiA) + P{B) + P{C)^P(A n B) - P (B nC) - P {A r^C) + P (A n B nC)
LetD =BuC .Then,

ee
P(A<jBuC)=P{AwD)= P{A) + P (D) - P (A n D)

Fr
But, Ar^D = A = {AnB)Kj{A nC)
P(AnD)= P[(AnB)'^{An C}]
= P (A n B) + P {A nC) - P [{A n B) r\{A n C)] for
ur
= P(AnB) + P(/lnC)-P{AnfJnC) ...(11)

and. P(D) = P(BuC) - P{B)-\-P{C)-P{BnC) ...(iii)


s
ook
Yo

From (i), (ii) and (iii), we get


P(AuB^C) = P(^) + P(B) + P(C)-P(BnC)-[P(AnB) + P(AnC)-P(AnBnC)]
eB

=> P(AuBuC) = P(A) + P(B) + P(C)-P(AnB)-P(AnC)-P(BnC) + P(AnBnC)


Q.E.D.
r

COROLLARY If A, B, C are mutually exclusive ezrents, then


ou
ad

P(AnB) = P(BnC) = P(AnC) = P(AnBnC) = 0.


P(AuBuC)= P(A) + P(B) + P(C)
Y

This is the addition theorem for three mutually exclusive events.


Re
nd

THEOREM 3 Let A and B be two events associated to a random experiment. Then,


(i) P(A r^B) = P{B)-P{Ar.B)
Fi

(ii) P (A n B) = P (A) - P (A n B)
(iii) P{{AnB)^{A nB))^ P (A) + P {B)-2P (A B)
rKOOL (i) Since A n B and A n B are mutually exclusive events such that
(A ri B) u (A n B) - B
P(AnB) + P(A oB) = P(B)
P(A nB) = P(B)-P(A nB)
(ii) Since A n B and A n B are mutually exclusive
events such that

(A n B) vj(A n B) = A
P{AnB)-hP{AuB) - P(A)
P{Ar^B) - P{A) - P {A B).
Fig. 20.4
20.40
APPLIED MATHEMATICS-X!

(iii) Since A n B and A B are mutually exclusive events. Therefore,


P {(A n B) u (A n 6)1 = P (A n B) + 6 (A n B)
= PiA)~PiAnB) + P{B)-P{Ar^B) [ Using (i) and (ii)]
= P{A) + P{B)-2P{A nB)
^ * P {A nB) is known as the probability of occurrence of 6 only. Q.E.D.
ki;makk2
P (A n 6) is known as the probability of occurrence of A only,
l^i-NIARK 3 P |(A n 6) u (A n 6)| is known as the probability of occurrence of exactly one of
tvN'o events A and B.

klAIAKK 4
If A mill B are tu’o events associated to a random exp>eriinent such that A c B, then
A n B ^ 1^.

ow
P(A nB) >0=> P{B)-P(An6) >0 => P (6)-P (A) >0 =>P(A)<P(B).
THEOREM 4 For any fivo events A and B, prozH’ that
P(Ar\B)<P{A)<PiA^B)<P(A) + P (B).
I’KcK'iF Since A n B cz A. Therefore, we have

e
Fl
re
P(AnB) < P(A). ...(i)

F
Also, A c A u 6 => P (A) < P (A u 6) ...(ii)
Now, P(AuB) = P(A) + PfB) - P{AnB)
ur
P(AuB)<P{A) + P(B)
or ...(iii)
sf
From (i), (ii) and (iii), we get
k
P (A nB) < P {A) <P (A u B) <P {A) + P (B).
Yo
Q.E.D.
oo

THEOREM 5 For any two events A and B, prove that the probability that exactly one of A, B occurs is
given byP{A) + P{B)-2P{A yj B) = P (A u B) - P {A n B).
B

I’Koor We have.
re

P (Exactly one of A, B occurs) - P {(A nB) ^(A n B)]


u
ad

= P(AnB) + P{AnB)
Yo

= P (A) - P{AnB) + P (6) - P(AnB)


d

= P (A) + P(B) - 2P (An B)


Re
in

= {P(A) + PiB)~P(AnB)] - P(AnB)


F

= P (A u B) - P(AnB). Q.E.D.

ILLUSTRATIVE EXAMPLES

l ype / PROBLEMS BASED UPON FORMULAE


(i) P(Au6)=P(A) + P(B)-P(AnB)
(ii) P(AyjByjC)=P{A) + P(B) + P{C)-PiAnB)-P(BnC)-P(AnC) +P(AnBnC)
1
EXAMPLE! Given P (A) = — and P (B) = -.Find P (A or B), if A and Bare mutually exclusive events.
5

SOLUTION Since A and B are mutually exclusive events.


P (A or B) = P(A u B) =P (A) + P (B) =- + - = -
5 5 5
20.41
PROBABILITY

liXAMPLLlAnndBnre two tmihinlli/ exclusive events of mi experiment. If PCtwt A') = 0.65,


P{AuB)= 0.65 and P(B) = pjind the vnhie of p.
SOLUTION By addition theorem for mutually exclusive events, wc have
P (A u B) = P(A) + P{B)
P(AuB) = l-P('notA') + P(B) [v P{A)=1-P(A)]
0.65 = 1 -0.65 + p => p = 0.30
LXAMl’l.ns A mid B are tzoo non-miitunlly exclusive events. If P(A) - 1,
-4 P(B)=-5 mid

P (A u B) =1 ,find the values ofP (A n B) and P (A n B).


/ P{B) = ^ and P {A u B) ^ ^
1
SOLUTION We have, P(A) =
4

w
By addition theorem, v\'e have
P (A u B) = P(A) + P(6) -P{Ar>B)
1 1 2
- = l + ±-P{AnB)

F lo
2 4 5
1 2 1 3
P{AnB) = - + -~

ee
=>
' 4 5 2 20

Fr
1
P(AnB) = P(A)-P(AnB)= -4 20 10

EXAMPLE 4
for
IfE and F are two events such that P (E) = ^ , P (F) - — and P (£ and F) = —, find
ur
(i) P( Eor F) (ii) P (not £ and not F).
s
ook

SOLUTION Wc have,
Yo

P (E) - ^ , P (F) = i and P (£ n f) =


eB

1 1 5
(i) P (£ or F) = P {£ u F) = P (£) + P (F) - P (£ n F) -^ ^ -
r

P (not E and (lef F) = P(£nF)


ou
ad

(ii)
= P (£ u F)
Y

= 1 - P (£ u F) = 1 - IP (E) + P (F) - P (£ n F)1


Re
nd

l4 2 8
Fi

EXAMPLE 5 The probalhlih/ that at least one ofjhe eveids A and B occurs is 0.6. If A and B occur
siimiltaneoiisli/ with probabiliti/ 0.2, then ifnd P(A) + P (B).
SOLUTION We have,
P (At least one of the events A and B occurs) = 0.6 i.e. P (A vj B) = 0.6
and. P (A and B occur simultaneously) = 0.2 i.e. P (A nB) = 0.2
Now,
P(A u B) = P(A) + P(B) -PiAr^B)
0.6 = P(A)+_P(B)-0.2 _
0.6 = l-P(A) + l-P(B)-0.2
0.6 = 2 - 0.2-[P (A)+ P(B)]
0.6 = 1.8-[P(A) + P(6)1
P(A) + P(B) =1.8-0.6 =1.2
20.42
APPLIED MATHEMATICS-XI

EXAMI-LL (. Check whether the following probabilities P (A) ami P (B) are consistently defined:
(i) P {A) =05, P (B) =07, P (A n B) = 0.6 (ii) P (A) =05, P {B) =0.4, P (A u B) =0.8
SOLUTION (i) We have, P (v4) =05,P(B)= 0.7, P (A n B) = 0.6
We know that P(AnB)<P(A) and P (A B) < P (B). But, for the given probabilities
P (A n B) ^ P (A). So, given probabilities are not consistently defined,
(ii) We have.
P (A) =0.5, P (B) =0.4 and P(A u B) = 0.8
P (A n B) = P (A) + P (B) - P (A u B) = 05 + 0.4 - 0.8 = 0.1
Clearly, P(AnB)<P(A) and P (A n B) < P (B). Hence, the given probabilities are
consistently defined.
liXAMPl.E 7 Events E and F are such that P (not E or not F) = 0.25. State whether E and Fare niutualh/
exclusive.

w
SOLUTION _ We have, P (not £ or not P) = 0.25
i.e.P(£ uF) =0.25 => P (£ n F) = 0.25 => 1 - P (£ n F) = 0.25 P(£nF)=l -0.25 = 0.75^0

Flo
Hence, £ and F are not mutually exclusive.
I \ AMPI I. s A, B, C are three mutually exclusive and exhaustive ei’ents associated with a random

e
^P(A)and P{C) = ^P{B).

re
experiment. Find P(A), it being given that P{B) =

rF
SOLUTION Let P (A) = p. Then,
P{B) = ~4 P(A) => P(B) = I2 p and P(C) = -2 P(B) => P(C) = -4^
ur
fo4

Since A, B, C are mutually exclusive and exhaustive events associated with a random
ks
experiment.
Yo

A u 6 uC = S
oo

P(A uBuC) = P(S)


B

P(A uB wC) = 1 [V P(S)=1J


re

=>
P(A) + P(B) + P(C) = 1 [By addition Theorem]
3 3 , 4 4
u
ad
Yo

EX AMPLE Four candidates A, S, C,D have appliedfor the assignment to coach a school cricket team. If
A is twice as likely to be selected as B, and B and C are given about the same chance of being selected, while
nd
Re

C is twice as likely to be selected as D, what are the probabilities that (i) C will be selected? (ii) A will not be
selected?
Fi

SOLUTION Let A^, A2, A3 and A4 be the events that candidates A, B, C and D respectively are
selected as school cricket team coach. Then,
It is given that
P(A3) = 2 P(A4), P (A2) = P (A3) and P (Aj) = 2P (A2)
P (Ai) = 4P (A4), P (A2) = P (A3) = 2 P (A4)
Clearly, Aj, A2, A3 and A4 are mutually exclusive and exhaustive events. Therefore,
Aj cj A2 A3 A4 = S
=> P(AiuA2uA3uA4) = P(S)
=> P(A]) + P(A2) + P(A3) + F(A4)=1
4P(A4) + 2P(A4)-f2P(A4)+F(A4)=l =>9P(A4)=1 ^P(A4)=~.
20.43
PROBABILITY

2
(i) Required probability = P (4 3) = 2 P{A^) = -
— 4 5
(ii) Required probability = P (/^-|) =1 -P =1 -4 P (^4) =1 ^
LXAMl’U: 10 Probabiliti/ that a truck stopped at a roadblock wiU have fault}/ brakes or badly ivorn tires
an 0.23 and 0.24, respectivch/. Abo, the probability is 0.38 that a truck stopped at the roadblock will have
faulti/ brakes and or badly working tires. What is the probability that a truck stopped at this roadblock zvill
have faulty brakes as well as badly worn tires?
SOLUTION Let B be the event that a truck stopped at the roadblock will have faulty brakes and
T be the event that it will have badly worn tires.
Tt is given that P (B) = 0.23, P (T) - 0.24 and P (B uT) = 0.38. We have to find P{Br^T).
We know that

w
P (B uT) = P (B) + P (T) -P (6 nT) [By addition theorem]
P (BnT) = P (B) + P (T) -P (BuT) = 0.23 + 0.24-0.38 -0.09

F lo
LXAMl’LC 11 The probability of two events A and B are 0.25 and 0.50 respectively. The probability of
their simultaneous occurrence is 0.14. Find the probability that neither A nor B occurs.

ee
SOLUTION We have, P(A) = 0.25, P(B) = 0.50 and P (A n B) = 0.14

Fr
Required probability = P(A nB)
= P{AuB) [-.● (A u B) = A n B]
= 1 - P (A u B) = 1 - [P(A) + P(6) - P (A n 6)]
for
ur
= 1 -(0.25+ 050-0.14) = 0.39

type // PROBLEMS BASED UPON ADDITION THEOREMS OF PROBABILITY BUT CAN BE SOLVED
s
ook

INDEPENDENTLY BY USING THE DEFINITION ONLY


Yo

\Ol I Following problems will be solved by using addition theorems but these problems can be solved
eB

otherwise also. Students are advised to do these problems without using addition theorem.
liXAMl’LL 12 Find the probability of getting an ezK’n number on the first die or a total of 8 in a single
throw of tzoo dice.
r
ou
ad

SOLUTION Let S be the sample space associated with the experiment of throwing a pair of dice.
Then, n{S) = 36.
Y

Total number of elementary events = 36


Re
nd

Let A and B be two events given by


A = Getting an even number on first die, B = Getting a total of 8.
Fi

Then, A n B = Getting an even number on first die and a total of 8.


Clearly, A - |(2,1),..., (2, 6), (4,1), (4, 2),..., (4, 6), (6,1), (6, 2),..., (6, 6)|,
6 = {(2, 6), (6,2), (3,5), (5, 3), (4,4)1 and, A n B = |(2, 6), (6, 2), (4,4)|.
P(A) =
18

36'
P(B) = 36
— and P(AnB) = ^
36
18 5 3 20 5
Required probability = P (A u B) = P(A) + P(B)
I XAMIM A die is throzvn twice. What is the probability that at least one of the tzvo throzvs comes up
ivith the number 4?
SOLUTION Let S be the sample space associated with the random experiment of throwing a die
twice. Then, n(S) = 36.
Total number of elementary events = 36
Considerthe events: A = First throw shows 4, B = Second throw shows 4
A n B = First and Second throw show 4 i.e. getting 4 in each throw.
20.44 APPLIED MATHEMATICS-XI

Clearly, /I = ((4,1), (4,2), (4,3), (4,4), (4,5), (4,6)|, B = {(1, 4), (2,4), (3,4), (4,4), (5,4), (6, 4)}
and, An 6 = 1(4, 4))
1
P(A) = ~ , P(6) = — and P (A n B) =
36 36 36

Required probability = Probability that at least one of the two throws shows 4.
1 11
= P(AuB)= P(A) + P(B) -P(AnB)=— + —-
36 36 36 36

EXAMPLE 14 Oiw iwmbcr is chosen from numbers 1 to 200. Find the pvobubiliti/ that it IS

divisible by 4 or 6?
SOt.UTrON Let S be the sample space. Then, n{S) = 200.
Total number of elementary events = 200
Let A be the event that the number selected is divisible by 4 and B be the event that the
number selected is divisible by 6. Then,

w
A = |4, 8,12,..., 2001, B = {6,12,..., 1981 and A n 6 = (12, 24,..., 192|
200 198 192
Clearly, n (A) = = 50, n(6)= — = 33 and n (A n 6) = = 16

F lo
4 6 12
50 33 16
P(A) = P{B) = and P (A n 6) =
200' 200 200

ee
Required probability = P (a number is divisible by 4 or 6)

Fr
50 33 16 67
= P(A u6) = P(A) + P(6)-P(AnB) =
for 200 200 200 ” S
EXAMPLE15
A card is drawn from a deck of 52 cards. Find the probability ofgettin<^ a kin^ ora heart ora
i
ur
red card.

SOLUTION Consider the following events:


s
ook
Yo

A = Getting a king, B = getting a heart card, C = Getting a red card.


Clearly,
eB

13 26
c 13 C 26
P(A) = P{B) = P(C) = 52
.
52
c1 52' 52
C1 52' c 52
1
our
ad

1 13
P {A n B) = P (Getting a king of heart) = P {B nC) = P (Getting a heart card) =
52' 52

P{C A) =P (Getting a red king) = ^,P {A r\ B r\C) = P


1
Y

(Getting a king of heart) = 52


Re
nd

Required Probability = P {A u BkjC)


= P(A) + P(B) + P(C)-P(An6)-P{6nC)-P(CnA)+P(AnBnC)
Fi

4 , 13
- + + 26 1 13 __2 ^ _ 7
52 52 52 52 52 52 52 ~ ^ “ 13
EXAMPLE 16 A drawer contains 50 bolts and 150 nuts. Fialf of the bolts and half of the nuts are rusted. If
one item is chosen at random, what is the probability that it is rusted or a bolt?
SOLUTION Let A be the event that the item chosen is rusted and B be the event that the item
chosen is a bolt. Clearly, there are 200 items in all, out of which 100 are rusted.
100 50 25
P(A)=- , P(B) = and P (A n B) =
200 200 200

Required probability = P (A u B)
100 50 25 125 5
= P(A) + P(B)-P(AnB) = +
200 200 200 200 8
i;XAMPl.E 17
Four cards are drawn at a time from a pack of 52 playing cards. Find the probabiliti/ of
getting all the four cards of the same suit.
SOLUTION Since 4 cards can be drawn at a time from a pack of 52 cards in ways. Therefore,
20.45
PROBABILITY

Total number of elementary events =


Consider the following events:
A = Getting all spade cards; B = Getting all club cards;
C = Getting all diamond cards, and D = Getting all heart cards.
Clearly, A, B, C and D are mutually exclusive events such that 13
13 13
13
C4 C4 , P(C) = C4 and P (D) = C4
P(A) = 52 , P(B) = 52 52 52
C4 C4 C4 "C4

ow
Required probability = P (A u B u C u D)
= P(A)-^P(B) + P(C) + P(D) [By addition Theorem]
13
C4 44
= 4
52 4165

e
C4

re
LXAMFLE 18 All is chosen at random from the numbers ranging from 1 to 50. What is the
probabiUhj that the integer chosen is a multiple of 2 or 3 or 10?

Flr
F
50
SOLUTION Out of 50 integers an integer can be chosen in Cl ways.
= 50.
Total number of elementary events =
Consider the following events:
ou 1

sr
A = Getting a multiple of 2, B = Getting a multiple of 3 and, C = Getting a multiple of 10.
Clearly, A = [2, 4,501, B = (3, 6,48}, C = |10, 20,..., 50j
AnB = {6,12, ...,481, BnC = {30}, AnC = {10, 20,...,50} and, AnBnC = {30}
fo
k
oo
8 1
25 16
— ,?(/! nB) = ,P(6nC) =
Y

50' 50 50'
reB

P(/4 nC) = -- and PiAr^Br^C) = ^


50 50
uY

Required probability = P {A B r\C)


P{A)-r P{B) + P{C)-P{Ar\B)-P{Ar\C)-P{BnA)+P{AnBnC)
ad
do

1 5 1 33
^ IB ^5 +

50 ^ 50 ^ 50 50 50 50 50 50
in

EX.AMI-LE19 In an essay competition, the odds in favour of competitors P,Q, R,S arel: 2,1:3,1:4,
Re

ai
idl :5 respectiveh/. Find the probability that one of them wins the compeiilion.
F

SOLUTION Let A, S, C, D be the events that the competitors P, Q, R and S respectively win the
competition. Then,
1 1
P{A) = -, P(B) =
4
, P(C) = 5- and P (D) = y6
Since only one competitor can win the competition. Therefore, A, B, C, D are mutually exclusive
events.

Required probability = P(AuBuCuD)


= P{A) + P{B) + P{C) + P{D) [By addition Theorem]
1 1 1 1 114
= — 4- — + — + — —
3 4 5 6 120
EXAMl’l E 20 (i) Two dice are thrown together. What is the probability that the sum of the numbers _ on

the two faces is neither divisible by 3 nor by 4? (ii) What is the probability that the sum of the numbers on
the two faces is divisible by 3 or 47
20.46
APPLIED MATHEMATICS-XI

SOLUTION (i) Let S be the sample space associated with the experiment of throwing a pair of
dice. Then, n{S) = 36.
Total number of elementary events = 36
Consider the following events.
A = The sum of the numbers on two faces in divisible by 3
B = The sum of the numbers on two faces is divisible by 4.
Then, ^ = [{1, 2), (2,1), (1, 5), (5,1), (3, 3), (2, 4), (4, 2), (3, 6), (6, 3), (4, 5), (5, 4), (6, 6))
B = 1(2, 2), (1, 3), (3,1), (2, 6), (6, 2), (4, 4), (3, 5), (5, 3), (6, 6)j and, A r\ B = [(6, 6))
P{A) = ^ = “/ P(B) = ~
ab a 36
= - and P (A n B) = —
4 36
(i) Required probability = P (A n B) = P {A u B) = \-P {A uB)
= ^~[PiA) + P{B)-PiAr^B)] = + 4

w
1 3 4 36 J 9

(ii) Required probability = P{AuB)= P{A) + P{B)-P{AnB) = i3 + l_± = ^9

Flo
4 36

LXAMi'Ln 21 An urn contains twenh/ white slips of paper numbered from 1 through 20, ten red slips of

ee
paper numbered from 1 through 70, forty yellow slips of paper numbered from 1 through 40 and ten blue
slips of paper numbered from 7 through 10. If these 80 slips of paper are thoroughly shujfled so that each

Fr
slip has the same probability of being drawn. Find the probabilities of drazuing a slip of paper that IS
(i) blue or white (ii) number 1.2,3,4 or 5 (Hi) red or yellow and numbered 1,2,3, or 4. (iv) numbered 5,
for
.75, 25 or 35. (v) white and numbered higher than 72 or yellow and numbered higher than 26.
ur
s
SOLUTION There are 80 slips of paper out of which one slip can be chosen in 80
C-| = 80 ways.
k
Yo
oo

So, total number of elementary events = 80


eB

(i) There are 10 blue and 20 white slips out of which one slip can be chosen in 30
= 30 ways.
Favourable number of ways = 30
r

Hence, P (Drawing a blue or white slip) = — = —


ou
ad

80 8

(ii) Consider the following events;


Y

W = Drawing a white slip numbered 1, 2,3,4 or 5,


Re
nd

R = Drawing a red slip numbered 1,2,3,4 or 5,


y = Drawing a yellow slip numbered 1, 2, 3, 4 or 5,
Fi

B = Drawing a blue slip numbered 1, 2, 3, 4 or 5


Clearly, these events are mutually exclusive.
Required probability = P (W u R u V u 6)
= P{W) + P(R)+P{Y) + PiB) = ~ + — +
80 80 80 80 ~80~4
(iii) Consider the following events:
R = Drawing a red slip numbered 1, 2, 3 or 4,
y = Drawing a yellow slip numbered 1, 2, 3, or 4
Clearly, R and y are mutually exclusive events such that P (R) = A
80
and P (y) = —.
80

Required probability = P (R ^y) = P (R) + P (y) = —+ A=i_


80 80 10
20.47
PROBABILITY

(iv) Consider the following events:


A = Drawing a slip numbered 5, B = Drawing a slip numbered 15
C = Drawing a slip numbered 25, D = Drawing a slip numbered 35
We observe that A, B, C and D are mutually exclusive events such that
4
P{A)=- [●.● There are 4 tickets, one of each colour numbered 5]
80

P{B)=3:80 [●.● There is one white and one yellow ticket each numbered 15]

P(C)=.^80 [●.● There is just one yellow ticket numbered 25]


1
and. [v There is just one ye]low ticket numbered 35]

w
80
4 2 1 1 1
Required probability - (/I uBuC ^ ^ ^ ^ ^ gg ” 10

Flo
(v) Consider the following events:

e
A = Drawing a white slip numbered higher than 12

re
B = Drawing a yellow slip numbered higher than 26

F
8 14
We observe that /\ and Bare mutually exclusive events such that P (/4) = — and P (^)
ur
r
Required probability = P (/4 u 6) = P (A) + P (B) = —+ ^^
fo
ks
Yo
Ivvc III PROBLEMS WHICH CAN BE SOLVED BY USING ADDITION THEOREMS ONLY
oo

EXAMI’H. 22 Tzoo cards are drawjj from a pack of 52 cards. Whal is the probabilily that either both aie
B

red or both are kings?


52
SOLUTION Out of 52 cards, two cards can be drawn in C2 ways.
re

So, total number of elementary events = ' C2.


u
ad

Consider the following events:


Yo

A = Two cards drawn are red cards, B = Two cards drawn are kings.
Required probability ^ P (.A u B) = P(A) + P(B) - P (A n B) (By addition Theorem] ...(i)
d
Re

Let us now find P (A), P (B) and P (A n 6).


in

26
There are 26 red cards, out of which 2 red cards can be drawn in C2 ways.
26
F

c.
P(A) = 52
C9
Since there are 4 kings, out of which 2 kings can be drawn in ‘^€2 ways.
*-2
P(B) = 52
C2
There are 2 cards which are both red and kings.
P (A o B) = Probability of getting 2 cards which are both red and kings.
'C2
= Probability of getting 2 red kings = 52
C,

Required probability = P(A) + P(B) - P (A n B)


26
C2 ^^2 ^€2 _ 325 1 1 55
52 52 52 1326 221 1326 221
C2 C2 C2
.48
MATHEMATICS-XI

EXAMIM.I-: 23 A basket contains 20 apples ami 10 oiwiges out of which 5 apples and 3 oranges are
defective. If a person takes out 2 at random zvhat is the probability that either both are apples or both are
good?
SOLUTION Out of 30 items, two can be selected
30
in ways.
So, total number of elementary events = C2-
Consider the following events: A = Getting two apples, B = Getting two good items
There are 20 apples, out of which 2 can be drawn in ~^C2 ways.
20

P{A) =
^2
30
C2
There are 8 defective pieces and the remaining 22 are good. Out of 22 good pieces, two can be
selected in ^€2 ways.
22
C2

w
P{B) = 30
C2

F lo
Since there are 15 pieces which are good apples out of which 2 can be selected in ways.
15
C2
P (A nB) = Probability of getting 2 pieces which are good apples = 30

ee
C2

Fr
20 22 15
C2 C2 C2 _ 316
Required probability = P{A) + P(B) -P{A r>B) = 30 r
+
for 30p 30^ 435
C2 (-2 C2
A die has two faces each with number ‘1', three faces each with number '2' and one face
ur
EXAMPLE 24

with number 'S’. If the die is rolled once, determine


s

(i) P (1) (ii) P (1 or 3) (hi) P (jiof 3)


ook
Yo

SOLUTION Let A, B, C be three events defined by


eB

A = Getting a face with number'r, B = Getting a face with number'2',


C = Getting a face with number '3'

P(A) = 7 = \,P(B) = I
our

Then,
ad

- and P (C) = -
6 3 6 2 ^ 6
(i) P(2) = P(^) = i
Y

(ii) P(1 or 3) == P {A uC) - P{A) + PiC)


Re

[●.● A and C are mutually exclusive]


nd

1
=> P (1 or, 3) = - + -
Fi

3 6 2

(hi) P(not 3) = P(C) = l-P(C) = 1 i = 1


6 6

EXAMPLE 25 The probability that a student will receive A, B, C or D grade are 0.40,0.35,0.15 and 0.10
respectively. Find the probability that a student will receive
(i) BorC grade (ii) at least C grade.
SOLUTION Let £^,£2 ,£3 and £4 denote respectively the events that a student will receive
A, B,C and D grades. Then,
P(£l) = 0.40, £(£3) = 0.35, £(£3) = 0.15 and P (£4) = 0.10
(i) Required probability = £(£3 vj £3)
= £(£3)+ £(£3) [●.● £3 and £3 are mutually exclusive events]
= 0.35 + 0.15 = 050
(ii) Required probability = Probability that the student receives C or D grade
= £(£3 ^£4)
20.49
PROBABILITY

= P(£3) + P(E4) [●.● £3 and £4 are mutually exclusive]


= 0.15 + 0.10 = 0.25
2
EXAMPLE 26 The probability that a person uhll get an electric contract is -5 nnd the probabilih/ thnt he
2
will not get plumbing contract is If the probability of getting at least one contract is ^— , xvhat is the
probability that he zuill get both?
SOLUTION Consider the following events:
A = Person gets an electric contract, B = Person gets plumbing contract
Clearly, P (A) = 5 P(B) = -andP(/iuB) =
7 d

w
We have to find P{An B). By addition theorem, we have
P (A u B) = P (A) + P (6) - P n 6)
2
-1 - P (A n B) => P{AnB) = |5 3 - 23 ^ 17
” 105

Flo
3 5 7j 7
EXAMPLE 27 If a pei'son visits his dentist, suppose the probability that he will have his teeth cleaned is

e
0.48, the probability that he will have cavity filled is 0.25, probability that he will have a tooPi extracted is

re
0.20, the probability that he will have a teeth cleaned and cavity filled is 0.09, the probability that he zuill

F
have his teeth cleaned and a tooth extracted is 0.12, the probability that he will have a cavity filled and
tooth extracted is 0.07, ami the probability that he will have his teeth cleaned, cavity iflled, and tooth
ur
r
extracted is 0.03. What is the probability that a person visiting his dentist will have at least one of these
things done to him?
fo
ks
SOLUTION Consider the following events:
Yo

C =The person will have his teeth cleaned, f =The person will have cavity filled
oo

E =The person will have a tooth extracted


eB

It is given that P(C) = 0.48, P(F) = 0.25, P (£) = 0.20,


P (C oF) = 0.09, P (C n£) = 0.12, P{EnF)= 0.07 and P (C n F n£) = 0.03.
ur
ad

Required probability = P (C u F u E)
Yo

= P(C) + P(F) + P(E)-P(CnF)-P(Fn£)-P(CnE) + P(CnFn£)


= 0.48 + 0.25 + 0.20 - 0.09 - 0.12 - 0.07 + 0.03 = 0.68
d

The probability that a patient visiting a dentist zvill have a tooth extracted is 0.06, the
Re

EXAMPLE 28
in

probability that he will have a cavity filled is 0.2 and the probability that he zuill have a tooth extracted as
F

well as cavity iflled is 0.03. What is the probability of that a patient has either a tooth extracted or a cavity
filled?
SOLUTION Let A be the event that the patient will have his tooth extracted, B the event that he
will have a cavity filled.
We have, P (A) = 0.06, P (B) - 0.2 and P (A n B) = 0.03 = 0.23
Required probability = P (A u B) = P (A) + P (B) - P (A n B) = 0.06 + 0.2 - 0.03
EXAMPLE 29 The probability that a person idsiting a dentist zuill have his teeth cleaned is 0.44, the
probability that he zuill have a cavity filled is 0.24. The probability that he zuill have his teeth cleaned or a
cavityiflled is 0.60. What is the probability that a person visiting a dentist zuill have his teeth cleaned and
cavity filled?
SOLUTION Let A be the event that the patient will have his teeth cleaned and Bbe the event that
he will have cavity filled.
We have, P (A) = 0.44, P(B) = 0.24 and P (A u B) = 0.60
= 0.08
Required probability = P {A r\B) = P (A) + P (B) - P{Au B) = 0.44 + 0.24 - 0.60
20.50
APPLIED MATHEMATICS-XI

2
FXAMI’l.n.lO
Probability that Hameed passes in Mathematics is - and the probabilih/ that he passes in
4 . 1
English is If the probability of passing both courses is what is the probability that Hameed will pass in
at least one of these subjects?
SOLUTION Let A be the event that Hameed passes in Mathematics and B be the event that he
passes in English.
We have, P (A) = —,P(B) = - and P (A n B) =-
3' 9 4

ow
Required probability = P(AuB)=^ P (A) + P (B) - P (A n B) = - + 1 = 11
-
3 99 4 ’ 36
Find the probability of at most two tails or at least two heads in a toss of three coins.
EXAMI’I L .^1

e
SOLUTION Consider the following events:

re
A = Getting at most two tails in a toss of three coins.
B = Getting at least two heads in a toss of three coins.

Frl
F
We have, S = {HHH, HHT, HTH, THH, HTT, THT, TTH, TTJ],
ou
/I = {HHH, HHT, HTH, THH, TTH, HTT, THT], B = {HHT, HTH, THH, HHH]

sor
P{A) =l,P(B) = t 12 and P {A n B)^8
= - 2
1

Required probability = P{A'wB)= P (A) + P (B) - P (A


kf nB) = - +
1 7
8 2 2
oo
8
I.XAMl’l I .^2
In a tozvn of 6000 people 1200 are over 50 years old and 2000 arefemale. It is known that
Y

30% of the females are over 50 years. What is the probability that a random chosen individual from the
B

town either female or over 50 years?


SOLUTION Consider the following events :
re
oY

A ~ A randomly chosen individual is a female


u

6
= A randomly chosen individual is over 50 years old.
ad

2000
Clearly, P (A) = 1200 _ 1
d

6000 6000 " 5


in

and, P {A n B) - P (An individual is a female over 50 years old) = 30% of 2000 _ 1


Re

6000 Soo "lO


F

1 1 1 13
Required probability = P (A u 6) = P{A) + P{B)-P{AnB)= -
3 5 10 30
i XAMl’I.I' .3.3
In class XI of a school, 40% of the students study Mathematics and 30% studi/ Biology.
10% of the class study both Mathematics and Biology. If a student is selected at random from the class,
find the probability that he will be studying Mathematics or Biology or both.
SOLUTION Consider the following events:
M = A student studies Mathematics, B = A student studies Biology.
40 30 10
We have, P(M) ~ P{B) = and P (A4 n B) =
100 ' 100 100

Required probability = P (A4 u B)


40
= P{M) + P{B)~P{MnB) = + 30 10 60 _ 3
100 100 100 100 " 5
20.51
PROBABILITY

rxAMPLi: 34 Tzoo students Anil and Ashima appeared in an examination. The probnhiliti/ that Anil will
qualify the examination is 0.05 and that Ashima will qualify the examination is 0.10. The probability that
both will qualify the examination is 0.02. Find the probability that:
(i) both Anil and Ashima will not qualify the exam.
(ii) at least one of them will not qualify the exam.
(iii) only one of them will qualify the exam.
SOLUTION Let £ and F denote the events that Anil and Ashima will qualify the examination.
Then, P (£)= 0.05, P (F) = 0.10 and P (£ n F) = 0.02
(i) Required probability = P (£ n F)
= P(EoF)=l-P(EuF)
= l-lP(E) + P(F)-P(£nFl =1 -(0.05 + 0.10-0.02)= 0.87
(ii) Required probability = P (At least one of tliem will not qualify tlie exam)

w
= 1 - P (Both of them will qualify the exam)
= 1 - P (E n F) = 1 - 0.02 = 0.98

F lo
(iii) P (Only one of them will qualify the exam) = P (£) + P (F) - 2P (E n F)
= 0.05 + 0.10-2x 0.02= 0.15-0.04 = 0.11

ee
class of 60 students 30 optedfor NCC 32 opted for NSS and 24 optedfor both NCC

Fr
LXAMI’LESS In a

and NSS. If one of these students is selected at random, find the probability that:
(i) the student opted for NCC or NSS (ii) the student has opted neither NCC nor NSS
for
ur
(iii) the student has opted NSS but not NCC.
SOLUTION Consider the following events:
s
ok

A=A sUident opted NCC, B = A student opted NSS


Yo

We have.
o

30 32 24
eB

^ 32_24 ^ 38 ^19
(i) Required probability = P (A u B) = P (A) + P (B) - P (A n B) = 60 60 60 60 30
r
ou
ad

11
(ii) Required probability = P (A n B) = P (A u B) = 1 - P (A n B) - 1 - 30 30
Y

2
^_2A
(iii) Required probability = P (A n B) = P (B) - P (A n B) 60 60 60 15
Re
nd

EXAMPLE 36 One of the four persons John, Rita, Aslani or Gurpreet will be promoted next month.
Fi

Consequently the sample space S consists offour elementary outcomes as given below.
S = I John promoted, Rita promoted, Aslam promoted, Gurpreet promoted}
You are told that the chances of John's promotion is same as that of Gurpreet. Rita's chances of promotion
arc twice as likely as Johns. Aslam's chances are four times that of John,
(i) Determine P (John promoted), P (Rita promoted), P (Aslam promoted), P (Gurpreet promoted)
(ii) If A = {John promoted or Gurpreet promoted },find P( A)
SOLUTION (i) Let P (John promoted) = p. Then, by hypothesis
P (Gurpreet promoted) =p,P (Rita promoted) = 2p and, P (Aslam promoted) = 4p.
We have, S = 1 John promoted, Rita promoted, Aslam promoted, Gurpreet promoted}
P (John promoted) +P (Rita promoted) +P (Aslam promoted) + P (Gurpreet promoted) = 1
1
=> p + 2p + 4p + p=^l => p =
1 .X n 2 1
Hence, P(John promoted) = -, P (Rita promoted) = = g = 4'
20.52
APPLIED MATHEMATICS-XI

P (Aslam promoted) = 4/j = —8 - and, P (Gurpreet promoted) = p = -


^ 8
(ii) A = {John promoted or Giirpreet promoted }
2 1
P {A) = P (John promoted) + P (Gurpreet promoted) = p + p = 2p =
4

EXAMPLE 37 If A and B are am/ huo events such that P (A u B) = 1 and P (A) ,find P (A n B).
SOLUTION Clearly, A nB and A are mutually exclusive events such that
A\jB = Au(AnB)
=>
PiA^B) = P(A) + P{AnB)

w
-=i-P(A) + P{AnB)
1
= 1-
=>
~+P(Ar^B) => P(AnB)=-

o
2 j 6

e
re
AI.ITLR We have, P{A uB) = i2

rFl
F
=> P(A) + P(B)-P(AnB)=i

or
ou
Fig. 20.5
1 -
|]+P(B)-P(/lnB)=i
ksf
●jJ 2

\3 + P{AnB}=-^ P{Ar^B)=-
oo

2 ^6
Y

EXAMPLE38 Figure 20.6 shows three events A, B and C and also the probabilities of the
B

various
intersections (for instance P (A r\B) =0.07) determine
re

(i) P(A) (ii) P(BnC) (iii) P(A u B) (iv) P(AnB)


(v) P{Br^C) (vi) Probabiliti/ of exactly one of the three events.
oYu

SOLUTION (i) We have, P{AnB)= 0.13 and P{AnB)= 0.07


ad

Since AnB and A nBare mutually exclusive events such that


d

A =(AnB)u(Ar^B).
in

P(A) = P(AnB) + P(AnB)


Re

=> P (A) =0.13+ 0.07 =0.20


F

(ii) It is evident from the Figure 20.6 that


P(A r^B)= 0.07, P (BnC) = 0.15
and P(AnBnC) =0.10
Also, A B, B oC and A B oC are mutually
exclusive events such that
B=(AnB)u(BnC)u(A nBnC)
P(B)=P(AnB) + P(BnC) + P(AnBnC)
P(B) = 0.07+ 0.15+ 0.10 = 0.32 Fig. 20.6
Now, P(BnC)=P(B)-P(BnC)
P (BnC) =0.32-0.15 =0.17
(iii) We have, P (A) = 0.20, P (B) = 0.32 and P(AnB)= 0.07
P (A u6) = P (A) + P(B) -P(AnB) = 0.20 + 0.32-0.07 = 0.45
20.53
PROBABILITY

(iv) We have, P (/I) = 0.20 and P (A n B) = 0.07


P(AnB)=P(A)-P(AnB) -0.20-0.07 =0.13
(v) It is evident from the Figure 20.6 that P (B nC) = 0.15.
(vi) Probability that exactly one of three events A, B and C occurs
= P(An6nC) + P(A nBnC)+P(AnBnC) = 0.13+0.10+ 0.28 = 0.51
FiX AMPLE 39 For a post three persons A, B and C appear in the intervieiv. The probalhlity of A
selected istzoice that of Band the probability ofB being selected is thrice that ofC. What are the individual
probabilities of A, B, C being selected?
SOLUTION Let A|, A2 and A3 be three events as defined below:
= Person A is selected, A2 = Person B is selected, A3 = Person C is selected.

ow
A1

We have, P (A^^) = 2P (A2) and P (A2) — 3P (A3) p (/\j) =6P (A3) and P(A2) = 3P (A3).
Since Aj, A-,, A3 are mutually exclusive and exhaustive events.
A| uAo u A3 = S

e
P (A| cj A2A3) = P (5)

re
P(Ai) + P(A2) + P(A3) = 1 [ Aj, A2, A3 are mutually exclusive]
6P(A3)+ 3P(A3) + P(A3) = 1

Frl
F
1
10P(A3) = 1 => P(A3) = —
ou
or
3
— and P (A2) = —
P(Ai) = 10 10
kfs
EXAMPLE 40 A box Contains 6 red, 4 white and 5 black balls. A person drazvs 4 balls from the box at
random. Find the probability that among the balls drazun there is atleast one ball of each colour.
oo

SOLUTION We observe that at least one ball of each colour can be drawn in one of the following
Y
B

mutually exclusive ways:


(i) 1 red, 1 white and 2 black balls, (ii) 2 red, 1 white and 1 black balls.
re

(iii) 1 red, 2 white and 1 black balls.


Thus, if we define three events A, B and C as follows:
oYu

A = Drawing 1 red, 1 white and 2 black balls, B = Drawing 2 red, 1 white and 1 black balls
ad

C = Drawing 1 red, 2 white and 1 black balls


d

We observe that A, B, C are mutually exclusive events.


in
Re

Required probability = P {A u B C)
= P (A) + P {B) + P (C) [By addition Theorem]
F

6
C, X X 5C2 ^C2 X 15+1 X +1 X ^C2 X
15
15
C4 C4 C4
6x4x10 + 15x4x5 + 6x6x5 24 X 720 48

15 15x14x13x12 91
C4
EXAMPLE 41 A die is haded in such a way that each odd number is tzvice as likely to occur ns each even
number. Find P(G), lohereGis the event that a number greater than 3 occurs on a single roll of the die.
face of the die",
SOLUTION Let Aj denote the event "Getting number / on the upper
/=!, 2, 3, 4,5, 6. Clearly, A,-; /=!, 2,..., 6 are mutually exclusive and exhaustive events.
It is given that P (A2) = P (A4) = P (A^) = p (say) and, P (A^ = P (A3) = P (A5) = 2p.
Now, A^ UA2 WA3 UA4UA5 vjA^ =S
P(AiuA2^A3uA4uA5uA6)=P(S)
20.54
APPLIED MATHEMATICS-XI

P (A^) + P {A2) + P (A3) + P (A4) + P (Ag) + P (Afi) = 1


1
2p + p + 2p + p + 2p + p = l 9p=l=^ p 9
Now, G = A4 UAg UA5
P(G) =-P (A4 uAg vjA^)
P(G)=P(A4) + P(Ag) + F(A6) [●-● A4, Ag, A^ are mutually exclusive]
P(G)=p+2p + p - 4p = ^
EXAMl’LL -J2 A tenm of medical students doing their internship have to assist during surgeries at a city
hospital The probabilities of surgeries rated as very complex, complex, routine, simple and very simple are
respectively 0.15, 0.20, 0.31, 0.26, 0.08. Find the probabilities that a particular surgery will be rated
(i) Complex or very complex (ii) neither very complex nor
very simple (Hi) routine or complex
(iv) routine or simple.

w
SOLUTION Consider the following events:
A = Surgery is very complex, B = Surgery is complex, C = Surgery is routine,

F lo
D ~ Surgery is simple, E = Surgery is very simple.
It is given that P (A) = 0.15 P (B) = 0.20, P (C) = 0.31, P (D) = 0.26 and P {£) = 0.08.

e
Clearly, A, B,C,D and Pare mutually exclusive events,
(i) Required probability = P (A'uB) = P (A) + P {B) = 0.15 + 0.20
Fre
= 0.35
for
(ii) Required probability = P (A n£) =P(aUE)
= 1 - P (A u£) = 1 - {P (A) + P (£)} = 1 - (0.15 + 0.08)
r
= 0.77
You

(iii) Required probability = P (C u P) = P (C) + P (6) = 0.31 + 0.20 = 0.51


oks

(iv) Required probability = P (C kjD) = P (C) + P (D) = 0.31 + 0.26 = 0.57


eBo

EXERCISE 20.4
1- (a) If and P be mutually exclusive events associated
ad
our

with a random experiment such


that P(A) = 0.4 and P(P) = 0.5, then find:
(i) P (A u P) (ii) P (A n P) (iii) P (A n P) (iv) P (A n B).
(b) A and P are two events such th^t P (A) = 054, P (P)
dY
Re

= 0.69 and P (A n P) = 0 35
Find (i) P (A u P) (ii) P (A n P) (hi) P (A n B) (iv) P (P o A)
Fin

(c) Fill in the blanks in the following table:


P(A) P(B) P(AnP) P (A w B)
1 1 1
(i)
3 5 15
(ii) 0.35 0.25 0.6
(iii) 0.5 0.35 0.7

2. If A and B are two events associated with a random experiment such that P(A) = 0.3,
P(B) = 0.4 and P (A u B) = 05, find P (A n B).
3. If A and 6 are two events associated with a random experiment such that P(A) = 0.5,
P(B) = 0.3 and P (A n B) = 0.2, find P (A u B).
4. If A and B are two events associated with a random experiment such that P (A u B) = 0 8
P(A n B) = 0.3 and P (A) = 05, find P(P).
20.55
PROBABILITY

5. Given two mutually exclusive events A and B such that P{A)='U2 andP(6)-l/3,
find P {A or B).
. There are three events A, B, C one of which must and only one can happen, the odds are 8 to
3 against A, 5 to 2 against B, find the odds against C.
7. One of the two events must happen. Given that the chance of one is two-third of the other,
find the odds in favour of the other.
xOli: students are advised to do thefoUozving exercises bi, using addition theorems and also by
using the definition only i.e. by calculating exhaustive number of cases and favourable number of
cases.

8. A card is drawn at random from a well-shuffled deck of 52 cards. Find the probability of its
being a spade or a king.
9. In a single throw of two dice, find the probability that neither a doublet nor a total of 9 will

ow
appear.
10. A natural number is chosen at random from amongst first 500. What is the probability that
the number so chosen is divisible by 3 or 5?
at least one of the two throws come up

e
11. A die is thrown twice. What is the probability that

Fl
re
with the number 3?
deck of 52 cards. Find the probability of getting an ace or a spade

F
12. A card is drawn from a
card.
ur
The probability that a student will pass the final examination in both English arid Hindi is
13.

or
0.5 and the probability of passing neither is 0.1. If the probability of passing the English
sf
examination is 0.75. What is the probability of passing the Hindi examination?
k
chosen from numbers 1 to 100. Find the probability that it is divisible
Yo
14. One number is
oo

by 4 or 6?
From a well shuffled deck of 52 cards, 4 cards are drawn at random. What is the probability
B

15.
that all the drawn cards are of the same colour.
re

16. 100 students appeared for two examinations. 60 passed the first, 50 passed the second and
30 passed both. Find the probability that a student selected at random has passed at least
u
ad
Yo

one examination.
A box contains 10 while, 6 red and 10 black balls. A ball is drawn at random from the box.
the probability that the ball drawn is either white or red?
d

What is
Re

:4,1:5 and 1: 6 respectively. Find


in

18. In a race , the odds in favour of horses A, B, C, D are 1:3,1


probability that one of them wins the race.
F

19. The probability that a person will travel by plane is 3/5 and that he will travel by tram is
1 /4. What is the probability that he (she) will travel by plane or train?
2U . Two cards are drawn from a well shuffled pack of 52 cards. Find the probability that either
both are black or both are kings.
21. In an entrance test that is graded on the basis of tw'o examinations, the probability of a
randomly chosen student passing the first examination is 0.8 and the probability of passing
the second examination is 0.7. The probability of passing at least one of them is 0.95. What is
the probability of passing both?
A box contains 30 bolts and 40 nuts. Half of the bolts and half of the nuts are rusted. If two
' ‘ items are drawn at random, what is the probability that either both are rusted or both are
bolts?
20.56
APPLIED MATHEMATICS-XI

23. An integer is chosen at random from first 200 positive integers. Find the probability that the
integer is divisible by 6 or 8. t }
24. Find the probability of getting 2 or 3 tails when a coin is tossed four times.
2d. Suppose an integer from 1 through 1000 is chosen at random, find the probability that the
integer is a multiple of 2 or a multiple of 9. r /
26. In a large metropolitan area, the probabilities are 0.87, 0.36, 0.30 that a family (randomly
chosen for a sample survey) owns a colour television set, a black and white television set, or
both kinds of sets. What is the probability that a family owns either any one or both kinds of
sets?

27. If A and Bare mutually exclusive events such that F (A) =0.35 and P (B) = 0.45, find
(i) £(AuB) (ii) P(AnB) (hi) P(AnB) (iv) F(AnB)
28. Asamplespaceconsistsof9elementary eventEj, £3,Eg, £9 whoseprobabiliHesare

w
£(£]) = £(£2) = 0.08, £(£3) = £(£4) = 0.1, P(Ef^) =P(Ey) = 0.2, £(£g) = £(£.) = 0.07
Suppose = {£1, £g}, B = {£2, £g, £9}
(i) Compute£(A), £(B) and £ (AnB).

Flo
(ii) Using the addition law of probability, find £ (A u B).
(hi) List the composition of the event AuS, and calculate £(AuB) by addine the

ee
probabilities of the elementary events,

Fr
(iv) Calculate £ (B) from £(6), also calculate £ (B) directly from the elementary events of
for
ur
ANSWERS

1. (a) (i)0.9 (ii) 0.1 (hi) 0.5 (iv) 0.4 ]. (b) (i) 0.88 (ii) 0.12 (hi) 0.19 (iv) 0.34
ks
Yo

(0 0.5 (hi) 0.15 2. 0.2 3. 0.6 4. 0.6 5. 5/6


oo

15
eB

6. 43 :34 7. 3:2 8. 4/13 11


9. 13/18 10. 233/500 11. —
36
4
r

12. — 33 92
13. 0.65 14. 8
ou

15.
ad

13 16. - 17. —
100 833 5 13
171
Y

17 55
18. 19. — 20. 185 1
21. 0.556 22. 23.
420 20 221 483 4
nd
Re

5
24. 25. 0.556 26. 0.93
8 27. (i) 0.8 (ii) 0 (hi) 0.35 (iv) 0.2
Fi

28. (i) 0.25, 0.32, 0.17 (ii) 0.40 (iii) 0.40 (iv) 0.68

HINTS TO SELECTED PROBLEMS


1. (ii) Use : £ (A n B) = £ (TO B) = 1 - £ (A u B)
(iii) Use : £ (A n B) = £ (B) - £ (A n B)
(iv) Use : £ (A n B) = £ (A) - £ (A n B)
6. It is given that A, B, C are mutually exclusive and exhaustive.
AkjBuC = S => £(AuBuC) £{S) => £(A) + £(B) + £(C)=1
7. Let AA, uB
6 be= two
S
events. Then A, 6 are mutually exclusive and exhaustive
£ (AuB)=l => P(A) + P{B)=1
PROBABILITY 20.57

=> -P(B) + P(B)=1


3 V P(/l) = |p(B)
- P(B) = |
Odds in favour of B are P (6): P (B) i.e. 3/5 ; 2/5 or 3 : 2

9. Let = Getting a doublet, B = Getting a total of 9. Then,


P{AnB) = P (aUB) =1 -P (A u 6) =1 -[P (A) + P{B)-P{Ar^ B)]
12. A = Getting an ace, B = Getting a spade card.
Required probability = P {A<jB) = P (A) + P {B) -P {AnB)

ow
13. Let E and H denote the events that the student will pass in English and Hindi examination
respectively. Then, we have P (£ n H) = 05, P (£ o H) = 0.1 and P (£) = 075.
Now, P{EnH) = 0.1
=> P (E^jH) = 0.1 => 1 -P(£uH) = 01 => P(£ uH) = 0.9

e
Fl
re
=> P (£) + P (H) - P(£ n H) = 0.9 => 0.75 + P{H)-05 = 0.9 => P (H) = 0.65.
15. Let A = 4 cards drawn are red, B = 4 cards drawn are black. Then, A, B are mutually

F
exclusive events.
ur
So, required probability = P {A B)=P (A) + P (B)
or
sf
21. Let A and B denote the events that a randomly chosen student passes first and second
examinations respectively. Then, P (A) = 0.8, P (B) = 07 and P (A u B) = 0.95
k
Yo
oo

Required probability =P {A r\B) =P (A) + P (B) - P (A vj B) = 0.8 + 07 - 0.95 = 055


25. Consider the following events:
B

A = Integer chosen is a multiple of 2, B = Integer chosen is a multiple of 9.


e

111 55
500
= 1,P(B) = 1000 and P (A n B) = 1000
ur

We have, P(A) =
1000 2
ad

1 111 55 555
Yo

Required probability = P (A u B) = P(A) + P{B)-P(AnB) = - + ^ ~


d

26. Consider the following events:


Re
in

A = Family owns colours television set, B = Family owns black and white television set
F

It is given that P(A) = 0.87, P(B) = 0.36 and P (A n B) = 0.30


Required probability = P (A u B) = P(A) + P(B) - P(A n B) = 0.87 + 0.36 - 0.30 = 0.93
27. It is given that A and B are mutually exclusive events. Therefore, P (A n B) = 0.
(i) P (A u B) = P(A) + P(B) = 0.35 + 0.45 = 0.8 (ii) P(AnB) = 0
(iii) P (A n B) = P(A) -P (A n B) = P(A) = 0.35
(iv) P(AnB)=P (AnB) = l-P(AuB)=l-(P(A) + P(B)} = 1 = 0.8 = 0.2
28. We have, P(£i) + P(£2) + PfEs) + £(£4) + + £(£7) + £(^8) = ^ => = 0-1
(i) P(A) = P(£i) + £(%) + £(£3) = 0.08 + 0.1 + 0.07 = 0.25
P(B) = £(£2) + £(%) + £(£3) + £(£9) = 0-08 + 0.1 + 0.07 + 0.07 = 0.32
P(A n B) = P(%) + £(£3) = 0.1 + 0.07 = 0.17
(ii) P(A u B) = P(A) + P(B) -£(A n B) = 025 + 0.32 - 0.17 = 0.40
20.58 APPLIED MATHEMATICS-XI

20.9 CONDITIONAL PROBABILITY

Let A and B be two events associated with a random experiment. Then, the probability of
occurrence of events under the condition thatBhas already occurred and P (B) 0, is called the
conditional probability and it is denoted by P (A/B). Thus, we have
P {A/B) = Probability of occurrence of A given that B has already occurred.
Similarly, P {B/A) when P (A) 0 is defined as the probability of occurrence of event P when A
has already occurred.
In fact, the meanings of symbols P{A/B) and P(B/A) depend on the nature of the events
A and B and also on the nature of the random experiment. These two symbols have the following
meaning also.
P {A/B) = Probability of occurrence of A when B occurs

w
OR

Probability of occurrence of A when B is taken as the sample space


OR

Flo
Probability of occurrence of A with respect to B.

e
and, P {B/A) = Probability of occurrence of B when A occurs

re
OR

F
Probability of occurrence of B when A is taken as the sample space.
OR
ur
r
Probability of occurrence of B with respect to A.
fo
In order to understand properly the meaning of conditional probability let us discuss the
ks
following illustrations.
Yo

ILLUSTRATION 1 let there be a bag containing 5 white and 4 red balls. Two balls are drazvn from the
oo

bag one after the other ivithoiit replacement. Consider the following events:
B

A = Drawing a white ball in the first draw. B = Drawing a red ball in the second draw.
re

Now,
P {B/A) = Probability of drawing a red ball in second draw given that a white ball
u

has already been drawn in the first draw


ad
Yo

^ P(B/A) = Probability of drawing a red ball from a bag containing 4 white and
4 red balls
d

=> P (B/A) = - = -
Re
in

8 2

For this random experiment P(A/B) is not meaningful because A cannot occur after the
F

occurrence of event B.
y^TE hi the above illustration
events A and B were subsets of two different sample spaces ns they are
● outcomes of txuo different trials which are performed one after the other.
ILLUSTRATION 2 Consider the random experiment of throwing a pair of dice and two events associated
with it given by
A =
The sum of the numbers on two dice is 8 = ((2, 6), (6, 2), (3, 5), (5, 3), (4, 4))
B = D-iere is an even number on first die = 1(2,1),..., (2, 6), (4,1),..., (4, 6), (6,1),..., (6, 6))
In this case, events A and B are the subsets of the same sample space. So, we have the following
meanings for P (A/B) and P (B/A).
P(A/B) = Probability of occurrence of A when B occurs
or

Probability of occurrence Awhen fls taken as the sample space.


PROBABILITY 20.59

Number of elementary events in B which are favourable to A


P{A/B) =
Number of elementary events in B
Number of elementary events favourable loAnB _ ^
P{A/B)^ 18
Number of elementary events favourable toB
Similarly,
P (B/A) = Probability of occurrence of B when A occurs.
or

Probability of occurrence of B when A is taken as the sample space


Number of elementary events mA which are favourable to 6
P{B/A) =
Number of elementary events in A
Number of elementary events favourable toA nB _ 3
P{B/A) = 5
Number of elementary events favourable to A

w
ILLUSTRATION 3 A die is throzvn twice ami the sum of the numbers appearing is observed to be 6. What
is the conditional probability that the number 4 has appeared at least once?

F lo
SOLUTION Consider the following events :
A ~ Number 4 appears at least once, B = The sum of the numbers appearing is 6.

ee
Required probability = P{A/B)

Fr
= Probability of occurrence of A when B is taken as the sample space
Number of elementary events favourable tOi4 which are favourable to B
for
Number of elementary events favourable to B
ur
Number of elementary events favourable to {A nB) _ 2
5
Number of elementary events favourable to B
s
ook
Yo

It follows from illustrations 2 and 3 that if A and B are two events associated with the same
eB

sample space S of a random experiment, then


Number of elementary events favourableto A n B
P(A/B) =
Number of elementary events favourable to B
our
ad

n (A n B)
n (A n B) »(S)
P(A/B) = [Dividing numerator and denominator byn (S)[
n{B)
Y

n{B)
Re

n{S)
nd

P{Ar^B)
P{A/B) =
Fi

P(B)
Similarly, we have
P (A n B)
P(B/A) =
P(A)
These results have also been derived in the next section by using multiplication theorem on
probability.

ILLUSTRATIVE EXAMPLES

n {A B n(A. B)
I’lipe I EXAMPLES BASED ON P {AIB) P {B/A)
n {B) ’ n(A)

EXAMPLE 1 A/<7/r dice is rolled. Consider thefoUoiuing events:


A = {1,3,5}, B = {2,3}, and C = {2, 3, 4, 5}. Fmd
20.60 APPLIED MATHEMATICS-XI

(i) P(A/B) and P(B/A) (ii) P(A/C) and P(C/A)


(iii) P (A u B/C) ami P {A n B/C) (iv) P (A n B/C)
SOLUTION We have, n (A) = 3,n(B) = 2,n(C) = 4
Clearly,
AnB = l3],AnC = [3,5},AuBnC = {2, 3,5| arid A n B n C = {3}
n(AnB) = l,n{AnC) = 2,n{Au BnC) = 3 and jj (A n 6 nC) = 1
(i) P(A/B) =
n{Ar^B) 1 n(AnB) 1
- — and, P (B/A) =
n{B) n{A) 3
n(A nC) n{AnC) 2
(ii) P(A/C) = = — = - and, P(C/A) =
n{C) 4 2 n{A) 3
n{A B nC) 3
(iii) P (A u B/C) =

ow
'i(C) 4

;i(A n B oC) 1
(iv) P (A n B/C) =
;>(C) 4

EXAMPLE 2 A coin is tossed three times. Find P (E/F) in each of the following:

e
(i) £ = Head on the third toss, F = Heads on first two tosses

Fl
re
(ii) £ = At least two heads, F = At most two heads
(iii) £ = At most two tails, F = At least one tail

F
SOLUTION The sample space associated to the given random experiment is given by
ur
S = {HHH, HHT, HTH, THH, HTT, THT, TTH, TTT}

r
(i) We have. fo
£ = F = {HHH, HHT}
ks
£nf = {HHH}
Yo
Clearly, ;;(£ n f) = 1 and j/(f) = 2
oo

n (£ n f) 1
P(£/f) =
n(F) 2
B

(ii) We have.
re

£ = {HHH, HHT, HTH, THH}, F = {TTT, THT, TTH, HTT, THH, HTH, HHT}
EnF =
u
ad

Clearly, ;i (£ n £) = 3 and n{F) = 7


Yo

n (£ n F) 3
P(E/F) =
n(F) 7
d
Re

(iii) We have.
in

E = {HHH, HHT, HTH, THH, HTT, THT, TTH}


F

F = {HHT, HTH, THH, HTT, THT, TTH, TTT}


EnF = [HHT, HTH, THH, HTT, THT, TTH}
Clearly, h (£ n £) = 6 and n (F) = 7
n (£ n £) 6
P(£/£) =
~Kf) 7

EXAMPLE 3 Two coins are tossed once. Find P (E/F) in each of the following:
(i) £ = Tail appears on at least one coin, F = One coin shows head
(ii) E = No tail appears, F = No head appears.
SOLUTION The sample space associated to the random experiment of tossing two coins is
given by S = {HH, HT, TH, TT}.
(i) We have.
£ = {HT,TH,TT},F = \HT,TH}
EnF = {HT,TH}
PROBABILITY 20.61

Clearly, n (£ n f) =2 and n (f) = 2


n (£ n £)
P{E/F) = ^ = 1
u{F) 2

(ii) We have.
£ = {HH\,F = {FT}
EnF = { J = 0
Clearly, jj (E n F) =0 and n (F) = 1
n (£ n F) 0
- = 0
Required probability =
n{F) 1

EXAMPLE 4 Mother, father mui son Hue up at random for a family picture. Find P {A/B), if A and B are
defined as follows:
A = Son on one end, B = Father in the middle
Total number of ways in which Mother (M), Father (F) and Son (S) can be lined up

w
SOLUTION

at random in one of the following ways:

F lo
MFS, MSF, FMS, FSM, SFM, SMF
We have.
A = {SMF, SFM, MFS, FMS} and B = {MFS, SFM)

e
Fre
AnB = {MFS, SFM)
Clearly, n (A n B) = 2 and n{B) = 2 for
n(AnB) 2
- = 1
Required probability = P{A/B) = n(B) 2
r
A and B are two events such that P (/4) 5^ 0. Find P {B/A), if
You

EXAMPLE 5
oks

(i) A is a subset of B (ii) A n B = (f)


eBo

SOLUTION (i) If A is a subset of B, then


A r-^B = A ^ n (A n B) = n (A)
n (A n B) = 1
P(B/A) =
our
ad

n{A)

(ii) If A n B = then n(A n 6) =0


P{B/A) = n (A n B) ^ 0 = 0
dY

H(^) " ')(/!)


Re

A couple has tiuo children. Find the probability that


Fin

EXAMPLE 6

(i) both the children are boys, if it is knozon that the older child is a boy.
(ii) both the children are girls, if it is known that the older child is a girl.
(iii) both the children are boys, if it is known that at least one of the children is a boy.
:th
SOLUTION Let 6; andG,-(i=l, 2) stand for the event that / child be a boy and a girl
respectively. Then, the sample space associated to the random experiment
IS

S = (Bj B2, B-| G2, Gj 62, G| G2 }●


(i) Consider the following events:
A = Both the children arc boys = {B^ 62}, B ^ The older child is a boy = {Bj B2, S| G2}
AnB = {Bj B2}. Clearly, iJ (A oB) = land n{B) = 2
n (A n B) 1
Required probability =P(A/6) = 2
n{B)
(ii) Consider the following events:
A =Both the children are girls = {Gj G2}, B = The older child is a girl = {G^ G2,Gi B2}
20.62 APPLIED MATHEMATICS-XI

■- A B — {Gj G2}
Clearly,»(A n B) =1 and n (B) = 2
n (/4 n B) 1
Required probability =
n{B) 2

(iii) Consider the following events:


A = Both the children are boys = {B^ B2}
B = At least one of the children is a boy = (B^ B2, B^ G2, G| B2}
AnB = {B1B2}
Clearly,«(A n B) = 1 and «(B) = 3
)]{AnB) 1
Required probability =
n{B) 3

EXAMPLE 7 A pair of dice is throum. If the two numbers appearing on them are different, find the

w
probability (i) the sum of the numbers is 6 (ii) the sum of the numbers is 4 or less (iii) thesumofthe
numbers is 4

SOLUTION Consider the following events:

F lo
A = Numbers appearing on two dice are different
B = The sum of the numbers on two dice is 6

ee
C = The sum of the numbers on two dice is 4 or less

Fr
D = The sum of the numbers on two dice is 4
Clearly,
A =
for
{{1, 2), (1, 3), (1, 4), (1,5), (1, 6), (2,1), (2, 3), (2, 4), (2,5), (2, 6), (3,1), (3, 2),
ur
(3,4), (3,5), (3, 6), (4,1), (4, 2)(4, 3), (4,5), (4, 6), (5,1), (5, 2) (5, 3), (5, 4),
(5,6), (6,1), (6, 2), (6, 3), (6, 4), (6,5)}
s

B = {(1,5), (5,1), (2, 4) (4, 2), (3, 3)1


ook
Yo

C = {(1,1), (1,2), (2,1), (1,3), (3,1), (2, 2)} and, D = {(1, 3), (3,1), (2, 2)}
eB

Clearly, AnB = {(1,5), (5,1), (2, 4), (4, 2)}, A n C = {(1, 2), (2,1), (1, 3), (3,1)}
and. AnD = {(1,3), (3,1)}
n(AnB) = 4,n(AnC) = 4,n{AnD) = 2 and »(v4) = 30
our
ad

(i) Required probability = P (B/A) = /I (A n B) _ 4 _ 2


n(A) “ “ 15
Y

(ii) Required probability = P(D/A) = M (A n D) _ 2 1


n(A) “ M “ 15
Re
nd

(iii) Required probability = P(C/A) = «(A nC) _ 4 _ 2


Fi

n (A) ~ 30 ^ 15
EXAMPLE 8 A die is thrown tiuiceand the sum of the numbers appearing is observed to be 6. What is the
probability that the number 4 has appeared at least once?
SOLUTION Consider the following events:
A = Sum of the numbers appearing on two dice is 6= {(1,5), (5,1), (2, 4), (4, 2), (3, 3)}
B = Number 4 has appeared at least once
= {(1, 4), (4,1), (2, 4), (4, 2), (3, 4), (4, 3), (4, 4), (4, 5), (5, 4), (4, 6), (6, 4)}
AnB = {(2, 4) (4, 2)}
n{A nB) 2
Required probability = P(6/A) =
n(A) 5

EXAMPLE 9 A die is thrown three times. Events A and B are defined as below:
A = Getting 4 on third die, B = Getting 6 on the first and 5 on the second throw
Fitid the probability of A given that B has already occurred.
20.63
PROBABILITY

SOLUTION The sample space S associated to the random experiment of throwing three dice
has 6x6x6 = 216 elements.
We have.
A = {(1,1,4),(1,2,4),(1,3,4),(1,4,4),(1,5,4),(1,6,4)
(2,1, 4), (2, 2, 4), (2, 3, 4), (2, 4, 4), (2,5, 4), (2, 6, 4)

(6,1, 4), (6, 2, 4), (6, 3, 4), (6, 4, 4), (6,5, 4), (6, 6, 4)1
B = {{6,5,1),(6,5,2),(6,5, 3),(6,5,4),{6,5,5),(6,5, 6)}
AnB = {(6,5,4)}
ji (A n B) _ 1
Required probability = P(A/B) = n(B) 6

EXAMIM.EIO A black and a red dice are rolled in order. Find the conditional probabilili/ of obtaining
(i) a Slim grater than 9, given that the black die resulted in a 5.

w
(ii) a sum 8, given that the red die resulted in a number less than 4.
(i) Consider the following events;

F lo
SOLUTION

A = Getting a sum greater than 9, B = Black die resulted in a 5


Clearly, A = {(5, 5), (6, 4), (4, 6), (6,5), (5, 6), (6, 6)}, B = {(5,1), (5, 2), (5, 3), (5, 4), (5,5), (5, 6)}
A n B = {(5, 5), (5, 6)}

e
Fre
Thus, ji(AnB) = 2, ?j(A) = 6 and n(B) = 6
n(AnB) 2 1
Hence, Required probability = P(A/B) = n{B) 6
for3

(ii) Consider the following events:


A = Getting 8 as the sum, B = Red die resulted in a number less than 4
r
You

Clearly, A = {(2, 6), (6, 2)(3,5), (5, 3), (4, 4)}


oks

B = {(6,1), (6, 2) ,(6, 3), (5,1), (5, 2), (5, 3), (4,1), (4, 2), (4, 3), (3,1), (3, 2), (3, 3)
eBo

(2,1), (2, 2), (2, 3), (1,1), (1,2), (1,3)}


AnB = {(6, 2),(5, 3)}
Thus, f!(AoB) = 2,n(B) = 18
(A n B) _ 2 _ 1
our
ad

Required probability = P(A/B) = n{B) 18

EXAMPLE 11 Consider the experiment of throwing a die, if a multiple of 3 comes up, throw the die again
and if any other number comes, toss a coin. Find the conditional probability of the event 'the coin shows a
dY
Re

tail' given that 'at least one die shows a three'.


SOLUTION The sample space S associated to the given random experiment is given by
Fin

S = {(3,1), (3, 2), (3, 3), (3, 4), (3, 5), (3, 6), (6,1), (6, 2), (6, 3), (6, 4), (6,5), (6, 6),
(1, H), (1, T), (2, H), (2, T), (4, H), (4, T), (5, H), (5, T)}
Consider the following events:
A = The coin shows a tail, B = At least one die shows a three
Clearly,A = {(1, T), (2, T), (4, T), (5, T)}, B = {(3,1), (3, 2), (3, 3), (3, 4), (3,5), (3, 6), (6, 3))
A n 6 = ({»
Thus, Ji (A r> B) = 0,n(A) = 4 and ii(B) = 7
n (AnB) 0
- = 0
Required probability = P (A/B) = n(B) 7

EXAM l-LE 12 ^ school, there arc WOO students, out of which 430 are girls. It is known that out of430,
10% of the girls study in class XU. What is the probability that a student chosen randomly studies in class
XII given that the chosen student is a girl?
20.64
APPLIED MATHEMATICS'XI

SOLUTION Let A be the event that a student chosen randomly studies in class XII and B be the
event that the randomly chosen student is a girl. We have to find P (A/B).
10
Clearly, (A n B) = 10% of 430 = 430 x = 43 and,»(6) = 430
100

Required probability = P(A/B) = n (A n B) _ 43 _ 1


n{B) " 430 “ io
EXAMPLE 13 An instructor hns a question bank consisting of 300 easi/ True/false questions, 200
difficult True/False questions, 500 ensij multiple choice questions and 400 difficult multiple choice
questions. If a question is selected at random from the question bank, what is the probability that it luill be
any easy question given that it is a multiple choice question?
SOLl?TlON Let A be the event that selected question is an easy question and B be the event that

w
the question selected is a multiple choice question.
We have.
n(A) = 300 + 500 = 800, n(B) = 500 + 400 = 900

Flo
A nB = ;■ ■
Selected question is an easy multiple choice question
Clearly, n (A ri B) = 500

e
Required probability = P(A/B) = n (A n B) _ 500 _ 5

re
tt(B) 900 " 9

F
Type II EXAMPLES BASED ON P {A/B) PJA , B) P{A B)
AND P (S/4)
ur
~P{A)

r
P(B)

EXAMPLE 14 Given that AandB are two events such


fo
that P(A) = 06 P(B)=03 and
P (A n B) = 0.2,find P (A/B) and P (B/A).
ks
SOLUTION We have,
Yo
oo

P (A n B) P (A n B)
P(A/B) = and P(B/A) =
P(B) P(A)
eB

0.2
P(A/B) =
0.3 ^ and P(6/A) = ^
0.6 ~ 3
^
ur

CXAMPLE15 lfP(A)=f,P(B) = —andP(AuB) =L f/„rf


ad
Yo

11 11 11
(i) P(Ar^B) (ii) P(A/B) (iii) P(B/A)
SOLUTION (i) We know that
d
Re
in

P(AkjB) = P (A)+ P(B)-P (An B)


P(AnB) = P(A) + P(B)-P(AuB) = A + 11 ±11
F

11 11

(ii) P(A/B) = P(AnB)_4/n 4


P(B) ■ 5/11 “ 5
(iii) P(B/A) = P(A nB) ^4/11 _ 2
P(A) 6/11 “ 3

EXAMPLE 16 EvaluateP(AuB),it2P(A] = P (B) = ■^andP(A/B) - -


t)

SOLUTION We have,

2 P (A) = P(B) = ^
13
p (,4) = A
26
and P (B) = 13
P(Ar^B)
Now, P (A/B) =
P(B)
20.65
PROBABILITY

2 P {An B) => P{A


—=
n . . nB) = —
2
5 5 13
13
5 2 11
P (A u 6) = P {A)+ P{B)-P {An B)=-^ + — ~ 13 13 26

Type III PROBLEMS BASED UPON THE MEANING OF CONDITIONAL PROBABILITY


EXAMPLE 17 Two integers are selected at random from integers 1 to 11. If the sum is even, find the
probabilit}/ that both the numbers are odd.

ow
SOLUTION Out of integers from 1 to 11, there are 5 even integers and 6 odd integers.
Consider the following events:
A = Both the numbers chosen are odd, B = The sum of the numbers chosen is even,

e
Required probability

re
= P{A/B)
= Probability that the two numbers chosen are odd if it is given that the sum

Frl
F
of the numbers chosen is even.
The number of ways of getting the sum as an
ou
sor
*“2
even number = ^€2 + ^C2
S/-" , 6f—
C2+ C2
The number of ways of selecting two odd numbers
=

kf
3
oo
5
Y

EXAMPLE 18 A die is thrown three times, if the first throw is a four, find the chance of getting 15 as the
B

sum.
re

SOLUTION Consider the following events:


oY

A = Getting 15 as the sum in a throw of three dice, B = Getting 4 on the first die.
u

Required probability
ad
d

= Probability of getting 15 as the sum of the numbers if there is 4 on the first die.
in

2 V Totalnumberofways = lx 6x 6 = 36
Re

36 There are two favourable elementary events viz. (4.6,5). (4,5,6).


F

18

EXERCISE 20.5

1. Ten cards numbered 1 through 10 are placed in a box, mixed up thoroughly and then one
card is drawn randomly. If it is known that the number on the drawn card is more than 3,
what is the probability that it is an even number?
2. Assume that each child born is equally likely to be a boy or a girl. If a family has two
children, what is the conditional probability that both are girls given that (i) the
youngest is a girl, (ii) at least one is a girl?
3. Given that the two numbers appearing on throwing two dice are different. Find t e
probability of the event 'the sum of numbers on the dice is 4'.
4. A coin is tossed tluee times, if head occurs on first two tosses, find the probability of getting
head on third toss.
20.66
APPLIED MATHEMATICS-Xl

\ A die is thrown three times, find the probability that 4 appears on the third toss if it is given
that 6 and 5 appear respectively on first two tosses.
6. Compute P{A/B), if P (B) = 05 and P (A nB) = 0.32
7. IfP(A) = 0.4, P (6) = 0.3andP(6/A) =05, find P{A n 6) and P(A/B).
8. If A and B are two events such that P (A) , P (B) - - and P {A u B) = — , find
3 5 30
P(A/B) andP(BM).

w
9. A couple has two children. Find the probability that both the children are (i) males, if it is
known that at least one of the children is male, (ii) females, if it is known that the elder
child is a female.
[CBSr-2010]

e
. ANSWERS
4

re
2. (i) i ei) i 1

o
1. 1 1
3. — 4. - 5.

r
7
15 2 6
16 2 5 1
6. — 7. 0.2, - 1 1

F
8. -
25 3 6 ' 2 (i) 3 (ii) -
oF
ul
- hints TO SELECTED PROBLEMS
I. The sample space associated to the given random experiment is given by

sr
S = {1, 2, 3, 4,5, 6,7,8, 9,10}

o
k
Consider the following events:
A = Number on the card drawn is even = (2, 4, 6, 8,10}
B = Number on the card drawn is greater than 3 = {4,5, 6,7,
of
8, 9,10}
o
Y
n {A n B) 4
Required probability = P{A/B) =
rB

11(B) 7

2. Consider the following events:


eY

A = Both children are girls, B = The youngest child is a girl, C = At least one child is a girl
Clearly, S = B^G2. G^B^, GjG^l A = (G^G^l B = and
u

C = {B-^G2, G-|B2, G]G2l


d
o
ad

n (A r>B) 1
(i) Required probability = P(A/B) =
11(B) 2
in

n (A n C) 1
(ii) Required probability = P (A/C) =
n(C)
Re

3
F

20.10 MULTIPLICATION THEOREMS ON PROBABILITY


In this section, we shall discuss some theorems which are helpful in computing the probabilities
of simultaneous occurrences of two or
more events associated with a random experiment.
THEOREM 1 If A and B are two ei’ents associated with a random experiment then
P(AnB) ^ P(A)P(B/A), ifP(A);^0
or.
P(AnB) = P(B)P (A/B), if P (B) ^ 0
I’KlIOl
Let S be the sample space associated with the given random experiment. Suppose S contains n
elementanj e’vents. Let /iq, ni2 and m be the number of elementary events favourable to A, B and A nB
respectively. Then, '
»?2
P(A) = n
P(B) = and P(A r,B) = —.
n n

Since m, elementary events are favourable to A out of which are favourable to B. Therefore,
P(B/A)=—. Similarly, we have P(A/B)= —
20.67
PROBABILITY

B S
/I
m
Now, P{AnB)=-
n

p n B) =— X =P (B/A) X P (A) m
i
w »'2
»Jl n
m

Again, P (A n B) = «

P (A n B) = = p(a/b)P(B) ●●■(ii) Fig. 20.7


H?2 ”
Q.E.D.

nchj:, l Prom (/) and (ii) in the above theorem, we obtain that
P(AnB) P (A n B)
and P(A/B) =

w
P(B/A) = P(B)
P(A)

ri;mark If a and Bare independent events, then P {A/B) = P {A) and P (B/A) = P(B).

Flo
P(Ar^B) = P(A) P(B)
Also, P (A u B) = P (A) + P (B) - P (A n B)

ee
= P(A) + P(B)-P(A)P(B) [vP(AnB) = P(A)P(B)l

Fr
= 1-1 + P(A) + P(B)-P(A)P(B) [Adding and subtracting 1]
= 1-[1-P(A)-P(B) + P(A)P(B)]
for
ur
= 1-P(A)P(B)
(Extension of multiplication theorem). If A^, Aj A„ are n events assoc,ated w,th a
ks
THEOREM 2
Yo

random experiment, then


oo

P(Ai oAt nA3 .... nA„) = P (Aj) P(A2/Ai) PiA^/AinAj)


P(A„/AinA2 0....nA„_i),
eB

whercP(A,/A|nA2...nA,_i)represcnts the . conditional


, ^ probability
, of the occurrence of
event A, , given that the events A^ A2^A,-_ j have already occurred.
r
ou

PROOF This theorem can be proved by using the principle of mathematical induction.
ad

If A, B,Care three events associated with a random experiment, then


Y

PARTICULAR CASE
P{AnBnC) = P (A) P (B/A) P (C/A n B).
nd
Re

ILLUSTRATIVE EXAMPLES
Fi

Type I ON FINDING THE PROBABILITY OF SIMULTANEOUS OCCURRENCE OFTWO OR MORE EVENTS


f„s 10 white and 15 black balls. Two balls are draivn in
EXAMPLE I A bag contains succession zvithout
-white and second is black?
replacement. What is the probability that first is
SOLUTION Consider the following events:
A = Getting a white ball in first draw, 6 = Getting a black ball in second draw.
Required probability
= Probability of getting a white ball in first draw and a black ball in second draw
= P (A and B)
= P (A n B)
= P(A)P(B/A) [By Multiplication Theorem]
10
2
Cl _ 10
Now, P(A) = 25 25 5
Cl
20.68
APPLIED MATHEMATICS-XI

and,
P (B/A) - Probability of getting a black ball in second draw when a white ball has
already been drawn in first draw
15
C,1 15 5 24 ball are left after drawing a white ball’
P(B/A) = 24
C1 24 in first-draw out of which 15 are black
Substituting these values in (i), we obtain
Required probability =P{AnB) = P(A)P{B/A) = -X- = i
5 8 4
EXAMPLE 2
card hi each oftlw two consecutive draws froni 17
we
II shuffled pack of cards, if the card drazun is not replaced after the first draw.
SOLUTION Let ^ be the event of drawing a diamond card in the first draw and Bbe the event of
drawing a diamond card in the second draw. Then,

w
13

P(A) = 52 -^ = - 1
C.1 52 ~ 4

Flo
After drawing a diamond card in first draw 51 cards are left out of which 12 cards are diamond
cards.

e
P (B/A) = Probability of drawing a diamond card in second draw when a diamond

re
card has already been drawn in first draw

rF
12

P(B/A) = 51
C,1 51 17
ur
fo
Required probability = P (A n B) = P(A)P(B/A) = lx —
1
ks
4 17 17
Yo

EXAMPLE 3 A bag contains 5 zvhite, 7 red and 8 black balls. Iffour balls are drawn one by one zvithout
oo

●placement, find the probability of getting all white balls.


B

SOLUTION Let A, B, C and D denote events of getting a white ball in first, second, third and
fourth draw respectively. Then,
re

Required probability = P(Ar^BnCnD)


u

= P (A) P (B/A) P (C/A nB)P (D/A n B n C)


ad
Yo

Now,
P(A) = Probability of drawing a white ball in first draw=-5-=1
20 4
When a white ball is drawn in the first draw there are 19 balls left in the bag, out of which 4 are
nd
Re
Fi

P(B/A) = —
19

wTr -''T replaced, therefore after drawing a white ball in second draw there
are 18 balls left in the bag, out of which 3 are white.
P(CMnB) = A
18
1

6
After drawing a white ball in third draw there are 17 balls left in the bag, out of which 2 are
white.

P(D/AnBnC) = —
17

Hence, Required probability = P (A r\B nC nD)


P (A) P (B/A) P (C/A nB)P (D/A nBnC)
1 4 1 2 1
- — x X— X

4 19 6 17 969
20.69
PROBABILITY

EXAMPLE4 A bag contains 19 tickets, numberedfrom ItolS.A . ticket is drnwn ami then another ticket
. , ,

is drawn without replacement. Find the probability that both tickets will show even numbers.
SOLUTION Let A be the event of drawing an even numbered ticket in first draw and B be the
event of drawing an even numbered ticket in the second draw. Then,
Required probability = P (.A n S) = P (>1) P {B/A)
Since there are 19 tickets, numbered 1 to 19, in the bag out of which 9 are even numbered viz. 2,4,
6,8,10,12,14,16,18.
9
P (A) = —
19

Since the ticket drawn in the first draw is not replaced, therefore second ticket drawn is from
the remaining 18 tickets, out of which 8 are even numbered.
8 4
P(B/A)=—
' 18 9

w
Substituting these values in (i), we get
4 ±
Required probability = P (A n B) = P(A)P(B/A) = 19 9 19

F lo
EXAMPLE 5 Ah urn contains 5 white and 8 black balls. Two successive drawings of three ballsjd a time
are made such that the balls are not replaced before the second draw. Find the probability that thefirst di,
aw

ee
gives 3 while balls and second draw gives 3 black balls.

Fr
SOLUTION Consider the following events:
/I = Drawing 3 white balls in first draw, B = Drawing 3 black balls in the second draw.
Required probability = P (A B) = P (A) P (B/A) for
ur
5C 3 10 5
Now, P(A) = 13 286 143
C3
s

After drawing 3 white8 balls in first draw 10 balls are left in the bag, out of which 8 are black balls.
ook
Yo

C3 56 _ 7
eB

P(B/A) = 10 120 ~ 15
C3
Substituting these values in (i), we obtain ^ ^ 7
r
ad
ou

Required probability = P (A n B) = P (A) P (B/A) = x — 429

EXAMPLE 6 Two balls are drawn from an urn containing 2 white, 3 red and 4 black balls one by one
Y

zoithout replacement. What is the probability that at least one ball is red?
Re

SOLUTION Let Ri and R2 denote the events of getting a red ball in first and second draws
nd

respectively. Then,
Fi

Required probability = P (R^ u R2)


= 1-P(RiuR2)
= 1-P(RioR_2)
-(i)
= 1-P(Ri)P(R2/Ri)
Now,
P(Rf) = Probability of not getting a red ball in first draw
^ Probability of getting an other colour (white or black) ball in first draw
_6 ^2
~ 9 3
When another colour ball is drawn in first draw there are 5 other cobur (white and black) balls
and 3 red balls, out of which one other colour ball can be drawn in C| ways.
P(R2/Ri)=^
20.70
APPLIED MATHEMATICS-XI

Substituting these values in (i), we obtain


Required probability = 1 -P(R\) P{R2/Ri) = 1 _ 2 ^ 5 _ _7
3 8 12

LXAMIM L 7 To test the quality of electric bulbs produced in a factory, two bulbs are randomlu selected
from a large sample without replacement. If either bulb is defective, the entire lot is rejected. Suppose a
sample of200 bulbs contains 5 defective bulbs. Find the probability that the sample will be rejected.
SOLUIJON Clearly, the sample will be rejected if at least one of the two bulbs is defective.

w
Therefore, if we consider the following events:
A = First bulb is defective, B = Second bulb is defective.
Then,
Required probability = P {A <J B)

e
= 1- P{AuB) = -[ -P(A r^B)

re
o
195 194
= 1~P{A)P{B/A)= 1- = 1 - 3783 _ 197

r
200 199 3980 ~ ^80

F
— EXERCISE 20.6

oF
ul
1. From a pack of 52 cards, two are drawn one by one witliout replacement Find the
probability that both of them are kings.
: From a pack of 52 cards, 4 are drawn one by one without replacement. Find the probability

sr
that all are aces (or, kings). ^

ko
3. Find the chance of drawing 2 white balls in succession from a bag containing 5 red and 7
white balls, the ball first drawn not being replaced. of
4. A b^ag contains 25 tickets, numbered from 1 to 25. A ticket is drawn and then another ticket
o
Y
IS
drawn without replacement. Find the probability that both tickets will show even
rB

numbers.
eY

5. From a deck of cards, three cards are drawn on by one without replacement. Find the
probability that each time it is a card of spade.
u

^ ot” drawn without replacement from a pack of 52 cards. Find the probability
d
o
ad

(i) both are kings (ii) the first is a king and the second is an ace
(iii) the first is a heart and second is red.
in

7. A bag contains 20 Hekets, numbered from 1 to 20. Two Hekets are drawn without
replacement. What is the probability that the first ticket has an even number and the second
Re

an odd number.
F

8. An urn contains 3 white, 4 red and 5 black balls. Two balls are drawn one by one without
replacement. What is the probability that at least one ball is black?
9. A bag contains 5 white, 7 red and 3 black balls. If three balls are drawn one by one without
replacement, find the probability that none is red.
10. A card is drawn from a well-shuffled deck of 52 cards and then a second card is drawn.
Find the probability that the first card is a heart and the second card is a diamond if the first
card IS not replaced.
11. An urn contains 10 black and 5 white balls. Two balls are drawn from the urn one after the
other without replacement. What is the probability that both drawn balls are black?
12. Three cards are drawn successively, without replacement from a pack of 52 well shuffled
cards. What is the probability that first two cards are kings and third card drawn is an ace?
13. A box of oranges is inspected by examining three randomly selected oranges drawn
without replacement. If all the three oranges are good, the box is approved for sale
20.71
PROBABILITY

otherwise it is rejected. Find the probability that a box containing 15 oranges out of which
12 are good and 3 are bad ones will be approved for sale.
drawn one after the other
14.
A bag contains 4 white, 7 black and 5 red balls. Three balls are are white, black and red
without replacement. Find the probability that the balls drawn
respectively.
ANSWERS

_ 11
1.
1
2.
1
3.Z 4.11 a.

22 50 850
221 270725
15
25 7.1
in) 4
1 8. —
6. (i) (iii) 19 22
221 663 204
2 44 1
13 3 13. — 14. —
10. 11. 12.
9. — 91 24
204 7 5525

w
65
HINTS TO SELECTED PROBLEMS

10. Consider the following events;

Flo
A = Getting heart card in first draw, & ^Getting a diamond card in second draw
15x — = —
Required probability = P (A n B) - P {A) P {B/A) = ^ 51 ~ 204

e
re
11. Let By (/■=!, 2) denote the event of getting a black ball in the (th ^raw.^

F
Required probability = P (6| n B2) ^ P P (62/B^} 15 14 7
ur
r
12. Consider the following events: fo
Ki = Getting a king in first draw, K2 = Getting a king in second draw
A3 = Getting an ace in third draw.
ks
Required probability = P (Kj n n A3)
Yo

4 3 4 2
oo

= P(K|)P(K2/Ki)P(A3/KioX2) =
X X

52 51 50 5525
eB

in ith draw.
13. Let Gy (i =1, 2, 3) denote the event of getting a good orange
Required probability = P(Gj 062^03) 12 11 10 44
ur

= P(Gi)P(G2/Gi)P(G3/GinG2)
— X — X — =
15 14 13 91
ad
Yo

20.11 MORE ON CONDITIONAL PROBABILITY


In section 20.3, we have introduced the concept of conditional probability which has been used
will obtain a formula for finding the
d

in multiplication theorem of probability. In this section, we


Re
in

conditional probability.
If A and 6 are two events associated with a random experiment, then
F

P(AnB)=P{A)P (B/A), if P{A)=^0 [Multiplication theorem]


or. P{AnB) = P{B)P (A/B), if P (B) ^0
P(A/B) = P(A)
P(B)
20.11.1 PROPERTIES OF CONDITIONAL PROBABILITY
stated and proved as
Following are some properties of conditional probability which
are

theorems.
THEOREM 1 Let A and B be two events associated with sample space S, then 0 < P (A/B) <1.
PItOOF We know that
A n 6 c 6
P(AoB)<P(B)
P{AnB) [V P(B)>0]
<1, if P(B) 7^0
P(B)
20.72
APPLIED MATHEMATICS-XI

Also, P (A n B) >0 and P (6) > 0. Therefore, P{AnB) >0


P{B)
Thus, we have 0 < P(AnB)
<1 or, 0<P(A/B)<1.
P{B)
Hence, 0<P{A/B)<1
Q.E.D.
THEOREM 2 If A is nu event associated with the sample space S of a random experiment then
P{S/A) = P(A/A)^1
PRCX:)F We have,

P(S/A) = P(SnA) ^P{A) =1


PiAf~~P{A)
Also, P{A/A) =
P (A n A) P (A)
= 1
P(A) P{A)
Hence, P{S/A)=P{A/A)=1
Q.E.D.

w
THEOREM 3 Let A and B be two events associated with a random experiment and S be the sample space
If C IS an ez’ent such that P (C) 0, then

F lo
/ \

P V {A kjB)/C y =P(A/C) + P(B/C)-P((Anfi)/C)


In pai ticular, if A and B are mutually exclusive events,

e
then

P (A uB)/C =P(A/C) + P{B/C)


Fre
for
I’KOOF We have.
r
P (A uB)/C ' _ P{{A u B) n C)
You
oks

P(C)
eBo

P {A uB)/C _ P {{A nC)\j{BnC)}


P{C)
ad
our

P (A uB)/C P{AnC)+ (BnC)-P{AnC)n(gnC)l


P(C)

P {A ^B)/C ^ P {A r^C) + P {B r^C) - P {A n B r-^C)


dY
Re

P{C)
Fin

P (A u6)/C P(A nC) +


PjBnC) P(AnBnC)
P(C) P(C) P(C)

^B)/Cj=P{A/C) + P(B/C)~p\^{A nS)/C


P {A

If A and B are mutually exclusive events, then P f (A nS)/C =0

P V (A nB)/C) =P(A/C) + P(B/C) Q.E.D.

theorem 4 If A and B are two events associated with a random experiment, then
P(A/B)=1-P{A/B)
PROOF We know that
P{S/B)=1
[See Theorem 2]
PROBABILITY 20.73

P {A uA)/B =1

=> P{A/B) + P{A/B)=1 [●.● A and A are mutually exclusive events]


P{A/B)=1-P{A/B) Q.E.D.

Following examples will illustrate the applications of the above formulae and properties for
conditional probability.

ILLUSTRATIVE EXAMPLES

EXAMPLE 1 If A and B arc two events such that P{A) = 05, P (6) = 0.6 and P (/I vj B) = 0.8, ifnd
P{A/B)andP(B/A).

w
SOLUTION We have, P (4) = 05, P (B) = 0.6 and P{AuB)= 0.8
We know that

P(AuB) = P{A) + P{B)-P{AnB)

o
P{AnB) = PiA) + P{B)-P{A^B)= 05 + 0.6 - 0.8 = 0.3

e
P{AnB) 0.3 1 P(AnB) 0.3 _ 3

re
^ and, P {B/A) =
P{A/B) =
P(B) 0.6

rFl P {A)~ ” 05 ~ 5

F
EXAMPLE 2 If A and B are two events such that P{A) =0.3, P(6) =0.6 and P (B/A) - 0.5, find
P{A/B)andP{A'uB).

r
ou
SOLUTION We have, P (A) = 0.3, P (B) = 0.6 and P (B/A) =05 sfo
P{Ar^B) = P{A)P{B/A) =0.3x05 = 0.15
k
P (/I n B) _ 0.15 _ 1
and. P(A/B) =
0.6 ” 4
oo

P(6)
Y

Thus, we have
eB

P{A) = 0.3, P{B) = 0.6 and P{AnB) = 0.15


P{AKjB)=PiA) + PiB)-P(AnB) = 0.3 + 0.6 - 0.15 = 0.75
r
You

EXAMPLES IfP {not A) = 0.7, P (B) = 0.7 and P^/A) = 0.5, then find P {A/B) and P(Au B).
ad

SOLUTION We have, P (not/I) = 0.7 or. P(/l) = 0.7


1 - P(A) = 0.7 [v P{A)=1-P{A)]
d

P{A) = 0.3
Re
in

P{AnB)
Now, P{B/A) =
P{A)
F

P(Ar^B)
05 =
0.3

P(AnB) = 0.15
P{AnB) 0.15 _ 3
P{A/B) =
P(B) 0.7 14

and, P{A^B) = P{A) + P{B)-P{Ar^B) = 0.3 + 0.7 - 0.15 = 0.85


EXAMPl.E 4 If A and B are tzoo events associated xvith a random experiment such that P {A) = 0.8,
P (B) = 05, P {B/A) = 0.4,find (i) P{AnB) (ii) P {A/B) (iii) P {A ^ B)
SOLUTION (i) We have.
P {B/A) =0.4
P{BnA) P(BnA)
= 0.4 V P{B/A) =
P{A) P{A)
20.74 APPLIED MATHEMATICS-XI

P{ArxB)
= 0.4
0.8
[v P (A) =0.8 (given)]
P(AnB)=0.32
(ii) We know that
P(AnB)
P(A/S) =
P(B)
0.32
=> P(A/B) = = 0,64 [●.● P (A n B) = 0.32 and P (B) = 05]
05

ow
(iii) We know that
P(AkjB)=P(A) + P{B)~P (An B)
P (A u 6) = 0.8 + 05 - 0.32 = 0.98 [●-● P (A) = 0.8, P (B) = 05 and P(AnB)= 0.32]
nXAMl’LES A fair die is rolled. Consider the events A = jl, 3,5], 6 = {2, 3} andC = (2, 3, 4,5}.

e
(i) P(A/B)andP(B/A) (ii) P(A/C)^7urfP(C/A)

re
(iii) P (A u B/C) and P (A n B/C)

Flr
SOLUTION We have,

F
S = {1,2, 3, 4,5, 6], A = {l,3,5f, B = {2,3} and C= {2, 3,4,5}
ou
/i(S) = 6, ;i(A) - 3, II (6) = 2 and ii (C) = 4

l,P(B)=^ = i P(C) = i=|,P(/lnB) = 6'


1 1

sr
P(AnC) = ~
6 2 6 3 6 3 6 3 '

P(BnC) =-=-, P (A n 6 nC)=i6 and P (A uB) =-=-


ko
6 3 6 3
of
(i) P(A/6) = P(A nB) _l/6 _1 and, P (6/A) = P(B nA) _l/6 _ 1
o
P(B) P(A) "l/2~3
Y
erB

P(AnC)_1/3 1
(ii) P(A/C) = and, P (C/A) = P(A nC) _l/3 _ 2
P(C) ~2/3~2 P(A) '1/2" 3
uY

P (A n 6) n C
(iii) P (A n B/C) = P(A nBnC) _ 1/6 _ 1
273 "4
ad

P(C)
do

P(C)

and. P(B/C) = P(BnC) _l/3 _1


in

p(C) “2/3 "2


Re

P(A uB/C) = P(A/C) + (B/C)-P(AnB/C} = i2 + i-i = 14


F

2 4
2 1 1
EXAMPLL6 Three events A, 6 mirf C have probabilities - respectively. Given that
1 1
P (A n C) =-5 mirf P(6 nC) = -,
4
find the values ofP (C/6) imrf P (A nC).
SOLUTION We have,

P(A)=1,P{B)
5'
= ^,P(C)
3
=i,P(A
2
nC) =75 and P (BnC) =74
P(C/B) = P(CnB)_ 1/4 3
P(B)

and.
P(A nC) = P(A uC) = 1 - {P(A) + P(C) -P(A nC)} = l 2 1_1^_ 3
- +
5 2 sJ’io
EXAMPLE?
^{2l)=|,P(B)=^iim/P(AnB)=-,/m(fP{A/B)niirfP(B/A).
8 2 4
20.75
PROBABILITY

SOLUTION We know that


P(AnB) P{AnB)
P(A/B) = and P(B/A) =
B(A)

TliGrcfore, to find P{A/B) and P (B/A), we need the values of P {A nB), P (A) and P (B). So,
let us first, compute these probabilities.
Now, P(A nB) ^ P (A uB)
3
= 1 - P (A u B) = 1 - {P (A) + P (B) - P (A n B)1 - 18 2 4 8

1
p (A) = 1 - P (A) = - and P (B) = l-P(B) = -
8

P(A nB) _ 3/8 P{AnB) _ 3/8 _ 3


P{A/B) = - and P(B/A) =
1/2 4 P(A ) 5/8 5
P(B )

w
EXAMI’LES A die is rolled tzoice and the sum of the numbers appearing on them is observed to be7. What
is the conditional probability that the number 2 has appeared at least once?

F lo
SOLUTION Consider the following events:
A = Getting number 2 at least once; B = Getting 7 as the sum of the numbers on two dice.

ee
We have.

Fr
A i(2,1), (2, 2), (2, 3), (2, 4), (2, 5), (2, 6), (1, 2), (3, 2), (4, 2), (5, 2) (6, 2) [
and. B = 1(2,5),(5,2),(6,1),(1,6),(3,4),(43)} for
When a die is rolled twice, there are 36 elementary events.
ur
P{A) =
11

36
P(B) = - and P{AnB) ^ ^
36 36
s

P{AnB) _ 2/36 _ 1
ook
Yo

So, Required probability = P(A/B) = P(B) “ 6/36 " 3


eB

EXAMl’LE 9 A black and a red die are rolled.


(i) Find the conditional probnbilitij of obtaining a sum greater than 9, given that the black die resulted
our
ad

in a 5.

(ii) Find the conditional probabiliti/ of obtaining the sum 8. given that the red die resulted in a number
less than 4.
Y
Re

SOLUTION Consider the following events:


nd

A = Getting a sum greater than 9, B - Getting 5 on black die


Fi

C = Getting 8 as the sum, D = Getting a number less than 4 on red die.


Clearly,
A = {(4, 6), {6, 4), (5,5), (6, 5), (5, 6), (6, 6)}, S = {(5,1), (5, 2), (5, 3), (5, 4), (5,5), (5, 6)}
C = {(2, 6), (6, 2), (4, 4), (3,5), (5, 3)}
and. D = {(1,1), (2,1), (3,1), (4,1), (5,1), (6,1), (1,2), (2, 2), (3, 2), (4, 2), (5, 2), (6, 2),
(1,3), (2, 3), (3, 3), (4, 3), (5, 3), (6, 3){
Clearly, n(A) = 6, n{B) = 6, n{C) -5, n{D) =18, n{AnB)=2 and n (C nD) = 2.
18
P(A) = ~,P(D) =
36 36
i,P(AnB)=—=
2 36

18
36 6 36 6

and, P (C n D) = — = —
^ ' 36 18
20.76 APPLIED MATHEMATICS-XI

P(Ar^B)
(i) Required probability =P{A/B) =
P{B)~~Y/6~3
(ii) Required probability = P (C/D) = P(CnD)_l/18 1
P{D) ^172^9
EXAMPLEio Two mU\^ersarcsclectcdat rmuiomfroin integers Uhroiighn.Ifthesum iseivn,find the
probuhility that both the numbers are odd.
SOLUTION! Out of integers from 1 to 11, there are 5 even integers and 6 odd integers.
Consider the following events:
A - Both the numbers chosen are odd , B = The sum of the numbers chosen is even
Since the sum of two integers is even if either both

ow
are even or both are odd.

P{A) = 11
- , P{B) = 11 and P {A n B) = 11
C2 ^2 ^2

Required probability == P (A/B) = P(AnB) _ %/ '■^C2 _ ^5 ^3

e
P(B) ^C2+^C2 ^C2+^C2 15 + 10“ 5

re
rFl 11
C,

F
EXAMPLE 11 107o of the bulbs produced in a factory are red colour and 2% are red and defective. If one
bulb is picked at random, determine the probability of its being defective if it is red.

r
ou
SOLUTION Consider the following events;
fo
ks
A = The bulb produced is red, B = The bulb produced is defective,
oo

10 1
It is given that P(A) = and P{A nB) = =
100 10 100 50
Y
B

Required probability = P{B/A) = PjAnB) _ 1/50 1


P{A) “ 1/20 “5
re

EXAMPLE 12 A couple has 2 children. Find the probability that both are boys, if it is known that (i) one of
ou

the children is a boy (ii) the older child is a boy.


Y
ad

SOLUTION Let B, and Gj stand for /**'' child be a boy and girl respectively. Then the sample
space can be expressed as S = {B, B2, G2, B2, G] G2}.
d

Consider the following events:


in
Re

A = Both the children are boys; B = One of the children is a boy; C = The older child is a boy.
F

Then, A={Bj B2I, B = [B^ G2, 62,G^ 62! and C ={Bj B2, B| G2)
A r\ B - {B-j B2I and A nC {B| 62)
(i) Required probability = P(A/B) = P(AnB) _ 1/4 _ 1
P(B) ~ J/l ~ 3
(ii) Required probability = P(A/C) = P{A nC) _ 1/4 _ 1
P(C) ~ 2/4 ~ 2
EXAMPLE 13 Consider a random experiment in which a coin is tossed and if the coin shows head it is
tossed again hut if it shows a tail then a die is tossed. If 8 possible outcomes are eqiialli/ likely, find the
probability that the die shoivs a number greater than 4 if it is known that the ifrst throw of the coin results
in a tail.

SOLUI ION The sample space S associated with the given random experiment isi
S = {(H.H),(H,T),(T,l).(r.2).(r.3Kr,4).(T.5). (7.6)}.
PROBABILITY 20.77

Let A be the event that the die shows a number greater than 4 and B be the event that the first
throw of the coin results in a tail. Then,
/I = {(T,5)(T.6)! and B = |(T, 1), (T, 2), (T, 3), (T, 4), (T, 5), (T, 6)1
P(AnB) _ n (A _ 2 ^ 1
Required probability = P{A/B) = n{B) 6 " 3
P{B)

EXAMPLE 14 A coin is tossed hvice niid the four possible outcomes are assumed to be equally likely, if A
is the event, 'both head and tail have appeared’, and B be the event, 'at most one tail is observed .find
P {A), P (fi), P {A/B) and P (B/A).
SOLUTION Here, S = {HH.HT.TH,TT},/\ and B = {HH, HT,TH].
AnB = {HT,TH}.
n{A) 2 1 m(B) n{AnB) _2_1_
f = i , p (B) = — and, P (A n B) =

w
Now, P{A) = 4 "2
n(S) 4 2 JI(S) 4 H(S)

P(AnB) _ 1/2 9 P(AnB) 1/2 = 1.


P{A/B) =
P(B) 3/4 = ^ and P{B/A) = P(A) 1/2

Flo
EXAMPLE 15 A bag contains 3 red and 4 black balls and another bag has 4 red and 2 black balls. One bag

e
is selected at random and from the selected bag a ball is drawn. Let A be the event that the first bag is

re
selected, B be the event that the second bag is selected and C be the event that the ball drawn is red. Find

F
P{A), P{B), P (C/A) and P (C/B).
ur
SOLUTION There are two bags. Therefore, P{A)=- and P (B) = -

r
Now, P(C/A) = Probability of drawing a red ball when first is selected
fo
ks
= Probability of drawing a red ball from first bag = -
Yo
oo

4 2
and, P(C/B) = Probability of drawing a red ball from second bag = —
B

A coin is tossed, then a die is throiun. Find the probability of obtaining a '6' given that head
re

EXAMPLE 16

came up.
Tlie sample space S associated to the given random experiment is given by
u

SOLUTION
ad

S = {(H,1),(H,2),(H, 3),(H,4),(H,5),(H,6), (T, 1), (T, 2), (T, 3), (T, 4), (T, 5), (T, 6)}
Yo

Consider the following events:


A = Getting head on the coin, B = Getting 6 on the dice.
d
Re
in

Clearly,
A = {(H,1),(H, 2), (H, 3), (H,4), (H,5),(H, 6)}and B = {(H, 6), (T, 6)}
F

P (A n 6) _ 1/12 ^ 1
Required probability = P (B/A) = P(A) 6/12 " 6
EXAMPLE 17 A committee of 4 students is selected at random from a group consisting of 8 boys and 4
girls. Given that there is at least one girl in the committee, calculate the probability that there are exactly 2
girls in the committee.
SOLUTION Consider the following events:
A = There is at least one girl on the committee, B = There are exactly 2 girls on the committee.
We have to find P(6/A).
P(AnB)
Clearly, P(6/A) =
P(A)
Now,
C4 = 1
70 _85
P(A)=1-P(A)=1- 12
C4 495 ~ 99
30.78 MATHEMATICS-Xil

6x28 _ 56
P{AnB) =P(Selecting 2 girls and 2 boys out 8 boys and 4 girls = 12
495 ’'l65

P(B/A) = P(AnB) ^ 56 . 85 _ 168


P{A) " 165 '99 "425
EXAMi’i.i-: 18 Two coins are tossed. What is the probabiUti/ of coming up tzvo heads if it is known that at
least one head comes up.
SOLUTION Consider the following events:
A = Getting at least one head, B = Getting two heads.
Clearly,
A = {HT, TH, HH}, B = {HH} and soAnB = {HH\

ow
P(A) = [v S = [HH, HT,TH,TT\]
4 4

Required probability = P (B/A) = P{Ar\B) _ 1/4 _ 1


P{A) ~ ~ 3

e
EXAMPLE ]‘» An instructor has a test bank consisting of 300 easy True/False questions, 200 difficult

re
rFl
TruelFalse questions, 500 easy multiple choice questions (MCQ) and 400 difficult multiple choice

F
questions. If a questions is selected at random from the test bank, what is the probability that it will be an
easy question given that it is a multiple choice question.

r
SOLUTION Consider the following events:
ou
£ =The question selected is an easy question, fo
ks
D = The question selected is a difficult question
T = The question selected is a True/False question,
oo

M = The question selected is a multiple choice question.


Y

Total number of questions = 300 + 200 + 500 + 400 = 1400


eB

^ 3 500 5
1400 7 1400 7 ^ ^ 1400 14
ur

900 9 500 5
P{EnM) =
ad

1400 14
Yo

Required probability = P (E/M) = PjEnM) _ 5/14 _ 5


P{M) ~9/14~9
d

EXAMPLE 20 A die is throzvn three times. Events A and


Re

B arc defined as follows:


in

A :4 on the third throw, B:6 on the first and 5 on the second throw.
F

Find the probability of A given that B has already occurred.


SOLUTION There are
6x6x6=216 elementary events associated with the random
experiment.
Clearly,
/I = {(1,1, 4), (1, 2, 4), (1, 3, 4), (1,4, 4), (1,5, 4), (1, 6, 4),(2,1, 4), (2, 2, 4), (2, 3, 4),
(2, 4, 4), (2,5, 4), (2, 6, 4), (3,1, 4), (3, 2, 4), (3, 3, 4), (3, 4, 4), (3,5, 4), (3, 6, 4)
(4,1, 4), (4, 2, 4), (4, 3, 4), (4, 4, 4), (4,5, 4), (4, 6, 4), (5,1, 4), (5, 2, 4), (5, 3, 4),
(5, 4, 4), (5,5, 4), (5, 6, 4), (6,1, 4), (6, 2, 4), (6, 3, 4), (6, 4, 4), (6, 5, 4), (6, 6, 4)}
B = {(6,5,1), (6,5, 2), {6,5, 3), (6, 5, 4), (6,5,5), (6,5, 6))
and. AnB = {(6,5, 4)1
We observe that n (/I) = 36, /; (B) = 6 and n{AnB)=l
1
P {AnB) = and P{B)= —
216 216
20.79
PROBABILITY

P{AnB) _ 1/216 _1
Required probability =
P(B) ~ 6/216 6
rxAMi’U-: 21 Three dice nre throzon at the same time. Find the probabilih/ of getting three tivo's if it is
known that the sum of the juimbers on the dice was n six.
SOLUTION Associated to the random experiment of throwing three dice there are

6x6x6 = 216 elementary events.


Consider the following events;
A =Sum of the numbers on the dice is six, B = Getting three twos
We have to find P{6/A).
We observe that
A = {(1, 2, 3), (1, 3, 2), (2, 3,1), (2,1, 3), (3,1, 2), (3, 2,1), (1,1,4), (1, 4,1), (4,1,1), (2, 2, 2)1
andB = {(2, 2, 2)}
10 1
P(A) = ,P{B) = — and P (A nB) =

w
216 216 216
P{AnB) _ 1/216 _ 1

F lo
Hence, Required probability = P(B/A) = P(A) “10/216 10
tXAMi’i.F: 22 In a hostel 60% of the students read Hindi newspaper, 40% read English newspaper and
20 % read both Hindi and English newspapers. A student is selected at random.

e
Fre
(i) Find the probabiliti/ that she reads neither Hindi nor English nezos papers.
(ii) If she reads Hindi newspaper, find the probability that she reads English newspaper.
for
(iii) If she reads English newspaper, find the probability that she reads Hindi newspaper.
SOLUTION Consider the following events:
r
H = Student reads Hindi newspaper, £ = Student reads English newspaper.
You
s

We have,
ook

40 20 _1
|,P(E) =
60
P(H) = = -and P(H n£) =
100 "5
eB

100 100_ 5 _
(i) Required probability = P (H n£)
= P(HuE)
our
ad

= l-P(Hu£)
4 1
3 2_1
= l-{P(H) + P(£)-P(Hn£)} = 1 - 5 5 5 5 5

P(Hn£)_ 1/5 _1
dY
Re

(ii) Required probability = P(E/H) = P (H) 3/5 3


Fin

P(H n£) _l/5 _1


(iii) Required probability = P (H/E) = P(£) “ 2/5 “ 2
I-XAMl’LII 23 An electronic assembly consists of tzoo subsystems say A and B. From previous testing
procedures, thefoUozoing probabilities are assumed to be knozim.
P (A fails) = 0.2, P (B fails alone) = 0.15, P (A azid B fail) = 0.15.
Evaluate the follozving probabilities:
(i) P (A fails/B has failed) (ii) P {A fails alone)
SOLUTION Consider the following events: £ = A fails, F = B fails.
It is given that
P (A fails) = 0.2, P (A and B fails) = O.l^and, P (B fails alone) = 0.15
P (£) = 0.2, P (£ n F) = 0.15 and P (E n F) = 0.15
Now, P(E nF)= 0.15
P(F)-P{Er>F)^ 0.15
P (F) = P (£ n F) + 0.15 = 0.15 + 0.15 = 0.30 [v P(£nF)= 0.15]
20.80
APPLIED MATHEMATICS-XI

(i) P (/I fails/B has failed) = P(£/F) = PjEnF) ^ 0.15 ^ 1


P (F) 0.30 2
(ii) P (A fails alone) = P {£ n F) = P (£) - P (£ n f) = 0.2 - 0.15 = 0.05
EXAMPLt 24 Consider the experiment of tossing a coin. If the coin shows hend toss it again hut if it
shozvs tail then throw a die. Find the conditional probahilitp of the event 'the die shoius a number greater
than 4, given that ‘there is at least one tail'.
SOLUTION The outcomes of the experiment can be represented in the following tree diag ram.

(H, H)

(H,T)

(Ti)
(7,2)
(7,3)

w
(T4)

F lo
(T5)
a, 6)
Fig. 20.8 Outcomes of the random experiment
The sample space S of the experiment is given as

e
Fre
S = {{H, H), {H, T), (T, 1), (T, 2), (T, 3), (T, 4), (T, 5), (T, 6)) for
The probabilities of these elementary events are;
P|(H,H)!=ixi =
fPi(T, l)) = ix-6 = 12 '
1 1
r
2 2
You
oks

^-i=^.Pr.41=ixl=12,
- -

P{{T, 2)=^
eBo

P{{T,5)} = i i-
Tand,P{(7,6)i = ^x- =
1 1 1.
2''6" 6 12
our
ad

Consider the following ev'ents;


A - The die shows a number greater than 4, B = There is at least one tail.
Clearly,
dY
Re

A = {(T, 5), (T, 6)}, B = {(H, T), (T, 1), (T, 2), (T, 3), (T, 4), (T, 5), (T, 6)}
Fin

and. AnB = {(T,5)(T,6)j


(/●/, H)
Head(H)
(H,7)
(TD
(T,2)
(T,3)
Tail{71

Fig. 20.9 Computation of probabilities of the outcomes of the experiement

P (B) = P {(H, T)} + P{T, 1)1 + P {(T, 2)) + P {(T, 3)} + P {(T, 4)} + P {(T, 5)} + P {(T, 6)}
20.81
PROBABILITY

T i l l 1 1 1 3
[See Fig. 20.8]
^ ’ 4 12 12 12 12 12 12 4
1 1
and. P{AnB) = P \{T,5)}+P {{T, 6)} = ;^
12
+—
12
=6
P(AnB) 1/6 4 2
Required probability =P(A/B) = P(B) 3/4 18 9

KllMARK Here, the elenienton/ events are not equally likely. So, we cannot say that
2 P(AnB) 2/8 2
-andsoP(/l/S)=-^^=—--
7
P(B)=-,P(AnB) =
8

EXAMPLE 2fa Consider the experiment of throwing a die, if a multiple of 3 comes up throw the die again

ow
and if any other number comes toss a coin. Find the conditional probability of the event the coin shozvs a
tail', given that 'at least one die shoxvs a 2'.
SOLUTION The sample space of the experiment is given by
S = {(3,1),(3, 2) (3, 3), (3, 4), (3,5), (3, 6), (6,1), (6, 2), (6, 3), (6, 4), (6, 5), (6, 6)

e
(1, H), (1, T), (2, H), (2, T), (4, H), (4, T), (5, H), (5, T)}

re
The probabilities of the elementary events are:

rFl 1

F
36 '

ixl=—,P{(3,5)}=-xl
or
ou
P{(3,4)} =
6 6 36 ' 6 6
2)} = -xi=^,P{(6,3)} = ^xi ^
1
ksf
= ^,P{(6,
1
P ((6, l)!=lx^
6 6 36 6 6 36 6 6 36
1
oo

1 1
P{{6, 4)} = 36
6 6
Y

.P{{l.T))AA- = f,P[(2, H)i = ix-2 = 12 '


B

1 1
P{(l,H)l=-x
6 2 12
re

1 1
P {(2,T)( = ix-
1

6
= 2 12 '
oYu

2 jL,P,,5.T» = ixi = l
ad

1 1
P{(5, =
d

Clearly, the elementary events are not equally likely.


in
Re

Consider the following events:


A = The coin shows a tail, B = At least one die shows a 2.
F

Clearly,
A = {(1, T), (2, T), (4, T), (5, T)}, B = {(3, 2), (6, 2), (2, H), (2, T)\ and, Ar^B = {(2, T)}
P(B)=P1(3,2)}+P{(6,2)}+ = 36^ — +
36
1
12
1
12 9

and.
P(AnS)=P{(2,T)} = i=^ 1

P (A n B) _ ]^2 _ ^ ^
Hence, Required probability =P{A/B) = P(B) 2 " 24'8
9
4
REMARK As the elementary events are not equally likely. Therefore, we cannot say thatP
1 PjAnB) ^1/20 _1
P (A nB) =—and so P (A/B) =
20 P(B) "4/20 4'
20.82
APPLIED MATHEMATICS-XI

EXERCISE 20.7

If P (A) = ^Id , P(S) 13


and P{A nB)=~,
13
imdP{A/B).
2. If A and B are events such that P (^) = 0.6, P(B)= 0.3 and P(AnB)= 0.2, find P (A/B) and
P(B/A).
3. IIA and B are two events such that P (A r> B) = 0.32 and P (B) = OB, find P (A/B).
4. lfP{A)=0A, P (B) = 0.8, P (B/A) = 0.6. Find P (A/B) and P (A u B).
5. If A and B are two events such that
(i) P(A)=l/3,P(B)=l/4andP(A uB)=5/12, find P (A/B) and P (B/A).
(ii) P (A) = —, P (6) = ^ and P (A u B) = ^, find P (A n B), P (A/B), P (S/A)
P (B) andP(A nB) =^, find P (A/B).

ow
(iii) {^) =
13 13

(iv) P(A) =-, P(B) = - and P (A nB) = i, find P(A/B), P(B/A), P{A/B) and P(A/B).
6. If A and S are two events such that 2 P (A) = P (B) = — and P (A/B) = - , find P (A u B).

e
13 5

Fl
re
7, If P (A) = ^ , P (B) = ^ and P (A u B) = ^, find

F
(i) P (A n B) (ii) P(A/B) (iii) P(B/A)
ur
8. A coin is tossed three Hmes. Find P (A/B) in each of the following:
(i) A = Heads on third toss, B = Heads on first two tosses
or
sf
(ii) A = At least two heads, B = At most two heads
(iii) A = At most two tails, B = At least one tail.
k
Yo
oo

9. Two coins are tossed once. Find P (A/B) in each of the following:
(i) A = Tail appears on one coin, B = One coin shows head,
B

(ii) A = No tail appears, B = No head appears.


10. A die is thrown three times. Find P (A/B) and P (B/A), if
e

A = 4 appears on the third toss, B = 6 and 5 appear respectively on first two tosses.
ur

11. Mother, father and son line up at random for a family picture. If A and B are. two
ad

: . events
Yo

given by A = Son on one end, B = Father in the middle, find P (A/B) and P (B/A).
12.
A dice is thrown twdce and the sum of the numbers appearing is observed to be 6. What IS
i_
the conditional probability that the number 4 has appeared at least once?
d
Re

13.
Two dice are thrown. Find the probability that the numbers appeared has the sum 8, if it is
in

known that the second die always exhibits 4.


F

14.
A pair of dice is thrown. Find the probability of getting 7 as the sum, if it is known that the
second die always exhibits an odd number.
15.
A pair of dice is thrown. Find the probability of getting 7 as the sum if it is known that the
second die always exhibits a prime number.
16.
A die is rolled. If the outcome is an odd number, what is the probability that it is prime?
17.
Athepair of dice is thrown. Find the probability of getHng
first die.
the sum 8 or more, if 4 appears on
18.
Find the probability that the sum of the numbers showing on two dice is 8, given that at
least one die does not show five.
19.
Two numbers are selected at random from integers 1 through 9. If the sum is even, find the
probability that both the numbers are odd.
20.
A die is thrown twice and the sum of the numbers appearing is observed to be 8. What is the
conditional probability that the number 5 has appeared at least once?
20.83
PROBABILITY

21 Two dice are thrown and it is known that the first die shows a 6. Find the probability that
the sum of the numbers showiirg on two dice is 7.
22. A pair of dice is thrown. Let £ be the event that the sum is greater than or equal to 10 and f
be the event "5 appears on the first-die". Find P {£/F).If F is the event "5 appears on at least
one die", find P (E/F).
23. The probability that a student selected at random from a class will pass in Mathematics is
4/5, and the probability that he/she passes in Mathematics and Computer Science is l/2.
What is the probability that he/she will pass in Computer Science if it is known that he/she
has passed in Mathematics?
24. The probability that a certain person will buy a shirt is 0.2, the probability that he will ^
trouser is 0.3, and the probability that he will buy a shirt given that he buys a trouser is 0.4.
Find the probability that he will buy both a shirt and a trouser. Find also the probability that
he will buy a trouser given that he buys a shirt.
25. In a school there are 1000 students, out of which 430 are girls. It is known that out of 430,
10% of the girts study in class XII. What is the probability that a student chosen randomly
studies in class XII given that the chosen student is a girl?

w
2ft. Ten cards numbered 1 through 10 are placed in a box, mbced up thoroughly and then one
what is the probability that it is an even number?
F lo
card is drawn randomly. If it is known that the number on the drawn card is more than 3,
27. Assume that each born child is equally likely to be a boy or a girl. If a family has two
children, what is the constitutional probability that both are girls? Given that

e
Fre
(i) the youngest is a girl (ii) at least one is girl. for ANSWERS

4 2 1 4. 0.3,0.96
1.
●> 3. 0.64
3 ' 3
r
9
3 115 11
You

4 4 2 5
oks

2 1 6.
5. (i) T (ii) (iii)
3 ' 2 11 ' 5 ' 3 9 4'2'4'8 26
eBo

1
17 (iii) ^7
7. (0 ^ (ii) - (iii) - 8. (i) (ii) 9. (i) 1 (ii) 0
5 3 2
2 1
1 J-
ad

13.
our

11. 12.
10. 5 6
6 ' 36 2
2 1
1 1 17.
15. 16.
14. - 2
6 6 3
1 1 _3
Re

5 2
3
dY

20. 21. 22. —


18. — 19.
8 5 6 3 'll
25

(i) I (iii) ^
Fin

1 4
5 26. - 27.
23. - 24. 0.12, 0.6 25.
7
10

HINTS TO SELECTED PROBLEMS

6. P (A n B) = P (B) P (A/B) = ^ X ^ ^ 13
5 2 11
P (A u B) = P (A) + P (B) - P (A n B) = -^ 13
7
13
4
26

7. 1
P (A n B) = P (A) -F P (B) - P (A u B) = ^ ^ 11 11

(ii) P(A/B) =
P (A o B) _ 4/11 ^ 4
P(B) 5/11 5

P(B/A) =
P(AnB) _ 4/11 ^ 2
(iii) 6/11 3
P(A)
8. (i) We have, A = {HHH, HTH, THH, TFH}; B = {HHH, HHT}
20.84
APPLIED MATHEMATICS-Xl

P(/l/B)=^
(ii) We have,
4 = {HHH, HTH, THH, HHT}, B = (TTT, T7H, HTT, THT, HHT, THH, HTH}
P{A/B)=^7
(iii) A = {HHH, HTH, THH, HHT, THT, HTT, TTH}
B = {THH, HTH, HHT, TTH, THT, HTT, TTT\
P(A/B)=-
7
9. (i) We have, A = {TH, HTj, B = \HT, TH}
P(A/B)=^1
(ii) We have, A = {HH\, B = {77}

w
P{A/B)=0
10. We have,
^ - 1(1, 7 4), (1, 2, 4),{1, 3, 4),(1, 4, 4), (1,5, 4),(1, 6, 4),(2,1,4), (2, 2, 4), (2, 3 4)

F lo
(2, 4, 4), (2,5, 4), (2, 6, 4), (3,1, 4), (3, 2, 4), (3, 3, 4), (3, 4, 4), (3,5, 4), (3, 6, 4),
(4,1,4), (4, 2, 4), (4, 3,4),(4,4, 4),(4,5,4),(4,6, 4),(5,1,4),(5, 2, 4),(5, 3 4) '

ee
(5, 4, 4), (5,5, 4), (5, 6, 4), (6,1, 4), (6, 2, 4), (6, 3, 4), (6, 4, 4), (6,5, 4), (6,6,4)}

Fr
B =
}(6,5,1), (6,5, 2), (6, 5, 3), (6,5, 4), (6, 5,5), (6,5, 6)}
We observe that»{A)1 = 36, n (B) = 6 and n{AnB)=l
36
P{A n 6) = P(A) = = — and P (B) = — for 1
ur
216 ' 216 6 216 36
P{AnB) 1/36 1 PjAn B) 1/216 1
Hence, P {A/B) = ~,P{B/A) =
s
P(B) 1/6 6
P{A) 1/36 6
ook
Yo

11.
The sample space S is given by S = {MfS, MSP, FSM, FMS, SMF SFM}
Clearly, /I = {MPS, FMS SMF, SFM}, B = {MFS, SFM} and so /I n B = {MFS SFM}
eB

P{A/B) =
P{A) 4/6 2
r

19
. Let A = Getting two odd numbers, B = Getting the sum
ou
ad

as an even number.
P(AnB)
Required probability = P{A/B) = 2 10
Y

25.
P(B)
C^C2 + ^C2)/^C2 ^C2 + ^C2 16
4 = Student chosen randomly studies in class XII, B = Randomly chosen student is a eirl
Re
nd

P(fl)=^and/>MnB)= «
Fi

1000
P (71 n B)
P{A/B)^ = 0.1
P(B)

20.12 INDEPENDENT EVENTS

DEFINITION^ Events are said to be huiepcndent. if the occurrence or non-occurrence of one does not affect
the probabiliti/ of the occurrence or non-occurrence of the other.
Suppose a bag contains 6 white and 3 red balls. Two balls are
drawn from the bag one after the
other. Consider the events
A = Drawing a white ball in first draw; B = Drawing a red ball in second draw.
If the ball drawn m the first draw is not replaced back in the bag, then events A and B are
dependent events because P (6) is increased or decreased according as the first draw results as a
white or a red ball. If the ball drawn in first draw is replaced back in the bag, then A and B are
independent events because P (B) remains ^me whether we get a white ball or a red ball in first
draw i.e. P (B) = P {B/A) and P (B) = P (B/A).
20.85
PROBABILITY

It is evident from the above discussion that if A and B are two independent events associated
with a random experiment, then
P{A/B) = and P{B/A) = P (B) and vice-versa.
THEOREM 1 If Amid B mr independent events associated with a random experiment, then
P{AnB) = P(/l)P(B)
the probability of simultaneous occurrence of two indcpiendent events is etpial to the product of their
probabilities.
[■"'.■'(’'I' By multiplication theorem, we have
P{/\nB) = P{A)P{B/A)
Since A and 6 are independent events, therefore P {B/A) = P (B).
Q.E.D.
Hence, P{AnB) = P (A) P (B).
THEOREM 2 If Ai, Ai, ■■■, Ai i>^depeudent events associated with a random experiment, then
P{A-^r^A2nA^...nA„) = P {A-i) P {A2) P (A,,)

w
; !●' By multiplication theorem, we have

F lo
P (Ai n ^2 n A3 n... n A„) =P (A^) P {A2/A1) P {A3/A1 n A2)...
... P(A„/A^ nA2 n...oA„_i)
Since Aj, A2,..., Ai- l' Ai '‘‘re independent events. Therefore,

e
Fre
P (A2/A;i) = P CA2), P (A3/A1 n A2) =P(A3),..., P(Ai/^i ^^^2 n... n A,-i) (A,,)
Q.E.D.
Hence, P {A-i nA^ n... A„) = P (Aj) P (A2)P (Ar)-
for
PAIRWISE INDEPENDENT EVENTS Let Aj, A2,..., Ai ben roeiils associated to a random expieriment.
These events are said to be pairwise indefiendent, if
r
You

P(A,.nAy) = P{A/) P(Ay} for />;; i,j - l,2,...,n


oks

MUTUALLY INDEPENDENT EVENTS Let A-^ , A2 , ■■■, A,^ be n events associated to a random
eBo

expieriment. These events are said to be miitualh/ independent if the probability of the simultaneous
occurrence of any finite number of them is equal to the piroduct of their sep>arate probabilities.
P(A,nAy) = P {Ai) P (Aj), lor i ^ j; i, j =1, 2,, n
our
ad

i.e.

P(A-nA^nA^.) = P(A)P(A^)P(A^.), ior i^j^k;i,j,k =1,2, n


dY
Re

P(A|nA2...nA,,) = P {A-^) P {A2)... P {A„)


Fin

REMARK 1 //A;|, A2 ... Ai pairwise independent events, then the total number of co?Td/h'ojis/or
their pairwise independence is C2 whereas for their m utual indepiendences there must be
"C2 + "C3+... + = 2’^ -n-1 conditions.
REMARK 2 It follows from the above definitions that mutually indepiendent events are always pairwise
indepiendent but the converse need not be true as illustrated below:
ILLUSTRATION 1 A lot contains 50 defective and 50 non-defective bulbs. Two bulbs are drazvn at
random, one at a time, with repilacement. The events A, B, C are defined as
A : "the first bulb is defective",
B : "the second bidb is non-defectwe",
C : "the two bulbs are both defective or both non-defective."
Determine whether (i) A, B, C are pairwise indepiendent (ii) A. B, C are mutually independent.
20.86
APPLIED MATHEMATICS-XI

SOLUTION We have.
1 1
P(C)=ixl + lxl=l
2 2 2 2 2
P (A n 6) = Probability that the first is defective and the second is non-defective
P (A n B) = i2 X ^
2 \ = P(A)P(B)
P (B n C) = Probability that both the bulbs are non-defective

P(BnC) = I X I = ^ = P(B)P{C)
and. P (A n C) = Probability that both the bulbs are defective
P(AnC) = 1 X i = i = P{A)P(C)

w
Hence, A, 6, C are pairwise independent.
Now, P (A nB r\C) = Probability that the first bulb is defective and the second Ii S
non-defective and the first and second are both defective

Flo
or both non-defective

e
= 0

re
1
and, P{A)P{B)P{C) =

rF
Clearly, P (A n 6 n C) ^ P (A) P (B) P (C). Thus, A, B, C are not mutually independent.
ur
RIIMARK 1
In case of two events only associated to a random experiment,
fo there is no distinction between
fhi'ir mutual independence and p^ainvise independence.
ks
THEOREM 3 If A and B are independent ei>ents associated with a random experiment, then prove that
Yo
oo

(i) A and B are independent events (ii) A and B are independent events
eB

(iii) A and B are also independent events.


SOLUTION Since A and B are independent events. Therefore,
ur

P(AnB) = P(A)P(B) -(i)


ad
Yo

(i) It is evident from the Venn-diagram (Fig. 20.5) that A n B and A n Bare mutually exclusive
events such that (A n B) u (A n 6) = B. Therefore, by addition theorem on probability, we have
P{AnB) + P{A nB) = P (B)
nd
Re

A B s
P(AnB) = P(B)-P{AnB)
Fi

= P(B)-P(A)P(B) [Using (i)]


= P(B)|]-P(A)) AnB Ar\B AnB

= P(B)P(A)
= P(A)P(B)
Thus, P(AnB) = P (A) P (B).
Fig. 20.10
Hence, A and B are independent events,
(ii) It is clear from the Venn-diagram (see Fig. 20.5) that A n B and A n 6 are mutually exclusive
events such that (A n B) u (A n B) = A. So, by addition theorem on probability, we have
P (A r^B) + P {A n B) = P(A)
P{AnB) = P(A)-P(AnB)
20.87
PROBABILITY

= P {A) - P {A) P {B) [Using (i)]


^ P(/\)|l -P(S)1
= P(>\)P(B)
Thus, P{AnB) = P{A)P{B).
Hence, A and 6 are independent events,
(iii) We have to show that A and B are^ndependent CTentsjf A and B are independent events.
For this it is sufficient to show that P(/4nB) = P(/4)P(B).
Now,
P{A nB) = P(d uB)
P{A nB) = 1 - P (A u B) (●.● X n B = d n B]
= l-[P(^) + P(B)-P(.4nB)l [By Addition Theorem]

w
= 1 -[P(.4) + P(B)-P(A)P(B)1 [Using (i)]

= (l-P(/i)|-P(B)ll-P(A)l^

F lo
= {l-P{A)\{l-P{B)] = P{A)P{B)
Q.E.D.
Hence, A and B are independent events.

ee
REM ARK 4 In what follows the term imlepeiuient events will mean mutuall}/ independent events.

Fr
REMARK 2
■ If A and B are independent events associated to a random experiment, then
Probabiliti/ of occurrence of at least one = P (A u 6) for
ur
= P(A) + P(B)-P(AoB)
= P(A) + P(6)-P(A)P(B)
s
= 1 -{1 -P (A) -P (6) + P (A) P {6)1_
ook
Yo

= 1-|1-P(A)1{1-P(B)|= 1-P(A)P(B)
eB

REMARK I. 1/A|, A2,... A„ are independent events associated with a random experiment, then
Probabiliti/ of occurrence of at least one = P (Aj u A2 u... u A„)
= 1 - P (A;i u A2 u... u A„)
r
ou
ad

= 1 - P (Ai n A2 n... n A„)


Y

= 1-P(Ai)P(A2)...P(A„)
INDEPENDENT EXPERIMENTS Two random are independent iffor every pair of events
Re
nd

A and B where A is associated with the first experiment and B with the second experiment, we have
Fi

P(AnB) = P{A)P (B)

ILLUSTRATIVE EXAMPLES

Type I PROBLEMS ON PROVING THE INDEPENDENCE OR DEPENDENCE OF EVENTS


EXAMPLE 1 A coin is tossed thrice and all eight outcomes are equally likely.
E : "The first throw results in head" T: "The last throw results in tail
Prove that events £ and F are independent.
SOLUTION Let $ be the sample space associated with the given random experiment. Then,
S = {HHH,HHT,THH,HTH.TTH,HTT,THT.TTT}, £ = {HHT, HTH, HTT, HHH},
F = {HHT,HTT,THT,TTT{, £nP = {HHT. HTT}
4 1 4 1
n(£)
P(£) = 1 = i, p(f) = - — and P {E r\F) = —
2 8 4
n(S) 8 2 8
20.88
APPLIED MATHEMATICS-XI

1 1 1
Clearly, P (£ n F) = — = — X —
= P(E) P{F).
4 2 2

Hence, E and F are independent events.


liXAMPLE 2 All unbiased die is throum tzvice. Let the event A be 'odd number on the first throw'and B
the event odd number on the second throzv’. Check the independence of events A and B.
SOLUTION Clearly,
Total number of elementary events = 36
An odd number on the first throw means an odd number on first throw and- any number on
second throw. Therefore,
18 1
favourable number of elementary events to event A is 3 x 6 = 18.
P(A)=^

ow
36 2

Similarly, p (B) =~
36
^
2

and, 1
P(AnB) = P (Getting an odd number on both throws) = —
36 4

e
Clearly, P(A r\B) = i-=lx-
1

re
4 2

Fl
- = P(A)P{B)

F
Hence, A and B are independent events.
ur
EXAMPLE 3 Three coins are tossed. Consider the events: E = three heads or three tails, F= At least tzoo

r
heads and G = At most tzoo heads. Of the pairs (E, F), (E, G) and (F, G) zvhich are independent? Which are
fo
dependent?
ks
SOLUTION The sample space S associated with the experiment is given by
Yo
S = {HHFI, HHT, HTH, THH, TTH, THT, HTT, TTT]
oo

Clearly, £ = {HHH, TTT}, F = {HHH, HHT, HTH, THH}


and.
G = {HHT, HTH, THH, HTT, THT, TTH, TTT}
eB

Also,
Er^F = {HHH}, £ n G = (TTT}, F n C = {HHT, HTH, THH}
1
and P(FnG)--
ur

8
Clearly, P (£ n F) = P {£) P (F), but P{EnG)^P{E)P (G) and P {F r^G) ^ P (F) P (G)
ad
Yo

So, £ and F are independents, £ and G are dependent events and F and G are also dependent
events.
d
Re

EXAMPLE 4 A fair coin and an unbiased die are tossed. Let A be the event 'head appears on the coin'and
in

B be the ei'ent 3 on the die. . Check whether A and B are independent event or not.
F

SOLUTION The sample space related to the experiment is given by


S = {{H, 1), (H, 2), (H, 3), (H, 4), {H, 5), {H, 6), (T, 1), (T, 2), (T, 3), (T, 4), (T, 5) (F, 6)}
We have,

A = {(H, 1), (H, 2), (H, 3), (H, 4), (H, 5), (H, 6)}, B = {(H, 3), (F, 3)} and so A n B) = {{H. 3)).
^ 12 ^
(S) 12
=16 and P (/I n S) = 12^
Clearly, P (A n B) = P (A) P (B). Hence, A and B are independent events.
EXAMPLE 5 A die is marked 1,2,3 in red and 4, 5, 6 in green is loosed. Let A be the event 'number is
even and B be the event 'number is red'. Are A and B independent?
SOLUTION We have,
A = {2, 4, 6}, B = jl, 2, 3} and A n B - {2}
J’W=7=~,P(B)=|=land
o 2 6 2
P(/lnB)=l6
20.89
PROBABILITY

Clearly, P (A n 6) * P (A) P (B). So, A and B are not independent events.


1 7 1
EXAMPLE 6 Events A and B are such that P (A) = — p (B) = — and P (not A or not B) = —. State
2 12 4
zvhether A and B are independent?
SOLUTION We have,

P (not A or not B) = —
4

P(A uB)=-4 => P{Xr^B) = i4 ^ l-P(AnB)=-4 => P(AnB)=^4


Thus, we have P (AnB = - and P(A) P(B) =
iZ-=-L
4 2 ^ 12 ~ 24
P (A n B) ^ P (A) P (B).

w
So, A and 6 are not independent events.
EXAMPLE 7 An urn contains four tickets zoith numbers 112,121,211,222 and one ticket is drawn. Let

Flo
Aj(i = l, 2, 3) be the event that thei'^' digitofthenumberon ticket drawn is 1. Discuss the independence

e
of the events A|, A2, A3.

re
SOLUTION We have,

F
p (/\^) = Probability that the first digit of the number on the drawn ticket is 1.
1
ur
r
2
fo
p (/!.,) = Probability that the second digit of the number on the drawn ticket is 1.
P(A,)=f
1
ks
2
Yo

P (A3) = Probability that the third digit of the number on the drawn ticket is 1.
oo

P (A,) = f
1
B

P (A^ n A2) = Probability that first and second digits of the number on the drawn
re

ticket are each equal to 1.


u

1
ad

P(AinA2) = -
Yo

Similarly,

P{A2nAi) = f PiA^r^A^) = i
d
Re
in

and. P {A-^ n A2 n A3) = Probability that all the digits of the number on the drawn
F

ticket are unity


P {A-^ r\ A2 A2,) — 0
We observe that
P (Ai n A2) = P (Ai) P (A2), P (A2 nA3) = P {A2) P (A3), P (A3 nA^) = P (A3) P (Aj)
But, P (Aj n A2 n A3) ^ P (A|) P (A2) P (A3).
Hence, Ap A2 and A3 are pairwise independent but not mutually independent.
EXAMPLES A die is thrown once. If A is the event "the number appearing is a multiple of 3" and B is the
event "the number appearing is even". Are the events A and B independent?
SOLUTION It is given that
1

P(^)=| = i,P(B)=| 2

and. P (A n B) = P (Number appearing is even and a multiple of 3)


= P (Number appearing is 6) = -
20.90 APPLIED MATHEMATICS-XI

Clearly, P{Ar^B) = P{A)xP (B).


Hence, A and B are independent events.
EXAMPLE 9 Xwo dice
are throivn together. Let A be the event "getting 6 on the first die" and B be the
event getting 2 on the second die". Are the events A and B independent?
SOLUTION
The elementary events favorable to A are: (6,1), (6, 2), (6, 3), (6, 4), (6,5), (6, 6)
and that to B are: (1,2), (2,2), (3,2), (4, 2), (5,2), (6,2).
So, there is only one elementary event viz. (6,2) favourable to {A n B).
Pi'^)=-^=^,P(B)=^=^andP{AnB)= ^
36 6 36 6 36

Clearly, P{Ar\B)=~ = ~x — = P(4) P(B).


36 6 6
Hence, A and B are independent events.
EXAMPLE !0 for a loaded die, the probabilities of outcomes are given as under:
P(l) = P{2) =^,P{3) = P(5) =P{6)= — and P(4) = i-10

w
10 10
The die is thrown two times. Let /I and B be the events as defined below

F lo
A= Getting same number each time, B = Getting a total score of 10 or more.
Determine whether or not A and B are independent events.

ee
SOLUTION We have,
4 = 1(1,1), (2, 2),(3, 3), (4, 4), (5,5), (6, 6)1 and, B = {(4, 6), (6, 4), (5,5), (6,5), (5, 6), (6, 6)}

Fr
So,4nB = l(6, 6),(5,5)).
P(A) = P(l, 1) + P(2, 2) + P( 3, 3) + P(4, 4) + P(5,5) + P(6, 6) for
ur
= P(l) P(l) + P(2) P(2) + P( 3) P(3) + P{4) P(4) + P(5) P(5) + P(6) P(6)
_2 2 2 2 1 1 3 3 1 1 1 1 1
— X 1 X i X i X 1 X 1 X ——
s

10 10 10 10 10 10 10 10 10 10 10 10 5
ook
Yo

P(B) = P(4, 6) + P(6, 4) + P(5,5) + P(6,5) + P(5, 6) + P(6, 6)


eB

= P(4) P(6) + P(6) P(4) + P(5) P(5) + P{6) P(5) + P(5) P(6) + P(6) P(6)
_31^131111111ii
X 1 X 1 X 1 X 1 X 1 X =
10 10 10 10 10 10 10 10 10 10 10 10 10
our
ad

and. 1 1 1 1 1
P(A n B) = P(5,5) + P(6, 6) = P(5) P(5) + P(6) P(6) = _ ^ ^ ^
10 10 10 10 50
Clearly, P(4 n B) = P(4) P(6).
Y

Hence, A and B are independent events.


Re
nd

EXAMPLE 11 In the above example, if the die were fair, determine xvhether or not the events A and B are
independent.
Fi

SOLUTION If the die is fair, then


P(A)=436 j,P{B)=—=- and P{AnB)- —
1

6 36 6 36 18
Clearly, P(AoB)=—
18
ixi = P(/l) P(B).
So, events A and B are not independent events.
EX AMPI. E12 In the two dice experiment, ifA is the event ofgetting the sum of the numbers on dice as 11
and B is the event of getting a number other than 5 on the first die, find P {A and B). Are A and B
independent events?
OR

Two dice are tossed. Find whether the following two events A and B are independent:
^ - {(x, y): x + y =11), B- l(.v, y): .r 5), where (x, y) denote a ti/pical sample point.
SOLUTION We have.
Total number of elementary events = 36
20.91
PROBABILITY

Number of elementary events favourable to A = 2


Number of elementary events favourable to 6 = 30

Now,
P {A nB) = P (Getting the sum of the numbers on dice as 11 when 5 does not
occur on first die)
1

36

Clearly, P (/I n 6) = —
1 1
— X —^ = P{A)P{B)
36 18 6

So, A and B are not independent events. ^ ^


EXAMPLt 13 Civeu that the events A and B are such that P (A) , P(A u B) =-and P {B)=p.
Find p, if the}/ are (i) imituaU}/ exclusive, (ii) independent.
SOLUTION (i) HA and B are mutually exclusive events, then

w
P{A<jB)=P{A) + P{B)
3 1

F lo
5 2^
=> =l-i=—
5 2 10

e
(ii) If A, B are independent events, then

Fre
1
P(AnB)=P(A)P(B)=-p for
P(AuB)=-
^ 3
r
P(yl) + P(B)-P(/lnB)=-
You
oks

1 p 3
2^25
eBo

2 5 2
1
our
ad

'’ = 5
1 1 1
EXAMPLE 14 If A and B are? two events such that P (A) = 4 , P (B) - --2 and P (A n B) = - , find
dY
Re

P (not A and not B).


SOLUTION We observe that
Fin

P(AnS)=i=ixl = P(A)P(B)
So, A and B are independent events.
Now,
P (not A and not 6) = P (A n B)_
P (not A and not 6) = P(A) P(B) [●.● A and 6 are independent events!
1 W 1^ 3 1 3
P (not A and not B) = 1-- 1- X

4j [ 2J 4 2 8

Type II BASED UPON THE FORMULA /> i.A - B;- PMl/’UnFOR INDEPENDENT EVENTS
EXAMPLE 15 Events E and F are independent. Find P (f), ifP{F)= 0.35 and P (£ w F) = 0.6.
SOLUTION We have,
P (£ vj F) = P{E) +P{F)-P{En F)
20.92
APPLIED MATHEMATICS-XI

P(EuF) = P{E) + PiF)~P{E)PiF) [●.● £ and F are independent]


P(£uF) = PiE) + P{F)(l-P(E)
0.6 = 0.35+ P(F)(1 -0.35) [Substituting the values of P (£) and P (EuF)]
=>
0.25 = (0.65) P (F)
0.25 5
P(F) =
0.65 13

EXAMPLE 16 If p (/\) = (X4^ p (gj =p, P (A B) = 0.6 ami A and B are given to be independent
events, find the value of p.
SOLUTION Since A and B are independent events. Therefore, P {A B) = P (A) P (B).
We know that

P(AkjB) = PiA) + P(B)-P{Ar> B)

ow
P(AijB)=P(A) + P(B)~P (4) P (B) [■.■P{AnB)=P{A)P{B)]
=>
P(AuB) = P(A) + P(B)[:-P (A)
0.6 = 0.4 + p (1 - 0.4) [●●● P {A) = 0.4, P (/I u B) = 0.6 and P (B) = p]

e
=>
0.2 = 0.6 p => p = 1/3.

re
Fl
rvAMPLE 17 let A and B be two independent events. The probabilih/ of their simultaneous occurrence is

F
1/8 and the probabilih/ that neither occurs is 3/8. Find P (/\) and P (B).
SOLUTION Let P (A) = .v and P (6) = y.
ur
We have,

r
Now,
P{Ar^B)= 1/8 and P{AnB) = 3/8
P{AnB) = 1/8
fo
ks
P{A)P(B) = 1/8 ^ ,vy = 1/8
Yo
...(i)
oo

Since A and B are independent events. Therefore, so are A and B.


Thus, P{A r\B) = —
eB

P{A)P{B) = I 8
ur

=I
ad
Yo

8
1 3
=>
1 -.T-y + .vy = -
8
d

5
Re

x + y-xy =
in

8
1 5
F

=>
8 8 [Using (i)]
3
X -r u = —
4 ...(ii)
Now, (.Y-y)^ = {x + yf -4.vy
(A--y)^ = -^-4x1
16 8
=- 1

16 [Using (j) and (ii)]


1
= ±T
4

1
CASH 1 Whenx-x/ = -
4

In this case, we have


1 and... Y + V = —3 => Y 1
Y-y = —
4 - A

2
and u =
j=>P{A)
4 = 12 nnd P(B) = /4
20.93
PROBABILITY

1
LAbt II When X - y =
4

In this case, we have


1 1 1 1
1 .. x + y = —3 ^ x
X -y = — and —4 and *y = —2 => P (A) = —4 and P (B) - —.2
4 ^ 4
1 1

Hence, P{A)=^ and P (B) = ^ or, P {A) = ^ and P (G) =


1

— 2 - 1
EXAMPLE 18 If A and B are two independent events such that P{A r\B) -^and P{A r\B) =-, then
find P{A) and P(B).

w
SOLUTION Let P{A) - x and P(B) = y. It is given that and B are independent events such
tliat

P(ylnG)=—and P(yln6)=j6

lo
15

P(A) and P(A) P(B)=io

e
re
15

1-P (/1)1p(B) = —and P(A)ll-P (B)|/ = 7h rF


F
, / 15 V
2 1

r
(l-.Y)i/ = —and.v(l-y)=-r
15 6
fo
u
2 1
and, Y-.Vl/ = — ●●●(ii)
ks
y--Tv = 6
15
Yo

Subtracting (i) from (ii), we obtain


oo

1 2 1 1
-Y - 1/ = => -Y -y = — => -Y = y +
...(iii) V
B

● 6 15 30 30

Putting .Y = y + ^ in (i), we obtain


e

30
ur

1 2
ad

y- y + — i/ = —
30 J- 15
Yo

●) 1 1

y-ir- :.V =
15
d

30
Re

9 29 2 „
in

\r 1/+ — = 0
' 30' 15
F

30y“-29y+4 = 0
30y“-24y-5y + 4 = 0
1 4
6y(5y-4)-l(5y-4)=0 (6y-1)(5y-4) = 0 => =

CASE! Wheny=~.
0

Putting y = 6^ in (iii), we obtain .r = 5


4
CASE 11 When y = -■
5
4 5
Putting y = — in (iii), we obtain .y =—.
5 6

Thus, P{A) = -5 and P(B) =-6 or P(A) = -6 and P(B) = ^5


20.94
APPLIED MATHEMATICS-XI

EXAMPLE 19 A mid B are tzvo candidates seeking admission in a college. The probabiliti/ that A is
selected is 0.7 and the probability that exactly one of them is selected is 0.6. Find the probability that B is
selected.

SOLUTION Let £ and f be the events defined by


£ = Candidate A is selected, F = Candidate B is selected
Clearly, £ and F are independent events.
f - - \
It is given that P(£) = 0.7 and P (Er^F)<j{EnF) =0.6
V

Now,

P (£n£)u(£nf) =0.6

ow
P(£) + P(F)-2P(£n£)=0.6
P(£)+ P(f)-2 P(£)P(f)= 0.6 [●.● £ and F are independent events]
0.7 + P(f)-2x0.7xP{f) = 0.6

e
0.7+P(f)-1.4 P(f)=0.6

re
Fl
0.4 P(F)= 0.7-0.6

F
=>
P(F)=]4
ur
r
Hence, the probability that Bis selected is -. fo
4
ks
Type III ON FINDING THE PROBABILITY OF SIMULTANEOUS OCCURRENCE OF INDEPENDENT
Yo
EVENTS
oo

EXAMPLE 20 A bag contains 5 white, 7 red and 4 black balls. Iffour balls are drawn one by one with
replacement, what is the probability that none is white?
eB

SOLUTION Let A, B, C and D denote the events of not getting a white ball in first, second, third
and fourth draw respectively. Since the balls are drawn with replacement. Therefore,
ur

A, B, C and D are independent events such that


ad

P{A) = P{B) = P(C) = P(D)


Yo

There are 16 balls out of which 11 are not white. Therefore, P (/4) =11/16 = P (B) = P (C) = P (D).
d

Required probability = P(AnBr^CnD) = P {A) P (B) P (C)


Re

P (D) =
in

16

EXAMPLE 21 A class consists of 80 students; 25 of them are girls and 55 boys; 10 of them are rich and the
F

remaining poor; 20 of them are fair complexioned. What is the probabiliti/ of selecting a fair complexioned
rich girl?
SOLUTION Consider the following events:
A = Selecting a fair complexioned student; B = Selecting a rich student;
C = Selecting a girl.

Clearly, P (A) = | = i, P (B) = ^ = i and P (C) = /?


25 5
80 16

Required probability = P{Ar\Br^C)


= P{A)P{B)P(C) [●.● A, B,C are independent events]
1 1 5 5
4"'8''l6 512
EXAMPLE 22 A bag contains 3 red and 5 black balls and a second bag contains 6 red and 4 black balls. A
ball is drawfrom each bag. Find the probability that both are (i) red (ii) black.
20.95
PROBABILITY

SOLUTION (i) Let A be the event that a red ball is drawn from first bag and B be the event that a
red ball is drawn from the second bag. Then, A and B are independent events such that
P (A) =-and
8
P(B)=^.
10
6
Required probability = P (AnB) = P{A)P{B) = —
10 40

(ii) Let A and B be the events of drawing a black ball from first and second bag respectively.
Tlien, A and B are independent events such that P (i4) =5/8 and P {B) = 4/10.
5 4 1
Required probability = P {A n B) - P {A) P (B) - — X
8

10 4

I XAMPUI23 A police-innu fires four bullets on a dacoit. The probability that the dacoit will be killed by
one bullet is 0.6. What is the probability that the dacoit is still alive?
:th
bullet. Then,
SOLUTION Let /A y; I = 1, 2, 3, 4 be the event that the dacoit is not killed by the /

w
P (Ay) = 1 - 0.6 = 0.4. If the dacoit is alive after four shots, then non of the four shots hits the
dacoit. As all 4 shots are independent.

F lo
Probability that the dacoit is still alive = P (A^ n A2 o A3 n A4)
= P(Ai)P(A2)P(A3)P(A4)
= (0.4)'* = 0.0256.

ee
Fr
LX.AMFLi: 24 Two dice are thrown. Find the probability of getting an odd number on the ifrst die and a
multiple of 3 on the other. for
SOLUTION Consider the following events:
ur
A = Getting an odd number on first die, B = Getting a multiple of 3 on the second die.
Clearly, A = {1, 3,5} and B = {3,6}
oks
Yo

^ "6"^ ^ ^
o

3
eB

Required probability = P (A nB)


1 1 1

= P (A) P (B) = -2 X —3 = -6 [A and B are independent events]


our
ad

ALITER There are 36 elementary events associated with the given experiment. An odd number
on the first die and a multiple of 3 on the other can be obtained in one of the following 6 ways:
Y
Re

(1, 3), (1,6), (3,1),(3,6),(5,1),(5, 6)


nd

1
Fi

Required probability 36 6

Type IV ON FINDING THE PROBABILITY OF OCCURRENCE OF AT LEAST ONE EVENT FOR


INDEPENDENT EVENTS

EXAMPLE 25 A bag contains 5 white, 7 red and 8 black balls. Four balls are drawn one by one with
replacement, what is the probability that at least one is white?
SOLUTION Let Ay be the event that ball drawn in /th draw is white 1 < / < 4.
Since the balls are drawn one by one with replacement. Therefore, Aj,A2,A3,A4 are
independent events such that
1
/ = 1,2, 3,4.
20 4 '
Required probability = P (Aj u A2 u A3 u A4)
20.96
APPLIED MATHEMATICS-XI

V Aj, ^2, A^,


= 1-P{A-^)P{A2)P{A^)P{A^)
are independent
= 1 -

1 I 1
EXAMPLE 26 A problem in mathematics is given to 3 students zuhose chances of solving it are —
2' 3' 4
What is the probabilih/ that the problem is solved?

w
SOLUTION Let A,B,C be the respective events of solving the problem. Tlien P(A)=-
2'
P(B)=-andP(C)=i.
3 ^4

e
Clearly/I, B, C are independent events and the problem is solved if at least one student solves it.

re
ro
Required probability = P {A kj B kjC)
= 1-P{A)P(B)P(C)

F
IV 1

Ful
= 1 - 1-i i_i 1 - = 1 -- 1 ^ 3
2Jl 3V 4, 4 “ 4
EXAMPLE 27 A call solve 90% of the problems gwen in a book and B can solve 70%. What is the

sr
probabilih/ that at least one of them will solve the problem, selected at random from the book?

ko
o
SOLUTION LetE and F be the events defined as follows:
E = A solves the problem, F = B solves the problem.
of
Clearly, £ and F are independent events such that
o
Y
90
F(E) = =^ and F(F) = 70 ^ 7
erB

100 10 100 10
uY

Required probability = F (£ u F)
= 1 -P(£) P(F) [●.● £ and F independent events]
\ /-
9 7 1
= 1 - 1 - — 1 - = 1 X - = 0.97
o
ad
d

10; 10 10 10

EXAMPLE 28 The odds against A solving a certain problem are 4 to 3 and the odds in favour ofB solving
in

■ same problem are 7 to 5. Find the probabilifij that the problem will be solved.
Re

UTION We are given that the odds against A are 4 to 3.


F

P{A) = ^
3

4+3 7

Uso given that the odds in favour of B are 7 to 5.


7 7
P{B) =
7+5 12

problem will be solved if at least one of them solves the problem. So, we have to find
u B). Since A and B are independent events.
3\r
P{A^B) =1~P(A)P(B) = 1-fl- 7 / 'v
1 -—
12
7 \ 16
21

ice, required probability is 16/21.


EXAMPLE 29 The probability that a teacher will give an un-announced test during any class meeting is
a student is absent tiuice, zvhaf is the probability that he zoill miss at least one test?
20.97
PROBABILITY

;lh
SOLUTION Let Ei be the event that the sUident misses / test (i = 1, 2). Then and £2 are

independent events such that P {E^ “ 5 “ ^ ^^2)-


Required probability = P (E-j u E2)
= 1-P(£i)P(£2) 1 E|, £9 are independent]
1 \ / 9
= 1 - 1 - 1-1
5A 5) 25

HXAMPLK 30 A imchine operates if all of its three components function. The probability that first
componentfails during the year is 0.24, the second componentfails is 0.10 and the third componentfails is
0.05. What is the probability that the machine luillfail during the year?
SOLUTION Consider tlie following events:
A = First component of the machine fails during the year
B = Second component of the machine fails during the year
C = Third component of the machine fails during the year

w
We have,
P (A) = 0.14, P (6) = 0.10 and P (C) = 0.05
Clearly, the machine will fail if at least one of its three
Required probability = P{t1cjBuC)
= 1 - P
= 1 -P(X nB nC)
F lo
u B uQ
components fails during the year.
for F
ree
[v A,B,C are independent events]
= 1-P(A)P(B)P(C)
= 0.14) (1 - 0.10) {1 - 0.05)
Your

= 1 -(0.86) (0.90) (0.95)


oks

/I coin is tossed and a die is thrown, find the probability that the outcome will be a head or
eBo

CXAMPLr:3i
anumber greater than 4, or both.
SOLUTION Let A be the event of getting head in a single toss of a coin and B be the event of
ad

a throw of a die. Then,


our

getting a number greater than 4 in


1
P (A) = iand
2
P (B) = -16 3

Required probability = P (A u 6)
Re
dY

= 1 - P(A)P(B) [●.● A and B are independent events]


nr n 2
Fin

= 1 - 1 - - 1 -
2A 3 3

ALITER In
- the above solution events A and B are subsets
, u of distinct
● sample spaces as^sociated
die is considered as one
with independent experiments. If tossing a coin and throwing a
experiment, then the sample space S associated with It IS
S = l(H,l),(H, 2),(H, 3),(H, 4),(H,5),(H, 6), (T,1),(T, 2), (T, 3), (T, 4), (T, 5), (T, 6))
The elementary events favourable to the event "Getting head number greater than 4 of both
ora

(H, 1), (H, 2), (H, 3), (H, 4), (H, 5), (H, 6), (T, 5), (T, 6).
are

Favourable number of elementary events - 8


Hence, required probability = 8/12 - 2/3
EXAMPLE 32 2« tzvo successivc tliivws of a pair of dice, determine the probability of getting a total of 8
each lime.
SOLUTION Let A denote the event of getting a total of 8 in first throw and B be the event of
getting a total of 8 in second throw. Then,
20.98
APPLIED MATHEMATICS-X!

P(A] = — and P(B) = —


36 36
Required probability = P{A nB)
= P{A)P{B) [●●● /I, B are independent events]
5 5 25
X —

36 36 1296
1
LXAMPLL 33
Probabilities of solving a speciifc problem indepieiideiitly Inj A and B are - and —
3
respcctweli/. If both fry to solve the problem independently, find the probability that
(i) the problem is solved
(ii) exactly one of them solves the problem.
SOLUTION
Let E be the event that the problem1 is solved by A and F be the event that the
1
problem is solved by B. It is given that P (£) - and P(F) = -.

w
2 ^ ^ 3
(i) The problem is solved if at least one of A and B solves the problem. Therefore,

F lo
Required probability = P (£ u F)
= 1-P(£)P(F) Iv B are independent events]

ee
1 2
= 1 -

Fr
2A 3 3
(ii) Required probability = P (£) + P (F) - 2 P (£ n F) for
= P (£) + P (F) - 2 P (£) P (F) = i2 + 1--2xi 1 1
ur
X — =
3 2 3 2
i-.XAMPLE 34 A scientist has to make a decision on each of the two independent events I and II Suppose
s

the probability of error in making decision on event I is 0.02 and that on event II is 0.05. Find the
ook
Yo

probability that the scientist will make the correct decision on


(i) both the events
eB

(ii) only one event


SOLUTION Consider the following events:
~ Scientist will make the correct decision on ev'ent /
r

B = Scientist will make the correct decision on event II


ad
ou

We have,
P{A) = 1 -0.02 = 0.98 and P (B) = 1-0.05 = 0.95
Y

(i) Required probability = P (A n B)


Re
nd

= P{A)P(B) [●.● A and B are independent events]


Fi

= 0.98 X 0.95 = 0.931


(ii) Required probability = Probability of occurrence of exactly one of A and 6
= P(A) + P(B)-2P(AnB)
= P (A) + P (6) - 2P (A) P (B) [■.■ A & B are ndependent events]
= 0.98 + 0.95 - 2 X 0.98 x 0.95 = 0.068
EXAMPLE 35 A town has two fire extinguishing engines functioning independently. The probabilitu of
availability of each engine, when needed, is 0.95. What is the probability that
(i) neither of them is available when needed? (ii) an engine is available tvhen needed?
(iii) exactly one engine is available lolien needed?
^LUTION Let A denote the event that first engine is available when needed and B the event
thatsecondengineisavailable when needed. Then, P (A) = P(6) = 0.95.
(i) Required probability = P {A r\B)
= P(A)P(B) [■●● A, 6 are independent]
20.99
PROBABILITY

= (0.05) X (0.05) = 0.0025


(ii) Required probability = P (A l^B) ^
= 1 -P{A) P(B) [v A, B are independent]
- 1 -(0.05) (0.05) = 0.9975
(iii) Required probability {A) + P (B) - 2P (A n B)
= P{A) + P{B)-2P(A)x P (B) = 0.95 + 0.95 - 2 x 0.95 x 0.95 = 0.095
CXAMI’IX 36 A company has estimated that the probabilities of success for three products introduced m
the market are- and - respectively. Assuming independence, find the probability that
3 5 3
(i) the three products are successful. (ii) none of the products is successful.
SOLUTION Consider the following events:
A = First product is successful, and 6 = Second product is successful,
C = Third product is successful

w
We have,

i3 , P (B) = -5 and P (C) = |3

F lo
P (A) =

(i) Required probability = P (All three products are successful)


= P(AnBnC)
['.● A, B, C are independent events]

e
= P{A)P(B)P{C)

Fre
- 1 -x-= —
" 3 "" 5 "" 3 45
for
(ii) Required probability = P (None of the products is successful)
= P(rAnBnC)
r
You

[●.● A, B, C are independent events]


oks

= P{A)P{B)P{C)
eBo

' 3 5 3 15
EXAMPLE 37 A can hit a target 4 times in 5 shots, B 3 times in 4 shots, and C 2 times in 3 shots.
ad
our

Calcfikie the probability that


(i) A, B, C all may hit (ii) B, C may hit and A may not.
(iii)‘jTiy two of A, B and C will hit the target
(iv) fWJte of them will hit the target.
dY
Re

SOLUTION Consider the following events:


Fin

E = A hits the target, f = B hits the target, and G =C hits the target
We have, P (£) = -p5 / P (P) = t4 P (G) = —J
(i) Required probability = P (A, B, C all may hit)
= P (£ n F n G)
= P(£)P(F)P(G) [v £, F, G are independent events]
=ix-x-=-
" 5 4 "" 3 5
(ii) Required probability = P (B, C may hit and A may not)
= P(E nFnC)
= P (£) P (F) P (G) [●.● £, F, G are independent events]
r 3 2 1
1 X — X —
5 j 4 3 10

(iii) Required probability = P (Any two of A, 6 and C will hit the target)
20.100
APPLIED MATHEMATICS-XI

= P(EnFoG)u(EnFoG)u(£nFnG))
= P(FnFnG} + F(£nFnG) + F(£nFnG)
- F {£) F (F) F (G) + F {£) F (F) F (G) F (G) + F (£) F (F) F (G)
4 3 1 1 3 2 4 1 2 13
“ - X—X — + -X—X — + -X — X —
54354354 3 30
(iv) Required probability = F (None of .4, B and C will hit the target)
1 1 1 1
= F(£ nFnG) = F(£)F(F)F(G) = - X —X — =

5 4 3 60
i-XAiViFLL 3s A combination lock on a suitcase has
3 wheels each labelled with nine digits from 1 to 9. If
an opening combination is a particular sequence of three digits with no repeats, what is the probabilitu of a
person guessing the right combination? '
SOLUTION
Let Aj, / = !, 2, 3 be the event that the digit wheel occupies the correct

w
on

position. Tlicn,
Required probability = P (A-i r\Aj nA^)

F lo
= P{A-i)PiA2/A^)P{A:^/A^nA2) = ix-x-
9 8 7
=
1 1

504
1

ee
Fr
EXERCISE 20.8
1 A com IS tossed thrice and all the eight outcomes are assumed equally likely. In which of the
for
following cases are the following events A and B are independent?
(i) A — the first throw results in head, B = the last throw results in tail
r
(ii) A = the number of heads is odd, B = the number of tails is odd
You
s

(iii) A = the number of heads is two, B = the last throw results in head
ook

2. 1 rove that in throwing a pair of dice, the occurrence of the number 4 on the first die is
eB

independent of the occurrence of 5 on the second die.


V A card is drawn from a pack of 52 cards so that each card is equally likely to be selected. In
which of the following cases are the events A and B independent?
our
ad

(i) A = the card drawn is a king or queen, B = the card drawn is a queen or jack
(li) A = the card drawn is black, B = the card drawn is a king
(iii) B = the card drawn is a spade, B - the card drawn in an ace
dY
Re

4. A com is tossed three times. Let the events A, B and C be defined as follows:
A = first toss is head, B = second toss is head, and C = exactly two heads are tossed in
Fin

a row.

Check the independence of (i) A and B (ii) B and C and (iii) C and A
5. betwoeventssuchthatF{A)=l/4,F(e)=l/3andF(Au£)=l/2,showthat
A and B are independent events.
6. Given two independent events A and B such that F (A) = 0.3 and F (6) = 0 6 Find
(i) F(AnB) (ii) F(A n B) (iii) F (A o 6) (iv) F (A n B)
(v) P(AuB) (vi) F(A/B) (vii) F (B/A)
7. If F (not B) - 0.65, P {A u B)= 0.85, and A and B are independent events, then find F (A).
nS) = 2/15andF(A n B) =1/6, then

9. A and B are two independent events. The probability that A and B occur is 1/6 and the
probability that neither of them occurs is 1/3. Find the probability of occurrence of two
events.

1(1. If A and S are two independentevcntssuch that F(A cj B) = 0.60 and F( A) = 0.2, find F (B).
11, A die is tossed twice. Find the probability of getting a number greater than 3 on each toss.
20.101
PROBABILITY

12. Given the probability that A can solve a problem is 2/3 and the probability that B can solve
the same problem is 3/5. Find the probability that none of the two will be able to solve the
problem.
13. An unbiased die is tossed twice. Find the probability of getting 4,5, or 6 on the first toss and
2, 3 or 4 on the second toss,
-t A bag contains 3 red and 2 black balls. One ball is drawn from it at random. Its colour is
noted and then it is put back in the bag. A second draw is made and the same procedure is
repeated. Find the probability of drawing (i) two red balls, (ii) two black balls, (iii) first red
and second black ball.
15. Three cards are drawn with replacement from well shuffled pack of cards. Find the
probability that the cards drawn are king, queen and jack.
io . An article manufactured by a company consists of two parts X and V. In the process of
manufacture of the partX, 9 out of 100 parts may be defective. Similarly, 5 out of 100 are

ow
likely to be defective in the manufacture of part Y. Calculate the probability that the
assembled product will not be defective.
17. The probability that A hits a target is 1/3 and the probability that B hits it, is 2/5. What is the
probability that the target will be hit, if each one of A and B shoots at the target?

e
Fl
10 . An anti-aircraft gim can take a maximum of 4 shots at an enemy plane moving away from

re
it. The probabilities of hitting the plane at the first, second, third and fourth shot are 0.4,0.3,

F
0.2 and 0.1 respectively. What is the probability that the gun hits the plane?
i” The odds against a certain event are 5 to 2 and the odds in favour of another event,
ur
r
independent to the former are 6 to 5. Find the probability that (i) at least one of the events
will occur, and (ii) none of the events will occur.
fo
ks
20. A die is thrown thrice. Find the probability of getting an odd number at least once.
Yo

Two bails are drawn at random with replacement from a box containing 10 black and 8 red
oo

_ i

balls. Find the probability that (i) both balls are red, (ii) first ball is black and second is
B

red, (iii) one of them is black and other is red.


22. An mm contains 4 red and 7 black balls. Two balls are drawn at random with replacement.
re

Find the probability of getting (i) 2 red balls, (ii) 2 black balls, (in) one red and one
u
ad

black ball.
Yo

3 ,5
— and —
23. The probabilities of two students A and B coming to the school in time are - 7

respectively. Assuming that the events, 'A coming in time' and 'B coming in time' are
d
Re

independent, find the probability of only one of them coming to the school in time. Write at
in

least one advantage of coming to school in time.


F

24. Two dice are thrown together and the total score is noted. The event E, F and G are
"a total 4", a total of 9 or more", and "a total divisible by 5", respectively. Calculate
P(£), P(f) and P(G) and decide which pairs of events, if any, are independent.
25. Let A and Bbe two independent events such that P(A) = p] and P(B) = p2-^GScnbe in words
the events whose probabilities are:
(i) P\Pl (ii) (l-pi)p2 (iii) 1 -(1 -p\) (1 -p2) (iv) Pi+P2=2piP2
ANSWERS

1. (i) 3. (ii) and (iii) 4. (i) independent (ii) dependent

(iii) independent 6. (i) 0.18 (ii) 0.12 (iii) 0.42 (iv) 0.28 (v) 0.72 (vi) 0.3 (vii) 0.6
7. 0.77 8.1or^
6 5
9. P(A) = -, P(6) = i or P( A) = ^, P(B) = ^
3 2 2 a
20.102
APPLIED MATHEMATICS-XI

1 2 1
10. 0.5 11. - 12. 13. -
4 15 3
9 4 6 6
l-l' (i) 25
(ii) ^
25
(iii) ^
25
15. 16. 0.8645
2197

17.^ 18. 0.696 (i) —


52
(ii) zr
25
20. -
7
5 77 77 8
... 16 .... 20 40 16 49 56 26
2!. (i) — (ii) (iii) 22. (i) (iii) 23. —
81 81 81 121 121 121 49

24. P(E) _ —,
12
P(f) = A
18
^ ^
36
^ ]sjq jg independent
25. (i) A and B occur (ii) A does not occur, but B occurs (iii) At least one of A andS occurs
(iv) Exactly one of A and B occurs.

w
HINTS TO SELECTED PROBLEMS
2. Here, n(S) = 36, A = {(4,1), (4,2), (4,3), (4,4), (4,5), (4,6)}
and B = ((1,5), (2,5), (3,5), (4, 5), (5, 5), (6, 5)].

Flo
So, A n fi = {(4, 5)}. Now, show that P {A nB) = P (A) P (B)
1 4 1 1

e
3. (iii) We have, P (A) = — -rP{B) =^ = — and P (A n 6) =

re
52 4 52 13 52

Clearly, P (A n B) = P (A)^ (B). So, Aand 6 are independent events.

rF
12. Required probability = P(A nB)=P{A)P{B)
ur
16. Let, A = Part X is not defective, B = Part V is not defective. fo
Required probability = P (A n B) =P (A) P(6) = —x —
100 100
ks
17. Required probability = P (A u B) =1 -P (A) P (B)
Yo
oo

18. The gun hits the plane, if it hits the plane in at least one shot.
Required probability = 1 - (1 - 0.4) (1 - 0.3) (1 - 0.2) (1 - 0.1)
B

6
19. Let A and B be the events. Then, P (A) = — =-and P(B)= —
re

2+5 7 6+5 11
5 52
(i) Required probability = P (A u B) =1 - P (A) P (6)
u

=1 x -=
ad

11 7 77
Yo

25
(ii) Required probability = P (A n B ) =P (A) P CB) =-x-
11 7 77
20. Let A,-denote the event of getting an odd number in y'*’ throw,/= 1 2 3 Then
d
Re
in

1
y = i, 2, 3.
2'
F

Required probability = P (A| u A2 u A3)


i\r 1V
= 1-P(A)P(A2)P(A3)= 1 1 1 1
2A 2A 2)~8
21. Consider the following events:
Rj = /'●’ ball drawn is red, Bj = /*^ ball drawn is black, where / = 1, 2
16
(i) Required probability = P {R^ r\ R2) = P{R^)P{R2) =
18 "" 18 81
10 20
(ii) Required probability = P{B-^nR2) = P(Bj)P(R2) = —

18
X

18 81
(iii) Required probability = P ((R^ n 83) u (B^ n R2))
= P(Ki 062) + P(B| n R2}
8 10 10 8 40
= P(Ri)P(B2) + P(Bi)P(R2) = ^ ^ ^ _
18 18 18 18 81
20.103
PROBABILITY

20.13 MORE ON THEOREMS OF PROBABILITY


In the previous sections, we have discussed those problems based on addition and
multiplication on theorems which require the use of only one of the two theorems. In this
section, we will discuss problems based upon the use of both the theorems. Following examples
will illustrate the same.

ILLUSTRATIVE EXAMPLES

EXAMPLI- I A bng contains 4 white ami 2 black balls. Another contains 3 white ami 5 black balls. If one
ball is drawn from each bag, find the probabilih/ that
(i) both are white; (ii) both are black (in) one is white and one is black.
SOLUTION Consider the following events:
W1 = Drawing a Whitehall from first bag, W2 = Drawing a white ball from second bag.
B\ = Drawing a black ball from first bag, 67 “ Drawing a black ball from second bag.
Clearly, P(W^)= 4/6, P(Bi)=2/6, P(W2) = 3/S and P(B2)=5/8.

w
(i) P (botla balls are white) = P ((white ball from 1st bag) and (white ball from 2nd bag)]
= P (W| n W2)
= P(Wi)P(W2)
- i 1 = - F lo [●.● Wj and W2 are independent events)]

e
' 6^" 8 4
(ii) P (both balls are black) = P [(black ball from 1st bag) Fre
and (black ball from 2nd bag)]
for
= P(Bi 062)
= P(Sl)P(f^2) [●.● and B2 are independent events]
r
You

2 5 5
oks

— X —

6 8 24
eBo

(hi) P (one white ball and one black ball)


= P [(black from 1st and white from 2nd) or ( white from 1st and black from 2nd)]
ad
our

= P[(BinW2)u(W^nB2)]
= P (B] n W2) + P (Wi n B2) [By addition theorem for mutually exclusive events]
= P (Bi ) P {W2) + P (W;i) P (62) [●●■ Bi and W2; B2 & are pairs of independent events]
Re
dY

2 3 4 5 13
= — X i X — = —
6 8 6 8 24
Fin

EXAMPI.I- 2 A box contains 3 red and 5 bine balls. Two balls are drawn one b}/ one at a time at random
without replacement. Find the probabilih/ of getting 1 red and 1 blue ball.
SOLUTION Consider the following events:
= Getting a red ball in first draw, R2 = Getting a red ball in second draw,
Bi = Getting a blue ball in first draw, 62 = Getting a blue ball in second draw.
Now, P (one red and one blue ball)
= P [(red ball in first draw and blue ball in second draw) or
(blue ball in first draw and red ball in second draw)]
= P[(Rl nB2)vj(B^ nP2)l
= P(Ri nB2) + P(Bi nR2> [Bv
! _ addition Theorem for mutually exclusive events]
. 3 5 5 3 15
= P (Ki) P (Bi/Ri) + P (Pi) P (P2/P1) = 8 7 8 7
— X — + — X — = —
28

EXAMPLIIS Two cards are drawn from a well shuffled pack of 52 cards without replacement. Wlmt is the
probabilih/ that one is a red queen and the other is a king of black colour?
20.104
APPLIED MATHEMATICS-XI

SOLUTION Consider the following events :


Rj = Getting a red queen in draw ; f = 1,2.
Kj = Getting a black king in draw ; / = 1,2
(
Required probability = P K (R^ n K2) <J{K^ n R2)
= P {Rl n K2) + P (Ki n R2)
= P (Ri) P {K2/R1) + P (Ri) P (R2/Ki)
1
X
1
+
2^1 2^i [2 2\ 2 2 1 2
52^Cl ^ 51 Cc-i
— X — + X
52 51
C C 52 51 152 51
'“1 663

EXAMPLE!
Tteo cards are drawn xuithout replacement from a well shuffled pack of 52 cards. Find the
probabilitp that one is a spade and other is a queen of red colour.

w
SOLUTION Consider the following events:
Sj ~ Getting a spade card indraw;/=1, 2
Qi = Getting a red queen indraw;/= 1, 2
Required probability = P f (Si n Q2) u (Qi n S2)
F lo
ee
= P(SinQ2)+P(QinS2)

Fr
IBy addition Theoreml
= ^(Si)P(Q2/S,)-rP(Qi)P(S2/Qi)
13
Cl 2^^1 13^ for
i = 2fH.A
X
1 1
+
ur
X
52 51 52
C1 C1 C1
51
C1 152 51 51

EXAMI LE 5 A bag contains 5 white and 3 black balls. Four balls are successively drawn out without
s
ook

replacement. What is the probabilih/ that they are alternately of different colours? '
Yo

SOLUTION Let Wj denote the event of drawing a white ball in /^‘' drawn and R denote the event
eB

of drawing a black ball in draw, where / = 1, 2, 3, 4.


Required probability = P [(Wi n 82 nW^ n B^) nW2 n W4)]
r

= P{W^nB2n W3 n B4) + P (Ri 0^30 63 n W4) (By addition Theorem]


ou
ad

= P fWi) P (B2/W1) P (W3/W1 n B2) P (B4/W1 n 62 n W3)


Y

P (Si) P (W2/S1) P {63/Bi n W2) P (W4/61 n W2 n 63)


+

[By Multiplication Theorem]


Re
nd

5 3 42 3524 1
-X-X-X-+-X-X X —
8 7 6 5
Fi

8 7 6 5 7

EXAMPLE 6 Cards are numbered 1 to 25. Tu’o cards are drawn one after the other. Find the probability
that the number on one card is multiple of 7and on the other it is a multifile of 11.
SOLUTION Two cards can be drawn in the following mutually exclusive ways:
(i) First card bears a multiple of 7 and second bears a multiple of 11
(ii) First card bears a multiple of 11 and second bears a multiple of 7.
Thus, if we define the following events;
A
1 - First card drawn bears a multiple of 7, A2 = Second card drawn bears a multiple of 7,
B
] - First card drawn bears a multiple of 1 i, 63 = Second card drawn bears a multiple of 11.
Then,
Required probability = P [(A-^ n Bn) u n A^)]
= P (Ai B2) + P {B^ n A2) [By addition Tlieorem]
= P (4i) P (Bn/A-i) + P (Bj) p (A2/B1)
20.105
PROBABILITY

Between 1 and 25, there are three multiples of 7 viz. 7,14, 21 and 2 multiples of 11 viz. 11, 22.
P (Ai) = ^ /MS2M1) =
25'
^('^2/Si) = ^ 24

Substituting these values in (i), we obtain


3 2 2 3 1

Required probability = ^^^ + ^^^ = 50


EXAMPLE? contains 4 red and 5 black balls and bag B contains 3 red and 1 black balls. One ball is
drmvn from bag A and two from bag B. Find the probabilih/ that out of 3 balls drauuh two are black and one
is red.

SOLUTION Two black and one red ball can be drawn from two bags in two mutually exclusive
ways:
(I) Drawing one black ball from bag A and two balls from bag B out of which one is black and
other is red

w
(II) Drawing one red ball from bag A two black balls from bag B.
Thus, if we define the following events:

F lo
= Drawing a black ball from bag A, £2 = Drawing one red and one black ball from bag B
£3 = Drawing one red ball from bag A, £4 = Drawing two black balls from bag B.

ee
7 ^2
p 7

P(£i)=^,P(£2) = = ^,P(E3)=fP(h) =
1
Then,

Fr
10 10 15
C, C2
Now, P (Two black balls and one red ball)
= P 1(1 black from bag A and one red and one black from bag B) or
for
ur
(one red from bag A and 2 black from bag B)]
= P[(Eio£2)u(£3n£4)]
s
ook

[By addition Theorem]


Yo

= P(£in£2) + P(£3n£4)
= P(£i)P(£.) + P(£3)f’(£4) [By mu Itiplication Theorem for independent events]
eB

5 X 21 .1—4 X —
— 7 ^ ^
9 45 9 15 135 15
EXAMPLE 8 A bag contains 5 red marbles and 3 black marbels. Three marbels are draum one bp one
r
ou
ad

without replacement. What is the probabilihj that at least one of the three marbles drawn be bloack, if the
first marbel is red?
Y

SOLUTION For / =1, 2, 3, let R,- denote the event "Getting a red marbel in i draw" and By
denote the event "Getting a black marbel in draw". Then,
Re
nd

Required probability = P |(Ri n R2 n 63) u(R^nB2nR3)u(RinB2 063)1


Fi

= P(R| n R2 ^ B3) + P (Ri n B2 nR3) + P (Ri oB2 n B3)


= P(R|) P(R2/Ri) P(B3/Ri oR2) + P(Rt) P(B2/Ri) P(R3/Ri ^B2)
+ P(R|)P(B2/Ri) PfBj/RioB,)
5 4 3 5 3 4 5 X —3 2 150 _ 25
"8''7''6''8''7%’'8 7 6 316 56
X — =

ALITBR Consider the following events;


A = Getting a red marbel in first draw
B = Getting at least one black marbel in the last two draws
...(i)
Required probability = P {Ar\B) = P(A) P(B/A) = P(A) (1 -P(B/A)1
Now,

P(A)=-
8

and, P(B/A) = P (Getting no black marbel in second and third draws when a red marbel has
already been drawn in first draw)
20.106
APPLIED MATHEMATICS-XI

= P (Getting red marbels in second and third draw from the bag when a red
marbel has already been drawn in first draw)
= i 1-1
7 6 "7
2
Substituting P{A) = - and P{B/A) = - in (i), we obtain
5 f 25
Required probability = - 1
8 \ 7 36

vwMPUi 4 Tln-ee groups of children contain 3 girls and 1 hoij; 2 girls and 2 boi/s; 1 girl and 3 Im/s
respectivehj. One child is selected at random from each group. Find the chance that the three selected
comprise one girl and 2 boi/s.
SOLUTION One girl and 2 boys can be selected in the following mutually exclusive ways:
Group 1 Group 2 Group 3

w
(I) Girl
Boy Boy
(11) Boy Girl Boy

F lo
(III) Boy Boy Girl

Thus, if we define Gj, G2, G3 as the events of selecting a girl from first, second and third group
respectively and 82, 63 as the events of selecting a boy from first, second and third group

ee
respectively. Then Bj, 62, 63, G^, G2, G3 are independent events such that

Fr
P{G,) = P(G2) = P{G,) =
t, P(B,)=\. P(B,) =|, P(B3) =1
for
Required probability = P (Selecting 1 girl and 2 boys)
ur
= (I or II or III)
s

- P(IuIIuIII)
ook
Yo

= P[(Gi nS2 ^63) u(6i nC2 nB3) ufBj 063 ^03)]


eB

= P (Gi n 82 n 63) + P (6^ n G2 n 63) + P (B^ n 62 n G3)


= P (G|) P (82) P (83) + P (8^) P (G2) P (83) + P (Si) P (B2) P(G3)
our

13
ad

444444444 32 32 32 32
EXAMPLE 10
The probabilities of A, B and C solving a problem are 1/3,2/7 and 3/8 respective!}/. If all the
Y

three try to solve the problem simultaneously, find the probability that exactly one of them can solve it.
Re

SOLUTION Let £i,£2and£3 be the events that the problem is solved by A, B and C
nd

respectively. Then,
Fi

P(£,) = i3' 8(82) = ~


7
and £(£3) = 8

Exactly one of A, B and C can solve the problem in the following mutually exclusive ways:
(I) A solves but 8 and C do not solve i.e. £j n £3 n £3
(II) 8 solves but/I and C do not solve i.e.Ej ^£3 083
(III) C solves but A and 8 do not solve i.e. n E-, n £3
Required probability = P (I or II or III)
= P[(£i 082 n £3) u(£i nE2 n£3)u(8i 083 083]
= P (El n £2^n £3)^+ P (£1 n £3 n £3) + P (Ei n 83 n £3)
= P (£1) P (£3) P (£3) + P (£1) P (£2) P (£3) + p (£1) p {£2) p (£3)
1 2 V 3^ 1 V2V 3 1 \f \ /
1 1 _ - + 1 1 1-2 3
3JI7
+ 1 --
3 7 / \ J V / V 3 7 8
/ \ / \
20.107
PROBABILITY

155225 253 25 5 5 25
- _x—X- + — X—X- + —X—X — +

3 7 8 3 7 3 7 8 168 42 28 56
EXAMIM.L II Three critics review a book. Odds in favour of the book are 5:2,4:3 and 3; 4 respectively
for three critics. Find the probability that the majority are in favour of the book.

rw
SOLUTION Let A, B and C denote the events that the book will be reviewed favourably by the
first, the second and the third critic respectively. Then A, B, C are independent events and we
are given that 3

^™dP(C) = ^4 ^
4

4+3 7

e
The book will be favourably reviewed by the majority of the reviewers if at least two (out of

r
three) review it favourably. This happens in any one of the following miitually exclusive ways:

luo
(I) 1st favours, 2nd favours and third does not favour i.e. n 6 n C

F
(II) 1st favours, 2nd does not favour and third favours i.e. ^ n B n C
(III) 1st does not favour, 2nd favours and third favours i.e. A nBnC

oF
(IV) 1 St favours, 2nd favours and third also favours i.e. A n 6 n C

rs
Required probability = P (I u II u 111 u IV)
= P (A n B nC) u(/\ n B _r\C)^(A nBnQ'^iAnBnC)

ok
= P {A nBnC) + P (An B nC) + P (A nBnC) + P {A nBnC)
[■.■ Four events are mutually exclusive]

fo
= P (A) P (B) P (C) + P (A) P (B) P (C) + P (A) P (B) P (C) + P (A) P (6) P (C)
[●.● A, B and C are independent]
o
Y
80 + 45 + 24 + 60 209
Y
544533243543
-X-+-X-X-+-X-X-+-X-X-
rB

343 343
777777777777
EXAMPLE E2 A, B and C shot to hit a target. If A hits the target 4 times in 5 trials; B hits it 3 times in 4
trials and C hits 2 times in 3 trials; what is the probability that the target is hit by a least 2 persons?
ue

SOLUTION Let Ej, £2 and £3 be the events that A hits the target, B hits the target and C hits the
d

target respectively. Then, £i, £3, £3 are independent events such tliat
P(£l) £(£2) = I and £(£3) =
no
ad

The target is hit by at least two persoiis in the following mutually exclusive ways:
i

(I) A hits, B hits and C does not hit i.e. n £2 n £3


Re
F

(II) A hits, B does not hit and C hits i.e. £3


(III) A does not hit, B hits and C hits i.e. £i n £2 n £3
(IV) A hits, B hits and C hits i.e. n £2 n £3
Required probability
= P (I u II w III u IV)
= P [(£1 n £2 n £3) u (£1 n £2 n
£3) u(Ei n£2 nE3)vj(£i ^£2 n£3)]
= P (El n£2 n£3) + P(£i n£2 ^£3) + P (Ei n£2 nE^) + P (£1 n£2 ^£3)
[●.● Four events are mutually exclusive]
= P (£1) P (£2) P (£3) + P (Ej) P (£2) P (^3) + P (^1) ^ ^^2) P (^3) + P P(P2) P (^3)
[v El, £2 and £3 are independent]
32432 1^_2_^1 + — 2 5
543543543543 5 15 10 5 6
20.108
APPLIED MATHEMATICS-XI

● \ AMM L13 A speaks truth in 60% of the cases and B in 90% of the cases. In what percentage of cases
arc the}/ like!}/ to (i) contradict each other in stating the same fact? (ii) agree in stating the same fact?
SOLUTION Let£be the event thatA speaks truth and £be the event that Bspeaks truth. Then, E
and F are independent events such that
60 3 . 90 9
P(£) = - and P (F) - —
100 5 ' ' 100 10

(i) A and B will contradict each other in narrating the same fact in the following mutually
exclusive ways:
(1) speaks truth and B tells a lie i.e. £ n F (II) /I tells a lie and B speaks truth i.e. £ n F
P {A and B contradict each other)

ow
= P (I or II) ^ P (I w 10
= P[(EnF)u(E nF)]
= P{EnF) + P(E n F) [●.● £ n F and £ n F are mutually exclusive]
= P(£)P(F) + P(£)P(F) [●.● E and F are in dependent]

e
3 f S'! 9 3 1 2 9
= — X 1 + 1 - 42

re
X = — X
5 I 10 10 5 10 s'" 10 100
5

rFl
F
Hence, in 42'^ cases A and B are likely to contradict each other,
(ii) ^ and B will agree in stating the same fact in the following mutually exclusive ways:

r
(I) A and B both speak truth (II) A and B both tell a
ou
lie.

P (A and B agree) = P ((£ n F) u (£ n F))


fo
ks
= P(£ n F) + P {£ n F)
oo

2 1 29 58
= P(£)F(F) + P(£)P(F) = +
5 10 5 10 50 100
Y
eB

Hence, A and B will agree in 58% cases.


EXAMPLE 14 The odds against a husband who is 45 i/ears old, living till he is 70 are 1; 5 and the odds
r

against his wife who is now 36, living till she is 61 are 5 :3. Find the probabiliti/ that
ou

(i) the couple will be alive 25 i/ears hence,


ad
Y

(ii) exactli/ one of them will be alive 25 years hence,


(iii) none of them zvill be alive 25 years hence,
d

(iv) at least one of them will be alive 25 years he nee.


Re
in

SOLUTION Let A be the event that the husband will be alive 25 years hence and B be the event
that the wife will be alive 25 years hence. Then, A and B are independent events such that
F

7+5
-
12
and P(B) = 5-^+ 3
=3,
8

(i) P (couple will be alive 25 years hence) = P {A and S) = P (/I n B)


= P (/I) P (B) [■.* A and 6 are independent events]
~ 12 8 '
(ii) Exactly one of them will be alive 25 years hence in two mutually exclusive ways:
(I) Husband will be alive 25 years hence and wife will not i.e. A nB
(II) Wife will be alive 25 years hence and husband will not i.e. A nB
P (Exactly one will be alive 25 years hence)
= P (lor ll)_
= P[{AnB)Kj{A nB)]
20.109
PROBABILITY

= P{AnB) +P{A nB) [●.● A nB and A nB are mutually exclusive]


= P (A) P (B) + P {A) P {B) [v A, 6 are independent events]
5 7 3 46 23
5

12
X
V
1 'Ufi Alx-
J\2) 8
= —
\12
X

12
X
96 48

(iii) P (None of them wil] be alive 25 years hence) = P (A n B)


A, B are independent
= P{A)P{B) events

7 5 35
1 —5 Wi
X 1 3
12 8 12 8 96

ow
(iv) P (At least one of them will be alive 25 years hence) = 1 - P (A) P (B)
35 61
= 1 - fi-AV 1- 3'i 96 96
12 / \

EXAMl’i .r A bag CQutnins 3 white, 3 black aud 2 red balls. One by one, three balls arc drawn without

e
replacing them. Find the probability that the third ball is red.

re
SOLUTION

rFl
LetOj be the event of drawing a ball other than a red ball in '/th draw and Rj be the

F
event of drawing a red ball in /th draw (1 < / < 3).
A red ball can be drawn in third draw in the following mutually exclusive ways:

r
(I) First draw gives an other colour ball, second draw gives an other colour ball and the third
ou
draw gives a red ball i.e. 0| n O2 ^
fo
ks
(II) First draw gives a red ball, second draw gives other colour ball and the thiid draw gives a
red ball i.e. nO? rA R3
oo

(III) First draw gives an other colour ball, second draw gives a red ball and the third draw gives
Y

a red ball i.e. O-j n R2 o R3


eB

P (Third ball is red)


= P (I or II or III)
r

= P(luIIuIII)
ou

= P[(Oi n02 nRs) u(Ri n02 n R3) u(0] nR2 oR3)]


ad
Y

= P(0^ n02 nR3) + P(Ri n02 nRj) + P(0] oR2 n R3)


[●.● Events are mutually exclusive]
d

= P(0|)P(02/0i)P(R3/0i n 0-y) + P (Ri) P (0-,/Ri) P (R3/R1 nOj)


Re
in

+
P (O,) P (R2/Oi) P (R3/O1 n R2)
6 5 2 2 6 1 6 2 1 5 1 J_ _ I
F

=-X-X-+-X-X-+-X-X-
87 6 87 6 87 6 28 ^ ^ ^ 28 ” 4
EXAMPLE 16 The probability of student A pmssing an examination is 3/7 and of student B passing is 5/7.
Assuming the two events “A passes", "B passes ns independent, ifnd the pmubability of:
(i) Only A passing the examination (ii)
1 Only one of them passing the examination
SOLUTION Consider the following events;
£j = A passes the examination, £3 ~ ^ passes the examination.
We have, P(£]) = I and £(£2) = ^
(i) Required probability = P (£j n £2)
E-[ and £2 are independent
= p(E,)R(£2)
£^ and £2 are also independent
3f 5 6
- 1--
7 7 49
20.110 APPLIED MATHEMATICS'XI

(ii) Required probability


= R[(£|nf2)^(£in£2)]
- P(£in£2) + P(£T_n£2) [●.● £j n £2 and £■] n £2 are mutually exclusive events]
= P (£|) P (£2) + P (Ej) P (Ej) ['.● and £2; £| & ^ are pairs of independent events]
7 7 7)7 '49
EXAMPLE 17 There are three unis A, B and C. Urn A contains 4 white balls and 5 blue balls. Urn B contains

ow
4 white balls and 3 blue balls. Urn C contains 2 white balls and 4 blue balls. One ball is drawn from each of
these urns. What is the probabilihj that out of these three balls draum, two are white balls and one is a blue ball?
SOLUTION Consider the following events;
£2 = Ball drawn from urn A is white, £2 = Ball drawn from urn B is white,

e
£3 = Ball drawn from urn C is white

re
P(£l) = f P(E2) = I and P (£,) = i
1
Then,

Flr
y / 6 3

F
P (El) = Ball drawn from urn/I is blue = 1- P(£i)=l-1=:5
' 9 9
ou
P (£2) = Ball drawn from urn B is blue ==1-P(£2)=1--=-
4

sr
^ 7 7
and. P {£3) = Ball drawn from urn C is blue = 1 - P (£,)
fo
k
=1 -— =—
3 3
oo
Two white balls and one blue ball can be drawn in the following mutually exclusive ways:
(I) White from urnA, white from urn B and blue from urn C i.e. £| n £2 n £3
Y
reB

(11) White from urn A, blue from urn B and white from urn C i.e. £2 n £3 n £3
(III) Blue from urn A, white from urn B and white from urn C i.e. £2 n £2 n £3
uY

Required probability = P (I) + P (II) + P (HI)


= P(IuIIulII)
= P(£2 n£2 nEj) + P(E^nE2 nE3)+P(Ej np2 .nE^)
ad
do

= P (£2) P(£2) £(£3) + P (£|) £(£2) £(£3) + P (£2) £(£2) £(£3)


in

£2, £2/^3 "‘ire independent events]


4 4 2 4 3 1 5 4 1
Re

64
- - X- X — + -X—X — + -X-X — =
97 397 397 3 189
F

EXAMPLE 18 A certain team wins with probability 0.7, loses xvith probability 0.2 and ties with
probability 0.1 the team plays three games. Find the probability that the team wins at least tzoo of the
games, but not lose.
SOLUTION
Let Wj, Lj and D,-; / = 1, 2, 3 denote respectively the events that the team wins, loses
and ties the r" game. Then,
P{Wj) - 0.7, £(£,-) = 0.2 and P(Dy) = 0.1; / =1,2,3
Required probability
= P (Team wins at least two games and does not lose any game)
= [(Wj n W2 n D3) u (Wj n D2 n W3) u (D, n W2 n W3) u {W2 n W2 n W3)]
= P (Wj n W2 0D3) + P (W2 nD2 n W3) + P (Dj n W2 n W3) + P (Wj n W2 n W3)
= P (W2) P(W2) £(03) + PfWj) P(D2) P(W3) + P(D2) P(W2) P(W3)+ P(W2) P(W2) P {W3)
= (0.7) (0.7) X 0.1 + (0.7) X (0.1) X (0.7) + (0.1) x (0.7) x (0.7) + (0.7) (0.7) (0.7)
= (0.049) X 3 + 0.343 = 0.49
20.111
PROBABILITY

EXAMPLE 19 A clerk was asked to mail three report cards to three students. He addresses three envelopes
but imfortunateli/ paid no attention to which report card be put in which envelope. What is the probabilth/
that exacth/ one of the students received his or her own card?
SOLUTION Consider the following events;
A = First report card is put in the correct envelope.
B = Second report card is put in the correct envelope.
C = Third report card is put in the correct envelope.
We have.
1
P{A) = P(B) = P(C) = -.
Required probability
= P [Exactly one of the report cards is put in the correct envelope]
= P{A) + P (B) + P (C) - 2[P n B) + P (6 nC) + P (C n A)] + 3 P(A n B nC)
= 3 P(A) -2[P(^) P(B/A) + P(B)P(C/B) + P(A) P{C/A)] + 3P(A) P(B/A) P{C/AnB)

w
1
= 3x--2
1 1 1 1
—X-+—X—+—X—
1 1 1 ^ 1
+ 3x—X—X—xl =
1 1 1 1^2x1.1
3 3 2 2 2 2
3 2 3 2 3 2

F lo
3

ALII FR Required probability


= P[(/lnBnC)u(AnBnC)u{^nBnC)]

ee
= P(v4nBnC) + (AnBnC) + P(XnBnC)] ^ __

Fr
= P{/4) P{B/A) P (C/A nB) + P{B)P {A/B)P {C/BnA) + P (C) P(A/C)P (B/A nC) for
1
— X
1 , 1 1 1
— xl+ — X—xl+ — X—xl = —
1 1 1 ^ 1

3 2 3 2 3 2 6 2
ur
example; 20 Neelam is taking up subjects Mathematics, Physics and Chemistry. She estimates that het
probabilities of receiving grade A in these courses are 0.2, 0.3 and 0.9 respectively. If the grades can be
oks
Yo

regarded ns independent events, find the probabilities that she receives,


o

(i) AllA's (ii) NoA's (Hi) Exactly two A's


eB

SOLUTION Consider the following events:


£ = Neelam receives grade A in Mathematics, F = Neelam receives grade A in Physics
our
ad

G = Neelam receives grade A in Chemistry


Then, P {£) = 0.2, P (f) = 0.3 and P (G) = 0.9
(i) Required probability = P (E r\F r^G)
Y

= P(£)P(F)P{G) ]●.● E, F, G, are independent events]


Re

= 0.2 X 0.3 X 0.9 = 0.054


nd

(ii) Required probability = P (£ n F nG)


Fi

= P(£)P(F)P(G) [v £, F, G, are independent events]


= 0.8 X 0.7 X 0.1 = 0,056

(iii) Required probability= P [(£ n P n G) u (Ej~' F n G) u (£ n F n G)]


= P{EnF nG) +P(E nF nG) + P(EnF r\G)^
= P (E) P (F) P (G) + P (E) P (F) P (G) + P (£) P (F) P (C)
= 0.2 X 0.3 X 0.1 + 0.8 X 0.3 x 0.9 + 0.2 x 0.7 x 0.9
= 0.006 + 0.216 + 0.126 = 0.348.
EXAMPLE 21 A doctor claims that 60% of the patients he examines are allergic to some type of weed,
What is. the probability that (i) exactly 3 of his next 4 patients are allergic to weeds? (h) none of his
next 4 patients is allergic to weeds?
SOLUTION Consider the following events:
A = First patient is allergic to weeds, B = Second patient is allergic to weeds
C = Third patient is allergic to weeds, D = Fourth patient is allergic to weeds
20.112
APPLIED MATHEMATICS-XI

Clearly, A, B, C, D are independent events such that


P{A) = P (B) = P (C) = P (D) = 60 _ 3
100 " 5
(i) Required probability
= P[(AnBnC nD) u{AnBnC_nD) ^{A r^B nC nD) u{A n B nC nD)]
= P{AnBnC nD) h_P(A nBr\C nD)_+ P(A nB nC nD) + P {A n B nC nD)
= P(A)P (B) P (C) P (D) + P (/I) P (B) P (C) P (D) + P (A) P (B) P (C) P (D)
+ P (A) P (B) P (C) P (D)
33 3 2332332332333 216
= 7>'-^x-rX7 + -x->^-x- + -x-x-x- + ix-x-x - = —

ow
3 5 5 5 5 5 5_ 5_ 5_ 5 _5 5 5 5 5 5 625
(ii) Required probability = P (A d B nC nD)
16
= P(A)P(B)P(C)P(D)= -
V 5 625

e
EXAMPLE 22
Two persons A and B throiv a die alternnteh/ till one of them gets a 'three' and wins the

re
game. Find their respectively probabilities of winning, if A begins.

Flr
SOLUTION We define the following events.

F
E = Person A gets a three, F = Person B gets a three.
Clearly, P {£)= iD P(f)=i6 P (£) =1-i6 =-6 and P (f) =1
ou 6
1 5
6

sr
A wins if he throws a 'three' in 1st or 3rd or 5th ... throws.

His probability of throwing a 'three' in first throw kfo


= P (£) = i6
oo
A will get third throw if he fails in first and B fails in second throw.

Probability of winning of /I in third throw = P (£ n f n £)


Y
reB

= P(E)P(F)P(E)=-x^x- = 5^^ X
1

6 6 6 6j 6
uY

Similarly, we have
Probability of wirming of A in fifth throw = P(£nfn£nPn£)
ad
do

= P(£)P(F)P(£)P(P)P(£)

= (P(£))2(P(f))2p(£)=f^’] 1
in

X —

6 j 6
Re

and so on.
F

Hence, probability of winning of A


= P[£u(£nfn£)u(£n£n£nFn£)u....]
= P(E) + P(E nF nE) + P{E nF nE nF nE) + ...
1 1 1
+ - + - +...
6 6 J 6 6y 6
16 6
2 a
a + ar + ar +... =
1-(5/6)2 n 1 -r

5
Thus, probability of winning of B = 1-Probability of winning of 2l = 1- —
11 11

EXAMPLE 23 A and B throw alternately a pair of dice. A loins if he throws 6 before B throws 7 and 61wins
'
if the throws 7 before A throws 6. Find their respective chance of winning, if A begins.
SOLUTION 6 can be thrown with a pair of dice in the following ways: (1, 5), (5,1), (4, 2), (2, 4),
(3, 3).
20.113
PROBABILITY

So, probability of throwing a '6' =


5 31
and, probability of not throwing a '6' = 1 - — = 00 36

Now, 7 can be thrown with a pair of dice in 6 ways, viz. (1, 6), (6,1), (2,5), (5,2), (4,3), (3,4).
1
So, probability of throwing a 7' = —
36 6
1 5
and.
probability of not throwing a 7' =1 - ^
Let £ and F be two events defined as;
E = throwing a '6' in a single throw of a pair of dice,

ow
and. F = throwing a 7' in a single throw of a pair of dice
31
Then, P(£)=v P(£)=^,P(F) = -6 and F(F)=-6
36' 36

A wins if he throws '6' in 1st or 3rd or 5th ... throws.

e
Probability of A throwing a '6' in first throw = £(£)= —

re
Fl
A will get third throw if he fails in first and B fails in second throw.

F
31 5 5
Probability of A throwing a '6' in third throw P (£ n F n £) = P (£) P (F) P (£) =
ur 36

r
Similarly,
Probability of A throwing a ‘6' in fifth throw = P (E_n F n £ nF n ^
_ fo _ _ _
ks
= P(£)P{F)P(£)P(F)P(£)
Yo

r 31 r5f 5
oo

X - X — and so on
36 6j 36
eB

Hence,
Probability of winning ofA= P(Eu(£ nFn£)u...)
ur

= P(£) + P(EnFn£) + P(EnFn£nFn£) + ...


5 rSl 5^ 5 ^f31 5f 5
ad
Yo

36 [ 36 6) 36 1 36 6 36
5/36 30
d

1-(31/36) X (5/6) 61
Re
in

30 31
Thus, probability of winning of B =1 - —
F

tXAMI’LE24 Three persons A, 6, C throzo a die in succession till one gets n 'six' and loins the game. Find
their respective probabilities of winning, if A begins.
SOLUTION Let £ be the event of 'getting a six' in a single throw of an unbiased die. Then,
1 1 5
P (£) = — and P (E) = 1 - 6 6
6

/I wins if he gets a 'six' in 1 st or 4th or 7th... throw. His probability of getting a 'six' in first throw
isP(£)=7-
D

A will get fourth throw if he fails in Brst, B fails in second and C fails in third throw.
Probability of winning of A in fourth throw ^ 1
= P(£nEn£n£) = P(E)P(£)P(£)P(E)=(||
V 6J
X —

6
20.114
APPLIED MATHEMATICS-XI

Similarly,

Probability of winning of 4 in 7th throw = — x -- and so on.


V 3J 6
Hence,
1 ^5 \3 1 / c \6 1
Probability of winning ofA=—+ — --+ - ●- + ...
6 IbJ 6 \6j 6
1/6 36

l-(5/6)''^ 91
B wins if he gets a 'six' in 2nd throw or 5th throw or 8th throw ...

/5fl
Probability of winning of B = — + - +
6,6 6 / 6 6y 6

low
/S'] 1
6, 6 30
/ c \3 91
1- ^
6

ee
30 25
Hence, probability of winning of C = 1
F +

Fr
91 91 91

EXERCISE20.9
for
ur
1. A bag contains 6 black and 3 white balls. Another bag contains 5 black and 4 white balls. If
one ball is drawn from each bag, find the probability that these two balls are of the same
colour.
k s
Yo

A bag contains 3 red and 5 black balls and a second bag contains 6 red and 4 black balls. A
●>
oo

ball is drawn from each bag. Find the probability that one is red and the other is black.
eB

.5.
Two balls are drawn at random with replacement from a box containing 10 black and 8 red
balls. Find the probability that (i) both the balls are red. (ii) first ball is black and second is
red. (iii) one of them is black and other is red.
r

4.
Two cards are drawn successively without replacement
ou

from a well-shuffled deck of


ad

52 cards. Find the probability of exactly one ace.


Y

■T.
A speaks truth in 75% and B in 80% of the cases. In what percentage of cases are they likely
to contradict each other in narrating the same incident?
nd
Re

6.
Kama! and Monica appeared for an interview for two vacancies. The probability of Kamal's
selection is 1/3 and that of Monika's selection is 1/5. Find the probability that
Fi

(i) both of them will be selected (ii) none of them will be selected
(iii) at least one of them will be selected (iv) only one of them will be selected.
7.
A bag contains 3 white, 4 red and 5 black balls. Two balls are drawn one after the other,
without replacement. What is the probability that one is white and the other is black?
8.
A bag contains 8 red and 6 green balls. Three balls are drawn one after another without
replacement. Find the probability that at least two bails drawn are green.
4.
Arun and Tarun appeared for an interview for two vacancies. The probability of Arun's
selection is 1/4 and that of Tarim's rejection is 2/3. Find the probability that at least one of
them will be selected.

10. A and 6 toss a coin alternately till one of them gets a head and wins the game. If A starts the
game, find the probability that B will win the game.
11. Two cards are drawn from a well shuffled pack of 52 cards, one after another without
replacement. Find the probability that one of these is red card and the other a black card?
20.115
PROBABILITY

12. Tickets are numbered from 1 to 10. Two tickets are drawn one after the other at random.
Find the probability that the number on one of the tickets is a multiple of 5 and on the other
a multiple of 4.
13. In a family, the husband tells a lie in 30% cases and the wife in 35% cases. Find the
probability that both contradict each other on the same fact.
14 A husband and wife appear in an interview for two vacancies for the same post. The
probability of husband's selection is 1/7 and that of wife's selection is 1/5. What is the
probability that
(i) both of them will be selected? (ii) only one of them will be selected?
(iii) none of them will be selected?
15. A bag contains 7 white, 5 black and 4 red balls. Four balls are drawn without replacement.

ow
Find the probability that at least three balls are black.
16. A, fi, and C are independent witness of an event wliich is known to have occurred. A speal«
the truth three times out of four, B four times out of five
and C five times out of six. What is the
probability that the occurrence wiU be reported truthfuUy by majority of three witnesses.
17. A bag contains 4 white balls and 2 black balls. Another contains 3 white balls and 5 black

e
balls. If one ball is drawn from each bag, find the probability that

re
(i) both are white

rFl
(ii) both are black (iii) one is white and one is black
bag contains 4 white, 7 black and 5 red balls. 4 balls are drawn with replacement What is

F
18. A
the probability that at least two are white? ^ i u
Three cards are drawn with replacement from a well shuffled pack of 52 cards. Fmd the

r
19.
ou
probability that the cards are a king, a queen and a jack.
fo , , „
A bag contains 4 red and 5 black baUs, a second bag contains 3 red and 7 black balls One
ks
20.
ball is drawn at random from each bag, find the probability that the (i) balls are of different
colours (ii) balls are of the same colour.
oo

21. A can hit a target 3 times in 6 shots, B: 2 times in 6 shots and C : 4 times in 4 shots. They fix a
Y

volley. What is the probability that at least 2 shots hit?


B

22. Tlie probability of student A passing an examination is 2/9 and of student B passing 5/9
Assuming the two events: ‘A passes', 'B passes' as independent, fmd the probability of: (i)
re

only A passing the examination (ii) only one of them passing the examination.
ou

23 There are three urns A, B, andC. Urn A contains 4 red balls and 3 black bdls. Urn B
Y
ad

■ contains 5 red balls and 4 black balls. Urn C contains 4 red and 4 black balls. One ball is
drawn from each of these urns. What is the probability that 3 balls drawn consist of 2 red
d

balls and a black ball?


the examination. His
24. X is taking up subjects - Mathematics, Physics and Chemistry i . , t'- j i.u
in
Re

in

probabilities of getting grade A in these subjects are 0.2, 0.3 and 0.5 respectively. Find the
F

probability that he gets


(i) Grade A in all subjects (ii) Grade A in no subject
(iii) Grade A in two subjects. ^ . ● cu
25. A and B take turns in throwing two dice, the first to throw 9 being awarded the prize. Show
that their chance of winning are in the ratio 9:8.
26. A, B and C in order toss a coin. The one to throw a head wins. What are their respective
chances of winning assuming that the game may continue indefinitely.
27. Three persons A, B, C throw a die in succession till one gets a 'six' and wms tlie game. Fm
their respective probabilities of winning. -
28 A and B take turns in throwing two dice, the first to throw 10 being awarded the prize,
show that if A has the first throw, their chance of winning are in the ratio 12 . ih
-79 There are 3 red and 5 black balls in bag 'A'; and 2 red and 3 black balls in bag
IS

drawn from bag 'A' and two from bag 'B'. Find the probability that out of the 3balls drawn
one is red and 2 are black.
20.116
APPLIED MATHEMATICS-XI

30.
Fatima and John appear in an interview for two vacancies for the same post. The
probabUity of Fatima's selection is - and that of John's
selection is ^. What is the probability
that (i) both of them will be selected? (ii) only one of them will be selected?
(iii) none of thorn will be selected?
31.
A bag contains 8 marbles of which 3 are blue and 5 are red. One marble is drawn at random
Its colour IS noted and the marble is replaced in the bag. A marble is again drawn from the
bag and its colour is noted. Find the probability that the marble will be
(i) blue followed by red. (ii) blue and red in any order. (iii) of the same colour.
32.
An urn contains 7 red and 4 blue bails. Two balls are drawn at random with replacement
rmd the probability of getting ^
(i) 2 red balls (ii) 2 blue balls (iii) One red and one blue ball.
33.
A card is drawn from a well-shuffled deck of 52 cards. The outcome is noted, the card is

ow
replaced and the deck reshuffled. Another card is then drawn from the deck,
(i) What is the probability that both the cards are of the same suit?
(ii) What is the probability that the first card is an ace and the second card is a red queen’
T "‘“‘If ■ K y°u your friend are among
100 studen s what is the probability that: (i) you both enter the same section? (ii) yon both

e
enter the different sections?

re
Fl
35. In a hockey match, both teams A and B scored same number of goals upto the end of the

F
decide the winner, the refree asked both the captains to throw a die alternately
and decide that the team, whose captain gets a first six, will be declared the winner. If the
ur
captain of team A was asked to start, find their respective

r
probabilities of winning the
match and state whether the decision of the refree was fair or not. * fo
36. A and B throw a pair of dice alternately. A wins the game if he gets a total of 7 and B wins
ks
the game if he gets a total of 10. If A starts the game, then find the probability that B wins.
Yo
oo

_ ANSWERS
14 21
, (i) —
16 (n)
.... —
20 40 32
eB

1. — 2. —
27 40
3- (iii) ^ 4.
81 81 81 221
1 8 7
5. 35%
(iv) I
6. (i) (ii) (iii)
ur

15 15 15
7. A 5 1 1
ad

8. — 9. 10. 11.^
Yo

22 13 2 3 51
4 1
12. — 13. 0.44 2 24
45 14. (i) — (ii) (iii) ^
d

35 7 35
23 107
Re

1 5
in

15. 16. 13
364 17. (i) - (ii) (iii) ^
120 24
67 24
F

6
18.
256
19.
2197
20. (1) ^
90
(ii)
47
21.
2
90 3
8 17
22. (i) — (ii) 43/81 23.
81 42
24. (i) 0.03 (ii) 0.28

(iii) 0.22 26. ill 27. ^ ^ ^ 29.


39 1
1 'I'l 9r 91' 91 30. (i) —
80 35

(ii)^
24
(iii)
35
31. (i) ^
64
(ii)
15
(iii) ^
17
32 32
49 16 56 1 1 17
32. (i) (ii) (iii) 33. (i) - (ii) 16
121 121 121 34. (i) (ii) 4^
4 338 33 33
35. Team A: — Team B:
11' —; The decision was fair as the two probabilities are almost equal.
5
36. —
17
20.117
PROBABILITY

HINTS TO SELECTED PROBLEMS

1. For i =1,2, let us define the following events:


.●til th
Bj = Drawing a black ball from/ bag, Wj = Drawing a white ball from i bag
So, required probability = P ((6] n 62))((W^ n W2))
= P{B^nB2) + P{W-^nW2) = P (B^) P (Bj) + P ^ (^2)
6. Consider the following events : K = Kamal is selected, M = Monica is selected. Then,
P(K) = 1/3, P(M)-1/5.
1 1 1
(i) Required probability = P (fC n M) = P (K) P (M) = 3 5 15
_ _ - — 2 4 8
(ii) Required probability = P (fC nM ) =P (K) P (M) = —x —- —

w
- — 2 4 7
(iii) Required probability = 1 - P (K) P (M)=l- —x —

F lo
(iv) Required probability = P(K nM ) + P {K n M) = P (K) P (M) + P {K) P{M)
9. Required probability =1 - P(y4)P(T)-l 3j"2
1 --

ee
4

Fr
11. Required probability = P [{R and B) or (6 and R)] = P (R B) + P (B n R) 26
26 26 26 26
= P(R)P(B/R) + P(B)P(R/B) = 52 for X
51
h
52
X
51
=
51
ur
.●th draw.
12. Let, Aj = Getting a multiple of 5 in / draw, Bj = Getting a multiple of 4 in
s
ook

where i = 1, 2. .
Yo

Required probability = P [(/\j n B2) P (B-j n A2)\


eB

= P {A^ n B2) + P (R] ■^2)


= P (v4i) P (B2/A1) + P (Bi) P (A2/BO = I
r
ad
ou

20. (i) Required probability = P ((R| n B7) cj (B^ n R2)) = P (Ri ^2) + P (R] ^2)
Y

= PiR^)P{B2) + P{R2)P(Bd= + 9 10
Re
nd

(ii) Required probability = P ((R| n R2)(Ri R2)) = P ^ ^2) + P (R) R2)


= P (Ri) P {R2) + P (Bi) P (R2) = ^9 10^ ^9
Fi

— X — + — X —
10

21. Required probability


= P 1(A n B nC) u(A n B nC) u(A n B nC) u (/I n B nC)]
= P(AnBnC) + P{ylnBnC) + P(AnBnC)+P(/lnBnC)
= P{A) P(R) P(C) + P (A) P(B) P (C) + P (d) P (B) P(C) + P(A) P (B) P (C)
20.14 THE LAW OF TOTAL PROBABILITY
THEOREM (Lazu of Total Probainlih/) Let S be the sample space ami let Bj, E2, B,, he « mutually
exclusive and exhaustive events associated with a random exp^eriment. IfA is any event which occurs with
£-| or £2 or... orE,,, then
P{A) = P (£i) P (A/£i) + P (£2) P (A/£2) + ... + P (R;,) P
20.118
APPLIED MATHEMATICS-XI

Fig. 20.11

I’KOOi-
Since £^,£2, aremutually exclusive and exhaustive events. Therefore,

^ ^^2 '^^3 ■■■ where E^- n£y = (j) for i ^ j.

w
Clearly,
={A n£j) u(A n£2) u(/I n£3)u... ^(A nE„)

F lo
P(A) = P(Ar^E-^) + PiAr^E2)+P{Ar^E2).. P (A n E„) . +
[By addition theorem]
But, P(AnEi) = P (El) P (A/Ei) for i = 1, 2,..., u

ee
Hence, P (A) = P (E^) P (A/E^) + P (E^) P (A/E^) + ... + P (^) P (A/E,,) Q.E.D.

Fr
The law of total probability as stated and proved above says that if an event A can occur in 11
rnutually exclusive ways, then the probability of occurrence of A is the sum of the probabilities
for
of all mutually exclusive ways as shown in the following tree diagram.
ur
£(£,)
s

£1
ook
Yo

£(£2) />.
£, ■/J
%7J
eB

£(£3)
£3
our
ad

0
Y
Re

£(£„-i) .1^5
nd

£ II-1

£(£.,)
Fi

£,
Fig. 20,12

ILLUSTRATIVE EXAMPLES

EXAMPLE 1 A bag coutains 4 red and 3 black balls. A second bag contains 2 red and 4 black balls One
bag IS selected at random. From the selected bag, one ball is drawn. Find the probabilitu that the ball drawn
IS red.

SOLUTION A red ball can be drawn in two mutually exclusive ways.


(I) Selecting bag I and then drawing a red ball from it.
(II) Selecting bag II and then drawing a red ball from it.
Let £|,£2 and a denote the events defined as follows:
20.119
PROBABILITY

P(£i) =
El = Selecting bag 1, P(A/Ej) = 4/7
Bogl
£2 = Selecting bag II, (4K, 3B)
A
A = Drawing a red ball
{2R, 4B)
Since one of the two bags is selected randomly. Bog II P(A/£2) = 2/6

P{Ei) = I and £(£2) =


P(E2) = Vi
Fig. 20.13

Now, P {A/El) = Probability of drawing a red ball when the first bag has been chosen.
4
[●.● First bag contains 4 red and 3 black balls]
7

and, P (A/E2) = Probability of drawing a red ball when the second bag has been selected
2
[●.● Second bag contains 2 red and 4 black balls]

w
6

Using the law of total probability, we have


1 4
= _x- + -x —
1 2

F lo
Required probability = P (A) = P (£|) P (A/£|) + P (£2) ^ (A/£2)
= —-
19

ee
2 7 2 6 42

Fr
iix.AMPLtz Find the probabilitij of dmnung ci onc-nipve coin from 0 purse zuitli two compartments
coins and other contains! fifty-paise coins and
one? of zi’hich contains 3 fiftij-paise coins and 2 one-rupee for
3 one-rupee coins.
SOLUTION A one rupee coin can be drawn in two mutually exclusive ways.
ur
(I) Selecting compartment I and then drawing a rupee coin from it.
s

coin from it.


ook

(II) Selecting compartment II and then drawing a rupee


Yo

Let El, £2 and A be the events defined as follows:


eB

El = tlae first compartment of the purse is chosen,


£2 = the second compartment of the purse is chosen,
A = a rupee coin is drawn from the purse.
our
ad

P(£,) = l/2
Comp-I P(A/E,) = 2/5
Y

3(50P), 2(K)
Re

A
nd
Fi

Comp-II
2(50P), 3(1 ?) P(A/E2) = 3/5
£(£2) = 1/2
Fig. 20.14

Since one of the two compartments is chosen randomly.


P(£i) = 1 = P(£2)
Also,
P (A/El) = Probability drawing a rupee coin given that the first compartment of the
purse is chosen
= - [-.■ First compartment contains 3 fifth paise coins and 2 one rupee coins]
20.120
APPLIED MATHEMATICS-XI

and.
P {A/E2) - Probability of drawing a rupee coin given that the second compartment of the
purse is chosen

= I [*.■ Second compartment contains 2 fifth paise coins and 3 one rupee coins]
By the law of total probability
P (Drawing a one rupee coin) = P{A) = P (£^) p (A/EA + P (EA P (A/EA = lx^ + ix-^=i
2 5 2 5 2
EXAMPLE 3 One bag contains 4 zvhite and 5 black balls. Another bag contains 6 zvhiteand 1 black balls.
A ball is transferredfromfirst bag to the second bag and then a ball is drawn from the second bag. Find the
probabilit}/ that the ball drawn is white.

ow
SOLUTION A white ball can be drawn from the second
bag in two mutually exclusive ways:
(I) By transferring a white ball from first bag to the second bag and then drawing a white ball
from it.

(II) By transferring a black ball from first bag to the second bag and then drawing a white ball
from it.

e
re
£1 = a white ball is transferred from the first bag to the second bag

rFl
£2 = 3 black ball is transferred from the first bag to the second bag

F
A
a white ball is drawn from the second bag
Since the first bag contains 4 white and 5 black balls.

r
ou
n£l) = ^ and P(£2> = I fo
ks
If £1 has already occurred, that is a white ball has already been transferred from first bag to the
oo

second bag, then the second bag contains 7 white and 7 black balls.
So, P{A/EA=^
Y
eB

^ 14
If £2 has already occurred, that is a black ball has been transferred from first bag to the second
r

bag, then the second bag contains 6 white and 8 black balls.
ou
Y

So, P{A/E2) = -
ad

14

Using law of total probability, we obtain


d

P (Getting a white ball) = p (A) = p (£^) p (A/E^) + P (£2) P (A/E2)


in
Re

4.7,5
— X — + — X —
6 58 29
F

9 14 9 14 126 63
EXAMPLE 4 There are two bags. The first bag contains 5 white and 3 black balls and the second bag
contains 3 white and 5 black balls. Two balls are drawn
at random from the first bag and are put into the
second bag without noticing their colours. Then two balls are drawn from the second bag Find the
probabilihj that the balls are white and black.
SOLUTION A white and
a black ball can be drawn from the second bag m
the following
mutually exclusive ways:
(I) By transferring 2 black balls from first bag to the second bag and then drawing a white
and a black ball from it. ^
(II) By transferring 2 white baUs from first bag to the second bag and then drawing a white
and a black ball from it. ^
(III) By transferring one white and one black ball from first bag to the second bag and then
drawing a white and a black ball from it.
20.121
PROBABILITY

LetEi, £2/ £3 and A be the events as defined below:


£^ = Two black balls are drawn from the first bag,
£2 = Two white balls are drawn from the first bag,
£3 = One white and one black ball is drawn from the first bag,
A = Two balls drawn from the second bag are white and black.
P(£,)=3/28 El

£{£3) = 5/14 E.
A

P(E) = 15/28 E,

w
Fig. 20.15

1 _ 15
^C2
X

F lo
We have, P(£]) = = "'"d P(23) = 8 28
8
28C2 ^ ^C2 14 C2

If Ej has already occurred, that is, if two black balls have been transferred from the first bag to

ee
the second bag, then the second bag will contain 3 white and 7 black balls.

Fr
Probability of drawing a white and a black ball from the second bag is 10
C2
for
ur
^Cix^C
P (A/£i) = 10
ks
15
C2
Yo
oo

Similarly, we have 4
SCiK^Ci ^ 5 qx _ _8_
eB

P(A/E2) = 10
C2 = ^ and P (A/£3) = 10
C2
15
r

By the law of total probability, we have


ou
ad

P (A) = P (El) P (A/£i) + P (£2) P i^/p2) + ^ (^3) ^


673
_ 3 7_ ^ 5 15^A
Y

“ 15 "^14''9^28''15 1260
nd

EXAMPLE 5 A bag contains 6 red and 5 blue balls and another bag contains 5 red and 8 blue balls. A ball
Re

drawn from the first bag and ivithoiit noticing its colour is put in the second bag. A ball is then drawn
Fi

IS

from the second bag. Find the probabilihj that the ball drawn is blue in colour.
SOLUTION A blue colour ball can be drawn from the second bag in the following mutually
exclusive ways:
(I) By transferring a blue ball from first bag to the second bag and then drawing a blue ball
from the second bag. P(£i) = 5/ll
(II) By transferring a red ball from first bag to the P(A/E,) = 9/14
second bag and then drawing a blue ball from
the second bag. A

Let £1, £2 and A be the events defined as follows:


£1 = Ball drawn from first bag is blue P(A/£2) = 8/14
£2 = Ball drawn from first bag is red P(£2) = 6A1
A = Ball drawn from the second bag is blue Fig. 20.16
20.122
APPLIED MATHEMATICS-XI

Since first bag contains 6 red and 5 blue balls, we have

P(£,) = and P (E^) = A


If £| has already occurred, that is, if a blue ball is transferred from the first bag to the second bag,
then the second bag contains 5 red and 9 blue balls, therefore the probability of drawing a blue
ball from the second bag is —.
^ 14
■■■

Similarly, we have P(A/E2) = —


14

Using the law of total probability, we obtain


P(A) = P (Ej) p (A/E^) + P {£2) P (A/Ej) = —X — +
11 14 11 14 154'
EXAMPLE 6 There are txvo bags, one of which contains 3 black and 4 zvhite, balls, zvhile the other
contains 4 black and 3 white balls. A fair die is cast, if the face I or 3 turns up, a ball is taken from the first

F low
bag, and if any other face turns up a ball is chosen from the second bag. Find the probabilitu ofchoosin? a
blackball. '' ^

SOLUTION Let £j, £2 and A be the events defined as follows:

Ej = The die shows 1 or 3, £3 = The die shows 2, 4, 5 or 6, and A = The ball drawn is black.
2
We have, P (£^) = -
e
6 T = 6 3
for Fre
If £1 occurs, then the first bag is chosen and the probability of drawing a black ball from it is
7

P (A/E^) = I
Your
eBo ks

If £2 occurs, then the second bag is chosen and the probability of drawing a black ball from it is
7

P(A/E,)=^
ad
our

Using the law of total probability, we obtain


1 3 2 4
Re

11
P(A) = P{E^)P(A/E^) + PiE^)P{A/E2) = — X — + — X —
3 7 3 7 21
Find Y

£(£,) = 1/3
P(A/Ed = 3/7

P{A/E,) = 4/7
£(£2) = ^
Fig. 20.17

EXAMPLE 7 A bag A contains 4 black and 6 red balls and bag B contains 7 black and 3 red balls. A die is
thrown. If lor 2 appears on it, then bag /I is chosen, otherwise bag B. Ifhuo balls are drawn at random
(without
black.
replacement) from the selected bag, ifnd the probability‘ of one of them bein'? red and another
20.123
PROBABILITY

SOLUTION Consider the following events:


El= Getting 1 or 2 on the die, £3 = Getting any number other than 1 and 2 on the die
A = Getting 1 red and 1 black ball from the bag.
P(E,) = 1/3
r(A/E,)=
10c,

7C, x3C 1
P(A/£,)=
10 C2
P(£,)=2/3

ow
Fig, 20.18

Clearly,

p(E,)=|=in%)=^=f 4
Cix _ 8

e
P(A/E^) = P (Getting 1 red and 1 black ball from bag A)

re
10 15
C2

rFl ^Ci __ 7

F
P(A/E2) = P (Getting 1 red and 1 black ball from bag B) 10
C2
15

8 2 7 22

r
1
ou
Required probability = P(A) - £(£]) P{A/E{) + £(£2) Pi^/^i) 3 ^ 15 3 ^ 15 45
fo
ks
is known that the
EXAMPLE 8 Two thirds of the students in a doss are boys and the rest girls. It _
probability of a girl getting a first class is 0.25 and that of a boy getting a first class is 0.28. Find the
oo

probnhility that a student chosen at random will get first class marks m the subject.
Y

SOLUTION Let £i, £2 and A be the events defined as follows:


B

£^ = a boy is chosen from the class, £3 = a girl is chosen from the class,
re

and. A = the students gets first class marks.


P (£t) = 2/ 3, P (£2) -1/3, P (A/Ei) = 0.28 and P (A/E2) = 0.25
ou

Then,
Y
ad

Using the law of total probability, we obtain


P (A) = P (El) P (A/£i) + P (£2) P (A/£2) = -- X 0.28 + - X 0.25 = 0.27
3
d

EXAMPLE In a bolt factory, machines A, B and C manufacture respectively 25%, 35% and 40% of the
in
Re

total bolts. Of their output 5,4 and 2 percent are respectively defective bolts. A bolt is drawn at landom
from the product. What is the probability that the bolt drawn is defective?
F

SOLUTION Let Ei, £2, £3 and A be the events defined as follows:


Ej = the bolt is manufactured by machine A; Ej = the bolt is manufactured by machine B,
£3 = the bolt is manufactured by machine C, and, A = the bolt is defective.
Then,
35 40
i p(£2) =
25
P(El) = £(£3) =
100 100 ' 100
the condition that it is
P (A/£i) = Probability that the bolt drawn is defective given
manufactured by machine A
= 5/100
Similarly, we obtain
P(A/£2) = — and P(A/£3) = —
100
100
20.124
APPLIED MATHEMATICS-XI

Using the law of total probability, we obtain


P{A) = P {Ej) P {A/E^) + P {£2) P {A/E2) + P (£3) P (A/E-^)
= -?5- A+ 35
X
4
+
40
X
2
= 0.0345
100 100 100 100 100 100

EXAMPLE 10 A person has undertaken a construction job. The probabilities are 0.65 that there will be
strike, 0.80 that the construction job will be completed on time if there is no strike, and 0.32 that the
construction job will be completed on time if there is a strike. Determine the probability that the
construction job will be completed on time.
SOLUTION Let A be the event that the construction job will be completed on time, £. be the
event that there will be a strike and £3 be the event that there will be no strike.
We have.

P(£i)=0.65,P(£,)=1-P(£,)=1 -0.65 = 0.35,P(/1/£i)=0.32 and P(/l/£2,= 0.80

w
By total probability theorem, we obtain
Required probability = P (A) = P (Ej) P (A/E-y) + P (£2) P (A/E2)

F lo
= 0.65 X 0.32 + 0.35 x 0.80 = 0.208 + 0.28 =0.488
example 11 /I box has 5 blue and 4 red balls. One ball is drawn at random and not replaced. Its colour is

ee
also not noted. Then another hall is drawn at random.
W}mt is the probability of second ball being blue?

Fr
SOLUTION
events:
First ball drawn from the box may be blue or red. So, let define the following
us

for
£1 = First ball drawn from the bag is blue, £3 = First ball drawn from the bag is red
ur
A — Second ball drawn from the bag is blue.
Clearly, P(£j) =5^ P(£J =1,y p(/i/£j) P{A/E,)A
s
ook

8 “ 8
Yo
eB

P(£,) = 5/9 P(A/£,)=4/8


r

A
ou
ad

P(E2) = 4/9 P{/l/£,) = 5/8


Y

Fig. 20.19
Re
nd

By total probability theorem, we obtain


Fi

P(2l) = P(£i) P(A/£j) + P (£j) P(A/£,) = 59 X i8 + i9 X ^8 ®9

£—77 : EXERCISE 20.10


^^^8^‘=‘^^t‘^^i^s5whiteand6blackballs.AnotherbagBcontains4whitea nd3blackballs
A ball IS transferred from bag A to the bag B and then a ball is taken out of the second bae‘
Fmd the probability of this ball being black.
2. A purse contains 2 silver and 4 copper coins. A second purse contains 4 silver and 3 copper
corns If a com is pulled at random from one of the two purses, what is the probability that it
IS a silver com? ^

3. bag contains 4 yellow and 5 red balls. Another bag contains 6 yellow and 3 red balls A
ball transferred from the first bag to the second bag and then a ball is drawn from the
second bag. Find the probability that ball drawn is yellow.
20.125
PROBABILITY

4. A bag contains 3 white and 2 black balls and another bag contains 2 white and 4 black balls.
One bag is chosen at random. From the selected bag, one ball is drawn. Find the probability
that the ball drawn is white.
5. The contents of three bags I, II and III are as follows:
Bag 1:1 white, 2 black and 3 red balls.
Bag II; 2 white, 1 black and 1 red ball;
Bag III: 4 white, 5 black and 3 red balls.
A bag is chosen at random and two balls are drawn. What is the probability that the balls
are white and red?
6. An unbiased coin is tossed. If the result is a head, a pair of unbiased dice is rolled and the
sum of the numbers obtained is noted. If the result is a tail, a card from a well shuffled pack
of eleven cards numbered 2,3,4,..., 12 is picked and the number on the card is noted. What
is the probability that the noted number is either 7 or 8?
7. A factory has two machines A and B. Past records show that the machine A produced 60%

w
of the items of output and machine B produced 40% of the items. Further 2% of the items
produced by machine A were defective and 1% produced by machine B were defective. If
The bag A contains 8 white and 7 black balls while
F lo
an item is drawn at random, what is the probability that it is defective?
the bag B contains 5 white and 4 black
balls. One ball is randomly picked up from the bag A and mixed up with the balls in bag B.
Then a ball is randomly drawn out from it. Find the probability that ball drawn is white.

e
Fre
9. A bag contains 4 white and 5 black balls and another bag contains 3 white and 4 black balls.
A ball is taken out from the first bag and without seeing its colour is put in the second bag.
for
A ball is taken out from the latter. Find the probability that the ball drawn is white.
10. One bag contains 4 white and 5 black balls. Another bag contains 6 white and 7 black balls.
r
A ball is transferred from first bag to the second bag and then a ball is drawn from the
You
oks

second bag. Find the probability that the ball drawn is white.
An urn contains 10 white and 3 black balls. Another urn contains 3 white and 5 black balls.
eBo

11.
Two are drawn from first urn and put into the second urn and then a ball is drawn from the
latter. Find the probability that it is a white ball.
A bag contains 6 red and 8 black balls and another bag contains 8 red and 6 black balls. A
ad
our

12.
ball is drawn from the first bag and without noticing its colour is put in the second bag. A
ball is drawn from the second bag. Find the probability that the ball drawn is red in colour.
13. Three machines Ej, Ej, £3 in a certain factory produce 50%, 25% and 25% respecUvely, of
the total daily output of electric bulbs. It is known that 4% of the hibes produced one each of
dY
Re

machines E, and E2 are defective, and that 5% of those produced on £3 are defective. If one
tube is picked up at random from a day's production, calculate the probability that it is
Fin

defective.
ANSWERS

29 7 118
39 19
1. 15 495
42 45
83 31 29
193 10. —
6. 7. 0.016 8.
63
150 72
792
59 59 17
11. 12. 13.
130 105 400

HINTS TO SELECTED PROBLEMS

2. Consider the following events:


£1 = Selecting first purse, £2 = Selecting second purse, A = Coin drawn is silver com.
20.126 APPLIED MATHEMATICS-XI

We have, P (£j) = P (£2) = i PWEi) = P(/l/£2) = i.


Required probability = P (A) = P (Ej) P (A/E^) + P {£2) P (A/E2).
4. Consider the following events:
Ej = Selecting first bag, £2 = selecting second bag, A = ball drawn is white.
Then, P (£j) = P (£2) =1/2, P (A/Ej) = 3/5, P (A/E2) = 2/6.
7
Required probability = P (A) = P (E^) P (A/Ej) + P (£3) P (A/E2) =-x~ + -x- =
2 5 2 6 15
5. Let£-[ = bag I is selected, £2 = bag II is selected, £3 = bag III is selected
and, A = Two balls drawn from the chosen bag are white and red.

ow
Then,P(£j) = P(£2)=P(£3)=i
P(A/E^) = , R(A/E2) = and P(A/E3) =
^C2 "C2 12
C2

e
re
.-. Required probabiUty = P(A) = P (£j) P{A/E^)+ £(£2) P (A/£2) + P (£3) P(A/£3)

rFl
6. Let£j = The coin shows a head, E| = The coin shows a tail, A = The noted number is 7 or 8.

F
Then,P(£i)=l/2,P(£2)=l/2,P(A/£i)=n/36 and P{A/£2) =2/11.
Required probability = P(A) = P (E^) P {A/£i) + P (£2) P(A/£2)

r
ou
20.15 BAYE’S THEOREM fo
ks
THEOREM (Bnye's Theorem) Let S be the sample space and let E-^, £2 be 11 mutually exclusive
and exhaustive events associated with a random experiment. If A is any event which occurs with £, or £9
oo

or... orE,,, then ^ ^


Y

P{Ei) P (A/Ei)
eB

P(Ei/A) = , i = 1, 2,..., /I
.S P(Ei)P{A/Ei)
/=!
r

iIRQOF Events £^,£2, are?? mutually exclusive and exhaustive events


You

such that
ad

S = £| u£2 u... u^,, where Ej nEj =^lorijt j


A = AnS
d

=> A = {An£i)u(An£2)u...u(An£;,)
Re
in

=> P(A) = P(An£i) + P(A n£2)+... + P(An^,) [By addition theorem]


F

=> P(A) = 'i P(An£,.)


I = 1

P(A) = j =I l P{Ej)P{A/Ei) [v P{AnEj)=P(Ej)PiA/^)] ...(i)

£. £2 £3' ●£„ S

^^£1 /InE^ Ar£^ AnE,

Fig. 20.20

Using multiplication theorem of probability, we obtain


P(AnE^) = P(A)P(Ei/A) for ! = 1,2,h
20.127
PROBABILITY

PjAnEj)
P{E^/A) =
P(A)
P{E^)P{A/Ei) (v P{AnEi)=P{Ei)P{A/Ei)]
P{Ei/A) =
P{A)
P{Et)P{A/Ej) [Using (i)]
P{Ei/A) =
Z P{Ei)P{A/Ei)
i = l

P{Ei)P{A/E^) , / = 1,2,...,n
Hence, P{Ej/A) =
I P{Ei)P{A/Ei)

ow
/ = i

Ninr I The events £t,£2,fl, are iisualli/ referred to as ‘hypothesis' and the probabilities
P(£i),~P(£2)/ ■■■' ^(^i) hnoum as the 'priori' probabilities os they exist before we obtain any
information from the experiment.

e
called the 'likelyhood probabilities'as they tell
us

re
N(Tn:2 The probabilities P (A/E^); =1, 2, ...,nare

rFl
how likely the event A under consideration occurs, given each and every prion probabilities.

F
NOTH 3 The probabilities P (Ej/A); i = 1, 2, n are called the ‘posterior probabilities’ as they are
determined after the results of the experiment are knozvn.

r
ou
The significance of Baye's theorem may be understood in the following manner :
fo
An experiment can be performed in n mutually exclusive and exhaustive ways £j, £2, E,,. The
ks
probability P (£;■) of the occurrence of event Ej} i = l, 2, n is known. The experiment is
performed and we are told that the event A has occurred. With this information the probability
oo

P (£,■) is changed to P (^-/A).Baye's theorem enables us to evaluate P (Ej/A) if all the P{Ej) {prion
Y
B

probabilities) and P{A/Ej) (likelyhood probabiKties) are known as explamed in the following
examples.
re
ou

ILLUSTRATIVEEXAMPLES
Y
ad

EXAMI-LE 1 ^ bolt factory, machines A, B and C manufacture respectively 25%, 35% and 40% of the
total bolts. Of their output 5,4 and 2 percent are respectively defective bolts. A bolt is drawn at random
d

from the product. If the bolt drawn is found to be defective, zuhat is the probability that it is manufactured
in
Re

by the machine B?
F

SOLUTION Let Ej, £2, E3 and A be the events defined as follows:


£| = Bolt is manufactured by machine A, £2 = Bolt is manufactured by machine B,
£3 = Bolt is manufactured by machine C, A = Bolt is defective
Then, P(E]) = (Probability tliat the bolt drawn is manufactured by machine A) = 25/100,
^ (Probability that the bolt drawn is manufactured by machine B)= 35/100,
p(£^j ^ (Probability that the bolt drawn is manufactured by machine C) = 40/100.
P{A/E-^) = Probability that the bolt drawn is defective given that it is manufactured by machine A
= 5/100

Similarly, we have P (A/E2) - f and P(A/£3)=^


100

Required probability = Probability that the bolt is manufactured by machine B given that
the bolt drawn is defective
= P(£2M)
20.128
APPLIED MATHEMATICS-XI

P{E2)P{A/E2)
P(Ei) P (A/E^) + P{E2) P (A/E2) + P{E^) P (A/E^)
35 ^ 4
100 100 140 140 _ 28
25 5 35 4 40 2
X + X + 125 +140 + 80 345 ~ 69
100 100 100 100 ' 100 ^ 100
EXAMPLE 2 Three urns contain 6 red, 4 black; 4 red, 6 black, and 5 red, 5 black balls respectively. One of
the urns is selected at random and a ball is dratunfrom it. If the ball draion is red, find the probability that
it is drawn from the first urn.
SOLUTION Let£j,£2,E3 and A be the events defined as follows:
£j = First urn is chosen, £2 = Second urn is chosen,
£3 = Third urn is chosen, and A = Ball drawn is red.
Since there are three urns and one of the three urns is chosen at random. Therefore,

w
P(£,) = P(£2) = £(£3) = i.

The probability of drawing a red ball from it is 6/10.


P(/l/£j) = A.
F lo
If £[ has already occurred, then first urn has been chosen which contains 6 red and 4 black balls.

ee
So,

Fr
10

Similarly, PiA/Ej) = — and PM/E,) = ~ for


10 10

We have to find P (£^/A), i.e. given that the ball drawn is red, what is the probability that it ISi
r
drawn from the first
You

urn.
s
ook

By Baye's theorem, obtain


P(£l)P(^/£i)
eB

P(E^/A) =
P (Ej) P (A/E-i) + P(£2) P (A/E2) + P(£3) P (A/E^)
1 6
our

— X
ad

1
3 10 A = 2
6 1 4 1 5 15 5
^ X
+ X — + — X
3 10 3 10 3 10
dY

EXAMPLE 3 A company has two plants to manufacture scooters. Plant I manufactures 70% of the
Re

scooters and Plant 11 manufactures 30%. At Plant /, 80% of the scooters are rated as of standard quality
Fin

and at Plant II, 90% of the scooters are rated as of standard quality. A scooter is chosen at random and is
found to be of standard quality. What is the probability that it has come from Plant II?
SOLUTION Let£|, £2 and A be the following events:
£1 = Plant I is chosen, £3 = Plant II is chosen, and A = Scooter is of standard quality.
70 30 80 90
Then, P{Ei) = . P(E2) =
100 100' P (>!/£]) = 100
and PiA/E^) =
100

We have to find P (£2/4).


By Baye's theorem.
30 90
P(£2) P (A/E^) ioo ion 17 17
P(£2/^) =
P(£l) P {A/E{) + P(£2) P {A/E2) 70
- X
80
+
30 90 56 + 27 "83
100 100 100 ^ 100
20.129
PROBABILITY

EXAMPLE 4 An insurance company insured 2000 scooter drivers, 4000 car drivers and 6000 truck
drivers. The probabilities of an accident involving a scooter driver, car driver and a truck driver are 0.01,
0.03 and 0.15 respectively. One of the insured person meets zvith an accident. What is the probability that
he is a scooter driver?
SOLUTION Let El, , E3 and A be the events defined as follows:
Et = Person chosen is a scooter driver, £2 = Person chosen is a car driver,
E3 = Person chosen is a truck driver, and Person meets with an accident.
Since there are 12000 drivers out of which scooter, car and truck drivers are 2000,4000 and 6000
respectively.
2000
= i3 and P(E2) = 6000 ^ 1_
P(Ei) = - and £(£2) = 12000 12000 ” 2
12000 6

It is given that
P (/4/£i) = Probability tliat a person meets with an accident given that he is a
scooter driver = 0.01.

w
Similarly, P(/1/E2) = 0.03 and P{A/E^) = 0.15.
We are required to find P (£1/^), i.e. given that the person meets with an accident, what is the
probability that he was a scooter driver?
By Baye's rule.
P(£i)P(A/£i)
F lo
ee
P{E^/A) =

Fr
P(£l) P (A/E{) + £(£2) P (A/£2) + £(£3) E (A/E^)
1 for
X 0.01 1 1
6.
P(£i/A) = 1 1 1 + 6 + 45 52
^ X 0.01 +
ur
X 0.03 + X 0.15
6 3 2
s

balls, urn B contains 3 white, 2 black and 4 red


ook

EXAMPLE 5 Urn A contains 2 icyhite, 1 black and 3 red


Yo

balls and urn C contains 4 white, 3 black and 2 red balls. One urn is chosen at random and 2 balls are
eB

drawn at random from the urn. If the chosen bails happen to be red and black, what is the probabdity that
both balls come from urn B?
Let £|, £2, £3 and A denote the following events.
our
ad

SOLUTION

E-i = Urn A is chosen, Ej = Urn B is chosen, £3 = Urn C is chosen, and A = two bails drawn at
random are red and black.
dY

Since one of the urns is chosen at random.


Re

P(£l) = i = P(£2) = £(£3)


Fin

If £j has already occurred, then urn A has been chosen. The urn A contains 2 white, 1 black and X

3 red balls. Therefore, the probability of drawing a red and a black ball is 6p
^2

3qxiq i- = 1
i.e, P{A/E{) = 6/-. 15 " 5
^2

^Ci X _ 2 ^Cix ^Ci _ 1


Similarly, P(A/£2) = and P (A/Eg) =
^2
9 ' 'C2 6

We are required to find P (E2/A). By Baye's theorem


P(E2)P(A/E2)
PiE^/A) =
P(£l) P(A/£i) + £(£2) P (A/E2) + £(£3) P {A/E^)
20.130
APPLIED MATHEMATICS-XI

1 2 2
X

3 9 9 20
=>
P{E2/A) = 1 1 1 2 1 1 1 2 1 53
X + X - + - X + +
3 5 3 9 3 6 5 9 6
EXAMPLES
There are 3 bags, each containing 5 ivhite balls and 3 black balls. Also there are 2 bags, each
containing 2 white balls and 4 black balls. A white ball is drawn at random. Find the probabiliti/ that this
white ball is from a bag of the first group.
SOLUTION Let El, £2 and A be the events defined as follows:

El ^ Selecting a bag from the first group, Ej ~ Selecting a bag from the second group
and, A = Ball drawn is white

Since there are 5 bags out of which 3 bags belong to first group and 2 bags to second group.

w
P(Ei) = f P(E,) = I
a 5

If E-i has already occurred, then a bag from the first group is chosen. The bag chosen contains 5

Flo
white balls and 3 black balls. Therefore, the probability of drawing a white ball from it is 5/8
P{A/Ei)=5/8

e
Similarly, P {A/E2) =2/6 =1/3.

re
We have to find P (Ei/A), i.e. given that the bail drawn is white, what is the probability that it is

F
drawn from a bag of the first group.
ur
r
By Baye's theorem.
fo 3
- X
5

P{Ei/A) = P{Ei)P{A/Ei) 5 8 45
ks
PiEi)P{A/Ei) + P{E2)P{A/E2) 3 5
2,1-61
Yo
X - +
oo

5 8 5 3
t.XA.MPLE 7 A card from a pack of 52 cards is lost. From the remaining cards of the pack, two cards are
B

drawn and arefound to be hearts. Find the probability of the missing card to be a heart.
re

SOLUTION Let £|,£2,£3,£4 and be the events as defined below;


£| - Missing card is a heart card, £2 = Missing card is a spade card,
u
ad

£3 = Missing card is a club card, £4 = Missing card is a diamond card


Yo

and.
A = Drawing two heart cards from the remaining cards.
Then,
d
Re

13

\.PiEl) =
13 13 1
P(E0=^ -,P(E4) = M = i
in

52 52
P(E,) =
52 52 4
F

P {A/El) = Probability of drawing two heart cards given that one heart card is missing
Co
P{A/Ei) = 51
C2
P {A/Ef) - Probability of drawing two heart cards given that one spade card is missing
Co
P{A/E2) = 51
C2
13 13
c
Similarly, P{A/Ef) = C2
51 ^ and PiA/E^) = 51
C2 ^2
By Baye's Theorem
Required probability = P (E^/A)
20.131
PROBABILITY

P{E{jP{A/E^)
P{Ei) P {AE{) + P(E2) P (A/E2) + ^(£3) P (A/E^) + PIE4) P {A/E^)
12
1
X
C2
4 51 C2
12 13 13 13
C2 1 C2 1 C2
1
X C2,l " 51 + ^ Cl +
4
^ Cl
51 C2
4 51 C2
4 51^^ 4 C2
12 11
66
C2
12 /-● , 13^ , 13/^ I 15p 66 + 78 + 78 + 78 50
C2 + ^2+ '“2 ^2

EXAMi’LL a Suppose a girl throws a die. If she gets a 5 or 6. she tosses a coin three times and notes the
number of heads. If she gets a 1,2,3, or 4, she tosses a coin once and notes whether a head or tad is obtained.
If she obtained exactly one head; what is the probability that she threw a 1,2,3, or 4 with the die?

w
SOLUTION Consider the following events;
El = Getting 5 or 6 in a single throw of a die
E2 = Getting 1,2,3, or 4 in a single throw of a die.

F lo
ee
A = Getting exactly one head.

Fr
2

Clearly, P (£1) = ^ 3
for
P(A/Ei) = Probability of getting exactly one head when a coin is tossed three times
ur
U 8
s
ook

P (A/E2) = Probability of getting exactly one head when a coin is tossed once only
Yo

1
eB

2
P{E2)P{A/E2)
Required probability = £(£2/^) P{£i)P(A/£i) + P(£2)P(71/E2)
r
ad
ou

2 1
X
8
3 2
Y

1 3 1 2 11
X + X
Re

3 8 2 3
nd

EXAMPLE 9 Given three identical boxes I, II and Ilf each containing tzvo coins. In box I both coins are
Fi

qold coins, in box II both are silver coins and in box III there is one gold and one sliver com. A person
chooses a box at random and takes out a coin. If the coin is ofgold, what is the probability that the other com
in the box is also of gold?
SOLUTION Consider tlae following events;
£^ = Box I is choosen, £2 = Box II is choosen, £3 = Box III is choosen.
A = The coin drawn is of gold.
1
Clearly, P (Ej) = P (£2) = £ (£3) “
2=1
P (A/£i) = Probability of drawing a gold coin from box I = -
P {AIE-f) = Probability of drawing a gold coin from box II = 0^
P (A/E3) = Probabilit)' of drawing a gold coin from box III = -
Probability that the other coin in the box is of gold
20.132
APPLIED MATHEMATICS-XI

- Probability that gold coin is drawn from the box I


= P{E^/A)

P(E^)P{A/E,) 3 2

P (£i) P (A/E^) + P (£2) P {A/E2) + P (£3) P (A/E^^ 1 X 1 +


1 n 1 5 3
3 3 3 2
EXAMPLE 10 Bag I contains 3 red and 4 black balls and Bag !1 contains 4 red and 5 black balls. One ball
IS
transferred from Bag I to Bag 11 and then a ball is drawn from Bag II. The ball so drazun isfound to be red
in colour. Find the probabilih/ that the transferred ball is black.
SOLUTION Consider the following events:
£l = Ball transferred from Bag I to Bag II is red
£2 = Ball transferred from Bag I to Bag II is black
A = Ball drawn from Bag II is red in colour.

w
Clearly,
1
andP(/i/£2)=i- =-

F lo
^ 10 5
4 2
P(E2)P(A/Eo)
X

Required probability = P{E2/A) 7 5 16

ee
P (E^) P (A/E^) + P (E2) P (A/E2) ”3^1 4.2 31

Fr
+ X
7 2 7 5
EXAMPLE 11
Suppose that 5% of men and 0.25% of xvomen have grey
for hair. grey haired person is
selected at random. What is the probability of this person being male? Assume that there are equal number
of males and females. '
r
You

SOLUTION Consider the following events:


s
ook

E] = Pers()n selected is male, £2 = Person selected is female.


/I = Person selected is grey haired.
eB

Clearly,

P(E-l) -PiE2)-~'P(A/Ei) =^and P(v4/£2) =


1
our
ad

400

Required probability = P (Ei/A) = P(£;)P(A/£i)


P (£1) P (A/E^) + P (Ej) P (A/E2)
dY
Re

1 5

2 "" 100 20
Fin

1 5 1 1 21
2 100 ^ 2 ^ 400
X

EXAMPLE 12 A bag contains 4 balls. Tzvo balls are drawn at random zoithout replacement and are found
to be white. What is the probability that all balls are zuhite?
SOLUTION Since two balls drawn are white. So, we have the following possibilities:
(i) The bag contains two white balls and 2 balls of other colour,
(ii) The bag contains 3 white bails and one ball of other colour,
(iii) The bag contains all white balls.
Consider the following events;
El = There are two white and two other colour balls in the bag,
E■2
There are three white and one other colour ball in the bag,
£3 = There are all white balls in the bag, 4 = Drawing 2 white balls from the bag
PROBABILITY 20.133

Clearly, P(£0 = P = £(£3) = 3


4
3r
^C2 i6 , P (A/E^) = ^2 1 C.
^.P{A/E^) =
= 1
P{A/E{) = 4 4
^2 C2 C2
1
xl
P(E3)P(/A/£3) 3 3
Required probability = P (£3//!) = — 1 1 1 1 1 5
X + X + X 1
X P{E^)P{A/E^) 3 6 3 2 3
j = l

EXAMPLE 13 A bag contains 3 red and 7 black balls. Two balls arc selected at random one-bif-one

ow
without replacement. If the second selected ball happens to be red, what is the probabiliti/ that the first
selected ball is also red?
SOLUTION Consider the following events:
E■^ = First ball drawn is red and second is of any colour

e
£2 = First ball drawn is black and second is of any colour, A = Second ball drawn is red.

re
Clearly, P (£,) = A P (Ej) =^, P (/l/Ej) and P (A/£,) =|
Frl
F
3 2
X
2
P(£i)P(/l/£t) 10 9
Required probability = P {Ei/A) =
ou
or
3 9
P(£,)P(d/£]) + P(£2)P(/l/£2) 3 ^2^ 7 X

10 9 10 9
kfs
EXAMPLE 14 Suppose that 6% of the people with blood group O are left handed and 10% of those with
other blood groups are left handed. 30% of the people have blood group O. Ifa left handed person is selected
oo

at random, what Is the probability that he/she will have blood group O?
Y

SOLUTION Consider the following events:


B

£^ = A person with blood group O is selected


re

£2 = A person with other blood groups is selected.


A = A left handed person is selected.
oYu
ad

We have.
10
30 70 _ 7
£(£l) = 100 P{A/£^) and P(A/£2) = 100
10' 100 ~10'
d

P(£|)P(A/£i)
in

Required probability = PfE^/A) =


Re

P(£i)P(A/£i) + P(E2)P(^/£2)
F

3^6
10"" 100 li-A
3
X
6
+
7
X
10 88 ”44
10 100 10 100

EXAMPLE 15 A man is known to speak truth 3 out of 4 times. He throws a die and reports that it is a six.
Find the probability that it is actually a six.
SOLUTION Let Ej, £2 and A be the events defined as follows:
£-, = Six occurs, £2 = Six does not occur, and A = The man reports that it is a six.
Clearly, £(£,) = j,
6
PiE,) = D

Now, P (A/Ej) = Probability that the man reports that there is a six on the die given that six
has occurred on the die
= Probability the man speaks truth =3/4
20.134 APPLIED MATHEMATICS-XI

and. P {A/E2) = Probability that the man reports that there is a six on the die given that
six has not occurred on the die
3 1
= Probability that the man does not speak trutli =1 — = —
4 4

We have to find P (£-| /A) i.e., the probability that there is six on the die given that the man has
reported that there is six.
By Baye's theorem
1 3

w
X
P{E{)P{A/E{) 6 4 3
P(Hi/A) = 1 3 5 1
P(£l)P(>l/£l) + P(£2)P(/i/£2) X - + X
8
6 4 6 4

liXA.viPLC 16 In n test, nji examinee either guesses or copies or knoius the answer to a multiple choice

e
question with four choices. The probability that he makes a guess is 1/3 and the probability that he copies

ro
re
the answer is 1/6. The probability that his anszver is correct, given that he copied if, is 1/8. Find the
probability that he knew the anszver to the question, giz>en that he correctly answered it.

F
SOLUTION Let £|^, £2 , £3 and A be the events defined as follows:

answer, and A = Examinee answers correctly.


Clearly, P(£i)=ip(£2)=l.
uFl
£] = Examinee guesses the answer, £2 = Examinee copies the answer, £3 = Examinee knows the

sr
3 6

ko
o
Since £j ,£2 ,£3 are mutually exclusive and exhaustive events. of
P(£i) + £(£2) + £(£3) = 1 ^ £(£3) = 1 -(£{£i) + £(£2)) = 1 - i3 -1=1
6 ~ 2
o
If £^ has already occurred, the examinee guesses, then there are four choices out of which only
Y
erB

one is correct. Therefore, the probability that he answers correctly given that he has made a
guess is 1/4 i.e. £ (A/£,) =1/4. It is given that £ (A/£2) =1/8.
uY

and, £(A/£3) = Probability that he answers correctly given that he knew the answer =1
By Baye's Theorem
Required probability = £(£3/^)
ad
do

£{£3)£(A/£3)
£(£l) £ (A/E^) + £(£2) £ (A/£2) + £(£3) £ (A/£3)
in

1
X 1
24
Re

2
1 1 1 1
1.1
F

+ X +
29
3 4 6 8 2

tXAMl’lJ; 17 A letter is knozim to have come either from TATANAGAR or CALCUTTA. On the
envelope just tzvo consecutive letters TA are visible. What is the probability that the letter has come from
(i) Calcutta (ii) Tatanagar?
SOLUTION Let £| be the event that the letter came from Calcutta and £2 be the event that the
letter came from Tatanagar. Let A denote the event that two consecutive letters visible on the
envelope are TA.
Since the letters have come either from Calcuttaor Tatanagar.
P(£l) = i = P(£2)
If Ej has occurred, then it means that the letter came from Calcutta. In the word CALCUTTA
there are 8 letters in which TA occurs in the end. Considering TA as one letter there are seven

letters out of which one can be in 7 ways.


PROBABILITY 20.135

1
P(A/E|) =
7

If £2 has occurred, then the letter came from Tatanagar. In the word TATANAGAR there are 9
letters in which TA occurs twice. Considering one of the two TA's as one letter there are 8 letters.
P(A/£2)=|8
We have to find P{Ei/A) and P (£2/A).
1 1

ow
X
4
P(E^)P(A/£i) 2 7
(i) P(Ei/A) = 1 1 2
P(£l)P(A/£i) + P(E2)P(A/E2) ^ X + X
11
2 7 2 8
1 2
X
7
P(E2)P(A/E2)

e
2 8
(ii) P(E2/A) = 1 1 1 2 11
P{E^)P{A/E^) + P{E2)P{A/E2)

re
— X + X

2 7 2 8

past experience, it is known that the probabilitk^

F
EXAMPLE 18 A doctor is to z’isit a patient. From the
that he will come bij train, bus, scooter or by other means

Frl of transport are respectively —, -, —and 5


ou
1 i and if he comes by train, bus and scooter respectively, but

sr
The probability that he will be late are
if he comes by other means of transport, then he will not
probability that he comes by train ?
kfo
be late. When he arrives, he is late. What is the
oo
SOLUTION Let E], £2, £3, £4 be the events that the doctor comes by train, bus, scooter and
Y

other means of transport respectively. It is given that


reB

P(El) =
|j,P(E2) = i,P(E3) = L and (P(£4)=|
uY

Let A denote the event that the doctor visits the patient late. It is given that
P{A/E-i) = Probability that the doctor will be late if he comes by train - -
ad
do

P{A/E2) = Probability that the doctor will be late if he comes by bus = ^


in

1
P (A/£3) = Probability that the doctor will be late if he comes by scootor
Re

12
F

and. P {A/E4) = Probability that the doctor will be late if he comes by other means of
transport
= 0

We have to find P {E■^/A).


By Baye's theorem
P(£l)P(A/Ei)
P(El/A) =
P (£1) P (A/Ei) + P (£2) P {A/E2) + P (£3) P (A/£3) + P (£4) P
3 1
- X

10 4
3 120 1
P{E^/A) = -3 X
1
+
1
X
1
- +
1
X
1
+ ^x 0 40 18 2
10 4 5 3 10 12 5

Hence, required probability is —.


20.136 APPLIED MATHEMATICS-XI

EXAMPLE 19 Letci-^, ^2 ' ^3 mutually exclusive diseases. Let S = {si, S2, S3..., s^) be the set of
observable symptoms of these diseases. For example, s^ is the shortness of breath, $2 is loss of weight, S3 is
fatigue etc. Suppose a random sample of W,000 patients contains 3200 patients zuith disease dy 3500 with
disease ^3 mid 3300 with disease d^- Also, 3100 patients with disease d^, 3300 zvith disease ^3 mid 3000
ivith disease d^, shoiu the symptom S. Knozving that the patient has symptom S, the doctor wishes to
determine the patient's illness. On the basis of this information, what should the doctor conclude?
SOLUTION Let Ej denote the event that the patient has disease dj ;i = 1, 2, 3 and A be the
event that the patient has symptom S. Then,
P[El) =
3200 _ 3500 _ 35 3300 _ 33
10000 ' 100 ' P(£2) = 10000 " Tm' P(£3) = 10000 ~ 100

w
3100 _ 31 3300 _ 33 30000 30
P(A n£i) =
10000 ” 100 ' P (A n £2) = 10000 ~1^ and, P(A nE^) = 10000 100

P(A/£i) = P (A n£^)^ 31/100 _ m P(A/£2) =


P (A n £3) _ 33/100 33
P(Ei) 32/100 “ 32'

o
e
P(£2) 35/100 35

re
P(A/£3) = PjA n£3) _ 30/100 _ 30
P(£3) “33/100“^

Frl
F
Using Baye's theorem
P(Pl/A) = P(£|)P(A/£Q
ou
P (£5) P (A/£i) + P (£2) P(A/£2) + P (£3) P (A/£3)

r
so
32 31
X kf
100 32 31
P(El/A) = 32 31 35 33 33 30 94
oo
- X +

100 32 100 35 *^ 100 33


X

P(E2)P(A/E2)
Y

P(E2/A) =
B

P (£1) P (A/£i) + P (£3) P (A/£3) 4- P (£3) P (A/£3)


35 33
re

100 35 33
P(E2/A} =
oY

32 31 35 33 33 30
u

X + ^ ^
94
ad

100 32 100 35 100 33


and,
d

P(£3/A) = P(£3)(A/£3)
in

P(£l) P(A/£]) + P(£2) P(A/£2) + P{E^) P(A/E^)


Re

33 30
F

100 33 30
P(E^/A) = 32 31 35 33 33 30
X + X
94
100 32 100 35 100 33

Clearly, P (E^/A) < P (E^/A) < P (£2/A) i.e. P (£2/A) is largest.


Thus, the doctor should conclude that the patient is most likely to have disease ^3.
EXAMPLE 20 Suppose that the reliability of a HIV test is specified asfollozvs:
Of people having HIV, 90% of the lest detect the disease but 10% go undetected. Of people free of HIV,
99 /o of the test are judged HIV-ive but 1% are diagnosed as showning HIV+ive. From a large population
ofzvhich only 0.1% have HIV, one person is selected at random, given the HIV test, and the pathologist
reports him/her as HIV +ive. What is the probability that the person actually has HIV?
SOLUTION Consider the following events:
£1 = The person selected is actually having HIV
E2 - The person selected is not having HIV
A = The person's HIV test is diagnosed as positive
20.137
PROBABILITY

We have.
0.1
P (El) = 0.1% = 100 = 0.001, P (£2) = ^ ^ (£1) = 1 ' 0-^01 = 0.999
P [A/E-i) = Probability that the person tested as HIV +ive given that he/she is
actually having HIV.
90
= 0.9
100

and.
p {A/E2) = Probability that the person tested as HIV +ive given that he/she is
actually not having HIV
1
= 0.01
100
P(£i)P(/\/£l)

w
Required probability = P{E-^/A) =
P(£i)P(A/Ei) + P(E2)P(A/£2)
0.001 X 0.9 90

F lo
0.001 X 0.9 + 0.999 x 0.01 1089

EXAMPLE 21 Suppose you have two coins which appear identical in your pocket. You knoiv that one is

ee
fair and one is 2-headed. If you lake one out, toss it and get a head, what is the probability that it was a fair

Fr
coin?

SOLUTION Consider the following events: for


El = Taking out a fair coin from the pocket.
ur
£2 =Taking out a two-headed coin from the pocket
s

A = Getting a head on the coin when it is tossed.


ook
Yo

We have,

P(El) =^, P{E2) P(A/Ei) = ^and P{AIEf)=\


eB

By Baye's theorem
r

P{Ef)P{AIEf)
ad
ou

Required probability = P(Ei//4) =


P(£i)P(/1/£i)+P(£2)P(A/£2)
1 1 1
Y

2 2 - 4 - 1
Re
nd

1 1 1 3 3
X + xl
2 2 2 4
Fi

EXAMPLE 22 Three bags contain a nwnber of red atid lohite balls are as follows:
Bag 1: 3 red balls; Bag 11: 2 red balls and 7 white ball; Bag III: 3 white balls
The probability that bag i will be chosen and a ball is selected from it is ^,(=1,2, 3. If a white ball is
selected, what is the probability that it came from (i) Bag II (ii) Bag III
SOLUTION Consider the following events:
El = Bag I is selected, Et = Bag II is selected, £3 = Bag III is selected
We have,

P(£;.)=y/ = 1,2,3
o

P(A/Ei) =Probabili,ty of drawing a white ball when bag I is selected = 0


P(v4/E2) = —3 and p(a/E^^ 3
20.138
APPLIED MATHEMATICS-XI

P{E2)P{A/E2)
(i) Required probability = P(£2/A) =
P(£l) P{A/E^) + P{E2) P{A/E2) + £(£3) P{A/E^)
2 1
X

6 3 1/9 _ 2
1 2 1
x0 + X + ^1 11/18 ”11
6 6 3 6

P(£3)PM/£3)
(ii) Required probability = P (£3 //I) =
P(£l) P(A/£i) + £(£2) P{A/E2) + £(£3) P(^/£3)
3
xl
6. 1/2 9
1^21
x0+ X +
3
xl 11/18 11
6 6 3 6

FX AMI LE 23 A shopkeeper sells three types of seeds Ai, A2 nnd A-^. They are sold as a mixture zuhere the

w
proportions are 4:4:2 respectively. The germination rates of three types of seeds are 45%. 60% and 35%.
Calculate the probability

F lo
(i) that it will not germinate given that the seed is of type A3
(ii) of a randomly chosen seed to germinate.

ee
(iii) that it is of type A2 given that a randomly chosen seed does not germinate.

Fr
SOLUTION Consider the following events: for
Ej = Seed chosen is of type Aj,i =\, 2, 3
ur
A = Seed chosen garminates.
s

£3 = Bag III is selected


ook
Yo

We have.

PlEd-f,P(E2)=f^ndP(E,)=^
eB

45 60 35
P(A/£^) = P(A/£2) =
our

P(A/£3) =
ad

100' 100' 100

(i) Required probability = £(A/E3) =1 - £(A/£3) =1 = 0.65


Y

100 100

(ii) Required probability = P(A) = £(£,) P(A/E^) + P(E2)P(A/£2) + £(£3) P(A/£3)


Re
nd

= 4 X 45 + — X4 60 2 35 _ 490 = 0.49
10 100 10 100 10"" 100 ” 1000
Fi

(iii) Required probability = £(£2/A)


^ £(£20 A) _ £(£2) £(A/£2) _ £(£2)(1 -£(A/£2))
£(A) £(A) 1-£(A)
4 r 60 4 40
X 1-
10 100/ ■ 10 ^100 _16
49 51 51
1-
100 100

EXERCISE 20.11
1. The contents of urns I, II, III are as follows:
Urn 1:1 white, 2 black and 3 red balls
Urn II: 2 white, 1 black and 1 red balls
20.139
PROBABILITY

Um in : 4 white, 5 black and 3 red balls.


One um is chosen at random and two balls are drawn. They happen to be white and red.
What is the probability that they come from Urns I, II, III?
2. A bag A contains 2 white and 3 red balls and a bag B contains 4 white and 5 red balls. One
ball is drawn at random from one of the bags and is found to be red. Find the probability
that it was drawn from bag B.
3. Three urns contain 2 white and 3 black balls; 3 white and 2 black balls and 4 white and 1
black ball respectively. One ball is drawn from an um chosen at random and it was found to

ow
be white. Find the probability that it was drawn from the first um.
4. The contents of three urns are as follows:
Um 1:7 white, 3 black balls, Um 2:4 white, 6 black balls, and Um 3:2 white, 8 black balls.
One of these urns is chosen at random with probabilities 0.20, 0.60 and 0.20 respectively.

e
From the chosen um two balls are drawn at random without replacement. If both these

re
balls are white, what is the probability that these came from um 3?
three itmes and notes the
Suppose a girl throws a die. If she gets 1 or 2, she tosses a coin

Flr
5.

number of tails. If she gets 3,4,5 or 6, she tosses a coin once and notes whether a head oir

F
'tail' is obtained. If she obtained exactly one 'tail', what is the probability that she threw 3,4,
5 or 6 with the die?
ou
Two groups are competing for the positions of the Board of Directors of a Corporation. The

sr
6.

probabilities that the first and the second groups will win are 0.6 and 0.4 respective y.
fo
Further, if the first group wins, the probability of introducing a new product is 0.7 and the
k
corresponding probability is 0.3 if the second group wins. Find the probability that the new
oo
product introduced was by the second group.
Suppose 5 men out of 100 and 25 women out of 1000 are good orators. An orator is chosen at
Y
7.

random. Find the probability that a male person is selected. Assume that there are equal
reB

number of men and women.


8. A letter is known to have come either from LONDON or CLIFTON. On the envelope just
uY

two consecutive letters ON are visible. What is the probability that the letter has come from
(i) LONDON (ii) CLIFTON?
ad
do

9. In a class, 5% of the boys and 10% of the girls have an IQ of more than 150. In this class, 60%
of the students are boys. If a student is selected at random and found to have an IQ of more
in

than 150, find the probability that the student is a boy.


A factory has three machines X, Y and Z producing 1000, 2000 and 3000 bolts per day
Re

10.

respectively. The machine X produces 1% defective bolts, Y produces 1.5% and Z produces
F

2% defective bolts. At the end of a day, a bolt is drawn at random and is found to be
defective. What is the probability that this defective bolt has been produced by machine X.
11. An insurance company insured 3000 scooters, 4000 cars and 5000 trucks. The probabilities
of the accident involving a scooter, a car and a truck are 0.02,0.03 and 0.04 respectively. One
of the insured vehicles meet with an accident. Find the probability that it is a (a) scooter (u)
car (iii) truck.
12. Suppose we have four boxes A, B, C, D containing coloured marbles as given below:
[arble Colour Red White Black
Box
1 6 3
A
6 2 2
B
8 1 1
C
6 4
D 0
20.140
APPLIED MATHEMATICS-Xl

One of the boxes has been selected at random and a single marble is drawn from it. If the
marble is red, what is the probability that it was drawn from box A ? box B ? box C ?
33. A manufacturer has three machine operators A, B and C. The first operator A produces 1%
defective items, whereas the other two operators B and C produce 5% and 7% defective
Items respectively. A is on the job for 50% of the time, B on the job for 30% of the time and C
on the job for 20% of the time. A defective item is produced. What is the probability that it
was produced by A? ^
14. An item is manufactured by three machines A, B and C. Out of the total number of items
manufactured during a specified period, 50% are manufacture on machine A, 30% on B and
20% on C. 2% of the items produced on A and 2% of items produced on B are defective and
3 /o of these produced on C are defective. All the items stored at one godown. One item is
drawn at random and is found to be defective. What is the probability that it was
manufactured on machine A?

ow
15. There are three coins. One is two-headed coin (having head on both faces), another is
i
biased coin that comes up heads 75% of the times and third is also a biased coin that comes
up tail 40% of the times. One of the three coins is chosen at random and tossed, and it shows
heads. What is the probability that it was the two-headed coin?
36.
In a factory, machine A produces 30% of the total output, machine B produces 25% and the

e
maclune C produces the remaining output. If defective items produced by machines A, B

Fl
re
and C are 1%, 1.2%, 2% respectively. Three machines working together produce 10000
herns m a day. An item is drawn at random from a day's output and

F
found to be defective,
rind the probability that it was produced by machine B?
ur
17.
A company has two plants to manufacture bicycles. Tlie first plant manufactures 60% of the
or
bicycles and the second plant 40%. Out of that 80% of the bicycles are rated of standard
quality at the first plant and 90% of standard quality at the second plant. A bicycle is picked
sf
up at random and found to be standard quality. Find the probability that it comes from the
k
Yo
second plant.
oo

18.
Three urns A, B and C contain 6 red and 4 white; 2 red and 6 white; and 1 red and 5 white
balls respectively. An urn is chosen at random and a ball is drawn. If the ball drawn is
B

found to be red, find the probability that the ball was drawn from urn A.
e

19. In a
group of 400 people, 160 are smokers and non-vegetarian, 100 are smokers and
vegetarian and the remaining are non-smokers and vegetarian. The probabilities of gettine
ur

a special chest disease are 35%, 20% and 10% respectively. A person is chosen from the
ad
Yo

group at random and is found to bo suffering from the disease. What is the probability that
20.
the selected person is a smoker and non-vegetarian?
A factory has three machines A, Sand C, which produce
d

100, 200 and 300 items of a


particular type daily. The machines produce 2%, 3% and 5% defective items respectively
Re
in

One day when the production was over, an item was picked up randomly and it was found
to be defective. Find the probability that it was produced by machine A.
F

21. A bag contams 1 white and 6 red balls, and a second bag contains 4 white and 3 red balls.
One of the bags is picked up at random and a ball is randomly drawn from it, and is found
to be white in colour. Find the probability that the drawn ball was from the first bag.
22. Ina certain college, 4% of boys and 1% of girls are taller than 1.75 metres. Furthermore, 60%
of the students in the colleges are girls. A student selected at random from the college is
found to be taller than 1.75 metres. Find the probability that the selected students is girl.
23. For A, B and C the chances of being selected as the manager of a firm are in the ratio 4T -2
respectively. The respective probabilities for them to introduce a radical change in
marketing strategy are 0.3, 0.8 and 0.5. If the change does take place, find the probability
that it is due to the appointment of 6 or C.
24. Three persons A, B and C apply for a job of Manager in a private company. Chances of their
selections (A, B and C) are in the ratio 1:2:4. The probabilities that A, B and C can introduce
changes to improve profits of the company are 0.8, 0.5 and 0.3 respectively If the changes
do not take place, find the probability that it is due to the appointment of C.
20.141
PROBABILITY

25. An insurance company insured 2000 scooters and 3000 motorcycles. The probability of an
accident involving a scooter is 0.01 and that of a motorcycle is 0.02. An insured vehicle met
with an accident. Find the probability that the accidented vehicle was a motorcycle.
26. Of the students in a college, it is known that 60% reside in a hostel and 40% do not reside in
hostel. Previous year results report that 30% of students residing in hostel attain A grade
and 20% of ones not residing in hostel attain A grade in their annual examination. At the
end of the year, one students is chosen at random from the college and he has an A grade.
What is the probability that the selected student is a hosteler?
27. There are three coins. One is two headed coin, another is a biased coin that comes up heads
75% of the time and third is an unbiased coin. One of the three coins is choosen at random
and tossed, it shows heads, what is the probability that it was the two headed coin?

ow
28. Assume that the chances of a patient having a heart attack is 40%. It is also assumed that
meditation and yoga course reduces the risk of heart attack by 30% and prescription of
certain drug reduces its chances by 25%. At a time a patient can choose any one of the two
options with equal probabilities. It is given that after going through one of the two options
and patient selected at random suffers a heart attack. Find the probability that the patient

e
re
followed a course of meditation and yoga?

rFl
Coloured balls are distributed in four boxes as shown in the following table:

F
29.

Box Colour

r
ou
Blue
Black White
fo Red
ks
I 3 4 5 6

2 2
oo

n 2 2
Y

1 2 3 1
III
eB

IV 4 3 1 5

A box is selected at random and then a ball is randomly drawn from the selected box. The
r

colour of the ball is black, what is the probability that ball drawn is from the box III.
ou
ad
Y

30. If a machine is correctly set up it produces 90% acceptable items. If it is incorrectly set up it
produces only 40% acceptable items. Past experience shows that 80% of the setups are
d

correctly done. If after a certain set up, the machine produces 2 acceptable items, find the
Re
in

probability that the machine is correctly set up.


Bag A contains 3 red and 5 black balls, white bag B contains 4 red and 4 black balls. Two
F

31.
balls are transferred at random from bag A to bag B and then a ball is drawn from bag B at
random. If the ball drawn from bag B is found to be red, find the probability that two red
balls were transferred from bag A to bag B.
32. By examiiung the chest X-ray, probability that T.B is detected when a person is actually
suffering is 0.99. The probability that the doctor diagnoses incorrectly that a person has T.B.is
on the basis of X-ray is 0.001. In a certain city 1 in 1000 persons suffers from T.B. A person '
selected at random is diagnosed to have T.B. What is the chance that he actually has T.B.?
33. A test for detection of a particular disease is not fool proof. The test will correctly detect the
disease 90% of the time, but will incorrectly detect the
disease 1% of the time. For a large
population of which an estimated 0.2% have the disease, a person is selected at random,
given the test, and told that he has the disease. What are the chances that the person
actually have the disease?
20.142 APPLIED MATHEMATICS-XI

34.
Let di,d2, ^3 be three mutually exclusive diseases. Let Sbe the set of observable symptoms
of these diseases. A doctor has the following information from a random sample of 5000
patients: 1800 had disease dp 2100 has disease ^2 ^rid the others had disease d^ ■
1500 patients with disease dy 1200 patients with disease ^2 s^id 900 patients with disease d^
showed the symptom. Which of the diseases is the patient most likely to have?
35. A is known to speak truth 3 times out of 5 times. He throws a die and reports that it IS one.

Find the probability that it is actually one.


3b. A speaks the truth 8 times out of 10 itmes. A die is tossed. He reports that it was 5. What is
the probability that it was actually 5?
37.
In answering a question on a multiple choice test a student either knows the

ow
answer or

guesses. Let 3/4 be the probability that he knows the answer and 1/4 be the probability
that he guesses. Assuming that a student who guesses at the answer will be correct with
probability 1/4. What is the probability that a student knows the answer given that he
answered it correctly?

e
38.
A laboratory blood test is 99% effective in detecting a certain disease when its infection is

re
rFl
present. However, the test also yields a false positive result for 0.5% of the healthy person
tested (i.e. if a healthy person is tested, then, with probability 0.005, the test will imply he

F
has the disease). If 0.1% of the population actually has the disease, what is the probability
that a person has the disease given that his test result is positive?

r
ou
39.
There are three categories of students in a class of 60 students:
fo
ks
A : Very hardworking ; B : Regular but not so hardworking; C : Careless and irregular
10 students are in category A, 30 in category B and rest in category C. It is found that the
oo

probability of students of category A, unable to get good marks in the final year
examination is 0.002, of category B it is 0.02 and of category C, this probability is 0.20. A
Y
B

student selected at random was found to be one who could not get good marks in the
examination. Find the probability that this student is of category C.
re
ou

ANSWERS
Y
ad

33 55 30 25 2 1
1. 2. 3. 4. 5. —
118 ' 118 ' 118 52 9 40 11
d

2 2 12 5
6. 7. (i) ^ (ii) 9. 3
in

8.
Re

9 3 17 17 7
F

0.1 r\ ^ r-\ ^ 10 1 2 8 5
10. 11. 0) — (h) - (iii) 12. 13.
19 19 19 15'5'15 34
5 20 3 36
14. 15. 16. 0.2 17. 18. —
11 47 7 61
28 2 1 3 3
19. 20. 21. 22. 23. -
45 23 5 11 5
7 3 9 4 14
24. 25. 26. 27. 28. —
10 4 13 9 29
156 81 18 110 90
29. 30. 31. 32. 33.
947 85 133 221 589
3 4 12 22 20
34. 35. 36. 37. — 38. 39.
13 9 13 133 231
20.143
PROBABILITY

HINTS TO SELECTED PROBLEMS

I. Let us define the following events E-j = Urn I is chosen, £2 - Urn II is chosen, £3 ~ Urn III is
chosen, and A = Two balls drawn are white and red.
Clearly, P(£i) =1/3 =£(£3) =£(£3)'
4/^ 3/^
1
Cix -’c, X ^Ci and, £ (A/E3) =
^1 ^
P(d/£i) = £ (A/E2) = 12
6/~
^2 ■'C2 C2
P(Ej)P(A/Ei)
P(E:/A) =
£ (£1) £ (v4/£i) + £ (£2) £ (A / £2) + £ (£3) P (21/£3)

w
2. Let £1 = Bag A is chosen, £3 = Bag B is chosen, and A = Ball drawn is red.
£(£3) =£(£2) =1/2, £ (A/£i) = 3/5 and £ (A/E2) =5/9.
5. Consider the following events:

o
£^ = Getting 1 or 2 in a throw of a die, £2 = Getting 3, 4, 5 or 6 in a throw of a die

e
re
A = Getting exactly one tail.
Clearly, £(£i) = ^ £(£2) = |-£('^/£i) ^
Frl
F
2 1
X

£(£2) P{A/E2) 3 2
ou
Required probability = £(£2/A) = 1 3 2 1 11

r
£(£l) £(A/Ei) + £(£2)£(A/£2) X + X

so
3 8 3 2

(1. Consider the following events:


kf
Ej = First group wins, £3 = Second group wins, A =New product is introduced.
oo

It is given that
Y

£(£i) = 0.6, £ (£2) = 0.4, £(d/£i) = 0.7, £ (A/E2) = 0.3


B

^ £(£2)£(A/£2)
Required probability = P {Ej/A) = £C£l)P(A/£i) + £(E2)£(2l7£2)
re

0.4 X 0.3 12 2
oY
u

0.6x0.7 + 0.4x0.3 54 9
ad

7. Let Ej = Person chosen is a man, £2 = Person chosen is a woman, A = Person is a good


d

orator.
1 25
in

— and £ (A/E2) = 1000


£(£;) = £ (£2) = -- £(A/£i) = 100
Re

Now, find £ (£iA) by using Bay's theorem.


F

12. Let Ej, £2, £3, £4 and A be the events as defined below.
El =BoxAisselected,£2 = Box Bis selected, £3 = Box C is selected, £4 =BoxD is selected.
A = Marble drawn is of red colour.
Since there are four boxes and one of them is selected at random. Therefore,

£(£i) = £(E2) = £(£3) = ^(^4)-^-


Clearly, £ (A/E^) = p i^/Ei) i^/^3) ^ ^
£ (£])£ (71/El)
£(£i/A) =
£ (£1) £ (A/£i) + £ (£2) £ (A/£2) + £ (£3) £ (t1/£3) + £ (£4) P (7I/E4)
1 1
X
1
4 10
1 1 1 6 1 8 1 0 15
+ X

4 10 4 10'*'4 ^10 4 10
X + X
20.144
APPLIED MATHEMATICS-XI

P{E2/A) = P{E2)P{A/E2)
P (£i) P (A/£i) + P (£2) P {A/E2) + P (£3) P (z4/£3) + P (£4) P (/\/£4)
1 6
X -
4 10 2
1 1 1 6 1
X + X + X
8
+
1
- X -
0 15 "5
4 10 4 10 4 10 4 10

P{£3/A) = P{E^)P{A/E^)
P (£i) P (^/£|) + P (£2) P (A/£2) + P (£3) P (A/£3) + P (£4) P (yl/£4) 15
13. Let £], £2, £3 and /4 be the following events;
£1 = Operator A performs the job, £2 = Operator B performs the job,
£3 = Operator C performs the job, A = Item produced is defective

low
Clearly, P (E^) = 20 _ 2
100 10 ^ 100 10 100 "10
1
£(/i/£i) =
100'
Required probability = P (£^/A)

ee
rF P(£l)P(A/£i)

Fr
P (£1) P (A/Ej) + P {£2) P (A/£2) + P (£3) P (A/£3)
5 1

5 1
10
3
X
100
5 2 7
for 5

5 + 15+14
5
34
u
X + X
10 100 10 100 10 100
ks
16. Let Ey £2, £3 be the events as defined below:
Yo
o

£] = Item is produced by machine A, £2 = Item is produced by machine B


Bo

£3 = Item is produced by machine C and, A = The event that the item is defective.
30
re

25 45
It is given that, P (£j) =
100' £(£2)
^
= —, £(£3) =
100 ^ 100'
ou

1.2
ad

P(^/£]) = p(a/E2) = 100


andP(/\/£3) = —
100
Y

Required probability = P{E2/A) = P(E2)P(A/E2)


P (£1) P (A/E^) + P (£2) P (A/£2) + P (£3) P (A/Ej) ■
nd
Re

27. Consider the following events:


Fi

£1 = Selecting two headed coin, £2 = Selecting biased coin,


£3 = Selecting unbiased coin, A = Getting head on the coin.
Clearly, P (£^) = £ (£2) = P (£3) = 1, p(^a/E^)=1, P (A/E2) =
75
100 |,PW£3) = i
P(£QP(A/£i)
Required probability = P (Ej/A) =
P(£i)P(/1/£i)+P(£2)P(A/E2)
28.
Consider the following events:
El = The patient follows a course of meditation and yoga,
£2 = The patient takes a certain drug, A = The patient suffers a heart attack.
Clearly,
40 75 40
and P (A/£2) =
100 100
20.145
PROBABILITY

P(£i)P(A/£i)
Required probability = P A) =
P(£i)P(A/£i) + P(£2)P(A/E2)
1 70 40
— X
14
2 100 100
1 70 40 1 75 . 40 29
— X + - X
2 100 100 2 100 100
30. Let A be the event that the machine produces 2 acceptable items. Let £^ represent the event
of correct setup and £2 represent the event of incorrect setup.
It is given that
P (£0 = 0.8, P (£2) = 0.2, P (A/£i) = 0.9 x 0.9, P (A/£2) = 0.4 x 0.4
^ P(£l)P(A/£i)
Required probability = P {Ei/A) = P(£i)P(A/£i) + P(£2)P(A/£2)

ow
32. Let El = The person selected is suffering from T.B
£2 = The person selected is not suffering from T.B, A = The doctor diagnoses correctly.
Then, P(£j) =1/1000, £(£2) =999/1000, P (A/£i) = 0.99 and P (A/£2> = 0.001

e
P (£i) P (A/£i)

Fl
re
Required probability = P (£i/A) = P(£)P(A/£i) + P(£2)P(A/£2)

F
33. Let£j,£2 and A be the following events:
ur
= The person selected has disease, £2 = The person selected does not have disease,
A = Test is positive. or
sf
It is given that
k
P(£i) = 0.002, £(£2) = 0.998, P (A/£i) = 0.90, P (A/£2) = 0.01.
Yo
oo

P(£l)P(A/£i)
.-. Required probability = P(£i/A) = P(£i)P(A/£i) + P(£2)P(A/£2)
B

34. Let Ej denote the event that the patient has disease d,-; / = 1, 2, 3 and A be the event that
e

the patient showed the symptom S.


ur

1800 2100 1100


Clearly, P(£i) = 5000' £(£2) = 5000' £(£3) =
ad

5000
Yo

P(A/£i) = = '500/5000 ^15 ^ 12 p ,^/p , ^ 9


P(£l) 1800/5000 18 21 H
d
Re

Compute P (£i/A), P (£2/A) and P {E^/A) and find the greatest of these.
in

37. Consider the following events:


F

El = Student knows the answer, £2 = Student guesses the answer,


A = Student answers correctly.
Clearly,
P(£l) = -, P (£2) = I, P (21/E2) = i, P (/l/£i) = 1
4 4 4
3 X 11
12
P(E,)P(A/Ei) 4
Required probability = P (S^/A) =
P (£1) P (/t/£i) + P {£2) P (A/E2) 4
3 X j ^ 14 ^ 14 13
38. Consider the following events:
£;^ = Person selected has the disease, £2 = Person selected does not have the disease,
A = Test result is positive
Clearly,
20.146
APPLIED MATHEMATICS-XI

1 999 99
P(£l) = £(£2) = P(A/E{} = and P{A/E2)=~
1000 ' 1000 ' 100 1000
P(£l)P(/l/£l)
Required probability = P iE^/A) =
P(£i)P(^/£i) + P(£2)P(/1/£2)
MULTIPLE CHOICE QUESTIONS (MCQs)
Mark the correct alternative in each of the following:
1. One card is drawn from a pack of 52 cards. The probability that it is the card of a king or
spade, is
(a) 1/26 (b) 3/26 (c) 4/13 (d) 3/13
2. Two dice are thrown together. The probability that at least one will show its digit greater

w
than 3, is

(a) 1/4 (b) 3/4 (c) 1/2 (d) 1/8

Flo
.1. Two dice are thrown simultaneously. The probability of obtaining a total score of 5, is

e
(a) 1/18 (b) 1/12 (c) 1/9 (d) none of these

re
4. Two dice are thrown simultaneously. The probability of obtaining total score of seven, is

rF
(a) 5/36 (b) 6/36 (c) 7/36 (d) 8/36
ur
r>.
The probability of getting a total of 10 in a single throw foof two dice, is
(a) 1/9 (b) 1/12 (c) 1/6 (d) 5/36
ks
6.
A card is drawn at random from a pack of 100 cards numbered 1 to 100. The probability of
Yo
oo

drawing a number which is a square, is


(a) 1/5
B

(b) 2/5 (c) 1/10 (d) none of these.


re

7. A bag contains 3 red, 4 white and 5 blue balls. All balls are different.Two balls are drawn at
random. The probability that they are of different colour, is
u
ad

(a) 47/66 (b) 10/33 (c) 1/3


Yo

(d) 1
8.
Two dice are thrown together. The probability that neither they show equal digits nor the
sum of their digits is 9 will be
d
Re
in

(a) 13/15 (b) 13/18 (c) 1/9 (d) 8/9


F

9.
Four persons are selected at random out of 3 men, 2 women and 4 children. The probability
that there are exactly 2 children in the selection, is
(a) 11/21 (b) 9/21 (c) 10/21 (d) none of these
1(1. The probabilities of happening of two events A and B are 0.25 and 0.50 respectively. If the
probability of happening of A and B together is 0.14, then probability that neither A nor B
happens, is
(a) 0.39 (b) 0.25 (c) 0.11 (d) none of these
I
1. A die is rolled, then the probability that a number 1 or 6 may appear, is
(a) 2/3 (b) 5/6 (c) 1/3 (d) 1/2
12. Six boys and six girls sit in a row randomly. The probability that all girls sit together, is
(a) 1/122 (b) 1/112 (c) 1/102 (d) 1/132
20.147
PROBABILITY

13. The probabilities of three mutually exclusive events A, B and C are given by 2/3,1/4 and
1/6 respectively. The statement
(a) is true (b) is false (c) nothing can be said (d) could be either
14. If
fl-3p) (l+4p) (1+p) are the probabilities of three mutually exclusive and
2 3 6

exhaustive events, then the set of all values of p is


(a) (0,1) (b) (-1/4,1/3) (c) (0,1/3) (d) (0,co)

15. A pack of cards contains 4 aces, 4 kings, 4 queens and 4 jacks. Two cards are drawn at
random. The probability that at least one of them is an ace, is
(c) 9/20 (d) 1/9
(a) 1/5 (b) 3/16

ow
16. If three dice are throw simultaneously, then the probability of getting a score of 5 is
(b) 1/6 (c) 1/36 (d) none of these
(a) 5/216
17. One of the two events must occur . If the chance of one is 2/3 of the other, then odds in
favour of the other are

e
(b)3:l (c) 2 : 3 (d) 3:2
(a) 1 :3

Fl
re
18. The probability that a leap year will have 53 Fridays or 53 Saturdays, is

F
(b) 3/7 (c) 4/7 (d) 1/7
2/7 ,. u 1
A person write 4 letters and addresses 4 envelopes. If the letters are placed m the envelopes
ur
or
19.
at random, then the probability that all letters are not placed in the right envelopes, is
sf
(a) 1/4 (b) 11/24 (c) 15/24 (d) 23/24
20. A and B are two events such that P (A) = 0.25 and P (B) = 050. The probability of both
k
Yo

happening together is 0.14. The probability of both A and B not happening, is


oo

(a) 0.39 (b) 0.25 (c) O.n (d) none of these


B

21. If the probability of A to fail in an examination isis -^ and that of 6 is —. Then, the probabiUty
re

10

that either A or B fails, is


u

(d) none of these


ad

(a) 1/2 (b) 11/25 . (c) 19/50


Yo

is drawn at random. The


22. A box contains 10 good articles and 6 defective articles. One item is
probability that it is either good or has a defect, is
d

(c) 40/64 (d) 24/64


Re

(a) 64/64 (b) 49/64


in

23. Three integers are chosen at random from the first 20 integers. The probability that their
F

product is even is
(b) 3/29 (c) 17/19 (d) 4/19
(a) 2/19
24. Out of 30 consecutive integers, 2 are chosen at random. The probability that their
sum IS

odd, is
(b) 16/29 (c) 15/29 (d) 10/29
(a) 14/29
ins 5. black balls, 4 white balls and 3 red balls. If
25. A bag contains a ball is selected randomwise.
the probability that it is black or red ball is
(b) 1/4 (c) 5/12 (d) 2/3
(a) 1/3
26. Two dice are thrown simultaneously. The probability of getting a pair of aces is
(b) 1/3 (c) 1/6 (d) none of these
(a) 1/36
27. An urn contains 9 balls two of which are red, three blue and four black. Three balls are
of the same colour is
drawn at random. The probability that they are

(b) 3/9 (c) 3/7 (d) 7/17


(a) 5/84
20.148
APPLIED MATHEMATICS-XI

28. Five persons entered the lift cabin on the ground floor of an 8 floor house. Suppose that
each of them independently and with equal probability can leave the cabin at any floor
beginning with the first, then the probability of all 5 persons leaving at different floor is
%
(a) ^
75 (b) ^P5 (c) ^ (d)
55
29. A box contains 10 good articles and 6 with defects. One item is drawn at random. The
probability that it is either good or has a defect is
(a) 64/64 (b) 49/64 (c) 40/64 (d) 24/64
30. A box contains 6 nails and 10 nuts. Half of the nails and half of the nuts are rusted. If one
Item is chosen at random, the probability that it is rusted or is a nail is

ow
(a) 3/16 (b) 5/16 (c) 11/16 (d) 14/16

31. If S is the sample space and R (A) = 1 P (B) and S = A u B, where A and B are two mutually

exclusive events, then P (A) =


(a) 1/4 (b) 1/2 (c) 3/4 (d) 3/8

e
32. If A and B are independent events such that 0 < P(A) < 1 and 0 < P(B) < 1, then which of the

re
Fl
following is not correct?

F
(a) A and 6 are mutually exclusive (b) A and B are independent
(c) A and 6 are independent
ur
(d) A and B are independent

r
mutually exclusive and exhaustive events of an experiment such
fo that
3 P (A) -2 P (B) - P (C), then P (A) is equal to
ks
(a) 1/11 (b) 2/11 (c) 5/11 (d) 6/11
Yo
34. Let A and S be two events such that P(A) 0.6, P(B) = 0.2 and P(A/B) = 05. Then P(A/B)
oo

equals
(a):^
eB

(b) (d) r
10 10 7
35. Let A and Bbe two events. If P(A) = 0.2, P(B) =0.4, F(A u B) = 0.6, then P(A/B) is equal to
ur

(a) 0.8 (b) 0.5 (c) 0.3 (d) 0


^
ad

Three186numbers are chosen from 1 to 20. The probability that they are not
Yo

consecutive is
187 188 18
(a) (b) (c)
190 190 (d) 20
190
d

C3
Re

37.
6 boys and 6 girls sit in a row at random. The probability that all the girls sit together i
in

IS

7^ (c) — (d) None of these


F

.38.
Without repetition of the numbens, four digit numbers are formed with the numbers 0 2 3
5. The probability of such a number divisible by 5 is ' ' '
(a) 1/5 (b) 4/5 (c) 1/30 (d) 5/9
If the probability for A to fail in an examination is 0.2 and that for B is 0 3 then the
probability that either A or B fails is
(a) > 0.5 (b) 0.5 (c) <0.5 (d) 0
Three digit numbers are formed using the digits 0, 2,4, 6,8. A number is chosen at random
out of these numbers what is the probability that this number has the same digits’
(a) 1/16 (b) 16/25 (c) 1/645 (d) 1/25
41.
In a leap year the probability of having 53 Sundays and 53 Mondays i
is
2 ^ , 3 4 n;
(a)
7
(b)
7
(C) 37 (d) ^
7
PROBABILITY 20.149

42. Assume that in a family, each child is equally likely to be a boy or a girl. A family with three
children is chosen at random. The probability that the eldest child is a girl given that the
family has at least one girl is

(a) I (b)i
1 2 —
43. If A and B are any two events having P{A vj 6) = — and P{A) = — / then probability of A n B
IS

(d)i
(a)i (b)|
44. A die is thrown and a card is selected at random from a deck of 52 playing cards. The
probability of getting an even number of the die and a spade card is

w
(a) i (b) 1 (01 (d) I
45. If A and B are two events, the probability that at least one of them occurs is

Flo
(a) P(A) + F(B)-2P(AnB) (b) P{A) + P{B)-P {AnB)
(c) P(A) + P(B) + P{AnB) (d) P(A) + P(B) + 2P{AnB)

e
A single letter is selected at random from the word 'PROBABILITY. The probability that it

re
46.
is a vowel, is

F
(b) ^
11
(c) ^
11
(d)^
11
ur
r
47. Seven persons are to be seated in a row. The probability that two particular persons sit next
to each other, is
fo
ks
(b)l (c)f (d)i
Yo
oo

48. The probability that at least one of the events A and B occurs is 0.6. If A and B occur
B

simultaneously with probability 0.2, then P(A) + P (B) =


(b) 0.8 (c)1.2 (d) 1.6
re

(a) 0.4
49. If one ball is drawn at random from each of three boxes containing 3 white and 1 black, 2
white and 2 black, 1 white and 3 black balls, then the probability that 2 white and 1 black
u
ad
Yo

balls will be drawn is

(a) — (b)l (c) ^


32
(d):^
16
32
d

from a pack of well-shuffled pack of 52


Re

50. A and B draw two cards each, one after another,


in

cards. The probability that all the four cards drawn are of the same suit is
13 X 24 none of these
F

44 11
(a) (b) (c)
85x49 85x49 17 X 25 X 49

51. A and B are two events such that P (A) = 0.25 and P (B) = 050. The probabUity of both
happening together is 0.14. The probability of both A and B not happening is
(a) 0.39 (b) 0.25 (c) 0.11 (d) none of these^ ^ ^
52. The probabilities of a student getting I, II and III division in an examination are —, - and -4
respectively. The probability that the student fails in the examination is
197 27 83
(a) (b) (c) (d) none of these
200 ' ' 100 ' 100
53. India play two matches each with West Indies and Australia. In any match the probabilities
of India getting 0,1 and 2 points are 0.45, 0.05 and 0.50 respectively. Assuming that the
outcomes are independent, the probability of India getting at least 7 points is
(a) 0.0875 (b) 1/16 (c) 0.1125 (d) none of these
20.150 APPLIED MATHEMATICS-XI

54. Three faces of an ordinary dice are yellow, two faces are red and one face is blue. The dice is
rolled 3 times. The probability that yellow red and blue face appear in the first second and
third throws respectively, is
1 1
(a) (c) ^ (d) none of these
36 30
1
55. If A and Bare two events such that P(A/B) =p, P(A) =p, P(B) = - and P(A uB) = —.
—, then
3 9
P =

(b)| (d) 44

w
56. A person writes 4 letters and addresses 4 envelopes. If the letters are placed in the
envelopes at random, then the probability that all letters are not placed in the right
envelopes, is

o
(a)i 11

e
15 23
(b) ^ (c) ^ (d) ^

re
24 24 24

^9. A speaks truth in 75% cases and 6 speaks truth in 80% cases. Probability that they

Frl
F
contradict each other in a statement, is
7 13
(a) ^ (b)
20 20
ou
r
58. Three integers are chosen at random from the first 20 integers. The probability that their
product is even is
so
kf
(a) ^
19
(b) (c) ^
17
(d).^19
29 19
oo

59. Out of 30 consecutive integers, 2 are chosen at random. The probability that their sum IS
Y

odd, is
B

14 16 15 10
(a) (b) (c) (d)
29 29 29 29
re

60. A bag contains 5 black balls, 4 white balls and 3 red balls. If a ball is selected randomwise,
oY
u

the probability that it is black or red ball is


ad

(a)l << (c) I12 (a) I


d

61. Two dice are thrown simultaneously. The probability of getting a pair of aces is
in

(B)i
Re

(a) (d) none of these


36
F

62. An urn contains 9 balls two of which are red, three blue and four black. Three balls are

drawn at random. The probability that they are of the same colour is

(a) 84 (b) I (c) I (d) L 17


63. A coin is tossed three times. If events A and B are defined as A -Two heads come, 6 = Last
should be head. Then, A and 6 are
(a) independent (b) dependent (c) both (d) mutually exclusive
64. Five persons entered the lift cabin on the ground floor of an 8 floor house. Suppose that
each of them independently and with equal probability can leave the cabin at any floor
beginning with the first, then the probability of all 5 persons leaving at different floors is
(a)
%
(d)
yD % 5^
20.151
PROBABILITY

65. A box contains 10 good articles and 6 with defects. One item is drawn at random. The
probability that it is either good or has a defect is 24
64 49 40
(a)^ (b) ^ (c)^ (d) ^
64 64 64
64
66. A box contains 6 nails and 10 nuts. Half of the nails and half of the nuts are rusted. If one
item is chosen at random, the probability that it is rusted or is a nail is
11 14
(a) A
16
(b)A
16
(c)ff
16
(d)
16

67. A bag contains 5 brown and 4 white socks. A man pulls out two socks. The probability that
these are of the same colour is
48

ow
18 30
(a) tI
108
(b)
108
(c)
108
(d)
108

6 S. If S is the sample space and P (A) = ^ P (B) and S = A u B, where A and B are two muluaUy

e
exclusive events, then P (A) =
(d) 3/8

re
(a) 1/4 (b) 1/2 (c) 3/4

rFl
69. If A and B are two events, then P {A nB) =

F
(a) P(A)P(B) (b) 1-P(A)-P(B)
(c) P(A) + P(B)-P(AnB) (d) P (B) - P (A n B

or
ou
70. If P (A u B) = 0.8 and P (A n B) = 0.3, then P (A ) + P (B) =
ksf
(a) 0.3 (b) 0.5 (c) 0.7 (d) 0.9
71. A bag X contains 2 white and 3 black balls and another bag Y contains 4 white and 2 black
oo

balls. One bag is selected at random and a ball is drawn from it. Then, the probability
Y

chosen to be white is
B

(b) 7/15 (c) 8/15 (d) 14/15


(a) 2/15 . -
72. Two persons A and B take turns in throwing a pair of dice. The first person to throw 9 from
re

both dice will be awarded the prize. If A throws first, then the probability that B wins the
oYu

game is
ad

(a) 9/17 (b) 8/17 (c) 8/9 (d) 1/9


73. The probability that in a year of 22nd century chosen at random, there will be 53 Sundays, is
d

(a) 3/28 (b) 2/28 (c) 7/28 (d) 5/28


in
Re

74. From a set of 100 cards numbered 1 to 100, one card is drawn at random. The probability
F

that the number obtained on the card is divisible by 6 or 8 but not by 24 is


(a) 6/25 (b) 1/4 (c) 1/6 (d) 2/5

75. If A and B are two events such that P( A) = and P(A n B) = then P(B/A) =
(d) 17/20
(a) 1/10 (b) 1/8 (c) 7/8 ^
76. If A and Bare two events associated to a random experiment such that P(A n B) = — and

P(B) =17/20, then P(A/B) =


(a) 14/17 (b) 17/20 (c) 7/8 (d) 1/8
77. Associated to a random experiment two events A and B are such that P(B) - 3/5,
P(A/B) =1/2 and P(A u B) = 4/5. The value of P(A) is
(a) 10 (b) ^ (c)
^
10 (a)|
20.152
APPLIED MATHEMATICS-XI

78. If P{A) = 3/10, P(B) =2/5 and P{A^B) = 3/5, then P{A/B) + P(B/A) equals
(a) 1/4 (b) in (c) 5/12 (d) 1/3
79. LetP(/l) =7/13, P(B) =9/13 and P(7l nB) = 4/13. Then, P{AjB) =
(a) 5/9 (b) 4/9 (c) 4/13 (d) 6/13

80. If P{A) = |, P{B) = A and P{A n B) = then P(Z/6) P{B/A) is equal to


(b) I
25
(c)^ (d) 1
42

81. If/I and Bare two events such that P(B) = P(A/B)=~, then P(AnB) equals

(b)|
1

ow
(a) ~ (c)
12 4 4

82. Let A and 6 be two events such that P(A) = —, P{B) =—o and P(AuB) = 4^. Then P(A/B)
o
P{A/B) is equals to

e
(b) T (c)^ (d) A

re
20

S3. liP(B)=~,P(A/B)=^
rFl
and P(A^B)=^,5 then P(Au~B) + P(A w B) =
25

F
j I

(a)l 4
(0-
1
(d) 1

or
ou
84. If P{A) = 0.4, P(6) = 0.8 and P(B/A) = 0.6, then P(A u B)
ksf
(a) 0.24 (b) 0.3 (c) 0.48 (d) 0.96
85. If P(B) = ~,P{A/B) = i2 and P(A u B) = -,5 then P{B/A) =
oo

D
Y

(a)l (b) /10 1


B

(0-
(d) I
re

86. If A and B are two events such that P(A) = 0.4, P(6) = 0.3 and P{A u B) = 05, then P(BnA)
equals
oYu

.)f (b)^
ad

1
(c) — (d)-
10

87. If A and B are two events such that A:^<^,B = ^


d

then
P(A n B)
(a) P(A/B) =
in
Re

P(B)
(b) P{A/B)=P(A) P(B)
F

(c) PiA/B)==P{B/A)=l P(A)


(d) P{A/B) =
P(B)
88. If A and Bare two events such that P(A) 0 and P(B) ^ 1, then P{A/B)
(a) 1-P(A/B) 1-P(A uB) P{A)
(b) 1-P(A/B) (c) (d)
P(B) P(B)
89. If the events A and B are
independent, then P{A r\ B) is equal to
P{A)
(a) P{A) + P(B) (b) P(A)-P{B) {c)P(A)P{B) (d)
P(S)

9(1. If A and B are two independent events with P{A) = and P(B) = 1, then P{A n B) equals
(a) ^
15
(b)
1
45
20.153
PROBABILITY

91. If ^ and B are two independent events such that P(/\)=0.3, P(AuB) =0.5, then
P{A/B)-P{B/A) =
1
(b) ^35 (c) ^
(a) f 70 (d)^
92. A flash light has 8 batteries out of which 3 are dead. If two batteries are selected without
replacement and tested, the probability that both are dead is 33
'X 1 9
(a) A (b) ^
14
(c) ^
64
(cl) ^
56

93. A bag contains 5 red and 3 blue balls. If 3 balls are drawn at random without replacement,
then the probability of getting exactly one red45ball is 135
15 15
(a) — (b) (c) (d)
56 196 392
29

ow
94. A bag contains 5 red and 3 blue balls. If 3 balls are drawn at random without replacement,
then the probability that exactly two of the three balls were red, the first ball being red, is
(a) i (b) I (c) II 28 (d) I:28

In a college 30% students fail in Physics, 25% fail in Mathematics and 10% fail m both. One

e
95.
student is chosen at random. The probability that she fails in Physics if she has failed in

re
Mathematics is

Fl
F
(a) —
1
(d)^20
ur
10 .

r
96. Three persons. A, B and C fire a target in turn starting with fo A.Their probabilities of hitting
the target are 0.4,0.3 and 0.2 respectively. The probability of two hits is
ks
(a) 0.024 (b) 0.452 (c) 0.336 (d) 0.138
Yo
-1 and
... ^-
two students. Their chances of solving a problem correctly are
oo

97. A and B are 3 4

error is ^ and they obtain the same


eB

respectively. If the probability of their making common


answer , then the probability of their answer to be correct is
(d) ^
ur

10
(b)
13
(c)^
40 12
ad

13 120
Yo

98. Two cards are drawn from a well shuffled deck of 52 playing cards with replacement. The
probability that both cards are queen is
d

kh
Re

(a)
in

99. A box contains 3 orange balls, 3 green balls and 2 blue balls. Three baUs are dnawn at
F

random from the box without replacement. The probability of drawing 2 green balls and
one blue ball is

(a)
167
(b) ^ (c)
21
(d)A
28
168 28

100. If two events arc independent, then


(a) they must be mutually exclusive
(b) the sum of their probabilities must be equal to 1
(c) (a) and (b) both are correct
(d) none of the above is correct
101. Two dice are thrown. If it is known that the sum of the numbers on the dice was less than 6,
the probability of getting a sum 3, is
(a)
1
.T
18
... 5
(b) -
18 «5 (d)|
20.154
APPLIED MATHEMATICS-Xl

102. If A and B are such that P(A u 6) =


10
^ and P(A uB) =-^, then P(A) + P(B) =
(a)
10
(b)- (d)^
ANSWERS

1. (c) 2. (b) 3. (c) 4. (b) 5. (b) 6. (c) 7. (a) 8. (b) 9. (c)


10. (a) 11- (c) 12. (d) 13. (b) 14. (b) 15. (c) 16. (c) 17. (d) 18. (b)
19. (d) 20. (a) 21. (c) 22. (a) 23. (c) 24. (c) 25. (d) 26. (a) 27. (a)
28. (a) 29. (a) 30. (c) 31. (a) 32. (a) 33. (b) 34. (a) 35. (d) 36. (b)
37. (c) 38. (d) 39. (c) 40. (d) 41. (b) 42. (d) 43. (c) 44. (c) 45. (a)

ow
46. (b) 47. (c) 48. (c) 49. (a) 50. (a) 51. (a) 52. (b) 53. (a) 54. (a)
55. (c) 56. (d) 57. (a) 58. (c) 59. (c) 60. (d) 61. (a) 62. (a) 63. (b)
64. (a) 65. (a) 66. (c) 67. (d) 68. (a) 69. (d) 70. (d) 71. (c) 72. (b)
73. (d) 74. (a) 75. (c) 76. (a) 77. (b) 78. (b) 79. (a) 80. (c) 81. (c)

e
82. (d) 83. (d) 84. (d) 85. (d) 86. (d) 87. (a) 88. (c) 89. (c) 90. (d)

re
91. (c) 92. (a) 93. (b) 94. (b)

rFl
95. (c) 96. (d) 97. (a) 98. (a) 99. (d)

F
100. (d) 101. (c) 102. (b)

F/LL IN THE BLANKS TYPE QUESTIONS (FBQs)

r
ou
fo
I. Let S = {1, 2, 3, 4, 5, 6) be tiw sample space associated to a random experiment and
A = IL 3,5} be an event, then A =
ks
2. If £i,£2'^3'^4 elementary events associated to a random experiment such that
oo

m) P(E2)-\. then P(E,)=


Y
eB

3. The probability of happening of an event A is 0.5 and that of B is 0.3. If A and 6 are

mutually exclusive events, then the probability of neither A nor B is


ur

4. If A and B are two events associated with a random experiment such that
ad

P(A) ~ 0.3, P(B) = 0.2 andP(4 nS) = 0.1, then the value of P(/4 n 6) =
Yo

5. IfP(A) =0.4, P(7l u6) =0.7 and the events are mutually exclusive, then P(B) =
6. If A and B are two events, then the probability of occurrence of exactly one of / and B
d
Re
in

is

7. If A and B are two events, then theprobability of occurrence of/I only is


F

1 7
8. If A and 6 are two events such that P(A ^ B) =-, P (A r\B) = -,P(71) then P{A nB)
IS

9. If A and 6
are two events such that p {Au B) + P (A nB) =- and P{A) = 2 P(B), then
8
P(A)=

i PMuB)=|
1
10. If AandB are two events such that P(AnB) and P (B) = —, then
j 6 3
P(A) =
11. If A and B are independent events, then P (A uB) =1 AT, where .v =
12. If A and 6 are independent events such that P(A) =p, P{B)=2p and P(Exactly one of
A, B) = —, then p =
20.155
PROBABILITY

13. If A and B are two events such that P (A u B) = -3 and P(A u B) = —


9,
then P (A) + P {B) =

14. If A and 6 are two events such that P(A/B) = P(A)-p, P{B) - -3 and P{A^B) 9
then

P =
of B.
15. Let A and B be two events. If P(A/B} =P(A), then A is
l-k
then
16. Let A and B be two events such that P (A) ^ 0, P (B) 1 and P(A/B) = P(B)'

k=.

17. If two events A and 6 are mutuall); exclusive, then P(A/B)

ow
18. If A and 6 are two events such that Ac B, then P(B/A) = ..
19. If 4 P(A) = 6 P( 6) = 10 P(A n B) = 1, then P (B/A) =
20. If A and B are two events, then P (A n B) =

e
21. If A and 6 are two events. then the probability of occurrence of A only is equal to

re
Fl
F
22. If A and B are two events, then the probability of occurrence of exactly one of A and B is
ur
equal to.
event A and B, if P(A) = P{A/B)
r
and P(B/A)=^, then A and B
23. For two fo are
ks
events. _
Yo

. Let A and B be two events for which P{A)=a, P(B) ^B,P(A nB) =c, then P(A nB) =
oo

24
eB

25. Let A, B, C be pairwise independent events with P(C) > 0 and P (A n B n C) = 0. If


P({A n B)/C) =1 - J, then .v
ur

26 LetA B CbethreeeventssuchthatP(A nBnC)=0,P(ExactlyoneofAandB occurs) = .v,


P (exactly one of B and C occurs) = i/, P(Exactly one of A and C occurs) = 2. Then
ad
Yo

P(AuBuC) =
27. If A and B are two events such that P(AuB)=(AnB), then P (exactly one of A and B
d
Re

occurs) =
in

ANSWERS
F

2. —
3
3. 0.2 4. 0.2 5. ^
1. {2,4,61 10
10
1
7. P(A)-P(AnB)
5
8. — 9. Z 10.
6. P(A) + P(B)-2P(AnB) 12 12 2

10 1
1 5
12. - 13. 14. 15. independent
11. P(A)P(B) 3' 12 9 3

17. 0 18. 1 19. - 20. P{B)-P{AnB)


16. P(AuB) 5

21. P(A)-P(AnB) 22. P(A) + P(B)-2P(A n B) 23. independent


-V+1/ + 2 27. 0
26.
24. b-c 25. P(A) + P(B) 2
20.156
APPLIED MATHEMATICS-XI

SHORT ANSWER QUESTIONS (VSAQs)


Answer each of the following questions in one word or one sentence or as per exact requirement of the
question:
I’ Three number s are chosen at random from numbers 1 to 30. Write
the probability that the
chosen numbers are consecutive.
2- n (> 3) persons are sitting in a row. Two of them are selected. Write the probability that they
are together.
3. A single letter is selected at random from the
that it is a vowel?
word 'PROBABILITY'. What is the probability
4. What is the probability that a leap year will have 53 Fridays or 53 Saturdays?
5. Three dice are thrown simultaneously. What is the probability of getting 15as the sum?

w
If the letters of the word 'MISSISSIPPI' are written down at random in a row, what is the
probability that four S's come together.
7. What is the probability that the 13th days of randomly chosen month is Friday?

F lo
8. Three of the six vertices of a regular hexagon are chosen at random. What is the probability
that the triangle with these vertices is equilateral.

ee
9- If £ and £2 are independent evens, write the value of P (E-^uE-y) r\(E n £2)).

Fr
hi. If A and B
are two independent events such that P (A n B) =- and P (A =-, then
write the values of P (A) and P (B). for ^
ur
41. A four digit number is formed using the digits 1, 2, 3, 5 with no repetitions. Write the
probability that the number is divisible by 5.
ks
Yo

12. When three dice


are thrown, write the probability of getting 4 or 5
oo

on each of the dice


simultaneously.
eB

13. Three digit numbers formed with the digits 0, 2, 4, 6 and 8. Write the probability
are
of
forming a three digit number with the same digits.
r

14. A ordinary cube has four plane faces, one face marked 2 and another face marked 3, find the
ou
ad

probability of getting a total of 7 in 5 throws.


Y

15. Three numbers


are chosen from 1 to 20. Find the probability that they are consecutive.
16. 6 boys and 6 girls sit in a row at random. Find the probability that all the girls sit together.
Re
nd

17. If A and Bare two independent events such that P (A) = O.Sand P (A u B) = 0.8. Find P (£).
Fi

18. An unbiased die with face marked 1, 2, 3, 4, 5, 6 is rolled four times. Out of 4 face values
obtained, find the probability that the minimum face value is not less than 2 and the
maximum face value is not greater than 5.
19. If A and B are two events write the
one of two events.
expression for the probability of occurrence of exactly
2(1. Write the probability that a number selected at random from the set of first 100 natural
numbers is a cube.
21. In a competition A, B and C are participating. The probability that A wins is twice that of B,
the probability that B wins is twice that of C. Find the probability that A losses.
22. If A, B, C are mutually exclusive and exhaustive events associated to a random experiment.
then write the value of P (A) P(B) + P(C).
20.157
PROBABILITY

23. If two events and 6 are such that P(^)=0.3, P(B) = 0.4 and P{AnB) = 05, find
P{B/Ar^B)
24. If A and B are two independent events, then write P (A n B) in terms of P (A) and P (B).
25. If P (/4) = 0.3, P (B) = 0.6, P (B/A) = 05, find P (A u 6).
2b. If SandC are independent events such that P(/\) = P(B) = P(C) = p, then find the
probability of occurrence of at least two of A, B and C.
27 If/4and6areindependenteventsthenwriteexpressionforP(exactlyoneof /4, B occurs).
28. If A and B are independent events such that P(.A) = p, P( B) = 2p and P (Exactly one of A and
B occurs) = - , find the value of p.
9

w
ANSWERS

4 3 13 4 1 1
144 2
8. —

Flo
2. 10
145 n n 7 216 165 84

10. P(A) = i,P(B) = i 11. 1/4 12. 1/27 13. 1/25 14. 5/6‘*

e
9. 0

re
F
18
15.
20
16. 1/132 17. 2/7 18. 16/81 19. P {A) + P (B) -2P{An B) 20. 1/25
C3
ur
r
21. 3/7 22. 1 23. 1/4 25. 0.75 26. 2p^-3p^ fo 27. P{A)PiB) + P{B)P{A)
28. 1/3,5/12
ks
Yo
oo
eB
ur
ad
Yo
d
Re
in
F
CHAPTER l22
DATA REPRESENTATION
AND VISUALIZATION

21.1 STATISTICS
The word "statistics" is used in both its singular as well as its plural senses,

low
In singular sense, statistics may be defined as the science of collecHon, presentation, analysis
and interpretation of numerical data.
In plural sense, statistics means numerical facts or observations collected with definite purpose.
For example, income and expenditure of persons in a particular locality, number of males and

ee
females in a particular town are statistics.

F
Fr
Statistics in plural sense have the following characteristics,
(i) A single observation does not form statistics. StatisHcs are a sum of total observatiODS.
(ii) Statistics are expressed quantitatively and not qualitatively,
for
ur
(iii) Statistics are collected with a definite purpose,
(iv) Statistics in an experiment are comparable and can be classified into various groups.
ks
Yo

21.2 STATISTICAL DATA


oo

The word data means information (its exact dichonary meaning is: given tacts).
When an investigator collects data himself with a definite plan or design in his (her) mind, it
eB

IS

called raw data or primary data. For example, if we list the marks scored m a test by a class of 50
students, it is raw data and as there is only one variable "Marks’. So, it ,
r

Similarly, if we record heights of males of age more than 20 years, in a locality, the data collected
ou
ad

is imivJinte raw data. Data in statistics is sometimes classified according to how rnany variables
are in a particular study. If we collect weight and height of class XI students in a school, there aie
Y

two variables "weight" and "height". So, the data collected is a bivariate data. Similarly, if you
want to find the average day temperature and average humidity level in a particular morith in
nd
Re

bivariate data. Bivariate data has many practical uses in real life,
Delhi. The data collected is a,
Fi

Data having more than two variables is called a imiltivariate data. For example, if we record
class. The data collected has three variables and is
height, weight and IQ level of students in a
called a multivariate daiei.
originally collected rather obtained from published or unpublished
source
Data which are not
are known as secondary data.
21.2.1 DATA MEASUREMENT SCALES
Measurement can be defined as the process of assigning numbers to objects according to some
set of rules. For example, if we measure someone's height, we first ask him (her) to stand agains
a wall and then we take a measuring tape. Keeping one end of the measuring tape on *e ground
wo extend it vertically until it reaches to the top of his (her) head and then we read off the
marked on the measuring tape to know the height. So, we assigm a number to the person equal to
his (her) weight according to a set of rules. The rules in this case are that the tape is stretAed
vertically up from the ground until it reaches the top of the head and then we read of how tall he
(she) is. This is measurement.
21.2
APPLIED MATHEMATICS-Xi

In statistics there are four data measurement scales:


(i) Nominal scale of measurement (ii) Ordinal scale of measurement
(iii) Interval scale of measurement (iv) Ratio scale of measurement
NOMINAL SCALE OF MEASUREMENT A nominal scale is the first level of measurement in which
we
assign numbers to objects where the different numbers indicate different objects. The
numbers have no real meaning other than differentiating between objects. For example, in case
of a very common variable "Gender" in statistics, if we assign 1 to all males and 2 to all females.
Then, it means all males are identified by ^ 1 and all females by a 2. This is nominal we could
have assigned males as a 1 males as n 2 or males as n 100 and females as a 200. It does not matter
what number we are assigning as long as all males get the same number and all females get
some other number. By assigning two distinct numbers to males and females, we are just
differentiating two groups. Thus, nominal scale is a naming scale where variables are simply
named or labeled with no specific order. Let us take another example of cricket players To
diffcrenHate players, numbers are marked on their T-shirts. The number that a player has on his

w
T-shirt does not provide any insight into the player’s position or ability or anything like that. It
)ust simply differentiates between players. So, if someone has the number 10 on his T-shirt and

F lo
someone has the number 15 on his T-shirt, it does not mean that the player having 15 on T-shirt
is better or has better
average run score in comparison to player having 10 on T-shirt. It just
means that they are not the same player.

ee
Here are some examples of nominal measurement scale for better understanding of this

Fr
measurement scale.
EXAMPLE 1
Select the degree of discomfort ofCOVID-19 patients:: for
1 - Mild, 2 - Moderate, 3 - Severe
ur
EXAMPLE 2 How wolild you describe your behavioural pattern?
E - Extroverted,
s

I - Introverted, A - Ambivert
ook
Yo

ORDINAL SCALE OF MEASUREMENT Ordinal is the second level measurement which is used for
ranking and ordering of the data without establishing the degree of variation between them.
eB

Ordinal indicates the order. Ordinal data is qualitative data which have naturally occurring
orders. For ordinal also, we assign numbers to objects just like nominal but here the numbers
our

also have meaningful order. For example, in a race we assign first, second, third place and so on
ad

because the first place person did better than the second, second did better than the third and so
on.
But, the number that they are assigned one, two or three indicates how they finished in the
Y

race. So the number for ordinal indicates placement of order. Thus, we can rank people with
Re

ordmal data. Here are some examples of ordinal scale:


nd

EXAMPLE 1
Hoiu satisfied are you with our product?
Fi

1~ Totally satisfied, 2-Satisfied, 3-Neutral, 4 - Dissatisfied, 5 - Totally dissatisfied


EXAMPLE 2 How happy are you with our service?
1 - Very Happy, 2 - Happy, 3 - Neutral, 4 - Unhappy, 5 - Very unhappy
INTERVAL SCALE OF MEASUREMENT Interval scale is the tliird level measurement after the
nommal scale and ordinal scale. Understanding tlie first two levels will help us to differentiate
interval measurement. A nominal scale is used when variables do not have a natural order or
ranking whereas in an ordinal scale, the rank of variables matters.
The interval scale is a quantitative measurement scale where there is order the difference
between the tu^o variables is meaningful and equal. It measures variables that exist alone a
common scale at equal intervals. ^
Interval scale contains ail the properties of the ordinal scale in addition to which, it offers a
calculation of difference between the variables. The main characteristic of this scale is the
equidistant difference between the objects. The difference between 40 degrees and 30 degrees
21.3
DATA REPRESENTATION AND VISUALIZATION

represents the same temperature difference as the difference between 90 degrees and 80
degrees. This is because each 10-degree interval has the same physical meaning.
Interval scales are not perfect as they do not have a true zero point even if one of the scale values
happens to carry the name "zero". The Fahrenlieit scale illustrates the issue as the zero degrees.
Fahrenheit does not represent the complete absence of temperature.
RATIO SCALE OF MEASUREMENT The ratio scale of measurement is the most informative scale.
It is an interval scale with additional property that its
zero position indicates the absence of the
quantity being measured. Ratio scale has most of the characteristics of other three variable

ow
measurement scales i.e. nominal, ordinal and interval. It is the 4th level of measurement and
tells us about the order and number of the object between the value of the scale.
The following are some examples for ratio scale:
What is your height?

e
LXAMPl.C 1

re
● Less than 155 cm
● 156 —170 cm

Frl
F
● 171 —175 cm
● More than 175 cm
ou
EXAMPLE 2 What is your weight in kgs?

sor
● Less than 50 kgs
● 51 — 70 kgs kf
● 71—90 kgs
oo
● 90 —no kgs
Y

● More than 110 kgs


B

SUMMARY By summarising the above discussion, we can say that nominal variables are used
to name or label a series of values. Ordinal scales provideinformation about the order of
re

choices, such as in customer satisfaction survey. Interval scales give us the order of the values
oY
u

and the ability to quantify the difference between each one. Ratio scale gives us order, interval
ad

values and the ability to calculate ratio since a true zero" can be defined.
d

21.3 CLASSIFICATION OF DATA


in

If the number of observations in the raw data is not very large, it can be easily presented and
Re

understood. But, when the number of observations is ver>' large a proper classification is
F

essential for analyis and interpretation. So, the next step in statistical investigation is
classification of data. Classification means arranging the data in different groups according to
the similarities. Broadly the data can be classified on following four bases: (i) Geographical (ii)
Chronological (iii) Qualitative and (iv) Quantitative. In geographical classification the data ^
are

classified on the basis of geographical or locational differences between various items. For
example, the product of food grains in India may be presented statewise.
In chronological classification the data are observed over a period of itme. For example, the
population of India may be presented from 1960 to 1990.
In qualitative classification data are classified on the basis of some attribute or quality such as
sex, religion, literacy etc.
In quantitative classification the data is classified or grouped according to some characteristics
that can be measured quantitatively, such as, height, weight,
etc. For example, the students of a
school may be classified according to weight as shown in the following table:
21.4
APPLIED MATHEMATICS-XI

Weight (in kgs) No. of students


40-50 305
50 - 60 455

60-70 784
70 - 80 200
80-90 10
90 -100 02

In the quantitative classification there are two elements viz (i) the variable i.e., the weight in the
above example and (ii) frequency i.e. the number of students in each class.
VARIABLE OR VARIATE A dmracteristic that varies in magnitude from observation to observation.

w
For example, weight, height, income, age etc. are variables.
A variable may be continuous or discrete. A continuous variable assumes all values within the
given range. For example, height of a person is a continuous variable.

F lo
A discrete variable cannot
take all possible values within the given range. For example, the number of machines in an
establishment is a discrete variable.

ee
FREQUENCY The number of times an observation occurs in the given data, is called thefrequena/ of the

Fr
observation.

21.4 DATA REPRESENTATION for


Frequency table or frequency distribution is a method
ur
to present raw data in the form from
which one can easily understand the information contained in the raw data.
Frequency distributions are of two types:
s
ok
Yo

(i) Discrete frequency distribution. (ii) Continuous or Grouped frequency distribution.


o

21.4.1 DISCRETE FREQUENCY DISTRIBUTION


eB

The process of preparing this type of distribution is very simple. The construction of a discrete
frequency distribution from the given raw data is done by the use of the method of tally marks.
r

In the first column of the frequency table we write all possible values of the variable from the
ou
ad

lowest to the highest.


We now look at the first value in the given raw data and put a bar (vertical line) in the second
Y

column opposite to it. Now we sec the second value in the given raw data and put a bar opposite
Re

to it in the second column. This process is repeated till all observations in the given raw data are
nd

exhausted. To facilitate counting blocks of five (j 111) are prepared and some space is left in
Fi

between each block. We finally count the number of bars corresponding to each value of the
variable and place it in the third column of frequency. The process will be clear from the
following example of the number of children in 20 families:
1,1, 2, 3, 4, 3, 2,1,1, 4, 5, 2, 4, 2, 2,1, 3, 3, 2, 5
The data may be put in the form of a discrete frequency distribution as follows:

No. of children Tally Bars Frequency


1 5
2 6
3 4

4 3

5 2
i
21.5
DATA REPRESENTATION AND VISUALIZATION

21.4.2 CONTINUOUS OR GROUPED FREQUENCY DISTRIBUTION


The above method of condensing the raw data is convenient only where the values in the raw
data are largely repeating and the difference between the greatest and the smallest observations
is not very large.
If the number of observations in data is large and the difference between the greatest and the
smallest observations is large, then we condense the data into classes or groups. For example, let
the marks obtained by 30 students of a class in a test be
39, 25, 5, 33,19, 21,12, 48,13, 21, 9,1,10, 8,12, 17,19,17, 41,40,12, 46, 37,17, 27, 30,
6, 2, 23,19.
We can arrange these marks as follows: __

ow
Marks (Class Number of students
Tally Bars Frec]uenc\/
intervals)

0-10 6

10-20 11

e
5

re
20-30

30 - 40
rFl 4

F
4
40-50

r
Such a presentation of data is known as the grouped frequency distribution.
ou
fo
In the above example 30 observations have been divided into 5 groups. These groups are called
classes. The class 0-10 means the marks obtained between 0 and 10 including 0 and excluding 10.
ks
The number of observations falling in a particular class is called the frequency of that class or
oo

class frequency. Thus, the class 0-10 has frequency 6 and the class 10-20 has 11 as class
frequency. In the class 0-10, we say that 0 is the lower limit and 10 is the upper limit of the class.
Y
eB

Similarly in the class 10-20,10 is the lower limit and 20 is the upper limit. The span of the class
i.e., the difference between the upper limit and the lower limit, is known as the class interval. For
example, in the class 10-20 the class interval is 20 -10 = 10.
ur

There are two methods of classifying the data according to the class intervals viz. (i) Exclusive
ad
Yo

Method, and (ii) Inclusive Method.


(i) EXCLUSIVE METHOD ^
d

When the class intervals are so fixed that the upper limit of one class is the lower limit of the ne
Re
in

class it is known as the exclusive method of classification. In this method the upper limit of a
class is not included in the class. Thus, in the class 0 -10 of marks obtained by students, a student
F

who has obtained 10 marks is not included in this class. He is counted in the next class 10-20.
(ii) INCLUSIVE METHOD . j , ● .u ^ i
In this method, the classes are so formed that the upper limit of a class is mcluded in that class.
The following example illustrates the method.
Wages f?) No. of workers
1000 - 1099 125
150
j 1100-1199 200
I 1200-1299 250
1300 -1399
1400 -1499 175

1500 - 1599 100

Total 1000
21.6
APPLIED MATHEMATICS-Xl

In the class 1000-1099, we include workers having wages between ? 1000 and ? 1099. If the
income
of a worker is exactly ? 1100 he is included in the next class 1100-1199.
Excl usive Method Inclusive Mcthoil
Wages (^) No. ofu’orkers Wages No. of workers
1000-1100 125 1000-1099 125
1100-1200 150 1100-1199 150
1200-1300 200 1200 -1299 200
1300 -1400 250 1300-1399 250
1400- 1500 175 1400-1499 175
1500-1600 100 1500 -1599 100
Total 1000 Total 1000

It is evident from the above example that both the inclusive and exclusive methods give us the
same
class frequency, although the class intervals are apparently different in the two cases. In

w
the above example in case of exclusive method the class interval is 100 whereas in case of
inclusive method the class interval is 99. However, 99 is not the correct class interval. Whenever
in

tervals and^to have


becomes a
2
b +
continuity.
where
2
F lo
inclusive method is used it is necessary to make an adjustment to determine the correct class
If a-b is a class in inclusive method, then in exclusive method it
_ (lower limit of a class) - (upper limit of previous class
2
)
for F
ree
In the above example, in inclusive method the difference between the lower limit of a class and
the upper limit of the preceding class is 1 i.e. h = 1. Therefore we subtract 1/2 from the lower
Your

Imiit of each class and add 1/2 in the upper limit of each class to make it continuous. The
ks

adjusted classes would then be as follows:


eBoo

Wages (?) No. of workers


999.5 -1099.5
ad

125
our

1099.5-1199.5 150
1199.5-1299.5 200
1299.5- 1399.5 250
Re

1399.5-1499.5
Y

175
Find

1499.5 -1599.5 100

In should be noted that before adjustment the class interval was 99 but after adjustment, it is 100.
The mid-value of a class is called the class mark. For example, tlae class-mark or mid-value of the
class 1000-1100 is 1050. In fact

Class mark = lower limit + upper limit


2

or
Class mark - lower limit + - (difference between the upper and lower limits)

21.4.3 CUMULATIVE FREQUENCY DISTRIBUTION


In secHon 21.4.2, the frequencies are grouped-frequencies or class-frequencies. If, however, the
frequency of the first class is added to that of the second and this sum is added to that of the third
and so on, then the frequencies so obtained are known as cumulative frequencies (c.f.).

(
DATA REPRESENTATiON AND VISUALIZATION 21.7

There are two types of cumulative frequencies viz. less than and greater than. For less than
cumulative frequencies we add up the frequencies from above and for greater than cumulative
frequencies we add up frequencies from below. The values of the variable and the
corresponding frequencies written together form a cumulative frequency distribution.
The cumulative frequency distributions for the frequency distribution in section 21.4.2 are as
follows:

Less than ciinndativefrequency distribution:


No. of workers
Wages (^)
(cumulative frequency)
Less than 1099.5 125

Less than 1199.5 275

Less than 1299.5 475

Less than 1399.5 725

w
Less than 1499.5 900

Less than 1599.5 1000

Greater tfum cumulative frequency distribution:

F lo No. of workers

ee
Wages (f) (cumulative frequency)

Fr
Less than 1099.5 1000

Less than 1199.5 875


for
Less than 1299.5 725
r
Less than 1399.5 525
You
s

275
ook

Less than 1499.5


Less than 1599.5 100
eB

RELATIVE FREQUENCY DISTRIBUTION Relative frequencies are very useful for the comparison
of two or more frequency distributions. The relative frequency of any class is the percentage of
our
ad

the total frequency.


Class frequency xlOO
i.e. Relative frequency =
Total frequency
dY
Re

ILLUSTRATIVE EXAMPLES
Fin

EXAMPLE! The frequency distributions of the ages of the inhabitants of two villages are given belozv:
No. of Inhabitants
Age group Village A Village B
0-5 18 35

5-10 26 38

10-20 25 40

20-30 29 32

30-40 21 27

40-50 14 23
50-60 6 18

60-70 4 15

Prepare relative frequency distributions.


21.8 APPLIED MATHEMATICS-XI

SOLUTION Calculation of relative frequency distribution


No. of Inhabitants Relative frequency
Age group
Village A Village B Village A Village B
18 35
0-5 18 35 X100 =12.6 X 100 =15.3
143 228
26 38
5-10 26 38 X100 =18.2 X 100 =16.7
143 228
25 40
10-20 25 40 X100 =175 X100 =175
143 228
29 32
20-30 29 32 X 100 =20.3 X100 =14.0
143 228

low
21 27
30-40 21 27 X 100 =14.7 X 100=11.8
143 228
14 23
40 - 50 14 23 X 100 =9.8 X 100 =10.1
143 228

ee
18
50-60 6

F 18 — X100 =4.2 X 100 =7.9

Fr
143 228
4 15
60-70 4 15 X100 =2.8 X100 =6.6
143 for
ur
228

Total 143 228 100.1 99.9


ks
Yo

EXERCISE 21.1
oo

1. In a sun'ey of 35 families in a v illage,


' the number of children per family was recorded
eB

and the following data obtained:


1 0 2 3 4 5 6
r
ou
ad

7 2 3 4 0 2 5
4
Y

5 12 6 3 2
7 6 5 4 3 7 8
nd
Re

9 7 9 4 5 4 3
Fi

Represent the data in the form of a discrete frequency distribution.


2. Construct a frequency table for the following ages (in years) of 30 students using equal
class intervals, one of them being 9 -12, where 12 is not included:
18,12,7,6,11,15,21,9,8,13,15,17,22,19,14,21,23,8,12,17,15,6,18,2 3,16,9,21 11 16
22.

3. From tire following observations prepare a frequency distribution table in the


ascending order starting with 10 -15 (exclusive method).
35,19,14,35,33,43,30,29,32,10,12,15,29,18,17, 54, 48,36,11, 60, 46,38, 25, 20,13,27,
23,18,16, 26, 44,18, 27,16, 37, 49, 21, 41,11, 52, 50, 34, 45, 57, 53, 24, 23, 26.
4. Classify the following data by taking class interval such that their mid-values are 17,22,
27, 32 and so on
30,42, 30,54, 40,48,15,17, 51,42,25,41, 30, 27,42,36, 28, 36,37,54,44,31, 36,40, 36 22
30,31,19,48, 16,42,32,21, 22,40,33,41,21.
21.9
DATA REPRESENTATION AND VISUALIZATION

[HIN1; Since, we have to classify the data in such a manner that the midvalues are, 17,22,27
etc. the first class should be 15 -19 (mid value = (15
+ 19) /2 = 17), second class 20 - 24 etc]
5. Present the following data of the percentage marks of 60 students in the form of a frequency
table with 10 classes of equal width one class being 40 - 49.
41,17,83,63,54,92,60,58,70,06,67,82,33,44,57,49,34,73,54,63,36,5 2,32,75,60,33,09,
79, 28,30,42,93,43, 80, 03, 32,57, 67,24, 64,63,11,35, 82,10, 23, 00, 41, 60,32, 72,53,92, 88,
62, 55, 60, 33, 40, 57
6. Following are the ages of 360 patients getting medical treatment in a hospital on a day
20-30 30 - 40 40 - 50 50-60 60-70
Age (in years) 10-20

90 50 60 80 50 30
No. of patients i

Construct the cumulative frequency distribution.


7. Form of a frequency distribution table from the following data, showing the scores of 30
students in a class test.

w
Marks No. of students

F lo
Below 10 5

Below 20 12

Below 30 20

ree
Below 40 26

F
Below 50 30

S. The number of smokers in two villages A and B in various age groups


for arc
given by the
following distribution:
r
No. of hilmbitants
You
oks

Age group Village B


Village A
eBo

10-20 32 12

20-30 60 44

30-40 48 56
our
ad

40 - 50 53 42

50-60 20 17

60-80 7 19
dY
Re

Prepare the relative frequency distribution.


9. From the following grouped frequency distribution construct'Less than' and 'more than'
Fin

cumulative frequency distributions.


Marks obtained No. of students
0-10 4

10-20 6

20-30 9

30 - 40 6

40-50 2

21.5 VISUALIZATION OF DATA


In the previous section, we have discussed the techniques of classification of raw data. In fact
classification of data is very useful in summarising the raw data and presenting them in a
systematic manner. However, these forms of presentation of data do not always prove to be very
interesting to the common man. One of the most appealing and convincing ways of presenting
21.10
APPLIED MATHEMATICS-XI

the statistical results is through diagrams and graphs. In this section, we shall discuss the
diagramatic presentation of frequency distributions.
There are various methods of diagramatic presentation of frequency distributions, which are m
use.

21.5.1 BAR GRAPH

A bar graph is a pictorial representation of the numerical data by a number of bars (rectangles)
of uniform width erected horizontally or vertically with equal spacing between them. Each
rectangle or bar represents only one value of the numerical data and so there are as many bars as
the number of values in the numerical data. Tlie height or length of a bar indicates on a suitable
scale the corresponding value of the numerical data.
In order to draw a bar graph, we draw two mutually perpendicular lines in the plane paper. The
horizontal line is called Ar-axis and the vertical line is known as the i/-axis. If the bars are drawn

w
vertically on horizontal line (x-axis), then the scale of heights of the bars or rectangles is shown
along i/-axis. If the bars are drawn horizontally on
the vertical line (i/-axis), then the scale of
height of the bars or rectangles is shown along .r-axis. The bars can be shaded, hatched or

Flo
coloured.

While constructing bar graphs the following points should be kept in mind;

e
(i) The width of the bars should be uniform throughout,

re
(ii) The gap between one bar and another should be uniform throughout,

F
(iii) Bars may be either horizontal or vertical. The vertical bars should be preferred because
they give a better look.
ur
r
Following examples will illustrate the construction of bar graphs. fo
ks
ILLUSTRATIVE EXAMPLES
Yo

EXAMPLE 1 The following table gives the number of students in


oo

Class IX in a school duriiw academic


years 1996-1997 to 2000-2001.
B

Academic Year 1996-97 1997-98 1998-99 1999-2000 2000-2001


re

Number of students 50 75 125 150 200


u
ad

Represent the above data by a bar graph.


Yo
d
Re
in

200
F

175

150

£ 125
T3
3 100
i i
I
i i
<A

o 75

i
o

S
I i I
z 25
o 1 (a
1
X
O' o
O' O' O' o o
O' O' O' o o
<N CN
ts. 00 O' c
O' O' O'
O' O' O' O' c
fS

Academic years
Fig.22.1 BarGraph
21.11
DATA REPRESENTATION AND VISUALIZATION

SOLUTION To represent the above data by a bar graph, we first draw a horizontal and a vertical
line. Since five values of the numerical data are given. So, we mark five points on the horizontal
line at equal distances and erect rectangles of the same widtli at these points. Tlie heights of the
rectangles are proportional to the numerical values of the data as shown in Fig. 22.1.
rXAMPLE2 The folloiViiig tabic gives the mmiber of vehicles passing through a biisij crossing in Delhi in
different time intervals on a particular day.
12 to 13 hrs 13tol4hrs 14 to Iirs
Time inlen'al 8lo9lirs 9 to 10 hrs 10 to 11 hrs 11 to 12 hrs
250 200 150 100
Number of vehicles 300 400 350

Represent the above data by a bar graph


SOLUTION Here 7 values of the data are given. So, mark 7 points on the horizontal axis at equal
distances and erect rectangles of the same width whose heights are proportional to the values of

ow
the numerical data.
y

450

400

e
350
i

Fl
re
300
1

F
■£ 250
1
I i
200
ur
O
Si I
150
% I i or
J
I 100
sf
2
I P
k
50
Yo
i X
oo

o
ir, ^ ^
i X X X X
B

J- — fs cn in

m
re

c (M
O'

Time intervals
u

Fig. 21.2
ad
Yo

21.5.2 PIE DIAGRAMS

Pie diagrams are used for representing relative frequency distributions only. In pie diagrams
d

the relative frequencies are represented by sectors of a circle. To draw a pie diagram we first
Re
in

draw a circle of arbitrary radius and then divide it into as many sectors as there are classes (or
values of the variable) in the given frequency distribution. The area of each sector is
F

proportional to the relative frequency of the class represented by the sector. But, the area of any
sector of a circle is proportional to the angle subtended by its arc at the centre. So, we make
angles at the centre proportional to the relative frequencies. In fact, we use the following
formula to find the sector angles.
Class frequency X 360
Sector angle =
Total frequency
Following examples will illustrate the procedure.
ILLUSTRATIVE EXAMPLES

LXAMl’LE 1 The following data relate to the cost of construction of a house in Delhi
Items Cement Steel Bricks Timber Labour Miscellaneous

10% 15% 25% 12%


Expenditure 207o 18%
21.12
APPLIED MATHEMATICS-XI

Represent the above data by a pie diagram.


SOLUTION
Calculation of central angles.
Items Expenditure (in percent) Central angle
20
Cement 20 X 360 = 72°
100

18
Steel 18 X 360 = 64.8°
100

Bricks 10
( 10
X 360 = 36°
100

w
15
Timber 15 x 360 =54°
1100

F lo
25
Labour 25 X 360 = 90°
UOO

ee
12

Fr
Miscellaneous 12 X 360 = 43.2°
_ UOO
for
Draw a circle of any radius and divide it into 6 parts according to the degree measures of the
ur
central angles calculated above and mark them as shown in Fig. 21.3.
s
ook
Yo
eB

Steel
18%
Cement
Bricks 20%
r
ad
ou

10%

Miscellaneous
Timber
Y

\ 12%
15%
Re
nd

Labour
25%
Fi

Fig. 21.3

The diagram so obtained is the pie diagram of the given data.


EXAMPLE!
Draw a pie diagram to represent the following data on the proposed outlay during the fourth
five year plan:

Items Industries and Irrigation


Agriculture minerals Communication Miscellaneous
and power
? (in crores) 6000 4000 2500 4500 3000

SOLUTION Total outlay = ? (6000 + 4000 + 2500 + 4500 + 3000 ) crores = ^ 20,000 crores.
21.13
DATA REPRESENTATION AND VISUALIZATION

Calculation of Central Angles


Items Anwuuf in crores) Central angle_
6000
6000 — X 360 = 108°
Agriculture 20000

f 4000 = 72°
4000 X 360
Industries and Minerals 20000

2500
2500 x 360 =45°
Irrigation and Power 20000

w
4500
4500 X 360 = 45° ,
Communication 20000

f 3000 X 360 =54°


3000

o
Miscellaneous

e
20000

re
20000 360'
Total

Frl
F
ou
r
Industries
Agriculture

so
& minerals
108°
72°
kf
Irrigation 45°
oo
& 54
power 45°
Y
B

Communication
re
oY
u

Fig. 21.4
ad

Draw a circle of any radius and divide it into 5 parts according to the measure of the central
angle. The diagram so obtained is the required pie diagram as shown in the Fig. 21.4.
d

21 6 GRAPHICAL REPRESENTATION OF FREQUENCY DISTRIBUTIONS


in
Re

In’the previous section, we have discussed diagramatic representation of frequency


distributions. In this section we shall discuss graphical representation of frequency
F

distributions. There are various methods of graphical representation of frequency distributions.


In this section, we shall only discuss three of them viz. (i) Histogram (ii) Frequency polygon
(iii) Cumulative frequency curve (Ogive).
21.6.1 HISTOGRAM . ●
A histogram or frequency histogram is a graphical representation of a frequency distribution i.
in

the form of rectangles with class intervals as bases and heights proportional to corresponding
frequencies such that there is no gap between any two successive rectangles.
A histogram is a two dimensional diagram. We shall discuss the construction of a histogi am for
the following frequency distribution separately,
(i) A continuous grouped frequency distribution with equal class-intervals,
(ii) A continuous grouped frequency distribution with unequal class-intervals,
(iii) A frequency distribution in which mid-points of class-intervals are given,
(iv) A grouped frequency distribution in which class-intervals are given in inclusive form.
21.14
APPLIED MATHEMATICS-XI

CONSTRUCTION OF A HISTOGRAM OF A CONTINUOUS GROUPED FREQUENCY DISTRIBUTION


In drawing the histogram of the above frequency distribution, we use the following algorithm.
ALGORITHM
STHI’ 1
Take a graph paper and draw two perpendicidar lines, one horizontal and one vertical,
intersecting at O (sap). Mark them as OX and OY.
STEP II
Take horizontal line OX as X-axis and vertical line OY as Y-axis.
STEP 111
C lioose a suitable scale and along X-axis represent class-limits.
Ckoose a suitable scale for Y-axis and mark frequencies along Y-axis.
STFP \ Construct rectangles with class intervals as ba.ses and respective frequencies as heights.
XQIE It should be noted that the scale for x-nxis may not be same as the scale for y-axis. The selection of
scale depends upon our convenience and the type of data.
Following illustrations will clarify the above algorithm.

ow
ILLUSTRATION 1 The following table gives the marks scored by 100 students in an entrance
examination.

Marks:
0-10JlO-20 20-30 30-40 40-50 50-60 60-70 70-80

e
No. ^students: {Frequency)

Fl
re
4 10 16 22 20 18 8 2

Represent this data in the form of a histogram.

F
SOLUTION We represent the class limits along .v-axis on a suitable scale and the frequencies
along y-axis on a suitable scale.
ur
or
Taking class-intervals as bases and the corresponding frequencies as heights, we construct
rectangles to obtain the histogram of the given frequency distribution
sf
as
shown in Fig. 21.5.
k
Yo
oo
B

22
'S'.

c
re

V
■o
20

16
u
ad

o
Yo

£ 12
£
3

2 8
d
Re
in

4
F

Marks

Fig.21.5

ILLUSI RATION 2 The following is the distribution of weights (in kg) of 50 p ersons:
Weight (in kg): 50-55 55-60 60-65 65-70 70-75 75-80 80-85 85-90
Number of persons: 12 8 5 4 5 7 6 3
Draw a histogram for the above data. '
SOLimoN We represent the class limits along X-axis on a suitable scale and the frequencies
along Y-axis on a suitable scale.
Since the scale on X-axis starts at 50, a kink (break) is indicated near the origin to signify that the
graph IS drawn to scale beginning at 50, and not at the origin.
21.15
DATA REPRESENTATION AND VISUALIZATION

12

10

c/3 6
U

Q-

O 4
lx
ru
Xl

ow
E
2
Z

0
Weight (in kg]

e
re
rFl
A HISTOGRAM OF A CONTINUOUS GROUPED FREQUENCY DISTRIBUTION

F
CONSTRUCTION OF
WITH UNEQUAL CLASS INTERVALS

In drawing the histogram of the above frequency distribution, we adjust the frequencies as

r
ou
given in the following algorithm. fo
ks
ALGORITHM
sri;p 1 Take a graph paper and dram two perpendicular lines, one horizontal and one vertical,
oo

intersecting at O (sap). Mark them as OX and OY.


Y

Take horizontal line OX as X-axis and vertical line OY as Y-axis.


eB

SILP 11

STHP 111 Choose a suitable scale along X-axis and represent class-limits on it.
Determine a class-interval which has the minimum class size. Let the minimum class size be h.
ur

STHPI\

STHI’ V Compute the adjusted frequencies of each class by using the following formula:
ad
Yo

Adjusted frequency of a class- , - X


Frequency of the class.
d

These adjusted frequencies are the heights of each rectangle of histogram but widths will be
Re
in

according to class limits.


Choose a suitable scale for Y-axis and mark adjusted frequencies along Y-axis.
F

STI-P VI

STiq’ VII Construct rectangles zvith class inteiwals as bases and respective adjusted frequencies
as

heights.
The histogram so obtained is the desired histogram of the given frequency distribution.
The above procedure is illustrated by the following illustrations.
ILLL'STR.\TION3 Represent the following data by means of histogram.
10-15 15-20 20-25 25-30 30-40 40-60 60-80
Weekly wages (in ^) _
5 12 12
No. of workers (Frequency) 7 9

SOLUTION Here, the class intervals are of unequal width. So, we shall first compute adjusted
frequencies of each class. The minimum class size is 15-10 =5. The adjusted frequencies
are

computed by using the following formula:


Minimum class-size
Adjusted frequency of a class = Frequency of the class.
Class-size
21.16
APPLIED MATHEMATICS-XI

The adjusted frequencies are computed in the following table:


Weekh/ wages (in^) Frequency Adjusted frequency
10-15 7 -x7=7
5

5
15-20 9 - X 9 = 9
5

20-25 8 -x8 = 8
5

25-30 5 -x5 =5
5

ow
30-40 12 — xl2=6
10

40-60 12 — xl2 = 3
20I

e
re
60-80

Fl — x8=2
20

F
Now, we construct rectangles with class-limits as bases and respective adjusted frequencies
ur as

heights.

r
y fo
ks
Yo
10
oo
eB

8
ur
ad

75
6
Yo

o
5
d

o
U 4
Re

QJ
in

X
E
Z
F

Weekly wages (in ?) ^


Fig. 21.7

ILLUSTRATION 4 Draw a histogram for the marks of students given below:


Marks: 0-10 10-30 30-45 45-50 50-60

No. of Students: 8 32 18 10 6

SOLUTION Here, minimum class-size = 5. So, we adjust the frequencies by using the following
formula:
21.17
DATA REPRESENTATION AND VISUALIZATION

Minimum class size


Adjusted frequency of a class = X Frequency of the class
Class size

The adjusted frequencies are computed as follows^


Class Intervals No. of students Adjusted frequency
(Marks) (Frequency)

0-10 8
Ax8=4
10

— X 32 = 8
10-30 32
20

5
— xl8 = 6
30-45 18
15

45-50 10 ^xl0=10
5

F low
— X 6 - 3
50-60 8
10

Now, we construct rectangles with class-limits as bases and respective adjusted frequencies as
heights.

re
for F
Your
s
eBook
ad
our
Re
Y
Find

Marks
Fig. 21.8

CONSTRUCTION OF A HISTOGRAM WHEN MID-POINTS OF CLASS-INTERVALS ARE GIVEN


To draw the histogram when mid-points of class intervals are given, we assume^^that the
frequency corresponding to the variate value a is spread over the interval " 2 ^ ^ 2'
h is the jump from one value to the other. Using this, we obtain the upper and lower class limits
of different classes from given mid-points as given in the following algorithm.
ALGORITHM

Ml.-lM Compute the difference between second and first mid-point. Let it be h.
21.18
APPLIED MATHEMATICS-XI

STEP II
Divide the difference li bi/ 2.
STEP llf
Subtract -^^fromfirst mui-point to get the loiver limit offirst class and add ^ tofirst mid-point
to get the upper limit offirst class.
STEP IV
Repeat first three steps for all other mid-points.
After ascertaining the lower and upper class limits of different classes from the given
mid-points, we construct the histogram by the procedure explained earlier.
Following illustration will illustrate the above procedure.
ILLUSTRATION 5 Construct a histogram from the following distribution of total marks obtained by 65
students of IX class in the final examination.

ow
Marks (mid-points): 150 160 170 180 190 200
No. of students: 8 10 25 12 7 3

SOLUTION Ascertainment of lower and upper class limits:

e
re
Since the difference between the second and first mid-point is 160 -150 = 10

rFl
F
/i=10=>- = 5
2

So, lower and upper limits of the first class are 150 - 5 and 150 -h 5 i.e. 145 and 155 respectively.

r
ou
First class interval is 145 -155.
fo
Using the same procedure, we get the classes of other mid-points as under:
ks
Marks:
145-155 155-165
oo

165-175 175-185 185-195 195-205


' No. of students:
Y

10 25 12 7 3
B

The histogram of the above frequency distribution is given in Fig. 21.9.


re

y
ou

28 -
Y
ad

24 -
d

20 ■
c
in
Re

01
●u
3
16 -
■.T
F

o 12 '

Z
8 -

4 -

O
X
Marks
Fig. 21.9

CONSTRUCTION OF HISTOGRAM WHEN CLASS INTERVALS ARE GIVEN IN INCLUSIVE FORM


i.e. CLASS INTERVALS ARE NOT CONTINUOUS
If the given grouped frequency distribution is not continuous, we first convert it into a

continuous frequency distribution by using the following procedure.


21.19
DATA REPRESENTATION AND VISUALIZATION

To convert a given frequency distribution into a continuous frequenc\j iiistribidion, we follow the
following steps:
STEP I Compute the difference between the lower limit of second clnss interval and upper limit offiist
class interval. Let the difference be h.
STEP II Subtract — from the lozoer limits of each class and add — to the upper limits of each class.
2 2

After converting the given class intervals into continuous intervals, we draw the histogram with
reference to continuous class intervals.
Following illustrations will illustrate the above procedure.
ILLUSTRATION 6 The following table presents the number of illiterate females in the age group (10-34)
in a toum.

w
15-19 20-24 25-29 30-34
Age group: 10-14

980 800 580 290


Number of Females: 300

F lo
Draio a histogram to represent the above data.
SOLUTION Tlie given frequency distribution is not continuous. So, we shall first convert it into a

ee
continuous frequency distribution.

Fr
The difference between the lower limit of a class and the upper limit of the preceding class is 1
i.e. /i = 1. To convert the given frequency distribution into a continuous frequency distribution,
we subtract -2 = -2 = 0.5 from each lower limit and add ^2 = 0.5 to each upper limit. The for
ur
distribution so obtained is as given below:
ks

24.5-29.5 29.5-34.5
Yo

9.5-14.5 14.5-19.5 19.5-24.5


Age groii^
oo

980 800 580 290


Number of Females: 300
eB

The histogram of the above frequency distribution is given in Fig. 21.10.


r
ou
ad
Y
nd
Re

1000
Fi

800

o> 600
rz

o
400
01
X)
£ 200
3
z

in in m in
in in
0 O' ●S' I?' ●S' O' ●S'
m

Age group
Fig. 21.10
21.20
APPLIED MATHEMATICS-XI

Draiv a histogram for the folloioing data:


Class-interval 25-29 30-34 35-39 40-44 45-49 50-54

Frequency: 5 15 23 20 10 7

SOLUTION The histogram of the above frequency distribution is given in Fig. 21.11.
Y

25

20

15

u
C

cr
P
u
U.
10

5
F low forFe
0

Class-interval
Fig. 21,11
eBoks
Your

:^1.6.2 FREQUENCY POLYGON


ad

In the previous section, we have learnt how to represent graphically a frequency distribution
our

using histogram. Frequency polygon is another method of representing frequency distributions


graphically. Frequency polygon of a given frequency distribution can be drawn in two ways,
Re

(i) By using histogram. (ii) Without using histogram.


We shall discuss the construction of frequency polygon in both ways.
In order to draw a frequency polygon by using a histogram, we may follow the followine
FindY

algorithm: ^
rilTH

Obtain the frequency distribution and draw a histogram representing it.


Obtain the mid-points of the upper horizontal side of each rectangle.
}oin these mid-points of the adjacent rectangles of the histogram by dotted line segments.
Obtain the mid - points of two class - intervals of zero frequency i.e. on X-axis, one adjacent to
the first, on its left and one adjacent to the last, on its right.
These class-intervals are known as imagined class intervals.
S1M’\ I
Complete the polygon by joining the mid-points of first and last class intervals to the mid
- points of imagined class-intervals adjacent to them.
21.21
DATA REPRESENTATION AND VISUALIZATION

„j intervals do )wt exist. For example, in the case of marks


Rl-MARk Sometimes imagined class obtained bp
below
the students in a test, zoe cannot go i zero and beyond maximum marks on the two sides. In such cases
the extreme line-segments are only partly drawn and brought down vertically
are so that they meet with
the vertical sides offirst and last rectangles.
ll.l.USTRATlON 1 For the following data, draio a histogram and a frequency polygon:
50-60 60-70 70-80 80-90 90-100
Marks: 0-10 10-20 20-30 30-40 40-50

Number of 10 4 6 7 3 2 2 3 9

I Students: ^
SOLUTION In Fig. 21.12 a histogram and a frequency polygon (in dotted lines) are drawn on the
same scale.

w
F lo
y

10

ee
Fr
9

7
for
VI
ur
c
o
T3
6
s
ook

VI
5
Yo

o
u
u 4
eB

Si
£
3
3
Z
our

2
ad

50 60 70 80 90 100 X
dY

O 10 20 30 40
Re

Marks
Fin

Fig. 21.12

ILLUSTRATION 2 2 The marks scored by 750 students in an examination are given in the form of a
frequency distribution table.
720-760 760-800 800-840 940-880
Marks: 600-640 640-680 680-720

172 59 18
156 284
Number of Students: 16 45

Represent this date in the form of a histogram and construct a frequency poiygon.
SOLUTION In Fig. 21.13 a histogram and a frequency table of the above frequency distribution
are drawn on the same scale.
21.22
APPLIED MATHEMATICS-X!

280

260

240

220

ow
200

180

160
c
aj
T3
140

e
D
t/;

re
120
u
Cl
Si 100
E

Flr
F
a
2 80

60

40
ou
sr
fo
20

k
oo
c o o o c o o
o VC o *9“ CO rg vC o
o o
If) VO vO vC rv. lx CO 3 CO
CO
Y

Marks
reB

Fig. 21.13

To construct a frequency polygon without using the


uY

histogram of a given frequency


distribution, we use the following algorithm.
ALGORITHM
ad
do

STEP [
Obtain the freqiienci/ cUstribiition.
STEP [1
Compute the mid-points of class intervals i.e. class marks.
in

STEP IH
Represent class marks on X-axis on a suitable scale.
Re

STEP IV Represent frequencies on Y-axis on a suitable scale.


F

STEP V
Plot the points {Xj, f), where Xj denotes class mark and f corresponding freq uenci/.
STEP VI
Join the points plotted in step K bi/ line segments.
Tfptzoo class intervals of zero fretinenaj, one at
STEP VII
the beginning ami the other at the end
Obtain their mid-points.
These classes are known as imagined classes.
STi-P vm
Complete the frequency polygon by joining the mid-points offirst and last class intervals to
the mid-points of the imagined classes adjacent to the m.
Following illustrations will illustrate the above procedure.
ILLUSTRATIONS
Construct a frequency polygon for the following data:
Age (in years): 0-2 2-4 4-6 6-8 8-10 10-12 12-14 14-16 16-18
Frequency: 2 4 6 8 9 6 5 3 1

SOLUTION First we obtain the class marks as


given in the following table.
21.23
DATA REPRESENTATION AND VISUALIZATION

Age (in years) Class Marks Frequency


1 2
0-2
2-4 3 4

4-6 5 6

6-8 7

8-10 9 9

10-12 11 6

12-14 13 5

14-16 15 3

17 1
16-18

ow
Now, we plot the points (1, 2), (3,4), (5,6), (7,8), (9, 9), (11, 6), (13, 5), (15, 3) and (17,1).
Now, we join the plotted points by line segments. The end points (1,2) and (17,1) are joined to
the mid-points (-1,0) and (19,0) respectively of imagined class-intervals to obtain the frequency
polygon.

e
re
10

rFl
F
9 (9. 9)

(7, 8)

or
ou
7
ksf
6 (5, 6) (11,6)
oo

5 (13. =.5)
Y

U
B

4 (3,4)
T

£ 3 (15, 3)
re

2 (1-2)
oYu

(17,1)
ad

(-1. 0) 1,
(19,0)

7 9 11 13 15 17 19
d

1 3 5
I

Age (in years)


in
Re

Fig. 21.14
F

11.LUSTR.ATION 4 The following are the scores of two groups of class IV students in a test of reading
ability.
Scores Ovup A Group B
1
50-52 4

47-49 10 3

15 4
44-46
41-43 18

38-40 20 12

35-37 12 17

13 22
32-34

Total 92 68
21.24
APPLIED MATHEMATICS-Xi

Construct n frequency polygon for each of these two groups on the same axes.
SOLUTION Here the class intervals are not continuous. Since the class marks do not alter by
changing the class intervals in continuous form. Therefore, we do not make them continuous.
We now calculate the class marks as given in the following table.
Scores Class Marks Group A Group B
50-52 51 4 2
47-49 48 10 3
44-46 45 15 4
41-36 42 18 8
38-40 39 20 12
35-37 36 12 17

w
32-34 33 13 22

We represent class marks on X-axis on a suitable scale and the frequencies on Y-axis on

F lo
a
suitable scale.

To obtain the frequency polygon for group A, we plot the points (51,4), (48,10), (45,15), (42,18),

ee
(39,20), (36, 12) and (33, 13) and join these points by line segments.

Fr
To obtain the frequency polygon for group A, we plot the points (51, 2), (48, 3), (45, 4), (42, 8),
(39,12), (36,17) and (33, 22) on the same scale and join these points by dotted line segments. for
The two frequency polygons are shown in Fig. 21.15.
ur
y
s
ook
Yo

24
(33, 22)
eB

22
(39, 20)
20 \
\
r

18
ad

(42,18)
ou

I \
I
16
'(36,17) \
I (45,15)
Y

14 I \
\
(33,13) /
Re

\ (39,12)
nd

12 I
u
c
(D
10
I I (36,12) \
Fi

I
3
CT-
\ (48,10)
tu
I \
p-
I
I
(42,8) \
6 \
I
\
I \
4 I, (51,4)
(45,4)
2 (48, 3) (51,2)
s

0 30 33 36 39 42 45 48 51 54 X
Scores
Fig.21.15

21.6.3 CUMULATIVE FREQUENCY CURVE (AN OGIVE)


In earlier sections, we have learnt about graphical representation of frequency distributions by
using bargraphs, histograms and frequency polygons. In this section, we will learn about the
21.25
DATA REPRESENTATION AND VISUALIZATION

construction of cumulative frequency polygon and cumulative frequency curves or ogives. The
technique of drawing the cumulative frequency polygons and cumulative frequency curves or
ogives is more or less the same. The only difference is that in case of simple frequency curves
and polygons the frequencies are plotted against class marks of the class intervals where as in
case of a cumulative frequency polygon or curves the cumulative frequencies are plotted against
the lower or upper limits of the class intervals depending upon the manner m which the senes
has been cumulated. There are two methods of constructing a frequency polygon and an ogive.
Let us now discuss the two methods,
(i) Less than method (ii) More than method

ow
LESS THAN METHOD

To construct a cumulative frequency polygon and an


ogive by less than method, we use the
following algorithm

e
ALGORITHM

Start zvith the upper limits of class mtervais and add class frequencies to obtain the cumulative

re
frequency distribution.

Frl
F
Mark upper class limits along X-axis on a suitable scale.
Mark cumulative frequencies along Y-axis on a suitable scale.
ou
Plot the points f.Y,-, fj), zvhere X: is the upper limit of a class andfi is corresponding cumulative

sor
frequency.
Join the points obtained in step IV by a free hand smooth curve to get the ogive and to get the
kf
cumulative frequency polygon join the points obtained in step IV by line segments.
oo

MORE THAN METHOD


Y

than method, we use the


To construct a cumulative frequency polygon and an ogive by more
B

following algorithm.
re
oY

ALGORITHM
u

Start zvith the lozver limits of the elass intervals and from thc^ total frequency subtract the
ad

frequency of each class to obtain the cumulative frequency distribution.


d

Mark the lower class limits along X-axis on a suitable scale.


in

Mark the cumulative frequencies along Y-axis on a suitable scale.


Re

Plot the points (Xj, f ), zvhere Xj is the lozver limit of a class and fj is the corresponding
F

cumulative frequency.
Join the points obtained in step IV by a free hand smooth curve to get the ogive and to get the
cumulative frequency polygon join these points by line segments.
Following illustrations will illustrate the above algoritiun.
■ ^ .1R.\T10N i Drazv an ogive and the cumulative frequency polygon for the follozuing frequency
distribution by less than method.
30-40 40-50 50-60
0-10 10-20 20-30
Marks:
23 51 6 3
10
No. of students: 7

SOLUTION We first prepare the cumulative frequency distribution table by less than method
as

given below:
21.26
APPLIED MATHEMATICS-XI

Marks No. of Students Marks less than


Cumulative Frequency
0-10 7 10 7

10-20 10 20 17

20-30 23 30 40

30-40 51 40 91

40-50 6 50 97

50-60 2 60 100

Otiier than the given class intervals, we assume a class -10 - 0 before the first class interval 0 -10
with 2ero frequency.

w
Now, we mark (he upper class limits (including the imagined class) along x-axis on a suitable

F lo
scale and the cumulative frequencies along y-axis on a suitable scale.
Thus, we plot the points (0,0), (10,7), (20,17), (30,40), (40,91), (50,97) and (60,100).

e
Now, we join the plotted points by a free hand curve to obtain the required ogive as shown in

Fre
lAese«afshownt'C2l“ for
r
You
oks
eBo
our
ad

W
c
(U
dY

3 60
Re
Fin

Q>
> 50
U3
(S

3
S 40
3
u

30

20

10

-I

0 10 20 30 40 50 60
Marks ►

Fig. 21.16 Cumulative frequency curve or ogive


21.27
DATA REPRESENTATION AND VISUALIZATION

1 Uii
■' iiii

100
U1 Wr
(60,100):
(50,97)
A

90 H"
i : (40,91)
I

80
IL
●j

/
70
ful
u
C

3
cr
60
/ 'Cumulative frequency polygon
o r
XI
50
n . 4u;

low
r
1
re

3
3 40 Hi
U TT^“
30

ee
t
20
i ^(20,17)1 £ rF
Fr
,n: ■:
..

43
10
~

(10,7)
L-J
0 10 20 30 40 for 50 60
u
Marks ^
ks
Fig. 21.17 Cumulative frequency polygon
Yo
o
Bo

ILLUSTRATION 2 Draw a cumulative frequenci/ curve and cumulative frequenci/ polygon foi tin
following frequency distribution by less than method.
re

20-29 30-39 40-49 50-59 60-69


(in years): 0-9 10-19
ou
ad

23 17 11 9
No. of persons: 5 15 20
Y

SOLUTION The given frequency distribution is not continuous So, we first make it continuous
nd
Re

and prepare the cumulative frequency distribution as under.


Fi

Frequency Age less than Cumulative Frequency


Age (in years)
5 9.5 5
0.5-9.5

15 19.5 20
9.5 -19.5

20 29.5 40
19.5-29.5

23 39.5 63
29.5 - 39.5

17 49.5 80
39.5 - 49.5

11 59.5 91
49.5 - 59.5

9 69.5 100
59.5 - 69.5
21.28
APOUED MATHEMATICS-XI

Now we plot points (9.5,5), (19.5,20), (29.5,40), (39.5,63, (49.5,80), (59.5,91) and (69.5,100) and
join them by a free hand smooth curve to obtain the required ogive as shown in Fig. 21.18.
u!Il
:Ui
.V
U:.
iB gs^tisiei
100
z c:

90
Ttg tSSSi
KS

iil ii i
T

80 S m»ssmsss»< ii ae
n
mi
11
m
ru 2
1 -Ii in in
i:li a 1 iii mmmm
70
i
: i::
iHi
Ii;! 11; S>
ii
Un
d :U: 13
(A
C ‘id TtZZ
r2iii [Hi TT ;it: n 13
o 60 li:
m
0^
ill U:
39.5,63) ●t; ;d TmI
U h 1
a.
50
21 ;n n;
Ztl lit inltHH ’●1 t| It i
o It t iiil
ii
un 2
4 40
QJ
!?t rtts
ni
to n
H3

w
m

H
rt
iS! B u TH
?

2 30 mm tssit
56 ifii
^JmSSIZS iHiHirr

F lo
20
Mm **’5:Kai«a '*®S|Si§i 2iff2 I iiz;
tnI
10 3t
a

ee
{imaistss OSS
U SZZS SXTZZ aSSI rt ?

aiiilill
!
I
a
5u It
IBBMffi warnims£

Fr
nZSSKZZSSss I ttiz WM TT?
22

-0.5 9.5 19.5 29.5 39.5 49.5 59.5 69.5


Age (in years) ►
for
ur
Fig. 21.18 Cumulative frequency curve

^w^in 21 polygon is obtained by joining these points by line segments


s

as
ook
Yo
eB

1 im nz
[2: Bx ton U2
u
nd i
tt :
2; 2t TIT : n

I
nt: n : isaBS)
100 t n
lj t«: g isnsis^
ii rr It!
nt1
u

ii2 !
tr
\lh iii: 11
our

tt I H- _(69.5,100)
ad

ft ni

BilKPiji:
-t;

90 2
; riii 02 ■U
n -1 i
i nt
ii II sS
i 2u Pin t
21
80
Y
Re

70
(A
nd

c
o
2 60
Fi

2 50
Q)
Si
H 40
3
2
30

20
Si
10

t;

-0.5 9.5 19.5 29.5 39.5 49.5 59.5 69.5


Age (in years) ►
Fig. 21.19 Cumulative frequency polygon
21.29
DATA REPRESENTATION AND VISUALIZATION
EXERCISE 21.2

1. The following table gives the population of India.


1941 1951 1961 1971
Year 1931

312.1 356.9 439.9 546.9


Population (in Millions) 275.5

Represent the above data diagrammatically by a bar diagram.


The following table gives the birth rate per thousand, of different countries over a certain
period. .

ow
Birth Rate
Coiintri/
India 33
40
I China
U.K. 20

e
16

re
Germany
Sweden 15

F
Newzealand 30

Frl
Represent the above data by a bar diagram.
ou
3. The following gives the expenditure of a family on different items.

sr
item of Expenditure Expenditure
Foot

Rent
kfo
40%
20%
oo
20%
Clothing
Y

10%
Education
reB

10%
Litigation
Miscellaneous 5%
uY

Represent the above data by a pie diagram.


The following table gives the areas^f different continents of the world:
ad
do

Continents Area Sin millions ofsq. kms)


in

Asia
Re

Africa
U.S.S.R. 20.5
F

' Europe i 4.9


24.3
I North America
17.9
I South America
8.5
Australia _

Represent the above data by a pie diagram,


Draw a pie diagram for the following data of investmerh pattern in a Five year plan:
14%
Agriculture
16%
Irrigation and Power
29%
Small Industries
17%
Transport
16%
Social Service
Miscellaneous 8%
21.30
APPLIED MATHEMATICS-XI

6.
The monthly profits in rupees of lOQ shops are distributed as follows:
_ Profit per s/iop 0-50 50 -100 100 -150 150 - 200 200 - 250 250 - 300
No. of shops 12 18 27 20 17 6

7.
Draw a histogram for the data and show the frequency polygon for it.
Draw a histogram and the two ^ves for tl^following data on the size of families:
No. of Children 0 1 2 3 4 5 6
No. offamilies 171 82 50 25 13 7 2
8.
The following table gives the height of trees:
Height No. of trees
Less than 7 28
Less than 14 57

w
Less than 21 92
Less than 28 134

Flo
Less than 35 216
Less than 42 287
Less than 49

ee
341
Less than 56

Fr
360
Represent the data in
the form of histogram, 'less than' ogive and 'more than' ogive.
for
ur
— — MULTIPLE CHOICE QUESTIONS (MCQs)
Mark the correct alternative in each of the folloiuing:
s
1. Tally marks are used to find
k
Yo

(a) class intervals (b) range


●7 Tu t - frequency (d) upper limits
oo

2. The difference between the highest and lowest values of the observations is called
eB

^ Th ^ (c) range (d) class-intervals


3. The difference between the upper and the lower class limits is called
(a) mid-points (b) class size
r

(c) frequency (d) mean


ou

4. In the class intervals 10 - 20,20 - 30,20 is taken


ad

in
(a) the interval 10 - 20 (b) the interval 20 - 30
Y

(c) both intervals 10 - 20, 20 - 30 (d) none of the intervals


5. In a frequency distribution, the mid-value of a class is 15 and the class intervals
Re
nd

is 4. The
lower limit of the class IS

(b) 12 (c) 13 (d) 14
Fi

oHte daltre:°^ '


(a) 47 and 37 (b) 37 and 47
_ -n u (c) 37.5 and 47.5 (d) 47.5 and 37.5
7. the number of times a particular item occurs in a given data is called its
(a) variation
(b) frequency
(c) cumulative frequency (d) class-size
8. The width of each of nine classes in a frequency distribution is 2.5 and the lower class
oimdary of the lowest class 10.6. Then the upper class boundary of the highest class i is

(a) 35.6 fb) 33.1 (c) 30.6 (d) 28.1


9. The following marks were obtained by the students in a test-
81, 72,90, 90, 86,85,92, 70, 71,83, 89,95, 85, 79,62
The range of the marks is
(a) 9 (b) 17 (c) 27 (d) 33
21.31
DATA REPRESENTATION AND VISUALIZATION

10. Tallys are usually marked in a bunch of


(a) 3 (b) 4 (c) 5 (d) 6
11. Let / be the lower class limit of a class-interval in a frequency distribution and }n be the
mid-point of the class. Then, the upper class limit of the class is
l + m
(a) 771 + — (b) (c) 2m -1 (d) W-2I
2

ANSWERS

1. (c) 2. (c) 3. (b) 4. (b) 5. (c) 6. (a) 7. (b) 8. (b)


9. (d) 10. (b) 11. (c)
FILL IN THE BLANKS TYPE QUESTIONS (FBQs)
!. The class-mark of the class 130-150 is
2. In a frequency distribution, the mid value of a class is 10 and the width of the class is 6. The

w
lower limit of the class is
3. In the class inter\'als 10-20,20-30, the number 20 is included in the class

F lo
●1. The width of each of five continuous classes in a frequency distribution is 5
class-limit of the lowest class is 10. The upper class limit of the highest class IS‘
5. The class marks of a frequency distribution are given as follows: 15,20,25,
and the lower
The class

ee
corresponding to the class mark 20 is

Fr
6. In a frequency distribution, the mid value of a class is 42 and the width of the class is 10,
then the class interval is
for
7. If / is the lower class limit of a class in a frequency distribution and m is the mid-point of the
class, then the upper class limit of the class interval is
r
A grouped frequency distribution table with classes of equal size, using 63-72
You

8.
s

(72 included) as one of the classes, is constructed for the following data :
ook

30,32,45,54,74, 78,108,112, 66,76,88,40,14,20,15,35,44,66,75,84, 95,96 102,110,88,74,


eB

112,14, 34,44
The number of classes in the distribution will be
9. A grouped frequency table with class intervals of equal sizes using 250-270 (270 not
our
ad

' included ) as one of the class interval is constructed for the following data:
268, 220, 368, 258, 242, 310, 272, 342, 310, 290, 300, 320, 319, 304,402, 318, 406, 242, 354,278,
210, 240, 330,316, 406, 215, 258, 236
dY

The frequency of the class 310-330 is


Re

10. The range of the data : 25,18, 20, 22,16, 6,17,15,12, 30, 32,10,19, 8,11, 20
Fin

is

11 The marks obtained by 17 students in a Mathematics test (out of 100) are given below:
' 91, 82,100,100, 96, 65, 82, 76, 79, 90,46, 64, 72, 68, 66, 48, 49.
The range of the data is
12. The class intervals in a frequency distribution are :
40 - 44, 45 - 49, 50 - 54, If the frequency distribution, is converted into a continuous
frequency distribution then the class intervals are
13. Let m be the mid-point and ii be the upper class limit of a class in a continuous frequency
distribution. The lower class limit of the class is
ANSWERS

1. 140 2. 7 3. 20-30 4. 35 5. 17.5-22.5


7. 2m -/ 8. 10 9. 6 10. 12 11. 54
6. 37 - 47
12. 39.5 - 44.5, 44.5 - 49.5, 49.5 - 54.5, 13. 2m - u
DATA INTERPRETATION -1
(CENTRAL TENDENCY)

JOii;.,.

In the previous chapter, we have studied frequency tables and their use in the graphical
presentation of raw data. The variable in a frequency table can be either qualitative or
quantitative. In the case of quantitative variables or those of qualitative variables which
can

be measured quantitatively the information contained in the raw data, or in the associated

w
frequency table can also be presented by means of few numerical values. Methods providing
such values are called measures of location or measures of central tendency and the numerical
value obtained is called an or

F lo
An average or a central value of a data or a statistical
describes the characteristics of the entire data or
scries is the value of the variable which
the associated frequency distribution In

e
Fre
other words, an average of a distribution is the value of the variable which is representative
of the entire distribution. Such a value is of great significance because it depicts the
characteristics of the enHre data. Since an average represents the entire data, therefore its
for
value lies somewhere in between two extremes i.e. the largest and smallest items. For this
average is called a measure of locaHon or a measure of central tendency.
r
reason an
You
oks

22.2 MEASURES OF CENTRAL TENDENCY


eBo

The commonly used measures of central tendency (or averages) are : (i) ArithmcHc mean (AM)
or simply mean (ii) Geometric Mean (iii) Harmonic Mean (iv) Median (v) Mode.
MEAN OF INDIVIDUAL OBSERVATIONS OR UNGROUPED DATA
ad
our

22.3 ARITHMETIC

■ -iNHION I/.Yi,.Y2,.Y3,....,.r„ flrenivalues of a variable X, then the arithmetic mean or simply the
mean of these values is denoted by x i^"d is defined as
dY
Re

.Y, + .Y^ + Y3 + .. ■ + .Y, If


Fin

X =
V 1=1

Here, the symbol Z1=1


denotes the sum x\ + -Y's + Y3... : .Y...

In other words, the arithmetic mean of a set of observations is equal to thei


ir sum divided by the total
number of observations.
The following examples illustrate the use of the above definition in finding the A.M. of
individual observations.

ILLUSTRATIVE EXAMPLES

KXAMl'LE 1 If the heights of 5 persoos ore 144 an. 152 an, 151 an, 158 an and 155 an
respectively. Find the mean height.
22.2
APPLIED MATHEMATICS-XI

SOLUTION We have.
144 + 152 + 151 + 158 + 155 760
Mean height = cm = 152 cm
5 5

EXAMPLE 2 Find the arithmetic mean of first 6 natural numbers.


SOLUTION First six natural numbers are 1, 2,3,4, 5,6. Let x be their arithmetic mean. Then,
X =
1 + 2 + 3 + 4 + 5+ 6 21 7
— = — = 3.5
6 6 2

EXAMPLE 3 Find the arithmetic mean of first ten odd natural numbers.

ow
SOLUTION First ten odd natural numbers are 1, 3, 5, 7, 9, 11, 13, 15, 17, 19 Let v be their
arithmetic mean. Then, , ^ -.-ci ^ ul uitir
X = 1 + 3 + 5 + 7 + 9 + 11 + 13 + 15 + 17 + 19 100
= 10
10 10

e
22.3.1 PROPERTIES OFARITHMETIC MEAN
Arithmetic mean has tlie following properties:

re
Flr
II

F
PROPERTY1 Ifx is the mean of n observations -Vj, ^2,..., .v,„ then prove that i.e.
1=1
the algebraic sum of deviations from mean is zero.
PROOF We have.
ou
sr
_ If”

X = - Z-v
II
=> ;iX = Z -V
1=1
fo
k
oo
Now, Z (Xi - X) = (Xi - X) + (.V2 - X) + ... + (r - X)
1=1
Y

n
reB

Z (Xi -X) = (a-j + A-, + ... + -Y„)-/lX


1=1
uY

II II

Zu-/-x) = Za.',.-iiX
1=1 1=1
ad
do

II

Z (-V,- ~X) = nX-nX


1=1 [Using (i)]
in

II

Z(T,-X) = 0
Re

1=1
F

II

Hence, 2](x,-X) = 0.
1=1

PROPERTY 2 If X is the mean of n obsenmtious then the mean of the observations


+ fl, a*2 + a,..., A'„ + aisX + a.
i.e. if each observation is increased by a, then the mean is also
increased by a.
PROOF We have.
1
X = - Z^ t

Let Xi be the mean of observations A'] + a,a'2 + a,...,x,j + a. Then,


+ 1?) + (a'2 + fl) + ... + (a~„ + a)
n
22.3
DATA INTERPRETATION-1 (CENTRAL TENDENCY)

(a.-| + ^2 + ■■■ + .Y„) + nn


Xi =
n

1/ \
Xi = -(.Xi + .Y-, + ... + .Y„) + —
n ■ ”

1 "

Xi = - I
+ n = X + Cl [Using (i)j
”l,=l
PROPERTY 3 Ifx isthemeanofXx,X2,.. x„, then the mean ofax^.axi,..., n.v is a X, zuheren is ●/ II

f.t’. if, each observation is multiplied by a non-zero number a, then the


any number different from zero i.e.
mean is also multiplied by a.

ow
PROOF We have,
if”
x = i» I E-v
/=1
I

Let X'' be the mean of /?.Y|, aX2,■ ■ ■, c^x„. Then,

e
re
X' =
ax^ + a%2 + ● ■. + ax n

rFl
F
n

a{x, + .V2 + ... + .Y,,)


X' =

r
n
ou
X' = a
1
II

E X.- = aX
fo [Using (i)]
ks
I
n
\ '* 1=1
oo

, then the mean of


PROPERTY 4 f/X mean of n observations a.'|,.Y2/-V3,-- ●f
XV
Y
eB

^2. ^3
f / — is zvhereais any non-zero number.
a n a a a
ur

i.e. if each observation is divided by a non-zero number, then the mean is also divided by it.
ad

PROOF We have.
Yo

n
1
d

I
n
Re
in

X
_ x^ X-.
Let X' be the mean of —' “f" - Then,
F

a a a

U X1 X
77
X' = -
n \ a a n

X' = -
1 f Yi + -Y2 + ●●● + II

n V a

X' = -
If Y-] + -Y2 + . ● ● + A." II

a V n

II
1 1
X' =- = -(X) [Using (i)]
a n a
V 1=1

X
X' = —
a
22.4
APPLIED MAI HEMAfICS-XI

If X is the mean of n observations x^, X2, x„, then the mean ofx■^ - a, X2 - a,
. . X
tt -a is X - a, where a is nn\/ real number.
We have,
n
1

n
\ ;=1

Let X' be the mean of .y, - a, Xo - a,., .x -a. Then,

ow
X' ^ (-^1 -n) + (x2-a) + .■■ + (y„ - a)
n

(yj + y, + ... + Y„) - na

e
X’ =
n

re
1 ” na

Flr
X' =
- Z -^ — = X a

F
[Using (i)]
/
a
ou
ILLUSTRATIVE EXAMPLES

sr
If the mean of 6,4, 7, p and 20 is 8, find the value of} 5.

SOLUTION Since 8 is the mean of 6,4,7, p, 10.


_ 6 + 4 + 7 + p + lO
5
fo
k
=> 40 = 27 + ;?=>;) = 13.
oo
Find the sum of the deviations of the variate values 3,4,6, 8,14 from their mean.
.I’i I .
Y

SOLUTION Recall that the deviations of the values Yj, X2, Y3, x„ about A are
reB

x-i - A, X2 - A, Y3 - A,..., Y„ - A.
uY

Let X be the mean of the values 3,4,6,8,14. Then,


3 + 4 + 6 + 8 + 14 35
X =
5 5 ^
ad
do

Now, sum of the deviations of the values 3,4,6,8,14 from thei r mean X = 7 is given by
= (3 - 7) + (4 - 7) + (6 - 7) + (8 - 7) + (14 - 7) = -4 - 3 - 1 + 1 + 7 = 0.
in

The mean of 40 observations was 260. It was detected on rechecking that the value of
Re

165 was wrongly copied as 125 for computation of mean.


F

Find the correct mean.


SOLUTION Here, n = 40, X = 160
1 1
So, X - -(2y, ) => 160 = —(Sy, ) ^ SYy = 160 X 40 = 6400
Incorrect value of Iy; = 6400
Now,

Correct value of Iy, = Incorrect value of Sy, - Incorrect item + Correct item

Correct value of Sy, = 6400 - 125 + 165 = 6440


Correct mean = Correct value of Iy./ —
6440
= 161.
n 40

CXAMlM.h ■! The mean of 100 itetns was found to be 30. If at the time of calculation two iterns were

wrongly taken as 32 and 12 instead of 23 and 1 IJind the correct mean.


22.5
DATA INTERPRETATION-1 (CENTRAL TENDENCY)

SOLUTION Here, n = 100, X = 30.

So, X = I.V, = nX => I.Y,- = 100 X 30 = 3000


n

Incorrect value of I.r, = 3000


Now, Correct value of lY,' = Incorrect value of Zy,- - (Sum of incorrect values)
+ (Sum of correct values)
Correct value of Zy,- = 3000 - (32 + 12) + (23 + 11 ) — 2990.
Correct value of Zy, 2990
= 29.9
Correct mean =
100

ow
n

LXAMIM.I ;The mean monthh/ salary of 10 members of a group is ? 1445, one more member whose
monthly salary is ? 1500 has joined the group.
Find the mean monthly salary of
11 members of the group.
SOLUTION Here, n = 10, X = 1445.

e
re
So,

rFl
X = -(Zy,) => hX = Zy,- => 10 x 1445 = Z.y,- => Z.v,- = 14450.

F
n

So total monthly wages of 10 persons =^14450.

r
ou
Monthly salary of one more person who joined the group is t 1500.
Total monthly wages of 11 persons =? 14450 + ^ 1500 =^15950
fo
ks
Total monthly wages
oo

So, average monthly salary to 11 persons 11


Y

15950
B

= 11450
11
number, what will be the
re

r.XAMPLE 6 The mean of 10 numbers is 20. If 5 is subtracted from every


new mean?
ou
Y

SOLUTION Let Yi, Y2, ..., Y]o be 10 numbers with their mean equal to 20. Then,
ad

X = -(El'i)
d

n
in
Re

20 =
■Y| + .Y2 + ●●● + Xio
10
F

...(i)
.V-| + Y2 + ... + Y^o — 200
New numbers are .y-, — 5, .Y2 - 5,..., Y]o - 5. Let X' be the mean of new numbers. Then,
X' =
(■Yi - 5) + (y2 - 5) + ■■■ + (y,o - 5)
10

X' =
(Yi +Y-2 +... + Y^n)-5xlO _ 200-50 [Using (i)]
10 10

X =15.

EXAMPLE 7 The mean of 16 numbers is 8. If 2 is added to every number, what will be the new mean?
SOLUTION Let Y^, Y2, Y3,..., Y^f, be 16 numbers with their mean equal to 8. Then,
1
X =-(Z.q)
n
22.6
APPLIED MATHEMATICS -XI

_ T] + A~2 4- ... +
16

.V] + a'2 + ... + .y,6 = 16 X 8 = 128

New numbers are Yi + 2, Y2 + 2, Y3 + 2,..., Y^g + 2.


Let X' be the mean of new numbers. The u.
^ _ (Yi + 2) + (X2 + 2) + ... + + 2)
16

'Y ^ + .^2 + ●●● + -^16) + 2x 16 128 + 32


16 16 [Using (i)]
160
X’ = = 10.
16

w
LXA.MPLt 8 The mean of 5 number is 18. If one numbers is excluded, their mean is 16 Find the
excluded number.

SOLUTION Here, n = 5, X = 18.


1

F lo
ee
X = -(lY,-) =5. Sy,- = nX => lY,- = 5 X 18 = 90.

Fr
n

So, total of 5 numbers = 90.


Let the excluded number be a. Then, total of 4 numbers for is 90 - a.
ur
90 ~ a
Mean of 4 numbers =
4
s
ook
Yo

90-a
16 = ^ 90 - i? = 64 =5- rt = 26.
4
eB

Hence, the excluded number is 26.

IIXAMPLE 9 If the mean ofifve observations x, x + 2, y + 4, y + 6, y + 8 is 11, find the mean offirst
r

three observations.
ad
ou

SOLUTION We have,
Y

11 = Y + (y + 2) + (y + 4) + (y + 6) + (y + 8)
Re

5
nd

55 = 5y + 20
Fi

5.V = 35 ^ Y = 7

Now, mean of first three observations

_ Y + (y + 2) + (y + 4) 3y 4- 6
= .v + 2 = 7 + 2 = 9
3 3 [●●● T = 7]

TXAMPLEIO If the mean of n observations ax^,ax2,ax^,ax„ is aX,showthat

(rtYj - aX) + {0x2 - aX) + ... + (flY„ - aX) = 0


SOLUTION We have.

aX = nx-^ + ax2 + . ■. + ax,


n
a.Xj + ax2 + ... + ax„ = n (aX)

Now, (i7Y, - aX) + (ax2 -aX)+ ... + (ax„ - aX)


DATA INTERPRETATION-1 (CENTRAL TENDENCY) 22.7

= {rt.v, + ^7.V2 + ... + ax„) - {nX -i- ^7X -i-... + aX)


n-terms

= n(flX)-n(rtX) = 0. [Using (i)]

EXAMPLE 11 The mean of n observnfions x^,X2,...,x„ is X . If (n - b) is nchled to each of the


observations, shozv that the mean of the new set of observations is X + (a - b).
SOLUTION We have.
x\ + .V2 + ... + -V ;;
X =
n

LetX'be the mean of x^ + {a - b), X2 + {(i - b), ...,x„ + {a -h). Then,


^ {xi + (fl - b)\ + {X2 + (fl - b)] + ■ ■ ■ + +{a- ^)1
n

w
X' =
,Y^ + X2 + ●●● + .y,, + n{a - b)
n

F lo
n(a - b)
X' _
= Xi + .Y2 +... + X = X + (a - b) [Using (i)]
^2
+
n n

EXAMPLE 12 If Yj, .V2, are n values of a variable X such that

ee
Fr
n n

Z(-v
1=1
I 2) = 110 and ^ (.v,- - 5) = 20.
1=1
f}j^ value of n and the mean.
for
SOLUTION We have,
ur
n

^ (y,- - 2) = 110 and ^ (y,- - 5) = 20.


s
ook

j=l i=l
Yo

(Y,-2) + {y2-2) + ... + (y„-2)^110


eB

and. (Y| - 5) + (y2 - 5) 4-... + (y„ - 5) = 20


(y, 4- Y2 + ... + .v„) - 2?! = 110 and (Yj 4- y'2 4-... 4- y„) - 5h = 20
our
ad

^ Y; - 2n = no and ^ Yj - 5/; - 20 1=1


1=1

S -2n = 110 and S -5n = 20


Y

=>
Re

Thus, we have
nd

S - 2« = 110
...(ii)
Fi

and. S - 5/; = 20
Subtracting (ii) from (i), we get
3» = 90 => « = 30

Putting n = 30 in (i), we get


I!

S - 60 = 110 => S = 170 => 2^ Y,- = 170


i=\

1 I " 170 _ 17
Mean = — Y y,- 30 “ 3
" I /=i
17
Hence, n = 30 and mean —●

EXAMPLE 13 The sum of the deviations of a set of n values Y],y'2,...,y„ measured from 50 is
-10 and the sum of deviations of the values from 46 is 70. Find the values of n and the mean.
22.8 APPLIED MATHEMATICS - XI

SOLUTION We have

2] (-Y,- - 50 = -10 and (.v,- - 46) = 17


1=1 1=1

2] Y,- - 50ir = -10


1=1

and. 2] Y,- - 4611 = 70 ...(ii)


(=1

Subtracting (ii) from (i), we get

ow
-4n = -80 => 1! = 20

Putting II = 20 in (i), we get

2; Y; = 50 X 20 = -10 => 2] Y,- = 990

e
1=1 (=1

re
1 990
Mean - - V ,y = 49.5
4m 20

Frl
F
Hence, n = 20 and mean = 49.5
EXERCISE 22.1
ou
r
If the heights of 5 persons are 140 cm, 150 cm, 152 cm, 158 cm and 161 cm respectively,

so
find the mean height. kf
Find the mean of 994, 996, 998, 1002 and 1000.
Find the mean of first five natural numbers.
oo

Find the mean of all factors of 10.


Y

Find the mean of first 10 even natural numbers.


B

. Find the mean of .v, j + 2, .v + 4, .r + 6 , .y + 8.


re

Find the mean of first five multiples of 3.


oY

Following are the weights (in kg) of 10 new born babies in a hospital on a particular day:
u

3.4, 3.6, 4.2, 4.5, 3.9, 4.1, 3,8, 4.5, 4.4, 3.6. Find the mean X .
ad

The percentage of marks obtained by students of a class in mathematics are : 64, 36, 47,
d

23, 0, 19, 81, 93, 72, 35, 3,1. Find their mean.
in

The numbers of children in 10 families of a locality arc:


Re

2,4, 3, 4, 2, 0, 3, 5,1,1, 5. Find the mean number of children per family.


F

Explain, by taking a suitable example, how the arithmetic mean alters by (i) adding a
constant k to each term, (ii) subtracting a constant k from each them, (iii) multiplying
each term by a constant k and (iv) dividing each term by a non-zero constant k.
The mean of marks scored by 100 students was found to be 40. Later on it was
discovered that a score of 53 was misread as 83. Find the correct mean.
The traffic police recorded the speed (in km/hr) of 10 motorists as 47,53,49, 60,39,42,55,
57, 52,48. Later on an error in recording instrument was found. Find the correct overage
speed of the motorists if the instrument recorded 5 km/hr less in each case.
The mean of five numbers is 27. If one number is excluded, their mean is 25. Find the
excluded number.

The mean weight per student in a group of 7 students is 55 kg. The individual weights
of 6 of them (in kg) are 52, 54, 55, 53, 56 and 54. Find the weight of the seventh student.
The mean weight of 8 numbers is 15. If each number is multiplied by 2, what will be the
new mean?
DATA INTERPRETATION-1 (CENTRAL TENDENCY! 22.9

The mean of 5 numbers is 18. If one number is excluded, their mean is 16. Find the
excluded number.
The mean of 200 items was 50. Later on, it was discovered that the two items were
misread as 92 and 8 instead of 192 and 88. Find the correct mean.
If M is the mean of .Yi , .^2, T3, .v^, .Y5 and .y^,, prove that
(.Yi - M) + {X2 -M) + (.Y3 - M) + (.Y4 - M) + (,Yg - M) + (.Yft - M) = 0.
Durations of sunshine (in hours) in Amritsar for first 10 days of August 1997 as reported
by the Meteorological Department are given below:
9.6, 5.2, 3.5, 1.5, 1.6, 2.4, 2.6, 8.4, 10.3, 10.9
10

(i) Find the mean X (ii) Verify that ^ (-t/ - X) - 0


;=1

Find the values of n and X in each of the following cases:


11 II

(i) I (^-12) 10 and ^ (.Y, - 3) = 62


/=! i=l

ri

10) = 30 and ^ (y,- - 6) = 150


(ii) Z(-'-
1=1 /=i

The sums of the deviations of a set of n values -Yi,.Y2, F low


...,.y„ measured from 15 and
- 3 are - 90 and 54 respectively. Find the value of n and mean.
Find the sum of the deviations of the variate values 3, 4, 6, 7, 8,14 from their mean.
If X is the mean of the ten natural numbers .Yj, .Y2, .Y3,..., show that
forFe
(●^1 X) + (.Y2-X) + ..- + (-i:io-X) = 0.
NSWERS

152.2 998 3 4.5


eBoks
Your

11 -Y + 4 9 4
ad

39.5 3 39.7 55.2 km/hr.


our

30 26
35 61 kg.
Re

50.9 5.6 (i) n = 8, X = 10.75 (ii) n = 30, X = 11


15
n = 8,X=- 0 0
FindY

22.'' ARITHMETIC MEAN OF GROUPED DATA OR DISCRETE FRFOuENC>


DISTRlB^TlOh

In a discrete frequency distribution the arithmetic mean may be computed by any one of the
following methods:
(i) Direct Method
(ii) Short-cut Method
(iii) Step Deviation Method.
22. .'RECTMETHOr

. :NiTi ■ If a variate X lakes values x■^, Yj, ..., x„ zoith corresponding frequencies f, A, A/ -v ,f II

respectively, then arithmetic mean of these values is


22.10 APPLIED MATHEMATICS -XI

X = /].V] + /2.V2 + ... + f„x II

/] + A + ● ● ● + /,
II

I/..V /

/-I
or. X =
N , where W = 2 '/ = /i + A + ●●● + /„
;=I

We may use the following algorithm to compute arithmetic mean by direct method.
ALGORITHM

STLP I
Prepare the frequency table in such a umj that its ifrst column consists of the values of the
variateand the second column the corresponding frequencies.
STl'.l' II
Multiply the frequency of each row with the corresponding values of variable to obtain
third column containing f^i-

w
STEP
Jil Find the sum of all entries in III column to obtain '.FfiXj.
STEP i\’

^fiXi
F lo
Find the sum of all the frequencies in II column to obtain Zf or N.

e
STEP V
Use the formula: ^ =

Fre
N

The following examples will illustrate the above algorithm.


for
r
ILLUSTRATIVE EXAMPLES
You
oks

EXAMPLE 1 Find the mean of the following distribution:


eBo

x: 4 6 9 10 15
/: 5 10 10 7

SOLUTION Calculation of Arithmetic Mean


our
ad

X.I
fi
4 5 20
dY
Re

6 10 60
Fin

9 10 90
10 7 70
15 8 120

N = If = 40 IfjXj = 360

Mean = X = IfjXj _ 360 = 9.


Xf 40
EXAMPLE 2
Following table shows the lueights of 12 students:
Weight (in kgs): 67 70 72 73 75

Number of Students: 4 3 2 2 1

Find the mean weight.


22.11
DATA INTERPRETATION-I (CENTRAL TENDENCY)

SOLUTION Calculation of Arithmetic Mean

Weight (in kgs) Frequeuci/ // Xi


X,I fi
67 4 268

70 3 210

72 2 144

73 2 146

75 1 75

= 843
N = I/; = 12
843
Mean = X = = 70.25 kg.

w
N 12

EXAMPLF.3 Find the mean of the following distribution:


.V;

/:
10

7
30 50

F lo
10
70

15
89

10

ee
SOLUTION Calculation of Mean

Fr
-T;I fi for //●Vy

10 7 70
r
8 240
30
You
s

500
ook

50 10

70 15 1050
eB

89 10 890

ly;. = N = 50 I.f-Xi = 2750


our
ad

Mean =
ZfXj _ 2750 = 55.
N 50
dY
Re

EXAMPLE 4 Jf the mean of the folhioing distribution is 6, find the value of p.


2 4 6 10 p +5
Fin

-v;

2 3 1 2
f 3

SOLUTION Calculation of Mean

X;I f fi^i

2 3 6

4 2 8

6 3 18

10 1 10

p +5 2 Ip +10

N = lf= 11 IfXi = 2p + 52
22.12 APPUED MATHEMATICS-XI

We have, N = Zfi = 11, = 2p + 52

Mean = 6^2^ => 66 = 2p + 52 ^ 2p = \4 => p ~ 7


N 11

Find the value ofp, if the mean of thefollozving distribution is 7.5.


x: 3 5 7 9 11 13
y- 6 8 15 8 4
P
SOLUTION Calculation of Mean

X;I
fi fiXi
3 6 18
5 40

w
7 15 105
9
P 9p

F lo
11 8 88
13 4 52

ee
N = Ifi=4\+p 'LfjXj = 303 + 9p

Fr
We have. Xfi = 41 + /;, IfjXi = 303 + 9p for
ur
Mean =
s

303 4- 9p
ook
Yo

7.5 =
41 + p
eB

7.5 X (41 + p)^ 303 + 9p


307.5 + 7.5p = 303 -r 9p ^ 9^ _ 7.5^ = 397.5 - 303 => l-5p = 4.5 => p = 3
our
ad

Find the missing frequencies in the following frequency distribution if if is known that
the mean of the distribution is 1.46.
Y

Number of accidents (x): 0 1 2 3 4 5 Total


Re

Frequency (f): 46
nd

7
7 25 10 5 200

Let the missing frequencies be /] and/2.


Fi

SOLUTION

Calculation of Mean

xI
fi fi^i
0 46 0
1
/i fi
2
A 2/2
3 25 75
4 10 40
5 5 25

N = 86 + /, + /. 140 + / + 2/2
22.13
DATA INTERPRETATION-! (CENTRAL TENDENCY)

We have; M = 200
-(i)
200 = 86 + /i + /2 => /i + A = 114
Also,
Mean = 1.46

1.46 =
N

140 + fj + ‘2-/2 ...(ii)


1.46 =
200
=i> 292 = 140 +/i+2/2 =>/i+2/2 =152

Solving (i) and (ii), we get /i - 76 and /o - 38.

w
BXERCfSE 22.2

Calculate the mean for the following distribution;

Flo
5 6 7 8 9
x:

e
14 11 3

re
4

- Find the mean of the following data:

F
23 25 27 29 31
A'.’ 19 21
ur
r
16 18 16 15 13
13 15

^. Find the mean of the following distribution;


fo
ks
10 12 20 25 35
Yo
.y;
oo

3 10 15 7 5
f--
4. Five coins were simultaneously tossed 1000 times and at each toss the number of heads
B

were observed. The number of tosses during which 0, 1, 2, 3, 4 and 5 heads were
re

obtained are shown in the table below. Find the mean number of heads per toss.
u

No. of tosses
ad

No. of heads per toss


Yo

38
0

1
144
d
Re

2 342
in

3
287
F

4 164
25
5

Total 1000

The mean of the following data is 20.6. Find the value of p.


10 15 25 35
y; P
3 10 25 7 5
/●
If the mean of the following data is 15, find p.
5 10 15 20 25
y;

6 6 10 5
P
22.14 APPLIED MATHEMATICS-XI

'7. Find the value of p for the following distribution whose mean is 16.6.
-v; 12 15 20 25 30
P
12 16 20 24 16 8 4

8. Find the missing value of p for the following distribution whose mean is 12.58.
.y; 5 10 12 20 25
P
f: 2 5 22 7 4 2

9. Find the missing frequency [p) for the following distribution whose mean is 7.68.
y; 3 5 7 9 11 13
6 8 15 8 4
P

w
10- Find the value of p, if the mean of the following distribution is 20.
y; 15 17 19 20+;? 23
f:

o
2 3 4
5p 6

e
11- Candidates of four schools appear in a mathematics test. The data were as follows:

re
Schools

rFl
No. of Candidates Average Score

F
I 60 75

r
n 48 80
ou
m Not available
fo
ks
55
IV 40 50
oo

If the average score of the candidates of all the four schools is 66, find the number of
Y

candidates that appeared from school III.


eB

12. Find the


missing frequencies in the following frequency distribution if it is known that
the mean of the distribution is 50.
ur

Y: 10 30 50 70 90
ad
Yo

/: 17 /i 32 A 19 Total 120.
d

ANSWERS
Re
in

1. 7.025 2. 25 3. 20 4. 2.47 5. p= 20
F

6. 8 7. 18 8. 157.9 10. p = l n. 52
-12. /i =28,/2=24
22.4.2 SHORT-CUT METHOD

If the values of y or (and)/are large, the calculation of AM by the direct method is quite
tedious and time consuming, because calculations involved are lengthy. In such a case to
minimize the time involved in calculation, we take deviations from an arbitrary point as
discussed below.
Let be values of
variable X with corresponding frequencies f\, f2>
a

respectively. Taking deviations about an arbitrary point 'A', we have


d] = Y/ - A, i = 1,2,3, ●●●, n
fidi= MXi-A);i = 1,2,3,- ●/»
22.15
DATA INTERPRETATION-1 (CENTRAL TENDENCY)

II II

1=1 (=1

II II II

i=l i=l i=l

II
II I!

=> ZfA = - AN N = Y.fi


1=1
i=l 1=1

1 AN

ow
I
N 1=1 1=1

II
1
II 1

N N 1=1
i=l

e
1

re
N i=l

Fl
F
Finding AM by using the above formula is known as the short-cut method.
ur
NO) i- The number 'A' is generally known as the assumed mean and is generally chosen in such a

r
way that the deviations are small. fo
Following algorithm may be used to find arithmetic mean by the short-cut method.
ks
Yo
oo

ALGORITHM

Prepare the frequency table in such a zuay that its first column consists of the values of the
eB

variable and the second column consists of the corresponding frequencies.


t^TEP II Choose a number 'A' (preferable among the values in first column) and take deiyiations
ur

d- = x,--A of the values Xj of variable X about A. Write these deviations against the
ad

corresponding frequencies in the third column.


Yo

!r>lEP ill Multiply the frequencies in column II with the corresponding deviations dj in column III to
prepare column IV consisting of f dj.
d
Re
in

STEP \\ Find the sum of all entries in column III to obtain Z-^'^' and the sum of all frequencies in
F

1=1

column II to obtain Z/'


1=1

1
STEP V Use the formula: X - A + ZM- ●
N V 1=1

Following examples will illustrate the above algorithm.

ILLUSTRATIVE EXAMPLES

EXAMPLE i The following table shows the zveights of 12 students:


67 70 71 73 75
Weight (in kg):
4 3 2 2 1
Number of students:
Find the mean weight by using short-cut method.
22.16 APPLIED MATHEMATICS - XI

SOLUTION Let the assumed mean be .<4 = 72

Calculation of Mean

weight No. of d,- = .V,- - A = Xj -72 fidi


in Kg students

a:.I fi
67 4 -5 -20

70 3 -2 -6

72 2 0 0

w
73 2 1 2

75 1 3 3

Flo
N = Z/. = 12 Tfd^ = -21
We have.

ee
Fr
N = 12, Z/;rf/ = -21, and A = 71
1
Mean = A +
N
(IM) for
ur
Mean = 72 +
(-21) = 72-
7 288-7 281
= 70.25 kg
s
12 4 4 4
k
Yo

Hence, mean weight = 70.25 kg.


oo
eB

EXAMI’U: 2 Find the mean xvage from the data given below:
Wage {in f): 800 820 860 900 920 980 1000
No. of lookers: 7 14 19 25 20 10
r

5
ou
ad

SOLUTION Let the assumed mean be /I = 900.


Y

Calculation of Mean

Wage (in ^ No. of workers dj = Xj - A = Xj - 900


nd

fid;
Re

.V.I
fi
Fi

800 7 - 100 - 700

820 14 - 80 - 1120

860 19 - 40 - 760

900 25 0 0

920 20 20 400

980 10 80 800

1000 5 100 500

iV = I/- = 100 Ifdi = -880


22.17
DATA INTERPRETATION-! (CENTRAL TENDENCY)

We have.

N = 100,1 fjdi = - 880 and ^ = 900

Mean X = A +
IM-
N

-880
Mean X = 900 = 900 - 8.8 = 891.2
100

Hence, mean wage = ? 891.2.

ow
-)-3 TEP-DE- lA.iON METHOD

Sometimes, during the application of the short-cut method for finding AM, the deviations d
are divisible by a common number h (say), in such a case the arithmetic is reduced to a great
extent by taking

e
re
.V;I - A .
; i = 1,2,3,●●●,/»

Frl
ii,
h

F
Xj = A + hUj I' i = 1,2,3,---,h
ou
or
fiXi = Afi + hfiUi, / = l,2,3,---,n
kfs
IJ II
oo
/”/
/=! 1=1 1=1
Y
B

II
1 AN
I
+ h
re

N V <=i N
oYu

|iN
ad

jj

X = A + /i
i=l
d

Finding AM by using this formula is known as the step-deviation method.


mean by step deviation method:
in

Following algorithm may be used to find the arithmetic


Re
F

uGORITHM

Ohiain the frequency distribution and prepare the frequency table in such a xvay that its
first column consists of the values of the variable and the second column corresponding
frequencies.
Choose a number ‘A’ (generally known as the assumed mean) and take deviations
dj = Xj - A about A. Write these deviations against the corresponding frequencies in the
third column.

Choose a number h, generally common factor of all dfs in III column, divide deviations dj byh
toget Uj. Writefhese against the corresponding dj's in the IV column.
Multiply the frequencies in 11 column with the corresponding ufs in IV column to prepare
V column of fjUj.
22.18 APPLIED MATHEMATICS - XI

STi-l* V
Tind the sum of nil entries in V column to obtain and the sum ofnllfrecjuencies in II

( "
column to obtain N =
V /=] >

5IHP VI Use the formula: ^ ^+h● fiii

Following examples will illustrate the above algorithm.


ILLUSTRATIVE EXAMPLES

IZXA.VIPLE 1 Find the mean wage from the following data:

ow
Wage (in ^ : 800 820 860 900 920 980 1000
No. of workers: 7 14 19 25 20 10 5

SOLUTION Let the assumed mean be /I = 900 and h = 20.

e
Fl
Calculation of Mean

re
F
Wage No. of di=Xi~A ll:I
■V, - 900
20
ur
(in f) X, workers f = .Y;I - 900

or
sf
800 7 - 100 -5 -35
k
Yo
820 14 - 80 -4 -56
oo

860 19 - 40 -2 -38
B

900 25 0 0 0
re

920 20 20 1 20
u
ad

980 10 80 4
Yo

40

1000 5 100 5 25
d

N = !/● = 100
Re

=-44
in

We have.
F

N = 100, S = -44, ^ = 900 and h = 20.

Mean = X = A + h
N

-44
X = 900 + 20 X = 900-8.8 = 891.2
100

Hence, mean wage = ? 891.2.


EXAMPLE
2 Apply step deviation method to find the AM of the follozving frequency distribution
Variate (y): 5 10 15 20 25 30 35 40 45 50
Frequency (f): 20 43 75 67 72 45 39 9 8 6
SOLUTION Let the assumed mean be /4 = 25 and h = 5.
22.19
DATA INTERPRETATION-1 (CENTRAL TENDENCY)

Calciilntion of Mean
Variate Freqiienq/ Deviations X, - 25
I/;I
5 /i"/
Xi fi d, =x,-25
- 20 - 4 - 80
5 20
- 15 -3 - 129
10 43

75 - 10 -2 - 150
15
-5 - 1 -67
20 67
0 0 0
25 72
5 1 45
30 45
10 2 78
35 39
9 15 3 27
40

w
20 4 32
45 8
25 5 30
50 6

F lo
N = S/;- = 384 IM=-214

ee
We have,
N = 384, A = 25, h = 5 and Z /,«,● = -214

Fr
1
Mean = X — A + h Z /,»,● for
N
('-214^ = 25 - 2.786 = 22.214
ur
Mean = 25 + 5 x
384
s

EXAMPLE 3 The weights in kilograms of 60 workers in a factonj are given in the following
ook
Yo

frequency table. Find the mean iveight of a zvorker.


eB

61 62 63 64 65
Weight (in kg) x: 60
14 16 10 7
5
No. of workers f
SOLUTION Let the assumed mean be A = 63
our
ad

Calculation of Mean

Weight (in kg) No. of workers dj = -V,- - 63 fA


Y

f
Re

X
nd

-3 - 15
60 5
- 16
Fi

61 8 -2
- 1 - 14
62 14
16 0 0
63
10 1 10
64
7 2 14
65

N = Z// = 60 UA = -21

We have, N = 60, A = 63 and = -21


Mean = A +
N

-2n 7
Mean = 63 + = 63 - — = 63 - 0.35 = 62.65
60 20

Hence, mean weight of a worker - 62.65 kg


22.20
APPLIED MATHEMATICS-XI

EXAMIM
L 4 The table belozu gives the distribution of villages under different hei<^hts from sea

level in a certain region. Compute the mean height of the region:


Height (in metres): 200 600 1000 1400 1800 2200
No. of village: 142 265 560 271 89 16
SOLUTION Let the assumed mean be /I = 1400 and h= 400
Calculation of Mean
Height No. of villages
(in metres) ■Y,- -1400
.fi di = Y,- - 1400 It;
fdh
400
X:I

ow
200 142 - 1200 -3 - 426
600 265 - 800 -2 - 530
1000 560 - 400 - 1 - 560
1400 271 0

e
0 0

re
1800 89 400
2200 16

rFl 800
1
2
89

F
32

N = Z/;- = 1343 Z fiii = -1395

r
We have, A = 1400, h = 400,1 fuj = -1395 and N = 1343
ou
Mean = A + /z — S/jn,-
1 fo
ks
N

-1395
oo

Mean = 1400 + 400 x = 1400 - 415.49 = 984.51


1343
Y
eB

- -- - EXERCISE 22.3
1. The number of telephone calls received at an exchange per interval for 250 successive
r

one-minute intervals are given in the following frequency


ou

table:
ad
Y

No. of calls (y) : 0 1 2 3 4 5 6


No. of intervals (f): 15 24 29 46 54 43 39
d

Compute the mean number of calls per interval.


Re
in

2. Five coins were simultaneously tossed 1000 times, and at each toss the number of heads
was observed. The number of tosses during which 0,1,2, 3, 4 and 5 heads were obtained
F

are shown in the table below. Find the mean number of heads per toss
No. of heads per toss (.r*); 0 1 2 3 4 5
No. of tosses (f): 38 144 342 287 164 25

3. The following table gives the number of branches and number of plants in theearden of
a school. °

No. of branches (y): 2 3 4 5 6

No. of plants (/■): 49 43 57 38 13


Calculate the average number of branches per plant.
4. The following table gives the number of children of 150 families in a village
No. of children (.r): 0 1 2 3 4 5
No. of families (/■): 10 21 55 42 15 7

Find the average number of children per family.


22.21
DATA INTERPRETATION-1 (CENTRAL TENDENCY)

5. The marks obtained out of 50, by 102 students in a Physics test are given in the frequency
table below;
15 20 22 24 25 30 33 38 45
Marks (.t);
8 11 20 23 18 13 3 1
Frequency (f): 5
Find the average number of marks.
6. The number of students absent in a class were recorded every day for 120 days and the
information is given in the following frequency table :
No. of students absent (.r); 0 1 2 3 4 5 6 7
No. of days (f): 1 4 10 50 34 15 4 2
Find the mean number of students absent per day.
In the first proof reading of a book containing 300 pages the following distribution of

w
misprints was obtained: 4 5
1 2 3
No. of misprints per page (x); 0 5 1
154 95 36 9

F lo
No. of pages if):
Find the average number of misprints per page.
8. The following distribution gives the number of accidents met by 160 workers in a factory

ee
Fr
during a month.
0 1 2 3 4
No. of accidents (:r):
1
No. of workers {[): 70 52 34 3 for
ur
Find the average number of accidents per worker,
Find the mean from the following frequency distribution of marks at a test in statistics :
s
ok

35 40 45 50
Yo

5 10 15 20 25 30
Marks (x):
o

15 50 80 76 72 45 39 9 8 6
No. of students (f):
eB

. — ANSWERS
r

4. 2.35
j. 3.62 (approx)
ou
ad

1. 3.54 2. 2.47
7. 0.73 8. 0.83
V 26.08 h. 3.53 (approx)
Y

'● 22.075
Re
nd

22 4 4 ARITHMETIC MEAN OF A CONTINUOUS FREQUENCY DISTRIBUTION


Uptill now we have been discussing about various methods for
Fi

of a discrete frequency distribution. In case of a conhnuous frequency distribution or


frequency distribution with class intervals arithmetic mean may be computed by S
any of L methods discussed so far. The values of .v.,.v„.V3 -,.ri. taken as the II

classes. It should be noted that the mid-value or


mid-points or class-marks of the various
class-marks of a class interval is equal to \ (lower limit + upper limit).
Following examples will illustrate the procedure.
ILLUSTRATIVE EXAMPLES

EXAMPLE 1 Find the mean of the fotlowing frequency distribution: 40 - 50


0-10 10-20 20-30 30-40
Class-interval:
15 8 10
10
No. of ivorkers f: 7
22.22
APPLIED MATHEMATICS - XI

SOLUTION
Calciilntion of Mean

Class- Mid- ■Y,- - 25


Freqiiena/ dj = Xj - 25 iti fd^i
interz’dl
vahies (x^ f. 10

0-10 5 7 - 20 -2 - 14
10-20 15 10 - 10 - 1 - 10
20-30 25 15 0 0 0
30-40 35 10 1 8
40-50 45 10 20 2 20

ow
N = Zfi = 50 ZfUi = 4
We have.

/4 - 25,/? = 10, N = 50 and IfjUj = 4.

e
Mean = A + hl—Zftij
Fl
re
N

F
4
Mean = 25 + 10 x — = 25.8
ur
50

EXAMPLE 2 Find the mean of the following frequency distribution: or


sf
Classes: 0-20 20-40 40-60 60-80 80-100
k
Yo
Frequency: 15 18 21 29
oo

17
B

SOLUTION
Calculation of Mean
re

Classes Mid X; - 50
Frequency d; = Y,. - 50 ih
fdh
u

20
ad

values (x^
Yo

0-20 10
d

15 -40 -2 -30
Re
in

20 - 40 30 18 -20 -1 -18
40 - 60 50 21
F

0 0 0
60 - 80 70 29 20 1 29
80 - 100 90 17 40 2 34

'Lfi = 100 I fUi = 15


We have.

A = 50, h = 20, N = 100 and Ifu; = 15


Mean = A + h —Ifu,-
N

15
Mean = 50 + 20 x = 50 + 3 = 53
100
22.23
DATA INTERPRETATION-1 (CENTRAL TENDENCY)

EXAMPLE 3 The following tnble gives the cUstrilnition of toln! household expenditure (in rupees)
of inanun! workers in a city.
Expenditure: 100-150 150-200 200-250 250-300 300-350 350-400 400-450 450-500
(in r)
33 28 30 22 16 7
40
Frequency: 24

Find the average expenditure (in f) per household.

w
SOLUTION Let the assumed mean be A = 325 and h = 50.
Calculation of Mean
x-A
fd‘i
Expenditure Frequency Midvalues d:I = X,-1 - A U:I -

e
h
(in Xi fi X:

re
ro
.V;I - 325
= .v.-325
I
50

F
125 - 200 -4 - 96
100 - 150 24

Ful
- 150 -3 - 120
150 - 200 40 175

225 - 100 -2 - 66
200 - 250 33

sr
- 50 -1 -28
250 - 300 275

ko
0 0 0
o
300 - 350 30 325
50 of 1 22
350 - 400 22 375

425 100 2 32
400 - 450 16
o
21
Y
7 475 150 3
450 - 500
erB

N = I/ = 200 Tfu. = -235


uY

We have.

N = 200, A = 325, h = 50, and I/;», = -235


o
ad
d

1
X = A + /i—I fjUi
N
in

-235 235
X = 325 + 50 X = 325 - = 325 - 58.75 = 266.25
Re

200 4
F

Hence, the average expenditure is ^ 266.25.


EXAMPLE 4 A frequency distribution of the life times of 400 T.V. picture tubes tested in a tube
company is given below. Find the average life of tube.
Frequency Life time (in hrs) Frequency
Life time (in hrs)
14 800-899 62
300-399
46 900-999 48
400-499
58 1000-1099 22
500-599
76 1100-1199 6
600-699
700-799 68

SOLUTION Here, the class-intervals arc formed by exclusive method. If we make the series
an inclusive one the mid-values remain same. So, there is no need to convert the series into
an inclusive form.
22.24
APPLIED MATHEMATICS-XI

Let the assumed mean be = 749.5 and h = 100.

Calculation of Mean
X; - A
Life time Frequency Mid-Values di = X; - A U: -
fdh
h
(in hrs): ,f X
Xi - 749.5
= A-,- - 749.5 Hi =
100
300 - 399 14 349.5 - 400 -4 - 56
400 - 499 46 449.5 - 300 -3 - 138
500 - 599 58 549.5 - 200 -2 - 116
600 - 699 76 649.5 - 100 -1 - 76
700 - 799 68 749.5 0 0 0
800 - 899 62 849.5 100 1 62

w
900 - 999 48 949.5 200 2 96
1000 - 1099 22 1049.5 300 3 66

F lo
1100 - 1199 6 1149.5 400 4 24

N = !/● = 400 S fUi = -138

ee
We have, N = 400,71 = 749.5, h = 100 and IfUj = -138

Fr
1
X = A + hl—lf-u.
N
for
ur
' -138 ^ 138
X = 749.5 + 100 X = 749.5 -
I 400 = 749.5 - 34.5 = 715
s

4
ook
Yo

Hence, the average life time of a tube is 715 hours.


eB

tX.AMPLE 5
If the mean of the following distributions 54, find the value ofir
Class: 0-20 20 - 40 40 - 60 60-80 80 - 100
Frequency: 7
P 10 9
our

13
ad

SOLUTION
Computation of Arithmetic Mean
dY

Class .V,-I - 50
Mid-values Frequency dj = Xj - 50 U: =
Re

20
(X,) (fj
Fin

0-20 10 7 - 40 -2 - 14
20 -40 30 - 20
V - 1
-P
40 -60 50 10 0 0 0
60 - 80 70 9 20 1 9
80 - 100 90 13 40 2 26

Ifi = 39+ p Z/Uf =21-p


We have.

A = 50, N =39 + p,h = 20, Ifyi^- = 21 - and X = 54


22.25
DATA INTERPRETATION-I {CENTRAL TENDENCY)

1
Mean = A. + h\-~
N

21 -p
54 = 50 + 20 X
39 + p

21-p
4 = 20 X
39+ p

l = sf^
39 + pJ
^ 39 + p = 105 - 5p 6p = 66 p = 11

LX AMl’l .r The following table gives weekly wages in rupees of workers in a


organization. The frequency of class 49-52 is missing. It is known that the mean of the fnqmncy
distribution is 47.2. Find the missing frequency.

w
46 - 49 49-52 52-55
40-43 43-46
Weekly wages {f): 7 27
60

F lo
31 58
Number of -workers:
SOLUTION Let the missing frequency be/, the assumed mean be A - 47 and h = 3.

ee
Calculation of Mean

Fr
mid-values /, Xj - 47.5 fdh
Class- di = Xi - 47.5 u.
for 3
Intervals
ur
-2 - 62
s

41.5 31 -6
40 - 43
ook
Yo

-3 - 1 -58
43 - 46 44.5 58
eB

0 0 0
46 - 49 47.5 60

49-52 50.5 / 3 1 /
our
ad

6 2 54
52-55 53.5 27

N-I/-=176 + / ZM=/'66
dY
Re

We have.
Fin

X = 47.2, A = 47.5 and h = 3

X ^A + h IN
^TfiUi

47.2 = 47.5 + 3 X \±1^\


\l76+/j
-0.3 = 3x
/-66
176 + /

-1 _ / - 66 ^ _176_/= 10/-660 ^ ll/ = 484 => / = 44


10 176 + /
Hence, the missing frequency is 44.
22.26
APPLIED MATHEMATICS-XI

EXAMPLE 7 The mean of the following frequenci/ table 50. But the frequencies f. and f.in class
20 - 40 and 60 - 80 are missing. Find the missing freq uenaes.

Class: 0-20 20 - 40 40 - 60 60- 80 80 - 100 Total


Frequency: /,17 32
h 19 120

SOLUTION Let the assumed mean be ^ = 50 and h = 20.


Calculation of Mean
Class Frequency Mid-values
U:I
X: - A
fdh
It
f, X

ow
0-20 17 10 -2 - 34
20-40
/. 30 - 1
-/
40-60 32 50

e
0 0

re
60 - 80
A 70 1
h
80 - 100 19

Frl 90 2

F
38

N=S/. =68 + /j+/2 - 4 - /| + /2


ou
or
We have. kfs
N = If = 120
[Given]
68 + + /2 = 120
oo

/i + A = 52
Y

Now,
B

Mean = 50
re

A + h
N I = 50
oYu

^ “ /i + A
ad

50 + 20 X = 50
120
d

50 + ijiA+A = 50
® =^‘^“A+A = o =>/i-/2=4 ...(ii)
in

6 6
Re

Solving equations (i) and (ii), we get A = 28 and A - 24.


F

EXAMPLE 8 Find the


maiks of students from the follozving cumulative frequency
mean

distribution:

Marks
Number ofstudents Marks
Numberof students
0 and above 80 60 and above 28
10 and above 77 70 and above 16
20 and above 72 80 and above 10
30 and above 65 90 and above 8
40 and above 55 100 and above. 0
50 and above 43
22.27
DATA INTERPRETATION-1 (CENTRAL TENDENCY)

SOLUTION Here we have, the cumulative frequency distribution. So, first we convert it into
an ordinary frequency distribution. We observe that there are 80 students getting marks
greater than or equal to 0 and 77 students have secured 10 and more marks. Therefore, the
number of students getting marks between 0 and 10 is 80 - 77 = 3.
Similarly, the number of students getting marks between 10 and 20 is 77 - 72 = 5 and so on.
Thus, we obtain the following frequency distribution.
Marks Number of Marks Number of
students students

50-60 15
0-10 3
60-70 12
10-20 5
70-80 6
20-30 7
80-90 2
30-40 10

w
90 -100 8
40-50 12

Now, we compute arithmetic mean by taking 55 as the assumed mean.

F lo
Computation of Mean

ee
X.I - 55
Marks Mid-value Frequency U;I fNi

Fr
10
(/,)
for
ur
3 -5 - 15
0 - 10 5
5 -4 - 20
10-20 15
s
7 -3 - 21
25
ook

20-30
Yo

10 -2 -20
30 - 40 35
12 - 1 - 12
eB

40-50 45
15 0 0
50-60 55
12 1 12
60 - 70 65
r

6 2 12
75
ad
ou

70 - 80
2 3 6
80 - 90 85
4 32
90 - 100 95
Y

Ifiq = -26
Re

Total If = 80
nd
Fi

We have.
N = Ifi = 80, Ifill, = -26, A = 55 and h = 10
X = A + hl^IfUj
N

-26
X = 55 + 10^ = 55 - 3.25 = 51.75 Marks.
80

EXAMPLE 9 Find the mean marks of the students from the following cumulative frequency
distribution:
Belozv Belozv Below Belozo Belozo Below Bclozv Below Below Below
Marks:
20 30 40 50 60 70 80 90 WO
W

9 17 29 45 60 70 78 83 85
Number of students: 5
22.28
APPLIED MATHEMATICS-XI

SOLUTION Here we have, cumulative frequency distribution less than type. First we

convert it into an ordinary frequency distribution. We observe that the number of students
getting marks less than 10 is 5 and 9 students have secured marks less than 20. Therefore,
number of students getting marks between 10 and 20 (inclusive 0 and exclusive 10) is 9 - 5 =
4. Similarly, the number of students getting marks between 20 and 30 is 17 - 9 = 8 and so on.
Thus, we have the following frequency distribution:
Marks: 0-10
10-20 20-30 30-40 40-50 50-60 60-70 70-80 80-90 90-100
Number of 5 4 12 16 15 10 8 5 2
students:

Let us now compute arithmetic mean by taking 55 and the assumed mean.
Computation of Mean
X:I - 55
Marks Mid-vnhie ll;
Frequency 10 fd‘i

w
0-10 5 5 -5 - 25
10-20 10

F lo
4 -4 - 16
20 - 30 25 8 -3 - 24
30-40 35 12 -2 - 24

ee
40-50 45 16 - 1 - 16

Fr
50 - 60 55 15 0 0
60-70 65 10
for 1 10
70-80 75 2 16
r
80-90 85 5 3 15
You
s

90 - 100 95 2
ook

Total
N = Zfi= 85
eB

= -56
We have.
^ - 85, Zfjiij = -56, h = 10 and A = 55
our
ad

X = A + h[~Zfiq
N
dY

-56
X = 55 -I-10 X
Re

= 55 - 6.59 = 48.41 Marks


85
Hence, mean marks scored by the students = 48.41.
Fin

- _ EXERCISE 22.4
The following table gives the distribution of total household expenditure (in rupees) of
manual workers in a city. ^
Expenditure Frequency Expenditure Frequency
(in rupees) (x^ (/;> (in rupees) (x) (.f)
100-150 24 300-350 30
150-200 40 350-400 22
200-250 33 400-450 16
250-300 28 450-500 7

Find the average expenditure (in rupees) per household.


22.29
DATA INTERPRETATION-I (CENTRAL TENDENCY)

^ A survey was conducted by a group of students as a part of their environment


awarenei programme, in which they coiiected the following data regarding the
number of plants in 20 houses in a locality. Find the mean number of plants per house.
Number os plants: 0-2 2-4 4-6
1
6-8
5
8-10
6
10-12
2
12-14
3
Number of houses: 1 2

Which method did you use for finding the mean, and why?
of 50 workers
3. Consider the following distribution of daily wages of a factory.
160 - 180 180 - 200
Daily wages (in?). 100 - 120 120 - 140 140 8- 160 6 10
Number of workers: 14 12

Find the mean daily wages of the workers of the factory by using
an appropriate method.
4. Thirty women were examined in a hospital by a doctor and number of per
minute recorded and summarised as foUows. Find the mean heart beats per mmute for
these women, choosing a suitable method.
65 - 68 68 - 71 71 - 74 74 - 77 77 - 80 80 - 83 83 - 86

w
Number of heat
beats per minute: 7 4 2
4 3 8

F lo
Number of women: 2
Find the mean of each of the following frequency distributions: (5 - 14)
5. Class interval: 0-6 6-12 12 - 18 18 - 24 24 - 30

ee
8 10 9 7
6
Frequency:

Fr
50-70 70 - 90 90 - 110 110 - 130 130 - 150 150 - 170
6. Class interval:
27 8 22
18 12 13 for
Frequency:
8-16 16-24 24-32 32-40
7. Class interval: 0-8
9
r
6 7 10 8
Frequency:
You

24-30
s

0-6 6-12 12-18 18-24


8. Class interval:
ook

5 10 12 6
7
Frequency:
eB

10-20 20-30 30-40 40-50


0. Class interval: 0-10
12 15 10 14
9
Frequency:
our
ad

0-8 8-16 16-24 24-32 32-40


10. Class interval:
9 10 8 8
5
Frequency:
32-40
dY

0-8 8-16 16-24 24-32


11. Class interval:
Re

6 4 3 2
5
Frequency:
Fin

10-30 30-50 50-70 70-90 90 - 110 110 - 130


12. Class interval:
12 20 3 2
5 8
Frequency:
35-45 45-55 55-65 65-75
13. Class interval: 25-35
10 8 12 4
6
Frequency:
40-44 45-49 50-54 55-59
25 - 29 30 - 34 35-39
14. Classes:
6 5 3 4
14 22 16
Frequency:
15. For the following distribution, calculate mean using all suitable
9-16
methods:
16-27
1-4 4-9
Size of item: 20
12 26
6
Frequency:
16. The weekly observations on cost of living index in a certain city for the year
2004 - 2005 are given below. Compute the weekly cost of living mdex.
22.30
APPLIED MATHEMATICS-XI

Cost of living Number of Cost of living Number of


Index Students Index Students
1400 - 1500 5 1700 - 1800 9
1500 - 1600 10 1800 - 1900 6
1600 - 1700 20 1900 - 2000 2

17. The following table shows the marks scored by 140 students in an examination of a
certain paper:
Marks: 0- 10 10- 20 20- 30 30- 40 40 - 50
Number of students: 20 24 40 36 20

ow
Calculate the
average marks by using all the three methods: direct method, assumed
mean deviation and shortcut method.

18. pe mean of the following frequency distribution is 62.8 and the sum of all the
frequencies is 50. Compute the missing frequency/j and /2.

e
Class: 0-20 20-40 40 - 60 60 - 80

re
Frequency: 5

Fl A 10 A
80 - 100
7
100 - 120
8

F
19. The following distribuHon shows the daily pocket allowance given to the children of a
^4«en?y^ tiuildmg. The average pocket allowance is ? 18.00. Find out the missing
ur
r
Class interval: 11-13
Frequency:
13-15 15-17 17-19 fo
19-21 21-23 23-25
ks
7 6 9 13 5 4
Yo
20. If the mean of the following distribution is 27, find the value of p.
oo

0- 10 10 - 20 20- 30 30-40 40- 50


Frequency: 8
eB

P 12 13 10
21. In a retail market, fruit vendors
boxes contained were selling mangoes kept in packing boxes. These
varying number of mangoes. The following was the distribution of
ur

mangoes according to the number of boxes.


ad

Number of mangoes: 50 - 52 53 - 55 56 - 58 59 - 61 62 -64


Yo

Number of boxes: 15 no 135 115 25


Find the mean number of
mangoes kept in a packing box. Which method of finding the
d

mean did you choose?


Re
in

22. The table below shows the daily expenditure on food of 25 households in a locality
Daily expenditure (in ?):
F

100 - 150 150 - 200 200 - 250 250 - 300 300 -350
Number of households: 4 5 12 2 2
Find the mean daily expenditure on food by a suitable method.
23. To find out the concentration of SO^ in the air (in parts per million, i.e., ppm), the data
was collected for 30 localities in a certain city and is presented below:
Concentration 0/SO2 Ppm) Frequency
0.00-0.04 4
0.04-0.08 9
0.08-0.12 9
0.12-0.16 2
0.16-0.20 4
0.20-0.24 2
Find the mean concentration of SO2 in the air.
22.31
DATA INTERPRETATION-! (CENTRAL TENDENCY)

24. A class teacher has the following absentee record of 40 students of a class for the whole
term. Find the mean number of days a student was absent,
(i) Number of days; 0-6 6- 12 12- 18 18 - 24 24- 30 30- 36 36 - 42
11 7 4 4 3 1
Numbe of students: 10

(ii) Number of days: 0-6 6-10 10 - 14 14 - 20 20 -28 28 - 38 38 - 40


10 7 4 4 3 1
Number of students: 11

25. The following table gives the literacy rate (in percentage) of 35 cities. Find the mean
literacy rate.
45 - 55 55 - 65 65 - 75 75 - 85 85 -95
Literacy rate (in %):
3 10 11 8 3
Number of cities:

26. The following is the cummulative frequency distribution (of less than type) of 1000

w
persons each of age 20 years and above. Determne the 60
mean age.70 80
40 50
Age below (in years): 30
100 220 350 750 950 1000
Number of persons:

Flo
27 If the mean of the following frequency distribution is 18, find the missing frequency.
Class interval: 11-13 13-15 15-17 17-19 19-21 21-23 23-25

e
5 4
3 6 9 13 /

re
Frequency:
28. Find the missing frequencies in the following distribution, if the sum of the frequencies

F
is 120 and the mean is 50.
0-20 20-40 40-60 60-80 80-100
Class:
ur
r
fo 19
Frequency: 17 h 32

29 The daily income of a sample of 50 employees are tabulated as follows:


ks
1-200 201-400 401-600 601-800
"C
Income (in ?):
Yo

14 7
oo

14 15
No. of employees:
Find the mean daily income of employees.
eB

30. The marks obtained by 110 students in an examination are given below:
Marks: 30-35 35-40 40-45 45-50 50-55 55-60 60-65
ur

16 28 23 18 8 3
Frequency: 14
ad

Find the mean marks of the students.


Yo

ANSW, iRS
3. 145.20 4. 75.9
1. 266.25 2. 8.1 plants
d
Re

6. 112.20 7. 21.4 8. 15.75


5. 8.15
in

10. 21 11. 16.4 12. 65.6


9. 26.333
17. 25.857
F

13. 49.5 14. 36.357 16. 1663.3


19. 20 20. p = 7 21. 57.19
18. /i = B,/2=12.
22. ^ 211 23. 0.099 ppm 24. (i) 14.1 days (ii) 12.475 days 25. 69.43%
26. 51.3 years 27. 8 28. /i=28,/2=24 29. ? 356.5 30.44.81

22.5 MEDIAN

DEFINITION Median of a distribution is the value of the variable zvhich divides the distribution into
tzoo equal parts i.e. it is the value of the variable such that the number of observations above it is equal
to the number of observations below it.
22.5.1 MEDIAN OF AN UNGROUPED DATA (OR INDIVIDUAL OBSERVATIONS)
If the values .v,- in the raw data are arranged in
middle, most value in the arrangement is called the median.
order of increasing or decreasing magnitude, then the
Thus, for the ungrouped data -Vj, X2,.. ●/ x,„ the median is computed by using the following
algorithm.
22.32
APPLIED MATHEMATICS-XI

ALGORITHM

'Arrange the observations (values of the variate) in ascenduw or ciescendiii<? order of


magnitude. a j
bll.i' Determine the total number of observations, say, n
STLTHl If n is odd, then
(n + 1 Y"
Median = Value of — observation
^ \ 2
If n is even, then
.11, \th
n ( W
Median Value of — observation + Value of +1 observation

ow
\2 ) [2
2

Following examples illustrate the above algorithm.


ILLUSTRATIVE EXAMPLES

e
re
EXA.MPLE 1

Fl
Find the median of the follozving data:

F
25, 34, 31, 23, 22, 26, 35, 28, 20, 32
ur
SOLUTION Arranging the data in ascending order, we get

r
20, 22, 23, 25, 26, 28, 31, 32, 34, 35 fo
Here, the number of observations n = 10 (even).
ks
Yo
-.th th
10 rio
oo

Value of observation + value of + 1 observation


Median = 2; 2
eB

Median = Value of 5*^'observation + Value of 6**^ observation 26 28


= 27
ur

2 2
ad

Hence, median of the given data is 27.


Yo

1.XAMPLE2
Find the median of the following values:
d

37, 31, 42, 43, 46, 25, 39, 45, 32


Re
in

SOLUTION Arranging the data in ascending order, we have


F

25, 31, 32, 37, 39, 42, 43, 45, 46


Here, the number of observations n=9 (odd)
Nth
9 + 1
Median = Value of
2 y observation = Value of 5th observation = 39.
EXAMPLE 3 The median of the observations 11,12,14,18, x+ 2, x+ 4,30,32, 35,41 arranged in
i
ascending order is 24. Find the value ofx.
SOLUTION Here, the number of observations n = 10. Since n is even, therefore
w \ th
/ n th
n
observation + - +1 observation
Median = — ^2
2

24 =
5* observation + 6‘'^observation
2
22.33
DATA INTERPRETATION-l (CENTRAL TENDENCY)

{X + 2) + (,Y + 4) 2.V + 6
- => 24 = -V + 3 X = 21.
=> 24 = =>24 =
2 2

Hence, a; = 21 ●
LX.AMi’l I 4 Find the median of the following datn:19, 25, 59, 48, 35, 31, 30, 32, 51. If 25 is
placed by 52, zolint will be the neiu median?
re

SOLUTION Arranging the given data in ascending order, we have


19, 25, 30,31,32,35, 48, 51,59
Here, the number of observations n=9 (odd)
Since the number of observations is odd. Therefore,
(9 + \ the observation
Median = Value of
2

low
Median = Value of 5“" observation = 32.
Hence, Median = 32
If 25 is replaced by 52, then the new observations arranged in ascending order are:
19, 30, 31, 32, 35, 48, 51, 52, 59
New median = Value of 5 observation = 35.

ee
F
Fr
EXERCISE 22.i-

Find the median of the following data (1-8)


for
ur
-■ 83, 37, 70, 29, 45, 63, 41, 70, 34, 54
133, 73, 89, 108, 94, 104, 94, 85, 100, 120
s
3. 31, 38, 27, 28, 36, 25, 35, 40
k
Yo
oo

4. 15, 6, 16, 8, 22, 21, 9, 18, 25


5. 41, 43, 127, 99, 71, 92, 71, 58, 57
eB

6. 25, 34, 31, 23, 22, 26, 35, 29, 20, 32


7. 12, 17, 3, 14, 5, 8, 7, 15
r
ou
ad

8 92, 35, 67, 85, 72, 81, 56, 51, 42, 69


9. Numbers 50, 42, 35, 2.y + 10, 2v - 8, 12, 11, 8 are written in descending order and then
Y

median is 25, find x.


Find the median of the following observations : 46, 64, 87, 41, 58, 77, 35, 90, 55, 92, 33. If
nd
Re

10.
92 is replaced by 99 and 41 by 43 in the above data, find the new median.
Find the median of the following data : 41, 43,127, 99, 61, 92, 71, 58, 57. If 58 is replaced
Fi

11.
by 85, what will be the new median?
, 2 The weiehts (in kg) of 15 students are : 31, 35, 27, 29, 32, 43, 37, 41 34, 28, 36 44 45 42,
30. Find^the median. If the weight 44 kg is replaced by 46 kg and 27 kg by 25 kg, fmd e
new median.

13 . The following observations have been arranged in ascending order. If the median of the
data is 63, find the value of .r;
29, 32, 48, 50, .y, .y + 2, 72, 78, 84, 95
ANSWERS

3. 33 4. 16 :>. 61
1. 49.5 2. 97
8. 68 9. 12 40. 58.58
6. 27.5 7. 10

n. 61.71 12. 35 kg, 35 kg. 13. 62


22.34
APPLIED MATHEMATICS - XI

22.5.2 MEDIAN OF DISCRETE FREQUENCY DISTRIBUTION


In case of a discrete frequency distribution ,f,. / f,fi = „ we calculate the median by
using the following algorithm.
ALGORITHM
STEP 1
Find the cumulative frequencies (cf.)
N "
STEP II Find —/ where ^
^ /=i

ill the cumulative frequenci/ (c.f.) just greater than


STEP
y and determine the corresponding
value of the variable.
STEP IV The value obtained in step III is the median.
ILLUSTRATION Obtain the median for thefollowing frequency distribution:
I 2 3 4 5 6 7 8 9

w
10 11 16 20 25 15 9 6
SOLUTION

-Y

F lo
Calculation of Median
/ Cf

e
Fre
1
8
2 10 18
3 11
for
29
4
16
r
45
5
You

20 65
oks

6 25 90
eBo

7 15 105
8 9 114
9
120
our
ad

M=120

N
dY

Here, N = 120 => — = 60


Re

2
Fin

We find that the cumulative frequency just greater than — i-G-,


i 60 is 65 and the value of x
corresponding to 65 is 5. Therefore, Median = 5. ^
22.5.3 MEDIAN OF A GROUPED OR CONTINUOUS FREQUENCY DISTRIBUTION
In order to calculate the median of a grouped or continuous frequency distribution , we use
me following algorithm.
ALGORITHM

STEP I
Obtain the frequency distribution.
STEP II
Pf'<-’P‘^re the cumulativefrequency column and obtain N = Ifj.
STEP III Find N/2.
SIEPJV See the cumulative frequency just greater than N/2 and determine the corresponding class.
This class is known as the median class.
22.35
DATA INTERPRETATION-l (CENTRAL TENDENCY)

STEP V L/sf the following formula:


N
- F
Median = / + ' ~ 'xh
f
where, I = lower limit of the median class
/ = frequency of the median class
h = with (size) of the median class
F = Cumulative frequency of the class preceding the median class N = 1/
ILLUSTRATION 3 Calculate the median from the following distribution:
Class: 5-20 30- 25 15-20 20-25 25-30 30-35 35-40 40-45
15 2 0 5 4 2 2
6
Frequency: 5
SOLUTION First we prepare the following cumulative table to compute the median

w
Class Frequency Cumulative freqiiencij

F lo
5 5
5-10
6 11
10-15
15 26
15-20

e
Fre
10 36
20-25
5 41
25-30
for
4 45
30-35
47
r
35-40 2
You

49
oks

40-45 2
eBo

N=49

We have, M = 49
ad
our

N 49
— = 24.5
2 2

The cumulative frequency just greater than A//2 is 26 and the corresponding class is
Re
dY

15 - 20. Thus, 15 - 20 is the median class such that


Fin

/ = 15,/ = 15,F = lland/7 = 5


N
-f 24.5-11 13.5
xh = 15 + X 5 = 15 + = 19.5
Median = / + — 3
15
/

ILLUSTRATIVE EXAMPLES

EXAMPLE 1 The number of students absent in a school zoas recorded every day for 147 days and the
raw data was presented in the form of the foUoivingfrcqueiicy table.
9 20 11 12 23 15 18 20
No. of students absent: 5 6 7
1
No. of days: 1 5 12 14 16 23 10 70 4 2 2
Obtain th.e median and describe what information it conveys.
22.36
APPLIED MATHEMATICS - XI

SOLUTION

Ccilailcition of median
A'.-I
fi cf
5 1 1
6 5 6
7 11 17
14 31
9 16 47
10 13 60
11 10

ow
70
12 70 140
13 4 144
15 1 145
18 1

e
146

re
20

rFl 1 147

F
N = = 147

We have.

r
ou
^
N = 147 => — =
2
147
2
= 73.5 fo
ks
The cumulative frequency just greater than N/2 is 140 and the corresponding value of
variable a is 12.
oo

Hence, median = 12. This means that for about half the number of days, more than 12
Y
eB

students were absent.

liXAMlM I : Calculate the median from the following data:


Marks:
ur

0-10 W-30 30- 60 60 - 80 80-90


No. of students: 5 15 30
ad

8 2
Yo

SOLUTION Here, the class intervals are of unequal width. If the class intervals are of
unequal width the frequencies need not be adjusted to make the class intervals equal.
d

Calculation of Median
Re
in

Marks
No. of students Cumulative frequency
F

(Frequenci/)
0-10 5 5
10-30 15 20
30-60 30 50
60-80 8 58
80-90 2 60

N = Ifi= 60
Here, N = 60 N/2 = 30

S'*"
30 - 60. Hence, 30 - 60 is the median class.
W2 = 30 is 50 and the corresponding class i IS

/=30,/= 30,F = 20,// = 30


22.37
DATA INTERPRETATION-1 (CENTRAL TENDENCY)

N
-F
2
Now, Median = / + — X h
/
30 - 20
Median = 30 + X 30 = 40
30

EXAMri.t 3 If the median of the folloiving frequency distribution is 46, find the missing
frequencies.
Variable: 10-20 20-30 30 - 40 40 - 50 50 - 60 60 - 70 70 - 80 Total
Frequency: 22 ; 30 ? 65 ? 25 18 229
frequency of the class 30 - 40 beand that of the class 50 - 60 be/2. The

ow
SOLUTION Let the
total frequency is 229.
12 + 30 + /, + 65 + A + 25 +18 = 229 => /, + A = 79
It is given that the median is 46.

e
Fl
Clearly, 46 lies in the class 40 - 50. So, 40 - 50 is the median class.

re
l = 40,;. = 10,/ = 65andF = 12 + 30 + /, = 42 + /i,N = 229

F
Now,
ur
N
or
sf
-F
Median = / + .2. x/i
k
/
Yo
oo

229
B

-(42-h/i)
46 = 40 + — xlO
re

65
u
ad

145-2/
Yo

46 = 40 +
13
d

2/ = 67 => /i = 33.5 or 34 (say)


Re
in

13
F

Since / + / = 79 . Therefore, A = 45.


Hence, / = 34 and/j = 45.
EXAMPLE 4 Find the median of the following frequency distribution:
60-69 70-79 80-89 90-99 100-109 110-119
Weekly wages {in
15 20 30 20
5
No. of days:
SOLUTION Here, the frequency table is given in inclusive form. So, we first transform it into
exclusive form by subtracHng and adding h/2 to the lower and upper limits respectively of
each class, where h denotes the difference of lower limit of a class and the upper limit of the
previous class.
Transforming the above table into exclusive form and preparing the cumulative frequency
table, we get
22.38
APPLIED MATHEMATICS-XI

Weekly wages (in No. of workers Cuniulnfive frequency


59.5- 69.5 5 5
69.5 - 79.5 15 20
79.5- 89.5 20 40
89.5- 99.5 30 70
99.5 - 109.5 20 90
109.5 - 119.5
98

N = If = 98

ow
We have, N = 98. .●. N/2 = 49
The cumulative frequency just greater than N/2 i 70 and the corresponding class is
IS

89.5 - 99.5. So, 89.5 - 99.5 is the median class.


/ - 89.5, h = 10, / = 30 and f = 40

e
re
Now, Median = I + -2.
N

Fl -/

F
xh
f
ur
49-40

r
Median = 89.5 + X 10 = 92.5
30 fo
ks
EXAMPLE 5
Compute the median form the folloiving data:
Yo
Mid-value: 115 125 135 145 155 165 175
oo

185 195
Frequency: 6 25 48 72 116 60 38 22 3
eB

SOLUTION Here, we are given the mid-values. So, should first find the upper and
lower limits of the various classes. The difference between two consecutive values is
h= 125-115= 10.
ur

Lower limit of a class = Mid-value - h/2, Upper limit


ad

= Mid-value + h/ 2.
Yo

Calculation of Median
d

Mid-value Class groups Frequency Cumulative


Re
in

Frequency
115 110 - 120 6
F

6
125 120 -130 25 31
135 130 -140 48 79
145 140 - 150 72 151
155 150 - 160 116 267
165 160 - 170 60 327
175 170 -180 38 365
185 180 - 190 22 387
195 190 - 200 3 390

N = If = 390
Wo have.

N = 390 .-. N
— _ 390 = 195
2 " 2
22.39
DATA INTERPRETATION-! (CENTRAL TENDENCY)

The cumulative frequency just greater than N/2 i.c., 195 is 267 and the corresponding class
is 150 -160. So, 150 -160 is the median class.
/ = 150,/ = 116,/i = 10,f = 151
Now,
N
- F
Median = / + — xh
f
195-151
Median = 150 + xlO = 153.80
116

Compute the median for the folloiving cumulative frequencij distribution:

ow
EXAMPLE 6

Less Less Less Less Less Less Less Less Less

than 20 than 30 than 40 than 50 than 60 than 70 than 80 than 90 than 100
0 4 16 30 46 66 82 92 200

SOLUTION We are given the cumulative frequency distribution. So, we first construct a

e
frequency table from the given cumulative frequency distribution and then we will make

re
necessary computations to compute median.

rFl
F
Class intervals Frequency (f) Cumulative frequencij (cf.)
20-30 4 4

or
ou
30-40 12
ksf 16

40-50 14 30

50-60 16 46
oo

60-70 20 66
Y
B

70-80 16 82

10 92
re

80-90
100
90 - 100
oYu
ad

N = 'Lf= 100
d

N
Here, N = If = 100 Y = 50
in
Re

N
We observe that tlae cumulative frequency just greater than ~ = 50 is 66 and the
F

corresponding class is 60 - 70.


So, 60 - 70 is the median class.
I =60, f = 20, f = 46 and h = 10
N
-F
Now, Median = I + — xh
f
50-46
Median = 60 + X 10 = 62
20

EXAMPLE 7 The median of the following data is 525. Find the values of x and y, if the total
frequency is 100
22.40 APPLIED MATHEMATICS-XI

Class interval
Frequency
0-100 2
100 - 200 5
200 - 300 X

300 - 400 12
400 - 500 17
500 - 600 20
600 - 700 y
700 - 800 9
800 - 900 7
900 - 1000 4
SOLUTION

Computation of Median
Class intervals Frequency (f) Cumulative frequency (cf)

w
0-100 2 2

F lo
100 - 200 5 7
200 - 300 X 7+x
300 - 400 12 19+x

e
Fre
400 - 500 17 36+ x
500 - 600 20 for 56+ x
600 - 700
y 56 +x + y
700 - 800 9
65+ x + y
r
800 - 900
You

7
72 +x + y
oks

900 - 1000 4
76 +x + y
eBo

Total = 100

We have.
our
ad

N = I. fi = 100
76 + x + y = 100 => X + y = 24
dY
Re

It is given that the median is 525. Clearly, it lies in the class 500 - 600
Fin

/ = 500,/i = 100,/ = 20,f = 36 + xandN = 100


Now,
N
--F
Median = / + xh
f

525 = 500 + 50 - (36 + x)


xlOO
20

525 - 500 = (14 - x) X 5


25 = 70 - 5a: => 5a: = 45 => a: = 9

Putting X = 9 in a: + y = 24, we get y = 15.


Hence, x = 9andy = 15.
22.41
DATA INTERPRETATION-1 (CENTRAL TENDENCY)

rxAMiM i: 8 If the ntedian of the distribution given below is 28.5, ifnd the value of x and \j.
Class interval: 0-10 10-20 20-30 30 -40 40- 50 50 - 60
20 15 y 5
No. of students: 5 X
Total 60

SOLUTION
Computation of Median
Class intenmls Frequency (f Cumulative frequency (cf
5 5
0-10
5 + .Y
10 - 20 Y

20 25+.Y
20-30

ow
15 40 + .Y
30-40
40-50 y 40 + Y + 1/
50-60 5 45 + Y + 1/

I = 60

e
re
We have.
Median = 28.5

rFl
F
Clealry, it lies in the class interval 20 - 30. So, 20 - 30 is the median class.
/ = 20, h = 10, / = 20, F = 5 +X and N = 60

r
N
ou
Median = / + -^
- F
x/j
fo
ks
Now,
/
30 - (5 + .c)-^x
oo

28.5 = 20 + 10
20
Y
eB

25-X
28.5 = 20 +
2
25 - Y
r

8,5 = => 25 - Y = 17 => Y = 8


ou

2
ad
Y

We have.
N = 60
d

45 + Y + )/ = 60 => Y -(● y = 15
Re
in

Putting ,r = 8 in Y + y = 15, wc' get y = 7


F

Hence, y = 8 and y = 7.
EXERCISE 22.6
. Find
1. Following are the lives ir. hours of 15 pieces of the components of aircraft engine
the median:
715, 724, 725, 710, 729, 745, 694, 699, 696, 712, 734, 728, 716, 705, 719.
The following is the distribution of height of students of a certain class in a certain city:
Height (in cms): 160 - 162 163 - 165 166 - 168 169 - 171 172 -174 18
15 118 142 127
No. of students:
Find the median height.
Following is the distribution of I.Q. of 100 students. Find the median I.Q.
I.Q.: 55-64 65-74 75-84 85-94 95-104 105-114 115-124 125-134 135-144
No of
22 33 22 8 2 1
Students: 1 2 9
22.42
APPLIED MATHEMATICS-XI

4. Calculate the median salary of the following data giving salaries of 280 persons:
Salary (in thousands): 5-10 10-15 15-20 20-25 25-30 30-35 35-40 40-45 45-50
No. of persons: 49 133 63 15 6 7 4 2 1

5. Calculate the median from the following data:


Marks below: 10 20 30 40 50 60 70 80
No. of students: 15 35 60 84 96 127 198 250

6. Calculate the missing frequency from the following distribution, it being given that the
median of the distribution is 24.
Age in years: 0- 10 10- 20 20 - 30 30 - 40 40 - 50
No. of persons: 5 25 ■> 18 7

ow
7. The following table gives the frequency distribution of married women by age at
marriage: ^ °

Age (hi years) Frequency Age (in years) Frequency


15-19 53 40-44

e
9

re
20-24 140 45-49 5
25-29

rFl 98 50-54 3

F
30-34 32 55-59 3
35-39 12 60 and above 2

or
ou
Calculate the median and interpret the results.
ksf
8. The following table gives the distribution of the life time of 400 neon lamps:
Lite time: (in hours) Number of lamps
oo

1500-2000 14
Y

2000-2500 56
B

2500-3000 60
3000-3500
re

86
3500-4000 74
oYu

4000-4500 62
ad

4500-5000 48
Find the median life.
d

9. The distribution below gives the weight of 30 students in a class. Find the median
in
Re

weight of students:
Weight (in kg): 40-45 45-50 50-55 55-60
F

60-65 65-70 70-75


No. of students: 2 3 6 6 3 2

10. Find the missing frequencies and the median for the following distribution if the mean
is 1.46.
No. of accidents: 0 1 2 3 4 5 Total
Frequency (No. of days): 46 7 7
25 10 5 200

11. An incomplete distribution is given below:


Variable: 10-20 20-30 30-40 40-50 50-60 60-70 70-80
Frequency: 12 30 65 25 18

You are given that the median value is 46 and the total number of items is 230.
(i) Using the median formula fill up missing frequencies,
(ii) Calculate the AM of the completed distribution.
22.43
DATA INTERPRETATION-1 (CENTRAL TENDENCY)

12. If the median of the following frequency distribution is 28.5 find the missing
frequencies:
Class interval: 0-10 10-20 20-30 30-40 40-50 50-60 Total
15 A 5 60
Frequency: 5 /i 20

13. The median of the following data is 525. Find the missing frequency, if it is given that
there are 100 observations in the data:

Class interval Frequency Class interval Frequency

2 500 - 600 20
0 - 100
100 - 200 5 600 - 700 A

w
700 - 800 9
200 - 300 /i
12 800 - 900 7
300 - 400
900 - 1000 4

o
400 - 500 17

e
14. If the median of the following data is 32.5, find the missing frequencies.

re
rFl
Class interval: 0-10 10-20 20-30 30-40 40-50 50-60 60-70 Total

F
9 12 3 2 40
Frequency: /i 5

15. Compute the median for each of the following data:

r
ou
(i) Marks No. ofstudents (ii) Marks
fo No. of students
0
ks
Less than 10 0 More than 150

10 More than 140 12


Less than 30
oo

25 More than 130 27


Less than 50
Y
eB

43 More than 120 60


Less than 70
65 More than 110 105
Less than 90
r

87 More than 100 124


Less than 110
You
ad

96 More than 90 141


Less than 130
100 More than 80 150
Less than 150
d

of a school was conducted


16. A survey regarding the height (in cm) of 51 girls of class X
Re
in

and the following data was obtained:


F

Height in cm Number of Girls


Less than 140 4

Less than 145 11

Less than 150 29

Less than 155 40

Less than 160 46

Less than 165 51

Find the median height.


17. A life insurance agent found the following data for distribution of ages of 10() policy
holders. Calculate the median age, if policies are only given to persons havmg age
18 years onwards but less than 60 years.
22.44
APPLIED MATHEMATICS - XI

Age in years Number of policy holders


Below 20 2
Below 25 6
Below 30 24
Below 35 45
Below 40 78
Below 45 89
Below 50 92
Below 55 98
Below 60 100
1^. The lengths of 40 leaves of a plant are measured correct to the nearest millimetre, and the
data obtained is represented in the following table:
Length (in mm): 118-126 127-135 136-144 145-153 154-162 163-171 172-180
No. of leaves: 3 5 9 12 5 4 2

w
Find the mean length of leaf.
An incomplete distribution is given as follows:

F lo
Variable: 0-10 10-20 20-30 30-40 40-50 50-60 60-70
Frequency: 10 20
7 40 7
25 15
You are given that the median value is 35 and the sum of all the frequencies is 170. Using

ee
the median formula, fill up the missing frequeiicies.

Fr
" The median of the distribution given below is 14.4. Find the values of t and i/, if the total
for
frequency is 20.
Class interval: 0-6
r
6-12 12-18 18-24 24-30
Frequency:
You
4 .V 5 1
y
s

The median of the following data is 50. Find the values of p and a, if the sum of all the
ook

frequencies is 90.
eB

Marks: 20-30 30-40 40-50 50-60 60-70 70-80 80-90


Frequency: p 15 25 20 8 10
our
ad

ANSWERS
.. 716 167.13 ems 99.35 .. ?13421 5. 59.35 . 25
Median = 24.5 years.
Nearly half the women were married between the ages 15 and 24.5 years.
dY
Re

Median life = 3406.98 hours ●- 56.67 kg


Fin

111- Missing Frequencies 76 and 38, Median = 1.


11. Missing frequencies 34 and 46, Mean = 45.87.
12. =8,A=7 (1
=9,/, =15 14. /j=3,/,=6
(i) 76.36 (ii) 116.67 11’ 149.03 cm
17. 35.76 years
18. 146.75 mm i‘'- Class 20-30 40-50
Frequency 35 25
- ' . .r = 4, y = 6 :■ p = 5,q = 7

22 5 U' VT,
Besides median, there are other measures which divide a distribution into a number of equal
parts. These measures are quartiles, quantiles, deciles, percentiles etc. Quartiles divide the
distribution into 4 equal parts, quantiles into 5 equal parts, septiles into 7 equal parts, octiles
into 8 equal parts, deciles into 10 equal parts and percentiles into 100 equal parts. Important
amongst these are quartiles, deciles and percentiles.
22.45
DATA INTERPRETATION-I (CENTRAL TENDENCY)

QUARTILES Qiiartiles are those lvalues of the variate zvhich divide th.e distribution into four equal pm ts.
As they divide the distribution into four equal parts, therefore number of quartiles is three.
They are known as lower quartile, middle quartile and upper quartile. These are also called
first, second and third quartiles and are denoted by , Qo and Q3 respectively.
The lower quartile is positioned in such a manner that 25% observations are below it and
75% observations are above it. Therefore, the lower quartile Q, is the central positional
value of the lower half of the distribution. The middle or second quarhle Q2is the central
positional value of the median and the upper quartile Q3 is the central positiorial value of
the upper half of the distribution, 25% observations are above the upper quartile and /b/o
observations are below it.

ow
NOTE It must be noted that Q, < Q2 < Q3 median falls between Q, and Q3.
22.6.1 COMPUTATION OF QUARTILES FOR INDIVIDUAL OBSERVATIONS
Let a'i,.Y2,.V3,... be n values of a variate X. To compute quartiles of these values we use the
following algorithm.

e
re
ALGORITHM

rFl
Arrange the observations (data) in ascending order of mag)iifude.

F
Determine the total number of observations, sa\j n.
ili Use the folloxvingformulas to compute Q,, Q2 and Q,.
th

or
f n +1
ou
Qi = of observation.
ksf
Value of' ^ ^ observation, if n is odd
oo

Q2 = th th
f n
Y

n
AM. of values of — and — +1 observation, if n is even
B

0
re

-.th

= Value of 3 ^ - observation.
oYu
ad

Following illustration will illustrate the above algorithm.


ILIA S I U .\TiON From the following information of wages of 30 workers in a factory compute the
d

median, lower and upper quartiles.


in
Re

S.No. Wages S.No.

330 16 240
F

1
320 17 330
2
550 18 420
3
470 19 380
4
210 20 450
5
500 21 260
6
270 22 330
7
120 23 440
8
680 24 480
9
490 25 520
10
400 26 300
11
170 27 580
12
440 28 370
13
480 29 380
14
620 30 350
15
22.46
APPLIED MATHEMATICS-XI

SOLUTION Arranging the wages in ascending order, we obtain the following table:
S.No Wages (f) S.No Wages (0
1 120 16 400
2 170 17 420
3 210 18 440
4 240 19 440
5 260 20 450
6 270 21 470
7 300 22 480
8 320 23 480
9 330 24 490
10 330 25 500

w
11 330 26 520
12

13

14

15

We haven = 30.
330

350

370

380
F lo 27

28

29

30
for F
ree
550

580

620

680

Lower Quartile:
Your

\th \th

Q, = Value offV — 30 + 1
ks

observation = Value of observation


eBoo

4 , 4

Qi - Value of (7.75)^ observation


ad
our

Qi = Value of 7 th observation
3
+- (Value of 8 th observation - Value o 7 th observation)
Re
Y
Find

Q, = 300 +1( 320 - 300) = 315


Middle quartile (Median):
Nth
n
Q2 = A.M.of and — + 1 observation
u

Value of 15 observation + Value of 16^ observation 380 + 400


Qi = = 390
2 2
Upper quartile:
\th Nth
n + 1 30 + 1
Q3 = Value of 3 observation = Value of 3 observation
4 J N 4 ,
22.47
DATA INTERPRETATION-! (CENTRAL TENDENCY)

Q3 = Value of 23.25‘^ observation


1
Q3 = Value of 23 rd observation-)- - (Value of 24 th observation
- Value of 23 rd observation)
1
Q3 = 480 + -^{490 - 480) = 482.50
Hence, ? 315, Q2 - ? 390 and Q3 = ^ 482.50.
22.6.2 COMPUTATION OF QUARTILES FOR A DISCRETE FREQUENCY DISTRIBUTION

ow
Foradiscrete frequency distribution Xj/f,;i = 1,2,...,n, we calculate quartiles by using the
following algorithm.
ALGORITHM

Obtain the discrete frequency distribution, sni/, x-J f,-)i = 1,2,..., n,

e
STEPl

Fl
re
STEP II Compute the cumulative frequencies.

F
ri

Obtain N = ^
ur
STEP III

STEI’ IV
/=!

of the variable. This value is the lozver quartile Qj.


f or
See the cumulative frequency just greater than N/4 and determine the corresponding value
ks
Yo
See the commutative frequena/ just greater than N/2 and determine the corresponding
oo

value of the variable. This value is the middle quartile Q2 i.e the median.
B

See the cumulative frequency just greater than and determine the correspwnding value of
the variable, which is equal to the upper quartile Q3 .
re

Following illustration illustrates the above algorithm.


u
ad

ILLUSTRATION From the following data compute the values of various quartiles:
Yo

30 40 50 60 70 80
Age in years: 20

132 153 140 51 3


No. of members:
3 61
d
Re

SOLUTION The cumulative frequency table is as given below:


in

Age in years No. of Students Cumulative Frequencies


F

20 3 3

30 61 64

40 132 196

50 153 349

60 140 489

70 51 540

80 3 543

N= 543

N 543
= 135.75
Lower Quartile: We have, “ —
22.48
APPLIED MATHEMATICS - XI

Cumulative frequency just greater than N/4 is 196 and the corresponding value of the
variable is 40.

Qj = Lower quartile = 40 years.

Middle Quartile (Median): We have, — = = 271.5


2 2

Cumulative frequency just greater than is 349 and the corresponding value of the
variables 50.

Q2 = Middle quartile = 50 years.


3N 3 X 543
Upper Quartile: We have, — = 407.25
4

Cumulative frequency just greater than 407.25 is 489 and the corresponding value of the

w
variable is 60.

So, Q3 = Upper quartile = 60 years.


22.6.3

F lo
COMPUTATION OF QUARTILES FOR A FREQUENCY DISTRIBUTION WITH CLASS
INTERVALS

e
Fre
The following algorithm is used to compute quartiles for continuous series.
ALGORITHM
for
jj_; Obtain the given frequency distribution
r
: 1! Compute cumulative frequencies and denote S f by N.
You
oks

iN
eBo

. I; Compute —, where / = 1 for lower quartile, i = 2 for middle quartile and j = 3 for
upper quartile.
ad
our

iN
See the cumulative frequency just greater than The corresponding class is the quartile
class.
Re
dY

iN
Fin

1 + -F
4
STEP Use the formula / = X h, where
/

/ = lower limit for the quartile class obtained in step IV


F = cumulative frequency of the class preceding the quartile class in step IV
f = frequency of the quartile class,
h = zvidth of the quartile class.
Following illustration will illustrate the above algorithm.
ILLUSTRATION Compute loioer, middle and upper quartiles for the folloiuing distribution:
Marks group: 5-10 10-15 15-20 20-25 25-30 30-35 35-40 40-45

No of Studezzt: 5 6 15 10 5 4 2 1
22.49
DATA INTERPRETATION-1 (CENTRAL TENDENCY)

SOLUTION

Marks groups (class) hJo. of students (Frequency) Cumulative frequency


5 5
5-10
6 11
10- 15
15 26
15-20
10 36
20-25
5 41
25-30
4 45
30-35
2 47
35-40
1 48
40-45
N = 48

w
N
Lower Quartile: We have, N = 48 => — = 12.
4
N

F lo
is 26. So, 15 - 20 is the lower
We find that the cumulative frequency just greater than ^
quartile class such that / = 15, / = 15, /i = 5, F = 11.

ee
Fr
N
-F
12-11
xh =15 + X 5 = 15 H— = 15.333.
Q, = ' + - 15 3 for
/
ur
2N
Middle quartile (Median): Wc have, N = 48 4
= 24.
s
ok
Yo

2N

The cumulative frequency just greater than ^ is 26. The corresponding class is 15 - 20. It is
o
eB

the median class such that I ■= 15, f = 15, h - 5,F = 11.

24-n
r

Q, =; + i X // = 15 + X 5 = 15 + 65 = 21.5
ad
ou

10
/
3N
Y

= 36.
Upper Quartile: We have, N = 48 ^ ^
Re
nd

3N
- F
36-36
Fi

X /i = 25 + X 5 = 25.
Q,=i + — 5
/
Hence, = 15.33, Q2 = 21.5 andQ^ = 25.

As we have learnt the three quartiles viz. Qy Q. and Q3 divide frequency distribution into
four equal parts. Let us consider the distribution of scores of a group of individuals in a test.
75% scores are less than or equal to the upper quartile Q3 and top 25% scores arc greater than
Q, So, the scores in the top 25% are assigned the quartile rank 1 i.e. scores greater than are
assigned the Quartile rank 1. The scores between Q-, and Q3 are assigned the ranking of
upper middle quartile i.e. Quartile rank 2. The quartile rank 3 is assigned ot scores ben^'een
Qi and Qj. Bottom 25% scores i.e. scores less than Qj are assigned the quartile rank 4.
QuartUe rank 4 Quartile rank 3-*^-*-Quartile rank 2-*-^ Quartile rank 1 —
Qi Q2 Q3

Fig. 22.1
22.50
APPLIED MATHEMATICS -XI

Quartile rankings are a measure of how well a mutual fund has performed against ail other
funds in its category. The rankings range from 1 to 4. Mutual funds with the highest percent
returns in the chosen time period are assigned to top quartile Qg whereas those with the
lowest returns are assigned to bottom quartile Qj. Quartile rankings are complied by sorting
the funds based on trailing returns over a period chosen by the user. Funds in the top 25%
are assigned the ranking of top quartile Qg, the next 25% are assigned a ranking of upper
middle quartile, the next 25% after that are assigned a ranking of lower middle quartile and
the lowest 25% are assigned the ranking of bottom quartile.
22.7 DECILES

ow
DECILES The nine points which divide the given distribution into ten equal parts are called deciles.
Deciles are denoted by D^, D2,-.., Dy. The first decile, D|, is the value of the variable such
that it exceeds 10% of the observations and is exceeded by 90% of the observations. Similarly,
Dy, the seventh decide, has 70% observations before it and 30% observations after it. The fifth

e
decide, D5, coincides with the median.

re
The methods for computing deciles are tlie same as those of computing quartiles in case of all

Frl
F
types of distributions.
22.7.1 COMPUTATION OF DECILES FOR INDIVIDUAL OBSERVATIONS
ou
r
Let a: I, a.'2, ATg,..., A.',, be n values of a variable X. To compute deciles of these values we use the
following algorithm. so
kf
ALGORITHM
oo

STEP 1 Arrange the observations in ascending order of magnitude.


Y
eB

STEP II Determine the total number of observations, say, n.


STEP 111 Use the following formula to compute ith decide.
ur

th
i{n + l)
oY

Di = Value of observation, i = 1,2,..., 9.


10
ad

ILLUSTRATION The following table gives the wages of 30 workers in a factory. Compute 4th and
d

6th deciles.
in

S.No. Wages (f)


Re

S.No Wages (f)


1 330
F

16 240
2 320 17 330
3 550 18 420
4 470 19 380
5 210 20 450
6 500 21 260
7 270 22 330
120 23 440
9 680 24 480
10 490 25 520
11 400 26 300
12 170 27 580
13 440 28 370
14 480 29 380
15 620 30 350
22.51
DATA INTERPRETATION-1 (CENTRAL TENDENCY)

SOLUTION Arranging the wages in ascending order of magnitude, we obtain the following.
S.No. Wages i f) S.No Wages i ?)
120 16 400
1

170 17 420
2
210 18 440
3

4 240 19 445

260 20 450
5

6 270 21 470

7 300 22 480

8 320 23 480

w
9 330 24 490

10 330 25 500

F lo
11 330 26 520

12 330 27 550

ee
13 350 28 580

Fr
14 370 29 620

15 380 30 680
for
ur
We have, n = 30
\th
;i + 1 30 + 1
observation = Value of 4 observation
s
= Value of 4
ook

10 )
Yo

10

= Value of 12.4*'^ observation


eB

D4 = Value 01 \22.4*'^ observation+ —


10
(Value of 13 th observation
r
ad

- Value of 12^^ observation


ou

D = 350 + —{370 - 350) = 358


Y

^ 10
Re
nd

4*^ decile = 358.


Fi

Nth Nth
f n +l 30 + 1
D. = Value of 6 observation = Value of 6 observation
" V 10 y 10

= Value of 18.6^*^ observation

De = Value of 18‘^ observation+ 4-


10
(Value of 19'*^ observation
- Value of 18^*^ observation

Dg = 440 + —(445
10^
- 440) = 443.

22.7.2 COMPUTATION OF DECILES FOR DISCRETE FREQUENCY DISTRIBUTION


Let Xj/fi ;i -1,2,3,...,n is a frequency distribution of a variable X. To compute deciles of
this frequency distribution we use the following algorithm.
22.52
APPLIED MATHEMATICS-XI

ALGORITHM
5TEP1
Obtain the given frequency distribution
STl-:p II
Compute the cumulativefrequencies and denote If. byN.
iN
STEP HI Compute to compute ith decile, where i = 1,2,3, ,9.
10
iN
IV See the cumulative frequency just greater than . The corresponding value of the variable
10
is the ith decile, where i = l,2,
ILLUSTRATION
Compute the third and deciles of the following frequency distribution.

ow
Income (?) No. of workers
58 2
59 3

e
60 6

re
61

rFl 15

F
62 10
63 5

or
64
ou
4
65 3
ksf
66 1
oo

SOLUTION The cumulative frequency distribution is as given below.


Y

Income (?)
B

No. of workers Cumulative frequency (c.f)


58 2 2
re

59 3 5
oYu

60 6 11
ad

61 15 26
62 10 36
d

63 5 41
in
Re

64 4 45
F

65 3 48
66 1 50

N = 2 / = 50

3N
Third decile (D3): N = 50 => = 15
10
3N
The cumulative frequency just greater than 10 is 26 and the corresponding value of the
variable is 61.

D3 = ? 61.
7N
Seventh decile (Dy): hf = 50 => = 35.
10
22.53
DATA INTERPRETATION-1 {CENTRAL TENDENCY)

7N
The cumulative frequency just greater than is 41 and the corresponding value of the variable is
10

64. Therefore, Dy = ? 64.


22.7.3 COMPUTATION OF DECILES FOR A FREQUENCY DISTRIBUTION WITH CLASS INTERVALS
We may use the following algorithm to compute various deciles for a continuous series:
step I Obtain the given frequency distribution
STEP II Compute the cumulative frequencies and denote E // by N.
/N
to obtain decile, where i = 1,2, 3, .9-

ow
STEP III Compute 10
/N
STEP IV See the cinnulativefrequency just greater than . The corresponding class is the i“‘ deciles
10
class.

e
STEPV Use the formula

re
zN _ p

rFl
F
10
aI = 1 + X h, where
f
I = lower limit of the deciles class obtained in step IV

or
ou
F = cumulative frequency of the class preceding the deciles class in step IV
ksf
f=frequency of the i‘’’ deciles class
h = width of the deciles class.
oo

Following illustration will illustrate the above algorithm.


Y

ILLUSTRATION Compute D3 and Dj for the following frequency distribution


B

40-50 50-60 60-70 70-80


Marks: 0-lQ 10-20 20-30 30-40
re

6 4 2 1
10 17 7
No. of Students: 3
oYu

SOLUTION The cumulative frequency distribution is as given below.


ad

Marks X No. of students Cumulative frequency (c.f.)


d

3
in

3
Re

0-10
10 13
10-20
F

17 30
20-30
7 37
30-40
6 43
40-50
4 47
50-60
2 49
60-70
1 50
70-80

N = I /;● = 50

3N
= 15
Computation 0/ O3 : N = 50 10
3N
The cumulative frequency just greater than 10
is 30 and the corresponding class is 20 - 30.
22.54
APPLIED MATHEMATICS-XI

So, 20 - 30 is the third deciles class such that / = 20, / = 17, h = 10, f = 13, N = 50.
3N
-F
10 15-13 20
Dg - / + X /7 = 20 + xl0 = 20 + — = 21.05
/ 17 17

Hence, D3 = 21.05 marks.

7N
Computation of Dy : N = 50 = 35
10

7N
The cumulative frequency just greater than 10 is 35 and the corresponding class is 30 - 40.
So, 30 - 40 is the third deciles class such that / = 30, / = 7, /j = 10, f = 30, JV = 50

w
Dy=l +
10

/
xh = 30 +

F lo
35-30
7
50
X 10 = 30 + — = 37.14
7

ee
Fr
Hence, 7th deciles is 37.14 marks.

22.8 PERCENTILES
for
ur
PERCENTILES The ninety nine points zvhich divide the given distribution into hundred equal parts
s

are
called the percentiles of the distribution.
ook
Yo

Percentiles are generally denoted by Pj, The first percentile P, is the value of
eB

the variable such that it exceeds 1% of the observations and i


is exceeded by 99% of the
observations. Similarly, is the median.
our
ad

The methods for computing percentiles are exactly same as those of computing deciles m
i
case of all types of frequency distributions. In fact, one has to replace 10 by 100 in the
Y

formulas for deciles.


Re
nd

22.8.1 COMPUTATION OF PERCENTILES FOR INDIVIDUAL OBSERVATIONS


Fi

Let Tj, Xy,..., be n values of a variable X. To compute percentiles of these values we use

the following algorithm.


ALGORITHM

1 Arrange the observations in ascending order of magnitude.


STEF
il Determine the total number of observations. Let the nimiber of observations be n.
III Use the following formula to compute ith percentile.
Ih
n + 1
Pj = Value of / observation, i = 1,2,...,99.
100 Jj
Above algorithm is illustrated in the following illustration.
22.55
DATA INTERPRETATION-1 (CENTRAL TENDENCY)

ILLUSTRATION From the following infor^nation of wages of 30 workers in a factory calculate 40th
percentile.
S.No. Wages (f) S.No Wages (f)
330 16 240
1
320 17 330
2
550 18 420
3
470 19 380
4
210 20 450
5
500 21 260
6
270 22 330
7
120 23 440
8

ow
680 24 480
9
490 25 520
10
11 400 26 300

12 170 27 580
440 28 370
13

e
Fl 380

re
14 480 29
15 620 30 350

F
SOLUTION Arranging the wages in ascending order we obtain the following table:
ur
or
S.No. Wages (f) S.No Wages (f)
400
1 120 16
f
ks
2 170 17 420
Yo

18 440
oo

3 210

4 240 19 440
B

260 20 450
5
re

270 21 470
6
300 22 480
u

7
ad

480
Yo

8 320 23

9 330 24 490
d

10 330 25 500
Re
in

11 330 26 520

27 550
F

12 350
350 28 580
13
14 370 29 620

380 30 680
15

We have, n = 30.
Ih
30 + 1
P40 = Value of < 40 observation = Value of 12.4“" observation
100

4
P40 ^ Value of 12 th observation+ — {Value of 13 th observation
-Value of 12th observation
22.56
APPLIED MATHEMATICS - XI

P40 = 350 + 10
370 - 350) = 350 + — X 20 = 358
10

Hence, 40th percentile is ? 358.


22.8.2 COMPUTATION OF PERCENTILES FOR A DISCRETE FREQUENCY DISTRIBUTION

= 1, 2,..., n be a frequency distribution of a variable X. To compute percentiles


for this frequency distribution we use the following algorithm.
ALGORITHM
STEP 1
Obtahi the given frequency distribution.
SIEMI Prepare the cumulative frequency distribution and denote I f by N
iN ' -
STEP 111 Compute 100 to compute ith percentile, where i = 1,2,..., 99.
iN
STEP IV See the cumulative frequency just greater than

w
100 ■ The corresponding value of the
Following illustration illustrates the above algorithm.
x:

/: 1
2

9
3 4

F lo
ILLUSTRATION 1 Compute 27th percentile from the follozving frequency distribution:
1 5 6 7 9

e
26 59 72 52 29 7 1

Fre
SOLUTION First we prepare the culumative frequency distribution as given below:
.V
/
for cf
0 1 1
r
1 9
You

10
s
ook

2 26 36
3 59 95
eB

4 72 167
5 52 219
our
ad

6 29 248
7 7 255
8 1 256
dY
Re

N = ,lf =256
Fin

27N 27 X 256
We have, N = 256 => = 69.12
100 100

The cumulative frequency just greater than 69.12 is 95 and the corresponding value of a: is 3.
Hence, -P27 = 3.

22.8.3 COMPUTATION OF PERCENTILES FOR A FREQUENCY DISTRIBUTION WITH CLASS


INTERVALS

To compute percentiles for a continuous frequency distribution, we may use the following
algorithm;
ALGORITHM
STEF I
Obtain the given frequency distribution.
22.57
DATA INTERPRETATION-1 (CENTRAL TENDENCY)

STFP II Compute theiN cumulative frequencies and denote .I f by N.


STEP in Compute to find ith percentile, where i = 1,2,..., 99.
iN
STEP IV See the cumulative frequency just greater than . The corresponding class is the ith
100
percentile class.
Use the formula
iN
-F
100
RI =/ + xh to obtain ith percentile.
/
zohere, / = loiver limit for the ith percentile class obtained in step IV
f=frequency of the ith percentile class,

w
h = uyidth of ith percentile class.
F = cumidative frequency of the class preceding the ith percentile class obtained in step IV.
Compute 70th and 90lh percentile from the following data:

Flo
ILLUSTKATION

Marks group: 0-10 10-20 20-30 30-40 40-50 50-60 60-70 70-80 80-90 90-100

e
7 5 4 3 2
7 10 16 11

re
No. of Students: 5
The cumulative frequency distribution is as given below.

rF
SOLUTION

Marks group: No. of students (f) Cumulativefrequency (c.f.)


ur
5 fo 5
0-10
7 12
10-20
ks
10 22
20-30
Yo

16 38
30-40
oo

11 49
40-50
B

7 56
50-60
re

5 61
60-70
70-80 4 65
u
ad

3 68
80-90
Yo

2 70
90 -100
N ='Lf= 70
d
Re
in

Cornputationof ^70 ● N = 70 => VON _ 70 X 70 = 49.


100 ~ 100
F

The cumulative frequency just greater than 49 is 56. So, the corresponding class i.e. 50 - 60 is
the 70th percentile class such that / = 50, / = 7, /i = 10, F - 49.
70N
-F
49-49
100 X /i = 50 -I- X 10 = 50.
P70 - / + 7
/
Hence, P70 = 50 marks.
90N 90 X 70
= 63.
Computation of P90: N = 70^
The cumulative frequency just greater than 63 is 65 and the corresponding class is 70 - 80. So,
70 - 80 is the 90th percentile class such that / = 70, / = 4, = 10, F = 61.
Hence, <P^ = 75 marks.
22.58
APPLIED MATHEMATICS-XI

EXERCISE 22.7

1. Compute first and third quartiles from the following data-


Roll No.: 1 2 3 4 5 6 7
Marks: 20 28 40 12 30 15 50

2. Compute Qj, Uy from the following data:


Marks: 10 20 30 40 50 80
No. ofstudents: 4 7 15 7 2

3. Find out median, first quartile, third quartile and third decile of the following
distribution:
Height (in inches): 58 59 60 61 62 63 64 65 66
No. of persons: 2 3 ■ 6 15 10 5 4 3 1

4. Calculate the median, 3rd decile and 20th percentile from the following data:

w
-v: 0-5 5-10 10-15 15-20 20-25
/: 7

F lo
18 25 30 20

5. In the following table the figures relating to the wages of 60 workers in a factory are
given. Compute lower and upper quartiles.

ee
Wages (in ^): 20-25 25-30 30-35 35-40 40-45

Fr
No. of Workers: 2 10 25 16 7

6. Draw a 'less than ogive' from the following data and


for hence find the value of lower
quartile.
ur
Variable: 0-5 5-10 10-20 20-30 30-40 40-60
s

Frequena/:
ook

5 7 15 20
Yo

8 5

7. Draw the "less than ogive" for the data given below and determine the median and two
eB

quartiles.
Marks: 0-10
10-20 20-30 30-40 40-50 50-60 60-70 70-80 80-90 90-100
our

No. of 5 20 40 70 85 65 50 35 20 10
ad

Candidates:

8. From the following frequency distribution draw a cumulative frequency curve and read
Y

of the value of all quartiles and 5th decile.


Re
nd

Marks:
0-5 5-10 10-15 15-20 20-25 25-30 30-35 35-40
No. of Students: 4
Fi

6 10 10 55 22 18 5

ANSWERS
1- Qi=15,Q3=40 2. Qi =20,Q3 =40,D7 =40
3. Median = 61, = 61,Q3 = 63, D3 = 61 4. Median = 15, D3 = ll, P20 “ 8.6
5. = 30.6, Qj = 37.5 6. Qi = 11
7. Median = 47.65, Q, = 34.12, Q3 = 63 8. Q, = 20.2, Q2 - 24, Q3 = 30, D5 = 24
22.9 PERCENTILE RANK
The term percentile was first used by Francis Galton, an English Victorian era Statistician, in
1885. The rth percentile of a distribution is the value of the observation such that r% of the
observations fall at or below it. For example, 25^^ percentile or quartile Q,
is that value of a
variable such that 25% of the observations fall at or less than that value and 75% of the
22.59
DATA INTERPRETATION-! {CENTRAL TENDENCY)
th

Observations are greater than the value. We can also say that the percentile rank of 25
percentile is 25. The formal definition of the term percentile rank is as given below.
PERCENTILE RANK The percentUc jwik of a specific score x (saij) hi a frequency distribution ofsco
res

is the number r (ranging between 0 and 200) indicating that r% scores of thefrequency distribution
are less than or equal to x.
In other words, the percentile rank of a specific score in a frequency distribution of scores is
the percentage of scores in the frequency distribution that are equal to or lower than it.
For example, if a candidate in CAT scored 492 marks out of 500 and his score is higher than
or equal to 97% of the candidates taking the test, then we say that his percentile rank is 97

ow
and 492 is 97*^ percentile. Thus, a percentile raiik of 95 indicates that 95% of the scores fall at
or below the score at the 95^ percentile.
It is evident from the above definition that the percentile rank of lower quartile of a frequency

e
distribution of scores is 25 and that of upper quartile is 75. The median has the percentile

re
rank 50.

Frl
F
22.9.1 PERCENTILE RANKS FOR INDIVIDUAL OBSERVATIONS
In order to compute the percentile ranks of individual scores (observations), we may follow
ou
the following algorithm:

r
so
ALGORITHM

Obtain the scores and arrange them in ascending order of magnitude as given below.
5TF.P I
kf
^\/ ^2' I ■ ■ ■' ■
oo

S11-P II Use the formula:


Y

1
/ + -e
B

Percentile rank of Xj = 2-xlOO


n
re

where I - Number of scores less than x^, e = Number of scores equal to


oY
u

= Number of scores or observations


ad

n
d

ILLUSTRATIVE EXAMPLES
in
Re

EXAMPLE 1 Following are the IQ (Intelligence Quotient) scores of 20 persons in an IQ test.


F

110, 260, 112, 136, 149, 207, 165, 293, 167, 172, 192, 200, 260, 226, 251, 254, 260,
291, 296, 260

Find the percentile rank of score 260 and write its interpretation.
SOLUTION Arranging the scores in ascending order, we obtain
no, 112,136,149,165,167,172,192, 200, 207, 228,251,254, 260, 260, 260,260,
291,293, 296
We find that

n = Number of scores (observations) = 20


I = Number of scores less than 260 = 13, e = Number of scores equal to 260 = 4
1
I + e 13+ - x4
Percentile rank of score 260 = 2^x100 = 2 X100 = 75
n 20
22.60
APPLIED MATHEMATICS -XI

Interpretation The percentile rank of score 260 is 75 means that 75% 0


persons in the groups
have IQ score less than or equal to 260.
EXAMPLE 2
FoUowmg are the marks scored by a group of students in a test on data interpretation ■
87, 89, 27, 93, 92, 86, 90, 87, 72
Find the 60th percentile and its percentile rank.
SOLUTION Arranging the marks in ascending order, we obtain the following list of marks:
27,72, 86, 87,87,89, 90,91, 93
Here n = 9
th
9 + n

ow
^60 = Value of ●! 60 score = Value of 6 th score = 89
100
Now, / Number of scores less than 89 = 5, e = Number of scores equal to 89 = 1
/A. 5 + - X 1

e
Percentile rank of 89 = ^ X 100 = 2
X 100 = 61.11

re
n 9

Frl
A school principal decides to provide the results of 25 students of class XI in tenns of
EXAMPLE 3

F
percentile ratiks instead of regular practice of providing results as percentages of total tnarks scored in

all the subjects. Following is the list of students and their tnarks scored out of500 marks.
ou
or
Name Marks scored Name Marks scored kfs Name Marks scored
Aditi 471 Bhaskar 445 Namrata 481
Anuj 472 Chetan 488 Patel 454
oo

Ali 477
Chirag 456 Petra 488
Ashok 477
Y

Fransis 435 Rachel 442


B

Akshay 461 John 444


Rupal 456
Amitabh 468 Michael 456 Shruti 477
re

Azam 459 Manu 469 Thomas 472


Aziz 471
oYu

Mathew 435 Vivek 477


ad

Vishal 465

Using the percentile rankformula, find the candidates scoring 88'^' percentile.
d

SOLUTION Arranging the marks scored in ascending order, we obtain the following table:
in
Re

Name Marks scored Name Marks scored


F

Fransis 435 Amitabh 468


Methew 435 Manu 469
Rachel 442 Aziz 471
John 444 Aditi 471
Bhaskar 445 Thomas 472
Patel 454
Anuj 472
Michael 456 Ali 477
Rupal 456 Ashok 477
Chirag 456 Shruti 477
Azam 459 Vivek 477
Akshay 461 Namrata 481
Vishal 465 Petra 488
Chetan 488
22.61
DATA INTERPRETATION-1 (CENTRAL TENDENCY)

Here, n = 25 and percentile rank = 88.


Let / be the number of students who have scored marks less than or equal to 88th percentile.
Then,
/ /
88 = - X 100 => 88 = — X 100 => / = 22
Thus 22 students have scored marks less than or equal to 88>1' percentile. We find that 22nd
student in the list have scored 477 marks and there are three students who have scored same
marks. Hence, Ali, Ashok, Shruti and Vivek have 88»- percentile rank.
2292 PERCENTILE RANKS FOR GROUPED FREQUENCY DISTRIBUTION
In order to find the percentile ranks for scores in a grouped frequency distribution, we may
follow the following algorithm.
ALGORITHM .. j .
and re-zvnte it in ascending order of scores
as
Sti;f 1 Obtain thefrequenc]/ distribution of scores

w
given below. Xn
Scores : -■^3 -T.; ^n-\
■^2

F lo
f\ h h f, /„
Frequency:
STEP II Prepare cumulative frequency distribution.

ee
STEP HI Use the following formula to find the percentile rank of score -Y;(●

Fr
C 1-1
xlOO
Percentile rank of score X; = N for
ur
where fj = frequency of score Xj
=z cumulativefrequency of score Xj_i (score just less than .y,)
s
C
ook
Yo

N = 'Lfi = Sum of all frequecies.


eB

ILLUSTRATIVE EXAMPLES
EXAMPLE 1 Thefrequenaj distribution of marks of 25 students is an examination is as given
r

below:
ou
ad

16 20 25 28 32 35 38 42 45 49
Marks:
2 3 1 2 4 2 6 1 2 2
Y

Number of students:
Find the percentile rank of 38 marks.
Re
nd

SOLUTION Let us first prepare the cumulative frequency distribution of marks as given
Fi

below.

Marks (,y,-) Number of students (f) Cumulativefrequency (Q)


2 2
16
3 5
20
1 6
25
2 8
28
4 12
32
2 14
35
6 20
38
1 21
42
2 23
45
2 25
49

N = S /;. = 25
22.62
APPLIED MATHEMATICS-XI

We find from the above table that:

= (Cumulative frequency of score just less than 38) = 14


c /-I

/ = (Frequency of score 38) = 6 and, A/ = E^-=25


4^'
1
c/-I 14 + - X 6
Percentile rank of score 38 =
N
X 100 = ^ X 100 = 68
25

EXERCISE 22.8
1. The scores of 10 students in an examination are as follows:

21, 42, 35, 19, 17, 35, 49, 32, 28, 37


Find the percentile rank of the student scoring 35 marks.
2. The marks scored by 20 students of class XI in
i half-yearly examination in Mathematics
are as follows:

w
24, 33,43,59,55, 68,87, 49, 25,99, 96,63, 79, 63, 81, 63,82, 63,54, 47
Find the percentile rank of 63 marks.
3. given
The frequency
F lo
distribuHon of marks scored by a group of 40 students of class XI is
i

ee
as
below:

Fr
Marks:
12 18 25 30 36 48 52 57 63 75 82 88 92 96 98
Number of students: 2 3 1 2 4 2 6 1
for 2 2 4 6 3 1 1
Fmd the percentile rank of 82 marks.
ur
ANSWERS
oks

1. 60 2. 60 3. 67.5
Yo
o

22.10 MODE
eB

In earlier classes we have studied about the computation of mode of raw data. In this
section, we shall learn about the computation of mode of a discrete frequency distribution
our
ad

and frequency distribution with class intervals. But, let us first recall the definition of mode.
MODE The mode or modal value of a distribution is that value of the variable for which the frequence
IS maximum. ■' j j
Y
Re

Thus, the mode of a distribution is that value of the variable around which the values
nd

of the
variable are clustered densely.
Fi

22.10.1 COMPUTATION OF MODE OF A SERIES OF INDIVIDUAL OBSERVATIONS


Jn order to compute the mode of a series of individual observations, we first convert it into a
iscrete series frequency distribution by preparing a frequency table. From the frequency
table, we identify the value havmg maximum frequency. The value of variable so obtained is
the mode or modal value.

Following examples will illustrate the procedure.

ILLUSTRATIVE EXAMPLES

EXAMPLE ] Find the mode of the following data:


120, 110, 130, no, 120, 140, 130, 120, 140, 120
22.63
DATA INTERPRETATION-1 (CENTRAL TENDENCY)

SOLUTION Let us Hrst form the frequency table for the given data as given below:
110 120 130 140
Value Xj-.
2 4 2 2
Frequency fj:
We observe that the value 120 has the maximum frequency.
Hence, the mode or modal value is 120.
EXAMPLE 2 Find the mode of the following data:
25, 16, 19, 48, 19, 20, 34, 15, 19, 20, 21, 24, 19, 16, 22, 16, 18, 20, 16, 19
SOLUTION The frequency table of the given data is as given below:
Value {Xi): 15 16 18 19 20 21 22 24 25 34 48
3 1 1 1 1 1 1
Frequency {fi): 1 4 1 5

We observe that the value 19 has the maximum frequency i.e. it occurs maximum number of

w
times. Therefore, mode of the given data is 19.
EXAMPLE 3

SOLUTION

Value (:r,-):
Frequency (//):
14

1
15

3
16

1
F lo
Find the value ofx, if the mode of the following data is 25:
15, 20, 25, 18, 14, 15, 25, 15, 18, 16, 20, 25, 20, x, 18
The frequency table of the given data is as given below:
18

3
20

3
for F
25

3 ree
X

It is given that the mode of the given data is 25. So, it must have the maximum frequency. That
Your

is possible only when x = 25.


oks

Hence, x=25.
eBo

22.10.2 COMPUTATION OF MODE BY GROUPING


Sometimes there are two or more values having the same frequency. In such cases one cannot
ad
our

say which is modal value and hence mode is said to be ill-dehned. Such a frequency
distribution is also known as bimodal or multimodal distribution. For such frequency
distribution mode is computed by grouping method.
Re
Y

Consider the following frequency distribution:


Find

9 10 11 12 13
X: 5 6 7
14 13 11 7 4 3
8 12 13
that modal value is 8, because the
From the above frequency distribution, we can dearly say
value 8 of variable x has occurred the maximum number of times i.e. 14. But, we find that the
difference between the maximum frequency and the frequencies of the values of the variable
on both sides of 8 which are very close to 8 is very small. This means that the values ot
variable are heavily concentrated on either side of 8. Therefore, if we find mode }ust by
inspection, an error is possible. In such cases, we prepare a groupmg table and an anal^is
table to find the mode. These tables help us in determining the correct value of mode. The
grouping table consists of six columns which are constructed by usmg the followmg
algorithm.
ALGORITHM
STEP 1 Obtain the discrete frequency distribution.
STEP 11 Take the column offrequencies as column I and encircle the maximum frequency in it.
22.64
APPLIED MATHEMATICS-XI

--'ii'T 111
Construct column II, containing the sum of the frequencies taken two at a time and encircle
the maximum frequency in if.
STi-P IV
Leave the first frequency and construct column IH, containing the sum of the frequencies
mken two at a time. Encircle the maximumfrequency in column III ^ ^
S Ti;!‘ \
Construct column IV_, containing the sum of three frequencies at a time and encircle the
maximum frequency in it.
STKl’ VI
Excliide the first frequency and compute the sum of the frequencies taken three at a time to
construct column V. Encircle the maximum frequency in this column.
■ I t-r
Exclude the ifrst two frequencies and compute the sum of the frequencies taken three at a

time to construct column VI. Encircle the maximumfrequency in this column.


grouping table, we prepare an analysis table by using the following

ow
ALGORITHM
STIT* 1
Prepare a table in which in the top most row zvrite all values of the variable and in the left
most column write column numbers from I to VI. ^
'-riv II
See the maxinmni frequency in the first column of the

e
grouping table and obtain the
corresponding value of the variable. Nozu, mark a bar (\) in the first rozv of the analysis

re
rFl
table against the value of the variable having the maximum frequency. Continue the same
procedure for the remaining five columns. ^

F
■^TEP 111
Find the total number of bars corresponding to each value of the variable. That value of the
variable zvhich has the maximum number of bars is the mode of the frequency distribution.

r
Following illustration will illustrate the grouping and analysis tables.
ou
ILLUSTRATION 4
fo
Compute the modal value for the follozving frequency distribution:
ks
A'; 95 105 115 225 135 145 155 165 175
oo

4 2
y- 18 22 21 19 10 3 2
Y

SOLUTION It is clear from the frequency distribution that the difference between the
eB

maximum frequency and frequency succeeding it is very small and values of the variable .v
method^^^ concentrated on its either side. So, we compute the modal value by grouping
are
r
ou

Grouping Table
ad
Y

x: f
Col. I Col. II Col. Ill Col. IV Col.V Col. VI
d

95 4
Re
in

6
105 2 24
F

20
115 18

45
125

135 21

145 19

29
155 10
13
32
165 3 15
5
175 2
22.65
DATA INTERPRETATION-l (CENTRAL TENDENCY)

Analysis Table
145 155 165 175
Col. No. 95 105 115 125 135

ni

rv

VI

2 5 4 2 1
Total

From the analysis table, it is clear that the value 125 has the maximum number of bars. So,

w
Modal value is 125.

COMPUTATION OF MODE FOR A CONTINUOUS FREQUENCY DISTRIBUTION

F lo
22.10.3

In case of a grouped or continuous frequency distribution with equal class intervals, we


use

the following algorithm to compute the mode.

e
Fre
ALGORITHM

STi;i’ 1 Obtain the continuous frequency distribution. for


^11-1’ II Determine the class of maxhnum frequency either by inspection or by grouping method.
This class is called the modal class.
r
Obtain the values of the following from the frequency distribution:
You

Slliiili
oks

I = lower limit of the modal class, f=frequency of the modal class


eBo

h = width of the modal class,


/j = frequency of the class preceding the modal class,
our
ad

f2 = frequency of the class following the modal class.


s[ i;p IV Substitute the values obtained in step III in the following formula:
/-/i
dY

Mode = / + x/i
Re

2/-/1-/2
Fin

Following examples will illustrate the above algorithm.


ILLUSTRATIVE EXAMPLES

LXAMPLE 1 Compute the mode for the following frequency distribution:


Size of items: 0-4 4-8 8-12 12-16 16-20 20-24 24-28 28-32 32-36 36-40
1 0
Frequency: 5 7 9 17 12 10 6 3

SOLUTION Here, the maximum frequency is 17 and the corresponding class is


12-16. So, 12-16 is the modal class such that I = I2,h = 4, f = 17, f = 9 and /i = 12.
Mode = / -I-
f-h xh
2/ - /i “ fz
22.66
APPLIED MATHEMATICS - XI

17-9 32
Mode = 12 +
34-9-12 ^4 = 12 + —
13
x4 = 12 + — = 12 + 2.46 = 14.46
13

EXAMPLE 2
For the following groupedfrequency distribution find the mode:
Class: 3-6 6-9 9-12 12-15 15-18 18-21 21-24
Frequency: 2 5 10 23 21 12 3

SOLUTION We observe that the class 12-15 has maximum frequency. Therefore, this is the
modal class such that / = 12, // = 3, / = 23, f = 10 and /, = 21.

Mode = / + /-/i xh
y-h-fi

w
23-10 13 13
Mode = 12 +
x3 = 12 + — x3 = 12+ — = 14.6
46 -10 - 21 15 5

F lo
Compute the value of mode for the followingfrequency distribution.
EXAMPLE 3

ee
Class: 100-110 110-120 120-130 130-140 140-150 150-160 160-170

Fr
Frequency: 4 6 20 32 33 8 2

SOLUTION Clearly, the difference between the maximum frequency and the frequency for
ur
preceding is very small. So, we shall determine the modal class by grouping method.
Grouping Table
s
ok
Yo

Class Frequency
o
eB

Col. I Col. II Col. Ill Col. IV Col.V Col. VI


100-110 4
10
r

110-120 6
ou
ad

30
26
120-130 20
58
52
Y

130-140 32
65
Re

85
nd

140-150 33 73
41
150-160
Fi

43
10
160-170 2

Analysis Table
Col. No. 100-110 110-120 120-130 130-140 140-150 150-160 160-170
I

II
I
III

IV

VI

Total 1 3 5 4 1
22.67
DATA INTERPRETATION-1 (CENTRAL TENDENCY)

Clearly, class 130-140 has maximum number of bars. So, 130-140 is the modal class.
/ = 130, h = 10, / = 32, /, = 20, A = 33
/-/i x/f
Now, Mode - ^ +
2/-/1-/2
32-20 12
Mode = 130 + X 10 = 130 + X 10 = 140.9
64 - 20 - 33 9

EXAMPLE 4 The follozoing data gives the distribution of total household expenditure (in rupees) of
znanual workers in a city:
Expenditure (in ^ Frequency Expenditure (in ^ Frequency

24 3000-3500 30
1000-1500

w
40 3500-4000 22
1500-2000
4000-4500 16

F lo
2000-2500 33

28 4500-5000 7
2500-3000

ee
Find the average expenditure zuhich is being done by the maximum number of manual workers.

Fr
SOLUTION We know that the mode is the value of the variable wlaich occurs maximum
number of times in a frequency distribution. So, the average expenditure done by the
for
maximum number of workers is the modal value. We observe that the class 1500-2000 has
ur
the maximum frequency 40. So, it is the modal class such that I = 1500,/i = 500,/ = 40,
/ = 24 and /; = 33.
ks

/-/i
Yo

x/i
oo

Mode = / +
2/-/1-/2
eB

40-24 16
Mode = 1500 + X 500 = 1500 + — X 500 = 1847.826
80 - 24 - 33 23

Calculate the value of mode for the folloioing frequency distribution:


r

EXAMPLE 5
ou
ad

Class: 1-4 5-8 9-12 13-16 17-20 21-24 25-28 29-32 33-26 37-40
Y

2 5 9 12 14 14 15 11 13
Frequency:
Re

SOLUTION Here, the classes are not in the inclusive form. So, we first convert them in
nd

inclusive form by subtracting h/2 from the lower limit and adding /j/2 to the upper limit of
Fi

each class, where h is the difference between the lower limit of a class and the upper limit of
the preceding class.

Class Frequency Class Frequency

24 20.5-24.5 30
0.5-4.5

40 24.5-28.5 22
4.5-S.5

40 28.5-32.5 22
8.5-12.5

33 32.5-36.5 16
12.5-16.5

28 36.5-36.5 7
16.5-20.5

To find the modal class, we use the grouping method:


22.68
APPLIED MATHEMATICS-XI

Grouping Table
Class:
Frequency:
Coll Col II Col III Col IV ColV Col VI

0.5-4.5 2
7
4.5-S.5 5 15
13
8.5-12.5
22
17
12.5-16.5 29
9
21
16.5-20.5 12 35

ow
26
20.5-24.5 14
40
28
24.5-28.5 14
43
29
28.5-32.5 15 40 39

e
Fl
26

re
32.5-36.5 11
24

F
36.5-40.5 13
ur
Analysis Table
or
sf
Col. No. 0.5- 4.5- 8.5- 12.5- 16.5- 20.5- 24.5- 28.5- 32.5- 36.5-
4.5 8.5 12.5 16.5 20.5 24.5 28.5 32.5 36.5 40.5
k
Yo
oo

I
B

n
re

ni

IV
u
ad
Yo

VI
d
Re

Total
in

1 3 5 4 1
F

Since 24.5-28.5 has the maximum number of bars. So, 24.5-28.5 is the modal class.

/ = 24.5, /i = 4, / = 14, /i = 14, /2 = 15


Mode = / + /-/i xh
2/-/1-A
14-14
Mode = 24.5 + X 4 = 24.5 + 0 = 24.5
28 - 14 - 15

I XAMFLE 6 The following table shows the age distribution of cases of a certain disease admitted
during a year in a particular hospital.
Age (in years): 5- 14 15 - 24 25 -34 35 -44 45- 54 55 - 64
No. of cases: 6 11 21 23 14 5
Find the average age for which maximum cases occurred.
22.69
DATA INTERPRETATION-1 (CENTRAL TENDENCY)

SOLUTION Here, class intervals are not in inclusive form. So, we first convert them in
limit of
inclusive form by subtracting/f/2 from the lower limit and adding /i/2 to the upper
each class, where h is the difference between the lower limit of a class and the upper limit of
the preceding class. The given frequency distribution in inclusive form is as follows.
34.5 34.5 - 44.5 44.5 - 54.5 54.5 - 64.5
Age (in years): 4.5 -14.5 14.5 - 24.5 24.5 -.—
11 21 23 14 5
6
No. of cases:
We observe that the class 34.5 - 44.5 has the maximum frequency. So, it ISi the modal class

such that

/ = 34.5, li = 10, / = 23, /i = 21 and fj = 14

ow
Mode = / +
/-/i x/i
2/ “ /i “ A
23-21
Mode = 34.5 + - X 10 = 34.5 + — xio = 36.31
46 - 21 - 14 11

e
re
EXERCISE 22.9

i Find the mode of the following data:

rFl
F
(i) 3, 5, 7,4, 5, 3, 5, 6, 8, 9, 5, 3, 5, 3, 6,9, 7, 4
(ii) 3, 3, 7,4, 5, 3, 5, 6, 8, 9, 5, 3, 5, 3, 6, 9, 7, 4

or
(iii) 15, 8, 26, 25, 24, 15, 18, 20, 24, 15, 19, 15
ou
2. The shirt sizes worn by a group of 200 persons, who bought the shirt from a store, are
as
ksf
follows:
37 38 39 40 41 42 43 44
oo

Shirt size:
25 39 41 36 17 15 12
Number of persons: 15
Y
B

Find the modal shirt size worn by the group.


3 Find the mode of the following distribution,
re

(i) Class-interval: 0-10 10-20 20-30 30-40 40-50 50-60 60-70 70-80
Frequency; 5 8 7 12 28 20 10 10
oYu

(ii) Class-interval: 10-15 15-20 20-25 25-30 30-35 35-40


ad

45 75 35 25 15
Frequency: 30
30-35 35-40 40-45 45-50 50-60
d

(iii) Class-interval: 25-30 14


25 34 50 42 38
in

Frequency:
Re

0-10 10-20 20-30 30-40 40-50 50-60 60-70


(iv) Class: 7
10 16 12 6
F

10
Frequency:
4. Compare the modal ages of two groups of students appearing for an entrance test:
16-18 18-20 20-22 22-24 24-26
Age (in years): 23
50 78 46 28
Group A:
89 40 25 17
54
Group B:
5. The marks in science of 80 students of class X are given below: Find the mode of the
marks obtained by the students in science.
Marks: 0-10 10-20 20-30 30-40 40-50 50-60 60-70 70-80 80-90 90-100
4 1 1
Frequency: 3 5 16 12 13 20 5

6 The following is the distribution of height of students of a certain class in a certain city:
163-165 166-168 169-171 172-174
Height (in ems): 160-162 18
15 118 142 127
No. of students:
Find the average height of maximum number of students.
22.70
APPLIED MATHEMATICS-XI

7. The following table shows the ages of the patients admitted in


i a hospital during a year:
Age (in years): 5-15 15-25 25-35 35-45 45-55 55-65
No. of students: 6 11 21 23 14 5
Find the mode and the mean of the data given above. Compare and interpret the two
measures of central tendency.
8. The following data gives the information on
the observed lifetimes (in hours) of 225
electrical components:
Lifetimes (in hours): 0-20 20-40 40-60 60-80 80-100 100-120
No. of components: 10 35 52 61 38 29
Determine the modal lifetimes of the components.
9. The following table gives the daily income of 50 workers of a factory:

low
Daily income (in ^) 100-120 120-140 140-160 160-180 180-200
Number of workers: 12 14 8 6 10
Find the mean, mode and median of the above data.
10. The following distribution gives the state-wise teacher-student ratio-- iin higher

ee
secondary schools of India. Find the mode and mean of this data. Interpret, the two

F
Fr
measures:

Number of students Number of Number of students Number of


for
ur
per Teacher StatesAI.T. per Teacher States/U.T.
15-20 3 35-40 3
k s
20 - 25 8
Yo

40 - 45 0
oo

25 - 30 9 45 - 50 0
eB

30 - 35 10 50-55 2

11- Find the mean, median and mode of the following data:
r

Classes: 0-50
50-100 100-150 150-200 200-250 250-300 300-350
ou
ad

Frequency: 2 3 5 6 5 3 1
Y

12. A student
, ,. noted the , number of cars passing through a spot on a road for 100 periods
each of 3 minutes and summarised it in the table given below. Find the mode of the data:
nd
Re

Number of cars: 0-10 10-20 20-30 30-40 40-50 50-60 60-70 70-80
Frequency: 7 14 13 12 20
Fi

11 15 8
13. The followin
ig frequency distribution gives the monthly consumption of electricity of 68
consumers o
f a locality. Find the median, mean and mode of the data and compare them.
Monthly consump- 65-85 85-105 105-125 125-145 145-165 165-185 185-205
tion: (in units)
No. of consumers: 4 5 13 20 14 8 4
14. 100
surnames were randomly picked up from a local telephone directly and the
frequency distribution of the number of letters in the English alphabets in the surnames
was obtained as follows;
Number of letters: 1-4 4-7 7-10 10-13 13-16 16-19
Number surnames: 6 30 40 16 4 4

Determine the median number of letters in the surnames. Find the mean number of
letters in the surnames. Also, find the modal size of the surnames.
22.71
DATA INTERPRETATION-I (CENTRAL TENDENCY)

15. Find the mean, median and mode of the following data:
Classes: 0-20 20-40 40-60 60-80 80-100 100-120 120-140
8 10 12 6 5 3
Frequency: 6
16. The following data gives the distribution of total monthly houshold expenditure of 200
families of a village. Find the modal monthly expenditure of the families. Also, find the
mean monthly ei^enditure: ■ -

Expenditure .frequency Expenditure Frequency


(in^ (in ^

1000-1500 24 3000-3500 30

1500-2000 40 3500-4000 22

w
2000-2500 33 4000-4500 16

2500-3000 28 4500-5000 7

F lo
17. The given distribution shows the number of runs scored by some top batsmen of the
world in one-day international cricket matches.

ee
Fr
Runs scored Number of Runs scored Number of
bastsman bastsman

3000-4000 4 7000-8000
for 6
ur
4000-5000 18 8000-9000 3
s
ok

9000-10000 1
Yo

5000-6000 9
o

6000-7000 7 10000-11000 1
eB

Find the mode of the data.

18. The frequency distribution table of agriculture holdings in a village is given


r
ad
ou

below:

Area of land (in hectares): 1-3 3-5 5-7 7-9 9-11 11-13
Y

Number of families: 20 45 80 55 40 12
Re
nd

Find the modal agriculture holdings of the village.


Fi

19. The monthly income of 100 families are given as below:


Income in (in ?) Number of families

0-5000 8

5000-10000 26

10000-15000 41

15000-20000 16

20000-25000 3

25000-30000 3

30000-35000 2

35000-40000 1

Calculate the modal income.


22.72
APPLIED MATHEMATICS-XI

ANSWERS
1- (i) 5 (ii) 3 (iii) 15 2. 40

3. (i) 46.67 (ii) 22.14 (iii) 38.33 (iv) 36

4. Group A: 18.93 years. Group B: 18.83 years 5. 53.17 b. 167.35

7. Mode = 36.8 years. Mean = 35.37 years 8. 65.625 hours

9. Mean = 145.20, Median = 138.57, Mode = 125


10. Mode = 30.6, Mean = 29.2 11. Median = 170.83, Mean = 169, Mode = 175
12. Mode = 44.7 cars

13. Median = 137 units. Mean = 137.05 units. Mode = 135.76 units

M. Median = 8.05, Mean = 8.32, Modal size = 7.88

low
15. Median = 61.66, Mean = 62.4, Mode = 65
16. ^ 1847.83, ? 2662.50 17. Mode = 4608.7 runs

18. 6.2 hectares 19. ^ 11875

ee
F MULTIPLE CHOICE QUESTIONS (MCQs)

Fr
Mark the correct alternative in each of thefollowing:
1. Which one of the following is not a measure of central value? for
ur
(a) Mean (b) Range (c) Median (d) Mode
2. The mean of n observationsis X. If k is added to each observation, then the
ks
new mean is
Yo

(a) X (b) X + k (c) X-k


oo

id) kX
3. The mean of n observations is X. Ifeach observation is multiplied by A:, the mean of new
eB

observations is
X
r

(a) kX
(b)y (c) X + k id) X-k
ou
ad

4. The mean of a set of seven numbers is 81. If one of the numbers is discarded, the mean of
Y

the remaining numbers is 78. The value of discarded number is


(a) 98 (b) 99 (c) 100 id) 101
nd
Re

5. For which set of numbers do the mean, median and mode all have the same value?
Fi

(a) 2,2,2,2,4 (b) 1, 3,3,3, 5 (c) 1,1,2,5,6 (d) 1,1,1,2, 5


6. For the set of numbers 2,2,4,5 and 12, which of the following statements is true?
(a) Mean = Median (b) Mean > Mode (c) Mean < Mode (d) Mode = Median
7. If the arithmetic mean of 7,5,13, x and 9 is 10, then the value of x is
(a) 10 (b) 12 (c) 14 (d) 16
8. If the mean of five observations x, x + 2, at + 4, x + 6, x + 8, is 11, tlien the mean of first three
observations is
(a) 9 (b) 11 (c) 13 (d) none of these
9. Mode is

(a) least frequent value (b) middlemost value


(c) most frequent value (d) none of these
22.73
DATA INTERPRETATION-1 (CENTRAL TENDENCY)

10. The following is the data of wages per day; 5,4,7,5,8,8,8,5,7,9, 5,7,9,10,8. The mode
of the data is
(a) 7 (b) 5 (c) 8 (d) 10
11 The median of the following data: 0,2,2,2, - 3,5, -1,5,5, - 3,6,6,5,6 is
(a) 0 (b) -1.5 (c) 2 (d) 3.5
12. The algebraic sum of the deviations of a set of n values from their mean is
(a) 0 (b) n-1 (c) n (d) n + l
13. A,B,C are three sets of values of x:
A: 2,3,7,1,3,2,3
B: 7,5,9,12,5,3,8
C; 4,4,11,7,2,3,4
Which one of the following statements is correct?

w
(a) Mean of A = Mode of C (b) Mean of C = Median of B
(c) Medianof B = Modeof A (d) Mean, Median and Mode of A are equal.

F lo
14. The mean of five numbers is 30. If one number is excluded, their mean becomes 28. The
excluded number is

ee
(a) 28 (b) 30 (c) 35 (d) 38

Fr
15. If the man of the observations : x,x-^3,x + 5,x+7,x+10 is 9, the mean of last three
observaiton is
for
ur
31 32 34 35

(a) J (b) j (d) 3


s
ook
Yo

16. If Xis the mean of Ti,T2,...,t„, then for fl?t0, the mean of axi,ax2,.-.,ax n>
eB

h. £2 T,
/● ● */ IS
a ' a a
r
ad
ou

( 1V U nx
fl+-
(a) a+- X (b) (c) a+-
(d) a) In
a) aj ajn
Y

17. Let Xbethemeanof Ti,X2,...,t„ and Y themeanof t/1,1/2'●●■'!/ «● ^ Z is the mean of


Re
nd

Xi,X2,...,x„,yi,y2/-wy«/ then Z is equal to


Fi

X+Y X+Y
.V X-hY (d)
(a) X-hY (b) — (c) n In

18. If Xi,X2,...,Xit arethemeansof n groups with Ml,M2/ ' /"jfc number of observations
respectively, then the mean X of all the groups taken together is given by

k _ 1 cf _
I n,Xi Z
1=1 1=1

(a) Z (b) (c) — (d)


In
1=1
I".
1=1

19. The median of the data: 4,4,5,7,6,7,7,12,3 is


(a) 4 (b) 5 (c) 6 (d) 7
22.74
APPLIED MATHEMATICS -XI

20. The median of the data : 78,56,22,34, 45, 54,39,68,54,84 ISi


(a) 45 (b) 49.5 (c) 54 (d) 56
21. There are 50 numbers. Each number is subtracted from 53 and the mean of the numbers
so obtained is found to be-3.5. The mean of the given numbers is
(a) 46.5 (b) 49.5 (c) 53.5 (d) 56.5
22. The mean of 100 observations is 50. If one of the observations which was 50 is replaced
by 150, the resulting mean will be
(a) 50.5 (b) 51 (c) 51.5 (d) 52
23. Mode of the data : 15,14,19,20,14,15,16,14,15,18,14,19,15,17 ,15 ISi
(a) 14 (b) 15 (c) 16 (d) 17
24. The empirical relation between mean, mode and median is

low
(a) Mode = 3 Median-2 Mean (b) Mode = 2 Median-3 Mean
(c) Median = 3 Mode-2 Mean (d) Mean = 3 Median-2 Mode
25. The mean of a, b, c, d and e is 28. If the mean of a, c, and e is 24, what is the mean of
b and d?
(a) 31 (b) 32 (c) 33 (d) 34

ee
F
26. The mean of 25 observations is 36. Out of these observations if the mean of first 13

Fr
observations is 32 and that the last 13 observations is 40, the 13* observaation is
(a) 23 (b) 36 (c) 38 (d) 40
for
ur
27. Which of the following is not a measure of central tendency?
(a) Mean (b) Median (c) Mode (d) Standard deviation
ks

28. The algebraic sum of the deviations of a frequency distribution from its mean is
Yo

(a) always positive


oo

(b) always negative


(c) 0 (d) a non-zero number
eB

29. The arithmetic mean of 1,2,3,..., nis


u + 1 n~l
r

n n
(a)
(b)^ (0^
ou

(d) 2 + ^
ad

30. For a frequency distribution, mean, median and mode are connected by the relation
Y

(a) Mode = 3 Mean - 2 Median (b) Mode = 2 Median - 3 Mean


Re
nd

(c) Mode = 3 Median - 2 Mean (d) Mode = 3 Median + 2 Mean


Fi

31. Which of the following cannot be determined graphically?


(a) Mean (b) Median (c) Mode (d) None of these
32. The median of a given frequency distribution is found graphically with the help of
(a) Histogram (b) Frequency curve (c) Frequency polygon (d) Ogive
33. The mode of a frequency distribution can be determined graphically from
(a) Histogram (b) Frequency polygon
(c) Ogive (d) Frequency curve
34. Mode is

(a) least frequent value (b) middle most value


(c) most frequent value (d) None of these
22.75
DATA INTERPRETATION-1 (CENTRAL TENDENCY)

35. The mean of n observations is X. If the first item is increased by 1, second by 2 and so on,
then the new mean is
n
(a) X+n (b) X-f- (0 (d) None of these

36. (3ne of the methods of determining mode is


(a) Mode = 2 Median - 3 Mean (b) Mode = 2 Median + 3 Mean
(c) Mode = 3 Median - 2 Mean (d) Mode = 3 Median + 2 Mean
37. If the mean of the following distribution is 2.6, then the value of y is
2 3 4 5
Variable (x): 1

4 5 1 2
Frequency: y
(c) 13 (d) 24

w
(a) 3 (b) 8
38. The relationship between mean, median and mode for a moderately skewed distribution

F lo
IS

(a) Mode = 2 Median - 3 Mean (b) Mode = Median - 2 Mean

ee
(c) Mode = 2 Median - Mean (d) Mode = 3 Median - 2 mean

Fr
39. The mean of a discrete frequency distribution Xj / /●; i = 1,2,..., n is given by
for
ur
(b) \
1=1
(a) (c)
1=1
(d)
s

I/-
ook
Yo

1=1
i=l
eB

40. If the arithmetic mean o{x,x + 3,x + 6,x-\r9, and .r +12 is 10, the x -
(a) 1 (b) 2 (c)6 (d) 4
our

41. If the median of the data: 24,25,26, t -h 2, t + 3,30,31,34 is 27.5, then ^ =


ad

(a) 27 (b) 25 (c)28 (d) 30


42. If the median of the data: 6,7, x - 2, a:, 17,20, written in ascending order, is 16. Then a: =
Y

(a) 15 (b) 16 (c)17 (d) 18


Re
nd

43. The median of first 10 prime numbers is


(b) 12 (c) 13 (d) 14
Fi

(a) 11
44. If the mode of the data: 64,60,48, t, 43,48,43,34 is 43, then a: -i- 3 =
(a) 44 (b) 45 (c) 46 (d) 48
45. If the mode of the data: 16,15,17,16,15, a:, 19,17,14 is 15, then a: =
(a) 15 (b) 16 (c) 17 (d) 19
46. The mean of 1, 3, 4, 5, 7, 4 is m. The numbers 3, 2, 2, 4, 3, 3, p have mean m -1 and
median q. Then, p + q =
(a) 4 (b) 5 (c)6 (d) 7

47. If the mean of a frequency distribution is 8. land ILfiX^ =132+5fc, S/ =20, then/c =
(a) 3 (b) 4 (c)5 (d) 6
48. If the mean of 6,7, x, 8, y, 14 is 9, then
(a) x + y = l\ (b) x + y = \9 (c) x-y = l9 (d) x-y = ll
22.76
APPLIED MATHEMATICS - XI

49. The mean of n observations is T . If the first observation is increased by 1, the second by
2, the third by 3, and so on, then the new mean is
n + l n + 1
(a) X + {2n + 1) (b) T + 2 (c) X + {n + 1) (d) -V

5u
50. If the mean of first n natural numbers is then 71 =

(a) 5 (b) 4 (c)9 (d) 10


■ The arithmetic mean and mode of a data are 24 and 12 respectively, then its median IS
i
(a) 25 (b) 18 {c)20 (d) 22
52. The mean of first n odd natural number is
n + 1 n

(a) ^

w
(b) 2 (c) n (d)

53. The mean of first H odd natural numbers is—, then77

F lo
=
81'
(a) 9 (b) 81 (c) 27 (d) 18

ee
54. If the difference of mode and median of a data is 24, then the difference of median and

Fr
mean is
(a) 12 (b) 24 (c)8 (d) 36
55. If the arithmetic mean of 7,8, .v, 11,14 is a:, then.v =
for
ur
(a) 9 (b) 9.5 (c) 10 (d) 10.5
56. If mode of a series exceeds its mean
s
by 12, then mode exceeds the median by
ook
Yo

(a) 4 (b) 8 (c)6 (d) 10


d7. If the mean of first n natural number is 15, then
eB

77 =

(a) 15 (b) 30 (c) 14 (d) 29


58. If the mean of observations x^, X2,..., a:,, is x, then the mean of 'a|+ a, X2+ a,. ..,
r

x,j + is
ou
ad

.Y
(a) ax (b) X ~ a (c) Y + a (d)
Y

o9. Mean of a certain number of observations is y . If each observation is divided by


Re
nd

m
{m ^ 0) and increased by 72, then the mean of new observationis
Fi

X n m
(a) ~
777
+ (b) - +m (c) Y + - (d) ^+-
n m n

-V,-25
60. If =
10 , If;Uj - 20, Tfi - 100, then x =
(a) 23 (b) 24 (c) 27 (d) 25
61. If 35 is removed from the data: 30,34,35,36,37,38,39,40, then the median increases bv
(a) 2 - ■ 1.5
(b) - - (c) 1 (d) 0.5
62. While computing mean of grouped data, we assume that the frequencies are
(a) evenly distributed over all the classes,
(b) centred at the class marks of the classes,
(c) centred at the upper limit of the classes,
(d) centred at the lower limit of the classes.
22.77
DATA INTERPRETATlON-l (CENTRAL TENDENCY)

63. In the formula X = <7 + //\N^


—Y/j' III' 1') finding the mean of grouped frequency
distribution Wj =
X: - a a - X:
X, + a
(b) h {Xj - rt) (c) (ci)
(a) h h h

64. For the following distribution;


5-10 10-15 15-20 20-25
Class: 0-5
15 12 20 9
10
Frequency:
the sum of the lower limits of the median and modal class is
(b) 25 (c) 30 (d) 35
(a) 15
65. For the following distribution;
20 30 40 50 60
Below; 10

w
12 27 57 75 80
Number of students; 3

F lo
the modal class is
(a) 10-20 (b) 20-30 (c) 30-40 (d) 50-60
66. Consider the following frequency distribution:

e
Fre
Class: 65-85 85-105 105-125 125-145 145-165 165-185 185-205
20 14 7 4
4 5 13
Frequency: for
The difference of the upper limit of the median class and the lower limit of the modal
class is
r
(b) 19 (c) 20 (d) 38
You

(a) 0
s
ook

S/i-rf,.
67. In the formula X = a + for finding the mean of grouped data fr,'® are deviations
eB

^fi '
from (7 of
(a) lower limits of classes (b) upper limits of classes
our
ad

(c) mid-points of classes (d) frequency of the class marks


68. The abscissa of the point of intersection of less than type and of the more than type
cumulative frequency curves of a grouped data gives its
dY

(d) all the three above


Re

(a) mean (b) median (c) mode


69. Consider the following frequency distribution;
Fin

6-11 12-17 18-23 24-29


Class: 0-5
10 15 8 11
13
Frequency;
The upper limit of the median class is
(b) 17.5 (c) 18 (d) 18.5
(a) 17

ANSWERS

2. Cb) 3. (a) 4. (b) 5. (b) 6. (b) 7. (d)


1- (b)
10. (c) 11. (d) 12. (a) 13. (d) 14. (d)
8. (a) 9. (c)
17. (b) 18. (c) 19. (c) 20. (c) 21. (d)
15. (c) 16. (b)
24. (a) 25. (d) 26. (b) 27. (d) 28. (c)
22. (b) 23. (b)
31. (a) 32. (d) 33. (a) 34. (c) 35. (c)
29. (a) 30. (c)
22.78
APPLIED MATHEMATICS-XI

36. (c) 37. (b) 38. (d) 39. (a) 40. (d) 41. (b) 42. (c)
43. (b) 44. (c) 45. (a) 46. (d) 47. (d) 48. (b) 49. (b)
50. (c) 51. (c) 52. (c) 53. (b) 54. (a) 55. (c) 56. (b)
57. (d) 58. (c) 59. (a) 60. (c) 61. (d) 62. (b) 63. (c)
64. (b) 65. (c) 66. (c) 67. (c) 68. (b) 69. (b)

FILL IN THE BLANKS QUESTIONS (FBQs)


n
1- If X represents the mean of n observations x x„, then the value of
IS /=!

2. If each observation of the data is increased by 5, then their mean is


by 5.

low
3. If the mean of the data x^,X2,. ..,x„ is X, then the mean of ax-^ + h,ax2-^b,....,ax„ + b
IS

4. If mode and median of the certain data are 3 and 3 respectively, then mean is
5. If Mode - Mean = 36, then Median - Mean =

ee
6. The mean and mode of a data are
rF 24 and 12 respectively, then median of the data

Fr
is

7. If the mean of 12 observations is 15. If two observations 20 and 25 are removed, then the
mean of remaining observations is
r
8. If Mean : Median = 2:3, then Mode : Mean = .
fo
u
9. The mean of a set of 12 observations is 10 and another set of 8 observations is 12. The
ks
mean of all 20 observations is
Yo
oo

10. The mean of the following distribution


x: 3 5 7 4
B

-r 2 a 5 b
re

is 5. Then the value of b is


n - If the mean of x, y, z is y, then mean of x and 2
u

is
ad
Yo

12. The mode of the data 2, j, 3,4,5,2,4,6 where x>2,i is

13. The mean of first 673 natural numbers is


nd

14. The mean of the observations 425,430,435,..., 495 is


Re

15. If themeanand medianofaunimodaldata


Fi

the data is
are
34.5 and 32.5 respectively, then mode of
16. The mean of first u odd natural numbers is
17. The mean of the observations 1, 3,5, 7,9,..., 99 is
18. If the mean of first n natural numbers is 20, then n =
19. If a mode exceeds a mean
by 12, then the mode exceeds median by
20. If the median of the observations Xj,X2,X;^,x^,x^, x^, Xj, .Vg is m, then the median of the

observations x^,x^,x^,x^ (where x^ <%2 <x^ <x^ <x^ <x^ <x^ <x^)is .
21. If mode-median = 2, then median - mean =
22. If the
average of a, b, c, d is the average of b and c, then the value oia-b-c + d is
22.79
DATA INTERPRETATION-1 {CENTRAL TENDENCY)

23. Given that a, b, c, d are non-zero integers such that a<b<c<d.li the mean and median of
. and b =
a, b, c, d are equal to zero, then a =
24. If a < b <2a and mean and median of a, b and 2a are 15 and 12 respectively, then
a =

25. If the mean of 26,19,15, 24 and x is then median of the data is


ANSWERS

4. 3 5. 12 6. 20
1. 0 2. increased by 5 3. aX + b

7. 13.5 8.5:2 9. 10.8 10. 6 11.1/ 12. 4 13. 337

ow
14. 460 15. 28.5 16. n 17. 2500 18. 39 19. 8 20. m

21. 1 22. 0 23.-d,-c 24. 11 25. 21

VERY SHORT ANSWER QUESTIONS (VSAQs)

e
Fl
re
1. If the ratio of mode and median of a certain data is 6:5, then find the ratio of its mean
and median.

F
2. If the mean of x -h 2,2x -h 3,3x -h 4,4x + 5 is x + 2, find x.
ur
or X
sf
3. If the median of scores and — (where x>0) is 6, then find the value of ,●
2 3 4 5 6 ^
k
Yo
oo

4. If the mean of 2,4,6,8, x, y is 5, then find the value ofx + ij.


B

5. If the mode of scores 3,4,3,5,4, 6,6, x is 4, find the value of x.


re

6. If the median of 33, 28,20,25,34, x is 29, find the maximum possible value of x.
7. If the median of the scores 1,2, x, 4,5 (where 1 < 2 < x < 4 < 5) is 3, then find the mean of
u
ad

the scores.
Yo

8. If the ratio of mean and median of a certain data is 2:3, then find the ratio of its mode
d

and mean.
Re
in

9. The arithmetic mean and mode of a data are 24 and 12 respectively, then find the
F

median of the data.


10. If the difference of mode and median of a data is 24, then find the difference of median
and mean.

11. Write the median class of the following distribution:


Classes: 0-10 10-20 20-30 30-40 40-50 50-60 60-70
4 4 8 10 12 8 4
Frequency:
12. What is the algebraic sum of deviations of a frequency distribution about its mean?
13. Which measure of central tendency is given by the x-coordinate of the point of
intersection of the 'more than' ogive and 'less than' ogive?
14. What is the value of the median of the data using the graph in the following hgure of less
than ogive and more than ogive?
22.80
APPLIED MATHEMATICS -XI

35 -●
g-
g 30 more than ogive
® 25 ●●

0)
●less than ogive
> 20 -
CD
5 15 ■ ●
£
5 10 -
5 ●

0
X
2 4 6 8 10 12 14 16 18 20
Marks —►

low
Fig. 22.2
15. Write the empirical relation between mean, mode and median.
16. Which measure of central tendency can be determined graphically?
17. Write the modal class for the following frequency distribution:

ee
Class-interval: 10-15 15-20 20-25 25-30 30-35 35-40
Frequency: 30 35 75 40
rF
Fr
30 15

18. A student draws a cumulative frequency curve for the marks obtained by 40 students of
a class as shown below. Find the median marks obtained by the students of the class.

r
Y fo
u
ks
Yo

U
oo

c
0) u>
3 C
ST
B

3
re

|o 40
is 30
3
u

u
20
ad
Yo

10

0 10 20 30 40 50 60 70 80
nd

upper limits of class intervals


Re

(Marks)
Fi

Fig. 22.3
19. Write the median class for the following frequency distribution:
Class-interval: 0-10 10 - 20 20 - 30 30 - 40 40 - 50 50 - 60 60 - 70 70 - 80
Frequency: 5 7 12 28 20 10 10
20. In the graphical representation of a frequency distribution, if the distance between mode
and mean is k times the distance between median and mean, then write the value of k.
21. Find the class marks of classes 10-25 and 35-55.

ANSWERS
1. 9:10 2.-1 3. 4 4. 10 5. 4
6. 30 7. 3 8. 5:2 9. 20 10. 12
11. 30-40 12. zero 13. Median 14. 4
15. Mode = 3 Median - 2 Mean 16. Median 17. 20-25 18. 50
19. 40-50 20. 3 21. 17.5,45
CHAPTER

DATA INTERPRETATION - II
(DISPERSION)

23.1 INTRODUCTION

In earlier chapters, we have learnt about methods of representing data graphically and in
tabular form. Such representations exhibit certain characteristics or salient features of the data.
We have also studied various methods of ftnding a representative value of the given data. This
value is called the central value for the given data and various methods for finding the central
value are known as the measures of central tendency. The measures of central tendency are:

w
mean (arithmetic mean), median and mode. We have learnt that the measures of central

F lo
tendency give us one single figure that represents the entire data i.e., they give us one single
figure around which the observations are concentrated. In other words, measures of central
tendency give us a rough idea where observations are centred. But the central values are
inadequate to give us a complete idea of the distribution as they do not tell us the extent to which

e
to make better interpretation from the
Fre
the observations vary from the central value. In order
data, we should also have an idea how the observations are scattered or how much they are
for
bunched around a central value. There can be two or more distributions having the same central
value but still there can be wide disparities in the formation of the distribution as discussed
below.
r
You

Consider following three distributions:


oks

(i) 1, 5, 9,13,17 (ii) 3, 6, 9,12,15 (iii) 7, 8, 9,10,11


eBo

In all these distributions we have the same number of observations and the same mean and
median both equal to 9. Therefore, if we are given that the mean of 5 observations is 9, we are
unable to say whether it is the average of first distribution or second distribution or third
ad
our

distribution.

Mean = median
Re

Distribution (i)
dY

0 2 5 9 13 17

Mean = median
Fin

Distribution (ii)
0 3 6 9 12 5

Mean = median
Distribution (iii)
0 7 8 9 10 11

Fig. 23.1

Let us now plot these distributions on a number line as shown below:


We observe that the dots representing observations in distribution (iii) are more close to each
other and are clustering around the mean and median (central value). So, we say that there is
more variability in the values of observations in distribution (i) in comparison to distributions
(ii) and (iii). We can also say that the distribution (iii) is more consistent than distributions
(i) and (ii).
23.2
APPLIED MATHEMATICS-XI

Let us now consider the runs scored by two batsmen and B2 in their last ten matches as given
below: ®

Match: 1 2 3 4 5 6 7 9 10
Batsman 30 91 0 64 42 80 30 5 117 71

rw
Batsman B2: 53 46 48 50 53 53 58 60 57 52
The mean and median of the scores are as under:
Mean Median
Batsman B 1 53 53

e
Batsman B2 53 53

e
We observe that the mean and median of the runs scored by both the batsmen Bj and B2 are
a::

lo
same. On the basis of this a natural question arises : Is the performance of two players same? The

r
answer is of course not in affirmative. Because the variability in the scores of batsman Bi is more

F
as he has scored runs from 0 (minimum) to 117 (maximum), where as the batsman B2 has scored

u
runs more consistently as the runs scored by him vary from 46 (minimum) to 60 (maximum). If

oF
the scores of batsmen 6j and B2 are plotted on a number line, we find that the points
representing scores of batsman B2 cluster around the central value (mean = median) while those

rs
corresponding to batsman B^ are scattered or more spread out.
it follows from the above discussion that the central values (mean, mode, median) are not

k
o
sufficient to give complete information about a distribution. VariabiUty in the values of the
observations of given data gives us better information about the data. So, variability is another
of
factor which is required to be studied in statistics. Like central value, we have a single number to
describe variability of a distribution. This single number is called the dispersion of the
o
Y
distribution and various methods of determining or measuring dispersion are called the
rY
B

measures >
of dispersion. In this chapter, we shall learn some of the important measures of
dispersion.
23.2 MEASURES OF DISPERSION
ue

As discussed above that the dispersion is the measure of variations in the values of the variable.
It measures the degree of scatteredness of the observations in a distribution around the central
od

value.
ad

Following are commonly used measures of dispersion:


in

(i) Range (ii) Quartile deviation (iii) Mean deviation (iv) Standard deviation.
In this chapter, we shall study all of these measures of dispersion except the quartile deviation.
Re
F

23.3 RANGE

RANGE The range is the difference between two extreme observations of the distribution.
If A and B are the greatest and smallest values respectively of observations in a distribution,
then its rang is A - B.
Thus,
Range of a distribution = Maximum value - Minimum value
In section 23.1, we have
Range of scores of batsman Bj =117-0=117
Range of scores of batsman B2 = 60 -46 = 14.
Clearly, the range of scores of batsman Bj is more than that of 63. Therefore, the scores of
batsman B^ are more scattered or dispersed while the scores are more close to each other for
batsman 82-
Range is the simplest but a crude measure of dispersion. As it is based upon two extreme
observations so it does not measure the dispersion of the data from a central value. Therefore

we
require some other measures of variability which depend upon the difference (or deviation)
23.3
DATA INTERPRETATION - II (DISPERSION)
mean deviation and
of the values from the central value. Such measures of dispersion are
standard deviation. Let us discuss them in detail.

23.4 MEAN DEVIATION


In this section, we will learn how to calculate mean deviation about mean and median for
various types of data.
23.4.1 MEAN DEVIATION FOR UNGROUPED DATA OR INDIVIDUAL OBSERVATIONS
Ifx-^, X2, ...,x,^aren values of a variable X, then the mean deviation from an average A (median or AM) is
given by
1
M.D. =
n
I \Xj -A\=- S|4|, where dj = .V;
1 = 1 n
/
-A

deviation of individual observations:


We may use the following algorithm to find mean

w
ALGORITHM

STI-I’l Compute the central value or average 'A' about which mean deviation is to be calculated.

F lo
Take deviations of the observations about the central value 'A' obtained in step I ignoring
± signs and denote these deviations by \ dj\.
STHIMII Obtain the total of these deviations i.e. S | dj\.

ee
Fr
sTi-.r IV Divide the total obtained in step III by the number of observations.

Following examples illustrate the procedure. for


ILLUSTRATIVE EXAMPLES
ur
EXAMPLE 1 Calcidate the mean deviation about median from the following data: 340,150, 210, 240,
s
ook

300,310,320.
Yo

SOLUTION Arranging the observations in ascending order of magnitude, we obtam


eB

150, 210, 240, 300, 310, 320, 340.


Clearly, the middle observation is 300. So, median = 300.
Calculation of Mean Deviation
our
ad

X;1 K-l=U-i-3oq
Y

340 40
Re

150
nd

150
210 90
Fi

240 60

300 0

310 10

320. 20.

Total d;=i:l.Y,--3oq = 370


370
M.D. = 1n ZK|=17 E|.r,-3001 = 7
= 52.8

EXAMPLE 2 The scores of a batsman in ten innings are■; 38, 70,48, 34,42, 55, 63,46,54,44. Find the
mean deviation about the median.
SOLUTION Arranging the data in ascending order, we obtain
34, 38,42, 44, 46,48, 54, 55, 63, 70
23.4
APPLIED MATHEMATICS-XI

Here n — 10. So, median is the A.M. of 5th and 6th observations
46 + 48
Median = = 47
2

Calculation of Mean Deviation


Xi I4-I=I-^/-47|
38 9
70 23
48 1
34 13
42 5

ow
55 8
63 16
46 1
54 7

e
44

re
3
Total

rFl L\dj\=8e

F
1 86
M.D. =
n
1141=:::::^
10
= 8.6

r
ou
EXAMPLE 3
Find the mean deviation from the mean for the data: fo 6,7,10,12,13,4,8,20
ks
SOLUTION
Let X be the mean of the given data. Then,
X
6 + 7+10 + 12 + 13 + 4 + 8 + 20
= 10
oo
Y

Computation of Mean Deviation


eB

Xi \di\=\xi-X\=\Xi-10\
r

6 4
ou

7 3
ad
Y

10 0
12 2
d

13 3
Re
in

4 6
8 2
F

20 10
Total
2 4 = 30
Now, 2|4 30 and n = 8
1 30
M.D.
-n ^141 = — = 3.75
8

Thus, the mean deviation from the mean for the given data is 3.75.
EXAMPLE 4 Calculate the mean deviation about the mean of the set offirst n natural numbers when n
IS odd natural number.

SOLUTION Since n is an odd natural number. Therefore, n = 2m +1 for some natural numberm.
Let X be the mean of first n natural numbers. Then,
X = 1 + 2 + 3 +...+ (?i-l) + n n(n + l) n+1
n 2n 2
23.5
DATA INTERPRETATION -11 (DISPERSION)
Im+1+1
X = = ?7I + 1
2

The mean deviation (M.D.) about mean is given by


M.D. = -n 2U-X1
r =l
2m + 1
1 [v /i = 2m + l]
M.D.=
2m +1 r=l
f 2jh+1

M.D.= L_i y |r-(7n + l)|+


2m + l

r =l r =771+1

ow
7/7 27/1+1

M.D.=
2771 + 1 r =l r =171+1
2m+1 2/71+1

1—J y r-(777 + 1) r=77i+


X 11 ●
1

M.D.= r +(777 + 1) y 1 +

e
2777 + 1 r=l r=l r=m+ 1

re
M.D.=
1 777 (777 + 1)

rFl+ 777(777 + 1) + ^ + 1) + (2777 + 1)} -(777 + 1)(777 + 1)

F
2t7i + 1 2 2 r

M.D.=
1
2
+ m (m +1) +12 (HI +1) (3m + 2) -(m +1)4J

r
ou
2/77 + 1

L^|!^!i^ + i(777 + l)(3777 + 2)-(777 +l)2|J fo


ks
M.D.=
7+l\ 2 2
777 + 1
|/77 + ( 3 777 + 2) - 2 (777 + 1)}
oo

M.D.=
2 (2777 +1)
Y

●77-lV7^-l
B

+ 1
777 + 1 777 (777 + 1) I 2 Jl 2 [V 77 = 2777 + 11
M.D.= (2777) =
re

2(2/77+1) 2/77 + 1 71

2-1
ou

1 r 7?-1 V 77 + 1'l
Y

77
ad

M.D. = -
n\ 2 A 2 J 4?1 _ ,
EXAMPLES Calculate the mean deviation about the mean of the set offirst n natural numbers when n
d

is even natural number.


natural number. Therefore, n = 2m for some natural number 777.
in
Re

SOLUTION Since 77 is an even


Let X be the mean of first /i natural numbers. Then,
F

_ 1 + 2+3+...+ 77 71(77 + 1) 77 + 1
X = 2
77 277
2/77 + 1 1 [v 77 = 2777]
X = = 777 + -
2 2

The mean deviation (M.D.) about mean is given by


M.D. = - X k-XI
77
r=l
2m
1
7/7+ —
n [■: n-2m + l]
M.D.=
2m r=l 2)
2m n
1
?77
lA
M.D.= Z ^ 777 + -
2)
+
I r- 777 + -
2)
2m r=l r =7/7+1
23.6
APPLIED MATHEMATICS-XI
m
1 1 2}n (

I f
M.D.= m + ~
n
+ m + -
2m 2j
Lr=l 2J
1 w m 2m 2m
M.D.= 1
m + -
2)
+
S - Z m +~
2)
y
r=m+ 1 r^m+l

1 in {m +1) 1 111
M.D = n
2m 2 + m m + -J+-{(m + l) + 2m} w + - m

w
2^
1
M.D = in (m +1) m(2m + l) ni(3m + l) w(2m + l)
2m 2 2 2 2

M.D= — (w + 1) m(3m + l) m

e
2m 2 2 4m
(-m-l + 3m + l)=- = " [v » = 2m]
2 4

ro
re
EXERCISE 23.1
1. Calculate the mean deviation about the median of the following observations:

F
(i) 3011, 2780, 3020, 2354, 3541, 4150, 5000
(ii) 38, 70, 48, 34, 42, 55, 63,46, 54, 44
(iii) 34, 66, 30, 38, 44, 50, 40, 60, 42, 51
Fl
u
sr
(iv) 22,24, 30, 27, 29, 31, 25, 28, 41, 42

ko
o
(v) 38, 70, 48, 34, 63, 42, 55, 44, 53, 47 of
2. Calculate the mean deviation from the mean for the following data:
(i) 4,7,8,9,10,12,13,17
o
Y

(ii) 13,17,16,14,11,13,10,16,11,18,12,17
erB

(iii) 38, 70, 48, 40, 42, 55, 63,46, 54, 44


uY

(iv) 36, 72, 46,42, 60, 45, 53,46, 51, 49


(v) 57, 64, 43, 67, 49, 59, 44,47, 61, 59
3. Calculate the
from their medians: deviation
mean
of the following income groups of five and seven members
do
ad

I
in

II
Income in ?
Income in ?
Re

4000
F

3800
4200
4000
4400
4200
4600
4400
4800
4600

4800
5800

4. The lengths (in cm) of 10 rods in a shop are given below:


/■^ i:- ^2.3, 55.2, 72.9, 52.8, 79.0,32.5,15.2, 27.9, 30.2
(i) rind mean deviation from median
(ii) Find mean deviation from the mean also.
5. In quesHon 1 (lii), (iv), (v) find the number of observations lying between X - M.D. and
A + M.D, where M.D. is the mean deviation from the mean.
23.7
DATA INTERPRETATION ■ II (DISPERSION)
ANSWERS

(iii) 8.7 (iv) 4.7 (V) 8.4


1. (i) 649.4 (ii) 8.6
(iii) 8.4 (iv) 7.2 (V) 74
2. (i) 3 (ii) 2.33
3. 320,457.14 4. (i) 16.4 (ii) 16.44 5. 6,5 and 6

23.4.2 MEAN DEVIATION OF A DISCRETE FREQUENCY DISTRIBUTION

w
If.v^/^,/=l,2,...,n is
is the frequency distribution, then mean deviation from an average A
(median or AM) is given by
n

M.D. = ^ -Al,where =N

e
deviation of a discrete frequency

e
We may use the following algorithm to find the mean

or
r
distribution.

F
ALGORITHM

Calculate the central value or average 'A' of the given frequency distribution about which
oF
ul
STKP 1
mean deviation is to be calculated.
value in step / ignoring signs and denote

sr
STl-P 11 Take deviations of the observations from the central

o
k
them by\dj\.
STEP 111 Multiply these deviations by respective frequencies and obtain of the total 1 f | (f\.
STl-P IV Divide the total obtained in step 111 by the number of observations i.e. N=-Lfto obtain the
o
Y
mean deviation.
rB

Following examples illustrate the above algorithm.


eY

ILLUSTRATIVE EXAMPLES
Calculate mean deviation about mean from the following data:
u

EXAMPLE!
17 23 27
3 9
X,-:
do
ad

12 9 5
10
fi-- 8
in

SOLUTION Calculation of mean deviation about mean.

lx,--151
Re

X; fi fi^i
F

12 96
3 24

6 60
9 10 90

2 24
17 12 204
72
23 9 207

12 60
27 5 135

N=lfi =44 I f X, = 660 1.x,-151=312

1 660
= 15
Mean = X - — {I.f Xj) - 44
N
312
= 7.09.
Mean deviation = M.D. = —
N
1 x, -151 = 44
23.8
APPLIED MATHEMATICS-Xl

EXAMPLE 2 Calculate the


mean deviation from the median for the following distribution:
Xi 10 15 20 25 30 35 40 45
7 3 5 6 8 4 9
SOLUTION We have to calculate mean deviarion about median. So, first we calculate median.
Xi
/● Cwnulativefrequency l4l=l->^/-30| ^141
10 7 7 20 140
15 3 10 15 45
20 18 10 80

ow
25 5 23 5 25
30 6 29 0 0
35 37 5 40
40 4

e
41 10 40

re
45

Fl 9 50 15 135

F
N= Ifj =50
2:.^ 141 =505
ur
Clearly, N = 50 => N/2 = 25.

r
The cumulative frequency just greater than N/2 is 29 and the corresponding value of .v is 30. fo
ks
Therefore, median = 30.
Yo
Clearly, If | j- - 30|= =505 and N =50.
oo

505
Mean deviation = — S = = 10.1
eB

N 50

r- , , . .u . — EXERCISE 23.2
ur

1.
Calculate the mean deviation from the median of the following frequency distribution:
ad
Yo

Heights in inches 58 59 60 61 62 63 64 65 66
No. of students 15 20 32 35 35 22 20 10 8
d
Re
in

2.
telephone calls received at an exchange in 245 successive one-minute
intervals are shown in the following frequency distribution:
F

Number of calls 0 1 2 3 4 5 6 7

Frequency 14 21 25 43 51 40 39 12

Compute the mean deviation about median.


3. Calculate the mean deviation about the
median of the following frequency distribution:
Xi 5 7 9 11 13 15 17
A 2 4 6 8 10 12 8

4. Find the mean deviation from the mean


for the following data:
(i) X/ 5 7 9 10 12 15
fi 6 2 2 2 6
23.9
DATA INTERPRETATION - II (DISPERSION)
2a 25
5 10 la
(ii) 3 5
4 6
7
fi
70 90
10 30 50
(iii) XL
16 8
4 24 28
fi
22 23 24
Size: 20 21
(iv) 4
5 1
6 4
Frequency:

5. Find the mean deviation from the median for the following data:
21 27 30
15
(i)
5 6 7
3

w
91 94 35
74 89 42 54
(ii) 3 4
12 2 4 5

Flo
20
1
answers

e
re
2. 1.49 3. 2.72
1. 1.703

F
(iii) 15.3 (iv) 0.32
4. (i) 3.38 (ii) 6.32
ur
r
5. (i) 5.93 (ii) 12.5 fo
23 4 3 MEAN DEVIATION OF A GROUPED OR CONTINUOUS FREQUENCY DISTRIBUTION
ks
Yo
oo
B

central value (median or mean).


re

Following examples will illustrate the procedure.


u

ILLUSTRATIVE EXAMPLES
ad
Yo

example 1 Find the mean deviation abont the median of the following frequency distribution:
24-30
12-18 18-24
0-6 6-12
d

Class: 5
Re

12 9
10
in

8
Frequency:
Calculation of Mean Deviation about the MMian
F

SOLUTION

Cumulative
Mid-Values lx,--141
Class (Xi) Frequency (f) frequency (c.f.)
8 11
0-6 3
5 50
10 18
6-12 9
1 12
12 30
12-18 15
7 63
9 39
18-24 21
13 65
5 44
24-30 27

N = I if =44 S:/-l:t,--141=278
23.10
APPLIED MATHEMATICS-XI
N
Here JV = 44,
so ^ 22 and the cumulative frequency just greater than — i
jisSO. Thus 12-18 is the
median class.
N/2-F
Median = / +
h, where/ = 12,/z =6,/ = 12,f =18.
X

/
22-18 4x6
Median = 12 + X 6=12 + = 14.
12 12
Clearly, -141=278
Mean deviation about median ^ .f\x- ~\4\ = 278
44
= 6.318

EXA.MPLE2 Calculate the


mean deviation from the median of the following data:

ow
Wages per week (in 10-20 20-30 30-40 40-50
No. of workers 4
50-60 60-70 70-80
6 10 20 10 6 4
SOLUTION
Calculation of Mean Deviation from Median

e
Wages per Mid-Values
Frequency Cumulative

re
week (in (Xi)

Fl fi frequency K-|=|A:y-45| ifldil

F
10-20 15 4 4
ur 30 120

r
20-30 25 6 10 fo 20 120
ks
30-40 35 10 20
Yo
10 100
oo

40-50 45 20 40 0 0
eB

50-60 55 10 50 10 100
ur

60-70 65 6 56 20 120
ad
Yo

70-80 75 4 60 30 120

N = If =60
d

^fi\di\ = 680
Re
in

Here,
N = 60. So, y = 30. The cumulative frequency just greater than Y = 30 is 40 and the N
F

corresponding class is 40-50. So, 40-50 is the median class.


/=40, / = 20, /?=10, F = 20.
So, N/2-F 30-20
Median = / + X /i = 40 + xl0=45.
/ 20
Thus, we have
^fi\Xj 45\~Xfj\dj\ = 680 and N = 60.
Mean deviation from median = — = 11.33.
N 60

EXAMPLE 3 Find the mean deviation from the mean


for the following data:
Classes : 10-20 20-30 30-40 40-50 50-60 60-70 70-80
Frequencies: 2 3 14 8 3 2
23.11
DATA INTERPRETATION ● II (DISPERSION)

SOLUTION Computation of Mean Deviation from Mean


Mid-value frequencies A';
I -XI
Classes fi^i = l.r,-45l
S Xj fi
2 30 30 60
10-20 15

3 75 20 60
20-30 25

280 10 80
30-40 35

14 630 0 0
40-50 45

8 440 10 80
50-60 55
60

ow
3 195 20
60-70 65

2 150 30 60
70-80 75

N = lfi=^0 lfi\xi-X\
= 1800 = 400

e
re
Clearly, N = 40 and 2 f Xj = 1800
S/- Xj _ 1^

Fl
F
X = 45
N 40
ur
From the above table, we get

r
2 fj\xj -X I =400 and N =2^; =40 fo
ks
400
M.D. = = 10.
Yo
40
oo

EXAMPLE 4 Find the mean deviation about the mean for the following data:
eB

10-20 20-30 30-40 40-50 50-60 60-70 70-80


Marks obtained:
14 8 3 2
2 3
Number of students: _
ur

SOLUTION In order to avoid the tedious calculations of computing mean(X), let us compute X
ad

by step-deviation method. The formula for the same is


Yo

(1 " , XI - a
= assumed mean and, h = common factor.
X = a+ h Y. f dj , where dj=- h
, a
d

N !=1 J
Re
in

Let us take the assumed mean a = 45 and h -10 and form the following table
F

.V;I - 45
Marks Number of Mid-points dj = I I
10
obtained students f A";/ = U,-45

-3 -6 30 60
10-20 2 15

-2 -6 20 60
20-30 3 25
-1 -8 10 80
30-40 8 35
0 0 0 0
40-50 14 45

1 8 10 80
50-60 55
2 6 20 60
60-70 3 65

3 6 30 60
70-80 2 75

^ tf I ^
N = 40 Ifdj^O = 400
23.12
APPLIED MATHEMATICS-XI

Clearly, N = 40, Ifi dj = 0.


- f I 0
X=a + h ~^fi^
N
= 45 + 10x— = 45.
40

It is evident from the table that | Xj ~X\ = 400


M.D. = 1 2^ |x,.-X| = 400
40
= 10.

_EXERCISE 23.3
I.
Compute the mean deviation from the median of the following distribution:
Class 0-10 10-20 20-30 30-40 40-50

ow
Frequency 5 10 20 5 10

“■ Find the mean deviation from the mean for the following data :
(i) Classes 0-100
100-200 200-300 300-400 400-500 500-600 600-700 700-800

e
Frequencies 4 8 9 10 7

re
5 4 3

(ii) Classes 95-105

rFl
105-115 115-125

F
125-135 135-145 145-155
Frequencies 9 13 16 26 30 12

r
(iii) Classes 0-10 10-20 20-30
ou
30-40 40-50 50-60
Frequencies 6 14
fo 16 4 2
ks
3- Compute mean deviation from mean of the following distribution:
oo

Marks 10-20 20-30 30-40 40-50 50-60 60-70 70-80


Y

80-90
eB

No. of students 8 10 15 25 20 18 9 5

The age distribution of 100 life-insuance policy holders is as follows:


r
ou
ad
Y

Age (on nearest


17-19.5 20-25.5 26-35.5 36-40.5 41-50.5 51-55.5 56-60.5 61-70.5
birth day)
No. of persons 5 16
d

12 26 14 12 6 5

Calculate the mean deviation from the median


Re
in

age.
Find the mean deviation from the mean
and from median of the following distribution:
F

Marks 0-10 10-20 20-30 30-40 40-50


No. of students 5 8 15 16 6

^ deviation about median age for the age distribution of 100 persons given
Age: 16-20 21-25 26-30 31-35 36-40 41-45 46-50 51-55

Number of persons 5 6 12 14 26 12 16 9

Calculate the mean deviation about the mean


for the following frequency distribution:
Class interval: 0-4 4-8 8-12 12-16 16-20

Frequency: 4 6 8 5 2
23.13
DATA INTERPRETATION - II (DISPERSION)

8. Calculate mean deviation from the median of the following data:


12-18 18-24 24-30
0-6 6-12
Class interval:

3 6 2
4 5
Frequency:

ANSWERS

(ui) 10.576 3. 14.218 4_ 10.605


1. 9 2. (i) 157.92 (ii) 12.005
5. 9.44,9.56 7. 0.99 8. 7.08

23.4.4 LIMITATIONS OF MEAN DEVIATION . .


Following are some limitations or demerits of mean deviation,
(i) In^a frequency distribution the sum of absolute values of deviations from *e mean s
always more *an the sum of the deviations from median. Therefore, mean deviation
ItaTZn is not very scientific. Thus, in many cases, mean devatton may g.ve

w
unsatisfactory results. ;c rw^i a
deviation about lugh,

F lo
(ii) In a distribution, where the degree of variability is very
medianthecalculated ^
tor sucn "
re
presentative central value. Thus, the mean

series can not be fully relied,


absolute values of deviations. Therefore, it

ee
(iii) In the computation of mean deviation we use

Fr
cannot be subjected to further algebraic treatment.
23.5 VARIANCE AND STANDARD DEVIATON
for
VARIANCE The varmce of a variate X is the arithmetic mean of the squares of all deviations of Xfrom
r
the arithmetic mean of the observations and is denoted b^Var(X) or a .
You

ThepositivesquarerootofthevarianceofavariateXisknoionasitsstanda rd deviation and is denoted


s
ook
eB

Thus, Standard deviation = + ^Var (X) and standard


Similar to the mean deviation, we shall discuss the calculation of variance
deviation in the following three cases;
our
ad

(i) Individual observations (ii) Discrete frequency distribution


(iii) Continuous or grouped frequency distribution.
dY

VARIANCE OF INDIVIDUAL OBSERVATIONS


Re

23.5.1

If X2,.--,x„ are n values of a variable X, then


Fin

II — j

Var(X) = -
1
I (.v,--X)2 or, o'
n
] i=X 1 (x,--X)“ ■
n
[' = 1

Now, Var(X) - -ji 1(j =I l (X/-X)^l = ^n ■ [1 =X 1 (x^ - 2x,-X + X )■


Var(X) = -n i =X l Xj^- i»/=1
X 2x, X + -n i=\
X X^
1 ” 9

1 ” T — 1 nX^
X Xi y +
Var(X) n /■ = 1 n

1 ”
X Xj =X
Var(X) = -M 1 =S 1 ,xi^-2X^+X^ n j = 1
23.14
APPLIED MATHEMATICS-XI

Var(X) = i £ -X^
n i=i

Var(X)=i S xf-ll
1 II II

Z X.-/
« J = 1 n i = i ●●●(ii)

^
tedious and ti™consummg.ta °f ™riance from the above formulae is quite
that case we take deviations
If4 = from an arbitrary point A (say).
n n n

I Xi - nA
/ = !

1 n
1 n

.^A - Z X:-A
n j = l
r « 1 = 1

1/ .^A
n

= X~A

w
n 1 = 1

1 n

F lo
=>
d =X - A, where d = ~ Z cL
N A
1 n

Var(X) = «- (x^ -X)2

ee
/ = l

Fr
Var{X) =- i (xj-A+A-Xf for
=>
Var(X) = 1 Z (dj-df
r
You
s

Var(X) = ”1 £ ^.2_1« 7 =£ 1 J
ook

« '=1 «,=:1
eB

Var(X) = 77- 7 =S 1 73^^ -2fl[77 7=P


Z 4')^ + nd^
n
our
ad

Var (X) = n1 7 =Z 1 -2 d^ + d'^


^2
dY

Var(X) = i Z 1 1 n
Re

n 7 = 1
s 42 -24
n 7 = 1 tt 7 = 1 ^
Fin

n2
Var (X) = 1 Z - 1 n
Thus, I di
« 7=1 yn i=\ ^ ...(hi)

Shnda°r7d^™"; *® '“‘’T °f individual observations, variance and


SoShtt? f° I for
Igorithm IS useful f”"rfindmg the variance when deviadons
°f *eareabove three formulas. Following
taken from the actual mean ®
ALGORITHM

STLIM Compute the mean X of the given obsen>ations Xi,X2,...,x„.


sriiP 11
Take the deviations of the observations from the mean i.e. find .ti- -X; 7 = 1,
«
STIiP in Square the deviations obtained i
in step II and obtain the sum 7 =Z 1 (Xj -X)^.
23.15
DATA INTERPRETATION - il (DISPERSION)

STKr IV Divide the sum i =I 1 {xj-Xf obtained in step III by n. This gives the value of variance of X.
Following examplos will illustrate the above algorithm.
ILLUSTRATIVE EXAMPLES

Ti/pe I ON FINDING VARIANCE AND STANDARD DEVIATION OF INDIVIDUAL OBSERVATIONS


EXAMPLE 1 Compute the variance and standard deviation of the following observations of marks of 5
students of a tutorial group:
Marks out of 25 : 8,12,13,15,22
SOLUTION Clearly,

w
8 + 12 + 13 + 15 + 22
X = 14
5

Flo
Calculation of variance
iXj-xf

e
Xi x^-X

re
-6 36
8

F
-2 4
12
-1 1
ur
13

r
1 1
15
22 8
fo 64
ks
I(x,--X)^=106
Yo
oo

Here, n = 5 and I(.v,-X)^=106


B

106
= 21.2
Var(X) =
re

S.D. = .JVar {X) = -Jill = 4.604.


u

EXAMPLE 2 Find the variance and standard deviation for the following data:
ad
Yo

65, 68, 58, 44,48,45, 60, 62, 60, 50


SOLUTION Let X be the mean of the given set of observations. Then,
- 65 + 68 + 58 + 44 + 48 + 45 + 60 + 62 + 60 + 50 _ 560
d

= 56
Re

X 10
in

10

Computation of Variance
F

Xj -X ~Xj -56
9 81
65
2 4
58
12 144
68
-12 144
44
-8 64
48
-11 121
45
4 16
60

62 6 36

60 4 16

-6 36
50
X(X-X)^=662
23.16
APPLIED MATHEMATICS-XI

Clearly, n=10 and S(o:,--X)^ = 662.


662
Variance = - Z{Xj-X)^ = 10 = 66.2 n

Hence, Standard deviation (ct) = VVariance = = 8.13


Ti/pe II ON PROVING RESULTS ON VARIANCE
EXAMPLES
Let Xj, X2, x^,, Xj, be n values of a variable X. If these values are changed to
Xi + a, X2 + a,..., x,j + a, where a e R, shozv that the variance remains unchanged.
SOLUTION Let Uj =Xj + a,i = l,2,...,nbe the n values of variable LI. Then,
II
1 1 ” 1
II
1
II
u
n ■
z ~ Z z X;I +na'i =
z Xi + a = X + a
1 = 1 » /= ] n
1 = 1 n :
/ = !

Ui-U = {Xj + a)~{X + a) = Xj -X, i = l, 2,..


II

w
=>
Z = Z (Xi-Xf
i = l f = 1

F lo
II
1 1
II

n .
Z n ■
Z
i = l 1 = 1

e
Var(L7) = Var(X).

Fre
EXAMPLE 4 tct ATj, X2,...., Xjj values of a variable X and let 'a' be a non-zero real number. Then, prove
that the variance of the observations ax^, ax2 is /
for
(X). Also, find their standard deviation.
SOLUTION Leti/i,H2,...,^,bethe»vaIuesofvariabIeUsuchthat(^- , / = 1,2,...,«. Then,
r
You
11
1 1
II
1
n

Z Z =
s

u a < -
Z f =
ook

n ■ n ,
/=! 1=1 n ,
/ = 1
eB

Uj - U = axj - aX for all / = 1, 2,.. ., n

-ti = a{xj-X)
our

(Uj-U)^ = a^(Xj~X)^
ad

Z a
^ Z (^/ -x)^
dY

/ = 1
Re

n
2 1
Fin

a
Z ■
” /=! n ●
1 = 1

Var(U) = a^ Var (X)


= V^r (U) = ^a^ Var (X) fliVvZTpO = U|ox-
liKMARK The variance of 20 observations is 5. If each observation is multiplied bu 2, then from the above
example,
New variance of the resulting observations = 2^x5 = 20

EXAMPLE 5 Letx^,x2,x^,..., X,, be n values of a variableX, and let Xj=a + hiij, i = 1, 2,..., n, where
UlrU2,...,i{,jarethevaluesofvariable U. Then, prove that Var (X) = }? Var(H), h 0.
SOLUTION We have,
Xj = a + h Uj , i = 1, 2,....,n
23.17
DATA INTERPRETATION - II (DISPERSION)
II

Z
i = l 1 = 1

II II

z = na +
h Z
1 = 1 i = l

*1
1

' Z ^
n ;
= a+ h - I
n :
1 = 1
1 = 1
_ -i n _ I "
X = a + hu V «,-=!
^;;Z
”i=l
^
X-I -X = {a + hUi)-{a + hU),i =l,2,...,n
= h{Ui-U\ij= h2 «
(Xi'Xf =
II

w
I (^ -X)2

F lo
1 = 1 1 = 1

[Dividing both sides by n]


1” Z(xi-X)2 =
n ” i=1

Var (X) = Var (U).
ree
for F
Type III ON FINDING THE DESIRED VALUES by using the formulas for mean and
VARIANCE OF INDIVIDUAL OBSERVATIONS
50 and 4 respectively. Find the
CXAMI’LF b If the mean and standard deviation of 100 observations
are
Your

sum of ali the observations and the sum of their squares.


ks

and standard deviation be X


SOLUTION Let Xi, X2,..Xaoo be 100 observations and their mean
eBoo

and o respectively. Then,


1 100 - 1
T -X and,
ad

X =
our

100 ,tl 100 i=l

100 ^
1 100 1 [v X =50 and a = 4]
50 = — Z^'
100
= 100
Re

1=1 i=l
Y

100 100

5000 = ^bOO = Z 2-250000


Find

X-
1

1 = 1 1=1

100 100 -1
Y,Xi =5000 and, = 251600
i=l
^^1 2
43, find the mean and
EXAMin.L7 Iffor a distribution of 18 observations I(a:,- -5) = 3 and I.{Xj -5)
Standard deviation.
SOLUTION We have
18 lo o
Y^{Xi -5) = 3 and, 2 -b) =^3
i=l «=1
18 18 18

V _ X 5 = 3 and, -10 + 125 = 43


: =1 1=1 '=1 '=1
23.18
applied MATHEMATICS-XI
18 18 18
-18 X 5 = 3 and, 10 2^.i--+18x25=43
/=! / = ! /=1
18 18

f = l
=93and, 2^7^-10x93 + 18x25=43
/=!

18 18
=93 and, =523
/=! /=1

1 Q'2

ow
7
' 1 18 18
523 1^93^^
I
S.D.= 9414-8649
Z-Vf 27.6586
18
/=1 18 118 324 18 18
= 1.536

Ii/fJi’ IV ON FINDING CORRECTED MEAN AND CORRECTED

e
VARIANCE OR S.D.
KXAMPU-;« For a group of200 candidates the

re
mean and S.D. werefound to be 40 and 15 respectively,
Later on it was found that the score 43

rFl
was misread as 34. Find the correct mean and correct S.D.

F
SOLUTION We have, n = 200, X = 40, cr = 15.
=> 2=»X =200 X 40 = 8000.

r
ou
Now,
Corrected Z .r,. = Correct Z .v; - (Sum of incorrect values) + (Sum of correct values) fo
ks
= 8000 - 34 + 43 = 8009.
Corrected mean = Corrected Z a:; _ ^ = 40.045
oo

n 200
and. <t = 15
Y
B

2
15 = Variance
1 1 ^2
re

15^ = (2-V/^)- IX;I


200 {200
ou

rSOOO'i^
Y

1
ad

225 =
200 200
1
225 =
(I.y,-2) -1600
d

200
in
Re

2 Xj^ = 200 X 1825 = 365000


Incorrect 2 Xi^ = 365000.
F

Corrected 2 =
;
(Incorrect 2 x,^) - (Sum of squares of incorrect values)
+ (Sum of squares of correct values)
Corrected 2 X,= 365000-(34)^ + (43)2 = 365693

So, Corrected CJ =
^” Corrected 2x,-2 J1{n Corrected 2x.- i
365693
200
8009^^
200
= 7^^28.465-1603.602 = 14 995

SOLUTION We have. - 20, X = 10 and a = 2.


1
X = - 2x,.1
n
23.19
DATA INTERPRETATION - II (DISPERSION)

I.Xi = nX = 20x10 = 200


Incorrected Z a:,- = 200.
and. a = 2
=4

- -(Mean)^ = 4
n

— Zx.^-100 = 4 [●.● Mean = 10]


t
20

Zx.^ = 104x20

ow
Incorrected I x-^ = 2080.
(i) When 8 is omitted from the data: If 8 is omitted from the data, then 19 observations are left.
Now, Incorrected Z Xj = 200

e
Corrected ZXj + S = 200

re
Corrected Z = 192

Frl
F
and.
Incorrected ^ = 2080
ou
r
Corrected Z x.^ +8^ = 2080
Corrected Z x-^ = 2080 - 64
so
I
kf
Corrected Z x^^ = 2016
oo
Y

Corrected mean =
Corrected ZXj _ 192 = 10.10
B

19 19

Corrected variance = — (Corrected Z - (Corrected mean)^


re

19
oY
u

2016 fl92^^ 38304 - 36864 _ 1440


ad

Corrected variance =
19 19 361 361
d

Corrected standard deviation = 1440 ^ 12ViO = 1.997.


in

V 361 ” 19
Re

(ii) When the incorrect observation 8 is replaced by 12:


F

Now, Incorrected Z = 200


Corrected Zxi -12 + 8=200
Corrected I-Xj = 200 -8 + 12 = 204
and. Incorrected £ a:- ^ = 2080
Corrected Zx-^ = 2080 - 8^ +12^ = 2160.
204
Now, Corrected mean = = 10.2
20

Corrected Variance = —
20
(Corrected Zx.' h - (Corrected mean)^

Corrected Variance =
2160 flOA'f
=>
20 I 20
23.20
APPLIED MATHEMATICS-XI

Corrected Variance =
2160x20-(204)^ 43200-41616 1584
(20)2 400 400

1584 19.899
Corrected standard deviation = = 1.9899
400 10 10

EXAMPLE 10 The mean and variance of 7 observations are 8 and 16 respectively. If 5 of the observations
are 2,4,10,12,14, find the remaining two observations.
SOLUTION Let x and y be the remaining two observations. Then,
Mean

2 + 4 + 10+ 12 + 14 +A-+ y = 8
7

42 + :n: + y = 56
x+y = 14 -(i)

w
and. Variance = 16

^ (2^ + 4^ +10^ +12^ +14^ + + /) - {Meanf = 16

i (4 + 16 + 100 + 144 + 196 +


F lo
+ y^) - 64 = 16

ee
460 + + y^ = 7 x 80

Fr
x^ + y^ = 100 ...(ii)
Now, (x + y)^ +{x-y)^ = 2(x^ +y^)
for
ur
196 + (x-yf = 2x100
(x-y)2 = 4
s
ook
Yo

x-y = ±2
eB

If^-y = 2, then a:+ y = 14 and x-y = 2 give x = 8, y = 6


Ifx-y = -2, thenx + y = 14 and x-y = -2 givex = 6,y = 8.
Hence, the remaining two observations are 6 and 8.
our
ad

Type V ON FINDINGTHE VARIANCE WHEN DEVIATIONS ARE TAKEN FROM AN ASSUMED MEAN

Following algorithm is helpful for finding the variances when deviations are taken from an
Y

assumed mean.
Re
nd

ALGORITHM

bliil’J Choose an assumed mean A (say).


Fi

STEI’il
Take the deviations dj of the observations from an assumed mean i.e. obtain
n

d^=Xj-A, i=l,2,...,n Take the total of these deviations i.e. Z dj.


i = 1

STi:iMll Square the deviations obtained in step 11 and obtain the total Z
; = 1

STHI* IV Substitute the values of j =1 1 d.^,


'
Z dj and n in the formula
f = 1

1 n \ r
1 « f
Var (X) = -
n
i j =Z i 4
y \n

EXAMPLE 11
The scores of a batsman in 10 matches zvere asfollozvs: 38,70,48,34,42,55,63,46,54,44
Compute the variance and standard deviation.
23.21
DATA INTERPRETATION - II (DISPERSION)

SOLUTION Let the assumed mean be A = 48.


Calculation of Variance

Xi d^=Xj-A
-10 100
38
22 484
70
0 0
48
-14 196
34
- 6 36
42
7 49
55
15 225
63
-2 4
46

w
6 36
54
-4 16
44

= 10, S4=14 and 2<f.^=1146


F lo
ldi=U ld,^=lU6

ee
Here, n

Fr
\ /
1 1146 14
= 112.64
Var (X) = -
-

n
j
10 10) for
Hence, S.D. = JVar (X) - V112.64 - 10.61
r
Tifpe VI ON FINDING VARIANCE AND STANDARD DEVIATION OF INDIVIDUAL OBSERVATIONS
You

Calculate the mean and standard deviation offirst n natural numbers.


s

EXAMPLE 12
ook

SOLUTION Here Xj = i; i=l, 2,..., n.


eB

LetX be the mean and a be the S.D. Then,


n(n + l) _« + l
I ,-=ii(l + 2+ 3 + ... + «) =
II
1 » 1
X I X:t = 2
2n
our
ad

n i = 1 n 1 = 1 n

A / 1 II

iJM f =S 1 ,v,-2 -(Xf


II

Now, a' X,-


i = 1 n 1 = 1
dY

n
y V
Re

2
o 1 -) ? 2,
2 = i(i2 + +... + >n ——
Fin

a
n K ^ J

=
n (« + l)(2n + l) + (n + l)(2n + l) (K + l)^_n^j2l
12
4
6n 2 J 6

n +1 n 2-1
Mean = and S.D. ——●
2 V 12
EXAMPLE 13 Find the mean and standard deviation offirst n terms of an A.P. whose ifrst term is a and
common difference is d.
SOLUTION The terms of the A.P. are: a, a+ d, a -hid, a + 3d,a + {r -1) d,a + {n 1) d.
LetX be the mean of these terms. Then,

X =-
n
+ + fr) +((7 + 2if)+---+('^ + ('1^1) ^)} “ -\la
2
+ («-l) d ● = a + {n-l) ^
Let a be the standard deviation of n terms of the A.P. then.
23.22
APPLIED MATHEMATICS-XI

n i2
2 1 II
a
n
Y, \a + {r-l) di -X \Jsmg:o^=-Y {Xi-X)'^
r=l 71
r =l

II -|2
2 1
a
n
Y ■ a + {r-l)d
r = l -{« +(«-!)
a

ow
*« r=i

a
2
il2r-(« + l)|2
Lr = l

e
^2 n

re
a
=—
4;i Z i 4^2-4 (n+l)r+(»
^ +1)2 ( .

Flr
F
2 I! tl II
a
4 -4(« + l) |;r +
L \r=1 ;
ou
l,r=l j r=l

sr
2_rf2 [4n(n + l)(2n+l) 4 (« +1) «(;; +1)
a
+ n (» + l)2
fo
4j? 6 2

2 _rf2 [2n(/n-l){2n + l) -??(??+ 1)2


k
oo
C7
4/j 3
Y

(«2 -1)
reB

2
a
= — «(« + !) {2 (2« + l)-3 (» + !)} 12
uY

«2-l
G =d
12
ad
do

EXERCISE 23.4
I ● Find the mean, variance and standard deviation for the following data-
(i) 2,4,5,6, 8,17. (ii)6,7,10,12,13,4, 8,12
in

(iii) 227,235,255,269,292,299,312,321,333,348. (iv) 15,22,27,11,9,21,14,9.


Re

2. The variance of 20 observations is 5. If each observation is multipHed by 2, find the variance


F

of the resulting observations.


3- The variance of 15 observations IS
i
4. If each observation is increased by 9, find the variance
of the resulting observations.
4. The mean of 5 observadons is 4.4 and their variance is 8.24. If three of the observations are 1,
2 and 6, find the other two observations.
The mean and standard deviation of 6 observations are
8 and 4 respectively. If each
observation is multiplied by 3, find the new mean and new standard deviation of the
resulting obser\-ations.
6. The mean and variance of 8 observations are
9 and 9.25 respectively. If six of the
observations are 6, 7,10,12,12 and 13, find the remaining two observations.
7- For a group of 200 candidates, the mean and standard deviations of scores were found to be
40 and 15 respectively. Later on it was discovered that the scores of 43 and 35 were misread
as
34 and 53 respectively. Find the correct mean and standard deviation.
23.23
DATA INTERPRETATION - II (DISPERSION)

8. The mean and standard deviation of 100 observations were calculated as 40 and 5.1
respectively by a student who took by mistake 50 instead of 40 for one observation. What
are the correct mean and standard deviation?
9. The mean and standard deviation of 20 observations are found to be 10 and 2 respectively.
On rechecking it was found that an observation 8 was incorrect. Calculate the correct mean
and standard deviation in each of the following cases:
(i) If wrong item is omitted (ii) if it is replaced by 12.

w
10. The mean and standard deviation of a group of 100 observations were found to be 20 and 3
respectively. Later on it was found that three observations were incorrect, which were
recorded as 21, 21 and 18. Find the mean and standard deviation if the incorrect
observations were omitted.

e
11. Show that the two formulae for the standard deviation of ungrouped data

re
ro
1 1
a =
Y,(Xi -X)^ and o' are equivalent, where X = - ^ x I ●

In n

F
ANSWERS

1. (i) 7,23.33,4.83 (ii) 9,9.25,3.04

F
ul (iii) 289.10,1539.77, 39.24 (iv) 16,38.68,6.22

sr
2.20 3.4 4.9,4 5.18,12 6. 4,8 7. 39.955,14.9

ko
o
8. Mean = 39.9 S.D. = 5 9. (i) 1.997 (ii) 1.98 10. 20,3.035

23.5.2 VARIANCE OF A DISCRETE FREQUENCY DISTRIBUTION


of
o
If X,' / fj; / = !, 2,..., n is a discrete frequency distribution of a variate X, then
Y
erB

Var (X) = —
N Ii
S= 1 f{x,-xf ■
uY

1
Also, Var (X) = —
N 1 = 1
S -2:c,-X+x2)
d
o
ad

1 II - 1 II
NX^
Var(X) = — -2X +
N
in

N / = 1 N I = 1

II
1 1
Z ifxf- -2X^+X^
Re

Var(X) = —
N / = r' ' N 1 = 1
F

ft
1
Var (X) = — -X2
N t=l ,
^2
I f n 9\ /
1 II
...(ii)
or. Var(X)=-
N
1 ifxf
=1 N j = l

If the values Xj of variable X or (and) frequencies^ are large the calculation of variance from the
above formulae is quite tedious and time consuming. In such a case, we take deviations of the
values of variable X from an arbitrary point A(say). li dj =Xj - A, i =1, 2,ii, then the above
formula reduces to
x2
1 ”
Var(X) =1N N t = l
...(hi)
23.24 APPLIED MATHEMATICS-XI

Sometimes dj=Xj-A are divisible by a common number h{say). If we define


^● = — h
= / =1/ 2,n, then we obtain the following formula for variance.
\ ( 2
1 n
1 «

Var (X) = ]? T7 ...(iv)


(N r = 1 i = 1

In order to compute variance by using the following formula


Var(X)=i
N
Z
f = 1
, we may use the following algorithm.

ALGORITHM

STHPI Obtain the given frequency distribution.


sn-:r ll Find the meanX of the given frequency distribution.
SlLPlll Compute deviations {Xj -X)from the mean X.

w
■"^ihpix Find the squares of deviations obtained in step III.

F lo
S11:P\ Multiply the squared deviations by respective frequencies and obtain the total Zf (.v,- -X)^.
^ 1 IT' VI Divide the total obtained in step V by N = Zf to obtain the variance.
Following example illustrates the above algorithm.

e
Fre
ILLUSTRATIVE EXAMPLES for
EXAMPLE 1
Find the variance and standard deviation of thefolloivingfrequen cy distribution:
r
Variable (xi) 2 4 6 10 12 14 16
You

Frequency (fi)
oks

4 4 5 15 8 5 4 5
eBo

SOLUTION
Calculation of Variance and Standard Deviation
Variable Frequency
fXi l-,-X=.V;-9
; i (Xf-Xf fiiXi-X)^
-Y,
A
our
ad

2 4 8 -7 49 196
4 4 16 -5 25 100
6 5 30 -3 9 45
dY
Re

8 15 120 -1 1 15
Fin

10 80 1 1 8
12 5 60 3 9 45
14 4 56 5 25 100
16 5 80 7 49 245

N= Zfi=50 Zf Xj =450 Zfi{Xj-Xf=754


Here, N = 50, ZfjXj= 450 and, Z f {Xj - X) ^ = 754
450
= 9
N 50

1 754
and, Var(X) =
N
● X^(.v,-X)2 50
= 15.08

Hence, S.D. = ^Var(X) = = 3.88


23.25
DATA INTERPRETATION - II {DISPERSION)

NOTE: Jjj practice the calculation ofS.D. and variance by the above algorithm is rarely used, because if
the actual mean is infractions the calculation is quite tedious a?id time consuming.
In order to compute the variance by using the following formula
2

Var(X)= , where dj =X{- A, we may use the following algorithm.


) vN

ALGORITHM
>!l i Take the deviations of observations from an assumed mean A(say) and denote these deviations
by dj.
STEP II
Multiply the deviations by the respective frequencies and obtain the total Ifdj.

ow
STEP 111 Obtain the squares of deviations obtained in step I i.e. d. .
STEP IV
Multiply the squared deviations by respective frequencies and obtain the total dj .
STEl' V Substitute the values in the formula
^2

e
re
\N

rFl
F
Following examples illustrate the above algorithm.
EXAMPLE 2 Calculate the variance and standard deviation from the data given below:

r
9.5
ou
7.5 8.5
Size of item 3.5 4.5 5.5 6.5
fo
ks
7 22 60 85 32
3
Frequency
oo

SOLUTION Let the assumed mean be A = 6.5


Y

Calculation of Variance and Standard Deviation


eB

Size of item f dj= Xi-6.5 dF fidi fA^


r

Xi
ou

27
ad
Y

3 -3 9 -9
3.5
4 -14 28
4.5 7 -2
d

1 -22 22
5.5 22 -1
Re
in

0 0 0
6.5 60 0
F

1 85 85
7.5 85 1

4 64 128
8.5 32 2

9 24 72
9.5 8 3

N=i:fi=2l7 Ifjdj =128 I, jfdj^ =362

Here, N = 217, .Ifidj =128 and I, f d^ = 362


1 , , 362 ri28^^
Var {XjJ^-Lfdl
N
— ,I jf dj
N
- 217 217
= 1.668-0.347 =1.321

Hence, S.D. = ^Var (X) = = 1349


23.26
APPLIED MATHEMATICS-XI

RtiMARK Sometimes deviations d, in the algorithm given above are divisible b^ a common number h. In
such a case, we define Uj = ^ i_h ’ i = l,2,...,n and the formula for computmg variance is

Var(X) = h^ if I 1 n

— Z fiL .
N [i= I N /=!' ^

EXAMPLE 3 Find the variance and standard deviation for the follozving distribution:
X: 4.5 14.5 24.5 34.5 44.5 54.5 64.5

ow
/: 1 5 12 22 17 9 4

SOLUTION
Calculation of Variance and Standard Deviation

4 = Xj - 345 X/ - 345

e
Xi “i =

re
10

rFl
F
4.5 1 -30 -3 -3 9 9

14.5 5 -20 -2 -10 4 20

r
ou
24.5 12 -10 -1
fo-12 1 12
ks
34.5 22 0 0 0 0 0
44.5 17 10
oo

1 17 1 17
Y

54.5 9 20 2 18 4 36
eB

64.5 4 30 3 12 9 36
r

N= Zf =70 I.fiUi=22 lfiU,^=130


ou
ad
Y

Here, N=70, Zfuj=20, ZfjU,^=m and /j=10


d

1 1 ^2
Var (X) = /?2
Re
in

{N N
F

Var (X) = 100


130 ^221 = 100
13 nil
70 70 7 35
= 100 [1.857 - 0.098] = 175.822

Hence, S.D. = ^ar (X) = Vl75.822 = 13.259


EXAMPLE 4
The following table gives the number offinished articles turned out per day by different
number of workers in a factory. Find the standard deviation of the daily output offinished articles.
Number of articles: 18 19 20 21 22 23 24 25 26 27

No. of workers: 3 7 11 14 18 17 13 8 5 4
DATA INTERPRETATION ■ II (DISPERSION) 23.27

SOLUTION Calculation of Standard Deviation


23
X
f fidi
18 3 -5 25 -15 75
19 7 -4 16 -28 112

20 11 -3 9 -33 99
21 14 -2 4 -28 56
22 18 -1 1 -18 18
23 17 0 0 0 0
24 13 1 1 13 13
25 8 2 4 16 32
26 5 3 9 15 45
27 4 4 16 16 64

w
\ =100 I Ifjdj =-62 1^4-^ =514
Clearly, N =100, .Ifidj =-62 and 1^4^=514
a'
N N
f 514
100

F lo 62 f
100
47556
10000

ee
Fr
47556 218.07
Hence, o = 2.1807
10000 100
EXAMPLE
If a is a positive integer and the frequency distribution:
for
ur
a: ; a 2fl 3a 4a 5a 6a
s

1 1 1
/: 2 1 1
ook
Yo

has a variance of 160. Determine the value of a.


eB

SOLUTION Computation of Variance

fi f^i
our
ad

a 2 la 2.2
4.2
Y

2. 1 2.
Re

3. 1 3. 9.2
nd

4. 1 4. 16.2
Fi

5. 1 5. 25.2
6. 1 6. 36.2

N=S4=7 IfjXj = 22. IfXi^ = 92.2


Here, N =7, IfjXj -11a, IfjXj^ =91a^ and Variance = 160
Now,
Variance = 160
n2
160=f —I/a-,-2
1

N } \ N

92.2 22.^2 => 644.2 _454^2 160.2


160 = 160 = 160 = . 2 =49=> .=7
7 7 ) 49 49
23.28 APPLIED MATHEMATICS-Xl

EXAMrLi- 6 There are 60 students in a class. The following is the frequency distribution of marks
obtained by the students in a test:
Marks: 0 1 2 3 4 5

Frequency: x-2 A 2a- A+1

ivhere x is a positive integer. Determine the mean and standard deviation of the marks.
SOLUTION It is given that there are 60 students in the class.
(a-2) + a + a^ +(a + 1)^ +2a + (a + 1) =60
2a^+7a-60=0
(2a+ 15) (a-4) =0

ow
a-4 = 0
[v A >0 2a + 15 0]
a =4

Thus, we obtain the following frequency distribution:

e
Marks: 0 1

Fl 2 3 4 5

re
F
Frequency: 2 4 16 25 8 5
ur
Computation of mean and standard deviation

r
Marks (a,-) Frequeiicy (fj) fiXi
fo
ks
Yo
0 2 0 0
oo

1 4 4 4
B

2 16 32 64
re

3 25 75 225
u
ad
Yo

4 32 128

5 5 25 125
d
Re
in

N=Z/- =60 IfjXj = 168 IfjXj^ =546


F

Here, N = 60, TfiXj = 168, S^a,2 =546


168
Mean = = 2.8
N 60

and. Variance =
(I

, 'i n ^ 546 n68^^ = 9.1-7.84=1.26
In / \ N 60 60

Hence, S.D. = VVariance = Vl.26 = 1.122


EXERCISE 23.5

1 ■ Find the standard deviation for the following distribution:


A : 4.5 14.5 24.5 34.5 44.5 54.5 64.5
/: 1 5 12 22 17 9 4

2. Table below shows the frequency /with which 'a' alpha particles were radiated from a
diskette
23.29
DATA INTERPRETATION - II (DISPERSION)

X: 0 1 2 3 4 5 6 7 9 10 11 12

51 203 383 525 532 408 273 139 43 27 10 4 2

Calculate the mean and variance.

3. Find the mean, and standard deviation for the following data;
10 20 30 40 50 60
(i) Year render:
15 32 51 78 97 109
No. of persons (cumulative):

2 3 4 5 6 7 8 9 10 11 12 13 14 15 16
(ii) Marks:
1 6 6 2 2 3 0 2 1 0 0 0 1
Frequency:
4. Find the standard deviation for the following data;
8 13 18 23
(i) x: 3

w
7 10 15 10 6
/:
4 5 6 7

F lo
(ii) .Y: 2 3

9 16 14 11 6
4

ANSWERS

ee
Fr
1. 13.26 1.x =3.88, a^=3.64 3. (i) X = 37.25 years, S.D. = 15.5 years, (ii) X =5.975,
S.D. = 2.85. 4. (i) 6.12 (ii) 1.38 for
ur
23.5.3 VARIANCE OF A GROUPED OR CONTINUOUS FREQUENCY DISTRIBUTION
In a grouped or continuous frequency distribution any of the methods discussed above for a
s

discrete frequency distribution can be used. We may use the following algorithm for computing
ook
Yo

variance of a grouped or continuous frequency distribution.


eB

ALGORITHM

STEPl Find the mid-points of various classes.


our
ad

STEPIl Take the deviations of these mid-points from an assumed mean. Denote these deviations b\j dj.
, ■ '4
STEP III Divide the deviations in step II by the class interval h and denote them b\j r.e. Uj - —.
Y
Re

Multiply the frequency of each class with the corresponding Uj and obtain I f Uj.
nd

j\-
Square the values of Uj and multiply them with the corresponding frequencies and obtain
Fi

STEP V

STEP VI Substitute the values of .I f Uj, 2 f h and N = -Yfj in the formula


f 1
Var(X) = i
N VN y

Following examples will illustrate the above algorithm.


ILLUSTRATIVE EXAMPLES
EXAMPLE 1 Calculate the mean and standard deviation for thefollowhig distribution:
30-40 40-50 50-60 60-70 70-80 80-90
Marks: 20-30

6 13 15 14 5 4
No. of students: 3
23.30
APPLIED MATHEMATICS-XI

SOLUTION
Calculation of Standard Deviation

Frequency Mid-values X;I -55


Class-interval Ui =
(fit (Xi) 10

20-30 3 25 -3 -9 9 27

30-40 6 35 -2 -12 4 24

40-50 13 45 -1 -13 1 13

50-60 15 55 0 0 0 0

60-70 14

ow
65 1 14 1 14

70-80 5 75 2 10 4 20

80-90 4 85 3 12 9 36

e
N = = 60
.Ifu. =2 =134

re
Here, N=60, TfiUj=2, lfu.^=134 and /i=10
rFl
F
Mean = X = A + h = 55 + 10
f 2 ^
= 55.333

r
N 60
ou
and. Var(X) = h^\(- Ifu^ ---
1
If Uj
n2 fo
^ = 100 134 ( 2']^ = 222.9
60 Is
ks
{N / V N

S.D. = fVar (X) = V222.9 = 14.94.


oo

EXAMPLE 2 The following table gives the distribution of income of 100 families in a village. Calculate
Y
eB

the standard deviation:

Income ? 0-1000 1000-2000 2000-3000 3000-4000 4000-5000 5000-6000


ur

No. of Families 18 26 30 12 10 4
ad
Yo

SOLUTION Calculation of Standard Deviation


d

Mid-values No. offamilies X:i - 2500


Income ?
fUi uF fi4
Re

(frequencies) fi
in

Xi 1000

0-1000 500 18 -2 -36 4 72


F

1000-2000 1500 26 -1 -26 1 26


2000-3000 2500 30 0 0 0 0
3000-4000 3500 12 1 12 1 12
4000-5000 4500 10 2 20 4 40
5000-6000 5500 4 3 12 9 36

I f =100 I^t^-=-18 i: f III =186


Here, W=100, Zfui=~ 18, .1 f III =186 and, /i=1000
/ \
n2
2 186 r-i8A^
^ =(1000)^ \ ● =1827600
100 100
V /

Hence, S.D. = fVar (X) = VTS7600 = 1351.88


DATA INTERPRETATION - II (DISPERSION) 23.31

EXAMPLE 3 Calculate the mean and standard deviation for the follozving data:
60 75 90 105 120
Wages upto (in f) 15 30 45
107 157 202 222 230
No. of workers 12 30 65

SOLUTION We are given the cumulative frequency distribution. So, first we will prepare the
frequency distribution as given below:
Class- Cumulative X; - 675
Mid-values Frequency
inteival frequency 15

0-15 12 7.5 12 -4 ^8 192

15-30 30 22.5 18 -3 -54 162

30-45 65 37.5 35 -2 -70 140

w
45-60 107 52.5 42 -1 -42 42

60-75 157 67.5 50 0 0 0

75-90 202 82.5 45 1 45 45

Flo
90-105 222 97.5 20 2 40 80

105-120 230 112.5 3 24 72

e
Ifu^ =733

re
I_/J =230 Ifj Uj = -105

F
Here, A =675, /i=15, N=230, I^h-=-105 and f =75?>
( \ -105
= 675 - 6.85 = 60,65
ur
r
Mean - A + h — 71 f Uj = 675 + 15
230
N fo
9 1 7 f 1
i-N ^fi^j ■
ks
and, Var (X) =
Yo
oo

733 r-105^^
Var(X) = 225- ● = 225(3.18-0.2025) = 669.9375
B

230 230
re

S.D. = .y Var (X) = V669.9375 = 25.883


EXAMPLE 4 The measurements of the diameters (in mm) of the heads of 107 screws are given below:
u
ad
Yo

Diameter (in mm) 33-35 36-38 39-41 42-44 45-47

19 23 21 17
No. of screws 17
d
Re
in

Calculate the standard deviation.

SOLUTION Here the class intervals are formed by the inclusive method. But, the mid-points of
F

class-intervals remain same whether they are formed by inclusive method or exclusive method.
So there is no need to convert them into an exclusive series.

Calculation of Standard Deviation


Diameter (in No. of screws oq -40
Mid-values Xi Ui = ftn fi4
mm) k 3

33-35 34 17 -2 -34 68

36-38 37 19 -1 -19 19

39-41 40 23 0 0 0

42-44 43 21 1 21 21

45^7 46 27 2 M 108

I if =107 TLf 1^=216


23.32 APPLIED MATHEMATICS-XI

Here N= 1^-107, S/^--22, .Yfiil=2\e, 2i=40and, h=3


^2
n 216 22
Var(X) -
N / V N
ZfiU^ =9j 107 107

Var(X) = 9(2.0187 -0.0420) = 9x1.9767 = 17.7903


S.D. = V17.7903 = 4.2178.
EXAMPLES
Calculate the mean and standard deviation for the following table given the age distribution
of a group of people:

ow
Age: 20-30 30-40 40-50 50-60 60-70 70-80 80-90

No. of persons: 3 51 122 141 130 51 2

SOLUTION Here A=55,h= 10.


Calculation of Mean and Standard Deviation

e
re
Mid-values Number of X;I -55
Age

Frl
=

(Xi) persons if)

F
10

20-30 25 3 -3 -9 9 27

30-40 35
ou
51 2 -102

or
4 204

40-50 45 122 1 -122 1 122


kfs
50-60 55 141 0 0 0 0

60-70 65 130
oo
1 130 1 130

70-80 75 51 2 102 4 204


Y
B

80-90 85 2 3 6 9 18

N = Zf =500 I^i^’2=705
re

Z^J4=5
oYu

Here, N = .If = 500, Ifuj = 5 and, IfUj^ = 705


ad

X = A + h(—N Zfuj]^55 + 10 5 ^
= 55.1
d

500
in

N /
Re

1
and. CT 2 = hU IfUi ^
N J \N
F

705 100
G ^ = 100■ (70500 - 5) = 70495 ^ 14099
500 I500, 50000 500 ” 100

V14099 118.739
CJ = 11.8739.
10 10
EXERCISE 23.6
1. Calculate the mean and S.D. for the following data:
Expenditure (in ?): 0-10 10-20 20-30 30-40 40-50

Frequency: 14 13 27 21 15

2. Calculate the standard deviation for the following data:


Class: 0-30 30-60 60-90 90-120 120-150 150-180 180-210

Frequency: 9 17 43 82 81 44 24
23.33
DATA INTERPRETATION - II (DISPERSION)

3. Calculate the A.M. and S.D. for the following distribution:


Class: 0-10 10-20 20-30 30-40 40-50 50-60 60-70 70-80
15 12 10 5 2 1
18 16
Frequency:
4. A student obtained the mean and standard deviation of 100 observaHons as 40 and 5.1
respectively. It was later found that one observation was wrongly copied as 50, the correct
figure being 40. Find the correct mean and S.D.
5. Calculate the mean, median and standard deviation of the following distribution:
Class-interval: 31-35 36-40 41-45 46-50 51-55 56-60 61-65 66-70
3 8 12 16 5 2 3
2
Frequency:
6. Find the mean and variance of frequency distribution given below:
l<x<3 3 <x <5 5<a;<7 7 <x<10
Xj-.
6 4 5 1
h--
7. The weight of coffee in 70 jars is shown in the following table:

w
200-201 201-202 202-203 203-204 204-205 205-206
Weight (in grams):
1 1

F lo
13 27 18 10
Frequency:
Determine the variance and standard deviation of the above distribution.
Mean and standard deviation of 100 observations were found to be 40 and 10 respectively.

ee
If at the time of calculation two observations were wrongly taken as 30 and 70 in place of 3

Fr
and 27 respectively, find the correct standard deviation.
9 While calculating the mean and variance of 10 readings, for a student wrongly used the
rea
ding of 52 for the correct reading 25. He obtained the mean and variance as 45 and 16
respectively. Find the correct mean and the variance.
r
10 Calculate mean, variance and standard deviation of the following frequency distribution.
You
s

40-50 50-60
ook

Class: 1-10 10-20 20-30 30-40


29 18 4 5 3
11
Frequency:
eB

ANSWERS
our

3. AM =26.01, S.D. = 17.47


ad

1. X =26.11, 0=12.86. 2. X =118.7,0 = 4251.

4. X =39.9,0=5. 5. X =50.35, 0=7.94, median =50.65


6. Mean =55, Variance = 4.26 7. Variance = 1.16 gm, S.D. 1.08 gm
dY
Re

8. 10.24 9. Mean = 42.3, Variance = 43.81 10. Mean = 21.5, Variance = 161, S.D. = 12.7
Fin

23.6 ANALYSIS OF FREQUENCY DISTRIBUTIONS


In this section, we shall see how we can use various measures of dispersion to compare two or
more series. In the earlier secHons of this chapter we have seen that the mean deviation and
standard deviation have the same units in which the data are given. Therefore, measures of
dispersion are unable to compare two or more series which are measured in different units even
if they have the same mean. Thus, we require those measures which are independent^ c of ■ the
units. The measure of variability which is independent of units is called coefficient of variation
(C.V.) and is defined as
C.V. = = X 100, where a and X are the standard deviation and mean of the data.
X

For comparing the variability of two series, we calculate the coefficient of variation for■ each
.
series. The series having greater C.V. is said to be more variable or conversely less consistent,
^

less uniform, less stable or less homogeneous than the other and the series havmg lesser C.V. is
said to be more consistent (or homogeneous) than the other.
23.34
APPLIED MATHEMATICS-XI

Let there be two frequency distributions with standard deviations and 03 and equal meanX.
Then,

C.V. (1st distribution) = -5:


X
x 100 and, C.V. (2nd distribution) =
02
^xlOO
X

51 xlOO
C.V. (1st distribution) - X
C.V. (2nd distribution) X 100 ^2
X

This means that the two distributions can


be compared on the basis of the values of their
standard deviations oj and 03 only.
Thus, if two series have equal means then the series with greater standard deviation (or
variance) is said to be more variable or dispersed than the other. Also the series with lesser value

w
of the standard deviation (or variance) is said to be more consistent than the other.
ILLUSTRATIVE EXAMPLES

F lo
EXAMPLE 1 An analysis of monthly luages paid to the workers of two firms A and B belonging to the
same industry gives thefollozuing residts:

ee
Firm A Firm B

Fr
Number of workers 1000 1200

Average monthly wages ^2800 ^2800


for
Variance of distribution of wages
ur
100 169

In which firm, A or B is there greater variability in individual wages?


s
SOLUTION We observe that the average monthly wages in both the firms is same i.e. Rs. 2800.
ook
Yo

Therefore, the firm with greater variance will have more variability. Thus, firm B has greater
variability in individual wages.
eB

EXAMPLE 2 An analysis of monthly zuages paid to workers in tzvo firms A and B, belonging to the same
industry, gives the follozoing residts:
r

Firm A Firm B
ou
ad

No. of zuage earners 586 648


Y

Mean of monthly zvages ^5253 ^5253

Variance of the distribution of zvages 100 121


Re
nd

(i) Which ifrm Aor B pays out larger amount as monthly zvages?
Fi

(ii) Which firm AorB shozus greater variability in individual wages?


SOLUTION (i) Firm A:
Number of wage earners (say) = 586
Meanof monthly wages (say) Xj = ^5253
Total monthly wage
Mean of monthly wages =
Number of workers

5253 =
Total monthly wages
586

Total monthly wages = ? (5253 x 586) = ? 3078258


Firm B:

Number of wage earners (say) U2 = 648


Mean of monthly wages = ? 5253
23.35
DATA INTERPRETATION - II (DISPERSION)

Total monthly wages


Mean of monthly wages = Number of workers

Total monthly wages


5253 =
648

Total monthly wages = ? (5253 x 648) = ^ 3403944


Clearly, firm B pays out larger amount as monthly wages,
(ii) Since firms A and B have the same mean. Therefore, the firm with greater variance will have
more variability individual wages.
Clearly, Variance of firm B > Variance of firm A.
Hence, firm B will have greater variability in individual wages.
EXAMPLE 3 The folloiuing values are calculated in respect of heights and zveights of the students of a
section of class XI:

w
Height Weight
Mean 162.6 cm 52.36 kg

F lo
Variance 127.69 cm^ 23.1361 kg^
Can zue say that the zveights shoiv greater variation than the heights?
SOLUTION In order to compare the variability of height and weight, we have to calculate their

ee
and 02 denote the standard deviations of height and weight

Fr
coefficients of variation. Let
respectively. Further, let for
and X2 be the mean height and weight respectively.
We have,
r
X2 = 52.36
You

X^ = 162.6,
s

^ = 127.69 2^ = 23.1361
ook

o and cj
1

= fV27^ = 11.3 and = V23.13^ = 4.81


eB

02
Now,
11.3
Coefficient of variation in heights =
_

xl00 = X 100 = 6.95


our
ad

X1 162.6

and.
_ ^2 xlOO = 4.81
X 100 = 9.18
Coefficient of variation in weights =
dY

52.36
X2
Re

Clearly, coefficient of variation in weights is greater than the coefficient of variation in heights.
Fin

So, weights shows more variability than heights.


EXAMPLE4 The sum and sum of squares corresponding to length x (in cm) and zveighty (in gm) of 50
plant products are given belozv:
50 50 50 2
S 'X1 = 212, S r. ^ = 902.8, i =I l xji = 261,
I
1
= 1457.6
/■ = 1 t = l

Which is more varying, the length or zveight?


SOLUTION We have.
50 50
Z XI = 212 and I X
I
^ = 902.80
f = 1 j = l

50
I XI ^2
i=l 2 I f 50 2 1 50
I
X and o Z X. Xi
X ~ 50 i
50 i = 1 J ^50 /.I
23.36 APPLIED MATHEMATICS-XI

212 2 902.80 212


X and a
X
50 50 50

X = 4.24 and <7


X ^ = 18.056-(4.24)2 ^ 18.056-17.9776 = 0.0784
X = 4.24 and dX = VO.0784 = 0.28
It is given that

w
50 50

/=!
S y, = 261 and
I1
S u/ = 1457.6
= 1

50
2:, y,

e
1 = 1 50 50 ^2
2 1 1
y and a
2:. y/

re
Y

ro
50 50
7 V 50 1=1

Y
261
and 2 1457.6 f261)^

F
G
50 y
50 50

F
ul
Gy = 29.152-(5.22)2 = 1.9036
y = 5.22 and

y = 5.22 and cry = 1.3797

sr
In order to determine the variability of length and weight, we will have to compute the

ko
o
coefficients of variations in lengths and weights. of 0.28
Coefficient of variation in lengths = ^xl00 = X 100 = 6.60
X 4.24
o
Y
1.3797
Coefficient of variation in weights = ^ x 100 = X 100 = 26.43
erB

y 5.22
uY

Clearly, coefficient of variation is weights is greater than the coefficient of variation in lengths.
Hence, weights have more variability than lengths.
EXAMPLES
The following is the record of goals scored by team A in football session.
d
o

0
ad

Nujnber of goals scored: 1 2 3 4

Number of matches: 1 9 7 5 3
in

For the team B, mean number of goals scored per match was 2 with a standard deviation 1.25 goals. Find
which team may be considered more consistent?
Re
F

SOLUTION In order to determine the consistency of teams we will have to find the coefficients
of variations of two teams.

Computation of mean and standard deviation of goals scored by team A.

No. of goals scored Xj r 2


No. of matches f fi^ fi^i
0 1 0 0
1 9 9 9
2 7 14 28
3 5 15 45
4 3 12 48

Zf = 25 Zfxi =50 X. 2 ^ 130


We have.
N
= Zf = 25, ZfXi = 50and Zfx.^ = 130
23.37
DATA INTERPRETATION ■ II (DISPERSION)

50
XA _ = 2
N 25

2 ^ 130 {50 = 5.2-4 = 1.2


and, a
A 25 25

= Vu = 1.095
It is given that Xg = 2andag - 1.25
Now, 1.095
Oa X 100 = X 100 =54.75
Coefficient of variation in goals scored by team ^ 2
Xa
1.25
X 100 = 62.50
Coefficient of variation of goals scored by team B -^
Xe
xl00 =
2

lesser than that of team


We observe that the coefficient of variation of goals scored by team A is
B. Hence, team A is more consistent.
EXAMPLE 6 Suppose that samples of polythene bags from two manufacturers, A and B, are tested by a

w
prospective buyerfor bursting pressure, ivith the following result^:
Bursting Pressure in kg

F lo
Number of bags manufactured by manufacturer
A

2
B

e
5-10

Fre
9 11
10-15 for
29 18
15-20
54 32
20-25
r
11 27
You

25-30
oks

5 13
30-35
eBo

Which set of the bags has Ihc highest average bursting pressure? Which has more uniform pressure?
SOLUTION For determining the set of bags having higher average bursting ^
compute mean and for finding out set of bags having more uniform pressure we compu
ad
our

coefficient of variation.
Manufacturer A:
Computation of mean and standard deviation
dY
Re

Mid-values
xT^\75
Bursting
I

f
Fin

X;I 5
pressure_
-2 -4 8
5-10 7.5 2

-1 -9 ■ 9
10-15 12.5 9

0 0 0
15-20 17.5 29

1 54 54
20-25 22.5 54
2 22 44
25-30 27.5 11

3 15 45
30-35 32.5 5

N=£^=110 S ti,- = 3 S fi Uj =78

X^ = a+ h
N J
23.38
APPLIED MATHEMATICS-XI

78
=> = 17.5+5X = 175 + 35 = 21
110 [●●● h ^5, a = 175]
1 f ^ ^2 160 78 17600 - 6084
a
A = 25^ = 25 = 2379
110 110 IlOx 110

= V2379 = 4.87
4.87
Coefficient of variation = -^4- x 100 = X 100 = 23.19
21

Manufacturer B:

ow
Bursting Mid-values
Xj-175
fi
pressure XiI
Ui =
5 fi »■ ^
5-10 7.5

e
9 -2 -18 36

re
10-15 12.5 11 -1 -11 11

Frl
15-20 17.5 18

F
0 0 0
20-25 22.5 32 1 32 32
ou
25-30 27.5 27

r
2 54 108

so
30-35 32.5 13 3 kf 39 117

N = I^=110 IfiUi=96 S f u.^ = 304


oo

Here,N=110, = 96,« = 17.5and h=5


Y
B

96
Xg = a-\-h = 175 + 5 X = 175 + 4.36 = 21.81
N 110
re

and, a B ^=/,2 n y r 2 yf f = 25-^


304 f 96 33440 - 9216
oY

(n ' }=25
u

= 50.04
110 110 110x110
ad

ag = ^/50.04 = 7.07
d

7.07
Coefficient of variation = ^ x 100 = xlOO = 32.41
21.81
in
Re

We observe that the average bursting pressure is higher for manufacturer B. So, bags
manufactured by B have higher bursting pressure.
F

The coefficient of variation


uniform pressure.
is less for manufacturer A. So, bags manufactured by A have more

EXERCISE 23.7
1. Two plants A and B of a factory show following results about the number of workers and
the wages paid to them
Plant A Plant B
No. of workers 5000 6000

Average monthly wages ?2500 ?^2500


Variance of distribution of wages 81 100

In which plant A or B is there greater variability in individual wages?


2. follows:
TTie means and standard deviations of heights ans weights of 50 students of a class are as
23.39
DATA INTERPRETATION - II (DISPERSION)

Weights Heights
Mean 63.2 kg 63.2 inch

5.6 kg 11.5 inch


Standard deviation
Which shows more variability, heights or weights?
3. Coefficient of variation of two distributions are 60% and 70% and their standard deviations
are 21 and 16 respectively. What are their arithmetic means?
4. Calculate coefficient of variation from the following data:
1000-1700 1700-2400 2400-3100 3100-3800 3800-4500 4500-5200
Income (in ?):
20 25 35 10
No. of families: 12 18

5, An analysis of the weekly wages paid to workers in two firms A and B, belonging to the

w
same industry gives the following results:
Firm A Firm B

Flo
586 648
No. of wage earners
^52.5 ^47.5
Average weekly wages

e
121

re
Variance of the 100

distribution of wages

rF
(i) Which firm A or S pays out larger amount as weekly wages?
ur
(ii) Which firm A or B has greater variability in individual wages? fo
6. The following are some particulars of the distribution of weights of boys and girls in a class:
Girls
ks
Boys
Yo
100 50
Number
oo

Mean weight 60 kg 45 kg
B

9 4
Variance
re

Which of the distributions is more variable?


7. The mean and standard deviation of marks obtained by 50 students of a class in three
u
ad

subjects, mathematics, physics and chemistry are given below:


Yo

Subject Mathematics Physics Chemistry


42 32 40.9
Mean
d

20
Re

Standard Deviation 12 15
in

Which of the three subjects shows the highest variability in marks and which shows the
F

lowest?

8. From the data given below state which group is more variable or Gj?
40-50 50-60 60-70 70-80
Marks 10-20 20-30 30-40
32 33 40 10 9
9 17
Group G|
30 25 43 15 7
10 20
Group G2
9. Find the coefficient of variation for the following data:
15-20 20-25 25-30 30-35 35-40
Size (in cms): 10-15
8 20 35 20 15
No. of items: 2

10. From the prices of shares X and Y given below: find out which is more stable in value:
X: 35 54 52 53 56 58 52 50 51 49
Y: 108 107 105 105 106 107 104 103 104 101
23.40
APPLIED MATHEMATICS-XI

11. Life of bulbs produced by two factories A and B are given below:
Length of life
550-650 650-750 750-850 850-950 950-1050
(in hours):
Factory A :
10 22 52 20 16
(Number of bulbs)
Factory B:
8 60 24 16 12
(Number of bulbs)
The bulbs of which factory are more consistent from the point of view of length of life?
12. Following are the marks obtained, out of 100, by two students Ravi and Hashina in 10 tests:
Ravi; 25 50 45 30 70 42 36 48 35 60

w
Hashina: 10 70 50 20 95 55 42 60 48 80
Who is more intelligent and who is more consistent?

Flo
ANSWERS

ee
1. Plant B 2. Heights 3. 35,22.85 4. 3.21 5. (i) Firm B (ii) Firm B

Fr
6. Boys 7. Highest: Chemistry Lowest: Mathematics 8. G 1 9. 21.75 10. y

11. Factory 12. Hashina is more intelligent and consistent. for


ur
MULTIPLE CHOICE QUESTIONS (MCQs)
k s
1. For a frequency distribution mean deviation from mean is computed by
Yo
oo

Zd
(a) M.D. = (b) M.D. =
eB

Zf\d\
Zfd Zf\d\
(c) M.D. = (d) M.D. =
2/
r

2/
ou
ad

2. For a frequency distribution standard deviation is computed by applying the formula


Y

(a) a =
Zfd^ f^/df (b) a = Z/df S/t/2
'^f J
Re
nd

s/
Fi

^2
(c) a =
Zfd^ Zfd Z fd S fd^
(d)
V I 2/
3. If V is the variance and a is the standard deviation, then
(a) V = ^ o
(b) v = -
a
(c) v = (d) v2 = a

4. The mean deviation from the median is


(a) equal to that measured from another value
(b) maximum if all observations are positive
(c) greater than that measured from any other value,
(d) less than that measured from any other value.
5. If n = 10, X = 12 and Zx^ = 1530, then the coefficient of variation i IS

(a) 36% (b) 41 % (c) 25 % (d) none of these


23.41
DATA INTERPRETATION ■ II (DISPERSION)

6. The standard deviation of the data:


2
x: 1 a a fl"
II 'C
"Co c1 ^2 '-1

IS
2>i
2'l
2 V* 1 + a fl + a
1 + rt
(a) (b)
2 2 2 2 ;

2h / 7 Y
1
(c) (d) none of these
V 2 2

7 The mean deviation of the series a,a + d,a + Id,a + In from its rnean is
(a)
(n + \)d (b)
nd (c) n(n + l)d
n{n + V) d ,..{2n + \)d
(d)
n {n +1)

w
2» + l
2«+T 2« + l

8. A batsman scores runs in 10 innings as 38, 70, 48, 34, 42, 55, 63, 46, 54 and 44. The mean

F lo
deviation about mean is
(b) 6.4 (c) 10.6 (d) 7.6
(a) 8.6

ee
9. The mean deviation of the numbers 3,4,5,6,7 from the mean is

Fr
(b) 5 (c) 1.2 (d) 0
(a) 25
10. The sum of the squares deviations for 10 observations taken from their mean 50 is 250. The for
coefficient of variation is
ur
(b) 40 % (c) 50 % (d) none of these
(a) 10 %
.V,, be values taken by a variable X and yy y2, ..., y„ be the values taken by a
s
11. Let Xy X2, ●●
ok
Yo

variable Y such that y,- = axj+b,i = 1, 2,...,«. Then,


o

(a) Var {Y) = Var (X) (b) Var (X) = fl Var (Y)
eB

(c) Var (X) = Var (X) + b (d) none of these


. If the standard deviation of a variable X is a, then the standard deviation of variable
r

12
ad
ou

aX + b .
IS
c
iia + b
Y

a
a
(b) - CJ (c) c (d)
(a) a o c c
Re

c
nd

is
13. If the S.D. of a set of observations is 8 and if each observation is divided by - 2, the S.D. of
Fi

the new set of observations will be


(b) -8 (c) 8 (d) 4
(a) -4
aX + b
, where fl, b, care constants
14. If two variates X and rare connected by the relation Y - ^
such thatflc <0, then
fl

(b) ay = - -
fl
(c) ay = - + b (d) none of these
(a) ay = -c ax

15. If for a sample of size 60, we have the following information Ix,-^ =18000 and Ix,- =960,
then the variance is
(b) 16 (c) 22 (d) 44
(a) 6.63
16. Let fl, b, c, d, e be the observations with mean m and standard deviation s. The standard
deviation of the observations a + k,b + k, c + k, d + k, e+ k is
23.42
APPLIED MATHEMATICS-XI

(a) s (b) ks (c) s + k


17. The standard deviation of first 10 natural numbers i
IS

(a) 5.5 (b) 3.87 (c) 2.97 (d) 2.87


18. Consider the first 10 positive integers. Ifmultiply each number by -1 and then add 1 to
we

each number, the variance of the numbers so obtained is


(a) 8.25 (b) 6.5 (c) 3.87 (d) 2.87
19. Consider the numbers 1,2,3,4,5,6, 7,8,9,10. If 1 is added to each number, the variance of
the numbers so obtained is
(a) 6.5 (b) 2.87 (c) 3.87 (d) 8.25
20. The mean of 100 observations is 50 and their standard deviation is 5. The sum of all squares

ow
of all the observations is
(a) 50,000 (b) 250,000 (c) 252500 (d) 255000
21. Let ari, a:2, ..., be n observations. Let i/,. =axj+b for f =1, 2,..., h, where and b are
constants. If the mean of Xj ® is 48 and their standard deviation is 12, the mean of y/® is 55 and

e
standard deviation of y,-® is 15, the values of <7 and b

re
are

(a) fl = 1.25,l7=-5

rFl (b) rt=-1.25,l;=5

F
(c) n = 2.5, b=-5 (d) a ^2.5,b =5
22. The mean deviation of the data 3,10,10, 4, 7,10, 5 from the mean is

or
ou
(a) 2 (b) 2.57 (c) 3 (d) 3.57
23. The mean deviation for n observations
ksf
Xi, a,*2, ..., -v„ from their mean X is given by
(a) Z -X) (b) 1 ± Ixf -X| (C) /=!
X (r, (d) 1",=1± (X, -X)2
oo
Y

24. Let X-I.X2,..X,; ben observations andX be their arithmeHc mean. The standard deviation
B

is given by
re

(a) £ (X;-xf (b) i 2 (i,.-X)2 (c) 1


(d)
1
oYu

1 = 1 "l=l n ●
ad

1 = 1 n
I = 1

25. The standard deviation of the observations 6,5,9,13,12,8,10 is


d

52 52
(a) 6 (b) ^/6
(c)y (d)
in
Re

26. The mean deviation of the data 2, 9,9,3,6,9,4 from the mean is
F

(a) 2.23 (b) 2.57 (c) 3.23 (d) 3.57


27. Variance of the data 2, 4, 5, 6, 8,17 is 23.33. The variance of 4, 8,10,12,16, 34 will be
(a) 23.33 (b) 25.33 (c) 46.66 (d) 93.32
28. A set of n values x^,X2,.. .,x,, has standard deviation a The standard deviation of n values
Xi +k, X2+ k,...,Xj,^i^ will be
(a) a (b) o + A: (c) a~k CT
(d) k
29. Let x^,X2,x^,x^,x^ be the observations with mean m and standard deviations. The
standard deviation of the observations brj, kx2, kx^, kx^, kx^ i
(a) k + s
Cb) fk (c)|A:|s (d) s
30. Coefficient of variation of two distribuHons are 50 and 60, and their arithmetic means are 30
and 25 respectively. Difference of their standard deviation is
u
23.43
DATA INTERPRETATION - II (DISPERSION)

(a) 0 (b) 1 (c) 1.5 (d) 2.5


C f-32
31. Given that — = is the relation between temperature in °C and °F. The standard
5 9

deviation of some temperature data in °C is 5. If the data were converted in °F, the variance
would be

(a) 81 (b) 57 (c) 36 (d) 25


32. When tested, the lives (in hours) of 5 bulbs were noted as follows: 1357,1090,1666,1494,
1623. The mean deviations (in hours) from their mean is
F
(a) 178 (b) 179 (c) 220 (d) 356
33. The following are the marks obtained by 9 students in Mathematics test: 50, 69, 20,33,53,
39,40, 65,59. The mean deviation from the median is
(a) 9 (b) 10.5 (c) 12.67 (d) 14.67

e ANSWERS
o
1- (d) 2. (a) 3. (c) 4. (d) 5. (c) 6. (a) 7. (c) 8. (a)

9. (c) 10. (a) 11. (a) 12. (c) 13. (d) 14. (b) 15. (d) 16. (a)

17. (d) 18. (b) 19. (c) 20. (a) 21. (a) 22. (b) 23. (b) 24. (c)

25. (d) 26. (b) 27. (d) 28. (a) 29. (c) 30. (a) 31. (a) 32. (b)

33. (c)

FILL IN THE BLANKS TYPES QUESTIONS (FBQs)

1. If X is the mean of n values x^, X2, x,j of variable X, then ^ (at/ -X) is always equal
i=1

to.

2. The mean deviation of the data is. when measvired from the median.
ad

3. The sum of the squares of the deviations of the values of the variable is
when taken about their arithmetic mean.
Re

4. IfX is the mean of n variables X2,...,A:„ofvariableXandflhasa ny valueotherthanX, is


than ^ (a:,- -a)^.
*=l

5. The standard deviation is to the mean deviation taken from the


arithmetic mean.

6. If the variance of data is 121 then the standard deviation of the data is
7. The standard deviation of data is of any change in origin, but
is. on the change of scale.
A
8. Coefficient of variation = X 100, then A =
Mean

9. A set of n values X2,---,x„ has standard deviation o.The standard deviation of ri values
flAT|, ax2, .../^xn
23.44
APPLIED MATHEMATICS-XI

10. If the variance of the data 2, 4, 5, 6, 8,17 is 23.33, then the variance of 4, 8,10,12,16, 34
IS.

11. The variance of first n natural numbers is

aX + b
12. If the standard deviation of a variable X is a, then the S.D. of the variable , where a, b,
c

c are constants, is
18 18
13. If x-^, X2,...XiQ are 18 observations such that (a::,-- 8) = 9 and {Xi~8f=45, then the
1=] 1=1

standard deviation of these observations is

w
14. If the standard deviation of first n natural numbers is 2, then n =

33
15. If the variance of 1, 2,3,4,5,..., 10 is —, then the standard deviation of 3,6,9,12,..., 30,

Flo
is

e
re
ANSWERS

F
1. zero 2. Least 3. Minimum 4. Less

5. greater than or equal 6. 11 7. independent, dependent


ur
r
8. standard deviation 9.|^7|a 10. 93.32
fo n. 12.
a
a
ks
12 c
Yo

13. 1.5 14. 7 15.


oo

2
eB

VERY SHORT ANSWER QUESTIONS (VSAQs)


ur

Answer each of the following questions in one word or one sentence or as per exact requirement of the
ad

question:
Yo

1. Write the variance of first n natural numbers.


2.
If the sum of the squares of deviations for 10 observations taken from their mean is 2.5, then
d
Re
in

write the value of standard deviation.


F

3.
If A^i, a:2 ,..., are n values of a variable X and J/i /1/2 / ● ● ● /
sre n values of variable Y such
that yj =aXj +b,i = 1,2,..., n, then write Var(Y) in terms of Var(X).
4.
aX + b
If X and Y are two variates cormected by the relation Y = and Var (X) = cr^, then
c

write the expression for the standard deviation of Y.


5. In a series of 20 observations, 10 observations are each equal to k and each of the remaining
half is equal to - k. If the standard deviation of the observations is 2, then write the value
ofk.

b. If each observation of a raw data whose standard deviation is a is multiplied by a, then


write the S.D. of the new set of observations.
23.45
DATA INTERPRETATION - II (DISPERSION)

, "C,j, then write


i;

7. If a variable X takes values 0,1,2,..., n with frequencies "Cq, ”Ci, C2 '●●●

variance X.
ANSIVERS

/I 2-1 3. Var (Y) = Var (X)


1. 2. 0.5
12
n

6. 1 fl I CT 7. -
a
4. a 5. ±2 4
c

ow
e
re
rFl
F
r
ou
fo
ks
oo
Y
B
re
ou
Y
ad
d
in
Re
F
CHAPTER 24
DATA INTERPRETATION-III
(SKEWNESS AND KURTOSIS)

24.1 INTRODUCTION

Recall that the central value of a distribution is the value of the variable which represents the
entire distribution i.e. it is that value of the distribution such that the maximum number of
observations are close to it. If two or more distributions have the same central value, then it does
:
not mean that they are alike. To distinguish them we compute the coefficient of dispersion by
taking the help of measures of dispersion. Dispersion of a distribution determines the extent of

w
scatterdness of the values of the variable around its central value. If the values of the variable are

Consider the following distributions:


I Variable:

Frequency:
0-5

10

0-5
5-10

30

5-10
10-15

60

10-15
F lo
closely concentrated around the central value the distribution is said to more consistent in
comparison to a distribution in which values of the variable are far from the central value.

15-20

60

15-20
20-25
for F
30

20-25
ree
25-30

10

25-30
II Variable:
10 40 90 30 20 10
Frequency:
Your
ks

y
eBoo

100
90 90
ad
our

80 80

70 70

60 60

50 50
Re

40
40
Y

30
30
Find

20 20

10 10

o X o 5 10 15 20 25 30 X
5 10 15 20 25 30

Fig. 24.1 (i) Fig. 24.1 (ii)

In both the distributions the value of the mean is 15 and standard deviation is 6 i.e. they have the
same central value and same coefficient of dispersion. But, these two distributions are not alike
in nature because they differ widely in their overall appearance as can be seen from their
frequency polygons shown in Fig. 24.2 (i) and 24.2 (ii). The first distribution is symrnetrical
about the central value whereas the second distribution is asymmetrical or skewed. It follows
from the above discussion that the measures of central value and dispersion are inadequate to
describe a distribution completely. Measures of skewness and kurtosis help us to distinguish
two or more distributions.
24.2
APPLIED MATHEMATICS-XI

24.2 SKEWNESS

Following are some definitions of skewness given by different statisticians:


DEFINITION 1 "Skeiuness refers to the asymmetry or lack of symmetry in the shape of a frequency
distribution
— Morris Hamberg
DEFINITION 2 "A distribution is said to be 'skewed' when the mean and the median fall at different points
in the distribution, and the balance (or centre of gravity) is shifted to one side or the other - to left or
’’'Sht"- -Garret
DEFINITION 3 “When a series is not symmetrical it is said to be a symmetrical or skewed".
— Croxton & Cowden

DEFINITION 4 "Measures of skewness tell us the direction and the extent of skewness. In symmetrical
distribution the mean, median and mode are identical. The more the mean moves azvay from the mode, the
larger the asymmetry or skcioness". —Simpson & Kafka

w
It follows from the above definitions that the term 'skewness' refers to lack of symmetry, i.e.
when the distribution is not symmetrical it is called a skewed distribution.

F lo
SYMMETRICAL DISTRIBUTION In a symmetrical distribution the frequencies are symmetrically
distributed on both sides of the centre point of the frequency curve. It is evident from Fig. 2.2 that in a

ee
symmetrical distribution the values of mean, median and mode coincide i.e. the spread of the frequencies is

Fr
same on both sides of the central point of the curve.
for
ur oks
Yo
o
eB

Fig, 24.2 (Mean = Median = Mode) Symmetrical distribution


our
ad

Mean = Median = Mode.

A distribution which is not symmetrical is called a skewed distribution and such a distribution
Y

could be either a positively skewed or negatively skewed as discussed below. The skewness of a
Re
nd

symmetrical distribution is zero.


Fi

POSITIVELY SKEWED DISTRIBUTION A distribution is said to be positively skewed distribution if the


frequencies are spread out over a greater range of values on the high-value end of the curve (i.e., the right
hand side) than they are on the low value end i.e., the excess tail is on the right-hand side.

Mode Mean

Median
Fig. 24.3 Positively skewed distribution
24.3
DATA INTERPRETATION-111 (SKEWNESS AND KURTOSIS)
is maximum and that of the
Thus, in a positively skewed distribution the value of the mean
mode is least, the median lies between the two i.e.
Mean > Median > Mode .

NEGATIVELY SKEWED DISTRIBUTION A distribution is said to be a negatively skewed if the


leytZcies are spread out over a swatter range of values on the low-value end of the curve (the left hand
side), than they are on the high value end i.e., the excess tail is on the left-hand side.
Thus, in a negatively skewed distribution the value of the mode is maximum and that of the
mean is least, the median lies in between the two i.e.
Mode > Median > Mean

w
F lo
Mean Mode

Median

ee
Fig. 24.4 Negatively skewed distribution

Fr
NOTE It Should be noted that in a moderately symmetrical distribution the interval between the mean and
the median is approximately one-third of the interval between the mean and mode i.e.
for
^ (Mean - Mode)
ur
Mean - Median =
s

3 (Mean - Median) = (Mean - Mode)


ook
Yo

or.

or. Mode = 3 Median - 2 Mean.


eB

24.2.1 TESTS OF SKEWNESS


Following tests can be applied to check whether distribution is skewed or not. A distribution is
our
ad

a skewed distribution i.e., skewness is present in it if:


(i) The values of mean, median and mode donot coincide,
dil Ouartiles are not equidistant from the median,
Y

(iii The sum of positive deviations from the median is not equal to the sum of the negative
Re

deviations i.e. the algebraic sum of the deviations about median is not zero,
nd

(iv) Frequencies are not equally distributed at points of equal deviation from the
Fi

(V) Frequency curve is not a bell-shaped curve i.e. when the data are plotted on a graph they
do not give the normal bell-shaped form.
It follows from the above points that a s
ymmetrical distribution satisfies the following
conditions:
(i) The values of mean, median and mode coincide
i.e. Mean = Median = Mode,
(ii) Quartiles are equidistant from the median i.e.
Q3 - Q2 = Qz " Q3 " = median - Qi-
(iii) The sum of the deviations from the median is zero,
(iv) Frequencies are equally distributed at points of equal deviations from the mode,
(v) Frequency curve is a normal bell-shaped curve.
24.3 MEASURES OF SKEWNESS
In the previous sections, we have seen that the term skewness refers to lack of symmetry. There
24.4
APPLIED MATHEMATICS-XI

are various methods of measuring skewness of a distribution. These measures of skewness


indicate the difference between ihe manner in which items are distributed in a given
distribution compared with a symmetrical distribution. In other words, they tell us the direction
and extent of asymmetry in a distribution. There are two types of measures of skewness;
(i) Absolute Measures of Skewness (ii) Relative Measures of Skewness
24.4 ABSOLUTE MEASURES OF SKEWNESS
I. BASED UPON MEAN, MEDIAN AND MODE The absolute
median and mode are: measures of skeumess based upon mean,

(i) Sj^ = Mean - Mode (ii) = Mean - Median


II. BASED UPON QUARTILES The absolute skewness S,, based upon the quartiles of a distribution is given
by the formula
-Q3 ^ Qi ~^Qz = Q3 + Qi - 2 (median)
The absolute measures of skewness are not very useful because the skewness computed by

w
using any one of these measures is expressed in the unit of value of the distribuHon and so it
cannot be compared with the skewness of another distribution expressed in different units.
KLMARk i The difference between mean and mode is used

F lo to measure the skewness because in


asijmmetnca distribution the values of mean, median and mode are equal and the mean moves azvau from
the mode when the observations are asymmetrical. Also, the distribution becomes more and more a

ee
symmetrical as the difference between mean and mode increases.

Fr
are used to measure the absolute skewness of a distribution due to the reason that in
for
a symmetrical distribution the middle quartile i.e. median is equidistant from the lower and upper
quartiles and lies between them. In a positively skewed distribution Q3 is fartherfrom median than Q-. IS
ur
from the median and in a negatively skewed distribution Qi is fartherfro
median. ^
m
the median than Q3 is from the
s
ook
Yo

24.5 RELATIVE MEASURES OF SKEWNESS


eB

To compare two or more distribuhons the absolute measures of skewness cannot be used. For
this purpose we compute the relative measures of skewness which are called the coefficients of
skewness. These are pure numbers independent of units of measurement. Hie foUowins are the
our
ad

coefficients of skewness: °
1. Karl Pearson's coefficient of skewness
2. Bowley's coefficient of skewness
dY

3. Kelly's coefficient of skewness


Re

4. Coefficient of skewness based on moments,


Fin

These measures of skewness


distributions.
are mainly used for comparison between two or more

24.5.1 KARL PEARSON’S COEFFICIENT OF SKEWNESS


The Karl Pearson's coefficient of skewness Sf. of a distribution is defined J7S
n Mean - Mode ^
Skp = Standard Deviation
Mean - Mode
Now, = 0 c=> = 0
s?o '
<=> Mean - Mode = 0
o Mean = Mode

<=> Distribution is symmetrical


Thus, a distribution is a symmetrical distribution iff S
'kp
= 0
24.5
DATA INTERPRETATION-III (SKEWNESS AND KURTOSIS)
Mean - Mode
>0 <=> >0
S.D.
Mean Mode > 0 [v S.D. >01
<=> Mean > Mode
<x> Distribution is positively skewed
>0.
Thus, a distribution is a positively skewed distribution iff S^.p
Mean - Mode
<0 <0
S.D.

o Mean - Mode < 0


<=> Mean < Mode
o Distribution is negatively skewed.
Thus, a distribution is a negatively skewed distribution iff S;,.p < 0.
It follows from the above discussion that the Karl Pearson's coefficient of skewness gives the
direction as well as the extent or degree of skewness. The degree of skewness is obtained from

w
the absolute value of Sj^p.
The Karl Pearson's coefficient of skewness cannot be

F lo used where the mode is ill-d^med.


However, in moderately skewed distributions mean, mode and median are connected by the
following relation

ree
Mean Mode = 3 (Mean - Median)

F
Therefore, for moderately skewed distribution, we have for
3 (Mean - Median)
Sjcp = S.D.

Theoretically, the value of this coefficient varies between - 3 and 3. However, in practice these
r
You

limits are rarely attained.


oks

ILLUSTRATIVE EXAMPLES
eBo

EX.-VMI’LH 1 Compute Karl Pearson's coefficient of skewness for a frequenci/ distribution having
mean - 50, mode - 56 and S.D. = 15.
ad
our

SOLUTION We have. Mean = 50, Mode = 56 and S.D. = 15.


Mean - Mode
Sk =
S.D.
dY
Re

50-56 6
= -0.4
Sjc = 15 15
Fin

Hence, Karl Pearson's coefficient of skewness is - 0.4.


EXAMPLE 2 Karl Pearson's coefficient of skewness of a distribution is 0.4. Its standard deviation is
and mean is 30. Find the mode.
= 0.4, 0 = 8 and X = 30.
SOLUTION We have, Sf.^
Mean - Mode
Skkp =
S.D.
30 - Mode
0.4 = => 3.2 = 30 - Mode => Mode = 30 - 3.2 = 26.8

Hence, mode = 26.8.


= 300,
EXAMPLES Calculate Karl Pearson's coefficient of skewness for the distribution for which mean
mode = 326 and o = 30.
SOLUTION We have. Mean = 100, Mode —126 and S.D. — 30.
24.6
APPLIED MATHEMATICS-XI

Mean - Mode
Sk =
S.D.
100-126 -26
s. = Si. = = - 0.87
Up 30 kp 30
EXAMPLl: 4
In a certain distribution thefollozoing results were obtained:
X = 45, Median = 48, Coefficient of skewness = - 0.4
The person who gave you the data failed to give the value of standard deviation and you are
required to estimate it with the help of the available information.
SOLUTION We have, X = 45, Median = 48 and Coeff. of skewness = - 0.4

Coefficient of skewness = 3 (X - Median)

ow
S.D.

0.4 =
3 (45 - 48)
=> S.D. =— = 225
S.D 0.4
EXAMPLE 5
Kar! Pearson's coefficient ofskenwess of a distribution is 0.32, its standard deviation is 6.5
and mean is 29.6. Find the mode of the distribution.

e
re
Fl
SOLUTION We have Sj. - 0.32, a = 65 and Mean = 29.6.
Kp

F
. - Mean - Mode
kp S.D.
ur
r
29.6 - Mode
=> 0.32 =
6.5 fo
ks
=> 0.32 X 6.5 =29.6-Mode => Mode = 29.6-0.32 x 6.5 => Mode = 29.6 - 2.08 => Mode = 27.52
Yo
oo

EXAMPLE 6
In a moderately skewed distribution the mean is ? 20 and the median is ^ 18.5. If the
coefficient of variation is 30%, find the Pearsonian coefficient of skewness of the distribution.
eB

SOLUTION We have, X = 20, Median = 18.5 and C.V. = 30.


We know that:
ur

C.V.=-^ X 100 =>30 =


CJ
X 100 ^ a = 6.
ad

X 20
Yo

Since the distribution is moderately skewed. Therefore,


„ _ 3 (Mean - Median) 3(20-185)
d

^kp => S
kp = = 0.75
Re

S.D.
in

EXAMPLE 7 A distribution showed the following measures of location: Mean = 45, Median = 48,
F

Coefficient ofskezvness = - 0.4. Estimate the standard deviation with the help of the above information.
SOLUTION We have,
- 3 (Mean - Median) 3(45-48) 3 X - 3 90
-0.4 =
Id. ^ S.D.
=> S.D.=
-0.4
— = 22.5
4
EXAMPLES
From a moderately skewed distribution of retail prices for men's shoes, it is found that the
mean price is 120 and the median price is X17. If the coefficient of variation is 20%, find the Pearsonian
coefficient of skewness of the distribution.
SOLUTION We have. Mean = 20, Median =17 and C.V. = 20
a
C.V. = X 100 =4> 20 = — X 100 a = 4.
X 20

And, _ 3 (Mean - Median) _ 3(20-17) 9


%p = S.D. 4
-=2.25
4
24.7
DATA INTERPRETATION-III (SKEWNESS AND KURTOSIS)

EXAMPLE 9 A frequency distribution gives the following results.


Coefficient of variation = 5, Standard deviation = 2, Karl Pearson's coeff of skewness = 0.5.
Find the mean and mode of the distribution.
= 05
SOLUTION We have, C.V. =5, ct = 2 and S/^-p
Now, C.V.=5
a
V C.V. = xlOO
^xl00=5 X
X

i xl00=5 ^ 5X=200 => X=40.


X

ow
Mean - Mode
And, S^.p = S.D.

0.5 = 20 Mode ^ X 2 = 20 - Mode Mode = 20-1-19.


EXAMPLE 10 In a liscreate series of 20 terms; the sum of the terms is 200; the sum of the squares of the

e
re
terms is 5000 and the median is 15. Find Karl Pearson's coefifcient ofskezvness.
SOLUTION We have, n = 20,1 Xj = 200, Z x- = 5000 and Median - 15.
rFl
F
1 200
= 10

r
Mean=- Zx; => Mean = 20
ou
n
fo
f _ [sOQO poof = 7250-100'=V1^= 12-24
ks
a
a = S.D.=
11 n
/
\n
- I XI
20 I 20
oo

-15
3 (Mean - Median)— 3(10-15) = -1.22
^ % 12.24
Y

^ ~
Now, 5k = 12.24
eB

kp S.D.

Hence, Karl Pearson's coefficient of skewness is -1.22


For a group of 20 items, X = 1452,1 -144280 auri made = 63.7. Find Pearson's
r

EXAMPLE 11
ou
ad
Y

coefficient of skewness.
SOLUTION We have, n = 20, X a; = 1452, X x/ = 144280 and Mode = 63.7.
d

1 1452
= 72.6
Re
in

X = - Zxi => X = 20
n
F

-Ir ^ 1
-ZXiI
f
S.D.= I
n \n

144280 1452 f 44.08


S.D.= = ^7214 -5270.:^ = a/1943.24 =
^20 V 20
Mean - Mode 72.6 - 63.7
= 0.20
Now, Skkp = - => Sif —
S.D kp 44.08

Hence, Pearson's coefficient of skewness is 0.20.


EXAMPLE 12 Compute the Karl Pearson's coefficient of skewness from the following36data: 42
18 24 30
6 12
Value (x): 5
18 15 10
4 7 9
Frequency (f):
SOLUTION Let A = 24 and h = 6.
24.8
APPLIED MATHEMATICS*XI

X;I
fi fm
h
fi4
6 4 - 3 -12 36
12 7 -2 -14 28
18 9 -1 -9 9
24 18 0 0 0
30 15 1 15 15

ow
36 10 2 20 40
42 5 3 15 45
N=Ijf=68
lfiU,^=173

e
re
Calculation of mean: We have,

Frl
F
X =A + h(~Zfi
vN J
X = 24 +6x—
68
= 25.32 [●●● /I = 24, h = 6]
ou
Calculation of Mode: Since mode is the value of the variable which occurs maximum number of

r
times i.e. it is that value of the variable which has the maximum
so
— '
frequency. Clearly the
maximum frequency 18 corresponds to 24. Hence, mode = 24. kf
Calculation of standard deviation: We have.
oo

n2
Y

a =h
B
re

173 15
a = 6 X
= 6 X .^2544 - 0.049 = 6 x 158 = 9.48
oY

68 Us [v h=6]
u
ad

Calculation of Karl Pearson's coefficient of skewness: We have,


Mean - Mode
d

25.38-24 1.32
=> .S = 0.139.
'kp
a
9.48 9.48
in
Re

EXAMPLE 13 Calculate the Karl Pearson's coefficient of skewness from the data given below
F

Size: 1 2 3 4 5 6 7
Frequency: 10 18 30 25 12 3 2
SOLUTION Taking the assumed mean /I = 4, we have
fi Ui=Xi-4 fn fi^^
1 10 -3 - 30 90
2 18 -2 -36 72
3 30 -1 - 30 30
4 25 0 0 0
5 12 1 12 12
6 3 2 6 12
2. 2 3. 18
N =2 = 100
Ij5^=-72 =234
24.9
DATA INTERPRETATION-III (SKEWNESS AND KURTOSiS)

Calculation of Mean: We have,


X -A + ^'ZfiUi
N
( 72 ^ 328 = 3.28 [vA=4]
X =4 +
100 100

Calculation of mode: Since mode is that value of the variable which has the maximum frequency.
Since the maximum frequency occurs corresponding to the value 3 of the variable X.
So, Mode = 3.
Calculation of standard deviation : We have.
^2
c =h
N

Here, /i =1.
1 n f ^ ^ 1234: 23400-(72)^ 134.96 = 1.3496

w
100^
<T =
N 100 100 100

Calculation of coefficient of skewness: We have.

Marks:
Sk

EXAMl’LE 14

No. of students:
=
Mean - Mode _ 3.28 - 3
S.D.

0-10
1.3496~

5
10-20
6
= 0.2074

20-30
11
F lo
From the following data, calculate Karl Pearson’s coefficient of skewness:
30-40
21
40-50
for
35 F ree
50 - 60
30
60 - 70
22
70 - 80
18

SOLUTION Taking^ = 35 and/i = 10, we have


r
You

-35
Mid-values No. of students f Uj= — 10
oks

Marks
eBo

-3 -15 45
0-10 5 5
-2 -12 24
10-20 15 6
ad
our

-1 -11 11
20-30 25 11
0 0 0
30-40 35 21
1 35 35
40-50 45 35
Re
dY

30 2 60 120
50-60 55
3 66 198
60-70 65 22
Fin

72. 288
70-80 7i 18 A
N=Z if =148 Ifuj =195 S fj =721

Calculation of mean : We have.


f 1
X = A+ h —'^fi^H
N
195 [v A = 35, h=10]
X = 35 +10 X = 35 + 13.176=48.176
148

Calculation of Mode: The maximum frequency 35 corresponding to the class 40 - 50. So, 40 - 50 is
the modal class.
/ = 35,/i=21,/2 = 30,/i=10,/ = 40.
24.10
APPLIED MATHEMATICS-XI

Now, Mode = / + f-h X h


2/-/1-/2
35-21 14 140
Mode = 40 + xl0 = 40 + —X 10=40 + = 40 + 7.37=47.37
70 - 21 - 30 19 19

Calculation of S.D.: We have.

a =h
1/n N

721 (\95^
a = 10 X
\us 148 = 10 X ^872 -1.736 = 10 x 1.771 = 17.71
Calculation of coefficient of skczvness: We have.
Mean - Mode 48.176 - 47.37

w
%p = o 17.71
= 0.046.

F lo
EX.^MPLE 15 Compute Karl Pearson's coefficient of skewness from the folloxuing data:
Marks: 0-10 10-20 20-30 30-40 40-50

ee
Frequency: 8 11 26 9 6

Fr
SOLUTION Taking >1 =25 and/i= 10, we have
Marks Mid-values
fi X:I -25 for fi^H
ur
^● =
10
s
0-10 5 8 -2
ook

-16 32
Yo

10-20 15 11 -1 -11 11
eB

20-30 25 26 0 0 0

30-40 35 9
r

1 9 9
ou
ad

40-50 45 6 2 12 24
Y

N = 60
zfu^=-e
Re
nd

Calculation of Mean: We have,


Fi

X=A + h(—l:fU:
X = 25 + 10 X — = 25-1=24.
60 [●●● A = 25, h = 10]

Calculation of Mode: Since the maximum frequency is corresponding to the class 20 - 30. So
20 - 30 is the modal class such that I = 20, h = 10, / = 26, /[ = 11 and /2 = 9.
Now, Mode = / + f-fl X h
2/-/1-/2
26-11 15 75
Mode = 20 + X 10 = 20 + — X 10 = 20 + — = 24.69
52-11-9 32 16

Calculation of S.D.: We have.


24.11
DATA INTERPRETATION-III (SKEWNESS AND KURTOSIS)

o = h

I76 X 60 - 36 67.26
76 = 11.21
a=10 = 10
^60 60 , 60^

Calculation of Karl Pearson's coefficient of skezoness: We have,


Mean - Mode 24. - 24.69 - 0.69
=> S = - 0.062
%p = S.D.
'kp 11.21 11.21

EXAMPLE 16 Calculate Karl Pearson's coefficient of skexvness from the follotoing


80
data: 100
Below 20 40 60
Profits: in lacs) 50 64 70
8 20
No. of companies:
SOLUTION The given frequency distribution is a cumulative frequency distribution. This can be

w
written as a frequency distribution as follows;
40-60 60-80 80-100
20-40

F lo
0-20
Profits: f? in lacs) 14 6
8 12 30
No. of companies:

ee
We shall now prepare a table for the computation of mean, mode and standard deviation as

Fr
given below;

Proifts No. of Mid-values Xj -50 for fi '


ur
20
^ in lacs companies f X;t

A =50, /J=20
ks

-16 32
10 -2
Yo

0-20 8
oo

-12 12
30 -1
20-40 12
eB

0 0 0
40-60 30 50
14 14
70 1
60-80 14
r
ou
ad

12 24
90 2
80-100 6
I.fUi=-2 I .^,,."=82
Y

N=70
Re
nd

Computation of Mean: We have, N —70, Uj = -2, A =50, and /i = 20


Fi

r-2'\
X=A + /i (Ur) =>X =50 + 20x = 49.42
[ N j 70

Computation ofS.D.: We have,


N=70, S_/i-i(,-=-2,Sj}i/;^=82,A=50and /(=20
1 82 -2f = 20
82 X 70 - 4
= 20
o =h
^70 70 V 70^
^N N

V5736 20 X 75.73 = 21.63


ct = 20x
70 70

' is of class 40-60. So, 40-60 is the modal


Computation of Mode: Since the maximum frequency '30
class such that I = 40, h = 20, / = 30, f = 12 and /2 -14.
24.12
APPLIED MATHEMATICS-XI
Now,

Mode = / + f-if xh
V-h-h
30-12 18
Mode = 40 +
60-12-14
X 20 = 40 + —X 20=40 + 10.58 = 50.58
34

Compilation of Karl Pearson's coefficient of skewness: We have,


„ _ Mean - Mode 49.42-5058
= -0.0536
S.D. 2L63

gives the data relating to marks obtained by the students who appeared
from ihZTlT ofsZoness

ow
Marks: 0-10 10-20 20-30 30-40 40 - 50 50 - 60 60-70 70-80
No. of students: 10 40 20 0 10 40 16 14

e
SOLUTION Here the maximum frequency corresponds to two classes

re
formula:

Fl
each other. So, mode is ill-defined and we
which are very far from
shall compute the coefficient of skewness by using the

F
_ 3 (Mean - Median)
ur
%p =

r
S.D.

Taking = 35 and =10, we obtain the following table:


fo
ks
Marks
Mid-values Xj
Yo
No. of students f cf -t;I - 35
^'f- =
oo

10
0-10
eB

5 10 10 -3 - 30 90
10-20 15 40 50 -2 -80 160
ur

20-30 25 20 70 -1 -20 20
ad

30-40 35 0 70
Yo

0 0 0
40-50 45 10 80 1 10 10
d

50-60 55 40 120 2 80 160


Re
in

60-70 65 16 136 3 48 144


F

70-80 75 14 150 4 56 224


N=Z^=150 ^fiUj=64 2 f i^= 808
Calculation of mean: We have.
64
35 +10 X = 35 + 4.27 = 39.27
150

Calculation of median : We have N = 150 => — = 75


2

The cumulaHve frequencyjust greater than ^ is 80 and it is corresponding to the class 40 - 50. So,
40 - 50 is the median class such that/ = 40,/i= 10,/=10 and P =70.
24.13
DATA INTERPRETATION-III (SKEWNESS AND KURTOSIS)

75-70
Median = / + — X /i = 40 + X 10 = 40 + 5 = 45
10
/

Calculation of S.D.: We have.

S.D. = /i
N

808 ( 64
10 X ^5.387 -0.182 =10 X 2.281 =22.81
S.D.=10x
i 150 150

Calculation of coefficient of skewness


-17.19

w
3 (Mean - Median) 3(39.27 -45) = 0.754
S.D. 22.81 22.81

Calculated Karl Pearson’s coefficient of skewness from thefollowmg data:

F lo
EXAMPLE 18

Weight (in lbs) 90-100 100-110 110-120 120-130 130-140 140-150 150-160 160-170 170-180

ee
21 19 10 3 2
18 22
No. of Students 4 2

Fr
SOLUTION Taking A = 135 and h = 10, we obtain the following table:
Calculation of Mean and Standard Deviation for
ur
2
X;I -135 fi^H fi^h
Weight Mid-values Xj fi =
s
(in lbs) 10
k
Yo
oo

4 -4 -16 64
90 -100 95
eB

2 -3 -6 18
100-110 105

18 -2 -36 72
110-120 115
r

22 -1 -22 22
ou

120-130 125
ad

21 0 0 0
130-140 135
Y

19 1 19 19
140 -150 145
nd
Re

10 2 20 40
150-160 155
Fi

3 3 9 27
160 -170 165

2 4 8 32
170 -180 175

N=I.f =101 Zfu^=-24 If It,? =294

Calculation of mean: We have,


X =A + h
N
-24
X =135 +10 X = 135-2.38=132.62
101

Calculation of standard deviation: We have,


^2
S.D. = /ix
24.14
APPLIED MATHEMATICS-XI

S.D. = 10x
294
r-24^
101 [ 101, = 10 X pSl - 0.06 = 1.689 X 10 = 16.89
Calculation ofMode: The maximum frequency is 22 corresponding to the class 120 -130. Since the
class 130 -140 has frequency 21, which is very close to 22. So, it is not clear which class is the
modal class and the distribution seems to be bimodal. To decide whether the distribution is
bimodal or not, we prepare the table and analysis table as given below:
Grouping Table
Weight Frequency
in (lbs)
I II III IV V VI

90-100 4

w
6

100-110 2 24

Flo
20

e
110-120

re
18

F
40 42

120-130 22
ur
r
43 fo 61
130-140 21
ks
62
Yo

40
oo

140-150 19
B

29 50
re

150-160 10 32
u
ad

13
Yo

160-170 3 14
d

4
Re
in

170-180 1
F

Analysis Table
Column No.
Class in which mode is expected to lie
110-120 120-130 130 -140 140 -150
I 1 1

II 1 1 1
m 1 1
IV 1 1 1
V 1 1
VI 1 1 1
2 5 5 3
24.15
DATA INTERPRETATlON‘lll (SKEWNESS AND KURTOSIS)

From the arralysis table we find that the classes 120-130 and 130 -140 repeat the ^
times i.e. 5. So^ the distribution is bimodal. Hence, we apply the following foimula to compute
the skewness:
3 (Mean - Median)
Sfcp = S.D.

Calcuintion o/wedian
fi c.f.
Weight in lbs
4 4
90 -100
2 6
100-110

ow
18 24
110-120
22 46
120 -130
21 67
130 -140
19 86
140 -150

e
96

re
10
150-160

160 -170

rFl 3 99

F
2 101
170 -180
hi =lfi = 101

or
ou
, N 101
ksf
We have, — = -— = 505
2 2
N .
The cumulative frequency just greater than — IS 67 and the corresponding class is 130 -140. So,
oo
Y

130 - 140 is the median class such that / = 130, /i = 10, / = 21, f = 46.
B

N
-F
re

505 - 46
Median = / + -^ X h =130 + X 10 =130-^2.14 =132.14
21
/
oYu
ad

Calculation of coefficient ofskeivness:


3 (Mean - Median) _ 3 (132.62 -132.14) _ 1-47 = 0.087
d

S.D. 16.89 16.89


in
Re

EXERCISE 24.1
F

1. Define the term skewness and write absolute and relative measures of skewness,
2. What are various measures of skewness ? Which measure is generally used and why ?
3. Define the Karl Pearson's coefficient of skewness.
4. What is skewness ? Explain the main types of skewed curves.
5. What is positive and negative skewness ?
6. What are the tests of skewness ?
7. From the following series calculate the Karl Pearson's coefficient of skewness: 15
12 13 14
Measurement: 11
6 4 3
3 9
Frequency:
8. Compute the Karl Pearson's coefficient of skewness from the30following 35d^ata: 40
15 20 25
Variable:
24 20 21
12 28 25
Frequency:
24.16
APPLIED MATHEMATICS-XI

9. Calculate Pearson's coefficient of skewness from the following:


Age: 0-10 10-20 20-30 30-40 40-50 50-60 60-70
No. of Persons: 10 12 24 32 28 11 3
10. Calculate Karl Pearson's coefficient of skewness from the following table:
Wages: 50-60 60-70 70-80 80-90 90-100 100-110 110-120
No. of Persons: 15 18 17 30 40 20 10
11. Compute Pearson's coefficient of skewness for the following data:
Wage (in 270-280 280-290 290-300 300-310 310-320 320-330 330-340 340-350
No. of Persons: 12 18 35 42 50 45 20 8

ow
12. Calculate Pearson's coefficient of skewness for the following data:
Wages (in f): 10 -15 15-20 20-25 25-30 30-35 35-40 40-45 45-50
No. of Workers: 8 16 30 45 62 32 15 6

e
13. Calculate Karl Pearson's coefficient of skewness from the data given below:

re
Wages (in V:

Fl
40-50 50-60 60-70 70-80 80-90
90-100 100-110 110-120 120-130 130-140

F
No. of Workers: 5 6 10 25 30 36 50 60 70
ur
14. Calculate Karl Pearson's coefficient of skewness for the following distribuhon:

r
Monthly Salary (in fo No. of Salesmen
ks
400 but less than 600 4
Yo
600 but less than 800
oo

10
800 but less than 1000 19
eB

1000 but less than 1200 12


1200 but less than 1400 4
1400 but less than 1600
ur

1
ad

15. Calculate Pearsonian coefficient of skewness from the following data:


Yo

Life Time (in hours): 300- 400- 500- 600- 700- 800- 900- 1000- 1100-
400 500 600
d

700 800 900 1000 1100 1200


Re
in

Number of Tubes: 14 46 58 76 68 62 48 22 6
F

resuffi Pearson's coefficient of skewness from the following data and interpret the
Income per day) No. of employees
0 and above 150
10 and above 140
20 and above 100
30 and above 80
40 and above 80
50 and above 70
60 and above 30
70 and above 14
80 and above 0
24.17
DATA INTERPRETATION-111 (SKEWNESS AND KURTOSIS)

17. Compute Karl Pearson's coefficient of skewness from the following data:
23.5-26.5 26.5-29.5 29.5-32.5 32.5-35.5 35.5-38.5
Variable: 20-23.5
399 194 27 10
17 193
Frequena/:
18 For a moderately skewed distribution of retail prices for men's shoes it was found that the
mean price is ? 200 and median price is ? 190. If the coefficient of variation is 20%, find the
Pearsonian coefficient of skewness of the distribution.
19. The following information was obtained from the records of a factory relating to wages.
A.M = ? 56.80, Median = ? 50.50, S.D. =? 12.40
Calculate the coefficient of variation and skewness.

ow
= 05. Find the
20. A frequency distribution gives the following results: C.V. = 5, S.D. = 2, S^p
mean and mode of the distribution. _
21. Find the coefficient of variation of a frequency distribution, given that X = 120,
Mode = 123 and = - 0.3.
- 0.64. Find the

e
22. A frequency distribution gives the following results: C.V. = 0.8, a — 5, S^p -

re
mean and mode of the distribution.
23. Compute mean, mode, standown deviation and Karl Pearson's Coefficient of skewness for

Frl
F
each of the following frequency distributions:
(i) Class: 5-15 15-25 25-35 35-45 45-55 55-65
35 20
Frequency: 20
ou 5 15 5

sor
0-10 10-20 20-30 30-40 40-50 50-60
(ii) Years:
16 18 25 20 9
No. of Pearsons: 12 kf
65 80 95 110 125 140
50
oo
(iii) Wages above (in V:
57 47 23 0
No. of workers: 100 82 69
Y
B

70-80 80-90 90-100 100-110 110-120 120-130 130-140 140-150


(iv) Daily wages (in V:
42 50 45 20 8
12 18 35
No. of workers:
re

24. Compute mean, median, standard deviation and Karl Pearson's coefficient for each of the
oY
u

following frequency distributions:


ad

10 15 20 25 30 35
(i) Variable: 5
d

6 8 7 9 12 5
Frequency: 3
in

2 3 4 5 6
(ii) Variable: 1
Re

12 7 2 1
Frequency: 2 6
F

40-50 50-60
(iii) Marks: 0-10 10-20 20-30 30-40
30 20 17 8 10
No. of Students: 15
20 30 40 50 60
(iv) Age (in years): Under 10
32 51 78 97 109
No. of persons: 15

25. Calculate Karl Pearson's coefficient of skewness from the following data:
20-25 25-30 30-35 35-40 40-45 45-50 50-55 55-60
Age:
50
50 70 80 180 150 120 70
No. of persons:
26. The mean, median and Karl Pearson's coefficient of skewness of a frequency distribution
65,70 and - 0.6 respectively. Find the mode and the standard deviation.
are

27. Which of the following two series is symmetrical ?


Series A: Mean = 22, Median = 24, S.D. = 10
Series B: Mean = 22, Median = 25, S.D. = 12.
24.18
APPLIED MATHEMATICS-XI

ANSWERS

7. 2 8. 0.9962 9. -0.358 10. -0.443


3

11. -0.331 12. -0.224 13. -0.682 14. 0.034


15. 0.243 16. -0.754 17. 0.0683 18. 0.25

19. C.y. = 21.83, Si, =-0.653 20. X = 40, Mode = 39


21. 8.82 22. X = 625, Mode = 628.2
23. (i) Mean = 39, Mode = 51.66, S.D. = 18.13, ,Sh

w
0.698
(ii) Mean = 30.2, Mode = 35.83, S.D. = 14.13, Si. = -0.37
Kp
(iii) Mean = 99.2, Mode = 124, S.D.: = 27.54, Su = -0.9
kp
(iv) Mean = 110.44, Mode = 116.15, S.D. . = 17.25, Sr- = -0.33
kp

o
e
24. (i) Mean = 21.9, Median = 25, S.D. = 8.7, St, =-1.068
kp

re
(ii) Mean = 3.134, Median = 3, S.D = 1.117, St'kp =0.357

Frl
F
(iii) Mean = 25.3, Median = 22.5, S.D. = 15.12, S^ = 055
kp
(iv) Mean = 29.95, Median = 31.29, S.D. = 15.48, S^ = - 0.259
kp
ou
r
25. -0.239 26. Mode = 80, S.D. = 25 27. Series A

so
HINTS TO THE SELECTED PROBLEMS
kf
13. Since the distribution is highly skewed distribution. Therefore, use the formla
oo

3 (Mean - Median)
Y

50
B

^4. The frequency distribution obtain fromthe given cumulative frequency distribution as
re

follows:
oY
u

Month!}/ Salary (in ?): 400-600 600-800 800-1000 1000-1200 1200-1400 1400-1600
ad

Number of Salesmen: 4 10 19 12 4 1
d

16. Prepare the frequency distribution from the given cumulative frequency distribution. The
in

frequency distribution will be bimodal. So, use the formula _ 3 (Mean - Median)
Re

S
kp S.D.
F

18. We have, X = 200, Median = 190 and C. V. = 20.


a cr
Now, C.V. = 20 X 100 =20 => X 100 = 20 => cr = 40.
X 200

■■■ _= 3 (Mean - Median) 3 (200 -190) = 0.75


S.D 40
a
21. Use:C.V.= X 100
X

_ 3 (Mean - Median) ,
26. Using Si,^ = S.D.
we obtain.

3(65-70) 15
-0.6 = => S.D.= = 25
S.D. 0.6

27. Compute S^.^ for both the series. The series having smaller coefficient of skewness in
i
magnitude will be more symmetrical.
24.19
DATA INTERPRETATION'llI (SKEWNESS AND KURTOSIS)

24.5.2 BOWLEY’S COEFFICIENT OF SKEWNESS


The Bowley's coefficient of skewness is based on quartiles and is defined
as

(Q^ - Median) - (Median -Qi) _ Q:^ +Qi -2 Medi^


Qa Qs

In a symmetrical distribution the first and the third quarhles are equidistant from the median
as

shown in Fig. 24.5.

Qi Median Qs
Fig. 24,5

Thus, The distribution is symmetrical

ow
Median -Qi =Qs “ Median
<=> Qg + Q,^ - 2 Median = 0
<=>
+Q-^ -2 Median = 0
Qa

e
re
= 0.

rFl
In a positively skewed distribution the top 25% of the values tend to be farther from the median

F
than the bottom 25 percent i.e. Q3 is farther from the median than Qi is from the median.

r
ou
fo
ks
oo
Y
eB

Ql Median Q3
ur

Fig. 24.6
ad
Yo

Q3 - Median > Median - Qi


Thus, distribution is positively skewed
d

Q3 -Median > Median -Qi


Re
in

<=> - 2 Median > 0


Q-^ +Qi -2 Median
F

o
>0
Qa “Qi
>0

If the distribution is negatively skewed, then the top 25% observations are nearer to the median
than the bottom 25% i.e. Q3 is nearer to the median than Qi is from the median.
Q3 - Median < Median - Qi
Thus, The distribuHon is negatively skewed
<=> Q3 - Median < Median -Qi
<» +Qj-2 Median <0
- 2 Median
<=>
<0
Q3-Q1
<=> <0.
24.20
APPLIED MATHEMATICS-Xl

Qi Median

Fig. 24.7

It follows from the above discussion that S. is zero, positive or negative according as the

ow
distribution is symmetrical, positively skewecTor negatively skewed. Also, the denominator in

the formula for is twice the quartile deviation, so the degree of skewness is measured
relative to the dispersion of the distribution.
The BouHei/s coe^cientofsk^^^^^ is also called the quartile measure ofskezouess and it varies betzoeen

e
t and I. Tins method of measuring skezvness is usefid in open-end distributions and where the extereme

re
values are present.

rFl
F
ILLUSTRATIVE EXAMPLES

r
EXAMPLE 1 hi a distribution, the difference between tzvo quartiles is 15, their sum is 35 and median 20
ou
find the coefficient of skezvness. fo
ks
SOLUTION
We have, Q3 - Qj -15, Q3 + Qj = 35 and. Median = 20.
_ Q3 Qi “ 2 Median 35 - 2 X 20 1
oo

=> S = -0.333
Q3-Q1 15 3
Y
eB

EXAMPLE 2 In a frequency distribution, the coefficient of skezvness based on quartiles is 0 5 If the sum
of the upper and loiuer quartiles is 28 and the median is 1 l,find the values of lower and upper quartiles.
SOLUTION The coefficient of skewness based on quartiles
r

is given bv
c _ Q3 + Qi - 2 Median
ou

^kB
ad
Y

Q3-Q1

We have, -05,Q3 +Qi =28 and Median = 11. Substituting these values in the above
d

tormula, we get
Re
in

05 =
28-2x11 11 6
Q3-Q1
F

Thus, we haveQ3 +Q^ =28 andQ3 -Qi=12


Solving these two equations, we get Q3 = 20 and Qj = 8.
Hence, Q3 =20 and = 8.

» CnfcH/ofe the coefficient of skewness if mode = 11,


quartiles = 8 and the sum of two quaritles is 22.
mean = 8, the difference of two
^
SOLUTION
We have, Q3 - 8, Q3 + Q] = 22, Mean = 8 and, Mode = 11.
Mode = 3 Median - 2 Mean
11 = 3 Median - 2x8
3 Median =27
Median = 9

Now, _ Q3 + Ql - 2 Median 22-2x9 1


=> ,S - = 05
Q3-Q1 'kB = 2
24.21
DATA INTERPRETATlON-ni (SKEWNESS AND KURTOSIS)

EXAMPLE 4 Calculate Boivlei/s coefficient of skezoness for the distribution for which
= 62, Q3 =195 and Median = 142.
SOLUTION We have, Qi =62, Median =142 andQ3 =195
195 + 62 - 2 X 142 27
Q3 + Ql -2 Median = - 0.203
195-62 133
Q3 “Ql
EXAMPLE 5 The marks obtained bi/ ten students in a test are:
43,12, 31, 20,17, 26, 35, 40, 5, 37
Compute the Bowley's coefficient of skewness for the above data,
SOLUTION First we shall compute quartiles from the given data. For this the given data is
arranged in ascending order of magnitude as given below:

ow
5,12,17, 20, 26, 31, 35, 37,40, 43
Here, n=10
111
n + n
Ql = Value of — observation = Value of 2.75^''' observation
^1 ^ 4

e
Fl
re
- 2nd observation)]
Ql = 2nd observation + 0.75 (3rd observation

F
Ql =12 + 0.75 X (17 -12) = 15.75
ur
th
—4 (j? +1)^^ observation = Value of 8.25 observation
Q3 = Value of
th
observation + 0.25 (9^ observation - 8
f
th or
observation)
ks
Yo

Q3 = 37 + 0.25 (40 - 37) = 37.75


oo

and,
B

th th r 26 + 31 = 28.5
Median = Mean of 5 and 6 observation =
re

35
Qg + Ql - 2 (Median) _ 37.75 + 15.75 - 2 x 285
u

= -0.158.
ad
Yo

37.75-15.75 22
Qs “Ql
EX.\MPLE6 From the information given beloiv, calculate Karl Pearson's coefficient ofskewness and also
d

Bowlei/’s quartiles coefficient of skewness:


Re
in

Measure: Place A Place B


F

150 140
Mean

142 155
Median

30 55
S.D.

195 260
Third quartile
62 80
First quartile
SOLUTION Coefficient of skewness for place A:
3 (Mean - Median) _ 3 (150 -142) _ 4 _ ^ g
S.D. 30 5

195 + 62 - 2 X 142 27
Q3 + Ql “ ^ Median = -0.203
195 - 62 133
Q3 “Ql
24.22
APPLIED MATHEMATICS-XI

Coefficients of skewness for place B:


^ _ 3 (Mean - Median) 3(140-155) 9
-=0.818
S.D. 55 11

_ Qs + Ql ~ 2. Median 260 + 80 - 2 X 155 30


= 0.166
S.D. 260 - 80 180

EXAMPLE 7 Calculate Bowley's coefficient of skewness from thefolloioing data:


Variable: 5 15 25 35 45
Frequency: 10 20 40 20 10

SOLUTION First we shall compute quartiles from the following table:

ow
Computation of quartiles
Variable X
Frequency f cf
5 10 10

e
15 20 30

re
25

rFl 40 70

F
35 20 90
45 10 100

r
ou
N = 100
fo
ks
Computation of lower quartile: We have, N = 100 => — = 25.
4
oo

The cumulative frequency just greater than — i.e. 25 is 30 and the corresponding value of X is 15.
Y
eB

Ql=15.
r

Computation of median: Wehave, A/=100 =5-— =50


ou

2
ad
Y

The cumiilaHve frequency just greater than ~ i.e. 50 is 70 and the corresponding value of Xis 25.
d

Median = 25.
Re
in

3N
Computation of upper quartile: We have, N = 100 => = 75.
F

The cumulaHve frequency just greater than ^ i.e. 75 is 90 and the corresponding value of
variable X is 35. Therefore, Q3 = 35.
Computation of skewness: We have, = 15, Q2 = 25 and Q3 = 35.
c _ Q3 + Qi - 2 Median 35 + 15 - 2 X 25
^kB ^ S
Xb - = 0.
Q3 ~Ql 35-15

Thus, Sfcg = 0 i.e. the distribution is a symmetric distribution which is also evident from the
distribution.
EXAMPLE 8
From the following data compute Bowley's coefficient of skewness:
Marks:
30-40 40-50 50-60 60-70 70-80 80-90 90-100
No. of students: 3 5 11 20 12 8 6
24.23
DATA INTERPRETATION-III (SKEWNESS AND KURTOSIS)

SOLUTION Calculation of coefficient of skewness


Marks No. of students f cf.
3 3
30-40
5 8
40-50
11 19
50-60
20 39
60-70
12 51
70-80
8 59
80-90
6 65
90 -100

w
N = 65

N 65
Calculation ofQ-^: We have, N = 65=>-^=-^ = 16.25

Flo
Clearly, the cumulative frequency just greater than N/4 is « and the

e
50 - 60. So, 50 - 60 is the lower quartile class such that / =50,/-ll,F-» and n

re
F
Qi = I + -4 f X h
ur
r
16.25-8 8.25 825 fo
X 10=50 + X 10=50 + =57.5
Ql=50 + 11 11 11
ks

Calculation of median: We have, y = y = 32.5


Yo
oo

Clearly, the cumulative frequency just greater than N/2 is 39 and the corresponding class is 60 -
B

70 So, 60 - 70 is the median class such that / = 60, -10, / - 20, f -19.
re

N-f
Median = l + — X h
u

f
ad
Yo

325-19 135
Median = 60 + X 10 = 60 + — = 66.75
20 2
3N 3 X 65
d

= 48.75
Calculation 0/Q3: We have, -y = —^
Re
in

Clearly, the cumulative frequency just greater than 3N/4 is 51 and *e corresponding class i
is
F

70 - 80. So, 70 - 80 is the upper quarUle class such that / - 70, / -12, ft - iu, +
1N.-F
X h
Q3=' + —
f
48.75 - 39 97.5
-xlO =70 + =78.125
Q3=70 + 12 12

Calculation of the coefficient of skewness: We have,


Q^ + Qt-2 Median
Q3-Q1
78.125 + 575 - 2 x 6675
78.125 - 575
[v Qi =575, Median =6675 andQs =78.125 ]
135.625-1335 2.125 = 0.1030
78.125 - 575 20.625
24.24
APPLIED MATHEMATICS-XI

EXAMPLE 9 The weekly zvciges earned by 100 workers of a factory are as follows. Find the Bowley's
coefficient ofskeiuness:
Daily wages (in^.h 12.5-17.5 17.5-22.5 22.5-27.5 27.5-32.5 32.5-37.5 37.5-42.5 42.5-47.5 47.5-52.5 52.5-57.5
No. of workers: 12 16 25 14 13 10 6 3 1
SOLUTION
Calculation of coefficient of skewness
Daily ivages (in No. of workers f cf
12.5-17.5 12 12
17.5 - 22.5 16 28
22.5 - 27.5 25 53
27.5 - 32.5 14 67

w
32.5 - 37.5 13 80
37.5 - 42.5 10 90

Flo
42.5 - 47.5 6 96

ee
47.5 - 52.5 3 99

Fr
52.5 - 57.5 1 100
N=100
for
ur
N
Calculation ofQ-^: We have, N = 100 => = 25.
4
k s

The cumulative frequency just greater than ^ i.e. 25 is 28 and the corresponding class is 17.5
Yo
oo

22.5. So, 17.5-22.5 is the lower quartile class such


eB

that/=175 h=5 f = 16 f=12

Ql=/ + ^ X h
r

f
ou
ad

25-12 65
Ql=175 + -x5=175-h —=2156
Y

16 16

Calculation of median: We have,/V =100 => — =50


Re
nd

The cumulative frequency just greater than ^ i.e. 50 is 53 and the corresponding class is 22.5 -
Fi

27.5. So, 22.5 - 27.5 is the median class such that 1=225, h=5,f = 25,F = 28.
N
-F
Median = / -f- -2. X h
f
50-28
Median = 225 -i- X 5 = 225 -I- 4.4 = 26.9
25
3N 3x100
Calculation of Q^: We have, — = 75.
4 4

The cumulaHve frequency just greater than ^ i.e. 75 is 80 and the corresponding class is 32.5 -
37.5. So, 32.5 - 37.5
is the upper quarHIe class such that / = 325, / = 13, f = 67 and h =5.
24.25
DATA INTERPRETATION-III (SKEWNESS AND KURTOSIS)

3N
-F
-X /j
Q3 - / +
/
75-67
Qs = 325 + X 5 = 325 + 3.08 = 3558
13

Calculation ofskeiuness : The Bowley's coefficient of skewness is given by


Q3 + Ql - 2 Median _ 3558 + 2156 - 2 x 26.9 _ 57.14 - 5^ _ 3.34 = 0.238
3558 - 2156 14.02 14.02
Q3 “Ql
EXAMPLE 10 Calculate Boxvlei/s coefficient of skewness from the folloiuing data:
10 20 30 40 50 60 70 80 90 100
Merits Below:

5 9 17 29 45 60 70 78 83 85
Number of students:
SOLUTION We are given the cumulative frequency distribution of less than type. So, we shall
first prepare the frequency distribution from it.
Calcidation of Boxoleffs coefficient of skewness

w
cf

F lof
Marks

5 5
0-10

4 9
10-20

ree
17
20-30 for F
12 29
30-40
16 45
40-50
15 60
50-60
Your
ks

10 70
60-70
eBoo

8 78
70-80
5 83
80-90
ad

85
our

90 -100 2

N = 85
Re

Calculation o/Qj; We have N = 85=>-j=-y = 21-25


Y
Find

. N
The cumulative frequency just greater than — i.e. 21.25 is 29 and the corresponding class is 30 -

40 So 30 - 40 is the lower quartile class such that / = 30, h =10, f = 12, F = 17.

X h
f
21.25-17
Ql = 30 + X 10 = 30+ 3.54 = 33.54
12

Calculation of Q2 (= Median): We have, M = 85 => — =— = 42.5.


, N
Tl-ie cumulative frequency just greater than — i.e. 42.5 is 45 and the corresponding class is
40 - 50. So, 40 - 50 is the median class such that / = 40, h = 10, / = 16, F = 29.
24.26
APPLIED MATHEMATICS-XI

N
-F
Median = / + -2. xh
f
425 - 29 135
=> Median =40 + X 10 = 40 + X 10 = 40+ 8.44 =48.44
16 16
3N
Calculation of : We have, N =85 — - 63.75. Clearly, the cumulative frequency just

greater than-^i.e. 63.75 lies in the class 60 - 70. So, 60- 70 is the upper quartile class such that
/ = 60,/t=10,/=10,F = 60.

ow
Q3=' + ^ xh
f
63.75 - 60
Q3=40 + X 10 = 63.75
10

e
Calculation of coefficient ofskeioness: We have,

Fl = 33.54, Median = 48.44 and Q3 = 63.75.

re
c _ Q3 + Qi - 2 Median

F
-

Q3
ur
or
63.75 + 33.54 - 2 X 48.44 97.29 - 96.88 0.41
= 0.014
63.75 - 33.54 30.21 30.21
sf
EXAMPLE 11
For the following frequnecy distribution, calculate the coefficient of skewness based on
k
Yo
qiiartiles:
oo

Class limits: 10- 19 20-29 30-39 40-49 50-59 60-69 70-79 80-89
B

Frequency: 5 9 14 20 25 15 4
re

SOLUTION Here the class intervals are formed by the inclusive method. So, first we convert

them into an exclusive series by subtracting and adding | in the lower and upper limits
u
ad
Yo

respectively, where h = difference between the lower limit of a class and the upper limit of the
preceding class.
d

Here, h = 1. So the class intervals in inclusive form are given below:


Re
in

Class limits
Frequency cf
F

9.5 -19.5 5 5

19.5-29.5 9 14

29.5 - 39.5 14 28
39.5-49.5 20 48
49.5 - 59.5 25 73
59.5 - 69.5 15

69.5 - 79.5
96
79.5 - 89.5 4 100
N = 100
24.27
DATA INTERPRETATION-IH (SKEWNESS AND KURTOSIS)
N
Computation o/Qj: We have, N = 100 => — = 25.
N .
. 25 is 28 and the corresponding class is
The cumulative frequency just greater than ^
i.e

29.5 - 39.5. So 29.5 - 39.5 is the lower quartile class such that I = 295, h = 10, / = 14 and F -14.
N
-F
X h
Qi = ' + -
f
25-14
Qi=295 + X 10 = 295 + 7.86 = 37.36
14
N
= 50.
Computation of Q2 (Median): We have, N= 100 => —
N .

The cumulative frequency just greater than y i.e., 50 is 73 and the corresponding class is

w
49.5 - 59.5. So, 49.5 - 59.5 is the median class such that / = 495, / = 25, h = 10, f = 48.

Flo
N
-f
●y
Median = / + — X h
f

ee
50-48

Fr
Median = 495 + X 10 = 495 + 0.8 = 50.3
25
3N
= 75.
Computation 0/Q3 : We have, IV =100 for
ur
3N
The cumulative frequency just greater than i.e. 75 is 88 and the corresponding class is
k s
Yo

59.5 - 69.5. So, 59.5 - 69.5 is the upper quartile class such that / =595, / =15, It =10, F =73.
oo

3N
eB

-F

Q3=/ + ^ f X h
r

75-73
ou
ad

X 10 =595+ - = 60.83
Q3 =595 + 15 3
Y

Computation of skewness: We have, Qt = 37.36, Q2 -50.3 andQ3 -60.83.


Q3 +Ql -2 Median
Re
nd

Q3-Q1
Fi

60.83+ 37.36-2(50.3) 2.41


= 0.103.
60.83 - 37.36 23.47

EXERCISE 24.2

1. Define Bowley's measure of skewness.


2. In a frequency distribution the coefficient of skewness based on quarriles is 0.6. If the sum
of the upper and lower quartiles is 100 and median is 38, find the value of the upper
quatirle.
3. For a distribution Bowley's coefficient of skewness is 0.36, Qi =8.6 and Median - 12.3.
What is the quartile coefficient of dispersion.
4. If the first quartile is 142 and the semi-i-inter quartile range is 18, find the median, assuming
that the distribution is symmetrical about mean or median.
24.28
APPLIED MATHEMATICS-XI

5. Find Bowley's coefficient of skewness for the following frequency distribution


No. ofchilderen: 0 1 2 3 4 5 6
No. of families: 7 10 16 25 18 11 8

6. Calculate measure of skewness based


Variable:
on quartiles and median from the following data:
10-20 20-30 30-40 40-50 50-60 60-70 70-80
Freqiieuci/: 358 2417 976 129 62 18 10

7- Calculate the Bowley's coefficient of skewness from the following:


Class limits: 10-15 15-20 20-25 25-30 30-35 35-40 40-45 45-50
Frequency: 2 5 7 13 21 15 8 3

ow
8. Compute the Bowley's coefficient of skewness from the following data:
Variable:
0-5 5- 10 10-15 15-20 20-25 25-30 30-35 35-40
Frequencij: 7 10 20 13 17 10 14 9

e
9. Following figures ralate to the size of capital of companies:

re
Fl
Capital in Lakhs of^ No. of companies

F
1-5
ur 20

r
j 6-10 fo 27

11-15 29
ks
I 16-20
Yo
38
oo

21-25 48
eB

' 26-30 53

31-35 70
I
ur

Find out the coefficient of skewness with the help of Bowley's measure of skewness.
ad
Yo

10. Calculate coefficient of variation and coefficient of skewness by Karl Pearson and Bowley's
method from the following data: ^
d

Marks: 0-10 10-20 20-30 30-40 40-50 50 - 60 60 - 70


Re
in

No. of students: 5 15 30 65 80 92 100


F

I'rom the following data compute quartiles and hence find out the coefficient of skewness.
Income (in No, of persons
Below 200 25

200 - 400
40

400 - 600 85

600 - 800 75

800- 1000 16

1000 and above 16


24.29
DATA INTERPRETATION-MI (SKEWNESS AND KURTOSIS)

12. Calculate the coefficient of skewness from the following data by using quartiles:
Marks No. of students Marks No. of students
Above 60 65
Above 0 180
/f
75 20
15 160
//
130
n
90 5
30
A5. JlOO

ANSWERS

3. 0.24 4. 160 5. 0
2. Q3 = 70
8. 0.06 9. -0.12

ow
6. 0.131 7. ^0.055

10. C.v.= 41%, Su =0.85, Skb = 0.09 12. -0.113


= 0.045
11. Qi =40059, Q2 =554.12, Qs=722, S^g
HINTS TO SELECTED PROBLEMS

e
2. We have, S^._ = 0.6, Q3 + Qi-100 and Median 38

re
.-. S
Q3 + Ql - 2 Median

rFl
F
kB
Q3
100 - 2 X

You might also like